You are on page 1of 785

WORKBOOK AND CASEBOOK

For Goodman and Gilmans


T e Pharmacological Basis of T erapeutics
NOTICE
Medicine is an ever-changing science. As new research and clinical experience broaden our knowledge,
changes in treatment and drug therapy are required. he authors and the publisher o this work have
checked with sources believed to be reliable in their e orts to provide in ormation that is complete
and generally in accord with the standards accepted at the time o publication. However, in view o the
possibility o human error or changes in medical sciences, neither the authors nor the publisher nor
any other party who has been involved in the preparation or publication o this work warrants that the
in ormation contained herein is in every respect accurate or complete, and they disclaim all responsi-
bility or any errors or omissions or or the results obtained rom use o the in ormation contained in
this work. Readers are encouraged to con irm the in ormation contained herein with other sources. For
example and in particular, readers are advised to check the product in ormation sheet included in the
package o each drug they plan to administer to be certain that the in ormation contained in this work
is accurate and that changes have not been made in the recommended dose or in the contraindications
or administration. his recommendation is o particular importance in connection with new or in re-
quently used drugs.
WORKBOOK AND CASEBOOK
For Goodman and Gilmans
T e Pharmacological Basis of T erapeutics

EDITORS
Douglas E. Rollins, MD, PhD Donald K. Blumenthal, PhD
Professor Emeritus, Pharmacology &Toxicology Associate Professor of Pharmacology &Toxicology
University of Utah Associate Dean for Interprofessional Education &
Salt Lake City, Utah Assessment, College of Pharmacy
University of Utah
Salt Lake City, Utah

New York Chicago San Francisco Lisbon London Madrid Mexico City
Milan New Delhi San Juan Seoul Singapore Sydney oronto
Copyright 2016, by The McGraw-Hill Companies, Inc. All rights reserved. Except as permitted under the United States Copyright Act of 1976, no part of this publication may
be reproduced or distributed in any form or by any means, or stored in a database or retrieval system, without the prior written permission of the publisher, with the exception that
the program listings may be entered, stored, and executed in a computer system, but they may not be reproduced for publication.

ISBN: 978-0-07-179337-7

MHID: 0-07-179337-2

The material in this eBook also appears in the print version of this title: ISBN: 978-0-07-179336-0,
MHID: 0-07-179336-4.

eBook conversion by codeMantra


Version 1.0

All trademarks are trademarks of their respective owners. Rather than put a trademark symbol after every occurrence of a trademarked name, we use names in an editorial fashion
only, and to the bene t of the trademark owner, with no intention of infringement of the trademark. Where such designations appear in this book, they have been printed with
initial caps.

McGraw-Hill Education eBooks are available at special quantity discounts to use as premiums and sales promotions or for use in corporate training programs. To contact a
representative, please visit the Contact Us page at www.mhprofessional.com.

TERMS OF USE

This is a copyrighted work and McGraw-Hill Education and its licensors reserve all rights in and to the work. Use of this work is subject to these terms. Except as permitted under
the Copyright Act of 1976 and the right to store and retrieve one copy of the work, you may not decompile, disassemble, reverse engineer, reproduce, modify, create derivative
works based upon, transmit, distribute, disseminate, sell, publish or sublicense the work or any part of it without McGraw-Hill Educations prior consent. You may use the work
for your own noncommercial and personal use; any other use of the work is strictly prohibited. Your right to use the work may be terminated if you fail to comply with these terms.

THE WORK IS PROVIDED AS IS. McGRAW-HILL EDUCATION AND ITS LICENSORS MAKE NO GUARANTEES OR WARRANTIES AS TO THE ACCURACY,
ADEQUACY OR COMPLETENESS OF OR RESULTS TO BE OBTAINED FROM USING THE WORK, INCLUDING ANY INFORMATION THAT CAN BE ACCESSED
THROUGH THE WORK VIA HYPERLINK OR OTHERWISE, AND EXPRESSLY DISCLAIM ANY WARRANTY, EXPRESS OR IMPLIED, INCLUDING BUT NOT
LIMITED TO IMPLIED WARRANTIES OF MERCHANTABILITY OR FITNESS FOR A PARTICULAR PURPOSE. McGraw-Hill Education and its licensors do not warrant
or guarantee that the functions contained in the work will meet your requirements or that its operation will be uninterrupted or error free. Neither McGraw-Hill Education nor
its licensors shall be liable to you or anyone else for any inaccuracy, error or omission, regardless of cause, in the work or for any damages resulting therefrom. McGraw-Hill
Education has no responsibility for the content of any information accessed through the work. Under no circumstances shall McGraw-Hill Education and/or its licensors be liable
for any indirect, incidental, special, punitive, consequential or similar damages that result from the use of or inability to use the work, even if any of them has been advised of the
possibility of such damages. This limitation of liability shall apply to any claim or cause whatsoever whether such claim or cause arises in contract, tort or otherwise.
CONTENTS

Pre ace. . . . . . . . . . . . . . . . . . . . . . . . . . . . . . . . . . . . . . . . . . . . . . . vi 28 Estrogens, Progestins, Androgens,


and Contraception . . . . . . . . . . . . . . . . . . . . . . . . . . . . . . 441
SECTION I 1 29 AC H, Adrenal Steroids, and
Pharmacology o the Adrenal Cortex . . . . . . . . . . . . . . 459
General Principles
30 Endocrine Pancreas and Pharmacotherapy
1 Pharmacodynamics . . . . . . . . . . . . . . . . . . . . . . . . . . . . . . . 2 o Diabetes Mellitus and Hypoglycemia . . . . . . . . . . . . 470
2 Pharmacokinetics. . . . . . . . . . . . . . . . . . . . . . . . . . . . . . . . 18 31 Drug T erapy o Mineral Ion Homeostasis
3 Clinical and Environmental oxicity. . . . . . . . . . . . . . . . 40 and Bone urnover Disorders . . . . . . . . . . . . . . . . . . . . 480
4 Special Populations (Children and Elderly) . . . . . . . . . . 67
SECTION VI 489
SECTION II 79
Drugs Af ecting Gastrointestinal Function
Neuropharmacology
32 Pharmacotherapy o Gastric Acidity, Peptic
5 Neurotransmission. . . . . . . . . . . . . . . . . . . . . . . . . . . . . . . 80 Ulcers, and Gastroesophageal Re ux Disease . . . . . . . 490
6 Cholinergic Pharmacology. . . . . . . . . . . . . . . . . . . . . . . . 95 33 Drugs Used or the reatment o Bowel Disorders . . . 497
7 Adrenergic, Dopaminergic, and
Serotonergic Pharmacology . . . . . . . . . . . . . . . . . . . . . . 118
SECTION VII 511
8 Psychopharmacology. . . . . . . . . . . . . . . . . . . . . . . . . . . . 149
9 Hypnotics, Sedatives, and Ethanol. . . . . . . . . . . . . . . . . 176 Chemotherapy o Microbial Diseases
10 Opioid Pharmacology . . . . . . . . . . . . . . . . . . . . . . . . . . . 194 34 General Principles o Antimicrobial T erapy . . . . . . . 512
11 Anesthetic Agents and T erapeutic Gases . . . . . . . . . . 210 35 Chemotherapy o Malaria . . . . . . . . . . . . . . . . . . . . . . . . 520
12 Pharmacotherapy o the Epilepsies . . . . . . . . . . . . . . . . 225 36 Chemotherapy o Protozoal In ections: Amebiasis,
13 Drug T erapy o Neurodegenerative Diseases. . . . . . . 238 Giardiasis, richomoniasis, rypanosomiasis,
14 Drug Addiction . . . . . . . . . . . . . . . . . . . . . . . . . . . . . . . . 254 Leishmaniasis, and Other Protozoal In ections. . . . . . . 530
37 Chemotherapy o Helminth In ections. . . . . . . . . . . . . 536
SECTION III 267 38 Sul onamides, rimethoprim-Sul amethoxazole,
Quinolones, and Agents or Urinary
Modulation o Cardiovascular Function
ract In ections. . . . . . . . . . . . . . . . . . . . . . . . . . . . . . . . . 542
15 Drug T erapy o Hypertension, Edema, and 39 Penicillins, Cephalosporins, and
Disorders o Sodium and Water Balance . . . . . . . . . . . 268 Other -Lactam Antibiotics . . . . . . . . . . . . . . . . . . . . . . 549
16 Drug T erapy o Myocardial Ischemia . . . . . . . . . . . . . 289 40 Aminoglycosides . . . . . . . . . . . . . . . . . . . . . . . . . . . . . . . 559
17 Pharmacotherapy o Heart Failure . . . . . . . . . . . . . . . . 301 41 Protein Synthesis Inhibitors and
18 Antiarrhythmic Drugs. . . . . . . . . . . . . . . . . . . . . . . . . . . 312 Miscellaneous Antibacterial Agents . . . . . . . . . . . . . . . 568
19 Drug T erapy o T romboembolic Disorders . . . . . . . 323 42 Chemotherapy o uberculosis, Mycobacterium
20 Drug T erapy o Dyslipidemias . . . . . . . . . . . . . . . . . . . 334 Avium Complex Disease, and Leprosy . . . . . . . . . . . . . 578
43 Anti ungal Agents . . . . . . . . . . . . . . . . . . . . . . . . . . . . . . 588
SECTION IV 345 44 Antiviral Agents and reatment
In ammation, Immunomodulation, o HIV In ection . . . . . . . . . . . . . . . . . . . . . . . . . . . . . . . . 600
and Hematopoiesis
21 Histamine, Bradykinin, and T eir Antagonists . . . . . . 346 SECTION VIII 625
22 Prostaglandins, NSAIDs, and Chemotherapy o Neoplastic Diseases
Pharmacotherapy o Gout. . . . . . . . . . . . . . . . . . . . . . . . 356 45 Cancer Chemotherapy and Cytotoxic Agents . . . . . . . 626
23 Immunotherapeutic Agents . . . . . . . . . . . . . . . . . . . . . . 375 46 argeted Anticancer T erapies. . . . . . . . . . . . . . . . . . . . 661
24 Pulmonary Pharmacology . . . . . . . . . . . . . . . . . . . . . . . 389
25 Hematopoietic Agents . . . . . . . . . . . . . . . . . . . . . . . . . . . 405 SECTION IX 689
SECTION V 421 Special Systems Pharmacology
47 Ocular Pharmacology . . . . . . . . . . . . . . . . . . . . . . . . . . . 690
Hormones and Hormone Antagonists 48 Dermatological Pharmacology. . . . . . . . . . . . . . . . . . . . 707
26 Introduction to Endocrinology:
T e Hypothalamic-Pituitary Axis . . . . . . . . . . . . . . . . . 422 Index . . . . . . . . . . . . . . . . . . . . . . . . . . . . . . . . . . . . . . . . . . . . . . 728
27 T yroid and Antithyroid Drugs . . . . . . . . . . . . . . . . . . . 431

v
PREFACE

T is Workbook and Casebook derives rom the 12th edition Mechanisms o Action (and, where appropriate, Mechanisms
o Goodman & Gilmans T e Pharmacological Basis of T era- o Resistance) able
peutics. T e organization o the parent text has been retained Clinical Cases
and many o the tables and gures are the same. T e editors
Key Concepts
have attempted to provide the most important concepts rom
the 12th edition o G&G in a user- riendly ormat without the Summary Quiz
elimination o salient pharmacological in ormation. It is our Summary able o Drugs that includes Drug Class, Drug
intention that the Workbook and Casebook will complement Name, Clinical Uses, and Common and Unique Clinically
the parent text and the second edition o Goodman & Gilmans Important oxicities
Manual of Pharmacology and T erapeutics and be use ul to
Additional Side Bars are included to highlight signi cant phar-
students and educators who might need a companion text in
macological in ormation. Narrative pharmacological in orma-
an advanced pharmacology course.
tion is provided in the orm o Clinical Cases to give a clinical
T e number o chapters in the Workbook and Casebook has been context or the therapeutic use o drugs and their adverse events.
reduced rom 67 to 48 by combining the substance o multiple We have omitted research data, chemical structures, re erences,
chapters rom the 12th edition into one Workbook and Casebook and the pharmacokinetic data o Appendix II, all o which can
chapter. For example, Chapter 9 Muscarinic Receptor Agonists be ound on the G&G Web site on AccessMedicine.com and
and Antagonists, Chapter 10 Anticholinesterase Agents, and AccessPharmacy.com, along with pharmacotherapeutic updates
Chapter 11 Agents Acting at the Neuromuscular Junction and and mechanistic animations.
Autonomic Ganglia in the 12th edition have been combined
We thank the contributors and editors o the 12th edition
into one Workbook chapter entitled Cholinergic Pharmacology.
o G&G, Christie Naglieri and James Shanahan o McGraw
An additional chapter, Chapter 4 Special Populations: Children
Hill, and the long line o contributors and editors who have
and the Elderly, has been added to the Workbook and Casebook
worked on Goodman & Gilman over 12 editions. It is an
based in part on content that is published in Updates in the
honor to ollow in the path o Louis Goodman and Al red
online version o Goodman & Gilman (Accessmedicine.com
Gilman, and it is a tribute to their writing and editing that
and Accesspharmacy.com).
their book is use ul and relevant 74 years af er the publication
Each Workbook and Casebook chapter begins with a statement o the rst edition.
o how it di ers rom the 12th edition so that the reader can
re er to the parent text to obtain additional in ormation i nec-
Douglas E. Rollins
essary. Workbook and Casebook chapters (except or chapters
Donald K. Blumenthal
on general principles) are ormatted as ollows:
Learning Objectives
Drugs Included in the Chapter

vi
SECTION

General Principles I
1. Pharmacodynamics 1

2. Pharmacokinetics 18

3. Clinical and Environmental oxicity 40

4. Special Populations (Children and Elderly) 67

1
CHAPTER

1 Pharmacodynamics
T is chapter will be most use ul a er having a basic understanding o the material in
PHARMACODYNAMIC Chapter 3, Pharmacodynamics: Molecular Mechanisms o Drug Action in Goodman
CONCEPTS AND & Gilmans T e Pharmacological Basis of T erapeutics, 12th Edition Additional in or-
NOMENCLATURE (key terms mation related to this chapter is provided in Chapter 1, Drug Invention and the Phar-
a re indica ted in ita lics) maceutical Industry and Chapter 7, Pharmacogenetics T e drugs presented in this
Pharmacodynamics is the studyo the bio- chapter are used to illustrate general pharmacodynamic principles T e mechanisms
chemical and physiological e ects o drugs o action and therapeutic uses o drugs described in this chapter are discussed in more
and their mechanisms o action. detail in subsequent chapters Neither a Mechanisms o Action able nor a Clinical
Summary able is included in this chapter because this in ormation is provided in
Pharmacodynamics re ers to the e ects o subsequent chapters
a drug on the body; in contrast, the e ects
o the bodyon the actions o a drug are In addition to the material presented here, Goodman & Gilmans T e Pharmacological
pharmacokineticprocesses (see Chapter 2). Basis of T erapeutics, 12th Edition contains:
The termdrug receptor or drug target A description in Chapter 1 o the process o drug invention and FDA approval
denotes the cellular macromolecule or able 1-1, ypical Characteristics o the Various Phases o the Clinical rials
macromolecular complexwith which the Required or Marketing o New Drugs
drug interacts to elicit a cellular response.
Figure 1-1, T e Phases, ime Lines, and Attrition T at Characterize the Invention o
Drugs commonlyalter the rate or magni- New Drugs
tude o an intrinsiccellular response rather
A comprehensive description in Chapter 3 o the various mechanisms o drug action,
than create newresponses.
including cellular pathways activated by physiological receptors, structural and unc-
Drug receptors are o ten located on the tional amilies o physiological receptors, second messengers, ion channels, nuclear
sur ace o cells but mayalso be located in receptors, and transcription actors
speci cintracellular compartments.
Chapter 3 also describes mechanisms o receptor desensitization and regulation, and
Manydrugs also interact with acceptors an example o pharmacodynamic interactions in a multicellular context
within the body; acceptors are entities
Chapter 7 includes a number o examples o genetic polymorphisms that a ect drug
that do not directlycause anychange in
pharmacodynamics, the impact o pharmacogenetics on drug development, and a
biochemical or physiological response.
discussion o pharmacogenetics in clinical practice
Proteins ormthe most important class o
drug receptors. Examples include: LEARNING OBJECTIVES
Physiological receptors or hormones, Understand key concepts and terms related to pharmacodynamics, including
growth actors, transcription actors, drug receptor agonism and antagonism
and neurotransmitters (see Table 1-1)
Know concepts and terms that are used to quanti y drug receptor interactions,
Enzymes o crucial metabolicor including a nity, e cacy, potency, KD and Ki.
regulatorypathways (eg, dihydro olate
reductase, acetylcholinesterase, and Understand how drug pharmacodynamic in ormation is used to predict benef -
cyclicnucleotide phosphodiesterases) cial and toxic drug e ects
Proteins involved in transport processes Know the modern process o drug invention and FDA approval, and post-market
(eg, Na+, K+-ATPase); secreted surveillance
glycoproteins (eg, Wnts) Know how genetic polymorphisms and other actors can a ect the pharma-
Structural proteins (eg, tubulin) codynamic properties o drugs and lead to variability in individual patient
responses to drugs
Speci cbindingo drugstoother cellular
constituentssuchasDNAisalsoexploited or Know how pharmacodynamics is applied to populations o patients to estimate
therapeuticpurposes(eg, manycancer che- population therapeutic windows or drug dosing
motherapeuticagentsandantiviral drugs).
Antibioticand other anti-in ectives o ten
target enzymes and biochemical processes
that are unique to the pathogen, resulting
in cytotoxicityor inhibition o proli eration.

2
Pharmacodynamics CHAPTER 1

TABLE 1-1 Physiological Receptors


EFFECTORS AND
STRUCTURAL FAMILY FUNCTIONAL FAMILY PHYSIOLOGICAL LIGANDS TRANSDUCERS EXAMPLE DRUGS
G Protein coupled Adrenergic receptors NE, Epi, DA Gs; AC Dobutamine, propranolol
receptors (GPCRs)

Muscarinic cholinergic ACh Gi and Gq; AC, Atropine, carbachol


receptors ion channels, PLC

Eicosanoid receptors Prostaglandins, leukotrienes, Gs, Gi, and Gq proteins Misoprostol, montelukast
thromboxanes

Thrombin receptors (PAR) Receptor peptide G12/13, GEFs Vorapaxar

Ion channels Ligand-gated ACh (M2), GABA, 5-HT Na+, Ca2+, K+, Cl Nicotine, gabapentin

Voltage-gated None (activated by Na+, Ca2+, K+, other ions Lidocaine, verapamil
membrane depolarization)

Transmembrane Receptor tyrosine kinases Insulin, PDGF, EGF, VEGF, SH2 domain and PTB- Herceptin, imatinib
enzymes growth actors containing proteins

Membrane-bound GC Natriuretic peptides Cyclic GMP Neseritide


Tyrosine phosphatases

Transmembrane, Cytokine receptors Interleukins and other Jak/STAT, soluble Anakinra


non enzymes cytokines tyrosine kinases

Toll-like receptors LPS, bacterial products MyD88, IARKs, NF-B (in development)

Nuclear receptors Steroid receptors Estrogen, testosterone Co-activators Estrogens, androgens, Cortisol

Thyroid hormone receptors Thyroid hormone Thyroid hormone

PPAR, PPARa Eiconsanoids, atty acids RXR Thiazolidinediones

Intracellular enzymes Soluble GC NO, Ca2+ Cyclic GMP Nitrovasodilators

AC, adenylyl cyclase; DA, dopamine; GC, guanylyl cyclase; GEF, guanine nucleotide exchange actor; LPS, lipopolysaccharide; NE, norepinephrine;
PAR, protease-activated receptor; PLC, phospholipase C; PPAR, peroxisome proli erator-activated receptor.

CASE 1-1
A man who su ers rom hay ever is at his local pharmacy looking or a nonprescrip- DRUG-RECEPTOR
tion drug that can provide relie o symptoms One o the allergy medicines he f nds on INTERACTIONS (key terms
the shel says the active ingredient is an antihistamine, diphenhydramine a re indica ted in ita lics)
a. What is an active ingredient in an allergy medication such as this? Manydrugs act on physiological receptors
Pharmaceuticals contain 1 or more pharmacologically active ingredients, which are and are particularlyselective because
termed active ingredients. T e quantities o each active ingredient in each dose (eg, physiological receptors have evolved to
tablet, capsule, or volume o a liquid medication) are indicated on the packaging. In recognize and respond to individual signal-
addition to active ingredients, most medications also contain other ingredients that are ing molecules with great selectivity.
pharmacologically inactive, but may improve the pharmacokinetics, appearance, taste, Drugs that bind to physiological receptors
shel -li e, or other properties that may enhance dosing and product e ectiveness. and mimicthe regulatorye ects o the
b. What kind o drug is an antihistamine and why is it so named? endogenous signaling compounds are
termed agonists.
As the name indicates, an antihistamine is an agent that antagonizes the action
o the endogenous mediator histamine. Histamine is released by mast cells and is Adrug that binds to the same
involved in a variety o allergic and in ammatory responses (see Chapter 21). T is recognition site as the endogenous
mans hay ever is caused by mast cell release o histamine and other substances that agonist (the primaryor orthosteric
result in his symptoms o sneezing, runny nose, itching o the nose and throat, and site on the receptor) is said to be
itchy, watery eyes. An antihistamine such as diphenhydramine will bind to hista- a primaryagonist.
mine receptors on cells throughout the body and block the e ects o histamine. (continues)
(Continued)
3
SECTION I General Principles

Full a gonis t Ina ctive compound


DRUG-RECEPTOR Pa rtia l a gonis t Inve rs e a gonis t
INTERACTIONS (key terms are L
indicated in italics) (Cont.) 200 LR a

)
Ri

s
t
i
Drugs that can elicit the same maximal

n
u

y
response as the endogenous ligand are

r
L

a
150

r
termed ull agonists (see Figure 1-1) and

t
i
LR i LR a

b
r
are said to possess ull e cacy.

a
(
e
s
Drugs that are onlypartlye ective in 100 L

o
activating a receptor regardless o the

p
LR i LR a

s
e
concentration employed are termed

R
f
50

o
partial agonists (see Figure 1-1).
l
e
L
v
Asyntopicinteraction is an interaction e
L
LR i Ra
between ligands that bind to the same 0
recognition site, or to recognition Log [Drug]
sites that overlap, on the receptor FIGURE 1-1 Regulation o the activity o a receptor with con ormation-selective drugs. The ordi-
macromolecule (see Figure 1-2). nate is the activity o the receptor produced by Ra, the active receptor con ormation (eg, stimu-
Allosteric(allotopic) agonists bind to an lation o adenylyl cyclase by a adrenergic receptor). I a drug Lselectively binds to Ra, it will
allostericor allotopicsite, a region on produce a maximal response. I Lhas equal a nity or Ri and Ra, it will not perturb the equilibrium
the receptor that is di erent romthe between them and will have no e ect on net activity; Lwould appear as an inactive compound.
I the drug selectively binds to Ri, then the net amount o Ra will be diminished. I Lcan bind to
primarysite (see Figure 1-2). receptor in an active con ormation Ra but also bind to inactive receptor Ri with lower a nity, the
Manyreceptors exhibit some constitutive drug will produce a partial response; Lwill be a partial agonist. I there is su cient Ra to produce
activityin the absence o a regulatory an elevated basal response in the absence o ligand (agonist-independent constitutive activity),
ligand; drugs that stabilize such receptors then activity will be inhibited; Lwill then be an inverse agonist. Inverse agonists selectively bind
in an inactive con ormation are termed to the inactive orm o the receptor and shi t the con ormational equilibrium toward the inactive
state. In systems that are not constitutively active, inverse agonists will behave like competitive
inverse agonists (see Figure 1-1). antagonists, which helps explain why the properties o inverse agonists and the number o such
Drugs that blockor reduce the action agents previously described as competitive antagonists were only recently appreciated. Recep-
o an agonist are termed antagonists tors that have constitutive activity and are sensitive to inverse agonists include benzodiazepine,
(see Figure 1-2). histamine, opioid, cannabinoid, dopamine, bradykinin, and adenosine receptors.
Antagonismmost commonlyresults
romcompetition o a drug with an
agonist or the primarysite on the c. What are histamine receptors and what kind o an antagonist is diphenhydramine?
receptor (re erred to as competitive
Histamine receptors are G proteincoupled receptors (GPCRs; see able 1-1). H 1
antagonists because o a syntopic
receptors couple to Gq/11 and activate the PLCIP3Ca2+ pathway and its many pos-
interaction; see Figure 1-2).
sible sequelae, including activation o PKC, Ca2+calmodulindependent enzymes
Noncompetitive antagonismresults rom (eNOS and various protein kinases), and PLA2. H 2 receptors link to Gs to activate
an antagonist that covalentlybinds the the adenylyl cyclasecyclic AMPPKA pathway, whereas H 3 and H 4 receptors
receptor or dissociates extremelyslowly couple to Gi/o to inhibit adenylyl cyclase and decrease cellular cyclic AMP. Activa-
romthe receptor (pseudo-irreversible) tion o H 3 receptors also can activate mitogen-activated protein (MAP) kinase and
such that the maximal response o the inhibit the Na+/H + exchanger, and activation o H 4 receptors mobilizes stored Ca2+
receptor is diminished with increasing in some cells.
concentration o antagonist (see
All the available H 1 receptor antagonists are actually inverse agonists (see Side
Figure 1-2).
Bar DRUG-RECEP OR IN ERAC IONS and Figure 1-1) that reduce constitutive
Noncompetitive antagonismcan also activity o the receptor and compete with histamine. Whereas histamine binding to
occur with antagonists that interact the receptor induces a ully active con ormation, antihistamine binding yields an
with allostericsites on the receptor inactive con ormation (see Figure 1-1). At the tissue level, the e ect seen is propor-
(allostericantagonists), reducing the tional to receptor occupancy by the antihistamine.
a nityo the receptor or agonist.
d. T e warning on the medication package says marked drowsiness may occur. T e
Drugs that act bycombining with the
package label also warns that excitability may occur, especially in children. How

agonist are termed chemical antagonists.


are these side ef ects related to the antihistamine ef ects o the active ingredient?
Functional antagonists act byindirectly
Some antihistamines such as diphenhydramine can cross the blood-brain barrier

inhibiting the cellular or physiological


(BBB) and have e ects on histamine receptors in the central nervous system (CNS).
e ects o the agonist.
T ere is substantial evidence that histamine unctions as a neurotransmitter in
(Continued)

4
Pharmacodynamics CHAPTER 1

the CNS. Histamine, histidine decarboxylase, enzymes that metabolize histamine,


DRUG-RECEPTOR
and H 1, H 2, and H 3 receptors are distributed widely but nonuni ormly in the CNS.
INTERACTIONS (key terms are
Histamine-containing neurons control both homeostatic and higher brain unc-
tions, including regulation o the sleep-wake cycle, circadian and eeding rhythms, indicated in italics) (Cont.)
immunity, learning, memory, drinking, and body temperature. Partial agonists and inverse agonists that
(Continued) compete with a ull agonist at the same
site on a receptor (ie, syntopicinteraction)
A will behave as competitive antagonists.
Co mpe titive Co mpe titive antag o nis m
t
100 Speci cityis a pharmacodynamicpropertyo
A
c
a drug that re ers toa drug's abilitytoelicit a
e
f
f
E
well-de ned response in a speci ctissue.
l

a
3 10
m
50 The chemical structure o a drug
xi

a
Control contributes to its speci city or a given
M
type o physiological receptor.
%
0 Adrug that interacts with a single type
L L L L
o receptor that is expressed on onlya
Log [A]
limited number o di erentiated cells
will exhibit high (narrow) speci city.
B
Ps e udo -Irreve rs ible
No nc o mpe titive
Drugs acting on a type o receptor that
A 100 antag o nis m is expressed ubiquitouslyon a varietyo
t
c
e
cells throughout the bodywill exhibit
f
f
E
widespread e ects, and could produce
l

a
m
50 serious side e ects or toxicities i the
xi
a
M
receptor serves important unctions in
%
multiple tissues.
0
Manyclinicallyimportant drugs exhibit
Log [A]
a low(broad) speci citybecause the
C drug is able to interact with multiple
100
Antag o nis m receptors in di erent tissues; such broad
t
speci citymight enhance the clinical
c
e
f
f
utilityo a drug, but also contribute to a
E
l
a
spectrumo adverse side e ects due to
m
50
xi
of -target interactions.
a
Allo s te ric
M
%
A
P 0
Log [A]

D
100 Po te ntiatio n
t
c
e
f
f
E
l
a
m
50
xi
a
M
%
0
Log [A]

FIGURE 1-2 Mechanisms o receptor antagonism and potentiation. A. Competitive antagonism


occurs when the agonist A and antagonist I compete or the same binding site on the receptor.
Response curves or the agonist are shi ted to the right in a concentration-related manner by
the antagonist such that the EC50 or the agonist increases (eg, L versus L, L, and L) with the
concentration o the antagonist. B. I the antagonist binds to the same site as the agonist but
does so irreversibly or pseudo-irreversibly (slow dissociation but no covalent bond), it causes a
shi t o the dose-response curve to the right, with urther depression o the maximal response.
Allosteric e ects occur when an allosteric ligand I or P binds to a di erent site on the receptor
to either inhibit (I) the response (see panel C) or potentiate (P) the response (see panel D). This
e ect is saturable; inhibition or potentiation reaches a limiting value when the allosteric site is
ully occupied.
5
SECTION I General Principles

e
QUANTITATIVE ASPECTS

s
A B

n
o
100 100
OF DRUG RECEPTOR

p
s
e
INTERACTIONS

R
l
50 50

a
m
Receptor occupancytheoryassumes that

xi
EC 50 EC 50

a
response emanates roma receptor occu-

M
0 0

%
pied bya drug, a concept that has its basis [A] Log [A]
in the lawo mass action. FIGURE 1-3 Graded responses (y axis as a percentage o maximal response) expressed as a
The dose-response (or concentration- unction o the concentration o drug A present at the receptor. The hyperbolic shape o the
response) curve is a depiction o the curve in panel A becomes sigmoid when plotted semi-logarithmically, as in panel B. The con-
observed e ect o a drug as a unction o centration o drug that produces 50% o the maximal response quanti es drug activity and is
its concentration in the receptor compart- re erred to as the EC50 (ef ective concentration or 50% response). The range o concentrations
needed to ully depict the dose-response relationship (~3 log 10 units) is too wide to be use ul in
ment (see Figure 1-3). the linear ormat o Figure 1-3A; thus, most dose-response curves use log [Drug] on the x axis,
The maximal asymptoticresponse as in Figure 1-3B. Dose-response curves presented in this way are sigmoidal in shape and have
occurs when the drug occupies all the 3 properties: threshold, slope, and maximal asymptote. These 3 parameters quantitate the activity
receptor sites. o the drug.
Some drug-receptor systems exhibit
an inverted U-shaped dose-response T e rst-generation H 1 antagonists can both stimulate and depress the CNS.
relationship with low-dose stimulation Stimulation occasionally is encountered in patients given conventional doses; they
and high-dose inhibition o response, become restless, nervous, and unable to sleep. Central excitation also is a strik-
an e ect known as hormesis. ing eature o overdose, which commonly results in convulsions, particularly in
The bimolecular drug-receptor interaction in ants. Central depression, on the other hand, usually accompanies therapeutic
that results in receptor activation can be doses o the older H 1 antagonists. Diminished alertness, slowed reaction times,
described as a series o 2 equilibrium and somnolence are common mani estations. Patients vary in their susceptibility
reactions as shown in Equation 1-1. and responses to individual drugs. T e ethanolamines (eg, diphenhydramine) are
k+1 k+2
particularly prone to causing sedation. T us, the development o second-generation
L+ R LR LR* nonsedating antihistamines was an important advance that allowed their general
k1 k2
(Equation 1-1)
use. T ese newer H 1 antagonists do not cross the blood-brain barrier appreciably.
The initial reaction is the reversible binding T eir sedative ef ects are similar to those o placebo.
o drug (L) toa receptor (R) which leads to
the ormation o the drug-receptor complex e. Are there other pharmacological properties of H1 antagonists that are clinically
(LR), and depends on both the orward or useful?
associationrate (k+1) and the reverse or Many o the rst-generation H 1 antagonists tend to inhibit responses to acetylcho-
dissociationrate (k1). line (ACh) that are mediated by muscarinic receptors and may be mani est during
The equilibriumdissociation constant clinical use (see Chapter 6). Some H 1 antagonists also can be used to treat motion
(KD) is de ned as the ratio o the sickness; the anticholinergic properties o H 1 antagonists may be largely responsible
o and on rate constants (k1/k+1; or this ef ect. Indeed, promethazine (a rst-generation H 1 antagonist) has perhaps
Equation 1-2); the af nityor equilibrium the strongest muscarinic-blocking activity among these agents and is the most
association constant (KA) is the reciprocal ef ective H 1 antagonist in combating motion sickness. T e second-generation H 1
o the equilibriumdissociation constant. antagonists have no ef ect on muscarinic receptors.

[L][R] k1
KD = = CASE 1-2
[LR] k+1 (Equation 1-2)
A pharmaceutical company research and development team is working to develop new
Thus a high-af nitydrug has a low drugs that target a novel molecular entity that might play a role in diabetes. Preclini-
KDand will bind a greater number cal data regarding the pharmacodynamics of a number of small molecules have been
o a particular receptor at a low collected and are being analyzed to determine which lead compounds might be worth
concentrationthana low-af nitydrug. carrying forward into clinical trials in humans.
Depending on the characteristics
a. What are the steps that the team has taken to this point and what must be done
o the drug (eg, ull agonist, partial
before the drug is tested in humans?
agonist, antagonist, inverse agonist; see
Figure 1-1), the drug-receptor complex Modern drug invention usually starts with a statement (or hypothesis) that a cer-
mayundergo a con ormational change tain protein or pathway plays a critical role in the pathogenesis o a certain disease,
that leads to the ormation o the and that altering the proteins activity would there ore be ef ective against that
activated complex(LR*). disease. T e usual approach to invention o a small-molecule drug is to screen
a collection o chemicals (library) or compounds with the desired eatures.
(Continued)
6
Pharmacodynamics CHAPTER 1

An alternative is to synthesize and ocus on close chemical relatives o a substance


QUANTITATIVE ASPECTS
known to participate in a biological reaction o interest (eg, congeners o a speci c
OF DRUG RECEPTOR
enzyme substrate chosen to be possible inhibitors o the enzymatic reaction), a par-
ticularly important strategy in the discovery o anticancer drugs. INTERACTIONS (Cont.)
Several variables a ect the requency o hits obtained in a screen. Among the most The apparent dissociation constant, Kapp,
important are the drugability o the target and the stringency o the screen in is a macroscopicequilibriumconstant
terms o the concentrations o compounds that are tested. T e slang term dru- that refects both the ligand binding
gability re ers to the ease with which the unction o a target can be altered in the equilibriumand the subsequent
desired ashion by a small organic molecule. I the protein target has a well-de ned equilibriumthat results in the ormation
binding site or a small molecule (eg, a catalytic or allosteric site), chances are excel- o the active receptor LR*.
lent that hits will be obtained. I the goal is to employ a small molecule to mimic or The ractional occupancyo a receptor,
disrupt the interaction between 2 proteins, the challenge is much greater. , is described byEquations 1-3 and 1-4;
T e validity o the target must also be con rmed. Modern techniques o molecular at a concentration o drug that results
biology o er new and power ul tools or validation o potential drug targets, to the in 50%occupancyo the receptor
extent that the biology o model systems resembles human biology. Genes can be ( = 0.5), the concentration o drug
inserted, disrupted, and altered in mice. One can thereby create models o disease equals the KD.
in animals or mimic the e ects o long-term disruption or activation o a given bio-
[ligand-receptor complexes]
logical process. I , or example, disruption o the gene encoding a speci c enzyme f=
or receptor has a bene cial e ect in a valid murine model o a human disease, one [total receptors]
may believe that the potential drug target has been validated. Mutations in humans [LR]
=
can also provide extraordinarily valuable in ormation. [R] + [LR] (Equation 1-3)
Following the path just described can yield a potential drug molecule that interacts
KA[L] [L]
with a validated target and alters its unction in the desired ashion (either enhanc- f= =
ing or inhibiting the unctions o the target). Now one must consider all aspects o 1+ KA[L] [L] + KD
the molecule in questionits a nity and selectivity or interaction with the target, its (Equation 1-4)
pharmacokinetic properties (absorption, distribution, excretion, metabolism), issues The concentration o an agonist drug
with regard to its large-scale synthesis or puri cation rom a natural source, its phar- that produces 50%o the maximal
maceutical properties (stability, solubility, questions o ormulation), and its sa ety. response is termed the EC50 (the hal -
(Continued) maximallyef ective concentration; see
Figure 1-3).
The abilityo a drug to activate a receptor
A Re la tive pote ncy
100 and generate a cellular response is a refec-
tion o its e cacy; relative e cacyre ers
80 Drug X Drug Y to a comparison o the maximal response
60
o 2 agonists; the more e cacious drug
40 exhibits a higher maximal response
EC 50 EC 50
20 (see Figure 1-4).
t
c
0 A ull agonist has ull e cacy; a partial
e

f
f
E
Log [Agonis t] agonist has partial e cacy.
l
a
m
Drug potencyre ers to a comparison o the
xi
a
concentrations o agonist drug required to
M
B Re la tive e ffica cy
%
elicit a given biological response and is a
100 Drug X mixed unction o both a nityand e cacy.
80
Incomparingthe EC50 o 2drugswiththe
60
Drug Y
same e cacy, the drugwiththe lower EC50
40 isthe more potent drug(see Figure 1-4).
20
The a nityo a competitive antagonist (Ki)
0 or its receptor can be determined in radio-
Log [Agonis t] ligand binding assays or bymeasuring the
FIGURE 1-4 Two ways o quanti ying agonism. A. The relative potency o 2 agonists (Drug X, unctional response o a systemtoa drug in
blue line; Drug Y, black line) obtained in the same tissue is a unction o their relative a nities and the presence o the antagonist (see text
intrinsic e cacies. The EC50 o Drug X occurs at a concentration that is one-tenth the EC50 o Drug and Equations 3-3 and 3-4in Chapter 3 o
Y. Thus, Drug X is more potent than Drug Y. B. In systems where the 2 drugs do not both produce Goodman &Gilmans The Pharmacological
the same maximal response characteristic o the tissue, the observed maximal response is a non- Basis o Therapeutics, 12th Edition).
linear unction o their relative intrinsic e cacies. Drug X is more e cacious than Drug Y; their
asymptotic ractional responses are 100% (Drug X) and 50% (Drug Y).

7
SECTION I General Principles

Be ore being administered to people, potential drugs are tested or general toxicity
by monitoring the activity o various systems in 2 species o animals or extended
periods o time. Compounds also are evaluated or carcinogenicity, genotoxicity,
and reproductive toxicity. Animals are used or much o this testing, although the
predictive value o results obtained in nonhuman species is certainly not per ect.
Usually 1 rodent (usually mouse) and 1 nonrodent (of en rabbit) species are used.
In vitro and ex vivo assays are utilized when possible, both to spare animals and to
minimize cost. I an unwanted e ect is observed, an obvious question is whether it
is mechanism-based (ie, caused by interaction o the drug with its intended target)
or due to an o -target e ect o the drug. I the latter, there is hope o minimizing
the e ect by urther optimization o the molecule.
Be ore clinical trials o a potential new drug may proceed in the United States (ie,
be ore the drug candidate can be administered to a human subject), the sponsor
must le an IND (Investigational New Drug) application, which is a request to the
United States. Food and Drug Administration (FDA) or permission to administer
the drug to human test subjects. T e IND describes the rationale and preliminary
evidence or e cacy in experimental systems, as well as pharmacology, toxicology,
chemistry, manu acturing, and so orth. It also describes the plan or investigating
the drug in human subjects (see Chapter 3).
b. In vitro binding studies show that KD values o the drugs that are being consid-
ered or clinical studies range rom 0.9 nM to 0.4 mM. What does a KD measure
and why is that value important to know?
In general, the drug-receptor interaction is characterized by (1) binding o drug to
receptor and (2) generation o a response in a biological system, as illustrated in
Equation 1-1 where the drug or ligand is denoted as L and the inactive receptor as
R. T e equilibrium dissociation constant (KD) is then described by ratio o the o
and on rate constants (k1/k+1) when L and R are at equilibrium with LR.
T e a nity constant or equilibrium association constant (KA) is the reciprocal o
the equilibrium dissociation constant (ie, KA = 1/KD); thus a high-a nity drug has
a low KD and will bind a greater number o a particular receptor at a low concentra-
tion than a low-a nity drug. As a practical matter, the a nity o a drug is in u-
enced most of en by changes in its o -rate (k- 1) rather than its on-rate (k+1).
Equation 1-3 shows the relationship o the ractional occupancy (f) o receptors
by agonist L in terms o the concentration o L (ie, [L]), and either KD (or KA). T is
relationship illustrates that when the concentration o the drug equals the KD (or
1/KA), f = 0.5, that is, the drug will occupy 50% o the receptors. Note that this rela-
tionship describes only receptor occupancy, not the eventual response that is of en
ampli ed by the cell. Many signaling systems reach a ull biological response with
only a raction o receptors occupied (a phenomenon that might occur in a system
with spare receptors).
c. Other preclinical studies were conducted with the target protein expressed in
cultured cells. T ese studies were used to determine dose-response curves or
each lead compound. What is a dose-response curve and what in ormation can it
provide regarding the drug-receptor interaction?
T e basic currency o receptor pharmacology is the dose-response (or concentration-
response) curve, a depiction o the observed e ect o a drug as a unction o its
concentration in the receptor compartment (see Figures 1-1 through 1-4). Drugs
that bind to physiological receptors and mimic the regulatory e ects o the endog-
enous signaling compounds are termed agonists. I the drug binds to the same
recognition site as the endogenous agonist (the primary or orthosteric site on the
receptor), the drug is said to be a primary agonist. Allosteric (allotopic) agonists
bind to a di erent region on the receptor re erred to as an allosteric or allotopic
site. Drugs that block or reduce the action o an agonist are termed antagonists.
Antagonism most commonly results rom competition with an agonist or the same
or overlapping site on the receptor (a syntopic interaction), but can also occur by
(Continued)
8
Pharmacodynamics CHAPTER 1

interacting with other sites on the receptor (allosteric antagonism), by combining


with the agonist (chemical antagonism), or by unctional antagonism by indirectly
inhibiting the cellular or physiological e ects o the agonist. Agents that are only
partly as e ective as agonists regardless o the concentration employed are termed
partial agonists and are said to have lower e cacy than ull agonists (see Figure 1-4,
panel B). Many receptors exhibit some constitutive activity in the absence o a regu-
latory ligand; drugs that stabilize such receptors in an inactive con ormation are
termed inverse agonists (see bottom-most curve in Figure 1-1).
In addition to determining whether a ligand is an agonist or antagonist, dose-
response curves as described here complement the in vitro binding studies
described in part b previously and provide estimates o drug concentrations that
might be appropriate or studies in animals and human subjects. In addition to
estimating relative e cacy, the relative potency o agonist drugs can be estimated
by comparison o EC50 values, the concentration o drug needed to obtain a hal -
maximal (ie, 50% o maximal) response. Dose response curves o antagonists gen-
erated in the presence o xed concentrations o agonist can be used to estimate the
a nity o a competitive antagonist or its receptor, the Ki (see text and Equations
3-3 and 3-4 in Chapter 3 o Goodman & Gilmans T e Pharmacological Basis of
T erapeutics, 12th Edition).
d. Studies by this research group in rodent models showed that some o the lead
compounds had unexpected side ef ects, including serious toxicities. What can
cause such unexpected ef ects in a whole animal model?
T e chemical structure o a drug contributes to its a nity, intrinsic activity, as well
as its speci city. A drug that interacts with a single type o receptor that is expressed
on only a limited number o di erentiated cells will exhibit high speci city. I how-
ever, a receptor is expressed ubiquitously on a variety o cells throughout the body,
drugs acting on such a widely expressed receptor will exhibit widespread e ects,
and could produce serious side e ects or toxicities i the receptor serves important
unctions in multiple tissues. Even i the primary action o a drug is localized, the
subsequent physiological e ects o the drug may be widespread. One example would
be immunosuppressant drugs (see Chapter 23) that speci cally inhibit cells o the
immune system; their use is limited by the risk o opportunistic systemic in ections.
Many clinically important drugs exhibit a broad (low) speci city because the drug
is able to interact with multiple receptors in di erent tissues. Such broad speci c-
ity might enhance the clinical utility o a drug, but also contribute to a spectrum
o adverse side e ects due to o -target interactions. It should always be considered
that a given drug has multiple mechanisms o action that depend on many actors,
including receptor speci city, the tissue-speci c expression o the receptor(s), drug
access to target tissues, drug concentration in di erent tissues, pharmacogenetics
(see able 1-2), and interactions with other drugs (eg, see Chapters 2 to 7 in Good-
man & Gilmans T e Pharmacological Basis of T erapeutics, 12th Edition).

CASE 1-3
Following the FDA approval o a new short-acting hypnotic drug to enhance sleep ol-
lowing pain ul surgical procedures, postmarketing surveillance shows that toxic side
e ects are relatively common among elderly patients, but uncommon in younger adults
a. What is postmarketing surveillance?
No drug is totally sa e; all drugs produce unwanted e ects in at least some people
at some dose. Many unwanted and serious e ects o drugs occur so in requently,
perhaps only once in several thousand patients, that they go undetected in the
relatively small populations (a ew thousand) in the standard Phase III clinical trial
(see able 1-1 in Goodman & Gilmans T e Pharmacological Basis of T erapeutics,
12th Edition). o detect and veri y that such events are in act drug-related would
require administration o the drug to tens or hundreds o thousands o people
(Continued)
9
SECTION I General Principles

TABLE 1-2 Examples o Genetic Polymorphisms Inf uencing Drug Response


GENE PRODUCT GENE DRUGSa RESPONSES AFFECTED
Targets and Receptors

Angiotensin-converting ACE inhibitors (eg, enalapril) Renoprotective e ects, hypotension, le t ventricular


enzyme (ACE) mass reduction, cough

Thymidylate synthase 5-Fluorouracil Colorectal cancer response

Chemokine receptor 5 (CCR5) Antiretrovirals, inter eron Antiviral response

2 Adrenergic receptor (ADBR2) 2 Antagonists (eg, albuterol, terbutaline) Bronchodilation, susceptibility to agonist-induced
desensitization, cardiovascular e ects (eg, increased heart
rate, cardiac index, peripheral vasodilation)

1 Adrenergic receptor (ADBR1) 1 Antagonists Blood pressure and heart rate a ter 1 antagonists

5-Lipoxygenase (ALOX5) Leukotriene receptor antagonists Asthma response

Dopamine receptors D2, D3, D4 Antipsychotics (eg, haloperidol, clozapine, Antipsychotic response (D2, D3, D4), antipsychotic-
thioridazine, nemonapride) induced tardive dyskinesia (D3) and acute akathisia (D3),
hyperprolactinemia in emales (D2)

Estrogen receptor Estrogen hormone replacement therapy High-density lipoprotein cholesterol

Serotonin transporter 5-HTT (SLC6A4) Antidepressants (eg, clomipramine, Clozapine e ects, 5-HT neurotransmission,
f uoxetine, paroxetine, f uvoxamine) antidepressant response

Serotonin receptor 5-HT2A(HTR2A) Antipsychotics Clozapine antipsychotic response, tardive dyskinesia,


paroxetine antidepression response, drug discrimination

HMG-CoA reductase Pravastatin Reduction in serum cholesterol

Vitamin Koxidoreductase VKORC1 War arin a Anticoagulant e ect, bleeding risk

Corticotropin releasing hormone Glucocorticoids Bronchodilation, osteopenia


receptor (CRHR1)

Ryanodine receptor (RYR1) General anesthetics Malignant hyperthermia

Modif ers

Adducin Diuretics Myocardial in arction or strokes, blood pressure

Apolipoprotein E Statins (eg, simvastatin), tacrine Lipid-lowering; clinical improvement in Alzheimer disease

Human leukocyte antigen Abacavir, carbamazepine, phenytoin Hypersensitivity reactions

G6PD de ciency Rasburicase a, dapsone a Methemoglobinemia

Cholesteryl ester trans er protein Statins (eg, pravastatin) Slowing atherosclerosis progression

Ion channels (HERG, KvLQT1, Erythromycin, cisapride, clarithromycin, Increased risk o drug-induced torsade de pointes,
Mink, MiRP1) quinidine increased QT interval (Roden, 2003; Roden, 2004)

Methylguanine-methyltrans erase DNA methylating agents Response o glioma to chemotherapy

Parkin Levodopa Parkinson disease response

MTHFR Methotrexate GI toxicity (Ulrich et al, 2001)

Prothrombin, actor V Oral contraceptives Venous thrombosis risk

Stromelysin-1 Statins (eg, pravastatin) Reduction in cardiovascular events and in repeat angioplasty

Inosine triphosphatase (ITPA) Azathioprine, mercaptopurine Myelosuppression

Vitamin D receptor Estrogen Bone mineral density


a
In ormation on genetics-based dosing, adverse events, or testing added to FDA-approved drug label (Grossman, 2007).

10
Pharmacodynamics CHAPTER 1

during clinical trials, adding enormous expense and time to drug development, and
delaying access to potentially bene cial therapies. In general, the true spectrum
and incidence o untoward e ects becomes known only af er a drug is released to
the broader market and used by a large number o people (Phase IV, postmarketing
surveillance; see Chapter 3, Case 3-2).
b. T e preclinical trials o this drug showed a therapeutic index o 25. What is the
therapeutic index and how is it calculated?
In preclinical studies o drugs, the median lethal dose (LD50) is determined in
experimental animals (see Figure 1-5, panel B). T e LD50/ED50 ratio is an indication
o the therapeutic index, which is a quantitative statement o how selective the drug
is in producing its desired e ects versus its adverse e ects. A therapeutic index o
25 indicates that the concentration o drug that is lethal in 50% o animals (LD50) is
25- old higher than the dose required to achieve a speci c therapeutic e ect in 50%
o the population tested (the median e ective dose, ED50).
A similar term to therapeutic index, the therapeutic window, is the range o steady-
state concentrations o drug that provides therapeutic e cacy with minimal toxic-
ity (see Figure 1-6). In clinical studies, the dose, or pre erably the concentration, o
a drug required to produce toxic e ects can be compared with the concentration
required or therapeutic e ects in the population to evaluate the clinical therapeutic
index. Since pharmacodynamic variation in the population may be marked, the
concentration or dose o drug required to produce a therapeutic e ect in most o
the population usually will overlap the concentration required to produce toxicity
in some o the population, even though the drugs therapeutic index in an indi-
vidual patient may be large. Also, the concentration-percent curves or e cacy and
toxicity need not be parallel, adding yet another complexity to determination o
the therapeutic index in patients. Finally, no drug produces a single e ect, and the
therapeutic index or a drug will vary depending on the e ect being measured.
(Continued)

The ra pe utic LD50 400


= =4
Index: ED50 100

100 A 100 B
g
g
Hypnos is De a th
n
n
Cumula tive
i
i
d
d
n
n
80 fre que ncy 80
o
o
p
p
dis tribution
s
s
e
e
R
R
s
s
l
l
60 60
a
a
u
u
d
d
vi
vi
i
i
d
d
n
n
I
I
40 40
f
f
Fre que ncy
o
o
e
e
dis tribution
g
g
a
a
t
t
ED50 ED50 ED99
n
n
20 20
e
e
c
c
r
r
LD1 LD50
e
e
P
P
0 0
5 7 10 20 50 100 200 400 800
Conce ntra tion (mg/L) Dos e (mg/kg)

FIGURE 1-5 Frequency distribution curves and quantal concentration-e ect and dose-e ect
curves. A. Frequency distribution curves An experiment was per ormed on 100 subjects, and the
e ective plasma concentration that produced a quantal response was determined or each
individual. The number o subjects who required each dose is plotted, giving a log-normal re-
quency distribution (blue bars). The gray bars demonstrate that the normal requency distribu-
tion, when summated, yields the cumulative requency distributiona sigmoidal curve that is
a quantal concentration-e ect curve. B. Quantal dose-ef ect curves. Animals were injected with
varying doses o a drug and the responses were determined and plotted. The calculation o the
therapeutic index, the ratio o the LD50 to the ED50, is an indication o how selective a drug is
in producing its desired e ects relative to its toxicity. See Chapter 3 in Goodman &Gilmans The
Pharmacological Basis o Therapeutics, 12th Edition or additional explanation.
11
SECTION I General Principles

The ra pe utic

g
window

n
i
d
n
100

o
p
s
e
R
s
t
n
The ra pe utic

e
i
t
a
re s pons e

P
50

f
o
e
g
a
Adve rs e

t
n
e
e ffe cts

c
r
e
P
1 2 3 4 6
Conce ntra tion of drug in pla s ma (ng/mL)
FIGURE 1-6 The relation o the therapeutic window o drug concentrations to the therapeutic
and adverse e ects in the population. The ordinate is linear; the abscissa is logarithmic.

c. Why might this toxicity primarily be seen in elderly patients in Phase IV trials?
Data on the correlation o drug levels with e cacy and toxicity must be interpreted
in the context o the pharmacodynamic variability in the population (eg, genetics,
age, disease, and the presence o coadministered drugs), as well as pharmacokinetic
variables. Determinants o inter-individual variation in response to drugs that are
due to pharmacokinetics include disease-related alterations such as impaired renal
and liver clearance due to renal and hepatic disease, circulatory ailure, altered drug
binding to plasma proteins, impaired GI absorption, and pharmacokinetic drug
interactions. T e e ects o these actors on variability o drug pharmacokinetics are
described more thoroughly in Chapters 2 and 5 to 7 in Goodman & Gilmans T e
Pharmacological Basis of T erapeutics, 12th Edition.
I the Phase III clinical trials did not include large numbers o elderly patients,
toxicities in this patient population might not be apparent until postmarketing sur-
veillance has indicated an increased risk o toxicities in the elderly. Elderly patients
may be more susceptible to toxicities or a variety o reasons, including age-related
decreases in cardiac, renal and hepatic unction, disease comorbidities, and poor
nutritional status (see Chapter 3).
T ere is a quantal concentration-response curve or e cacy and adverse e ects
(see Figure 1-5B); or many drugs, the concentration that achieves e cacy in all the
population may produce adverse e ects in some individuals. T us, a population
therapeutic window expresses a range o concentrations at which the likelihood o
e cacy is high and the probability o adverse e ects is low (see Figure 1-6), but it
does not guarantee either e cacy or sa ety. T ere ore, use o the population thera-
peutic window to adjust dosage o a drug should be complemented by monitoring
appropriate clinical and surrogate markers or drug e ect(s).

CASE 1-4
A 64-year-old male patient diagnosed with atrial f brillation is started on anticoagulant
therapy with war arin to lower his risk o stroke due to f brin clots Despite calculating
initial war arin dosing based on the patients age, weight, and sex, laboratory monitoring
o anticoagulant activity a er the f rst week o therapy shows that the patient is receiving
too much war arin (his INR was too high indicating he is over anti-coagulated) A er
several weeks o dose adjustment, the patients anticoagulant therapy is optimized at a
war arin concentration that is 10- old lower than the initial dose
a. What actors might have contributed to this patients high sensitivity to war arin?
Both pharmacokinetic and pharmacodynamic polymorphisms a ect war arin dosing
(see Chapter 19). T e anticoagulant war arin is catabolized by CYP2C9, and war arins
(Continued)
12
Pharmacodynamics CHAPTER 1

action is partly dependent on the baseline level o reduced vitamin K (catalyzed by


vitamin K epoxide reductase; Figure 1-7). Inactivating polymorphisms in CYP2C9
are common, with 2 to 10% o most populations being homozygous or low-activity
variants, and these polymorphisms are associated with lower war arin clearance, a
higher risk o bleeding complications, and lower dose requirements (see able 30-2 in
Goodman & Gilmans T e Pharmacological Basis of T erapeutics, 12th Edition). Com-
bined with genotyping or a common polymorphism in VKORC1, inherited variation
in these 2 genes account or 20 to 60% o the variability in war arin doses needed to
achieve the desired INR (see Figure 1-7), and use o these tests in the clinic can result
in ewer bleeding complications and a shorter time o trial-and-error dosing to achieve
the desired steady-state level o anticoagulation.
T e target o war arin is VKORC1, an enzyme that reduces vitamin K epoxide to
vitamin K hydroquinone (see Figure 1-7 and able 1-2; Chapter 19). War arin inhi-
bition o VKORC1 blocks the reduction o vitamin K epoxide, which inhibits the
synthesis o a number o vitamin K-dependent clotting actors. Several genetic vari-
ations in VKORC1 are in strong linkage disequilibrium and have been designated
haplotypes A and B (or non-A). VKORC1 variants are more prevalent than those o
CYP2C9. T e prevalence o VKORC1 genetic variants is higher in Asians, ollowed
by European Americans and A rican Americans.
Polymorphism in VKORC1 explains ~30% o the variability in war arin dose require-
ments. Compared with VKORC1 non-A/non-A homozygotes, the war arin dose
requirement is decreased by ~25% in heterozygotes and ~50% in A/A homozygotes.
T e clinical relevance o these genetic polymorphisms remains uncertain. T e goal
o war arin therapy is to maintain a patient within a target INR range, most of en
an INR value between 2 and 3. T e risk o serious bleeding increases with INR val-
ues more than 4 and is highest during initiation o war arin therapy. Variations in
VKORC1 have a greater e ect than CYP2C9 variants on war arin responses early in
therapy. Patients with VKORC1 haplotype A have signi cantly higher INR values
in the rst week o war arin therapy than non-A homozygotes; those with 1 or 2
VKORC1 haplotype A alleles achieve a therapeutic INR more rapidly and are more
likely to have an INR o 4 or greater than patients with 2 non-A alleles. Both the
VKORC1 haplotype and CYP2C9 genotype have a signi cant e ect on the war arin
dose af er the rst 2 weeks o therapy.
(Continued)

5
CYP 2C9
wa rfa rin hydroxywa rfa rin
)
g
m
(
e
s
o
D
VKORC1
g
n
i
t
r
vita min K
a
vita min K
t
e poxide
S
d
e
d
n
e
m
m
o
c
e
R
prothrombotic a ntithrombotic
(a ctiva te d clotting (hypofunctiona l clotting
fa ctors ) fa ctors ) 0
CYP 2C9 AA AB BB AA AB BB AA AB BB
ge notype
VKORC1 homozygous he te ro- homozygous
va ria nt zygous common
ge notype BB AB AA
FIGURE 1-7 Pharmacogenetics o war arin dosing. War arin is metabolized by CYP2C9 to inactive
metabolites, and exerts its anticoagulant e ect partly via inhibition o VKORC1 (vitamin Kepoxide
hydrolase), an enzyme necessary or reduction o inactive to active vitamin K. Common polymor-
phisms in both genes, CYP2C9 and VKORC1, impact on war arin pharmacokinetics and pharmaco-
dynamics, respectively, to a ect the population mean therapeutic doses o war arin necessary to
maintain the desired degree o anticoagulation (o ten measured by the international normalized
ratio [INR] blood test) and minimize the risk o too little anticoagulation (thrombosis) or too much
anticoagulation (bleeding).
13
SECTION I General Principles

Based on evidence that genetic variations a ect war arin dose requirements and
responses to therapy, the FDA amended the prescribing in ormation or war arin in
2007 to indicate that lower war arin initiation doses be considered or patients with
CYP2C9 and VKORC1 genetic variations. E orts to acilitate the rational incorpo-
ration o genetic in ormation into patient care have included the development o a
war arin dosing algorithm and point-o -care methods or CYP2C9 and VKORC1
genotyping. In a 2009 study o more than 4000 patients, the International War arin
Pharmacogenetics Consortium compared the accuracy o a pharmacogenetic algo-
rithm that included VKORC1 and CYP2C9 genotypes with 2 conventional clinical
approaches: one based on clinical in ormation to adjust the initial dose, and the
other using a xed-dose approach. T e pharmacogenetic algorithm predicted the
war arin dose signi cantly better than the other 2 approaches. Moreover, the phar-
macogenetic algorithm signi cantly improved the dose prediction or patients who
required either high or low doses o war arin (<21 mg/wk or >49 mg/wk).
Vitamin K is required or synthesis o unctional clotting actors by serving as a
co actor (see Figure 1-7; Chapter 19). Relative de ciency o vitamin K may increase
the sensitivity o patients to war arin and may result rom inadequate diet (eg, post-
operative patients on parenteral uids), especially when coupled with the elimina-
tion o intestinal ora by antimicrobial agents. Gut bacteria synthesize vitamin K
and are an important source o this vitamin. Consequently, antibiotics can cause
a high INR in patients who were previously adequately controlled on war arin. In
addition to an e ect on reducing intestinal ora, cephalosporins containing het-
erocyclic side chains also inhibit steps in the vitamin K cycle. Low concentrations
o coagulation actors may result rom impaired hepatic unction, congestive heart
ailure, or hypermetabolic states, such as hyperthyroidism; generally, these condi-
tions increase the INR.
Frequently cited drug interactions that increase INR and enhance the risk o hem-
orrhage in patients taking war arin include decreased metabolism due to CYP2C9
inhibition by amiodarone, azole anti ungals, cimetidine, clopidogrel, cotrimoxa-
zole, disul ram, uoxetine, isoniazid, metronidazole, sul npyrazone, tolcapone, or
za rlukast, and displacement rom protein binding sites caused by loop diuretics
or valproate.

CASE 1-5
A 42-year-old man is diagnosed with chronic myelogenous leukemia (CML) He is
prescribed imatinib, a small molecule anticancer drug that targets a disease-causing
enzyme in leukemic cells
a. What is the molecular target o imatinib in CML?
T e molecular target o imatinib in CML is the abnormal protein tyrosine kinase
BCR-ABL, which is caused by a chromosomal translocation (see Chapter 46). BCR-
ABL is constitutively active which causes the uncontrolled proli eration o leuke-
mic cells. Imatinib binds at the active site o the kinase and blocks its activity, thus
blocking cell proli eration.
Imatinib mesylate was the rst molecularly targeted protein kinase inhibitor to
receive FDA approval. Imatinib was identi ed through high-throughput screening
against the BCR-ABL kinase. T e lead compound o this series, a 2-phenylamino-
pyrimidine, had low potency and poor speci city, inhibiting both serine/threonine
and tyrosine kinases. T e addition o a 3-pyridyl group at the 3 position o the
pyrimidine enhanced its potency. Further modi cations resulted in improved activ-
ity against PDGFR and c-KI (2 other disease-causing tyrosine kinases) and loss o
serine/threonine kinase inhibition. Introduction o N-methylpiperazine as a polar
side chain greatly improved water solubility and oral bioavailability, yielding ima-
tinib, an inhibitor o the closed, or inactive, con guration o the kinase.
(Continued)

14
Pharmacodynamics CHAPTER 1

b. T e patients cancer goes into remission or 24 months, but then comes back,
despite daily doses o imatinib. What is the likely mechanism o resistance to
imatinib that has developed in this patient?
Resistance to imatinib and other small molecule tyrosine kinase inhibitors arises
rom point mutations in 3 separate segments o the kinase domain. T e contact
points between imatinib and the enzyme become sites o mutations in drug-
resistant leukemic cells; these mutations prevent tight binding o the drug and
lock the enzyme in its open con guration, in which it has access to substrate. Such
mutations hold the enzyme in its open and enzymatically active con rmation. T e
most common resistance mutations a ect amino acids 255 and 315, both o which
serve as contact points or imatinib; these mutations con er high-level resistance to
imatinib. Some mutations, such as at amino acids 351 and 355, con er low levels o
resistance to imatinib.
c. What is the therapeutic strategy to overcome resistance to imatinib in this patient?
T e strategy depends on the sites o mutation in BCR-ABL that lead to the resis-
tance. Mutations at amino acids 351 and 355 might lead to a clinical response with
dose escalation o imatinib. Mutations at other amino acids that con er high resis-
tance to imatinib will require using a di erent tyrosine kinase inhibitor such as
nilotinib or dasatinib. Dasatinib is una ected by mutation at 255 but is ine ective
in the presence o mutation at 315. Nilotinib retains inhibitory activity in the pres-
ence o most point mutations (except at 315) that con er resistance to imatinib.

KEY CONCEPTS
Pharmacodynamics re ers to the e ects o a drug on the body
Most drugs act by binding to physiological receptors and alter the rate or mag-
nitude o an intrinsic cellular response rather than create new responses
Important pharmacodynamic properties o a drug include its a nity, e cacy,
and specif city
T e e ects o a drug depend on many pharmacodynamic actors, including
receptor specif city, the tissue-specif c expression o the receptor(s), drug con-
centration in the receptor compartment on target tissues, drug concentrations
in di erent tissues expressing the receptor, pharmacogenetics, and interactions
with other drugs at the receptor
No drug is totally sa e; all drugs produce unwanted e ects in at least some
people at some dose
T e concentration or dose o drug required to produce a therapeutic e ect in
most o the population usually will overlap the concentration required to pro-
duce toxicity in some o the population

SUMMARY QUIZ

QUESTION 1-1 Pindolol and some other adrenergic receptor antagonists have an
additional property that is re erred to as intrinsic sympathomimetic activity (ISA) T is
additional property indicates these agents are
a ull agonists
b inverse agonists
c partial agonists
d neutral antagonists
e noncompetitive antagonists

15
SECTION I General Principles

QUESTION 1-2 A 28-year-old woman is brought to the emergency department a er


overdosing on oxycodone, an opiate analgesic She is unconscious and barely breathing
when she arrives at the hospital, but revives and is breathing normally within 2 minutes
a er receiving an injection o naloxone A er about an hour, she requires another dose
o naloxone as the symptoms o opiate overdose begin to redevelop Naloxone is likely
acting as a
a strong stimulant that counteracts the CNS-depressant e ects o the opiate
b diuretic that increases renal excretion o the opiate
c drug that stimulates respiratory centers in the brain
d competitive antagonist o opioid receptors
e noncompetitive antagonist o opioid receptors

QUESTION 1-3 T e equilibrium dissociation constant (KD) is a measure o a drugs


a e cacy
b a nity or its receptor
c specif city
d sa ety
e solubility in water

QUESTION 1-4 T e therapeutic window is an indication o the


a length o time a drug should be administered to achieve optimal e ects
b time o day when a drug should be administered or optimal e ects
c range o concentrations over which a drug is maximally e ective or all patients
d range o concentrations over which a drug is sa e and e cacious or most patients
e age range o patients that is optimal or a given drug

QUESTION 1-5 Preclinical characterization o a new drug indicates that it is an inverse


agonist or adenosine receptors It interacts syntopically with adenosine In the presence
o adenosine, this drug will behave like a(n)
a ull agonist with additive e ects to adenosine at high concentrations
b partial agonist with additive e ects to adenosine at high concentrations
c partial agonist with inhibitory e ects to adenosine at high concentrations
d inert compound with no additive or inhibitory e ects to adenosine at high
concentrations
e competitive antagonist with inhibitory e ects on receptor activation

SUMMARY QUIZ ANSWER KEY


QUESTION 1-1 Answer is c. Several blockers (eg, pindolol, acebutolol, bucindolol)
activate receptors partially in the absence o catecholamines; however, the intrinsic
activities o these drugs are less than that o a ull agonist such as isoproterenol,
epinephrine, or norepinephrine T us these are partial agonists and are said to have
intrinsic sympathomimetic activity Substantial sympathomimetic activity would be
counterproductive to the response desired rom a antagonist; however, slight residual
activity may, or example, prevent pro ound bradycardia or negative inotropy in a resting
heart T e potential clinical advantage o this property, however, is unclear and may be
disadvantageous in the context o secondary prevention o myocardial in arction
Although only limited data are available, blockers with slight partial agonist
activity may produce smaller reductions in resting heart rate and blood pressure
Hence, such drugs may be pre erred as antihypertensive agents in individuals with
diminished cardiac reserve or a propensity or bradycardia

16
Pharmacodynamics CHAPTER 1

Nonetheless, the clinical signif cance o partial agonism has not been substantially
demonstrated in controlled trials but may be o importance in individual patients
Agents such as pindolol block exercise-induced increases in heart rate and cardiac
output (see Chapters 15 and 16)
QUESTION 1-2 Answer is d. Naloxone is a competitive antagonist o various
opioid receptors (see Chapter 10) Opioid antagonists, particularly naloxone, have
an established use in the treatment o opioid-induced toxicity, especially respiratory
depression Its specif city is such that reversal by this agent is virtually diagnostic or
the contribution o an opiate to the depression Naloxone acts rapidly to reverse the
respiratory depression associated with high doses o opioids
It should be used cautiously because it also can precipitate withdrawal in dependent
subjects and cause undesirable cardiovascular side e ects By care ully titrating
the dose o naloxone, it usually is possible to rapidly antagonize the respiratory-
depressant actions without eliciting a ully expressed withdrawal syndrome T e
duration o action o naloxone is relatively short, and it o en must be given repeatedly
or by continuous in usion
Under ordinary circumstances, opioid antagonists such as naloxone produce ew
e ects in the absence o an exogenous agonist However, under certain conditions
(eg, shock), when the endogenous opioid systems are activated, the administration o
an opioid antagonist alone may have visible consequences
QUESTION 1-3 Answer is b. T e equilibrium dissociation constant (KD) is a measure
o a drugs a nity or its receptor It is the reciprocal o the a nity constant or equilib-
rium association constant (KA; ie, KA = 1/KD) T us a high-a nity drug has a low KD
and will bind a greater number o a particular receptor at a low concentration than a
low-a nity drug
QUESTION 1-4 Answer is d. A population therapeutic window expresses a range o
concentrations at which the likelihood o e cacy is high and the probability o adverse
e ects is low (see Figure 1-6) It does not guarantee either e cacy or sa ety T ere ore,
use o the population therapeutic window to adjust dosage o a drug should be comple-
mented by monitoring appropriate clinical and surrogate markers or drug e ect(s)
QUESTION 1-5 Answer is e. Many receptors exhibit some constitutive activity in the
absence o a regulatory ligand Inverse agonists selectively bind to the inactive orm
o such receptors and shi the con ormational equilibrium toward the inactive state
(see Figure 1-1) Inverse agonists typically bind such receptors at the same site as the
endogenous agonist; ligands that interact syntopically with a ull agonist will behave as
competitive antagonists
T e characteristic pattern o competitive antagonism is the concentration-
dependent production o a parallel shi to the right o the agonist dose-response
curve with no change in the maximal response (see Figure 1-2A) T e magnitude o
the rightward shi o the curve depends on the concentration o the antagonist and its
a nity or the receptor
In systems that are not constitutively active, inverse agonists will behave like
competitive antagonists, which helps explain why the properties o inverse agonists
and the number o such agents previously described as competitive antagonists were
only recently appreciated Receptors that have constitutive activity and are sensitive to
inverse agonists include benzodiazepine, histamine, opioid, cannabinoid, dopamine,
bradykinin, and adenosine receptors

17
CHAPTER

2 Pharmacokinetics
T is chapter will be most use ul a er having a basic understanding o the material in
Chapter 2, Pharmacokinetics in Goodman & Gilmans T e Pharmacological Basis of
T erapeutics, 12th Edition. T is chapter also draws rom content in Chapter 5, Mem-
brane ransporters and Drug Response; Chapter 6, Drug Metabolism; and parts o
Chapter 7, Pharmacogenetics. T e drugs presented in this chapter are used to illustrate
general pharmacokinetic principles. T e pharmacokinetic, pharmacodynamic, and
therapeutic uses o drugs described in Chapter 2 are discussed in more detail in subse-
quent chapters. Neither a Mechanisms o Action able nor a Clinical Summary able is
included in this chapter because this in ormation is provided in subsequent chapters.
In addition to the material presented here, the 12th Edition contains:
A review in Chapter 2 o cell membranes and their physicochemical properties that
a ect the movement o drugs to their site o action
A discussion in Chapter 2 o di erent routes o drug administration and their rela-
tive advantages and disadvantages
A detailed discussion in Chapter 2 o renal drug excretion, biliary and ecal excre-
tion, and excretion by other routes
A comprehensive discussion o design and optimization o dosage regimens in
Chapter 2, including examples o dosing calculations that take into consideration
bioavailability, clearance, and distribution
Appendix II Design Optimization o Dosage Regimens: Pharmacokinetic Data in
Goodman & Gilmans T e Pharmacological Basis of T erapeutics, 12th Edition pro-
vides a summary o basic pharmacokinetic data or a number o drugs that are in
common clinical use, as well as in ormation that is use ul in individualizing dosing
in a given patient
Chapter 5 contains a detailed review o membrane transporters, including their role
in absorption, distribution, and clearance o xenobiotics, and their role in adverse
drug responses
able 5-1 Regulation o ransporter Expression by Nuclear Receptors provides
detailed in ormation about drug regulation o transporter expression mediated
through speci c nuclear receptors
A description in Chapter 5 o the role o e ux transporters in the blood-brain
barrier (BBB) and blood-cerebrospinal uid (CSF) barrier
Chapter 6 provides a comprehensive discussion o drug metabolism enzymes
and illustrations that show many o the key chemical reactions catalyzed by
these enzymes

CHARACTERISTICS OF
A DRUG THAT PREDICT LEARNING OBJECTIVES
ITS MOVEMENT AND Understand key concepts and terms that determine drug pharmacokinetics,
AVAILABILITY AT ITS SITES including absorption, distribution, metabolism, and excretion.
OF ACTION Understand the mechanisms by which drugs cross membranes and the physico-
Molecular size and structural eatures chemical actors that in uence this trans er.
Degree o ionization Be able to predict the ionization state o a drug that is a weak acid or base
Relative lipid solubilityo its ionized and knowing the drugs pKa and the pH o the uid.
nonionized orms Describe the role o membrane transporters in drug absorption, distribution,
Its binding to serumand tissue proteins and clearance.

18
Pharmacokinetics CHAPTER 2

Know the major mechanisms by which drugs are metabolized and excreted
PASSIVE TRANSPORT
rom the body.
VERSUS ACTIVE
Understand the major pharmacokinetic mechanisms that can result in drug TRANSPORT ACROSS
interactions. CELLULAR BARRIERS
Know the pharmacokinetic mechanisms that give rise to interpatient variability
Passive transport is the dominant transport
in drug response and toxicity.
mechanismin the disposition o most
Understand the clinical pharmacokinetic principles used to calculate dosing drugs (Figure 2-1).
required to achieve steady-state drug concentrations in plasma. In passive di usion, the drug molecule
Understand how route o administration can a ect the bioavailability and clear- usuallypenetrates cellular barriers by
ance o drugs. di usion along a concentration gradient
byvirtue o itssolubilityinthe lipidbilayer.
CASE 2 1 Paracellular transport through
intercellular gaps occurs across capillary
In Case 1-1, a gentleman is seeking an allergy medicine at his local pharmacy to relieve
endotheliumand is important in
his hay ever symptoms. One o the nonprescription drugs he nds contains diphen-
ltration across the glomerulus in the
hydramine, which can cause drowsiness or other CNS e ects as discussed in part d o
kidneybut is limited in some tissues
Case 1-1.
such as the CNSwhere capillaries have
a. How is diphenhydramine able to cause CNS e ects? tight intercellular junctions.
First-generation antihistamines such as diphenhydramine can cross the BBB and Facilitateddi usion describes a carrier-
cause CNS depression such as somnolence. Diphenhydramine and the other rst- mediated transport process in which
generation antihistamines contain a tertiary amino group linked by a 2- or 3-atom there is no input o energy, and there ore
chain to 2 aromatic substituents (see Figure 32-3 in Goodman & Gilmans T e enhanced movement o the involved
Pharmacological Basis of T erapeutics, 12th Edition). T e only ionizable group in substance is down a chemical gradient.
(Continued) Active transport is characterized bya direct
requirement or energy, movement against
an electrochemical gradient, saturability,
PAS S IVE TRANS P ORT ACTIVE selectivity, and competitive inhibition by
TRANS P ORT
cotransported compounds.
Pa ra ce llula r Diffus ion Fa cilita te d ABC
tra ns port diffus ion tra ns porte rs
WEAK ELECTROLYTES AND
THE INFLUENCE OF PH
Manydrugs are weakacids or bases that
are present in solution as both the nonion-
ized and ionized species.
Nonionized molecules are usuallymore
lipid soluble and can di use readily
across the cell membrane.
Ionized molecules usuallyare less
able to penetrate the lipid membrane
because o their lowlipid solubility.
FIGURE 2-1 The variety o ways drugs move across cellular barriers in their passage through- The transmembrane distribution o a weak
out the body. Transmembrane movement o drug generally is limited to unbound drug; thus electrolyte is in uenced byits pKa and the
drug-protein complexes constitute an inactive reservoir o drug. Unbound drugs cross mem-
pHgradient across the membrane.
branes either by passive processes or by mechanisms involving the active participation o
components o the membrane. In passive trans er, the drug molecule usually penetrates by di - The ratio o nonionized to ionized drug
usion along a concentration gradient by virtue o its solubility in the lipid bilayer. Such trans er at a given pHis readilycalculated rom
is directly proportional to the magnitude o the concentration gradient across the membrane, the Henderson-Hasselbalch equation
to the lipid-water partition coe cient o the drug, and to the membrane sur ace area exposed (Equation 2-1), where pKa is the drugs
to the drug. The greater the partition coe cient, the higher is the concentration o drug in the
acid dissociation constant.
membrane and the aster is its di usion. A ter a steady-state is attained, the concentration o
the unbound drug is the same on both sides o the membrane i the drug is a nonelectrolyte. [protonated form]
For ionic compounds, the steady-state concentrations depend on the electrochemical gradi- log = pKa pH
[unprotonated form]
ent or the ion and on di erences in pH across the membrane, which will in uence the state o
ionization o the molecule disparately on either side o the membrane and can e ectively trap (Equation 2-1)
drug on 1 side o the membrane. Carrier-mediated active transport is characterized by a direct
(continues)
requirement or energy.
19
SECTION I General Principles

A
WEAK ELECTROLYTES AND
We a k Acid HA A + H+ pKa = 4.4
THE INFLUENCE OF PH (Cont.) nonionize d ionize d

Figure 2-2 illustrates the partitioning o a B [1] [1000] 1001 = [HA] + [A ]


weakacid (pKa = 4.4) between plasma
(pH= 7.4) and gastricjuice (pH= 1.4). HA A + H+
At steadystate, an acidicdrug will accumu- P la s ma pH = 7.4

late on the more basicside o the membrane Lipid Mucos a l Ba rrie r


and a basicdrugon the more acidicside, a
Ga s tric juice pH = 1.4
phenomenon known as iontrapping.
In the kidneytubules where a lipid soluble [1] [0.001] 1.001 = [HA] + [A ]

(uncharged) drug can be reabsorbed by HA A + H+


passive di usion, excretion o the drug
can be promoted byaltering the pHo the FIGURE 2-2 In uence o pH on the distribution o a weak acid between plasma and gastric
juice separated by a lipid barrier. A. The dissociation o a weak acid, pKa = 4.4. B. Dissociation o
urine to avor the ionized state (A or BH+). the weak acid in plasma (pH 7.4) and gastric acid (pH 1.4). The uncharged orm, HA, equilibrates
Alkaline urine avors excretion o weak across the membrane. Blue numbers in brackets show relative concentrations o HA and A. In
acids; or example, elevation o urine pH this example, the ratio o nonionized to ionized drug in plasma is 1:1000; in gastric juice, the
with sodiumbicarbonate will increase ratio is 1:0.001, as given in brackets. The total concentration ratio between the plasma and the
the excretion o weakacids such as gastric juice there ore would be 1000:1 i such a system came to a steady-state. For a weak base
with a pKa o 4.4 (eg, chlordiazepoxide), the ratio would be reversed, as would the thick horizon-
aspirin (pKa ~3.5) and urate (pKa ~5.8). tal arrows, which indicate the predominant species at each pH. Accordingly, at steady state, an
Acid urine avors excretion o weakbases. acidic drug will accumulate on the more basic side o the membrane and a basic drug on the
more acidic side.

diphenhydramine and these other agents is the tertiary amino group, which has
DRUG ABSORPTION, a pKa ~9.0. T us, at physiological pH (pH 7.4), the drug is largely unionized
BIOAVAILABILITY, AND (see Figure 2-2) and very lipophilic and can easily di use through lipid membranes,
ROUTES OF ADMINISTRATION including the BBB.
Absorptionis the movement o a drug rom b. Are there other antihistamines that this patient could take that do not cause
its site o administration intothe central drowsiness?
compartment (Figure 2-3). T e second-generation antihistamines, such as loratadine (see Chapter 21), do not
Bioavailabilityis the ractional extent to cause drowsiness. T ese second-generation histamine H 1 antagonists are ionized at
which a dose o drug reaches its site o physiological pH and as a consequence do not readily cross the BBB. For example,
action or a biological uid romwhich the the pKa o loratadine is ~4.3, and is thus largely ionized at pH 7.4 and not able to
drug has access to its site o action. di use readily through lipid membranes.
A raction o the administered and (Continued)
absorbed dose o drug will be inactivated
or diverted in the intestine and liver be ore
it can reach the general circulation and be TIS S UE RES ERVOIRS
distributed to its sites o action. bound fre e
THERAP EUTIC
UNWANTED S ITE
I the metabolicor excretorycapacityo S ITE OF ACTION
OF ACTION
Re ce ptors
the liver and the intestine or the drug bound fre e CENTRAL bound fre e
is large, bioavailabilitywill be reduced COMPARTMENT
substantially(frst-pass e ect).
The route o drugadministration can a ect DRUG
ABS ORP TION
[FREE DRUG]
CLEARANCE
the time course o drug e ects, extent o DOS E
LIBERATION EXCRETION
absorption, bioavailability, rst-pass e ect,
variabilityo drug e ects, and adverse prote in bound me ta bolite s
e ects (see Table 2-1). drug

Knowledge o drugs that undergosigni -


cant metabolismor require active transport
BIOTRANS FORMATION
across the intestinal and hepaticmem-
branes can be used topredict some phar-
macokineticinteractions because drugs may FIGURE 2-3 The interrelationship o the absorption, distribution, binding, metabolism, and
excretion o a drug and its concentration at its sites o action. Possible distribution and binding o
compete or metabolismand transport.
metabolites in relation to their potential actions at receptors are not depicted.
20
Pharmacokinetics CHAPTER 2

TABLE 2-1 Some Characteristics o Common Routes o Drug Administration DISTRIBUTION OF DRUGS
LIMITATIONS AND Distribution re ers to the movement o
ROUTE ABSORPTION PATTERN SPECIAL UTILITY PRECAUTIONS drug into interstitial and intracellular uids
Intravenous Absorption circumvented Valuable or Increased risk o adverse ollowing absorption or systemicadminis-
Potentially immediate emergency use e ects tration (see Figure 2-3).
e ects Permits titration o Must inject solutions Distribution re ects a number o physi-
Suitable or large dosage slowly as a rule ological actors and the particular physico-
volumes and or irritating Usually required or Not suitable or oily
substances, or complex high-molecular-weight solutions or poorly
chemical properties o the individual drug.
mixtures, when diluted protein and peptide soluble substances Physiological actors that determine the
drugs rate o deliveryand potential amount o
drug distributed into tissue include:
Subcutaneous Prompt rom aqueous Suitable or some Not suitable or large
solution poorly soluble volumes Cardiacoutput
Slow and sustained rom suspensions and or Possible pain or Regional blood ow
repository preparations instillation o slow- necrosis rom irritating
release implants substances Capillarypermeability
Tissue volume
Intramuscular Prompt rom aqueous Suitable or moderate Precluded during
solution volumes, oily vehicles, anticoagulant therapy During the initial distribution phase, the
Slow and sustained rom and some irritating May inter ere with liver, kidney, brain, and other well-per-
repository preparations substances interpretation o certain used organs receive most o the drug.
Appropriate or diagnostic tests (eg,
sel -administration creatine kinase)
The second distribution phase to muscle,
(eg, insulin) most viscera, skin, and at is slower and
mayrequire minutes toseveral hours be ore
Oral ingestion Variable, depends on Most convenient and Requires patient the concentration o drug in tissue is in
many actors economical; usually compliance equilibriumwith that in blood.
more sa e Bioavailability
potentially erratic and The second phase involves a ar larger
incomplete raction o bodymass (eg, muscle) than
does the initial phase and generally
accounts or most o the extravascularly
distributed drug.
With exceptions such as the brain, di u-
c. Besides the CNS, where else in the body might the net charge o a drug be sion o drug into the interstitial uid occurs
a ected by pH? rapidlybecause o the highlyperme-
T e partitioning o a weak acid (pKa = 4.4) between plasma (pH = 7.4) and gas- able nature o the capillaryendothelial
tric juice (pH = 1.4) is depicted in Figure 2-2. Assume that the gastric mucosal membrane.
membrane behaves as a simple lipid barrier with a high electrical resistance that Tissue distribution is determined bythe
is permeable only to the lipid-soluble, nonionized orm o the acid. T e ratio o partitioning o drug between blood and
nonionized to ionized drug at each pH is readily calculated rom the Henderson- the particular tissue.
Hasselbalch equation (see Equation 2-1). In the example o Figure 2-2, the ratio Lipid solubilityand transmembrane pH
o nonionized to ionized drug in plasma is 1:1000; in gastric juice, the ratio is gradients are important determinants
1:0.001, as given in brackets in Figure 2-2. T e total concentration ratio between o tissue uptake or drugs that
the plasma and the gastric juice there ore would be 1000:1 i such a system came are either weakacids or bases;
to a steady-state. For a weak base with a pKa o 4.4 (eg, chlordiazepoxide), the however, ion trapping associated
ratio would be reversed, as would the thick horizontal arrows in Figure 2-2, which with transmembrane pHgradients is
indicate the predominant species at each pH. Accordingly, at steady state, an acidic generallynot large because the pH
drug will accumulate on the more basic side o the membrane and a basic drug on di erence between tissue and blood
the more acidic side. (~7.0 vs 7.4) is small.
T e e ects o net charge are observable elsewhere in the body, such as in the kidney The most important determinant o
tubules. Urine pH can vary over a ride range, rom 4.5 to 8. As urine pH drops (as blood-tissue partitioning is the relative
[H +] increases), weak acids (A) and weak bases (B) will exist to a greater extent in binding o drug to plasma proteins and
their protonated orms (HA and BH +); the reverse is true as pH rises, where A and tissue macromolecules that limits the
B will be avored. In the kidney tubules where a lipid-soluble (uncharged) drug can concentration o ree drug.
be reabsorbed by passive di usion, excretion o the drug can be promoted by alter-
ing the pH o the urine to avor the ionized state (A or BH +). T us, alkaline urine
(Continued)

21
SECTION I General Principles

avors excretion o weak acids; acid urine avors excretion o weak bases. Elevation
PLASMA PROTEIN BINDING
o urine pH (by giving sodium bicarbonate) will promote urinary excretion o weak
OF DRUGS
acids such as aspirin (pKa~3.5) and urate (pKa~5.8). T is principle o ion trapping is
Manydrugs circulate in the bloodstream an important process in drug distribution.
bound to plasma proteins. T e establishment o concentration gradients o weak electrolytes across membranes
Albumin is a major carrier or acidic with a pH gradient is a physical process and does not require an active electrolyte
drugs. transport system. All that is necessary is a membrane pre erentially permeable to
1-Acid glycoprotein binds basicdrugs. one orm o the weak electrolyte and a pH gradient across the membrane. T e estab-
lishment o the pH gradient, however, is an active process.
Certain drugs maybind to proteins
that unction as speci chormone
carrier proteins, such as the binding
o estrogen or testosterone to sex CASE 2 2
hormonebinding globulin or the A 68-year-old woman has symptoms o angina when she climbs stairs or engages in
binding o thyroid hormone to thyroxin- strenuous activity. She is prescribed nitroglycerin (glyceryl trinitrate, see Chapter 16)
binding globulin. to take prophylactically be ore she engages in any activity that might cause angina
Plasma protein binding is a nonlinear, symptoms.
saturable process.
a. T e patients pharmacist instructs her that she is to place the nitroglycerin tab-
Binding o a drug to plasma proteins limits let under her tongue a couple o minutes be ore strenuous activity to prevent
its concentration in tissues and at its site o angina. Why is this route o administration used or this drug?
action because onlyunbound ( ree) drug
Sublingual administration o nitroglycerin results in rapid onset o action (peak
is in equilibriumacross membranes (see
concentrations within 4 minutes o administration) and avoids the rst-pass
Figure 2-3).
e ect. Absorption rom the oral mucosa has special signi cance or certain
AppendixIIin Goodman &Gilmans The drugs despite the act that the sur ace area available is small. Venous drainage
Pharmacological Basis o Therapeutics, 12th rom the mouth is to the superior vena cava, bypassing the portal circulation
Edition provides plasma protein binding and thereby protecting the drug rom rapid intestinal and hepatic rst-pass
percentages or a number o commonly metabolism. Nitroglycerin is e ective when retained sublingually because it
used drugs. is nonionic and has very high lipid solubility. T us, the drug is absorbed very
The extent o plasma protein binding also rapidly. Nitroglycerin also is very potent; absorption o a relatively small amount
maybe a ected bydisease-related actors. produces the therapeutic e ect (unloading o the heart; see Chapter 16).
Changes in protein binding due Nitroglycerin that is swallowed undergoes nearly complete rst-pass metabolism
to disease states and drugdrug as the result o enzymatic denitration in the liver. T us, its bioavailability is only
interactions are clinicallyrelevant ~0.01 (ie, ~1%) when swallowed.
mainly or a small subset o the b. o prevent rst-pass e ects, what other routes o administration might be
so-called high-clearance drugs o e ective or this agent?
narrowtherapeuticindexthat are
Because nitroglycerin has very high lipid solubility, it can be absorbed through
administered intravenously.
the skin and administered via controlled-release transdermal patches. T is route
Drug excretion bythe kidneys, transport, o administration is e ective or patients requiring chronic administration o
and metabolismcan be limited bybinding nitroglycerin to prevent symptoms o angina, but can result in the development
to plasma protein. o tolerance unless the patch is removed or 8-12 hours each day (see Case 16-1).

CASE 2 3
A 59-year-old man who was diagnosed with coronary artery disease undergoes angio-
plasty and receives a coronary stent. o prevent platelet thrombosis o the stent, he is
prescribed a standard dosing regimen o clopidogrel, a drug that inhibits platelet aggre-
TISSUE BINDING OF DRUGS gation (see Chapter 19). A er several weeks o clopidogrel therapy, his platelet unction
is measured and it is determined that the dose o clopidogrel is too low to e ectively
Manydrugs accumulate in tissues at higher
inhibit platelet aggregation.
concentrations than those in the extracel-
lular uids and blood. a. What kind o drug is clopidogrel and how might this explain the poor response
Tissue accumulation maybe a result o to therapy with this agent?
active transport or, more commonly, bind- Clopidogrel is a prodrug that requires bioactivation, primarily by CYP2C19. T ere
ing to cellular constituents such as proteins, is wide interindividual variability in the capacity o clopidogrel to inhibit platelet
phospholipids, or nuclear proteins. aggregation, and some patients are designated resistant to the antiplatelet e ects
(continues)
(Continued)
22
Pharmacokinetics CHAPTER 2

TABLE 2-2 ABC Transporters Involved in Drug Absorption, Distribution, and TISSUE BINDING OF DRUGS
Excretion Processes (Cont.)
TRANSPORTER TISSUE PHYSIOLOGICAL Tissue binding is generallyreversible and
NAME GENE DISTRIBUTION FUNCTION SUBSTRATES can serve as a reservoir that prolongs drug
MDR1 (ABCB1) Liver Natural Characteristics: Neutral or action in that same tissue or at a distant
Kidney detoxi cation cationic compounds with bulky site reached through the circulation.
Intestine system against structure Tissue binding and accumulation also
BBB xenobiotics Anticancer drugs: etoposide,
can produce local toxicity.
BTB doxorubicin, vincristine
BPB Ca2+ channel blockers: diltiazem, Manylipid-soluble drugsare storedbyphysi-
verapamil cal solution in the neutral at; hence, at may
HIV protease inhibitors: indinavir, serve as a reservoir or lipid-soluble drugs.
ritonavir
Some drugs, divalent metal ion chelating
Antibiotics/Antifungals:
erythromycin, ketoconazole agents, and heavymetals mayaccumulate
Hormones: testosterone, in bone by adsorption onto the bone
progesterone crystal sur ace and eventual incorporation
Immunosuppressants: cyclosporine, into the crystal lattice.
tacrolimus
Others: digoxin, quinidine,
exo enadine, loperamide DRUG MEMBRANE
TRANSPORTERS
MRP1 (ABCC1) Ubiquitous Leukotriene C4 Characteristics: Amphiphilic with at
(Kidney, BCSFB, secretion rom least one negative net charge Transporters are membrane proteins that
BTB) leukocyte Anticancer drugs: vincristine (with control the in ux(uptake) o essential
GSH), methotrexate nutrients and ions, and the e uxo cellu-
Glutathione conjugates: lar waste, environmental toxins, drugs, and
leukotriene C4, glutathione conjugate
other xenobiotics.
o ethacrynic acid
Glucuronide conjugates: estradiol- The unctions o membrane transport-
17-D-glucuronide, bilirubin mono(or ers maybe facilitated (equilibrative, not
bis)glucuronide requiring energy) or active (requiring
Sulfated conjugates: estrone-3- energy); see Figure 2-1.
sul ate (with GSH)
HIV protease inhibitors: saquinavir The transport o drugsis primarilymediated
Antibiotics: grepa oxacin by2major super amilies, ABC(ATP-binding
Others: olate, GSH, oxidized cassette) andSLC(solute carrier) transporters.
glutathione Most ABCproteins are primaryactive
MRP2 (ABCC2) Liver Excretion Characteristics: Amphiphilic with at transporters, which relyon ATPhydrolysis
Kidney o bilirubin least one negative net charge (similar toactivelypump their substrates across
Intestine glucuronide and to MRP1) membranes (Table 2-2).
BPB GSH into bile Anticancer drugs: methotrexate, There are 49 known genes or ABC
vincristine
protein which include P-glycoprotein
Glutathione conjugates:
leukotriene C4, glutathione conjugate (P-gp, encoded byABCB1, also
o ethacrynic acid termed MDR1) and the cystic brosis
Glucuronide conjugates: estradiol- transmembrane regulator (CFTR,
17-D-glucuronide, bilirubin mono(or encoded byABCC7).
bis)glucuronide The SLCsuper amilyincludes genes that
Sulfate conjugate of bile salts:
taurolithocholate sul ate encode acilitated transporters and ion-
Amphipathic organic anions: coupled secondaryactive transporters
statins, angiotensin II receptor (Table 2-3).
antagonists, temocaprilat Approximately315 SLCtransporters
HIV protease inhibitors: indinavir, have been identi ed in the human
ritonavir
Others: GSH, oxidized glutathione
genome, manyo which serve as
drug targets or in drug absorption
and disposition, including the
serotonin (5-HT) and dopamine
transporters (SERT,encoded by
SLC6A4; DAT,encoded bySLC6A3).
(continues)
23
SECTION I General Principles

DRUG MEMBRANE TRANSPORTER TISSUE PHYSIOLOGICAL


NAME GENE DISTRIBUTION FUNCTION SUBSTRATES
TRANSPORTERS (Cont.)
MRP3 (ABCC3) Liver ? Characteristics: Amphiphilic with
Uptake and e uxtransporters determine
Kidney at least one negative net charge
the plasma and tissue concentrations o Intestine (Glucuronide conjugates are
endogenous compounds and xenobiotics, better substrates than glutathione
and therebycan in uence the systemicor conjugates.)
site-speci ctoxicityo drugs. Anticancer drugs: etoposide,
methotrexate
Geneticmutations and polymorphisms
Glutathione conjugates:
in drug membrane transporters can lead leukotriene C4, glutathione conjugate
to signi cant variabilityin individual o 15-deoxy-delta prostaglandin J2
patient drug disposition and response ( or Glucuronide conjugates:
examples, see Table 2-4). estradiol-17-D-glucuronide,
etoposide glucuronide, morphine-3-
glucuronide, morphine-6-glucuronide,
acetaminophen glucuronide,
hymecromone glucuronide, and
harmol glucuronide
DISTRIBUTION OF DRUGS Sulfate conjugates of bile salts:
taurolithocholate sul ate
INTO THE CNS
Bile salts: glycocholate, taurocholate
Distribution o drugs into the CNS romthe Others: olate, leucovorin
blood is unique because:
MRP4 (ABCC4) Ubiquitous ? Characteristics: Nucleotide analogs
Brain capillaryendothelial cells have (Kidney, Anticancer drugs:
continuous tight junctions, thus drug Prostate, 6-mercaptopurine, methotrexate
penetration into the brain depends on Lung, Muscle, Glucuronide conjugates:
transcellular rather than paracellular Pancreas, Testis, estradiol-17-D-glucuronide
transport. Ovary, Bladder, Sulfate conjugates:
Gallbladder, dehydroepiandrosterone sul ate
The BBB, which consists o brain BBB, BCSFB) Cyclic nucleotides: cAMP, cGMP
capillaryendothelial cells and Diuretics: urosemide,
pericapillaryglial cells, have unique trichlormethiazide
properties that limit drug transport. Antiviral drugs: ade ovir, teno ovir
Antibiotics: ce azolin, ce tizoxime
At the choroid plexus, a blood-CSF Others: olate, leucovorin,
barrier is present with epithelial cells taurocholate (with GSH)
that are joined bytight junctions, rather
than endothelial cells. MRP5 (ABCC5) Ubiquitous ? Characteristics: Nucleotide analogs
Anticancer drugs: 6-mercaptopurine
The more lipophilica drug, the more likely Cyclic nucleotides: cAMP, cGMP
it is to cross the BBB(see Case 2-1). HIV protease inhibitors: ade ovir
Drugs maypenetrate into the CNSby MRP6 (ABCC6) Liver ? Anticancer drugs: doxorubicin*,
speci cuptake transporters normally Kidney etoposide*
involved in the transport o nutrients and Glutathione conjugates:
endogenous compounds romblood into leukotriene C4
the brain and CSF. Other: BQ-123 (cyclic peptide)
The unctional BBBand blood-CSFbarrier BCRP (MXR) Liver Normal heme Anticancer drugs: methotrexate,
involve e uxtransporters that are capable (ABCG2) Intestine metabolism/ mitoxantrone, camptothecins SN-38,
o removing a large number o chemically BBB transport during topotecan, imatinib
diverse drugs romthe CNS(see Figure 2-4, maturation o Glucuronide conjugates:
andTables 2-2 and 2-3). erythrocytes? 4-methylumbelli erone glucuronide,
estradiol-17-D-glucuronide
MDR1 (P-gp) and the organicanion Sulfate conjugates:
transporting polypeptide (OATP) are 2 dehydroepiandrosterone sul ate,
o the more notable e uxtransporters estrone-3-sul ate
expressed in the BBBand blood-CSF Antibiotics: nitro urantoin,
barrier (BCSFB). uoroquinolones
Statins: pitavastatin, rosuvastatin
E uxtransporters are expressed in Others: cholesterol, estradiol,
brain capillaryendothelial cells and the dantrolene, prazosin, sul asalazine,
choroid plexus. phytoestrogens, PhIP, pheophorbide A

24
Pharmacokinetics CHAPTER 2

TRANSPORTER TISSUE PHYSIOLOGICAL


NAME GENE DISTRIBUTION FUNCTION SUBSTRATES

MDR3 (ABCB4) Liver Excretion o Characteristics: Phospholipids


phospholipids
into bile

BSEP (ABCB11) Liver Excretion o bile Characteristics: Bile salts


salts into bile

ABCG5 and Liver Excretion o plant Characteristics: Plant sterols


ABCG8 Intestine sterols into bile
and intestinal
lumen

Representative substrates and cytotoxic drugs with increased resistance (*) are included in this
table (cytotoxicity with increased resistance is usually caused by the decreased accumulation
o the drugs). Although MDR3 (ABCB4), BSEP (ABCB11), ABCG5, and ABCG8 are not directly
involved in drug disposition, inhibition o these physiologically important ABC transporters will
lead to un avorable side e ects.
BBB, blood-brain barrier; BTB, blood-testis barrier; BPB, blood-placental barrier; BCSFB, blood-
cerebrospinal uid barrier.

TABLE 2-3 Families in the Human Solute Carrier Super amily


NUMBER OF FAMILY SELECTED DRUG EXAMPLES OF LINKED HUMAN
GENE NAME FAMILY NAME MEMBERS SUBSTRATES DISEASES
SLC1 High-a nity glutamate and neutral 7 Amyotrophic lateral sclerosis
amino acid transporter

SLC2 Facilitative GLUT transporter 14

SLC3 Heavy subunits o the heteromeric 2 Melphalan Classic cystinuria type I


amino acid transporters

SLC4 Bicarbonate transporter 10 Hemolytic anemia, blindness-auditory


impairment

SLC5 Na+ glucose cotransporter 8 Dapagli ozin Glucose-galactose malabsorption


syndrome

SLC6 Na+- and Cl-dependent 16 Paraoxetine, uoxetine X-linked creatine de ciency syndrome
neurotransmitter transporter

SLC7 Cationic amino acid transporter 14 Melphalan Lysinuric protein intolerance

SLC8 Na+/Ca2+ exchanger 3 N,N-Dimethylarginine

SLC9 Na+/H+ exchanger 8 Thiazide diuretics Congenital secretory diarrhea

SLC10 Na+ bile salt cotransporter 6 Benzothiazepines Primary bile salt malabsorption

SLC11 H+ coupled metal ion transporter 2 Hereditary hemochromatosis

SLC12 Electroneutral cation-Cl 9 Gitelmans syndrome


cotransporter amily

SLC13 Na+-sul ate/carboxylate 5 Sul ate conjugates,


cotransporter cysteine conjugates

SLC14 Urea transporter 2 Kidd antigen blood group

SLC15 H+-oligopeptide cotransporter 4 Valacyclovir

25
SECTION I General Principles

NUMBER OF SELECTED DRUG EXAMPLES OF LINKED HUMAN


GENE NAME FAMILY NAME FAMILY MEMBERS SUBSTRATES DISEASES

SLC16 Monocarboxylate transporter 14 Salicylic acid, Muscle weakness


atorvastatin

SLC17 Vesicular glutamate transporter 8 Sialic acid storage disease

SLC18 Vesicular amine transporter 3 Reserpine Myasthenic syndromes

SLC19 Folate/thiamine transporter 3 Methotrexate Thiamine-responsive megaloblastic


anemia

SLC20 Type III Na+-phosphate cotransporter 2

SLC21/SLC0 Organic anion transporter 11 Pravastatin

SLC22 Organic cation/anion/zwitterion 18 Pravastatin, met ormin Systemic carnitine de ciency syndrome
transporter

SLC23 Na+-dependent ascorbate 4 Vitamin C


transporter

SLC24 Na+/(Ca2+-K+) exchanger 5

SLC25 Mitochondrial carrier 27 Sengers syndrome

SLC26 Multi unctional anion exchanger 10 Salicylic acid, Congenital Cl-losing diarrhea
cipro oxacin

SLC27 Fatty acid transporter protein 6

SLC28 Na+-coupled nucleoside transport 3 Gemcitabine, cladribine

SLC29 Facilitative nucleoside transporter 4 Dipyridamole,


gemcitabine

SLC30 Zinc ef ux 9

SLC31 Copper transporter 2 Cisplatin

SLC32 Vesicular inhibitory amino acid 1 Vigabatrin


transporter

SLC33 Acetyl-CoA transporter 1

SLC34 Type II Na+-phosphate cotransporter 3 Autosomal-dominant


hypophosphatemic rickets

SLC35 Nucleoside-sugar transporter 17 Leukocyte adhesion de ciency type II

SLC36 H+-coupled amino acid transporter 4 D-Serine, cycloserine

SLC37 Sugar-phosphate/phosphate 4 Glycogen storage disease (non-la)


exchanger

SLC38 System A and N, Na+-coupled neutral 6


amino acid transporter

SLC39 Metal ion transporter 14 Acrodermatitis enteropathica

SLC40 Basolateral iron transporter 1 Type IVhemochromatosis

SLC41 MgtE-like magnesium transporter 3

SLC42 Rh ammonium transporter 3 Rh-null regulator

SLC43 Na+-independent system-L-like 2


amino acid transporter

26
Pharmacokinetics CHAPTER 2

TABLE 2-4 Examples o Genetic Polymorphisms Inf uencing Drug Response


GENE PRODUCT GENE DRUGSa RESPONSES AFFECTED
Drug Metabolism and Transport

CYP2C9 Tolbutamide, war arin,a phenytoin, nonsteroidal Anticoagulant e ect o war arin
anti-in ammatory

CYP2C19 Mephenytoin, omeprazole, voriconazole,a Peptic ulcer response to omeprazole;


hexobarbital, mephobarbital, propranolol, cardiovascular events a ter clopidogrel
proguanil, phenytoin, clopidogrel

CYP2D6 blockers, antidepressants, anti-psychotics, Tardive dyskinesia rom antipsychotics, narcotic


codeine, debrisoquine, atomoxetine,a side e ects, codeine e cacy, imipramine dose
dextromethorphan, encainide, ecainide, requirement, blocker e ect; breast cancer
uoxetine, guanoxan, N-propylajmaline, recurrence a ter tamoxi en
perhexiline, phenacetin, phen ormin,
propa enone, sparteine, tamoxi en

CYP3A4/3A5/3A7 Macrolides, cyclosporine, tacrolimus, Ca2+ E cacy o immunosuppressive e ects o


channel blockers, midazolam, ter enadine, tacrolimus
lidocaine, dapsone, quinidine, triazolam,
etoposide, teniposide, lovastatin, al entanil,
tamoxi en, steroids

Dihydropyrimidine dehydrogenase Fluorouracil, capecitabine a 5-Fluorouracil toxicity

N-acetyltrans erase (NAT2) Isoniazid, hydralazine, sul onamides, amona de, Hypersensitivity to sul onamides, amona de
procainamide, dapsone, ca eine toxicity, hydralazine-induced lupus, isoniazid
neurotoxicity

Glutathione trans erases (GSTM1, Several anticancer agents Decreased response in breast cancer,
GSTT1,GSTP1) more toxicity and worse response in acute
myelogenous leukemia

Thiopurine methyltrans erase (TPMT) Mercaptopurine,a thioguanine,a azathioprine a Thiopurine toxicity and e cacy, risk o second
cancers

UDP-glucuronosyl-trans erase (UGT1A1) Irinotecan,a bilirubin Irinotecan toxicity

P-glycoprotein (ABCB1) Natural product anticancer drugs, HIVprotease Decreased CD4 response in HIV-in ected patients,
inhibitors, digoxin decreased digoxin AUC, drug resistance in
epilepsy

UGT2B7 Morphine Morphine plasma levels

Organic anion transporter (SLC01B1) Statins, methotrexate, ACE inhibitors Statin plasma levels, myopathy; methotrexate
plasma levels, mucositis

COMT Levodopa Enhanced drug e ect

Organic cation transporter (SLC22A1, Met ormin Pharmacologic e ect and pharmacokinetics
OCT1)

Organic cation transporter (SLC22A2, Met ormin Renal clearance


OCT2)

Novel organic cation transporter (SLC22A4, Gabapentin Renal clearance


OCTN1)

CYP2B6 Cyclophosphamide Ovarian ailure


a
In ormation on genetics-based dosing, adverse events, or testing added to FDA-approved drug label.

27
SECTION I General Principles

S mall inte s tine : Live r: Kidne y: Brain c apillarie s :


METABOLISM abs o rptio n he pato biliary tubular s e c re tio n barrie r func tio n
trans po rt
(BIOTRANSFORMATION)
OF DRUGS
The majorityo therapeuticagents are S LC

lipophiliccompounds ltered through the S LC


ATP
ABC
glomerulus and reabsorbed into the sys- ABC S LC ABC
ABC
temiccirculation during passage through ATP ATP ATP ATP
S LC
the renal tubules. ABC S LC S LC ABC
ATP

The metabolismo drugs and other xeno- S LC


biotics into more hydrophilicmetabolites
Blood
is essential or their elimination romthe
body, as well as or termination o their FIGURE 2-4 Transepithelial or transendothelial ux. Transepithelial or transendothelial ux
biological and pharmacological activity. o drugs requires distinct transporters at the 2 sur aces o the epithelial or endothelial barriers.
In general, biotrans ormation reactions These are depicted diagrammatically or transport across the small intestine (absorption), the
generate more polar, inactive metabolites kidney and liver (elimination), and the brain capillaries that comprise the BBB. Asymmetrical
transport across a monolayer o polarized cells, such as the epithelial and endothelial cells o
that are readilyexcreted romthe body brain capillaries, is called vectorial transport. Vectorial transport is important in the e cient trans-
through the urine or bile, but in some er o solutes across epithelial or endothelial barriers. From the viewpoint o drug absorption and
cases, metabolites with potent biological disposition, vectorial transport plays a major role in hepatobiliary and urinary excretion o drugs
activityor toxicproperties are generated. rom the blood to the lumen and in the intestinal absorption o drugs. In addition, ef ux o drugs
Drug metabolismor biotrans ormation rom the brain via brain endothelial cells and brain choroid plexus epithelial cells involves vecto-
rial transport. The ABC transporters mediate only unidirectional ef ux, whereas SLC transporters
reactions are classi ed as either phase I mediate either drug uptake or ef ux.
unctionalization reactions or phase IIbio-
synthetic(conjugation) reactions.
Phase 1 unctionalization reactions
introduce or expose a unctional group o the drug. T is variability re ects, at least in part, genetic polymorphisms in
on the parent compound through the CYPs involved in the metabolic activation o clopidogrel, most importantly
oxidation, reduction, or hydrolytic CYP2C19. Clopidogrel-treated patients with the loss-o - unction CYP2C19*2
reactions and generallyresult in the loss allele exhibit reduced platelet inhibition compared with those with the wild-type
o pharmacological activity, although CYP2C19*1 allele and experience a higher rate o cardiovascular events. Even
there are examples o retention or patients with the reduced- unction CYP2C19*3, *4, or *5 alleles may derive less
enhancement o activity. bene t rom clopidogrel than those with the ull- unction CYP2C19*1 allele.
Although CYP3A4 also contributes to the metabolic activation o clopidogrel, poly-
Phase 1 enzymes lead to the
morphisms in this enzyme do not appear to in uence clopidogrel responsiveness.
introduction o unctional groups such
as OH, COOH, SH, O, or NH2. b. What kinds o pharmacokinetic drug interactions might be expected with
Prodrugs are pharmacologically clopidogrel?
inactive compounds that are CYP2C19 is responsible or metabolizing a number o drugs (see able 2-4) and
convertedrapidlytobiologicallyactive there are potential drug interactions that can occur among these drugs. For example,
metabolites, o tenbythe hydrolysiso omeprazole, a protein pump inhibitor, which is used to reduce gastric acidity (see
an ester or amide linkage. Chapter 32), is also metabolized by CYP2C19 and can act as a CYP2C19 inhibi-
Phase 2 conjugation reactions lead to tor. Concomitant administration o omeprazole and other proton pump inhibitors
the ormation o highlypolar conjugates that are inhibitors o CYP2C19, with clopidogrel, produces a small reduction in the
bycovalent linkage o a unctional inhibitory e ects o clopidogrel on platelet aggregation. T is reduction in the anti-
group on the parent compound or phase platelet e ects o clopidogrel can increase the risk o cardiovascular events.
Imetabolite and endogenouslyderived
glucuronicacid, sul ate, glutathione, CASE 2 4
amino acids, or acetate.
Acetaminophen is among the most widely used drugs and is an active ingredient in
Drugs that are poorlymetabolized remain many over-the-counter analgesics and cold medications, as well as in many prescrip-
in the body or longer periods o time and tion products in combination with codeine and other analgesics.
their pharmacokineticpro les showmuch
longer elimination hal -lives than drugs a. How is acetaminophen normally metabolized?
that are rapidlymetabolized. Although many drugs require prior oxidative metabolism by phase 1 enzymes,
acetaminophen and some other drugs can undergo phase 2 conjugation (glucuroni-
dation and/or sul ation) without prior oxidative metabolism (see able 2-6). One o
(Continued)

28
Pharmacokinetics CHAPTER 2

the important phase 2 enzymes that metabolizes acetaminophen are the sul otrans-
BIOTRANSFORMING
erase (SUL ) SUL 1 iso orms that are recognized as phenol SUL s, because they
ENZYMES (Xe nob iot ic
catalyze the sul ation o phenolic molecules such as acetaminophen, minoxidil, and
17-ethinyl estradiol. Me t ab olizin g Enzyme s;
Tab le 2 5)
b. Package labels on products containing acetaminophen warn o possible liver
damage when daily dosing o acetaminophen exceeds 4000 mg. What is the Phase 1 oxidation reactions are carried
mechanism o liver toxicity by acetaminophen? out bycytochrome P-450s (CYPs), avin-
containing monooxygenases (FMO), and
Acetaminophen, which is normally metabolized by glucuronidation and sul ation, epoxide hydrolases (EH).
is also a substrate or oxidative metabolism by CYP2E1 and CYP3A4, which gen-
erate the toxic metabolite N-acetyl-p-benzoquinone imine (NAPQI) that, under Phase 2 enzymes include the glutathione-
normal dosing, is readily neutralized through conjugation with glutathione (GSH). S-trans erases (GST), UDP-glucuronos-
However, an overdose o acetaminophen can lead to depletion o cellular GSH lev- yltrans erases (UGT), sul otrans erases
els, thereby increasing the potential or NAPQI to interact with other cellular com- (SULT), N-acetyltrans erases (NAT), and
ponents resulting in toxicity and cell death. Acetaminophen toxicity is associated methyltrans erases (MT).
with increased levels o NAPQI and hepatic necrosis (see Chapter 3). Conjugation reactions usuallyrequire
the substrate to have oxygen (hydroxyl
or epoxide groups), nitrogen, or sul ur
CASE 2 5
atoms that serve as acceptor sites or a
A 26-year-old woman is lling a prescription or oral contraceptives and is asked by her hydrophilicmoiety, such as glutathione,
pharmacist whether she is taking any other medications, including herbal remedies. T e glucuronicacid, sul ate, or anacetyl group.
woman tells the pharmacist that she takes St Johns wort, an over-the-counter herbal Xenobioticmetabolizing enzymes are
remedy used or depression. ound in most tissues in the bodywith the
a. What are the possible drug interactions that might occur with St. Johns wort? highest levels located in the GItract (liver,
small and large intestines).
Hyper orin, a component o St. Johns wort, activates a type 2 nuclear receptor,
the pregnane X receptor (PXR). T e type 2 nuclear receptors include the pregnane The liver is considered the major
X receptor (PXR), constitutive androstane receptor (CAR), and the peroxisome metabolicclearing house or both
endogenous chemicals (eg, cholesterol,
(Continued)
steroid hormones, attyacids, and
proteins) and xenobiotics.
Drugs that are orallyadministered are
TABLE 2-5 Xenobiotic Metabolizing Enzymes
absorbed bythe gut and taken to the
ENZYMES REACTIONS liver through the portal vein.
Pha se 1 oxygena ses Xenobiotic-metabolizingenzymeslocated
in the epithelial cells o the GI tract are
Cytochrome P450s (P450 or CYP) C and O oxidation, dealkylation, others
responsible or the initial metabolic
Flavin-containing monooxygenases (FMO) N, S, and P oxidation processing o most oral medications.
Following oral administration o a drug,
Epoxide hydrolases (mEH, sEH) Hydrolysis o epoxides
a signi cant portion o the dose maybe
Pha se 2 tra nsfera ses metabolicallyinactivated in either the
intestinal epitheliumor the liver be ore
Sul otrans erases (SULT) Addition o sul ate
the drug reaches the systemiccirculation;
UDP-glucuronosyltrans erases (UGT) Addition o glucuronic acid this so-called irst-pass metabolism
signi cantlylimits the oral bioavailability
Glutathione-S-trans erases (GST) Addition o glutathione o highlymetabolized drugs.
N-acetyltrans erases (NAT) Addition o acetyl group Tissues o the nasal mucosa and lung
playimportant roles in the metabolism
Methyltrans erases (MT) Addition o methyl group
o drugs that are administered through
Other enzymes aerosol sprays and are also the rst line
o contact with hazardous chemicals
Alcohol dehydrogenases Reduction o alcohols that are airborne.
Aldehyde dehydrogenases Reduction o aldehydes Phase 1 oxidation enzymes are located
primarilyin the endoplasmicreticulum,
NADPH-quinone oxidoreductase (NQO) Reduction o quinones
whereas the phase 2 conjugation enzyme
mEH and sEH are microsomal and soluble epoxide hydrolase. UDP, uridine diphosphate; systems are mainlycytosolic.
NADPH, reduced nicotinamide adenine dinucleotide phosphate.

29
SECTION I General Principles

proli erator-activated receptors (PPARs). PXR, discovered because it is activated by


CYTOCHROME P-450
the synthetic steroid pregnenolone-16-carbonitrile, is also activated by a number
SUPERFAMILY: THE CYPS
o other drugs, including antibiotics (ri ampicin and troleandomycin), Ca2+ channel
The CYPs are a super amilyo enzymes, blockers (ni edipine), statins (mevastatin), antidiabetic drugs (troglitazone), HIV
all o which contain a molecule o heme protease inhibitors (ritonavir), and anticancer drugs (paclitaxel).
that is noncovalentlybound to the poly- Activated PXR is an inducer o CYP3A4, which can metabolize steroids ound in
peptide chain. oral contraceptives. PXR also induces the expression o genes encoding certain
CYPs use O2, plus H+ derived romthe drug transporters and phase 2 enzymes, including UDP-glucuronosyltrans erases
co actor-reduced nicotinamide adenine (UG s) and sul otrans erases (SUL s). T us, PXR acilitates the metabolism and
dinucleotide phosphate (NADPH), to carry elimination o many xenobiotics, including drugs, with notable consequences (see
out the oxidation o substrates. Figure 2-6).
CYPs catalyze diverse reactions, including b. How might concomitant administration o St. Johns wort a ect the e cacy o
N-dealkylation, O-dealkylation, aromatic drugs this patient might be taking such as the oral contraceptives?
hydroxylation, N-oxidation, S-oxidation,
By activating PXR and inducing the expression CYP3A4, drugs that are metabo-
deamination, and dehalogenation
lized by CYP3A4 will be cleared more quickly. CYP3A4 is the most abundantly
(Table 2-6).
expressed CYP in the liver and is responsible or metabolizing ~50% o all clinically
The CYPs are promiscuous in their capacity use ul drugs (see Figure 2-5 and Side Bar CY OCHROME P-450 SUPERFAMILY:
to bind and metabolize multiple sub- HE CYPS). T us, the e cacy o the oral contraceptives this woman will be taking
strates; extensive overlapping substrate will be reduced, increasing the chance o an unplanned pregnancy.
speci cities bythe CYPs is one o the
underlying reasons or the predominance
o drugdrug interactions involving drugs CASE 2 6
metabolized byCYPs. A 63-year-old, 84-kg man with mild heart ailure is started on a pharmacotherapy
When2coadministereddrugsare both regimen with digoxin to improve cardiac output (see Chapter 17).
metabolizedbya single CYP,theycompete
a. Af er administration o an intravenous dose o 500 g o digoxin, the plasma
or bindingtothe enzymesactive site
concentration o drug is determined to be 0.75 ng/mL. What is the volume o
whichcanresult in the inhibition o
distribution o digoxin in this patient?
metabolismo 1or botho the drugs,
leadingtoelevatedplasma levels. Dividing the amount o drug in the body by the plasma concentration yields a
volume o distribution or digoxin o ~667 L, or a value ~13 times greater than the
There are 57 putatively unctional CYP
total-body volume o an 84-kg man.
genes in humans, which are named with
the root CYP ollowed bya number des- b. What might account or this surprisingly large volume o distribution?
ignating the amily, a letter denoting the Digoxin distributes pre erentially to muscle and adipose tissue and to its speci c
sub amily, and another number designat- receptors (Na+, K+-A Pase), leaving a very small amount o drug in the plasma to
ing the CYP orm(eg, CYP3A4 is amily3, be measured. For drugs that are bound extensively to plasma proteins but that are
sub amilyA, and gene number 4). not bound to tissue components, the volume o distribution will approach that o
Most drugsare metabolizedbyonly12CYPs the plasma volume because drug bound to plasma protein is measurable in the
(CYP1A1, 1A2, 1B1, 2A6, 2B6, 2C8, 2C9, 2C19, assay o most drugs. In contrast, certain drugs have high volumes o distribution
2D6, 2E1, 3A4, and3A5; see Figure 2-5). even though most o the drugs in the circulation are bound to albumin because
The most active drug-metabolizing- these drugs are also sequestered elsewhere.
CYPs are those in the CYP2C, CYP2D, and T e volume o distribution may vary widely depending on the relative degrees o
CYP3Asub amilies. binding to high-a nity receptor sites, plasma and tissue proteins, the partition
CYP3A4, the most abundantly coe cient o the drug in at, and accumulation in poorly per used tissues. As might
expressed CYPin liver, is involved in be expected, the volume o distribution or a given drug can di er according to
the metabolismo more than 50%o patients age, gender, body composition, and presence o disease.
clinicallyused drugs (see Figure 2-5). c. How would the maintenance dose o oral digoxin be calculated in order to grad-
CYP1A, CYP1B, CYP2A, CYP2B, and ually bring this patient to therapeutic plasma concentrations?
CYP2Esub amilies are not signi cantly A steady-state plasma concentration o 0.7 to 0.9 ng/mL is selected as an appro-
involved in the metabolismo priate conservative target based on prior knowledge o the action o the drug
therapeuticdrugs, but theydo catalyze in patients with heart ailure to maintain levels at or below the 0.5 to 1.0 ng/mL
the metabolicactivation o many range. Equation 2-11 (p. 38), the patients creatinine clearance rate (CLCR = 56
protoxins and procarcinogens to their mL/min), and digoxin pharmacokinetic values (see Appendix II Design Opti-
ultimate reactive metabolites. mization o Dosage Regimens: Pharmacokinetic Data in Goodman and Gilmans
(continues) (Continued)

30
Pharmacokinetics CHAPTER 2

T e Pharmacological Basis of T erapeutics, 12th Edition) such as clearance


CYTOCHROME P-450
(CL = 0.88CLCR + 0.33) and oral bioavailability (F = 0.7 13) are used to calculate
SUPERFAMILY: THE CYPS
the appropriate dosing rate, as described in the section on Maintenance Dose in
Chapter 2 in Goodman and Gilmans T e Pharmacological Basis of T erapeutics, 12th (Cont.)
Edition. Digoxin clearance in this patient is estimated to be 77 mL/min (4.6 L/h). There are large di erences in levels o
Using Equation 2-11, the maintenance dosing rate or this 84-kg man is calculated expression o each CYPamong individuals
to be 0.12 mg/24 h ( or details o the calculation, see Example in the Maintenance due to the presence o geneticpolymor-
Dose section o Chapter 2 in Goodman and Gilmans T e Pharmacological Basis of phisms and di erences in gene regulation.
T erapeutics, 12th Edition). Human CYPgenes that exhibit polymor-
In practice, the dosing rate would be rounded to the closest dosage size, 0.125 phisms include CYP2A6, CYP2C9, CYP2C19,
mg/24 h, which would result in a steady-state plasma concentration o 0.78 ng/mL and CYP2D6 (see Table 2-4).
(0.75 125/120). Digoxin is a well-characterized example o a drug that is di cult Some drugs are ligands or nuclear
to dose and must be monitored regularly. While guidelines based on calculations o receptors and act as CYPinducers that
the sort suggested here are use ul, it is clear that tablet sizes are limiting and tablet can increase not onlytheir own rates o
sizes intermediate to those available are of en needed. Because the coe cient o metabolism, but also induce metabolism
variation or the clearance equation when used or digoxin treatment in this patient o other coadministered drugs (see
group is large (52%), it is common or patients who are not monitored regularly to Table 2-7 and Figure 2-6).
require hospital admission to adjust medication. Monitoring the clinical status o
One potential consequence o CYP
patients (new or increased ankle edema, inability to sleep in a recumbent position,
induction is a decrease in plasma
decreased exercise tolerance), whether accomplished by home health ollow-up or
drug concentration over the course
regular visits to the clinician, is essential to avoid untoward results.
o treatment, resulting in loss o
d. Digoxin has a narrow therapeutic index, and plasma levels 1.0 ng/mL usually drug ef cacy.
are associated with e cacy and minimal toxicity. What are the maximum and In addition toinducing CYPs, drugs
minimum plasma concentrations associated with once-daily dosing with 0.125 that are ligands or nuclear receptors
mg o digoxin? can induce the transcription o a
T is calculation rst requires estimation o digoxins volume o distribution based on battery o target genes that a ect
available pharmacokinetic data (Appendix II) and an estimate o digoxins elimi- pharmacokinetics, including drug
nation t 1/2 as described in the Example in the section on Dosing Interval or Inter- transporters and phase 2enzymes, which
mittent Dosage in Chapter 2 o Goodman and Gilmans T e Pharmacological Basis can lead toa varietyo drug interactions.
of T erapeutics, 12th Edition. With once-daily dosing, the plasma concentrations CYPinduction mayalso increase the
would uctuate minimally (0.7-0.88 ng/mL) about the steady-state concentration rate o metabolicactivation o protoxins
o 0.78 ng/mL, well within the recommended therapeutic range o 0.5-1.0 ng/mL. and carcinogens, resulting in increased
I twice the daily dose (2 0.125 mg) was given every other day, the average toxicityand carcinogenesis.
steady-state concentration would remain at 0.78 ng/mL, while the predicted maxi-
CYP3A4 is induced bynuclear receptors
mum concentration would be 0.98 ng/mL and the minimum concentration would
pregnane Xreceptor (PXR) and
be 0.62 ng/mL. While this result would maintain a therapeutic concentration and
constitutive androstane receptor (CAR),
avoid large excursions rom the average steady-state concentration between doses,
thus its level is highlyin uenced bya
it does not avor patient compliance. Dosing must be compatible with the patients
number o drugs and other xenobiotics.
routine and every other day dosing is problematic in this patient population.
e. Af er starting the patient on a maintenance dose o 0.125 mg o digoxin each day,
how long will it take to reach a steady-state therapeutic level?
T e rule o thumb or most drugs is that the time to reach 94% o steady-state con-
centrations af er starting or changing dosing is our hal -lives. Using Equation 2-10
(p. 38), the t1/2 is 3.1 days ( or details o the calculation, see Example in the Dosing
Interval or Intermittent Dosage section o Chapter 2 in Goodman and Gilmans T e
Pharmacological Basis of T erapeutics, 12th Edition). T us, it will take at least 12 days
to reach steady-state concentrations o drug in this patient.
I target concentrations o drug need to be achieved more rapidly, it may be possi-
ble to use a loading dose. In the case o this patient, Equation 2-12 (p. 39) would be
used to develop a loading dose strategy ( or details o the calculation, see Example
in the Loading Dose section o Chapter 2 in Goodman and Gilmans T e Pharmaco-
logical Basis of T erapeutics, 12th Edition). Here a target Cp o 0.9 ng/mL is chosen
as a target below the recommended maximum o 1.0 ng/mL. o avoid toxicity, this
oral loading dose, would be given as an initial 0.25-mg dose ollowed by a 0.25-mg
(Continued)

31
SECTION I General Principles

TABLE 2-6 Major Reactions Involved in Drug Metabolism


REACTION EXAMPLES
I. Oxidative reactions

N-Dealkylation RNHCH3 RNH2 + CH2O Imipramine, diazepam, codeine, erythromycin,


morphine, tamoxi en, theophylline, ca eine

O-Dealkylation ROCH3 ROH + CH2O Codeine, indomethacin, dextromethorphan

Aliphatic hydroxylation RCH2CH3 RCHOHCH3 Tolbutamide, ibupro en, phenobarbital,


meprobamate, cyclosporine, midazolam

Aromatic hydroxylation Phenytoin, phenobarbital, propanolol, ethinyl


estradiol, amphetamine, war arin

N-Oxidation RNH2 RNHOH Chlorpheniramine, dapsone, meperidine


R1 R1
NH N OH
R2 R2

S-Oxidation R1 R1 Cimetidine, chlorpromazine, thioridazine,


S S O omeprazole
R2 R2

Deamination OH Diazepam, amphetamine


RCHCH3 O
R C CH3 + NH3
NH2 R C CH3
NH2

II. Hydrolysis reactions Carbamazepine (see Figure 6-4)

O Procaine, aspirin, clo brate, meperidine, enalapril,


R 1 COOH + R 2 OH cocaine
R 1 COR 2
O Lidocaine, procainamide, indomethacin
R 1 COOH + R 2 NH2
R 1 CNHR 2

III. Conjugation reactions

Glucuronidation COOH COOH Acetaminophen, morphine, oxazepam, lorazepam


O R
O
R+ OH OH + UDP
OH O OH
OH UDP OH

Sul ation P AP S + ROH R O S O2 OH + P AP Acetaminophen, steroids, methyldopa

Acetylation CoAS CO CH3 + RNH2 RNH CO CH3 + CoA-S H Sul onamides, isoniazid, dapsone, clonazepam

Methylation RO-, RS-, RN- + AdoMet RO-CH3 + AdoHomCys L-Dopa, methyldopa, mercaptopurine, captopril

Glutathionylation GSH + R R-GSH Adriamycin, os omycin, busul an

PAPS, 3-phosphoadenosine-5phosphosul ate; PAP 3-phosphoadenosine-5-phosphate.

32
Pharmacokinetics CHAPTER 2

A
CYP 1A1/2 EXCRETION OF DRUGS
Es te ra s e s
CYP 1B1 Drugs are eliminated romthe bodyeither
Epoxide
unchanged bythe process o excretion or
Othe rs hydrola s e converted to metabolites.
CYP 2A6
Excretoryorgans, the lung excluded, elimi-
CYP 2B6 DPYD
nate polar compounds more ef ciently
than substances with high lipid solubility.
CYP 2C8/9 Lipid-soluble drugs thus are not readily
eliminated until theyare metabolized to
more polar compounds.
CYP 2C10
The most important organ or excreting
drugs and their metabolites is the kidney.
CYP 2D6 CYP 3A4/5
Excretion o drugs and metabolites in
the urine involves 3 distinct processes:
CYP 2E1
Glomerular ltration
B TP MT NATs Active tubular secretion
GSTs
Passive tubular reabsorption
Othe rs
Substances excreted in the eces are princi-
pallyunabsorbed orallyingested drugs or
drug metabolites excreted either in the bile
or secreted directlyinto the intestinal tract
and not reabsorbed.
Excretion romthe lung is important mainly
or the elimination o anestheticgases.
S ULTs UGTs Drugs excreted in breast milkare potential
sources o unwanted pharmacological
FIGURE 2-5 The raction o clinically used drugs metabolized by the major phase 1 and phase
e ects in the nursing in ant.
2 enzymes. The relative size o each pie section represents the estimated percentage o drugs
metabolized by the major phase 1 (panel A) and phase 2 (panel B) enzymes, based on studies
in the literature. In some cases, more than a single enzyme is responsible or metabolism o a
single drug. CYP, cytochrome P-450; DPYD, dihydropyrimidine dehydrogenase; GST, glutathione- CLINICAL
S-trans erase; NAT, N-acetyltrans erase; SULT, sul otrans erase; TPMT, thiopurine methyltrans erase; PHARMACOKINETICS
UGT, UDP-glucuronosyltrans erase
Clinical pharmacokinetics attempts to pro-
vide a quantitative relationship between
dose and e ect, and a rameworkwithin
which to interpret measurements o con-
centrations o drugs in biological uids and
TABLE 2-7 Nuclear Receptors That Induce Drug Metabolism their adjustment through changes in dos-
ing or the bene t o the patient.
RECEPTOR LIGANDS
The 4most important parameters governing
Aryl hydrocarbon receptor (AHR) Omeprazole drug disposition are:
Constitutive androstane receptor (CAR) Phenobarbital Bioavailability, the raction o drug
absorbed as such into the systemic
Pregnane X receptor (PXR) Ri ampin circulation
Farnesoid X receptor (FXR) Bile acids Volume o distribution, a measure o the
apparent space in the bodyavailable to
Vitamin D receptor Vitamin D
contain the drug based on howmuch
Peroxisome proli erator activated receptor (PPAR) Fibrates is given versus what is ound in the
systemiccirculation
Retinoic acid receptor (RAR) all-trans-Retinoic acid
Clearance, a measure o the bodys
Retinoid X receptor (RXR) 9-cis-Retinoic acid ef ciencyin eliminating drug romthe
systemiccirculation
(continues)

33
SECTION I General Principles

CLINICAL RXR

PHARMACOKINETICS (Cont.)
PXR
Elimination t1/2, the time it takes or Lig and Lig and
the plasma concentration o a drug to
OH
be reduced by50%and a measure o
CYP3A4
the rate o removal o drug romthe
systemiccirculation

VOLUME OF DISTRIBUTION
RNAPII
The volume o distribution (V) relates the Lig and Co
-a c tiv
amount o drug in the bodyto the concen- PXR RXR a to TBP
r
tration o drug (C) in the blood or plasma. TATA
The volume o distribution de ned in

Equation 2-2 considers the bodyas a


single homogeneous compartment (ie, FIGURE 2-6 Induction o drug metabolism by nuclear receptormediated signal transduction.
a one-compartment model), in which When a drug such as atorvastatin (Ligand) enters the cell, it can bind to a nuclear receptor such
all drug administration occurs directly as the pregnane X receptor (PXR). PXR then orms a complex with the retinoid X receptor (RXR),
into the central compartment, and binds to DNA upstream o target genes, recruits coactivator (which binds to the TATA box bind-
ing protein, TBP), and activates transcription by RNA polymerase II (RNAP II). Among PXR target
distribution o drug is instantaneous
genes are CYP3A4, which can metabolize the atorvastatin and decrease its cellular concentration.
throughout the volume (V). Thus, atorvastatin induces its own metabolism.
Amount o drug in body/ V= C or
V= amount o drug in the body/ C
(Equation 2-2)
Adrugs volume o distribution there ore dose 6 to 8 hours later, with care ul monitoring o the patient and the nal
re ects the extent towhich it is present in 0.125-mg dose given 12 to 14 hours later.
extravascular tissues and not in the plasma.
. I the patients creatinine clearance was 20 mL/min, how would this a ect the
It is reasonable to viewVas an imaginary loading dose?
volume, because or manydrugs the
As shown in Equation 2-12 (p. 39), drug elimination rate is not a actor in determining
volume o distribution exceeds the known
the loading dose. T us, the loading dose or a patient with renal insu ciency would
volume o anyand all bodycompartments.
be the same or a patient with normal renal unction. However, the maintenance dose
For drugs that are bound extensivelyto would have to be adjusted or the decreased clearance o the digoxin.
plasma proteins but that are not bound to
tissue components, the volume o distribu- Additional in ormation and examples regarding individualizing dosing or a given
tion will approach that o the plasma vol- patient are provided in Appendix II o Goodman and Gilmans T e Pharmacological
ume because drug bound to plasma protein Basis of T erapeutics, 12th Edition.
is measurable in the assayo most drugs.
Certain drugs have high volumes o distribu-
tion even though most o the drug in the KEY CONCEPTS
circulation is bound toalbumin because Understanding and employing pharmacokinetic principles can increase the prob-
these drugs are alsosequestered elsewhere. ability o therapeutic success and reduce the occurrence o adverse drug e ects.
The volume o distribution mayvarywidely
Absorption o drug rom its site o administration into the central compart-
depending on:
ment is the rst step in the drug reaching its site o action; bioavailability is the
The relative degrees o binding to high- ractional extent to which an administered dose is absorbed and reaches its site
af nityreceptor sites, plasma, and tissue o action.
proteins
A er absorption, a drug typically distributes most rapidly to well-per used tis-
The partition coef cient o the drug in at
sues (liver, kidney, brain), and more slowly to muscle, most viscera, skin, and at.
Accumulation in poorlyper used tissues
issue distribution is also determined by physicochemical properties o a drug,
The volume o distribution or a given drug including lipid solubility, drug ionization, and the relative binding o drug to
can di er according to a patients: plasma proteins and tissue macromolecules.
Age
While many phase 1 reactions result in the biological inactivation o the drug,
Gender phase 2 reactions produce a metabolite with improved water solubility, a change
Bodycomposition that acilitates the elimination o the drug rom the tissue, normally via e ux
Presence o disease transporters.

34
Pharmacokinetics CHAPTER 2

Lipid-soluble drugs are not readily excreted until they are metabolized to more
polar compounds.
T e kidney is the most important organ or excreting drugs and their
metabolites.
DRUG CLEARANCE
Pharmacokinetic drugdrug interactions are among the leading causes o
adverse drug reactions (ADRs). Clearance romthe systemcirculation (CL)
is the most important concept to consider
When a constant drug dosage is given, steady state is not reached until at least when designing a long-termdrug dosing
our hal -lives have passed. rate that maintains steady-state concentra-
Genetic polymorphisms in membrane transporters and drug metabolizing tions o a drug (Css) within a therapeutic
enzymes can result in marked inter-patient responses to drugs, including serious window(see Chapter 1 or de nition o
drug toxicities. therapeuticwindow).
Assuming complete bioavailability, the
steady-state concentration o drug in
SUMMARY QUIZ the body(Css) will be achieved when the
rate o drug elimination (CL) equals the
QUESTION 2-1 T e term rst-pass e ect re ers to the rate o drug administration (dosing rate)
a. e ect a new drug has on the body a er its rst administration. as described in Equation 2-3.
b. time it takes or a drug to be detected in the urine or eces a er oral administration. Dosing rate= CLCss
c. ability o the intestines and liver to reduce the bioavailability o a drug.
(Equation 2-3)
d. time it takes or a drug to reach therapeutic concentrations in the target tissue.
e. initial e ect a drug has on target tissues.
Systems or elimination o drugs such as
metabolizing enzymes and transporters
QUESTION 2-2 T e term blood-brain barrier (BBB) re ers to a usuallyare not saturated, thus the
absolute rate o elimination o the drug
a. noncellular barrier that prevents drugs rom entering the CNS unless transported by
is essentiallya linear unction o its
speci c carriers.
concentration in plasma ( rst-order
b. cellular barrier that includes brain capillary endothelial cells that limits drug entry kinetics), where a constant raction o
into the brain. drug in the bodyis eliminated per unit
c. virtual or conceptual barrier that can explain the behavior o some drug e ects on o time.
the CNS. I mechanisms or elimination o a
d. physical barrier that prevents blood rom entering the brain. given drug become saturated, the
e. device that is used to prevent blood-borne drugs rom entering the brain. kinetics approach zero-order, in
which a constant amount o drug is
QUESTION 2-3 Phase 1 drug metabolism di ers rom phase 2 metabolism in that eliminated per unit o time, and the
relationship o clearance (CL) to drug
a. phase 1 metabolism always occurs prior to phase 2 metabolism.
concentration (C) is described by
b. phase 1 metabolism occurs in the intestine where orally administered drugs can rst Equation 2-4 which is analogous to the
be metabolized, whereas phase 2 metabolism occurs in the blood. Michaelis-Menten equation where Km
c. phase 1 metabolism occurs in the liver soon a er a drug is absorbed, whereas phase represents the concentration at which
2 metabolism occurs a er a drug is excreted into the urine. hal the maximal rate o elimination
d. phase 1 metabolic enzymes activate prodrugs, whereas phase 2 metabolic enzymes is reached (in units o mass/volume)
inactivate drugs. and vm is equal to the maximal rate o
elimination (in units o mass/time).
e. phase 1 metabolic reactions unctionalize drugs, whereas phase 2 metabolic reactions
conjugate drugs. CL= vm /(Km + C) (Equation 2-4)
QUESTION 2-4 T e most important pharmacokinetic concept to consider when Clearance o a drug (CL) is its rate o
designing a rational long-term pharmacotherapy is
elimination byall routes normalized to
a. bioavailability. the concentration o drug (C) in some
b. route o administration. biological uid where measurement can
c. volume o distribution. be made as described in Equation 2-5.
d. clearance. CL= rate o elimination/C
e. elimination t1/2. (Equation 2-5)
(continues)

35
SECTION I General Principles

DRUG CLEARANCE (Cont.) MEC for


a dve rs e
For a single dose o a drug with Pe a k e ffe ct re s pons e
complete bioavailabilityand rst-
order kinetics o elimination, systemic Ons e t of

)
clearance maybe determined rom

p
e ffe ct The ra pe utic

C
(
mass balance and the integration o window

t
c
e
f
Equation 2-5 over time as described in

f
E
Equation 2-6 where AUCis the total area

g
u
r
MEC for

D
under the curve o drug concentration
de s ire d
in the systemiccirculation as a unction Dura tion of a ction re s pons e
o time romtime zero to in nityas
shown in Figure 2-7.

CL= Dose / AUC


la g pe riod Time
(Equation 2-6)
FIGURE 2-7 Temporal characteristics o drug e ect and relationship to the therapeutic window
(eg, single dose, oral administration). A lag period is present be ore the plasma drug concentra-
tion (Cp) exceeds the minimum e ective concentration (MEC) or the desired e ect. Following
onset o the response, the intensity o the e ect increases as the drug continues to be absorbed
and distributed. This reaches a peak, a ter which drug elimination results in a decline in Cp and in
RATE OF DISTRIBUTION the e ects intensity. E ect disappears when the drug concentration alls below the MEC. Accord-
AND EQUILIBRIUM OF ingly, the duration o a drugs action is determined by the time period over which concentrations
DRUG INTO TISSUES exceed the MEC. An MEC exists or each adverse response, and i drug concentration exceeds
this, toxicity will result. The therapeutic goal is to obtain and maintain concentrations within the
The rate o distribution and equilibration therapeutic window or the desired response with a minimum o toxicity. Drug response below
o a drug into a tissue will depend on the the MEC or the desired e ect will be sub-therapeutic; above the MEC or an adverse e ect, the
ratio o the per usion o the tissue to the probability o toxicity will increase. Increasing or decreasing drug dosage shi ts the response
partition o drug into the tissue. curve up or down the intensity scale and is used to modulate the drugs e ect. Increasing the
dose also prolongs a drugs duration o action but at the risk o increasing the likelihood o
In manycases, groups o tissues with simi- adverse e ects. Unless the drug is nontoxic (eg, penicillins), increasing the dose is not a use ul
lar per usion-partition ratios all equilibrate strategy or extending the duration o action. Instead, another dose o drug should be given,
at essentiallythe same rate such that only timed to maintain concentrations within the therapeutic window. The area under the blood
one apparent phase o distribution is seen. concentration-time curve (area under the curve, or AUC, shaded region) can be used to calculate
the clearance (see Equation 2-6) or rst-order elimination. The AUC is also used as a measure o
I plasma and tissue reservoirs are in rapid bioavailability (de ned as 100% or an intravenously administered drug). Bioavailability will be
equilibrium, drug distribution will appear less than 100% or orally administered drugs, due mainly to incomplete absorption and rst-pass
as being in a singlecentralcompartment metabolism and elimination.
(see Figure 2-3).
Clearance o drug roma single central com-
partment occurs in a rst-order ashion, as QUESTION 2-5 A drug that is a weak acid (pKa = 5) is largely excreted unchanged by
de ned in Equation 2-5. the kidneys. o increase its rate o excretion
The decline o drug plasma concentration a. the pH o the urine should be increased.
(C) with time or a drug introduced into b. the pH o the urine should be decreased.
this central compartment is depicted in
c. the pH o the urine should be le unchanged.
Figure 2-8Aand is described byEquation 2-7
where kis the elimination rate constant. d. CYP liver enzymes should be induced to increase metabolism o the drug.
e. the patient should drink more water to increase urine output.
C= [dose / V][ekt]
(Equation 2-7)
SUMMARY QUIZ ANSWER KEY
The multicompartment model o drug
disposition can be viewed as though the QUESTION 2-1 Answer is c. A drug given orally must be absorbed rst rom the GI
blood and highlyper used lean organs tract, and the absorbed drug then passes through the liver, where metabolism and bili-
such as heart, brain, liver, lung, and kidneys ary excretion may occur be ore the drug enters the systemic circulation. Accordingly, a
cluster as a single central compartment, raction o the administered and absorbed dose o drug will be inactivated or diverted
whereas more slowlyper used tissues such in the intestine and liver be ore it can reach the general circulation and be distributed
as muscle, skin, at, and bone behave as the to its sites o action. I the metabolic or excretory capacity o the liver and the intestine
nal compartment (ie, the tissue compart- or the drug is large, bioavailability will be reduced substantially ( rst-pass e ect).
ment or reservoir; see Figure 2-3). T is decrease in availability is a unction o the anatomical site rom which absorption
(continues)

36
Pharmacokinetics CHAPTER 2

A B
32 32 RATE OF DISTRIBUTION
C op = 31 AND EQUILIBRIUM OF
N
N
o
C p
O
O
DRUG INTO TISSUES (Cont.)
I
I
T
T
16 16
A
A
o
V = Dos e / C
R
R
p
The decline o drug plasma concentration
T
T
N
N
with time or a multicompartment system
E
E
8 8
C
C
)
)
N
N
is shown in Figure 2-8B.
L
L
O
O
m
m
C
C
/
/
g
g
Two di erent terms have been used to


G
G
4 4
(
(
U
U
describe the volume o distribution or drugs
R
R
D
D
that ollowmultiple exponential decay.
A
A
M
M
2 2
Varea is the ratio o clearance to the
S
S
A
A

t1 /2 t1 /2
L
L
rate o decline in concentration during
P
P
1 1 the elimination ( nal) phase o the
0 2 4 6 8 10 12 0 2 4 6 8 10 12
logarithmicconcentration versus time
TIME (hours ) TIME (hours )
curve (see Equation 2-8).
FIGURE 2-8 Plasma concentration-time curves ollowing intravenous administration o a drug
(500 mg) to a 70-kg patient. A. Drug concentrations are measured in plasma at 2-hour intervals CL dose
Varea = =
ollowing drug administration. The semilogarithmic plot o plasma concentration (Cp) versus time k k AUC
appears to indicate that the drug is eliminated rom a single compartment by a rst-order pro- (Equation 2-8)
cess (see Equation 2-7) with a t1/2 o 4 hours (k = 0.693/t1/2 = 0.173 h -1). The volume o distribution
(V) may be determined rom the value o Cp obtained by extrapolation to t = 0 (Cp0 = 16 g/mL). The volume o distribution at steady-
Volume o distribution (see Equation 2-2) or the 1-compartment model is 31.3 L, or 0.45 L/kg state (Vss) represents the volume in
(V= dose/Cp0). The clearance or this drug is 90 mL/min; or a 1-compartment model, CL = kV.
which a drug would appear to be
B. Sampling be ore 2 hours indicates that in act the drug ollows multiexponential kinetics.
distributed during steadystate i the
The terminal disposition t1/2 is 4 hours, clearance is 84 mL/min (see Equation 2-6), Varea is 29 L
(see Equation 2-7), and Vss is 26.8 L. The initial or centraldistribution volume or the drug (V1 = drug existed throughout that volume
dose/Cp0) is 16.1 L. The example chosen indicates that multicompartment kinetics may be over- at the same concentration as that in
looked when sampling at early times is neglected. In this particular case, there is only a 10% error the measured plasma or blood (see
in the estimate o clearance when the multicompartment characteristics are ignored. For many Equation 2-9, where VCis the volume o
drugs, multicompartment kinetics may be observed or signi cant periods o time, and ailure distribution in the central compartment
to consider the distribution phase can lead to signi cant errors in estimates o clearance and and VTis the volume term or drug in
in predictions o the appropriate dosage. Also, the di erence between the centraldistribution
the tissue compartment).
volume and other terms re ecting wider distribution is important in deciding a loading dose
strategy. The idealized 1-compartment model discussed earlier does not describe the entire time
Vss = VC + VT
course o the plasma concentration. That is, certain tissue reservoirs can be distinguished rom
the central compartment, and the drug concentration appears to decay in a manner that can be (Equation 2-9)
described by multiple exponential terms (Figure 2-8B).
Changesin the pattern or ratioo blood ow
tovarioustissueschangeswill leadtochanges
in the rates o drug distribution totissues.
takes place; other anatomical, physiological, and pathological actors can in uence
bioavailability, and the choice o the route o drug administration must be based on an Disease states that cause altered
understanding o these conditions. Moreover, knowledge o drugs that undergo sig- regional blood owor reduced
ni cant metabolism or require active transport across the intestinal and hepatic mem- per usion can lead to altered e ects o
branes instructs our understanding o adverse events in therapeutics, because some drugs that varydepending on relative
drugs are substrates or the same drug metabolizing enzymes or drug transporters and per usion o speci ctissues.
thus compete or metabolism and transport.
Diseases such as hepatic cirrhosis may a ect rst-pass metabolism by shunting
o blood away rom hepatic metabolizing enzymes.

QUESTION 2-2 Answer is b. T e distribution o drugs into the CNS rom the blood
is unique. One reason or this is that the brain capillary endothelial cells have continu-
ous tight junctions; there ore, drug penetration into the brain depends on transcel-
lular rather than paracellular transport. T e unique characteristics o brain capillary
endothelial cells and pericapillary glial cells constitute the BBB. At the choroid plexus,
a similar blood-CSF barrier is present, except that it is epithelial cells that are joined by
tight junctions rather than endothelial cells. T e lipid solubility o the nonionized and
unbound species o a drug is there ore an important determinant o its uptake by the
brain; the more lipophilic a drug, the more likely it is to cross the BBB. T is situation

37
SECTION I General Principles

2 Ste a d y-Sta te
DOSING REGIMENS TO Atta ine d a fte r a pproxima te ly four ha lf-time s
ACHIEVE A STEADY-STATE Time to s te a dy-s ta te inde pe nde nt of dos a ge
DRUG CONCENTRATION
Cs s

N
Equation 2-3 indicates that a steady-

O
I
state concentration eventually will be

T
A
R
achieved when a drug is administered at

T
1

N
a constant rate, that is, when the rate o

E
C
Stea d y-Sta te Co nc e n tra tio n s
drug elimination equals the rate o drug

N
O
Proportiona l to dose /dos age inte rva l
availability.

C
Proportional to F/CL
With regular intermittent dosage the Flu c tu a tio n s
Proportiona l to dos e inte rva l/ha lf-time
concentration o drug rises with absorption Blunte d by s low a bs orption
and alls byelimination during each dosing 0
cycle, and at steadystate, the entire cycle 0 1 2 3 4 5 6
is repeated identicallyin each interdose TIME (multiple s of e limina tion ha lf-time )
interval (see Figure 2-9). FIGURE 2-9 Fundamental pharmacokinetic relationships or repeated administration o drugs.
Equation 2-10 provides an approxima- The blue line is the pattern o drug accumulation during repeated administration o a drug at
tion o the time required to reach steady intervals equal to its elimination hal -time when drug absorption is 10 times as rapid as elimina-
state a ter a dosage regimen is initiated tion. As the rate o absorption increases, the concentration maxima approach 2 and the minima
approach 1 during the steady state. The black line depicts the pattern during administration o
or changed (ie, our hal -lives to reach
equivalent dosage by continuous intravenous in usion. Curves are based on the 1-compartment
~94%o a newsteadystate) and a model. Average concentration (Css) when the steady state is attained during intermittent drug
means to estimate the appropriate administration:
dosing interval.
Fdosing rate = CLCss
t1/2 0.693 Vss / CL where F is ractional bioavailability o the dose and T is dosage interval (time). By substitution o
(Equation 2-10) in usion rate or Fdosing rate, the ormula is equivalent to Equation 2-3 and provides the con-
centration maintained at steady state during continuous intravenous in usion.
To maintain the chosen steady-state or
target plasma drug concentration (CP),
the dosing rate is adjusted according o en is used in drug design to alter drug distribution to the brain; or example, the
to Equation 2-11 (where Fis ractional so-called second-generation antihistamines, such as loratadine, achieve ar lower brain
bioavailability o the dose) such that concentrations than do agents such as diphenhydramine and thus are nonsedating (see
the rate o input equals the rate o loss; Case 2-1). Drugs may penetrate into the CNS by speci c uptake transporters normally
Equation 2-11 is a similar relationship involved in the transport o nutrients and endogenous compounds rom blood into the
to what was previously described in brain and CSF.
Equation 2-3.
Another important actor in the unctional BBB involves membrane transporters
Dosing rate = target Cp CL/F that are e ux carriers present in the brain capillary endothelial cell and capable o
(Equation 2-11) removing a large number o chemically diverse drugs rom the cell. MDR1 (P-gp) and
the organic aniontransporting polypeptide (OA P) are 2 o the more notable o these.
The dosing interval or intermittent dosing
T e e ects o these exporters are to dramatically limit access o the drug to the tissue
depends on the total uctuation in drug
expressing the e ux transporter.
concentrations that can be tolerated.
I the dosing interval Twere chosen
QUESTION 2-3 Answer is e. Drug metabolism or biotrans ormation reactions are
to be equal to the t1/2, then the total classi ed as either phase 1 unctionalization reactions or phase 2 biosynthetic
uctuation would be 2- old; this is o ten (conjugation) reactions.
a tolerable variation.
Phase 1 reactions introduce or expose a unctional group on the parent compound
I a drug is relativelynontoxicsuch such as occurs in hydrolysis reactions. Phase 1 reactions generally result in the loss
that a concentration manytimes that o pharmacological activity, although there are examples o retention or enhancement
necessary or therapycan be tolerated o activity. In rare instances, metabolism is associated with an altered pharmacological
easily, the maximal-dose strategycan activity. Prodrugs are pharmacologically inactive compounds designed to maximize
be used, and the dosing interval can be the amount o the active species that reaches its site o action. Inactive prodrugs are
much longer than the elimination t1/2 converted rapidly to biologically active metabolites o en by the hydrolysis o an ester
( or convenience). or amide linkage.

38
Pharmacokinetics CHAPTER 2

Phase 2 conjugation reactions lead to the ormation o a covalent linkage between


DOSING REGIMENS TO
a unctional group on the parent compound or phase 1 metabolite and endogenously
derived glucuronic acid, sul ate, glutathione, amino acids, or acetate. T ese highly ACHIEVE A STEADY-STATE
polar conjugates generally are inactive and are excreted rapidly in the urine and eces. DRUG CONCENTRATION
An example o an active conjugate is the 6-glucuronide metabolite o morphine, which (Cont.)
is a more potent analgesic than its parent (see Chapter 10). For drugs with a narrowtherapeutic
index, it maybe important toestimate
QUESTION 2-4 Answer is d. Clearance is the most important concept to consider when
the maximal and minimal concentrations
designing a rational regimen or long-term drug administration. T e clinician usually that will occur or a particular dosing
wants to maintain steady-state concentrations o a drug within a therapeutic window or interval (see the section Dosing Interval
range associated with therapeutic ef cacy and a minimum o toxicity or a given agent. or Intermittent Dosage in Chapter 2o
Assuming complete bioavailability, the steady-state concentration o drug in the body will Goodman&GilmansThePharmacological
be achieved when the rate o drug elimination equals the rate o drug administration (see Basiso Therapeutics, 12th Edition or a
Equation 2-3). Adjusting the dosing rate to achieve therapeutic steady-state concentra- detailed discussion and an example o
tion o drug thus requires an appreciation o the rate o clearance. When dosing rate is the calculations).
higher than clearance rate, drug will accumulate with possible toxic e ects. When dosing
rate is lower than clearance rate, subtherapeutic concentrations will result. The loadingdose is one or a series o doses
that maybe given at the onset o therapy
T e concept o clearance is extremely use ul in clinical pharmacokinetics because with the aimo achieving the target con-
its value or a particular drug usually is constant over the range o concentrations centration rapidly(ie, i the time required
encountered clinically. T is is true because systems or elimination o drugs such as to attain steadystate is long relative to the
metabolizing enzymes and transporters usually are not saturated, and thus the absolute need to achieve therapeuticconcentrations
rate o elimination o the drug is essentially a linear unction o its concentration in o drug).
plasma. T at is, the elimination o most drugs ollows rst-order kinetics, where a con- Equation 2-12 provides an estimate or
stant raction o drug in the body is eliminated per unit o time.
the magnitude o a loading dose.
QUESTION 2-5 Answer is a. o increase the rate o excretion o a drug that is a weak Loading dose = target Cp Vss/F
acid, the pH o the urine should be made more alkaline to increase the amount o (Equation 2-12)
drug in the urine that is ionized (see answer to Case 2-1c). In the proximal and distal
tubules, the nonionized orms o weak acids and bases undergo net passive reabsorp- Note that the loading dose o a drug is
tion. T e concentration gradient or back-di usion is created by the reabsorption o not a ected bydrug elimination and
water with Na+ and other inorganic ions. Because the tubular cells are less permeable thus is not routinelya ected byhepatic
to the ionized orms o weak electrolytes, passive reabsorption o these substances or renal insuf ciency.
depends on the pH. When the tubular urine is made more alkaline, weak acids are Disadvantages o using a loading dose
largely ionized and thus are excreted more rapidly and to a greater extent. When the include abruptlyexposing a patient
tubular urine is made more acidic, the raction o drug ionized is reduced, and excre- to what maybe a toxicconcentration
tion is likewise reduced. Alkalinization and acidi cation o the urine have the opposite o drug and the long time required
e ects on the excretion o weak bases. In the treatment o drug poisoning, the excre- or drug concentration to all i the
tion o some drugs can be hastened by appropriate alkalinization or acidi cation o the concentration is excessive.
urine. Whether alteration o urine pH results in a signi cant change in drug elimina- Therapeuticdrug monitoring to better
tion depends on the extent and persistence o the pH change and the contribution o estimate CL/Fmaybe use ul to re ne
pH-dependent passive reabsorption to total drug elimination. T e e ect is greatest or dosing using Equation 2-11 to achieve
weak acids and bases with pKa values in the range o urinary pH (5-8). However, alka- desired target plasma concentrations.
linization o urine can produce a 4- to 6- old increase in excretion o a relatively strong
acid such as salicylate when urinary pH is changed rom 6.4 to 8.0 and the raction o
When the goal o measurement is
nonionized drug is reduced rom 1% to 0.04%. adjustment o dosage, the sample should
be taken well a ter the previous dose,
that is, just be ore the next planneddose,
whenthe concentrationisat itsminimum,
except or drugs that act onlyduring the
initial period o each dosing interval.
Although our hal -lives are required to
achieve steady-state concentrations with
constant (nonloading) dosing, it may
be important to monitor be ore steady-
state has been achieved with drugs that
are toxicto minimize damage.

39
CHAPTER

3 Clinical and Environmental


Toxicity
Ch 3 Cl n l nd Env on n l ox y o l on o Ch 4, D ug
SELECT ANTIDOTES,
ox y nd Po on ng nd Ch 67 Env on n l ox ology: C nog n nd
CARCINOGENS, AND H vy M l o Goodman & Gilmans T e Pharmacological Basis o T erapeutics,
METALS INCLUDED IN 12 h Ed on. T h ology ( n lud ng h h n o on) o d ug
THIS CHAPTERa,b,c n on d n Ch 3 d u d n v ou o u qu n h .T M h -
Acetylcysteine (MUCOMYST) n o A on nd h Cl n l Su y l l d o h o only u d
n do , ox l , nd h l h l o h u d h u lly o
A atoxin B1
h vy l o on ng. In dd on o h l n d h , h 12 h Ed on
Arsenic n lud :
Atropine Ad l d d on o d ug ox y ng n n l
Benzo[a]pyrene Ad l d d on o d ug y ng nd l n l l n hu n
Cadmium Al o o n ou o no on l d o d ug ox y nd o on ng
Chromium Ad on o v ou hod o d on n ng o on d n
De erasirox(EXJADE) A d u on o nv on n l k n nd k n g n
De eroxamine (DESFERAL) A d l d d u on o nog n nd h o v n on
Dimercaprol A d u on o h n l o h n o l x ou
Dimercaptosuccinicacid (DMSA),
succimer (CHEMET) LEARNING OBJECTIVES
Diphenhydramine (BENEDRYL) Und nd h do - on l on h o d ug nd how u d o qu n-
EDTACaNa2 y y nd ox y.
Ethanol Und nd how d ug d o y nd y n hu n .
Flumazenil Know h d n y o h u d ug ox y.
Fomepizole D h d n y o d ug-d ug n on nd know how h y ly
Lead o d ug h y.
Mercury Know h n l o n g ng d ug o on ng n lud ng h u o
Naloxone (NARCAN) n do .
Penicillamine Know h ox yo u nd h on h vy l x ou .
Physostigmine Und nd h h n o on nd h u u o l h l o
Pralidoxime chloride (2-PAM) o h n o u l x ou .
Sodium2,3-dimercaptopropane sul onate
(DMPS)not approved byFDA, but
approved or use in Germany MECHANISMS OF ACTION OR TOXICITYOF SELECT ANTIDOTES, CARCINOGENS, AND
HEAVYMETALS
a
Acompletelist ofantidotesisshowninTable3-1;
b
examples ofimportant carcinogens are shown DRUG CLASS DRUG MECHANISM OF ACTION
inTable 3-2; c a list oftoxicmetals withfrequent Antidotes or Common Acetylcysteine Binds with the toxic metabolite o
environmental or occupational exposure is shown Acute Poisons acetaminophen
inTable 3-3.
Atropine Muscarinic receptor antagonist
(see Chapter 6)

Diphenhydramine H1 histamine receptor antagonist


(see Chapter 21)

De eroxamine Chelates iron

40
Clinical and Environmental Toxicity CHAPTER 3

DRUG CLASS DRUG MECHANISM OF ACTION

De erasirox Chelates iron

Ethanol Competitive inhibitor o alcohol


dehydrogenase (see Chapter 9)

Fomepizole Inhibitor o alcohol dehydrogenase


(see Chapter 9)

Flumazenil Benzodiazepine receptor antagonist


(see Chapter 9)

Naloxone Opiate receptor antagonist


(see Chapter 10)

Physostigmine Cholinesterase inhibitor


(see Chapter 6)

Pralidoxime (2-PAM) Cholinesterase reactivator (see


Chapter 6)

Carcinogens A atoxin B1 8,9-epoxide metabolite reacts with


amines in biological macromolecules
Forms DNA adducts (see Figure 3-11)

Benzo[a]pyrene Genotoxic carcinogen that orms


DNA adducts and reactive oxygen
species

Toxic Heavy Metals Arsenic As3+ orms covalent bonds with


sul ur and alters proteins
containing cysteine

Cadmium Induces the ormation o reactive


oxygen species through an unknown
mechanism

Chromium CrVI is taken into cells by anion


transporters where it is
reduced to CrIII
Criii orms covalent adducts with DNA

Lead Inhibition o calcium transporters and


channels that lead to alterations o
almost all neurotransmitter pathways
Inhibition o -aminolevulinate
dehydratase
Inhibition o errochelatase
(see Figure 3-8)

Mercury Hg 2+ and methylHg (MeHg) orm


covalent bonds with sul ur and alter
proteins containing cysteine

Heavy Metal Dimercaprol Forms chelation complexes between


Chelators its sul hydryl groups and metals

Sodium Forms chelation complexes between


2,3-dimercaptopropane its sul hydryl groups and metals
sul onate (DMPS)

EDTA CaNa2 Chelates divalent and trivalent metals

Penicillamine Chelator o copper, mercury, zinc,


and lead

Dimercaptosuccinic acid Forms chelation complexes between


(DMSA), Succimer its sul hydryl groups and metals

41
SECTION I General Principles

TABLE 3-1 Some Common Antidotes and Their Indications


ANTIDOTE POISONING INDICATION(S)
Acetylcysteine Acetaminophen

Atropine sul ate Organophosporus and carbamate pesticides

Benztropine Drug-induced dystonia

Bicarbonate, sodium Na+ channel blocking drugs

Bromocriptine Neuroleptic malignant syndrome

Calcium gluconate or chloride Ca2+ channel blocking drugs, Fluoride

Carnitine Valproate hyperammonemia

Crotalidae polyvalent immune Fab North American crotaline snake envenomation

Dantrolene Malignant hyperthermia

De eroxamine Iron

Digoxin immune Fab Cardiac glycosides

Diphenhydramine Drug-induced dystonia

Dimercaprol (BAL) Lead, mercury, arsenic

EDTA, CaNa2 Lead

Ethanol Methanol, ethylene glycol

Fomepizole Methanol, ethylene glycol

Flumazenil Benzodiazepines

Glucagon hydrochloride adrenergic antagonists

Hydroxocobalamin hydrochloride Cyanide

Insulin (high dose) Ca2+ channel blockers

Leucovorin calcium Methotrexate

Methylene blue Methemoglobinemia

Naloxone hydrochloride Opioids

Octreotide acetate Sul onylurea-induced hypoglycemia

Oxygen, hyperbaric Carbon monoxide

Penicillamine Lead, mercury, copper

Physostigmine salicylate Anticholinergic syndrome

Pralidoxime chloride (2-PAM) Organophosphorus pesticides

Pyridoxine hydrochloride Isoniazid seizures

Succimer (DMSA) Lead, mercury, arsenic

Thiosul ate, sodium Cyanide

Vitamin K1 (phytonadione) Coumarin, indanedione

42
Clinical and Environmental Toxicity CHAPTER 3

TABLE 3-2 Examples of Important Carcinogensa


CARCINOGEN
CLASS EXAMPLE SOURCE MECHANISM
Genotoxic

Nitrosamines Nicotine-derived Tobacco products Metabolic activation to orm DNA adducts


nitrosaminoketone (NNK)

Polycyclic aromatic Benzo[a]pyrene Fossil uel combustion, tobacco Metabolic activation to orm DNA adducts or ROS
hydrocarbons smoke, charbroiled ood

Aromatic amines 2-Aminonaphthalene Dyes Metabolic activation to orm DNA adducts

Fungal toxins A atoxin B1 Corn, peanuts, and other ood Metabolic activation to orm DNA adducts

Non-genotoxic

Liver toxicants Ethanol Beverages, environment Toxicity and compensatory proli eration; depletion o GSH

Phorbol esters Tetradecanoyl phorbol Horticulture; rubber and Activation o PKC iso orms
acetate gasoline production

Estrogens Diethylstilbestrol Drugs, environment Activation o estrogen-receptor signaling

Metals Arsenic Environment, occupation Inhibition o DNA repair; activation o signal transduction
pathways

Irritants Asbestos Environment, occupation Stimulation o in ammation; ormation o ROS

Dioxins TCDD Waste incineration, herbicides, Activation o the aryl hydrocarbon receptor
paper-pulp bleaching
a
Compounds in this table are classi ed as group 1 carcinogens by the International Agency or Research on Cancer (IARC), with the exception
o the phorbol esters, which have not been examined. TCDD, 2,3,7,8 tetrachlorodibenzo-p-dioxin; ROS, reactive oxygen species; GSH,
glutathione; PKC, protein kinase C.

TABLE 3-3 Toxic Metals with Frequent Environmental or Occupational Exposure a


COMMON SOURCE OF ORGAN SYSTEMS MOST
METAL CERCLA PRIORITY EXPOSURE SENSITIVE TO TOXICITY IARC CARCINOGEN CLASSIFICATION
As 1 Drinking water CV, skin, multiple other Group 1, carcinogenic to humansliver, bladder,
lung

Pb 2 Paint, soil CNS, blood, CV, renal Group 2A, probably carcinogenic

Hg 3 Air, ood CNS, renal Group 2B, possibly carcinogenic (MeHg +); group 3,
not classi able (Hg 0, Hg 2+)

Cd 7 Occupational, ood, Renal, respiratory Group 1, carcinogenic to humanslung


smoking

Cr6+ 18 Occupational Respiratory Group 1, carcinogenic to humanslung

Be 42 Occupational, water Respiratory Group 1, carcinogenic to humanslung

Co 49 Occupational, ood, water Respiratory, CV Group 2B, possibly carcinogenic

Ni 53 Occupational Respiratory, skin (allergy) Group 1, carcinogenic (soluble Ni compounds);


group 2B, possibly carcinogenic (metallic Ni)lung
a
The Agency or Toxic Substances and Disease Registry (ATSDR) has both detailed monographs and brie summaries or each o these
compounds, available at http://www.atsdr.cdc.gov. The International Agency or Research on Cancer (IARC) also has monographs available at
http://monographs.iarc. r. CERCLA, Comprehensive Environmental Response, Compensation, and Liability Act. CNS, central nervous system; CV,
cardiovascular.

43
SECTION I General Principles

100
A

,
g
n
i
d
n
o
p
Y
s
T
e
I
R
L
A
50

n
T
o
R
i
t
a
O
l
M
u
p
o
P
f
o
%
0
2 5 10 20 50

7.0 98
B
95
)
s
)
t
90
s
i
6.0
t
n
i
n
u
u
80
t
i
t
b
i
b
o
r
o
p
r
(
p
5.0 50
(
Y
Y
T
I
T
L
I
L
A
A
T
20
T
R
R
O
4.0
O
M
10
M
%
LD50 5

3.0 2
2 5 10 20 50
DOS E (mg/kg, log s ca le )

Co mpo und A
Co mpo und B

FIGURE 3-1 Dose-response relationships. A. The toxic response to a chemical is evaluated at


several doses in the toxic or lethal range. The midpoint o the curve representing percent o
population responding (response here is death) versus dose (log scale) represents the LD50, or
the concentration o drug that is lethal in 50% o the population. B. A linear trans ormation o
the data in panel A is provided by plotting the log o the dose administered versus the percent
o the population mortality, in probit units. For a discussion o probit units, see Chapter 4 in
Goodman and Gilmans The Pharmacological Basis of Therapeutics, 12th Edition.

7.0
98
90
)
80 EFFECTS
e
6.0
l
)
a
s
c
t
70
i
s
n
t
u
i
b
ED 60
t
o
i
b
r
p
DES IRABLE
o
5.0 50
(
r
UNDES IRABLE
p
(the ra pe utic)
g
(
40
n
e
i
d
s
LD
n
30
n
o
o
p
p
s
20
s
4.0
e
e
R
NON-DELETERIOUS DELETERIOUS
R
10
%
(s ide e ffe cts ) (toxic e ffe cts )
5
3.0 pha rma cologica l
2
10 20 50 100 200 800
pa thologica l
DOS E (mg/kg)
FIGURE 3-2 Comparison o e ective dose (ED), and lethal dose (LD). For a dis- ge notoxic
cussion o probit units, see Chapter 4 in Goodman and Gilmans The Pharmacologi-
cal Basis of Therapeutics, 12th Edition. Note that the abscissa is a logarithmic scale. FIGURE 3-3 Spectrum o the e ects o pharmaceuticals.
44
Clinical and Environmental Toxicity CHAPTER 3

CASE 3-1
An w h l o o d o h n o hy n on.
a. Describe how the e ects o the chemical are initially quanti ed?
Ev lu on o h do - on o h do - l on h u lly o-
n o ox olog . T g d d do - on l on h n n nd v du l
nd qu n l do - on l on h n h o ul on ( Ch 1 nd 2).
G d d do o d ug g v n o n nd v du l u u lly ul n g gn ud
o on h do n d. In qu n l do - on l on h , h
n g o h o ul on d n h do d; h l on-
h qu n l n h h d o h n o n n gv n
nd v du l ( F gu 3-1). T qu n l do - on h no non x ly
o n n ox ology nd u d o d n h d n l h l do (LD50) o
d ug nd o h h l . T LD50 o o ound d n d x n lly,
u u lly y d n on o h h l o o (o lly o n on -
lly) v l do n h l h l ng .
b. Describe how the chemicals toxicity and therapeutic index are determined?
F gu 3-2 llu h l on h w n qu n l do - on u v o h
h u o d ug o g n d n v do (ED50), h on n-
on o d ug wh h 50% o h o ul on w ll h v h d d on , nd
qu n l do - on u v o l h l y y h g n .T 2 u v n
u d og n h u nd x ( I), wh h qu n h l v y o d ug.
I = LD50/ED50
D ug how w d ng o I, o 1 o 2 o o h n 100. D ug w h low I
u d n d w h u on. Ag n h ll n o h go y n lud h
d gly o d d g l ( Ch 17) nd n h oh u gn (
Ch 45 nd 46). Ag n w h v y h gh I x ly nd n lud o
n o ( g, n ll n) ( Ch 39), unl h known ll g on .

GUIDELINES FOR THE STUDY OF NEW DRUGS IN HUMANS


PHASE OF STUDY EXPLANATION
Notice o Claimed Must be led once a drug is judged ready to be studied in humans. Includes:
Investigational In ormation o composition and source
Exemption or a Chemical and manu acturing in ormation
New Drug (IND) All data rom animal studies
Proposed clinical plans and protocols
Names and credentials o physicians who will conduct clinical trials
Compilation o key data relevant to study the drug in humans made available to investigators in Institutional Review Boards

Phase I Establish the e ects o the drug as a unction o dosage in 20-100 healthy volunteers:
Designed to prevent severe toxicity
Determine the probable limits o the sa e clinical dosage range
Pharmacokinetic measurements are o ten conducted in this phase

Phase II Per ormed in 100-200 patients with target disease:


Goal is to determine limited ef cacy and the doses to be used in subsequent trials

Phase III Per ormed in 1000 to 3000 patients:


To establish and con rm sa ety and ef cacy
Double-blind and crossover study designs are o ten used
Drug ormulated as intended or market

Phase IV Post-marketing surveillance


FDA requires drug manu acturers to per orm
FDA operates a voluntary reporting system available to physicians, pharmacists, and consumers
To identi y additional drug toxicity not apparent during the drug development process

45
SECTION I General Principles

TYPES OF THERAPEUTIC DRUG TOXICITYa


TYPE OF TOXICITY DESCRIPTION EXAMPLE
Dose-dependent Incidence and seriousness proportional to the Drug overdose is a dramatic example
Reactions concentration o the drug in the body and
the duration o exposure

Pharmacological Toxicity Progression o clinical e ects to toxic e ects CNS depression produced by barbiturates rom anxiolysis, to
dependent upon dose sedation, to somnolence, to coma
May also occur at therapeutic doses Phototoxicity associated with exposure to sunlight in patients
treated with tetracycline

Pathological Toxicity Drug in overdose produces a distinct Acetaminophen overdose producing hepatic necrosis
pathological reaction

Genotoxic E ects Injury to DNA leading to mutagenic or A atoxin exposure


carcinogenic toxicities

Allergic Reactions Result rom previous sensitization to Type I: Anaphylactic reactions mediated by IgE antibodies
a particular chemical or to one that is Type II: Cytolytic reactions mediated by IgG and IgM antibodies
structurally similar Type III: Arthrus reactions mediated predominantly by IgG
Type IV: Delayed hypersensitivity reactions mediated by sensitized
T-lymphocyctes and macrophages

Idiosyncratic Reactions An abnormal reaction to a chemical that is Genetically determined resistance to the anticoagulant action o
peculiar to a given individual. May take the war arin due to an alteration in vitamin Kepoxide reductase (see
orm o extreme sensitivity to low doses or Chapter 19)
extreme insensitivity to high doses o drugs
a
A complete discussion o the types o drug toxicity can be ound in Goodman and Gilmans The Pharmacological Basis of Therapeutics,
12th Edition, Chapter 4.

CASE 3-2
A 35-y -old wo n ng d o o h g l ux w h d ug h h n
n ly ov d y h Food nd D ug Ad n on (FDA). Af 2 on h o
h y h l ng nd n gh ly od o ux d n h d, u h
hy n ll o ll h h h d ug ng ll d u o o l w h
d hy h , nd n w d ug w ll d.
a. What is the process o drug approval by the FDA?
F w h n on - h d o h d ug d n ln l l h h k l .
F d l l w n h Un d S nd h l on d on qu h h udy
o n w d ug n hu n ondu d n od n w h ng n gu d l n . S
S d B GUIDELINES FOR HE S UDY OF NEW DRUGS IN HUMANS.
b. What are the di erent types o therapeutic drug toxicity?
In h u , d ug y lly odu nu ou , u u u lly only 1
ough h y go l o n; o o h oh und l
o h d ug o h h u nd on ( F gu 3-3). S d
o d ug u u lly oh o u no d l ou no do h y lw y n -
h d on nu on o h d ug; h y n lud u h d y ou h
o u ng w h yl n d n h y. O h und l y
h z d ox ( Sd B YPES OF HERAPEU IC
DRUG OXICI Y).
c. Why can a drug approved by the FDA or use in humans and still cause
serious toxicity?
So ox o h u l n d d d u on h known
h olog l h n ; how v , o n no un l h o k ng
od h h h u ox y o l o d ug o ully d.
(Continued)
46
Clinical and Environmental Toxicity CHAPTER 3

In h Un d S , h ov l y o n w d ug y lly u only 500


o 3000 x o d u j . Su h y l k ly o d n y ox o u ng
n 1% o o o n v ng d ug. T Adv Ev n R o ng Sy -
o h FDA l u on 2 gn l o d d ug v n . F , h FDA
qu (Cod o F d l R gul on , l 21, Volu 5, S on 314.80) d ug
nu u o o o k ng u v ll n o on d ug , nd
l gul on x o non on odu . S ond, h FDA o
volun y o ng y M dW h, v l l o o h h l h o on l
nd on u . Ho l y l o u o dv d ug v n o o
nv g o n l dv d ug v n , nd h nv g on y d
v l l o ndu y nd h gov n n . Un o un ly, ny n on l d w ll
gn n ly und h o d y nd o l y u l o dv
d ug v n u o und o ng nd u d ul o h
d no n o o o l n x o u o h v n o d on d ug
v l l on h o n k .
A n x l , o k ng u v ll n d n d h ox y o d
w h h o on n o - odul ng d ug d . T d ug w known
o nh n GI o l y nd w k d n h Un d S n o
g o o h g l f ux. Po k ng u v ll n v l d h d w
(Continued)

A
Me c hanis ms o f
Che mic al Inte rac tio ns

P HARMACO- P HARMACO-
KINETIC DYNAMIC

biotra ns forma tion non-re ce ptor

dis tribution re ce ptor

a bs orption

excre tion

B
Clas s ific atio n o f
Che mic al Inte rac tio ns

ADDITIVE

SYNERGISTIC

P OTENTIATION

ANTAGONIS M

functiona l

che mica l

dis pos itiona l

re ce ptor

FIGURE 3-4 Mechanisms and classi cation o drug interactions.

47
SECTION I General Principles

o d w h olong on o h Q n v l nd d o on o v n ul
hy h . I w w hd wn o h k , nd u qu n - on ol
ud d on d n d ko hy h .C d now l d
n d u on h ough n nv g on l og n g d y h
nu u .T u , h d n on o d ug ox y x nd yond h g
o d ug d v lo n nd ov l.

DRUGDRUG INTERACTIONS
TYPE OF INTERACTION DESCRIPTION EXAMPLE
Absorption One drug causes an increase or decrease in Ranitidine increases GI pH and may increase absorption o basic
(Pharmacokinetic) the GI absorption o another drug drugs such as triazolam

Protein Binding One drug displaces another rom protein The anticoagulant e ects o war arin may be enhanced by
(Pharmacokinetic) binding sites displacement rom plasma proteins by valproic acid therapy

Metabolism A drug in uences the metabolism o one Phenobarbital, phenytoin, and valproate increase the metabolism
(Pharmacokinetic) or several other drugs by enhancing or o carbamazepine
inhibiting hepatic CYPs Metabolism o carbamazepine is inhibited by erythromycin
(see Chapter 12)

Receptor Binding One drug binds to a receptor and prevents Buprenorphine binds opioid receptors with high af nity, and can
(Pharmacodynamic) the pharmacological action o another drug prevent euphoria rom concomitant use o abused opioid drugs
at that receptor

Therapeutic Action The therapeutic e ect o 1 drug impacts the When aspirin, an inhibitor o platelet aggregation, is given
(Pharmacodynamic) therapeutic e ect o another drug concomitantly with heparin, an anticoagulant, there may be an
increased risk o bleeding

CASE 3-3
A 38-y -old n ng d o zu d o d w h z n .
CLASSIFICATION Al hough h zu qu n y d d, v l o d dd d o u h du
OF DRUGDRUG h zu qu n y.
INTERACTIONS
a. Is a drugdrug interaction o concern in this patient?
Additivethe combined ef ect o 2 drugs
equals the sumo the ef ect o each agent P n o only d w h o h n 1 d ug, h v nd v du l d y
given alone ho , nd y l o u ng ov - h - oun (O C) d on , v n ,
nd o h n u l u l n .T oly h u ln u o h l h
Synergisticthe combined ef ect o qu on d on o o n l d ug n on ( F gu 3-4).
2 drugs exceeds the sumo the ef ects
o each drug given alone A d ug n qu n ly nf u n h ol o 1 o v l o h d ug (
Ch 2), nd h lly no l w h h CYP . A no h n
Potentiationthe creation o a toxic lly n o d y CYP2E1 o h ox ol NAPQI ( F gu 3-6).
ef ect o 1 drug due to the presence o In k o h nol, o n ndu o h 2E1 o nzy , y l d o n d
another drug u l y o no h n o on ng ov do . S l ly, nu o
Antagonismthe inter erence o 1 drug ond-g n on dn n h n ( n dn , zol ) w ov d
with the action o another drug o h k wh n h y w no d o l d o Q n v l olong on nd
Chemicalantagonismreactionbetween hydy hy h wh n o d n dw h ol d n o .
2 chemicals to neutralize their ef ects b. What are the di erent types o drugdrug interactions?
Dispositional antagonismalteration D ug n on g n lly nvolv d ug o on, o n nd ng, o-
o the disposition o a substance sothat l , o nd ng, o h u on ( S d B DRUGDRUG
less o the agent reaches the target organ IN ERAC IONS). Fu h d ugd ug n on n l d dd v ,
Receptor antagonismblockade o the yn g , o n on, o n gon ( S d B CLASSIFICA ION OF
ef ect o 1 drug with another drug that DRUGDRUG IN ERAC IONS).
competes at the receptor site

48
Clinical and Environmental Toxicity CHAPTER 3

CH3 OH H3 C CH2 OH
Me tha nol Ethano l H2 O 2
+ NADP H
NAD
+ O2
Fome pizole Alcohol Ca ta la s e
X de hydroge na s e CYP 2E1
NADH
+ NADP H 2H2 O
H+ O + H2 O
H2 C O H3 C CH
Forma lde hyde Ac e talde hyde
NAD+

Alde hyde
de hydroge na s e X Dis ulfira m
NADH
+ O O
H+
HC OH H3 C C OH
Fola te -de pe nde nt Formic a cid Ac e tic ac id
pa thway CoA + ATP

CO 2
+ Thiokina s e
H2 O
AMP + 2P ;

C
-hydroxy--me thyl CoA S C CH3 Trica rboxylic
gluta ryl CoA a cid cycle
Ac e tyl Co A

Fa tty a cids
Ke tone Chole s te rol
bodie s

FIGURE 3-5 Metabolism o ethanol and methanol.

CASE 3-4
A 20-y -old n go o oll g y. A on h yo o k h
un h o n unl l d k. An hou l h n x n ng n n n h d-
h nd lu d v on. H k n o h g n y oo wh d n d h
h h n o on d w h h nol.
a. How does methanol produce toxicity?
On on l ohol, h nol (CH 3OH), l o known hyl nd wood l ohol.
I n o n ndu l g n nd olv n ound n odu u h n
ov , h ll , nd n z ; h nol dd d o ndu l-u h nol o
k un o hu n on u on.
Sv ol do n d v lo du o h u ul on o o d(
F gu 3-5), nd h o yd on n v , lly n h on x
o o .T v u ld u n o dw h h nol n ox on o -
n n o h ln l u nd o u on qu n o nju y o g ngl on
ll o h n u d y h ol , o d, w h u qu n nf -
on, o hy, nd o n l l l l ndn .
b. What are the signs and symptoms o methanol poisoning?
M h nol o on ng on o h d h , GI d , nd n( lly l d o
n nju y), d ul y h ng, l n , nd lu d v on o d
w h hy o d k .T ln l u n l o n lud n o o
h n .
(Continued)

49
SECTION I General Principles

ac e tamino phe n
HNCOCH3

HNCOCH3 HNCOCH3

OH

CYP
s ulfa te glucuronide
NCOCH3

NAP QI
(toxic
inte rme dia te )

Gluta thione Nucle ophilic


Ce ll Ma cromole cule s
HNCOCH3 HNCOCH3

Gluta thione Ma cromole cule s


OH OH

Me rca pturic Acid Ce ll De a th

FIGURE 3-6 Pathways o acetaminophen metabolism and toxicity. The toxic intermediate
NAPQI is N-acetyl-p-benzoquinoneimine.

c. What are the treatment options or this patient?


l 3-1 l o on n do nd h nd on . M h nol dly
o d v h o l ou , nh l on, nd h ough h k n, w h h l 2
ou ng o l v n o ndu l ng . M h nol l o ol z d
( F gu 3-5) y ADH nd ALDH, w h d g ng on qu n . Co on
w n h nol nd h nol o ADH o h o h u o h nol n
n do n h nol o on ng. Fo zol (4- hyl y zol ), n ADH nh -
o , u ul n h nol nd hyl n gly ol o on ng.
Follow ng h nh on o ol o h nol w h h h nol o o -
zol , h od ly u d o v ly ov h n ng h nol.

CASE 3-5
A 16-y -old wo n ough o h g n y oo u h ook n ov do
o no h n. H v l gn no l nd h l x o ng
d ugh h v ng d u d .
a. What are the general principles o the management o a poisoned patient?
T jo y o o on ng x o u o d o US o on on ol n
judg d o non ox o only n lly ox . Wh n ox y x d, o do
o u, h o y o o on ng n o u o v l un on un l h
d ug o h l l n d o h ody. B u o h u on o on
nd n du on o on o o d ug , h n o o on ng u
o nd go l d d. T go l o o v l hy olog l un on
o n .T ond go l o k h on n on o o on n u
low o l y v n ng o on nd nh n ng l n on. T h d
go l o o h ox olog l o h o on h o .
(Continued)
50
Clinical and Environmental Toxicity CHAPTER 3

TABLE 3-4 Top Five Agents Involved in Drug-Related Deaths


Cocaine

Opioids

Benzodiazepines

Alcohol

Antidepressants

U.S. DHHS.

Knowl dg o h l v ox y o d ug h g nn ng o h o n g-
n o o on d n . T ollow ng l u hu n o on ng n -
v: l 3-4 l h o 5 g n nvolv d n d ug- l d d h ; l 3-5 l
h u n o qu n ly nvolv d n hu n o on ng x o u ; l 3-6
l h o on o d w h h l g nu o hu n l .
T ABC n on o gn y ( w y, h ng, ul on) o u-
l ly ugh nd l o h n o u o on ng, u w h h dd on
o l n D (d l y) nd E ( x o u ) ( l 3-7). In v , ndo -
h l nu on, h n l v n l on, h olog l lood u u o ,
nd/o x o o l ul o y u o y n y nd o .
A ully o n d d lh o y y l o v l l d on o
h l h gh l d n o on ng v n . O n, n o v on o
hy l y o nd gn y h only dd on l lu o o on ng d g-
no . G ou o hy l gn nd y o o dw h o on ng
ynd o known ox d o . l 3-8 d o only n oun d
ox d o .
T o y lly v l l u n d ug ox ology n uno y d gn d
od o on d ug o u u h h n , u , nzod -
z n , nn , o n , nd o .A u o on ng w h h u n
n u u lly d n d on l n l g ound , nd h ul o h y
n qu n ly v l l nough o gu d l z on. Add on lly, d on o
d ug o h ol on u n uno y do no n h h d d
d ug on l o h u n ly o v d o on ng lln . Wh n ng on
(Continued)

TABLE 3-5 Substances Most Frequently Involved in Human Poisoning Exposures


SUBSTANCE %
Analgesics 12.5

Personal care products 9.1

Cleaning substances 8.7

Sedatives/hypnotics/antipsychotics 6.2

Foreign bodies 5.1

Topical preparations 4.5

Cold and cough medications 4.5

Antidepressants 4.0

Data rom Bronstein AC, Spyker DA, Cantilena LR, et al. 2007. Annual Report o the American
Association o Poison Control CentersNational Poison Data System (NPDS): 25th annual report.
Clin Toxicol, 2008, 46:9271057.

51
SECTION I General Principles

TABLE 3-6 Poisons Associated with the Largest Number of Human Fatalities
Sedatives/hypnotics/antipsychotics

Acetaminophen

Opioids

Antidepressants

Cardiovascular drugs

Stimulants and street drugs

Alcohols

Bronstein AC, Spyker DA, Cantilena LR, et al. 2007. Annual Report o the American Association
o Poison Control CentersNational Poison Data System (NPDS): 25th annual report. Clin Toxicol,
2008, 46:9271057.

o no h n o n nno l ly x lud d v h x o u h o y,
u qu n on o h d ug o nd d. An l o d og (ECG)
y u ul d ng h lo k , N + h nn l lo k d , o K+ h nn l lo k-
d o dw h d on l . Fu h l o o y n ly , u h
u o lood g d n on , u h , o l lood oun , nd
oh ng, hould lo d o h nd v du l o on ng u n .
b. How are the principles o absorption, distribution, metabolism, and elimination
di erent or a drug a er excessive exposure than a er a therapeutic dose?
T n l o h ok n ( o on, d u on, ol , nd
l n on) d d n Ch 2. ox ok n (h h ok n o
d ug und u n h odu ox y o x v x o u ) yd
gn n ly o on ng, nd h d n y o oundly l n
d on nd ogno . Ing ng l g h n h u do o h u -
l y olong o on, l o n nd ng nd n volu o
d u on, nd h ng ol .
Wh n on on d w h o n l o on ng, 2 ox ok n qu on hould
o o n h l n n nd:
How long do n y o n n d o on o d (d ug o on
nd dyn )?
How long w ll k n n ox d n og (d ug l n on nd
dyn )?
(Continued)

TABLE 3-7 ABCDE: Initial Treatment Approach for Acute Poisoning


Airway Maintain patency

Breathing Maintain adequate oxygenation and ventilation

Circulation Maintain per usion o vital organs

Disability Assess or central nervous system dys unction


If neurological disability is noted, consider:
Oxygen administration (check pulse oximetry)
Dextrose administration (check blood glucose
concentration)
Naloxone administration (consider empiric trial)
Thiamine (for adult patients receiving dextrose)

Exposure Assess toxidrome (see Table 3-8)

52
Clinical and Environmental Toxicity CHAPTER 3

TABLE 3-8 Common Toxidromes


DRUG CLASS EXAMPLE(S) MENTAL STATUS HR BP RR T PUPIL SIZE OTHER
Sympathomimetic Cocaine, Amphetamine Agitation Tremor, diaphoresis

Anticholinergic Diphenhydramine, Delirium Ileus, ushing


Belladonna atropa

Cholinergic Organophosphates Somnolence/coma SLUDGE,a asciculation

Opioid Heroin, Oxycodone Somnolence/coma

Sedative-hypnotic Benzodiazepines, Somnolence/coma


Barbiturates

Salicylate Aspirin Con usion Diaphoresis, vomiting

Ca2+ channel blocker Verapamil

HR, heart rate; BP, blood pressure; RR, respiratory rate; T, temperature. aSLUDGE, muscarinic e ects o Salivation, Lacrimation, Urination,
De ecation, Gastric cramping, and Emesis.

c. How does acetaminophen produce toxicity?


A no h n ol z d o non ox glu u on d nd ul onjug , nd
o h ghly v ol N- yl-p- nzoqu non n (NAPQI) v CYP
oo . NAPQI d o olog lly v n d , nd u h
n d o n du o d ug ol . A h u do ng, NAPQI
nd o nu l o h l glu h on ; u , n ov do , glu h on d l on yl d
o h holog l nd ng o h n o ( F gu 3-6). T h ghly v
NAPQI ol nd ov l n ly o ll o ol ul , l d ng o dy un on
o nzy y nd u u l nd ol d y. Fu h o ,d l -
on o n llul GSH nd h h o y h ghly u l o ox d v
nd o o .
A u ov do n u v h d g , nd h nu o d n lo
d l o on ng w h no h n on nu o g ow. Ch on u o l h n
2 g/d y no y lly o dw hh dy un on, u ov u o no-
h n- on n ng n o nd ov - h - oun o n on odu k d n h
Un d S h l d o h gh n d w n o h o l y o ox y.
A u ng on o o h n 7.5 g o no h n, o u o u h -
u do , n ul n ox y. T o ou u dv o n ov -
do o no h n o n lly lh n o . R n l u ul n o
nd hy ogly o l o yo u .
In dul , h o ox y y o u ng on o ngl do o 10 o 15 g
(150-250 g/kg) o no h n; do o 20 o 25 g o o o n lly
l. Cond on o CYP ndu on ( g, h vy l ohol on u on) o GSH d l -
on ( g, ng o lnu on) n h u l y oh nju y, wh h
h n do u n d, l un o only, w h do n h h u ng .
Sy o h o u du ng h 2d y o u o on ng y no h n
f g d ( g, n u , do n l n, no x ) nd l h o n-
l ou n o h n ox on. Pl h n n o l v d,
o k dly o, g nn ng ~12 o 36 hou ng on. Cl n l nd -
on o h d g n w h n 2 o 4 d y o ng on o ox do ,
w h gh u o l n, nd h o g ly, j und , nd o gulo hy. R n l
n o nk n l lu y o u . L v nzy no l y lly
k 72 o 96 hou ng on. T on o h n h lo hy o wo n-
ng o gulo hy yond h nd oo ogno . B o y o h l v
v l n lo ul n o w h ng o h o l . In non l ,
h h l on v l ov od o w k o on h .
(Continued)
53
SECTION I General Principles

d. How is the diagnosis o acetaminophen toxicity established?


A no h n ov do on u d l g n y. S v l v d g
o u n 90% o n w h l on n on o no h n o h n
300 g/ L 4 hou ng on o 45 g/ L 15 hou h ng on o
h d ug. M n lh d g n n d wh n h d ug on n on
l h n 120 g/ L 4 hou o 30 g/ L 12 hou ng on.
e. How should acetaminophen poisoning be treated?
E ly d gno nd n o no h n ov do n l oo z
ou o . P h 10% o o on d n who do no v n
d v lo v lv d g ; 10 o 20% o h v n u lly d o h lu
d n n v u o v . A v d h o l, g v n w h n 4 hou o
ng on, d no h n o on y 50 o 90% nd h d
hod o g d on n on. G l v g g n lly no o nd d.
N- yl y n (NAC)( l 3-1) nd d o ho ko h nju y.
NAC h y hould n u d n u d o no h n o on ng
o lood on n on o v l l ,w h n n d y
ul u qu n ly nd h h ko h o ox y low. NAC un on y
d ox y ng NAPQI. I o h l GSH o nd y onjug d ly w h
NAPQI y v ng GSH u u .T o vd n h n o -
l h d no h n ox y, NAC y o g n x h nju y y n -
ox d n nd n - nf oy o . Ev n n h n o v d h o l,
h l o on o NAC, nd n h hould v d h o l vo d d no
NAC d n on d l y d u o on n o h o l-NAC n on.
Adv on o NAC n lud h ( n lud ng u , wh h do no qu
d ug d on nu on), n u , vo ng, d h , nd n hyl on .
An o l lo d ng do o 140 g/kg g v n, ollow d y h d n on o 70
g/kg v y 4 hou o 17 do . Wh v l l , h n v nou lo d ng do
150 g/kg y n v nou n u on n 200 L o 5% d x o ov 60 nu , ol-
low d y 50 g/kg y n v nou n u on n 500 L o 5% d x o ov 4 hou ,
h n 100 g/kg y n v nou n u on n 1000 L o 5% d x o ov 16 hou .
In dd on o NAC h y, gg v u o v w n d. T n lud
n g n o h nd n l lu h y o u nd n u on h n
o o und d. Hy ogly n ul o l v lu , nd l glu o
hould on o d lo ly. Ful n n h lu n nd on o l v
n l n on, nd l v n l n n hould on d ly n h ou
o n o n who d v lo v l v nju y d NAC h y.

CASE 3-6
A 2-y -old-g l ough o h g n y oo f ng ng o o h g-
n n o h on l .
a. How should iron poisoning be established in this patient?
A h o y o ng on y n qu n ly u n o n h y. A on-
on o h d gno d w h h u n o l on.
In h v lu on o h ld hough o h v ng d on, olo o on n
h g on n nd n g n yd n on o h on n on o on
n l n o d. I h l l h n 63 ol (3.5 g/L), h h ld
no n d d ng . How v , vo ng hould ndu d wh n h
on n h o h, nd n x- y hould k n o v lu h nu o ll
n ng n h ll ow l ( on l d o qu ).
L g oun o ou l ox , u l n dul . Mo d h
o u n h ld n, ul ly w n h g o 12 nd 24 on h . A l l 1
o 2 g o on y u d h, u 2 o 10 g u u lly ng d n l .T
qu n y o on o on ng l o v l l y n h hou hold, ul ly
(Continued)
54
Clinical and Environmental Toxicity CHAPTER 3

h u ly h n gn n y. T olo d ug o ng o ny o h
o lly v l l l gv h h n o ndy. All on -
on hould k n h ld oo o l .
b. What are the signs and symptoms o iron poisoning and how does iron produce
toxicity?
S gn nd y o o v o on ng y o u w h n 30 nu ng -
on o y d l y d o v l hou . T y n lud do n l n, d h , o
vo ng o own o loody o h on n on n ng ll . O ul on-
n llo o y no , l ud , d ow n , hy v n l on du o do ,
nd d ov ul oll . I d h do no o u w h n 6 hou , h y
n n od o n ov y, ollow d y d h n 12 o 24 hou . T o-
o v nju y o h o h y ul n ylo no o g ng. H -
o h g g o n nd h d g o n n nd ng u o y.
c. How should this patient be treated?
I on n h u GI n d y l v g w h od u on o
ho h olu on, l hough h l n l n qu on l . Wh n h l
on n on o on g h n h o l on- nd ng y (63 ol; 3.5
g/L), d ox n hould d n d. Sho k, d hyd on, nd d-
no l hould d n h onv n on l nn . Mo o n h
d o d gno nd h y. W h ly v n, h o l y o
on o on ng n du d o h gh 45% o low 1%.
D ox n ol d h on h l o Streptomyces pilosus nd
d h lly o o n h l- l g nd. D ox n h h d l
o o k ly h gh n y o on (K = 1031) ou l d w h
v y low n y o l u (K = 102). In v o, ov on o h o d n
nd n nd, o l x n, o n n. I on n h oglo n o y o-
h o no ov d y d ox n .
D ox n oo ly o d o l d n on, nd n l d n -
on qu d. Fo v on ox y ( u on l v l g h n 500 g/dL), h
n v nou ou d. T d ug d n d 10 o 15 g/kg/hou y on-
n n u on. F o n u on (45 g/kg/h) h v nu d n w ; d
olu u u lly o d w h hy o n on. D ox n y gv n n u-
ul ly n od ly ox ( u on 350-500 g/dL) do o 50 g/kg w h
x u do o 1 g. Hy o n on l o n o u w h h n u ul ou .
d. What are the toxic e ects o de eroxamine therapy?
D ox n u nu o ll g on , n lud ng u u , wh l , h,
nd n hyl x . O h dv n lud dy u , do n l d o o , d h ,
v ,l g , nd hy d . O on l o o on h v n
o d. D ox n y u n u o ox y du ng long- , h gh-do h y
o n u on-d nd n h l jo ; o h v u l nd ud o y h ng h v
nd d. A ul on y ynd o h n o d w h h gh-do (10-25
g/kg/h) d ox n h y; hy n , hy ox , v , nd o no h l
o n n y o . Con nd on o h u o d ox n n lud n l n u -
n y nd nu ; du ng gn n y, h d ug hould u d only l ly nd d.

CASE 3-7
A 48-y -old nh o un y- qu d n u on . H d no l
n ll n odu . W h n 15 nu o k ng h do , h d v lo gh n n
h h , nd nu n nd w ll ng o h l . H w ll g n y v ;h
d n h ho nd n o d o h ho l.
a. What are the di erent types o adverse reactions to medications?
T d n y o ox y o d ug n lud do -d nd n on h y
n x n on o d ug h u , holog l ox y, g no ox y,
(Continued)
55
SECTION I General Principles

ll g on , nd d o yn on ( Sd B YPES OF HERA-
PEU IC DRUG OXICI Y).
b. He does not remember ever-receiving penicillin be ore so what type o reaction
is this?
H yh v v d n ll n h ld nd no known wh h d on
w . An ll gy n dv on h ul o v ou n z on
o ul h l o o on h u u lly l . Su h on
d d y h un y . Fo low- ol ul -w gh h l o u
n ll g on, o ol odu u u lly h n, o n ng
w h n ndog nou o n o o n n g n o l x. Su h n g n ndu
h yn h o n od , u u lly l n od o l 1 o2w k .
Su qu n x o u o h h l ul n n n g n- n ody n on h
ovok h y l n on o ll gy. Do - on l on h u u lly
no n o h ovo on o ll g on . All g on h v
n d vd d n o4g n l go d on h h n o unolog l
nvolv n ( Sd B YPES OF HERAPEU IC DRUG OXICI Y).
An hyl x d d y IgE n od . T F o on o IgE n nd o -
o on ll nd o h l .I h F o on o h n ody ol ul h n
nd n g n, v ou d o ( g, h n , l uko n , nd o gl nd n )
l d nd u v od l on, d , nd n nf o y on . T
n g o h y o on h g on n l (GI) ( ood ll -
g ), h k n (u nd o d ), h o y y (hn nd
h ), nd h v ul u ( n hyl ho k). T on nd o o u
qu kly h ll ng w h n n g n o wh h h nd v du l h n n z d
nd d d hy n v y on .
c. What would be the treatment or this type o reaction?
T u o n h n o d ly d ug- ndu d n hyl x n o-
du d n Ch 7 o Goodman and Gilmans T e Pharmacological Basis o T era-
peutics, 12 h Ed on.

CASE 3-8
A 56-y -old wo n v ou ly d v lo d v h o o yo n f v ng
ul hox zol - ho x d-do odu (sept r a ) o u n y n -
on. Sh now n h ho l o h l n u g y. H ho l h h
k All g o sept r a on h on ov . Follow ng h u g y h d v lo
u n y n on nd g v n ba ct r im.
a. What is the issue with the administration o BACTRIM to this patient?
ba ct r im no h d n o x d-do o n on o ul hox zol -
ho ( Ch 38). So h hould on n h ba ct r im w ll u
h y o dv on sept r a . T y o d on o o v -
ou ly v n l nd o h l k o knowl dg y ho l ou h
d on ng d n d. In h , n ly h h y h
d d no know h h n h d h o yo o on o sept r a , o h
ba ct r im w l odu .
b. How can medication errors be avoided?
Ov h d d on d l n on h n g v n o h du on o d-
on o nd dv d ug v n (ADE ). M d on o n o u n ny
o h d on ng o u o , wh l ADE nju l d o
h u o nonu o d on . I l vd h d on o 50 o 100
o o on h n ADE . So ADE , u h v ou ly unknown ll g ,
un v n l , u o d on o n v n d. d on lly, h
5 R gh o d on d n on h v n ugh on ho lw d :
Right drug, right patient, right dose, right route, right time. (Continued)
56
Clinical and Environmental Toxicity CHAPTER 3

However, accomplishing a reduction in medication errors involves scrutiny o the


HEALTH EFFECTS OF
systems involved in prescribing, documenting, transcribing, dispensing, adminis-
LEAD TOXICITY
tering, and monitoring a therapy.
Neurotoxicity
c. How can medication errors be prevented?
Cognitive delays and behavior changes

Good medication use practices have mandatory and redundant checkpoints, such
in children
as having a pharmacist, a doctor, and a nurse all review and con rm that an ordered
dose o a medication is appropriate or a patient prior to the drugs administration. Neuromuscular de ectslead palsy
In such a system, medication errors occur only when several holes in the medica- Encephalopathy
tion administration sa eguards exist and are simultaneously aligned. Several practical Cardiovascular e ects
strategies have been suggested to reduce medication errors within hospitals and other Increased blood pressure
health care settings (see able 3-9), and these strategies are being constantly revised.
Renal e ects
Depressed glomerular fltration
CASE 3-9

Hematological e ects
A 3-year-old boy is brought to a medical clinic because of the recent onset of lethargy
and behavior changes. T e diagnosis of lead intoxication is made.
Hypochromicmicrocyticanemia
Immunosupression
a. How could this child have been exposed to lead?
Gastrointestinal e ects
In the United States, paint containing lead or use in and around households was
Severe intestinal painlead colic
banned in 1978, while the use o tetraethyl lead in gasoline was phased out and even-
tually eliminated between 1976 and 1996. T e economic bene t o the reduction in Carcinogenesis
lead exposure due to these 2 measures is estimated at hundreds o billions o dollars Probablycarcinogenicto humans
per year. Despite these bans, past use o lead carbonate and lead oxide in paint and cancers o lung, brain, kidney,
tetraethyl lead in gasoline remain the primary sources o lead exposure. Lead is not and stomach
degradable and remains throughout the environment in dust, soil, and the paint o
older homes. Young children of en are exposed to lead by nibbling sweet-tasting paint
chips or eating dust and soil in and around older homes. Renovation or demolition
o older buildings may cause substantial lead exposure. etraethyl lead was used as
an antiknock agent in gasoline, which resulted in high levels o lead in air pollution.
Removal o lead rom gasoline caused lead levels in air pollution to drop by more
than 90% between 1982 and 2002. Lead was commonly used in plumbing and can
leach into drinking water. Acidic oods and beverages dissolve lead when stored in
containers with lead in their glaze or lead-soldered cans, which was a signi cant
problem through the middle o the 20th century and remains a problem in develop-
ing countries. Lead exposure also has been traced to other sources such as lead toys,
(Continued)
TABLE 3-9 Best Practice Recommendations to Reduce Medication Administration
Errorsa
Short Term
Maintain unit-dose distribution systems for nonemergency medications
Have pharmacies prepare intravenous solutions
Remove inherently dangerous medications (eg, concentrated KCl) from patient care areas
Develop special procedures for high-risk drugs
Improve drug-related clinical information resources
Improve medication administration education for clinicians
Educate patients about the safe and accurate use of medications
Improve access of bedside clinicians to pharmacists
Long Term
Implement technology-based safeguards:
Computerized order entry
Computerized dose and allergy checking
Computerized medication tracking
Use of bar codes or electronic readers for medication preparation and administration

See Massachusetts Hospital Association.


a

http://macoalition.org/documents/Best_Practice_Medication_Errors.pdf

57
SECTION I General Principles

S uccinyl CoA + Glycine non-Western olk medicines, cosmetics, retained bullets, artists paint pigments,
ashes and umes rom painted wood, jewelers wastes, home battery manu acture,
-a minolevulina te syntha s e
and lead-type. Blood lead levels in the general population have steadily decreased
-Aminolevulina te (-ALA) since the 1970s. Between 1976 and 2002, mean blood levels in children 1 to 5 years o
age dropped rom 15 to 1.9 g/dL. T e Centers or Disease Control and Prevention
-a minolevulina te de hydra ta s e (CDC) recommends screening o children at 6 months o age and the use o aggres-
sive lead abatement or children with blood lead levels greater than 10 g/dL.
Porphobilinoge n
porphobilinoge n de a mina s e
Lead exposure occurs through ingestion or inhalation. GI absorption o lead var-
uroporphyrinoge n III cosyntha s e ies considerably with age and diet. Children absorb a much higher percentage o
ingested lead (~40% on average) than adults (<20%).
Uroporphyrinoge n III
uroporphyrinoge n de ca rboxyla s e
b. What is the mechanism o lead toxicity?
Lead toxicity results rom molecular mimicry o other divalent metals. Lead takes
Coproporphyrinoge n III the place o zinc or calcium in a number o important proteins. Because o its size
coproporphyrinoge n oxida s e and electron a nity, lead alters protein structure and can inappropriately activate
or inhibit protein unction. Speci c organ systems that are targets or lead are CNS,
Protoporphyrin IX cardiovascular, renal, hematopoietic, and gastrointestinal.
fe rroche la ta s e + Fe 2+ c. What are the health ef ects o lead poisoning?
Figure 3-7 shows the sites o action o the actions o lead on heme biosynthesis.
He me Lead causes a hypochromic microcytic anemia. Figure 3-8 shows the health e ects
Action produce d by le a d:
o lead poisoning in relation to the blood lead concentration.
Inhibition
Pos tula te d inhibition
d. What are the treatment options or this patient?
FIGURE 3-7 Heme biosynthesis and T e most important response to lead poisoning is removal o the source o lead
actions o lead. Lead inter eres with the exposure. Supportive measures should be undertaken to relieve symptoms.
biosynthesis o heme at several enzymatic Chelation therapy is warranted or children and adults with high blood lead levels
steps. Steps that def nitely are inhibited (>45 g/dL and >70 g/dL, respectively) and/or acute symptoms o lead poisoning.
by lead are indicated by black blocks.
Although chelation therapy is e ective at lowering blood lead levels and relieving
Steps at which lead is thought to act but
where evidence or this is inconclusive are immediate symptoms, it does not reduce the chronic e ects o lead beyond the bene t
indicated by blue blocks. o lead abatement alone. In rats, chelators enhance mobilization o lead rom the sof
tissues to the brain and may increase the adverse neurodevelopmental e ects o lead.
Chelators e ective in treating acute lead poisoning are listed in the Summary able
at the end o the chapter.

BLOOD LEAD (g/dL)


Childre n 150 Adults

de a th
100 e nce pha lopa thy
e nce pha lopa thy fra nk a ne mia
ne uropa thy
fra nk a ne mia longevity
colic 50 he moglobin synthe s is

40 pe riphe ra l ne uropa thie s


he moglobin infe rtility (me n)
urina ry coproporphyrin a nd -ALA ne phropa thy
ne rve conduction ve locity urina ry coproporphyrins a nd -ALA
30
sys tolic blood pre s s ure (me n)
vita min D me ta bolism he a ring a cuity
20 e rythrocyte protoporphyrin (me n)

e rythrocyte protoporphyrin (wome n)


e rythrocyte protoporphyrin
10
ne ura l deve lopme nt
de cre a s e incre a s e
1
FIGURE 3-8 Mani estations o lead toxicity associated with varying concentrations o lead in
the blood o children and adults. -ALA, -aminolevulinate
58
Clinical and Environmental Toxicity CHAPTER 3

CASE 3-10
A 12-month-old boy is brought to the hospital because of weakness, nausea, vomiting,
diarrhea, and dyspnea for the past week. His father admitted to bringing home elemen-
tal mercury so that his older children could shine silver coins. T e older children
admitted to pushing the liquid mercury around the table some of which fell onto the
carpet. So that it would not be detected the older children spread it around with their
feet. Neither the parents nor the 3 older children were ill. A 24-hour urine collected
from the toddler showed an elevated mercury concentration.
a. Why was the youngest child intoxicated when the older children or the adults
were not?
Most likely it is because he was crawling or close to the oor which would have the
highest concentration o mercury vapor.
Figure 3-9 shows the health e ects o mercury in relation to the concentration in
air and the corresponding urinary concentrations.
Metallic mercury (Hg0) vapor is readily absorbed through the lungs (~70-80%),
but GI absorption o metallic mercury is negligible. Once absorbed, Hg0 distributes
throughout the body and crosses membranes such as the blood-brain barrier (BBB)
and the placenta via di usion. Hg0 is oxidized by catalase in the erythrocytes and
other cells to orm Hg2+. Shortly a er exposure, some Hg0 is eliminated in exhaled
air. A er a ew hours, distribution and elimination o Hg0 resemble the properties
o Hg2+. A er exposure to Hg0 vapor, it is oxidized to Hg2+ and retained in the brain.
Inhalation o high levels o mercury vapor over a short duration is acutely toxic to
the lung. Respiratory symptoms o mercury exposure start with cough and tight-
ness in the chest and can progress to interstitial pneumonitis and severely com-
promised respiratory unction. Other initial symptoms include weakness, chills,
metallic taste, nausea, vomiting, diarrhea, and dyspnea. Acute exposure to high
doses o mercury is toxic to the CNS with symptoms, including tremor, emotional
lability, insomnia, muscular atrophy, parathesia, and cognitive def cits.
b. How can this exposure be limited?
It is unlikely that the metallic mercury can be removed by cleaning the carpet in
place. T e carpet will need to be removed rom the house and discarded properly.
(Continued)

Co nc e ntratio n o f Me rc ury
Air (g/m 3 ) Urine (g/L)
TARGET ORGAN EFFECTS
lung 1100 a cute a ffe cts : pne umonitis
ne rvous sys te m e re this m; gros s tre mors
ora l tis s ue s gingivitis
kidneys ne phrotic syndrome
le ns of eye 500 me rcuria le ntis

200
pe riphe ra l ne uropa thy
100 de cre a s e d ve rba l e nzymuria
ne rvous sys te m inte llige nce s core s
& kidneys
50 EEG cha nge s
(s lowe r &
tre mor a tte nua te d
25 re s pons e )

5 uppe r norma l ra nge of urine leve ls


FIGURE 3-9 The concentration o mercury vapor in the air and related concentrations o
mercury in urine are associated with a variety o toxic ef ects.

59
SECTION I General Principles

c. What are the treatment options or this patient?


W h x ou o ll u y, n on o x o u l nd -
o y u o y qu d. E y u d w h n 30 o 60 nu o o l
x o u o no g n u y, ov d d h n w k nd l nd h
no o o v nju y. T would no n y n h n who w o on d
w h u yv o .M n n n o l oly l n nd f u d o n o
n x o d o no g n u y. Ch l on h y n l n n
w h u no g n o ll u y x o u . Fo u y h l o ,
h Su y l h nd o h h .

CASE 3-11
A 33-y -old n, n ly vd o K ny , u d o h v ng l v n
du o ox n B1 x o u .
a. What are some common carcinogens? How do environmental contaminants
cause cancer?
F gu 3-10 how h h l d o n nv on n l nog n x o u
o nog n . l 3-2 l o o n nog n .
b. How do humans become exposed to excessive amounts o af atoxin B1?
Af ox n odu d y Aspergillus f avus, ungu h o on on -
n n o ood , lly o n, nu , o on d, nd nu . A. f avus un-
d n n g on w h ho nd w l , nd ul , h o llul no
ou o l n u o l nd o l g on o L n A ,A ,
nd Sou h A . Hu n x o u o f ox n n h Un d S v y
nd no hough o h v gn n on h l h.
c. What is the mechanism by which af atoxin B1 is thought to be carcinogenic?
T h n o f ox n nog n h n x n v ly ud d (
F gu 3-11). T 8,9- ox d o f ox n B1 d ly w h n n olog -
l o ol ul . Af ox n B1 8,9- ox d o ddu w h d oxygu no n
nd l u n h n d d n h lood o u n o hu n nd l o o y
n l x o d o f ox n, ov d ng v d n o h v yo h hw y n
v vo. Af ox n ly o DNA ddu d oxygu no n du , -
ng h h N1 o N7 o on. T N7-gu n n ddu w h d nn ,
l d ng o G n v on . Hu n f ox n x o u o dw hh -
o llul no ng n AGG o AG u on n odon 249 o h 53
(Continued)

Initiatio n Pro mo tio n

Ce ll
De ath
Me tabo lic
Ac tivatio n

Ca rcinoge n
DNA Adduc t Clo nal Ang io g e ne s is
Expos ure Abs o rptio n Mutatio ns Me tas tas is
Fo rmatio n Expans io n & Invas io n
(Ge notoxic)

Me tabo lic
De to xific atio n

DNA Re pair
FIGURE 3-10 Carcinogenesis: initiation and promotion. There are many steps that occur
between the exposure to a genotoxic carcinogen and the development o cancer. Processes
in gray lead to the development o cancer, while those in blue reduce the risk. Nongenotoxic
carcinogens act by enhancing steps leading to cancer and/or inhibiting protective processes.
A chemopreventive agent acts by inhibiting steps leading to cancer or by increasing protective
processes.
60
Clinical and Environmental Toxicity CHAPTER 3

Hydro xylate d GS H N-ac e tylc ys te ine


me tabo lite s (de to x) c o njug ate s (de to x) c o njug ate s
Oltipra z,
CYPs GTP s , ITCs GS Ts GS H
1A2, 3A4
Chlorophyllin O O O O

O DNA
O O
A. fla vus H
in fo o d DNA Adduc ts
Abs o rptio n CYPs
O 1A2, 3A4 O
CH3 CH3
P rima ry H O O O O
P re ve ntion Aflato xin B 1 Aflato xin B 1
in live r e po xide

Albumin

Albumin
adduc ts
FIGURE 3-11 Metabolism and actions o a atoxin B1. Following absorption, a atoxin B1 under-
goes activation by CYPs to its 8,9-epoxide, which can be detoxi ed by glutathione S-trans erases
(GSTs) or by spontaneous hydration. Alternatively, it can react with cellular macromolecules such
as DNA and protein, leading to toxicity and cancer. Oltipraz, green tea polyphenols (GTPs), and iso-
thiocyanates (ITCs) decrease a atoxin carcinogenesis by inhibiting the CYPs involved in activating
a atoxin and increasing the synthesis o the co actor GSH or GSTs involved in detoxi cation.

u o u o g n , ul ng n h l n o n gn n w h y n .
T u on l o n v o v d n g og h l g on w h l d
f ox n x o u .
Af ox n x o u nd h B v u wo k yn g lly o u h o llul
no . M ny o h g on w h l v d f ox n x o u l o h v h gh
l v l o nd h B n on. S ly, f ox n o h B x ou
n h ko h o llul no 3.4- o 7.3- old, v ly; ho
x o d o o h h v 59- old n d ko n o d o un x o d
nd v du l .
T n on w n f ox n nd h B h on l o h
n d n d n o h o llul no no w ll und ood. H
B nf u n h ol o f ox n B1 y u gul ng CYP , n lud ng 3A4,
nd d ng glu h on S- n v y. In dd on, h o llul ol -
on o d g don y h B n on n h l k l hood h
f ox n- ndu d DNA ddu w ll u u on . T h o ox nd u o -
o o ng o h B l o ould ov d o vo l nv on n
o h ol on nd nv on o n d ll .
d. What treatment options are available or a person likely to be exposed to af atoxin?
T l l on h w n f ox n ol ( F gu 3-11) nd -
nog n y k n l ng g o h o v n v g h od y
ol . Inh ng CYP v yo n ng glu h on onjug on w ll
du h n llul on n on o h 8,9- ox d nd hu v n DNA
ddu o on. Ol z, g n oly h nol (G P ), nd o h o y n
(I C ) d f ox n nog n y nh ng h CYP nvolv d n v -
ng f ox n nd n ng h yn h o h o o GSH o GS nvolv d n
d ox on ( F gu 3-11).
Ch o v n on o h o llul no n o l x o d o f ox n l o
n h v d yl ng n on w h h B. B u o h ong n -
on w nh B nd f ox n n nog n , h h Bv n
w ll du h n v y o o l o h ndu on o n y f ox n. P y
v n on o f ox n x o u h ough h nd o f uo n o ng o o o
ov ho w h ung l on n on n l o du hu n x o u . A o
o - v y v n on o h o ov ood o g o l h
d o A. f avus, wh h qu w nd hu d nv on n .
(Continued)
61
SECTION I General Principles

Y no h o h u d o h h o v n on o f ox n h o no-
gn h u o n o ol ul . Chlo o hyll n, n ov - h - oun
x u o w - olu l hlo o hyll l , nd gh ly o f ox n n h GI ,
o ng o l x h no o d ( F gu 3-11).

KEY CONCEPTS
Ph ology n w h ox ology wh n h hy olog l on o
d ug n dv .
T h u nd x ( I) qu n h l v yo d ug.
ox ok n (h h ok n o d ug und u n h o-
du ox y o x v x ou ) yd gn n ly f o on ng.
T o h l o v d nl o oy n l , wh n o ly qu l -
d, ly o hu n ox y.
Ex o u o x n l n l o ox g n n h gh do n y
nd v l d hod o d ov o l h z d o hu n x o d o u h
low do .
F d l l w n h Un d S nd h l on d on qu h h udy
o n w d ug n hu n ondu d n o d n w h ng n gu d l n (
S d B GUIDELINES FOR HE S UDY OF NEW DRUGS IN HUMANS).
M d on o l vd o 50 o 100 o o on h n
dv d ug v n .
T o yo o on ng n o u o v l un on un l h d ug o
h l l n d o h ody; n do un o only n d d.
W h nv on n l x o u , on h o on d o ul on x o u o
low-do ox n ov long od o .
T n o on o no l ll o l gn n y ul g o ,
nd xog nou h l n 1o o o h g .
M l n o n l o nv on n l ox n ; h y u qu ou nv -
on n l on n n h o o o h n u l nd n h o og n ou .
T o o n on o x ou o l o l n h ou o
h x ou .
n o u l n ox on of n nvolv h u o h l o .
Follow ng h on x o u o ox l , h l on h y do no how
ln l n yond ho o on o x o u lon , nd n o
y do o h h n good.

SUMMARY QUIZ

QUESTION 3-1 F v d ug h h v on o h h u nd x hown low.


Wh h d ug w ll qu h lo on o ng o vo d ox y?
. I=2
. I=5
. I = 50
d. I = 10
. I = 100

QUESTION 3-2 A o h w h 3 h ld n l h n h g o 5y on n d
ou g o v n o on ng n h h ld n. On v gy o
. ll h h ld n no o k d n unl gv n o h y n dul .
. k ll d n n h gh n .

62
Clinical and Environmental Toxicity CHAPTER 3

. k n h ll d n n h ld- n k g.
d. k ll l n ng odu und h k h n nk nd ll h h ld n no o o n
h doo .
. k d n h g g.

QUESTION 3-3 A 28-y -old n ough o h g n y oo . Phy l x -


n on v l h h h l v on, l on, u n on, d h , o h n,
vo ng, n dh , nd on d u l . I l k ly h h h n o-
on d w h
. h o n.
. n.
. h nol.
d. o g no ho h d .
. d z .

QUESTION 3-4 A 23-y -old wo n w h u d d ug ov do ough o h


g n y oo . On hy l x n on, h un on v nd h on
h llow. H n l h y hould o
. u .
. k n h h w ll oxyg n d.
. g ox ology od n wh o on h h ng d.
d. g h x- y.
. g n l o d og .

QUESTION 3-5 A 35-y -old n ng d w h C N 2ED A o u


x ou o
. u y.
. n .
. on.
d. o .
. l d.

QUESTION 3-6 Af h on x ou o l d, 43-y -old n hould


dw h
. ED A C N 2.
. d ol.
. d ox n .
d. u .
. non o h ov .

QUESTION 3-7 A 27-y -old wo n d gno d w h n o on ng. A l k ly


ou o h x o u
. d nk ng w .
. h y.
. h hu nd.
d. y h l n.
. d n l ll ng .

QUESTION 3-8 B nzo[ ] y n , k y nog n n o o ok , nog n


u o ddu w h
. n h n .
. DNA.

63
SECTION I General Principles

. y n .
d. l u n .
. do n .

SUMMARY QUIZ ANWSER KEY

QUESTION 3-1 An w a. T h u nd x ( I), qu n h l v yo


d ug. Cl ly, h h gh h o, h h d ug. I = LD50/ED50 ( F gu 3-2).
D ug how w d ng o I, o 1 o o h n 100. D ug w h low I u
d n dw h u on. Ag n h ll n o h go y n lud h d
gly o d d g l nd n h oh u g n . Ag n w h v y h gh I
x ly nd n lud o n o ( g, n ll n), unl h known
ll g on .

QUESTION 3-2 An w c. Po on ng v n on g y go z d ng
v , qu ng no h v o h ng on h o h nd v du l, o v , qu -
ng u n d d on o u ul. P v v n on g h o
v , nd v l y o v o on ng v n on d d n l 4-7
n Ch 4 o Goodman and Gilmans T e Pharmacological Basis o T erapeutics, 12 h
Ed on. On v gy o k n h ll d on n h ld-
n on n .

QUESTION 3-3 An w d. Of n, n o v on o hy l y o nd gn y
h only dd on l lu o o on ng d gno . G ou o hy l gn nd
y o o dw h o on ng ynd o known ox d o .
l 3-8 d o only n oun d ox d o .T n h h h -
gn nd y o o o g no ho h o on ng.

QUESTION 3-4 An w b. T
ABC n on o gn y o ul ly
ugh nd l o h n o u o on ng ( l 3-7). In v ,
ndo h l nu on, h n l v n l on, h olog l lood u u -
o , nd/o x o o l ul o y u o y n y nd o .

QUESTION 3-5 An w e. ED A C N 2 v o h n o u l d o-
on ng, ul ly n o n on w h d ol, u no n v h l o o
u yo n n v vo.

QUESTION 3-6 An w e. A h l o o ound h o l o l x w h


l, y lly 5- o 6- d ng . Fo on o o l x w n h l -
o nd l hould vn o v l nd ng o olog l l g nd . In
o u x o u o h gh do o o l , h l on h y du ox y.
How v , ollow ng h on x o u , h l on h y do no how l n l n
yond ho o on o x o u lon nd, n o , do o h h n
good. Ch l on h y y n h n u o ox o h vy l nd only
o nd d o u o on ng .

QUESTION 3-7 An w a. T y ou o x o u o n h ough


d nk ng w . A n n u lly l h ou o o l nd o k n o w ll nd ng
w .L v l o n n d nk ng w v g 2 g/L ( ) n h Un d S u
n o h n 50 g/L (5 h EPA nd d) n v w ll w , ul ly
n C l o n , N v d , nd A zon . D nk ng w o oh o h wo ld
wh w ll w h n o o d o vn w o n lln , ul ly w n,
Ch n , A g n n , Ch l , B ngl d h, nd n Ind , o on n d
w h u h h gh l v l o n (o v l hund d og l ),
nd w d d o on ng h v ul d.

64
Clinical and Environmental Toxicity CHAPTER 3

QUESTION 3-8 An w b. B nzo[ ] y n , k y nog n n o o ok , n


x l o g no ox nog n h o ohd DNA ddu nd v
oxyg n (ROS).

SUMMARYTABLE OF SELECT ANTIDOTES, HEAVY METALS, AND HEAVYMETAL CHELATORSa


TOXICITIES
CLASS AND
SUBCLASSES NAMES CLINICAL USES COMMON UNIQUE: CLINICALLY IMPORTANT
Antidotes Acetylcysteine Treatment o acute Nausea Rare anaphylaxis
or Common acetaminophen poisoning Rash
Acute Poisons

Atropine Treatment o poisoning Anticholinergic symptoms See Chapter 6


with organophosphate and (see Chapter 6)
carbamate pesticides

Diphenhydramine Treatment o drug-induced Sedation, dry mouth, dry See Chapter 21


dystonia eyes (see Chapter 21)

De eroxamine Parenteral treatment o acute Pruritus, wheals, rash, Anaphylaxis


and chronic iron poisoning dysuria, abdominal pain, Cataract ormation has been reported
diarrhea, and ever

De erasirox Oral treatment o chronic


iron overload

Ethanol Used to treat acute methanol See Chapter 9 See Chapter 9


and ethylene glycol poisoning

Fomepizole Used to treat acute methanol


and ethylene glycol poisoning

Flumazenil Treatment o the CNS See Chapter 9 See Chapter 9


depression produced by
benzodiazepines

Naloxone Treatment o opiate-induced See Chapter 10 See Chapter 10


CNS depression

Physostigmine Treatment o anticholinergic See Chapter 6 See Chapter 6


syndrome

Pralidoxime (2-PAM) Cholinesterase reactivator See Chapter 6 See Chapter 6


used in the treatment o
organophosphate poisoning

Toxic Heavy Lead See Figure 3-7 Cognitive delays and


Metals behavior changes, encephalopathy,
neuromuscular de cits (lead palsy),
elevated blood pressure, hypochromic
microcytic anemia (see Figure 3-8),
abdominal pain (lead colic), anorexia,
and constipation

Mercury See Figure 3-9


Tremors, emotional lability,
muscular atrophy,
parathesia, and cognitive de cits

Cadmium Local irritation along absorption route


Inhaled cadmium results in lung
irritation and pneumonitis
Nausea, vomiting, and diarrhea

65
SECTION I General Principles

TOXICITIES
CLASS AND
SUBCLASSES NAMES CLINICAL USES COMMON UNIQUE: CLINICALLY IMPORTANT

Chromium Tubular and glomerular renal injury


Inhaled chromium results in lung and
upper respiratory tract irritation and
decreased lung unction
GI pain, vomiting, and diarrhea

Arsenic Myocardial ischemia, cardiac


arrhythmia, peripheral vascular disease
(black oot disease)
Hyperkeratinization o the palms o
hands and soles o eet
GI cramping, vomiting, GI
hemorrhaging
Headache, lethargy, seizures, coma
Anemia and leukopenia

Heavy Metal Dimercaprol Used to treat acute exposures Increased blood pressure Pain ul sterile abscess at injection site
Chelators to arsenic, gold, mercury, and Nausea, vomiting, Contraindicated or treatment o
in combination with EDTA headache, burning chronic exposure to heavy metals
CaNa2 to treat lead exposure sensation in mouth and because it does not prevent
throat, and abdominal pain neurotoxic e ects

Sodium Treatment o acute exposure


2,3-dimercaptopropane to lead, arsenic, and mercury
sul onate (DMPS)

EDTA CaNa2 Treatment o acute Malaise, atigue, Rapid IVadministration causes


lead poisoning excessive thirst hypocalcemic tetany
Renal toxicity due to chelation o
essential metals, particularly zinc, in
proximal renal tubular cells
Used in pregnancy only under conditions
where bene ts outweigh risks

Penicillamine Used or treatment o Dryness and scaling o skin Long-term treatment results in
exposure to copper, mercury, cutaneous reactions such as urticaria,
zinc, and lead maculopapular reactions, pemphigoid
lesions, lupus erythematosus, and
dermatomyositis

Dimercaptosuccinic acid Approved or the treatment Nausea, vomiting, diarrhea, Transient elevations in hepatic
(DMSA), Succimer o children with acute lead and loss o appetite transaminases
poisoning
Used o label or treatment o
adults with lead, arsenic, and
mercury poisoning
a
carcinogens are not appropriate or this summary table. Only the toxicities o select heavy metals are shown.

66
CHAPTER

Special Populations
(Children and Elderly) 4
T ere is no speci c chapter on the topic o pharmacotherapy o special populations
PEDIATRIC
(children and elderly) in Goodman & Gilmans T e Pharmacological Basis of T era-
peutics, 12th Edition. However, this is an important area o clinical pharmacology PHARMACOKINETICS
because the pharmacotherapy o children and the elderly requires consideration o the ABSORPTION
di erences in pharmacokinetics and pharmacodynamics that can signi cantly a ect Absorption o drugs romthe gastrointes-
the sa ety and e cacy o drugs used in these special populations. Moreover, most ran- tinal (GI) tract is reduced in neonates and
domized controlled clinical trials exclude young children and the aged, which makes it changes with maturation making predic-
di cult or the clinician to make evidence-based decisions regarding appropriate drugs tion o medication bioavailabilityo orally
and dosing regimens to use in these patients. administered drugs verydi cult.
T e content o this chapter is drawn rom a variety o sources, including a number o Young children have higher gastricpH
chapters in Goodman & Gilmans T e Pharmacological Basis of T erapeutics, 12th Edi- than adults; adult levels o gastricacidity
tion including Chapters 1 to 3 in Section I: General Principles, and later chapters in are not reached until 3to7years o age.
which the pharmacotherapy o children or the elderly is discussed in the context o Acid-labile drugs (eg, penicillin,
speci c agents. Content regarding general principles o pharmacotherapy in these ampicillin, and na cillin) have greater
special populations is drawn rom online Updates published as part o the online version bioavailability.
o Goodman & Gilmans T e Pharmacological Basis of T erapeutics, 12th Edition related to Weakacids (eg, phenobarbital,
pediatric pharmacology (speci cally, T e History of Pediatric Drug T erapy: Learning from phenytoin) are ionized in the GI tract
Errors Not rials and Pediatric Pharmacokinetics: Why Kids Are Not Small Adults), and rom o the neonate and young child and
Hazzards Geriatric Medicine and Gerontology, 6th Edition (speci cally, Chapter 8 General thus are more slowlyabsorbed than
Principles o Pharmacology and Chapter 24 Appropriate Approach to Prescribing). in adults.
Neither a Mechanisms o Action able nor a Clinical Summary able is included in this
Weakbases(eg, penicillin, ampicillin,
chapter because this in ormation is provided or speci c agents in subsequent chapters.
anderythromycin) will be more quickly
In addition to the material provided here, Goodman & Gilmans T e Pharmacological
absorbedthaninthe adult GItract.
Basis of T erapeutics, 12th Edition contains:
Neonatesandin antshave prolongedrates
Appendix II with pharmacokinetic data or a number o drugs with di erences in
o gastricemptyingcomparedtoadults,
pharmacokinetic parameters that occur in children, the aged, and individuals with
withadult rateso gastricemptyingnot
speci c disease states
developinguntil 6to8monthso age.
Hazzards Geriatric Medicine and Gerontology, 6th Edition contains: Biliary unction develops over the rst
able 8-5 Changes in Pharmacokinetics and Pharmacodynamics with Aging and month o li e; the reduced levels o bile
Suggested Dose Adjustments or Older Patients acid salts and pancreaticenzymes in the
neonate mayreduce the absorption o
LEARNING OBJECTIVES lipophilicdrugs.
Describe the important pharmacokinetic and pharmacodynamic di erences -Glucuronidase and UDP-glucuronyl
between adults and children that can a ect sa ety and e cacy o drugs used in trans erase have higher activities in the
in ants and children. neonate GItract than in adults which
mayreduce drug absorption.
Know the FDAs role in providing in ormation to clinicians to improve sa e and
e ective use o drugs in young children, including breast- eeding in ants. The development o intestinal ora in
the neonate, which depends primarily
Describe the important changes in the pharmacokinetics and pharmacodynamics
on diet, can contribute to dif erences in
o drugs that occur in older adults.
drug metabolismcompared with adults.
Know the classes o medications that should be avoided in older adults because Dermal absorption is higher in neonates
o central nervous system (CNS) e ects. and in ants due to underdeveloped stra-
Know the steps that should be taken to optimize drug regimens in older adults. tumcorneumand increased skin hydration.
Intramuscularinjectionsare generallyavoided
inneonates, in ants, andchildrenbecause
intramuscularabsorptionisunpredictable due
todecreasedmuscle tone andcontraction,
andvariable blood owandoxygenation.

67
SECTION I General Principles

PEDIATRIC CASE 4-1


PHARMACOKINETICS In 1956, newborns receiving the antibiotic sul soxazole were ound to have a high
DISTRIBUTION incidence o kernicterus, which leads to yellow discoloration o the brain, seizures,
Volume o distribution (V) is important and death.
in children due to age-related changes in a. What is kernicterus and what causes it?
bodycomposition.
Kernicterus re ers to the yellow discoloration o the brain that is caused by high
Premature in ants have a higher concentrations o bilirubin in the brain. It is a condition only seen in young chil-
percentage o bodyweight that is water dren and can lead to brain damage and death i not treated.
(85%total bodywater) than term
Bilirubin is a yellow pigment that is the breakdown product o heme, 80% o which
in ants (75%total bodywater).
originates rom circulating hemoglobin and 20% rom other heme-containing pro-
Adult total bodywater (55%) is reached teins such as the CYPs. Bilirubin is hydrophobic, associates with serum albumin,
by12 years o age. and must be metabolized urther by glucuronidation to assure its elimination.
The Vo drugs that distribute in total

T e ailure to e ciently metabolize bilirubin by glucuronidation leads to elevated
bodywater and extracellular uid are serum levels and a clinical symptom called hyperbilirubinemia or jaundice. High
higher or in ants than adults. levels o ree bilirubin in the plasma can enter the brain causing kernicterus.
Body at increases with age and b. How did sul soxazole lead to kernicterus in these newborns?
development.
Bilirubin is metabolized by glucuronosyl trans erase (UG 1A1) and the glucuroni-
Premature in ants do not have

dated bilirubin is excreted in the urine. In ants have limited expression o glucuro-
appreciable body at compared to term nosyl trans erase and thus cannot e ciently metabolize bilirubin. Sul soxazole
in ants (16%body at). displaces bilirubin rom plasma proteins, thus increasing the ree raction o biliru-
Drugs that are at soluble have lower V

bin in the plasma and enhancing the movement o bilirubin into the brain. More-
in in ants and children than adults. over, in ants also have an immature BBB, which allows more o the ree bilirubin to
Premature in ants have increased
cross into and damage the brain.
membrane permeabilityallowing easier c. What other in ant drug toxicities have resulted rom low glucuronosyl trans er-
access o drugs into compartments such ase expression in newborns?
as the CNS.
In 1959, gray baby syndrome was described in premature in ants receiving the
In ants have an immature blood-brain

antibiotic chloramphenicol. T is syndrome was caused by chloramphenicol accu-
barrier (BBB), which allows toxic mulation and resulted in hypothermia, vomiting, acidosis, hypotension, cyanosis,
substances to more readilycross into a characteristic gray color, and death in some in ants. Because o immature gluc-
and damage the brain. uronosyl trans erase, in ants cannot metabolize chloramphenicol to the inactive
Protein binding, both 1-acid glycoprotein chloramphenicol glucuronate. In ants also have reduced renal capacity and dimin-
and albumin, are decreased in the neonate. ished renal excretion o chloramphenicol and its metabolites also contributed to the
Serumalbumin concentrations do not
accumulation o the drug in these children.
reach adult levels until 1 year o age. d. What other drugs have altered metabolism in children because o immature
Neonatal serumalbumin is 80%o that

phase 2 metabolic enzymes?
in adults and has decreased binding Morphine is metabolized by glucuronosyl trans erase to the 20- old more active
a nity or manydrugs. metabolite, 6-glucuronide morphine. T us, higher serum concentrations o mor-
Bilirubin and ree attyacids are present

phine are required or in ants to obtain e ective analgesia.
in higher concentrations in neonates
and compete with some drugs that bind
CASE 4-2
to albumin (see Case 4-1).
The reduced plasma protein binding

An 83-year-old man has occasional pain rom mild osteoarthritis. He also has
o drugs results in a larger raction o hypertension and is overweight.
ree drug in neonatal serumand thus a. What are the considerations in using a nonsteroidal anti-in ammatory drug
greater drug ef ect. (NSAID; Chapter 22) to treat this patients arthritis symptoms?
Epidemiologic and clinical studies have demonstrated an association between
NSAID use and GI bleeds and renal impairment in older persons. T e e ects o
NSAIDs on renal unction increase salt and water retention, which will contribute
to this patients hypertension. Age generally is correlated with an increased prob-
ability o developing serious adverse reactions to NSAIDs (see Chapter 22), and
caution is warranted such as choosing a lower starting dose or elderly patients.
(Continued)

68
Special Populations (Children and Elderly) CHAPTER 4

NSAIDs are labeled with a black box warning related to cardiovascular risks and
PEDIATRIC
are speci cally contraindicated ollowing coronary artery bypass gra (CABG)
PHARMACOKINETICS
surgery. Patients at increased risk o cardiovascular disease or thrombosis are
likely to be particularly prone to cardiovascular adverse events while on NSAIDs. EXCRETION
T is includes patients with rheumatoid arthritis as the relative risk o myocardial In ants and small children have reduced
in arction is increased in these patients compared to patients with osteoarthritis renal unction (20-40%o adult unction).
or no arthritis. T e risk appears to be related to actors inf uencing drug Adult ltration rates are not reached until
exposure, such as dose, t 1/2, degree o COX-2 selectivity, potency, and treatment approximately3 years o age.
duration. T us, the lowest possible dose should be prescribed or the shortest
Reduced renal unction in children is due to:
possible period.
Reduced glomerular ltration rate (GFR)
b. What are the alternatives to NSAIDs in this patient?
Tubular cell immaturity
Alternative approaches should be considered be ore NSAIDs are prescribed or
Reduced nephron length
indications such as osteoarthritis in elderly patients. Possible non-pharmacologic
approaches, such as gentle exercise and weight reduction, may be bene cial alterna- Reduced solute gradient
tives to treatment with NSAIDs. Decreased responsiveness to
When pharmacologic therapy is required, a drug therapy with a less-adverse event antidiuretichormone
pro le, such as acetaminophen, should be used. Even when normalized or bodysur ace area,
renal plasma ow, glomerular ltration,
CASE 4-3 tubular secretion, tubular reabsorption, and
the concentrating and acidi ying unctions
A 90-year-old woman develops symptoms o a cold and buys an over-the-counter cold o the kidneyare lowcompared with adults.
medication at the grocery store. T e medication contains diphenhydramine, acet-
Renal clearance o drugs excreted almost
aminophen, and phenylephrine. She takes the recommended adult dose but soon a er
entirelybyglomerular ltration (eg, ami-
taking the medication she becomes very con used and disoriented.
noglycosides and vancomycin) will change
a. What is likely causing the signs o con usion? in a manner that corresponds to matura-
Diphenhydramine is a rst-generation antihistamine that is a common ingredient tion o renal unction.
in over-the-counter cold and allergy medications (see Chapter 21). T is agent is Premature in ants have lower ltration
able to cross the BBB where it has signi cant anticholinergic e ects, including rates and are slowto develop the renal
con usion and somnolence. T e elderly have reduced BBB unction and are also at capacitythat termneonates will have
a higher risk o adverse drug e ects when taking drugs that have anticholinergic developed by1 weeko age.
properties. Premature in ants require lower doses,
b. What symptoms are associated with strong anticholinergic drugs in longer dosing intervals, or both to
older patients? maintain the same steady-state plasma
concentrations as the ull-termin ant.
In the elderly, drugs with strong anticholinergic properties are associated with
adverse e ects such as con usion, dry mouth, dry eyes, urinary retention, constipa- Excretion o drugs that depend on tubular
tion, and postural hypotension. secretion (eg, penicillins, sul onamides,
urosemide, and chloramphenicol) has
c. What other drugs have strong anticholinergic properties that should be avoided reduced rates o clearance in the neonate.
in elderly patients?
T e 2012 Beers Criteria (see Side Bar PO EN IALLY INAPPROPRIA E
MEDICA IONS FOR HE ELDERLY) lists the ollowing medications with strong
anticholinergic properties that should be avoided in older adults.
First-generation antihistamines, as a single agent or as part o a combination prod-
uct (see Chapter 21):
Brompheniramine
Carbinoxamine
Chlorpheniramine
Clemastine
Cyproheptadine

(Continued)

69
SECTION I General Principles

Dexbrompheniramine
PEDIATRIC
PHARMACKINETICS Dexchlorpheniramine
METABOLISM Diphenhydramine (oral)
In ants have reduced hepaticdrug metabo-
Doxylamine
lismenzymes.
Neonatal phase 1 activity( unctionaliza- Hydroxyzine
tion reactions catalyzed byCYPs, see Promethazine
Chapter 2) are intact but not at ull
capacityuntil 6 months to 1 year o age. riprolidine
As a result, oxidation o manydrugs is
Antispasmodics (see Chapter 6)
impaired in the neonate.
Belladonna alkaloids
Lacko one CYPenzyme maybe

compensated or byanother. Clidinium-chlordiazepoxide


Neonatal phase 2 activity(conjugation Dicyclomine
reactions, see Chapter 2) is 50 to 70%o
Hyoscyamine
the adult rate.
Glucuronidation is signi cantly Propantheline
reduced in the neonate and drug Scopolamine
glucuronidation is much less than
in the adult (eg, acetaminophen Antiarrhythmics (see Chapter 18)
glucuronidation). Disopyramide
The glucuronidation pathwaymaytake
ertiary tricyclic antidepressants ( CAs), alone or in combination (see Chapter 8)
up to 3 to 4 years to ullydevelop which
can slowmetabolismo endogenous Amitriptyline
substrates such as bilirubin and drugs
Chlordiazepoxide-amitriptyline
such as morphine and chloramphenicol.
Higher serumconcentrations Clomipramine
o morphine are required or Doxepin more than 6 mg/d
in ants because in ants cannot
metabolize morphine to the active Imipramine
6-glucuronide metabolite. Perphenazine-amitriptyline
The sul ation pathwayis well developed
rimipramine
in neonates and partlycompensates
or the lowrates o glucuronidation Antipsychotics (see Chapter 8)
(eg, acetaminophen is excreted in the
T ioridazine
urine primarilyas sul ate conjugates
in the neonate, whereas adults excrete Mesoridazine
acetaminophen in the urine primarilyas
Skeletal muscle relaxants (see Chapter 9)
the glucuronide conjugate).
Carisoprodol
Chlorzoxazone
Cyclobenzaprine
Metaxalone
Methocarbamol
Orphenadrine

70
Special Populations (Children and Elderly) CHAPTER 4

CASE 4-4 PEDIATRIC MEDICATION


A 72-year-old woman occasionally takes diazepam when she eels anxious and has SAFETY AND THE FDA
trouble sleeping. Her physician has prescribed diazepam or her since she was 50. Once a drug is approved or either adults
Recently the patient has had trouble maintaining her balance a er taking diazepam. or older children, it is requentlyused of -
label or other ages and indications.
a. What might be causing this patients di culty in maintaining her balance.
Pediatricproduct development studies,
Older adults have increased sensitivity to benzodiazepines (see Chapter 9), which
other than or vaccines or some childhood
increases the risk o alls, ractures, motor vehicle accidents, cognitive impairment,
illnesses such as otitis media, tend tobe
and delirium in older adults. Benzodiazepines and other hypnotic drugs increase pos-
small due tothe limited number o children
tural sway, which may contribute to this patients di culty in maintaining balance.
available or clinical trial participation.
b. What are the actors that increase benzodiazepine sensitivity in older adults? Postmarket pediatricexperience is
T e increased sensitivity o older adults to benzodiazepines is due to 3 actors: crucial to understand sa e use in the
(1) drug clearance is reduced; (2) there is increased distribution o benzodiazepines pediatricpopulation.
to the brain due to changes in lean body mass; (3) based on studies in animal Federal legislation in the past several
models, there is increased pharmacodynamic sensitivity o the brain to any given years has expanded the FDAa authority
concentration o benzodiazepine. to expand clinical studies o drugs in the
c. Are there benzodiazepines that are sa er in older adults? pediatricpopulation (see FDAPediatric
Product Development http://www. da.
Longer-acting benzodiazepines like diazepam are more likely to produce CNS
gov/Drugs/DevelopmentApprovalProcess/
adverse events such as daytime somnolence, con usion, and impaired motor coor-
DevelopmentResources/ucm049867.htm).
dination. Although short-acting benzodiazepines might reduce the risk o alls, all
benzodiazepines increase risk o cognitive impairment, delirium, alls, ractures, In 2012, as part o the Food and Drug
and motor vehicle accidents in older adults. Administration Sa etyand Innovation Act,
or FDASIA, Congress permanentlyreautho-
Benzodiazepines may be appropriate in some older adults to treat seizure disorders,
rized the Best Pharmaceuticals or Children
rapid eye movement sleep disorders, benzodiazepine withdrawal, ethanol with-
Act o 2002 (BPCA) and the Pediatric
drawal, severe generalized anxiety disorder, periprocedural anesthesia, and in end-
Research EquityAct o 2003 (PREA), and
o -li e care. I a benzodiazepine is determined to be appropriate in treating an older
gave FDAnewauthorities to encourage
adult, a prudent approach would be to avoid using agents with long elimination
more research into, and more development
hal -li e and high doses o benzodiazepine therapy.
o , treatments or children.
Nonbenzodiazepine hypnotics (ie, eszopiclone, zolpidem, zaleplon) are benzodi-
BPCAprovides an incentive or drug
azepine receptor agonists that have adverse e ects in older adults similar to those

companies to conduct FDA-requested


o benzodiazepines and have minimal improvement in sleep latency and duration.
pediatricstudiesbygrantingan additional
Chronic use (longer than 90 days) should be avoided with these agents.
6monthso marketingexclusivity.
d. What other medications have greater CNS ef ects in the elderly? PREArequires drug companies to study
Older adults are more sensitive to the e ects o opiates, anesthetic agents, and psy- their products in children under certain
chotropic agents including antipsychotics (see Side Bar CHANGES IN RESPONSE circumstances; when required, pediatric
O DRUGS WI H AGING). studies must be conducted with the
In the case o opiate analgesics, pharmacokinetics do not change with aging, but same drug and or the same use or
studies in animal models indicate increased sensitivity may be due to altered which theywere approved in adults.
expression o opiate receptors. In contrast, sensitivity to anesthetic induction doses FDAcan waive studies in children i
o thiopentone is increased in older adults due to drug distribution, not an age- the studies are not necessaryand can
related change in receptor sensitivity. Reduced clearance o thiopentone rom the grant extensions or de erred pediatric
central drug compartment produces higher drug concentrations in the CNS. studies at a sponsors request i there
is good cause or a delayin completing
KEY CONCEPTS the studies.
Be ore BPCAand PREAbecame law,
Neonates and young children have very di erent pharmacokinetics (drug more than 80%o the drugs approved
absorption, distribution, metabolism, and excretion) and pharmacodynam- or adult use were being used in children,
ics compared to adults, which can signi cantly reduce medication sa ety and even though the sa etyand ef ectiveness
e cacy. had not been established in children;
Older adults constitute a very heterogeneous group ranging rom healthy, t, todaythat number has been reduced
community-dwelling individuals taking no regular medication to rail institu- to about 50%.
tionalized individuals with multiple comorbidities and polypharmacy. (Continued)
T e most consistent and marked change in pharmacokinetics in older adults is
the increase in interindividual variability.
71
SECTION I General Principles

Reduced hepatic and renal clearance are generally the most signi cant
PEDIATRIC MEDICATION
pharmacokinetic changes in aging, which can a ect maintenance dose and
SAFETY AND THE FDA
dosing interval.
(CONT.)
Available methods to estimate renal unction in older adults are not reliable,
The FDASa etyand Innovation Act o 2012 particularly in rail or acutely ill older adults.
has had a dramaticimpact on pediatric
labeling o drugs, and labeling o drugs or T ere is an increased risk o adverse drug e ects in older adults.
breast- eeding women. When prescribing drug therapies in older adults, it is important to use the
FDAs O ce o PediatricTherapeutics (OPT) minimal dose required to obtain bene t.
was created bya mandate o Congress to Optimal drug therapy in the older patient should include:
assure access or children toinnovative, An assessment o the aims o therapy in the individual
sa e and ef ective medical products, and in
collaboration with another branch o the Risks o drug therapy
agency(Center or Drug Evaluation and Possible drugdrug interactions
Research [CDER]) developed the NewPedi- T e e ect o disease on drug e ects
atricLabeling In ormation Database which
contains pediatriclabeling changes based Monitoring o drug e cacy and adverse drug e ects once drug therapy
on keypediatricin ormation romthe stud- is initiated
ies submitted in response toPREAand BPCA
(http://www.accessdata. da.gov/scripts/sda/ SUMMARY QUIZ
sdNavigation.c m?sd=labelingdatabase).
In 2014, FDApublished a nal rule setting QUESTION 4-1 T e dosing o morphine in an in ant to control pain typically requires
standards or howin ormation about medi- using higher doses than an adult. T e major reason or the higher dosing is
cines used during pregnancyand breast- a. increased binding o drug to plasma proteins.
eeding is presented in the labeling o
prescription drugs and biological products. b. reduced entry o drug into the CNS.
This labeling replaces the previous c. reduced opiate sensitivity in young children.
product letter categoriesA, B, C,D, d. reduced metabolism o drug.
and Xused toclassi ythe risks o using e. increased clearance o drug.
prescription drugs during pregnancy.
QUESTION 4-2 T e bioavailability o orally administered drugs is di cult to predict in
The nal rule requires the use o 3
subsections in the labeling titled in ants because
Pregnancy,Lactation,andFemales a. the rate o transit o drugs through the in ant GI tract is very rapid.
and Males o Reproductive Potential b. in ants have very high rst-pass metabolism.
that provide details about use o the c. absorption o drugs rom the GI tract is reduced in neonates and changes with
drug or biological product. maturation.
ThePregnancyandLactation d. the clearance o drugs by the kidney is very rapid in in ants.
subsections also includes 3
subheadings:risksummary,clinical e. all o the above.
considerations,anddata. QUESTION 4-3 Nonsteroidal anti-inf ammatory drugs (NSAIDs) such as ibupro en
should be avoided in the elderly because NSAIDs
a. have little e cacy in treating pain in older adults.
b. are more readily converted to toxic metabolites in older adults.
c. impair renal unction in older adults.
d. impair hepatic unction in older adults.
e. impair uptake o many nutrients rom the GI tract in older adults.

QUESTION 4-4 Certain tricyclic antidepressants such as amitriptyline should be


avoided in elderly patients because they
a. have anticholinergic e ects.
b. have e ects on the heart that increase the risk o arrhythmias.
c. cannot be adequately monitored or adherence.
d. can cause serious GI bleeding.
e. are ine ective in most elderly patients.
72
Special Populations (Children and Elderly) CHAPTER 4

12
AUC IV
PHARMACOKINETICS
IN THE ELDERLY
AUC EV (young)
10 ABSORPTION AND
AUC EV (old)
BIOAVAILABILITY
n
o
8 Absorption o drugs is o ten slower in
i
t
a
r
the aged (see Figure 4-1) so the maximal
t
n
e
plasma concentration is reached later (lon-
c
6
n
o
ger Tmax) and is lower (lower Cmax).
C
a
The extent o absorption is usuallycom-
m
4
s
plete in older adults so the area under the
a
l
P
curve (bioavailability) is not af ected.
2 Oral bioavailabilitymaybe decreased or
drugs that require an acidicenvironment
0 in older adults with age-related hypo-
0 2 4 6 8 10 chlorhydria secondarytoatrophicgastritis
Time (5-10%o older adults) and in those taking
medications that raise gastricpHsuch as
FIGURE 4-1 Theoretical plasma concentrationtime curves or the same drug given intravas-
H2-antagonists and proton pump inhibitors.
cularly (IV) and extravascularly (EV) in a young adult versus an elderly adult. The bioavailability is
the ratio o the area under the curve (AUC) or the EVroute to the AUC or the IVroute. The intra- Factors associated with age-related
vascular AUC is not a ected by age. With aging, the extravascular curve shows delayed and lower changes in absorption and bioavailability
maximal plasma concentration and AUC may be increased or drugs that undergo rst-pass are shown inTable 4-1.
hepatic metabolism. (Reproduced with permission from Halter JB, Ouslander JG, Tinetti ME, Studenski
S, High KP, &Asthana S (Eds). Hazzards Geriatric Medicine and Gerontology, 6th ed. McGraw-Hill,
Inc., 2009. Fig 8-1.)

TABLE 4-1 Factors Associated with Bioavailability o Drugs Administered Through Common Extravascular Routes and
Description o Age-Related Changes
ABSORPTION FIRST-PASS CLEARANCE
AGE RELATED
ROUTE PROPERTIES OF DRUG DESCRIPTION AGE RELATED CHANGES DESCRIPTION CHANGES
Oral Particle size ormulation Gut lumen Gastric pH may be less acidic, Gut wall CYP450 Not known
Lipid solubility decomposition altering ionisation metabolism
Ionization

Passive absorption Complete but slower Hepatic metabolism Reduced


Increased by longer 3050%
gastrointestinal transit time
and possible increase in
permeability o epithelium
Decreased by reduced
per usion

Active transport (e.g., Probably decreased


iron, vitamin B-12)

Gut wall Not known


P-glycoprotein
transports drugs back
into lumen

Sublingual Particle size Rapid into blood Not known ?Reduced Nil
Lipid solubility vessels at base o per usion
Potency tongue

Rectal (local and Lipid solubility Varies with rectal Not known Nil
systemic action) Ionisation contents ?Reduced per usion

(Continued)

73
SECTION I General Principles

ABSORPTION FIRST-PASS CLEARANCE


AGE-RELATED
ROUTE PROPERTIES OF DRUG DESCRIPTION AGE-RELATED CHANGES DESCRIPTION CHANGES

Subcutaneous Particle size (small particles Slow Not known Nil


Injection absorbed by capillaries, ?Reduced per usion ?Proteolysis o protein
large particles absorbed by ?Changes to lymphatics drugs in lymph nodes
lymphatics)
Protein complex
pH
Use o vasoconstrictors

Intramuscular Lipid solubility Slow ( aster than Not known Nil


Injection Particle size (small particles subcutaneous due to ?Reduced per usion ?Proteolysis o protein
absorbed by capillaries, better per usion) ?Changes to lymphatics drugs in lymph nodes
large particles absorbed by
lymphatics)

Percutaneous Lipid solubility Slow Not known Nil


Heat dependent ?Reduced per usion

Intranasal (local Lipid solubility Variable Not known Nil


and systemic) ?Reduced per usion

Inhaled (local Particle size Minimal systemic Not known Lung metabolism and Not known
and systemic) Powders absorption ?E ects o reduced alveolar clearance
Aerosol solutions inhaler area, low-grade inf ammation,
(type and how used) ventilation/per usion
Gases: gas partition mismatch, decreased di usion
coe cient (blood) and transport across alveolar
capillary membrane

Ophthalmic Formulation: drops, Minimal (drainage Not known Nil


(topical) suspensions, ointments through nasolacrimal
canal)

Reproduced with permission rom Halter JB, Ouslander JG, Tinetti ME, Studenski S, High KP, &Asthana S (Eds). Hazzards Geriatric Medicine and
Gerontology, 6th ed. McGraw-Hill, Inc., 2009. Table 8-1.

PHARMACOKINETICS SUMMARY QUIZ ANSWER KEY


IN THE ELDERLY
DISTRIBUTION QUESTION 4-1 Answer is d. Morphine is metabolized by glucuronosyl trans erase to
the 20- old more active metabolite, 6-glucuronide morphine. T e phase 2 glucuronida-
Age-related changes in plasma protein tion pathway is signi cantly reduced in the neonate and as a result, drug glucuronida-
binding, tissue binding properties, and tion is much less than in the adult. T e glucuronidation pathway may take up to 3 to 4
body at/body water ratios can alter years to ully develop. T us, higher serum concentrations o morphine are required or
the V(Table 4-2). in ants to obtain e ective analgesia.
The BBBmaybecome less ef ective
QUESTION 4-2 Answer is c. Absorption o drugs rom the GI tract is reduced in
with aging.
neonates and changes with maturation making prediction o bioavailability o orally
Tissue per usion maydecrease with aging, administered drugs very di cult (see Side Bar PEDIA RIC PHARMACOKINE ICS
which mayslowdistribution, particularly ABSORP ION). Some o the actors that alter absorption o drugs in the GI tract that
to less highlyper used tissues such as change with age during the rst several years o li e include gastric pH, gastric emp-
muscle and at. tying time, biliary unction, metabolic enzymes in the GI tract, and development o
It maybe necessaryto change loading intestinal f ora.
dose o drugs in older adults because
o changes in V.

74
Special Populations (Children and Elderly) CHAPTER 4

QUESTION 4-3 Answer is c. Epidemiologic and clinical studies have demonstrated an


PHARMACOKINETICS
association between NSAID use and GI bleeds and renal impairment in older persons
(see Case 4-2). Age generally is correlated with an increased probability o develop- IN THE ELDERLY
ing serious adverse reactions to NSAIDs, and caution is warranted such as choosing a METABOLISM AND
lower starting dose or elderly patients and limiting use to the shortest time possible. CLEARANCE
NSAIDs are labeled with a black box warning related to cardiovascular risks and are Anumber o age-related changes in
speci cally contraindicated ollowing coronary artery bypass gra (CABG) surgery. hepaticblood owand unction alter
QUESTION 4-4 Answer is a. ertiary tricyclic antidepressants should be avoided in metabolismo drugs in the elderly
older adults because they have strong anticholinergic e ects (see Case 4-3). In the (Table 4-3).
elderly, drugs with strong anticholinergic properties are associated with adverse Hepaticblood owisreducedby~50%with
e ects such as con usion, dry mouth, dry eyes, urinary retention, constipation, and normal aging, which reduces clearance o
postural hypotension. ow-limiteddrugsby~50%but hasnoefect
on capacity-limited drugs (see Figure 4-2).
Hepaticblood owmayalso be reduced in
diseases prevalent in the elderlysuch as
TABLE 4-2 Factors That Determine Volume o Distribution: Ef ects o Aging heart ailure.
AGE RELATED Phase 1 metabolismis reduced by30 to
FACTOR EFFECT ON V CHANGES CLINICAL APPLICATION 50%in normal aging.
Plasma protein Highly protein bound Decreased albumin Not usually clinically Phase2metabolismappearstobemaintained
binding drugs are generally less which binds acidic signi cant. in normal aging, but reduced in rail aging.
able to cross membranes drugs, eg, war arin, In older adults with reduced hepatic
and have smaller V NSAIDs, phenytoin.
metabolismthere maybe a higher dose
Increased 1-acid
glycoprotein which requirement or prodrugs to obtain the
binds basic drugs, same AUC or the active drug (ie, the
eg, verapamil, metabolite) as in a young person.
propranolol. Decreased drug clearance associated with
Tissue binding Drugs which are tightly Changes in body Drugs bound to muscle, eg, aging results in an increase in drug hal -li e,
properties bound to tissues have composition digoxin, have decreased and is prolonged even more or lipid-soluble
large V (sarcopenia, increased Vwith aging. There ore drugsdue toanincrease inV(see Figure 4-3).
adiposity) may decreased loading dose
a ect V. required.

Lipid: water Lipid soluble drugs Relative increase in Loading dose o water
coe cient can pass through lipid proportion o body at soluble drugs, eg,
membranes o cells and decrease in body gentamicin, digoxin,
more easily and have water (muscle mass). decreased with aging
higher Vthan water There ore, higher V to avoid toxicity rom
soluble drugs or lipid soluble high initial Cp.
drugs and lower V or
water soluble drugs
with aging.

Transporters Passive acilitated Unknown. Drug interactions


di usion (move drug may occur at level o
in same direction as transporters.
concentration gradient). Passive transporters
Active transport (use ATP include the Organic Anion
to move drug against Transport Proteins (OAT-P)
concentration gradient). or benzyl penicillin,
digoxin, and pravastatin.
Active transporters include
MDR1 (P-gp) or many
cationic or neutral drugs,
eg, digoxin, macrolide
antibiotics, verapamil.

Reproduced with permission rom Halter JB, Ouslander JG, Tinetti ME, Studenski S, High KP, &
Asthana S (Eds). Hazzards Geriatric Medicine and Gerontology, 6th ed. McGraw-Hill, Inc., 2009.
Table 8-2.

75
SECTION I General Principles

A B
PHARMACOKINETICS IN
10 10
THE ELDERLYEXCRETION
9
Renal drug clearance is reduced with aging
8
(Table 4-4).
GFRis reduced by10 to 40%with aging. 7

e
e
c
c
n
n
a
Much o the age-related decline in renal 6

a
r
r

a
a
e
e
unction maybe related to disease,

l
l
5

C
C
c
particularlyhypertension, atherosclerosis,

c
i
i
t
t
4

a
a
andheart ailure, ratherthannormalaging.

p
p
e
e
H
H
3
Available methodstoestimate renal unction
in older adults are not reliable, particularlyin 2
rail or acutelyill older adults. 1
The Cockcro tGault equation(see below) 0 0
can give an estimate o renal unction 0 Old Young Old P ha s e I Young
or dose adjustment o renallyexcreted Old P ha s e II
drugs; however, the maintenance dose He pa tic Blood Flow Enzyme Ca pa city
o drugswith narrowtherapeuticindices
Flow-limite d drug Ca pa city-limite d drug
shouldbe guidedbytherapeuticdrug
monitoringandclinical response. FIGURE 4-2 The e ects o normal aging on hepatic drug clearance. With aging, hepatic blood
f ow is reduced by approximately 50%, which is associated with a 50% reduction in clearance
Tubular secretion is reduced with aging, to o f ow-limited drugs, but has little e ect on clearance o capacity-limited drugs (A). In normal
a similar or greater extent than GFR. aging, phase 1 metabolism is reduced, which reduces hepatic clearance o capacity-limited drugs
Tubular secretion maybe urther reduced metabolized by these enzymes, while phase 2 metabolism is probably preserved, although it may
bypolypharmacythat increases competi- be reduced in railty. Changes in enzyme capacity do not a ect the clearance o f ow-limited drugs
tion o drugs or transporters. (B). (Reproduced with permission from Halter JB, Ouslander JG, Tinetti ME, Studenski S, High KP, &Asthana
S [Eds]. Hazzards Geriatric Medicine and Gerontology, 6th ed. McGraw-Hill, Inc., 2009. Figs 8-3A&B.)
Overall tubular unction is decreased in the
elderly, with impaired abilitytoconcentrate
or dilute urine maximally. TABLE 4-3 Changes in Hepatic Clearance with Aging
Cockcro tGault Equation PROCESS OF AGE RELATED
HEPATIC CLEARANCE DESCRIPTION CHANGES CLINICAL APPLICATION
Creatinine clearance (mL)/min
Hepatic blood f ow Portal venous f ow Decreases by 30-50% Reduced clearance
(140 age) weight (kg) (~80%) and hepatic by 30-50% o high
=
72 serumcreatinine (mg/dL) arterial f ow (~20%) extraction ratio drugs,
0.85 or emales eg, morphine and
verapamil. Less impact
on low extraction
ratio drugs, eg,
carbamazepine, war arin,
diazepam.

Protein binding Only ree drug Decreased albumin: Only signi cant or drugs
is cleared. acidic drugs have that are highly protein
Protein binding higher raction bound (>90%) with low
a ected by disease unbound and hepatic extraction ratios,
and competition increased hepatic eg, war arin, phenytoin,
rom other drugs. clearance. diazepam.
Increased 1-acid
glycoprotein: basic
drugs have lower
raction unbound
and decreased
hepatic clearance.

Scavenger cells Kup er cells scavenge Possible reduction in Not known. May reduce
large protein drugs. scavenger unction hepatic clearance.
Liver sinusoidal (in animal studies).
endothelial cells
(LSECs) may scavenge
smaller particles.
(Continued)
76
Special Populations (Children and Elderly) CHAPTER 4

PROCESS OF AGE RELATED CHANGES IN RESPONSE TO


HEPATIC CLEARANCE DESCRIPTION CHANGES CLINICAL APPLICATION
DRUGS WITH AGING
Trans er into Trans er across Structural changes in Not known. May reduce
hepatocyte the LSECs and the LSECs and the space hepatic clearance.
Decreased sensitivityo the baroreceptor
hepatocyte apical o Disse may reduce re exoccurs with aging which changes the
membrane by passive trans er. Changes cardiovascular response to manydrugs.
or active transport. in hepatocyte -Adrenergicreceptor antagonists
membrane transport (see Chapter 7) maybe less ef ective in
not known.
controlling hypertension (see Chapter
Metabolism Biotrans ormation Reduced phase I Probably preserved 15) in the elderly.
to more hydrophilic metabolism in vivo clearance o drugs The acute bloodpressure response to
metabolites that in normal aging by that undergo capacity-
calciumchannel blockers (see Chapter 15)
may be equally, less 3050%. Phase II limited phase II
or more active than metabolism appears metabolism in healthy maybe greater in the aged.
parent drug to be maintained aging, eg, temazepam, Aging increases the sensitivityto many
in healthy aging, salicylic acid. CNSdrugs.
reduced in rail aged.
Older adultsare more sensitive tothe
Trans er into bile Active transport Unknown. In aged Unknown. sedative andrespiratorydepressant efects
ductule into bile canaliculi. rodents, increased o benzodiazepines(see Chapter 9), and
Bile enters small biliary P-glycoprotein experience more cognitive impairment.
intestine and drug or expression and
metabolite reabsorbed unction. Older adults are more sensitive to
(enterohepatic cycle) opiates (see Chapter 10) and general
or excreted in aeces. anesthetics (see Chapter 11).
Reproduced with permission rom Halter JB, Ouslander JG, Tinetti ME, Studenski S, High KP,
Exposure to anypsychotropicmedication
&Asthana S (Eds). Hazzards Geriatric Medicine and Gerontology, 6th ed. McGraw-Hill, Inc., 2009. (see Chapter 8), regardless o class,
Table 8-3. is associated with an almost two old
increased risko alls in older adults.
Psychotropic(see Chapter 8) and
anticholinergic(seeChapter6)medications
are stronglyassociated with cognitive
impairment and deliriumin the aged, as
12
well as anticholinergicadverse ef ects
Old (lipophilic) such as urinaryretention and postural
Old (hydrophilic) hypotension (see Chapter 6), and may
Young
reduce the ef ectiveness o cholinesterase
)
inhibitor therapyin managing Alzheimer
n
I
(
disease and related dementias.
n
o
i
t
Antipsychoticdrugs increase the risko
a
r

t
n
drug-induced parkinsonismin the elderly
e
c
n
which might be misdiagnosed as a new
o
C
medical condition (ie, Parkinson disease).
a
m
s
a
l
P
STEPS IN OPTIMIZING
DRUG REGIMENS
0 IN OLDER ADULTS
0 10
Reviewcurrent drug therapy.
Time
Discontinue unnecessarydrug therapy.
FIGURE 4-3 The theoretical e ects o aging on hal -li e. The decrease in clearance with aging
prolongs hal -li e. For lipid-soluble drugs, the increase in volume o distribution (V) prolongs hal - Consider anynewsymptomto be caused
li e urther. For water-soluble drugs, the decrease in Vreduces hal -li e, but in most cases, the size byadverse drug events.
o the e ect o aging on Vis smaller than the e ect on clearance, resulting in an overall increase Consider nonpharmacologictherapies.
in hal -li e compared to young subjects. The hal -li e determines the time course o drug accumu-
lation and elimination, and the choice o dose interval. The increase in hal -li e that is observed Substitute with sa er alternatives (see Side
or many drugs with aging means that it will take longer or drugs to reach steady-state, longer to Bar POTENTIALLYINAPPROPRIATEMEDICA-
be eliminated a ter they are ceased, and dosing intervals may need to be increased. (Reproduced TIONSFORTHEELDERLY).
with permission from Halter JB, Ouslander JG, Tinetti ME, Studenski S, High KP, &Asthana S [Eds]. Reduce the dose.
Hazzards Geriatric Medicine and Gerontology, 6th ed. McGraw-Hill, Inc., 2009. Fig 8-4.)
77
SECTION I General Principles

POTENTIALLY TABLE 4-4 Changes in Renal Drug Clearance with Aging


INAPPROPRIATE PROCESS
MEDICATIONS FOR OF RENAL EFFECTS OF CLINICAL
THE ELDERLY CLEARANCE AGE APPLICATION EXAMPLE

The American GeriatricSociety(AGS) Glomerular Decreased GFR, Estimates used to Gentamicin clearance correlates
ltration extent unclear, adjust maintenance with Cockcro t-Gault estimates
has developed and regularlyupdates
~10-40% doses o drugs or renal o creatinine clearance.
the Beers Criteria (medication list o impairment
PotentiallyInappropriate Medications
or the Elderly) based on evidence-based Tubular secretion Decreased Reduction in renal Ratio o procainamide clearance
recommendations: (active) clearance may be to creatinine clearance decreases
greater than reduction with aging.
https://www.dcri.org/

in GFR. Digoxin excreted by passive
trial-participation/the-beers-list With polypharmacy, glomerular ltration and active
The list is updated based on a comprehen- increased risk o drug- tubular secretion. Serum digoxin
sive, systematicreviewand grading o the drug interactions levels increase with number
through competition o concurrent P-gp inhibitors,
evidence on drug-related problems and or transporters. e.g., verapamil, erythromycin,
adverse drug events in older adult. amiodarone, spironolactone,
The 2012 AGSBeers Criteria or Potentially atorvastatin.
Inappropriate Medication Use in Older
Tubular Unknown I impaired, would Changes in clearance o lithium,
Adults is available online at: reabsorption reduce the e ect o which, like sodium, is reely
http://www.americangeriatrics.org/
(passive) reduced glomerular ltered at glomerulus and 80%
les/documents/beers/2012AGSBeersCr ltration on clearance. reabsorbed in the proximal
iteriaCitations.pd tubule, consistent with changes
in GFR with aging.

Reproduced with permission rom Halter JB, Ouslander JG, Tinetti ME, Studenski S, High KP, &
Asthana S (Eds). Hazzards Geriatric Medicine and Gerontology, 6th ed. McGraw-Hill, Inc., 2009.
Table 8-4.

78
SECTION

Neuropharmacology II
5. Neurotransmission . . . . . . . . . . . . . . . . . . . . . . . . . . . . . . . . . . . . . . . . . . . . . . . . . . . . 80

6. Cholinergic Pharmacology . . . . . . . . . . . . . . . . . . . . . . . . . . . . . . . . . . . . . . . . . . . . . 95

7. Adrenergic, Dopaminergic, and Serotonergic Pharmacology . . . . . . . . . . . . . . . 118

8. Pyschopharmacology . . . . . . . . . . . . . . . . . . . . . . . . . . . . . . . . . . . . . . . . . . . . . . . . . 149

9. Hypnotics, Sedatives, and Ethanol . . . . . . . . . . . . . . . . . . . . . . . . . . . . . . . . . . . . . . 176

10. Opioid Pharmacology . . . . . . . . . . . . . . . . . . . . . . . . . . . . . . . . . . . . . . . . . . . . . . . . 194

11. Anesthetic Agents and T erapeutic Gases . . . . . . . . . . . . . . . . . . . . . . . . . . . . . . . 210

12. Pharmacotherapy o the Epilepsies . . . . . . . . . . . . . . . . . . . . . . . . . . . . . . . . . . . . . 225

13. Drug T erapy o Neurodegenerative Diseases . . . . . . . . . . . . . . . . . . . . . . . . . . . . 238

14. Drug Addiction . . . . . . . . . . . . . . . . . . . . . . . . . . . . . . . . . . . . . . . . . . . . . . . . . . . . . . 254

79
CHAPTER

5 Neurotransmission
DRUGS INCLUDED IN Chapter 5, Neurotransmission is a compilation o Chapter 8 Neurotransmission: T e
Autonomic and Somatic Motor Nervous Systems, and Chapter 14 Neurotransmission
THIS CHAPTER
and the Central Nervous System in Goodman & Gilmans T e Pharmacological Basis of
The pharmacologyo specifcdrugs is not T erapeutics, 12th Edition. An understanding o the material in these chapters will be
included in this chapter. The drugs pre- help ul in ollowing the material presented in this chapter. In addition to the material
sented in the cases are used as examples to presented here, the 12th Edition includes:
illustrate various aspects o neurotransmis-
A detailed discussion o the peripheral and central nervous systems (CNS)
sion; the specifcpharmacologyo these
drugs is presented in the subsequent An intricate discussion o the neurotransmitters in the peripheral and CNS, their
chapters o this section. synthesis, storage, mechanisms o release and termination o e ect
Details o the cholinergic, adrenergic, and somatic nervous systems that are also
addressed in Chapters 6 and 7 o this book
A detailed discussion o chemical transmission o impulses in the CNS

LEARNING OBJECTIVES
Understand the characteristics o the parasympathetic, sympathetic, and enteric
nervous systems.
Understand the di erence between the autonomic and somatic nervous systems.
Know the predominant transmitters at ganglionic sites in the parasympathetic
and sympathetic nerves.
Know the transmitters and their target receptors in parasympathetic and
sympathetic nervous system.
Know the e ect that agonist and antagonists have at each target receptor.
Understand the blood-brain barrier (BBB).
Understand the intricacies o chemical transmission within the CNS and how
the interaction o central neurons control the pharmacological e ects o drugs
acting in the brain.

CASE 5-1
A 47-year-old woman is given a drug to treat her overactive bladder. She is told that the
drug is similar to atropine and that it will decrease her requency o urination. She is
cautioned to be aware o the possibility o dry eyes, dry mouth, blurred vision, consti-
pation, drowsiness, dizziness, and con usion.
a. What are the divisions o the peripheral autonomic nervous system that describe
the diverse actions o this drug?
T e e erent nerves o the involuntary (autonomic) nervous system supply all
innervated structures o the body except skeletal muscle, which is served by the
somatic nerves (see Figures 5-1 and 5-2). T e peripheral autonomic nervous system
is composed o the sympathetic, parasympathetic, and enteric divisions (see Side
Bar DIVISIONS OF HE PERIPHERAL AU ONOMIC NERVOUS SYS EM).
b. Why does this drug have such a broad constellation o side e ects involving so
many di erent organs?
Atropine is a drug that blocks cholinergic muscarinic receptors located on the
membranes o many e ector organs (see Figure 5-1 and able 5-1), and it blocks
all the peripheral actions o acetylcholine, the parasympathetic neurotransmitter
(Continued)
80
Neurotransmission CHAPTER 5

cilia ry ga nglion
iris
cilia ry body

t
t
e
e
la crima l gla nd III

c
c
t
t
mechano- and chemoreceptors

o
o
b
b
s phe nopa la tine ga nglion inte rna l ca rotid

u
u
of carotid sinus

l
l
b
b
and carotid body

a
a
chorda tympa ni

r
r
(
(
c
c
s ublingua l gla nd

r
r
VII

a
a
otic

n
n
i
i
IX

a
a
s ubma xilla ry gla nd ga nglion

l
l
)
)
pa rotid gla nd X

o
o
a rch of a orta

u
u
t
t
va s os e ns itive a nd

f
f
l
l
o
o
s upe
uperior
rior
w
w
che more ce ptive e ndings
c
c
e
e
middle
r
r
cervica l ga nglia
v
v
i
i
c
c
a
a
he a rt
l
l
c
c
o
o
inferior
r
r
d
d
1
tra che a s te lla te ga nglion
bronchi
lungs 2
pulmona ry ve s s e ls
3
pa ra ve rte bra l
ga nglionic
live r 4 cha in
bile ducts
ga ll bla dde r 5 t
oo
lr
t
t
sa
h
h
p
p
r
o
o
ce lia c ga nglion do
a
a
t
t
6
h
h
r
r
r
r
a
a
al
a
a
o
o
s ple e n ve n tr
c
c
v
v
r
r
gre a te r
i
i
a
a
s toma ch
e
e
c
c
c
c
7
r
r
c
c
s ma ll bowe l s pla nchnic
t
t
i
i
c
c
e
e
o
o
o
o
b
b
proxima l colon
r
r
l
l
d
d
le
r
r
u
u
a
a
8
ss
m
m
pa ra ve rte bra l
le
l
l
g
g
b
b
a

ga nglion
a
a
s

r
a
a
a dre na l
t
n
n
s
r
r
9
sp

p
g
g
me dulla white ra mus
o
o
la
l
l
u
u
i
i
la
o
o
gra y ra mus
n
t
t
n
n

n
f
f
ch
l
l
c
o
o
10
i
i
c
c
h

w
w
kidne y
n

ic
c
c
ic
h
h
ure te r
a
a
11
i
i
n
n
s ke le ta l mus cle
12
dis ta l s upe rior
colon me s e nte ric 1
l
l
u
u
ga nglion
m
m
2 blood ve s s e ls
b
b
a
a
re ctum
r
r
3
c
c
o
o
p
r
r
e
d
d
urina ry 4
l
v
bla dde r
i
c
5
n
e
r
e xte rna l
v
1
e
s
s
ge nita lia
a
a
2
c
c
r
r
s
a
a
a
3
l
l
infe rior
c
o
o
r
u
u
me s e nte ric
a
4
t
t
l
f
f
l
l
ga nglion
g
o
o
a
5
w
w
n
g
l
i
a
To blood ve s s e ls To s we a t gla nds a nd
a nd ha ir follicle s s pe cia lize d blood ve s s e ls
of lowe r limb of lowe r limb

S e gme nta l pos tga nglionic S e gme nta l pos tga nglionic
a dre ne rgic fibe rs from choline rgic fibe rs from
pa ra ve rte bra l ga nglia to pa ra ve rte bra l ga nglia to
blood ve s s e ls a nd ha ir s we a t gla nds a nd ce rta in
follicle s via gra y ra mii blood ve s s e ls via gra y
a nd s pina l ne rve s ra mi a nd s pina l ne rve s

FIGURE 5-1 The autonomic nervous system. Schematic representation o the autonomic nerves and e ector organs
based on chemical mediation o nerve impulses. Blue, cholinergic; grey, adrenergic; dotted blue, visceral a erent; solid lines,
preganglionic; broken lines, postganglionic. In the rectangle at the right are shown the ner details o the rami cations
o adrenergic bers at any 1 segment o the spinal cord, the path o the visceral a erent nerves, the cholinergic nature o
somatic motor nerves to skeletal muscle, and the presumed cholinergic nature o the vasodilator bers in the dorsal roots
o the spinal nerves. The asterisk (*) indicates that it is not known whether these vasodilator bers are motor or sensory or
where their cell bodies are situated.

81
SECTION II Neuropharmacology

S OMATIC S YS TEM

Va rious
le ve ls Motor ne uron S ke le ta l Nicotinic
of ACh (s tria te d) Re ce ptors
s pina l cord mus cle Nm
Nm

AUTONOMIC S YS TEM
P a ra s ympa the tic
Ga nglion
Cra nia l Mus ca rinic
a nd ACh Nn Re ce ptors
s pina l ACh M

S mooth
M mus cle ,
ca rdia c
tis s ue ,
S ympa the tic
s e cre tory
Ga nglion gla nds
Thora cic
a nd ACh Nn Adre ne rgic
lumba r Re ce ptors
Nn NE /
S ympa the tic
choline rgic fibe r Mus ca rinic
ACh
/ Re ce ptors
Adre na l Epi/NE (s we a t gla nds )
me dulla (80%/20%)
ACh M

S we a t
gla nds

FIGURE 5-2 Schematic representation o the somatic motor nerves and the e erent nerves o the autonomic nervous system. The principal
neurotransmitters, acetylcholine(ACh) and norepinephrine (NE), are shown in grey. The receptors or these transmitters, nicotinic (N) and mus-
carinic (M) cholinergic receptors, and adrenergic receptors are shown in grey rectangles to the right. The somatic nerves innervate skeletal
muscle directly without a ganglionic relay. The autonomic nerves innervate smooth muscles, cardiac tissue, and glands. Both parasympathetic
and sympathetic systems have ganglia where ACh is the transmitter o the preganglionic bers; ACh acts on nicotinic receptors on the postgan-
glionic nerves. ACh is also the neurotransmitter at cells o the adrenal medulla, where it acts on nicotinic ACh receptors to cause release o the
catecholamines epinephrine (Epi) and NE into the circulation. Epi represents ~80% o the released catecholamines. ACh is the predominant neu-
rotransmitter o postganglionic parasympathetic nerves and acts on muscarinic receptors. NE is the principal neurotransmitter o postganglionic
sympathetic nerves, acting on or adrenergic receptors. Note that somatic nerves orm a specialized synaptic junction, termed the motor end
plate. Autonomic nerves orm a more di use pattern with multiple synaptic sites. The ganglia in the parasympathetic system are near or within
the organ being innervated with generally a one-to-one relationship between pre- and postganglionic bers. In the sympathetic system, the
ganglia are generally ar rom the e ector cells (eg, within the sympathetic chain ganglia). Preganglionic sympathetic bers may make contact
with a large number o postganglionic bers.

(see Chapter 6). T us, it is easy to see why its side e ects involve so many organs.
Although atropine has minimal e ects on the CNS, there are de nitely musca-
rinic receptors in the brain and their antagonism may lead to drowsiness and
con usion.
c. What is the enteric nervous system?
T e processes o mixing, propulsion, and absorption o nutrients in the GI tract
are controlled locally through a restricted part o the peripheral nervous system
called the enteric nervous system (ENS; see Chapter 33). T e ENS actually com-
prises components o the sympathetic and parasympathetic nervous systems and
has sensory nerve connections through the spinal and nodose ganglia.
(Continued)
82
Neurotransmission CHAPTER 5

TABLE 5-1 Responses o Ef ector Organs to Autonomic Nerve Impulses


ADRENERGIC CHOLINERGIC
ORGAN SYSTEM SYMPATHETIC EFFECT a
RECEPTOR SUBTYPEb PARASYMPATHETIC EFFECTa RECEPTOR SUBTYPEb
Eye
Radial muscle, iris Contraction (mydriasis)++ 1
Sphincter muscle, iris Contraction (miosis)+++ M3, M2
Ciliary muscle Relaxation or ar vision+ 2 Contraction or near vision+++ M3, M2
Lacrimal glands Secretion+ Secretion+++ M3, M2
Hea rt c
Sinoatrial node in heart rate++ 1 > 2 in heart rate+++ M2 >> M3
Atria in contractility and conduction 1 > 2 in contractility++ and M2 >> M3
velocity++ shortened AP duration
Atrioventricular node in automaticity and 1 > 2 in conduction velocity; AV M2 >> M3
conduction velocity++ block+++
His-Purkinje system in automaticity and 1 > 2 Little e ect M2 >> M3
conduction velocity
Ventricle in contractility, conduction 1 > 2 Slight in contractility M2 >> M3
velocity, automaticity and rate o
idioventricular pacemakers+++
Blood Vessels
(Arteries and arterioles)d
Coronary Constriction+; dilation e++ 1, 2; 2 No innervation h
Skin and mucosa Constriction+++ 1, 2 No innervation h
Skeletal muscle Constriction; dilatione, ++ 1; 2 Dilation h (?)
Cerebral Constriction (slight) 1 No innervation h
Pulmonary Constriction+; dilation 1; 2 No innervation h
Abdominal viscera Constriction +++; dilation + 1; 2 No innervation h
Salivary glands Constriction+++ 1, 2 Dilation h++ M3
Renal Constriction++; dilation++ 1, 2; 1, 2 No innervation h
(Veins)d Constriction; dilation 1, 2; 2
Endothelium NO synthase h M3
Lung
Tracheal and bronchial Relaxation 2 Contraction M2 = M3
smooth muscle
Bronchial glands secretion, 1 Stimulation M2, M3
secretion 2
Stomach
Motility and tone (usually)i+ 1, 2; 1, 2 i+++ M2 = M3
Sphincters Contraction (usually)+ 1 Relaxation (usually)+ M3, M2
Secretion Inhibition 2 Stimulation++ M3, M2
Intestine
Motility and tone Decrease h+ 1, 2; 1, 2 i+++ M3, M2
Sphincters Contraction+ 1 Relaxation (usually)+ M3, M2
Secretion 2 ++ M3, M2
Gallbla dder a nd Ducts Relaxation+ 2 Contraction+ M
Kidney
Renin secretion +; ++ 1; 1 No innervation
Urinary Bladder
Detrusor Relaxation+ 2 Contraction+++ M3 > M2
Trigone and sphincter Contraction++ 1 Relaxation++ M3 > M2
Ureter
Motility and tone 1 (?) M
Uterus Pregnant contraction 1
Relaxation 2 Variable j M
Nonpregnant relaxation 2

(Continued)
83
SECTION II Neuropharmacology

ADRENERGIC CHOLINERGIC
ORGAN SYSTEM SYMPATHETIC EFFECT a
RECEPTOR SUBTYPEb PARASYMPATHETIC EFFECTa RECEPTOR SUBTYPEb
Sex Orga ns, male Ejaculation+++ 1 Erection+++ M3

Pilomotor muscles Contraction++ 1


Sweat glands Localized secretion k++ 1
Generalized secretion+++ M3, M2
Spleen Capsule Contraction+++ 1
Relaxation+ 2
Adrenal Medulla Secretion o epinephrine and N(3)2(4)3; M
norepinephrine (secondarily)
Skeletal Muscle Increased contractility; 2
glycogenolysis; K+ uptake
Liver Glycogenolysis and 1
gluconeogenesis+++ 2

Pancreas
Acini secretion+ Secretion++ M3, M2
Islets ( cells) secretion+++ 2
secretion+ 2
Fat Cellsl Lipolysis+++; thermogenesis 1; 1, 2, 3
Inhibition o lipolysis 2

Sa livary Gla nds K+ and water secretion+ 1 K+ and water secretion+++ M3, M2

Nasopha ryngeal Gla nds Secretion++ M3, M2

Pineal Glands Melatonin synthesis

Posterior Pituita ry ADH secretion 1


Autonomic Nerve Endings
Sympathetic terminal
Autoreceptor Inhibition o NE release 2A > 2C(2B)
Heteroreceptor Inhibition o NE release M2, M4
Parasympathetic terminal
Autoreceptor Inhibition o ACh release M2, M4
Heteroreceptor Inhibition ACh release 2A > 2C
a
Responses are designated + to +++ to provide an approximate indication o the importance o sympathetic and parasympathetic nerve
activity in the control o the various organs and unctions listed.
b
Adrenergic receptors: 1, 2 and subtypes thereo ; 1, 2, 3. Cholinergic receptors: nicotinic (N); muscarinic (M), with subtypes 1-4. When
a designation o subtype is not provided, the nature o the subtype has not been determined, unequivocally. Only the principal receptor
subtypes are shown. Transmitters other than ACh and NE contribute to many o the responses.
c
In the human heart, the ration o 1 to 2 is about 3:2 in atria and 4:1 in ventricles. While M2 receptors predominate, M3 receptors are also present.
d
The predominant 1 receptor subtype in most blood vessels (both arteries and veins) is 1A, although other 1 subtypes are present in speci c
blood vessels. The 1D is the predominant subtype in the aorta.
e
Dilation predominates in situ owing to metabolic autoregulatory mechanisms.
Over the usual concentration range o physiologically released circulating epinephrine, the receptor response (vasodilation) predominates
in blood vessels o skeletal muscle and liver; receptor response (vasoconstriction) in blood vessels o other abdominal viscera. The renal and
mesenteric vessels also contain speci c dopaminergic receptors whose activation causes dilation.
g
Sympathetic cholinergic neurons cause vasodilation in skeletal muscle beds, but this is not involved in most physiological responses.
h
The endothelium o most blood vessels releases NO, which causes vasodilation in response to muscarinic stimuli. However, unlike the
receptors innervated by sympathetic cholinergic bers in skeletal muscle blood vessels, these muscarinic receptors are not innervated and
respond only to exogenously added muscarinic agonists in the circulation.
i
While adrenergic bers terminate at inhibitory receptors on smooth muscle bers and at inhibitory receptors on parasympathetic
(cholinergic) excitatory ganglion cells o the myenteric plexus, the primary inhibitory response is mediated via enteric neurons through NO,
P2Yreceptors, and peptide receptors.
j
Uterine responses depend on stages o menstrual cycle, amount o circulating estrogen and progesterone, and other actors.
k
Palms o hands and some other sites (adrenergic sweating).
l
There is signi cant variation among species in the receptor types that mediate certain metabolic responses. All three adrenergic receptors
have been ound in human at cells. Activation o 3 receptors produces a vigorous thermogenic response as well as lipolysis. The signi cance
is unclear. Activation o receptors also inhibits leptin release rom adipose tissue.
ADH, antidiuretic hormone; AP, action potential; AV, atrioventricular

84
Neurotransmission CHAPTER 5

d. What are the di erences between the parasympathetic and sympathetic nervous
DIVISIONS OF THE
systems?
PERIPHERAL AUTONOMIC
T e sympathetic system is distributed to e ectors throughout the body, whereas NERVOUS SYSTEM
parasympathetic distribution is much more limited (see Figure 5-1). T e ganglia
in the parasympathetic system are near or within the organ being innervated with Sympatheticnervous system
generally a one-to-one relationship between pre- and postganglionic bers (see Parasympatheticnervous system
Figure 5-2). In the sympathetic system the ganglia are generally ar rom the e ec- Entericnervous system
tor cells. Preganglionic sympathetic bers may make contact with a large number
o postganglionic bers.
In contrast to the parasympathetic and sympathetic nervous systems, somatic
nerves innervate skeletal muscle directly without a ganglionic relay (see Case 5-2).
STEPS INVOLVED
Acetylcholine (ACh) is the predominant neurotransmitter o postganglionic para- IN JUNCTIONAL
sympathetic nerves and acts on muscarinic receptors. Norepinephrine (NE) is the
TRANSMISSION
principal neurotransmitter o postganglionic sympathetic nerves acting on or
adrenergic receptors. Transmitter synthesis
Both parasympathetic and sympathetic systems have ganglia where ACh is the Transmitter storage
transmitter o the preganglionic bers; ACh acts on nicotinic receptors on the post- Transmitter release
ganglionic nerves. ACh is also the transmitter at cells o the adrenal medulla, where
Transmitter recognition and binding by
it acts on nicotinic receptors to cause release o epinephrine and norepinephrine into
target receptor
the circulation.
Termination o action

CASE 5-2
A 35-year-old man is undergoing abdominal surgery. T e anesthesiologist explains
in a presurgery meeting with the patient that a neuromuscular blocking drug will be
administered to cause complete muscle relaxation prior to the initiation o surgery.
a. What is the di erence between the autonomic nervous system that innervates many
visceral organs and the somatic nervous system that innervates skeletal muscle?
T e somatic nerves innervate skeletal muscle directly without a ganglion relay (see
Figure 5-2). At each neuromuscular junction, the axon terminal loses its myelin
sheath and orms a terminal aborization that lies in apposition to a specialized
sur ace o the muscle membrane termed the motor end plate (see Chapter 6).
b. What are the steps involved in the transmission o a nerve impulse?
T e steps involved in excitatory and inhibitory neurotransmission are depicted
schematically in Figure 5-3. T e steps involved in the junctional transmission
o the nerve impulse are outlined in the Side Bar S EPS INVOLVED IN JUNC-
IONAL RANSMISSION, the details o which are characteristic o a speci c
nerve transmission system, that is, parasympathetic, sympathetic, somatic, and in
the CNS.
ransmitter synthesis: Small molecules like ACh and NE (see Side Bar NEU-
RO RANSMI ERS or a list o neurotransmitters) are synthesized in nerve ter-
minals; peptides are synthesized in cell bodies and transported to nerve terminals.
ransmitter storage: Synaptic vesicles store transmitters, o en in association with
various proteins and requently with A P.
ransmitter release: Release o transmitter occurs by exocytosis. Depolarization
results in an in ux o Ca++, which in turn appears to bind proteins called synapto-
tagmins. T e storage vesicles dock and then use with sca olding proteins on the
presynaptic membrane, which is ollowed by exocytotic release o their contents
into the synaptic cle (see Figure 5-3).
ransmitter recognition: Receptors exist on postsynaptic cells, which recognize the
transmitter (see Side Bar NEURO RANSMI ERS). Binding o a neurotransmit-
ter to its receptor initiates a signal transduction event (see Figures 5-4 and 5-6).
(Continued)

85
SECTION II Neuropharmacology

ermination of action: A variety o mechanisms terminate the action o synaptically


NEUROTRANSMITTERS released transmitter, including hydrolysis and reuptake into neurons by speci c
PERIPHERAL CENTRAL transporters (see Chapters 6 and 7 or speci c examples).
Acetylcholine Acetylcholine c. What is the role o ion channels in the promulgation o the nerve impulse and
Norepinephrine Norepinephrine
controlling neuronal excitability?
Voltage-dependent ion channels provide or rapid changes in ion permeability
Epinephrine Epinephrine along axons and within dendrites and or excitation-secretion coupling that releases
Dopamine Dopamine neurotransmitters rom presynaptic sites.
T e electrical excitability o neurons is achieved through modi cation o ion chan-
Serotonin Serotonin
nels in neuronal plasma membranes (see Figures 5-3 through 5-6). T e relatively
Amino acids high extracellular concentration o Na+ (140 mM) compared to its concentration
intracellularly (14 mM) means that increases in permeability to Na+ causes depo-
-amino-butyric acid larization, ultimately leading to the generation o action potentials.
(GABA)
Increases in the concentration o intracellular Ca++ a ects multiple processes in the
Glycine cell and are critical to the release o neurotransmitters. able 5-2 lists the various
subtypes o Ca++ channels.
Aspartic acid
T e Cl gradient across the plasma membrane explains the act that activation o
Glutamic acid Cl channels causes an inhibitory postsynaptic potential (IPSP) that dampens neu-
Peptide ronal excitability and inactivation o these channels can lead to hyperexcitability.
neurotransmitters or Figure 14-3 in Goodman and Gilmans T e Pharmacological Basis of T erapeutics,
neuromodulators (see 12th Edition shows the structures o 3 amilies o Cl channels.
Table 14-7 Goodman
and Gilmans The
Pharmacological + 1. AP 2. EP S P 3. AP
E
L
A
DOCKING COMP LEX
N
Basis of Therapeutics,
I
A
T
)
V
R
0
N
B
m
12th Edition)
E
M
ne urexin
T
(
Na +
O
E

M
P
synta xin
Ra b 3

p
excita tory

a
+ syna ptobrevin

g
input Na + syna ptota gmin

c
+

ti

p
n
a ne uro-
K+
y

tra ns mitte r
s
+

tra ns porte r
Ca 2+
Na + Ca 2+ volta ge -s e ns itive
Ca 2+ cha nne l
K+ volta ge -s e ns itive pos tsyna ptic
Na + cha nne l re ce ptor-ga te d
K+
Na + ion cha nne l

+

inhibitory +
1 2
+
input Cl + SYNAP TIC VES ICLES
excita tory
+ 1. AP 2. IP S P 3. Inhibition
E
L
tra ns mitte r
A
N
I
A
T
)
V
R
inhibitory
N
0
B
m
E
M
T
(
tra ns mitte r
O
E

M
P
FIGURE 5-3 Steps involved in excitatory and inhibitory neurotransmission. 1. The nerve action
potential (AP) consists o a transient sel -propagated reversal o charge on the axonal membrane.
(The internal potential Ei goes rom a negative value, through zero potential, to a slightly positive
value primarily through increases in Na+ permeability and then returns to resting values by an
increase in K+ permeability.) When the AP arrives at the presynaptic terminal, it initiates release o
the excitatory or inhibitory transmitter. Depolarization at the nerve ending and entry o Ca2+ initiate
docking and then usion o the synaptic vesicle with the membrane o the nerve ending. Docked
and used vesicles are shown. 2. Combination o the excitatory transmitter with postsynaptic recep-
tors produces a localized depolarization, the excitatory postsynaptic potential (EPSP), through an
increase in permeability to cations, most notably Na+. The inhibitory transmitter causes a selective
increase in permeability to K+ or Cl, resulting in a localized hyperpolarization, the inhibitory post-
synaptic potential (IPSP). 3. The EPSP initiates a conducted AP in the postsynaptic neuron; this can
be prevented, however, by the hyperpolarization induced by a concurrent IPSP. The transmitter is
dissipated by enzymatic destruction, by reuptake into the presynaptic terminal or adjacent glial
cells, or by di usion. Depolarization o the postsynaptic membrane can permit Ca2+ entry i voltage-
gated Ca2+ channels are present. (Reproduced with permission rom Brunton L, Parker K, Blumen-
thal D, Buxton I (eds). Goodman &Gilmans Manual of Pharmacology and Therapeutics. New York:
McGraw-Hill, 2008, p 94. Copyright 2008 by The McGraw-Hill Companies, Inc. All rights reserved.)
86
Neurotransmission CHAPTER 5

A Io n c hanne ls 1 s ubunits for Ca 2 +, Na + cha nne ls

I II III IV

outs id e

me mb ra ne

ins id e

Volta ge C
s e ns ing
S 4 tra ns me mbra ne ina ctiva tion
s e gme nt re gion P KA s ite

P KC s ite
N
Ina ctiva tion trime r

Glycosyla tion s ite


Pore re gion

modula tion by P KA, P KC

B Multi-s ubunit as s e mbly o f Ca 2 + c hanne ls


2 a nd
N
2

2

C Cytos olic
S C
N
N S
1

C N
2+ C
Ca

C S truc ture dive rs ity o f K+ c hanne ls


+
K cha nne l Inwa rd
re ctifie r
+
K cha nne l

C
N


C
C N
N

FIGURE 5-4 Structural similarities o voltage-dependent Na+, Ca2+, and K+ channels. A. The subunit in both Ca2+ and
Na+ channels contains 4 subunits, each with 6 transmembrane hydrophobic domains. The hydrophobic regions that con-
nect segments 5 and 6 in each domain orm the pore o the channel. Segment 4 in each domain includes the voltage-
sensor. (Adapted with permission rom Catterall W. From ionic currents to molecular mechanisms: The structure and
unction o voltage-gated sodium channels. Neuron, 2000, 26:1325. Copyright Elsevier.) B. The Ca2+ channel also requires
several auxiliary small proteins (2, , , and ). The 2 and subunits are linked by a disul de bond. Regulatory subunits
also exist or Na+ channels. C. Voltage-sensitive K+ channels (Kv) and the rapidly activating K+ channel (KA) share a similar
putative hexaspanning structure similar in overall con guration to 1 repeat unit within the Na+ and Ca2+ channel struc-
ture, while the inwardly recti ying K+ channel protein (Kir) retains the general con guration o just loops 5 and 6. Regula-
tory subunits (cystosolic) can alter Kv channel unctions. Channels o these 2 overall moti s can orm heteromultimers.

87
SECTION II Neuropharmacology

A TM2
Exte rior

Inte rior

Hydrophobic
ring

L
L L
L L

FIGURE 5-5 Predicted 3-D structure o a ligand-gated ion channel receptor in a postsynaptic membrane. A. These channels consist o a cylindrical
membrane-embedded structure with a central pore. The second transmembrane domain (TM2) o each subunit lines the pore and bends inward to
block ion f ow through the channel. B. A highly conserved leucine residue (L) in the TM2 bend o each subunit is believed to protrude into the pore
to orm a tight hydrophobic ring, which may act as a barrier to the f ow o hydrated ions across the channel. (Redrawn with permission rom Nestler
EJ, Hyman SE, Malenka RC (eds). Molecular Neuropharmacology, 2nd ed. New York: McGraw-Hill, 2009, p 174. Copyright 2009 by The McGraw-Hill
Companies, Inc. All rights reserved.)

A B
Ne urotra ns mitte r

Re ce ptor
Ion
Re ce ptor Ion cha nne l cha nne l

P la s ma Ion cha nne l



me mbra ne

Effe ctors (AC, P LC)

S e cond me s s e nge rs (cAMP , Ca 2 +)

Alte re d prote in kina s e /phos pha ta s e a ctivitie s

Effe cts on othe r ne urona l proce s s e s


C D
Ne urotrophic fa ctors S te roid
hormone

P rote in S te roid
tyros ine re ce ptor
kina s e
Alte re d ge ne Effe cts on
e xpre s s ion othe r
S ubs tra te s , ne urona l
s ca ffold prote ins proce s s e s

Nucle us

DNA

FIGURE 5-6 General patterns o signal transduction in the brain. A. Neurotransmitter activation o a receptor that contains an integral ion channel.
B. Neurotransmitter activation o G proteincoupled receptor. A ter activation, the subunits o the G protein can directly regulate an ion channel (left),
and the subunit can activate second messenger-dependent signaling involving protein kinases and protein phosphatases, which can, in turn, a ect
ion channels and other neuronal processes (right). C. Neurotrophic actors promote receptor dimerization, which leads to activation o receptor protein
tyrosine kinase activity and its sequelae. D. Steroid hormone activation o a cytoplasmic receptor. A ter the receptor-hormone complex orms, it enters
the nucleus and regulates gene expression. (Redrawn with permission rom Nestler EJ, Hyman SE, Malenka RC (eds). Molecular Neuropharmacology,
2nd ed. New York: McGraw-Hill, 2009, p 76. Copyright 2009 by The McGraw-Hill Companies, Inc. All rights reserved.)
88
Neurotransmission CHAPTER 5

TABLE 5-2 Subtypes o Ca 2+ Channela


Ca 2+ CHANNEL UNOFFICIAL NAME Ca 2+ CURRENT TYPE PRIMARY LOCATIONS ANTAGONISTS FUNCTION
Cav 1.1 BKchannel L Skeletal muscle DHP, PAA, BZP EC coupling

Cav 1.2 L Cardiac muscle DHP, PAA, BZP EC coupling


Endocrine cells Ca2+ homeostasis
Neurons Regulation o transcription

Cav 1.3 L Endocrine cells DHP, PAA, BZP Hormone secretion


Neurons Regulation o transcription

Cav 1.4 L Retina DHP, PAA, BZP Tonic neurotransmitter release

Cav 2.1 SK1 P/Q Nerve terminals -Agatoxin IVA Neurotransmitter release
Dendrites Dendrite Ca2+ transients

Cav 2.2 SK2 N Nerve terminals -CTX-GVIA Neurotransmitter release


Dendrites Riluzole Dendrite Ca2+ transients

Cav 2.3 SK3 R Cell bodies SNX-482 Ca2+-dependent action


Dendrites potentials
Nerve Terminals Dendritic Ca2+ transients

Cav 3.1 Gardos channel T Cardiac muscle Mibe radil Repetitive ring o pacemaker
Skeletal muscle cells
Neurons

Cav 3.2 T Cardiac muscle Mibe radil Repetitive ring o pacemaker


Neurons cells

Cav 3.3 T Neurons Mibe radil Repetitive ring o pacemaker


cells

DHP = dihydropyridines (ni edipine). PAA = Phenylalkylamines (verapamil). BZP = benzothiazepines (diltiazem). -CTX = -conotoxin
a
Five classes o Ca2+ channel identi ed as L, P/Q, N, R, and T channels exist.

CASE 5-3 RECEPTORS IN THE


A 6-year-old boy is accidentally exposed to a large oral dose o the pesticide malathi- PERIPHERAL AND CENTRAL
one, a cholinesterase inhibitor. NERVOUS SYSTEMS
a. Why are the symptoms that this boy might experience predictable? Muscarinicacetylcholine
Acetylcholinesterase catalyzes the hydrolysis o the neurotransmitter acetylcholine Nicotinicacetylcholine
(ACh) (see Chapter 6). When the enzyme is inhibited, there is an abundance o ACh at Adrenergic
the ganglion in parasympathetic and sympathetic bers, at the parasympathetic e ec- Adrenergic
tor organs, and at the somatic neuromuscular junctions (see Figure 5-2). Although
Serotinergic
poisoning by anticholinesterase agents is complex (see Chapter 6), knowing the organs
innervated by parasympathetic, sympathetic, and somatic bers can allow or the pre- Dopamine
diction o symptoms to be experienced by this boy (see Figure 5-1 and able 5-1). Amino acid receptors (GABA, Glutamate,
Aspartate, Glycine)
b. How are the actions o neurotransmitters terminated?
Opioid receptors (, , ) (see Chapter 10)
Figure 5-7 shows the various mechanisms by which neurotransmitters are termi-
nated. Released neurotransmitter is inactivated by reuptake into the nerve terminal Somatostatin
(dopamine, NE, GABA), by degradation (ACh, peptides), or by uptake and metab- Neurotensin
olism by glial cells in the CNS (glutamate). Cholecystokinin
Purinergicreceptors (see Table 14-10
CASE 5-4 Goodman andGilmans The Pharmacological
A 23-year-old woman develops seasonal allergies, involving severe itching eyes, lac- Basis of Therapeutics, 12th Edition)
rimation, and rhinorrhea. She commonly takes an over-the-counter product that
contains diphenhydramine (a rst-generation antihistamine), but it makes her drowsy
(Continued)
89
SECTION II Neuropharmacology

Pre s ynaptic
ne uro n

11 10

Ne urotra ns mitte r 9

Ca 2 +
2
AP Glial
1 c e ll
5 6
NT
8
3
4

G prote in Na +

Po s ts ynaptic
ne uro n

FIGURE 5-7 Transmitter release, action, and inactivation. Depolarization opens voltage-depen-
dent Ca2+ channels in the presynaptic nerve terminal. (1) The inf ux o Ca2+ during an action
potential (AP) triggers (2) the exocytosis o small synaptic vesicles that store neurotransmitter
(NT) involved in ast neurotransmission. Released neurotransmitter interacts with receptors in the
postsynaptic membranes that either couple directly with ion channels (3) or act through second
messengers, such as (4) GPCRs. Neurotransmitter receptors in the presynaptic nerve terminal
membrane (5) can inhibit or enhance subsequent exocytosis. Released neurotransmitter is inacti-
vated by reuptake into the nerve terminal by (6) a transport protein coupled to the Na+ gradient,
or example, DA, NE, and GABA; by (7) degradation (ACh, peptides); or by (8) uptake and metabo-
lism by glial cells (Glu). The synaptic vesicle membrane is recycled by (9) clathrin-mediated
endocytosis. Neuropeptides and proteins are stored in (10) larger, dense core granules within the
nerve terminal. These dense core granules are released rom (11) sites distinct rom active zones
a ter repetitive stimulation.

and she is unable to work productively (see Chapter 21). Her doctor recommends that
she switch to loratadine, which should make her less sleepy.
a. What is the di erence between diphenhydramine and loratadine?
T e antihistamines are discussed in detail in Chapter 21 (see speci cally Case
21-1). Both diphenhydramine and loratadine are antagonists o histamine H 1 recep-
tors. Diphenhydramine is an ethanolamine and readily penetrates the CNS causing
the drowsiness experienced by this patient. Loratadine is a tricyclic piperidine that
does not penetrate the CNS and does not cause drowsiness.
b. What process in the brain keeps loratadine rom producing sleepiness?
Drugs acting in the CNS have to cross the blood-brain barrier (BBB) or the
blood-CSF barrier (see Chapter 2). T ese 2 barriers are ormed by brain capillary
endothelial cells and epithelial cells o the choroid plexus, respectively. Lipophilic
molecules such as diphenhydramine di use reely across the BBB and accumulate
in the brain. Loratadine is less lipophilic and does not cross the BBB. In addition,
diphenhydramine is an anticholinergic and interacts with muscarinic receptors,
whereas loratadine does not inhibit muscarinic receptors.

90
Neurotransmission CHAPTER 5

CASE 5-5
A 62-year-old man with a diagnosis o Alzheimers disease has mild cognitive loss. He
is given a drug that may improve his cognition, but his doctor explains to the amily
that the patients loss o cognitive ability will likely progress (see Chapter 13).
a. How is the pharmacological treatment o a disorder o the CNS di erent rom
treating a disorder in the peripheral nervous system?
T e identi cation o targets or drugs that a ect the CNS and behavior is a dif -
cult scienti c challenge. Complicating this e ort is the act that a CNS-active drug
(such as this patient is receiving to improve cognition) may act at multiple sites
with disparate and even opposing e ects. In addition, many CNS disorders involve
multiple brain regions and pathways, which can rustrate e orts to use a single
therapeutic agent.
b. What are the neurotransmitters in the CNS?
T e neurotransmitters in the CNS are generally the same as in the peripheral ner-
vous system (see Side Bar NEURO RANSMI ERS). In addition, there are many
other neurotransmitters in the CNS such as amino acids and peptides (see able 5-3).
c. How does this di erence make it di cult to treat disorders o the CNS?
In the peripheral nervous system, there are limited transmitters and receptors
which recognize them; thus, agonists or antagonists have speci c predictable
e ects. In the CNS there are more transmitters and receptors that unction to regu-
late brain activity and behavior. Figure 5-6 shows the multiple patterns o signal
transduction in the brain. Speci c therapeutic approaches to neurological and psy-
chiatric disorders are discussed in Chapters 8, 9, 12, 13, and 14.

TABLE 5-3 Examples o Neuropeptides


Calcitonin Family Tachykinins
Calcitonin Neurokinin A (substance A)
Calcitonin gene-related peptide (CGRP) Neurokinin B
Hypothalamic Hormones Neuropeptide K
Oxytocin Substance P
Vasopressin VIP-Glucagon Family
Hypothalamic Releasing and Inhibitory Glucagon
Hormones Glucagon-like peptide (GLP-1)
Corticotropin-releasing actor (CRF or CRH) Pituitary adenylyl cyclaseactivating peptide
Gonadotropin-releasing hormone (GnRH) (PACAP)
Growth hormone releasing hormone (GHRH) Vasoactive intestinal polypeptide (VIP)
Somatostatin (SST) Some Other Peptides
Thyrotropin releasing hormone (TRH) Agouti-related peptide (ARP)
Neuropeptide Y Family Bombesin
Neuropeptide Y(NPY) Bradykinin (BK)
Neuropeptide YY(PYY) Cholecystokinin (CCK; multiple orms)
Pancreatic polypeptide (PP) Cocaine- and amphetamine-regulated
Opioid Peptides transcript (CART)
-endorphin (also pituitary hormone) Galanin
Dynorphin peptides Ghrelin
Leu-enkephalin Melanin-concentrating hormone (MCH)
Met-enkephalin Neurotensin
Pituitary Hormones Nerve growth actor (NGF)
Adrenocorticotropic hormone (ACTH) Orexins (or Hypocretins)
-Melanocyte-stimulating hormone (-MSH) Orphanin GQ (or Nociceptin) (also grouped
Growth hormone (GH) with opioids)
Follicle-stimulating hormone (FSH)
-lipotropin (-LPH)
Luteinizing hormone (LH)

Modi ed with permission rom Nestler EJ, Hyman SE, Malenka RC (eds). Molecular
Neuropharmacology, 2nd ed. New York: McGraw-Hill, 2009, p 184, Table 7-1. Copyright 2009
by The McGraw-Hill Companies, Inc. All rights reserved.
91
SECTION II Neuropharmacology

CASE 5-6
A 56-year-old woman with advanced breast cancer is prescribed morphine or pain.
Although the morphine e ectively di uses the pain, the patient develops severe
constipation.
a. What is meant by drug specif city and nonspecif city in the CNS?
T e e ect o a drug in the CNS is considered to be speci c when it a ects an identi -
able molecular mechanism unique to target cells that bear receptors or that drug.
Conversely, a drug is regarded as being nonspeci c when it produces e ects at a
variety o di erent target cells, thus a ecting a diverse set o neurobiological systems.
Although relie o pain is the goal when administering an opiate, one must also
contend with potential o -target e ects, including respiratory depression and con-
stipation (see Chapter 10).

CASE 5-7
A 75-year-old woman has been taking alprazolam or anxiety and sleep or the past
10 years. Lately she has developed mild dementia and has orgotten to re ll her pre-
scription. A er 3 days without taking her alprazolam, she becomes extremely anxious
and shaky and has a tonic-clonic seizure.
a. What property o CNS drugs has led to this state o hyperexcitability in
this patient?
Alprazolam is a CNS depressant used commonly to treat anxiety and insomnia (see
Chapter 9).
Acute, excessive stimulation o the cerebrospinal axis (not the case with this
patient) normally is ollowed by depression, which is in part a consequence o neu-
ronal atigue and exhaustion o stores o transmitters. Acute, drug-induced depres-
sion (also not the case with this patient) is not ollowed by stimulation. However,
chronic drug-induced sedation or depression (such as with this patient) may be
ollowed by prolonged hyperexcitability upon abrupt cessation o the medication.
T is e ect is also common in alcohol withdrawal (see Chapter 9).

KEY CONCEPTS
T e autonomic nervous system, also called the vegetative, visceral, or involuntary
nervous system, regulates body unctions that occur without control (see able 5-1).
T e autonomic nervous system consists o nerves, ganglia, and plexuses that
innervate the heart, blood vessels, glands, other visceral organs, and smooth
muscle in various tissues (see Figure 5-1).
T e autonomic nervous system comprises the parasympathetic, sympathetic,
and enteric nervous systems.
T e somatic nervous system innervates skeletal muscle.
T e CNS is made up o various anatomic regionscerebral cortex, limbic sys-
tem, diencephalon, midbrain and brainstem, cerebellum, and spinal cord with a
complex assembly o interacting neurons and nuclei that regulate their own and
each others activities generally through chemical neurotransmission.
T e parasympathetic and sympathetic systems have ganglia where ACh is the
transmitter o the preganglionic bers (see Figure 5-2). In these ganglia, ACh
acts on nicotinic receptors on the postganglionic nerves.
ACh is also the neurotransmitter at cells o the adrenal medulla where it acts on
nicotinic ACh receptors.

92
Neurotransmission CHAPTER 5

ACh is the predominant neurotransmitter o postganglionic parasympathetic


nerves and acts on muscarinic receptors in the target tissues.
NE is the principal neurotransmitter o postganglionic sympathetic nerves
acting on and adrenergic receptors.
In the CNS there are several other transmitters beside ACh and NE, including
neuropeptides (see Side Bar NEURO RANSMI ERS and able 5-3).
In the peripheral nervous system, the actions o agonists and antagonist are
predictable based on their interactions with speci c receptors (see able 5-1).
In the CNS, the actions o agonist and antagonists are not as predictable as in
the peripheral nervous system due to interacting neurons and nuclei.

SUMMARY QUIZ

QUESTION 5-1 A 42-year-old man has just been prescribed a new drug. A er several
doses he notices dry mouth, dry eyes, and a rapid heart rate. T is is most likely due to
an inhibition o which o the ollowing neurotransmitter:
a. Norepinephrine
b. Serotonin
c. Glutamate
d. Acetylcholine
e. Epinephrine

QUESTION 5-2 A 23-year-old man is accosted by 3 men late at night. Immediately


his heart rate increases due to an increase in epinephrine in his circulation. T is
release o epinephrine rom the adrenal medulla is the result o the action o acetyl-
choline on
a. muscarinic receptors.
b. adrenergic receptors.
c. nicotinic receptors.
d. adrenergic receptors.
e. glutamate receptors.

QUESTION 5-3 A 56-year-old woman goes to an ophthalmologist or an eye examina-


tion. T e ophthalmologist administers an eye drop to dilate her pupils prior to the eye
examination. T e medication in the eye drop is most likely acting on
a. serotonin receptors.
b. 1 adrenergic receptors.
c. 2 adrenergic receptors.
d. nicotinic receptors.
e. acetylcholinesterase.

QUESTION 5-4 T e neurotransmitter at the neuromuscular junction is


a. acetylcholine.
b. norepinephrine.
c. serotonin.
d. aspartate.
e. epinephrine.

93
SECTION II Neuropharmacology

QUESTION 5-5 A 33-year-old woman with a brain in ection (meningitis) is adminis-


tered her antibiotic directly into the cerebrospinal f uid because the
a. bacteria causing the brain in ection are not sensitive to the antibiotic.
b. blood brain barrier excludes the antibiotic i it is administered systemically.
c. antibiotic binds to brain cells and is not available to the bacteria.
d. antibiotic is activated by the cerebrospinal f uid.
e. bacteria are mostly located in the cerebrospinal f uid.

QUESTION 5-6 An 80-year-old woman is administered a CNS depressant drug or


anxiety. When she abruptly stops the drug, a er taking it or many years, she is likely to
experience
a. increased sedation.
b. hyperexcitability.
c. a depressed mood.
d. nausea.
e. blurred vision.

SUMMARY QUIZ ANSWER KEY


QUESTION 5-1 Answer d. ACh is the predominant neurotransmitter o postgangli-
onic parasympathetic nerves. A cholinergic antagonist acts on muscarinic receptors to
produce the side e ects noted in this patient (see Figures 5-1 and 5-2, and able 5-1).
QUESTION 5-2 Answer c. ACh is the neurotransmitter at cells o the adrenal medulla,
where it acts on nicotinic ACh receptors to cause the release o the catecholamines,
epinephrine (80%) and norepinephrine (20%), into the circulation.
QUESTION 5-3 Answer b. T e eye drop used is most likely an 1 adrenergic agonist
that is acting on receptors to cause contraction (mydriasis) o the radial muscle in the
iris (see able 5-1).
QUESTION 5-4 Answer a. ACh is the transmitter at the neuromuscular junction o
somatic nerves (see Figure 5-2).
QUESTION 5-5 Answer b. Drugs acting in the CNS have to cross the BBB or the
blood-CSF barrier (see Chapter 2). T ese 2 barriers are ormed by brain capillary endo-
thelial cells and epithelial cells o the choroid plexus, respectively. Lipophilic molecules
di use reely across the BBB and accumulate in the brain, whereas less lipophilic drugs
(such as most antibiotics) do not easily penetrate the brain and are o en administered
directly into the cerebrospinal f uid thus bypassing the BBB (see Case 5-4.)
QUESTION 5-6 Answer b. Chronic drug-induced sedation or depression (such as with
this patient) may be ollowed by prolonged hyperexcitability upon abrupt cessation o
the medication (see Case 5-7.)

94
CHAPTER

CholinergicPharmacology 6
Chapter 6 Cholinergic Pharmacology is a combination o Chapter 9 Muscarinic
Receptor Agonists and Antagonists Chapter 10 Anticholinesterase Agents, and
DRUGS (OR AGENTS)
Chapter 11 Agents Acting at the Neuromuscular Junction and Autonomic Ganglia in INCLUDED IN
Goodman and Gilmans T e Pharmacological Basis of T erapeutics, 12th Edition. An THIS CHAPTER
understanding o the material in these chapters will be help ul in ollowing the material Acetylcholine (MIOCHOL-E)
presented in this chapter. In addition to the material presented here the above chapters Aldicarb (TEMIK)Insecticide
in the 12th Edition include:
Ambenoniumchloride (MYTELASE)
A complete discussion o the properties and subtypes o muscarinic receptors
Atracurium(TACRIUM, others)
T e pharmacological e ects o acetylcholine (ACh)
Atropine
A detailed discussion o the pharmacological e ects o atropine, a muscarinic
receptor antagonist Benztropine mesylate (COGENTIN, others);
see Chapter 13
A discussion o the toxicology o drugs with antimuscarinic properties
Bethanechol (URECHOLINE)
A review o the structure o acetylcholinesterase (AChE)
Carbachol (MIOSTAT,ISOPTOCARBACHOL)
A discussion o the mechanism o AChE
Carbaryl (SEVIN)Insecticide
A description o the nicotinic acetylcholine receptor
Cevimeline (EVOXAC)
A description o the actions o neuromuscular blocking agents on various organ
systems Chlorpyri os (DURSBAN, LORSBAN)
Insecticide
A discussion o the pathophysiology and treatment o malignant hyperthermia
Cisatracurium(NIMBEX)
A discussion o ganglionic transmission
Cyclopentolate hydrochloride
A description o the pharmacological actions o nicotine (CYCLOGYL, others)
LEARNING OBJECTIVES Dantrolene (DANTRIUM, others)
Understand cholinergic pharmacology, including the di erent types and sub- Dari enacin (ENABLEX)
types o cholinergic receptors expressed in the central nervous system (CNS), Diazinon (SPECTRICIDE, others)Insecticide
autonomic nervous system (ANS), and neuromuscular junction. Dicyclomine hydrochloride (BENTYL, others)
Know the mechanisms o synthesis, storage, release, and destruction o ACh Donepezil (ARICEPT)
in the CNS, parasympathetic and sympathetic ganglia, parasympathetic nerve Doxacurium
bers, and the neuromuscular junction.
Edrophonium
Know the uses and toxicities o muscarinic receptor agonists and antagonists.
Fesoterodine (TOVIAZ)
Know the uses and toxicities o drugs and agents that inhibit AChE. Flavoxate (Urispas)
Describe the uses and toxicities o drugs that block neuromuscular Galantamine (REMINYL)
transmission.
Glycopyrrolate (ROBINUL, others)
Homatropine hydrobromide (ISOPTO
MECHANISM OF ACTION MUSCARINIC AGONISTS AND ANTAGONISTS HOMATROPRINE)
DRUG CLASS DRUG MECHANISM OF ACTION Ipratropium(ATROVENT,others)
Muscarinic Receptor Acetylcholine Neurotransmitter at nicotinic and Malathion (CHEMATHION,
Agonists muscarinic receptors (see Figures 5-2, MALA-SPRAY)Insecticide
6-1, and Table 6-1 or details)
Mecamylamine (INVERSINE)
Pilocarpine Muscarinic receptor agonist Methacholine (PROVOCHOLINE)
Methacholine Muscarinic receptor agonist Methscopolamine bromide (PAMINE)
Carbachol Muscarinic receptor agonist Mivacurium(Not available in the United States)
Neostigmine bromide (PROSTIGMIN-oral)
Bethanechol Muscarinic receptor agonist
(continues)
Cevimeline Muscarinic receptor agonist

95
SECTION II Neuropharmacology

DRUGS (OR AGENTS) DRUG CLASS DRUG MECHANISM OF ACTION


INCLUDED IN THIS Muscarinic Receptor Atropine Muscarinic receptor antagonist
CHAPTER (Cont.) Antagonists
Scopolamine Muscarinic receptor antagonist
Neostigmine methylsul ate
(PROSTIGMIN-parenteral) Ipratropium Muscarinic receptor antagonist

Nicotine gumor lozenge (NICOTINEPOLACRI- Tiotropium Muscarinic receptor antagonist


LEX,NICORETTE, COMMIT,THRIVE, others)
Oxybutynin Muscarinic receptor antagonist
Nicotine nasal sprayor vapor inhaler
(NICOTROL) Tolterodine Muscarinic receptor antagonist
Nicotine transdermal patch (HABITROL, Trospium chloride Muscarinic receptor antagonist
others)
Dari enacin Muscarinic receptor antagonist
Onabotulinumtoxin A(BOTOX)
Oxybutynin (DITROPAN, others; OXYTROL- Soli enacin Muscarinic receptor antagonist
transdermal; GELNIQUE-gel) Fesoterodine Muscarinic receptor antagonist
Pancuronium
Flavoxate Muscarinic receptor antagonist
Physostigmine salicylate (ANTILIRIUM)
Pilocarpine (ISOPTOCARPINE) Glycopyrrolate Muscarinic receptor antagonist
Pilocarpine hydrochloride (SALAGEN, others) Dicyclomine hydrochloride Muscarinic receptor antagonist
Pipecuronium Homatropine hydrobromide Muscarinic receptor antagonist
Pralidoxime (2-PAM; PROTOPAMCHLORIDE)
Cyclopentolate hydrochloride Muscarinic receptor antagonist
Propoxur (BAYGON)Insecticide
Pyridostigmine bromide (MESTINON-oral; Tropicamide Muscarinic receptor antagonist
REGINOL, MESTINON-parenteral) Benztropine mesylate Muscarinic receptor antagonist
Rimabotulinumtoxin B(MYOBLOC)
Trihexyphenidyl hydrochloride Muscarinic receptor antagonist
Rivastigmine (EXELON)
Rocuronium(ZEMURON, others) Methscopolamine bromide Muscarinic receptor antagonist

SarinNerve gas
Scopolamine (TRANSDERMSCOP)
Soli enacin (VESICARE) CASE 6-1
SomanNerve gas A 65-year-old man with urinary retention and inadequate emptying o the bladder is
Succinylcholine (ANECTINE, QUELICIN) being treated with bethanechol.
TabunNerve gas a. Why is he treated with this drug?
Tacrine (COGNEX) Bethanechol is a muscarinic receptor agonist. Parasympathetic stimulation causes
Tiotropium(SPIRIVA) detrusor muscle contraction, increased voiding pressure, and ureteral peristalsis
Tolterodine (DETROL) (see ables 5-1, 6-2, and 6-3). Bethanechol has utility in treating urinary retention
Trihexyphenidyl hydrochloride (ARTANE, and inadequate emptying o the bladder when organic obstruction is absent, as in
others); see Chapter 13 postoperative retention, diabetic autonomic neuropathy, and certain types o neu-
rogenic bladder.
Trimethaphan
Tropicamide (MYDRIACYL, others) b. What side e ects should he be aware o while taking bethanechol?
Trospiumchloride (SANCTURA) T e side e ects this patient might experience are a predictable consequence o
muscarinic receptor stimulation (see able 5-1), or example, diaphoresis, diarrhea,
Varenicline (CHANTIX) abdominal cramps, lacrimation, salivation, bradycardia, and bronchial secretion.
Vecuronium(NORCURON, others)
c. What are the contraindications to the use o bethanechol?
T e important contraindications to using bethanechol are asthma, chronic obstruc-
tive pulmonary disease (COPD), urinary or GI tract obstruction, acid-peptic
disease, cardiovascular disease accompanied by bradycardia, hypotension, and
hyperthyroidism (muscarinic agonists may precipitate atrial brillation in hyper-
thyroid patients).

96
Cholinergic Pharmacology CHAPTER 6

He micholinium

AcCoA + Choline Choline

ChAT Na +
Mitochondrion
ACh 3Na +
Ve s a micol
ADP Na + , K+
-ATP a s e
Choline rgic ACh ATP
va ricos ity Co-T
2K+

Ca 2+ N

M
VAMP S

Ca 2+ Co-T Choline
ACh
Ace ta te

Botulinium
toxin S NAP S
Effe ctor ce ll
ACh me mbra ne

AChE
nAChR
(N n -Nm )
mAChR
(M1 -M 5 )

FIGURE 6-1 A cholinergic neuroe ector junction showing eatures o the synthesis, storage, and release o acetylcholine (ACh) and receptors
on which ACh acts. The synthesis o ACh in the varicosity depends on the uptake o choline via a sodium-dependent carrier. This uptake can be
blocked by hemicholinium. Choline and the acetyl moiety o acetyl coenzyme A, derived rom mitochondria, orm ACh, a process catalyzed by
the enzyme choline acetyl trans erase (ChAT). ACh is transported into the storage vesicle by another carrier that can be inhibited by vesamicol.
ACh is stored in vesicles along with other potential cotransmitters (Co-T) such as ATP and VIP at certain neuroe ector junctions. Release o ACh
and the Co-T occurs on depolarization o the varicosity, which allows the entry o Ca2+ through voltage-dependent Ca2+ channels. Elevated
[Ca2+]in promotes usion o the vesicular membrane with the cell membrane, and exocytosis o the transmitters occurs. This usion process
involves the interaction o specialized proteins associated with the vesicular membrane (VAMPs, vesicle-associated membrane proteins) and the
membrane o the varicosity (SNAPs, synaptosome-associated proteins). The exocytotic release o ACh can be blocked by botulinum toxin. Once
released, ACh can interact with the muscarinic receptors (M), which are GPCRs, or nicotinic receptors (N), which are ligand-gated ion channels, to
produce the characteristic response o the e ector. ACh also can act on presynaptic mAChRs or nAChRs to modi y its own release. The action o
ACh is terminated by metabolism to choline and acetate by acetylcholinesterase (AChE), which is associated with synaptic membranes.

CASE 6-2
A 56-year-old man reports to the emergency room a er eating mushrooms that he has
oraged. He has brought some o the whole, uncooked mushrooms with him.
a. Although his symptoms now, one hour a er ingesting the mushrooms, only
include mild abdominal pain and diarrhea, what symptoms should one look or?
It is important to identi y the speci c mushroom, i possible, to be able to be aware
o the possible toxicities. Amanita muscaria mushrooms contain muscarine, a
potent muscarinic receptor agonist, and symptoms will be related to stimulation o
muscarinic receptors (see able 5-1). Mushrooms o the Psilocybe or Panaeolus spe-
cies contain psilocybin which will cause short-lasting hallucinations. Mushrooms
o the Amanita phalloides or related species contain amatoxins which can cause
hepatic and renal ailure, and ultimately death.
(Continued)

97
SECTION II Neuropharmacology

TABLE 6-1 Characteristics o Subtypes o Nicotinic Acetylcholine Receptors nAChRs


RECEPTOR Primary MAIN SYNAPTIC MOLECULAR
Receptor Subtype a LOCATION MEMBRANE RESPONSE MECHANISM AGONISTS ANTAGONISTS
Skeletal Muscle (Nm) Skeletal Excitatory; end-plate Increased cation ACh Atracurium
(1)2 1 adult (1)2 neuromuscular depolarization; skeletal permeability Nicotine Vecuronium
1 etal junction muscle contraction (Na+; K+) Succinylcholine d-Tubocurarine
(postjunctional) Pancuronium
-Conotoxin
-Bungarotoxin

Peripheral Neuronal Autonomic Excitatory; depolarization; Increased cation ACh Trimethaphan


(Nn) ganglia; adrenal ring o postganglion permeability Nicotine Mecamylamine
(3)2( 4)3 medulla neuron; depolarization and (Na+; K+) Epibatidine
secretion o catecholamines Dimethylphenyl-
piperazinium

Central Neuronal CNS; pre- and Pre- and postsynaptic Increased cation Cytosine, epibatidine Mecamylamine
(CNS) postjunctional excitation permeability Anatoxin A Dihydro--erythrodine
(4)2( 4)3 Prejunctional control o (Na+; K+) Erysodine
(-btox-insensitive) transmitter release Lophotoxin

(7)5 (-btox-sensitive) CNS; pre- and Pre- and postsynaptic Increased Anatoxin A Methyllycaconitine
postsynaptic excitation permeability -Bungarotoxin
Prejunctional control o (Ca2+) -Conotoxin ImI
transmitter release
a
Nine (2-10) and three ( 2- 4) subunits have been identi ed and cloned in human brain, which combine in various con ormations to orm
individual receptor subtypes. The structure o individual receptors and the subtype composition are incompletely understood. Only a nite
number o naturally occurring unctional nAChR constructs have been identi ed. -btox, -bungarotoxin.

TABLE 6-2 Characteristics o Muscarinic Acetylcholine Receptor Subtypes mAChRs


SIZE; CELLULAR AND DISEASE
RECEPTOR CHROMOSOME TISSUE LOCATIONa CELLULAR RESPONSEb FUNCTIONAL RESPONSEc RELEVANCE
M1 460 aa CNS; Most abundant Couples by Gq/11 Increased cognitive unction Alzheimers disease
11q 1213 in cerebral cortex, Activation o PLC; IP3 and (learning and memory) Cognitive
hippocampus, DAG Increased seizure activity dys unction
striatum and thalamus Ca2+ and PKC Decrease in dopamine release Schizophrenia
Autonomic ganglia Depolarization and excitation and locomotion
Glands (gastric and ( sEPSP) Increase in depolarization o
salivary) Activation o PLD2, PLA2; AA autonomic ganglia
Enteric nerves Increase in secretions

M2 466 aa Widely expressed Couples by Gi/Go Hea rt: Alzheimers disease


7q 3536 in CNS, hind (PTX-sensitive) SA node: slowed spontaneous Cognitive
brain, thalamus, Inhibition o AC, cAMP depolarization; hyperpolarization, HR dys unction
cerebral cortex, Activation o inwardly AVnode: decrease in conduction Pain
hippocampus, recti ying K+ channels velocity
striatum, heart, Inhibition o voltage-gated Atrium: re ractory period,
smooth muscle, Ca2+ channels contraction
autonomic nerve Hyperpolarization and Ventricle: slight contraction
terminals inhibition Smooth muscle:
Contraction
Periphera l nerves:
Neural inhibition via autoreceptors
and heteroreceptor
Ganglionic transmission
CNS:
Neural inhibition
Tremors; hypothermia; analgesia

98
Cholinergic Pharmacology CHAPTER 6

SIZE; CELLULAR AND DISEASE


RECEPTOR CHROMOSOME TISSUE LOCATIONa CELLULAR RESPONSE b
FUNCTIONAL RESPONSE c
RELEVANCE
M3 590 aa Widely expressed in Couples by Gq/11 Smooth muscle: Chronic obstructive
1q 4344 CNS (< than other Activation o PLC; contraction (predominant pulmonary disease
mAChRs), cerebral IP3 and DAG in some, eg, bladder) (COPD)
cortex, hippocampus Ca2+ and PKC Gla nds: Urinary
Abundant in smooth Depolarization and excitation secretion (predominant in incontinence
muscle and glands ( sEPSP) salivary gland) Irritable bowel
Heart Activation o PLD2, PLA2; AA Increases ood intake, disease
body weight at
deposits
Inhibition o DA release
Synthesis o NO

M4 479 aa Pre erentially Couples by Gi/Go Autoreceptor- and heteroreceptor- Parkinson disease
11p 1211.2 expressed in CNS, (PTX-sensitive) mediated inhibition o transmitter Schizophrenia
particularly orebrain, Inhibition o AC, cAMP release in CNS and periphery. Neuropathic pain
also striatum, Activation o inwardly Analgesia; cataleptic activity
cerebral cortex, recti ying K+ channels Facilitation o DA release
hippocampus Inhibition o voltage-gated
Ca2+ channels
Hyperpolarization and
inhibition

M5 532 aa Substantia nigra Couples by Gq/11 Mediator o dilation in cerebral Drug dependence
15q 26 Expressed in low Activation o PLC; arteries and arterioles (?) Parkinson disease
levels in CNS and IP3 and DAG Facilitates DA release Schizophrenia
periphery Ca2+ and PKC Augmentation o drug-seeking
Predominant mAchR Depolarization and excitation behavior and reward (eg, opiates,
in neurons in VTA ( sEPSP) cocaine)
and substantia nigra Activation o PLD2, PLA2;
AA
a
Most organs, tissues, and cells express multiple mAChRs.
b
M1, M3, and M5 mAChRs appear to couple to the same G proteins and signal through similar pathways. Likewise, M2 and M4 mAChRs couple
through similar G proteins and signal through similar pathways.
c
Despite the act that in many tissues, organs, and cells multiple subtypes o mAChRs coexist, one subtype may predominate in producing a
particular unction; in others, there may be equal predominance.
PLC, phospholipase C; IP3, inositol-1,4,5-triphosphate; DAG, diacylglycerol; PLD2, phospholipase D; AA, arachidonic acid; PLA, phospholipase A;
AC, adenylyl cyclase; DA, dopamine; cAMP, cyclic AMP; SA node, sinoatrial node; AVnode, atrioventricular node; HR, heart rate; PTX, pertussis
toxin; VTA, ventral tegmentum area.

TABLE 6-3 Some Pharmacological Properties o Choline Esters and Natural Alkaloids
MUSCARINIC ACTIVITY
MUSCARINIC SUSCEPTIBILITYTO URINARY EYE ANTAGONISM NICOTINIC
AGONIST CHOLINESTERASES CARDIOVASCULAR GASTROINTESTINAL BLADDER (TOPICAL) BY ATROPINE ACTIVITY
Acetylcholine +++ ++ ++ ++ + +++ ++

Methacholine + +++ ++ ++ + +++ +

Carbachol - + +++ +++ ++ + +++

Bethanechol - +++ +++ ++ +++ -

Muscarine a - ++ +++ +++ ++ +++ -

Pilocarpine - + +++ +++ ++ +++ -


a
Not used therapeutically

99
SECTION II Neuropharmacology

b. T e mushrooms he has with him have been identif ed as Amanita muscaria.


What is the treatment or this type o mushroom poisoning?
reatment with atropine e ectively blocks the e ects o muscarinic receptor stimu-
lation. Large doses may be required. I the patient is showing signs o CNS excita-
tion and hallucinations, these symptoms should be treated with a benzodiazepine
(see Chapter 9). Atropine o en exacerbates the delirium.

CASE 6-3
A 48-year-old woman is being treated with oxybutynin or low bladder capacity and
urinary requency.
a. What kind o drug is oxybutynin and why is it e ective in this patient?
Oxybutynin is a muscarinic receptor antagonist that is indicated or the treatment
o overactive bladder (see able 6-4). T is drug lowers intravesicular pressure,
increases bladder capacity, and reduces the requency o contractions by antagoniz-
ing parasympathetic control o the bladder (see ables 5-1 and 6-3).
b. What side e ects should she be cautioned about?
T e side e ects o oxybutynin are a predictable consequence o muscarinic recep-
tor antagonism (see able 5-1). T ese e ects include xerostomia, blurred vision,
constipation, and dyspepsia. CNS-related antimuscarinic e ects, including drowsi-
ness, dizziness, and con usion can occur and are particularly problematic in elderly
patients. Dry mouth and dry eyes are the most common reasons or discontinuation.

MECHANISMS OF ACTION OF ACETYLCHOLINESTERASE (AChE) INHIBITORS


DRUG CLASS DRUG MECHANISM OF ACTION
ReversibleAChE Inhibitors Edrophonium AChE inhibition
Physostigmine salicylate AChE inhibition
Neostigmine bromide AChE inhibition
Neostigmine methylsul ate AChE inhibition
Ambenonium chloride AChE inhibition
Pyridostigmine bromide AChE inhibition
Tacrine AChE inhibition
Donepezil AChE inhibition
Rivastigmine AChE inhibition
Galantamine AChE inhibition
Carbamate Insecticides Carbaryl (SEVIN) Insecticide AChE inhibition
Propoxur AChE inhibition
Aldicarb AChE inhibition
Organophosphate Insecticides Diazinon AChE inhibition
Chlorpyri os AChE inhibition
Malathion AChE inhibition
Organophosphate Nerve Gases Tabun AChE inhibition
Sarin AChE inhibition
Soman AChE inhibition

Cholinesterase Reactivators Pralidoxime (2-PAM) AChE reactivator

100
Cholinergic Pharmacology CHAPTER 6

TABLE 6-4 Muscarinic Receptor Antagonists Used in the Treatment o Overactive Urinary Bladder
t 1/2
NONPROPRIETARY NAME TRADE NAME hours PREPARATIONSa DAILY DOSE ADULT
Oxybutynin DITROPAN, others 2-5 IR 10-20 mg b
ER 5-30 mg b
OXYTROL Transdermal patch 3.9 mg
GELNIQUE Topical gel 100 mg

Tolterodine DETROL 2-9.6c IR 2-4 mg b,d


6.9-18c ER 4 mg b,d

Trospium chloride SANCTURA 20 IR 20-40 mg e


35 ER 60 mg e

Soli enacin VESICARE 55 IR 5-10 mg b

Dari enacin ENABLEX 13-19 ER 7.5-15 mg

Fesoterodine TOVIAZ 7 ER 4-8 mg


a
Preparations are designated as ollows: IR, immediate-release tablet; ER, extended-release tablet or capsule.
b
Doses may need to be reduced in patients taking drugs that inhibit CYP3A4.
c
Longer times in indicated ranges are seen in poor metabolizers.
d
Doses should be reduced in patients with signi cant renal or hepatic impairment.
e
Doses should be reduced in patients with signi cant renal impairment; dosage adjustments also may be needed in patients with hepatic
impairment.
Doses may need to be reduced in patients taking drugs that inhibit CYPs 3A4 or 2D6.

CASE 6-4 POTENTIAL EFFECTS


A 14-year-old boy is brought to the emergency room a er drinking a tea made with OF AGENTS THAT
jimson weed seeds. INHIBIT ACh E
a. What is jimson weed and what are the likely symptoms this boy may experience? Stimulation o muscarinicreceptor
responses at autonomice ector organs
Jimson weed is a common plant that contains atropine and other belladonna
alkaloids. T e seeds contain high concentrations o these alkaloids and poison- Stimulation, ollowed bydepression or
ing results rom their ingestion. T e symptoms o various doses o atropine are paralysis, o all autonomicganglion and
shown in able 6-5 and are predictable as a consequence o muscarinic receptor skeletal muscle (nicotinicactions)
antagonism. Stimulation, with occasional subsequent
(Continued) depression, o cholinergicreceptor sites in
the CNS

TABLE 6-5 E ects o Atropine in Relation to Dose


DOSE mg EFFECTS
0.5 Slight cardiac slowing; some dryness o mouth; inhibition o sweating

1 De nite dryness o mouth; thirst; acceleration o heart, sometimes preceded by slowing; mild dilation o pupils

2 Rapid heart rate; palpitation; marked dryness o mouth; dilated pupils; some blurring o near vision

5 Above symptoms marked; di culty in speaking and swallowing; restlessness and atigue; headache; dry, hot skin; di culty in
micturition; reduced intestinal peristalsis

10 Above symptoms more marked; pulse rapid and weak; iris practically obliterated; vision very blurred; skin f ushed, hot, dry,
and scarlet; ataxia, restlessness, and excitement; hallucinations and delirium; coma

The clinical picture o a high (toxic) dose o atropine may be remembered by an old mnemonic device that summarizes the symptoms: Red as a
beet, Dry as a bone, Blind as a bat, Hot as f restone, and Mad as a hatter.

101
SECTION II Neuropharmacology

b. I his symptoms are serious and li e threatening, what is the appropriate


SYMPTOMS OF ACUTE
treatment?
ORGANOPHOSPHATE
Measures to limit absorption o the atropine should be initiated without delay i the
POISONING
poison has been taken orally, as in this case. For symptomatic treatment, slow intra-
MuscarinicMild venous injection o physostigmine (an AChE inhibitor) will increase ACh at mus-
Miosis carinic receptors. Physostigmine will rapidly abolish the delirium and coma caused
Ocular pain by high doses o atropine, but carries some risk in mild atropine intoxication.
Because physostigmine is metabolized rapidly, the patient may again lapse into
Conjunctival congestion a coma within 1 to 2 hours and repeated doses may be needed. I marked excite-
Diminished vision ment is present, and more speci c treatment is not available, a benzodiazepine (see
Salivation Chapter 9) is the most appropriate treatment to induce sedation and prevent sei-
Lacrimation zures. Support o respiration and control o hyperthermia may be necessary.
Rhinorrhea
CASE 6-5
Bronchoconstriction
Increased bronchial secretions A 23-year-old woman is brought to the emergency room a er deliberately ingesting a
bottle o organophosphate insecticide.
Anorexia
Nausea and vomiting a. Why is it important to identi y the specif c product this woman ingested?
Diarrhea T ere are two types o insecticides in this class: carbamate insecticides and
organophosphate insecticides (see able 6-6). T e carbamate insecticides are
Abdominal cramps
reversible and inhibit AChE in a ashion identical to other carbamoylating
MuscarinicSevere agents (physostigmine and neostigmine), while the organophosphate insecticides
Extreme salivation, lacrimation, and inhibit AChE in an irreversible manner by alkylphosphorylation. T e organophos-
sweating phate inhibition o AChE is initially reversible, but ages into an enzyme inhibi-
Involuntaryde ecation and urination tion that is resistant to hydrolysis and reactivation (see Figure 6-2). T e symptoms
o poisoning rom both insecticides resemble each other, but poisoning rom
Penile erection
an organophosphate insecticide will bene t rom the early administration o an
Bradycardia AChE reactivator (see below).
Hypotension
b. What symptoms is she likely to experience and what is the time rame that these
NicotinicMild symptoms may appear?
Generalized muscle weakness T e e ects o acute intoxication by anticholinesterase (anti-ChE) insecticides are
Fatigability mani ested by stimulation o muscarinic and nicotinic receptors (see able 5-1).
Involuntarytwitching and scattered In addition, there may be CNS e ects except with those agents that have low lipid
asciculations solubility that cannot cross the blood-brain barrier. T e symptoms o poisoning
with AChE inhibitors are highlighted in the Side Bar SYMP OMS OF ACU E
NicotinicSevere
ORGANOPHOSPHA E POISONING.
Severe muscle weakness
A er the inhalation o vapors or aerosols, systemic e ects may occur within min-
Paralysis utes, whereas the systemic e ects are delayed a er GI or percutaneous absorption.
Central Nervous System T e duration o toxic symptoms is determined largely by the properties o the
Con usion compound: its lipid solubility, whether it must be activated to orm the oxon, the
stability o the organophosphate-AChE bond, and whether aging o the phos-
Ataxia
phorylated enzyme has occurred. Delayed symptoms appearing a er 1 to 4 days
Slurred speech and marked by persistent low blood ChE and severe muscle weakness are termed
Loss o refexes the intermediate syndrome.
Generalized seizures c. What is the appropriate treatment or this ingestion?
Coma Atropine in su cient dosage (large doses may be required) e ectively antagonizes
Central respiratoryparalysis the e ects at muscarinic receptor sites. Atropine is virtually without e ect against
the peripheral neuromuscular junction (nicotinic) e ects.
T e nicotinic e ects o acute organophosphate poisoning can be reversed by prali-
doxime (2-PAM), a cholinesterase reactivator. T e mechanism o action o prali-
doxime is shown in Figure 6-2. T e reactivation o AChE is most pronounced at the
skeletal neuromuscular junction. Because pralidoxime has weak anti-ChE activity,
it is not recommended or the treatment o overdosage with physostigmine or neo-
stigmine or poisoning with the carbamoylating insecticides such as carbaryl.

102
Cholinergic Pharmacology CHAPTER 6

TABLE 6-6 Chemical Classif cation o Representative Organophosphorus Compounds o Particular Pharmacological or
Toxicological Interest
R1 O (S )
General ormula P
R2 X

Group A, X = halogen, cyanide, or thiocyanate leaving group; group B, X = alkylthio, arylthio, alkoxy, or aryloxy leaving group; group C,
thionophosphorus or thio-thionophosphorus compounds; group D, quaternary ammonium leaving group. R1 can be an alkyl (phosphonates),
alkoxy (phosphorates) or an alkylamino (phosphoramidates) group.
GROUP STRUCTURAL FORMULA COMMON, CHEMICAL, AND OTHER NAMES COMMENTS
A i-C 3 H7 O O DFP; Isof urophate; diisopropyl f uorophosphate Potent, irreversible inactivator
P
i-C 3 H7 O F

(CH3 )2 N O
Tabun Extremely toxic nerve gas
P
Ethyl N-dimethylphosphoramidocyanidate
C 2 H5 O CN

i-C 3 H7 O O
P Sarin (GB) Extremely toxic nerve gas
CH3 F Isopropyl methylphosphonof uoridate

CH3 CH3
CH3 C C
Soman (GD) Extremely toxic nerve gas; greatest
O O
CH3 H Pinacolyl methylphosphonof uoridate potential or irreversible action/rapid aging
P
CH3 F

B C 2 H5 O O Paraoxon (MINTACOL), E 600 Active metabolite o parathion


P O,O-Diethyl O-(4-nitrophenyl)-phosphate
C 2 H5 O O NO 2

CH3 O O Malaoxon Active metabolite o malathion


P O,O-Dimethyl
CH3 O S CHCOOC 2 H5 S-(1,2-dicarboxyethyl)-phosphorothioate

CH2 COOC 2 H5

C C 2 H5 O S Parathion Agricultural insecticide, resulting in


P O,O-Diethyl O-(4-nitrophenyl)-phosphorothioate numerous cases o accidental poisoning;
C 2 H5 O O NO 2 phased out in 2003

CH3 Diazinon, Dimpylate Insecticide; use limited to non-residential


C 2 H5 O S
O,O-Diethyl O-(2-isopropyl-6-methyl-4- agricultural settings
P N
pyrimidinyl) phosphorothioate
C 2 H5 O O CH3
N CH
CH3

Cl Chlorpyri os Insecticide; use limited to non-residential


S Cl O,O-Diethyl O-(3,5,6-trichloro-2-pyridyl) agricultural settings
N
H5 C 2 O P
phosphorothioate
H5 C 2 O O
Cl

CH3 O S Malathion Widely employed insecticide o greater


P O,O-Dimethyl S-(1,2-dicarbethoxyethyl) sa ety than parathion or other agents
CH3 O S CHCOOC 2 H5 phosphorodithioate because o rapid detoxi cation by higher
organisms
CH2 COOC 2 H5

D C 2 H5 O O I Echothiophate (PHOSPHOLINE IODIDE), MI-217 Extremely potent choline derivative;


P Diethoxyphosphinylthiocholine iodide administered locally in treatment o glaucoma;
+
C 2 H5 O S CH2 CH2 N(CH3 )3 relatively stable in aqueous solution

103
SECTION II Neuropharmacology

A. As s ocia tion of ACh Te tra he dra l tra ns ition s ta te Ace tyl e nzyme Hydrolys is of a ce tyl e nzyme
Glu 334
Glu 334 Glu 334 S e r 203 Glu 334
S e r 203 S e r 203 His 447 S e r 203
His 447 His 447 His 447

Gly 122
Gly 121

choline

B. Edrophonium complex C. Reve rs ible ne os tigmine binding Dime thyl ca rba moyl e nzyme Hydrolys is of dime thyl
Glu 334 S e r 203 ca rba moyl AChE
Glu 334 Glu 334 Glu 334 S e r 203
S e r 203 S e r 203
His 447 His 447 His 447

His 447

Age d monois opropyl E. Re a ctiva tion of


D. Reve rs ible DFP binding Diis opropyl phos phoryl AChE
phos phoryl AChE DFP-AChE by 2-PAM
Glu 334 Glu 334 Glu 334
S e r 203 S e r 203 S e r 203 Glu 334
S e r 203

His 447 His 447 His 447


His 447

Gly 122 Gly 122 Gly 122


Gly 121 Gly 121
Gly 121
2PAM

ca rbon oxyge n nitroge n hydroge n phos phorus fluorine

FIGURE 6-2 Steps involved in the hydrolysis o acetylcholine by acetylcholinesterase and in the inhibition and reactivation o the enzyme.
Only the 3 residues o the catalytic triad are depicted. The associations and reactions shown are: A. Acetylcholine (ACh) catalysis: binding o ACh,
ormation o a tetrahedral transition state, ormation o the acetyl enzyme with liberation o choline, rapid hydrolysis o the acetyl enzyme with
return to the original state. B. Reversible binding and inhibition by edrophonium. C. Neostigmine reaction with and inhibition o AChE: revers-
ible binding o neostigmine, ormation o the dimethyl carbamoyl enzyme, slow hydrolysis o the dimethyl carbamoyl enzyme. D. Diisopropyl
f uorophosphate (DFP) reaction and inhibition o AChE: reversible binding o DFP, ormation o the diisopropyl phosphoryl enzyme, ormation
o the aged monoisopropyl phosphoryl enzyme. Hydrolysis o the diisopropyl enzyme is very slow and is not shown. The aged monoisopropyl
phosphoryl enzyme is virtually resistant to hydrolysis and reactivation. The tetrahedral transition state o ACh hydrolysis resembles the conjugates
ormed by the tetrahedral phosphate inhibitors and accounts or their potency. Amide bond hydrogens rom Gly121 and Gly122 stabilize the car-
bonyl and phosphoryl oxygens. E. Reactivation o the diisopropyl phosphoryl enzyme by pralidoxime (2-PAM). 2-PAM attack o the phosphorus
on the phosphorylated enzyme will orm a phospho-oxime with regeneration o active enzyme. The individual steps o phosphorylation reaction
and oxime reaction have been characterized by mass spectrometry.

CASE 6-6
A 35-year-old woman with myasthenia gravis is being treated with ambenonium
chloride.
a. What is myasthenia gravis?
Myasthenia gravis is a neuromuscular disease characterized by weakness and
marked atigability o skeletal muscle; exacerbations and partial remissions occur
requently. T e de ect in myasthenia gravis is in synaptic transmission at the neuro-
muscular junction (see Figure 6-3). Myasthenia gravis is caused by an autoimmune
response primarily to the ACh receptor at the postjunctional end plate. A related
disease is Lambert-Eaton syndrome, in which antibodies are directed against Ca++
channels that are necessary or presynaptic release o ACh.
b. How is edrophonium used in the diagnosis o myasthenia gravis?
Although the diagnosis o myasthenia gravis usually can be made rom the his-
tory, signs, and symptoms, the use o the anti-ChE medication, edrophonium,
is o en use ul in con rming the diagnosis. T e edrophonium test is per ormed
(Continued)
104
Cholinergic Pharmacology CHAPTER 6

ANATOMY of the Motor End P la te P HYS IOLOGY P HARMACOLOGY


mye lin ne rve a ction te trodotoxin
s he a th pote ntia l ba tra chotoxin
(AP ) loca l a ne s the tics
a xon
node of he micholinium
ve s icula r botulinus toxin
Ra nvie r
a ce tylcholine proca ine , Mg 2+
S chwa nn re le a s e 4-a minopyridine
ce ll la ck of Ca 2+

s ubne ura l de pola riza tion (EP P ) exce s s of Ca 2+


s pa ce B (incre a s e d cura re a lka loids
pe rme a bility s na ke -toxins
A to Na + a nd K+) s uccinylcholine
te rmina l de ca me thonium
hydrolys is
me mbra ne choline s te ra s e
of a ce tylcholine
pos tjunctiona l by choline s te ra se
inhibitors
me mbra ne Ca 2+
mitochondria mus cle a ction ve ra tridine
pote ntia l quinine
te trodotoxin
s a rcopla s ma s pre a d of excita tion me ta bolic pois ons
in mus cle la ck of Ca 2+
proca ine
mus cle contra ction da ntrole ne
myofibrils
e nha nce me nt
blocka de
A B de pola riza tion
a nd pha s e block
B

FIGURE 6-3 Sites o action o agents at the neuromuscular junction and adjacent structures.
The anatomy o the motor end plate, shown at the le t, and the sequence o events rom lib-
eration o acetylcholine (ACh) by the nerve action potential (AP) to contraction o the muscle
ber, indicated by the middle column, are described in Chapter 8 o Goodman and Gilmans
The Pharmacological Basis o Therapeutics, 12th Edition. The modi cation o these processes by
various agents is shown on the right; an arrow marked with an X indicates inhibition or block;
an unmarked arrow indicates enhancement or activation. The insets are enlargements o the
indicated structures. The highest magni cation depicts the receptor in the bilayer o the postsyn-
aptic membrane.

by rapid intravenous injection o 2 mg o edrophonium chloride, ollowed


45 seconds later by an additional 8 mg i the rst dose is without e ect; a posi-
tive response consists o improvement in strength, unaccompanied by lingual
asciculations.
c. Why is ambenonium chloride used to treat this patient?
Ambenonium chloride along with pyridostigmine and neostigmine is the stan-
dard anti-ChE drug used in the symptomatic treatment o myasthenia gravis.
Following AChE inhibition, receptors at the end plate presumably are exposed to
concentrations o ACh that are su cient or channel opening and production o
a postsynaptic endplate potential (see Figures 6-1, 6-3, and 6-4).
d. What are the expected side e ects o this treatment?
An excessive dose o an anti-ChE drug results in a cholinergic crisis. T is condi-
tion is characterized by weakness resulting rom generalized depolarization o the
motor endplate and other eatures resulting rom stimulation o muscarinic recep-
tors. T e weakness may resemble myasthenia weakness. Cholinergic crisis and
myasthenia gravis can be distinguished by cautiously per orming the edrophonium
test, limiting the dose to 2 mg and with acilities or respiratory resuscitation avail-
able. A urther decrease in strength indicates cholinergic crisis, while improvement
signi es myasthenia weakness.
105
SECTION II Neuropharmacology

Pre g ang lio nic


Me mbra ne
axo n
Pote ntia l
0 Action Pote ntia l
Inte rne uro n
o r S IF c e ll S pike S low
S e ns o ry EP S P La te, S low
ne uro n EP S P EP S P
mV IP S P

Nn M2 M1 Pe ptide rgic

Po s tg ang lio nic 100 20 ms e c 10 s e c 1 min


ne uro n

FIGURE 6-4 Postsynaptic potentials recorded rom an autonomic postganglionic nerve cell
body a ter stimulation o the preganglionic nerve ber. The preganglionic nerve releases ACh
onto postganglionic cells. The initial excitatory postsynaptic potential (EPSP) results rom the
inward Na+ current (and perhaps Ca2+ current) through the nicotinic receptor channel. I the EPSP
is o su cient magnitude, it triggers an action potential spike, which is ollowed by a slow inhibi-
tory postsynaptic potential (IPSP), a slow EPSP, and a late, slow EPSP. The slow IPSP and slow EPSP
are not seen in all ganglia. The electrical events subsequent to the initial EPSP are thought to
modulate the probability that a subsequent EPSP will reach the threshold or triggering a spike.
Other interneurons, such as catecholamine-containing, small, intensely f uorescent (SIF) cells, and
axon terminals rom sensory, a erent neurons also release transmitters and that may inf uence
the slow potentials o the postganglionic neuron. A number o cholinergic, peptidergic, adrener-
gic, and amino acid receptors are ound on the dendrites and soma o the postganglionic neuron
and the interneurons. The preganglionic ber releases ACh and peptides; the interneurons store
and release catecholamines, amino acids, and peptides; the sensory a erent nerve terminals
release peptides. The initial EPSP is mediated through nicotinic (Nn) receptors, the slow IPSP and
EPSP through M2 and M1 muscarinic receptors, and the late, slow EPSP through several types o
peptidergic receptors.

MECHANISMS OF ACTION OF DRUGS ACTING AT THE NEUROMUSCULAR JUNCTION (NMJ) AND AUTONOMIC GANGLIA
DRUG CLASS DRUG MECHANISM OF ACTION
Depolarizing Neuromuscular Blocking Agents Succinylcholine (see Table 6-7) Block ACh at nicotinic receptors

Competitive Neuromuscular Blocking Agents Pancuronium (see Table 6-7) Block ACh at nicotinic receptors

Atracurium (see Table 6-7) Block ACh at nicotinic receptors

Vecuronium (see Table 6-7) Block ACh at nicotinic receptors

Pipecuronium (see Table 6-7) Block ACh at nicotinic receptors

Doxacurium (see Table 6-7) Block ACh at nicotinic receptors

Mivacurium (see Table 6-7) Block ACh at nicotinic receptors

Rocuronium (see Table 6-7) Block ACh at nicotinic receptors

Cisatracurium (see Table 6-7) Block ACh at nicotinic receptors

Botulinum Toxins Onabotulinum toxin A Block presynaptic release o Ach

Rimabotulinum toxin B Block presynaptic release o Ach

Antispasticity and Antirigidity Agents Dantrolene Inhibits Ca++ release o sarcoplasmic reticulum o skeletal
muscle

Ganglionic Stimulating Agents Nicotine Stimulation o cholinergic receptors on autonomic ganglia;


eventually blocks these receptors by persistent depolarization

Nicotinic ACh Receptor Agonists Varenicline Partial to ull agonist at nicotinic ACh receptors

Ganglionic Blocking Drugs Mecamylamine (see Table 6-8) Block ACh at ganglionic nicotinic receptors

Trimethaphan (see Table 6-8) Block ACh at ganglionic nicotinic receptors

106
Cholinergic Pharmacology CHAPTER 6

TABLE 6-7 Classif cation o Neuromuscular Blocking Agents


PHARMACOLOGICAL TIME OF CLINICAL
AGENT CHEMICAL CLASS PROPERTIES ONSET MIN DURATION MIN a MODE OF ELIMINATION
a

Succinyl choline Dicholine ester Ultrashort duration; 0.8-1.4 6-11 Hydrolysis by plasma
(ANECTINE, others) depolarizing cholinesterases
D-Tubocurarine b Natural alkaloid (cyclic Long duration; 6 80 Renal and hepatic elimination
benzyl-isoquinoline) competitive
Metocurine b Benzylisoquinoline Long duration; 4 110 Renal elimination
competitive
Atracurium (TRACRIUM, Benzylisoquinoline Intermediate duration; 3 45 Ho mann elimination;
others) competitive hydrolysis by plasma esterases
Cisatracurium (NIMBEX) Benzylisoquinoline Intermediate duration; 2-8 45-90 Ho mann and renal
competitive elimination
Doxacurium b Benzylisoquinoline Long duration; 4-8 120 Renal elimination
competitive
Mivacurium Benzylisoquinoline Short duration; 2-3 15-21 Hydrolysis by plasma
competitive cholinesterases
Pancuronium (generic) Ammonio steroid Long duration; 3-4 85-100 Renal and hepatic elimination
competitive
Pipecuronium b Ammonio steroid Long duration; 3-6 30-90 Renal elimination; hepatic
competitive metabolism and clearance
Rocuronium (ZEMURON, Ammonio steroid Intermediate duration; 0.9-1.7 36-73 Hepatic elimination
others) competitive
Vecuronium (NORCURON, Ammonio steroid Intermediate duration; 2-3 40-45 Hepatic and renal elimination
others) competitive
Gantacurium c Asymmetric mixed- Ultra-short duration, 1-2 5-10 Cysteine adduction and ester
onium chloro umarate competitive hydrolysis
a
Time o onset and clinical duration achieved rom therapeutic doses.
b
D-Tubocurarine, doxacurium, metocurine, and pipecuronium are no longer available in the United States.
c
Gantacurium is in investigational status.

TABLE 6-8 Usual Predominance o Sympathetic or Parasympathetic Tone at Various E ector Sites, and Consequences o
Autonomic Ganglionic Blockade
SITE PREDOMINANT TONE EFFECT OF GANGLIONIC BLOCKADE
Arterioles Sympathetic (adrenergic) Vasodilation; increased peripheral blood f ow; hypotension
Veins Sympathetic (adrenergic) Dilation: peripheral pooling o blood; decreased venous return;
decreased cardiac output
Heart Parasympathetic (cholinergic) Tachycardia
Iris Parasympathetic (cholinergic) Mydriasis
Ciliary muscle Parasympathetic (cholinergic) Cycloplegia ocus to ar vision
Gastrointestinal tract Parasympathetic (cholinergic) Reduced tone and motility; constipation; decreased gastric and
pancreatic secretions

Urinary bladder Parasympathetic (cholinergic) Urinary retention


Salivary glands Parasympathetic (cholinergic) Xerostomia
Sweat glands Sympathetic (cholinergic) Anhidrosis
Genital tract Sympathetic and parasympathetic Decreased stimulation

107
SECTION II Neuropharmacology

CASE 6-7
A 48-year-old man is undergoing abdominal surgery. Succinylcholine is used as a mus-
cle relaxant during the surgery.
a. What is succinylcholine and how does it act?
Succinylcholine is a depolarizing neuromuscular blocking agent. Its initial action is
to open channels in the same manner as ACh (see Figures 6-1 and 6-3). However, it
persists longer than ACh primarily because o its resistance to AChE. T e long-lasting
depolarization results in a brie period o repetitive excitation (resulting in muscle as-
ciculations) ollowed by block o neuromuscular transmission and f accid paralysis.
b. What are the contraindications or the use o succinylcholine?
Succinylcholine and other depolarizing agents can release K+ rom intracellular sites.
T is may be a actor in the production o the prolonged apnea in patients receiving these
drugs. T is is a li e-threatening complication and precludes the use o succinylcholine in
patients with so -tissue trauma, burns, or patients with congestive heart ailure who are
receiving digoxin or diuretics. Succinylcholine is also contraindicated or should be given
with great caution to patients with nontraumatic rhabdomyolysis, ocular lacerations,
spinal cord injuries with paraplegia or quadriplegia, or muscular dystrophies.
Hepatic disease that results in a de ciency o butyrylcholinesterase (which is
responsible or the rapid hydrolysis o succinylcholine) may also be responsible
or a prolonged apnea.

CASE 6-8
A 49-year-old man is undergoing abdominal surgery. Vecuronium is used as a muscle
relaxant during the surgery.
a. What is vecuronium, how does it act, and how does it di er rom
succinylcholine?
Vecuronium is a competitive blocker at the neuromuscular junction (see Figure 6-3).
It combines with the nicotinic ACh receptor at the end plate and thereby
competitively blocks the binding o ACh. able 6-7 shows the classi cation and
di erences between the neuromuscular blocking agents.
b. What are the contraindications or the use o vecuronium?
Vecuronium is eliminated by hepatic metabolism and caution should be exercised
when using this drug in patients with hepatic dys unction.

EFFECTS OF ACUTE CASE 6-9


NICOTINE POISONING A 2-year-old girl is brought to the emergency room a er having ingested 3 o her
Nausea mothers nicotine patches.
Salivation a. What is the danger rom this ingestion?
Abdominal pain Nicotine is a ganglionic stimulating agent and is readily absorbed rom the intes-
Vomiting tine. T e major action o nicotine consists initially o transient stimulation and
Headache subsequently o a more persistent depression o all autonomic ganglia and neuro-
muscular blockade by receptor desensitization. Nicotine markedly stimulates the
Cold sweat
CNS presumably due to the release o excitatory amino acids.
Dizziness
b. What are the symptoms o nicotine poisoning?
Disturbed hearing and vision
T e toxic e ects o acute nicotine poisoning are shown in the Side Bar EFFEC S
Mental con usion
OF ACU E NICO INE POISONING.
Muscle weakness
c. What is the appropriate treatment or this patient?
Hypotension
reatment options are limited. Attempts to limit absorption should be instituted
Seizures
(see Chapter 3). reatment o seizures with a benzodiazepine (see Chapter 9) may
Death romrespiratory ailure be appropriate. Respiratory assistance and treatment o shock may be necessary.

108
Cholinergic Pharmacology CHAPTER 6

KEY CONCEPTS
Muscarinic acetylcholine receptors in the peripheral nervous system are located
primarily on e ector organs that are innervated by postganglionic parasympa-
thetic nerves (see Figure 5-2).
Muscarinic acetylcholine receptors are also located in the CNS.
T e e ects o drugs that are muscarinic receptor agonists and antagonist can
be predicted rom knowledge o parasympathetic nerve innervation (see
ables 5-1 and 6-3).
Acetylcholine (ACh) is the neurotransmitter that interacts with muscarinic
receptors at the postganglionic parasympathetic nerve endings (see Figure 6-1).
ACh is also the neurotransmitter at nicotinic acetylcholine receptors o the
parasympathetic and sympathetic autonomic ganglia (see Figure 5-2).
ACh is also the neurotransmitter at nicotinic acetylcholine receptor at the
motor end plate o skeletal muscle (see Figures 5-2 and 6-3).
ACh at these synaptic sites is destroyed by acetylcholine esterase (AChE) (see
Figure 6-2).
Drugs and other agents that are inhibitors o AChE have e ects that are pre-
dicted rom knowledge o an excess o ACh at the above synaptic sites (see
able 5-1).
Drugs that block the nicotinic acetylcholine receptor at the motor endplate o
skeletal muscle are used as adjuvants during surgery to relax muscles.

SUMMARY QUIZ

QUESTION 6-1 A 35-year-old man has ingested Amanita muscaria mushrooms. He is


experiencing a heart rate o 40 beats per minute. T is bradycardia is most likely due to
an interaction o the chemical(s) in the mushrooms with which receptors at the sino-
atrial node?
a. 1 Adrenergic
b. 1 Adrenergic
c. 2 Adrenergic
d. M2 muscarinic
e. Nicotinic

QUESTION 6-2 A 49-year-old woman is treated with pilocarpine hydrochloride or


xerostomia (dry mouth) ollowing head and neck radiation treatments. As a result
o taking this drug she may experience which o the ollowing side e ects?
a. Sweating
b. Dry eyes
c. Dry skin
d. Con usion
e. achycardia

QUESTION 6-3 A 54-year-old woman is receiving Botox injections to remove acial


wrinkles. Botox (botulinum toxin) acts by
a. stimulating the release o ACh.
b. stimulating the release o norepinephrine.
c. blocking the release o ACh.
d. blocking the release o norepinephrine.
e. blocking muscarinic receptors.
109
SECTION II Neuropharmacology

QUESTION 6-4 A 23-year-old man has deliberately ingested an organophosphate


insecticide. His initial symptoms o salivation, lacrimation, and diarrhea are a conse-
quence o inhibition o
a. butyrylcholinesterase.
b. acetylcholinesterase.
c. Na+, K+-A Pase.
d. tyrosine hydroxylase.
e. monoamine oxidase.

QUESTION 6-5 T e patient in Question 6-4 should be treated with atropine and which
additional drug in the ollowing list?
a. Physostigmine
b. Bethanechol
c. Pralidoxime
d. Morphine
e. Gentamicin

QUESTION 6-6 A 65-year-old man with the diagnosis o Alzheimers disease is being
treated with donepezil. T is drug acts by
a. stimulating the release o ACh.
b. blocking the reuptake o norepinephrine.
c. inhibiting monamine oxidase.
d. blocking the release o ACh.
e. inhibiting AChE.

QUESTION 6-7 A 53-year-old man is scheduled to receive vecuronium as an adjuvant


muscle relaxant during abdominal surgery. T e dose o vecuronium may have to be
adjusted i this patient is also receiving
a. gentamicin.
b. penicillin.
c. ibupro en.
d. acetaminophen.
e. prednisone.

QUESTION 6-8 A 72-year-old woman receives succinylcholine as an adjuvant muscle


relaxant during knee surgery. T is drug acts by
a. blocking ACh at nicotinic receptors o neuromuscular endplate.
b. blocking ACh at nicotinic receptors in the adrenal medulla.
c. increasing Na+ and K+ permeability o the postsynaptic neuromuscular membrane.
d. blocking the transmission o the action potential along the nerve axon.
e. blocking the release o ACh rom neuromuscular presynaptic vesicles.

QUESTION 6-9 A 3-year-old boy is brought to the emergency room because he has
ingested a large amount o a nicotine-containing product. Serious nicotine toxicity in
this child is the result o
a. blockade o transmission at autonomic ganglia and neuromuscular junctions.
b. blockade o muscarinic receptors.
c. stimulation o adrenergic receptors.
d. blockade o adrenergic receptors.
e. stimulation o muscarinic receptors.

110
Cholinergic Pharmacology CHAPTER 6

SUMMARY QUIZ ANSWER KEY


QUESTION 6-1 Answer is d. Although the concentration o muscarine is low in
these mushrooms, the symptom o bradycardia observed in this patient is most likely
attributable to muscarine. Other chemicals that may be involved in the neurotoxicity
o mushrooms o the Amanita species are muscimol, ibotenic acid, and other isoxazole
derivatives. T e receptors in the sinoatrial node innervated by parasympathetic nerves
are M2 muscarinic receptors (see able 5-1).
QUESTION 6-2 Answer is a. Side e ects with pilocarpine treatment typi y choliner-
gic stimulation, with sweating being the most common complaint. T e other answers
would be side e ects seen a er treatment with a muscarinic receptor antagonist.
QUESTION 6-3 Answer is c. Botulinum toxins act peripherally to reduce muscle con-
traction. T e anaerobic bacterium Clostridium botulinum produces a amily o toxins
targeted to presynaptic proteins that block the release o ACh (see Figure 6-1). By
blocking ACh release, BO OX produces accid paralysis o skeletal muscle and dimin-
ished activity o parasympathetic and sympathetic cholinergic synapses. T e FDA has
issued a sa ety alert or BO OX, warning o respiratory paralysis rom unexpected
spread o the toxin rom the site o injection.
QUESTION 6-4 Answer is b. T e e ects o acute intoxication by anti-ChE agents such
as an organophosphate insecticide are mani ested by muscarinic and nicotinic signs
and symptoms, and, except or compounds o extremely low lipid solubility, by signs
re erable to the CNS (see Side Bar SYMP OMS OF ACU E ORGANOPHOSPHA E
POISONING).
QUESTION 6-5 Answer is c. T e AChE reactivator pralidoxime can be o great bene t
in the therapy o anti-ChE intoxication, but its use is supplemental to the administra-
tion o atropine. Although the phosphorylated esteratic site o AChE undergoes hydro-
lytic regeneration at a slow or negligible rate, nucleophilic agents such as the oxime,
pralidoxime, reactivate the enzyme more rapidly than does spontaneous hydrolysis (see
Figure 6-2). T e reactivating action o oximes is most marked at the skeletal neuromus-
cular junction.
QUESTION 6-6 Answer is e. A de ciency o intact cholinergic neurons, particularly
those extending rom subcortical areas such as the nucleus basalis o Meynert, has been
observed in patients with progressive dementia o the Alzheimer type (see Chapter 13).
Donepezil along with tacrine and rivastigmine are AChE inhibitors, which have the
requisite af nity and hydrophobicity to cross the blood-brain barrier and exhibit a
prolonged duration o action. T ey tend to slow the decline in cognitive unction and
behavioral mani estations or limited intervals o time.
QUESTION 6-7 Answer is a. Vecuronium is a competitive-type neuromuscular
blocking drug. Gentamicin is an aminoglycoside antibiotic (see Chapter 40) that also
produces neuromuscular blockade by inhibiting ACh release rom the preganglionic
terminal (through competition with Ca++) and to a lesser extent by noncompetitively
blocking the ACh receptor (see Figure 6-1).
QUESTION 6-8 Answer is c. Succinylcholine is a depolarizing neuromuscular
blocking drug that acts to depolarize the membrane in the same manner as ACh by
opening ligand-gated ion channels (ie, Nm acetylcholine receptors; see Figures 6-1
and 6-3). However, succinylcholine persists or a longer duration at the neuromuscular
junction because o its resistance to AChE. T e initial depolarization is ollowed by
block o neuromuscular transmission and accid paralysis.
QUESTION 6-9 Answer is a. T e major action o nicotine consists initially o transient
stimulation and subsequently o a more persistent depression o all autonomic ganglia.
T e e ects o high doses o nicotine on the neuromuscular junction are similar to those
on ganglia.

111
SECTION II Neuropharmacology

SUMMARYTABLE CHOLINERGIC PHARMACOLOGY


TOXICITIES
CLASS AND
SUBCLASSES NAMES CLINICAL USES COMMON UNIQUE; CLINICALLY IMPORTANT
Muscarinic Receptor Acetylcholine Neurotransmitter at
Agonists nicotinic and muscarinic
receptor sites; used topically
to induce miosis during
ophthalmologic surgery

Pilocarpine Used topically or treatment Parasympathomimetic O limited concern i used


o glaucoma and to induce e ects i absorbed topically; but caution should be
miosis; treatment o systemically taken in patients with asthma,
xerostomia ollowing head COPD, urinary or GI obstruction,
and neck radiation or Sjgren or cardiovascular disease
syndrome accompanied by hypotension or
bradycardia

Methacholine Administered cautiously Parasympathomimetic Emergency resuscitation


by inhalation or the e ects i absorbed equipment and medications to
diagnosis o bronchial airway systemically treat severe bronchospasm should
hyper-reactivity be available during testing

Carbachol Used topically or treatment Parasympathomimetic O limited concern i used


o glaucoma and to induce e ects i absorbed topically; but caution should be
miosis systemically taken in patients with asthma,
COPD, urinary or GI obstruction,
or cardiovascular disease
accompanied by hypotension or
bradycardia

Bethanechol Treatment o urinary Parasympathomimetic Caution should be taken in


retention and inadequate e ects i absorbed patients with asthma, COPD,
emptying o the bladder systemically urinary or GI obstruction,
when obstruction is absent or cardiovascular disease
Treatment o decreased accompanied by hypotension or
GI motility and decreased bradycardia
lower esophageal sphincter
pressure

Cevimeline Long-lasting sialogogic May have ewer side Caution should be taken in
e ect and enhances lacrimal e ects than pilocarpine patients with asthma, COPD,
secretions in Sjgren urinary or GI obstruction,
syndrome or cardiovascular disease
accompanied by hypotension or
bradycardia

Muscarinic Receptor Atropine Treatment o patients Predictable e ects o Urinary or GI obstruction, angle-
Antagonists with acute MI in whom muscarinic blockade closure glaucoma
excessive vagal tone causes (see Tables 5-1 and 6-2): Use with extreme caution in
bradycardia; treatment o xerostomia, constipation, patients with benign prostatic
Amanita muscaria poisoning; blurred vision, dyspepsia, hyperplasia
treatment o AChE poisoning and cognitive impairment

Scopolamine Treatment o motion sickness Xerostomia, drowsiness, Mydriasis and cycloplegia can
and blurred vision in some occur by inadvertent trans er o
individuals drug to the eye; rare but severe
psychotic episodes are possible

Ipratropium Treatment o chronic Predictable consequences Urinary or GI obstruction, angle-


obstructive pulmonary o muscarinic receptor closure glaucoma
disease (COPD; see blockade (see Tables 5-1 Use with extreme caution in
Chapter 24) and 6-2) patients with benign prostatic
hyperplasia

112
Cholinergic Pharmacology CHAPTER 6

TOXICITIES
CLASS AND
SUBCLASSES NAMES CLINICAL USES COMMON UNIQUE; CLINICALLY IMPORTANT

Tiotropium Treatment o chronic Predictable consequences Urinary or GI obstruction, angle-


obstructive pulmonary o muscarinic receptor closure glaucoma
disease (see Chapter 24) blockade (see Tables 5-1 Use with extreme caution in
and 6-2) patients with benign prostatic
hyperplasia

Oxybutynin Treatment o overactive Predictable consequences Reduce dose in patients taking


bladder by increasing o muscarinic receptor drugs that inhibit CYP3A4
capacity and reducing blockade (see Tables 5-1 Urinary or GI obstruction, angle-
requency and 6-2) closure glaucoma
Use with extreme caution in
patients with benign prostatic
hyperplasia

Tolterodine Treatment o overactive Predictable consequences Reduce dose in patients taking


bladder by increasing o muscarinic receptor drugs that inhibit CYP3A4
capacity and reducing blockade (see Tables 5-1 Urinary or GI obstruction, angle-
requency and 6-2) closure glaucoma
Use with extreme caution in
patients with benign prostatic
hyperplasia

Trospium chloride Treatment o overactive Predictable consequences Dosage adjustment is necessary


bladder by increasing o muscarinic receptor or patients with impaired renal
capacity and reducing blockade (see Tables 5-1 unction
requency and 6-2) Urinary or GI obstruction, angle-
closure glaucoma
Use with extreme caution in
patients with benign prostatic
hyperplasia

Dari enacin Treatment o overactive Predictable consequences Reduce dose in patients taking
bladder by increasing o muscarinic receptor drugs that inhibit CYP3A4 or
capacity and reducing blockade (see Tables 5-1 CYP2D6
requency and 6-2) Urinary or GI obstruction, angle-
closure glaucoma
Use with extreme caution in
patients with benign prostatic
hyperplasia

Soli enacin Treatment o overactive Predictable consequences Reduce dose in patients taking
bladder by increasing o muscarinic receptor drugs that inhibit CYP3A4
capacity and reducing blockade (see Tables 5-1
requency and 6-2)

Fesoterodine Treatment o overactive Predictable consequences Urinary or GI obstruction, angle-


bladder by increasing o muscarinic receptor closure glaucoma
capacity and reducing blockade (see Tables 5-1 Use with extreme caution in
requency and 6-2) patients with benign prostatic
hyperplasia

Flavoxate Treatment o overactive Predictable consequences Urinary or GI obstruction, angle-


bladder by increasing o muscarinic receptor closure glaucoma
capacity and reducing blockade (see Tables 5-1 Use with extreme caution in
requency and 6-2) patients with benign prostatic
hyperplasia

Glycopyrrolate Used to reduce GI tone and


motility (see Chapter 33)

Dicyclomine Used or the treatment Predictable consequences Urinary or GI obstruction, angle-


hydrochloride o diarrhea-predominant o muscarinic receptor closure glaucoma
irritable bowel syndrome (see blockade (see Tables 5-1 Use with extreme caution in
Chapter 33) and 6-2) patients with benign prostatic
hyperplasia
113
SECTION II Neuropharmacology

TOXICITIES
CLASS AND
SUBCLASSES NAMES CLINICAL USES COMMON UNIQUE; CLINICALLY IMPORTANT

Homatropine Topical administration to Predictable consequences Urinary or GI obstruction, angle-


hydrobromide produce mydriasis and o muscarinic receptor closure glaucoma
cycloplegia blockade (see Tables 5-1 Use with extreme caution in
and 6-2) patients with benign prostatic
hyperplasia

Cyclopentolate Topical administration to Predictable consequences Urinary or GI obstruction, angle-


hydrochloride produce mydriasis and o muscarinic receptor closure glaucoma
cycloplegia blockade (see Tables 5-1 Use with extreme caution in
and 6-2) patients with benign prostatic
hyperplasia

Tropicamide Topical administration to Predictable consequences Urinary or GI obstruction, angle-


produce mydriasis and o muscarinic receptor closure glaucoma
cycloplegia blockade (see Tables 5-1 Use with extreme caution in
and 6-2) patients with benign prostatic
hyperplasia

Benztropine Used to treat Parkinsons Predictable consequences Urinary or GI obstruction, angle-


mesylate disease and drug-induced o muscarinic receptor closure glaucoma
extrapyramidal symptoms blockade (see Tables 5-1 Use with extreme caution in
(see Chapters 8 and 13) and 6-2) patients with benign prostatic
hyperplasia

Trihexyphenidyl Used to treat Parkinsons Predictable consequences Urinary or GI obstruction, angle-


hydrochloride disease and drug-induced o muscarinic receptor closure glaucoma
extrapyramidal symptoms blockade (see Tables 5-1 Use with extreme caution in patients
(see Chapters 8 and 13) and 6-2) with benign prostatic hyperplasia

Methscopolamine Used in certain combination Predictable consequences Urinary or GI obstruction, angle-


bromide products or the temporary o muscarinic receptor closure glaucoma
relie o symptoms o allergic blockade (see Tables 5-1 Use with extreme caution in
rhinitis, sinusitis, and the and 6-2) patients with benign prostatic
common cold hyperplasia

ReversibleAChE Edrophonium Used or the diagnosis o Predictable consequences See Side Bar POTENTIAL EFFECTS
Inhibitors myasthenia gravis o muscarinic receptor OF ANTI-CHE AGENTS
stimulation (see Tables 5-1
and 6-2)

Physostigmine Used to treat intoxication Predictable consequences See Side Bar POTENTIAL EFFECTS
salicylate by anticholinergic drugs or o muscarinic receptor OF ANTI-CHE AGENTS; cardiac
substances stimulation (see Tables 5-1 arrhythmias i administered rapidly
and 6-2)

Neostigmine Used or the treatment o Predictable consequences See Side Bar POTENTIAL EFFECTS
bromide (oral) myasthenia gravis o muscarinic receptor OF ANTI-CHE AGENTS
stimulation (see Tables 5-1
and 6-2)

Neostigmine Used or the treatment o Predictable consequences See Side Bar POTENTIAL EFFECTS
methylsul ate myasthenia gravis o muscarinic receptor OF ANTI-CHE AGENTS
(parenteral) stimulation (see Tables 5-1
and 6-2)

Ambenonium Used or the treatment o Predictable consequences See Side Bar POTENTIAL EFFECTS
chloride myasthenia gravis o muscarinic receptor OF ANTI-CHE AGENTS
stimulation (see Tables 5-1
and 6-2)

Pyridostigmine Used or the treatment o Predictable consequences See Side Bar POTENTIAL EFFECTS
bromide myasthenia gravis o muscarinic receptor OF ANTI-CHE AGENTS
stimulation (see Tables 5-1
and 6-2)

114
Cholinergic Pharmacology CHAPTER 6

TOXICITIES
CLASS AND
SUBCLASSES NAMES CLINICAL USES COMMON UNIQUE; CLINICALLY IMPORTANT

Tacrine Approved or the treatment Predictable consequences See Side Bar POTENTIAL EFFECTS
o Alzheimers disease o muscarinic receptor OF ANTI-CHE AGENTS
stimulation (see Tables 5-1
and 6-2)

Donepezil Approved or the treatment Predictable consequences See Side Bar POTENTIAL EFFECTS
o Alzheimers disease o muscarinic receptor OF ANTI-CHE AGENTS
stimulation (see Tables 5-1
and 6-2)

Rivastigmine Approved or the treatment Predictable consequences See Side Bar POTENTIAL EFFECTS
o Alzheimers disease o muscarinic receptor OF ANTI-CHE AGENTS
stimulation (see Tables 5-1
and 6-2)

Galantamine Approved or the treatment Predictable consequences See Side Bar POTENTIAL EFFECTS
o Alzheimers disease o muscarinic receptor OF ANTI-CHE AGENTS
stimulation (see Tables 5-1
and 6-2)

Carbamate Insecticides Carbaryl Insecticide Excessive muscarinic Acute toxicity results rom excessive
receptor stimulation stimulation o muscarinic and
(see Tables 5-1 and 6-2); nicotinic receptors
low toxicity rom dermal
absorption

Propoxur Insecticide Excessive muscarinic Acute toxicity results rom


receptor stimulation (see excessive stimulation o muscarinic
Tables 5-1 and 6-2) and nicotinic receptors

Aldicarb Insecticide Excessive muscarinic Acute toxicity results rom


receptor stimulation (see excessive stimulation o muscarinic
Tables 5-1 and 6-2) and nicotinic receptors

Organophosphate Diazinon Insecticide Excessive muscarinic Acute toxicity results rom


Insecticides receptor stimulation (see excessive stimulation o muscarinic
Tables 5-1 and 6-2) and nicotinic receptors
Stability o phosphorylated
cholinesterase is enhanced
through aging

Chlorpyri os Insecticide Excessive muscarinic Acute toxicity results rom


receptor stimulation (see excessive stimulation o muscarinic
Tables 5-1 and 6-2) and nicotinic receptors
Stability o phosphorylated
cholinesterase is enhanced
through aging

Malathion Insecticide Excessive muscarinic Acute toxicity results rom


receptor stimulation (see excessive stimulation o muscarinic
Tables 5-1 and 6-2) and nicotinic receptors
Acute toxicity arises only rom
deliberate poisoning because
mammals detoxi y malathion by
a plasma carboxyl-esterase which
dictates species resistance
Stability o phosphorylated
cholinesterase is enhanced
through aging

115
SECTION II Neuropharmacology

TOXICITIES
CLASS AND
SUBCLASSES NAMES CLINICAL USES COMMON UNIQUE; CLINICALLY IMPORTANT
Organophosphate Tabun Nerve gas agent Potent toxin; toxicity is due to
Nerve Gases stimulation o muscarinic and
nicotinic receptors
Stability o phosphorylated
cholinesterase is enhanced
through aging

Sarin Nerve gas agent Potent toxin; toxicity is due to


stimulation o muscarinic and
nicotinic receptors
Stability o phosphorylated
cholinesterase is enhanced
through aging

Soman Nerve gas agent Potent toxin; toxicity is due to


stimulation o muscarinic and
nicotinic receptors
Stability o phosphorylated
cholinesterase is enhanced
through aging

Cholinesterase Pralidoxime (2-PAM) Used to reactivate Reverses peripheral neuromuscular


Reactivators cholinesterase a ter blockade o organo-phosphate
an organo-phosphate poisoning; does not enter the CNS
intoxication

Depolarizing Succinylcholine Depolarizing neuromuscular Should be administered Li e-threatening hyperkalemia; rarely


Neuromuscular blocking agent used as an only by an anesthesiologist may cause ganglionic blockade
Blocking Agents adjuvant during surgical in a setting where Histamine release which may result
anesthesia to obtain acilities or respiratory in bronchospasm, hypotension,
relaxation o skeletal muscle and cardiovascular and excessive bronchial and
resuscitation are available salivary secretion
(see Chapter 11)

Competitive Pancuronium Competitive neuromuscular Should be administered Persistent blockade with di culty
Neuromuscular blocking agent used as an only by an anesthesiologist in complete reversal o blockade
Blocking Agents adjuvant during surgical in a setting where
anesthesia to obtain acilities or respiratory
relaxation o skeletal muscle; and cardiovascular
long duration o action resuscitation are available
(see Chapter 11)

Atracuium Competitive neuromuscular Should be administered Histamine release which may result
blocking agent used as an only by an anesthesiologist in bronchospasm, hypotension,
adjuvant during surgical in a setting where and excessive bronchial and
anesthesia to obtain acilities or respiratory salivary secretion
relaxation o skeletal muscle; and cardiovascular
intermediate duration o resuscitation are available
action (see Chapter 11)

Vecuronium Competitive neuromuscular Should be administered


blocking agent used as an only by an anesthesiologist
adjuvant during surgical in a setting where
anesthesia to obtain acilities or respiratory
relaxation o skeletal muscle and cardiovascular
Intermediate duration o resuscitation are available
action (see Chapter 11)

Pipecuronium Competitive neuromuscular Should be administered Persistent blockade with di culty


blocking agent used as an only by an anesthesiologist in complete reversal o blockade
adjuvant during surgical in a setting where acilities
anesthesia to obtain or respiratory and
relaxation o skeletal muscle; cardiovascular resuscitation
long duration o action are available (see Chapter 11)

116
Cholinergic Pharmacology CHAPTER 6

TOXICITIES
CLASS AND
SUBCLASSES NAMES CLINICAL USES COMMON UNIQUE; CLINICALLY IMPORTANT

Doxacurium Competitive neuromuscular Should be administered Persistent blockade with di culty


blocking agent used as an only by an anesthesiologist in complete reversal o blockade
adjuvant during surgical in a setting where acilities
anesthesia to obtain or respiratory and
relaxation o skeletal muscle; cardiovascular resuscitation
long duration o action are available (see Chapter 11)

Mivacurium Competitive neuromuscular Histamine release which may result


blocking agent used as an in bronchospasm, hypotension,
adjuvant during surgical and excessive bronchial and
anesthesia to obtain salivary secretion
relaxation o skeletal muscle;
short duration o action (Not
available in the United States)

Rocuronium Competitive neuromuscular Should be administered


blocking agent used as an only by an anesthesiologist
adjuvant during surgical in a setting where
anesthesia to obtain acilities or respiratory
relaxation o skeletal muscle; and cardiovascular
intermediate duration o resuscitation are available
action but with rapid onset (see Chapter 11)

Cisatracurium Competitive neuromuscular Should be administered


blocking agent used as an only by an anesthesiologist
adjuvant during surgical in a setting where acilities
anesthesia to obtain relaxation or respiratory and
o skeletal muscle; intermediate cardiovascular resuscitation
duration o action are available (see Chapter 11)

Botulinum Toxins Onabotulinum Treatment o strabismus and Diplopia, eyelid drop, and distant
toxin A blepharospasm and as a spread o toxin with paralysis o
cosmetic procedure to reduce respiratory muscles
acial wrinkles (see Chapter 48)

Rimabotulinum Treatment o strabismus and Diplopia, eyelid drop, and distant


toxin B blepharospasm and as a spread o toxin with paralysis o
cosmetic procedure to reduce respiratory muscles
acial wrinkles (see Chapter 48)

Antispasticity and Dantrolene Treatment o malignant Hepatotoxicity has been reported


Antirigidity Agents hyperthermia; treatment and continued use requires
o spasticity and rigidity in monitoring o liver unction
nonambulatory patients

Ganglionic Stimulating Nicotine Presence in tobacco products Nicotine poisoning rom the
Agents con erring dependence on accidental ingestion o nicotine-
its users (see Chapter 14) containing insecticides, or, in
children, rom ingestion o tobacco
products; death rom respiratory
ailure may occur rapidly

Nicotinic ACh Varenicline Used as an aid to smoking FDA has issued a warning about
Receptor Agonists cessation mood and behavioral changes

Ganglionic Blocking Mecamylamine Initially used to treat Visual disturbances, dry Marked hypotension, constipation,
Drugs hypertension, but now mouth, urinary hesitancy, syncope, urinary retention,
surpassed by sa er agents decreased sexual potency, paralytic ileus, and cycloplegia
(see Table 6-8) diarrhea, abdominal pain

Trimethaphan Initially used to treat Visual disturbances, dry Marked hypotension, constipation,
hypertension, but now mouth, urinary hesitancy, syncope, urinary retention,
surpassed by sa er agents decreased sexual potency, paralytic ileus, and cycloplegia
(see Table 6-8) diarrhea, abdominal pain

117
CHAPTER

7 Adrenergic, Dopaminergic, and


SerotonergicPharmacology
T is chapter is a combination o Chapter 12 Adrenergic Agonists and Antagonists
DRUGS INCLUDED IN
and Chapter 13 5-Hydroxytryptamine (Serotonin) rom Goodman and Gilmans T e
THIS CHAPTER Pharmacological Basis of T erapeutics, 12th Edition. An understanding o the material
Acebutolol (SECTRAL, others) in those chapters will be help ul in ollowing the material presented in this chapter.
Albuterol (VENTOLIN-HFA, PROVENTIL-HFA, Further details o the therapeutic use o the drugs mentioned in this chapter can be
others) ound in subsequent chapters. In addition to the material presented here the above
chapters in the 12th Edition include:
Al uzosin (UROXATRAL)
able 12-1 Chemical Structures and Main Clinical Uses o Important Sympathomi-
Almotriptan (AXERT)
metic Drugs
Alosetron (LOTRONEX)
A detailed discussion o the pharmacology o the endogenous catecholamines
Apomorphine (APOKYN) epinephrine (E), norepinephrine (NE), and dopamine (DA)
Apraclonidine (IOPIDINE) T e pharmacology o miscellaneous sympathomimetic amines, including amphet-
Ar ormeterol (BROVENA) amine, methamphetamine, methylphenidate, and ephedrine
Aripiprazole (ABILIFY) A discussion o the therapeutic uses o sympathomimetic drugs in shock, hypoten-
Atenolol (TENORMIN, others) sion, cardiac arrhythmias, nasal decongestion, allergic reactions, narcolepsy, weight
reduction, and attention de cit/hyperactivity disorder (ADHD)
Betaxolog (BETAOPTIC, others)
Details o the physiological unctions o serotonin, including the multiple receptor
Betaxolol (LOKREN, KERCONE, others)
subtypes o 5-H
Bitolterol (TORNALATE)discontinued in
A discussion o the pharmacology o ergot and the ergot alkaloids, including lysergic
the United States
acid diethylamide (LSD)
Brimonidine (ALPHAGAN, others)
T e physiological unctions o dopamine, including a detailed discussion o dopamine
Bromocriptine (PARLODEL) receptor subtypes
Bucindolol (SANDORMIN)
Bunazosinnot available in the LEARNING OBJECTIVES
United States Understand the pharmacology o the major adrenergic and serotonergic ago-
Buspirone (BUSPAR, others) nists and antagonists.
Cabergoline (DOSTINEX) Know the mechanisms o action o the adrenergic and serotonergic agonists
Carteolol (OCCUPRES, others) and antagonists.
Carvedilol (COREG) Know the therapeutic uses, adverse e ects, toxicities, and contraindications
o the adrenergic and serotonergic agonists and antagonists.
Celiprolol (SELECTOL)
Citralopam(CELEXA)
Clonidine (CATAPRES, others; Transdermal, MECHANISM OF ACTION OF ENDOGENOUS CATECHOLAMINES
CATAPRES-TTS)
DRUG CLASS DRUG MECHANISM OF ACTION
Clozapine (FAZACLO)
Endogenous Epinephrine (E) Stimulates both and adrenergic receptors
Desvenla axine (PRISTIQ) Catecholamines Responses are shown in Table 5-1
Dexmedetomide (PRECEDEX) (see Figure 7-1)
Norepinephrine Neurotransmitter at postganglionic sympathetic nerves
Dexmethylphenidate (FOCALIN, FOCALINXR) (NE) Potent and 1 adrenergic receptor agonist (see Table 5-1)
Dextroamphetamine (DEXIDRINE,
Dopamine (DA) Immediate metabolic precursor o NE and E; central
ADDERALXR, others)
neurotransmitter important or the regulation o
Dobutamine (DOBUTREX) movement and behavior (see Figure 7-2)
Dolasetron (ANZEMET)
Dopamine Fenoldopam Agonist or peripheral D1 receptors and binds with
Dopamine Derivatives moderate a nity to 2 receptors
Dopexamine (DOPACARD)not currently Dopexamine Intrinsic activity at D1 and D2 receptors as well as 2
available in the United States receptors
(continues)

118
Adrenergic, Dopaminergic, and Serotonergic Pharmacology CHAPTER 7

MECHANISM OF ACTION OF -ADRENERGIC RECEPTOR AGONISTS AND DRUGS INCLUDED IN THIS


ANTAGONISTS CHAPTER (Cont.)
DRUG CLASS DRUG MECHANISM OF ACTION Doxazosin (CARDURA, others)
1-Selective Adrenergic Phenylephrine Agonist at 1 adrenergic receptors (see Duloxetine (CYMBALTA)
Receptor Agonists Table 5-1)
(see Figure 7-3) Eletriptan (RELPAX)
Mephentermine Agonist at 1 adrenergic receptors (see Table 5-1)
Ephedrine
Meteraminol Agonist at 1 adrenergic receptors (see Table 5-1)
Midodrine Agonist at 1 adrenergic receptors (see Epinephrine (solution or injection, EPIPEN)
Table 5-1) Escitralopam(LEXAPRO)
2-Selective Adrenergic Clonidine Agonist at 2 adrenergic receptors (see Table 5-1) Esmolol (BREVIBLOC, others)
Receptor Agonists (see Suppresses the out ow o the sympathetic
Figure 7-3) nervous system activity rom the brain Fenoldopam(CORLOPAM)
See Chapter 15 Fenoterol (BEROTEC)withdrawn
Dexmedetomidine Agonist at 2 adrenergic receptors (see Table 5-1) rommarket
Suppresses the out ow o the sympathetic Fluoxetine (PROZAC, others)
nervous system activity rom the brain
See Chapter 15 Fluvoxamine (LUVOX,others)
Apraclonidine Agonist at 2 adrenergic receptors (see Table 5-1) Formoterol (FORADIL, others)
2 Receptormediated reduction in the Frovatriptan (FROVAL)
ormation o aqueous humor (see Chapter 47)
Does not cross the blood-brain barrier (BBB) Granisetron (KYTRIL, others)
Brimonidine Agonist at 2 adrenergic receptors (see Table 5-1) Guanabenx(WYTENSIN)
2 Receptormediated reduction in the Guan acine (TENEX,others; INTUNIV,
ormation o aqueous humor (see Chapter 47) sustained release)
Can cross the BBB
Indoraminnot available in the
Guan acine Agonist at 2 adrenergic receptors (see Table 5-1)
Suppresses the out ow o the sympathetic United States
nervous system activity rom the brain Isocarboxazid (MARPLAN)
See Chapter 15
Isoetharine (BRONKOSOL)no longer
Guanabenz Agonist at 2 adrenergic receptors (see marketed in the United States
Table 5-1)
Suppresses the out ow o the sympathetic Isoproterenol (ISUPREL)
nervous system activity rom the brain Ketanserin (SUFREXAL)not available in
See Chapter 15 the United States
Methyldopa Metabolized to -methylnorepinephrine in
Levalbuterol (XOPENEX)
the brain where it activates 2 receptors and
suppresses the out ow o the sympathetic Levobetaxolol (BETAXON, others)
nervous system activity rom the brain Levobunolol (BETAGAN, others)
See Chapter 15
Lisdexam etamine (VYVANASE)
Tizanidine Agonist at 2 adrenergic receptors (see Table 5-1)
See Chapter 15 Mephenterminediscontinued in the
1 Adrenergic Prazosin Antagonist at 1 adrenergic receptors (see United States
Receptor Antagonists Table 5-1) to inhibit vasoconstriction induced Metaproterenol (ALUPENT,orciprenaline
(see Figure 7-4) by endogenous catecholamines in Europe)
See Chapter 15
Metaraminol
Terazosin Antagonist at 1 adrenergic receptors (see
Table 5-1) to inhibit vasoconstriction induced Methamphetamine (DESOXYN)
by endogenous catecholamines Methyldopa (ALDOMET,others)
See Chapter 15
Methylphenidate (RITALIN, RITALINSR,
Doxazosin Antagonist at 1 adrenergic receptors (see
CONCERTA, META-DATE, others; DAYTRANA,
Table 5-1) to inhibit vasoconstriction induced
by endogenous catecholamines transdermal)
See Chapter 15 Methylsergide (SANSERT)
Al uzosin Antagonist at 1 adrenergic receptors (see Metipranolol (OPTIPRANOLOL, others)
Table 5-1) with a nity or all 1 receptor subtypes
Metoprolol (LOPRESSOR, others)
Tamsulosin Antagonist at 1 adrenergic receptors (see
Table 5-1); some selectivity or 1A in prostate Midodrine (PROAMATINE, others)
Silodosin Antagonist at 1 adrenergic receptors (see (continues)
Table 5-1) some selectivity or 1A in prostate

119
SECTION II Neuropharmacology

DRUGS INCLUDED IN THIS DRUG CLASS DRUG MECHANISM OF ACTION


CHAPTER (Cont.) 2 Adrenergic Yohimbine Competitive antagonist that is selective or
Receptor Antagonists 2-adrenergic receptors
Milnacipran (SAVELLA)
(see Figure 7-4)
Moda nil (PROVIGIL) Phenoxybenzamine Nonselective antagonist at 2 adrenergic
receptors
Naphazoline (PRIVINE, NAPHCON, others)
Naratriptan (AMERGE) Phentolamine Nonselective antagonist at 2 adrenergic
receptors
Nebivolol (NEBILET,BYSTOLIC)
Norepinephrine (LEVOPHED) Additional Adrenergic Ergot Alkaloids Partial agonists and antagonists at receptors,
Receptor Antagonists DA receptors, and serotonin receptors
Olanzapine (ZYPREXA) (see Figure 7-4)
Ondansetron (ZOFRAN, others) Indoramin Selective 1 receptor antagonist

Oxymetazoline (AFRIN, OCU-CLEAR, others) Ketanserin Antagonist at 1 receptors and 5-HT receptors
Palonosetron (ALOXI) Urapidil Antagonist at 1 adrenergic receptors
Paroxetine (PAXIL)
Bunazosin An 1 selective antagonist
Pemoline (CYLERT,others)discontinued
in the United States
Pergolide (PERMAX) MECHANISM OF ACTION OF MISCELLANEOUS SYMPATHOMIMETIC AGONISTS
Phenelzine (NARDIL) DRUG CLASS DRUG MECHANISM OF ACTION
Phenoxybenzamine (DIBENZYLINE) Miscellaneous Amphetamine Amphetamine exerts most o its e ects in the CNS
Phentolamine (ORAVERSE) Sympathomimetic through the release o biogenic amines rom their
Agonists storage sites by targeting the neuronal dopamine
Phenylephrine (NEO-SYNEPHRINE, others) (see Figure 7-3) transporter (DAT) and the vesicular monoamine
Pindolol (VISKIN, others) transporter 2 (VMAT2) (see Table 7-1 and Figure 7-2)
Pirbuterol (MAXAIR) Methamphetamine In the brain, methamphetamine releases DA and
Pramipexole (MIRAPEX) other biogenic amines, and inhibits neuronal and
vesicular monoamine transporters as well as MAO
Prazosin (MINIPRES, others)
Methylphenidate In the brain, methylphenidate is predominantly a
Procaterol (MASCACIN)not available in dopamine and norepinephrine uptake inhibitor
the United States
Propranolol (INDERAL, INDERALLA, others) Dexmethylphenidate Mechanism o action is similar to methylphenidate

Propylhexedrine (BENZEDREX,others) Lisdexamphetamine Mechanism o action is similar to methylphenidate


Pseudoephedrine (SUDAPHED, others) Pemoline Dopaminergic but precise mechanism o action
Quetiapine (SEROQUEL) has not been determined
Respiridone (RISPERDAL, others) Ephedrine Agonist at both and adrenergic receptors; also
Ritodrinenot available inthe UnitedStates enhances the release o NE rom sympathetic neurons

Rizatriptan (MAXALT,others) Propylhexedrine Sympathomimetic drug that acts as a


vasoconstrictor or local application to the nasal
Ropinirole (REQUIP) mucosa or the eye
Rotigotine (NEUPRO)
Naphazoline Sympathomimetic drug that acts as a
Salmeterol (SEREVENT) vasoconstrictor or local application to the nasal
Sertraline (ZOLOFT) mucosa or the eye
Sibutramine (MERIDIA) Oxymetazoline Sympathomimetic drug that acts as a
Silodosin (RAPAFLO) vasoconstrictor or local application to the nasal
mucosa or the eye
Sumatriptan (IMITREX,others)
Xylometazoline Sympathomimetic drug that acts as a
Tamsulosin (FLOMAX)
vasoconstrictor or local application to the nasal
Terazosin (HYTRIN, others) mucosa or the eye
Terbutaline (BRETHINE) Pseudoephedrine Stereoisomer o ephedrine with vasoconstrictor
Timolol (TIMOPTIC, others, or ophthalmic activity
use; BLOCADREN, others, or systemicuse) Modaf nil CNS stimulant with actions on NE, DA, and 5-HT
Tizanidine (ZANAFLEX,others) neurons, but precise mechanism o action has not
(continues) been determined

120
Adrenergic, Dopaminergic, and Serotonergic Pharmacology CHAPTER 7

CASE 7-1 DRUGS INCLUDED IN THIS


A 56-year-old woman is being treated with prazosin or her elevated blood pressure. CHAPTER (Cont.)
Tranylcypromine (PARNATE, others)
a. What class o drug is prazosin and how is it used to treat hypertension?
Urapidilnot available in the United States
T e major e ects o prazosin result rom its blockade o 1 receptors in arterioles
and veins (see Figure 7-4). T is leads to a all in peripheral vascular resistance and Venla axine (EFFEXOR)
in venous return to the heart. Unlike other vasodilating drugs, administration o Xylometazoline (OTRIVIN, others)
prazosin usually does not increase heart rate (see answer to b below). In addition, Yohimbine (YOCON, APHRODYNE)
prazosin decreases cardiac preload and thus has little tendency to increase cardiac Zolmitriptan (ZOMIG)
(Continued)
HO H HO H O HO H H
MAO MAO
HO C CH2 NH2 HO C C H HO C CH2 N

OH OH OH CH3

No re pine phrine DOPGAL Epine phrine

COMT COMT
AR AD

CH3 O H CH3 O H H

HO C CH2 NH2 HO C CH2 N


HO H OH HO H O
OH OH CH3
HO C C H HO C C OH Me tane phrine
No rme tane phrine
OH H OH
DOPEG DOMA

COMT

CH3 O H OH

HO C C H

OH H
MOPEG

ADH AR

CH3 O H O
MAO MAO
HO C C H

OH
MOPGAL

AD

CH3 O H O

HO C C OH

OH
VMA

FIGURE 7-1 Steps in the metabolic disposition o catecholamines. Norepinephrine and epinephrine are f rst oxidatively deaminated to a short-lived
intermediate (DOPGAL) by monoamine oxidase (MAO). DOPGALthen undergoes urther metabolism to more stable alcohol or acid deaminated metab-
olites. Aldehyde dehydrogenase (AD) metabolizes DOPGALto 3,4-dihydroxymandelic acid (DOMA) while aldehyde reductase (AR) metabolizes DOPGAL
to 3,4-dihydroxyphenyl glycol (DOPEG). Under normal circumstances DOMA is a minor metabolite with DOPEG being the major metabolite produced
rom norepinephrine and epinephrine. Once DOPEG leaves the major sites o its ormation (sympathetic nerves; adrenal medulla), it is converted to
3-methoxy, 4-hydroxyphenylglycol (MOPEG) by catechol-0-methyl trans erase (COMT). MOPEG is then converted to the unstable aldehyde (MOPGAL) by
alcohol dehydrogenase (ADH) and f nally to vanillyl mandelic acid (VMA) by aldehyde dehydrogenase. VMA is the major end product o norepinephrine
and epinephrine metabolism. Another route or the ormation o VMA is conversion o norepinephrine or epinephrine into normetanephrine or meta-
nephrine by COMT either in the adreneal medulla or extraneuronal sites, with subsequent metabolism to MOPGAL and thence to VMA.
121
SECTION II Neuropharmacology

TH
Dopa Tyros ine

Adre ne rg ic AAADC MAO


Dopa mine
varic o s ity

ATP NP Y DA
VAMP S NP Y NE DH
NE ATP NE NE
Re s e rpine
Coca ine
Ca 2+
Ca 2+ 2
P1

NP Y
NE

P
Ade nos ine

T
P

A
AT E

Y
P N
rNTP a s e
Y2 2

A
E

T
N

P
NP Y NE
ATP
AMP

ADP

NP Y

Effe c to r c e ll
me mbrane

P 2X1 P 2X7 P 2Y Y1 Y5 ,

FIGURE 7-2 An adrenergic neuroe ector junction showing eatures o the synthesis, stor-
age, release, and receptors or norepinephrine (NE), the cotransmitters neuropeptide Y(NPY),
and ATP. Tyrosine is transported into the varicosity and is converted to DOPA by tyrosine
hydroxylase (TH) and DOPA to dopamine (DA) by the action o aromatic l -amino acid decar-
boxylase (AAADC). Dopamine is taken up into the vesicles o the varicosity by a transporter,
VMAT2, that can be blocked by reserpine. Cytoplasmic NE also can be taken up by this
transporter. Dopamine is converted to NE within the vesicle via the action o dopamine-
-hydroxylase (DH). NE is stored in vesicles along with other cotransmitters, NPY and ATP,
depending on the particular neuroe ector junction. Release o the transmitters occurs upon
depolarization o the varicosity, which allows entry o Ca2+ through voltage-dependent Ca2+
channels. Elevated levels o Ca2+ promote the usion o the vesicular membrane with the
membrane o the varicosity, with subsequent exocytosis o transmitters. This usion process
involves the interaction o specialized proteins associated with the vesicular membrane
(VAMPs, vesicle-associated membrane proteins) and the membrane o the varicosity (SNAPs,
synaptosome-associated proteins). In this schematic representation, NE, NPY, and ATP are stored
in the same vesicles. Di erent populations o vesicles, however, may pre erentially store di erent
proportions o the cotransmitters. Once in the synapse, NE can interact with and adrenergic
receptors to produce the characteristic response o the e ector. The adrenergic receptors are
GPCRs. and Receptors also can be located presynaptically where NE can either diminish
(2), or acilitate () its own release and that o the cotransmitters. The principal mechanism
by which NE is cleared rom the synapse is via a cocaine-sensitive neuronal uptake trans-
porter, NET. Once transported into the cytosol, NE can be restored in the vesicle or metabo-
lized by monoamine oxidase (MAO). NPYproduces its e ects by activating NPY receptors, o
which there are at least f ve types (YI through Y2). NPY receptors are GPCRs. NPYcan modi y its
own release and that o the other transmitters via presynaptic receptors o the Y2 type. NPYis
removed rom the synapse by metabolic breakdown by peptidases. ATP produces its e ects
by activating P2X receptors or P2Yreceptors. P2X receptors are ligand-gated ion channels;
P2Yreceptors are GPCRs. There are multiple subtypes o both P2X and P2Y receptors. As with
the other cotransmitters, ATP can act prejunctionally to modi y its own release via receptors
or ATP or via its metabolic breakdown to adenosine that acts on P1 (adenosine) receptors.
ATP is cleared rom the synapse primarily by releasable nucleotidases (rNTPase) and by cell-
f xed ectonucleotidases.

122
Adrenergic, Dopaminergic, and Serotonergic Pharmacology CHAPTER 7

Adre ne rg ic Ag o nis ts MECHANISMS BY WHICH


INDIRECT-ACTING
SYMPATHOMIMETIC
Dire c t-ac ting Mixe d-ac ting Indire c t-ac ting DRUGS INCREASE NE
AND E TO STIMULATE
ADRENERGIC RECEPTORS
S e le c tive No n-s e le c tive Re le as ing Uptake MOA COMT Release or displace NE romsympathetic
Ag e nts Inhibito r* Inhibito rs * Inhibito rs *
nerve varicosities
Blockthe transport o NEinto sympathetic
1 -phe nyle phrine 1 2 -oxyme ta zoline e phe drine a mphe ta mine coca ine s e le giline e nta ca pone
2 -clonidine 1 2 -is oprote re nol ( 1 2 1 2 a nd tyra mine nerve terminals
1 -dobuta mine 1 2 1 2 -e pine phrine re le a s ing a ge nt)
2 -te rbuta line 1 2 1 -nore pine phrine Blockthe catecholamine metabolizing
enzymes MAOand COMT

Re s p ons e s a re not re d uc e d by p rior Re s p ons e is re d uc e d by Re s p ons e s a re a b olis he d by p rior


tre a tme nt with re s e rp ine or g ua ne thid ine . p rior tre a tme nt with tre a tme nt with re s e rp ine or
Re s p ons e may by p ote ntia te d by c oc a ine , re s e rp ine or g ua ne thid ine . g ua ne thid ine .
re s e rp ine a nd g ua ne thid ine .

FIGURE 7-3 Classif cation o adrenergic receptor agonists (sympathomimetic amines) or drugs
that produce sympathomimetic-like e ects. For each category, a prototypical drug is shown.
(*Not actually sympathetic drugs but produce sympathomimetic-like e ects.)

output and rate, in contrast to vasodilators such as hydralazine that have minimal
dilatory e ects on veins (see Chapter 15).
b. What is prazosins mechanism o action?
Blockade o 1 adrenergic receptors inhibits vasoconstriction induced by endog-
enous catecholamines; vasodilation may occur in both arteriolar resistance ves-
sels and in veins. T e result is a all in blood pressure due to decreased peripheral
resistance. T e magnitude o such e ects depends on the activity o the sympathetic
nervous system at the time the antagonist is administered, and thus is less in supine
than in upright subjects and is particularly marked i there is hypovolemia. For
most receptor antagonists, the all in blood pressure is opposed by baroreceptor
re exes that cause increases in heart rate as well as uid retention. Prazosin appears
to depress barore ex unction in hypertensive patients.
(Continued)

Adre ne rgic Re ce ptor Anta gonis ts

Alpha Re ce ptor Anta gonis ts Be ta Re ce ptor Anta gonis ts

No n-s e le c tive 1 -s e le c tive 2 -s e le c tive No n-s e le c tive 1 -s e le c tive No n-s e le c tive 1 -s e le c tive
(Firs t Ge ne ra tion) (S e c ond Ge ne ra tion) (Third Ge ne ra tion) (Third Ge ne ra tion)

phe noxybe nza mine pra zos in yohimbine na dolol a ce butolol ca rte olol be ta xolol
phe ntola mine te ra zos in pe nbutolol a te nolol ca rve dilol* ce liprolol
doxa zos in pindolol bis oprolol bucindolol ne bivolol
a lfuzos in propra nolol e s molol la be ta lol*
ta ms ulos in timolol me toprolol
indora min s ota lol
ura pidil levobunolol
buna zos in me tipra nolol

FIGURE 7-4 Classif cation o adrenergic receptor antagonists. Drugs marked by an asterisk (*)
also block 1 receptors.

123
SECTION II Neuropharmacology

TABLE 7-1 Characteristics of Plasma Membrane Transporters for Endogenous Catecholamines


TYPE OF TRANSPORTER SUBSTRATE SPECIFICITY TISSUE REGION/CELL TYPE INHIBITORS
Neuronal

NET DA > NE > Epi All sympathetically Sympathetic nerves Desipramine


innervated tissue Chroma n cells Cocaine
Adrenal medulla Capillary endothelial cells Nisoxetine
Liver Syncytiotrophoblast
Placenta

DAT DA > NE > Epi Kidney Endothelium Cocaine


Stomach Parietal and endothelial cells Imazindol
Pancreas Pancreatic duct

Non-neuronal

OCT1 DA > Epi >> NE Liver Hepatocytes Isocyanines


Intestine Epithelial cells Corticosterone
Kidney (not human) Distal tubule

OCT2 DA >> NE > Epi Kidney Medullary proximal and distal tubules Isocyanines
Brain Glial cells o DA-rich regions, some Corticosterone
nonadrenergic neurons

ENT (OCT 3) Epi >> NE > DA Liver Hepatocytes Isocyanines


Brain Glial cells, others Corticosterone
Heart Myocytes O-methyl-isoproterenol
Blood vessels Endothelial cells
Kidney Cortex, proximal and distal tubules
Placenta Syncytiotrophoblasts (basal membrane)
Retina Photoreceptors, ganglion amacrine cells

NET, norepinephrine transporter, originally known as uptake 1; DAT, dopamine transporter; ENT (OCT3), extraneuronal transporter, originally
known as uptake 2; OCT 1, OCT 2, organic cation transporters; Epi, epinephrine; NE, norepinephrine; DA, dopamine.

c. What are the major side e ects o prazosin that this patient should be aware o ?
A major potential adverse e ect o prazosin and its congeners is the rst-dose
e ect; marked postural hypotension and syncope sometimes are seen 30 to 90 min-
utes a er an initial dose o prazosin. Syncopal episodes also have occurred with a
rapid increase in dosage or with the addition o a second antihypertensive drug to
the regimen o a patient who already is taking a large dose o prazosin. It is recom-
mended that patients take their rst dose immediately prior to bedtime.

CASE 7-2
A 48-year-old man with the diagnoses o essential hypertension is being treated with
clonidine.
a. What type o drug is clonidine?
Clonidine, an 2-selective adrenergic agonist (see Figure 7-3), is used primarily or the
treatment o systemic hypertension (see Chapter 15). Clonidines ef cacy as an anti-
hypertensive agent is somewhat surprising, because many blood vessels contain post-
synaptic 2 adrenergic receptors that promote vasoconstriction. Indeed, clonidine, the
prototypic 2 agonist, was initially developed as a vasoconstricting nasal decongestant.
b. What is clonidines mechanism o action?
Clonidines capacity to lower blood pressure results rom activation o 2 receptors in
the cardiovascular control centers o the CNS; such activation suppresses the out ow o
sympathetic nervous system activity rom the brain. Although the exact mechanism by
which clonidine lowers blood pressure is not completely understood, the e ect appears
to result, at least in part, rom activation o 2 receptors in the lower brainstem region.
(Continued)
124
Adrenergic, Dopaminergic, and Serotonergic Pharmacology CHAPTER 7

c. What side e ects o clonidine should this patient be warned about?


T e major adverse e ects o clonidine are dry mouth and sedation. T ese responses
occur in at least 50% o patients and may require drug discontinuation. However,
they may diminish in intensity a er several weeks o therapy. Sexual dys unction
also may occur. Marked bradycardia is observed in some patients.
Withdrawal reactions ollow abrupt discontinuation o long-term therapy with
clonidine in some hypertensive patients.

MECHANISM OF ACTION OF ADRENERGIC RECEPTOR AGONISTS AND ANTAGONISTS


DRUG CLASS DRUG MECHANISM OF ACTION
Adrenergic Receptor Agonists (see Isoproterenol Potent adrenergic receptor agonist with very low a nity or
Figure 7-3) receptors; power ul e ect on all receptors and almost no
action on receptors (see Table 5-1)

Dobutamine Direct agonist interactions with - and -adrenergic receptors

2 Selective Adrenergic Receptor Metaproterenol Agonist at 2 adrenergic receptors (see Table 5-1)
Agonists (see Figure 7-3) See Chapter 24

Albuterol Agonist at 2 adrenergic receptors (see Table 5-1)


See Chapter 24

Levalbuterol Agonist at 2 adrenergic receptors (see Table 5-1)


See Chapter 24

Pirbuterol Agonist at 2 adrenergic receptors (see Table 5-1)


See Chapter 24

Terbutaline Agonist at 2 adrenergic receptors (see Table 5-1)


See Chapter 24

Isoetharine Agonist at 2 adrenergic receptors (see Table 5-1)

Bitolterol Agonist at 2 adrenergic receptors (see Table 5-1)

Fenoterol Agonist at 2 adrenergic receptors (see Table 5-1)

Procaterol Agonist at 2 adrenergic receptors (see Table 5-1)

Salmeterol Agonist at 2 adrenergic receptors (see Table 5-1)


See Chapter 24

Formoterol Agonist at 2 adrenergic receptors (see Table 5-1)


See Chapter 24

Ar ormoterol Agonist at 2 adrenergic receptors (see Table 5-1)


See Chapter 24

Ritodrine Agonist at 2 adrenergic receptors (see Table 5-1)

Nonselective Adrenergic Receptor Carteolol Antagonist at 1 and 2 adrenergic receptors (see Table 5-1)
Antagonists (see Figure 7-4) See Chapter 47

Propranolol Antagonist at 1 and 2 adrenergic receptors (see Table 5-1)


See Chapters 15, 16, 17, and 18

Nadolol Antagonist at 1 and 2 adrenergic receptors (see Table 5-1)


See Chapters 15, 16, 17, and 18

Levobunolol Antagonist at 1 and 2 adrenergic receptors (see Table 5-1)


See Chapter 47

Metipranolol Antagonist at 1 and 2 adrenergic receptors (see Table 5-1)


See Chapter 47
(Continued)
125
SECTION II Neuropharmacology

DRUG CLASS DRUG MECHANISM OF ACTION


Timolol Antagonist at 1 and 2 adrenergic receptors (see Table 5-1)
See Chapters 15, 16, 17, 18, and 47

Pindolol Antagonist at 1 and 2 adrenergic receptors (see Table 5-1)


See Chapters 15, 16, 17, and 18

1 Selective Adrenergic Receptor Metoprolol Antagonist at 1 adrenergic receptors


Antagonists (see Figure 7-4) See Chapters 15, 16, 17, and 18

Betaxolol Antagonist at 1 adrenergic receptors


See Chapter 47

Levobetaxolol Antagonist at 1 adrenergic receptors


See Chapter 47

Bisoprolol Antagonist at 1 adrenergic receptors


See Chapters 15, 16, 17, and 18

Acebutolol Antagonist at 1 adrenergic receptors


See Chapters 15, 16, 17, and 18

Atenolol Antagonist at 1 adrenergic receptors


See Chapters 15, 16, 17, and 18

Esmolol Antagonist at 1 adrenergic receptors that has a very short


duration o action

Adrenergic Receptor Antagonists Labetalol Competitive antagonist at 1 and adrenergic receptors


with Additional Cardiovascular E ects See Chapter 15
(Third Generation Blockers)
(see Figure 7-4) Carvedilol Competitive antagonist at 1 and adrenergic receptors,
but also has antioxidant and anti-in ammatory e ects
(see Figure 7-5)
See Chapters 15 and 17

Bucindolol Antagonist at adrenergic receptors with weak 1 adrenergic


receptor blocking properties

Celiprolol Third-generation 1 adrenergic receptor antagonist

Nebivolol Third-generation 1 adrenergic receptor antagonist with


endothelial NO-mediated vasodilator activity (see Figure 7-5)

P la te le t/le ucocyte LDL oxida tion


a ggre ga tion a nd a dhe s ion Lipid pe roxida tion
LDL oxida tion Endothe lia l dys function
S mooth mus cle ce ll prolife ra tion Apoptos is

Antioxida nt
Ca 2 + Agonis t NO
Blocka de ROS

2
L-type
VGCC Ce ll me mbra ne
AC s GC
Va s cula r
s mooth mus cle
Ca 2 + cAMP ATP cGMP GTP

Co ntrac tio n Vas o dilatio n


FIGURE 7-5 Mechanisms underlying actions o vasodilating blockers in blood vessels. ROS,
reactive oxygen species; sGC, soluble guanylyl cyclase; AC adenylyl cyclase; L-type VGCC, L-type
voltage gated Ca2+ channel. (Modified with permission from Toda N. Vasodilating -adrenoceptor
blockers as cardiovascular therapeutics. Pharmacol Ther, 2003;100:215-234. Copyright Elsevier.)
126
Adrenergic, Dopaminergic, and Serotonergic Pharmacology CHAPTER 7

CASE 7-3
A 62-year-old woman with chronic obstructive airway disease is being treated with
albuterol.
a. What kind o drug is albuterol?
Albuterolol, a selective 2 receptor agonist (see Figure 7-3), is administered either
by inhalation or orally or the symptomatic relie o bronchospasm. Some o the
major adverse e ects o receptor agonists in the treatment o asthma or obstruc-
tive pulmonary disease (COPD) are caused by stimulation o 1 receptors in the
heart. Accordingly, drugs with pre erential af nity or 2 receptors compared with
1 receptors have been developed. However, this selectivity is relative, not absolute,
and is lost at high concentrations o these drugs.
b. How does albuterol a ect airways so that this patient might expect
improvement?
In the treatment o asthma and COPD, receptor agonists are used to activate
pulmonary receptors that relax bronchial smooth muscle and decrease airway
resistance. Although this action appears to be a major therapeutic e ect o these
drugs in patients with asthma, evidence suggests that receptor agonists also may
suppress the release o leukotrienes and histamine rom mast cells in lung tissue,
enhance mucociliary unction, decrease microvascular permeability, and possibly
inhibit phospholipase A2. See Chapter 24 or a detailed discussion o the treatment
o pulmonary disease with 2 receptor agonists.
c. What adverse e ects should this patient be alerted to?
T e major adverse e ects o receptor agonists occur as a result o excessive activa-
tion o receptors. achycardia is a common adverse e ect o systemically admin-
istered receptor agonists. Stimulation o heart rate occurs primarily by means o
1 receptors. Patients with underlying cardiovascular disease are particularly at risk
or signi cant reactions. However, the likelihood o adverse e ects can be greatly
decreased in patients with lung disease by administering the drug by inhalation
rather than orally or parenterally.
T e risk o adverse cardiovascular e ects also is increased in patients who
are receiving MAO inhibitors. In general, at least 2 weeks should elapse
between the use o MAO inhibitors and administration o 2 agonists or other
sympathomimetics.
remor is a relatively common adverse e ect o the 2-selective receptor agonists.
olerance generally develops to this e ect. Feelings o restlessness, apprehension,
and anxiety may limit therapy with these drugs, particularly oral or parenteral
administration.

CASE 7-4
A 51-year-old man has su ered a myocardial in arction. In the intensive care unit, he is
started on the blocker, metoprolol.
a. Why is this patient started on a blocker soon a er the onset o his myocardial
in arction?
A great deal o interest has ocused on the use o receptor antagonists in the treat-
ment o acute myocardial in arction and in the prevention o recurrences or those
who have survived an initial attack. Numerous trials have shown that receptor
antagonists administered during the early phases o acute myocardial in arction
and continued long-term may decrease mortality by ~25%.
b. What is the mechanism o action o metoprolol?
T e pharmacodynamics and pharmacokinetic parameters o receptor antagonists
are shown in able 7-2. T e precise mechanism by which receptor antagonists
(Continued)
127
SECTION II Neuropharmacology

TABLE 7-2 Pharmacological/Pharmacokinetic Properties of Adrenergic Receptor Blocking Agents


MEMBRANE INTRINSIC
STABILIZING AGONIST LIPID EXTENT OF ORAL PLASMA PROTEIN
DRUG ACTIVITY ACTIVITY SOLUBILITY ABSORPTION % AVAILABILITY % t 1/2 HOURS BINDING %
Classical non-selective blockers: First generation

Nadolol 0 0 Low 30 30-50 20-24 30

Penbutolol 0 + High ~100 ~100 ~5 80-98

Pindolol + +++ Low >95 ~100 3-4 40

Propranolol ++ 0 High <90 30 3-5 90

Timolol 0 0 Low to 90 75 4 <10


moderate

1-Selective blockers: Second genera tion

Acebutolol + + Low 90 20-60 3-4 26

Atenolol 0 0 Low 90 50-60 6-7 6-16

Bisoprolol 0 0 Low 90 80 9-12 ~30

Esmolol 0 0 Low NA NA 0.15 55

Metoprolol +* 0 Moderate ~100 40-50 3-7 12

Non-selective blockers with additional actions: Third genera tion

Carteolol 0 ++ Low 85 85 6 23-30

Carvedilol ++ 0 Moderate >90 ~30 7-10 98

Labetalol + + Low >90 ~33 3-4 ~50

1-selective blockers with additional actions: Third genera tion

Betaxolol + 0 Moderate >90 ~80 15 50

Celiprolol 0 + Low ~74 30-70 5 4-5

Nebivolol 0 0 Low NA NA 11-30

*Detectable only at doses much greater than required or blockade.

decrease mortality and morbidity a er a myocardial in arction is not known, but


the avorable e ects may stem rom decreased myocardial oxygen demand, redistri-
bution o myocardial blood ow, and antiarrhythmic actions. T ere is likely much
less bene t i adrenergic receptor antagonists are administered or only a short
time. In studies o secondary prevention, the most extensive, avorable clinical trial
data are available or propranolol, metoprolol, and timolol. In spite o these proven
bene ts, many patients with myocardial in arction do not receive a adrenergic
receptor antagonist.
c. What are the adverse e ects o blockers?
T e most common adverse e ects o receptor antagonists arise as pharmaco-
logical consequences o blockade o receptors (see able 5-1); serious adverse
e ects unrelated to receptor blockade are rare. Receptor blockade may cause
or exacerbate heart ailure in patients with decompensated heart ailure, acute
myocardial in arction, or cardiomegaly. Abrupt discontinuation o receptor
antagonists a er long-term treatment can exacerbate angina and may increase
the risk o sudden death.
(Continued)
128
Adrenergic, Dopaminergic, and Serotonergic Pharmacology CHAPTER 7

A major adverse e ect o receptor antagonists is caused by blockade o 2 recep-


tors in bronchial smooth muscle. Drugs with selectivity or 1 receptors or those
with intrinsic sympathomimetic activity at 2 receptors seem less likely to induce
bronchospasm. Since the selectivity o current blockers or 1 receptors is modest,
these drugs should be avoided i at all possible in patients with asthma.
T e adverse e ects o receptor antagonists that are re erable to the CNS may
include atigue, sleep disturbances (including insomnia and nightmares), and
depression.
Adrenergic blockade may blunt recognition o hypoglycemia by diabetic patients;
it also may delay recovery rom insulin-induced hypoglycemia. Receptor antago-
nists should be used with great caution in patients with diabetes who are prone to
hypoglycemic reactions; 1-selective agents may be pre erable or these patients.

CASE 7-5
A 58-year-old woman has su ered rom mild heart ailure or 5 years. She has been
treated with carvedilol. She recently changed to a di erent physician who questions the
use o a blocker to treat heart ailure.
a. Why would her new physician question the use o a blocker to treat heart
ailure?
It is a common clinical observation that administration o receptor antagonists
can worsen markedly or even precipitate congestive heart ailure in compensated
patients with multiple orms o heart disease, such as ischemic or congestive cardio-
myopathy. Consequently, the hypothesis that receptor antagonists might be ef ca-
cious in the long-term treatment o heart ailure originally seemed counterintuitive
to many physicians.
Carvedilol, metoprolol, and bisoprolol all have been shown to reduce the mortality
rate in large cohorts o patients with stable chronic heart ailure regardless o sever-
ity. reatment o patients with congestive heart ailure with receptor antagonists
should only be undertaken by physicians with experience in using these drugs or
this purpose.
b. What is the mechanism o action o carvedilol or this indication?
A number o mechanisms have been proposed to play a role in the bene cial e ects
o receptor antagonists in heart ailure. Since chronic excess catecholamines may
be toxic to the heart, especially through activation o 1 receptors, inhibition o
the pathway may help preserve myocardial unction. Also, antagonism o recep-
tors in the heart may attenuate cardiac remodeling, which ordinarily might have
deleterious e ects on cardiac unction. Interestingly, activation o receptors and
elevation o cellular cyclic AMP may promote myocardial cell death by apoptosis.
In addition, properties o certain receptor antagonists that are due to other, unre-
lated properties o these drugs may be potentially important. For example, a erload
reduction by drugs such as carvedilol may be clinically relevant (see able 7-3
and Figure 7-5).

TABLE 7-3 Third Generation Receptor Antagonists with Putative Additional Mechanisms of Vasodilation
NITRIC OXIDE 2 RECEPTOR 1 RECEPTOR Ca 2+ ENTRY K+ CHANNEL ANTIOXIDANT
PRODUCTION AGONISM ANTAGONISM BLOCKADE OPENING ACTIVITY
Celiprolola Celiprolola Carvedilol Carvedilol Tilisolola Carvedilol
Nebivolol Carteolol Bucindolola Betaxolol
Carteolol Bopindolola Bevantolola Bevantolola
Bopindolola Nipradilola
Nipradilola Labetalol
a
Not currently available in the United States, where most are under investigation or use.

129
SECTION II Neuropharmacology

MECHANISM OF ACTION OF DRUGS ACTING AT SEROTINERGIC AND DOPAMINERGIC RECEPTORS


DRUG CLASS DRUG MECHANISM OF ACTION
Selective Serotonin (5-HT) Reuptake Fluoxetine Inhibition o presynaptic reaccumulation o neuronally
Inhibitors (SSRIs) Citalopram released 5-HT
Escitalopram
Fluvoxamine
Paroxetine
Sertraline

Serotonin (5-HT)/Norepinephrine (NE) Duloxetine Selective reuptake inhibitor o 5-HT and NE


Reuptake Inhibitors (SNRIs) Venla axine
Desvenla axine
Minacipran
Sibutramine

MAO Inhibitors That Alter 5-HT Concentrations Phenelzine MAO inhibition that results in alteration o 5-HT (see
Tranylcypromine Figure 7-6)
Isocarboxazid

5-HT Receptor Agonists Buspirone 5-HT1A receptor-selective partial agonist

Almotripan 5-HT1B/1D receptor-selective agonist (see Table 7-4)


Eletriptan
Frovatriptan
Naratriptan
Rizatriptan
Sumatriptan
Zolmitriptan

Ergot Alkaloid Derivative Methylsergide 5-HT2A/2C receptor antagonist with some partial agonist
activity (see Table 7-4)

5-HT Receptor Antagonists Ondansetron 5-HT3 receptor antagonist (see Table 7-4)
Dolasetron
Granisetrol
Palonosetron
Alosetron

Clozapine 5-HT2A/2C receptor antagonist (see Table 7-4 and Chapter 8)

Respiradone 5-HT2A and D2 receptor antagonist (see Table 7-4 and


Chapter 8)

Quetiapine 5-HT and DA receptor antagonist (see Table 7-4 and


Chapter 8)

Olanzepine 5-HT and DA receptor antagonist (see Table 7-4 and


Chapter 8)

Dopamine (DA) Receptor Agonists Bromocriptine Ergot derivative that is a D1/D2 receptor agonist

Pergolide Ergot derivative that is a D1/D2 receptor agonist

Cabergoline Ergot derivative that is a strong D2 receptor agonist

Apomorphine Non-ergot alkaloid with agonist properties at dopamine


receptors with an order o potency D4>D2>D3>D5

Rotigotine Dopamine receptor agonist with pre erential binding to


D2 and D3 receptors

Pramipexole Selective D2 receptor agonist (see Chapter 13)

Ropinirole Selective D2 receptor agonist (see Chapter 13)

Dopamine (DA) Receptor Antagonists Aripiprazole Partial agonist at D2 receptors


(see Chapter 8)

130
Adrenergic, Dopaminergic, and Serotonergic Pharmacology CHAPTER 7

H COOH
C C NH2
L-TRYP TOP HAN
H H
N
H
O2 tryptopha n
te tra hydropte ridine hydroxyla s e

H COOH
HO
L-5-HYDROXY- C C NH2
TRYP TOP HAN H H
N
H
L-a roma tic
vita min B6 a mino a cid
de ca rboxyla s e
H H
HO
5-HYDROXY- C C NH2
TRYP TAMINE H H 5-HT
(S EROTONIN, 5-HT) N N-a ce tyla s e
H

MAO

H O
HO
5-HYDROXYINDOLE C C
H H
ACETALDEHYDE
N
H
a lde hyde a lde hyde
NAD NADH
de hydroge na s e re ducta s e

H H H
HO O HO
C C C C OH
H OH H H
N N
H H
5-HYDROXYINDOLE 5-HYDROXY-
ACETIC ACID TRYP TOP HOL
(5-HIAA)

H H O
HO
C C NH C
H H CH3
N
N-ACETYL-5-HT
H
hydroxyindole
O-me thyltra ns fe ra s e

H H O
H3 CO
C C NH C
H H CH3
N
MELATONIN H

FIGURE 7-6 Synthesis and inactivation o serotonin. Enzymes and co- actors are shown in blue.

CASE 7-6
A 36-year-old woman has been going through a di cult divorce or the past year. wo
weeks ago her ather died. She now presents to her physician with moderate situational
depression and is started on the selective serotonin reuptake inhibitor (SSRI) uoxetine.
a. Why is a drug that modulates serotonin (5-HT) at nerve endings e ective in
treating depression?
T e synthesis and inactivation o serotonin is shown in Figure 7-6 and the actions
and clinical indications o serotonergic drugs are presented in able 7-5. A multitude
(Continued)
131
SECTION II Neuropharmacology

TABLE 7-4 Serotonin Receptor Subtypes


5-HT1, 5-HT2, 5-HT47 STRUCTURAL FAMILIES 5-HT3

5-HTgated ion channel


G proteincoupled receptor
GENE SIGNAL SELECTIVE SELECTIVE
SUBTYPE STRUCTURE TRANSDUCTION LOCALIZATION FUNCTION AGONIST ANTAGONIST
5-HT1A Intronless AC Raphe nuclei Autoreceptor 8-OH-DPAT WAY100135
Cortex Hippocampus
5-HT1Ba Intronless AC Subiculum Autoreceptor
Globus pallidus
Substantia nigra
5-HT1D Intronless AC Cranial blood vessels Vasoconstriction Sumatriptan
Globus pallidus
Substantia nigra
5-HT1E Intronless AC Cortex
Striatum
5-HT1Fb Intronless AC Brain and periphery
5-HT2A (D receptor) Introns PLC and PLA2 Platelets Aggregation -Methyl-5-HT, Ketanserin
Smooth muscle Contraction DOI, MCPP LY53857
Cerebral cortex Neuronal excitation MDL 100, 907
5-HT2B Introns PLC Stomach undus Contraction -Methyl-5-HT, LY53857
DOI SB206553
5-HT2C Introns PLC Choroid plexus CSF production -Methyl-5-HT, LY53857
PLA2 Hypothalamus Neuronal excitation DOI, MK2.2 SB 2022084
Mesulergine
5-HT3 (M receptor) Introns Ligand-operated Parasympathetic nerves Neuronal excitation 2-Methyl-5-HT Ondansetron
ion channel Solitary tract Tropisetron
Area postrema
5-HT4 Introns AC Hippocampus GI tract Neuronal excitation Renzapride GR 113808
5-HT5A Introns AC Hippocampus Unknown
5-HT5B Introns Unknown Pseudogene
5-HT6 Introns AC Hippocampus Neuronal excitation SB 271046
Striatum
Nucleus accumbens
5-HT7 Introns AC Hypothalamus Unknown 5-carboxamino-
Hippocampus GI tract tryptamine
a
Also re erred to as 5-HT1DB.
b
Also re erred to as 5-HT1EB.
AC, adenylyl cyclase; PLC, phospholipase C; PLA2; phospholipase A2; 8-OH-DPAT, 8-hydroxy-(2-N,N-dipropylamino)-tetraline; DOI, 1-(2,5-dimethoxy-
4-iodophenyl) isopropylamine;
MCPP, metachlorphenylpiperazine; MK212.

o brain unctions are in uenced by 5-H , including sleep, cognition, sensory per-
ception, motor activity, temperature regulation, nociception, mood, appetite, sexual
behavior, and hormone secretion. All o the cloned 5-H receptors are expressed in
the brain, o en in overlapping domains (see able 7-4). Although patterns o 5-H
receptor expression in individual neurons have not been extensively de ned, it is
likely that multiple 5-H receptor subtypes with similar or opposing actions are
expressed in individual neurons, leading to a tremendous diversity o actions.
(Continued)
132
Adrenergic, Dopaminergic, and Serotonergic Pharmacology CHAPTER 7

TABLE 7-5 Serotonergic Drugs: Primary Actions and Clinical Indications


RECEPTOR ACTION DRUG EXAMPLES CLINICAL DISORDER
5-HT1A Partial agonist Buspirone, ipsaperone Anxiety, depression

5-HT1D Agonist Sumatriptan Migraine

5-HT2A/2C Antagonist Methysergide, risperidone, ketanserin Migraine, depression, schizophrenia

5-HT3 Antagonist Ondansetron Chemotherapy-induced emesis

5-HT4 Agonist Cisapride GI disorders

SERT (5-HT transporter) Inhibitor Fluoxetine, sertraline Depression, obsessive-compulsive disorder, panic disorder,
social phobia, post-traumatic stress disorder

T e e ects o 5-H active drugs in anxiety and depressive disorders, like the e ects
o selective serotonin reuptake inhibitors (SSRIs), strongly suggest a role or 5-H EVIDENCE SUGGESTING
in the neurochemical mediation o these disorders. THAT 5-HT IS A KEY
b. What are the adverse e ects o f uoxetine? MEDIATOR IN THE
Excessive stimulation o brain 5-H 2 receptors may result in insomnia, increased PATHOGENESIS OF
anxiety, irritability, and decreased libido, e ectively worsening prominent depres- MIGRAINE
sive symptoms. Excess activity at spinal 5-H 2 receptors causes sexual side e ects, Plasma and platelet concentrations o
including erectile dys unction, anorgasmia, and ejaculatory delay. 5-HTvarywith the di erent phases o the
Stimulation o 5-H 3 receptors in the CNS and periphery contributes to GI e ects, migraine attack.
which are usually limited to nausea but may include diarrhea and emesis. Some Urinaryconcentrations o 5-HTand its
patients experience an increase in anxiety, especially with the initial dosing o metabolites are elevated during most
SSRIs. With continued treatment, some patients also report a dullness o intel- migraine attacks.
lectual abilities and concentration. In addition, a phenomenon o a residual at
Migraine maybe precipitated byagents (eg,
a ect can occur in an otherwise success ul treatment with SSRIs.
reserpine and enfuramine) that release
c. Although this patient is otherwise healthy and is taking no other drugs, o what 5-HT romintracellular storage sites.
drug interactions should she be warned?
Most antidepressants, including the SSRIs, exhibit drugdrug interactions based on
their routes o metabolism CYPs (see Chapter 8). Paroxetine and, to a lesser degree,
uoxetine are potent inhibitors o CYP2D6. T e other SSRIs, outside o uvoxamine,
are at least moderate inhibitors o CYP2D6. T is inhibition can result in dispropor-
tionate increases in plasma concentrations o drugs metabolized by CYP2D6.
Another important drugdrug interaction with SSRIs occurs via a pharmacody-
namic mechanism. MAOIs enhance the e ects o SSRIs due to inhibition o sero-
tonin metabolism (see Figure 7-6 and Chapters 5 and 8). Administration o these
drugs together can produce synergistic increases in extracellular brain serotonin,
leading to the serotonin syndrome. Symptoms o the serotonin syndrome include
hyperthermia, muscle rigidity, myoclonus, tremors, autonomic instability, con u-
sion, irritability, and agitation; this can progress toward coma and death.

CASE 7-7
A 23-year-old woman su ers rom migraine headaches. She is given a prescription or
the 5-H receptor agonist sumitriptan and told to take it as soon as a migraine starts.
a. Why is she given a drug that is a 5-HT receptor agonist?
T e evidence that 5-H is important in the pathogenesis o migraine headaches is
shown in the Side Bar EVIDENCE SUGGES ING HA 5-H IS A KEY MEDIA-
OR IN HE PA HOGENESIS OF MIGRAINE. Consistent with this 5-H
hypothesis, 5-H receptor agonists have become a mainstay or acute treatment
o migraine headaches. T e triptans are e ective, acute antimigraine agents. T eir
(Continued)
133
SECTION II Neuropharmacology

5-HT1A 5-HT1D/1B
Re ce ptor Re ce ptor
5-HT

5-HT

S oma tode ndritic P re s yna ptic


Autore ce ptors (5-HT1A) Autore ce ptors (5-HT1D/1B)

FIGURE 7-7 Two classes o 5-HT autoreceptors with di erential localizations. Somatodendritic
5-HT1A autoreceptors decrease raphe cell f ring when activated by 5-HT released rom axon collat-
erals o the same or adjacent neurons. The receptor subtype o the presynaptic autoreceptor on
axon terminals in the orebrain has di erent pharmacological properties and has been classif ed
as 5-HT1D (in humans) or 5-HT1B (in rodents). This receptor modulates the release o 5-HT. Postsyn-
aptic 5-HT1 receptors are also indicated.

ability to decrease, rather than exacerbate, the nausea and vomiting o migraine is
an important advance in the treatment o the condition.
b. What is the mechanism o action o sumitriptan?
T ere remains a controversy about the relative importance o vascular versus neu-
rological dys unction in migraine; thus the mechanism o the ef cacy o 5-H 1B/1D
agonists in migraine is not resolved.
According to a prominent pathophysiological model o migraine, unknown events
lead to the abnormal dilation o carotid arteriovenous anastomoses in the head and
shunting o carotid arterial blood ow, producing cerebral ischemia and hypoxia.
Based on this model, an e ective antimigraine agent would close the shunts and
restore blood ow to the brain. Indeed, sumatriptan has the capacity to produce
this vascular e ect with a pharmacological speci city that mirrors the e ects o
these agents on 5-H 1B and 5-H 1D receptor subtypes.
An alternative hypothesis concerning the signi cance o one or more 5-H 1 receptors
in migraine pathophysiology relates to the observation that both 5-H 1B and 5-H 1D
receptors serve as presynaptic autoreceptors, modulating neurotransmitter release
rom neuronal terminals (see Figure 7-7). 5-H 1 agonists may block the release o
proin ammatory neuropeptides at the level o the nerve terminal in the perivascular
space, which could account or their ef cacy in the acute treatment o migraine.
c. What are the adverse e ects o sumitriptan?
Rare but serious cardiac events have been associated with the administration o
5-H 1 agonists, including coronary artery vasospasm, transient myocardial isch-
emia, atrial and ventricular arrhythmias, and myocardial in arction, predominantly
in patients with risk actors or coronary artery disease. Orally administered triptans
can cause paresthesias; asthenia and atigue; ushing; eelings o pressure, tightness,
or pain in the chest, neck, and jaw; drowsiness; dizziness; nausea; and sweating.
T e triptans are contraindicated in patients who have a history o ischemic or
vasospastic coronary artery disease (including history o stroke or transient isch-
emic attacks), cerebrovascular or peripheral vascular disease, hemiplegic or basilar
migraines, other signi cant cardiovascular diseases, or ischemic bowel diseases.

KEY CONCEPTS
Catecholamines and sympathomimetic drugs are classi ed as direct-acting,
indirect-acting, or mixed-acting (see Figures 7-3 and 7-4).
T e pharmacological e ects o adrenergic agonists and antagonists can be
predicted rom the distribution o and adrenergic receptors (see able 5-1).
(Continued)
134
Adrenergic, Dopaminergic, and Serotonergic Pharmacology CHAPTER 7

Drugs that are selective or a particular adrenergic receptor subtype may have
therapeutic bene ts, but the selectivity is relative, not absolute, and is of en lost
at higher concentrations.
Adverse e ects and toxicity o adrenergic agonists and antagonist can mostly be
predicted rom the distribution o and adrenergic receptors (see able 5-1).
Adrenergic agonists and antagonists have broad and varied therapeutic uses,
including the treatment o shock, hypotension, hypertension, cardiac arrhyth-
mias, congestive heart ailure, nasal decongestion, asthma and chronic pul-
monary disease (COPD), allergic reactions, glaucoma, narcolepsy, weight
reduction, and attention-de cit/hyperactivity disorder (ADHD).
Serotonin (5-H ) and dopamine (DA) are neurotransmitters in the CNS and
also have prominent peripheral actions.
T e multiple actions o 5-H and DA involve interaction with distinct 5-H
and DA receptor subtypes, respectively.
Many o the therapeutic e ects o 5-H and DA agonists and antagonists in the
CNS are discussed in detail in Chapter 8 Psychopharmacology and Chapter 13
Drug T erapy o Neurodegenerative Diseases.

SUMMARY QUIZ

QUESTION 7-1 Despite its ability to stimulate receptors in the sympathetic nervous
system, norepinephrine has relatively little capacity to increase bronchial air ow
because the receptors in bronchial smooth muscle are largely o the
a. 2 subtype.
b. 1 subtype.
c. D2 subtype.
d. 5-H 4 subtype.
e. M3 subtype.

QUESTION 7-2 A 38-year-old man is taking tranylcypromine, an MAO inhibitor, or


depression. Af er a celebratory dinner, he develops a severe headache and chest pain.
At hospital his blood pressure is 190/135 mm Hg. His hypertensive crisis is likely due
to the ingestion o
a. green salad.
b. chocolate cake.
c. red wine.
d. broiled salmon.
e. wheat bread.
QUESTION 7-3 A 53-year-old woman with COPD is using an albuterol inhaler or symp-
tomatic relie o bronchospasm. She does not like using her inhaler because it causes
a. her lips to turn black.
b. her heart to beat aster.
c. numbness in her ngers and toes.
d. transient blindness.
e. transient diminished hearing.
QUESTION 7-4 A 23-year-old male college student is taking amphetamine on a regular
basis to stay alert while studying. T e mechanism o action o amphetamine that causes
this e ect is
a. due to the release o epinephrine (E) rom the adrenal medulla.
b. due to the release o histamine rom mast cells.

135
SECTION II Neuropharmacology

c. due to the stimulation o receptors in the rontal cortex o the brain.


d. due to the release o norepinephrine (NE) rom central adrenergic neurons.
e. the result o inhibition o monoamine oxidase (MAO).

QUESTION 7-5 A 34-year-old woman has been requently using a nasal spray contain-
ing the adrenergic receptor agonist oxymetazoline or nasal decongestion. She has
recently noticed that it is less e ective and her symptoms are worse. T is loss o e cacy
is most likely due to
a. the act that her spray container is empty.
b. degradation o the oxymetazoline.
c. a manu acturing de ect in the nasal spray container.
d. a loss o innervation to her nasal mucosa.
e. rebound hyperemia o her nasal mucosa.

QUESTION 7-6 A 32-year-old woman with the diagnosis o a pheochromocytoma


is scheduled or surgery to remove her adrenal tumor. T e best drug to control her
episodes o severe hypertension prior to surgery would be
a. a nonselective adrenergic antagonist such as phenoxybenzamine.
b. a selective 2 adrenergic receptor agonist such as clonidine.
c. a selective 2 adrenergic receptor agonist such as terbutaline.
d. a nonselective adrenergic receptor antagonist such as propranolol.
e. a selective 1 adrenergic receptor antagonist such as terazosin.

QUESTION 7-7 T e typical dose o oral propranolol or the treatment o hypertension


is 320 mg/day. T e typical dose o IV propranolol to treat a li e-threatening arrhythmia
is 1 to 3 mg administered slowly. T e reason or the discrepancy in these doses is
a. adrenergic receptors are more sensitive to IV than oral propranolol.
b. the IV dose avoids the rst pass metabolism o oral propranolol.
c. treatment o hypertension requires a higher dose o propranolol than does treat-
ment o a cardiac arrhythmia.
d. the density o 2 receptors in the heart is greater than the density o 2 receptors on
blood vessels.
e. oral propranolol is excreted by the kidney at a aster rate than IV propranolol.

QUESTION 7-8 A 32-year-old woman is taking sertraline or mild depression. T is drug


increases the availability o serotonin (5-H ) at the postsynaptic membrane because it
a. stimulates 5-H 4 receptors.
b. enhances the release o 5-H rom presynaptic nerve endings.
c. inhibits the presynaptic uptake o 5-H .
d. blocks MAO which degrades 5-H .
e. enhances the synthesis o 5-H .

QUESTION 7-9 A 35-year-old man is taking buspirone or anxiety. T is drug acts as a(n)
a. partial agonist at 5-H 1A receptors.
b. inhibitor o 5-H reuptake into presynaptic nerve terminals.
c. agonist at dopamine receptors.
d. agonist at 2 adrenergic receptors.
e. antagonist at M3 receptors.

QUESTION 7-10 A major mechanism or the termination o dopamines (DA) postsyn-


aptic e ect is
a. downregulation o DA postsynaptic receptors.
b. degradation o DA in postsynaptic nerve terminals.
136
Adrenergic, Dopaminergic, and Serotonergic Pharmacology CHAPTER 7

c. decreased synthesis o DA.


d. blockage o postsynaptic adrenergic receptors.
e. reuptake o DA into presynaptic nerve terminals.

SUMMARY QUIZ ANSWER KEY


QUESTION 7-1 Answer is a. An important actor in the response o any cell or organ
to sympathomimetic amines is the density and proportion o - and adrenergic recep-
tors (see able 5-1). For example, NE has relatively little capacity to increase bronchial
air ow, since the receptors in bronchial smooth muscle are largely o the 2 subtype.
QUESTION 7-2 Answer is c. MAO is an enzyme involved in the inactivation o
catcholamines (see Figure 7-1). Patients who have received MAO inhibitors may expe-
rience severe hypertensive crises i they ingest cheese, beer, or red wine. T ese and
related oods, which are produced by ermentation, contain a large quantity o tyra-
mine, and to a lesser degree, other phenylethylamines. When GI and hepatic MAOs are
inhibited, the large quantity o tyramine that is ingested is absorbed rapidly and reaches
the systemic circulation in high concentration. A massive and precipitous release o NE
can result with consequent hypertension that can be severe enough to cause myocardial
in arction or a stroke (see Figures 7-1 and 7-2).
QUESTION 7-3 Answer is b. Some o the major adverse e ects o receptor agonists
in the treatment o asthma or chronic obstructive pulmonary disease (COPD) are
caused by stimulation o 1 receptors in the heart. Accordingly, drugs with pre erential
a nity or 2 receptors compared with 1 receptors have been developed. However,
this selectivity is relative, not absolute, and is lost at high concentrations o these drugs.
Moreover up to 40% o the receptors in the human heart are 2 receptors, activation
o which can cause cardiac stimulation.
QUESTION 7-4 Answer is d. T e alerting e ect o amphetamine, its anorectic e ect,
and at least a component o its locomotor-stimulating action presumably are mediated
by release o NE rom central noradrenergic neurons. Amphetamine appears to exert
most or all o its e ects in the CNS by releasing biogenic amines rom their storage
sites in nerve terminals; the neuronal dopamine transporter (DA ) and the vesicu-
lar monoamine transporter 2 (VMA 2) appear to be two o the principal targets o
amphetamines action (see Figure 7-2).
QUESTION 7-5 Answer is e. -Adrenergic receptor agonists are used extensively as
nasal decongestants in patients with allergic or vasomotor rhinitis and in acute rhinitis
in patients with upper respiratory in ections. T ese drugs probably decrease resistance
to air ow by decreasing the volume o the nasal mucosa; this may occur by activation
o adrenergic receptors in venous capacitance vessels in nasal tissues that have erec-
tile characteristics.
Intense constriction o these vessels may cause structural damage to the mucosa.
A major limitation o therapy with nasal decongestants is loss o e cacy, rebound
hyperemia, and worsening o symptoms with chronic use or when the drug is stopped.
Although the mechanisms are uncertain, possibilities include receptor desensitization
and damage to the mucosa.
QUESTION 7-6 Answer is a. One use o phenoxybenzamine is in the treatment o
pheochromocytoma. Pheochromocytomas are tumors o the adrenal medulla and
sympathetic neurons that secrete enormous quantities o catecholamines into the
circulation. T e usual result is hypertension, which may be episodic and severe. T e
vast majority o pheochromocytomas are treated surgically; however, phenoxyben-
zamine is of en used in preparing the patient or surgery. T e drug controls episodes
o severe hypertension and minimizes other adverse e ects o catecholamines, such
as contraction o plasma volume and injury o the myocardium. -Receptor antago-
nists also are used to treat pheochromocytoma, but only af er the administration o
a nonselective -receptor antagonist.
137
SECTION II Neuropharmacology

QUESTION 7-7 Answer is b. Propranolol is highly lipophilic and is almost completely


absorbed af er oral administration. However, much o the drug is metabolized by the
liver during its rst passage through the portal circulation; on average, only ~25%
reaches the systemic circulation.
QUESTION 7-8 Answer is c. A highly speci c mechanism or altering synaptic avail-
ability o 5-H is inhibition o presynaptic reaccumulation o neuronally released
5-H . T rough this mechanism serotonin reuptake inhibitors (SSRIs), such as sertra-
line, potentiate and prolong the action o 5-H released by neuronal activity.
Newly ormed 5-H is rapidly accumulated in synaptic vesicles (through VMA 2),
where it is protected rom MAO. 5-H released by nerve-impulse ow is reaccumulated
into the presynaptic terminal by the 5-H transporter, SER (SLC6A4) (see Chapter 2).
Presynaptic reuptake is a highly e cient mechanism or terminating the action o 5-H
released by nerve-impulse ow. MAO localized in postsynaptic elements and surround-
ing cells rapidly inactivates 5-H that escapes neuronal reuptake and storage.
QUESTION 7-9 Answer is a. Long-chain arylpiperazines such as buspirone are selec-
tive partial agonists at 5-H 1A receptors. Other closely related arylpiperazines act as
5-H 1A-receptor antagonists. Buspirone, the rst clinically available drug in this series,
has been e ective in the treatment o anxiety (see Chapter 8).
QUESTION 7-10 Answer is e. Synaptically released DA is subject to both transporter
clearance and metabolism (see Figure 7-8). T e DA transporter (DA ) is not selective

P RESYNAP TIC TERMINAL De pola riza tion


(Na + a nd
Die ta ry Ca 2 + e ntry)
phe nyla la nine
He pa tic
AADC
PH DOPA DA
Tyros ine TH
Tyr
COMT
HVA HVA DOPAC
VMAT2
MAO
ALDH
DA
DA

DA
D2
Ne uro nal re uptake a utore ce ptor
(DAT, NET), the n s tora ge
(VMAT2), or me ta bolis m
Syna ptic DA

Po s ts ynaptic uptake
D1/D2 fa mily [(OCT1, OCT2,
of re ce ptors ENT (OCT3)]

P OSTSYNAP TIC CELL

DA
Effe ctor
re s pons e COMT

ALDH
HVA HVA 3MT
MAO

FIGURE 7-8 Dopaminergic terminal. Illustrated are the sequence o events (synthesis, release,
postsynaptic receptors, reuptake, and metabolism) that take place at synaptic terminals.
138
Adrenergic, Dopaminergic, and Serotonergic Pharmacology CHAPTER 7

or DA; moreover, DA can also be cleared rom the synapse by the NE transporter,
NE . Reuptake o DA by the DA transporter is the primary mechanism or termination
o DA action and allows or either vesicular repackaging o transmitter or metabolism.

SUMMARYTABLE ADRENERGIC, DOPAMINERGIC, AND SEROTONERGIC PHARMACOLOGY


TOXICITIES
CLASS AND
SUBCLASSES NAMES CLINICAL USES COMMON UNIQUE; CLINICALLY IMPORTANT
Endogenous Epinephrine (E) Used to provide rapid, emergency Restlessness, throbbing Contraindicated in patients who are
Catecholamines relie o anaphylactic reactions headaches, tremor, and receiving a nonselective adrenergic
Used in combination with local palpitations receptor antagonist, since unopposed
anesthetics to prolong their actions on vascular 1 receptors may
action (see Chapter 11) lead to severe hypertension
Used to restore cardiac rhythm in Cerebral hemorrhage and cardiac
patients in cardiac arrest arrhythmia
Norepinephrine (NE) Used as a vasoconstrictor to raise Similar to epinephrine Excessive dose may result in severe
or support blood pressure under hypertension; extravasation at the site o
certain intensive care conditions IVinjection may result in tissue necrosis
Dopamine (DA) Used in treatment o severe Nausea, vomiting, Extravasation may cause tissue necrosis
congestive heart ailure tachycardia, anginal pain, Should be avoided or used with
Also used to treat cardiogenic and arrhythmia, headache, reduced dose in patients receiving a
septic shock hypertension MAO inhibitor
Dopamine Fenoldopam Dopamine derivative used or Headache, ushing, Tachycardia or bradycardia
Derivatives control o severe hypertension dizziness
Dopexamine Used to treat congestive heart Tachycardia and hypotension at high
ailure, sepsis, and shock in usion rates
1-Selective Phenylephrine Used as a nasal decongestant and
Adrenergic a mydriatic
Receptor Agonists
(see Figure 7-3) Mephentermine Used to prevent hypotension CNS stimulation Excessive rise in blood pressure and
which accompanies spinal arrhythmias
anesthesia (Discontinued in the
United States)
Meteraminol Used in the treatment o
hypotensive states
Midodrine Used to treat autonomic Supine hypertension
insu ciency and postural
hypotension
2-Selective Clonidine Used to treat hypertension (see Dry mouth and sedation Marked bradycardia
Adrenergic Chapter 15) Sexual dys unction may Contact dermatitis with transdermal
Receptor Agonists Used o -label to treat diarrhea occur patch
(see Figure 7-3) in patients with autonomic Withdrawal reaction upon abrupt
neuropathy and use ul in treating discontinuation o long-term therapy
withdrawal in patients addicted to Use with caution in patients with
narcotic, alcohol, and tobacco cardiovascular disease
Used preoperatively or sedation
and anxiolysis
Transdermal administration may
be use ul to treat menopausal hot
ashes
Dexmedetomidine Used to treat hypertension (see Dry mouth and sedation Marked bradycardia
Chapter 15) Sexual dys unction may Withdrawal reaction upon abrupt
Used o -label to treat diarrhea occur discontinuation o long-term therapy
in patients with autonomic Use with caution in patients with
neuropathy and use ul in treating cardiovascular disease
withdrawal in patients addicted to
narcotic, alcohol, and tobacco
Used preoperatively or sedation
and anxiolysis

139
SECTION II Neuropharmacology

TOXICITIES
CLASS AND
SUBCLASSES NAMES CLINICAL USES COMMON UNIQUE; CLINICALLY IMPORTANT
Apraclonidine Used topically to reduce Minimal e ects on Use with caution in patients with
intraocular pressure systemic cardiovascular cardiovascular disease
parameters

Brimonidine Used topically to reduce Hypotension and Use with caution in patients with
intraocular pressure sedation cardiovascular disease

Guan acine Used to treat hypertension (see Dry mouth and sedation Marked bradycardia
Chapter 15) Sexual dys unction may Withdrawal reaction upon abrupt
Used o -label to treat diarrhea occur discontinuation o long-term therapy
in patients with autonomic Use with caution in patients with
neuropathy and use ul in treating cardiovascular disease
withdrawal in patients addicted to
narcotic, alcohol, and tobacco
A sustained-release orm is
approved or treatment o
attention def cit/hyperactivity
disorder (ADHD) in children aged
6-17 years

Guanabenz Used to treat hypertension (see Dry mouth and sedation Use with caution in patients with
Chapter 15) cardiovascular disease

Methyldopa Used to treat hypertension (see Use with caution in patients with
Chapter 15) cardiovascular disease

Tizanidine Muscle relaxant used or the Dry mouth and sedation Use with caution in patients with
treatment o spasticity cardiovascular disease

1 Adrenergic Prazosin Used to treat hypertension Syncopal reaction a ter Orthostatic hypotension
Receptor f rst dose
Antagonists (see Headache and dizziness
Figure 7-4) rarely limit treatment

Terazosin Used to treat hypertension Syncopal reaction a ter Orthostatic hypotension


Also used to manage benign f rst dose
prostatic hypertrophy (BPH) Headache and dizziness
rarely limit treatment

Doxazosin Used to treat hypertension Syncopal reaction a ter Orthostatic hypotension


Also used to manage BPH f rst dose
Headache and dizziness
rarely limit treatment

Al uzosin Used to treat BPH Syncopal reaction a ter Orthostatic hypotension


f rst dose
Headache and dizziness
rarely limit treatment

Tamsulosin Used to treat BPH Syncopal reaction a ter Orthostatic hypotension


f rst dose Impaired ejaculation has been
Headache and dizziness reported
rarely limit treatment

Silodosin Used to treat BPH Syncopal reaction a ter Orthostatic hypotension


f rst dose
Headache and dizziness
rarely limit treatment

(Continued)
140
Adrenergic, Dopaminergic, and Serotonergic Pharmacology CHAPTER 7

TOXICITIES
CLASS AND
SUBCLASSES NAMES CLINICAL USES COMMON UNIQUE; CLINICALLY IMPORTANT
2 Adrenergic Yohimbine Although not approved, it is used Enhanced motor activity
Receptor o -label to treat male sexual and tremors
Antagonists (see dys unction
Figure 7-4)
Phenoxybenzamine Used in the treatment o Hypotension and re ex cardiac
pheochromocytoma stimulation may cause alarming
tachycardia, arrhythmias, or ischemic
cardiac events

Phentolamine Used in the treatment o Hypotension and re ex cardiac


pheochromocytoma; used to stimulation may cause alarming
prevent dermal necrosis a ter the tachycardia, arrhythmias, or ischemic
inadvertent extravasation o an cardiac events
receptor agonist Should be used with caution in
Use to reverse the duration o patients with peptic ulcer disease
so t-tissue anesthesia because o enhanced gastric acid
secretion

Additional Ergot Alkaloids Used in the treatment o migraine Nausea and vomiting Contraindicated in women who
Adrenergic (see narrative in Chapter 13 o Leg weakness, are, or who may become pregnant
Receptor Goodman and Gilmans 12th numbness and tingling due to increased motor activity
Antagonists (see Edition) o f ngers and toes o the uterus
Figure 7-4) Contraindicated in patients
with coronary artery disease,
hypertension, impaired hepatic or
renal unction

Indoramin Used or the treatment o


hypertension, BPH, and the
prophylaxis o migraine

Ketanserin Used to treat hypertension

Urapidil Used to treat hypertension

Bunazosin Used to treat hypertension

Miscellaneous Amphetamine Used or the treatment o Excessive CNS Aggressiveness, changes in libido,
Sympatho- narcolepsy and ADHD stimulation, with paranoid hallucinations, panic
mimetic Agonists dizziness, tremor, and states, and suicidal tendencies
(see Figure 7-3) hyperactive re exes Cardiovascular e ects include
hypertension and arrhythmias
Potential or abuse (see Chapter 14)

Methamphetamine Excessive CNS Aggressiveness, changes in libido,


Schedule II drug with a high stimulation, with paranoid hallucinations, panic
potential or abuse dizziness, tremor, and states, and suicidal tendencies
Used o -label or weight hyperactive re exes Cardiovascular e ects include
reduction hypertension and arrhythmias
Potential or abuse (see Chapter 14)

Methylphenidate Used to treat narcolepsy and Insomnia, abdominal Large doses may lead to convulsions
Dex- ADHD pain, anorexia, weight Contraindicated in patients with
methylphenidate loss glaucoma
Lisdexamphetamine

Pemoline Used to treat ADHD; discontinued Insomnia, abdominal Large doses may lead to convulsions
or use in the United States pain, anorexia, weight
loss

(Continued)

141
SECTION II Neuropharmacology

TOXICITIES
CLASS AND
SUBCLASSES NAMES CLINICAL USES COMMON UNIQUE; CLINICALLY IMPORTANT
Propylhexedrine Used to treat allergic or Tachycardia and Loss o e cacy and rebound
vasomotor rhinitis restlessness hyperemia
Use with caution in patients
with cardiovascular disease and
men with prostatic enlargement
Contraindicated in patients taking
MAO inhibitors

Naphazoline Used to treat conjunctival Loss o e cacy and rebound


hyperemia and irritation (see hyperemia
Chapter 47) Use with caution in patients
with cardiovascular disease and
men with prostatic enlargement
Contraindicated in patients taking
MAO inhibitors

Oxymetazoline Used to treat conjunctival Loss o e cacy and rebound


hyperemia and irritation (see hyperemia
Chapter 47) Use with caution in patients
Also used as a nasal decongestant with cardiovascular disease and
men with prostatic enlargement
Contraindicated in patients taking
MAO inhibitors

Xylometazoline Used to treat conjunctival Loss o e cacy and rebound


hyperemia and irritation (see hyperemia
Chapter 47) Use with caution in patients
with cardiovascular disease and
men with prostatic enlargement
Contraindicated in patients taking
MAO inhibitors

Pseudoephedrine Used to treat allergic or Tachycardia and Use with caution in patients
vasomotor rhinitis restlessness with cardiovascular disease and
men with prostatic enlargement
Contraindicated in patients taking
MAO inhibitors

Modaf nil Used to treat narcolepsy Tachycardia and Use with caution in patients
restlessness with cardiovascular disease and
men with prostatic enlargement
Contraindicated in patients taking
MAO inhibitors

Adrenergic Isoproterenol Used in emergencies to stimulate Palpitations, tachycardia, Cardiac ischemia and arrhythmias
Receptor Agonists heart rate in patients with headache, and ushing may occur in patients with coronary
(see Figure 7-3) bradycardia, heart block, or artery disease
torsades de pointes

Dobutamine Indicated or the short- Increased blood pressure Patients with atrial f brillation are at
term treatment o cardiac and heart rate risk o marked increases in ventricular
decompensation that may occur rate because dobutamine acilitates
a ter cardiac surgery or in patients AVconduction
with congestive heart ailure or May increase size o a myocardial
myocardial in arction in arct by increasing myocardial O2
demand

(Continued)

142
Adrenergic, Dopaminergic, and Serotonergic Pharmacology CHAPTER 7

TOXICITIES
CLASS AND
SUBCLASSES NAMES CLINICAL USES COMMON UNIQUE; CLINICALLY IMPORTANT
2-Selective Metaproterenol Used or the long-term treatment Muscle tremor, anxiety, Dysrhythmias and cardiac e ects
Adrenergic o obstructive airway diseases, restlessness, tachycardia
Receptor Agonists asthma, and acute bronchospasm Selectivity is relative
(see Figure 7-3) (see Chapter 24) and is lost at high
concentrations

Albuterol Used or the long-term treatment Muscle tremor, anxiety, Dysrhythmias and cardiac e ects
o obstructive airway diseases restlessness, tachycardia
and or treatment o acute Selectivity is relative
bronchospasm (see Chapter 24) and is lost at high
concentrations

Levalbuterol Used or the long-term treatment Muscle tremor, anxiety, Dysrhythmias and cardiac e ects
o obstructive airway diseases restlessness, tachycardia
and or treatment o acute Selectivity is relative
bronchospasm (see Chapter 24) and is lost at high
concentrations

Pirbuterol Used or the long-term treatment Muscle tremor, anxiety, Dysrhythmias and cardiac e ects
o obstructive airway diseases restlessness, tachycardia
and or treatment o acute Selectivity is relative
bronchospasm (see Chapter 24) and is lost at high
concentrations

Terbutaline Used or the long-term treatment Muscle tremor, anxiety, Dysrhythmias and cardiac e ects
o obstructive airway diseases restlessness, tachycardia
and or treatment o acute Selectivity is relative
bronchospasm (see Chapter 24) and is lost at high
concentrations

Isoetharine Used or the long-term treatment Muscle tremor, anxiety, Dysrhythmias and cardiac e ects
o obstructive airway diseases restlessness, tachycardia
and or treatment o acute Selectivity is relative
bronchospasm (see Chapter 24) and is lost at high
concentrations

Bitolterol Used or the long-term treatment Muscle tremor, anxiety, Dysrhythmias and cardiac e ects
o obstructive airway diseases restlessness, tachycardia
and or treatment o acute Selectivity is relative
bronchospasm (see Chapter 24) and is lost at high
concentrations

Fenoterol Used or the long-term treatment Muscle tremor, anxiety, Dysrhythmias and cardiac e ects
o obstructive airway diseases restlessness, tachycardia
and or treatment o acute Selectivity is relative
bronchospasm (see Chapter 24) and is lost at high
concentrations

Procaterol Used or the long-term treatment Muscle tremor, anxiety, Dysrhythmias and cardiac e ects
o obstructive airway diseases restlessness, tachycardia
and or treatment o acute Selectivity is relative
bronchospasm (see Chapter 24) and is lost at high
concentrations

Salmeterol Prolonged duration o action Muscle tremor, anxiety, Dysrhythmias and cardiac e ects
(>12 hours) restlessness, tachycardia Increased risk o atal or near atal
Used or the long-term treatment Selectivity is relative asthma attacks
o obstructive airway diseases and is lost at high
Agent o choice or nocturnal concentrations
asthma (see Chapter 24)

(Continued)

143
SECTION II Neuropharmacology

TOXICITIES
CLASS AND
SUBCLASSES NAMES CLINICAL USES COMMON UNIQUE; CLINICALLY IMPORTANT
Formoterol Prolonged duration o action (>12 Muscle tremor, anxiety, Dysrhythmias and cardiac e ects
hours) restlessness, tachycardia Increased risk o atal or near atal
Used or the long-term treatment Selectivity is relative asthma attacks
o obstructive airway diseases and is lost at high
Agent o choice or nocturnal concentrations
asthma
Approved or prophylaxis o
exercise-induced bronchospasm
(see Chapter 24)

Ar ormoterol Prolonged duration o action Selectivity is relative Dysrhythmias and cardiac e ects
(>12 hours) and is lost at high Increased risk o atal or near atal
Used or the long-term treatment concentrations asthma attacks
o obstructive airway diseases, Skeletal muscle tremor,
including chronic bronchitis (see insomnia, and increases
Chapter 24) in blood glucose

Ritodrine Used as a uterine muscle relaxant Muscle tremor, anxiety, May actually increase maternal
to arrest preterm labor restlessness, tachycardia morbidity
Selectivity is relative
and is lost at high
concentrations

Nonselective Carteolol Used in the treatment o chronic Use with caution in patients with
Adrenergic open-angle glaucoma (see pulmonary disease or congestive
Receptor Chapter 47) heart ailure
Antagonists (see
Figure 7-4) Propranolol Used in the treatment o Use o blockers may Use with caution in patients with
hypertension, angina, acute increase atigue, sleep congestive heart ailure
coronary syndromes and disturbances, and May exacerbate asthma or COPD
congestive heart ailure depression Abrupt discontinuation a ter long-
The later use should only be May blunt recognition o term use may exacerbate angina and
undertaken by physicians hypoglycemia increase the risk o sudden death
experienced in the use o
blockers to treat CHF
(see Chapters 15, 16, 17, and 18)

Levobunolol Used in the treatment o chronic Use with caution in patients with
open-angle glaucoma (see pulmonary disease or congestive
Chapter 47) heart ailure

Metipranolol Used in the treatment o chronic Use with caution in patients with
open-angle glaucoma (see pulmonary disease or congestive
Chapter 47) heart ailure

Nadolol Used in the treatment o Use o blockers may Used with caution in patients with
hypertension, angina, acute increase atigue, sleep congestive heart ailure
coronary syndromes and disturbances, and May exacerbate asthma or COPD
congestive heart ailure depression Abrupt discontinuation a ter long-
The later use should only be May blunt recognition o term use may exacerbate angina and
undertaken by physicians hypoglycemia increase the risk o sudden death
experienced in the use o
blockers to treat CHF
(see Chapters 15, 16, 17, and 18)

(Continued)

144
Adrenergic, Dopaminergic, and Serotonergic Pharmacology CHAPTER 7

TOXICITIES
CLASS AND
SUBCLASSES NAMES CLINICAL USES COMMON UNIQUE; CLINICALLY IMPORTANT
Timolol Ophthalmic preparations used in Use o blockers may Use with caution in patients with
the treatment o chronic open- increase atigue, sleep congestive heart ailure
angle glaucoma (see Chapter 47) disturbances, and May exacerbate asthma or COPD
Also used or the treatment o depression Abrupt discontinuation a ter long-
hypertension, angina, acute May blunt recognition o term use may exacerbate angina and
coronary syndromes and hypoglycemia increase the risk o sudden death
congestive heart ailure
The later use should only be
undertaken by physicians
experienced in the use o
blockers to treat CHF
(see Chapters 15, 16, 17, and 18)

Pindolol Used in the treatment o chronic Use o blockers Used with caution in patients with
open-angle glaucoma may increase atigue, congestive heart ailure
Also used or the treatment o sleep disturbances, May exacerbate asthma or COPD
hypertension, angina, acute and depression Abrupt discontinuation a ter long-
coronary syndromes and May blunt recognition o term use may exacerbate angina and
congestive heart ailure hypoglycemia increase the risk o sudden death
The later use should only be
undertaken by physicians
experienced in the use o
blockers to treat CHF
(see Chapters 15, 16, 17, and 18)

1 Selective Metoprolol Used to treat hypertension, Use o blockers Selectivity is relative and dependent
Adrenergic angina, tachycardia, heart may increase atigue, on dose
Receptor ailure, vasovagal syncope, and sleep disturbances, Should be avoided i possible in
Antagonists (see as secondary prevention o and depression patients with asthma or COPD
Figure 7-4) myocardial in arction May blunt recognition o Contraindicated or the treatment
Also used or migraine prophylaxis hypoglycemia o acute myocardial in arction in
(see Chapters 15, 16, and 17) patients with heart rates <45 bpm,
heart block greater than f rst degree,
systolic blood pressure <100 mm Hg,
or moderate to severe chronic heart
ailure

Betaxolol Used to treat hypertension, Use o blockers Selectivity is relative and dependent
angina pectoris, and glaucoma may increase atigue, on dose
sleep disturbances, Should be avoided i possible in
and depression patients with asthma or COPD
May blunt recognition o
hypoglycemia

Levobetaxolol Used to treat hypertension, Use o blockers Selectivity is relative and dependent
angina pectoris, and glaucoma may increase atigue, on dose
sleep disturbances, Should be avoided i possible in
and depression patients with asthma or COPD
May blunt recognition o
hypoglycemia

Bisoprolol Used to treat hypertension and Use o blockers Selectivity is relative and dependent
moderate to severe chronic heart may increase atigue, on dose
ailure, cardiac arrhythmias, and sleep disturbances, Should be avoided i possible in
ischemic heart disease and depression patients with asthma or COPD
(see Chapters 15, 16, 17, and 18) May blunt recognition o
hypoglycemia

(Continued)

145
SECTION II Neuropharmacology

TOXICITIES
CLASS AND
SUBCLASSES NAMES CLINICAL USES COMMON UNIQUE; CLINICALLY IMPORTANT
Acebutolol Used to treat hypertension, Use o blockers Selectivity is relative and dependent
ventricular and atrial cardiac may increase atigue, on dose
arrhythmias, and acute myocardial sleep disturbances, Should be avoided i possible in
in arction and depression patients with asthma or COPD
(see Chapters 15, 16, and 18) May blunt recognition o
hypoglycemia

Atenolol Used to treat hypertension, Use o blockers Selectivity is relative and dependent
angina, tachycardia, and as may increase atigue, on dose
secondary prevention o sleep disturbances, Should be avoided i possible in
myocardial in arction (see and depression patients with asthma or COPD
Chapters 15, 16, and 17) May blunt recognition o
Also used to treat Gravesdisease hypoglycemia
until anti-thyroid medications can
take e ect (see Chapter 27)

Esmolol Used when short duration o


blockade is needed such as
during surgery to control heart
rate or in the treatment o
supraventricular tachycardia (see
Chapter 18)

Adrenergic Labetalol Used to treat hypertension (see Use o blockers Should be avoided in patients with
Receptor Chapter 15) may increase atigue, asthma or COPD
Antagonists sleep disturbances,
with Additional and depression
Cardiovascular May blunt recognition o
E ects (Third hypoglycemia
Generation
Blockers) (see Carvedilol Used to treat hypertension, mild Use o blockers Should be avoided in patients with
Figure 7-4) to severe congestive heart ailure, may increase atigue, asthma or COPD
and acute myocardial in arction sleep disturbances,
(see Chapters 15, 16, and 17) and depression
May blunt recognition o
hypoglycemia

Bucindolol Used to treat hypertension, mild Use o blockers Should be avoided in patients with
to severe congestive heart ailure, may increase atigue, asthma or COPD
and acute myocardial in arction sleep disturbances,
(see Chapters 15, 16, and 17) and depression
May blunt recognition o
hypoglycemia

Celiprolol Used to treat hypertension and Use o blockers Should be avoided in patients with
angina may increase atigue, asthma or COPD
sleep disturbances,
and depression
May blunt recognition o
hypoglycemia

Nebivolol Used to treat hypertension Use o blockers Should be avoided in patients with
may increase atigue, asthma or COPD
sleep disturbances,
and depression
May blunt recognition o
hypoglycemia

Selective Fluoxetine Used to treat depression and Sedation (see Chapter 8) Signif cant drug interactions due to
Serotonin Citalopram anxiety (see Chapter 8) inhibition o CYPs (see Chapter 8)
(5-HT) Reuptake Escitalopram
Inhibitors (SSRIs) Fluvoxamine
Paroxetine
Sertraline
(Continued)
146
Adrenergic, Dopaminergic, and Serotonergic Pharmacology CHAPTER 7

TOXICITIES
CLASS AND
SUBCLASSES NAMES CLINICAL USES COMMON UNIQUE; CLINICALLY IMPORTANT
Serotonin(5-HT)/ Duloxetine Used to treat depression, anxiety, See Chapter 8 See Chapter 8
Norepinephrine peripheral neuropathy, and
(NE) Reuptake f bromyalgia (see Chapter 8)
Inhibitors (SNRIs)
Venla axine Used to treat depression, anxiety, See Chapter 8 See Chapter 8
and panic disorders (see Chapter 8)

Desvenla axine Used to treat depression, anxiety, See Chapter 8 See Chapter 8
and panic disorders (see Chapter 8)

Minacipran Approved or f bromyalgia

Inhibitor o the Sibutramine Used as an appetite suppressant


Reuptake o
5-HT, NE, and DA
(dopamine)

MAO Inhibitors Phenelzine Used to treat depression (see See Chapter 8 See Chapter 8
That Alter 5-HT Tranylcypromine Chapter 8)
Concentrations Isocarboxazid

5-HT Receptor Buspirone Used to treat anxiety (see See Chapter 8 See Chapter 8
Agonists Chapter 8)

Almotripan Used to treat an acute attack o Paresthesias, asthenia Contraindicated in patients who
Eletriptan migraine, but not intended or use and atigue, ushing, have a history o ischemic or
Frovatriptan in the prophylaxis o migraine tightness in the chest, vasospastic coronary artery disease,
Naratriptan dizziness, nausea, and cerebrovascular or peripheral
Rizatriptan sweating vascular disease, hemiplegic or
Sumatriptan basilar migraines, or other signif cant
Zolmitriptan cardiovascular diseases, or ischemic
bowel disease
Naratriptan is also contraindicated
in patients with severe renal or
hepatic impairment
Eletriptan is contraindicated in hepatic
disease Almotriptan, rizatriptan,
sumatriptan, and zolmitriptan are
contraindicated in patients who have
taken an MAO inhibitor within the
preceding 2 weeks (see Chapter 8)

Methylsergide Used or the prophylactic A complication o prolonged


treatment o migraine (not treatment is in ammatory
available in the United States) pulmonary, coronary, and
endocardial f brosis

5-HT Receptor Ondansetron Used to treat chemotherapy- See Chapter 33 See Chapter 33
Antagonists Dolasetron induced nausea (see Chapter 33)
Granisetrol
Palonosetron

Alosetron Used in the treatment o irritable See Chapter 33 See Chapter 33


bowel syndrome (see Chapter 33)

Ketanserin Used to treat hypertension (not


available in the United States)

Clozapine Atypical antipsychotics See Chapter 8 See Chapter 8


Respiradone
Quetiapine
Olanzepine

(Continued)
147
SECTION II Neuropharmacology

TOXICITIES
CLASS AND
SUBCLASSES NAMES CLINICAL USES COMMON UNIQUE; CLINICALLY IMPORTANT
Dopamine (DA) Bromocriptine Used or the treatment o Nausea, dizziness, and Hallucination and other CNS e ects
Receptor Agonists Parkinsons disease (see headache such as psychosis
Chapter 13)
Also used to treat acromegaly (see
Chapter 26)

Pergolide Used or the treatment o Has been linked to valvular heart


Parkinsons disease (see disease
Chapter 13)
Used o -label to treat hyper-
prolactinemia (see
Chapter 26)

Cabergoline Used to treat hyper-prolactinemia Hypotension, nausea, Has been linked to valvular heart
(see Chapter 26) dizziness disease

Apomorphine Used to treat Parkinsons disease See Chapter 13 See Chapter 13


(see Chapter 13)

Rotigotine Used to treat Parkinsons disease See Chapter 13 See Chapter 13


(see Chapter 13; not available in
the United States due to problems
with patch release o drug)

Pramipexole Used to treat Parkinsons disease Sedation


(see Chapter 13)

Ropinirole Used to treat Parkinsons disease Sedation


(see Chapter 13)
Also used to treat restless leg
syndrome

Dopamine Aripiprazole An atypical antipsychotic (see See Chapter 8 See Chapter 8


(DA) Receptor Chapter 8)
Antagonists

148
CHAPTER

Psychopharmacology 8
Chapter 8 Psychopharmacology is a combination o Chapter 15, Drug T erapy o
DRUGS INCLUDED
Depression and Anxiety Disorders and Chapter 16, Pharmacology o Psychosis and
Mania rom Goodman and Gilmans T e Pharmacological Basis of T erapeutics, 12th IN THIS CHAPTER
Edition. An understanding o the material in those chapters will be help ul in ollowing Amitriptyline (ELAVIL, others)
the material presented in this chapter. In addition to the material presented here, the Amoxapine (ASENDIN)
above chapters in the 12th Edition include:
Aripiprazole (ABILIFY)
A characterization o depressive and anxiety disorders
Asenapine (SAPHRIS, others)
able 15-2, T e Potencies o Antidepressants at the Human ransporters or Norepi-
Atomoxetine (STRATTERA)
nephrine (NE ), Serotonin (SER ), and Dopamine (DA )
Bupropion (WELLBUTRIN, ZYBAN, others)
able 15-4, Potencies o Selected Antidepressants at Muscarinic, Histamine H 1, and
alpha1-Adrenergic Receptors Buspirone (BUSPAR)
A discussion o the long-term adaptive e ects o antidepressants that enhance the Carbamazepine (TEGRETOL, others)
e ectiveness o therapy Chlorpromazine (THORAZINE, others)
A discussion o the pathophysiology o psychosis Citalopram(CELEXA)
A discussion o the chemistry o antipsychotic agents Clomipramine (ANAFRANIL)
T e role o dopamine and serotonin receptors in antipsychotic therapy Clozapine (CLOZARIL, others)
A detailed discussion o the use antipsychotic agents or nonpsychotic disorders Desipramine (NORPRAMIN)
A discussion o the adverse e ects o antipsychotic agents that are not predicted by Desvenla axine (PRISTIQ)
monoamine receptor a nities Doxepin (ADAPIN, SINEQUAN)
Novel treatments or psychosis and mania Droperidol (INAPSIN, others)
Duloxetine (CYMBALTA)
LEARNING OBJECTIVES
Escitalopram(LEXAPRO)
Understand the mechanisms o action, therapeutic uses, and adverse e ects o
antidepressant drugs. Fluoxetine (PROSAC, SYMBYAX,others)
Know how antidepressant drugs are used to manage depression. Fluphenazine (PROLIXIN, others)
Fluvoxamine (LUVOX)
Know the mechanisms o action, therapeutic uses, and adverse e ects o
antipsychotic drugs and drugs used to treat mania. Haloperidol (HALDOL, others)
Understand the pharmacotherapy o acute and chronic psychoses and Iloperidone (FANAPT)
bipolar disorder. Imipramine (TOFRANIL, others)
Isocarboxazid (MARPLAN)
Lamotrigine (LAMICTAL)
Lithium
Loxapine (LOXITANE)
Maprotiline (LUDIOMIL)
Mianserin (DEPNON, others)not
approved in the United States
Milnacipran (IXEL, others)not approved
in the United States
Mirtazapine (REMERON, others)
Molindone (MOBAN)use discontinued
Ne azodone (DUTONIN, others)
(continues)

149
SECTION II Neuropharmacology

DRUGS INCLUDED MECHANISMS OF ACTION OF PSYCHOPHARMACOLOGICAL AGENTS


IN THIS CHAPTER (Cont.) DRUG CLASS DRUG MECHANISM OF ACTION
Nortriptyline (PAMELOR) Monamine Oxidase Inhibitors Tranylcypromine Irreversible inhibitors o MAO-A and
Olanzapine (ZYPREXA) (MAOIs) Phenelzine MAO-B that increase noradrenergic
Paliperidone (INVEGA) Isocarboxazid and serotonergic neurotransmission
Selegiline by inhibiting the catabolism o
Paroxetine (PAXIL) norepinephrine (NE) and serotonin
Perphenazine (TRILAFON, others) (5-HT) (see Figure 8-1)
Selegiline is a selective MAO-B inhibitor
Phenelzine (NARDIL) that is also used to treat Parkinsons
Protriptyline (VIVACTIL) disease (see Chapter 13)
Quetiapine (SEROQUEL) Serotonin, Norepinephrine Amitriptyline Increase noradrenergic and serotonergic
Reboxetine (EDRONAX)not available in Reuptake Inhibitors (SNRIs) Doxepin neurotransmission by blocking the
the United States Tertiary Amine Tricyclics Imipramine norepinephrine (NET) and serotonin
(Tricyclic Antidepressants, TCAs) Clomipramine (SERT) transporters at presynaptic
Risperidone (RESPERDAL) Trimipramine terminals (see Figure 8-1)
Selegiline (EMSAM) Clomipramine is somewhat selective or
inhibition o serotonin uptake
Sertindole (SERDOLECT,others)not
approved in the United States Serotonin, Norepinephrine Amoxapine Increase noradrenergic and serotonergic
Sertraline (ZOLOFT) Reuptake Inhibitors (SNRIs) Desipramine neurotransmission by blocking the
Secondary Amine Tricyclics, Maprotiline norepinephrine (NET) and serotonin
Tranylcypromine (PARNATE) Tricyclic Antidepressants (TCAs) Nortriptyline (SERT) transporters at presynaptic
Trazodone (DESYREL) Protriptyline terminals (see Figure 8-1)
Trifuoperazine (STELAZINE, others) Selective Serotonin Reuptake Fluoxetine Blockade o serotonin reuptake results in
Trimipramine (SURMONTIL) Inhibitors (SSRIs) Fluvoxamine enhanced and prolonged serotonergic
Paroxetine neurotransmission
Valproicacid, divalproexsodium(DEPAK- Sertraline
ENE, DEPAKOTE, others) Citalopram
Venla axine (EFFEXOR) Escitalopram

Ziprasidone (GEODON) Atypical Antidepressants Duloxetine Inhibit the reuptake o both NE and
and SNRIs Venla axine 5-HT, but do not have a tricyclic
Desvenla axine structure and thus avoid some o
Milnacipran the side e ects o TCAs

Atomoxetine Inhibits the reuptake o NE

Trazodone Antagonist o the 5-HT2 amily o


Ne azodone receptors
Mirtazapine
Mianserin

Bupropion Enhances both noradrenergic and


dopaminergic neurotransmission via
reuptake inhibition o NE and dopamine
(DA) and may involve the presynaptic
release o NE and DA

Reboxetine NE reuptake inhibitor

Anxiolytic Drugs See Chapter 9 See Chapter 9


(Benzodiazepines)

Anxiolytic Drugs (not Buspirone Partial agonist at 5-HT1A receptors and


benzodiazepines) antagonist at D2 receptors

Typical Antipsychotic Drugs Chlorpromazine Dopamine antagonists, but also


(Phenothiazines) Perphenazine antiadrenergic, anticholinergic,
Tri uoperazine antiserotonergic, and antihistaminergic
Fluphenazine (see Figure 8-2)

150
Psychopharmacology CHAPTER 8

DRUG CLASS DRUG MECHANISM OF ACTION


Typical Antipsychotic Molindone Dopamine antagonists, but also
Drugs (Other) Loxapine antiadrenergic, anticholinergic,
Haloperidol antiserotonergic, and antihistaminergic
Droperidol (see Figure 8-2)

Atypical Antipsychotic Drugs Aripiprazole Reduced dopaminergic


Olanzapine neurotransmission through 2
Quetiapine mechanisms: (1) D2 antagonism
Risperidone (olanzapine) and (2) partial D2 agonism
Asenapine (aripiprazole)
Clozapine Many o the atypical antipsychotic
Ziprasidone drugs also are antagonists at serotonin
Paliperidone receptors (eg, olanzapine, iloperidone,
Iloperidone clozapine, asenapine, paliperidone,
Sertindole risperidone, quetiapine, sertindole, and
ziprasidone)
Some atypical antipsychotics are
antagonists at adrenergic, cholinergic,
and histaminergic receptors as well
(see Figure 8-2)

Anticonvulsants Used to Treat Valproic acid Valproate exhibits non-speci c binding


Mania Carbamazepine to voltage-gated Na+ channels (see
Lamotrigine Chapter 12)
Carbamazepine and lamotrigine
have speci c high a nity or the open-
channel con guration o the alpha
subunit o the Na+ channel (see
Chapter 12)

Lithium Lithium carbonate Plausible mechanisms o action o Li+


Lithium citrate a. Li+ is not a substrate or the Na+
pump and there ore cannot maintain
membrane potential
b. Li+ can inter ere with the activity o
both stimulatory and inhibitory G
proteins (Gs and Gi) by keeping them
in their inactive state
c. Inhibition o inositol
monophosphatase and inter erence
with phosphatidylinositol pathway
(see Figure 8-2) leading to decreased
cerebral inositol concentrations
d. Li+ treatment also leads to consistent
decreases in the unctioning o
protein kinases in brain tissue,
including PKC
e. Li+ and valproate treatment also
inhibits the activity o glycogen
synthase kinase-3 (GSK-3)
. LI+ and valproate both reduce
arachidonic acid turnover in brain
membrane phospholipids
g. Li+ and valproate both interact
with nuclear regulatory actors that
a ect gene expression, including
increased expression o Bcl-2, which
is associated with protection against
neuronal degeneration/apoptosis

151
SECTION II Neuropharmacology

P RESYNAP TIC P OSTSYNAP TIC


5-HT

5-HTR 1D/2A TCAs


2 AR
Nora dre ne rgic 1 AR
ne uron IP 3
P IP 2 IP
I Li+
NE NE NE
synthe s is
Re upta ke
AR
MAO
NET

TCAs
S NRIs
Modula tion
of ce ll s igna ling
MAOIs S S RIs pa thways a nd
TCAs ce ll function
S NRIs

S ERT
MAO Re upta ke

5-HT 5-HT 5-HT


synthe s is 5-HTR 1-7

S e rotone rgic
ne uron 5-HTR 1A/1D/7
2 AR

NE

FIGURE 8-1 Sites o action o antidepressants. Schematics representing noradrenergic (top) and
serotonergic (bottom) nerve terminals. SSRIs, SNRIs, and TCAs increase noradrenergic or seroto-
nergic neurotransmission by blocking the norepinephrine or serotonergic transporter at presyn-
aptic terminals (NET, SERT). MAOIs inhibit the catabolism o norepinephrine and serotonin. Some
antidepressants such as trazodone and related drugs have direct e ects on serotonergic recep-
tors that contribute to their clinical e ects. Chronic treatment with a number o antidepressants
desensitizes presynaptic autoreceptors and heteroreceptors, producing long-lasting changes in
monoaminergic neurotransmission. Post-receptor e ects o antidepressant treatment, including
modulation o GPCR signaling and activation o protein kinases and ion channels, are involved in
the mediation o the long-term e ects o antidepressant drugs. Note that NE and 5-HT also a ect
each others neurons.

CASE 8-1
A 39-year-old woman is taking amitriptyline or major depression.
a. How do antidepressants act to elevate the mood o patients?
Many di erent antidepressants have estab ished track records o e cacy or treat-
ing major depression. However, they a su er some imitations in e cacy, since at
east 20% o a depressed patients are re ractory to mu tip e di erent antidepres-
sants at adequate doses. In monoamine systems, reuptake o the transmitter is the
main mechanism by which neurotransmission is terminated; thus, inhibition o
reuptake can enhance neurotransmission, presumab y by s owing c earance o the
transmitter rom the synapse and pro onging the dwe -time o the transmitter
in the synapse. Enhancing neurotransmission may subsequent y ead to adaptive
changes (see Figure 8-1).

(Continued)
152
Psychopharmacology CHAPTER 8

P RESYNAP TIC P OSTSYNAP TIC


A2 R
D2 /D3 R Anti-P sy
Dopa mine rgic
ne uron
+ Ca 2+
+ IP 3
cAMP
Li+ P IP 2 IP
D2 R I
DA DA DA
synthe s is cAMP Li+
Re upta ke +
MAO
DAT D1 R

S ome
S timula nts Anti-P sy

FIGURE 8-2 Sites o action o antipsychotic agents and Li+. In varicosities (terminals) along
terminal arborizations o dopaminergic neurons projecting rom midbrain to orebrain, DA is
synthesized and stored in vesicles. Following exocytotic release, DA interacts with postsynaptic
receptors (R) o D1 and D2 types, and presynaptic D2 and D3 autoreceptors. Termination o DA
action occurs primarily by active transport o DA into presynaptic terminals via the DA trans-
porter DAT, with secondary deamination by mitochondrial monoamine oxidase (MAO). Stimula-
tion o postsynaptic D1 receptors activates the Gs-adenylyl cyclase-cAMP pathway. D2 receptors
couple through Gi to inhibit adenylyl cyclase and through Gq to activate the PLC-IP3-Ca2+ path-
way. Activation o the Gi pathway can also activate K+ channels, leading to hyperpolarization.
Lithium inhibits the phosphatase that liberates inositol (I) rom inositol phosphate (IP). Li+ can
also inhibit depolarization-evoked release o DA and NE, but not 5-HT. D2-like autoreceptors sup-
press synthesis o DA by diminishing phosphorylation o rate-limiting TH, and by limiting DA
release. In contrast, presynaptic A2 adenosine receptors (A2R) activate AC and, through cyclic AMP
production, TH activity. All antipsychotic agents act at D2 receptors and autoreceptors; some also
block D1 receptors. Stimulant agents inhibit DA reuptake by DAT, thereby prolonging the dwell
time o synaptic DA. Initially in antipsychotic treatment, DA neurons release more DA, but ol-
lowing repeated treatment, they enter a state o physiological depolarization inactivation, with
diminished production and release o DA, in addition to continued receptor blockade. (T-line),
inhibition or blockade; +, elevation o activity; , reduction o activity.

Long-term e ects o antidepressant drugs evoke adaptive or regu atory mecha-


nisms that enhance the e ectiveness o therapy. T ese responses inc ude increased
adrenergic or serotonergic receptor density or sensitivity, increased receptor-G
protein coup ing and cyc ic nuc eotide signa ing, induction o neurotrophic actors,
and increased neurogenesis in the hippocampus. Persistent antidepressant e ects
depend on the continued inhibition o 5-H or NE transporters, or enhanced sero-
tonergic or noradrenergic neurotransmission achieved by an a ternative pharmaco-
ogica mechanism. For examp e, chronic treatment with some antidepressants that
interact direct y with monoamine transporters (eg, SSRIs, SNRIs, or NE reuptake
inhibitors) reduces the expression and activity o 5-H or NE transporters in the
brain, which resu ts in enhanced serotonergic or noradrenergic neurotransmission.
b. What are the major considerations when starting an antidepressant medication?
Fo owing initiation o antidepressant drug treatment, there is genera y a thera-
peutic ag asting 3 to 4 weeks be ore a measurab e therapeutic response becomes
evident. T is is the reason that e ectroconvu sive therapy may be the treatment o
choice or agitated, depressed patients with a high risk o suicide. Some patients
may respond to antidepressant treatment sooner than 3 to 4 weeks; others may
require more than 8 weeks or an adequate response. In genera , i a patient does
not respond to a given antidepressant a er an 8-week tria on an adequate dose,
then switching to another antidepressant with a di erent mechanism o action is
a reasonab e next step (eg, SSRI to SNRI). I a partia response has been observed,
other drugs may be added to the primary SSRI or SNRI medications; these additive
medications inc ude the antidepressant drug bupropion, thyroid hormone (triiodo-
thyronine), or an atypica antipsychotic (aripiprazo e or o anzapine).
(Continued)
153
SECTION II Neuropharmacology

A er the success u initia treatment phase, a 6- to 12-month maintenance treat-


ment phase is typica , a er which the drug is gradua y withdrawn. I a patient has
experienced 2 separate episodes o major depression or is chronica y depressed (ie,
>2 years), i e ong treatment with an antidepressant is advisab e.
A controversia issue regarding the use o a antidepressants is their re ationship to
suicide. Data estab ishing a c ear ink between antidepressant treatment and suicide
are acking. T e FDA has issued a b ack box warning regarding the use o SSRIs
and a number o other antidepressants in chi dren and ado escents, particu ar y
during the ear y phase o treatment, due to the possibi ity o an association between
antidepressant treatment and suicide.
c. What are the adverse ef ects that this patient might expect?
Amitripty ine is a tricyc ic antidepressant. CAs are potent anticho inergics and
antagonists at histamine H 1 receptors; H 1 receptor antagonism contributes to the
sedative e ects o CAs ( ab e 8-1). Antagonism o muscarinic acety cho ine recep-
tors contributes to cognitive du ing as we as a range o adverse e ects mediated
by the parasympathetic nervous system (b urred vision, dry mouth, tachycardia,
constipation, di cu ty urinating). Some to erance does occur or these anticho in-
ergic e ects, which are mitigated by titration strategies to reach therapeutic doses
over a reasonab e period o time. Antagonism o 1-adrenergic receptors contrib-
utes to orthostatic hypotension and sedation. Weight gain is another side e ect o
this c ass o antidepressants.
CAs a so have quinidine- ike e ects on cardiac conduction that can be i e-threat-
ening with overdose and imit the use o CAs in patients with coronary heart
disease. T is is the primary reason that no more than a 1-week supp y shou d be
provided to a new patient; even during maintenance treatment, on y a very imited
supp y shou d be avai ab e to the patient at any given time. Like other antidepres-
sant drugs, CAs a so ower the seizure thresho d.
d. What are this patients options i the side ef ects o amitriptyline become
intolerable?
T e SSRIs are a so e ective in treating major depression. A o the SSRIs show a
c ear improvement in sa ety margin compared to the CAs and are much sa er in
overdose, and in c inica practice have a ected a broad range o psychiatric, behav-
iora , and medica conditions, or which they are used, on- and o - abe .
In addition to use as antidepressants, SSRIs a so are anxio ytics with demonstrated
e cacy in the treatment o genera ized anxiety, panic, socia anxiety, and obsessive-
compu sive disorders.

CASE 8-2
A 48-year-old man has developed depression a er the death o his wi e. He is prescribed
venla axine.
a. How does venla axine dif er rom other antidepressants?
Many o der CAs b ock both SER and NE , but with a high side e ect burden.
Four medications with a nontricyc ic structure that inhibit the reuptake o both
5-H and norepinephrine have been approved or use in the United States or treat-
ment o depression, anxiety disorders, and pain: ven a axine and its demethy ated
metabo ite, desven a axine; du oxetine; and mi nacipran (approved on y or bro-
mya gia pain in the United States). T e rationa e behind the deve opment o these
newer agents was that targeting both SER and NE , ana ogous to the e ects o
some CAs, might improve overa treatment response.
(Continued)

154
TABLE 8-1 Antidepressants: Chemical Structures, Dose and Dosage Forms, and Side E ects
DOSE AND DOSAGE AMINE
NONPROPRIETARY NAME t r a d e n a me FORMS EFFECTS SIDE EFFECTS

Norepinephrine Reupta ke Inhibitors: Usuala Dosage Agitation Seizures Sedation Hypo- Anticholinergic GI Ef ects Weight Sexual Cardiac
Tertiary Amine Tricyclics Dose Form tension Ef ects Gain Ef ects Ef ects
R1 (mg/day)

R3 R2
R1 R2 R3

Amitriptyline (el avil and others) 100-200 O, I NE, 5-HT 0 2+ 3+ 3+ 3+ 0/+ 2+ 2+ 3+


C H C=CH(CH2)2N(CH3)2

Clomipramine (a n a f r a n il ) 100-200 O NE, 5-HT 0 3+ 2+ 2+ 3+ + 2+ 3+ 3+


C CI N-CH2)3 N(CH3)2

Doxepin (a d a pin , sin eq ua n ) 100-200 O NE, 5-HT 0 2+ 3+ 2+ 2+ 0-+ 2+ 2+ 3+


O H C=CH(CH2)2N(CH3)2

Imipramine (t o f r a n il and others) 100-200 O, I NE, 5-HT 0/+ 2+ 2+ 2+ 2+ 0/+ 2+ 2+ 3+


C H N-(CH2)3N(CH3)2

P
s
y
(+)-Trimipramine (su r mo n t il ) 75-200 O NE, 5-HT 0 2+ 3+ 2+ 3+ 0/+ 2+ 2+ 3+

c
h
C H

o
p
h
Norepinephrine Reupta ke Inhibitors:

a
Secondary Amine Tricyclics

r
m
a
Amoxapine (a sen d in ) 200-300 O NE, DA 0 2+ + 2+ + 0/+ + 2+ 2+

c
o
l
o
Desipramine (n o r pr a min ) 100-200 O NE + + 0/+ + + 0/+ + 2+ 2+

g
y
Maprotiline (l u d io mil ) 100-150 O NE 0/+ 3+ 2+ 2+ 2+ 0/+ + 2+ 2+

Nortriptyline (pa mel o r ) 75-150 O NE 0 + + + + 0/+ + 2+ 2+

Protriptyline (viva c t il ) 15-40 O NE 2+ 2+ 0/+ + 2+ 0/+ + 2+ 3+

Selective Serotonin Reuptake Inhibitors

C
H
()-Citalopram (c el exa ) 20-40 O 5-HT 0/+ 0 0/+ 0 0 3+ 0 3+ 0

A
P
T
(+)- Escitalopram (l exa pr o ) 10-20 O 5-HT 0/+ 0 0/+ 0 0 3+ 0 3+ 0

E
R
()-Fluoxetine (pr o za c ) 20-40 O 5-HT + 0/+ 0/+ 0 0 3+ 0/+ 3+ 0/+

8
Fluvoxamine (luvox) 100-200 O 5-HT 0 0 0/+ 0 0 3+ 0 3+ 0
1
()-Paroxetine (pa xil ) 20-40 O 5-HT + 0 0/+ 0 0/+ 3+ 0 3+ 0
5
5
1
DOSE AND DOSAGE AMINE
5
NONPROPRIETARY NAME FORMS EFFECTS SIDE EFFECTS
6
t r a d e n a me

(+)-Sertraline (zo l o f t ) 100-150 O 5-HT + 0 0/+ 0 0 3+ 0 3+ 0


NHCH3

S
E
C
T
I
O
N
I
I
Cl
Cl

()-Venla axine (ef f exo r ) 75-225 O 5-HT, NE 0/+ 0 0 0 0 3+ 0 3+ 0/+

Atypical Antidepressants

N
e
()-Atomoxetine (st r at t er a ) 40-80 O NE 0 0 0 0 0 0/+ 0 0 0

u
(children:

r
o
mg/kg)

p
h
a
Bupropion (wel l bu t r in ) 200-300 O DA, ?NE 3+ 4+ 0 0 0 2+ 0 0 0

r
m
a
(+)-Duloxetine (c ymba lt a ) 80-100 O NE, 5-HT + 0 0/+ 0/+ 0 0/+ 0/+ 0/+ 0/+

c
o
l
o
()-Mirtazapine (r emer o n ) 15-45 O 5-HT, NE 0 0 4+ 0/+ 0 0/+ 0/+ 0 0

g
y
Ne azodone (ser zo n e) 200-400 O 5-HT 0 0 3+ 0 0 2+ 0/+ 0/+ 0/+

Trazodone (d esyr el ) 150-200 O 5-HT 0 0 3+ 0 0 2+ + + 0/+

Monoamine Oxidase Inhibitors

Phenelzine (n a r d il ) 30-60 O NE, 5-HT, DA 0/+ 0 + + 0 0/+ + 3+ 0

Tranylcypromine (pa r n at e) 20-30 O NE, 5-HT, DA 2+ 0 0 + 0 0/+ + 2+ 0

()-Selegiline (el d epr yl ) 10 O DA, ?NE, ?5-HT 0 0 0 0 0 0 0 + 0

Note: Most o the drugs are hydrochloride salts, but SURMONTIL and LUVOX are maleates; CELEXA is a hydrobromide, and REMERON is a ree-base. Selegiline is approved or early Parkinson
disease, but may have antidepressant e ects, especially at daily doses 20 mg, and is under investigation or administration by transdermal patch.
a
Both higher and lower doses are sometimes used, depending on an individual patients needs and response to the drug; see the literature and FDA-approved dosage recommendations.
O, oral tablet or capsule; I, injectable; NE, norepinephrine; 5-HT, serotonin, DA, dopamine; 0, negligible; 0/+, minimal; +, mild; 2+, moderate; 3+, moderately severe; 4+, severe.
Other signi cant side e ects or individual drugs are described in the Summary Table.
Psychopharmacology CHAPTER 8

b. Why is there a delay in the onset o antidepressant ef ect with SSRI antidepres-
sants and SNRIs such as venla axine?
SNRIs inhibit both SER and NE . Depending on the drug, the dose, and the
potency at each site, SNRIs cause enhanced serotonergic and/or noradrenergic
neurotransmission. Simi ar to the action o SSRIs, the initia inhibition o SER
induces activation o 5-H 1A and 5-H 1D autoreceptors. T is action decreases sero-
tonergic neurotransmission by a negative eedback mechanism unti these seroto-
nergic autoreceptors are desensitized. T en, the enhanced serotonin concentration
in the synapse can interact with postsynaptic 5-H receptors.
c. What side ef ects might this patient expect with venla axine?
T e side e ects o ven a axine are shown in ab e 8-1. Ven a axine dose reductions
are suggested or patients with rena or hepatic impairment (see ab e 8-2). T e
SNRIs have desirab e sa ety advantages over the CAs. SNRIs have a side e ect
pro e simi ar to that o the SSRIs, inc uding nausea, constipation, insomnia, head-
aches, and sexua dys unction. T e immediate re ease ormu ation o ven a axine
can induce sustained diasto ic hypertension (systo ic b ood pressure > 90 mm Hg
at consecutive week y visits) in 10 to 15% o patients at higher doses; this risk is
reduced with the extended-re ease orm.

CASE 8-3
A 53-year-old man with mild depression has been treated with a monoamine oxidase
inhibitor. His physician is now switching his antidepressant therapy to uoxetine.
a. What drug interactions should he be cautioned about while taking the SSRI?
Most antidepressants, inc uding the SSRIs, exhibit drugdrug interactions based
on their routes o metabo ism (see ab e 8-2). Paroxetine and, to a esser degree,
f uoxetine are potent inhibitors o CYP2D6. T e other SSRIs, outside o f uvox-
amine, are at east moderate inhibitors o CYP2D6. T is inhibition can resu t in dis-
proportionate increases in p asma concentrations o drugs metabo ized by CYP2D6
when doses o these drugs are increased. F uvoxamine direct y inhibits CYP1A2
and CYP2C19; f uoxetine and f uvoxamine a so inhibit CYP3A4. See Chapter 2 or
a ist o drugs metabo ized by these CYPs.
b. He had previously been taking tranylcypromine, an MAO inhibitor. What con-
cerns are there or switching his medication to an SSRI?
Another important drugdrug interaction with SSRIs occurs via a pharmacody-
namic mechanism. MAOIs enhance the e ects o SSRIs due to inhibition o sero-
tonin metabo ism. Administration o these drugs together can produce synergistic
increases in extrace u ar brain serotonin, eading to the serotonin syndrome.
Symptoms o the serotonin syndrome inc ude hyperthermia, musc e rigidity, myoc-
onus, tremors, autonomic instabi ity, con usion, irritabi ity, and agitation; this
can progress toward coma and death. Other drugs that may induce the serotonin
syndrome inc ude substituted amphetamines such as methy enedioxymetham-
phetamine (Ecstasy), which direct y re eases serotonin rom nerve termina s. T e
primary treatment is stopping a serotonergic drugs, administering nonse ective
serotonin antagonists, and supportive measures.
Since current y avai ab e MAOIs bind irreversib y to MAO and b ock the enzy-
matic metabo ism o monoaminergic neurotransmitters, SSRIs shou d not be
started unti at east 14 days o owing discontinuation o treatment with an
MAOI; this a ows or synthesis o new MAO. For a SSRIs but f uoxetine, at east
14 days shou d pass prior to beginning treatment with an MAOI o owing the end
o treatment with an SSRI. Since the active metabo ite o f uoxetine, norf uoxetine,
has a t1/2 o 1 to 2 weeks, at east 5 weeks shou d pass between stopping f uoxetine
and beginning an MAOI.
(Continued)

157
SECTION II Neuropharmacology

TABLE 8-2 Disposition o Antidepressants


ELIMINATION t 1/2, h PARENT PREDOMINANT CYP INVOLVED
DRUG DRUG (Active Metabolite) TYPICAL CP ng/mL IN METABOLISM
Tricyclic Antidepressants

Amitriptyline 16 (30) 100-250

Amoxapine 8 (30) 200-500

Clomipramine 32 (70) 150-500

Desipramine 30 125-300

Doxepin 18 (30) 150-250 2D6, 2C19, 3A3/4, 1A2

Imipramine 12 (30) 175-300

Maprotiline 48 200400

Nortriptyline 31 60-150

Protriptyline 80 100-250

Trimipramine 16 (30) 100-300

Selective Serotonin Reuptake Inhibitors

R,S-Citalopram 36 75-150 3A4, 2C19

S-Citalopram 30 40-80 3A4, 2C19

Fluoxetine 53 (240) 100-500 2D6, 2C9

Fluvoxamine 18 100-200 2D6, 1A2, 3A4, 2C9

Paroxetine 17 30-100 2D6

Sertraline 23 (66) 25-50 2D6

Serotonin Norepinephrine Reuptake Inhibitors

Duloxetine 11 2D6

Venla axine 5 (11) 2D6, 3A4

Other Antidepressants

Atomoxetine 5-20 (child: 3) 2D6, 3A3/4

Bupropion 11 75-100 2B6

Mirtazapine 16 2D6

Ne azodone 24 3A3/4

Reboxetine 12

Trazodone 6 800-1600 2D6

Values shown are elimination t1/2 values or a number o clinically used antidepressant drugs; numbers in parentheses are t1/2 values o active
metabolites. Fluoxetine (2D6), uvoxamine (1A2, 2C8, 3A3/4), paroxetine (2D6), and ne azodone (3A3/4) are potent inhibitors o CYPs; sertraline
(2D6), citalopram (2C19), and venla axine are less potent inhibitors. Plasma concentrations are those observed at typical clinical doses.
In ormation was obtained rom manu acturerssummaries and Appendix II in Goodman and Gilmans The Pharmacological Basis of Therapeutics,
12th Edition, which the reader should consult or important details.

158
Psychopharmacology CHAPTER 8

CASE 8-4
A 49-year-old man is in hospital or surgery on a herniated vertebral disc. On the sec-
ond day ollowing surgery, he becomes agitated, belligerent, paranoid, and aggressive.
He is given haloperidol to control his behavior.
a. What kind o drug is haloperidol and how does it act?
Like most antipsychotic drugs ha operido is an antagonist at D2 receptors (see
Figure 8-2; ab es 8-3 and 8-4).
b. What are the advantages o using a drug such as haloperidol in this setting?
De irium o owing surgery genera y requires on y short-term therapy. Because
anticho inergic drug e ects may worsen de irium and dementia, high-potency typi-
ca antipsychotic drugs (eg, ha operido ) or atypica antipsychotic agents with im-
ited antimuscarinic properties (eg, risperidone) are o en the drugs o choice.
c. While not a consideration in this setting, what are the side ef ects o the chronic
use o haloperidol?
Excessive D2 b ockade, as is o en the case with the use o high-potency typica agents
(eg, ha operido ), not on y increases risk or motor neuro ogica e ects (eg, muscu ar
rigidity, bradykinesia, tremor, akathisia) (see Figure 8-3), but a so s ows mentation
(bradyphrenia), and inter eres with centra reward pathways, resu ting in patient
comp aints o anhedonia. Rare y used are ow-potency typica agents (eg, ch or-
promazine), which a so have high a nities or H 1, M, and 1 receptors that cause
many undesirab e e ects (sedation, anticho inergic properties, orthostasis). Concerns
regarding Q c pro ongation (eg, thioridazine) urther imit their c inica use u ness.
d. What are other options in treating this patient?
Intramuscu ar (IM) administration o ziprasidone, aripiprazo e, or o anzapine repre-
sents an option or treating agitated and minima y cooperative patients, and presents
ess risk o drug-induced parkinsonism than ha operido . Q c pro ongation associated
with intramuscu ar droperido and intravenous administration o ha operido have
curtai ed the use o those particu ar ormu ations. reatment continues unti agitated
or ha ucinatory behaviors are contro ed and the under ying etio ogies are addressed.

CASE 8-5
A 32-year-old woman with a long history o drug abuse has been diagnosed with
schizophrenia.
a. What are the goals o short-term therapy with this patient?
T e immediate goa s o acute antipsychotic treatment are the reduction o agi-
tated, disorganized, or hosti e behavior, decreasing the impact o ha ucinations,
the improvement o organization o thought processes, and the reduction o socia
withdrawa . Doses used are o en higher than those required or maintenance
treatment o stab e patients.
b. What are the goals o long-term therapy with this patient?
T e need or ong-term treatment poses issues a most exc usive y to the chronic
psychotic i nesses, schizophrenia and schizoa ective disorder, a though ong-term
antipsychotic treatment is sometimes used or manic patients, or ongoing psy-
chosis in dementia patients, or l -dopa psychosis, and or adjunctive use in SSRI-
unresponsive major depression.
T e choice o antipsychotic agents or ong-term schizophrenia treatment is based
primari y on avoidance o adverse e ects and, when avai ab e, prior history o
patient response. Since schizophrenia spectrum disorders are i e ong diseases,
treatment acceptabi ity is paramount to e ective i ness management. Whether
atypica antipsychotic agents are superior to typica antipsychotic agents has been
the subject o signi cant and contentious debate.
(Continued)
159
1
TABLE 8-3 Chemical Structures, Dosages or Acute Psychosis and Schizophrenia Maintenance, and Metabolic Risk Prof le a
6
0
ORAL DOSAGE (MG/DAY)
NONPROPRIETARY NAME t r a d e n a me ACUTE PSYCHOSIS MAINTENANCE METABOLIC SIDE EFFECTS

S
DOSAGE FORMS 1ST EPISODE CHRONIC 1ST EPISODE CHRONIC WEIGHT GAIN LIPIDS GLUCOSE

E
C
T
Typical Antipsychotic Agents

I
O
Phenothia zines

N
I
I
Chlorpromazine (t h o r a zin e) O, S, IM 200-600 400-800 150-600 250-750 +++ +++ ++

Perphenazine (t r il a f o n ) O, S, IM 12-50 24-48 12-48 24-60 +/

Tri uoperazine (st el a zin e) O, S, IM 5-30 10-40 2.5-20 10-30 +/

Fluphenazine (pr o l ixin ) O, S, IM 2.5-15 5-20 2.5-10 5-15 +/

N
e
Fluphenazine decanoate Depot IM Not or acute use 5-75 mg/2 wks +/

u
r
o
Other Typical Agents

p
h
a
Molindone (mo ba n ) O, S 15-50 30-60 15-50 30-60

r
m
a
Loxapine (l o xit a n e) O, S, IM 15-50 30-60 15-50 30-60 +

c
o
l
Haloperidol (h a l d o l ) 2.5-10 5-20 2.5-10 5-15 +/

o
g
y
O
OH
F C (CH2 )3 N

Cl
O, S, IM

Haloperidol decanoate Depot IM Not or acute use 100-300 mg/month +/

Atypical Antipsychotic Agents

Aripiprazole (a bil if y) O, S, ODT, IM 10-20 15-30 10-20 15-30 +/

Asenapine (sa ph r is , syc r est ) ODT 10 10-20 10 10-20 +/

Clozapine (c l o za r il , fa zc l o ) O, ODT 200-600 400-900 200-600 300-900 ++++ +++ +++

(Continued )
ORAL DOSAGE (MG/DAY)
NONPROPRIETARY NAME t r a d e n a me ACUTE PSYCHOSIS MAINTENANCE METABOLIC SIDE EFFECTS
DOSAGE FORMS 1ST EPISODE CHRONIC 1ST EPISODE CHRONIC WEIGHT GAIN LIPIDS GLUCOSE
Iloperidone (fa n a pt ) O 12-24b 8-16 + +/ +/

Olanzapine (zypr exa ) O, ODT, IM 7.5-20 10-30 7.5-15 15-30 ++++ +++ +++

Paliperidone (in veg a ) O 6-9 6-12 3-9 6-15 + +/ +/

Paliperidone palmitate (su st en n a )c Depot IM See note c on dosing + +/ +/

Quetiapine (ser o q u el , ser o q u el xr ) O 200-600 400-900 200-600 300-900 + + +/

Risperidone (r isper d a l ) O, S, ODT 2-4 3-6 2-6 3-8 + +/ +/

RISPERDAL CONSTA Depot IM Not or acute use 25-50 mg/2 wks

Sertindole (ser d o l ec t , ser l ec t )d O 4-16 12-20 12-20 12-32 +/

Ziprasidone (g eo d o n , zel d o x )e O, IM 120-160 120-200 80-160 120-200 +/

Dosage Forms: O, tablet; S, solution; IM, acute intramuscular; ODT, orally dissolving tablet.
a
For urther in ormation on antipsychotic dosing in psychotic disorders, see Expert Consensus Panel or Optimizing Pharmacologic Treatment o Psychotic Disorders. The expert consensus
guideline series. Optimizing pharmacologic treatment o psychotic disorders. J Clin Psychiatry, 2003, 64(Suppl 12):297.

P
s
y
Note that doses in rst-episode, younger, or antipsychotic-nave patients are lower than or chronic schizophrenia patients. Dose in elderly schizophrenia patients is approximately 50% o that

c
h
used in younger adults with schizophrenia; dosing or dementia-related psychosis is approximately 25%.

o
Due to orthostasis risk, dose titration o iloperidone is 1 mg bid on day 1, increasing to 2, 4, 6, 8, 10, and 12 mg bid on days 2, 3, 4, 5, 6, and 7 (as needed). Sa ety data exist or daily doses up to

p
b

h
16 mg bid.

a
r
c
Paliperidone palmitate dosing: in acute schizophrenia, deltoid IM loading doses o 234 mg at day 1 and 156 mg at day 8 provide paliperidone levels equivalent to 6 mg oral paliperidone

m
during the rst week, and peaking on day 15 at a level comparable to 12 mg oral paliperidone. No oral antipsychotic needed in rst week. Maintenance IM doses can be given in deltoid or

a
c
gluteus every 4 weeks a ter day 8. Maintenance dose options: 39, 78, 117, 156, or 234 mg every 4 weeks. Failure to give initiation doses (except or those switching rom depot) will result in

o
l
o
subtherapeutic levels or months.

g
Not available in the United States.

y
d

e
Oral dose must be given with ood (500 kcal) to acilitate absorption. Food increases the absorption o single doses o 20-, 40-, and 80-mg capsules by 48%, 87%, and 101%, respectively.

C
H
A
P
T
E
R
8
1
6
1
SECTION II Neuropharmacology

TABLE 8-4 Potencies o Antipsychotic Agents at Neurotransmitter Receptorsa


DOPAMINE SEROTONIN 5HT2A/D2 DOPAMINE MUSCARINIC ADRENERGIC HISTAMINE
D2 5-HT1A 5-HT2A 5-HT2C RATIO D1 D4 M1 1A 2A H1
Typical Agents

Haloperidol 1.2 2100 57 4500 47 120 5.5 >10,000 12 1130 1700

Fluphenazine 0.8 1000 3.2 990 3.9 17 29 1100 6.5 310 14

Thiothixene 0.7 410 50 1360 72 51 410 >10,000 12 80 8

Perphenazine 0.8 420 5.6 130 7.4 37 40 1500 10 810 8.0

Loxapine 11 2550 4.4 13 0.4 54 8.1 120 42 150 4.9

Molindone 20 3800 >5000 10,000 >250 >10,000 >2000 >10,000 2600 1100 2130

Thioridazine 8.0 140 28 53 3.5 94 6.4 13 3.2 130 16

Chlorpromazine 3.6 2120 3.6 16 1 76 12 32 0.3 250 3.1

Atypical Agents

Asenapine b 1.4 2.7 0.1 0.03 0.05 1.4 1.1 >10,000 1.2 1.2 1.0

Ziprasidone 6.8 12 0.6 13 0.1 30 39 >10,000 18 160 63

Sertindole b 2.7 280 0.4 0.90 0.2 12 13 >5000 1.8 640 130

Zotepine b 8.0 470 2.7 3.2 0.3 71 39 330 6.0 210 3.2

Risperidone 3.2 420 0.2 50 0.05 240 7.3 >10,000 5.0 16 20

Paliperidone 4.2 20 0.7 48 0.2 41 54 >10,000 2.5 4.7 19

Iloperidone 6.3 90 5.6 43 0.9 130 25 4900 0.3 160 12

Aripiprazole 1.6 6.0 8.7 22 5.0 1200 510 6800 26 74 28

Sulpiride b 6.4 >10,000 >10,000 >10,000 >1000 >10,000 54 >10,000 >10000 >5000 >10,000

Olanzapine 31 2300 3.7 10 0.1 70 18 2.5 110 310 2.2

Quetiapine 380 390 640 1840 2.0 990 2020 37 22 2900 6.9

Clozapine 160 10 5.4 9.4 0.03 270 24 6.2 1.6 90 1.1


a
Data are averaged Ki. values (nM) rom published sources determined by competition with radioligands or binding to the indicated cloned
human receptors. Data derived rom receptor binding to human or rat brain tissue is used when cloned human receptor data is lacking.
b
Not available in the United States.
NIMH Psychoactive Drug Screening Program (PDSP) Ki Database: http://pdsp.med.unc.edu/pdsp.php (Accessed June 30, 2009).

c. She is prescribed the atypical antipsychotic drug aripiprazole. What is the mech-
anism o action o aripiprazole?
Present y, antipsychotic agents inc ude many di erent chemica structures with a
range o activities at di erent neurotransmitter receptors (eg, 5-H 2A antagonism,
5-H 1A partia agonism). As a resu t, structure- unction re ationships that were
re ied upon in the past have become ess important. Instead, receptor- unction re a-
tionships and unctiona assays are more c inica y re evant. Aripiprazo e represents
a good examp e o how an examination o the structure provides itt e insight into
its mechanism, which is based on dopamine receptor partia agonism. Detai ed
know edge o receptor a nities (see ab e 8-4) and the unctiona e ect at speci c
(Continued)

162
Psychopharmacology CHAPTER 8

A
100 USE OF ANTIPSYCHOTIC
AGENTS IN NONPSYCHOTIC
EP S
80 DISORDERS
5-HT2
y
Anti-psych Anxietydisorders
c
60
n
a
D2 Tourette disorder
p
u
c
c
40 Huntingtons disease
O
%
Autism
20
Antiemeticuse
0
0 2 5 10 20 30 40
Ola nza pine dos e (mg/day)
B
100

EP S
80
5-HT2
Anti-psych
y
60
c
n
D2
a
p
u
c
40
c
O
%
20

0
1 2 4 6 8 10 15
Ris pe ridone dos e (mg/day)

FIGURE 8-3 Receptor occupancy and clinical response or antipsychotic agents. Typically, in D2
receptor occupancy by the drug more than 60% provides antipsychotic e ects, receptor occu-
pancy more than 80% causes extrapyramidal symptoms (EPS). Atypical agents combine weak
D2 receptor blockade with more potent 5-HT2A antagonism/inverse agonism. Inverse agonism
at 5-HT2 receptor subtypes may contribute to the reduced EPS risk o olanzapine (Panel A) and
risperidone (Panel B) and e cacy at lower D2 receptor occupancy (olanzapine, Panel A). Aripipra-
zole is a partial D2 agonist that can achieve only 75% unctional blockade.

receptors (eg, u , partia , or inverse agonism or antagonism) can provide impor-


tant insight into the therapeutic and adverse e ects o antipsychotic agents.
T e reduction in dopaminergic neurotransmission is present y achieved through 1
o 2 mechanisms: D2 antagonism or partia D2 agonism, o which aripiprazo e is the
on y current examp e.
d. What are expected side ef ects o aripiprazole?
Whi e concerns over extrapyramida symptoms (EPS) and tardive dyskinesia have
abated with the introduction o the atypica antipsychotic agents into c inica
practice, there has been increased concern over metabo ic e ects o antipsychotic
treatment (see ab e 8-3): weight gain, dys ipidemia (particu ar y hypertrig yceri-
demia), and an adverse impact on g ucose-insu in homeostasis, inc uding new-
onset type 2 DM, and diabetic ketoacidosis (DKA), with reported ata ities rom
the atter. C ozapine and o anzapine have the highest metabo ic risk and are on y
used as ast resort. O anzapine has been re egated in most treatment a gorithms to
third-tier status and is considered on y a er ai ure o more metabo ica y benign
agents such as aripiprazo e, ziprasidone, asenapine, i operidone, risperidone,
and pa iperidone.

163
SECTION II Neuropharmacology

ADVERSE EFFECTS OF LITHIUM


ORGAN SYSTEM ADVERSE EFFECT
CNS Fine postural hand tremor, incoordination, ataxia, slurred speech,
seizures reported at therapeutic plasma concentrations, central
appetite stimulation with weight gain

Renal Decreased ability to concentrate urine, polydipsia, polyuria, and a


picture o nephrogenic diabetes insipidus

Endocrine Benign, di use, nontender thyroid enlargement in patients


with normal thyroid unction; 7-10% o patients develop overt
hypothyroidism

Cardiovascular Benign and reversible T wave and the appearance o U waves on ECG;
rare reports o e ects on cardiac conduction

Skin Allergic reactions such as dermatitis, olliculitis, and vasculitis;


worsening o other dermatological conditions is common

Reproductive Pregnancy category risk D; risk o cardiovascular abnormalities in


newborn i used early in pregnancy; Li+ reely crosses the placenta
and may cause etal or neonatal Li+ toxicity such as neonatal goiter,
CNS depression, hypotonia, and cardiac murmur

Other Benign increase in circulating polymorphonuclear leukocytes; metallic


taste; worsening o myasthenia gravis

CASE 8-6
A 38-year-old woman is being treated with lithium or bipolar disorder.
a. What is bipolar disorder and how ef ective is Li+ in treating the mania and
depression associated with bipolar disorder?
Patients who experience periods o hypomania and major depression have bipo ar
II disorder, those with mania at any time have bipo ar I, and those with hypomania,
but ess severe orms o depression have cyc othymia.
reatment with Li+ idea y is conducted in patients with norma cardiac and rena
unction. Occasiona y, patients with severe systemic i nesses are treated with Li+,
provided that the indications are compe ing, but the need or diuretics, nonste-
roida anti-inf ammatory agents, or other medications that pose potentia phar-
macokinetic prob ems o en prec udes ithium use in those with mu tip e medica
prob ems. reatment o acute mania and the prevention o recurrences o bipo ar
i ness in adu ts or ado escents are uses o Li+ approved by the FDA.
Whi e Li+, va proate, and carbamazepine have e cacy in acute mania, in c ini-
ca practice these are usua y combined with atypica antipsychotic drugs, even in
manic patients without psychotic eatures, due to their de ayed onset o action. Li+,
carbamazepine, and va proic acid preparations are on y e ective with dai y dosing
that maintains adequate serum eve s, and require serum eve monitoring.
b. What are the options or this patient?
Li+ is e ective in acute mania but is rare y emp oyed as a so e treatment or reasons
noted above, and because 5 to 7 days are required or c inica e ect. T e anticonvu -
sant sodium va proate provides more rapid antimanic e ects than Li+, with thera-
peutic bene t seen within 3 to 5 days. T e most common orm o va proate in use is
diva proex sodium, which is pre erred over va proic acid due to ower incidence o
GI and other adverse e ects.
Carbamazepine is e ective or acute mania. Immediate re ease orms o carbamaze-
pine cannot be oaded or rapid y titrated over 24 hours as with va proate due to the

(Continued)
164
Psychopharmacology CHAPTER 8

deve opment o neuro ogica adverse e ects such as dizziness or ataxia, even within
the therapeutic range (6-12 g/mL); the extended-re ease orm o carbamazepine is
FDA-approved or acute mania.
c. What therapeutic options are there or prophylaxis or uture depressive episodes?
T e choice o ongoing prophy axis is determined by the need or continued anti-
psychotic drug use and or use o a mood-stabi izing agent. Both aripiprazo e and
o anzapine are e ective as monotherapy or mania prophy axis, but o anzapine use
is eschewed out o concern or metabo ic e ects, and aripiprazo e shows no bene t
or prevention o depressive re apse.
T e overriding concern guiding bipo ar treatment is the high recurrence rate. Indi-
vidua s who experience mania have an 80 to 90% i etime risk o subsequent manic
episodes. As with schizophrenia, ack o insight, poor psychosocia support, and sub-
stance abuse a inter ere with treatment adherence. T e anticonvu sants amotrigine,
carbamazepine, and diva proex have data supporting their use in bipo ar prophy axis.
Lamotrigine was e ective in 2 arge, 18-month- ong maintenance tria s or bipo ar
patients whose most recent mood episode was manic or depressed, with greater
e ect on depressive re apse. T e abi ity to provide prophy axis or uture depressive
episodes combined with data in acute bipo ar depression has made amotrigine a
use u choice or bipo ar treatment, given that bipo ar I and II patients spend arge
amounts o time in depressive phases (32 and 50%, respective y).
Bipo ar disorder is a i etime i ness with high recurrence rates. Individua s who
experience an episode o mania shou d be educated about the probab e need or
ongoing treatment. Stopping mood stabi izer therapy can be considered in patients
who have experienced on y 1 i etime manic episode, particu ar y when there may
have been a pharmaco ogica precipitant (eg, substance or antidepressant use), and
who have been euthymic or extended periods. For bipo ar II patients, the impact o
hypomania is re ative y imited, so the decision to recommend pro onged mainte-
nance treatment with a mood stabi izer is based on c inica response and risk:bene t
ratio. Discontinuation o maintenance Li+ treatment in bipo ar I patients carries
a high risk o ear y recurrence and o suicida behavior over a period o severa
months, even i the treatment had been success u or severa years.

CASE 8-7
A 71-year-old man with bipolar disorder has been treated success ully with Li+
since age 40. He now has developed hypertension, mild congestive heart ailure,
and osteoarthritis.
a. What are issues that might arise rom the coadministration o drugs or
these conditions?
T e majority o o der patients on Li+ therapy are those maintained or years on the
medication. Less than 10% o individua s with bipo ar disorder experience their
rst manic episode at age 50 or above. E der y patients requent y take numer-
ous medications or other i nesses and the potentia or contraindications or
drugdrug interactions is substantia . For patients nave to Li+, these issues can
be addressed re ative y easi y prior to commencement o Li+ treatment. T e more
di cu t c inica decision revo ves around switching treatment in stab e patients
who have taken Li+ or years or decades with exce ent c inica response. In addi-
tion, age-re ated reductions in tota body water and creatinine c earance reduce
the sa ety margin or Li+ treatment in o der patients. argeting ower maintenance
serum eve s (0.6-0.8 mEq/L) may reduce the risk o toxicity. As GFR drops be ow
60 mL/min, strong consideration must be given to a search or a ternative agents,
despite ithiums therapeutic advantages. Li+ toxicity occurs more requent y in
e der y patients, in part as the resu t o concurrent use o oop diuretics and angio-
tensin-converting enzyme inhibitors. Anticonvu sants, especia y extended-re ease
diva proex, are a reasonab e a ternative to Li+.
(Continued)
165
SECTION II Neuropharmacology

Li+ is comp ete y tered, and 80% is reabsorbed in the proxima tubu es. Li+ competes
with Na+ or reabsorption, and Li+ retention can be increased by Na+ oss re ated to
diuretic use, or ebri e, diarrhea , or other GI i ness. Heavy sweating eads to a pre er-
entia secretion o Li+ over Na+; however, the rep etion o excessive sweating using ree
water without e ectro ytes can cause hyponatremia, and promote Li+ retention. T iazide
diuretics dep ete Na+ and can cause signi cant reductions in Li+ c earance that resu t
in toxic eve s. T e K+-sparing diuretics triamterene, spirono actone, and ami oride
have modest e ects on the excretion o Li+, with concomitant y sma er increases in
serum eve s. Loop diuretics such as urosemide seem to have imited impact on Li+
eve s. Rena excretion can be increased by administration o osmotic diuretics or
acetazo amide, but not su cient y or the management o acute Li+ intoxication.
T rough a teration o rena per usion, some nonsteroida anti-inf ammatory agents
can aci itate rena proxima tubu ar resorption o Li+ and thereby increase serum con-
centrations. T is interaction appears to be particu ar y prominent with indomethacin,
but a so may occur with ibupro en, naproxen, and COX-2 inhibitors, and possib y ess
so with su indac and aspirin. Angiotensin-converting enzyme inhibitors, particu ar y
the rena y c eared isinopri , a so cause Li+ retention, with iso ated reports o toxicity
among stab e ithium-treated patients switched rom osinopri to isinopri .
b. How should his bipolar disorder be managed in light o his age and coexisting
medical conditions?
Because o the ow therapeutic index or Li+, periodic determination o serum con-
centrations is crucia . Li+ cannot be used with adequate sa ety in patients who cannot
be tested regu ar y. Concentrations considered to be e ective and acceptab y sa e are
between 0.6 and 1.5 mEq/L. T e range o 1.0 to 1.5 mEq/L is avored or treatment
o acute y manic or hypomanic patients. Somewhat ower va ues (0.6-1.0 mEq/L) are
considered adequate and are sa er or ong-term prophy axis. Serum concentrations
o Li+ have been ound to o ow a c ear dose-e ect re ationship between 0.4 and 1.0
mEq/L, but with a corresponding dose-dependent rise in po yuria and tremor as
indices o adverse e ects. Nonethe ess, patients who maintain trough eve s o 0.8 to
1.0 mEq/L experience decreased re apse risk compared to those maintained at ower
serum concentrations. T ere are patients who may do we with serum eve s o 0.5
to 0.8 mEq/L, but there are no current c inica or bio ogica predictors to permit a
priori identi cation o these individua s. Individua ization o serum eve s is o en
necessary to obtain a avorab e risk-bene t re ationship.
T e concentration o Li+ in b ood usua y is measured at a trough o the dai y osci -
ations that resu t rom repetitive administration (ie, rom samp es obtained 10-12
hours a er the ast ora dose o the day). Peaks can be 2 or 3 times higher than trough
eve s at steady state. When the peaks are reached, intoxication may resu t, even when
concentrations in morning samp es o p asma at the dai y nadir are in the acceptab e
range o 0.6 to 1 mEq/L. Sing e dai y doses generate re ative y arge osci ations o
p asma Li+ concentration but ower mean trough eve s than with mu tip e dai y dos-
ing and are associated with a reduction in the extent and risk or po yuria; moreover,
sing e night y dosing means that peak serum eve s occur during s eep, so comp aints
regarding CNS adverse e ects are minimized. Whi e re ative y uncommon, GI com-
p aints are a compe ing reason or mu tip e dai y dosing or using de ayed re ease Li+
preparations, bearing in mind the increased po yuria risk rom these strategies.

CASE 8-8
A 28-year-old woman is taking lithium or management o the mania phase o bipolar
disorder. She has entered the depressive phase o the disorder and takes an overdose o
her lithium.
a. What are the acute toxic ef ects o lithium overdose?
T e occurrence o toxicity is re ated to the serum concentration o Li+ and its rate
o rise o owing administration. Acute intoxication is characterized by vomiting,
(Continued)
166
Psychopharmacology CHAPTER 8

pro use diarrhea, coarse tremor, ataxia, coma, and convu sions. Symptoms o mi der
toxicity are most ike y to occur at the absorptive peak o Li+ and inc ude nausea,
vomiting, abdomina pain, diarrhea, sedation, and ne tremor. T e more serious
e ects invo ve the nervous system and inc ude menta con usion, hyperref exia,
gross tremor, dysarthria, seizures, and crania nerve and oca neuro ogica signs,
progressing to coma and death. Sometimes both cognitive and motor neuro ogica
damage may be irreversib e, with persistent cerebe ar tremor being the most com-
mon. Other toxic e ects are cardiac arrhythmias, hypotension, and a buminuria.
b. What is the treatment or acute overdose o lithium?
T ere is no speci c antidote or Li+ intoxication, and treatment is supportive (see
Chapter 3), inc uding intubation i indicated, and continuous cardiac monitoring.
Leve s greater than 1.5 mEq/L are considered toxic, but inpatient medica admission
is usua y not indicated (in the absence o symptoms) unti eve s exceed 2 mEq/L.
Care must be taken to assure that the patient is not Na+- and water-dep eted. Dia ysis
is the most e ective means o removing Li+ and is necessary in severe poisonings,
that is, in patients exhibiting symptoms o toxicity or patients with serum Li+ con-
centrations more than or equa to 3 mEq/L in acute overdoses. A review o 213 case
reports o acute Li+ toxicity between 1948 and 1984 ound that comp ete recovery
occurred with an average maxima eve o 2.5 mEq/L, permanent neuro ogica
symptoms with mean eve s o 3.2 mEq/L, and death with mean maxima eve s o
4.2 mEq/L. T e most common neuro ogica seque ae are due to cerebe ar damage,
and mani est c inica y as ataxia, tremor, dysarthria, and dysmetria.

KEY CONCEPTS
Inhibition o monoamine reuptake is a major mechanism by which antidepres-
sant drugs enhance neurotransmission (see Figure 8-1).
Monoamine oxidase inhibitor (MAOI) antidepressants inhibit monoamine
metabolism (see Figure 8-1).
With antidepressant drugs there is a therapeutic lag lasting 3 to 4 weeks
be ore a measurable response becomes evident.
Sudden withdrawal o antidepressants can precipitate a withdrawal syndrome.
CAs are potent antagonists at histamine H 1 and muscarinic receptors that
account or many o their side e ects (see Chapters 6 and 21).
MAOIs have many serious ood and drug interactions.
All antipsychotic drugs reduce dopaminergic neurotransmission by D2 block-
ade (or in the case o aripiprazole modulation o DA activity by virtue o being
a partial D2 agonist).
A Parkinson-like syndrome, a risk o long-term therapy with antipsychotic
drugs that is dose- and drug-dependent, can be predicted based on receptor
occupancy (see Figure 8-3).
Alteration o a patients metabolic prof le (see able 8-3) is also a serious side
e ect o some antipsychotic medications.
Bipolar disorder is generally treated with Li+; the anticonvulsants valproic acid and
carbamazepine, and antipsychotic drugs are use ul to manage the acute manic phase.
T e narrow therapeutic indices or Li+, valproic acid, and carbamazepine
requires care ul monitoring o their plasma concentrations.

SUMMARY QUIZ

QUESTION 8-1 A 56-year-old man who has a 30-year history o smoking cigarettes is
being treated or schizophrenia with clozapine. He is hospitalized or an acute exac-
erbation o his psychoses; his clozapine therapy is continued. During the third week

167
SECTION II Neuropharmacology

o his hospital stay, he has a seizure that is thought to be due to clozapine toxicity. T e
clozapine toxicity in this patient is likely due to
a. increased GI absorption o clozapine.
b. decreased renal excretion o clozapine.
c. a decrease in his blood-brain barrier unction.
d. decreased metabolism o clozapine.
e. a pharmacy mistake.
QUESTION 8-2 A 54-year-old woman is taking Li+ or bipolar disorder. Her Li+ plasma
concentrations have consistently been 0.8 mEq/L (therapeutic range 0.6-1.5 mEq/L). She
has developed osteoarthritis or which she is prescribed ibupro en. One week later dur-
ing a routine visit it is noted that her Li+ plasma concentration is 1.5 mEq/L. T e cause
o the rise in this patients plasma Li+ concentration is due to the e ect o ibupro en to
a. increase Li+ GI absorption.
b. acilitate Li+ reabsorption in the renal proximal tubule.
c. decrease Li+ metabolism in the liver.
d. displace Li+ rom serum albumin.
e. acilitate Li+ reabsorption rom the distal colon.
QUESTION 8-3 A 22-year-old woman is being treated with amitriptyline or depres-
sion. She, and her amily, should be cautioned about not seeing a therapeutic e ect or
a. 24 hours.
b. 12 hours.
c. 3 days.
d. 1 week.
e. 3 to 4 weeks.
QUESTION 8-4 A 75-year-old man with obsessive compulsive disorder is treated with
risperidone. A er 3 weeks o therapy, he develops bradykinesia, masked acies, and
reduced arm movements when walking. T ese symptoms are due to
a. 1-adrenergic receptor antagonism.
b. -adrenergic receptor antagonism.
c. 5-H receptor stimulation.
d. D2 receptor antagonism.
e. blockade o norepinephrine uptake into presynaptic terminals.
QUESTION 8-5 An 86-year-old man with severe dementia has become very aggressive
and poses a danger to himsel and his amily. His geriatrician is reluctant to prescribe
an antipsychotic medication to control his behavior because o the risk o
a. increased mortality.
b. increased dementia.
c. decreased peripheral blood per usion.
d. skin cancer.
e. glaucoma.
QUESTION 8-6 A 43-year-old woman is being treated with olanzapine or schizophrenia.
Although olanzapine appeared to be improving her behavioral symptoms, she stopped
using the drug likely due to
a. an unusual taste.
b. increased libido.
c. weight gain.
d. her urine turned green.
e. hirsutism.
168
Psychopharmacology CHAPTER 8

QUESTION 8-7 A 32-year-old woman is treated or schizophrenia with clozapine. Her


physician insists on regular monitoring o
a. serum aminotrans erases.
b. serum Na+.
c. serum K+.
d. serum Ca++.
e. complete blood count (CBC).
QUESTION 8-8 A 43-year-old man with the diagnosis o bipolar disorder has devel-
oped a particularly severe orm o mania with psychotic symptoms. T e patients physi-
cian should begin treatment with which o the ollowing anticonvulsants?
a. Phenytoin
b. Gabapentin
c. Valproic acid
d. Lamotrigine
e. opiramate

SUMMARY QUIZ ANSWER KEY


QUESTION 8-1 Answer is d. Changes in smoking status can be especially problem-
atic or clozapine-treated patients and will alter serum levels by 50% or more. Within 2
weeks o smoking discontinuation (eg, hospitalization in nonsmoking environment), the
absence o aryl hydrocarbons will cause upregulated CYP1A2 activity to return to base-
line levels, with a concomitant rise in serum clozapine concentrations (see able 8-5).
QUESTION 8-2 Answer is b. T rough alteration o renal per usion, some nonsteroidal
anti-in ammatory agents can acilitate renal proximal tubular resorption o Li+ and
thereby increase serum concentrations.
QUESTION 8-3 Answer is e. Following initiation o antidepressant drug treatment,
there is generally a therapeutic lag lasting 3 to 4 weeks be ore a measurable therapeu-
tic response becomes evident (see Case 8-1).
QUESTION 8-4 Answer is d. With the exception o the D2 partial agonist aripipra-
zole, all other antipsychotic agents possess D2 antagonist properties, the strength o
which determines the likelihood or extrapyramidal syndrome (EPS), akathisia, long-
term tardive dyskinesia risk, and hyperprolactinemia. T e mani estations o EPS are
described in able 8-6, along with the usual treatment approach (also see Figure 8-3).
Parkinsonism resembling its idiopathic orm occurs when striatal D2 occupancy
exceeds 78%, and o en responds to dose reduction or switching to an antipsychotic
with weaker D2 antagonism. In situations where this is neither possible nor desirable,
anti-parkinsonian medication may be employed. Clinically, there is a generalized slow-
ing and impoverishment o volitional movement (bradykinesia) with masked acies
and reduced arm movements during walking. T e syndrome characteristically evolves
gradually over days to weeks as the risk o acute dystonia diminishes. T e most notice-
able signs are slowing o movements, and sometimes rigidity and variable tremor at
rest, especially involving the upper extremities. Bradykinesia and masked acies may be
mistaken or clinical depression. Elderly patients are at greatest risk.
QUESTION 8-5 Answer is a. Perhaps the least understood adverse e ect is the
increased risk or cerebrovascular events and all-cause mortality among elderly demen-
tia patients exposed to antipsychotic medications. All antipsychotic agents carry a
mortality warning in the drug label regarding their use in dementia patients. T e
cerebrovascular adverse event rates in 10-week dementia trials range rom 0.4 to 0.6%
or placebo to 1.3 to 1.5% or risperidone, olanzapine, and aripiprazole. T e mortality

(Continued)
169
SECTION II Neuropharmacology

TABLE 8-5 Metabolism o Common Antipsychotic Drugs


AGENT METABOLIC PATHWAYS EFFECT OF CYP INHIBITION EFFECT OF CYP INDUCTION
Atypical Antipsychotic Agents

Aripiprazole 2D6 and 3A4 convert aripiprazole to active 2D6 PMs experience 80% in 3A4 induction maximum
metabolite dehydro-aripiprazole. Metabolite aripiprazole AUC, and 30% in concentration and AUC o
has longer t1/2 (75 vs. 94 hours) and represents metabolite AUC (net e ect is aripiprazole and metabolite
40% o AUC at steady state. 60% in AUC or active moiety). by 70%.
Aripiprazole t1/2 146 hrs in PM.
2D6 inhibitors aripiprazole
AUC by 112% and metabolite
AUC by 35%.
Ketoconazole (a potent 3A4
inhibitor) with a 15-mg single
dose o aripiprazole the AUC
o aripiprazole and its active
metabolite by 63% and
77%, respectively.

Asenapine Primarily glucuronidation (UGT 1A4), and Fluvoxamine, 25 mg twice daily or Smoking had no e ect on
limited oxidation via CYP 1A2, and to a lesser 8 days, Cmax by 13% and AUC 29%. clearance or other kinetic
extent 2D6 and 3A4. No active metabolites. Paroxetine both Cmax and AUC by parameters. Carbamazepine
13%. Valproate, a UGT 1A4 inhibitor, both Cmax and AUC by 16%.
Cmax 2%, and AUC 1%.

Clozapine Multiple enzymes convert clozapine to active Fluvoxamine Cp 5-10 old. 2D6 Loss o smoking-related 1A2
metabolite N-desmethylclozapine. The mean inhibition may levels as induction serum levels by
contributions o CYPs 1A2, 2C19, 3A4, 2C9, much as 100%. 50%. Carbamazepine clozapine
and 2D6 are 30%, 24%, 22%, 12%, and 6%, levels on average by 50%.
respectively. CYP1A2 is the most important
orm at low concentrations, which is in
agreement with clinical ndings.

Iloperidone 2D6 and 3A4 convert iloperidone to active Ketoconazole AUC o a single Impact o 3A4 inducers
metabolites P88 and P95. In 2D6 EM, the t1/2 o 3-mg iloperidone dose and its not documented.
P88 and P95 are 26 and 23 hours, respectively; metabolites P88 and P95 by 57%,
in PM, 37 and 31 hours, respectively. Only 55%, and 35%, respectively.
P88 has a nity or D2. P88 accounts or 19.5% Fluoxetine 3-mg single dose AUC
and 34.0% o total exposure in EM and PM, o iloperidone and P88 metabolite
respectively. P95 has Ki o 3.91 nM or 5HT2A and 2-3 old, and P95 AUC by 50%.
4.7 nM or 1A, and accounts or Paroxetine AUC o iloperidone
48% and 25% o total exposure in EM and and P88 metabolite 1.6- old, and
PM, respectively. reduces P95 AUC by 50%. Paroxetine
(8-12 mg twice daily) steady state
Cmax o iloperidone and P88 by 1.6-
old, and steady state Cmax o P95
by 50%. Combined use steady
state Cmax o iloperidone and P88
by 1.4- old, and steady state Cmax
o P95 1.4- old.

Olanzapine Direct glucuronidation or 1A2 mediated Increase in olanzapine Cmax Carbamazepine use clearance
oxidation to N-desmethylolanzapine (inactive). ollowing uvoxamine is 54% in by 50%. Olanzapine Cp lower in
emale nonsmokers and 77% in smokers (with equal dosing).
male smokers. The mean increase
in olanzapine AUC is 52% and
108%, respectively.

Paliperidone 59% excreted unchanged in urine, 32% Unlikely to have much o an e ect. Carbamazepine use steady state
excreted as metabolites. Phase 2 metabolism Cmax and AUC by 37%.
accounts or no more than 10%.

170
Psychopharmacology CHAPTER 8

AGENT METABOLIC PATHWAYS EFFECT OF CYP INHIBITION EFFECT OF CYP INDUCTION


Quetiapine 3A4 mediated sul oxidation to active metabolite Ketoconazole (200 mg once daily Phenytoin increases clearance
norquetiapine, t1/2 12 hours. Steady state mean or 4 days), oral clearance o 5- old.
Cmax and AUC o norquetiapine are ~25% and quetiapine by 84%, resulting in a
~50% o that or quetiapine. 335% in maximum Cp.

Risperidone 2D6 converts risperidone to active metabolite Fluoxetine and paroxetine In a drug interaction study
9-OH risperidone. risperidone concentration ~ 2.5 old o risperidone 6 mg/day 3
In 2D6 PMs, hal -lives are: risperidone, 20 hours; and 3-9 old, respectively. Fluoxetine weeks, ollowed by 3 weeks o
9-OH risperidone, 30 hours did not a ect 9-OH risperidone carbamazepine, concentration o
conc., but paroxetine lowered 9-OH active moieties (risperidone + 9-OH
risperidone (13%). Net e ect: 2D6 risperidone) was decreased 50%.
inhibition levels o active moiety
up to 75%.

Ziprasidone 3A4 (~1/3) Aldehyde Oxidase (~2/3) Concomitant ketoconazole AUC Carbamazepine AUC by 35%.
by 35%

Haloperidol Multiple CYP pathways, particularly 2D6, Hal -li e prolonged in CYP 2D6 PMs Carbamazepine and phenytoin
3A4, and minor pathway 1A2. Only active Individuals with only one unctional haloperidol clearance ~32%, with
metabolite, reduced haloperidol ( ormed 2D6 allele experience 2- old greater Cp (mean, 47%). Discontinuation
by ketone reductase). Reduced haloperidol trough serum levels, those with no o carbamazepine Cp 2-3 old.
inhibits CYP2D6 and may be re-oxidized unctioning alleles 3-4 old higher.
to the parent drug.
Therapeutic serum levels not well de ned; 5-20
ng/mL used as a target or dosing.

Chlorpromazine CYP2D6. Over 10 identi ed human metabolites, Case report o uoxetine- 3A4/PGP inducers (eg,
most inactive. chlorpromazine interaction, but no phenobarbital, carbamazepine)
Chlorpromazine is a moderate 2D6 inhibitor, serum level data on extent o e ect. decrease chlorpromazine
and also induces its own metabolism. Levels levels by ~35%.
drop 25-33% during weeks 1-3 o treatment. Carbamazepine discontinuation
Cp (30-80%).

AUC, area under the curve; PM, poor metabolizer; EM, extensive metabolizer; t1/2, hal li e; Cp , plasma concentration; Cmax, maximum plasma
concentration.
a
May have multiphasic elimination with much longer terminal t1/2.

warning indicates a 1.6 to 1.7- old increased mortality risk or drug versus placebo.
Mortality is due to heart ailure, sudden death, or pneumonia. T e underlying etiol-
ogy or antipsychotic-related cerebrovascular and mortality risk is unknown, but the
f nding o virtually equivalent mortality risk or typical agents compared to atypical
antipsychotic drugs (including aripiprazole) suggests an impact o reduced D2 signaling
regardless o individual antipsychotic mechanisms.
QUESTION 8-6 Answer is c. Weight gain is a signif cant problem during long-term use
o antipsychotic drugs and represents a major barrier to medication adherence, as well
as a signif cant threat to the physical and emotional health o the patient (see able 8-3).
Weight gain has e ectively replaced concerns over EPS as the adverse e ect causing
the most consternation among patients and clinicians alike. Appetite stimulation is the
primary mechanism involved, with little evidence to suggest that decreased activity (due
to sedation) is a main contributor to antipsychotic-related weight gain. Recent animal
studies indicate that medications with signif cant H 1 antagonism induce appetite stimu-
lation through e ects at hypothalamic sites. T e low-potency phenothiazine chlorprom-
azine and the atypical antipsychotic drugs olanzapine and clozapine are the agents o
highest risk, but weight gain o some extent is seen with nearly all antipsychotic drugs,
partly related to the act that acutely psychotic patients may lose weight; in placebo-
controlled acute schizophrenia trials, the placebo cohort inevitably loses weight. For
clozapine and olanzapine, massive weight gains o 50 kg or more are not uncommon,
(Continued)

171
SECTION II Neuropharmacology

TABLE 8-6 Neurological Side E ects o Antipsychotic Drugs


TIME OF ONSET
REACTION FEATURES AND RISK INFO PROPOSED MECHANISM TREATMENT
Acute dystonia Spasm o muscles o Time: 1-5 days. Young, Acute DA antagonism Anti-parkinsonian
tongue, ace, neck, back antipsychotic nave agents are diagnostic
patients at highest risk and curative a

Akathisia Subjective and objective Time: 5-60 days Unknown Reduce dose or change
restlessness; not anxiety drug; clonazepam,
or agitation propranolol more e ective
than anti-parkinsonian
agentsb

Parkinsonism Bradykinesia, rigidity, Time: 5-30 days. Elderly at DA antagonism Dose reduction;
variable tremor, mask greatest risk change medication;
acies, shuf ing gait anti-parkinsonian agentsc

Neuroleptic malignant Extreme rigidity, Time: weeks-months. DA antagonism Stop antipsychotic


syndrome ever, unstable BP, Can persist or days a ter immediately; supportive
myoglobinemia; stopping antipsychotic care; dantrolene and
can be atal bromocriptine d

Perioral tremor Perioral tremor (may Time: months or years o Unknown Anti-parkinsonian agents
(rabbit syndrome) be a late variant o treatment o ten help c
parkinsonism)

Tardive dyskinesia Oro acial dyskinesia; Time: months, years o Postsynaptic DA Prevention crucial;
rarely widespread treatment. Elderly at 5- old receptor supersensitivity, treatment unsatis actory.
choreoathetosis greater risk. Risk potency up-regulation May be reversible with
or dystonia o D2 blockade early recognition and drug
discontinuation
a
Treatment: diphenhydramine 25-50 mg IM, or benztropine 1-2 mg IM. Due to long antipsychotic t1/2, may need to repeat, or ollow with
oral medication.
b
Propranolol o ten e ective in relatively low doses (20-80 mg/day in divided doses). 1-Selective adrenergic receptor antagonists are
less e ective.
Non-lipophilic adrenergic antagonists have limited CNS penetration and are o no bene t (eg, atenolol).
c
Use o amantadine avoids anticholinergic e ects o benztropine or diphenhydramine.
d
Despite the response to dantrolene, there is no evidence o abnormal Ca2+ transport in skeletal muscle; with persistent antipsychotic e ects
(eg, long-acting injectable agents), bromocriptine may be tolerated in large doses (10-40 mg/day). Anti-parkinsonian agents are not e ective.

and mean annual weight gains o 13 kg are reported in schizophrenia clinical trials, with
20% o subjects gaining more than or equal to 20% o baseline weight.
QUESTION 8-7 Answer is e. Clozapine possesses a host o unusual adverse e ects
aside rom seizure induction, the most concerning o which is agranulocytosis.
Clozapines introduction in the United States was based on its e cacy in re ractory
schizophrenia, but came with FDA-mandated CBC monitoring that is overseen by
industry-created registries. Now that several generic orms o clozapine are available in
addition to proprietary CLOZARIL, clinicians must veri y with each manu acturer the
history o prior exposure. T e overall agranulocytosis incidence is slightly under 1%,
with highest risk during the initial 6 months o treatment, peaking at months 2 to 3 and
diminishing rapidly therea er. T e mechanism is immune-mediated, and patients who
have verif able clozapine-related agranulocytosis should not be rechallenged. Increased
risk is associated with certain HLA types and advanced age. An extensive algorithm
guiding clinical response to agranulocytosis, and lesser orms o neutropenia, is avail-
able rom manu acturer Web sites, and must be ollowed, along with the current rec-
ommended CBC monitoring requency.

172
Psychopharmacology CHAPTER 8

QUESTION 8-8 Answer is c. T e pharmacology and chemistry o the anticonvulsants


with signif cant data or acute mania (valproic acid compounds, carbamazepine) and or
bipolar maintenance (lamotrigine) are covered extensively in Chapter 12. T ese com-
pounds are o diverse chemical classes, but share the common property o unctional
blockade o voltage-gated Na+ channels, albeit with di ering binding sites. Valproate
exhibits non-specif c binding to voltage-gated Na+ channels, while carbamazepine (and
its congeners) and lamotrigine have specif c high a nity or the open-channel conf gu-
ration o the alpha subunit o voltage-gated Na+ channels. T ese anticonvulsants have
varying a nities or voltage-dependent Ca2+ channels, and di er in their ability to acili-
tate GABA-ergic (valproate) or inhibit glutamatergic neurotransmission (lamotrigine).
T e extent to which any o these actions is necessary or antimanic or other mood sta-
bilizing activity is unknown, but the ailure o phenytoin, gabapentin, and topiramate to
be e ective antimanic and mood-stabilizing medications suggests that potent blockade
o voltage-gated Na+ channels (which gabapentin and topiramate lack) is necessary but
not su cient, since phenytoin is very active at these channels.
While Li+, valproate, and carbamazepine have e cacy in acute mania, in clinical
practice these are usually combined with atypical antipsychotic drugs, even in manic
patients without psychotic eatures, due to their delayed onset o action.
Li+ is e ective in acute mania, but is rarely employed as a sole treatment or rea-
sons noted above, and because 5 to 7 days are required or clinical e ect. T e anti-
convulsant sodium valproate provides more rapid antimanic e ects than Li+, with
therapeutic benef t seen within 3 to 5 days.
Lamotrigine has no role in acute mania due to the slow, extended titration neces-
sary to minimize risk o Stevens-Johnson syndrome.

SUMMARYTABLE PSYCHOPHARMACOLOGICAL DRUGS


TOXICITIES
CLASS AND SUBCLASSES NAMES CLINICAL USES COMMON UNIQUE: CLINICALLY IMPORTANT
Monoamine Oxidase Tranylcypromine Used to treat depression See Table 8-1 Hypertensive crisis resulting rom
Inhibitors (MAOIs) Phenelzine Selegiline is also used to treat oods containing tyramine
Isocarboxazid Parkinsons disease (see Drug interactionsopioid
Selegiline Chapter 13) agonist and SSRIs and SNRIs may
cause serotonin syndrome and
other antidepressants such as
TCAs and buproprion should be
avoided

Serotonin, Amitriptyline Used to treat depression Blurred vision, dry mouth, Quinidine-like e ects on cardiac
Norepinephrine Reuptake Doxepin tachycardia, di culty conduction
Inhibitors (SNRIs) Imipramine urinating, constipation, Lowering o seizure threshold
Tertiary Amine Tricyclics Clomipramine and sedation Orthostatic hypotension
(TCAs) Trimipramine Weight gain (see rom antagonism o
Table 8-1) 1-adrenergic receptors

Serotonin, Amoxapine Used to treat depression Blurred vision, dry mouth, Quinidine-like e ects on cardiac
Norepinephrine Reuptake Desipramine tachycardia, di culty conduction
Inhibitors (SNRIs) Maprotiline urinating, constipation, Lowering o seizure threshold
Secondary Amine Nortriptyline and sedation Orthostatic hypotension
Tricyclics (TCAs) Protriptyline Weight gain (see rom antagonism o
Table 8-1) 1-adrenergic receptors

173
SECTION II Neuropharmacology

TOXICITIES
CLASS AND SUBCLASSES NAMES CLINICAL USES COMMON UNIQUE: CLINICALLY IMPORTANT

Selective Serotonin Fluoxetine E ective in treating depression, Insomnia, increased Abrupt withdrawal may
Reuptake Inhibitors Fluvoxamine generalized anxiety, panic anxiety, irritability, and precipitate dizziness, headache,
(SSRIs) Paroxetine social anxiety, and obsessive decreased libido nervousness, nausea, and
Sertraline compulsive disorder (OCD) (see Table 8-1) insomnia
Citalopram Fluvoxamine is approved or This withdrawal may be more
Escitalopram OCD but not or depression intense or paroxetine
Citalopram is approved or Paroxetine is associated with an
use in premenstrual increase risk o congenital cardiac
dysphoric disorder mal ormations

Serotonin- Venla axine Treatment o depression, Nausea, constipation, The immediate release orm o
Norepinephrine Reuptake Desvenla axine anxiety disorders, and insomnia, headache, and venla axine can induce sustained
Inhibitors (SNRIs) Duloxetine bromyalgia and neuropathic sexual dys unction diastolic hypertension
Milnacipran pain (duloxetine) (see Table 8-1) Abrupt withdrawal o venla axine
Milnacipran is approved only may precipitate a withdrawal
or bromyalgia pain syndrome

Atypical Antidepressants Atomoxetine Approved or attention de cit Insomnia, nausea, Suicide-related events, emotional
hyperactivity disorder in constipation, headache, lability
children, adolescents, and erectile dys unction Contraindicated in patients with
adults (see Table 8-1) symptomatic cardiovascular
disease

Trazodone E ective antidepressants Somnolence, weight gain, Trazodone is associated with


Ne azodone increased appetite priapism
Mirtazapine Mirtazapine and Mirtazapine is associated with
Mianserin mianserin are quite agranulocytosis
sedating (see Table 8-1) Ne azodone withdrawn rom
market a ter rare cases o liver
ailure; generic ne azodone still
available (see Table 8-1)

Bupropion Used to treat depression, See Table 8-1 Seizures at high doses
prevention o seasonal (see Table 8-1)
depressive disorder, and as a
smoking cessation treatment

Reboxetine Used or the treatment o Insomnia, dry mouth, Glaucoma


depression constipation Suicidal ideation
Aggressive behavior

Anxiolytic Drugs See Chapter 9 See Chapter 9 See Chapter 9 See Chapter 9
(Benzodiazepines)

Anxiolytic Drugs (Not Buspirone Used to treat general anxiety Dizziness Contraindicated in patients with
benzodiazepines) disorder Somnolence severe renal or liver impairment
and in patients with symptomatic
cardiovascular disease

Typical Antipsychotic Chlorpromazine Low-potency drugs used or Anticholinergic Lowers seizure threshold
DrugsPhenothiazines Perphenazine the treatment o acute and symptoms o dry (see Tables 8-3 and 8-6)
Tri uoperazine chronic psychoses, including mouth, blurred vision,
Fluphenazine schizophrenia constipation, and urinary
retention

174
Psychopharmacology CHAPTER 8

TOXICITIES
CLASS AND SUBCLASSES NAMES CLINICAL USES COMMON UNIQUE: CLINICALLY IMPORTANT

Other Typical Molindone High-potency drugs used or Anticholinergic Extrapyramidal symptoms at


Antipsychotic Drugs Loxapine the treatment o acute and symptoms o dry doses that exceed 80% receptor
Haloperidol chronic psychoses, including mouth, blurred vision, occupancy
Droperidol schizophrenia constipation, and urinary (see Figure 8-3)
retention Haloperidol is associated with
prolonged QT interval
Metabolic side e ects
(see Table 8-3)
Neuroleptic malignant syndrome
(NMS)

Atypical Antipsychotic Aripiprazole Used or the treatment o acute Weight gain, anxiety, Extrapyramidal symptoms at
Drugs Olanzapine and chronic psychoses insomnia doses that exceed 80% receptor
Quetiapine Most antipsychotics (except occupancy (see
Risperidone quetiapine) are ine ective Figure 8-3) Metabolic side e ects
Asenapine as monotherapy or bipolar (see Table 8-3)
Clozapine depression
Ziprasidone
Paliperidone
Iloperidone
Sertindole

Anticonvulsants Used to Valproic acid Used to treat acute mania See Chapter 12 See Chapter 12
Treat Mania Carbamazepine (valproic acid, carbamazepine)
Lamotrigine and or bipolar maintenance
(lamotrigine) (see Chapter 12)

Lithium Lithium Treatment o bipolar disorder See Side Bar ADVERSE See Side Bar ADVERSE EFFECTS
carbonate Also used or augmentation o EFFECTS OF LITHIUM OF LITHIUM
Lithium citrate unipolar depression in patients Fine postural hand tremor Low therapeutic index and
who are inadequate responders Ataxia, slurred speech, requires periodic measurement o
to antidepressant therapy and incoordination serum concentrations
Weight gain Seizures reported at therapeutic
concentrations
Nephrogenic diabetes insipidus

175
CHAPTER

9 Hypnotics, Sedatives, and Ethanol


Chapter 9 Hypnotics, Sedatives, and Ethanol is a combination o Chapter 17, Hypnotics
DRUGS INCLUDED
and Sedatives and Chapter 23, Ethanol and Methanol rom Goodman and Gilmans T e
IN THIS CHAPTER Pharmacological Basis of T erapeutics, 12th Edition. An understanding o the material
Acamprosate (CAMPRAL) in those chapters will be help ul in ollowing the material presented in this chapter. In
Alprazolam(XANAX) addition to the material presented here, the above chapters in the 12th Edition include:
Amobarbital (AMYTAL) A detailed discussion o the pharmacological properties o benzodiazepines, includ-
Butabarbital (BUTISOL, others) ing animal models o anxiety, the e ects o benzodiazepines on the electroencepha-
logram (EEG) and sleep stages, the molecular targets o benzodiazepines, and
Carisoprodol (SOMA) GABAA receptormediated electrical events
Clomethiazolenot approved in the able 17-1 Benzodiazepines: Names and Structures
United States
A discussion o miscellaneous sedative hypnotic drugs used less commonly today
Chloral Hydrate (NOCTEC)
A detailed discussion o the management o insomnia
Chlorazepate (TRANXENE, others)
T e history o ethanol consumption details o the pharmacological e ects o ethanol
Chlordiazepoxide (LIBRIUM, others)
Clonazepam(KLONIPIN) In addition, the mechanisms o action o methanol and acute methanol poisoning,
including treatment with ethanol and omepizole are covered in this book Chapter 3
Diazepam(VALIUM, others) Clinical and Environmental oxicity.
Disul ram(ANTABUSE)
Estazolam(PROSOM, others) LEARNING OBJECTIVES
Eszopiclone (LUNESTA) Know the pharmacological e ects, mechanisms o action, untoward e ects, and
Etomidate (AMIDATE) therapeutic uses o benzodiazepines.
Flumazenil (ROMAZICON) Know the pharmacological e ects, mechanisms o action, untoward e ects, and
Flurazepam(DALMANE, others) therapeutic uses o barbiturates.
Lorazepam(ATIVAN) Understand the management o insomnia.
Mephobarbital (MEBERAL) Know the e ects o ethanol on physiological systems.
Meprobamate Describe the issues o physical dependence and tolerance o chronic ethanol use.
Methohexital (BREVITAL) Understand the genetics o ethanol disposition.
Midazolam(VERSED) Know the drugs used to treat alcoholism.
Nalme ene (REVEX)
Naltrexone (REVIA, VIVITROL)
Oxazepam(SERAX)
Paraldehydeno longer used in the MECHANISMS OF ACTION OF SEDATIVES, HYPNOTICS, AND DRUGS USED TO
United States TREAT ALCOHOLISM
Pentobarbital (NEMBUTAL) DRUG CLASS DRUG MECHANISM OF ACTION
Phenobarbital (LUMINAL, others) Benzodiazepines Alprazolam Figure 9-1 shows the benzodiazepine binding site
Propo ol (DIPRIVAN) Chlordiazepoxide on the GABAA receptor
Clonazepam Unlike barbiturates, benzodiazepines do
Quazepam(DORAL) Chlorazepate not activate GABAA receptors directly; rather
Ramelteon (ROZEREM) Diazepam benzodiazepines act allosterically by modi ying
Estazolam the e ects o GABA
Secobarbital (SECONAL)
Flurazepam Benzodiazepines and related compounds can
Temazepam(RESTORIL) Lorazepam act as agonists, antagonists, or inverse agonists
Thiopental (PENTOTHAL) Midazolam Agonists at the binding site increase, and
Oxazepam inverse agonists decrease, the amount o
Triazolam(HALCION) Quazepam chloride current generated by GABAA receptor
Zaleplon (SONATA) Temazepam activation
Triazolam
Zolpidem(AMBIEN)

176
Hypnotics, Sedatives, and Ethanol CHAPTER 9

DRUG CLASS DRUG MECHANISM OF ACTION


Novel Zolpidem Zolpidem is a nonbenzodiazepine that acts as an
Benzodiazepine agonist on GABAA receptors
Receptor Agonists
Zaleplon Zaleplon is a nonbenzodiazepine
that pre erentially binds to the benzodiazepine-
binding site on GABAA receptors containing the
1 receptor subunit

Eszopiclone Eszopiclone is believed to act by enhancing


GABAA receptor unction

Benzodiazepine Flumazenil Flumazenil binds with high a nity to speci c sites


Receptor Antagonist on the GABAA receptor, where it competitively
antagonizes the binding and allosteric e ects o
benzodiazepines and other ligands

Melatonin Congeners Ramelteon Agonist at melatonin MT1 and MT2 receptors to


promote sleep

Barbiturates Amobarbital Mechanism o action o barbiturates on GABAA


Butabarbital receptors (see Figure 9-1):
Mephobarbital Barbiturates enhance the binding o GABA to
Methohexital GABAA receptors in a Cl dependent ashion;
Pentobarbital however, barbiturates also promote the binding
Phenobarbital o benzodiazepines
Secobarbital Barbiturates potentiate GABA-induced Cl
Thiopental currents by prolonging periods during which
bursts o channel opening occur
Only and (not ) subunits o GABAA
receptors are required
Barbiturate-induced increases in Cl
conductance are not a ected by the deletion
o the tyrosine and threonine residues in the
subunit that govern the sensitivity o GABAA
receptors to activation by agonists

Miscellaneous Paraldehyde General central nervous system (CNS) depressant


Sedative-Hypnotic no longer used in the United States
Drugs
Chloral Hydrate Rapidly metabolized to trichloroethanol which
exerts barbiturate-like e ects on the GABAA
receptor

Meprobamate Mechanism o action is not clear

Carisoprodol Metabolized to meprobamate

Etomidate Intravenous anesthetic (see Chapter 11)

Clomethiazole Probably acts as an allosteric modulator at the


barbiturate site on the GABAA receptor

Propo ol Mechanism is not entirely known but thought


to act through enhancement o GABAA receptor
unction

Drugs Used to Treat Naltrexone Opioid antagonist (see Chapter 10)


Alcoholism
Disul ram Inhibits aldehyde dehydrogenase (ALDH) causing
accumulation o acetaldehyde (see Figure 9-2)

Nalme ene Opioid antagonist (see Chapter 10)

Acamprosate An analog o GABA, it is an agonist at GABAA


receptors and is probably an allosteric modulator

177
SECTION II Neuropharmacology

GABA s ite
CATEGORIES OF
Be nzo diaze pine s ite Barbiturate s ite
BENZODIAZEPINES BASED
ON THEIR ELIMINATION t 1/2
Ultra-short actingmidazolam
S te ro id s ite
Short-acting agents (t 1/2 <6 hours) a ne s the tics
triazolam, the nonbenzodiazepine or a nxioge nics
zolpidem(t 1/2 ~2 hours), and eszopiclone
(t 1/2 5-6 hours)
Intermediate-acting agents (t 1/2 6-24
hours), including estazolamand Pic ro toxin s ite
temazepam Cl convuls a nts

Long-acting agents (t 1/2 >24 hours),


including furazepam, diazepam, and FIGURE 9-1 Pharmacologic binding sites on the GABAA receptor. (Reproduced with permission
quazepam from Nestler EJ, Hyman SE, Malenka RC (eds). Molecular Neuropharmacology, 2nd ed. New York:
McGraw-Hill, 2009, p 135. Copyright 2009 by The McGraw-Hill Companies, Inc. All rights reserved.)

CATEGORIES OF INSOMNIA CASE 9-1


Transient insomnia lasts less than 3 days A 35-year-old woman comes to her physician complaining o not being able to sleep
and usuallyis caused bya brie environ- or the past week. She is prescribed zolpidem to be taken at bedtime.
mental or situational stressor a. What type o drug is zolpidem and what is its mechanism o action?
Short-terminsomnia lasts rom3 days Zolpidem is a nonbenzodiazepine sedative-hypnotic drug approved or the short-term
to 3 weeks and is usuallycaused bya treatment o insomnia. Although the actions o zolpidem are due to agonist e ects on
personal stressor such as illness, grie ,
(Continued)
or job problems
Long-terminsomnia is insomnia that has H3 C CH2 OH
CH3 OH
lasted or more than 3 weeks; no speci c Me tha nol Ethano l H2 O 2
stressor maybe identi able NAD+ NADP H
+ O2
Fome pizole Alcohol Ca ta la s e
X de hydroge na s e CYP 2E1
NADH
+ NADP H
+ H2 O 2H2 O
H+ O
H2 C O H3 C CH
Forma lde hyde Ac e talde hyde
NAD+

Alde hyde
de hydroge na s e X Dis ulfira m
NADH
+ O O
H+
HC OH H3 C C OH
Fola te -de pe nde nt Formic a cid Ac e tic ac id
pa thway CoA + ATP

CO 2
+ Thiokina s e
H2 O
AMP + 2P ;

C
-hydroxy--me thyl CoA S C CH3 Trica rboxylic
gluta ryl CoA a cid cycle
Ac e tyl Co A

Fa tty a cids
Ke tone Chole s te rol
bodie s

FIGURE 9-2 Metabolism o ethanol and methanol.


178
Hypnotics, Sedatives, and Ethanol CHAPTER 9

GABAA receptors and generally resemble those o benzodiazepines, it produces only


weak anticonvulsant e ects in experimental animals, and its relatively strong sedative
actions appear to mask anxiolytic e ects in various animal models o anxiety.
b. What side e ects should this woman be warned o ?
A er discontinuation o zolpidem, the bene cial e ects on sleep reportedly persist
or up to 1 week, but mild rebound insomnia on the rst night also has occurred.
olerance and physical dependence develop only rarely and under unusual circum-
stances. At therapeutic doses (5-10 mg), zolpidem in requently produces residual
daytime sedation or amnesia, and the incidence o other adverse e ects (eg, GI
complaints or dizziness) also is low. As with the benzodiazepines, large overdoses
o zolpidem do not produce severe respiratory depression unless other agents (eg,
ethanol) also are ingested. Hypnotic doses increase the hypoxia and hypercarbia o
patients with obstructive sleep apnea.
c. What considerations should her physician be aware o when treating insomnia
in this patient?
T e categories o insomnia are shown in the Side Bar CA EGORIES OF INSOM-
NIA. T is patients insomnia would be classi ed as short-term and hypnotics such
as zolpidem should be used intermittently, perhaps even skipping a dose a er 1
to 2 nights o good sleep. However, i her insomnia persists and becomes a long-
term problem, it raises a new set o issues. Side e ects o hypnotic agents may limit
their use ulness or insomnia management (see Summary able at the end o the
chapter). T e use o hypnotics or long-term insomnia is problematic or many
reasons. Long-term hypnotic use leads to a decrease in e ectiveness and may pro-
duce rebound insomnia on discontinuance. I a sedative-hypnotic has been used
regularly or more than 2 weeks, it should be tapered rather than discontinued
abruptly. In some patients on hypnotics with a short t 1/2, it is easier to switch rst
to a hypnotic with a long t1/2 and then to taper. T e onset o withdrawal symptoms
rom medications with a long t1/2 may be delayed. Consequently, the patient should
be warned about the symptoms associated with withdrawal e ects.
Hypnotics that act at GABAA receptors, including the benzodiazepine hypnotics and
the newer agents zolpidem, zopiclone, and zaleplon, are pre erred to barbiturates
because they have a greater therapeutic index, are less toxic in overdose, have smaller
e ects on sleep architecture, and have less abuse potential. Compounds with a shorter
t1/2 are avored in patients with sleep-onset insomnia but without signi cant daytime
anxiety who need to unction at ull e ectiveness during the day. T ese compounds
also are appropriate or the elderly because o a decreased risk o alls and respiratory
depression. However, the patient and physician should be aware that early-morning
awakening, rebound daytime anxiety, and amnestic episodes could also occur. T ese
undesirable side e ects are more common at higher doses o the benzodiazepines.

CASE 9-2
A 23-year-old male college student is brought to the ER because he cannot be aroused.
T e patient has a history o depression and he has been acting depressed over his
classes lately. T e patient has a prescription or lorazepam or anxiety and insomnia. In
the ER he is given umazenil intravenously.
a. What type o drug is f umazenil and what is its role in treating a benzodiazepine
overdose?
Flumazenil, the only member o this class, is an imidazobenzodiazepine that
behaves as a speci c benzodiazepine receptor antagonist. Flumazenil binds with
high a nity to speci c sites on the GABAA receptor, where it competitively antago-
nizes the binding and allosteric e ects o benzodiazepines and other ligands. Flu-
mazenil antagonizes both the electrophysiological and behavioral e ects o agonist
and inverse-agonist benzodiazepines. T e drug is given intravenously.
(Continued)

179
SECTION II Neuropharmacology

T e primary indications or the use o f umazenil are the management o suspected


benzodiazepine overdose and the reversal o sedative e ects produced by benzodi-
azepines administered during either general anesthesia or diagnostic and/or thera-
peutic procedures.
b. What cautions should be taken with this patient?
T e administration o a series o small injections o f umazenil is pre erred to a single
bolus injection. A total o 1 mg f umazenil given over 1 to 3 minutes usually is su -
cient to abolish the e ects o therapeutic doses o benzodiazepines; patients with
suspected benzodiazepine overdose should respond adequately to a cumulative dose
o 1 to 5 mg given over 2 to 10 minutes; a lack o response to 5 mg f umazenil strongly
suggests that a benzodiazepine is not the major cause o sedation. Additional courses
o treatment with f umazenil may be needed within 20 to 30 minutes should sedation
reappear. Flumazenil is not e ective in single-drug overdoses with either barbiturates
or tricyclic antidepressants. o the contrary, the administration o f umazenil in these
settings may be associated with the onset o seizures, especially in patients poisoned
with tricyclic antidepressants. Seizures or other withdrawal symptoms also may be
precipitated in patients who had been taking benzodiazepines or protracted periods
and in whom tolerance and/or dependence may have developed.

CASE 9-3
A 66-year-old man is brought to the ER with extreme anxiety, agitation, irritation, and
con usion. It is learned that, because o early dementia, he recently did not renew his
prescription or alprazolam, which he had been taking or the past 3 years or anxiety.
a. What is the likely cause o this patients change in mental status?
Chronic benzodiazepine use poses a risk or development o dependence and
abuse. Mild dependence may develop in many patients who have taken therapeutic
doses o benzodiazepines on a regular basis or prolonged periods. Withdrawal
symptoms may include temporary intensi cation o the problems that originally
prompted their use (eg, insomnia or anxiety). Dysphoria, irritability, sweating,
unpleasant dreams, tremors, anorexia, and aintness or dizziness also may occur,
especially when withdrawal o the benzodiazepine occurs abruptly. Hence, it is
prudent to taper the dosage gradually when therapy is to be discontinued.
b. What are the options or treating this patient?
One option would be to reinstitute the alprazolam and once it has its calming
e ects, begin a slow tapering o the dose. Another option would be to begin a ben-
zodiazepine with a longer t1/2 such as f urazepam and once it has its calming e ects,
discontinue its use and let the long hal -li e taper the e ects and avoid withdrawal.
Each o these options would depend on the ability o the patient or a caregiver to
administer the drug.

CASE 9-4
A 43-year-old woman has been treated with secobarbital or insomnia or the past
10 years.
a. What distinguishes secobarbital rom other barbiturates?
able 9-1 shows the distinguishing eatures o selected barbiturates, including seco-
barbital. Secobarbital has a relatively short t 1/2 compared to other barbiturates such
as phenobarbital.
b. What is the mechanism o action o barbiturates?
See the Mechanisms o Action able above. T e barbiturates reversibly depress
the activity o all excitable tissues. T e barbiturates exert several distinct e ects on
excitatory and inhibitory synaptic transmission. For example, ()-pentobarbital
(Continued)
180
Hypnotics, Sedatives, and Ethanol CHAPTER 9

TABLE 9-1 Structures, Trade Names, and Major Pharmacological Properties o Selected Barbiturates
R3 O
N C R 5a
3
(or S )a O C2 5C
N C R 5b
H O

COMPOUND DOSAGE
(TRADE NAMES) R3 R5a R5b FORMSb t 1/2 (hours) THERAPEUTIC USES COMMENTS

Amobarbital H C2H5 CH2CH2CH(CH3)2 IM, IV 10-40 Insomnia, pre-op Only Na+ salt administered
(AMYTAL) sedation, emergency parenterally
management o
seizures

Butabarbital H C2H5 CH(CH3)CH2CH3 Oral 35-50 Insomnia, pre-op Redistribution shortens


(BUTISOL, sedation duration o action o single
others) dose to 8 hours

Mephobarbital CH3 C2H5 Oral 10-70 Seizure disorders, Second-line anticonvulsant


(MEBARAL) daytime sedation

Methohexital CH3 CH2CH=CH2 CH(CH3)CCCH2CH3 IV 3-5c Induction and Only Na+ salt available;
(BREVITAL) maintenance o single dose provides 5-7
anesthesia min o Anesthesiac

Pentobarbital H C2H5 CH(CH3)CH2CH2CH3 Oral, IM, IV, 15-50 Insomnia, pre-op Only Na+ salt administered
(NEMBUTAL) rectal sedation, emergency parenterally
management o
seizures

Phenobarbital H C2H5 Oral, IM, IV 80-120 Seizure disorders, First-line anticonvulsant;


(LUMINAL, status epilepticus, only Na+ salt administered
others) daytime sedation parenterally

Secobarbital H CH2CH=CH2 CH(CH3)CH2CH2CH3 Oral 15-40 Insomnia, Only Na+ salt available
(SECONAL) preoperative
Sedation

Thiopental H C2H5 CH(CH3)CH2CH2CH3 IV 8-10c Induction and Only Na+ salt available;
(PENTOTHAL) maintenance o single dose provides brie
anesthesia, pre-op o anesthesiac
sedation, emergency
management o
seizures
a
O except in thiopental, where it is replaced by S. bIM, intramuscular injection; IV, intravenous administration. c Value represents terminal t1/2 due
to metabolism by the liver; redistribution ollowing parenteral administration produces e ects lasting only a ew minutes

potentiates GABA-induced increases in chloride conductance and depresses


voltage-activated Ca2+ currents at similar concentrations (<10 M) in isolated
hippocampal neurons. Barbiturates can produce all degrees o depression o the
CNS, ranging rom mild sedation to general anesthesia.
c. Why are barbiturates now not commonly used as sedatives?
T e barbiturates were once used extensively as sedative-hypnotic drugs, but are
associated with physical dependence. Except or a ew specialized uses, they have
been largely replaced by the much sa er benzodiazepines.
d. Describe the tolerance and physical dependence that occurs with barbiturates.
Pharmacodynamic ( unctional) and pharmacokinetic tolerance to barbiturates
can occur. T e ormer contributes more to the decreased e ect than does the latter.
(Continued)

181
SECTION II Neuropharmacology

olerance to the e ects on mood, sedation, and hypnosis occurs more readily
and is greater than that to the anticonvulsant and lethal e ects; thus, as toler-
ance increases, the therapeutic index decreases. Pharmacodynamic tolerance
to barbiturates con ers cross-tolerance to all general CNS-depressant drugs,
including ethanol.
Chronic administration o barbiturates markedly increases the protein and lipid
content o the hepatic smooth endoplasmic reticulum, as well as the activities
o glucuronyl trans erase and CYPs 1A2, 2C9, 2C19, and 3A4. T e induction o
these enzymes increases the metabolism o a number o drugs and endogenous
substances, including steroid hormones, cholesterol, bile salts, and vitamins K and
D. T is also results in an increased rate o barbiturate metabolism, which partly
accounts or the tolerance to barbiturates. Repeated administration, especially o
phenobarbital, shortens the t1/2 o barbiturates that are metabolized as a result o the
induction o microsomal enzymes.
Like other CNS depressant drugs, barbiturates are abused, and some individuals
develop a dependence on them (see Chapter 14). Moreover, the barbiturates may
have euphoriant e ects.
Withdrawal rom barbiturates can be serious, resulting in seizures and death.
e. What are the adverse e ects that can be seen with the chronic use o
barbiturates?
Drowsiness may last or only a ew hours a er a hypnotic dose o barbiturate,
but residual CNS depression sometimes is evident the ollowing day, and subtle
distortions o mood and impairment o judgment and ne motor skills may be
demonstrable. Residual e ects also may take the orm o vertigo, nausea, vomiting,
or diarrhea, or sometimes may be mani ested as overt excitement. T e user may
awaken slightly intoxicated and eel euphoric and energetic; later, as the demands o
daytime activities challenge possibly impaired aculties, the user may display irrita-
bility and temper.
Rarely, ex oliative dermatitis may be caused by phenobarbital and can prove atal;
the skin eruption may be associated with ever, delirium, and marked degenerative
changes in the liver and other parenchymatous organs.
Barbiturates combine with other CNS depressants to cause severe depression; etha-
nol is the most requent o ender, and interactions with rst-generation antihista-
mines also are common.
Barbiturates competitively inhibit the metabolism o certain other drugs; however,
the greatest number o drug interactions results rom induction o hepatic CYPs
and the accelerated disappearance o many drugs and endogenous substances (see
answer to Case 9-4d above).
Because barbiturates enhance porphyrin synthesis, they are absolutely contraindi-
cated in patients with acute intermittent porphyria or porphyria variegata.

DISTINGUISHING CASE 9-5


FEATURES OF FETAL A 35-year-old woman has been an abstainer rom alcohol all o her adult li e. Recently
ALCOHOL SYNDROME she has read on the Internet that the French population have a relatively low mortality
Acluster o cranio acial abnormalities rom coronary heart disease and that this protection is due to their widespread wine
consumption. She talks to her physician about this issue.
CNSdys unction
Pre- and/or postnatal stunting o growth a. Should this patient be advised to consume alcohol in moderate amounts?
In most countries, the risk o mortality due to coronary heart disease (CHD) is
correlated with a high dietary intake o saturated at and elevated serum choles-
terol levels. France is an exception to this rule, with relatively low mortality rom
CHD despite the consumption o high quantities o saturated ats (the French
paradox). Epidemiological studies suggest that widespread wine consumption
(Continued)

182
Hypnotics, Sedatives, and Ethanol CHAPTER 9

(20-30 g ethanol per day) is 1 o the actors con erring a cardioprotective e ect,
with 1 to 3 drinks per day resulting in a 10 to 40% decreased risk o coronary heart
disease compared with abstainers. One possible mechanism by which alcohol could
reduce the risk o CHD is through its e ects on blood lipids. Changes in plasma
lipoprotein levels, particularly increases in high-density lipoprotein (HDL; see
Chapter 20), have been associated with the protective e ects o ethanol.
b. Should abstainers rom alcohol be advised to consume ethanol in moderate
amounts?
T e answer is no. T ere have been no randomized clinical trials to test the e cacy
o daily alcohol use in reducing rates o coronary heart disease and mortality, and
it is inappropriate or physicians to advocate alcohol ingestion solely to prevent
heart disease.
c. What are the e ects o ethanol on various physiological systems?
able 9-2 lists the e ects o ethanol on various physiological systems. Most
notable are the CNS where ethanol is a general CNS depressant and perturbs
the balance between excitatory and inhibitory inf uences in the brain, result-
ing in anxiolysis, ataxia, and sedation; the cardiovascular system where ethanol
can result in cardiomyopathy; skeletal muscle where ethanol results in decreased
strength, and the gastrointestinal system where ethanol can produce signi cant
pathology in the pancreas and liver.

TABLE 9-2 Ef ects o Ethanol on Physiological Systems


PHYSIOLOGICAL SYSTEM EFFECT
Central Nervous System Ethanol is a general CNS depressant. Chronic abuse leads to dependence, tolerance, and craving or ethanol
(see Chapter 14). Alcoholism is a progressive illness, and brain damage rom chronic alcohol abuse contributes
to the de cits in cognitive unctioning and judgment seen in alcoholics. Alcoholism is a leading cause o
dementia in the United States. Chronic alcohol abuse results in shrinkage o the brain owing to loss o both
white and gray matter. It is important to note that ethanol itsel is neurotoxic, and although malnutrition or
vitamin de ciencies probably play roles in complications o alcoholism such as Wernicke encephalopathy and
Korsako s psychosis, most o the alcohol-induced brain damage in Western countries is due to alcohol itsel .
Additional severe neurological syndromes associated with chronic heavy use o alcohol include cerebellar
degeneration with associated atrophy o the cerebellar vermis, and a peripheral neuropathy.
Heavy doses o ethanol over multiple days or weeks are also associated with several temporary but disturbing
alcohol-inducedpsychiatric syndromes.

Neurochemical Pathways Alcohol perturbs the balance between excitatory and inhibitory inf uences in the brain, resulting in anxiolysis,
ataxia, and sedation. This is accomplished by either enhancing inhibitory or antagonizing excitatory
neurotransmission. Ethanol likely produces its e ects by simultaneously altering the unctioning o a number o
proteins that can a ect neuronal excitability (see Table 9-3). Many o the prominent e ects are on ligand-gated
and voltage-gated ion channels and GPCR systems.
While no de nitive data on the mechanisms or alcohol-induced psychiatric conditions are available, it is logical
to assume that alcohol-related changes in CNS pathways (NE and 5-HT levels, the balance between GABAA and
NMDA receptor activity, dopaminergic activity) may operate in a manner similar to those seen in depression,
anxiety, and schizophrenic disorders.

Ion Channels Substantial data implicate the GABAA receptor as an important target or the in vivo actions o ethanol.
Stimulation o this multisubunit, ligand-gated Cl channel system contributes to eelings o sleepiness, muscle
relaxation, and the acute anticonvulsant properties associated with all GABA-boosting drugs.
The nicotinic ACh receptor is also sensitive to the e ects o ethanol. Drinking acutely increases ACh in the
ventral tegmental area, with a subsequent increase in DA in the nucleus accumbens.
Ethanol inhibits the unction o the NMDA and kainate receptor subtypes; AMPA receptors are largely resistant
to alcohol.
Ethanol enhances the activity o large-conductance, Ca2+-activated K+ channels in neurohypophyseal terminals,
perhaps contributing to the reduced release o oxytocin and vasopressin a ter ethanol consumption.

(Continued)

183
SECTION II Neuropharmacology

PHYSIOLOGICAL SYSTEM EFFECT


Other Neurotransmitter Dopamine-related systems have central importance regarding the eelings o reward and craving
Systems associated with all intoxicating substances. O special importance are alterations in DA activity in the
ventral tegmental and related areas, especially the nucleus accumbens, which are likely to play a major
role in eelings o euphoria and reward. Acute alcohol intoxication results in an increase in synaptic
DA; repeated alcohol administration is associated with changes in both D2 and D4 receptors that
may be important in the perpetuation o alcohol use as well as in relapse.
The acute administration o ethanol is associated with a signi cant increase in 5-HT in the synaptic space;
continued use o ethanol produces an upregulation o 5-HT receptors.
Cannabinoid receptors, especially CB1 encoded by the gene CNR1, are also a ected by ethanol. Activation o
CB1 occurs with acute ethanol administration and a ects the release o DA, GABA, and glutamate, and reward
circuits o the brain. Antagonists o CB1 receptors, such as rimonabant, may block the e ect o ethanol on
dopaminergic systems.

Protein Kinases and Intracellular signal-transduction cascades, such as MAPK, tyrosine kinases, and neurotrophic actor receptors,
Intracellular Signaling also are thought to be a ected by ethanol. Ethanol enhances the activities o several iso orms o adenylyl
Enzymes cyclase, with AC7 being the most sensitive. This promotes increased production o cyclic AMP and thus
increased activity o PKA. Ethanols actions appear to be mediated by activation o Gs and promotion o the
interaction between Gs and adenylyl cyclase.

Cardiovascular System Ethanol intake greater than 3 standard drinks per day elevates the risk or heart attacks and bleeding-related
strokes. Indeed, vascular-related diseases are among the leading causes o early death in alcohol-dependent
individuals. The risk includes a six old increased risk or coronary artery disease, a heightened risk or cardiac
arrhythmias, and an elevated risk o congestive heart ailure.
Serum Lipoproteins and Cardiovascular E ects
In France, there is relatively low mortality rom coronary heart disease (CHD) despite the consumption o
high quantities o saturated ats (the French paradox). Epidemiological studies suggest that widespread
wine consumption (20-30 g ethanol per day) is one o the actors con erring a cardioprotective e ect, with
1-3 drinks per day resulting in a 10-40% decreased risk o coronary heart disease compared with abstainers.
In contrast, daily consumption o greater amounts o alcohol leads to an increased incidence o noncoronary
causes o cardiovascular ailure, such as arrhythmias, cardiomyopathy, and hemorrhagic stroke, o setting the
bene cial e ects o alcohol on coronary arteries; that is, alcohol has a J-shaped dose-mortality curve. One
possible mechanism by which alcohol could reduce the risk o CHD is through its e ects on blood lipids.
Changes in plasma lipoprotein levels, particularly increases in high-density lipoprotein (HLD) (see Chapter 20),
have been associated with the protective e ects o ethanol.
Hypertension
Heavy alcohol use can raise diastolic and systolic blood pressure. The prevalence o hypertension attributable
to excess alcohol consumption is not known, but studies suggest a range o 5-11%.
Cardiac Arrhythmias
Alcohol has a number o pharmacological e ects on cardiac conduction, including prolongation o the QT
interval, prolongation o ventricular repolarization, and sympathetic stimulation. Atrial arrhythmias associated
with chronic alcohol use include supraventricular tachycardia, atrial brillation, and atrial f utter.
Cardiomyopathy
Ethanol is known to have dose-related toxic e ects on both skeletal and cardiac muscle. Numerous studies
have shown that alcohol can depress cardiac contractility and lead to cardiomyopathy.
Stroke
Clinical studies indicate an increased incidence o hemorrhagic and ischemic stroke in persons who drink
more than 40-60 g alcohol per day.

Skeletal Muscle Chronic, heavy, daily alcohol consumption is associated with decreased muscle strength, even when
adjusted or other actors such as age, nicotine use, and chronic illness. Heavy doses o alcohol also
can cause irreversible damage to muscle, re lected by a marked increase in the activity o creatine
kinase in plasma.

Body Temperature Ingestion o ethanol causes a eeling o warmth because alcohol enhances cutaneous and gastric blood f ow.
Increased sweating also may occur. Heat, there ore, is lost more rapidly, and the internal body temperature
alls. A ter consumption o large amounts o ethanol, the central temperature-regulating mechanism becomes
depressed, and the all in body temperature may become pronounced.

Diuresis Alcohol inhibits the release o vasopressin (antidiuretic hormone) rom the posterior pituitary gland, resulting in
enhanced diuresis. The volume loading that accompanies imbibing complements the diuresis that occurs as a
result o reduced vasopressin secretion.

(Continued)

184
Hypnotics, Sedatives, and Ethanol CHAPTER 9

PHYSIOLOGICAL SYSTEM EFFECT


Gastrointestinal System Esophagus
Alcohol requently is either the primary etiologic actor or one o multiple causal actors associated with
esophageal dys unction.
Stomach
Heavy alcohol use can disrupt the gastric mucosal barrier and cause acute and chronic gastritis. Alcohol is not
thought to play a role in the pathogenesis o peptic ulcer disease. Unlike acute and chronic gastritis, peptic ulcer
disease is not more common in alcoholics. Nevertheless, alcohol exacerbates the clinical course and severity o
ulcer symptoms. It appears to act synergistically with Helicobacter pylori to delay healing (see Chapter 32).
Intestines
Many alcoholics have chronic diarrhea as a result o malabsorption in the small intestine. The major symptom
is requent loose stools. The rectal ssures and pruritus ani that requently are associated with heavy drinking
probably are related to chronic diarrhea.
Pancreas
Heavy alcohol use is the most common cause o both acute and chronic pancreatitis in the United States.
While pancreatitis has been known to occur a ter a single episode o heavy alcohol use, prolonged heavy
drinking is common in most cases. Acute alcoholic pancreatitis is characterized by the abrupt onset o
abdominal pain, nausea, vomiting, and increased levels o serum or urine pancreatic enzymes.
Liver
Ethanol produces a constellation o dose-related deleterious e ects in the liver. The primary e ects are atty
in ltration o the liver, hepatitis, and cirrhosis. Because o its intrinsic toxicity, alcohol can injure the liver in the
absence o dietary de ciencies. The accumulation o at in the liver is an early event and can occur in normal
individuals a ter the ingestion o relatively small amounts o ethanol. This accumulation results rom inhibition
o both the tricarboxylic acid cycle and the oxidation o at, in part owing to the generation o excess NADH
produced by the actions o ADH and ALDH (see Figure 9-2). Fibrosis, resulting rom tissue necrosis and
chronic inf ammation, is the underlying cause o alcoholic cirrhosis.
Vitamins and Minerals
The almost complete lack o protein, vitamins, and most other nutrients in alcoholic beverages
predisposes those who consume large quantities o alcohol to nutritional de ciencies. Alcoholics o ten
present with these de ciencies owing to decreased intake, decreased absorption, or impaired utilization
o nutrients. The peripheral neuropathy, Korsako s psychosis, and Wernickes encephalopathy seen
in alcoholics probably are caused by de ciencies o the B complex o vitamins (particularly thiamine),
although direct toxicity produced by alcohol itsel has not been ruled out.

Sexual Function Despite the widespread belie that alcohol can enhance sexual activities, the opposite e ect is generally noted.
Both acute and chronic alcohol use can lead to impotence in men. Increased blood alcohol concentrations
lead to decreased sexual arousal, increased ejaculatory latency, and decreased orgasmic pleasure. Many emale
alcoholics complain o decreased libido, decreased vaginal lubrication, and menstrual cycle abnormalities.

Hematological and Chronic alcohol use is associated with anemia. Microcytic anemia can occur because o chronic blood loss and
Immunological E ects iron de ciency. Macrocytic anemia and increases in mean corpuscular volume are common and may occur in
the absence o vitamin de ciencies. Normochromic anemia also can occur owing to e ects o chronic illness on
hematopoiesis.
Alcohol also a ects granulocytes and lymphocytes. E ects include leukopenia, alteration o lymphocyte subsets,
decreased T-cell mitogenesis, and changes in immunoglobulin production.

TABLE 9-3 Impact o Ethanol on Key Neurochemical Systems


NEUROTRANSMITTER SYSTEM EFFECTS
GABAA GABA release, receptor density

NMDA Inhibition o postsynaptic NMDA receptors; with chronic use, up-regulation

DA Synaptic DA, e ects on ventral tegmentum/nucleus accumbens reward

ACTH CNS and blood levels o ACTH

Opioid Release o endorphins, activation o receptors

5-HT in 5-HT synaptic space

Cannabinoid CB1 activity changes in DA, GABA, glutamate activity

185
SECTION II Neuropharmacology

CASE 9-6
A 43-year-old man has been chronically abusing alcohol (up to one- h o whisky per
day) over the past 10 years. He is recently incarcerated and has had no access to alcohol
or the past 3 days.
a. What is this man at risk o ?
Physical dependence to ethanol is demonstrated by the elicitation o a withdrawal
syndrome when alcohol consumption is terminated. T e symptoms and severity are
determined by the amount and duration o alcohol consumption and include sleep
disruption, autonomic nervous system (sympathetic) activation, tremors, and in
severe cases, seizures. In addition, 2 or more days a er withdrawal, some individu-
als experience delirium tremens, characterized by hallucinations, delirium, ever,
and tachycardia. Delirium tremens can be atal.
b. How should alcohol withdrawal be treated?
Alcohol withdrawal syndrome should be treated with sedatives such as a long-
acting benzodiazepine, and general supportive care such as intravenous f uids and
seizure precautions.

CASE 9-7
A 40-year-old man has been consuming 4 to 5 alcoholic drinks each night a er work
and an additional 2 to 3 drinks each night at home. Recently, he has been consuming
alcohol prior to leaving or work in the morning. He is seeking help or his drinking
behavior.
a. What are the goals o any alcohol treatment program?
T e core o care is a process o interventions and sessions that help enhance
changes in how the person views their problem, along with e orts to help
them alter the problematic behaviors. With cognitive-behavioral approaches
serving as the core o treatment, and a 20% or greater rate o spontaneous
remission in alcohol use disorders, the role o medications can be di cult to
evaluate. T us, only those pharmacological approaches that have been shown
to be superior to placebo through double-blind control trials are worth consid-
ering in clinical practice.
b. What is the role o pharmacotherapy o alcoholism?
Currently, 3 drugs are approved in the United States or treatment o alcoholism:
disul ram, naltrexone, and acamprosate (see able 9-4). Disul ram has a long
history o use but has allen into dis avor because o its side e ects and problems
with patient adherence to therapy. Naltrexone and acamprosate were introduced
more recently. T e goal o these medications is to assist the patient in maintain-
ing abstinence.
(Continued)

TABLE 9-4 Oral Medications or Treating Alcohol Abuse


MEDICATION USUAL DOSE MECHANISM/EFFECT
Disul ram 250 mg/day (range 125-500 mg/day) Inhibits ALDH with resulting acetaldehyde a ter drinking. Abstinence is
rein orced to avoid the resulting adverse reaction.

Naltrexone 50 mg/day Opioid receptor antagonist; elt to drinking through eelings o reward
with alcohol and/or craving.

Acamprosate 666 mg three times daily Weak antagonist o NMDA receptors, activator o GABAA receptors; may mild
protracted abstinence syndromes with eelings o a need or alcohol.

186
Hypnotics, Sedatives, and Ethanol CHAPTER 9

c. What are the drugs available to treat alcoholism?


Naltrexone helps to maintain abstinence by reducing the urge to drink and increas-
ing control when a slip occurs. It is not a cure or alcoholism and does not pre-
vent relapse in all patients. Naltrexone works best when used in conjunction with
some orm o psychosocial therapy, such as cognitive behavioral therapy. T ere is
evidence that naltrexone blocks activation by alcohol o dopaminergic pathways in
the brain that are thought to be critical to reward.
Acamprosate is an analogue o GABA. A number o double-blind, placebo-con-
trolled studies have demonstrated that acamprosate (1.3-2 g/d) decreases drinking
requency and reduces relapse drinking in abstinent alcoholics and appears to have
e cacy similar to that o naltrexone.
Disul ram, given alone, is a relatively nontoxic substance, but it inhibits aldehyde
dehydrogenase (ALDH) activity (see Figure 9-2) and causes the blood acetaldehyde
concentration to rise to 5 to 10 times above the level achieved when ethanol is given
to an individual not pretreated with disul ram. Acetaldehyde, produced as a result
o the oxidation o ethanol by alcohol dehydrogenase (ADH), ordinarily does not
accumulate in the body because it is urther oxidized almost as soon as it is ormed.
Following the administration o disul ram, both cytosolic and mitochondrial
orms o ALDH are irreversibly inactivated to varying degrees, and the concentra-
tion o acetaldehyde rises.
T e ingestion o alcohol by individuals previously treated with disul ram gives rise
to marked signs and symptoms o acetaldehyde poisoning. Within 5 to 10 minutes,
the ace eels hot and soon a erward becomes f ushed and scarlet in appearance. As
the vasodilation spreads over the whole body, intense throbbing is elt in the head
and neck, and a pulsating headache may develop. Respiratory di culties, nausea,
copious vomiting, sweating, thirst, chest pain, considerable hypotension, ortho-
static syncope, marked uneasiness, weakness, vertigo, blurred vision, and con usion
are observed. T e acial f ush is replaced by pallor, and the blood pressure may all
to shock levels.

CASE 9-8
A 22-year-old pregnant woman continues to drink 4 to 5 beers and 2 to 3 glasses
o wine each day despite being cautioned against the use o alcohol while she is
pregnant
a. What are possible teratogenic e ects o ethanol?
Children born to alcoholic mothers display a common pattern o distinct dysmor-
phology known as etal alcohol syndrome (FAS). T e diagnosis o FAS typically is
based on the observance o a triad o abnormalities shown in the Side Bar DIS IN-
GUISHING FEA URES OF FE AL ALCOHOL SYNDROME.
b. What is the likelihood that the child o this woman will have FAS?
T e incidence o FAS is believed to be in the range o 0.5 to 1 per 1000 live births
in the general US population, with rates as high as 2 to 3 per 1000 in A rican
American and Native American populations. A lower socioeconomic status o the
mother rather than racial background per se appears to be primarily responsible or
the higher incidence o FAS observed in those groups. Children who do not meet
all the criteria or a diagnosis o FAS still may show physical and mental de cits
consistent with a partial phenotype, termed etal alcohol e ects (FAEs) or alcohol-
related neurodevelopmental disorders.
T e incidence o FAEs is likely higher than that o FAS, making alcohol consump-
tion during pregnancy a major public health problem.

187
SECTION II Neuropharmacology

KEY CONCEPTS
Benzodiazepine can be categorized based on their elimination t 1/ 2 (see
Side Bar CA EGORIES OF BENZODIAZEPINES BASED ON HEIR
ELIMINA ION t 1/2).
Benzodiazepines are pharmacologically interchangeable.
Benzodiazepines act by modulating the e ects o GABA at the GABAA receptor.
Most benzodiazepines are metabolized extensively by hepatic CYPs; oxazepam
and lorazepam are conjugated directly (see able 9-5).
Flumazenil is a benzodiazepine antagonist used to reverse e ects o
benzodiazepines.
T e novel benzodiazepine agonists zolpidem, zaleplon, and zopiclone have
largely replaced benzodiazepines in the treatment o insomnia.
Ethanol is metabolized by sequential hepatic oxidation rst to acetaldehyde by
ADH, and then to acetic acid by ALDH (see Figure 9-2).
Ethanol has serious and long-lasting e ects on many physiological systems
(see able 9-2).

TABLE 9-5 Major Metabolic Relationships among Some o the Benzodiazepinesa


N-DESALKYLATED
COMPOUNDS 3-HYDROXYLATED COMPOUNDS

a
Compounds enclosed in boxes are marketed in the United States. The approximate hal -lives o the various compounds are denoted in
parentheses; S (shortacting), t1/2 <6 hours; I (intermediate-acting), t1/2 = 6-24 hours; L (long-acting), t1/2 = >24 hours. All compounds except
clorazepate are biologically active; the activity o 3-hydroxydesalkylf urazepam has not been determined. Clonazepam (not shown) is an
N-desalkyl compound, and it is metabolized primarily by reduction o the 7-NO2 group to the corresponding amine (inactive), ollowed by
acetylation; its t1/2 is 20-40 hours.

188
Hypnotics, Sedatives, and Ethanol CHAPTER 9

SUMMARY QUIZ ANSWER KEY

QUESTION 9-1 A 43-year-old man with severe hepatic cirrhosis requires a sedative or
insomnia. Which o the ollowing sedatives would be the best choice or this patient?
a. Phenobarbital
b. Diazepam
c. Lorazepam
d. Secobarbital
e. Flurazepam

QUESTION 9-2 A 19-year-old college student has overdosed on phenobarbital. A er


standard supportive care, the physician in the ER should do which o the ollowing to
hasten the elimination o phenobarbital?
a. Acidi y the urine
b. Acidi y the blood
c. Alkalinize the blood
d. Alkalinize the urine
e. Use a drug to stimulate hepatic CYP 2C19

QUESTION 9-3 A 32-year-old woman is taking ramelteon or chronic insomnia.


Ramelteon binds to
a. melatonin receptors M1 and M2.
b. muscarinic receptors M3.
c. nicotinic receptors.
d. 1-adrenergic receptors.
e. D2 dopaminergic receptors.

QUESTION 9-4 A 43-year-old woman is prescribed a benzodiazepine or anxiety. T e


choice o which benzodiazepine to prescribe should be based on
a. volume o distribution.
b. plasma hal -li e.
c. protein binding.
d. the indications approved by the FDA.
e. creatinine clearance.

QUESTION 9-5 A 53-year-old man with 15 years o alcohol abuse has developed weak-
ness in his legs and the onset o heart ailure. T e primary treatment is
a. digoxin.
b. metoprolol.
c. creatine.
d. abstinence rom alcohol.
e. thiamine.

QUESTION 9-6 A 33-year-old Japanese man becomes ushed and light-headed a er


one glass o wine. T is reaction is likely due to
a. the rate he drank the glass o wine.
b. increased absorption o the alcohol.
c. inhibition o monoamine oxidase.
d. decreased renal excretion o alcohol.
e. a variant in aldehyde dehydrogenase.

189
SECTION II Neuropharmacology

SUMMARY QUIZ ANSWER KEY

QUESTION 9-1 Answer is c. T e benzodiazepines are metabolized extensively by


hepatic CYPs, particularly CYPs 3A4 and 2C19. Some benzodiazepines, such as oxaze-
pam and lorazepam, are conjugated directly and are not metabolized by these enzymes.
T us, the benzodiazepine o choice in a patient who might have diminished hepatic
metabolism by CYPs would be either oxazepam or lorazepam.

QUESTION 9-2 Answer is d. T e treatment o acute barbiturate intoxication is based


on general supportive measures, which are applicable in most respects to poisoning by
any CNS depressant. Hemodialysis or hemoper usion is necessary only rarely, and the
use o CNS stimulants is contraindicated because they increase the mortality rate. I
renal and cardiac unctions are satis actory, and the patient is hydrated, orced diuresis
and alkalinization o the urine will hasten the excretion o phenobarbital. Measures to
prevent or treat atelectasis should be taken, and mechanical ventilation should be initi-
ated when indicated. See Chapter 3 Clinical and Environmental oxicity.

QUESTION 9-3 Answer is a. Ramelteon is a synthetic tricyclic analog o melatonin.


Binding o agonists, such as melatonin, to M 1 receptors promotes the onset o sleep
while melatonin binding to M 2 receptors shi s the timing o the circadian system.
Ramelteon binds to both M 1 and M 2 receptors with high af nity but, unlike mela-
tonin, it does not bind appreciably to quinone reductase 2, the structurally unrelated
M 3 receptor. Ramelteon is not known to bind to any other classes o receptors, such as
nicotinic, acetylcholine, neuropeptide, dopamine, or opiate receptors, or the benzodi-
azepine-binding site on GABAA receptors.

QUESTION 9-4 Answer is b. T e therapeutic uses and routes o administration o


individual benzodiazepines that are marketed in the United States are summarized in
able 9-6. Most benzodiazepines can be used interchangeably. For example, diazepam
can be used or alcohol withdrawal, and most benzodiazepines work as hypnotics. In
general, the therapeutic choice o a given benzodiazepine depends on its t1/2 (see Side Bar
CA EGORIZA ION OF BENZODIAZEPINES BASED ON HEIR ELIMINA ION t)
and may not match the FDA-approved indications. Benzodiazepines that are use ul as
anticonvulsants have a long t1/2, and rapid entry into the brain is required or ef cacy in
treatment o status epilepticus. A short elimination t1/2 is desirable or hypnotics, although
this carries the drawback o increased abuse liability and severity o withdrawal a er drug
discontinuation. Antianxiety agents, in contrast, should have a long t1/2 despite the draw-
back o the risk o neuropsychological de cits caused by drug accumulation.

QUESTION 9-5 Answer is d. Ethanol is known to have dose-related toxic e ects on


both skeletal and cardiac muscle. Numerous studies have shown that alcohol can
depress cardiac contractility and lead to cardiomyopathy. Echocardiography demon-
strates global hypokinesis. Approximately hal o all patients with idiopathic cardiomy-
opathy are alcohol-dependent. Although the clinical signs and symptoms o idiopathic
and alcohol-induced cardiomyopathy are similar, alcohol-induced cardiomyopathy has
a better prognosis i patients are able to stop drinking. Since 40 to 50% o persons with
alcohol-induced cardiomyopathy who continue to drink die within 3-5 years, absti-
nence remains the primary treatment.

QUESTION 9-6 Answer is e. Homozygotes with a non unctional aldehyde dehydro-


genase 2*2 (ALDH2*2) occur in 5 to 10% o Japanese, Chinese, and Korean individu-
als, or whom severe adverse reactions occur a er consumption o one drink or less.
Consequently, their risk or severe repetitive heavy drinking is close to zero. T is reac-
tion operates through the same mechanism that occurs with drinking a er taking the
ALDH2 inhibitor, disul ram (see Figure 9-2). Heterozygotes or this polymorphism
(ALDH2*2, 2*1) make up 30 to 40% o Asian individuals who, a er consuming etha-
nol, experience a acial ush and an enhanced sensitivity to beverage alcohol, but who
do not necessarily report an overall adverse response to ethanol.

190
Hypnotics, Sedatives, and Ethanol CHAPTER 9

TABLE 9-6 Trade Names, Routes o Administration, and Therapeutic Uses o Benzodiazepines
USUAL SEDATIVE-
COMPOUND (TRADE ROUTES OF EXAMPLES OF HYPNOTIC DOSAGE,
NAME) ADMINISTRATIONa THERAPEUTIC USESb COMMENTS t 1/2, Hoursc mg d

Alprazolam (XANAX) Oral Anxiety disorders, Withdrawal symptoms 122


agoraphobia may be especially severe

Chlordiazepoxide Oral, IM, IV Anxiety disorders, Long-acting and sel - 103.4 50-100, qdqid e
(LIBRIUM, others) management o alcohol tapering because o
withdrawal, anesthetic active metabolites
premedication

Clonazepam Oral Seizure disorders, Tolerance develops to 235


(KLONOPIN) adjunctive treatment in anticonvulsant e ects
acute mania and certain
movement disorders

Clorazepate Oral Anxiety disorders, Prodrug; activity due to 2.00.9 3.75-20, bidqid e
(TRANXENE, others) seizure disorders ormation o nordazepam
during absorption

Diazepam (VALIUM, Oral, IM, IV, rectal Anxiety disorders, Prototypical 4313 5-10, tidqid e
others) status epilepticus, benzodiazepine
skeletal muscle
relaxation, anesthetic
premedication

Estazolam (PROSOM) Oral Insomnia Contains triazolo ring; 1024 1-2


adverse e ects may
be similar to those o
triazolam

Flurazepam Oral Insomnia Active metabolites 7424 15-30


(DALMANE) accumulate with chronic
use

Lorazepam (ATIVAN) Oral, IM, IV Anxiety disorders, Metabolized solely by 145 2-4
preanesthetic Conjugation
medication

Midazolam (VERSED) IV, IM Preanesthetic and Rapidly inactivated 1.90.6


intraoperative
medication

Oxazepam (SERAX) Oral Anxiety disorders Metabolized solely by 8.02.4 15-30, tidqid e
Conjugation

Quazepam (DORAL) Oral Insomnia Active metabolites 39 7.5-15


accumulate with chronic
use

Temazepam Oral Insomnia Metabolized mainly by 116 7.5-30


(RESTORIL) Conjugation

Triazolam (HALCION) Oral Insomnia Rapidly inactivated; may 2.91.0 0.125-0.25


cause disturbing daytime
side e ects
a
IM, intramuscular injection; IV, intravenous administration; qd, once a day; bid, twice a day; tid, three times a day; qid, our times a day. b The
therapeutic uses are identi ed as examples to emphasize that most benzodiazepines can be used interchangeably. In general, the therapeutic
uses o a given benzodiazepine are related to its t1/2 and may not match the marketed indications.
c
Hal -li e o active metabolite may di er. d For additional dosage in ormation, see Chapter 11 (anesthesia), and Chapter 12 (seizure disorders).
e
Approved as a sedative-hypnotic only or management o alcohol withdrawal; doses in a nontolerant individual would be smaller.
Recommended doses vary considerably depending on speci c use, condition o patient, and concomitant administration o other drugs.

191
SECTION II Neuropharmacology

SUMMARYTABLE: SEDATIVE HYPNOTIC DRUGS AND DRUGS USED TO TREAT ALCOHOLISM


TOXICITIES
CLASS AND
SUBCLASSES NAMES CLINICAL USES COMMON UNIQUE: CLINICALLY IMPORTANT
Benzodiazepines Alprazolam The therapeutic uses and At peak concentrations The paradoxical e ect o
Chlordiazepoxide routes o administration hypnotic doses can nightmares has been reported with
Clonazepam o benzodiazepines are be expected to cause f urazepam
Chlorazepate summarized in Table 9-6 varying degrees o Amnesia, restlessness,
Diazepam In general, the lightheadedness, sleepwalking, and hypomania
Estazolam therapeutic uses o a lassitude, increased have been reported with various
Flurazepam given benzodiazepine reaction time, impaired benzodiazepines
Lorazepam depend on its elimination mental and motor Disinhibition reactions are also
Midazolam t1/2 (see Side Bar unctions, con usion, reported
Oxazepam CATEGORIZATION OF and amnesia; these Chronic benzodiazepine use is
Quazepam BENZODIAZEPINES BASED e ects can impair driving associated with dependence,
Temazepam ON THEIR ELIMINATION skills, particularly when abuse, and withdrawal
Triazolam HALF-LIFE) and may not combined with ethanol
match FDA-approved
indications

Novel Benzodiazepine Zolpidem Approved or the short- May produce daytime Hypnotic doses increase the
Receptor Agonists term treatment o sedation or amnesia hypoxia and hypercarbia o
insomnia patients with obstructive sleep
apnea

Zaleplon Used to treat chronic or Dependence and a withdrawal


transient insomnia syndrome may be less than or
other sedatives

Eszopiclone Used or long-term Bitter taste Minor withdrawal syndrome o


treatment o insomnia abnormal dreams, anxiety, and
nausea

Benzodiazepine Flumazenil Available only or IVuse Duration o clinical e ect Flumazenil will precipitate
Receptor Antagonist to treat benzodiazepine usually only 30 to 60 withdrawal in patients who have
overdose (see Chapter minutes been using benzodiazepines
3) and to reverse A series o small injections chronically
the sedative e ects is pre erred to a single
o benzodiazepines large injection; additional
administered during courses o treatment
surgery or diagnostic may be required should
procedures sedation reappear

Melatonin Congeners Ramelteon Approved or the Dizziness, nausea, atigue No tolerance to its reduction in
treatment o insomnia sleep onset
Not a controlled
substance

Barbiturates Amobarbital All are used as sedatives, Residual CNS Pharmacodynamic and
Butabarbital mephobarbital and depression the day a ter pharmacokinetic tolerance to
Mephobarbital phenobarbital are also administration barbiturates can occur
Methohexital used to treat seizures Residual e ects may also Depression o respiratory drive
Pentobarbital See Table 9-1 or details o include vertigo, nausea, Induction o liver microsomal drug-
Phenobarbital di erences in plasma t1/2 vomiting, or diarrhea metabolizing enzymes
Secobarbital and uses Barbiturates may produce Ex oliative dermatitis caused by
Thiopental excitement rather than phenobarbital
depression in some Severe CNS depression especially
patients and may worsen when combined with other CNS
a patients perception o depressants such as ethanol
pain Because barbiturates cause
enhanced porphyrin synthesis they
are contraindicated in patients
with acute intermittent porphyria
or porphyria variegata

192
Hypnotics, Sedatives, and Ethanol CHAPTER 9

TOXICITIES
CLASS AND
SUBCLASSES NAMES CLINICAL USES COMMON UNIQUE: CLINICALLY IMPORTANT

Miscellaneous Paraldehyde General CNS depressant Irritating to throat and Intravenous use is associated with
Sedative-Hypnotic no longer used in the stomach a ter oral use injuries to tissues
Drugs United States

Chloral hydrate May be used to


treat patients with
paradoxical reactions to
benzodiazepines

Meprobamate Used as a sedative anti- Drowsiness and ataxia Abuse potential is high
anxiety agent Impairment o motor coordination
and prolongation o reaction time
Withdrawal syndrome on abruptly
stopping

Carisoprodol Muscle relaxant Drowsiness and ataxia Has abuse potential as a popular
Major metabolite is street drug
meprobamate Impairment o motor coordination
and prolongation o reaction time
Withdrawal syndrome on abruptly
stopping

Etomidate Used as an IVanesthetic See Chapter 11 See Chapter 11


(see Chapter 11)

Clomethiazole Used or hypnosis Physical dependence and


in elderly and tolerance can occur and are
institutionalized patients associated with a withdrawal
especially or withdrawal syndrome upon abruptly stopping
rom ethanol the drug

Propo ol Used in the induction and See Chapter 11 See Chapter 11


maintenance o general
anesthesia (see Chapter
11)

Drugs Used to Treat Naltrexone Approved or the Nausea, abdominal Excessive doses can cause liver
Alcoholism treatment o ethanol cramps damage
dependence Contraindicated in patients with
liver ailure or acute hepatitis

Disul ram Used to promote By itsel disul ram may Severe reaction when ethanol,
abstinence rom ethanol cause lassitude, urticarial even in small quantities, is
rash, restlessness, tremor, co-ingested
headache, dizziness, and Reactions may include nausea,
mild GI disturbances vomiting, f ushing, sweating, chest
pain, hypotension, blurred vision,
and con usion

Nalme ene Approved or use in Dizziness, nausea, Agitation, hypotension,


treatment o opiate vomiting hallucinations
overdose (see Chapter 10)
Oral dosage orms used to
treat ethanol dependence
are not approved in the
United States

Acamprosate Promotes abstinence Insomnia, sexual Diarrhea, low or high blood


rom ethanol impotence, headaches pressure, irregular heart rhythm

193
CHAPTER

10 Opioid Pharmacology
T is chapter will be most use ul a er having a basic understanding o the material in
DRUGS INCLUDED IN THIS
Chapter 18, Opioids, Analgesia, and Pain Management in Goodman & Gilmans T e
CHAPTER Pharmacological Basis of T erapeutics, 12th Edition. In addition to the material pre-
Al entanil (ALFENTA) sented here, the 12th Edition includes:
Benzonatate (TESSALON, others) A history o analgesic use
Buprenorphine; injection (BUPRENEX); oral A detailed description o the endogenous opioid systems: ligands and receptors
(SUBUTEX); in combination with naloxone A discussion o opioid receptor classes, distribution, binding/coupling requirements
(SUBOXONE) or opiate ligands, and the unctional consequences o acute and chronic opiate
Butorphanol (STADOL, others) receptor activation
Codeine Figure 18-8 Structures o morphine-related opiate agonists and antagonists
Dextromethorphan marketed or over-the- Figure 18-9 Chemical structures o piperidine and phenylpiperidine analgesics
counter sales Details o the cellular and molecular mechanisms o opioid tolerance, dependence,
Di enoxin (MOTOFEN) available onlyin and withdrawal
combination with atropine Nonanalgesic therapeutic uses o opioids
Diphenoxylate (LOMOTIL, others) available able 18-4 Resources or Pain Management
onlyin combination with atropine
able 18-5 World Health Organization Analgesic Ladder
Fentanyl citrate (SUBLIMAZE, others);
transdermal patch (DURAGESIC, oth- LEARNING OBJECTIVES
ers); entanyl buccal tablets, buccal lm, Understand the mechanisms o action, adverse e ects, and therapeutic uses o
lollipop-like lozenges (FENTORA, ONSOLIS, opiate receptor agonists.
ACTIQ,others)
Know the mechanisms o action and therapeutic uses o opiate receptor antagonists.
Hydrocodone (LORTAB, VICODIN, others)
Know the role o opiate receptor agonists in the management o acute and
Hydromorphone (DILAUDID,others)
chronic pain.
Levorphanol (LEVO-DROMORAN)
Know the di erent methods o administration o opiate receptor agonists that
Loperamide (IMODIUM, others) improve analgesic e cacy while reducing side e ects.
Meperidine (pethidine, DEMEROL, others)
Methadone (DOLOPHINE, others)
Methylnaltrexone (RELISTOR) ACTIONS AND SELECTIVITIES OF SOME OPIOIDS AT , , RECEPTORS
Morphine sul ate; liposomal ormulation OPIATE LIGANDS RECEPTOR TYPES
(DEPODUR); preservative- ree or spinal
delivery(DURAMORPH, DEPODUR, others)
Nalbuphine (NUBAIN, others) Agonists

Nalme eneno longer marketed in the Fentanyl +++


United States Hydromorphone +++
Nalorphine (NALLINE, others) Levorphanol +++
Naloxone (NARCAN, others)
Methadone +++
Naltrexone (REVIA, VIVITROL, others)
Morphine a +++
Naltrindole ( receptor antagonist used
only or biomedical research) Su entanil +++ + +
Oxycodone (PERCODAN); in combina- Buprenorphine P
tion with acetaminophen (PERCOCET); Butorphanol P +++
extended release (OXYCONTIN)
(Continued)
Oxymorphone (NUMORPHAN, others)
(continues)

194
Opioid Pharmacology CHAPTER 1 0

OPIATE LIGANDS RECEPTOR TYPES DRUGS INCLUDED IN THIS


CHAPTER (Cont.)
Nalbuphine ++ Pentazocine (TALWIN); in combination
with acetaminophen (TALCEN); in combi-
Antagonists
nation with naloxone (TALWINNX)
Naloxone b
Propoxyphene (DARVON, others)
Naltrexone b Propoxyphene napsylate (DARVON-N)
Naltrindole b Remi entanil (ULTIVA)
a
Prototypical -pre erring. Su entanil (SUFENTA, others)
b
antagonist; P partial agonist. The number o + or symbols is an indication o potency. Tapentadol (NUCYNTA)
Adapted with permission rom Raynor K, Kong H, Chen Y, Yasuda K, Yu L, Bell GI, Reisine T, Tramadol (ULTRAM)
Pharmacological characterization o the cloned kappa-, delta-, and mu-opioid receptors. Mol
Pharmacol, 1994;45:330-334.

MECHANISMS OF ACTION OF OPIOID AGONISTS AND ANTAGONISTS


DRUG CLASS DRUG MECHANISM OF ACTION
Opioid Receptor Morphine Morphine and its congeners act through
Agonist Codeine activation (Side Bar ACTIONS AND SELECTIVITIES
Hydromorphone OF SOME OPIOIDS AT , , AND RECEPTORS) o
Hydrocodone the GPCR receptors: (mu opioid receptor, MOR);
Oxycodone (delta opioid receptor, DOR); or (kappa opioid
Oxymorphone receptor, KOR) (see also Figure 10-1)
Meperidine Agonist binding results in con ormational changes
Methadone in the GPCR, initiating G protein activation (see
Propoxyphene Chapter 1); the Gi/Go coupling results in a large
Fentanyl number o intracellular events, including:
Al entanil Inhibition o adenyl cyclase activity
Su entanil Reduced opening o voltage-gated Ca2+
Remi entanil channels
Levorphanol Stimulation o K+ current through several
Tramadol channels
Tapentadol Activation o PKC and PLC
Tramadol and tapentadol also inhibit uptake o
norepinephrine and serotonin
Opioid Receptor Nalbuphine MOR antagonist and KOR agonist
Agonists/Antagonists
and Partial Agonists
Butorphanol MOR antagonist and KOR agonist
Pentazocine Weak MOR antagonist or partial MOR agonist and
KOR agonist
Buprenorphine Partial MOR agonist
Antidiarrheal Agents Diphenoxylate Congener o meperidine that activates MOR
to produce constipation (see Chapter 33); only
available in combination with atropine to reduce
abuse liability
Di enoxin Metabolite o diphenoxylate that acts similar to
parent compound
Loperamide Congener o meperidine that activates MOR to
produce constipation (see Chapter 33)
Opioid Receptor Naloxone Opioid receptor antagonist
Antagonist
Naltrexone Opioid receptor antagonist
Methylnaltrexone Opioid receptor antagonist at peripheral sites
(Continued)
195
SECTION II Neuropharmacology

FACTORS EXACERBATING DRUG CLASS DRUG MECHANISM OF ACTION


OPIATE-INDUCED Antitussives Dextromethorphan Congener o the codeine analog methorphan
RESPIRATORY DEPRESSION NMDA-receptor antagonist
Antitussive MOA is not clear (see Chapter 24)
Other medications
Benzonatate Antitussive action is believed to be mediated
Other depressant medications such as through stretch or cough receptors in the lung, as
alcohol, tranquilizers, sedative-hypnotics well as by a central mechanism (see Chapter 24)
Sleep
Natural sleep produces a decrease in the CASE 10-1
sensitivityto CO2
A 22-year-old man is brought to the emergency room ollowing a skiing injury to his
Obstructive sleep apnea increases the like-
knee. Although he is in considerable pain, it is relieved by a small dose o morphine.
lihood o atal respiratorydepression
He reports that his pain is still present, but it is less bothersome. He says the morphine
Age eels good.
Newborns and elderlypatients are at
greater risk a. What are the mechanisms or the dif erent causes o pain?
Disease All pain is not the same and a number o variables contribute to the patients pain
report and there ore to the e ect o the analgesic. Many clinical pain syndromes, such
Patients with chroniccardiopulmonaryor as ound in cancer, typically represent a combination o in ammatory and neuro-
renal diseases can mani est a desensiti- pathic mechanisms. T e Side Bar PAIN S A ES di erentiates the various types o
zation to increased CO2
COPD (Continued)
Enhanced respiratorydepression is seen
in patients with chronicobstructive
PAG OPIATE ACTION 2
pulmonarydisease (COPD)
Relief of pain Pe ria que ducta l
PAG
gray
GABA-e rgic
Removal o a pain ul condition (which can ne uron
Dors a l
stimulate respiration) will reduce the ra phe
(tonica lly a ctive ) Locus
ventilatorydrive and lead to apparent MOR a ctiva tion 1 coe rule us
respiratorydepression (inhibits GABA re le a s e )
Me dullope ta l ne uron
(GABA-R) Me dulla 4
Me dulla
S PINAL OPIATE 3
ACTION
C-fibe r te rmina l

MOR
MOR
Ca 2+

K+

S pina l cord

2nd orde r ne uron

FIGURE 10-1 Mechanisms o opiate action in producing analgesia. Top le t: Schematic o orga-
nization o opiate action in the periaqueductal gray (PAG). Top right: Opiate-sensitive pathways
in PAG opiate actions block the release o GABA rom tonically active systems that otherwise
regulate the projections to the medulla (1) leading to an activation o PAG out ow resulting in
activation o orebrain (2) and spinal (3) monoamine receptors that regulate spinal cord projec-
tions (4) which provide sensory input to higher centers and mood.
Bottom le t: Schematic o primary a erent synapse with second order dorsal horn spinal neuron,
showing pre- and postsynaptic opiate receptors coupled to Ca2+ and K+ channels, respectively.
Opiate receptor binding is highly expressed in the super cial spinal dorsal horn (substantia gela-
tinosa). These receptors are located presynaptically on the terminals o small primary a erents
(C- bers) and postsynaptially on second-order neurons. Presynaptically, activation o MOR blocks
the opening o the voltage-sensitve Ca2+ channel, which otherwise initiates transmitter release.
Postsynaptically, MOR activation enhances opening o K+ channels, leading to hyperpolarization.
Thus, an opiate agonist acting at these sites jointly serves to attenuate the a erent-evoked exci-
tation o the second-order neuron.
196
Opioid Pharmacology CHAPTER 1 0

Injury
PAIN STATES
Tis s ue Injury
P G, BK, K
Acute nociception
Acute activation o small high-threshold
Loca l re le a s e of a ctive S e ns itiza tion sensorya erents (A and C bers)
fa ctors (P G, BK, K) generates transient input into the spinal
cord, which in turn leads to activation o
neurons that project contralateral to the
Pe rs is te nt a ctiva tion/ thalamus and thence to the somatosen-
s e ns itiza tion of A/C fibe rs
sory cortex; examples include a hot
co ee cup, a needle stick, or an incision
Activity in a s ce nding pa thways Tissue injury (Figure 10-2)
+ Following tissue injuryor local infam-
s pina l fa cilita tion
Fa cilita tion mation (eg, local skin burn, toothache,
rheumatoid joint), an ongoing pain
Exa gge ra te d output for
state arises that is characterized by
give n s timulus input burning, throbbing, or aching, and an
abnormal pain response (hyperalgesia);
such tissue injury-evoked pain is o ten
Ongoing pa in + Hype ra lge s ia re erred to asnociceptivepain
Nerve injury (Figure 10-3)
FIGURE 10-2 Mechanistic ow diagram o tissue injuryevoked nociception. BK, bradykinin; K,
Injuryto a peripheral nerve yields complex
cytokines; PG, prostagladins. anatomical and biochemical changes in
the nerve and spinal cord that induce
spontaneous dysesthesias (shooting,
pain and Figures 10-2 and 10-3 show mechanistic diagrams o these types o pain.
burning pain) and allodynia (light
Although nociceptive pain (as in this patient) usually is responsive to opioid analge-
touch hurts); this pain state is said to
sics, neuropathic pain is typically considered to respond less well to opioid analgesics.
be neuropathic
b. What are the mechanisms o action or morphine and other opiate ligands to
produce analgesia?
T e analgesic actions o opiates a er systemic delivery are believed to represent
actions in the brain (supraspinal), spinal cord, and in some instances in the periphery.
T e supraspinal and spinal actions o opiates are shown schematically in Figure 10-1.
c. What are the mechanisms o action by which morphine and other opiate ligands
alter mood and have rewarding properties that may be important in addiction?
T e mechanisms by which opioids produce euphoria, tranquility, and other altera-
tions o mood (including rewarding properties) are complex and not entirely clear.
(Continued)

Ne uroma
S pina l
s e ns itiza tion
Ne rve Injury

Pe riphe ra l ne rve
de ge ne ra tion...Ne uroma

S ponta ne ous S pina l A a ffe re nt


a ffe re nt a ctivity s e ns itiza tion fibe rs

S ponta ne ous dys e s the s ia s Allodynia


(s hooting, burning pa in) (light touch hurts )

FIGURE 10-3 Mechanistic ow diagram o nerve injuryevoked nociception.


197
SECTION II Neuropharmacology

Neural systems thought to mediate opioid rein orcement overlap with, but are
distinct rom, those involved in physical dependence and analgesia. Behavioral and
pharmacological data point to a pivotal role o the mesocorticolimbic (MCL) dopa-
mine system, a basal orebrain circuit long implicated in reward and motivation
(see Figure 10-4).

CASE 10-2
Following surgery or a hip replacement, a 64-year-old woman is treated with a par-
enteral opiate or pain. Upon release rom hospital, she is given a prescription or oral
oxycodone or pain. T ree days a er discharge she is complaining o constipation.
a. What is the cause o her constipation?
Opiates have important e ects upon all aspects o GI unction (see Chapter 33). It
is estimated that 40 to 95% o patients treated with opioids develop constipation
and that changes in bowel unction can be demonstrated even with acute dos-
ing. Opiate agonists suppress local neurogenic networks that provide a rhythmic
(Continued)

Hippoca mpus

Pre fronta l
cortex
(P FC)

N. Accumbe ns
(NAc)
Amygda la Ve ntra l te gme nta l
a re a (VTA)

P FC

Glu

VTA

DA

GABA
MOR

NAc opia te

MOR

VP
Rewa rd

FIGURE 10-4 Schematic pathways underlying rewarding properties o opiates.


Upper panel: This sagittal section o rat brain displays simpli ed DA and GABA inputs rom
the ventral tegmental area (VTA) and pre rontal cortex (PFC), respectively, into the nucleus
accumbens (NAc).
Lower panel: Neurons are labeled with their primary neurotransmitters. At a cellular level, MOR
agonists reduce excitability and transmitter release at the sites indicated by inhibiting Ca2+ in ux
and enhancing K+ current (see Figure 10-1). Thus, opiate-induced inhibition in the VTA on GABA-ergic
interneurons or in the NAc reduce GABA-mediated inhibition and increase out ow rom the ventral
pallidum (VP), which appears to correlate with a positive rein orcing state (enhanced reward).

198
Opioid Pharmacology CHAPTER 1 0

inhibition o muscle tone leading to concurrent increases in basal tone in the cir-
cular muscle o the small and large intestine. T is results in enhanced high-ampli-
tude phasic contractions, which are nonpropulsatile. T e upper part o the small
intestine, particularly the duodenum, is a ected more than the ileum. A period
o relative atony may ollow the hypertonicity. T e reduced rate o passage o the
intestinal contents, along with reduced intestinal secretion, leads to increased water
absorption, increasing viscosity o the bowel contents, and constipation.
b. What are the options or treatment o this patient?
T e peripherally limited antagonists such as methylnaltrexone have a very impor-
tant role in the management o the constipation and the reduced GI motility
present in the patient undergoing chronic opioid therapy (as or chronic pain or
methadone maintenance) and have been approved by the FDA or that use. With
distribution restricted to the periphery, these agents do not alter central opioid ago-
nist actions. Worrisome reports o GI per oration in this setting are under review
by FDA. Other strategies or the management o opioid-induced constipation are
described in Chapter 33.

CASE 10-3
A 43-year-old markedly obese man with the history o sleep apnea is being treated with
hydrocodone or chronic back pain.
a. What considerations should be taken into account when treating chronic pain
with opiates?
Management o pain is an important element in any therapeutic intervention.
Failure to adequately manage pain can have important negative consequences
on physiological unction such as autonomic hyper-reactivity (increased blood
pressure, heart rate, suppression o gastrointestinal motility, reduced secretions),
reduced mobility leading to deconditioning, muscle wasting, joint sti ening, and
decalci cation, and can contribute to deleterious changes in the psychological state
(depression, helplessness syndromes, anxiety). By many hospital-accrediting orga-
nizations, and by law in many states, appropriate pain assessment and adequate
pain management are considered to be standard o care, with pain being considered
the h vital sign.
able 10-1 shows dosing data or clinically employed opioid analgesics. T ese are
only guidelines, but they can give a place to start a patients treatment.
Numerous societies and agencies have published guidelines or the use o strong
opioids in treating pain, including the American Academy o Pain Medicine, the
American Pain Society, the Federation o State Medical Boards (FSMB), and the
Drug En orcement Agency. While slightly di erent in particulars, all guidelines to
date share the criteria established by the FSMB (see able 10-2).
b. Why is this patient at risk o respiratory depression while taking the
hydrocodone?
T is patients history o sleep apnea puts him at increased risk o respiratory
depression with opiate therapy (see Side Bar FAC ORS EXACERBA ING OPIA E-
INDUCED RESPIRA ORY DEPRESSION). Although e ects on respiration are
readily demonstrated, clinically signi cant respiratory depression rarely occurs
with standard analgesic doses in the absence o other contributing variables. It
should be stressed, however, that respiratory depression represents the primary
cause o morbidity secondary to opiate therapy.
c. What is a possible mechanism or respiratory depression in this patient?
Morphine-like opioids depress respiration through MOR and DOR receptors in part
by a direct depressant e ect on rhythm generation, with changes in respiratory pat-
tern and rate observed at lower doses than changes in tidal volume. A key property
(Continued)

199
2
TABLE 10-1 Dosing Data or Clinically Employed Opioid Analgesics
0
0
RECOMMENDED STARTTNG DOSE RECOMMENDED STARTING DOSE
APPROXIMATE
(ADULTS >50 KG) (CHILDREN AND ADULTS <50 KG)a
APPROXIMATE EQUI EQUI ANALGESIC

S
DRUG ANALGESIC ORAL DOSE PARENTERAL DOSE ORAL PARENTERAL ORAL PARENTERAL

E
C
Opioid Agonists

T
I
O
Morphine b 30 mg q34h (around-the-clock 10 mg q34h 15 mg q34h 5 mg q34h 0.3 mg/kg q34h 0.1 mg/kg q34h

N
dosing) 60 mg q34h (single
dose or intermittent dosing

I
I
Codeine c 130 mg q34h 75 mg q34h 30 mg q34h 30 mg q2h (IM/SC) 1 mg/kg q34h d Not recommended
Hydromophone (DILAUDID)b 7.5 mg q34h 1.5 mg q34h 4 mg q34h 1 mg q34h 0.06 mg/kg q34h 0.015 mg/kg q34h
Hydrocodone (in LORCET, LORTAB, VICODIN, 30 mg q34h Not available 5 mg q34h Not available 0.2 mg/kg q34h d Not available
others, typically with acetominophen)

N
Levorphanol 4 mg q68h 2 mg q68h 2 mg q68h 1 mg q68h 0.04 mg/kg q68h 0.02 mg/kg q68h

e
u
r
Meperidine (DEMEROL) 300 mg q23h 100 mg q3h Not recommended 50 mg q3h Not recommended 0.75 mg/kg q23h

o
p
h
Methadone (DOLOPHINE, others) 20 mg q68h 10 mg q68h 2.5 mg q12h 2.5 mg q12h 0.2 mg/kg q12h 0.1 mg/kg q68h

a
r
m
Oxycodone (REXICODONE, OXYCONTIN, also 30 mg q34h Not available 5 mg q34h Not available 0.2 mg/kg q34h d Not available

a
in PERCOCET, PERCODAN, TYLOX, others)g

c
o
l
Oxymorphone b (NUMORPHAN) Not available 1 mg q34h Not available 1 mg q34h Not recommended Not recommended

o
g
y
Propoxyphene (DARVON) 130 mg e Not available 65 mg q46h e Not available Not recommended Not recommended
Tramadol (ULTRAM) 100 mg e 100 mg 50100 mg q6h e 50-100 mg q6h e Not recommended Not recommended
Opioid Agonist Antagonists or Partial Agonists
Buprenorphine (BUPRENEX) Not available 0.30.4 mg q68h Not available 0.4 mg q68h Not available 0.004 mg/kg q68h
Butorphanol (STADOL) Not available 2 mg q34h Not available 2 mg q34h Not available Not recommended
Nalbuphine (NUBAIN) Not available 10 mg q34h Not available 10 mg q34h Not available 0.1 mg/kg q34h
Published tables vary in the suggested doses that are equi-analgesic to morphine. Clinical response is the criterion that must be applied or each patient; titration to clinical response is necessary.
Because there is not complete cross-tolerance among these drugs, it is usually necessary to use a lower than equi-analgesic dose when changing drugs and to retitrate to response. Caution:
Recommended doses do not apply to patients with renal or hepatic insuf ciency or other conditions a ecting drug metabolism and kinetics.
a
Caution: Doses listed or patients with body weight less than 50 kg cannot be used as initial starting doses in babies less than 6 months o age. Consult the Clinical Practice Guideline for Acute
Pain Management: Operative or Medical Procedures and Trauma section on management o pain in neonates or recommendations. bFor morphine, hydromorphone, and oxymorphone, rectal
administration is an alternate route or patients unable to take oral medications, but equi-analgesic doses may di er rom oral and parenteral doses because o pharmacokinetic di erences.
c
Caution: Codeine doses above 65 mg o ten are not appropriate due to diminishing incremental analgesia with increasing doses but continually increasing constipation and other side e ects.
d
Caution: Doses o aspirin and acetaminophen in combination opi-oid/NSAID preparations must also be adjusted to the patients body weight. Maximum acetaminophen dose: 4 g/day in adults,
90 mg/kg/day in children.
e
Doses or moderate pain not necessarily equivalent to 30 mg oral or 10 mg parenteral morphine.
Risk o seizures: parenteral ormulation not available in the United States.
g
Oxycontin is an extended-release preparation containing up to 160 mg o oxycodone per tablet and recommended or use every 12 hours. It has been subject to substantial abuse. Modi ed rom
Agency or Healthcare Policy and Research, 1992.
Opioid Pharmacology CHAPTER 1 0

TABLE 10-2 Guidelines or the Use o Opioids to Treat Chronic Pain


Evaluation of the patient: A complete medical history and physical must be conducted and documented in the medical record.
Treatment plan: The treatment plan should state objectives that are used to determine treatment success.
Informed consent and agreement: The physician should discuss the risks, bene ts, and alternatives to chronic opioid therapy with the patient.
Many practitioners have developed an opioid contractthat outlines the responsibilities o the physician and the patient or continued
prescription o controlled substances.
Periodic review: At reasonable intervals, the patient should be seen by the physician to review the course o treatment and document results
o consultation, diagnostic, testing, laboratory results, and the success o treatment.
Consultation: The physician should re er the patient or consultation when appropriate.
Documentation/medical records: The physician should keep actual and complete medical records that include:
(a) medical history and physical examination; (b) diagnostic, therapeutic, and laboratory results; (c) evaluations and consultations;
(d) treatment objectives; (e) discussion o risks and bene ts; ( ) treatment; (g) medications including date, type, dosage, and quantity
prescribed; (h) instructions and agreements; and (i) periodic reviews.
Compliance with controlled substances law and regulations: To prescribe, dispense or administer controlled substances, the physician must be
licensed in the state and comply with applicable state and ederal regulations.

o opiate e ects on respiration is the depression o the ventilatory response to


increased PCO2. T e ventilatory response to increased PCO2 is important in patients
with sleep apnea. T is e ect is mediated by opiate depression o the excitability o
brainstem chemosensory neurons. In addition to the e ects on the CO2 response,
opiates will depress ventilation otherwise driven by hypoxia through an e ect upon
carotid and aortic body chemosensors. Importantly, with opiates, hypoxic stimu-
lation o chemoreceptors still may be e ective when opioids have decreased the
responsiveness to CO2, and inhalation o O2 may remove the residual drive resulting
rom the elevated PCO2 and produce apnea.

CASE 10-4
A 56-year-old woman with advanced breast cancer is being treated with methadone
or her pain.
a. Why is methadone a good choice or this patient?
T e outstanding properties o methadone are its analgesic activity, its ef cacy by
the oral route, its extended duration o action in suppressing withdrawal symptoms
in physically dependent individuals, and its tendency to show persistent e ects with
repeated administration.
b. What are the major considerations in using methadone to treat this patients pain?
Side e ects, toxicity, and conditions that alter sensitivity, as well as the treatment o
acute intoxication, are similar to those described or morphine. During long-term
administration, there may be excessive sweating, lymphocytosis, and increased
concentrations o prolactin, albumin, and globulins in the plasma. Ri ampin and
phenytoin accelerate the metabolism o methadone and can precipitate withdrawal
symptoms. Unlike other opioids, methadone is associated with long Q syndrome
and is additive with agents known to prolong the Q interval. Serious cardiac
arrhythmias, including torsades de pointes, have been observed during treatment
with methadone.
Care must be taken when escalating the dosage because o the prolonged t 1/2
o the drug and its tendency to accumulate over a period o several days with
repeated dosing. Iatrogenic overdoses have occurred during initiation o therapy
and dosing titration with methadone because o too-rapid titration or the con-
comitant use o depressant drugs. T e peak respiratory depressant e ects o
methadone typically occur later and persist longer than the peak analgesic e ects
so it is necessary to exercise vigilance and strongly caution patients against sel -
medicating with CNS depressants, particularly during treatment initiation and
dose titration.
(Continued)
201
SECTION II Neuropharmacology

TABLE 10-3 Oral Morphine to Methadone Conversion Guidelines


CONVERSION RATIOS
DAILY MORPHINE DOSE mg/24 h, ORALLY MORPHINE oral : METHADONE oral
<100 3 : 1

101-300 5 : 1

301-600 10 : 1

601-800 12 : 1

801-1000 15 : 1

>1000 20 : 1

While not necessarily relevant to this patient, able 10-3 shows the oral morphine
to methadone guidelines.
he pain state (see Side Bar PAIN S A ES) in any given clinical condition is
not typically constant and will vary over time. In chronic pain states, the daily
course o the pain may luctuate, or example, being greater in the morning
hours or upon awakening. Arthritic states display lares that are associated with
an exacerbated pain condition. Changes in the magnitude o pain occur during
the daily routine resulting in breakthrough pain during episodic events such
as dressing changes (incident pain). hese examples emphasize the need or
individualized management o increased or decreased pain levels with baseline
analgesic dosing supplemented with the use o short-acting rescue medica-
tions as required. In the ace o ongoing severe pain, analgesics should be dosed
in continuous or around-the-clock ashion rather than on an as-needed
basis. his provides more consistent analgesic levels and avoids unnecessary
su ering.
c. What are other options or the treatment o this patients pain?
T ere are a variety o routes o administration o opiates or the treatment o
chronic pain.
Patient-controlled analgesia (PCA)
With this modality, the patient has limited control o the dosing o opioid rom an
in usion pump programmed within tightly mandated parameters. PCA can be used
or intravenous, epidural, or intrathecal administration o opioids. T is technique
avoids delays inherent in administration by a caregiver and generally permits better
alignment between pain control and individual di erences in pain perception and
responsiveness to opioids.
Spinal delivery
Administration o opioids into the epidural or intrathecal space provides more
direct access to the rst pain-processing synapse in the dorsal horn o the spinal
cord. T is permits the use o doses substantially lower than those required or oral
or parenteral administration
Local drug action
Opioid receptors on peripheral sensory nerves respond to locally applied opioids
during in ammation. Peripheral analgesia permits the use o lower doses, applied
locally, than those necessary to achieve a systemic e ect.
Rectal administration
T is route is an alternative or patients with dif culty swallowing or other oral
pathology and who pre er a less invasive route than parenteral administration.
(Continued)
202
Opioid Pharmacology CHAPTER 1 0

Inhalation
Opioids can be delivered by nebulizer. However, this delivery method is rarely used
due to erratic absorption rom the lung and highly variable therapeutic e ect.
Oral transmucosal administration
Opioids can be absorbed through the oral mucosa more rapidly than through the
stomach. Bioavailability is greater owing to avoidance o rst-pass metabolism, and
lipophilic opioids are absorbed better by this route than are hydrophilic compounds
such as morphine. A transmucosal delivery system that suspends entanyl in a dis-
solvable sugar-based lollipop or rapidly dissolving buccal tablet have been approved
or the treatment o cancer pain; in this setting, transmucosal entanyl relieves pain
within 15 minutes, and patients easily can titrate the appropriate dose.
Transnasal administration
Butorphanol, a(n) KOR agonist/MOR antagonist, has been employed intranasally.
A transnasal pectin-based entanyl spray is currently in clinical trials or the treat-
ment o cancer-related pain.
Transdermal and iontophoretic administration
ransdermal entanyl patches are approved or use in sustained pain. T e opioid
permeates the skin, and a depot is established in the stratum corneum layer.
Iontophoresis is the transport o soluble ions through the skin by using a mild elec-
tric current. T is technique has been employed with morphine. Unlike transdermal
opioids, a drug reservoir does not build up in the skin, thus limiting the duration
o both desired and undesired e ects. Patient-controlled iontophoretic transdermal
entanyl systems have been developed, but none is currently marketed.

CASE 10-5
A 19-year-old man is brought to the emergency room with a suspected heroin over-
dose. He is unresponsive and his respirations are depressed. He was discharged rom
30 days in a detoxi cation program earlier in the day. Naloxone is administered
intravenously.
a. What is naloxone and why is it ef ective in this patient?
A variety o agents bind competitively to 1 or more o the opioid receptors, display
little or no intrinsic activity, and robustly antagonize the e ects o opiate receptor
agonists. Opioid antagonists, particularly naloxone, have an established use in the
treatment o opioid-induced toxicity, especially respiratory depression. Its speci c-
ity is such that reversal by this agent is virtually diagnostic or the contribution o
an opiate to the respiratory depression. Naloxone acts rapidly to reverse the respira-
tory depression associated with high doses o opioids.
b. What precautions must be taken with the administration o naloxone?
It should be used cautiously because it also can precipitate withdrawal in dependent
subjects and cause undesirable cardiovascular side e ects. By care ully titrating the
dose o naloxone, it usually is possible to rapidly antagonize the respiratory-depres-
sant actions without eliciting a ully expressed withdrawal syndrome. T e duration
o action o naloxone is relatively short, and it o en must be given repeatedly or by
continuous in usion.
c. What are other therapeutic uses o opiate antagonists?
T ere is considerable interest in the use o opiate antagonists such as naltrexone as an
adjuvant in treating a variety o nonopioid dependency syndromes such as alcoholism
(see Chapters 9 and 14), where an opiate antagonist decreases the chance o relapse.
Buprenorphine in a xed-dose combination with naloxone is approved or the
treatment o opioid dependence.
(Continued)

203
SECTION II Neuropharmacology

T e potential utility o opiate antagonists in the treatment o shock, stroke, spinal


cord and brain trauma, and other disorders that may involve mobilization o endoge-
nous opioid peptides has been reported; but opioid antagonists have ailed to demon-
strate neuroprotective bene ts and their study in trauma has been largely abandoned.

CASE 10-6
A 17-year-old boy is brought to the emergency room ollowing an injury to his arm
while playing soccer. He is administered meperidine by intramuscular injection or pain.
a. What is meperidine and how does it act as an analgesic?
Meperidine is predominantly an MOR agonist that produces a pattern o e ects
similar but not identical to those already described or morphine. T e major use
o meperidine is or analgesia. Unlike morphine and its congeners, meperidine is
not used or the treatment o cough or diarrhea. T e analgesic e ects o meperidine
are detectable ~15 minutes a er oral administration, peak in 1 to 2 hours, and
subside gradually.
b. What cautions should be taken with the use o meperidine?
T e pattern and overall incidence o untoward e ects that ollow the use o meperi-
dine are similar to those observed a er equi-analgesic doses o morphine, except
that constipation and urinary retention may be less common.
Meperidine and its congeners are contraindicated in patients taking MAO inhibitors
or within 14 days a er discontinuation o an MAO inhibitor. Severe reactions may
ollow the administration o meperidine to patients being treated with MAO inhibi-
tors. wo basic types o interactions can be observed. T e most prominent is an
excitatory reaction (serotonin syndrome) with delirium, hyperthermia, headache,
hyper- or hypotension, rigidity, convulsions, coma, and death. T is reaction may
be due to the ability o meperidine to block neuronal reuptake o 5-H , resulting in
serotonergic overactivity. Conversely, the MAO inhibitor interaction with meperi-
dine may resemble acute narcotic overdose owing to the inhibition o hepatic CYPs.
Meperidine is N-demethylated to normeperidine, which then may be hydrolyzed
to normeperidinic acid and subsequently conjugated. In patients or addicts who
are tolerant to the depressant e ects o meperidine, large doses repeated at short
intervals may produce an excitatory syndrome including hallucinations, trem-
ors, muscle twitches, dilated pupils, hyperactive re exes, and convulsions. T ese
excitatory symptoms are due to the accumulation o normeperidine, which has a
t 1/2 o 15 to 20 hours, compared to 3 hours or meperidine. Since normeperidine
is eliminated by the kidney and the liver, decreased renal or hepatic unction
increases the likelihood o toxicity. As a result o these properties, meperidine
is not recommended or the treatment o chronic pain because o concerns over
metabolite toxicity. It should not be used or longer than 48 hours or in doses
more than 600 mg/d.

KEY CONCEPTS
Opioids are a mainstay o pain treatment, but rational therapy may involve,
depending on the pain state, 1 or more drug classes, such as NSAIDs,
anticonvulsants, and antidepressants (see able 10-4).
Opioid analgesics provide symptomatic relie o pain, but the underlying
disease remains.
Opioids activate , , and opiate receptors to produce their pharmacological
e ects.
Sustained administration o an opiate agonist leads to tolerance, physical
dependence, and a withdrawal syndrome upon abrupt discontinuation.
(Continued)

204
Opioid Pharmacology CHAPTER 1 0

TABLE 10-4 Summary o Drug Target and Site o Action o Common Drug Classes and Relative Ef cacy by Pain State
DRUG CLASS REPRESENTATIVE
AGENTS IN PARENTHESES DRUG ACTION SITE OF ACTIONa RELATIVE EFFICACY IN PAIN STATESb
NSAIDs (ibupro en, aspirin Nonspeci c COX inhibitors Peripheral and spinal Tissue injury >> acute stimuli = nerve injury = 0
acetominophen)

COX 2 inhibitor (celecoxib) COX2-selective inhibitor Peripheral and spinal Tissue injury >> acute stimuli = nerve injury = 0

Opioids (morphine) receptor agonist Supraspinal and spinal Tissue injury = acute stimuli nerve injury > 0 (see
this chapter)

Anticonvulsants (gabapentin) Na+ channel block, 2 Supraspinal and spinal Nerve injury > tissue injury = acute stimuli = 0
subunit o Ca2+ channel

Tricyclic antidepressants Inhibit uptake o 5-HT/NE Supraspinal and spinal Nerve injury tissue injury >> acute stimuli = 0
(amitryptiline)
a
Studies based on local delivery in preclinical models, eg, intracranial microinjection or intraventricular injections, lumbar intrathecal delivery or
topical/subcutaneous application at injury site.
b
Pain states are de ned by preclinical models: acute: hot plate/tail ick/acute mechanical compression; tissue injury: intraplantar injections
o irritants, ocal thermal injury; nerve injury: compression/ligation o sciatic nerve or its branches or o nerve roots; systemic delivery o
chemotherapeutics.

Respiratory depression represents the primary cause o morbidity secondary to


opiate therapy.
T e majority o patients treated with opioids develop constipation.
Naloxone and naltrexone are centrally acting opioid antagonists; methylnaltrex-
one is a peripherally acting opioid antagonist.
In addition to the traditional oral and parenteral ormulations or opioids,
many other methods o administration have been developed in an e ort to
improve therapeutic e cacy and reduce side e ects (see Case 10-4c).
Morphine and meperidine should be avoided in patients with impaired renal
unction due to metabolite toxicity.

SUMMARY QUIZ

QUESTION 10-1 A 55-year-old woman is having a colonoscopy. Aside rom other


medications or sedation, it is likely she will receive which o the ollowing analgesics?
a. Methadone
b. Meperidine
c. Buprenorphine
d. Fentanyl
e. Pentazocine

QUESTION 10-2 A 42-year-old man with chronic pain is brought to the emergency
room because o over-sedation and respiratory depression while using entanyl patches.
He is given intravenous naloxone. He is not given oral naloxone because naloxone
a. is not absorbed rom the GI tract.
b. undergoes rst-pass metabolism in the liver.
c. is metabolized to an inactive metabolite in the GI lining.
d. is excreted unchanged in the urine.
e. is destroyed by stomach acid.

205
SECTION II Neuropharmacology

QUESTION 10-3 A 48-year-old man is being treated with a long-acting opiate or pain
associated with terminal cancer. He is also prescribed a transmucosal entanyl ormula-
tion (lollipop) or breakthrough pain. T e transmucosal ormulation is an e ective
analgesic because it
a. avoids rst pass metabolism o entanyl.
b. avoids nausea and vomiting that is associated with the systemic use o entanyl.
c. delivers entanyl directly to opiate receptors in the mouth.
d. avoids constipation.
e. avoids respiratory depression.

QUESTION 10-4 A 50-year-old woman with back pain is administered an opiate ago-
nist. A er 2 weeks o therapy, she notices that she needs to increase the dose to get the
same analgesic e ect. She is experiencing
a. physical dependence.
b. addiction.
c. tolerance.
d. withdrawal.
e. drug-seeking behavior.

QUESTION 10-5 A 43-year-old man with postsurgical pain is placed on an intravenous


patient-controlled analgesia pump with morphine as the analgesic. T is orm o analge-
sic administration is pre erred to oral administration because morphine
a. is not absorbed orally.
b. administered orally is rapidly eliminated in the urine.
c. administered orally does not penetrate the brain.
d. administered orally causes more severe constipation.
e. administered parenterally bypasses rst-pass metabolism.

QUESTION 10-6 T e morphine metabolite that may be responsible or most o mor-


phines analgesic activity is
a. desmethylmorphine.
b. morphine-6-glucuronide.
c. morphine sul ate.
d. N-acetylmorphine.
e. hydroxymorphine.

QUESTION 10-7 A 20-year-old man is given codeine to relieve the pain o a sprained
ankle. A er two days the man returns to his doctor saying that the codeine is not e ec-
tive. A likely cause is due to
a. poor absorption.
b. rapid urinary excretion.
c. poor metabolism.
d. high protein binding.
e. poor brain penetration.

QUESTION 10-8 A 53-year-old man is requesting meperidine or his chronic back pain.
His physician is reluctant to use meperidine or the treatment o chronic pain because o
a. metabolite toxicity.
b. poor oral absorption.
c. increased addiction potential.
d. patient noncompliance.
e. likelihood that meperidine will be diverted or sale on the street.
206
Opioid Pharmacology CHAPTER 1 0

QUESTION 10-9 A 32-year-old woman has been taking an oxycodone-acetaminophen


product or chronic arthritic pain. On a ollow-up appointment her serum aminotrans-
erases are elevated and she is slightly jaundiced. She admits to increasing the dose o
the oxycodone combination product over the past several weeks. T e most likely cause
o her liver injury is
a. oxycodone.
b. hepatitis A.
c. hepatitis C.
d. arthritis.
e. acetaminophen.

SUMMARY QUIZ ANSWER KEY


QUESTION 10-1 Answer is d. Fentanyl citrate and su entanil citrate have widespread
popularity as anesthetic adjuvants (see Chapter 11). T ey are used commonly either
intravenously, epidurally, or intrathecally. T e analgesic e ects o entanyl and su en-
tanil are similar to those o morphine and other opioids. Fentanyl is ~100 times more
potent than morphine, and su entanil is ~1000 times more potent than morphine. T e
time to peak analgesic e ect a er intravenous administration o entanyl and su entanil
(~5 minutes) is notably less than that or morphine and meperidine (~15 minutes).
Recovery rom analgesic e ects also occurs more quickly.
QUESTION 10-2 Answer is b. Although absorbed readily rom the GI tract, naloxone
is almost completely metabolized by the liver be ore reaching the systemic circulation
and thus must be administered parenterally. T e drug is absorbed rapidly rom par-
enteral sites o injection and is metabolized in the liver primarily by conjugation with
glucuronic acid; other metabolites are produced in small amounts. T e t 1/2 o naloxone
is ~1 hour, but its clinically e ective duration o action can be even less.
QUESTION 10-3 Answer is a. Opioids can be absorbed through the oral mucosa more
rapidly than through the stomach. Bioavailability is greater owing to avoidance o rst-
pass metabolism, and lipophilic opioids are better absorbed by this route than are hydro-
philic compounds such as morphine. A transmucosal delivery system that suspends
entanyl in a dissolvable sugar-based lollipop or rapidly dissolving buccal tablet have been
approved or the treatment o cancer pain; in this setting, transmucosal entanyl relieves
pain within 15 minutes, and patients easily can titrate the appropriate dose.
QUESTION 10-4 Answer is c. Sustained administration o an opiate agonist (days to
weeks) leads to progressive loss o drug e ect. Here tolerance re ers to a decrease in the
apparent e ectiveness o a drug with continuous or repeated agonist administration,
and with the removal o the agonist tolerance disappears over several weeks. T is toler-
ance is ref ected by a reduction in the maximum achievable e ect or a right-shi in the
dose-e ect curve.
QUESTION 10-5 Answer is e. With most opioids, including morphine, the e ect o
a given dose is less a er oral than a er parenteral administration because o variable
but signi cant rst-pass metabolism in the liver. For example, the bioavailability o
oral preparations o morphine is only ~25%. T e shape o the time-e ect curve also
varies with the route o administration, so the duration o action o en is somewhat
longer with the oral route. I adjustment is made or variability o rst-pass metabo-
lism and clearance, adequate relie o pain can be achieved with oral administration
o morphine.
QUESTION 10-6 Answer is b. Morphine-6-glucuronide has pharmacological actions
indistinguishable rom those o morphine. Morphine-6-glucuronide given systemi-
cally is approximately twice as potent as morphine in animal models and in humans.
With chronic administration, the 6-glucuronide accounts or a signi cant portion
(Continued)
207
SECTION II Neuropharmacology

o morphines analgesic actions. Indeed, with chronic oral dosing, the blood levels o
morphine-6-glucuronide typically exceed those o morphine. Given its greater MOR
potency and its higher concentration, morphine-6-glucuronide may be responsible or
most o morphines analgesic activity in patients receiving chronic oral morphine.
QUESTION 10-7 Answer is c. Codeine has an exceptionally low a nity or opioid
receptors, and the analgesic e ect o codeine is due to its conversion to morphine.
CYP2D6 catalyzes the conversion o codeine to morphine. Well-characterized genetic
polymorphisms in CYP2D6 lead to the inability to convert codeine to morphine thus
making codeine ine ective as an analgesic or ~10% o the Caucasian population.
QUESTION 10-8 Answer is a. Meperidine is N-demethylated to normeperidine,
which then may be hydrolyzed to normeperidinic acid and subsequently conjugated. In
patients or addicts who are tolerant to the depressant e ects o meperidine large doses
repeated at short intervals may produce an excitatory syndrome, including hallucina-
tions, tremors, muscle twitches, dilated pupils, hyperactive ref exes, and convulsions.
T ese excitatory symptoms are due to the accumulation o normeperidine, which has
a t1/2 o 15 to 20 hours, compared to 3 hours or meperidine. As a result o these prop-
erties, meperidine is not recommended or the treatment o chronic pain because o
concerns over metabolite toxicity. It should not be used or longer than 48 hours or in
doses more than 600 mg/d.
QUESTION 10-9 Answer is e. Doses o acetaminophen greater than 4 g/day may
result in hepatotoxicity (see Chapters 2 and 22). T is amount o acetaminophen is
easily reached in patients who are taking an acetaminophen-opiate combination
product, and who escalate their dose o the opiate-acetaminophen analgesic when
tolerance occurs. Recently the FDA has lowered the amount o acetaminophen that
can be ormulated in combination with opiate analgesics. Patients will still have
access to over-the-counter products that contain acetaminophen, which they may
use to supplement their pain therapy.

SUMMARYTABLE OF OPIOID PHARMACOLOGY


TOXICITIES
CLASS AND
SUBCLASSES NAMES CLINICAL USES COMMON UNIQUE; CLINICALLY IMPORTANT

Opioid Morphine Treatment o acute and chronic pain Nausea, vomiting, Respiratory depression (see Side
Receptor Codeine dizziness, constipation, Bar FACTORS EXACERBATING
Agonist Hydromorphone pruritus, tolerance, and OPIOID-INDUCED RESPIRATORY
Hydrocodone physical dependence DEPRESSION)
Oxycodone Mental clouding, urinary retention,
Oxymorphone hypotension
Meperidine Propoxyphene has been associated
with cardiac toxicity

Methadone Long-acting MOR agonist used or


treatment o chronic pain and opioid
abstinence syndrome
Treatment o heroin users

Propoxyphene Used to treat mild to moderate pain

Fentanyl Potent MOR agonist with short


Al entanil duration o action used or acute pain
Su entanil and as an adjunct to general anesthesia
Remi entanil Fentanyl topical patch and other orms
o oral mucosal delivery are used or
chronic pain
Remi entanil is used or short pain ul
procedures requiring rapid onset and
o set

208
Opioid Pharmacology CHAPTER 1 0

TOXICITIES
CLASS AND
SUBCLASSES NAMES CLINICAL USES COMMON UNIQUE; CLINICALLY IMPORTANT

Levorphanol Available or IV, IM, and PO


administration as an analgesic

Tramadol Synthetic codeine analogues that are Nausea, vomiting, Less respiratory depression than
Tapentadol weak MOR agonists dizziness, dry mouth, equi-analgesic doses o morphine
Used to treat mild to moderate pain sedation and headache Abrupt discontinuation may
Possess monoamine reuptake inhibitor Less constipating than precipitate withdrawal syndrome
activity equivalent doses o
morphine

Opioid Nalbuphine Generally considered to be a KOR Should not be used in combination


Receptor agonist and MOR antagonist that is with MOR agonists
Agonist/ used as an analgesic
Antagonists
and Partial Butorphanol Generally considered to be a KOR
Agonists agonist and MOR antagonist that is
used to treat acute pain

Pentazocine Used to treat mild to moderate pain

Buprenorphine MOR partial agonist used as an May produce an abstinence


analgesic syndrome in patients taking MOR
Fixed-dose combination with naloxone agonist or several weeks
is used to treat opioid dependence

Antidiarrheal Diphenoxylate Used to treat diarrhea Available only in combination with


Agents (see Chapter 33) atropine because o abuse liability

Di enoxin

Loperamide Low abuse liability because o poor


solubility and CNS penetration

Opioid Naloxone Used to treat opioid-induced toxicity Will precipitate a withdrawal


Receptor Naltrexone syndrome in long-term opiate users
Antagonists
Methylnaltrexone Used in the management o opioid- Does not alter central opiate actions
induced constipation Reports o GI per oration

Antitussives Dextromethorphan Over-the-counter treatment or Produces ewer subjective Should not be used to suppress
cough o ten in combination with or GI side e ects than a productive cough
antihistamines, decongestants, codeine
expectorants, and bronchodilators

Codeine Also used as a cough suppressant (see


Chapter 24)

Benzonatate Used in the treatment o cough Drowsiness, dizziness, and Should not be used to suppress
dysphagia a productive cough

209
CHAPTER

11 AnestheticAgents and
TherapeuticGases
T is chapter will be most use ul a er having a basic understanding o the material in
DRUGS INCLUDED IN
Chapter 19, General Anesthetics and T erapeutic Gases and Chapter 20, Local Anes-
THIS CHAPTER thetics in Goodman & Gilmans T e Pharmacological Basis of T erapeutics, 12th Edition.
Articaine (SEPTOCAINE) In addition to the material presented here, the chapters in the 12th Edition include:
Benzocaine (large number o preparations Details o the general principles o surgical anesthesia
or topical application) A detailed discussion o the mechanisms o anesthesia
Bupivicane (MARCAINE, SENSORCAINE, T e chemical structures o parenteral, inhalational, and local anesthetics
others)
A detailed discussion o therapeutic gases
Carbon dioxide
Figure 20-3 which is a depiction o the local anesthetic receptor site
Chloroprocaine (NESACAINE)
able 20-1 Susceptibility to Block ypes o Nerve Fibers
Cocaine
A detailed discussion o the mechanisms o action o local anesthetics
Des urane (SUPRANE)
A detailed discussion o the various ways that local anesthetics are administered
Dexmedetomidine (PRECEDEX)
Dibucaine (NUPERCAINAL, others) LEARNING OBJECTIVES
Dyclonine (over-the-counter products Learn the mechanisms o action o parenteral, inhalational, and local
such as SUCRETS, ORAJELand OVERNIGHT anesthetics.
COLDSOREPATCH, and SKINSHIELD Understand the ways that anesthetics (parenteral, inhalational, and local) are
LIQUIDBANDAGE) used to acilitate surgical and other pain ul medical procedures.
En urane (ETHRANE)
Know the major pharmacological properties and toxicities o anesthetic drugs.
Etomidate (AMIDATE)
Fospropo ol (LUSEDRA)
Halothane (FLUOTHANE) MECHANISMS OF ACTION OF ANESTHETIC AGENTS
Helium DRUG CLASS DRUG MECHSM OF ACTION
Iso urane (FORANE, others) Parenteral Anesthetics Sodium thiopental Barbiturates act on GABAA
Ketamine (KETALAR) Methohexital receptors (see Chapter 9)
Lidocaine (XYLOCAINE, others); transdermal Propo ol Sedative and hypnotic
patch (LIDODERM); oral patch (DENTI- Fospropo ol The action o propo ol is
PATCH); combination with prilocaine in an mediated by its action on
occlusive dressing (EMLA); combination GABAA receptors to increase
with tetracaine (PLIAGIS) chloride conduction and
hyperpolarize neurons;
Mepivacaine (CARBAOCAINE, POLOCAINE, ospropo ol is a prodrug o
others) propo ol and has a similar
Methohexital (BREVITAL) mechanism o action
Nitricoxide Etomidate Modulator o GABAA receptors
Nitrous oxide Ketamine Inhibits NMDA receptors (see
Oxygen Chapter 5)
Pramoxine (various preparations including Inhalational Anesthetics Halothane See Side Bar MECHANISMS OF
creams, lotions, sprays, gel, wipes, and Iso urane GENERAL ANESTHESIA
oams available or topical application) En urane
Prilocaine (CITANEST) Des urane
Sevo urane
Procaine (NOVOCAINE)
Proparacaine (ALCAINE, OPHTHAINE, others) Nitrous oxide Potent and selective inhibitor
o NMDA-activated currents
Propo ol (DIPRIVAN)
(continues) (Continued)
210
Anesthetic Agents and Therapeutic Gases CHAPTER 1 1

DRUG CLASS DRUG MECHANISM OF ACTION DRUGS INCLUDED IN


Anesthetic Adjuncts Xenon Potent and selective inhibitor THIS CHAPTER (Cont.)
o NMDA-activated currents
Ropivacaine (NAROPIN, others)
Benzodiazepines See Chapter 9 Sevo urane (ULTRANE, others)
Dexmedetomidine Selective 2-adrenergic Tetracaine (PONTOCAINE)
receptor agonist that Thiopental Sodium(PENTOTHAL, others)
produces sedation and
analgesia, but not general Xenon
anesthesia

Analgesics See Chapter 10


MECHANISMS OF GENERAL
Neuromuscular blocking See Chapter 6
agents ANESTHESIAa
Cellular mechanisms
Therapeutic Gases Oxygen Increases tissue oxygenation
Hyperbaric oxygen is Inhalational anesthetics can hyperpolarize
administered at greater neurons.
than atmospheric pressure Inhalational and parenteral anesthetics
to improve oxygenation o
damaged tissues have e ects on synaptictransmission and
lesser e ects on action-potential genera-
Carbon dioxide Product o metabolism tion or propagation.
Hypocarbia constricts
Molecular mechanisms
cerebral vessels
Increase sensitivityo the GABAArecep-
Nitric oxide Critical endogenous cell tor to GABA, thus enhancing inhibitory
signaling molecule that neurotransmission.
activates soluble guanylate
cyclase (see Chapter 1) Ketamine, nitrous oxide, and xenon inhibit
NMDAreceptors, which are glutamate-
Helium Inert gas with low density, low gated cation channels.
solubility, and high thermal
conductivity Halogenated inhalational anesthetics,
xenon, and nitrous oxide activate K+ chan-
Local Anesthetics Lidocaine Local anesthetics act at the nels known as two-pore domain channels
Bupivicaine nerve cell membrane to located in pre-and postsynapticsites.
Articaine prevent the generation and
Chloroprocaine conduction o nerve impulses a
Details canbe foundin Chapter 19ofGoodman
Mepivacaine They do this by direct andGilmansThePharmacologicalBasisof
Prilocaine interaction with voltage-gated Therapeutics, 12th Edition.
Ropivacaine Na+ channels (see Figure 11-4)
Procaine to decrease or prevent the
Tetracaine large transient ux o Na+
Dibucaine that normally is produced by
Dyclonine a slight depolarization o the
Pramoxine membrane (see Chapters 5
Benzocaine and 6)
Proparacaine COMPONENTS OF THE
Cocaine ANESTHETIC STATE
Amnesia
CASE 11-1 Immobilityin response to noxious stimuli
Analgesia
A 32-year-old woman is about to be anesthetized or her rst time or a surgical procedure.
Unconsciousness
a. What is anesthesia? What are important general issues concerning anesthesia to
be aware o ?
General anesthetics depress the CNS to a su cient degree to permit the per or-
mance o surgery and other noxious or unpleasant procedures. T e components
o anesthesia are shown in the Side Bar COMPONEN S OF HE ANES HE IC
S A E. T e general principles o anesthesia are listed in the Side Bar GENERAL
PRINCIPLES OF SURGICAL ANES HESIA.
(Continued)
211
SECTION II Neuropharmacology

GENERAL PRINCIPLES OF 200


SURGICAL ANESTHESIAa 100
100
Minimize potential deleterious direct and

L
m
indirect e ects o anestheticagents and

/
g

AL
techniques t12 = 6.8 + 2.8 10

)
L
m
min
Sustain physiologichomeostasis, includ-

/
g
10 1 5 10 15


(
ing preventing excessive blood loss, tissue Minute s

l
a
t
ischemia, reper usion o ischemictissues,

n
e
t12 = 719 +

p
uid shi ts, exposure to cold environment, 329 min

o
i
h
and impaired coagulation

t
1.0

m
u
Improve postoperative outcomes bychoos-
r
e
S
ing techniques that blockor treat compo-
nents o the surgical stress response
0.1
a
Details canbe foundinChapter 19ofGoodman 0 4 8 12 16 20 24
andGilmansThePharmacologicalBasisof Hours
Therapeutics, 12thEdition. FIGURE 11-1 Thiopental serum levels a ter a single intravenous induction dose. Thiopental
serum levels a ter a bolus can be described by 2 time constants, t1/2 and t1/2 . The initial all
is rapid (t1/2<10 minutes) and is due to redistribution o drug rom the plasma and the highly
per used brain and spinal cord into less well-per used tissues such as muscle and at. During this
redistribution phase, serum thiopental concentration alls to levels at which patients awaken
(AL, awakening level; see insetthe average thiopental serum concentration in 12 patients a ter
a 6 mg/kg intravenous bolus o thiopental). Subsequent metabolism and elimination is much
slower and is characterized by a hal -li e (t1/2 ) o more than 10 hours. (Adapted with permission
from Burch PG, and Stanski DR, The role of metabolism and protein binding in thiopental anesthesia.
Anesthesiology, 1983;58:146-152. Copyright Lippincott Williams &Wilkins. http://lww.com.)

Inevitably, anesthetics also suppress normal homeostatic re exes. T e most promi-


nent physiological e ect o anesthesia induction, associated with the majority o
both intravenous and inhalational agents, is a decrease in systemic arterial blood
pressure. Airway maintenance is essential ollowing induction o anesthesia, as
nearly all general anesthetics reduce or eliminate both ventilatory drive and the
re exes that maintain airway patency. T ere ore, ventilation generally must be
assisted or controlled or at least some period during surgery.
Patients commonly develop hypothermia (body temperature <36C) during sur-
gery. Prevention o hypothermia has emerged as a major goal o anesthetic care.
Nausea and vomiting in the postoperative period continue to be signi cant prob-
lems ollowing general anesthesia and are caused by an action o anesthetics on the
chemoreceptor trigger zone and the brainstem vomiting center.
Pain control can be complicated in the immediate postoperative period. T e respi-
ratory suppression associated with opioids can be problematic among postoperative
patients who still have a substantial residual anesthetic e ect. Regional anesthetic
techniques are an important part o a perioperative multimodal approach that
employs local anesthetic wound in ltration, epidural, spinal, and plexus blocks.
Nonsteroidal anti-in ammatory drugs, opioids, 2-adrenergic receptor agonists,
and NMDA-receptor antagonists are commonly used as analgesics.
b. What are the dif erent types o anesthesia?
T e type o anesthesia generally depends on the type o surgery being per ormed and
whether the surgical procedure is being per ormed on an outpatient basis. T ere are par-
enteral, inhalational, and local anesthetics. T ese are discussed in the ollowing cases.
c. What are the mechanisms o general anesthesia?
T e mechanisms by which general anesthetics produce their e ects have remained
one o the great mysteries o pharmacology. T e current thinking on the cellular and
(Continued)

212
Anesthetic Agents and Therapeutic Gases CHAPTER 1 1

molecular mechanisms o anesthesia is shown in the Side Bar MECHANISMS OF


ANES HESIA.
In principle, general anesthetics could interrupt nervous system unction at a
myriad o anatomic sites, including peripheral sensory neurons, the spinal cord,
the brainstem, and the cerebral cortex. Delineation o the precise anatomic sites o
action is di cult because many anesthetics di usely inhibit electrical activity in the
central nervous system (CNS).

CASE 11-2
T e patient in Case 11-1 will be given a parenteral anesthetic as an inducing agent.
a. Why give an inducing agent?
Inhalational anesthetics are of en irritating and onset o anesthesia can be variable.
Consequently, anesthesia is usually begun with a parenteral agent that has a rapid
and predictable onset. T e commonly used parenteral anesthetics and their phar-
macological properties are shown in able 11-1.
b. What are actors that control onset and termination o ef ect o a parenteral
anesthetic?
Hydrophobicity is the key actor governing the pharmacokinetics o parenteral
anesthetics. Af er a single intravenous bolus, these drugs pre erentially partition
into the highly per used and lipophilic tissues o the brain and spinal cord where
they produce anesthesia within a single circulation time. Subsequently blood levels
all rapidly, resulting in drug redistribution out o the CNS back into the blood.
T e anesthetic then di uses into less per used tissues such as muscle and viscera,
and at a slower rate into the poorly per used but very hydrophobic adipose tissue.
ermination o anesthesia af er single boluses o parenteral anesthetics primarily
re ects redistribution out o the CNS rather than metabolism (see Figure 11-1).
Af er redistribution, anesthetic blood levels all according to a complex interaction
between the metabolic rate and the amount and lipophilicity o the drug stored in
(Continued)

TABLE 11-1 Pharmacological Properties of Parenteral Anesthetics


IV INDUCTION MINIMAL HYPNOTIC INDUCTION DOSE t 1/2 CL mL PROTEIN VSS
DRUG FORMULATION DOSE mg/kg LEVEL g/mL DURATION min HOURS min -1 kg -1 BINDING % L/kg
Thiopental 25 mg/mL in aqueous 3-5 15.6 5-8 12.1 3.4 85 2.3
solution + 1.5 mg/mL
Na2CO3; pH = 10-11

Methohexital 10 mg/mL in aqueous 1-2 10 4-7 3.9 10.9 85 2.2


solution + 1.5 mg/mL
Na2CO3; pH = 10-11

Propo ol 10 mg/mL in 10% 1.5-2.5 1.1 4-8 1.8 30 98 2.3


soybean oil, 2.25%
glycerol, 1.2% egg
PL, 0.005% EDTA or
0.025% Na-MBS; pH
= 4.5-7

Etomidate 2 mg/mL in 35% PG; 0.2-0.4 0.3 4-8 2.9 17.9 76 2.5
pH = 6.9

Ketamine 10, 50, or 100 mg/mL 0.5-1.5 1 10-15 3.0 19.1 27 3.1
in aqueous solution;
pH = 3.5-5.5

t1/2, phase hal -li e; CL, clearance; Vss, volume o distribution at steady state; EDTA, ethylenediaminetetraacetic acid; Na-MBS, Na-metabisul te;
PG, propylene glycol; PL, phospholipid.

213
SECTION II Neuropharmacology

150

Dia ze pa m

)
n
i
Thiope nta l

m
(
100

2
1
t
e
v
i
t
i
s
n
Mida zola m

e
S
-
50

xt
Ke ta mine

e
t
n
o
C
Propofol
Etomida te
0
0 1 2 3 4 5 6 7 8
Infus ion Dura tion (hours )

FIGURE 11-2 Context-sensitive hal -time o general anesthetics. The duration o action o
single intravenous doses o anesthetic/hypnotic drugs is similarly short or all and is determined
by redistribution o the drugs away rom their active sites (see Figure 11-1). However, a ter pro-
longed in usions, drug hal -lives and durations o action are dependent on a complex interaction
between the rate o redistribution o the drug, the amount o drug accumulated in at, and the
drugs metabolic rate. This phenomenon has been termed the context-sensitive hal -time; that
is, the t1/2 o a drug can be estimated only i one knows the contextthe total dose and over
what time period it has been given. Note that the hal -times o some drugs such as etomidate,
propo ol, and ketamine increase only modestly with prolonged in usions; others (eg, diazepam
and thiopental) increase dramatically. (Reproduced with permission from Reves JG, Glass PSA, Lubar-
sky DA, et al: Intravenous anesthetics, in Miller RD et al, (eds): Millers Anesthesia, 7th ed. Philadelphia:
Churchill Livingstone, 2010, p 718. Copyright Elsevier.)

the peripheral compartments. T us, parenteral anesthetic hal -lives are context-
sensitive, and the degree to which a t1/2 is contextual varies greatly rom drug to
drug, as might be predicted based on their di ering hydrophobicities and meta-
bolic clearances (see able 11-1 and Figure 11-2). For example, af er a single bolus
o thiopental, patients usually emerge rom anesthesia within 10 minutes; however,
a patient may require more than a day to awaken rom a prolonged thiopental
in usion.
c. What are the dif erences between the parenteral anesthetic agents?
Each o these drugs will produce anesthesia and the choice o which agent to use
is usually based on its pharmacological properties. able 11-1 lists the pharmaco-
logical properties o the parenteral anesthetic agents. Most individual variability
in sensitivity to parenteral anesthetics can be accounted or by pharmacokinetic
actors. For example, in patients with lower cardiac output, the relative per usion
o the brain and the raction o anesthetic dose delivered to the brain are higher;
thus, patients in septic shock or with cardiomyopathy usually require lower doses
o anesthetic. T e elderly also typically require a smaller anesthetic dose, primarily
because o a smaller initial volume o distribution.

CASE 11-3
For the patient in Case 11-1, a er induction o her anesthesia with a parenteral agent
she is to be maintained with an inhalational anesthetic.
a. What are the choices o inhalational anesthetic agents to maintain anesthesia?
able 11-2 lists the widely varying physical properties o the inhalational agents in
clinical use. T ese properties are important because they govern the pharmaco-
kinetics o the inhalational agents. Ideally, an inhalational agent would produce a
rapid induction o anesthesia and a rapid recovery ollowing discontinuation.
(Continued)

214
Anesthetic Agents and Therapeutic Gases CHAPTER 1 1

TABLE 11-2 Properties of Inhalational Anesthetic Agents


EC50c FOR
PARTITION COEFFICIENT AT 37C
ANESTHETIC MACa MACAWAKEb SUPPRESSION OF VAPOR PRESSURE RECOVERED AS
AGENT vol % vol % MEMORY vol % mm Hg at 20C Blood:Gas Brain:Blood Fat:Blood METABOLITES %
Halothane 0.75 0.41 243 2.3 2.9 51 20

Iso urane 1.2 0.4 0.24 250 1.4 2.6 45 0.2

En urane 1.6 0.4 175 1.8 1.4 36 2.4

Sevo urane 2 0.6 160 0.65 1.7 48 3

Des urane 6 2.4 664 0.45 1.3 27 0.02

Nitrous oxide 105 60.0 52.5 Gas 0.47 1.1 2.3 0.004

Xenon 71 32.6 Gas 0.12 0


a
MAC (minimum alveolar concentration) values are expressed as vol %, the percentage o the atmosphere that is anesthetic. A value o MAC
greater than 100% means that hyperbaric conditions would be required.
b
MACawake is the concentration at which appropriate responses to commands are lost.
c
EC50 is the concentration that produces memory suppression in 50% o patients. , Not available.

b. How is potency measured or inhalational anesthetics?


T e potency o general anesthetic agents usually is measured by determining the concen-
tration o general anesthetic that prevents movement in response to surgical stimula-
tion. For inhalational anesthetics, anesthetic potency is measured in minimum alveolar
concentration (MAC) units, with 1 MAC de ned as the minimum alveolar concen-
tration that prevents movement in response to surgical stimulation in 50% o subjects.
c. What are the pharmacokinetic principles o inhalational anesthetics?
In considering the pharmacokinetics o anesthetics, one important parameter is
the speed o anesthetic induction. Anesthesia is produced when anesthetic partial
pressure in the brain is equal to or greater than MAC. Because the brain is well per-
used, anesthetic partial pressure in brain becomes equal to the partial pressure in
alveolar gas (and in blood) over the course o several minutes. T ere ore, anesthesia
is achieved shortly af er alveolar partial pressure reaches MAC. While the rate o
rise o alveolar partial pressure will be slower or anesthetics that are highly soluble
in blood and other tissues, this limitation on speed o induction can be overcome
largely by delivering higher inspired partial pressures o the anesthetic.
Elimination o inhalational anesthetics is largely the reverse process o uptake. For
agents with low blood and tissue solubility, recovery rom anesthesia should mirror
anesthetic induction, regardless o the duration o anesthetic administration. For inha-
lational agents with high blood and tissue solubility, recovery will be a unction o the
duration o anesthetic administration. T is occurs because the accumulated amounts
o anesthetic in the at reservoir will prevent blood (and there ore alveolar) partial
pressures rom alling rapidly. Patients will be arousable when alveolar partial pressure
reaches MACawake, a partial pressure somewhat lower than MAC (see able 11-2).
d. What governs the speed o induction o an inhalational anesthetic?
It is essential to understand that inhalational anesthetics distribute between tis-
sues (or between blood and gas) such that equilibrium is achieved when the partial
pressure o anesthetic gas is equal in all tissues. When a person has breathed an
inhalational anesthetic or a su ciently long time that all tissues are equilibrated
with the anesthetic, the partial pressure o the anesthetic in all tissues will be equal
to the partial pressure o the anesthetic in inspired gas. Note, however, that while
(Continued)
215
SECTION II Neuropharmacology

1 Nitrous 1
Oxide De s flura ne

S evoflura ne

Is oflura ne

I
Ha lotha ne

F
0.5 0.5

/
A
F
0 0
0 10 20 30
Minute s

FIGURE 11-3 Uptake o inhalational general anesthetics. The rise in end-tidal alveolar (FA)
anesthetic concentration toward the inspired (FI) concentration is most rapid with the least
soluble anesthetics, nitrous oxide and des urane, and slowest with the most soluble anesthetic,
halothane. All data are rom human studies. (Reproduced with permission from Eger EI, II: Inhaled
anesthetics: Uptake and distribution, in Miller RD et al, (eds): Millers Anesthesia, 7th ed. Philadelphia:
Churchill Livingstone, 2010, p 540. Copyright Elsevier.)

the partial pressure o the anesthetic may be equal in all tissues, the concentration
o anesthetic in each tissue will be di erent. Indeed, anesthetic partition coe -
cients are de ned as the ratio o anesthetic concentration in 2 tissues when the
partial pressures o anesthetic are equal in the 2 tissues. Blood:gas, brain:blood,
and at:blood partition coe cients or the various inhalational agents are listed in
able 11-2. T ese partition coe cients show that inhalational anesthetics are more
soluble in some tissues (eg, at) than they are in others (eg, blood), and that there is
signi cant range in the solubility o the various inhalational agents in such tissues.
For example, des urane has a very low blood:gas partition coe cient (0.42) and
also is not very soluble in at or other peripheral tissues (see able 11-2). For this
reason, the alveolar (and blood) concentration rapidly rises to the level o inspired
concentration (see Figure 11-3). Indeed, within 5 minutes o administration, the
alveolar concentration reaches 80% o the inspired concentration. T is provides or
a very rapid induction o anesthesia and or rapid changes in depth o anesthesia
ollowing changes in the inspired concentration. Emergence rom anesthesia also is
very rapid with des urane. T e time to awakening ollowing des urane is shorter
than with halothane or sevo urane and usually does not exceed 5 to 10 minutes in
the absence o other sedative agents.

CASE 11-4
During anesthesia or the patient in Case 11-1, she will require endotracheal intubation.
a. What drugs are chosen or this?
Endotracheal intubation requently requires the administration o a neuromuscular
blocking drug to relax the muscles o the jaw. T e pharmacology o these drugs is
discussed in Chapter 6.
b. Why was the patient in Case 11-1 administered a benzodiazepine sedative prior
to her surgery?
As adjuncts, benzodiazepines are used or anxiolysis, amnesia, and sedation prior
to induction o anesthesia or or sedation during procedures not requiring general
anesthesia. T e benzodiazepine most requently used in the perioperative period is
midazolam ollowed distantly by diazepam, and lorazepam. T e pharmacology o
these drugs is discussed in Chapter 9.
(Continued)
216
Anesthetic Agents and Therapeutic Gases CHAPTER 1 1

c. Why will this patient require an analgesic prior to surgery, during the surgery,
USES OF LOCAL
and during the immediate postoperative period?
ANESTHETICSa
With the exception o ketamine, neither parenteral nor currently available inha-
lational anesthetics are e ective analgesics. T us, analgesics typically are admin- Topical
istered with general anesthetics to reduce anesthetic requirement and minimize Local infltration
hemodynamic changes produced by pain ul stimuli. Opioids are the primary Field block
analgesics used during the perioperative period because o the rapid and pro ound
Nerve block
analgesia they produce. During the perioperative period, opioids of en are given
at induction to preempt responses to predictable pain ul stimuli (eg, endotracheal Intravenous regional anesthesia
intubation and surgical incision). Subsequent doses either by bolus or by in usion Spinal anesthesia
are titrated to the surgical stimulus and the patients hemodynamic response. Epidural anesthesia
a
Details ofeachcanbe foundinChapter 20of
CASE 11-5 GoodmanandGilmansThePharmacologicalBasis
A 33-year-old man is about to undergo a dental procedure. His dentist says that the ofTherapeutics, 12thEdition.
local anesthetic he will be using contains epinephrine and asks i there is any history
o a heart condition or high blood pressure.
a. What is the mechanism o action o local anesthetics?
Local anesthetics act at the cell membrane to prevent the generation and the conduction
o nerve impulses. Local anesthetics block conduction by decreasing or preventing the
large transient increase in the permeability o excitable membranes to Na+ that normally
is produced by a slight depolarization o the membrane. T is action o local anesthetics is
due to their direct interaction with voltage-gated Na+ channels (see Figure 11-4). As the
anesthetic action progressively develops in a nerve, the threshold or electrical excitability
gradually increases, the rate o rise o the action potential declines, impulse conduction
slows, and the sa ety actor or conduction decreases. T ese actors decrease the prob-
ability o propagation o the action potential, and nerve conduction eventually ails.
b. Why would the local anesthetic contain epinephrine?
T e duration o action o a local anesthetic is proportional to the time o contact
with nerve. Consequently, maneuvers that keep the drug at the nerve prolong the
period o anesthesia. In clinical practice, a vasoconstrictor, usually epinephrine, is
of en added to local anesthetics. T e vasoconstrictor per orms a dual service. By
decreasing the rate o absorption, it not only localizes the anesthetic at the desired
site, but also allows the rate at which it is destroyed in the body to keep pace with the
rate at which it is absorbed into the circulation. T is reduces its systemic toxicity.
c. Are their any concerns about the use o epinephrine with local anesthetics?
Some o the vasoconstrictor agents used with local anesthetics may be absorbed system-
ically, occasionally to an extent su cient to cause untoward reactions. T ere also may
be delayed wound healing, tissue edema, or necrosis af er local anesthesia. T ese e ects
seem to occur partly because sympathomimetic amines increase the oxygen consump-
tion o the tissue; this, together with the vasoconstriction, leads to hypoxia and local tis-
sue damage. T e use o vasoconstrictors in local anesthetic preparations or anatomical
regions with limited collateral circulation could produce irreversible hypoxic damage,
tissue necrosis, and gangrene, and there ore are contraindicated. It should be stressed
that untoward cardiovascular e ects o local anesthetic agents may result rom their
inadvertent intravascular administration, especially i epinephrine also is present.

CASE 11-6
A 54-year-old man is having a procedure or which he has been told he will receive
spinal anesthesia.
a. What is spinal anesthesia?
Spinal anesthesia occurs ollowing the injection o local anesthetic into the cere-
brospinal uid (CSF) in the lumbar space. For a number o reasons, including the
(Continued)
217
SECTION II Neuropharmacology

A 1 s ubunit s ubunit

N I II III IV N

outs id e

me mb ra ne

ins id e
C C
Volta ge C
s e ns ing
S 4 tra ns me mbra ne ina ctiva tion
s e gme nt re gion
+
N
ne utra l

P KA s ite

P KC s ite

Ina ctiva tion trime r

Glycosyla tion s ite


modula tion by P KA, P KC Pore re gion

B clos e d (not conducting) ope n ina ctiva te d

FIGURE 11-4 Structure and unction o voltage-gated Na+ channels. A. A two-dimensional representation o the (center), 1 (le t), and 2
(right) subunits o the voltage-gated Na+ channel rom mammalian brain. The polypeptide chains are represented by continuous lines with
length approximately proportional to the actual length o each segment o the channel protein. Cylinders represent regions o transmembrane
helices. indicates sites o demonstrated N-linked glycosylation. Note the repeated structure o the 4 homologous domains (I-IV) o the
subunit. Voltage Sensing. The S4 transmembrane segments in each homologous domain o the subunit serve as voltage sensors. (+)
represents the positively charged amino acid residues at every third position within these segments. Electrical eld (negative inside) exerts a
orce on these charged amino acid residues, pulling them toward the intracellular side o the membrane; depolarization allows them to move
outward. Pore. The S5 and S6 transmembrane segments and the short membrane-associated loop between them (P loop) orm the walls o the
pore in the center o an approximately symmetrical square array o the 4 homologous domains (see panel B). The amino acid residues indicated
by circles in the P loop are critical or determining the conductance and ion selectivity o the Na+ channel and its ability to bind the extracellular
pore-blocking toxins tetrodotoxin and saxitoxin. Inactivation. The short intracellular loop connecting homologous domains III and IVserves as
the inactivation gate o the Na+ channel. It is thought to old into the intracellular mouth o the pore and occlude it within a ew milliseconds
a ter the channel opens. Three hydrophobic residues (isoleucinephenylalaninemethionine; IFM) at the position marked h appear to serve as
an inactivation particle, entering the intracellular mouth o the pore and binding to an inactivation gate receptor there. Modulation. The gating
o the Na+ channel can be modulated by protein phosphorylation. Phosphorylation o the inactivation gate between homologous domains III
and IVby PKC slows inactivation. Phosphorylation o sites in the intracellular loop between homologous domains I and II by either PKC or PKA
reduces Na+ channel activation. (Adapted with permission from Catterall W, From ionic currents to molecular mechanisms: the structure and function
of voltage-gated sodium channels. Neuron, 2000;26:13-25. Copyright Elsevier.) B. The 4 homologous domains o the Na+ channel subunit
are illustrated as a square array, as viewed looking down on the membrane. The sequence o con ormational changes that the Na+ channel
undergoes during activation and inactivation is diagrammed. Upon depolarization, each o the 4 homologous domains sequentially undergoes
a con ormational change to an activated state. A ter all 4 domains have activated, the Na+ channel can open. Within a ew milliseconds a ter
opening, the inactivation gate between domains III and IVcloses over the intracellular mouth o the channel and occludes it, preventing urther
ion conductance.

ability to produce anesthesia o a considerable raction o the body with a dose o


local anesthetic that produces negligible plasma levels, spinal anesthesia remains
one o the most popular orms o anesthesia.
Currently in the United States, the drugs most commonly used in spinal anesthesia
are lidocaine, tetracaine, and bupivacaine. General guidelines are to use lidocaine
or short procedures, bupivacaine or intermediate to long procedures, and tetra-
caine or long procedures.
(Continued)
218
Anesthetic Agents and Therapeutic Gases CHAPTER 1 1

b. What problems might occur with spinal anesthesia?


Persistent neurological de cits ollowing spinal anesthesia are extremely rare. T or-
ough evaluation o a suspected de cit should be per ormed in collaboration with a
neurologist. Neurological sequelae can be both immediate and late. Possible causes
include introduction o oreign substances (such as disin ectants or talc) into the
subarachnoid space, in ection, hematoma, or direct mechanical trauma.
A more common sequela ollowing any lumbar puncture, including spinal anes-
thesia, is a postural headache with classic eatures. T e incidence o headache
decreases with increasing age o the patient and decreasing needle diameter. Head-
ache ollowing lumbar puncture must be thoroughly evaluated to exclude serious
complications such as meningitis. reatment usually is conservative, with bed rest
and analgesics.
c. What are the other ways that local anesthetics can be administered?
Other ways that local anesthetics are used during surgical procedures are shown in
the Side Bar USES OF LOCAL ANES HE ICS. T e details o each o these can be
ound in Chapter 20 o Goodman and Gilmans T e Pharmacological Basis of T era-
peutics, 12th Edition.

KEY CONCEPTS
General anesthetics have low therapeutic indices and thus require administra-
tion by specially trained personnel.
All general anesthetics produce a relatively similar anesthetic state, but they are
quite dissimilar in their secondary actions (side e ects) on other organ systems.
A er a single intravenous bolus, a parenteral anesthetic partitions into the
highly per used and lipophilic tissues o the brain and spinal cord where it
produces anesthesia within a single circulation time.
ermination o a parenteral anesthetic a er a single bolus primarily re ects
redistribution out o the CNS (see Figure 11-1).
Parenteral anesthetics ultimately accumulate in atty tissue prolonging recovery
i multiple doses are given.
Inhalation anesthetics have a low sa ety margin.
A general anesthetic is rarely given as the sole agent; anesthetic adjuncts such
as sedatives, analgesics, or neuromuscular blocking agents are used to aug-
ment speci c components o anesthesia (Side Bar COMPONEN S OF ANES-
HE IC S A E).
Local anesthetics bind reversibly to a speci c receptor site within the pore o
Na+ channels in nerves and block ion movement through the pore.
T e duration o action o a local anesthetic is proportional to the time o con-
tact with the nerve; consequently, maneuvers (the addition o a vasoconstrictor
agent such as epinephrine) that keep the drug at the nerve prolong the period
o anesthesia.
Local anesthetics are administered clinically in various ways including topical,
in ltration, eld blocks, nerve blocks, intravenous regional, spinal, and epidural.

SUMMARY QUIZ

QUESTION 11-1 Current evidence supports the view that most intravenous general
anesthetics act predominantly through
a. 5-H receptors.
b. D2 dopamine receptors.
c. GABAA receptors.
219
SECTION II Neuropharmacology

d. 2-adrenergic receptors.
e. 1-adrenergic receptors.

QUESTION 11-2 ermination o anesthesia a er a single bolus o a parenteral


anesthetic primarily re ects
a. redistribution out o the CNS.
b. metabolism by the liver.
c. excretion by the kidney.
d. enhanced protein plasma protein binding.
e. hydrolysis by plasma esterases.

QUESTION 11-3 A 23-year-old man is having outpatient surgery or a torn anterior


cruciate ligament as the result o a skiing injury. He is most likely to receive which o
the ollowing drugs as a parenteral anesthetic?
a. Phenobarbital
b. Diazepam
c. Propranolol
d. Lidocaine
e. Propo ol

QUESTION 11-4 A 35-year-old woman is anesthetized with des urane or outpatient


surgery. T e induction with des urane is rapid because des urane is not very soluble in
at and because des urane
a. has a high blood:gas partition coef cient.
b. is highly protein bound.
c. is metabolized rapidly to an active metabolite.
d. increases cerebral vascular resistance.
e. has a low blood:gas partition coef cient.

QUESTION 11-5 I the patient in Question 11-4 is anesthetized with equipment in


which the CO2 absorbent is not well hydrated, she is at risk o poisoning with
a. NO.
b. O2.
c. CO.
d. H 2S.
e. He.

QUESTION 11-6 A 19-year-old woman is undergoing dental surgery. Immediately


upon in ltration o the local anesthetic, tetracaine with epinephrine, she begins to
wheeze and have an allergic reaction. T e cause o her allergic reaction is likely
a. epinephrine.
b. sul te.
c. tetracaine.
d. nickel.
e. penicillin.

QUESTION 11-7 A 44-year-old man is undergoing surgery on the h nger o his


right hand. A local anesthetic without epinephrine is used because epinephrine in this
setting may cause
a. methemoglobinemia.
b. renal impairment.
c. liver injury.

220
Anesthetic Agents and Therapeutic Gases CHAPTER 1 1

d. gangrene.
e. necrosis o the optic nerve.

QUESTION 11-8 While eating mussels, a 36-year-old woman develops numbness and
tingling o her lips and tongue. She begins to have dif culty breathing and rapidly
becomes unconscious. Her apparent respiratory paralysis is the result o
a. the block o Na+ channels.
b. serotonin syndrome.
c. malignant hyperthermia.
d. depletion o norepinephrine.
e. inhibition o monoamine oxidase (MAO).

SUMMARY QUIZ ANSWER KEY

QUESTION 11-1 Answer is c. Current evidence supports the view that most intrave-
nous general anesthetics act predominantly through GABAA receptors and perhaps
through some interactions with other ligand-gated ion channels such as NMDA recep-
tors and two-pore K+ channels.

QUESTION 11-2 Answer is a. ermination o anesthesia a er a single bolus o a par-


enteral anesthetic primarily re ects redistribution out o the CNS rather than metabo-
lism (see Figure 11-1).

QUESTION 11-3 Answer is e. Propo ol is the most commonly used parenteral anes-
thetic in the United States. Propo ol is advantageous or procedures where rapid return
to a preoperative mental status is desirable.

QUESTION 11-4 Answer is e. Des urane has a very low blood:gas partition coef cient
(0.42) and also is not very soluble in at or other peripheral tissues (see able 11-2). For
this reason, the alveolar (and blood) concentration rapidly rises to the level o inspired
concentration. Indeed, within 5 minutes o administration, the alveolar concentration
reaches 80% o the inspired concentration (see Figure 11-3). T is provides or a very
rapid induction o anesthesia and or rapid changes in depth o anesthesia ollowing
changes in the inspired concentration. Emergence rom anesthesia also is very rapid
with des urane or the same reasons.

QUESTION 11-5 Answer is c. Inhaled anesthetics are administered via a circle sys-
tem circuit that permits unidirectional ow o gas. T is system permits rebreathing
o exhaled gases that contain CO2. o prevent rebreathing o CO2 (which can lead to
hypercarbia), CO2 absorbers are incorporated into the anesthesia delivery circuits. T ese
CO2 absorbers contained either Ca(OH)2 or Ba(OH)2 and smaller quantities o more
potent alkalis, NaOH and KOH. Interaction o inhaled anesthetics with these strong
alkalis results in the ormation o CO. T e amount o CO produced is insigni cant as
long as the CO2 absorbent is suf ciently hydrated. With almost complete desiccation o
the CO2 absorbents, substantial quantities o CO can be produced. T is e ect is greatest
with des urane and can be prevented by the use o well-hydrated, resh CO2 absorbent.

QUESTION 11-6 Answer is b. Although allergic responses to agents o the amide type
are uncommon, solutions o such agents may contain preservatives such as methylpara-
ben that may provoke an allergic reaction. Local anesthetic preparations containing a
vasoconstrictor, such as epinephrine, also may elicit allergic responses due to the sul te
added as an antioxidant or the catecholamine/vasoconstrictor.

QUESTION 11-7 Answer is d. Epinephrine-containing solutions should not be


injected into tissues supplied by end arteries or example, ngers and toes, ears, the
nose, and the penis. T e resulting vasoconstriction may cause gangrene. For the same
(Continued)
221
SECTION II Neuropharmacology

reason, epinephrine should be avoided in solutions injected intracutaneously. Since


epinephrine also is absorbed into the circulation, its use should be avoided in those or
whom adrenergic stimulation is undesirable.

QUESTION 11-8 Answer is a. T is womans symptoms are consistent with ingestion


o saxitoxin in the mussels she is eating. Saxitoxin blocks the pores o the Na+ channel
(see Figure 11-4). Saxitoxin is elaborated by the dino agellates Gonyaulax catenella and
Gonyaulax tamarensis and retained in the tissues o clams and other shell sh that eat
these organisms. Given the right conditions o temperature and light, the Gonyaulax
may multiply so rapidly as to discolor the ocean, causing the condition known as red
tide. Shell sh eeding on Gonyaulax at this time become extremely toxic to humans
and are responsible or periodic outbreaks o paralytic shell sh poisoning. Saxitoxin in
nanomolar concentrations speci cally blocks the outer mouth o the pore o Na+ chan-
nels in the membranes o excitable cells. As a result, the action potential is blocked.

SUMMARYTABLE: ANESTHETIC AGENTS AND THERAPEUTIC GASES


TOXICITIES
CLASS AND
SUBCLASSES NAMES CLINICAL USES COMMON UNIQUE: CLINICALLY IMPORTANT
Parenteral Sodium thiopental Used or induction o general Excitatory symptoms such Respiratory depression
Anesthetics Methohexital anesthesia as cough, hiccup, muscle Fatal attacks o porphyria in
Protectant against cerebral tremors, twitching, and patients with acute intermittent
ischemia hypertonus upon induction porphyria or variegate porphyria
Dose-dependent decrease
in blood pressure

Propo ol Induction and maintenance Similar to thiopental Slightly greater degree o


Fospropo ol o anesthesia Greater degree o respiratory depression than
Fospropo ol is a prodrug hypotension than with thiopental
o propo ol thiopental Propo ol in usion syndrome
(PRIS) characterized by metabolic
acidosis, hyperlipidemia,
rhabdomyolysis, and enlarged liver

Etomidate Induction o anesthesia in Pain at injection site Respiratory depression is less than
patients at risk or hypotension Myoclonic movements with thiopental
Nausea and vomiting

Ketamine Induction o anesthesia in Emergence delirium characterized


patients at risk or hypotension by hallucinations, vivid dreams,
and bronchospasm and delusions
Pro ound analgesia and amnesia

Inhalational Halothane Used or maintenance o Dose-dependent decrease Fulminant hepatic necrosis


Anesthetics general anesthesia in blood pressure (halothane hepatitis)
Des urane is a widely used Sinus bradycardia Increased intracranial pressure
anesthetic or outpatient
surgery because o its
rapid onset o action and
rapid recovery
Sevo urane is also used or
outpatient anesthesia because
o its rapid recovery pro le

Iso urane Dose-dependent decrease Concentration-dependent


in blood pressure depression o ventilation
Increased intracranial pressure

(Continued)

222
Anesthetic Agents and Therapeutic Gases CHAPTER 1 1

TOXICITIES
CLASS AND
SUBCLASSES NAMES CLINICAL USES COMMON UNIQUE: CLINICALLY IMPORTANT
Inhalational En urane Dose-dependent decrease Concentration-dependent
Anesthetics in blood pressure depression o ventilation
(Cont.) Increased intracranial pressure

Des urane Dose-dependent decrease Concentration-dependent


in blood pressure depression o ventilation
Increased intracranial pressure
Carbon monoxide (CO) production
with CO2 absorbers that are not
su ciently hydrated

Sevo urane Dose-dependent decrease Carbon monoxide (CO) production


in blood pressure with CO2 absorbers that are not
su ciently hydrated
A degradation product produced
by the interaction o sevo urane
with the CO2 absorbent soda
lime has been associated with
nephrotoxicity

Nitrous oxide Weak anesthetic agent with


signi cant analgesic e ects
Used primarily as an adjunct to
other inhalational or intravenous
anesthetics

Xenon Inert gas not approved in


the United States
Use is limited by availability

Anesthetic Benzodiazepines See Chapter 9 See Chapter 9 See Chapter 9


Adjuncts
Dexmedetomidine Short-term sedation in critically Hypotension and
ill patients bradycardia
Sedation prior to and/or
during surgical or other
medical procedures in
nonintubated patients

Analgesics See Chapter 10 See Chapter 10 See Chapter 10

Neuromuscular See Chapter 6 See Chapter 6 See Chapter 6


blocking agents

Therapeutic Oxygen Treatment o tissue hypoxia See Chapter 20 o See Chapter 20 o Goodman and
Gases Goodman and Gilmans The Gilmans The Pharmacological Basis
Pharmacological Basis of of Therapeutics, 12th Edition
Therapeutics, 12th Edition

Carbon dioxide Used or insuf ation during See Chapter 20 o See Chapter 20 o Goodman and
endoscopic procedures Goodman and Gilmans The Gilmans The Pharmacological Basis
Used to ood surgical eld Pharmacological Basis of of Therapeutics, 12th Edition
during cardiac surgery, and Therapeutics, 12th Edition
to adjust pH during cardiac
bypass surgery

Nitric oxide Used to treat persistent


pulmonary hypertension
in the newborn
Selectively dilates pulmonary
vasculature

223
SECTION II Neuropharmacology

TOXICITIES
CLASS AND
SUBCLASSES NAMES CLINICAL USES COMMON UNIQUE: CLINICALLY IMPORTANT
Helium Used or pulmonary unction
testing, the treatment o
respiratory obstruction, laser
airway surgery, as a label in
imaging studies, and or gas
mixtures used or diving at depth

Local Lidocaine Used to provide local anesthesia Drowsiness, tinnitus, Respiratory depression and arrest,
Anesthetics Also used as an antiarrhythmic dysgeusia, dizziness, seizures, coma
agent (see Chapter 18) twitching

Bupivicaine Long-acting local anesthesia More cardiotoxic than


equie ective doses o lidocaine

Articaine Used or local anesthesia

Chloroprocaine Prolonged sensory and


motor block a ter epidural or
subarachnoid administration o
large doses

Mepivacaine Toxicity in neonate due to lower


pH o neonatal blood

Prilocaine May cause methemoglobinemia

Ropivacaine

Procaine Hydrolyzed in vivo to


para-aminobenzoic acid
which inhibits the action
o sul onamides

Tetracaine Potential or systemic toxicity


because o slow metabolism

Dibucaine

Dyclonine

Pramoxine

Benzocaine May cause methemoglobinemia

Proparacaine Local anesthetic or


ophthalmic use

Cocaine Used or topical anesthesia o Potential or abuse


the upper respiratory tract (see Chapter 14)

224
CHAPTER

Pharmacotherapyof the Epilepsies 12


T is chapter will be most use ul a er having a basic understanding o the material in
DRUGS INCLUDED IN
Chapter 21, Pharmacotherapy o the Epilepsies in Goodman & Gilmans T e Pharma-
cological Basis of T erapeutics, 12th Edition. In addition to the material presented here, THIS CHAPTER
the 12th Edition includes: Acetazolamide (DIAMOX,others)
A detailed discussion o the nature and mechanisms o seizures, including models Carbamezepine (TEGRETOL, CARBATROL,
o epilepsy others)
A discussion o the genetic approaches to the epilepsies Clonazepam(KLONOPIN, others)
T e structures o the antiseizure drugs Clorazepate (TRANXENE, others)
A detailed discussion o the general principles that determine the choice o drugs Ethosuximide (ZARONTIN, others)
or therapy o the epilepsies Felbamate (FELBATOL)withdrawn rom
market
LEARNING OBJECTIVES
Fosphenytoin (CEREBYX,others)
Know the di erent seizure types and the mechanisms that are responsible or
Gabapentin (NEURONTIN, others)
each type.
Lacosamide (VIMPAT)
Understand the mechanisms o action o the antiseizure drugs and how they are
used to treat the di erent types o seizures. Lamotrigine (LAMICTAL, others)
Levetiracetam(KEPPRA, others)
Describe the toxicities, adverse e ects, and salient drug interactions o drugs
used to treat the epilepsies. Methsuximide (CELONTIN)
Know the general principles o drug therapy o the epilepsies. Oxcarbazepine (TRILEPTAL, others)
Phenobarbital (LUMINAL, others)
Phenytoin (DILANTIN, others)
MECHANISMS OF ACTION OF ANTISEIZURE DRUGSa
Pregabalin (LYROCA, others)
DRUG CLASS DRUG MECHANISMS OF ACTION
Ru namide (BANZEL)
Hydantoins Phenytoin Enhance ast inactivation o voltage-gated Na+
Fosphenytoin channels (see Figure 12-1) Tiagabine (GABITRIL)
Topiramate (TOPAMAX,others)
Barbiturates Phenobarbital GABAA receptor allosteric modulator (see Figure 12-2)
AMPA/kainate receptor antagonist Valproicacid (DEPAKENE, others)
Vigabatrin (SABRIL)
Iminostilbenes Carbamazepine Enhance ast inactivation o voltage-gated Na+
Oxycarbazepine channels (see Figure 12-1) Zonisamide (ZONEGRAN, others)
GABAA receptor allosteric modulator (see Figure 12-2)

Succinimides Ethosuximide T-type Ca2+ channel blocker (see Figure 12-3)


Methsuximide

Valproic Acid Valproic Acid Enhance ast inactivation o voltage-gated Na+


channel (see Figure 12-1)
T-type Ca2+ channel blocker (see Figure 12-3)

Benzodiazepines Clonazepam GABAA receptor allosteric modulator (see Figure 12-2)


Clorazepate

Other Antiseizure Gabapentin Ligand o 2 subunit o T-type Ca2+ channel


Drugs Pregabalin

Acetazolamide Inhibitor o brain carbonic anhydrase

Lamotrigine Enhances ast inactivation o voltage-gated Na+


channel (see Figure 12-1)
T-type Ca2+ channel blocker (see Figure 12-3)

Levetiracetam Binds to synaptic vesicle protein SV2A


(Continued)
225
SECTION II Neuropharmacology

SUMMARY OF THE DRUG CLASS DRUG MECHANISMS OF ACTION


MECHANISMS OF ACTION Tiagabine GABA uptake inhibitor
OF ANTISEIZURE DRUGS GABA-transaminase inhibitor (see Figure 12-2)

Limit the sustained, repetitive ring o Topiramate Enhances ast inactivation o voltage-gated Na+
voltage-activated Na+ channels (see channel (see Figure 12-1)
Figure 12-1) GABAA receptor allosteric modulator (see Figure 12-2)
AMPA/kainate receptor antagonist
Enhanced -aminobutyricacid (GABA)- Inhibitor o brain carbonic anhydrase
mediated synapticinhibition either bya
presynapticor postsynapticaction (see Felbamate Enhances ast inactivation o voltage-gated Na+
Figure 12-2) channel (see Figure 12-1)
GABAA receptor allosteric modulator (see Figure 12-2)
Inhibition o voltage-activated Ca2+ NMDA receptor antagonist
channels responsible or T-type Ca2+
currents (see Figure 12-3) Zonisamide Inhibitor o T-type Ca2+ channel current (see
Figure 12-3)
Inhibitor o brain carbonic anhydrase

Lacosamide Enhances slow inactivation o voltage-gated Na+


channel (see Figure 12-1)

Ru namide Enhances slow inactivation o voltage-gated Na+


channel (see Figure 12-1)

Vigabatrin GABA uptake inhibitor


GABA-transaminase inhibitor (see Figure 12-2)
a
More details available in Table 21-2 o Goodman and Gilmans The Pharmacologicial Basis of
Therapeutics, 12th Edition.

GABA

viga ba trin GABA-T

s uccinic
va lproa te s e mia lde hyde
s uccinic
s e mia lde hyde
de hydroge na s e
me ta bolite s
tia ga bine
GAT-1
GAT-1
GABA

GABA binding s ite


be nzo-
Ope n Ina ctiva te d dia ze pine s ba rbitura te s

Na + Na +

A A

ca rba ma ze pine I
la motrigine
I phe nytoin Na + va lproa te
Na + topira ma te zonis a mide Cl

FIGURE 12-1 Antiseizure drug-enhanced Na+ channel FIGURE 12-2 Enhanced GABA synaptic transmission. In the presence
inactivation. Some antiseizure drugs (shown in blue text) o GABA, the GABAA receptor (structure on le t) is opened, allowing an
prolong the inactivation o the Na+ channels, thereby in ux o C1, which in turn increases membrane polarization. Some anti-
reducing the ability o neurons to ire at high requen- seizure drugs (show in larger blue text) act by reducing the metabolism o
cies. Note that the inactivated channel itsel appears to GABA. Others act at the GABAA receptor, enhancing C1 in ux in response
remain open, but is blocked by the inactivation gate (I). to GABA. Gabapentin acts presynaptically to promote GABA release; its
A, activation gate. molecular target is currently under investigation. GABA molecules, GABA-T,
GABA transaminase; GAT-1, GABA transporter.

226
Pharmacotherapy o the Epilepsies CHAPTER 1 2

Ca 2+ Ca 2+ GENERAL PRINCIPLES AND


va lproa te e thos uximide
CHOICE OF DRUGS FOR THE
THERAPY OF THE EPILEPSIES
Earlydiagnosis and treatment o seizure
disorders with a single appropriate agent
o ers the best prospect o achieving
prolonged seizure- ree periods with the
lowest risko toxicity.
The ef cacycombined with the unwanted
Ca 2+ Ca 2+
e ects o a given drug determine which
FIGURE 12-3 Antiseizure drug-induced reduction o current through T-type Ca2+ channels. particular drug is optimal or a given
Some antiseizure drugs (shown in blue text) reduce the ow o Ca2+ through T-type Ca2+ channels patient.
thus reducing the pacemaker current that underlies the thalamic rhythm in spikes and waves Unless extenuating circumstances such as
seen in generalized absence seizures.
status epilepticus exist, onlymonotherapy
should be initiated.
Dosage is increased at appropriate inter-
vals as required or control o seizures or as
limited bytoxicity.
I compliance has been con rmed yet
seizures persist, another drug should be
TABLE 12-1 Interactions o Anti-Seizure Drugs with Hepatic Microsomal substituted.
Enzymes I therapywith a second single drug
INDUCES INHIBITS METABOLIZED BY is inadequate, combination therapyis
DRUG CYP UGT CYP UGT CYP UGT warranted.
It is wise to select 2 drugs that each act by
Carbamazepine 2C9/3A Yes 1A2/2C8 No
2C9/3A4 distinct mechanisms.
Side e ects o each drug and the potential
Ethosuximide No No No No ? ? drug interactions also should be consid-
Gabapentin No No No No No No ered when using 2 drugs (see Table 12-1).
Measurement o plasma drug concen-
Lacosamide No No No No 2C19 ?
tration at appropriate intervals greatly
Lamotrigine No Yes No No No Yes acilitates the initial adjustment o dosage
to minimize dose-related adverse e ects
Levetiracetam No No No No No No without sacri cing seizure control.
Oxcarbazepine 3A4/5 Yes 2C19 Weak No Yes Once initiated, antiseizure drugs are typi-
callycontinued or at least 2 years.
Phenobarbital 2C/3A Yes Yes No 2C9/19 No
When discontinuing, medications should
Phenytoin 2C/3A Yes Yes No 2C9/19 No be tapered slowlyover a period o
Pregabalin No No No No No No
several months.

Primidone 2C/3A Yes Yes No 2C9/19 No

Ru namide 3A4 2C9/19 No ? No Yes

Tiagabine No No No No 3A4 No

Topiramate No No 2C19 No

Valproate No No 2C9 Yes 2C9/19 Yes

Vigabatrin No No No No No No

Zonisamide No No No No 3A4 Yes

CYP, cytochrome P450; UGT, uridine diphosphate-glucuronosyltrans erase.

227
SECTION II Neuropharmacology

CASE 12-1
A 12-year-old girl has her rst tonic-clonic seizure while at school. Her seizure was pre-
ceded by lip smacking and lasted about 1 minute during which she lost consciousness.
a. During subsequent neurological tests, her parents ask or a classi cation o epi-
leptic seizures so that they might understand where their child alls in the spec-
trum o epilepsies. What is a use ul classi cation o epilepsies?
able 12-2 classi es epileptic seizures into partial seizures and generalized seizures.
T e type o epileptic seizure is one determinant o the drug selected or therapy.
Apart rom this epileptic seizure classi cation, an additional classi cation speci-
es epileptic syndromes, which re er to a cluster o symptoms requently occurring
together and include seizure types, etiology, age o onset, and other actors. More than
50 distinct epileptic syndromes have been identi ed and categorized into partial versus
generalized epilepsies. T e partial epilepsies may consist o any o the partial seizure
types (see able 12-2) and account or roughly 60% o all epilepsies. T e etiology com-
monly consists o a lesion in some part o the cortex, such as a tumor, developmental
mal ormation, or damage due to trauma or stroke. Such lesions o en are evident on
brain magnetic resonance imaging (MRI). Alternatively, the etiology may be genetic.
T e generalized epilepsies are characterized most commonly by one or more o the
generalized seizure types listed in able 12-2 and account or ~40% o all epilepsies.
T e etiology is usually genetic. T e most common generalized epilepsy is re erred
to as juvenile myoclonic epilepsy, accounting or ~10% o all epileptic syndromes.
T e age o onset is in the early teens, and the condition is characterized by myo-
clonic, tonic-clonic, and o en absence seizures. Like most o the generalized-onset
epilepsies, juvenile myoclonic epilepsy is a complex genetic disorder that is prob-
ably due to inheritance o multiple susceptibility genes; there is a amilial cluster-
ing o cases, but the pattern o inheritance is not mendelian. T e classi cation o
epileptic syndromes guides clinical assessment and management, and in some
instances, selection o antiseizure drugs.
b. What seizure type is this patient exhibiting?
Although it is di cult to make an absolute diagnosis without all o the clinical data,
it would seem that this patient has had a complex partial seizure with a secondarily
generalized tonic-clonic seizure.
c. Should antiseizure medication be started immediately in this patient?
Early diagnosis and treatment o seizure disorders with a single appropriate agent
o ers the best prospect o achieving prolonged seizure- ree periods with the lowest
risk o toxicity. An attempt should be made to determine the cause o the epilepsy
with the hope o discovering a correctable lesion, either structural or metabolic.
T e e cacy combined with the unwanted e ects o a given drug determines which
particular drug is optimal or a given patient.
T e rst decision to make is whether and when to initiate treatment. For example,
it may not be necessary to initiate antiseizure therapy a er an isolated tonic-clonic
seizure in a healthy young adult who lacks a amily history o epilepsy and who has a
normal neurological examination, a normal EEG, and a normal brain MRI scan. T e
odds o seizure recurrence in the next year (15%) are similar to the risk o a drug reac-
tion su ciently severe to warrant discontinuation o medication. On the other hand, a
similar seizure occurring in an individual with a positive amily history o epilepsy, an
abnormal neurological examination, an abnormal EEG, and an abnormal MRI carries
a risk o recurrence approximating 60%, odds that avor initiation o therapy.
d. What other general principles o the treatment o seizure disorders should be
ollowed in this patient?
T e general principles o therapy o the epilepsies are outlined in the Side Bar
GENERAL PRINCIPLES AND CHOICE OF DRUGS FOR HE HERAPY OF
HE EPILEPSIES.

228
Pharmacotherapy o the Epilepsies CHAPTER 1 2

TABLE 12-2 Classif cation o Epileptic Seizures


CONVENTIONAL ANTI- RECENTLY DEVELOPED
SEIZURE TYPE FEATURES SEIZURE DRUGS ANTI-SEIZURE DRUGS
Partial Seizures

Simple partial Diverse mani estations determined by the region


o cortex activated by the seizure (eg, i motor
cortex representing le t thumb, clonic jerking
o le t thumb results; i somatosensory cortex
representing le t thumb, paresthesia o le t thumb Carbamazepine,
results), lasting approximating 20-60 seconds. phenytoin, valproate
Key feature is preservation of consciousness.
Complex partial Gabapentin, lacosamide,
Impaired consciousness lasting 30 seconds to
lamotrigine, levetiracetam,
2 minutes, o ten associated with purposeless
ru namide, tiagabine,
movements such as lip smacking or hand wringing.
Partial with secondarily topiramate, zonisamide
Simple or complex partial seizure evolves into Carbamazepine,
generalized tonic-clonic a tonic-clonic seizure with loss consciousness phenobarbital, phenytoin,
seizure and sustained contractions (tonic) o muscles primidone, valproate
throughout the body ollowed by periods o
muscle contraction alternating with periods o
relaxation (clonic), typically lasting 1-2 minutes.

Generalized Seizures

Absence seizure Abrupt onset o impaired consciousness associated Ethosuximide, valproate, Lamotrigine
with staring and cessation o ongoing activities clonazepam
typically lasting less than 30 seconds.

Myoclonic seizure A brie (perhaps a second), shocklike contraction Valproate, clonazepam Levetiracetam
o muscles that may be restricted to part o one
extremity or may be generalized.

Tonic-clonic seizure As described earlier in table or partial with Carbamazepine, Lamotrigine, levetiracetam,
secondarily generalized tonic-clonic seizures phenobarbital, phenytoin, topiramate
except that it is not preceded by a partial seizure. primidone, valproate

Modi ed with permission rom Leppik IE, Kelly KM, deToledo-Morrell L et al. Basic research in epilepsy and aging. Epilepsy Res, 2006, 68 (Suppl 1): 21.
Copyright Elsevier.

CASE 12-2
T e patient in Case 12-1 had a second tonic-clonic seizure and it was decided that she
should be treated with phenytoin.
a. What is the mechanism o action o phenytoin?
T e mechanisms o action o antiseizure drugs are listed in the able MECHANISMS
OF AC ION OF AN ISEIZURE DRUGS. Phenytoin limits the repetitive ring o
action potentials evoked by a sustained depolarization o mouse spinal cord neurons
maintained in vitro. T is e ect is mediated by a slowing o the rate o recovery o
voltage-activated Na+ channels rom inactivation, an action that is both voltage-
(greater e ect i membrane is depolarized) and use-dependent (see Figure 12-1).
b. What is unique about the elimination o phenytoin?
Phenytoin is one o the ew drugs or which the rate o elimination varies as a
unction o its concentration (ie, the rate is nonlinear). T e plasma t1/2 o phe-
nytoin ranges between 6 and 24 hours at plasma concentrations below 10 g/mL
but increases with higher concentrations; as a result, plasma drug concentration
increases disproportionately as dosage is increased, even with small adjustments or
levels near the therapeutic range.
c. How should phenytoin treatment be monitored?
A good correlation usually is observed between the total concentration o phenyt-
oin in plasma and its clinical e ect. T us, control o seizures generally is obtained
(Continued)
229
SECTION II Neuropharmacology

with total concentrations above 10 g/mL, while toxic e ects such as nystagmus
develop at total concentrations around 20 g/mL.
d. What untoward ef ects might this patient expect rom long-term phenytoin
treatment?
oxic e ects associated with chronic phenytoin treatment are primarily dose-
related cerebellar-vestibular e ects but also include other CNS e ects, behavioral
changes, increased requency o seizures, GI symptoms, gingival hyperplasia, osteo-
malacia, and megaloblastic anemia. Hirsutism is an annoying untoward e ect in
young emales. Usually, these phenomena can be diminished by proper adjustment
o dosage. Serious adverse e ects, including those on the skin, bone marrow, and
liver, probably are mani estations o drug allergy. Although rare, they necessitate
withdrawal o the drug.
Gingival hyperplasia occurs in ~20% o all patients during chronic therapy and is
probably the most common mani estation o phenytoin toxicity in children and
young adolescents. It may be more requent in those individuals who also develop
coarsened acial eatures. T e overgrowth o gingival tissue appears to involve
altered collagen metabolism. oothless portions o the gums are not a ected.
T e condition does not necessarily require withdrawal o medication and can be
minimized by good oral hygiene.
A variety o endocrine e ects have been reported. Inhibition o release o antidi-
uretic hormone (ADH) has been observed in patients with inappropriate ADH
secretion. Hyperglycemia and glycosuria appear to be due to inhibition o insulin
secretion. Osteomalacia, with hypocalcemia and elevated alkaline phosphatase
activity, has been attributed to both altered metabolism o vitamin D and the
attendant inhibition o intestinal absorption o Ca2+.
Hypersensitivity reactions include morbilli orm rash in 2 to 5% o patients and
occasionally more serious skin reactions, including Stevens-Johnson syndrome and
toxic epidermal necrolysis.

CASE 12-3
A 7-year-old boy is diagnosed with absence seizures. T is patient has an older brother
who also has absence seizures that progressed to tonic-clonic seizures within 1 year.
a. What is an absence seizure and what is thought to be the mechanism o seizure
activity in this type o seizure?
In contrast to partial seizures, which arise rom localized regions o the cerebral
cortex, generalized-onset seizures arise rom the reciprocal ring o the thalamus
and cerebral cortex. T e EEG hallmark o an absence seizure is generalized spike-
and-wave discharges at a requency o 3 per second (3 Hz).
T ese reverberatory, low- requency rhythms are made possible by a combination o
actors, including reciprocal excitatory synaptic connections between the neocortex
and thalamus as well as intrinsic properties o neurons in the thalamus. One intrin-
sic property o thalamic neurons that is pivotally involved in the generation o the
3-Hz spike-and-wave discharges is a particular type o Ca2+ current, the low thresh-
old ( -type) current. -type Ca2+ channels are activated at a much more negative
membrane potential (hence low threshold) than most other voltage-gated Ca2+
channels expressed in the brain.
b. What are the mechanisms o action o valproic acid?
T e principal mechanism by which antiabsence-seizure drugs (ethosuximide,
valproic acid) are thought to act is by inhibition o the -type Ca2+ channels (see
Figure 12-3). T us, inhibiting voltage-gated ion channels is a common mechanism
o action among antiseizure drugs, with antipartial-seizure drugs inhibiting
voltage-activated Na+ channels and antiabsence-seizure drugs inhibiting voltage-
activated Ca2+ channels (see Figures 12-1 and 12-3).
(Continued)
230
Pharmacotherapy o the Epilepsies CHAPTER 1 2

c. Should therapy with valproic acid be monitored by plasma concentrations?


Valproate plasma concentrations associated with therapeutic e ects are ~30 to
100 g/mL. However, there is a poor correlation between the plasma concentration
and e cacy.
d. What untoward ef ects might be expected with valproate therapy?
T e most common side e ects are transient GI symptoms, including anorexia, nau-
sea, and vomiting in ~16% o patients. E ects on the CNS include sedation, ataxia,
and tremor; these symptoms occur in requently and usually respond to a decrease
in dosage. Rash, alopecia, and stimulation o appetite have been observed occasion-
ally and weight gain has been seen with chronic valproic acid treatment in some
patients. Valproic acid has several e ects on hepatic unction. Elevation o hepatic
transaminases in plasma is observed in up to 40% o patients and o en occurs
asymptomatically during the rst several months o therapy.
A rare complication is a ulminant hepatitis that is requently atal. Pathological
examination reveals a microvesicular steatosis without evidence o inf ammation or
hypersensitivity reaction. Children younger than 2 years with other medical con-
ditions who were given multiple antiseizure agents were especially likely to su er
atal hepatic injury. At the other extreme, there were no deaths reported or patients
older than the age o 10 years who received only valproate.
Interactions o antiseizure drugs with hepatic microsomal enzymes can be ound
in able 12-1. T is knowledge will help predict drug interactions when using more
than one antiseizure medication.

CASE 12-4
In the patient in Case 12-3, there was initial consideration o treatment with ethosuximide.
a. Why is valproate a better choice or this patient than ethosuximide?
Ethosuximide and valproate are considered equally e ective in the treatment o
absence seizures. Between 50 and 75% o newly diagnosed patients are ree o seizures
ollowing therapy with either drug. I tonic-clonic seizures are present (as with the
sibling o this patient) or emerge during therapy, valproate is the agent o rst choice.
b. What is the mechanism o action o ethosuximide?
Ethosuximide reduces low threshold Ca2+ currents ( -type currents) in thalamic
neurons (see Figure 12-3). T e thalamus plays an important role in generation o
3-Hz spike-and-wave rhythms typical o absence seizures. Neurons in the thalamus
exhibit large-amplitude -type currents that underlie bursts o action potentials
and likely play an important role in thalamic oscillatory activity such as 3-Hz spike-
and-wave activity. At clinically relevant concentrations, ethosuximide inhibits the
-type current, as is evident in voltage-clamp recordings o acutely isolated, ventro-
basal thalamic neurons rom rats and guinea pigs.
c. What side ef ects might have been expected i this patient were treated with
ethosuximide?
T e most common dose-related side e ects o ethosuximide are gastrointestinal
complaints (nausea, vomiting, and anorexia) and CNS e ects (drowsiness, lethargy,
euphoria, dizziness, headache, and hiccough). Some tolerance to these e ects devel-
ops. Parkinson-like symptoms and photophobia also have been reported. Restlessness,
agitation, anxiety, aggressiveness, inability to concentrate, and other behavioral e ects
have occurred primarily in patients with a prior history o psychiatric disturbance.
Urticaria and other skin reactions, including Stevens-Johnson syndrome, as well
as systemic lupus erythematosus, eosinophilia, leukopenia, thrombocytopenia,
pancytopenia, and aplastic anemia also have been attributed to ethosuximide. T e
leukopenia may be transient despite continuation o the drug, but several deaths
have resulted rom bone marrow depression. Renal or hepatic toxicity has not
been reported.
231
SECTION II Neuropharmacology

CASE 12-5
A 3-year-old boy with the diagnosis o Lennox-Gastaut syndrome has been treated
or 6 months with phenytoin. T e phenytoin has been reducing the seizure requency,
but the decision is made to add lamotrigine to the regimen to urther reduce seizure
requency.
a. What is Lennox-Gastaut syndrome?
Lennox-Gastaut syndrome is a disorder o childhood characterized by multiple
seizure types, mental retardation, and re ractoriness to antiseizure medication.
b. What is the mechanism o action o lamotrigine?
Lamotrigine blocks sustained repetitive ring o mouse spinal cord neurons and
delays the recovery rom inactivation o recombinant Na+ channels, mechanisms
similar to those o phenytoin and carbamazepine (see Figure 12-1). T is may
well explain lamotrigines actions on partial and secondarily generalized seizures.
Lamotrigine is e ective against a broader spectrum o seizures than phenytoin and
carbamazepine, suggesting that lamotrigine may have actions in addition to regu-
lating recovery rom inactivation o Na+ channels. T e mechanisms underlying its
broad spectrum o actions are incompletely understood. One possibility involves
lamotrigines inhibition o glutamate release in rat cortical slices treated with
veratridine, a Na+ channel activator, raising the possibility that lamotrigine inhibits
synaptic release o glutamate by acting at Na+ channels themselves.
c. What precaution with the dose o lamotrigine should be taken with this patient?
Patients who are already taking a hepatic enzymeinducing antiseizure drug (such
as carbamazepine, phenytoin, phenobarbital, or primidone, but not valproate)
should be given lamotrigine initially at 50 mg/d or 2 weeks. T e dose is increased
to 50 mg twice per day or 2 weeks and then increased in increments o 100 mg/d
each week up to a maintenance dose o 300 to 500 mg/d divided into 2 doses.
d. What untoward ef ects might be expected rom lamotrigine?
T e most common adverse e ects are dizziness, ataxia, blurred or double vision,
nausea, vomiting, and rash when lamotrigine was added to another antiseizure
drug. A ew cases o Stevens-Johnson syndrome and disseminated intravascular
coagulation have been reported. T e incidence o serious rash in pediatric patients
(~0.8%) is higher than in the adult population (0.3%).

CASE 12-6
A 23-year-old woman is diagnosed with simple partial seizures. reatment with carba-
mazepine is initiated.
a. What is the mechanism o action o carbamazepine?
Like phenytoin, carbamazepine limits the repetitive ring o action potentials
evoked by a sustained depolarization o mouse spinal cord or cortical neurons
maintained in vitro. T is appears to be mediated by a slowing o the rate o recov-
ery o voltage-activated Na+ channels rom inactivation (see Figure 12-1).
T e predominant pathway o metabolism in humans involves conversion to the
10,11-epoxide. T is metabolite is as active as the parent compound in various ani-
mal models, and its concentrations in plasma and brain may reach 50% o those o
carbamazepine, especially during the concurrent administration o phenytoin or
phenobarbital. T e 10,11-epoxide is metabolized urther to inactive compounds,
which are excreted in the urine principally as glucuronides.
b. How should the therapy with carbamazepine be monitored?
T ere is no simple relationship between the dose o carbamazepine and concen-
trations o the drug in plasma. T erapeutic concentrations are reported to be 6 to
(Continued)

232
Pharmacotherapy o the Epilepsies CHAPTER 1 2

12 g/mL, although considerable variation occurs. Side e ects re erable to the CNS
are requent at concentrations more than 9 g/mL.
c. What toxicity might be expected in this patient?
During long-term therapy, the more requent untoward e ects o the drug include
drowsiness, vertigo, ataxia, diplopia, and blurred vision. T e requency o seizures
may increase, especially with overdosage. Other adverse e ects include nausea,
vomiting, serious hematological toxicity (aplastic anemia, agranulocytosis), and
hypersensitivity reactions (dangerous skin reactions, eosinophilia, lymphadenopa-
thy, splenomegaly). A late complication o therapy with carbamazepine is retention
o water, with decreased osmolality and concentration o Na+ in plasma, especially
in elderly patients with cardiac disease.
A transient, mild leukopenia occurs in ~10% o patients during initiation o ther-
apy and usually resolves within the rst 4 months o continued treatment; transient
thrombocytopenia also has been noted. In ~2% o patients, a persistent leukopenia
may develop that requires withdrawal o the drug.
d. Is carbamazepine used or other disorders?
Carbamazepine is the primary agent or treatment o trigeminal and glossopha-
ryngeal neuralgias. It is also e ective or lightning-type (tabetic) pain associ-
ated with bodily wasting. Most patients with neuralgia bene t initially, but only
70% obtain continuing relie . Adverse e ects require discontinuation o medica-
tion in 5 to 20% o patients. T e therapeutic range o plasma concentrations or
antiseizure therapy serves as a guideline or its use in neuralgia. Carbamazepine
is also used in the treatment o bipolar a ective disorders, as discussed urther in
Chapter 8.

KEY CONCEPTS
T erapy o epilepsies is symptomatic in that available drugs inhibit seizures, but
neither e ective prophylaxis nor cure is available.
Reducing the rate o recovery o Na+ channels rom inactivation would limit the
ability o a neuron to re at high requencies, an e ect that likely underlies the
e ects o carbamazepine, lamotrigine, phenytoin, topiramate, valproic acid, and
zonisamide against partial seizures.
T e principal mechanism by which anti-absence-seizure drugs (ethosuximide,
valproic acid) are thought to act is by inhibition o the -type Ca2+ channel.
Activation o the GABAA receptor inhibits the postsynaptic cell by increasing
the ow o Cl- ions into the cell, which tends to hyperpolarize the neuron.
T e antiseizure drug tiagabine inhibits the GABA transporter, GA -1, and
reduces neuronal and glial uptake o GABA.
T e goal o treating patients with epilepsy is to select the appropriate drug or
combination o drugs that best controls seizures at an acceptable level o untow-
ard e ects.
Measurement o drug concentrations in plasma acilitates optimizing antisei-
zure medication, especially when therapy is initiated, a er dosage adjustments,
in the event o therapeutic ailure, when toxic e ects occur, or when multiple-
drug therapy is instituted.
When initiating combination antiseizure therapy, it is wise to select 2 drugs that
act by distinct mechanisms (eg, one that promotes Na+ channel inactivation and
another that enhances GABA-mediated synaptic inhibition).
T e goal o treating status epilepticus is rapid termination o behavioral and
electrical seizure activity; the longer the episode is untreated, the more dif cult
it is to control and the greater the risk o permanent brain damage.

233
SECTION II Neuropharmacology

SUMMARY QUIZ

QUESTION 12-1 A 28-year-old man is being treated with phenytoin or tonic-clonic sei-
zures. His drug plasma concentration is in the low therapeutic range and he is still having
occasional seizures. His dose is increased slightly. Within 2 weeks he is ataxic, lethargic,
and has nystagmus. A repeat o his plasma concentration shows that he is now slightly
above the upper limit o the therapeutic range. T e reason or the dramatic rise in his
plasma concentration ollowing a modest increase in his dose is most likely because o
a. renal ailure.
b. liver ailure.
c. nonlinear elimination.
d. metabolic acidosis.
e. poor GI absorption o Ca2+.

QUESTION 12-2 A 34-year-old woman is being treated with carbamazepine or complex


partial seizures. Carbamazepine is metabolized to an active metabolite, the
a. 10,11-epoxide metabolite.
b. N-acetyl metabolite.
c. desmethyl metabolite.
d. hydroxyl metabolite.
e. para-hydroxy-phenyl metabolite.

QUESTION 12-3 A 19-year-old woman is being treated with ethosuximide, most likely
or which type o seizure?
a. Simple partial
b. Complex partial
c. onic-clonic
d. Absence
e. Status epilepticus

QUESTION 12-4 A 29-year-old woman is being treated with valproic acid or simple
partial seizures. She is at risk or developing a rise in her plasma
a. calcium.
b. hepatic transaminases.
c. blood urea nitrogen (BUN).
d. potassium.
e. glucose.

QUESTION 12-5 A 33-year-old man is taking valproic acid or tonic-clonic seizures.


Lamotrigine is added to improve seizure control. T e plasma concentrations o
lamotrigine may be increased because valproate inhibits
a. hydroxylation.
b. glucuronidation.
c. acetylation.
d. urinary excretion.
e. biliary excretion.

QUESTION 12-6 A 44-year-old man has levetiracetam added to his therapy because he
is re ractory to his current antiseizure regimen. Levetiracetam acts by
a. inactivation o voltage-gated Na+ channels.
b. enhanced GABA synaptic transmission.
(Continued)
234
Pharmacotherapy o the Epilepsies CHAPTER 1 2

c. an unknown mechanism.
d. reducing current through -type Ca2+ channels.
e. antagonizing D2 dopaminergic receptors.

QUESTION 12-7 A 20-year-old woman is taking topiramate as monotherapy or


re ractory generalized tonic-clonic seizures. She should be warned to increase her
dose o
a. levothyroxine.
b. metoprolol.
c. insulin.
d. botox.
e. oral contraceptive.

QUESTION 12-8 A 32-year-old woman is being treated with vigabatrin because her
complex seizures have been re ractory to all other therapies. Vigabatrin is reserved or
use in patients such as this although its availability is restricted due to
a. renal ailure.
b. liver ailure.
c. heart ailure.
d. vision loss.
e. hearing loss.

SUMMARY QUIZ ANSWER KEY

QUESTION 12-1 Answer is c. Phenytoin is one o the ew drugs or which the rate
o elimination varies as a unction o its concentration (ie, the rate is nonlinear). T e
plasma t1/2 o phenytoin ranges between 6 and 24 hours at plasma concentrations below
10 g/mL but increases with higher concentrations; as a result, plasma drug concentra-
tion increases disproportionately as dosage is increased, even with small adjustments
or levels near the therapeutic range.

QUESTION 12-2 Answer is a. T e predominant pathway o metabolism in humans


involves conversion to the 10,11-epoxide. T is metabolite is as active as the parent
compound in various animal models, and its concentrations in plasma and brain may
reach 50% o those o carbamazepine, especially during the concurrent administration
o phenytoin or phenobarbital.

QUESTION 12-3 Answer is d. Ethosuximide is e ective against absence seizures but


not partial or tonic-clonic seizures.

QUESTION 12-4 Answer is b. Elevation o hepatic transaminases in plasma is


observed in up to 40% o patients treated with valproic acid and o en occurs asymp-
tomatically during the rst several months o therapy.

QUESTION 12-5 Answer is b. Lamotrigine is completely absorbed rom the gastro-


intestinal tract and is metabolized primarily by glucuronidation. T e plasma t1/2 o a
single dose is 24 to 30 hours. Administration o phenytoin, carbamazepine, or pheno-
barbital reduces the t1/2 and plasma concentrations o lamotrigine. Conversely, addition
o valproate markedly increases plasma concentrations o lamotrigine, likely by inhibit-
ing glucuronidation.

QUESTION 12 -6 Answer is c. he mechanism by which levetiracetam exerts its


antiseizure e ects is unknown. No evidence or an action on voltage-gated Na+
channels, or either GABA- or glutamate-mediated synaptic transmission has
emerged.

235
SECTION II Neuropharmacology

QUESTION 12-7 Answer is e. Reduced estradiol plasma concentrations occur with


concurrent topiramate, suggesting the need or higher doses o oral contraceptives
when coadministered with topiramate.

QUESTION 12-8 Answer is d. Due to progressive and permanent bilateral vision loss,
vigabatrin must be reserved or patients who have ailed several alternative therapies;
its availability is restricted under the conditions o the SHARE special distribution
program.

SUMMARYTABLE: ANTISEIZURE DRUGS


TOXICITIES
CLASS AND
SUBCLASSES NAMES CLINICAL USES COMMON UNIQUE: CLINICALLY IMPORTANT
Hydantoins Phenytoin Used to treat partial and tonic- Gingival hyperplasia Cardiac arrhythmia
Fosphenytoin clonic seizures Ataxia Osteomalacia
Not active against absence Megaloblastic anemia
seizures Inhibition o insulin secretion
Skin hypersensitivity reactions
including Stevens-Johnson
syndrome

Barbiturates Phenobarbital Used to treat generalized tonic- See Chapter 9 See Chapter 9
clonic and partial seizures

Imminostilbenes Carbamazepine Used to treat generalized tonic- Drowsiness, ataxia, vertigo, Acute intoxication results in
clonic and both simple and diplopia, blurred vision, stupor or coma, hyperirritability,
complex partial seizures nausea, and vomiting convulsions, and respiratory
Transient leukopenia and depression
thrombocytopenia Aplastic anemia
Transient elevation o hepatic
transaminases
May reduce plasma concentrations
o oral contraceptives

Oxcarbazepine Used to treat partial seizures Induction o CYP3A4


and reduction o plasma
concentrations o oral
contraceptives

Succinimides Ethosuximide Ef ective against absence seizures, Nausea, vomiting, anorexia Parkinson-like symptoms,
but not tonic-clonic seizures Drowsiness, lethargy, photophobia, anxiety, aggression,
euphoria, dizziness, and inability to concentrate
headache, hiccough Hypersensitivity skin reactions
including Stevens-Johnson
syndrome
Transient leukopenia
Death rom bone marrow
depression has been reported

Methsuximide No longer in common use

Valproic Acid Valproic acid Broad-spectrum antiseizure Anorexia, nausea, and Inhibits drugs that are substrates
drug ef ective against absence, vomiting or CYP2C9, including phenytoin
myoclonic, partial, and tonic-clonic Sedation, ataxia, and and phenobarbital
seizures tremor Inhibits UGT and thus the
metabolism o lamotrigine and
lorazepam
Elevation o hepatic transaminases
Rarely may cause ulminant
hepatitis

236
Pharmacotherapy o the Epilepsies CHAPTER 1 2

TOXICITIES
CLASS AND
SUBCLASSES NAMES CLINICAL USES COMMON UNIQUE: CLINICALLY IMPORTANT
Benzodiazepines Clonazepam Used to treat absence seizures and See Chapter 9 Tolerance to antiseizure ef ects
myoclonic seizures in children develops a ter 1-6 months

Clorazepate Used in combination with other See Chapter 9


drugs to treat partial seizures

Other Antiseizure Gabapentin Ef ective, when used with other Somnolence, dizziness, Pregnancy Category C
Drugs Pregabalin drugs, against partial seizures ataxia, and atigue

Lamotrigine Used as monotherapy and add-on Dizziness, ataxia, blurred o Stevens-Johnson syndrome
therapy o partial and secondarily double vision, nausea, and and disseminated intravascular
generalized tonic-clonic seizures vomiting coagulation have been reported
in adults and Lennox-Gastaut
syndrome in children and adults

Levetriacetam Used or re ractory partial seizures Somnolence, asthenia, and


or uncontrolled generalized tonic- dizziness
clonic seizures
Adjunctive therapy or re ractory
generalized myoclonic seizures

Tiagabine Ef ective as add-on therapy o Dizziness, somnolence, and May be contraindicated in


re ractory partial seizures with or tremor generalized absence epilepsy
without secondary generalization

Topiramate Used as initial monotherapy (in Somnolence, atigue, Renal calculi due to inhibition o
patients at least 10 years old) weight loss, and carbonic anhydrase
and as adjunctive therapy ( or nervousness Cognitive impairment
patients as young as 2 years o
age) or partial onset or primary
generalized tonic-clonic seizures,
or Lennox-Gastaut syndrome in
patients 2 years o age or older,
and or migraine prophylaxis in
adults

Acetazolamide Sometimes ef ective in absence Use ulness is limited by the rapid


seizures development o tolerance

Felbamate Withdrawn rom the market


because o cases o aplastic
anemia

Zonisamide Add-on therapy or patients with Somnolence, ataxia, Renal calculi due to inhibition o
re ractory partial seizures anorexia, nervousness, and carbonic anhydrase
atigue Metabolic acidosismore
requent and severe in younger
patients

Lacosamide Add-on therapy or adults with


re ractory partial seizures

Ru namide Reduces tonic-clonic seizure


requency in children with
Lennox-Gastaut syndrome

Vigabatrin Adjunctive therapy or re ractory Progressive and permanent


partial complex seizures in adults bilateral vision loss
May also be o limited bene t in Vigabatrin reserved or use in
children with in antile spasms patients who have ailed other
therapies

237
CHAPTER

13 Drug Therapyof
Neurodegenerative Diseases
T is chapter wi be most use u a er having a basic understanding o the materia
DRUGS INCLUDED IN THIS
in Chapter 22, reatment o Centra Nervous System Degenerative Disorders in
CHAPTER Goodman & Gilmans T e Pharmacological Basis o T erapeutics, 12th Edition. In addition
Amantadine (SYMMETREL) to the materia presented here, the 12th Edition inc udes:
Apomorphine (APOKYN) A discussion o the se ective vu nerabi ities o speci c CNS neurons
Baclo en (LIORESAL) A detai ed discussion o the ro o genetics and environment in neurodegenerative
Carbidopa (LODOSYN) disorders
Carbidopa/levodopa (SINEMET,ATAMET, T e common ce u ar mechanisms o neurodegeneration
others; orallydisintegratingtablet, PARCOPA) A detai ed discussion o the neura mechanisms and neuroprotective mechanisms o
Dantrolene (DANTRIUM) parkinsonism
Donepezil (ARICEPT) A comp ete discussion o the genetics o Huntingtons disease
Entacapone (COMTAN; fxed combination Chemica structures o the drugs used to treat and manage neurodegenerative disorders
with carbidopa/levodopa, STAVELO) Figure 22-8 Monosynaptic muscle stretch ref ex with descending control via inhibitory
Galantamine (NIVALIN, others) interneurons
Levodopa (L-DOPA, LARODOPA)
LEARNING OBJECTIVES
Memantine (NAMENDA)
Understand the pathophysio ogy o neurodegenerative diseases, inc uding
Pramipexole (MIRAPEX) Parkinsons disease (PD), A zheimers disease (AD), Huntingtons disease (HD),
Rasagiline (AZILECT) and amyotrophic atera sc erosis (ALS).
Riluzole (RILUTEK) Understand the ro e o drugs in the treatment and management o the symp-
Rivastigmine (EXELON) toms o neurodegenerative diseases.
Ropinirole (REQUIP) Know the mechanisms o action and the adverse ef ects o drugs that are used
Selegiline (ELDEPRYL; oral disintegrating to treat and manage neurodegenerative diseases.
tablet, EMSAM;transdermal patch, ZELAPAR)
Tacrine (COGNEX)discontinued in the MECHANISMS OF ACTION OF DRUGS USED TO TREAT AND MANAGE
United States NEURODENERATIVE DISORDERS
Tetrabenazine (XENAZINENITOMAN)
DRUG CLASS DRUG MECHANISM OF ACTION
Tizanidine (ZANAFLEX)
Drugs Used to Treat Levodopa Decarboxylated in the brain to dopamine (DA)
Tolcapone (TASMAR) Parkinsons Disease

Carbidopa Peripheral DOPA-decarboxylase inhibitor

Carbidopa/levodopa Combination prevents decarboxylation o


combination levodopa in the periphery allowing more to be
available to enter the brain

Entacapone Catechol-O-methyltrans erase (COMT) inhibitor


that blocks the peripheral conversion o
levodopa to 3-O-methyDOPA

Tolcapone Catechol-O-methyltrans erase (COMT) inhibitor


that blocks the peripheral conversion o
levodopa to 3-O-methyDOPA

Ropinirole DA receptor agonist

Pramipexole DA receptor agonist

Selegiline Selective monoamine oxidase B(MAO-B) inhibitor

(Continued)
238
Drug Therapy of Neurodegenerative Diseases CHAPTER 1 3

DRUG CLASS DRUG MECHANISM OF ACTION

Rasagiline Selective monoamine oxidase B (MAO-B)


inhibitor

Amantadine Antiviral agent that alters DA release in the


striatum
Blocks NMDA glutamate receptors

Apomorphine DA receptor agonist

Drugs Used to Treat Donepezil Reversible antagonist o cholinesterase


Alzheimers Disease

Rivastigmine Reversible antagonist o cholinesterase

Galantamine Reversible antagonist o cholinesterase

Tacrine Reversible antagonist o cholinesterase

Memantine Noncompetitive antagonist o the NMDA-type


glutamate receptor

Drugs Used to Treat Tetrabenazine Inhibits the vesicular monoamine transporter


Huntingtons Disease 2 (VMAT2) and causes presynaptic depletion o
catecholamines

Drugs Used to Riluzole Inhibits presynaptic release o glutamate,


Treat ALS blocks postsynaptic NMDA- and kainate-type
glutamate receptors, and inhibits voltage-
dependent Na+ channels

Baclo en GABAB receptor agonist muscle relaxant

Tizanidine 2 adrenergic receptor agonist

Dantrolene Acts directly on muscle bers to impair Ca2+


release rom the sarcoplasmic reticulum

CASE 13 1
A 68-year-o d man is diagnosed with Parkinsons disease (PD).
a. What is the pathophysiology o Parkinsons disease?
T e dopaminergic de cit in PD arises rom a oss o the neurons in the substantia
nigra pars compacta that provide innervation to the striatum (caudate and puta-
men). T e current understanding o the pathophysio ogy o PD is based on the
nding that the striata DA content is reduced in excess o 80%. T is para e ed the
oss o neurons rom the substantia nigra, suggesting that rep acement o DA cou d
restore unction.
CARDINAL FEATURES OF
b. How is dopamine synthesized in neurons? PARKINSON DISEASE (PD)
DA, a catecho amine, is synthesized in the termina s o dopaminergic neurons rom
Bradykinesia (slowness and povertyo
tyrosine and stored, re eased, and metabo ized by processes described in Chapter
movement)
5 and summarized in Figure 13-1. T e actions o DA in the brain are mediated by
a ami y o DA receptor proteins. wo types o DA receptors are identi ed in the Muscular rigidity
mamma ian brain using pharmaco ogica techniques: D1 receptors, which stimu ate Resting tremor (which usuallyabates
the synthesis o the intrace u ar second messenger cyc ic AMP; and D2 receptors, during voluntarymovement)
which inhibit cyc ic AMP synthesis as we as suppress Ca2+ currents and activate Impairment o postural balance leading to
receptor-operated K+ currents. disturbances o gait and alling
(Continued)

239
SECTION II Neuropharmacology

P RESYNAP TIC TERMINAL De pola riza tion


(Na + a nd
Die ta ry Ca 2 + e ntry)
phe nyla la nine
He pa tic
AADC
PH DOPA DA
Tyros ine TH
Tyr
COMT
HVA HVA DOPAC
VMAT2
MAO
ALDH
DA
DA

DA
D2
Ne uro nal re uptake a utore ce ptor
(DAT, NET), the n s tora ge
(VMAT2), or me ta bolis m
Syna ptic DA

Po s ts ynaptic uptake
D1/D2 fa mily [(OCT1, OCT2,
of re ce ptors ENT (OCT3)]

P OSTSYNAP TIC CELL

DA
Effe ctor
re s pons e COMT

ALDH
HVA HVA 3MT
MAO

FIGURE 13-1 Dopaminergic nerve terminal. Dopamine (DA) is synthesized rom tyrosine in
the nerve terminal by the sequential actions o tyrosine hydrolase (TH) and aromatic amino
acid decarboxylase (AADC). DA is sequestered by VMAT2 in storage granules and released
by exocytosis. Synaptic DA activates presynaptic autoreceptors and postsynaptic D1 and D2
receptors. Synaptic DA may be taken up into the neuron via the DA and NE transporters (DAT,
NET), or removed by postsynaptic uptake via OCT3 transporters. Cytosolic DA is subject to
degradation by monoamine oxidase (MAO) and aldehyde dehydrogenase (ALDH) in the
neuron, and by catechol-O-methyltrans erase (COMT) and MAO/ALDH in non-neuronal cells;
the nal metabolic product is homovanillic acid (HVA). See structures in Figure 13-4. PH,
phenylalanine hydroxylase.

c. How is dopamine neurotransmission af ected by Parkinsons disease?


Considerab e e ort has been devoted to understanding how the oss o dopami-
nergic input to the neurons o the neostriatum gives rise to the c inica eatures o
PD. T e basa gang ia can be viewed as a modu atory side oop that regu ates the
ow o in ormation rom the cerebra cortex to the motor neurons o the spina
cord (see Figure 13-2). T e neostriatum is the principa input structure o the basa
gang ia and receives excitatory g utamatergic input rom many areas o the cortex.
Most neurons within the striatum are projection neurons that innervate other basa
gang ia structures. A sma but important subgroup o striata neurons consists o
interneurons that connect neurons within the striatum but do not project beyond
its borders. Acety cho ine (ACh) and neuropeptides are used as transmitters by
these striata interneurons.
(Continued)
240
Drug Therapy of Neurodegenerative Diseases CHAPTER 1 3

T e key eature o this mode o basa gang ia unction, which accounts or the
symptoms observed in PD as a resu t o oss o dopaminergic neurons, is the di -
erentia e ect o DA on the direct and indirect pathways (see Figure 13-3). T e
dopaminergic neurons o the substantia nigra pars compacta (SNpc) innervate a
parts o the striatum; however, the target striata neurons express distinct types o
DA receptors. T e striata neurons giving rise to the direct pathway express pri-
mari y the excitatory D1 dopamine receptor protein, whereas the striata neurons
orming the indirect pathway express primari y the inhibitory D2 type. T us, DA
re eased in the striatum tends to increase the activity o the direct pathway and
reduce the activity o the indirect pathway, whereas the dep etion that occurs in
PD has the opposite e ect. T e net e ect o the reduced dopaminergic input in PD
is to increase marked y the inhibitory out ow rom the SNpc and g obus pa idus
interna (GPi) to the tha amus and reduce excitation o the motor cortex.
d. What is the prognosis or this patient?
Progressive oss o dopamine (DA)-containing neurons is a eature o norma aging;
however, most peop e do not ose the 70 to 80% o dopaminergic neurons required
to cause symptomatic PD. Without treatment, PD progresses over 5 to 10 years
to a rigid, akinetic state in which patients are incapab e o caring or themse ves.
Death requent y resu ts rom comp ications o immobi ity, inc uding aspiration
pneumonia or pu monary embo ism. T e avai abi ity o e ective pharmaco ogica
treatment has radica y a tered the prognosis o PD; in most cases, good unctiona
mobi ity can be maintained or many years. Li e expectancy o adequate y treated
patients is increased substantia y, but overa morta ity remains higher than that o
the genera popu ation. Common y used medications or the treatment o PD are
summarized in ab e 13-1.

+
Ce re bra l corte x
Glu
+ Glu + Glu
+
S tria tum Ce re bra l corte x
VA/VL Glu
DA
ACh tha la mus + Glu + Glu
D1 D2
(s timula tory) (inhibitory)
S tria tum VA/VL
GABA
DA tha la mus
GABA ACh
Glu +
D1 D2
GPe STN GABA
S Npc GABA
GABA Glu + GABA
Glu +
GP i/S Npr
Glu To s p ina l c ord GPe STN
S Npc GABA
+ GABA
a nd b ra ins te m Glu +
GP i/S Npr
FIGURE 13-2 Schematic wiring diagram o the basal ganglia. Glu
The striatum is the principal input structure o the basalganglia and + To s p ina l c ord
a nd b ra ins te m
receives excitatory glutamatergic input rom many areas o cerebral
cortex. The striatum contains projection neurons expressing predom- FIGURE 13-3 The basal ganglia in Parkinsons disease. The pri-
inantly D1 or D2 dopamine receptors, as well as interneurons that use mary de ect is destruction o the dopaminergic neurons o the
acetylcholine (ACh) as a neurotransmitter. Out ow rom the striatum SNpc. The striatal neurons that orm the direct pathway rom the
proceeds along two routes. The direct pathway, rom the striatum to striatum to the SNpr and GPi express primarily the excitatory D1
the substantianigra pars reticulata (SNpr) and globus pallidus interna dopamine receptor, whereas the striatal neurons that project to the
(GPi), uses the inhibitory transmitter GABA. The indirect pathway, GPe and orm the indirect pathway express the inhibitory D2 dopa-
rom the striatum through the globus pallidus externa (GPe) and mine receptor. Thus, loss o the dopaminergic input to the striatum
the subthalamic nucleus (STN) to the SNpr and GPi consists o two has a dif erential ef ect on the two out ow pathways; the direct
inhibitory GABAergic links and one excitatory glutamatergic pro- pathway to the SNpr and GPi is less active (structures in light blue),
jection (Glu). The substantia nigra pars compacta (SNpc) provides whereas the activity in the indirect pathway is increased (structures
dopaminergic innervation to the striatal neurons, giving rise to both in dark blue). The net ef ect is that neurons in the SNpr and GPi
the direct and indirect pathways, and regulates the relative activity become more active. This leads to increased inhibition o the VA/VL
o these two paths. The SNpr and GPi are the output structures o the thalamus and reduced excitatory input to the cortex. (See legend
basal ganglia and provide eedback to the cerebral cortex through to Figure 132 or de nitions o anatomical abbreviations.)
the ventroanterior and ventrolateralnuclei o the thalamus (VA/VL).
241
SECTION II Neuropharmacology

CASE 13 2
T e patient in Case 13-1 is ear y in the course o his Parkinsons disease
a. What are the treatment options at this stage o his disease?
Common y used medications or the treatment o PD are summarized in ab e 13-1.
Pharmaco ogica treatment o PD shou d be tai ored to the individua patient. Drug
therapy is not ob igatory in ear y PD; many patients can be managed or a time
with exercise and i esty e interventions. For patients with mi d symptoms, MAO-B
inhibitors, amantadine, or (in younger patients) anticho inergics are reasonab e
choices. In most patients, treatment with a dopaminergic drug, either evodopa
(Continued)

TABLE 13-1 Commonly Used Medications for the Treatment of Parkinsons Disease
AGENT TYPICAL INITIAL DOSE DAILY DOSE RANGE COMMENTS
Levodopa Formulations

Carbidopa/levodopa 25 mg carbidopa + 100 mg levodopa 200-1200 mg levodopa


(25/100tablet), 2-3x daily

Carbidopa/levodopa 50 mg carbidopa + 200 mg levodopa 200-1200 mg levodopa Bioavailability 75% o immediate-


sustained-release (50/200 sustained-releasetablet) 2x daily release orm

Carbidopa-levodopa orally 25 mg carbidopa + 100 mg levodopa 200-1200 mg levodopa


disintegrating tablets (PARCOPA) (25/100tablet), 2-3x daily

COMT Inhibitors

Entacapone 200 mg with each dose o levodopa/ 600-2000 mg


carbidopa

Tolcapone 100 mg with carbidopa/levodopa 100-300 mg May be hepatotoxic. Use only in


patients not responding satis actorily
to other treatments. Requires
monitoring o liver unction

Carbidopa/levodopa/entacapone 12.5 mg carbidopa + 50 levodopa 150-1200 mg levodopa


+ 200 mg entacapone (STALEVO 50), 3x
daily

DA Agonists

Apomorphine 2 mg subcutaneous 6-18 mg subcutaneous Trimethobenzamide or other


antiemetic is used to reduce nausea
when initiating therapy

Bromocriptine 1.25 mg 2.5-15 mg daily Ergot; long-term use is associated


with cardiac valve brosis

Pramipexole 0.125 mg 3x daily 1.5-4.5 mg

Ropinirole 0.25 mg 3x daily 1.5-24 mg

Ropinirole sustained-release 2 mg per day 2-24 mg

MAO Inhibitors

Rasagiline 1 mg daily 0.5-1 mg

Selegiline 5 mg 2x daily 2.5-10 mg

Other Medications

Trihexyphenidyl HCl 1 mg 2x daily 2-15 mg

Amantadine 100 mg 2x daily 100-200 mg

242
Drug Therapy of Neurodegenerative Diseases CHAPTER 1 3

or a DA agonist, is eventua y required. Large contro ed c inica tria s provide


convincing evidence or a reduced rate o motor uctuation in patients in which
DA agonists are used as initia treatment. T is bene t was, however, accompanied
by an increased rate o adverse e ects, especia y somno ence and ha ucinations.
Practitioners pre er a DA agonist as initia therapy in younger patients in order to
reduce the occurrence o motor comp ications. In o der patients or those with
substantia comorbidity, evodopa/carbidopa is genera y better to erated.

CASE 13 3
As this (the patient rom Case 13-1) patients disease progresses, it is recommended
that he be treated with evodopa.
a. What is levodopa, and how is it used in the treatment o Parkinsons disease?
Levodopa, the metabo ic precursor o DA, is the sing e most e ective agent in the
treatment o PD. Levodopa is itse arge y inert; both its therapeutic and adverse
e ects resu t rom the decarboxy ation o evodopa to DA. T e metabo ism o
evodopa is shown in Figure 13-4.
Entry o evodopa into the CNS across the b ood-brain barrier is mediated by a
membrane transporter or aromatic amino acids, and competition between dietary
protein and evodopa may occur at this eve . In the brain, evodopa is converted to
DA by decarboxy ation primari y within the presynaptic termina s o dopaminer-
gic neurons in the stratium (see Figure 13-5). T e DA produced in the presynaptic
termina s is responsib e or the therapeutic e ectiveness o evodopa in PD; a er
re ease, it is either transported back into dopaminergic termina s by the presynaptic
uptake mechanism or metabo ized by the actions o MAO and catecho -O-methy -
trans erase (COM ) (see Figure 13-4).
b. What other drugs should be administered with levodopa?
In c inica practice, evodopa is a most a ways administered in combination with
a periphera y acting inhibitor o aromatic l -amino acid decarboxy ase, such as
carbidopa or benserazide (avai ab e outside the United States), drugs that do not
penetrate we into the CNS. I evodopa is administered a one, the drug is arge y
decarboxy ated by enzymes in the intestina mucosa and other periphera sites so
that re ative y itt e unchanged drug reaches the cerebra circu ation and probab y
(Continued)

Levodopa 3-O-Me thyldopa

HO NH2 COMT H3 CO NH2

COOH COOH
HO HO

AADC

Dopa mine 3-Me thoxytyra mine


NH2
COMT H3 CO NH2
HO
OH HO
DH
MAO MAO
ALDH Nore pine phrine ALDH

3,4-Dihydroxyphe nyl- 3-Me thoxy-4-hydroxy-


a ce tic Acid (DOPAC) phe nyla ce tic Acid (HVA)
HO H3 CO
COOH COMT COOH
HO HO

FIGURE 13-4 Metabolism o levodopa (L-DOPA). ALDH, aldehyde dehydrogenase; COMT,


catechol-O-methyltrans erase; DH, dopamine -hydroxylase; AADC, aromatic l -amino acid
decarboxylase; MAO, monoamine oxidase.
243
SECTION II Neuropharmacology

Pe riphe ry CNS (S triatum)


3-O-MD DOPAC
s e le giline
e nta ca pone (low dos e ) MAO-B
COMT ra s a giline
tolca pone AADC
L-DOPA L-DOPA DA

ca rbidopa AADC tolca pone COMT

DA 3MT

FIGURE 13-5 Pharmacological preservation o L-DOPA and striatal dopamine. The principal site
o action o inhibitors o catechol-O-methyltrans erase (COMT) (such as tolcapone and entaca-
pone) is in the peripheral circulation. They block the O-methylation o levodopa (L-DOPA) and
increase the raction o the drug available or delivery to the brain. Tolcapone also has ef ects
in the CNS. Inhibitors o MAO-B, such as low-dose selegiline and rasagiline, will act within the
CNS to reduce oxidative deamination o DA, thereby enhancing vesicular stores. AADC, aromatic
l -amino acid decarboxylase; DA, dopamine; DOPAC, 3,4-dihydroxyphenylacetic acid; MAO, mono-
amine oxidase; 3MT, 3-methoxyltyramine; 3-O-MD, 3-O-methyl DOPA.

ess than 1% penetrates the CNS. In addition, DA re ease into the circu ation by
periphera conversion o evodopa produces undesirab e e ects, particu ar y nau-
sea. Inhibition o periphera decarboxy ase, with carbidopa, marked y increases
the raction o administered evodopa that remains unmetabo ized and avai ab e
to cross the b ood-brain barrier (see Figure 13-5) and reduces the incidence o GI
side e ects.
Drugs or the treatment o PD inc ude inhibitors o the enzyme COM , which,
together with MAO, metabo izes evodopa and DA. COM trans ers a methy group
rom the donor S-adenosy -l -methionine, producing the pharmaco ogica y inactive
compounds 3-O-methy DOPA ( rom evodopa) and 3-methoxytyramine ( rom DA;
see Figure 13-4). T e principa therapeutic action o the COM inhibitors is to b ock
this periphera conversion o evodopa to 3-O-methy DOPA, increasing both the
p asma t1/2 o evodopa as we as the raction o each dose that reaches the CNS.
wo COM inhibitors present y are avai ab e or this use in the United States, to ca-
pone and entacapone. T e action o entacapone is attributab e principa y to periph-
era inhibition o COM . T e common adverse e ects o these agents are simi ar
to those o evodopa/carbidopa a one and inc ude nausea, orthostatic hypotension,
vivid dreams, con usion, and ha ucinations. An important adverse e ect associ-
ated with to capone is hepatotoxicity. Up to 2% o the patients treated with to capone
have increased serum a anine aminotrans erase and aspartate transaminase; and at
east 3 ata cases o u minant hepatic ai ure in patients taking to capone have been
observed, eading to the addition o a b ack box warning to the abe .
c. What are the adverse ef ects o levodopa that might be expected in this patient?
A common prob em is the deve opment o the wearing o phenomenon: each
dose o evodopa e ective y improves mobi ity or a period o time, perhaps 1 to
2 hours, but rigidity and akinesia return rapid y at the end o the dosing interva .
Increasing the dose and requency o administration can improve this situation,
but this o en is imited by the deve opment o dyskinesias, excessive and abnorma
invo untary movements. Dyskinesias are observed most o en when the p asma
evodopa concentration is high, a though in some individua s dyskinesias or dysto-
nia may be triggered when the eve is rising or a ing. T ese movements can be as
uncom ortab e and disab ing as the rigidity and akinesia o PD. In the ater stages o
PD, patients may uctuate rapid y between being o , having no bene cia e ects
rom their medications, and being on but with disab ing dyskinesias, a situation
ca ed the on/o phenomenon.
(Continued)
244
Drug Therapy of Neurodegenerative Diseases CHAPTER 1 3

In addition to motor comp ications and nausea, severa other adverse e ects may
be observed with evodopa treatment. A requent and troub ing adverse e ect is
the induction o ha ucinations and con usion, especia y in e der y patients or in
patients with preexisting cognitive dys unction. T is adverse e ect o en imits the
abi ity to treat parkinsonian symptoms adequate y.
Periphera decarboxy ation o evodopa and re ease o DA into the circu ation
may activate vascu ar DA receptors and produce orthostatic hypotension. Admin-
istration o evodopa with nonspeci c inhibitors o MAO, such as phene zine and
trany cypromine, marked y accentuate the actions o evodopa and may precipitate
i e-threatening hypertensive crisis and hyperpyrexia; nonspeci c MAO inhibi-
tors shou d a ways be discontinued at east 14 days be ore evodopa is administered
(note that this prohibition does not inc ude the MAO-B subtype-speci c inhibitors
se egi ine and rasagi ine, which are o en administered sa e y in combination with
evodopa). Abrupt withdrawa o evodopa or other dopaminergic medications may
precipitate the neuro eptic ma ignant syndrome o con usion, rigidity, and hyper-
thermia, a potentia y etha adverse e ect.
d. What other drugs can be used or the treatment o PD?
An a ternative to evodopa is the use o drugs that are direct agonists o striata
DA receptors, an approach that o ers severa potentia advantages. Since enzymatic
conversion o these drugs is not required or their activity, they do not depend
on the unctiona capacities o the nigrostriata neurons. T e direct DA receptor
agonists in c inica use have durations o action substantia y onger than that o
evodopa; they are o en used in the management o dose-re ated uctuations in
motor state, and may be he p u in preventing motor comp ications. Fina y, it has
been suggested that DA receptor agonists may have the potentia to modi y the
course o PD by reducing endogenous re ease o DA as we as the need or exogenous
evodopa, thereby reducing ree radica ormation.
wo ora y administered DA receptor agonists common y used or treatment o
PD are ropiniro e and pramipexo e.
wo isoenzymes o MAO oxidize monoamines. Whi e both isoenzymes (MAO-A
and MAO-B) are present in the periphery and inactivate monoamines o intesti-
na origin, the isoenzyme MAO-B is the predominant orm in the striatum and is
responsib e or most o the oxidative metabo ism o DA in the brain. wo se ective
MAO-B inhibitors are used or the treatment o PD: se egi ine and rasagi ine. Both
agents exert modest bene cia e ects on the symptoms o PD.
Se egi ine has been used or many years as a symptomatic treatment or PD and
is genera y we to erated in younger patients with ear y or mi d PD. In patients
with more advanced PD or under ying cognitive impairment, se egi ine may
accentuate the adverse motor and cognitive e ects o evodopa therapy. Metabo ites
o se egi ine inc ude amphetamine and methamphetamine, which may cause anxiety,
insomnia, and other adverse symptoms.
Un ike se egi ine, rasagi ine does not give rise to undesirab e amphetamine metabo-
ites. In randomized contro ed c inica tria s, rasagi ine monotherapy was e ective
in ear y PD.
Amantadine, an antivira agent used or the prophy axis and treatment o in uenza A
(see Chapter 44), has antiparkinsonian activity. Amantadine appears to a ter DA re ease
in the striatum, has anticho inergic properties, and b ocks NMDA g utamate receptors.

CASE 13 4
A 72-year-o d woman is diagnosed with A zheimers disease (AD).
a. What is the pathophysiology o AD?
T e patho ogica ha marks o AD are amy oid p aques, which are extrace u ar
accumu ations o amy oid- peptides (A), and intrace u ar neuro bri ary tang es
(Continued)
245
SECTION II Neuropharmacology

Ne uron
Mitochondrion A

Apo E4
Trunca te d
Apo E4 Oligome rs
S igna ling
Nucle us mole cule s
Impa ire d
syna ps e Microglia l
Ta u ce ll Ne urite
Ne urofibrilla ry
ta ngle s

Amyloid
pla que

FIGURE 13-6 Molecular and cellular processes presumed to participate in AD pathogenesis.


(From Roberson ED, Mucke L. 100 years and counting: Prospects for defeating Alzheimers disease.
Science, 2006, 314:781784. Reprinted with permission from AAAS.)

composed o the microtubu e-associated protein tau (see Figure 13-6). Whi e the
deve opment o amy oid p aques is an ear y and invariant eature o AD, tang e bur-
den accrues over time in a manner that corre ates more c ose y with the deve op-
ment o cognitive impairment. T e current consensus is that A accumu ation is an
upstream event that triggers tau patho ogy, resu ting in impaired neurona unction
and ce oss.
T e most striking neurochemica disturbance in AD is a de ciency o acety cho ine
(ACh). T e anatomica basis o the cho inergic de cit is atrophy and degenera-
tion o subcortica cho inergic neurons, particu ar y those in the basa orebrain
(nuc eus basa is o Meynert) that provide cho inergic innervation to the cerebra
cortex. T e se ective de ciency o ACh in AD, as we as the observation that cen-
tra cho inergic antagonists such as atropine can induce a con usiona state that
bears some resemb ance to the dementia o AD, has given rise to the cho inergic
hypothesis, which proposes that a de ciency o ACh is critica in the genesis o the
AD symptoms. A though viewing AD as a cho inergic de ciency syndrome akin
to the dopaminergic de ciency syndrome o PD provides a use u ramework, it is
important to note that the de cit in AD is ar more comp ex. AD invo ves mu tip e
neurotransmitter systems, inc uding g utamate, 5-H , and neuropeptides, and there
is destruction o not on y cho inergic neurons but a so the cortica and hippocam-
pa targets that receive cho inergic input.
b. What treatment is available or the cognitive symptoms o AD?
Augmentation o the cho inergic transmission is current y the mainstay o AD
treatment. T ree drugsdonepezi , rivastigmine, and ga antamineare wide y
used or this purpose; a ourth drug, tacrine, was the rst drug approved to treat
AD but is rare y used now because it has much more extensive side e ects com-
pared to the newer agents (see ab e 13-2).
A 4 agents are reversib e antagonists o cho inesterases, enzymes that act to imit
cho inergic neurotransmission by cata yzing the c eavage o acety cho ine in the
synaptic c e into cho ine and acetate (see Chapter 6).
Memantine is used either as an adjunct or as an a ternative to cho inesterase inhibi-
tors in AD and is a so common y used to treat other neurodegenerative dementias.
Memantine is a noncompetitive antagonist o the NMDA-type g utamate receptor.
(Continued)
246
Drug Therapy of Neurodegenerative Diseases CHAPTER 1 3

TABLE 13-2 Cholinesterase Inhibitors Used for the Treatment of Alzheimers Disease
DONEPEZIL RIVASTIGMINE GALANTAMINE TACRINEa
Brand name ARICEPT EXELON, generic RAZADYNE, generic COGNEX

Enzymes inhibited b AChE AChE, BuChE AChE AChE, BuChE

Mechanism Noncompetitive Noncompetitive Competitive Noncompetitiv

Typical maintenance dose c 10 mg once daily 9.5 mg/24h (transdermal) 8-12 mg twice daily 20 mg, our times daily
3-6 mg twice daily (oral) (immediate-release)
16-24 mg/day (extended-release)

FDA-approved indications Mildsevere AD Mildmoderate AD, Mildmoderate AD Mildmoderate AD


Mildmoderate PDDd

Metabolism e CYP2D6, CYP3A4 Esterases CYP2D6, CYP3A4 CYP1A2


a
Tacrine was the rst cholinesterase inhibitor approved or the treatment o AD, but is now rarely used because o hepatotoxicity and adverse
ef ects.
b
AChE (acetylcholinesterase) is the major cholinesterase in the brain; BuChE (butyrylcholinesterase) is a serum and hepatic cholinesterase that is
upregulated in AD brain.
c
Typical starting doses are one-hal o the maintenance dose and are given or the rst month o therapy.
d
PDD, Parkinsons disease dementia.
e
Drugs metabolized by CYP2D6 and CYP3A4 are subject to increased serum levels when co-administered with drugs known to inhibit
these enzymes, such as ketoconazole and paroxetine. Similarly, tacrine levels are increased by co-administration with the CYP1A2 inhibitors
theophylline, cimetidine, and uvoxamine.

It interacts with the Mg2+ binding site o the channe to prevent excessive activation
whi e sparing norma unction. Memantine signi cant y reduces the rate o c inica
deterioration in patients with moderate to severe AD.
c. What treatment is available or the behavioral symptoms o AD?
In addition to cognitive dec ine, behaviora and psychiatric symptoms in dementia
(BPSD) are common, particu ar y in midd e stages o the disease. T ese symptoms,
inc uding irritabi ity and agitation, paranoia and de usiona thinking, wandering,
anxiety, and depression, are a major source o caregiver distress and o en precipi-
tate nursing home p acement.
Atypica antipsychotics, such as respiridone, o anzepine, and quetiapin (see Chapter 8),
are the most ef cacious therapy or agitation and psychosis in AD. Risperidone and
o anzapine are e ective, but their use is o en imited by adverse e ects, inc uding
parkinsonism, sedation, and a s. In addition, the use o atypica antipsychotics in
e der y patients with dementia-re ated psychosis has been associated with a higher
risk o stroke and overa morta ity, eading the FDA to order inc usion o a boxed
warning in the prescribing in ormation or a drugs in this c ass. Un ortunate y,
there are ew e ective a ternatives.
Antidepressants (see Chapter 8) can be use u or BPSD, particu ar y when depres-
sion or anxiety contribute. Because o the adverse anticho inergic e ects o tricyc ic
agents, serotonergic antidepressants are avored. T ese agents are genera y we
to erated. razodone has modest bene ts, but or the most part, se ective serotonin
reuptake inhibitors (SSRIs) are the pre erred c ass o drugs.
T e typica AD patient presenting in ear y stages o disease shou d probab y be
treated with a cho inesterase inhibitor. Patients and ami ies shou d be counse ed
that a rea istic goa o therapy is to induce a temporary reprieve rom progres-
sion, or at east a reduction in the rate o dec ine, rather than ong-term recovery
o cognition. As the disease progresses, memantine can be added to the regimen.
Behaviora symptoms are o en treated with a serotonergic antidepressant or, i
they are severe enough to warrant the risk o higher morta ity, an atypica antipsy-
chotic. E iminating drugs ike y to aggravate cognitive impairments, particu ar y
(Continued)
247
SECTION II Neuropharmacology

anticho inergics, benzodiazepines, and other sedative/hypnotics, rom the patients


regimen is another important aspect o AD pharmacotherapy.

CASE 13 5
A 39-year-o d man is diagnosed with Huntingtons disease (HD).
a. What is the pathophysiology o Huntingtons disease?
HD is characterized by prominent neurona oss in the striatum (caudate/putamen)
o the brain. Atrophy o these structures proceeds in an order y ashion, rst a ecting
the tai o the caudate nuc eus and then proceeding anterior y rom mediodorsa
to ventro atera .
Se ective vu nerabi ity a so appears to under ie the most conspicuous c inica ea-
ture o HD, the deve opment o chorea. In most adu t-onset cases, the medium
spiny neurons that project to the GPi and SNpr (the indirect pathway) appear to be
a ected ear ier than those projecting to the g obus pa idus externa (GPe; the direct
pathway; see Figure 13-5). T e disproportionate impairment o the indirect path-
way increases excitatory drive to the neocortex, producing invo untary chorei orm
movements (see Figure 13-7).
b. What drugs are available or the treatment o patients with Huntington disease?
reatment or symptomatic HD emphasizes the se ective use o medications. None
o the current y avai ab e medications s ows the progression o the disease.
reatment is needed or patients who are depressed, irritab e, paranoid, exces-
sive y anxious, or psychotic. Depression can be treated e ective y with standard
antidepressant drugs with the caveat that drugs with substantia anticho inergic
pro es can exacerbate chorea. F uoxetine (see Chapter 8) is e ective treatment or
both the depression and the irritabi ity mani est in symptomatic HD. Carbamaze-
pine (see Chapter 12) a so has been ound to be e ective or depression. Paranoia,
de usiona states, and psychosis are treated with antipsychotic drugs, but usua y
at ower doses than those used in primary psychiatric disorders (see Chapter 8).
T ese agents a so reduce cognitive unction and impair mobi ity and thus shou d
be used in the owest doses possib e and shou d be discontinued when the psychi-
atric symptoms reso ve.
(Continued)

+
Ce re bra l corte x
Glu
+ Glu + Glu
S tria tum VA/VL
DA tha la mus
ACh
D1 D2
GABA
GABA
Glu +
GPe STN
S Npc GABA
GABA Glu +
GP i/S Npr
Glu
+ To s p ina l c ord
a nd b ra ins te m

FIGURE 13-7 The basal ganglia in Huntingtons disease. HD is characterized by loss o neurons
rom the striatum. The neurons that project rom the striatum to the GPe and orm the indirect
pathway are af ected earlier in the course o the disease than those which project to the GPi. This
leads to a loss o inhibition o the GPe. The increased activity in this structure, in turn, inhibits the
STN, SNpr, and GPi, resulting in a loss o inhibition to the VA/VL thalamus and increased thalamo-
cortical excitatorydrive. Structures in light blue have reduced activity in HD, whereas structures
in dark blue have increased activity. (See legend to Figure 13-2 or de nitions o anatomical
abbreviations.)
248
Drug Therapy of Neurodegenerative Diseases CHAPTER 1 3

T e movement disorder o HD per se on y rare y justi es pharmaco ogica therapy.


For those with arge-amp itude chorea causing requent a s and injury, tetrabena-
zine has recent y become avai ab e in the United States or the treatment o chorea
associated with HD. etrabenazine and the re ated drug reserpine are inhibitors o
the vesicu ar monoamine transporter 2 (VMA 2) and cause presynaptic dep etion
o catecho amines.

CASE 13 6
A 53-year-o d man is diagnosed with amyotrophic atera sc erosis (ALS).
a. What is ALS?
ALS (or Lou Gehrig disease) is a disorder o the motor neurons o the ventra horn
o the spina cord ( ower motor neurons) and the cortica neurons that provide
their a erent input (upper motor neurons). T e disorder is characterized by rapid y
progressive weakness, musc e atrophy and ascicu ations, spasticity, dysarthria, dys-
phagia, and respiratory compromise. Sensory, autonomic, and ocu omotor unction
is genera y spared.
T e patho ogy o ALS corresponds c ose y to the c inica eatures: T ere is promi-
nent oss o the spina and brainstem motor neurons that project to striated musc es
(a though the ocu omotor neurons are spared), as we as oss o the arge pyramida
motor neurons in ayer V o the motor cortex, which are the origin o the descend-
ing corticospina tracts.
ALS usua y is progressive and ata . Most patients die o respiratory compromise
and pneumonia a er 2 to 3 years, a though some individua s have a more indo ent
course and survive or many years.
b. What treatment options are available or patients with ALS?
Ri uzo e is an agent with comp ex actions in the nervous system. It inhibits g uta-
mate re ease, but it a so b ocks postsynaptic NMDA- and kainate-type g utamate
receptors and inhibits vo tage-dependent Na+ channe s.
In c inica tria s ri uzo e has modest but genuine e ects on the surviva o patients
with ALS. Meta-ana yses o the avai ab e tria s indicate that ri uzo e extends surviva
by 2 to 3 months.
Spasticity is an important component o the c inica eatures o ALS and the eature
most amenab e to present orms o treatment. Spasticity o en eads to considerab e
pain and discom ort and urther reduces mobi ity, which a ready is compromised
by weakness. Spasticity is de ned as an increase in musc e tone characterized by an
initia resistance to passive disp acement o a imb at a joint, o owed by a sudden
re axation (the so-ca ed c asped-kni e phenomenon).
T e best agent or the symptomatic treatment o spasticity in ALS is bac o en, a
GABAB receptor agonist.
izanidine is an agonist o 2 adrenergic receptors in the CNS. It reduces musc e
spasticity and is assumed to act by increasing presynaptic inhibition o motor
neurons.
Benzodiazepines (see Chapter 9) such as c onazepam are e ective antispastic-
ity agents, but they are sedating and may contribute to respiratory depression in
patients with advanced ALS.
Dantro ene a so is approved in the United States or the treatment o musc e spasm.
In contrast to the other agents discussed, dantro ene acts direct y on ske eta
musc e bers, impairing Ca2+ re ease rom the sarcop asmic reticu um. Because it
can exacerbate muscu ar weakness, it is not used in ALS but is e ective in treating
spasticity associated with stroke or spina cord injury and in treating ma ignant
hyperthermia.

249
SECTION II Neuropharmacology

KEY CONCEPTS
Neurodegenerative disorders are characterized by progressive and irreversib e
oss o neurons rom speci c regions o the brain.
Current y avai ab e therapies or neurodegenerative disorders a eviate the dis-
ease symptoms but do not a ter the under ying neurodegenerative processes.
T e dopaminergic de cit in Parkinsons disease arises rom a oss o the neurons
in the substantia nigra pars compacta that provide innervation to the striatum.
Drug therapy is not ob igatory in ear y Parkinsons disease; pharmaco ogica
treatment o Parkinsons disease shou d be tai ored to the individua patient.
A rea istic goa or the treatment o a patient with A zheimers disease is to
induce a temporary reprieve rom progression, or at east a reduction in the rate
o dec ine, rather than ong-term recovery o cognition.
Huntingtons disease is a dominant y inherited disorder characterized by
prominent neurona oss in the striatum o the brain and by the gradua onset
o motor incoordination and cognitive dec ine in mid i e.
At present there is no ef ective treatment or Huntingtons disease; therapy is
aimed toward contro o the motor and behaviora eatures o the disorder.
Amyotrophic atera sc erosis is a progressive degenerative disease o spina
motor neurons eading to weakness and eventua y para ysis.
T e drug ri uzo e is the on y treatment estab ished to a ter the course o amyotrophic
atera sc erosis; its ef ect is modest, pro onging surviva by on y a ew months.

SUMMARY QUIZ

QUESTION 13-1 A 72-year-o d man with Parkinsons disease is being treated with se e-
gi ine. Over the past 2 weeks he has noticed an increase in insomnia and anxiety. It is
ike y these symptoms are due to
a. dopamine.
b. ephedrine.
c. amphetamine.
d. serotonin.
e. caf eine.

QUESTION 13-2 A 75-year-o d woman with Parkinsons disease is being treated with
the combination o evodopa/carbidopa. Entacapone is added. T is patient is treated
with this combination o drugs to increase the amount o which o the o owing sub-
stance to reach the CNS?
a. Carbidopa
b. Levodopa
c. Entacapone
d. Acety cho ine
e. Dopamine

QUESTION 13-3 A 66-year-o d woman with A zheimers disease is being treated with
the cho inesterase inhibitor donepezi . T e patients ami y earns that donepezi is
re ated to pesticides. When the ami y expresses concern they are to d that donepezi
may decrease the rate o cognition dec ine by increasing the concentration o which
substance in the brain?
a. Acety cho ine
b. Serotonin
(Continued)
250
Drug Therapy of Neurodegenerative Diseases CHAPTER 1 3

c. Dopamine
d. Epinephrine
e. G utamate

QUESTION 13-4 A 73-year-o d man with A zheimers disease is being treated with
donepezi . Because his cognition is deteriorating, memantine is added. Memantine is
a noncompetitive antagonist o the
a. dopamine D2 receptor.
b. NMDA-type g utamate receptor.
c. adrenergic receptor.
d. adrenergic receptor.
e. 5-H 2 serotonergic receptor.

QUESTION 13-5 A 57-year-o d man with amyotrophic atera sc erosis is treated with
ri uzo e. T e patients ami y is to d that the drug may extend surviva by
a. 2 to 3 days.
b. 2 to 3 months.
c. 2 to 3 years.
d. an inde nite period o time.
e. none o the above; because it is not thought to extend surviva .

QUESTION 13-6 A 65-year-o d man with Parkinsons disease is treated with pramipex-
o e. T is drug acts as an agonist at striata
a. 5-H serotonergic receptors.
b. D1 dopaminergic receptors.
c. norepinephrine adrenergic receptors.
d. D2 dopaminergic receptors.
e. opiate receptors.

SUMMARY QUIZ ANSWER KEY

QUESTION 13-1 Answer is c. Metabo ites o se egi ine inc ude amphetamine and
methamphetamine, which may cause anxiety, insomnia, and other adverse symptoms.
Un ike se egi ine, rasagi ine does not give rise to undesirab e amphetamine metabo ites.

QUESTION 13-2 Answer is b. Drugs or the treatment o PD inc ude inhibitors


o the enzyme COM , which, together with MAO, metabo ize evodopa and DA.
COM trans ers a methy group rom the donor S-adenosy -l -methionine, produc-
ing the pharmaco ogica y inactive compounds 3-O-methy DOPA ( rom evodopa)
and 3-methoxytyramine ( rom DA; see Figure 13-5). When evodopa is administered
ora y, near y 99% o the drug is metabo ized in the periphery and does not reach the
brain. Most is converted by aromatic l -amino acid decarboxy ase (AADC) to DA,
which causes nausea and hypotension. Addition o an AADC inhibitor such as carbi-
dopa reduces the periphera ormation o DA but increases the raction o evodopa
that is methy ated by COM . T e principa therapeutic action o the COM inhibitor
entacapone is to b ock this periphera conversion o evodopa to 3-O-methy DOPA,
increasing both the p asma t 1/2 o evodopa as we as the raction o evodopa that
reaches the CNS.

QUESTION 13-3 Answer is a. T e most striking neurochemica disturbance in AD is


a de ciency o acety cho ine. T e anatomica basis o the cho inergic de cit is atrophy
and degeneration o subcortica cho inergic neurons, particu ar y those in the basa
orebrain (nuc eus basa is o Meynert) that provides cho inergic innervation to the
cerebra cortex.
(Continued)
251
SECTION II Neuropharmacology

Donepezi is a reversib e antagonist o cho inesterases, enzymes that act to imit


cho inergic neurotransmission by cata yzing the c eavage o acety cho ine in the syn-
aptic c e into cho ine and acetate (see Chapter 6). Cho inesterase inhibitors are the
usua rst- ine therapy or symptomatic treatment o cognitive impairments in mi d or
moderate AD. Augmentation o the cho inergic transmission is current y the mainstay
o AD treatment.

QUESTION 13-4 Answer is b. Memantine is used either as an adjunct or an a ternative


to cho inesterase inhibitors in AD, and is a so common y used to treat other neurode-
generative dementias. Memantine is a noncompetitive antagonist o the NMDA-type
g utamate receptor. It interacts with the Mg2+-binding site o the channe to prevent
excessive activation whi e sparing norma unction.

QUESTION 13-5 Answer is b. Ri uzo e is an agent with comp ex actions in the nervous
system. It inhibits g utamate re ease, but it a so b ocks postsynaptic NMDA- and
kainate-type g utamate receptors and inhibits vo tage-dependent Na+ channe s. In
c inica tria s ri uzo e has modest but genuine ef ects on the surviva o patients with
ALS. Meta-ana yses o the avai ab e tria s indicate that ri uzo e extends surviva by
2 to 3 months.

QUESTION 13-6 Answer is d. wo ora y administered DA receptor agonists are


common y used or treatment o PD: ropiniro e and pramipexo e. Ropiniro e and
pramipexo e have se ective agonist activity at D2 c ass sites (speci ca y at the D2 and D3
receptor) and itt e or no activity at D1 c ass sites.

SUMMARYTABLE: DRUGS USED TO TREAT NEURODENERATIVE DISORDERS


TOXICITIES
CLASS AND
SUBCLASSES NAMES CLINICAL USES COMMON UNIQUE: CLINICALLY IMPORTANT
Drugs Used to Treat Levodopa Treatment o Parkinsons Nausea Hallucinations and con usion
Parkinsons Disease disease Orthostatic hypotension
Contraindicated with nonspeci c
inhibitors o MAO
Abrupt withdrawal may precipitate
neuroleptic malignant syndrome

Carbidopa/levodopa Treatment o Parkinsons Nausea Hallucinations and con usion


combination disease Orthostatic hypotension
Contraindicated with nonspeci c
inhibitors o MAO
Abrupt withdrawal may precipitate
neuroleptic malignant syndrome

Entacapone Combined with Nausea Vivid dreams, con usion, orthostatic


levodopa/carbidopa to hypotension
treat Parkinsons disease

Tolcapone Combined with Nausea Associated with hepatotoxity


levodopa/carbidopa to Vivid dreams, con usion, orthostatic
treat Parkinsons disease hypotension

Ropinirole Treatment o Parkinsons Nausea Hallucinations and con usion


disease Fatigue and
somnolence

Pramipexole Treatment o Parkinsons Nausea Hallucinations and con usion


disease Fatigue and
somnolence

252
Drug Therapy of Neurodegenerative Diseases CHAPTER 1 3

TOXICITIES
CLASS AND
SUBCLASSES NAMES CLINICAL USES COMMON UNIQUE: CLINICALLY IMPORTANT
Selegiline Selective inhibitor o Anxiety and May exacerbate the adverse motor and
MAO-B used to treat insomnia due to its cognitive ef ects o levodopa
Parkinsons disease amphetamine and Contraindicated or use with meperidine
meth-amphetamine Used with caution in patients taking SSRIs
metabolites

Rasagiline Selective inhibitor o May exacerbate the adverse motor and


MAO-B used to treat cognitive ef ects o levodopa
Parkinsons disease Contraindicated or use with meperidine
Used with caution in patients taking SSRIs

Amantadine Antiviral agent used to Dizziness, lethargy,


treat Parkinsons disease anti-cholinergic
Its ef ects are modest ef ects, and sleep
disturbance

Apomorphine Intermittent therapy o Severe nausea and Hallucinations, dyskinesia, and abnormal
Parkinsons disease vomiting requiring behavior
pretreatment with an
antiemetic not o the
5-HT3 antagonist class

Drugs Used to Treat Donepezil Used to treat Alzheimers GI distress, muscle See Chapter 6
Alzheimers Disease disease cramping, abnormal Used with caution in patients with
dreams bradycardia or syncope

Rivastigmine Used to treat Alzheimers GI distress, muscle See Chapter 6


disease cramping, abnormal Used with caution in patients with
dreams bradycardia or syncope

Galantamine Used to treat Alzheimers GI distress, muscle See Chapter 6


disease cramping, abnormal Used with caution in patients with
dreams bradycardia or syncope

Tacrine Used to treat Alzheimers GI distress, muscle See Chapter 6


disease cramping, abnormal Used with caution in patients with
dreams bradycardia or syncope

Memantine Used as an adjunct or Headache and Dosage reduced in patients with


an alternative to treat dizziness renal impairment
Alzheimers disease

Drugs Used to Treat Tetrabenazine Treatment o chorea Depression


Huntingtons Disease associated with Hypotension
Huntingtons disease

Drugs Used to Riluzole Treatment o amyotrophic Nausea and diarrhea Associated with hepatic injury
Treat ALS lateral sclerosis (ALS)
May prolong survival by
2-3 months

Baclo en Treatment o spasticity Sedation Abrupt withdrawal o intrathecal


o ALS baclo en may cause rebound spasticity,
Intrathecal administration rhabdomyolitis, and multiple organ
may avoid sedation ailure

Tizanidine Treatment o spasticity Drowsiness, asthenia,


o ALS and dizziness

Dantrolene Treatment o spasticity Associated with hepatotoxicity


o ALS

253
CHAPTER

14 Drug Addiction
T is chapter will be most use ul a er having a basic understanding o the material
in Chapter 24, Drug Addiction in Goodman & Gilmans T e Pharmacological Basis of
T erapeutics, 12th Edition. T e speci c pharmacology (including the mechanisms o
action) o drugs mentioned in Chapter 14 is discussed in previous or subsequent chap-
ters. T e drugs presented in Chapter 14 are discussed in relation to their ability to pro-
duce tolerance, physical dependence, and addiction. No Mechanisms o Action Table
nor Clinical Summary Table are included as a part o Chapter 14 because new drugs
are not introduced. T e ew drugs that are used therapeutically to treat speci c drug
addictions are discussed in the narratives o the clinical cases. In addition to the mate-
rial presented here, the 12th Edition includes:
able 24-2 Dependence among Users 1990 to 1992
Figure 24-2 Cocaine-induced changes in CNS dopamine release
Figure 24-3 Nicotine concentrations in blood resulting rom ve di erent nicotine
delivery systems
A detailed discussion o the variables a ecting the onset and continuation o drug
abuse and addiction
A detailed discussion o the di erent types o tolerance

LEARNING OBJECTIVES
Understand the pharmacological principles o tolerance, physical dependence,
and withdrawal.
Describe the characteristic withdrawal syndromes or the commonly abused
drugs.
Know the patterns o abuse behavior and the toxicity o the commonly abused drugs.
Know the available pharmacological interventions or the acute treatment and
long-term management o the commonly abused drugs.

CASE 14-1
A 56-year-old obese woman is being treated with immediate-release oxycodone or
chronic back pain. She is also taking a muscle relaxant, cyclobenzaprine, and a short-
acting benzodiazepine or sleep.
a. What concerns are there with the use o an opiate-like immediate-release oxyco-
done or the treatment o chronic pain?
Rapi -onset, short- uration opioi s are excellent or acute short-term use, such as
uring the postoperative perio . As tolerance an physical epen ence evelop,
however, the patient may experience the early symptoms o with rawal between
oses, an uring with rawal, the threshol or pain ecreases. T e iversion o
prescription opioi s such as oxyco one an hy roco one to illegal markets has
become an important source o opiate abuse in the Unite States (see answer to
Case 14-1c below).
T e risk or a iction is highest in patients complaining o pain with no clear phys-
ical explanation or in patients with evi ence o a chronic, non-li e-threatening is-
or er. Examples are chronic hea aches, backaches, ab ominal pain, or peripheral
neuropathy. Even in these cases, an opioi may be consi ere as a brie emergency
treatment, but long-term treatment with opioi s shoul be use only a er other
alternatives have been exhauste .
(Continued)
254
Drug Addiction CHAPTER 1 4

b. Over the past year this patient has doubled her dose o oxycodone. Why has she
increased the dose o oxycodone?
While abuse an a iction are complex con itions combining many variables
(see able 14-1), there are a number o relevant pharmacological phenomena that
occur in epen ent o social an psychological imensions. First are the changes in
the way the bo y respon s to a rug with repeate use. olerance, the most com-
mon response to repetitive use o the same rug, can be e ne as the re uction in
response to the rug a er repeate a ministrations. Figure 14-1 shows an i ealize
ose-response curve or an a ministere rug. As the ose o the rug increases, the
observe e ect o the rug increases. With repeate use o the rug, however, the
curve shi s to the right (tolerance). T us, a higher ose is require to pro uce the
same e ect that was once obtaine at a lower ose. As outline in able 14-2, there
are many orms o tolerance, likely arising through multiple mechanisms.
olerance to some rug e ects evelops much more rapi ly than to other e ects o
the same rug. For example, tolerance evelops rapi ly to the euphoria pro uce
by opioi s such as heroin, an a icts ten to increase their ose in or er to reex-
perience that elusive high. In contrast, tolerance to the gastrointestinal e ects o
opioi s evelops more slowly. T e iscrepancy between tolerance to euphorigenic
e ects (rapi ) an tolerance to e ects on vital unctions (slow), such as respiration
an bloo pressure, can lea to potentially atal over oses in se ative abusers trying
to reexperience the euphoria they recall rom earlier use (see Case 14-5).
(Continued)

TABLE 14-1 Multiple Simultaneous Variables Af ecting Onset and Continuation o


Drug Abuse and Addiction
Agent (drug)
Availability
Cost
Purity/potency
Mode o administration
Chewing (absorption via oral mucous membranes)
Gastrointestinal
Intranasal
Subcutaneous and intramuscular
Intravenous
Inhalation
Speed o onset and termination o e ects (pharmacokinetics: combination o agent and host)

Host (user)
Heredity
Innate tolerance
Speed o developing acquired tolerance
Likelihood o experiencing intoxication as pleasure
Metabolism o the drug (nicotine and alcohol data already available)
Psychiatric symptoms
Prior experiences/expectations
Propensity or risk-taking behavior

Environment
Social setting
Community attitudes
Peer in uence, role models
Availability o other rein orcers (sources o pleasure or recreation)
Employment or educational opportunities
Conditioned stimuli: environmental cues become associated with drugs a ter repeated use in
the same environment

255
SECTION II Neuropharmacology

TABLE 14-2 Types o Tolerance


Innate (pre-existing sensitivity or
insensitivity) S e ns itiza tion

t
c
Acquired

e
f
f
e
Pharmacokinetic (dispositional or

ve
metabolic)

i
t
a
l
Pharmacodynamic

e
R
Learned tolerance
Tole ra nce
Behavioral
Conditioned
Acute tolerance
Reverse tolerance (sensitization)
Cross-tolerance Dos e
FIGURE 14-1 Shi ts in a dose-response curve with tolerance and sensitization. With tolerance,
there is a shi t o the curve to the right such that doses higher than initial doses are required to
achieve the same e ects. With sensitization, there is a le tward shi t o the curve such that or a
given dose, there is a greater e ect than seen a ter the initial dose.

c. What are the options or treatment o this patient?


For chronic a ministration, long-acting opioi s are pre erre . While metha one is
long-acting because o its metabolism to active metabolites, the long-acting version
o oxyco one has been ormulate to release slowly, thereby changing a short-acting
opioi into a long-acting one. Un ortunately, this mechanism can be subverte by
breaking the tablet an making the ull ose o oxyco one imme iately available.
T is has le to iversion o oxyco one to illicit tra c because high- ose oxyco one
pro uces euphoria that is sought by opiate abusers.

CASE 14-2
A 43-year-old man is admitted to the hospital because o a ractured hip. He has a long
history o alcohol consumption. Recently he has begun taking a drink o alcohol in the
morning.
a. Is it appropriate to assume that he has developed tolerance and physical depen-
dence to alcohol?
Heavy consumers o alcohol not only acquire tolerance but also inevitably evelop a
state o physical epen ence. T is o en lea s to rinking in the morning to restore
TABLE 14-3 Alcohol Withdrawal bloo alcohol levels iminishe uring the night. Eventually, they may awaken uring
Syndrome the night an take a rink to avoi the restlessness pro uce by alling alcohol levels.

Alcohol craving b. What are the major signs and symptoms o alcohol withdrawal?
Tremor, irritability T e alcohol-with rawal syn rome (see able 14-3) generally epen s on the size o
Nausea the average aily ose an usually is treate by resumption o alcohol ingestion.
Sleep disturbance
With rawal symptoms are experience requently but usually are not severe or li e-
Tachycardia
Hypertension threatening until they occur in conjunction with other problems, such as in ection,
Sweating trauma, malnutrition, or electrolyte imbalance. In the setting o such complica-
Perceptual distortion tions, the syn rome o elirium tremens becomes likely.
Seizures (6-48 hours a ter last drink)
Visual (and occasionally auditory or c. What is the appropriate treatment to prevent and treat alcohol withdrawal?
tactile) hallucinations (12-48 hours a ter A patient who presents in a me ical setting with an alcohol-with rawal syn rome
last drink) shoul be consi ere to have a potentially lethal con ition. Although most mil cases
Delirium tremens (48-96 hours a ter last o alcohol with rawal never come to me ical attention, severe cases require general
drink; rare in uncomplicated withdrawal)
evaluation; attention to hy ration an electrolytes; vitamins, especially high- ose
Severe agitation
Con usion thiamine (see Chapter 9); an a se ating me ication that has cross-tolerance with
Fever, pro use sweating alcohol. o block or iminish the symptoms escribe in able 14-3, a short-acting
Tachycardia benzo iazepine such as oxazepam can be use at a ose o 15 to 30 mg every 6 to 8
Nausea, diarrhea hours accor ing to the stage an severity o with rawal; some authorities recommen
Dilated pupils
(Continued)
256
Drug Addiction CHAPTER 1 4

a long-acting benzo iazepine unless there is emonstrate liver impairment. Anti-


convulsants such as carbamazepine have been shown to be e ective in alcohol
with rawal, although they appear not to relieve subjective symptoms as well as ben-
zo iazepines. A er me ical evaluation, uncomplicate alcohol with rawal can be
treate e ectively on an outpatient basis. When there are me ical problems, a history
o seizures, or simultaneous epen ence on other rugs, hospitalization is require .
d. What are the options or this patient i he desires treatment or his alcoholism
upon discharge rom the hospital?
Detoxi cation is only the rst step o treatment. Complete abstinence is the
objective o long-term treatment, an this is accomplishe mainly by behavioral
approaches. Me ications that ai in the prevention o relapse are un er evelop-
ment. Disul ram (AN ABUSE; see Chapter 9) has been use ul in some programs
that ocus behavioral e orts on ingestion o the me ication. Disul ram blocks
al ehy e ehy rogenase, the secon step in ethanol metabolism, resulting in the
accumulation o acetal ehy e, which pro uces an unpleasant f ushing reaction
when alcohol is ingeste . Knowle ge o this unpleasant reaction helps the patient
to resist taking a rink. Although quite e ective pharmacologically, isul ram has
not been oun to be e ective in controlle clinical trials because so many patients
aile to ingest the me ication.
Naltrexone (REVIA; see Chapter 10), an opioi receptor antagonist that blocks the
rein orcing properties o alcohol, is FDA-approve as an a junct in the treatment
o alcoholism. Chronic a ministration o naltrexone resulte in a ecrease rate o
relapse to alcohol rinking in the majority o publishe ouble-blin clinical trials.
It works best in combination with behavioral treatment programs that encourage
a herence to me ication an abstinence rom alcohol. A epot preparation o nal-
trexone with a uration o 30 ays (VIVI ROL) was approve by the FDA in 2006;
it greatly improves me ication a herence, the major problem with the use o me i-
cations in treating alcoholism.
Acamprosate (CAMPRAL; see Chapter 9), another FDA-approve me ication or
alcoholism, is a competitive inhibitor o the N-methyl-d-aspartate (NMDA)type
glutamate receptor. T e rug appears to normalize the ysregulate neurotrans-
mission associate with chronic ethanol intake an thereby to attenuate one o the
mechanisms that lea to relapse.

CASE 14-3
A 75-year-old woman with early dementia has been taking alprazolam as a sleep aid or
the past 10 years. She has consistently taken the same prescribed dose, but when her
prescription expired she orgot to renew it.
a. Is she likely to have developed tolerance or physical dependence to the
benzodiazepine?
When a benzo iazepine is taken or up to several weeks, there is little tolerance or
physical epen ence an no i culty in stopping the me ication when the con i-
tion no longer warrants its use. A er several months, the proportion o patients TABLE 14-4 Benzodiazepine
who emonstrate physical epen ence increases, an re ucing the ose or stopping Withdrawal Symptoms
the me ication abruptly pro uces with rawal symptoms (see able 14-4).
Following moderate dose usage
b. What symptoms might she experience as a result o abruptly stopping the Anxiety, agitation
benzodiazepine? Increased sensitivity to light and sound
Paresthesias, strange sensations
It can be i cult to istinguish with rawal symptoms (see able 14-4) rom the
Muscle cramps
reappearance o the anxiety symptoms or which the benzo iazepine was prescribe Myoclonic jerks
initially. Some patients may increase their ose over time because tolerance e nitely Sleep disturbance
evelops to the se ative e ects. Many patients an their physicians, however, con- Dizziness
ten that antianxiety bene ts continue to occur long a er tolerance evelops to the Following high-dose usage
se ating e ects. Moreover, these patients continue to take the me ication or years Seizures
Delirium
(Continued)
257
SECTION II Neuropharmacology

accor ing to me ical irections without increasing their ose an are able to unction
very e ectively as long as they take the benzo iazepine. T e egree to which toler-
ance evelops to the anxiolytic e ects o benzo iazepines is a subject o controversy.
c. How might this patients withdrawal syndrome be treated?
I patients receiving long-term benzo iazepine treatment by prescription wish to
stop their me ication, the process may take months o gra ual ose re uction.
With rawal symptoms (see able 14-4) may occur uring this outpatient etoxi-
cation, but in most cases the symptoms are mil . I anxiety symptoms return, a
nonbenzo iazepine such as buspirone may be prescribe , but this agent usually is
less e ective than benzo iazepines or treatment o anxiety in these patients. Some
authorities recommen trans erring the patient to a long t 1/2 benzo iazepine uring
etoxi cation; others recommen an anticonvulsant, carbamazepine or phenobar-
bital. Controlle stu ies comparing i erent treatment regimens are lacking. Since
patients who have been on low oses o benzo iazepines or years usually have no
a verse e ects, the physician an patient shoul eci e jointly whether etoxi ca-
tion an possible trans er to a new anxiolytic are worth the e ort.
A er etoxi cation, the prevention o relapse requires a long-term outpatient
rehabilitation program similar to the treatment o alcoholism. No speci c me ica-
tions have been oun to be use ul in the rehabilitation o se ative abusers; but, o
course, speci c psychiatric isor ers such as epression or schizophrenia, i pres-
ent, require appropriate me ications.

CASE 14-4
A 53-year-old man has been smoking since he was a teenager. He now desires to
quit smoking.
a. What are the issues o nicotine addiction that should be considered in this patient?
T e basic pharmacology o nicotine an agents or smoking cessation are iscusse
in Chapter 6. Because nicotine provi es the rein orcement or cigarette smoking,
the most common cause o preventable eath an isease in the Unite States,
it is arguably the most angerous epen ence-pro ucing rug. T e epen ence
pro uce by nicotine can be extremely urable, as exempli e by the high ailure
rate among smokers who try to quit. Although more than 80% o smokers express
a esire to quit, only 35% try to stop each year, an ewer than 5% are success ul in
unai e attempts to quit.
Cigarette a iction is inf uence by multiple variables. Nicotine itsel pro uces rein-
orcement; users compare nicotine to stimulants such as cocaine or amphetamine,
although its e ects are o lower magnitu e. While there are many casual users o alco-
hol an cocaine, ew in ivi uals who smoke cigarettes smoke a small enough quan-
tity (5 cigarettes per ay) to avoi epen ence. Nicotine is absorbe rea ily through
the skin, mucous membranes, an lungs. T e pulmonary route pro uces iscernible
CNS e ects in as little as 7 secon s. T us, each pu pro uces some iscrete rein orce-
ment. With 10 pu s per cigarette, the 1-pack-per- ay smoker rein orces the habit
200 times aily. T e timing, setting, situation, an preparation all become associate
repetitively with the e ects o nicotine.
T ere is evi ence or tolerance to the subjective e ects o nicotine. Smokers typically
report that the rst cigarette o the ay a er a night o abstinence gives the best
eeling. Smokers who return to cigarettes a er a perio o abstinence may experi-
ence nausea i they return imme iately to their previous ose. Persons naive to the
e ects o nicotine will experience nausea at low nicotine bloo levels, an smokers will
experience nausea i nicotine levels are raise above their accustome levels.
Negative rein orcement re ers to the bene ts obtaine rom the termination o an
unpleasant state. In epen ent smokers, the urge to smoke correlates with a low
bloo nicotine level, as though smoking were a means to achieve a certain nicotine
(Continued)
258
Drug Addiction CHAPTER 1 4

level an thus avoi with rawal symptoms. Some smokers even awaken uring the
TABLE 14-5 Nicotine Withdrawal
night to have a cigarette, which ameliorates the e ect o low nicotine bloo levels
Symptoms
that coul isrupt sleep. T us, smokers may be smoking to achieve the rewar o
nicotine e ects, to avoi the pain o nicotine with rawal, or most likely a combina- Irritability, impatience, hostility
tion o the two. Nicotine with rawal symptoms are liste in able 14-5. Anxiety
Dysphoric or depressed mood
b. What options are available to help him with his desire to quit smoking? Dif culty concentrating
T e nicotine with rawal syn rome (see able 14-5) can be alleviate by nicotine- Restlessness
replacement therapy, available with a prescription (eg, NICO ROL inhaler an Decreased heart rate
nasal spray) or without (eg, NICORE E gum an others; COMMI lozenges an Increased appetite or weight gain
others; an NICODERM CQ trans ermal patch an others). Figure 24-3 in Good-
man and Gilmans T e Pharmacological Basis of T erapeutics, 12th E ition shows the
bloo nicotine concentrations achieve by i erent metho s o nicotine elivery.
Because nicotine gum an a nicotine patch o not achieve the peak levels seen with
cigarettes, they o not pro uce the same magnitu e o subjective e ects as smok-
ing. T ese metho s o, however, suppress the symptoms o nicotine with rawal.
T us, smokers shoul be able to trans er their epen ence to the alternative eliv-
ery system an gra ually re uce the aily nicotine ose with minimal symptoms.
Although this results in more smokers achieving abstinence, most resume smoking
over the ensuing weeks or months. Comparisons with placebo treatment show large
bene ts o nicotine replacement at 6 weeks, but the e ect iminishes with time.
T e nicotine patch pro uces a stea y bloo level an seems to have better patient
compliance than that observe with nicotine gum. Veri e abstinence rates at
12 months are reporte to be in the range o 20%. T e necessary goal o complete
abstinence contributes to the poor success rate; when ex-smokers slip an begin
smoking a little, they usually relapse quickly to their prior level o epen ence.
T e search or better me ications to treat nicotine a iction has become an impor-
tant goal o the pharmaceutical in ustry, an other types o me ication have been
teste in clinical trials. A sustaine -release preparation o the anti epressant bupro-
pion (ZYBAN; see Chapter 8) improves abstinence rates among smokers an remains
a use ul option. T e cannabinoi CB1 receptor inverse agonist rimonabant improves
abstinence rates an re uces the weight gain seen requently in ex-smokers. Un ortu-
nately, the CB1 inverse agonist mechanism le to a high requency o epressive an
neurologic symptoms, en ing its evelopment in the Unite States. Varenicline, a
partial agonist at the 42 subtype o the nicotinic acetylcholine receptor, improves
abstinence rates but has also been linke to risk o eveloping suici al i eation.
Varenicline partially stimulates nicotinic receptors, thereby re ucing craving an
preventing most with rawal symptoms. It has high receptor a nity, thus blocking
access to nicotine, so i the treate smoker relapses, there is little rewar an absti-
nence is more likely to be maintaine .

CASE 14-5
A 16-year-old woman is brought to the emergency room with a heroin overdose. T is
has occurred 8 hours a er she was discharged rom a detoxi cation center.
a. Why is heroin so addictive?
Injection o a heroin solution pro uces a variety o sensations escribe as warmth,
taste, or high an intense pleasure (rush) o en compare with sexual orgasm.
T ere are some i erences among the opioi s in their acute e ects, with morphine
pro ucing more o a histamine-releasing e ect an meperi ine pro ucing more
excitation or con usion.
T us, the popularity o heroin may be ue to its availability on the illicit mar-
ket an its rapi onset. A er intravenous injection, the euphoric e ects begin
in less than a minute. Heroin has high lipi solubility, crosses the bloo -brain
barrier quickly, an is eacetylate to the active metabolites 6-monoacetyl mor-
phine an morphine. A er the intense euphoria, which lasts rom 45 secon s
(Continued)
259
SECTION II Neuropharmacology

to several minutes, there is a perio o se ation an tranquility (on the no )


lasting up to an hour. T e e ects o heroin wear o in 3 to 5 hours, epen ing
on the ose. Experience users may inject 2 to 4 times per ay. T us, the heroin
a ict is constantly oscillating between being high an eeling the sickness o
early with rawal (see Figure 14-2). T is pro uces many problems in the homeo-
static systems regulate at least in part by en ogenous opioi s. For example, the
hypothalamic-pituitary-gona al axis an the hypothalamic-pituitary-a renal axis
are abnormal in heroin a icts. Women on heroin have irregular menses, an
men have a variety o sexual per ormance problems. Moo also is a ecte . Heroin
a icts are relatively ocile an compliant a er taking heroin, but uring with-
rawal, they become irritable an aggressive.
b. What is likely to have led to the overdose in this particular patient?
Base on patient reports, tolerance evelops early to the euphoria-pro ucing e ects
o opioi s. T ere also is tolerance to the respiratory epressant, analgesic, se ative,
an emetic properties. Heroin users ten to increase their aily ose, epen ing on
their nancial resources an the availability o the rug. I a supply is available, the
ose can be increase progressively by 100- ol . Even in highly tolerant in ivi u-
als, the possibility o over ose remains i tolerance is excee e . Over ose is likely
to occur when potency o the street sample is unexpecte ly high, when the heroin
is mixe with a ar more potent opioi , such as entanyl (SUBLIMAZE, others), or
a er release rom a etoxi cation program (loss o tolerance) with the in ivi ual
returning to her previously use ose o heroin, as occurre with this patient.
c. What treatment options are available to this patient?
Opioi with rawal signs an symptoms (see able 14-6) can be treate by 3 i erent
approaches. T e rst an most commonly use approach epen s on cross-tolerance
an consists o trans er to a prescription opioi me ication an then gra ual ose
re uction. T e same principles o etoxi cation apply as or other types o physical
epen ence. It is convenient to change the patient rom a short-acting opioi such
as heroin to a long-acting one such as metha one. Detoxi cation an subsequent
maintenance o opiate epen ence with metha one is speci cally limite to accre ite
opioi treatment programs (O Ps) an is regulate by Fe eral Opioi reatment
(Continued)

Me thado ne
h
g
i
H
l
a
m
r
o
N
k
c
i
S
He ro in

0 6 12 18 24

Time (hours )

FIGURE 14-2 Di erences in responses to heroin and methadone. A person who injects heroin
( ) several times per day oscillates (gray line) between being sick and being high. In contrast,
the typical methadone patient (blue line) remains in the normalrange (indicated in shaded
blue) with little uctuation a ter dosing once per day. The ordinate values represent the subjects
mental and physical state, not plasma levels o the drug.

260
Drug Addiction CHAPTER 1 4

TABLE 14-6 Characteristics o Opioid Withdrawal


SYMPTOMS SIGNS
Regular withdrawal

Craving or opioids Pupillary dilation

Restlessness, irritability Sweating

Increased sensitivity to pain Piloerection (goose esh)


Tachycardia

Nausea, cramps Vomiting, diarrhea

Muscle aches Increased blood pressure

Dysphoric mood Yawning

Insomnia, anxiety Fever

Protracted withdrawal

Anxiety Cyclic changes in weight, pupil


Insomnia size, respiratory center sensitivity
Drug craving

Stan ar s. T e initial ose o metha one is typically 20 to 30 mg. T is is a test ose to


etermine the level nee e to re uce observe with rawal symptoms. T e rst ays
total ose then can be calculate epen ing on the response an then re uce by
20% per ay uring the course o etoxi cation.
A secon approach to etoxi cation involves the use o oral cloni ine (CA APRES,
others), a me ication approve only or the treatment o hypertension. Cloni ine is
an 2 a renergic receptor agonist that ecreases a renergic neurotransmission rom
the locus ceruleus (see Chapter 7). Many o the autonomic symptoms o opioi with-
rawal such as nausea, vomiting, cramps, sweating, tachycar ia, an hypertension
result rom the loss o opioi suppression o the locus ceruleus system uring the
abstinence syn rome. Cloni ine, acting upon istinct non-opioi receptors but by
cellular mechanisms that mimic opioi e ects, can alleviate many o the symptoms
o opioi with rawal, but not the generalize aches an opioi craving. When using
cloni ine to treat with rawal, the ose must be titrate accor ing to the stage an
severity o with rawal, beginning with 0.2 mg orally; postural hypotension is a com-
mon si e e ect. A similar rug, lo exi ine (currently in clinical trials in the Unite
States), has greater selectivity or 2A a renergic receptors an is associate with less
o the hypotension that limits the use ulness o cloni ine in this setting.
A thir metho o treating opioi with rawal involves activation o the en ogenous
opioi system without me ication. T e techniques propose inclu e acupuncture
an several metho s o CNS activation using transcutaneous electrical stimulation.
While attractive theoretically, this has not yet been oun to be practical. Rapi
antagonist-precipitate opioi etoxi cation un er general anesthesia has receive
consi erable publicity because it promises etoxi cation in several hours while the
patient is unconscious an not experiencing with rawal iscom ort. A mixture o
me ications has been use , but morbi ity an mortality as reporte in the lay press
are unacceptable, with no emonstrate a vantage in long-term outcome.
T e most success ul treatment or heroin a iction consists o stabilization on
metha one in accor ance with state an e eral regulations. Patients who relapse
repeate ly uring rug- ree treatment can be trans erre irectly to metha one
without requiring etoxi cation. T e ose o metha one must be su cient to
prevent with rawal symptoms or at least 24 hours. T e intro uction o buprenor-
phine, a partial agonist at -opioi receptors (see Chapter 10), represents a major
(Continued)
261
SECTION II Neuropharmacology

change in the treatment o opiate a iction. T is rug pro uces minimal with-
rawal symptoms when iscontinue an has a low potential or over ose, a long
uration o action, an the ability to block heroin e ects. reatment can take place
in a quali e physicians private o ce rather than in a special center, as require
or metha one. When taken sublingually, buprenorphine (SUBU EX) is active,
but it also has the potential to be issolve an injecte (abuse ). A buprenor-
phine-naloxone combination (SUBOXONE) is also available. When taken orally
(sublingually), the naloxone moiety is not e ective, but i the patient abuses the
me ication by injecting, the naloxone will block or iminish the subjective high
that coul be pro uce by buprenorphine alone.
Another pharmacological option is opioi antagonist treatment. Naltrexone
(REVIA, others; see Chapter 10) is an antagonist with a high a nity or the -opioi
receptor (MOR); it will competitively block the e ects o heroin or other MOR ago-
nists. Naltrexone has almost no agonist e ects o its own an will not satis y craving
or relieve protracte with rawal symptoms. For these reasons, naltrexone treatment
oes not appeal to the average heroin a ict, but it can be use a er etoxi cation
or patients with high motivation to remain opioi - ree. Physicians, nurses, an
pharmacists who have requent access to opioi rugs make excellent can i ates or
this treatment approach. A epot ormulation o naltrexone that provi es 30 ays o
me ication a er a single injection (VIVI ROL) has been approve or the treatment
o alcoholism. T is ormulation eliminates the necessity o aily me ication an pre-
vents relapse when the recently etoxi e patient leaves a protecte environment.

CASE 14-6
A 26-year-old man is brought to the emergency room as the result o a motor vehicle
accident. A urine drug screen is positive or benzoylecgonine. A blood alcohol concen-
tration is just below the legal limit or driving.
a. What does the urine drug screen indicate?
T e major route or cocaine metabolism involves hy rolysis o each o its 2 ester
groups. Benzoylecgonine, pro uce on loss o the methyl group, represents the
major urinary metabolite an can be oun in the urine or 2 to 5 ays a er a binge.
As a result, the benzoylecgonine test is a vali metho or etecting cocaine use; the
metabolite remains etectable in the urine o heavy users or up to 10 ays.
b. What are the e ects o cocaine that may have led to the accident?
T e general pharmacology an me icinal use o cocaine as a local anesthetic are
iscusse in Chapter 11. Cocaine pro uces a ose- epen ent increase in heart
rate an bloo pressure accompanie by increase arousal, improve per ormance
on tasks o vigilance an alertness, an a sense o sel -con ence an well-being.
Higher oses pro uce euphoria, which has a brie uration an o en is ollowe
by a esire or more rug. Repeate oses may lea to involuntary motor activity,
stereotype behavior, an paranoia. Irritability an increase risk o violence are
oun among heavy chronic users. O particular concern in rivers who are taking
cocaine is increase risk-taking an a sense o invincibility.
c. Why is cocaine addiction so dif cult to treat?
A orm o mala aptive memory begins with the a ministration o substances (eg,
cocaine) or behaviors (eg, the thrill o gambling) that irectly an intensely activate
brain rewar circuits. Activation o these circuits motivates normal behavior an
most humans simply enjoy the experience without being compelle to repeat it. For
some (~16% o those who try cocaine) the experience pro uces strong con itione
associations to environmental cues that signal the availability o the rug or the
behavior. T us, ref exive activation o rewar circuits becomes involuntary an with
a very rapi onset. T e cues acquire strong salience that overwhelms other behav-
iors. T e in ivi ual becomes rawn into compulsive repetition o the experience
(Continued)
262
Drug Addiction CHAPTER 1 4

ocusing on the imme iate pleasure espite negative long-term consequences an


TABLE 14-7 Cocaine Withdrawal
neglect o important social responsibilities. O course, un erlying this behavior are
Symptoms and Signs
poorly un erstoo changes in neural circuits.
Since cocaine with rawal (see able 14-7) is generally mil , treatment o with- Dysphoria
rawal symptoms usually is not require . T e major problem in treatment is not Depression
etoxi cation but helping the patient to resist the urge to resume compulsive Sleepiness
Fatigue
cocaine use. Rehabilitation programs involving in ivi ual an group psychother- Cocaine craving
apy base on the principles o alcoholics anonymous, an behavioral treatments Bradycardia
base on rein orcing cocaine- ree urine tests, result in signi cant improvement in
the majority o cocaine users. Nonetheless, there is great interest in n ing a me i-
cation that can ai in the rehabilitation o cocaine a icts.
d. What other medical conditions is this patient at risk o ?
Other risks o cocaine, beyon the potential or a iction, inclu e car iac arrhyth-
mias, myocar ial ischemia, myocar itis, aortic issection, cerebral vasoconstric-
tion, an seizures. Death rom trauma also is associate with cocaine use.

CASE 14-7
A 35-year-old woman is an occasional user o cannabis.
a. What is the mechanism o action o cannabis?
T e cannabis plant has been cultivate or centuries both or the pro uction o
hemp ber an or its presume me icinal an psychoactive properties. T e smoke
rom burning cannabis contains many chemicals, inclu ing 61 i erent cannabi-
noi s that have been i enti e . One o these, -9-tetrahy rocannabinol (-9- HC),
pro uces most o the characteristic pharmacological e ects o smoke marijuana.
Cannabinoi receptors CB1 (mainly CNS) an CB2 (peripheral) have been i enti-
e an clone . An arachi onic aci erivative, anan ami e, has been propose as
an en ogenous ligan or CB receptors. While the physiological unction o these
receptors an their en ogenous ligan s are incompletely un erstoo , they are likely
to have important unctions because they are isperse wi ely with high ensities
in the cerebral cortex, hippocampus, striatum, an cerebellum.
b. What are the pharmacological e ects o marijuana?
T e pharmacological e ects o -9- HC vary with the ose, route o a ministra-
tion, experience o the user, vulnerability to psychoactive e ects, an setting o use.
Intoxication with marijuana pro uces changes in moo , perception, an motiva-
tion, but the e ect most requently sought is the high an mellowing out. T is
e ect is escribe as i erent rom the high pro uce by a stimulant or opiate.
E ects vary with ose, but typically last ~2 hours. During the high, cognitive unc-
tions, perception, reaction time, learning, an memory are impaire . Coor ination
an tracking behavior may be impaire or several hours beyon the perception o
the high, with obvious implications or the operation o a motor vehicle an per or-
mance in the workplace or at school.
Marijuana also pro uces complex behavioral changes such as gi iness an
increase hunger. T ere are unsubstantiate claims o increase pleasure rom sex
an increase insight uring a marijuana high. Unpleasant reactions such as panic
or hallucinations an even acute psychosis may occur; several surveys in icate that
50 to 60% o marijuana users have reporte at least one anxiety experience. T ese
TABLE 14-8 Marijuana Withdrawal
reactions are seen commonly with higher oses an with oral ingestion rather than
Syndrome
smoke marijuana, because smoking permits the titration o ose accor ing to the
e ects. While there is no convincing evi ence that marijuana can pro uce a last- Restlessness
ing schizophrenia-like syn rome, association stu ies suggest a correlation o early Irritability
marijuana use with an increase risk o later eveloping schizophrenia. Numerous Mild agitation
clinical reports suggest that marijuana use may precipitate a recurrence o psycho- Insomnia
Sleep EEG disturbance
sis in people with a history o schizophrenia.
Nausea, cramping

263
SECTION II Neuropharmacology

KEY CONCEPTS
olerance to a drug is the reduction in response to the drug with repeated use.
Physical dependence is a state that develops as a result o tolerance to a drug
produced by a resetting o homeostatic mechanisms in response to repeated
drug use.
Withdrawal symptoms are characteristic or a given category o drugs and tend
to be opposite to the original e ects produced by the drug be ore tolerance
developed.
Addiction is usually characterized by compulsive drug-taking.
olerance, physical dependence, and withdrawal are biological phenomena and
in themselves do not imply that the individual is involved in misuse or addic-
tive behavior.
Fear o producing a medical addict o en results in needless su ering among
patients with pain due to limited use o appropriate medications.
Opioids should never be withheld rom patients with cancer out o ear o pro-
ducing addiction.
Abuse o combinations o drugs is common; alcohol is so widely available that
it is o en combined with all categories o abused drugs.
Patients with a history o alcohol- or other drug-abuse problems have an increased
risk or the development o benzodiazepine abuse and should rarely, i ever, be
treated with benzodiazepines on a chronic basis.
Smokers may be smoking to achieve the reward o nicotine e ects, to avoid the
pain o nicotine withdrawal, or most likely a combination o the two.
T e management o drug abuse and addiction must be individualized accord-
ing to the drugs involved and the associated psychosocial problems o the indi-
vidual patient.

SUMMARY QUIZ

QUESTION 14-1 A 24-year-old woman has increased her dosage o hydrocodone to


achieve the same analgesic e ect. T is is a demonstration o
a. physical dependence.
b. rst-pass metabolism.
c. tolerance.
d. an adverse e ect.
e. addiction.

QUESTION 14-2 A 28-year-old man has been taking the same dose o oxycodone or
several weeks as the result o a knee injury. He has not needed to increase his dose
o oxycodone to achieve analgesia. He develops irritability and muscle aches upon
abruptly stopping his oxycodone. T is is a demonstration o
a. physical dependence.
b. rst-pass metabolism.
c. tolerance.
d. an adverse e ect.
e. addiction.

QUESTION 14-3 A 47-year-old man has consumed approximately 1 bottle o whiskey


daily or the past 5 years. He is brought to the emergency room one evening because
he is belligerent. In the emergency room he does not appear sedated despite a blood
(Continued)
264
Drug Addiction CHAPTER 1 4

alcohol concentration o 275 mg/dL. Within minutes o his blood being drawn he
develops a respiratory arrest and is intubated success ully. Because o tolerance to
alcohols sedative e ect the therapeutic index o alcohol in this patient is
a. increased.
b. decreased.
c. unchanged.
d. irrelevant.
e. negative rein orcement.

QUESTION 14-4 A 33-year-old woman who smokes 1 pack o cigarettes per day has
recently begun getting up at 3 AM to smoke a cigarette. T is is a demonstration o
a. positive rein orcement.
b. negative rein orcement.
c. insomnia.
d. depression.
e. increased therapeutic index o nicotine.

QUESTION 14-5 A 34-year-old man is a recreational user o cocaine. He is also a regu-


lar consumer o alcohol. T e e ect o alcohol on cocaine metabolism is demonstrated
by the presence in the urine o
a. benzoylecgonine.
b. methanol.
c. benzocaine.
d. cocaethylene.
e. methylecgonine.

QUESTION 14-6 A 19-year-old woman is brought to the emergency room because o


ear ul hallucinations and suicidal thoughts a er consuming methylenedioxymetham-
phetamine (MDMA). MDMA is thought to interact with
a. 2 adrenergic receptors.
b. 1 adrenergic receptors.
c. D2 dopaminergic receptors.
d. opioid receptors.
e. 5-H 2 serotonergic receptors.

QUESTION 14-7 A 48-year-old woman is undergoing chemotherapy or advanced


breast cancer. o allay her nausea and vomiting, she is given
a. diphenhydramine.
b. pseudoephedrine.
c. -9- HC.
d. LSD.
e. 10, 11-epoxycarbamezepine.

SUMMARY QUIZ ANSWER KEY


QUESTION 14-1 Answer is c. olerance, the most common response to repetitive use
o the same drug, can be de ned as the reduction in response to the drug a er repeated
administrations. Figure 14-1 shows an idealized dose-response curve or an administered
drug. As the dose o the drug increases, the observed e ect o the drug increases. With
repeated use o the drug, however, the curve shi s to the right (tolerance). T us, a higher
dose is required to produce the same e ect that was once obtained at a lower dose.

265
SECTION II Neuropharmacology

QUESTION 14-2 Answer is a. T e appearance o a withdrawal syndrome when


administration o the drug is terminated is the only actual evidence o physical depen-
dence. Withdrawal signs and symptoms occur when drug administration in a physi-
cally dependent person is terminated abruptly.
QUESTION 14-3 Answer is b. Experience with alcohol can produce greater tolerance
(acquired tolerance) such that extremely high blood levels (300-400 mg/dL) can be
ound in alcoholics who do not appear grossly sedated. In these cases, the lethal dose o
alcohol does not increase proportionately to the sedating dose, and thus the margin o
sa ety (therapeutic index) is decreased. T e concept o therapeutic index is introduced
in Chapter 3.
QUESTION 14-4 Answer is b. Negative rein orcement re ers to the bene ts obtained
rom the termination o an unpleasant state. In dependent smokers, the urge to smoke
correlates with a low blood nicotine level, as though smoking were a means to achieve
a certain nicotine level and thus avoid withdrawal symptoms (see able 14-5). Some
smokers even awaken during the night to have a cigarette, which ameliorates the e ect
o low nicotine blood levels that could disrupt sleep. T us, smokers may be smoking to
achieve the reward o nicotine e ects, to avoid the pain o nicotine withdrawal, or most
likely a combination o the two.
QUESTION 14-5 Answer is d. Ethanol is requently abused with cocaine, as it reduces
the irritability induced by cocaine. Dual addiction to alcohol and cocaine is common.
When cocaine and alcohol are taken concurrently, cocaine may be transesteri ed to
cocaethylene, which is equipotent to cocaine in blocking DA reuptake.
QUESTION 14-6 Answer is e. T e phenethylamine hallucinogens include mescaline,
dimethoxymethamphetamine (DOM), methylenedioxyamphetamine (MDA), and
MDMA, have a relatively high af nity or 5-H 2 receptors (see Chapter 7), but they
di er in their af nity or other subtypes o 5-H receptors. T ere is a good correlation
between the relative af nity o these compounds or 5-H 2 receptors and their potency
as hallucinogens in humans.
QUESTION 14-7 Answer is c. Marijuana has medicinal e ects, including antiemetic
properties that relieve side e ects o anticancer chemotherapy. An oral capsule con-
taining -9- HC (dronabinol; MARINOL, others) is approved or anorexia associated
with weight loss in patients with HIV in ection and or cancer chemotherapy-induced
nausea and vomiting.

266
SECTION

Modulation of Cardiovascular
Function III
15. Drug T erapy o Hypertension, Edema,
and Disorders o Sodium and Water Balance 268

16. Drug T erapy o Myocardial Ischemia 289

17. Pharmacotherapy T erapy o Heart Failure 301

18. Antiarrhythmic Drugs 312

19. Drug T erapy o T romboembolic Disorders 323

20. Drug T erapy o Dyslipidemias 334

267
CHAPTER
Drug Therapyof Hypertension,
15 Edema, and Disorders of Sodium
and Water Balance
T is chapter will be most use ul a er having a basic understanding o the material in
DRUGS INCLUDED IN THIS
the section on hypertension in Chapter 27 Drug T erapy o Myocardial Ischemia and
CHAPTER Hypertension in Goodman & Gilmans T e Pharmacological Basis of T erapeutics, 12th
Acetazolamide (DIAMOX,others) Edition (pp 765-785), as well as material in Chapter 25 Regulation o Renal Volume
Adrenergicreceptor antagonists (many; and Vascular Volume and Chapter 26 Renin and Angiotensin In addition to the mate-
Table 15-1; see Chapter 7) rial presented here, the 12th Edition contains:
Aliskiren (TEKTURNA) T e molecular structures o drugs used to treat hypertension, edema, and other
Amiloride (MIDAMOR, others) disorders o Na+ and water balance
Amiloride/hydrochlorothiazide (generic) Figure 25-1 T e anatomy and nomenclature o the nephron
Amlodipine (NORVASC) Figure 25-5 Changes in the extracellular uid volume and weight with diuretic therapy
Bendro umethiazide (NATURETIN) An extensive discussion o the unctions and e ects o the renin-angiotensin system
in Chapter 26
Benzapril (LOTENSIN, others)
Bumetanide (BUMEX,others) LEARNING OBJECTIVES
Candesartan cilexetil (ATACAND) Understand the mechanisms o action o drugs used to treat hypertension,
Captopril (CAPOTEN, others) edema, and other disorders o Na+ and water balance
Chlorothiazide (DIURIL) Know the untoward e ects o drugs used to treat hypertension, edema, and
Chlorthalidone (HYGROTON) other disorders o Na+ and water balance
Clevidipine (CLEVIPREX) Know which patients should be treated and when treatment should be initiated
Clonidine (CATAPRES, others) Know which drugs are most e ective in treating individual hypertensive
Conivaptan (VAPRISOL) patients with speci c comorbidities, including diabetes mellitus, congestive
Desmopressin acetate (1-deamino-8-d- heart ailure, and renal disease
arginine vasopressin, DDAVP, others) Know which drugs can be used in combination
Diazoxide Know how to set treatment goals based on target blood pressures and other
Dichlorphenamide (DARAMIDE) clinical measures
Diltiazem(CARDIZEM)
Doxazosin (CARDURA, others)
Enalapril (VASOTEC, others) MECHANISMS OF ACTION OF DIURETICS
Enalaprilat (VASOTECINJECTION, others) DRUG CLASS DRUG MECHANISM OF ACTION
Eplerenone (INSPRA, others) Inhibitors o Carbonic Acetazolamide Inhibit carbonic anhydrase
Ethacrynicacid (EDECRIN, others) Anhydrase Dichlorphenamide (CA) in the proximal tubule
Methazolamide resulting in abolition o
Felodipine (PLENDIL) NaHCO3 reabsorption
Fenoldopam(CORLOPAM) Inhibition o CA in the
collecting duct is a secondary
Fosinopril (MONOPRIL, others)
site o action
Furosemide (LASIX,others)
Osmotic Diuretics Glycerin Isosorbide Act at the loop o Henle
Glycerin (OSMOGLYN) Mannitol (primary site o action) to
Guanabenz(WYTENSIN) Urea diminish passive reabsorption
Guanadrel o NaCl by reducing medullary
tonicity
Guan acine (INTUNIV,TENEX) Act in the proximal tubule
Human recombinant ANP(carperitide, (secondary site) by increasing
available onlyin Japan) the osmolality o tubular uid
thereby reducing luminal
Human recombinant BNP(nesiritide, Na+ concentration and Na+
NATRECOR) reabsorption
(continues)
(Continued)
268
Drug Therapy of Hypertension and Na + Balance Disorders CHAPTER 1 5

DRUG CLASS DRUG MECHANISM OF ACTION DRUGS INCLUDED IN THIS


Inhibitors o the Na+-K+-2Cl Furosemide Inhibit activity o the Na+-K+- CHAPTER (Cont.)
Symporter (Loop Diuretics, Bumetanide 2Cl symporter in the thick
High-Ceiling Diuretics) Ethacrynic acid ascending limb o the loop o
Hydralazine (APRESOLINE)
Torsemide Henle Hydrochlorothiazide (HYDRODIURIL)
Highly e cacious diuretics Hydro umethiazide (SALURON)
because ~25% o the ltered
Na+ load is normally reabsorbed Indapamide (LOZOL)
by the thick ascending limb Irbesartan (AVAPRO)
Inhibitors o the Na+-Cl Bendro umethiazide Inhibit activity o Na+-Cl Isosorbide (ISMOTIC)
symporter (Thiazide and Chlorothiazide symporter in the distal Isradipine (DYNACIRC)
Thiazide-Like Diuretics) Hydrochlorothiazide collecting tubule
Lisinopril (PRINIVIL, ZESTRIL, others)
Hydro umethiazide Less e cacious than loop
Methyclothiazide diuretics because ~90% o the Losartan (COZAAR)
Polythiazide ltered Na+ load is reabsorbed Mannitol (OSMITROL, others)
Trichlormethiazide be ore reaching the distal
Chlorthalidone collecting tubule Methazolamide (GLAUCTABS)
Indapamide Methyclothiazide (ENDURON)
Metolazone
Methyldopa (ALDOMET,others)
Quinethazone
Metolazone (MYKROX,ZAROXOLYN)
Inhibitors o Renal Epithelial Triamterene Inhibit renal epithelial Na+
Minoxidil (LONITEN)
Na+ Channels (K+-Sparing Amiloride channels in the late distal
Diuretics) tubule and collecting duct Moexipril (UNIVASC, others)
Cause small increases in NaCl Nicardipine (CARDENE)
excretion and are antikaliuretic
Ni edipine (PROCARDIAER)
Mineralocorticoid Receptor Spironolactone Competitively inhibit Nisoldipine (SULAR)
Antagonists (Aldosterone Eplerenone aldosterone binding to the
Antagonists, K+-sparing mineralocorticoid receptor Nitroprusside
Diuretics) which blocks the synthesis o Olmesartan medoxomil (BENICAR)
aldosterone-induced proteins Perindopril (ACEON)
(AIPs) in epithelial cells in
the late distal tubule and Polythiazide (RENESE)
collecting duct Prazosin (MINIPRESS, others)
Diuretic Atrial Natriuretic Human recombinant ANP Activate natriuretic peptide Quinapril (ACCUPRIL, others)
Peptides (Inhibitors o the (carperitide, available only in receptors on the sur ace Quinethazone (HYDROMOX)
Cyclic Nucleotide-Gated Japan) o epithelial cells in the
Nonspeci c Cation Channel) Human recombinant BNP inner medullary collecting
Ramipril (ALTACE, others)
(nesiritide) duct which stimulates the Reserpine
ormation o cyclic GMP that Spironolactone (ALDACTONE, others)
subsequently inhibits the cyclic
nucleotide-gated nonspeci c Telmisartan (MICARDIS)
cation channel (CNGC); the Terazosin (HYTRINand others)
CNGC is the primary channel
Terlipressin (LUCASSIN)
or Na+ entry in these cells
Tolvaptan (SAMSCA)
Torsemide (DEMADEX,others)
CASE 15 1 Trandolapril (MAVIK, others)
A 43-year-old man has a blood pressure o 138/88 taken during his annual examination Triamterene (DYRENIUM)
He has no other health problems and his blood laboratory results are in the normal range Triamterene/hydrochlorothiazide
He is modestly overweight and has a amily history o cardiovascular disease (DYAZIDE, MAXZIDE, others)
a. What, i any, are the therapeutic options or this patient? Trichlormethiazide (NAQUA)
T is patient is considered to have prehypertension (see able 15-1). It is appropriate to Urea (UREAPHIL, currentlynot available in
recommend nonpharmacological approaches to reduce his risk o developing hyperten- the United States)
sion. T e li estyle modi cations that could lower this patients blood pressure include: Valsartan (DIOVAN)
Reduction in body weight Vasopressin (8-l -arginine vasopressin,
Restricting sodium consumption (the Dietary Approaches to Stop Hypertension PITRESSIN, others)
[DASH] diet may be particularly use ul) Verapamil (CALAN, others)
(Continued)
269
SECTION III Modulation o Cardiovascular Function

OVERVIEW OF TABLE 15-1 Criteria or Hypertension in Adults


NEPHRON ANATOMY, BLOOD PRESSURE (mm Hg)
NOMENCLATURE, AND
CLASSIFICATION SYSTOLIC DIASTOLIC
FUNCTION (see Figure 25-1
in Goodma n & Gilma ns The Normal <120 and <80
Pha rma cologica l Ba sis of Prehypertension 120-139 or 80-89
Thera peutics, 12th Edition
Hypertension, stage 1 140-159 or 90-99
The basicurine- orming unit o the kidney
is the nephron. Hypertension, stage 2 160 or 100
The nephronconsistso a ltering apparatus,
the glomerulus, connected toa long tubule Restriction o ethanol intake to modest levels
portion that reabsorbs substances the body Increased physical activity
must conserve, and leaves behind and/or
secretessubstances that must be eliminated. b. A year later at his next annual examination, the patients blood pressure is
142/91. What, i any, are the therapeutic options or this patient?
The nephron can be divided into 4 major
anatomical and unctional regions: T is patients prehypertension has progressed to stage 1 hypertension (see
able 15-1). In addition to emphasizing the importance o the li estyle modi cations
The proximal tubule
listed above, this patient should be started on antihypertensive drug therapy.

The loop o Henle (which consists o the


proximal straight tubule, the intermediate c. What are the considerations in choosing an appropriate antihypertensive drug
tubule, the thickascending limb) therapy or this patient?
The distal convoluted tubule Choice o an antihypertensive drug should be driven by the likely bene t in an
individual patient, taking into account concomitant diseases such as diabetes mel-
The collecting duct
litus, problematic adverse e ects o speci c drugs, and cost. National guidelines

Normally~65%o ltered Na+ is reab- recommend diuretics as pre erred initial therapy or most patients with uncompli-
sorbed in the proximal tubule; this part o cated stage 1 hypertension who are unresponsive to nonpharmacological measures.
the tubule is highlypermeable to water. Patients also are commonly treated with other drugs (see able 15-2): receptor
Approximately25%o ltered Na+ is reab- antagonists, ACE inhibitors/A 1-receptor antagonists, and Ca2+ channel blockers.
sorbed in the loop o Henle. (Continued)
The distal convoluted tubule activelytrans-
ports NaCl and is impermeable to water. TABLE 15-2 Classi cation o Antihypertensive Drugs by Their Primary Site or
Mechanism o Action
The ine control o ultra iltrate
composition and volume takes place Diuretics
in the collecting duct. 1. Thiazides and related agents (hydrochlorothiazide, chlorthalidone, chlorothiazide,
indapamide, methylclothiazide, metolazone)
The transport that occurs in a speci c 2. Loop diuretics ( urosemide, bumetanide, torsemide, ethacrynic acid)
segment o the nephron is primarily 3. K+-sparing diuretics (amiloride, triamterene, spironolactone)
determined bythe transporters present in
Sympatholytic drugs (see Chapter 7)
the epithelial cells and the whether they
1. receptor antagonists (metoprolol, atenolol, betaxolol, bisoprolol, carteolol, esmolol,
are located on the luminal or basolateral nadolol, nebivolol, penbutolol, pindolol, propranolol, timolol)
membrane. 2. receptor antagonists (prazosin, terazosin, doxazosin, phenoxybenzamine, phentolamine)
3. Mixed - receptor antagonists (labetalol, carvedilol)
4. Centrally acting adrenergic agents (methyldopa, clonidine, guanabenz, guan acine)
5. Adrenergic neuron blocking agents (guanadrel, reserpine)
Ca 2+ channel blockers (verapamil, diltiazem, nisoldipine, elodipine, nicardipine, isradipine,
amlodipine, clevidipine, ni edipine a)
PRINCIPLES OF DIURETIC
Angiotensin-converting enzyme inhibitors (captopril, enalapril, lisinopril, quinapril, ramipril,
ACTION benazepril, osinopril, moexipril, perindopril, trandolapril)
Diuretics increase the rate o urine ow. AngII receptor antagonists (losartan, candesartan, irbesartan, valsartan, telmisartan,
Clinicallyuse ul diuretics also increase the eprosartan, olmesartan)
rate o Na+ excretion (natriuresis), as well
Direct renin inhibitor (aliskiren)
as an anion such as Cl.
Most diuretics are used to reduce extracel- Vasodila tors
1. Arterial (hydralazine, minoxidil, diazoxide, enoldopam)
lular luid volume by decreasing total
2. Arterial and venous (nitroprusside)
NaCl content.
a
Extended-release ni edipine is approved or hypertension.
270
Drug Therapy of Hypertension and Na + Balance Disorders CHAPTER 1 5

d. He is prescribed hydrochlorothiazide (12.5 mg daily). What is the mechanism o


PRINCIPLES OF DIURETIC
action o this drug in reducing blood pressure?
ACTION (Cont.)
T e initial action o the thiazide diuretics is to decrease extracellular volume by
interacting with a thiazide-sensitive NaCl cotransporter expressed in the distal con- With chronicdiureticadministration,
voluted tubule in the kidney (see Figure 15-1), which enhances Na+ excretion in the renal compensatorymechanisms adjust
urine, leading to a all in cardiac output and a consequent reduction in mean arte- Na+ excretion to match Na+ intake (see
rial pressure. However, the hypotensive e ect is maintained during long-term ther- Figure 25-5 in Goodman &Gilmans The
apy due to decreased vascular resistance; cardiac output returns to pretreatment Pharmacological Basis of Therapeutics,
values and extracellular volume returns almost to normal due to compensatory 12th Edition).
responses such as activation o the renin-angiotensin system (RAS; Figure 15-2). Diuretics are classi ed basedon their mech-
T e explanation or the long-term vasodilation induced by these drugs is unknown. anismo action and their site o action in the
Hydrochlorothiazide may open Ca2+-activated K+ channels, leading to hyperpo- nephron (Table 15-3and Figure 15-1).
larization o vascular smooth muscle cells, which leads in turn to closing o L-type
Ca2+ channels and lower probability o opening, resulting in decreased Ca2+ entry
and reduced vasoconstriction. Hydrochlorothiazide also inhibits vascular carbonic
anhydrase, which hypothetically may alter smooth muscle cell cytosolic pH and
thereby cause opening o Ca2+-activated K+ channels with the consequences noted.
T e major action o thiazide diuretics on the NaCl cotransporterexpressed pre-
dominantly in the distal convoluted tubules and not in vascular smooth muscle
or the hearthas contributed to repeated suggestions that these drugs decrease
peripheral resistance as an indirect e ect o negative sodium balance. T at thiazides
lose e cacy in treating hypertension in patients with co-existing renal insu ciency
is compatible with this hypothesis.

TABLE 15-3 Excretory and Renal Hemodynamic Ef ects o Diureticsa


DIURETIC MECHANISM CATIONS ANIONS URIC ACID RENAL HEMODYNAMICS
(PRIMARY SITE OF ACTION) Na + K+ H+b Ca 2+ Mg 2+ Cl HCO3 H2PO4 ACUTE CHRONIC RBF GFR FF TGF
Inhibitors o CA (proximal + ++ - NC V (+) ++ ++ I - - - NC +
tubule)

Osmotic diuretics ++ + I + ++ + + + + I + NC - I
(loop o Henle)

Inhibitors o Na+-K+-2Cl- ++ ++ + ++ ++ ++ +c +c + - V(+) NC V(-) -


symport (thick ascending
limb)

Inhibitors o Na+-Cl- symport + ++ + V(-) V(+) + +c +c + - NC V(-) V(-) NC


(distal convoluted tubule)

Inhibitors o renal epithelial + - - - - + (+) NC I - NC NC NC NC


Na+ channels (late distal
tubule, collecting duct)

Antagonists o + - - I - + (+) I I - NC NC NC NC
mineralocorticoid receptors
(late distal tubule, collecting
duct)
a
Except or uric acid, changes are or acute ef ects o diuretics in the absence o signi cant volume depletion, which would trigger complex
physiological adjustments.
H , titratable acid and NH4+.
b +

c
In general, these ef ects are restricted to those individual agents that inhibit carbonic anhydrase. However, there are notable exceptions in which
symport inhibitors increase bicarbonate and phosphate (eg, metolazone, bumetanide).
++, +, (+), -, NC, V, V(+), V(-), and I indicate marked increase, mild to moderate increase, slight increase, decrease, no change, variable ef ect,
variable increase, variable decrease, and insu cient data, respectively. For cations and anions, the indicated ef ects re er to absolute changes in
ractional excretion.
RBF, renal blood ow; GFR, glomerular ltration rate; FF, ltration raction; TGF, tubuloglomerular eedback; CA, carbonic anhydrase.

271
SECTION III Modulation o Cardiovascular Function

CA inhibitors Na +Cl symport inhibitors


Inte rs titia l Tra ns port of
s pa ce Tight junction Lume n Lume n
diure tics into
proxima l tubule
Na + Na + Proxima l Blood in
Na + K+
Symporte r Antiporte r tubule K+ K+
H+
HCO 3 H+ HCO 3
Na + Na + ATPa s e
HCO 3
Symporte r Na +
H2 CO 3 H2 CO 3 Dis ta l
CA Cl Cl
convolute d Cl Cl
H2 O CA Glome rulus tubule
CO 2

CO 2
Blood
out Colle cting Thiazide
CA duct
inhibito rs diure tic s
Thick sys te m
a s ce nding
limb
K+-s pa ring diure tic
Na +K+2Cl symport inhibitors

Lo o p MR-MRA
(ina ctive ) MRA MRA
K+
diure tic s Nucle us
K +
MR-Aldo MR
K+ + Aldo Aldo
ATPa s e Na + mRNA
Na
Na + 2Cl Symporte r 2Cl (+)
Aldo-induce d prote ins
Cl Cl
K+
(+)
+ Na + Na +
K+ K
K+ Na + Na +
ATPa s e
2+ Re na l pa pilla
Ca K+ K+ K+
K+
Mg 2+

Na + c hanne l
inhibito rs

FIGURE 15-1 Summary o the site and mechanism o action o diuretics. Three important eatures o this summary gure are
worth special note. 1. Transport o solute across epithelial cells in all nephron segments involves highly specialized proteins,
which or the most part are apical and basolateral membrane integral proteins. 2. Diuretics target and block the action o epi-
thelial proteins involved in solute transport. 3. The site and mechanism o action o a given class o diuretics are determined by
the speci c protein inhibited by the diuretic. Aldo, aldosterone; CA, carbonic anhydrase; MR, mineralocorticoid receptor; MRA,
mineralocorticoid receptor antagonist.

MACULA DENSA (MD) PATHWAY NSAIDs


(? )
Tubula r (+) NaCl Re abs o rptio n
()
Na Cl De live ry of at
Re a bs orption Mac ula De ns a () P GI2 /P GE 2 ()
Na Cl to MB
by (+)
()
Proxima l
Loop ()
Tubule Diuretics (? ) S HORT
LOOP
RENIN
s
NEGATIVE
i
INTRARENAL BARORECEP TOR PATHWAY Re nin
s
FEEDBACK
e
RELEAS E (+) Inhibitors
r
u
Blo o d Pre s s ure
i
(+)
r
()
t
a
in
N
AngII ()
Pre -g lo me rular Ve s s e ls
e
r
(+) Forma tion
u
()
s
s
Vasodilators
e
r
ADRENERGIC RECEP TOR PATHWAY
P
(+) ACE
ARTERIAL () Re na l (+) Ac tivatio n o f Activa tion Inhibitors
() BLOOD Sympa the tic 1 Adre ne rg ic of AT1 R ()
High
P RES S URE Pre s s ure Tone Re c e pto rs o n JGCs
(+) Ba rore ce ptors (+) ()
Diuretics AT1
Blocke rs
Circula ting Ce ntra lly Acting
NE/Epi Sympa tholytics Blocke rs
P ha rma cologica l Inte rve ntions
P hys iologica l inte rre la tions hips
LONG LOOP NEGATIVE FEEDBACK
Ma jor pa thways modula ting re nin re le a s e

FIGURE 15-2 Schematic portrayal o the 3 major physiological pathways regulating renin release. ACE, angiotensin-converting
enzyme; AngII, angiotensin II; AT1 R, angiotensin subtype 1 receptor; JGCs, juxtaglomerular cells; MD, macula densa; NE/Epi,
norepinephrine/epinephrine; NSAIDs, nonsteroidal anti-in ammatory drugs; PGI2/PGE2, prostaglandins I2 and E2.

272
Drug Therapy of Hypertension and Na + Balance Disorders CHAPTER 1 5

MECHANISMS OF ACTION OF INHIBITORS OF THE RENIN-ANGIOTENSIN SYSTEM (RAS; see Table 15-4)
DRUG CLASS DRUG MECHANISM OF ACTION
Angiotensin Converting Enzyme Captopril Prevent the conversion o AngI to active AngII by inhibiting angiotensin
Inhibitors (ACEIs) Enalapril converting enzyme (ACE), thus blocking AngIIs pressor responses and
Enalaprilat cardiovascular structural ef ects (see Figures 15-2, 15-3, and 15-4)
Lisinopril Note: Many o the drugs are ester-containing prodrugs that have better
Benzapril bioavailability than the active molecules (see Figure 26-9 rom Goodman &
Fosinopril Gilmans The Pharmacological Basis of Therapeutics, 12th Edition or
Trandolapril structures o drugs and prodrugs)
Quinapril
Ramipril
Moexipril
Perindopril

Angiotensin Receptor Blockers (ARBs) Candesartan cilexetil Competitive (but insurmountable) inhibitors o angiotensin AT1 receptors
Irbesartan (see Figures 15-2 and 15-4) that block AngIIs pressor responses and
Losartan cardiovascular structural ef ects (see Figure 15-3)
Olmesartan medoxomil Note: Some o the drugs are prodrugs that have better bioavailability
Telmisartan than the active molecules (see Figure 26-10 rom Goodman &Gilmans The
Valsartan Pharmacological Basis of Therapeutics, 12th Edition or structures o drugs
and prodrugs)

Direct Renin Inhibitors (DRIs) Aliskiren Competitive inhibitor o renin preventing the conversion o
angiotensinogen to AngI (see Figures 15-2 and 15-4)

CASE 15 2
A 68-year-old woman is being treated with 25 mg o hydrochlorothiazide or her
hypertension During her most recent checkup, her blood pressure was 161/93 and
she was also diagnosed with type 2 diabetes
a. What changes should be made in her treatment?
T is patient has stage 2 hypertension (see able 15-1) and an important comorbid-
ity, type 2 diabetes mellitus. T e risk o cardiovascular disease, disability, and death
in hypertensive patients is increased markedly by concomitant cigarette smoking,
diabetes, or elevated low-density lipoprotein; the coexistence o hypertension with
(Continued)

TABLE 15-4 Ef ects o Anti-hypertensive Agents on Components o the RAS


DIRECT RENIN INHIBITORS ACE-INHIBITORS ARBs DIURETICS Ca 2+ CHANNEL BLOCKERS BLOCKERS

PRC

PRA

AngI

AngII

ACE

Bradykinin

AT1 receptors inhibition

AT2 receptors stimulation

PRC, plasma renin concentration; PRA, plasma renin activity; ACE, angiotensin converting enzyme; ARB, angiotensin receptor blocker.

273
SECTION III Modulation o Cardiovascular Function

DIURETIC THERAPY
Ang io te ns in II
Diuretics are used to treat hypertension
(see Table 15-2), edema, or volume over-
load associated with various disease states
(see Figure 15-5). Alte re d Alte re d Alte re d
Pe riphe ral Re nal Cardiovas c ular
Restriction o dietaryNa+ intake is also Re s is tanc e Func tio n S truc ture
important in reducing hypertension
and edema, but compliance can be
MECHANIS MS MECHANIS MS MECHANIS MS
problematic.
I. Dire ct e ffe ct to incre a s e Na +
The pathophysiological mechanisms o I. Dire ct va s ocons triction
re a bs orption in proxima l
I. Non-he modyna mica lly-
me dia te d e ffe cts :
edema ormation and the e ects o diuret- II. Enha nce me nt of tubule . A. Incre a s e d expre s s ion of
proto-oncoge ne s
ics and dietaryNa+ restriction are shown in pe riphe ra l nora dre ne rgic II. Re le a s e of a ldos te rone
B. Incre a s e d production of
ne urotra ns mis s ion: from a dre na l cortex
Figure 15-5. A. Incre a s e d NE re le a s e (incre a s e d Na + re a bs orption
growth fa ctors
B. De cre a s e d NE re upta ke C. Incre a s e d synthe s is of
An algorithm or treating edema in patients a nd incre a s e d K+ excre tion extra ce llula r ma trix
C. Incre a s e d va s cula r in dis ta l ne phron) prote ins
caused byrenal, hepatic, or cardiacdisor- re s pons ive ne s s
III. Alte re d re na l he modyna mics :
ders is shown in Figure 15-6. III. Incre a s e d sympa the tic A. Dire ct re na l va s ocons triction II. He modyna mica lly
B. Enha nce d nora dre ne rgic me dia te d e ffe cts :
dis cha rge (CNS )
Diureticresistance to a less e ective ne urotra ns mis s ion in A. Incre a s e d a fte rloa d
(ca rdia c)
diureticsuch as a thiazide mayrequire sub- IV. Re le a s e of ca te chola mine s kidney
B. Incre a s e d wa ll te ns ion
from a dre na l me dulla C. Incre a s e d re na l sympa the tic
stituting with a more ef cacious drug such tone (CNS ) (va s cula r)

as a loop diuretic, but resistance to loop


diuretics can also occur. RES ULT RES ULT RES ULT

NSAIDcoadministration, which reduces the Ra pid Pre s s or Re s pons e S low Pre s s or Re s pons e
Va s cula r a nd Ca rdia c
production o prostaglandins (especially Hypertrophy and Remodeling
PGE2), is a common cause o diureticresis-
FIGURE 15-3 Summary o the 3 major ef ects o AngII and the mechanisms that mediate them.
tance that can be prevented. NE, norepinephrine.
There are several options available to over-
come resistance to loop diuretics:
Bed rest to increase renal circulation
Increasing dose o the diureticto the
ceiling dose these risk actors increases cardiovascular morbidity and mortality to a degree
Administering smaller doses more that is compounded by each additional risk actor. Because the purpose o treating
requently hypertension is to decrease cardiovascular risk, other dietary and pharmacological
interventions may be required to treat these comorbid conditions. ACE inhibitors/
Continuous intravenous in usion
A 1-receptor antagonists should be rst-line drugs in the treatment o diabetics
Combination therapywith 2 diuretics with hypertension in view o these drugs well-established bene ts in slowing the
with di erent sites o action (eg, a development and progression o diabetic glomerulopathy.
thiazide and loop diuretic), although
patients should be monitored to b. How would this patients treatment di er i she did not have diabetes?
avoid serious complications such as Patients with uncomplicated stage 2 hypertension will likely require a diuretic and
hyponatremia, hypokalemia, and another drug rom a di erent class. Subsequently, doses can be titrated upward and
volume depletion additional drugs added to achieve goal blood pressures (blood pressure <140/90 mm
Reducing salt intake Hg in uncomplicated patients). Some o these patients may require our di erent
drugs to reach the goal.
Administering diuretics shortly
be ore ood intake to ensure e ective c. T is patient is prescribed enalapril in addition to her hydrochlorothiazide. What
concentrations o diureticin the tubule adverse e ects are possible that the patient should be counseled to be aware o ?
lumen when salt load is highest T ere are several cautions in the use o ACE inhibitors such as enalapril in patients
with hypertension. Following the initial dose o an ACE inhibitor, there may be a
considerable all in blood pressure in some patients; this response to the initial dose
is a unction o plasma renin activity prior to treatment. T e potential or a large
initial drop in blood pressure is the reason or using a low dose to initiate therapy,
especially in patients who may have a very active RAS supporting blood pressure,
such as patients with diuretic-induced volume contraction or congestive heart ailure.
(Continued)

274
Drug Therapy of Hypertension and Na + Balance Disorders CHAPTER 1 5

Ang io te ns ino g e n
PHYSIOLOGICAL FACTORS
AND PHARMACOLOGICAL
DRI Re nin
AGENTS REGULATING
RENIN RELEASE
Ang io te ns in I (1-10) se e Figure 15-2)
As p-Arg-Va l-Tyr-Ile -His -Pro-P he -His -Le u Physiological actors
Bra dykinin 1 2 3 4 5 6 7 8 9 10
Arterial blood pressure (intrarenal
chyma s e baroreceptor pathway)
ACE ACE-I ACE
Dietarysalt intake (macula densa
pathway)
Ina ctive
pe ptide s Ang io te ns in II (1-8) Activation o the sympatheticnervous
system( adrenergicreceptor pathway)
ARB
Prostaglandins (PGI2/PGE2)
Activation o juxtaglomerular
AT1 angiotensin AT1 receptors
Pharmacological agents
FIGURE 15-4 Inhibitors o the RAS. ACE-I, angiotensin-converting enzyme inhibitor; ARB, Loop diuretics
angiotensin receptor blocker; DRI, direct renin inhibitor. Nonsteroidal anti-in ammatoryagents
(NSAIDs)
ACEinhibitors, ARBs, and renin
inhibitors
Centrallyacting sympatholyticdrugs
1
Adrenergicblockers
DIURETICS
n
o
i
e
+
t
t
MABP MABP
e
a
a
r
N
R
c
x
E
MABP S e ns e d ECFV

Na + excre tion ra te "S e ns e d"


In s p rip a
n la u s

ECFV
te a h )
)
st n

(=
n
e o

rs c e e r
c o rc u n o

io
g ti

p de

tit
e m
(= c Ve

ia
l

2 5
i

Ne t Na + = Na + inta ke
l

die ta ry 6
c
Othe r
o
y
Na + Na + excre tion
r
m
los s e s
a
4 Tota l
)
p
O
n
a
s
othe r los s e s
a
inta ke
o
g
t
m
(e g , swe a t,
h
r
m
ECFV
n
t
e
e
m
u
l
fe ce s, e tc.)
d
r
u
L
e
e
p
n
=
t
s
3
(
H2 O inta ke
(=

Na + Th irs
t
Psc a
cae
ivrte

re s triction Ne t Na +
iitotsy

ECFV
n)

ADH
e
a

me ch
l

a n is m H2 O excre tion

FIGURE 15-5 Interrelationships among renal unction, Na+ intake, water homeostasis, distribu-
tion o extracellular uid volume, and mean arterial blood pressure. Pathophysiological mecha-
nisms o edema ormation. 1. Rightward shi t o renal pressure natriuresis curve. 2. Excessive
dietary Na+ intake. 3. Increased distribution o extracellular uid volume (ECFV) to peritoneal cavity
(eg, liver cirrhosis with increased hepatic sinusoidal hydrostatic pressure) leading to ascites orma-
tion. 4. Increased distribution o ECFVto lungs (eg, le t-sided heart ailure with increased pulmonary
capillary hydrostatic pressure) leading to pulmonary edema. 5. Increased distribution o ECFVto
venous circulation (eg, right-sided heart ailure) leading to venous congestion. 6. Peripheral edema
caused by altered Starling orces causing increased distribution o ECFVto interstitial space (eg,
diminished plasma proteins in nephrotic syndrome, severe burns, and liver disease).

275
SECTION III Modulation o Cardiovascular Function

Chro nic Mo de rate Cirrho s is


re nal Ne phro tic or Mild
failure s yndro me s eve re CHF
CHF S pironola ctone :
Titra te up to 400 mg/d C Cr
C r
a s ne e de d. < l Cl
50
CrCl >

C r
C

> l
< 50

0
5
50
a dd a dd

Loop diure tic: Titra te s ingle da ily dos e up to ce iling dos e * a s ne e de d


Drop Thia zide Thia zide :
50 to 100 mg/d HCTZ

Loop diure tic: Incre a s e fre que ncy of ce iling doe s a s ne e de d:


Furos e mide, up to 3X da ily; Bume ta nide, up to 4X da ily; Tors e mide, up to 2X da ily

A
d
d
K+-s pa ring diure tic: Thia zide diure tic:
If CrCl > 75 & urina ry [Na ]:[K] ra tio is < 1 CrCl >50, us e 25 to 50 mg/d HCTZ
(Note : May a dd K+-s pa ring diure tic to loop CrCl 20 to 50, us e 50 to 100 mg/d HCTZ
a nd/or thia zide diure tic a t a ny point in a lgorithm CrCl <20, us e 100 to 200 mg/d HCTZ
for K+ home os ta s is.)

While ma inta ining othe r diure tics, switch loop a ge nt to continuous infus ion

FIGURE 15-6 Braters algorithm or diuretic therapy o chronic renal ailure, nephrotic syndrome,
congestive heart ailure, and cirrhosis. Follow algorithm until adequate response is achieved. I
adequate response is not obtained, advance to the next step. For illustrative purposes, the thi-
azide diuretic used in Braters algorithm is hydrochlorothiazide (HCTZ). An alternative thiazide-type
diuretic may be substituted with appropriate dosage adjustment so as to be pharmacologically
equivalent to the recommended dose o HCTZ. Do not combine two K+-sparing diuretics because of
the risk of hyperkalemia. CrCl indicates creatinine clearance in mL/min, and ceiling dose re ers to the
smallest dose o diuretic that produces a near-maximal ef ect. Ceiling doses o loop diuretics and
dosing regimens or continuous intravenous in usions o loop diuretics are disease-state-speci c.
Doses are or adults only.

With continuing treatment, there usually is a progressive all in blood pressure that in
most patients does not reach a maximum or several weeks.
ACE inhibitors not only prevent conversion o angiotensin I (AngI) to AngII, but
also block degradation o bradykinin and substance P. T e increase in bradykinin
and substance P can give rise to a common (5-20% o patients) adverse e ect, dry
cough, and may also contribute to the less requent (0.1-0.5%) occurrence o angio-
edema. Angioedema is a rare but serious and potentially atal adverse e ect o the
ACE inhibitors. Patients starting treatment with these drugs should be explicitly
warned to discontinue their use with the advent o any signs o angioedema; once
ACE inhibitors are stopped, angioedema disappears within hours. AngII receptor
antagonists have a reduced risk o dry cough and angioedema and can be used as
alternative RAS inhibitors in patients who cannot tolerate the dry cough or who
develop angioedema.
T e attenuation o aldosterone production by ACE inhibitors also in uences K+
homeostasis. T ere is only a very small and clinically unimportant rise in serum K+
when these agents are used alone in patients with normal renal unction. However,
substantial retention o K+ can occur in some patients with renal insu ciency or
diabetes. Furthermore, the potential or developing hyperkalemia should be con-
sidered when ACE inhibitors are used with other drugs that can cause K+ retention,
including the K+-sparing diuretics (amiloride, triamterene, and spironolactone),
NSAIDs, K+ supplements, and receptor antagonists. Some patients with diabetic
nephropathy may be at greater risk o hyperkalemia.

276
Drug Therapy of Hypertension and Na + Balance Disorders CHAPTER 1 5

MECHANISMS OF ACTION OF SYMPATHOLYTIC AGENTS USED IN TREATING HYPERTENSION


DRUG CLASS DRUG MECHANISM OF ACTION
Adrenergic Receptor Many (see Table 15-2 and Blocking adrenergic receptors in the heart results in reduction in cardiac
Antagonists Chapter 7) output by lowering myocardial contractility and heart rate, thus lowering
arterial blood pressure
Blockade o adrenergic receptors in the juxtaglomerular complex reduces
renin secretion, thereby reducing plasma AngII
Some o the adrenergic blockers have additional ef ects on lowering blood
pressure through ef ects on blocking 2 adrenergic receptors (labetalol,
carvedilol), or increasing NO production (nebivolol)

1 Adrenergic Receptor Prazosin Block 2 adrenergic receptors in arteriolar and venous vascular smooth muscle
Antagonists Terazosin reduces arteriolar resistance and increases venous capacitance, respectively
Doxazosin

Centrally Acting Adrenergic Methyldopa Converted to -methylnorepinephrine and stored in central adrenergic
AgentsMethyldopa neurons; it inhibits adrenergic neuronal out ow rom the brainstem to the
peripheral sympathetic nervous system

Centrally Acting Adrenergic Clonidine Stimulate the 2A subtype o 2 adrenergic receptors in the brainstem,
Agents2 Adrenergic Guanabenz reducing sympathetic out ow rom the CNS
Receptor Antagonists Guan acine

Adrenergic Neuron Blocking Guanadrel Guanadrel acts as a alse (inactive) neurotransmitter in postganglionic
Agents Reserpine adrenergic neurons
Reserpine acts at central and peripheral adrenergic neurons to delete
catecholamines

MECHANISMS OF ACTION OF L-TYPE CA2+ CHANNEL BLOCKERS


DRUG CLASS DRUG MECHANISM OF ACTION
Dihydropyridines Nisoldipine Inhibit L-type voltage-gated Ca2+ channels in arteriolar vascular smooth
Felodipine resulting in smooth muscle relaxation
Nicardipine
Isradipine
Amlodipine
Clevidipine
Ni edipine

Nondihydropyridines Verapamil Inhibit L-type voltage-gated Ca2+ channels in arteriolar vascular smooth
Diltiazem resulting in smooth muscle relaxation
The nondihydropyridine calcium channel blockers also inhibit L-type Ca2+
channels in heart, thereby lowering cardiac output by ef ects on SA node
(bradycardia) and reduced contractility

CASE 15 3
A 31-year-old woman is being treated or mild hypertension with valsartan and hydro-
chlorothiazide She is in good health and becomes pregnant
a. Are the drugs she is taking or her hypertension sa e in pregnancy?
All o the thiazide-like drugs cross the placenta, but they have not been shown to
have direct adverse e ects on the etus. However, ACE inhibitors, A 1-receptor
antagonists, and the direct renin inhibitor, aliskiren, are teratogenic and should be
discontinued as soon as pregnancy is detected.
b. What changes in her drug therapy should be made?
T is woman had chronic hypertension prior to her pregnancy, but is considered
low risk because she had mild hypertension with no evidence o organ damage. o
minimize risk to the etus, she should not receive an antihypertensive drug unless
(Continued)
277
SECTION III Modulation o Cardiovascular Function

P re gna nt wome n with chronic hype rte ns ion

Low ris k High ris k

No a ntihype rte ns ive drugs Hos pita liza tion a t initia l vis it

Ultra s ound e xa mina tion a t 16-20 wk, Antihype rte ns ive drugs a re ne e de d to ke e p s ys tolic be low
re pe a t a t 30-32 wk, a nd 140 a nd dis a tolic be low 90 mm Hg
monthly a fte r tha t until te rm
Ultra s ound e xa mina tion a t 16-20 wk, re pe a t a t
28 wk a nd the n e ve ry 3 wk until de live ry

Nons tre s s te s t a nd/or biophys ica l profile a t 28 wk


1. Antihype rte ns ive drugs if s e ve re a nd the n we e kly
hype rte ns ion de ve lops

2. If pre e cla mps ia de ve lops , if a nti- 1. Hos pita liza tion if the re is e xa ce rba tion to s e ve re
hype rte ns ive drugs a re us e d, or if hype rte ns ion, if the re is pre e cla mps ia , or if the re is
the re is a bnorma l fe ta l growth, e vide nce of a bnorma l fe ta l growth
the n be gin imme dia te fe ta l te s ting
with nons tre s s te s t or biophys ica l 2. Fre que nt e va lua tion of ma te rna l a nd fe ta l we ll-be ing
profile . Continue s e ria l te s ting
until de live ry 3. Cons ide r de live ry a t 36-37 wk

FIGURE 15-7 Algorithm or the management o pregnant women with chronic hypertension.
(Reproduced with permission from Sibai BM. Chronic hypertension in pregnancy. Obstet Gynecol.
2002;100:369-377.)

her systolic blood pressure exceeds 160 mm Hg or her diastolic pressure exceeds
110 mm Hg (see Figure 15-7). I her blood pressure exceeds these values, methyl-
dopa is the drug o choice because o its record o sa ety and e cacy. Hydralazine
and a -adrenergic blocker such as labetalol can also be used. Patients at high risk
or who develop hypertension during pregnancy should be managed according to
the algorithm shown in Figure 15-7.

MECHANISMS OF ACTION OF DIRECT VASODILATORS


DRUG CLASS DRUG MECHANISM OF ACTION
Arterial vasodilators Hydralazine Hydralazine directly relaxes arteriolar smooth muscle, but the mechanism is unclear
Minoxidil Minoxidil is bioactivated to orm minoxidil sul ate which relaxes arteriolar smooth
muscle by activating ATP-modulated K+ channels (KATP channels)
Diazoxide Diazoxide activates vascular smooth muscle K+ channels
Fenoldopam Fenoldopam is a short-acting D1 dopamine receptor agonist that has minimal
adrenergic ef ects
Arterial and venous vasodilators Nitroprusside Acts as a nonselective vasodilator by releasing NO through a mechanism(s) that
is unclear

CASE 15 4
A 68-year-old man has taken diltiazem or his hypertension and his chronic angina
or more than 5 years He su ers a mild myocardial in arction and during his stay in
the hospital, he is taken o diltiazem and placed on a regimen that includes captopril,
carvedilol, and aspirin
a. What is the rationale or taking this patient o diltiazem?
Diltiazem is a calcium channel blocker that is e ective in treating hypertension
and stable angina because o its e ects on relaxing vascular smooth muscle, thus
reducing arterial blood pressure and dilating coronary arteries. Its calcium chan-
nel blocking activity also reduces cardiac output by slowing heart rate and lowering
cardiac contractility. Because o its negative inotropic e ects, it can impair cardiac
output during and af er a myocardial in arction.
(Continued)
278
Drug Therapy of Hypertension and Na + Balance Disorders CHAPTER 1 5

b. What is the rationale or placing this patient on captopril during his stay in
the hospital?
T e bene cial e ects o ACE inhibitors in acute myocardial in arction are particu-
larly large in hypertensive and diabetic patients. Unless contraindicated (eg, cardio-
genic shock or severe hypotension), ACE inhibitors should be started immediately
during the acute phase o myocardial in arction and can be administered along
with thrombolytics, aspirin, and adrenergic receptor antagonists. In high-risk
patients (eg, large in arct, systolic ventricular dys unction), ACE inhibition should
be continued long term. One o the bene ts o long-term ACE inhibitor therapy is
to inhibit AngII-stimulated remodeling o the cardiovascular system, which causes
hypertrophy o vascular and cardiac cells and increased synthesis and deposition o
collagen by cardiac broblasts (see Figure 15-3). T is remodeling process is thought
to be an important eature in the development o chronic heart ailure. Current rec-
ommendations are to use ACE inhibitors as rst-line agents or the treatment o heart
ailure and to reserve ARBs or treatment o heart ailure in patients who cannot tol-
erate or have an unsatis actory response to ACE inhibitors.
c. Within an hour o starting the patient on captopril, he begins to notice swelling
in his nose, throat, mouth, glottis, larynx, lips, and tongue. What is likely to be
causing this adverse e ect and how can it be reversed?
T e patient is experiencing angioedema, which is caused by an increase in circu-
lating bradykinin. A small number o patients receiving ACE inhibitors experi-
ence angioedema that results rom inhibition o ACE, the enzyme that degrades
bradykinin and substance P. o reverse the drugs e ect, the ACE inhibitor should
be stopped, and the angioedema will disappear within hours. While the e ects o
the angioedema are diminishing, the patients airway should be protected, and i
necessary, epinephrine, an antihistamine, and/or a glucocorticoid should be admin-
istered. A rican Americans have a 4.5 times greater risk o ACE inhibitorinduced
angioedema than Caucasians. Although rare, angioedema o the intestine (visceral
angioedema) characterized by emesis, watery diarrhea, and abdominal pain also
has been reported. As an alternative to an ACE inhibitor in this patient, an ARB
such as valsartan can be substituted. T e incidence o angioedema with ARBs is
much lower than with ACE inhibitors. Valsartan is approved or heart ailure and
to reduce cardiovascular mortality in clinically stable patients with lef ventricular
ailure or lef ventricular dys unction ollowing myocardial in arction.
d. What is the rationale or starting this patient on carvedilol?
Adrenergic receptor antagonists that do not have intrinsic sympathomimetic
activity (ie, partial agonists) improve mortality in myocardial in arction patients.
T ey should be started early and continued inde nitely in patients who can toler-
ate them. Carvedilol is a mixed receptor antagonist that is approved or use in
patients with lef ventricular dys unction.

MECHANISMS OF ACTION OF VASOPRESSIN RECEPTOR AGONISTS AND ANTAGONISTS (see Figure 15-8)
DRUG CLASS DRUG MECHANISM OF ACTION
Vasopressin AgonistV1 Receptor Agonists Vasopressin Activate V1 receptors in GI and vascular smooth muscle to cause contraction
Terlipressin

Vasopressin AgonistV2 Receptor Agonist Desmopressin Activate V2 receptors in collecting duct to increase water permeability (antidiuresis)
Activate extrarenal V2 receptors on vascular endothelium to release procoagulant
actor VIII and von Willebrand actor

Vasopressin AntagonistsSelective V2 Tolvaptan Inhibit V2 receptors in collecting duct to increase ree water excretion without
Receptor (V2R) Antagonists (Aquaretics) excretion o electrolytes

Vasopressin AntagonistsNonselective V1aR/ Conivaptan Inhibit V2 receptors in collecting duct to increase ree water excretion without
V2R Antagonist (Aquaretics) excretion o electrolytes
Inhibition o V1a receptors in mesangial cells and vascular smooth muscle cells
o vasa recta and ef erent arteriole acts to increase inner medullary blood ow

279
SECTION III Modulation o Cardiovascular Function

Ra pid de cre a s e
in inne r V1a Re ce ptor Vas o pre s s in
me dulla ry
blood flow

V2 Re c e pto r
Ra pid a ctiva tion
of VRUT in a pica l
me mbra ne of IMCD

Ra pid a ctiva tion


of Na +K+2Cl
symporte r to a pica l
me mbra ne of TAL

Ra pid tra ns loca tion


of Na +K+2Cl
symporte r to a pica l
me mbra ne of TAL
Long-te rm incre a s e
Long-te rm incre a s e in expre s s ion of
in expre s s ion of a qua porin-2
Na +K+2Cl in principa l ce lls of
symporte r in TAL colle cting duct

Rapid ins e rtio n o f


Incre a s e d
pre fo rme d aquapo rin-2
me dulla ry
into apic al me mbrane
os mola lity
o f princ ipal c e lls
in c o lle c ting duc t

Inc re as e d pe rme ability o f


c o lle c ting duc t to wate r

Re nal c o ns e rvatio n o f wate r

FIGURE 15-8 Mechanisms by which vasopressin increases the renal conservation o water. Gray
and black arrows denote major and minor pathways, respectively. IMCD, inner medullary collect-
ing duct; TAL, thick ascending limb; VRUT, vasopressin-regulated urea transporter.

CASE 15 5
A 74-year-old male patient with heart ailure is hospitalized because o signi cant pul-
monary edema that is impairing his breathing He is receiving valsartan and urosemide
a. What are the mechanisms o action o these agents?
Valsartan is an angiotensin receptor blocker (ARB) that lowers blood pressure and
prevents vascular and cardiac remodeling by blocking the e ects o AngII on A 1
receptors (see Figure 15-3). It has been shown to reduce morbidity and mortality in
patients with heart ailure. Furosemide is a highly e cacious diuretic that inhibits
the Na+-K+-2Cl symporter in the thick ascending limb o the loop o Henle; hence,
it is termed a loop diuretic.
b. What additional agent(s) might be added to improve natriuresis and improve
hemodynamics?
An agent that might be use ul in this patient is nesiritide which is recombinant
human B-type natriuretic peptide (BNP). BNP is an endogenous peptide released
by cardiac ventricular muscle when it is stretched. BNP activates natriuretic peptide
receptors on the sur ace o epithelial cells in the inner medullary collecting duct
which stimulates the ormation o cyclic GMP that subsequently inhibits the cyclic
nucleotide-gated nonspeci c cation channel (CNGC); the CNGC is the primary
channel or Na+ entry in these cells. BNP also relaxes vascular smooth muscle and
lowers vascular resistance.
Eplerenone might also be use ul in this patient. It is a diuretic that acts on the
mineralocorticoid receptor to blocks the synthesis o aldosterone-induced proteins
(AIPs) in epithelial cells in the late distal tubule and collecting duct. It has been
(Continued)
280
Drug Therapy of Hypertension and Na + Balance Disorders CHAPTER 1 5

shown to reduce morbidity and mortality in patients when added to other agents in
patients with moderate to severe heart ailure. However, because it is a K+-sparing
diuretic, it could cause hyperkalemia when used in combination with valsartan.
c. A er several days in the hospital, the patient still has signif cant edema, but
he has become hyponatremic. What agent might be use ul in causing diuresis
without loss o Na+?
A vasopressin receptor antagonist such as tolvaptan or conivaptan might be used
to induce diuresis without signi cant loss o Na+. T ese agents are re erred to as
aquaretics because they can increase renal- ree water excretion without corresponding
loss o Na+ as with diuretics such as urosemide.
olvaptan is a selective oral V2R antagonist that is approved or clinically sig-
ni cant hypervolemic and euvolemic hyponatremia. T e drug is labeled with a
black box warning against too rapid correction o hyponatremia (can have serious
and atal consequences) and the recommendation to initiate therapy in a hospital
setting capable o close monitoring o serum Na+. olvaptan is contraindicated in
patients receiving drugs that inhibit CYP3A4.
Conivaptan is a nonselective V1aR/V2R antagonist that is FDA approved or the
treatment o hospitalized patients with euvolemic hyponatremia and hypervol-
emic hyponatremia. T e drug is available only or intravenous in usion. In CHF
patients, conivaptan increases renal ree water excretion without a change in
systemic vascular resistance.

KEY CONCEPTS
Nonpharmacological approaches to treat moderately elevated blood pressure
may be suf cient in many patients and can augment the e ects o antihyperten-
sive drugs
Choice o antihypertensive drugs or individual patients may be complex and
should be driven by the likely bene t to patient, taking into consideration the
patients concomitant diseases, adverse e ects o speci c drugs, and cost
Diuretics are pre erred as initial therapy or most patients with uncomplicated
stage 1 hypertension who are unresponsive to nonpharmacological approaches,
although patients are also commonly treated with other drugs: receptor
antagonists, ACE inhibitors/A 1 receptor antagonists, and Ca2+ channel
blockers
Patients with congestive heart ailure should ideally be treated with a diuretic,
receptor antagonist, ACE inhibitor/A 1 receptor antagonist, and (in selected
patients) spironolactone
ACE inhibitors or A 1 receptor antagonists should be rst-line drugs in the
treatment o diabetics with hypertension
o achieve stringent control o hypertension, many patients require 2, 3, or 4
appropriately selected drugs used at optimal doses

SUMMARY QUIZ

QUESTION 15-1 A 46-year-old man with uncomplicated stage 1 hypertension is placed


on lisinopril and hydrochlorothiazide A er several weeks, the patient develops a dry
cough What is the underlying cause o the cough?
a Bradykinins that accumulate because o the e ects o lowering blood pressure
b Bradykinins that accumulate because o the e ects o lisinopril
c T e diuresis caused by hydrochlorothiazide dries out mucus membranes in the upper
respiratory tract
d T e dry cough is unrelated to the pharmacological e ects o either drug
281
SECTION III Modulation o Cardiovascular Function

QUESTION 15-2 A 55-year-old woman who begins taking amlodipine to lower her
blood pressure develops edema in her ankles a er several weeks o taking the drug
What is causing the edema?
a T e reduced blood pressure is causing reduced water excretion by the kidneys
b Amlodipine has direct e ects on the kidney to reduce water excretion
c Amlodipine increases capillary hydrostatic pressure by dilating precapillary arteri-
oles without dilating postcapillary vessels
d T e edema is unrelated to the pharmacological e ects o the drug

QUESTION 15-3 An elderly man with stage 2 hypertension is taking valsartan and
hydrochlorothiazide He also takes ibupro en or his arthritis He monitors his blood
pressure daily, and tells you that his blood pressure is o en greater than 140/90 What
is likely causing these high blood pressure readings?
a T e valsartan-hydrochlorothiazide combination is known to be ine ective
b T e ibupro en is reducing the e ectiveness o the antihypertensives
c He is probably not using the blood pressure cu correctly
d He is probably not taking his antihypertensive drugs as prescribed

QUESTION 15-4 A patient with long-standing hypertension is taking aliskiren, amlo-


dipine, and triamterene What adverse e ects might occur with this drug combination?
a T ere is a risk o hyperkalemia with this drug combination
b T ere is a risk o hypokalemia with this drug combination
c T ere is a risk o hypernatremia with this drug combination
d T ere is a risk o hyponatremia with this drug combination

QUESTION 15-5 A 71-year-old man with chronic renal ailure with edema has been
titrated up to the maximal single daily dose o bumetanide, but he still has signi cant
edema What option should initially be considered to reduce the edema?
a Increase the single dose o bumetanide above the ceiling dose
b Increase the requency o dosing o bumetanide
c Add a K+-sparing diuretic
d Add a thiazide diuretic
e Begin IV in usion o bumetanide

SUMMARY QUIZ ANSWER KEY


QUESTION 15-1 Answer is b. Between 5 and 20% o patients taking ACE inhibitors
develop dry cough which some patients cannot tolerate T e cough is caused by the
accumulation o bradykinin and substance P that is a direct result o inhibiting ACE,
which is the enzyme that degrades these endogenous peptides For patients who cannot
tolerate the cough, ARBs are a good option because there is a much lower incidence o
this adverse e ect
QUESTION 15-2 Answer is c. T e calcium channel blockers are selective or inhibiting
arteriolar smooth muscle compared to venous smooth muscle T is can result in an
increase in capillary hydrostatic pressure which orces uid into the interstitial space
o alleviate the edema, calcium channel blockers are o en administered with a thiazide
diuretic to prevent the edema T is combination also has additive antihypertensive
e ects since thiazides lower blood pressure through a di erent mechanism

282
Drug Therapy of Hypertension and Na + Balance Disorders CHAPTER 1 5

QUESTION 15-3 Answer is b. It is most likely that ibupro en is opposing the actions
o the antihypertensive drugs NSAIDs are known to reduce the e ectiveness o many
antihypertensive drugs, especially those that inhibit the renin-angiotensin system (see
Figure 15-2) It is also possible that the patient is not adherent to his antihypertensive
medications, or his salt intake is excessive Another possibility is that other drugs,
including over-the-counter drugs or herbal preparations, are the cause For example,
NSAIDs, sympathomimetic decongestants, cyclosporine, erythropoietin, ephedra (ma
huang), and licorice are known to reduce the e ectiveness o many antihypertensive
drugs Be ore changing medications or altering dosing it is important to identi y and
eliminate, i possible, causes o the resistant hypertension
QUESTION 15-4 Answer is a. Both aliskiren, a direct renin inhibitor, and triamterene,
a K+-sparing diuretic, can increase plasma K+ concentrations When used in combina-
tion they can be additive in their e ects and can cause hyperkalemia
QUESTION 15-5 Answer is b. Patients with chronic renal ailure and other disease
states o en require aggressive diuretic therapies to control edema Figure 15-6
illustrates Braters algorithm or diuretic therapy in various disease states including
chronic renal ailure According to the algorithm, the next step a er titrating the single
daily dose o a loop diuretic up to the ceiling dose is to increase the requency o
dosing In the case o bumetanide, the algorithm recommends increasing the requency
o the ceiling dose to 4X daily Nothing is gained by increasing a single dose above
the ceiling dose (the dose that gives near maximal e ect or a given disease state) A
K+-sparing diuretic can be added to the loop diuretic to maintain K+ homeostasis and
may improve therapeutic response T e next step in the algorithm is to add a thiazide
diuretic which may result in a synergistic interaction T e last step in the algorithm
is to switch to continuous in usion o the loop diuretic while maintaining the
other diuretics

SUMMARYTABLE: DRUGS USED IN THE TREATMENT OF HYPERTENSION, EDEMA, AND DISORDERS OF NA+ AND
WATER BALANCE
TOXICITIES
CLASS AND UNIQUE; CLINICALLY
SUBCLASSES NAMES CLINICAL USES COMMON IMPORTANT
DiureticsInhibitors o Acetazolamide Edema when used in combination Most adverse ef ects, Sul onamide-related
Carbonic Anhydrase with other diuretics contraindications, and toxicities, including bone
Prophylaxis and treatment o drug interactions are marrow suppression, skin
high-altitude sickness secondary to urinary toxicities, renal lesions, and
Familial periodic paralysis alkalinization or metabolic allergic reactions
Correcting metabolic alkalosis, acidosis At high doses, drowsiness
especially diuretic-induced H+ and paresthesias
excretion

Dorzolamide Open-angle glaucoma and other


brinzolamide situations that require lowering
intraocular pressure

DiureticsOsmotic Glycerin Treatment o dialysis Headache, nausea, Can cause pulmonary


Diuretics Isosorbide disequilibrium syndrome by and vomiting caused edema in patients with
Mannitol increasing the osmolality o the by hyponatremia due heart ailure
Urea (currently not extracellular uid to extraction o water Contraindicated in patients
available in the United By increasing plasma osmotic rom intracellular who are anuric due to
States) pressure they are used to reduce compartments severe renal disease
intraocular pressure and reduce
cerebral edema

283
SECTION III Modulation o Cardiovascular Function

TOXICITIES
UNIQUE; CLINICALLY
CLASS AND SUBCLASSES NAMES CLINICAL USES COMMON IMPORTANT

DiureticsInhibitors Furosemide Hypertension (although short t1/2 Overzealous use o loop Ototoxicity (including
o Na+-K+-2Cl Symport Bumetanide limits use) diuretics can cause hearing loss and vertigo) is
(Loop Diuretics, High- Ethacrynic acid Chronic congestive heart ailure serious depletion o usually reversible
Ceiling Diuretics) Torsemide to reduce venous and pulmonary total-body Na+ Irreversible ototoxicity can
congestion I dietary K+ intake is not occur at high doses, with
Acute pulmonary edema su cient, hypokalemia rapid IVadministration,
Edema o nephrotic syndrome, may develop increasing and during concomitant
chronic kidney disease, and liver the risk o arrhythmias therapy with other drugs
cirrhosis known to be ototoxic
Hyperuricemia,
hyperglycemia, and
dyslipidemia
Contraindicated in severe
hyponatremia and volume
depletion, hypersensitivity
to sul onamides (only the
sul onamide-based agents)

DiureticsInhibitors Bendro umethiazide Widely used or the treatment I dietary K+ intake is not Most serious adverse
o Na+-Cl Symport Chlorothiazide o hypertension either alone su cient, hypokalemia ef ects o thiazides are
(Thiazide and Thiazide- Hydrochlorothiazide or in combination with other may develop increasing related to abnormalities
Like Diuretics) Hydro umethiazide antihypertensive drugs (can have the risk o arrhythmias o uid and electrolyte
Methyclothiazide additive or synergistic ef ects Hypokalemia can balance (extracellular
Polythiazide when used in combination with increase risk o atal volume depletion,
Trichlormethiazide other classes) arrhythmias when used hypotension, hypokalemia,
Chlorthalidone Best initial therapy or in combination with hyponatremia,
Indapamide uncomplicated hypertension quinidine (see Chapter 18) hypochloremia, metabolic
Metolazone Edema associated with heart alkalosis, hypomagnesemia,
Quinethazone ailure, hepatic cirrhosis, and hypercalcemia, and
renal disease (except when GFR hyperuricemia)
is <30-40 mL/min) Thiazide diuretics have
Ca2+ nephrolithiasis caused atal or near- atal
Treatment o nephrogenic hyponatremia
diabetes insipidus by increasing Impaired glucose
proximal tubular water tolerance, dyslipidemias
reabsorption (secondary to Contraindicated in
volume contraction) and by patients with sul onamide
blocking the ability o the hypersensitivity
distal convoluted tubule to
orm dilute urine

DiureticsInhibitors Triamterene and xed Used in combination with thiazide Nausea, vomiting, Li e-threatening
o Renal Epithelial Na+ dose combination and loop diuretics to augment diarrhea, and headache hyperkalemia, thus
Channels (K+-sparing triamterene/ diuresis and antihypertensive contraindicated in patients
Diuretics) hydrochloro-thiazide ef ects and of set the kaliuretic with hyperkalemia or those
ef ects o the thiazide and loop at risk o hyperkalemia
diuretics

Amiloride and xed Used in combination with thiazide Nausea, vomiting, leg Li e-threatening
dose combination and loop diuretics to augment cramps, and dizziness hyperkalemia, thus
amiloride/ diuresis and antihypertensive contraindicated in patients
hydrochlorothiazide ef ects, and of set the kaliuretic with hyperkalemia or those
ef ects o the thiazide and loop at risk o hyperkalemia
diuretics

284
Drug Therapy of Hypertension and Na + Balance Disorders CHAPTER 1 5

TOXICITIES
UNIQUE; CLINICALLY
CLASS AND SUBCLASSES NAMES CLINICAL USES COMMON IMPORTANT

DiureticsMineralo- Spironolactone Similar to other K+-sparing Can bind to other steroid Li e-threatening
corticoid Receptor Eplerenone diuretics, spironolactone is o ten receptors and may hyperkalemia, thus
Antagonists (Aldosterone co-administered with thiazide or cause gynecomastia, contraindicated in patients
Antagonists, K+-Sparing loop diuretics to treat edema and impotence, decreased with hyperkalemia or those
Diuretics) hypertension libido, hirsutism, and at risk o hyperkalemia
Spironolactone is the diuretic o menstrual irregularities Contraindicated in patients
choice in patients with hepatic Can induce diarrhea and with peptic ulcers
cirrhosis other gastric disturbances
When added to standard therapy, Can cause CNS ef ects
morbidity and mortality are such drowsiness, ataxia,
reduced in patients with heart and headache
ailure (see Chapter 17)

DiureticsAtrial Human recombinant Use is limited to patients


Natriuretic Peptides ANP (carperitide, with acutely decompensated
(Inhibitors o the Cyclic available only in congestive heart ailure who are
Nucleotide-Gated Japan) short o breath at rest to improve
Nonspeci c Cation Human recombinant hemodynamics (should not be
Channel) BNP (nesiritide) used in place o diuretics)

Angiotensin Converting Captopril Treatment o hypertension as Hypotension, especially May cause hyperkalemia in
Enzyme Inhibitors (ACEIs) Enalapril monotherapy or in combination in patients with elevated patients taking K+-sparing
Enalaprilat with a Ca2+ channel blocker, plasma renin activity diuretics, K+ supplements,
Lisinopril adrenergic blocker, or diuretic Dry cough (5-20% receptor blockers, or
Benzapril Prevention or delay o heart o patients) due to NSAIDs
Fosinopril ailure in patients with systolic accumulation o Acute renal ailure in
Trandolapril dys unction bradykinin, substance P patients with reduced renal
Quinapril Prevention o cardiovascular and/or prostaglandins in per usion
Ramipril events in patients with acute the lungs Patients who are pregnant
Moexipril myocardial in arction and in should discontinue
Perindopril patients who are high risk o ACEIs due to teratogenic
cardiovascular events potential
Prevention or delay o renal Angioedema (0.1-0.5% o
disease in patients with type 1 patients)
diabetes mellitus

Angiotensin Receptor Candesartan cilexetil Treatment o hypertension as Hypotension in patients Hyperkalemia in patients
Blockers (ARBs) Irbesartan monotherapy or in combination whose blood pressure is with renal disease or those
Losartan with a Ca2+ channel blocker, highly dependent on the taking K+-sparing diuretics
Olmesartan medoxomil adrenergic blocker, or diuretic RAS or with combination or K+ supplements
Telmisartan Irbesartan and losartan therapy with other Acute renal ailure in
Valsartan are approved or diabetic antihypertensive drugs patients with reduced renal
nephropathy Incidence o cough is per usion
Losartan is approved or stroke lower than ACEIs Incidence o angioedema is
prophylaxis lower than ACEIs
Valsartan is approved or Patients who are pregnant
heart ailure and to reduce should discontinue
cardiovascular events in patients ARBs due to teratogenic
with le t ventricular dys unction potential
ollowing myocardial in arction

285
SECTION III Modulation o Cardiovascular Function

TOXICITIES
UNIQUE; CLINICALLY
CLASS AND SUBCLASSES NAMES CLINICAL USES COMMON IMPORTANT
Direct Renin Aliskiren Treatment o hypertension as Mild GI symptoms at Not recommended in
Inhibitors (DRIs) monotherapy or in combination high doses pregnant patients
with other antihypertensive Incidence o cough and
agents, particularly with ACE angioedema less than
inhibitors or ARBs (which increase with ACE inhibitors
plasma renin activity; see Table 15-2)
Cardioprotective and
renoprotective in combination
therapy
Recommended in patients
intolerant to other antihypertensive
drugs or or urther blood pressure
control
Adrenergic Receptor Many (see Table 15-2 Ef ective therapy or all grades o Should be avoided in Risk o hypoglycemic
Antagonists and Chapter 7) hypertension as monotherapy or patients with reactive reactions may be increased
in combination with other agents airway disease (asthma) in diabetics taking insulin
Highly pre erred or hypertensive or with SA or AVnodal Sudden discontinuation
patients with conditions such dys unction or in can cause rebound
as MI, ischemic heart disease, or combination with other hypertension; dosage
congestive heart ailure drugs that inhibit AV should be tapered
conduction, such as gradually over 10-14 days
verapamil prior to discontinuation
1 Adrenergic Receptor Prazosin Not recommended as First-dose phenomenon Patients with pheo-
Antagonists Terazosin monotherapy or hypertensive (in up to 50% o patients) chromocytoma can have a
Doxazosin patients in which symptomatic vasoconstrictor response to
Used primarily in conjunction with orthostatic hypotension epinephrine resulting rom
diuretics, blockers, and other occurs within 30-90 activation o unblocked
antihypertensive agents minutes (or longer) o the vascular 2 adrenergic
Use ul or hypertensive patients initial dose o the drug or receptors
with benign prostatic a ter a dosage increase
hyperplasia because they also
improve urinary symptoms
Centrally Acting Methyldopa Current use largely limited to Transient sedation May precipitate severe
Adrenergic treatment o hypertension in Diminution in psychic bradycardia and sinus arrest
AgentsMethyldopa pregnancy, where it has a record energy may persist, Hepatotoxicity, sometimes
or sa ety depression occurs associated with ever
occasionally >20% o patients who
Dryness o the mouth receive methyldopa or
Diminished libido a year develop a positive
Parkinsonian signs Coombs test and 1-5% o
Hyper-prolactinemia that these patients will develop
may cause gynecomastia a hemolytic anemia
and galactorrhea that requires prompt
discontinuation o the drug
Centrally Acting Clonidine No xed place or these drugs in Sedation and xerostomia Symptomatic bradycardia
Adrenergic Agents2 Guanabenz the treatment o hypertension are prominent and sinus arrest in patients
Adrenergic Receptor Guan acine because o CNS ef ects and Postural hypotension and with dys unction o the
Antagonists lack o evidence demonstrating erectile dys unction may SA node and AVblock
reduction in risk o adverse be prominent in patients with AV
cardiovascular events nodal disease; sudden
discontinuation may cause
a withdrawal syndrome
consisting o headache,
apprehension, tremors,
abdominal pain, sweating,
tachycardia, rebound
hypertension

286
Drug Therapy of Hypertension and Na + Balance Disorders CHAPTER 1 5

TOXICITIES
UNIQUE; CLINICALLY
CLASS AND SUBCLASSES NAMES CLINICAL USES COMMON IMPORTANT

Adrenergic Neuron Guanadrel (not Rarely used because o Undesirable ef ects that Occasional psychotic
Blocking Agents marketed in the adverse ef ects are related to sympathetic depression that can lead
United States) blockade, including to suicide with reserpine
Reserpine hypotension during Reserpine must be
standing and exercise, discontinued at rst
general eeling o atigue sign o depression;
and lassitude reserpine-induced
CNS ef ects with reserpine depression may last
include sedation and several months a ter drug
inability to per orm is discontinued
complex tasks

Ltype Ca2+ Nisoldipine Used alone or in combination Re ex tachycardia


Channel Blockers Felodipine with other antihypertensive drugs
Dihydropyridines Nicardipine May be more e cacious than
Isradipine other agents as monotherapy in
Amlodipine elderly subjects and in A rican
Clevidipine Americans, population groups in
Ni edipine which the low renin status is more
prevalent
May be a pre erred treatment
in patients with isolated systolic
hypertension
Parenteral administration o the
short-acting clevidipine may
be use ul in treating severe or
perioperative hypertension

Ltype Ca2+ Verapamil Used alone or in combination Direct negative May worsen symptoms in
Channel Blockers Diltiazem with other antihypertensive drugs chrontropic and patients with heart ailure
Nondihydropyridines May be more e ciacious than inotropic ef ects due to direct negative
other agents as monotherapy in Concurrent use o a inotropic ef ects
elderly subjects and in A rican receptor antagonist
Americans, population groups in may magni y
which the low renin status is more negative chronotropic
prevalent ef ects o these drugs
May be a pre erred treatment or cause heart block in
in patients with isolated systolic susceptible patients
hypertension

Arterial Vasodilators Hydralazine Hydralazine no longer a rst-line Hydralazine causes Hydralazine can cause
Minoxidil drug; may have utility in the symptoms o severe autoimmune reactions,
Diazoxide treatment o severe hypertension, vasodilation, including including drug-induced
Fenoldopam in some patients with CHF, and headache, nausea, lupus syndrome, serum
in hypertensive emergencies ushing, hypotension, sickness, hemolytic
in pregnant women (especially palpitations, tachycardia, anemia, vasculitis, and
preeclampsia) dizziness, and angina rapidly progressive
Systemic minoxidil best reserved pectoris glomerulonephritis
or severe hypertension that Minoxidil has strong
responds poorly to other antinatriuretic ef ects,
antihypertensive medications, causes re ex increases
especially in male patients with in sympathetic cardiac
renal insu ciency ef ects, and hypertrichosis

Arterial and Venous Nitroprusside Used primarily to treat Excessive vasodilation At high doses, conversion
Vasodilators hypertensive emergencies, also to cyanide can be
in situations when short-term toxic but prevented by
reduction o cardiac preload coadministration o sodium
and/or a terload is desired thiosul ate

287
SECTION III Modulation o Cardiovascular Function

TOXICITIES
UNIQUE; CLINICALLY
CLASS AND SUBCLASSES NAMES CLINICAL USES COMMON IMPORTANT

Vasopressin AgonistV1 Vasopressin Used to induce GI smooth muscle V1-mediated ef ects Vasoconstriction o
Receptor Agonists Terlipressin contraction to treat postoperative include cutaneous coronary and peripheral
ileus, abdominal distension, vasoconstriction, nausea, arteries
and dispel intestinal gas prior to belching, cramps, and an Vasopressin should be
abdominal x-ray imaging urge to de ecate administered only at low
Used to vasoconstrict splachnic Major V2 receptor- doses and with extreme
and portal arteries to reduce mediated adverse ef ect caution in individuals
bleeding associated with o vasopressin is water suf ering rom vascular
various abdominal procedures, intoxication; must not disease, especially coronary
hemorrhagic disorders such be administered to artery disease
as esophogeal varices, and patients with primary or Arrhythmia and decreased
surgeries in patients with portal psychogenic polydipsia cardiac output
hypertension Peripheral vasoconstriction
Terlipressin is pre erred or and gangrene with large
bleeding esophageal varices doses o vasopressin
because o increased sa ety, and is
ef ective in treating patients with
hepatorenal syndrome

Vasopressin AgonistV2 Desmopressin Drug o choice or treatment Mild acial ushing and May cause transient
Receptor Agonist (DDAVP) o central diabetes insipidus to headache thrombo-cytopenia in
control polyuria and polydypsia Major adverse ef ect individuals with type IIb
Reduces bleeding time by is water intoxication; vWD and is contraindicated
increasing release o vWF and patients receiving in such patients
actor VIII in patients with certain desmopressin to maintain
bleeding disorders hemostasis should be
Primary nocturnal enuresis advised to reduce uid
Relieves post-lumbar puncture intake
headache Must not be administered
to patients with primary
or psychogenic polydipsia
Tachyphylaxis to
desmopressins
procoagulant ef ects
usually occurs a ter several
days owing to depletion
o actor VIII and vWF

Vasopressin Antagonists Tolvaptan Used in disease states with water GI ef ects, hyperglycemia, Metabolized by CYP3A4
Selective V2 Receptor (V2R) excess and hyponatremia (chronic and pyrexia (drug-drug interactions)
Antagonists (Aquaretics) heart ailure, cirrhosis, nephrosis) Less common adverse
to increase renal ree water ef ects: li e-threatening
without changing electrolyte thrombosis, ventricular
excretion brillation, urethral and
Used in patients with euvolemic vaginal hemorrhage,
or hypervolemic hyponatremia respiratory ailure, diabetic
associated with SIADH ketoacidosis, ischemic colitis,
increase in prothrombin
time, rhabdomyolysis

Vasopressin Antagonists Conivaptan Treatment o hospitalized patients In usion site reaction Metabolized by CYP3A4
Nonselective V1aR/V2R with euvolemic hyponatremia and (drug should only (drug-drug interactions)
Antagonist (Aquaretics) hypervolemic hyponatremia be in used into large
veins and in usion sites
changed daily)
Headache, hypertension,
hypotension,
hypokalemia, and pyrexia

288
CHAPTER

Drug Therapyof Myocardial


Ischemia 16
T is chapter will be most use ul a er having a basic understanding o the material
DRUGS INCLUDED
related to angina at the beginning o Chapter 27, reatment o Myocardial Ischemia
and Hypertension in Goodman & Gilmans T e Pharmacological Basis of T erapeutics, IN THIS CHAPTER
12th Edition. In addition to the material presented here, the 12th Edition contains: Antithrombin agents (see Chapter 19)
able 27-1 Organic Nitrates Available or Clinical Use, which provides in ormation Antiplatelet drugs (see Chapter 19)
about the preparations, usual doses, and routes o administration o organic nitrates Adrenergicreceptor antagonists
T e molecular structures o drugs used to treat myocardial ischemia (see Chapter 7)
Numerous dihydropyridines (ni edipine
LEARNING OBJECTIVES [ADALAT,PROCARDIA, others], amlodipine
Understand the mechanisms o action o drugs used to treat myocardial isch- [NORVASC, others], elodipine [PLENDIL,
emia and relieve symptoms o angina. others], isradipine [DYNACIRC, others],
Understand the goals o treatment based on a knowledge o the underlying nicardipine [CARDENE, others], nisoldipine,
cause(s) o angina. nimodipine, clevidipine, others)
Diltiazem(CARDIZEM, DILACOR-XR, others)
Know which drugs are most e ective in treating di erent types o angina.
Isosorbide dinitrate (ISDN; ISORDIL, SORBI-
Know which drugs are used in combination to treat angina.
TRATE, DILATRATE-SR, others)
Know the untoward e ects and contraindications o the major classes o drugs Isosorbide-5-mononitrate (ISMN; IMDUR,
used to treat angina. ISMO, others)
Know the role o drug-eluting stents to treat patients with acute coronary Nitroglycerin (glyceryl trinitrate; NITRO-BID,
syndromes. NITROSTAT,NITROL, NITRO-DUR, others)
Paclitaxel drug-eluting stents
MECHANISMS OF ACTION OF DRUGS USED TO TREAT MYOCARDIAL ISCHEMIA
Ranolazine (RANEXA)
DRUG CLASS DRUG MECHANISM OF ACTION Sildena l (VIAGRA, REVATIO)
Organic Nitrates Nitroglycerin (glyceryl Prodrugs that are the source o Sirolimus drug-eluting stents
(Nitrovasodilators, trinitrate) NO that increase intracellular
NO Donors) Isosorbide dinitrate (ISDN) cGMP and cause relaxation o Tadala l (CIALIS, ADCIRCA)
Isosorbide-5-mononitrate large veins (reduced preload Thrombolyticagents (see Chapter 19)
(ISMN) reduces myocardial O2 demand),
Vardena l (LEVITRA)
relaxation o arteriolar resistance
vessels (reduced a terload reduces Verapamil (CALAN, ISOPTIN, others)
myocardial O2 demand), and
relaxation o coronary vessels
(improved myocardial O2 supply) SYMPTOMS OF
MYOCARDIAL ISCHEMIA
Adrenergic Receptor see Chapter 7 Block myocardial adrenergic
Antagonists receptors to slow heart rate (improved Angina pectoris, the primarysymptomo
myocardial O2 supply and reduced ischemicheart disease, is caused bytran-
O2 demand) and reduce ventricular sient episodes o myocardial ischemia that
contractility (reduced O2 demand) are due to an imbalance in the myocardial
oxygen supplydemand relationship (see
(Continued) Figure 16-1).
Angina pectoris is a heavy, pressing
substernal discom ort (rarelydescribed
aspain) o ten radiating to the
Le t shoulder
Flexor aspect o the le t arm
Jaw
Epigastrium
(continued)

289
SECTION III Modulation of Cardiovascular Function

SYMPTOMS OF DRUG CLASS DRUG MECHANISM OF ACTION


MYOCARDIAL ISCHEMIA Ca2+ Channel Verapamil Blocking voltage-sensitive L-type
(Cont.) Antagonists Diltiazem Ca2+ channels relaxes coronary
(Ca2+ Entry Blockers) Numerous dihydropyridines arteries (improved myocardial O2
Asigni cant number o patients note (ni edipine, amlodipine, supply) and relaxes peripheral
discom ort in a di erent location or o a elodipine, isradipine, arteriole resistance vessels (reduced
di erent character: nicardipine, nisoldipine, a terload reduces myocardial O2
nimodipine, clevidipine, demand)
Women, the elderly, and diabetics are
others)
more likelyto experience myocardial
ischemia with atypical symptoms. Antiplatelet Drugs see chapter 19 Inhibiting platelet aggregation
Some patients experience myocardial at site o ruptured coronary
atherosclerotic plaque prevents
ischemia with no symptoms occlusion o coronary blood ow
o angina (silent ischemia),
but with electrocardiographic, Statins see chapter 20 Correct dyslipidemias that lead
echocardiographic, or radionuclide to coronary atherosclerosis and
evidence o ischemia. stabilize coronary plaques

Na+ Channel Blocker Ranolazine Blocks late inward Na+ current in


(Late Inward Na+ cardiac myocytes that leads to
Current Blocker) increased ventricular diastolic wall
tension (reduced O2 demand)

CLASSIFICATION OF MYOCARDIAL ISCHEMIA


TYPE OF MYOCARDIAL
ISCHEMIA ETIOLOGY THERAPIES
Typical angina (stable angina, Decreased coronary vessel Therapy is directed at reducing myocardial O2 demand (nitrates and
exertional angina, exercise- radius due to coronary adrenergic antagonists; see Figure 16-1)
induced angina) atherosclerosis results in
ischemia when combined
with exertion

Unstable angina and non-ST- Acute or subacute worsening Therapies directed at decreasing myocardial oxygen demand (nitrates and
segment-elevation myocardial o anginal symptoms caused adrenergic antagonists) have limited e cacy
in arction (acute coronary by decreased coronary ow Additional therapies are directed at the atherosclerotic plaque and
syndrome) due to thrombosis at the site consequences or prevention o its rupture including:
o ruptured coronary plaque antiplatelet agents
with partial or total occlusion antithrombin agents
o the vessel thrombolytic agents
angioplasty and mechanotherapy with intracoronary stents (see Side Bar
DRUG-ELUTING INTRACORONARYSTENTS)
coronary bypass surgery in selected patients
Reduce coronary vasospasm with IVnitroglycerin, and possibly a Ca2+ channel
blocker to increase O2 supply (see Figure 16-1)

Acute myocardial in arction Ischemia with loss o Therapy is directed at reducing the size o the in arct, preserving viable tissue
myocardial tissue caused by reducing O2 demand o the myocardium, and preventing ventricular
by blockage o one or more remodeling
coronary vessels Nitroglycerin to relieve ischemic pain but should not be used i hypotension
limits the administration o blockers
Reper usion therapies are critical with angioplasty and stents having better
outcomes than thrombolytic therapy

Variant or Prinzmetal angina Intermittent decrease in ow Increase myocardial O2 supply with coronary vasodilators (see Figure 16-1);
due to localized coronary Ca2+ channel blockers reduce mortality and the incidence o MI
vasospasm Long-acting nitrates can occasionally be e ective alone but additional Ca2+
channel blocker therapy is usually required

290
Drug Therapy o Myocardial Ischemia CHAPTER 1 6

Ag e nts de c re as ing O2 de mand Ag e nts inc re as ing O2 s upply


BALANCE
He a rt ra te
Adre ne rgic a nta gonis ts Corona ry Va s odila tors
blood flow
S ome Ca 2+ e ntry blocke rs Contra ctility (e s p. Ca 2+ e ntry blocke rs )
= O S upply
O2 De mand > 2
Pre loa d Re giona l
Orga nic nitra te s myoca rdia l Als o s ta tins,
Ca 2+ e ntry blocke rs Afte rloa d blood flow a nti-thrombotics
IS CHEMIA

FIGURE 16-1 Pharmacological modif cation o the major determinants o myocardial O2 supply. When myocardial O2 requirements exceed O2
supply, an ischemic episode results. This f gure shows the primary hemodynamic sites o actions o pharmacological agents that can reduce O2
demand (le t side) or enhance O2 supply (right side). Some classes o agents have multiple e ects. Stents, angioplasty, and coronary bypass
surgery are mechanical interventions that increase O2 supply. Both pharmacotherapy and mechanotherapy attempt to restore a dynamic
balance between O2 demand and O2 supply.
INTERACTIONS OF
NITRATES WITH
CASE 16-1 PHOSPHODIESTERASE 5
A 52-year-old man who su ers rom angina when he climbs stairs or participates in simi- (PDE5) INHIBITORS
lar activities receives a prescription or nitroglycerin (glyceryl trinitrate). He is instructed Erectile dys unction is common in
to take a tablet 1 or 2 minutes be ore he expects to climb stairs to prevent the angina. patients with coronary artery dis-
a. What is the mechanism o action o nitroglycerin that prevents angina rom ease; thus many men desiring ther-
developing in this patient? apy or erectile dys unction may be
Nitroglycerin and the other organic nitrates used to treat angina are prodrugs receiving antianginal therapy.
that are sources o nitric oxide (NO). NO activates the soluble iso orm o guanylyl T e PDE5 inhibitors (sildena l,
cyclase, thereby increasing intracellular levels o cyclic GMP and leading to the tadala l, and vardena l) block the
relaxation o smooth muscle cells in a broad range o tissues. T e NO-dependent cyclic GMP-speci c PDE5 am-
relaxation o vascular smooth muscle leads to vasodilation. Low concentrations ily o phosphodiesterases which
o nitroglycerin pre erentially dilate veins more than arterioles. T is venodilation are widely expressed in vascular
decreases venous return, leading to a all in le and right ventricular chamber size smooth muscle, including the
and end-diastolic pressures, but usually results in little change in systemic vascular smooth muscle o the corpus cav-
resistance. Systemic arterial pressure may all slightly, and heart rate is unchanged or ernosum and penile arteries.
may increase slightly in response to a decrease in blood pressure. Nitroglycerin can T e accumulation o cyclic GMP
dilate epicardial stenoses and reduce the resistance to ow through such areas result- in vascular smooth muscle that
ing in an increase in blood ow that would be distributed pre erentially to ischemic occurs with NO stimulation o
myocardial regions as a consequence o vasodilation induced by autoregulation. guanylyl cyclase can be enhanced
By their e ects on the systemic circulation, the organic nitrates also can reduce by inhibiting PDE5 activity.
myocardial O2 demand (see Figure 16-1). T e major determinants o myocardial Given alone, the PDE5 inhibi-
O2 consumption include le ventricular wall tension, heart rate, and myocardial tors cause modest drops in blood
contractility. Ventricular wall tension is a ected by a number o actors that may be pressure, but in men who are
considered under the categories o preload and a erload. Organic nitrates decrease taking organic nitrate vasodila-
both preload and a erload as a result o respective dilation o venous capacitance tors, a all in blood pressure o
and arteriolar resistance vessels. T e ability o nitrates to dilate epicardial coronary more than 25 mm Hg can occur.
arteries, even in areas o atherosclerotic stenosis, is modest, and the preponderance Because o the possibility o
o evidence continues to avor a reduction in myocardial work, and thus in myocar- extreme hypotension, PDE5
dial O2 demand, as their primary e ect in chronic stable angina. inhibitors are contraindicated
b. How and when should the nitroglycerin be administered to prevent anginal pain? or patients receiving any orm
Anginal pain may be prevented when the drug is used prophylactically immedi- o nitrate and should not be
ately prior to exercise or stress. T e smallest e ective dose should be prescribed. prescribed to such patients.
Sublingual organic nitrates should be taken at the time o an anginal attack or in Patients should be questioned
anticipation o exercise or stress. Such intermittent treatment provides reproduc- about the use o PDE5 inhibitors
ible cardiovascular e ects. Peak concentrations o nitroglycerin are ound in plasma within the past 24 hours be ore
within 4 minutes o sublingual administration; the drug has a t1/2 o 1 to 3 minutes nitrates are administered; a
because o rapid and complete rst-pass metabolism by the liver. T e onset o action period longer than 24 hours may
o nitroglycerin may be even more rapid i it is delivered as a sublingual spray rather be needed to avoid drug interac-
than as a sublingual tablet. tions, especially with tadala l
which has the longest hal -li e.
(Continued)
291
SECTION III Modulation of Cardiovascular Function

Frequently repeated or continuous exposure to high doses o organic nitrates leads


to a marked attenuation in the magnitude o most o their pharmacological e ects.
T e magnitude o tolerance is a unction o dosage and requency o use. In patients
who require constant administration o an organic nitrate to prevent symptoms o
angina, tolerance can develop rapidly. An e ective approach to preventing tolerance
or restoring responsiveness is to interrupt therapy or 8 to 12 hours each day, which
allows the return o ef cacy. It is usually most convenient to omit dosing at night in
patients with exertional angina either by adjusting dosing intervals o oral or buccal
preparations or by removing cutaneous nitroglycerin.
Patients should be instructed to seek medical attention immediately i 3 tablets
taken over a 15-minute period do not relieve a sustained attack because this situa-
tion may be indicative o myocardial in arction (MI), unstable angina, or another
cause o the pain. Patients also should be advised that there is no virtue in trying to
avoid taking sublingual nitroglycerin or anginal pain.
c. What are the expected adverse e ects o nitroglycerin therapy?
Untoward responses to the therapeutic use o organic nitrates are almost all secondary
to actions on the cardiovascular system. Headache is common and can be severe. It
usually decreases over a ew days i treatment is continued and o en can be controlled
by decreasing the dose. ransient episodes o dizziness, weakness, and other mani es-
tations associated with postural hypotension may develop, particularly i the patient is
standing immobile, and may progress occasionally to loss o consciousness, a reaction
that appears to be accentuated by alcohol. It also may be seen with very low doses o
nitrates in patients with autonomic dys unction. Even in severe nitrate syncope, posi-
tioning and other measures that acilitate venous return are the only therapeutic mea-
sures required. All the organic nitrates occasionally can produce drug rash.
d. What drug combinations should be avoided?
Erectile dys unction is a requently encountered problem whose risk actors parallel
those o coronary artery disease. T us many men desiring therapy or erectile dys-
unction already may be receiving (or may require, especially i they increase physi-
cal activity) antianginal therapy. T e combination o a phosphodiesterase 5 (PDE5)
inhibitor (sildena l, tadala l, vardena l) used or erectile dys unction with organic
nitrate vasodilators can cause extreme hypotension (see Side Bar IN ERAC IONS
OF NI RA ES WI H PHOSPHODIES ERASE 5 (PDE5) INHIBI ORS). PDE5
inhibitors should not be prescribed to patients receiving any orm o nitrate and
patients should be questioned about the use o PDE5 inhibitors within 24 hours
be ore nitrates are administered. A period o longer than 24 hours may be needed
ollowing administration o a PDE5 inhibitor or sa e use o nitrates, especially with
tadala l because o its prolonged t1/2. In the event that patients develop signi cant
hypotension ollowing combined administration o a PDE5 inhibitor and a nitrate,
uids and adrenergic receptor agonists, i needed, should be used or support.

CASE 16-2
A 67-year-old woman with mild heart ailure (LVEF = 45%) has anginal pain with
exercise and is prescribed a adrenergic blocker.
a. How does therapy with a adrenergic blocker prevent anginal pain?
T e e ectiveness o adrenergic receptor antagonists in the treatment o exertional
angina is attributable primarily to a all in myocardial O2 consumption at rest and
during exertion, although there also is some tendency or increased ow toward
ischemic regions. T e decrease in myocardial O2 consumption is due to a negative
chronotropic e ect (particularly during exercise), a negative inotropic e ect, and a
reduction in arterial blood pressure (particularly systolic pressure) during exercise
(see Figure 16-1). T e reduction in arterial blood pressure is secondary to a drop in
cardiac output due to the negative chronotropic and inotropic e ects o blockers.
(Continued)
292
Drug Therapy o Myocardial Ischemia CHAPTER 1 6

b. What is the rationale underlying the choice o a adrenergic blocker or


this patient?
Because there is a proven mortality bene t rom the use o adrenergic receptor
antagonists in patients with heart disease, this class o drugs represents the rst line
o therapy. Adrenergic receptor antagonists are e ective in reducing the severity
and requency o attacks o exertional angina and are recommended or patients
with mild heart ailure (LVEF 40%; able 16-1). Moreover, several adrenergic
receptor antagonists have been shown to reduce mortality in patients with conges-
tive heart ailure and treatment o patients with heart ailure with adrenergic
antagonist drugs has become standard therapy or many such patients (see Chap-
ters 12 and 28 in Goodman & Gilmans T e Pharmacological Basis of T erapeutics,
12th Edition). Most adrenergic receptor antagonists appear to be equally e ective
in the treatment o exertional angina. imolol, metoprolol, atenolol, and proprano-
lol have been shown to exert cardioprotective e ects.
(Continued)

TABLE 16-1 Recommended Drug Therapy or Angina in Patients with Other Medical Conditions
RECOMMENDED TREATMENT AND
CONDITION ALTERNATIVES FOR ANGINA DRUGS TO AVOID
Medical Conditions

Systemic hypertension receptor antagonists


(Ca2+ channel antagonists)

Migraine or vascular headaches receptor antagonists


(Ca2+ channel antagonists)

Asthma or chronic obstructive pulmonary disease Verapamil or diltiazem receptor antagonists


with bronchospasm

Hyperthyroidism receptor antagonists

Raynauds syndrome Long-acting, slow-release Ca2+ antagonists receptor antagonists

Insulin-dependent diabetes mellitus receptor antagonists (particularly


i prior MI) or long-acting, slow-release
Ca2+ channel antagonists

Non-insulin-dependent diabetes mellitus receptor antagonists or long-acting,


slow-release Ca2+ channel antagonists

Depression Long-acting, slow-release Ca2+ receptor antagonists


channel antagonists

Mild peripheral vascular disease receptor antagonists or Ca2+


channel antanogists

Severe peripheral vascular disease with rest ischemia Ca2+ channel antagonists receptor antagonists

Cardiac Arrhythmias and Conduction Abnormalities

Sinus bradycardia Dihydropyridine Ca2+ channel antagonists receptor antagonists, diltiazem, verapamil

Sinus tachycardia (not due to heart ailure) receptor antagonists

Supraventricular tachycardia Verapamil, diltiazem, or receptor


antagonists

Atrioventricular block Dihydropyridine Ca2+ channel antagonists receptor antagonists, diltiazem, verapamil

Rapid atrial f brillation (with digitalis) Verapamil, diltiazem, or receptor antagonists

Ventricular arrhythmias receptor antagonists


(Continued)

293
SECTION III Modulation of Cardiovascular Function

RECOMMENDED TREATMENT AND


CONDITION ALTERNATIVES FOR ANGINA DRUGS TO AVOID

Left Ventricular Dysfunction

Congestive heart ailure

Mild (LVEF 40%) receptor antagonists

Moderate to severe (LVEF <40%) Amlodipine or elodipine (nitrates)

Le t-sided valvular heart disease

Mild aortic stenosis receptor antagonists

Aortic insu ciency Long-acting, slow-release dihydropyridines

Mitral regurgitation Long-acting, slow-release dihydropyridines

Mitral stenosis receptor antagonists

Hypertrophic cardiomyopathy receptor antagonists, nondihydropyridine Nitrates, dihydropyridine Ca2+


Ca2+ channel antagonists channel antagonists

Reproduced with permission rom Gibbons RJ, Chatterjee K, Daley J et al. ACC/AHA/ACP-ASIM Guidelines or the management o patients
with chronic stable angina: A report o the American College o Cardiology / American Heart Association Task Force on Practice Guidelines
(Committee on Management o Patients With Chronic Stable Angina). J Am Coll Cardiol 1999;33:2092-197. Copyright 1999 by the American
College o Cardiology Foundation.

c. I the patients angina is not adequately controlled by monotherapy with a


adrenergic receptor antagonist, what other classes o antianginal drugs could
be added to the adrenergic blocker therapy in this patient?
T e concurrent use o organic nitrates and adrenergic receptor antagonists can
be very e ective in the treatment o typical exertional angina. T e additive ef cacy
primarily is a result o the blockade by one drug o a re ex e ect elicited by the
other. Adrenergic receptor antagonists can block the baroreceptor-mediated
re ex tachycardia and positive inotropic e ects that are sometimes associated
with nitrates, whereas nitrates, by increasing venous capacitance, can attenuate
the increase in le ventricular end-diastolic volume associated with receptor
blockade. Concurrent administration o nitrates also can alleviate the increase in
coronary vascular resistance associated with blockade o adrenergic receptors.
When angina is not controlled adequately by a receptor antagonist plus nitrates,
additional improvement sometimes can be achieved by the addition o a Ca2+ chan-
nel blocker, especially i there is a component o coronary vasospasm. T e di erences
among the chemical classes o Ca2+ channel blockers can lead to important adverse
or salutary drug interactions with receptor antagonists (see text in Chapter 27 in
Goodman & Gilmans T e Pharmacological Basis of T erapeutics, 12th Edition).

CASE 16-3
A man with hypertension and occasional symptoms o angina with exercise is prescribed
amlodipine.
a. What is the mechanism o action o amlodipine that is benef cial in treating
hypertension and angina?
Amlodipine inhibits L-type voltage-sensitive Ca2+ channels in vascular smooth
muscle, especially in arterial beds. T e basis or the use o Ca2+ channel antagonists
in hypertension comes rom the understanding that hypertension generally is the
result o increased peripheral vascular resistance. All o the Ca2+ channel block-
ers lower blood pressure by relaxing arteriolar smooth muscle and decreasing
(Continued)
294
Drug Therapy o Myocardial Ischemia CHAPTER 1 6

peripheral vascular resistance. Ca2+ channel blockers are e ective when used alone
TREATMENT OF
or in combination with other drugs or the treatment o hypertension.
CLAUDICATION AND
T e utility o Ca2+ channel antagonists in the treatment o exertional, or exercise- PERIPHERAL VASCULAR
induced, angina results rom an increase in coronary blood ow owing to coronary
DISEASE
arterial dilation, rom a decrease in myocardial O2 demand (secondary to a decrease
in arterial blood pressure, heart rate, or contractility), or both (see Figure 16-1). Most patients with peripheral artery
Amlodipine, as with other dihydropyridine Ca2+ channel blockers, has potent vaso- disease also have coronaryarterydisease
dilator activity but minimal e ects on cardiac contractility, SA nodal activity, or AV and the therapeuticapproaches overlap.
nodal activity at concentrations that cause vasodilation ( able 16-2). In contrast to Reductions in cardiovascular morbidityand
the dihydropyridines, verapamil and diltiazem have direct negative inotropic and mortalityin patients with peripheral
chronotropic e ects (see able 16-2). arterial disease have been documented
b. What are the potential side e ects o the Ca2+ channel antagonists? with antiplatelet therapy, ACEinhibitors,
and treatment o hyperlipidemia.
T e pro le o adverse reactions to the Ca2+ channel blockers varies among the
drugs in this class. Patients receiving immediate-release short-acting ormulations Risk actor and li estyle modi cations
o ni edipine develop headache, ushing, dizziness, and peripheral edema; these (physical exercise, rehabilitation, and
preparations are not appropriate in the long-term treatment o angina or hyperten- smoking cessation) remain cornerstones o
sion. Dizziness and ushing are much less o a problem with sustained-release or- therapy or patients with claudication.
mulations and with the dihydropyridines having a long t1/2 and relatively constant Pentoxi ylline and cilostazol are drugs used
concentrations o drug in plasma. Peripheral edema may occur in some patients speci cally or lower extremityclaudication.
with Ca2+ channel blockers and most likely results rom increased hydrostatic pres- Pentoxi ylline is a rheological modi er
sure in the lower extremities owing to precapillary dilation and re ex postcapillary that increases the de ormabilityo red
constriction. Some other adverse e ects o these drugs are due to actions in non- blood cells.
vascular smooth muscle such as the lower esophageal sphincter (which can cause Cilostazol is PDE3 inhibitor that inhibits
or aggravate gastroesophageal re ux), constipation (a common side e ect o vera-
platelet aggregation and vasodilation
pamil, but less requent with other Ca2+ channel blockers), and urinary retention
byincreasing intracellular cyclicAMPin
(a rare adverse e ect).
manycells, including platelets.
c. What are the important contraindications or the use o Ca2+ channel blockers?
T e use o intravenous verapamil with an intravenous adrenergic receptor antag-
onist is contraindicated because o the increased propensity or AV block and/or
severe depression o ventricular unction. Patients with ventricular dys unction, SA
or AV nodal conduction disturbances, and systolic blood pressures below 90 mm
Hg should not be treated with verapamil or diltiazem, particularly intravenously
(see able 16-1).

TABLE 16-2 Ca 2+ Channel Blockers: Chemical Structures and Some Relative Cardiovascular Ef ectsa
CHEMICAL STRUCTURE VASODILATION SUPPRESSION OF SUPPRESSION OF SUPPRESSION OF
GENERIC NAME (TRADE NAME) CORONARY FLOW CARDIAC CONTRACTILITY AUTOMATICITY SA NODE CONDUCTION AV NODE
5 1 1 0

5 1 1 0

295
SECTION III Modulation of Cardiovascular Function

CHEMICAL STRUCTURE VASODILATION SUPPRESSION OF SUPPRESSION OF SUPPRESSION OF


GENERIC NAME (TRADE NAME) CORONARY FLOW CARDIAC CONTRACTILITY AUTOMATICITY SA NODE CONDUCTION AV NODE

NR NR NR NR

5 0 1 0

5 1 1 0

3 2 5 4

4 4 5 5

a
Relative e ects are ranked rom no ef ect (0) to prominent (5). NR, not ranked.

CASE 16-4
A 58-year-old man with a long history o exertional angina begins to develop more re-
quent episodes o anginal pain that are also more intense. His cardiologist tells him he
has unstable angina.
a. What is the medical therapy or patients with unstable angina?
T e term unstable angina pectoris has been used to describe a broad spectrum o
clinical entities characterized by an acute or subacute worsening in a patients angi-
nal symptoms. More recently, e orts have been directed toward identi ying patients
with unstable angina on the basis o their risks or subsequent adverse outcomes
such as MI or death. T e term acute coronary syndrome has been use ul in this
context. Common to most clinical presentations o acute coronary syndrome is dis-
ruption o a coronary plaque, leading to local platelet aggregation and thrombosis
at the arterial wall, with subsequent partial or total occlusion o the vessel.
T e principal therapeutic goal in unstable angina is to increase myocardial blood
ow. Medical therapy or unstable angina involves the administration o aspirin,
(Continued)
296
Drug Therapy o Myocardial Ischemia CHAPTER 1 6

which reduces mortality, nitrates, adrenergic receptor blocking agents, and hepa-
DRUG-ELUTING
rin, which are e ective in controlling ischemic episodes and angina. Because vaso-
INTRACORONARY STENTS
spasm occurs in some patients with unstable angina, Ca2+ channel blockers may
o er an additional approach to the treatment o unstable angina. However, there is Intracoronarystents can ameliorate angina
insuf cient evidence to assess whether such treatment decreases mortality except and reduce adverse events in patients with
when the underlying mechanism is vasospasm. In contrast, therapy directed toward acute coronarysyndromes.
reduction o platelet unction and thrombotic episodes clearly decreases morbidity Luminal subacute restenosis caused by
and mortality in patients with unstable angina (see Chapters 34 and 30 in Goodman smooth muscle proli eration within the
& Gilmans T e Pharmacological Basis of T erapeutics, 12th Edition), and adminis- lumen o the stented arteryreduces the
tration o statins to treat dyslipidemias (see Chapter 20) also can reduce the risk o long-termef cacyo stents in a substantial
adverse cardiac events. minorityo patients.
b. What other therapies might be considered to relieve angina symptoms in Drug-eluting stents containing either
this patient? paclitaxel or sirolimus can reduce the rate o
Percutaneous coronary interventions (PCI) such as angioplasty and coronary artery restenosis compared tobare metalstents.
stent deployment, or (less commonly) emergency coronary bypass surgery, can Long-term(6 months) therapywith clopi-
complement pharmacological treatment. In some subsets o patients, percutaneous dogrel and li elong therapywith aspirin
or surgical revascularization may have a survival advantage over medical treatment maybe needed to prevent stent throm-
alone. Intracoronary stents can ameliorate angina and reduce adverse events in bosis in patients receiving drug-eluting
patients with acute coronary syndromes. However, the long-term ef cacy o intra- stents, but these long-termantiplatelet
coronary stents is limited by subacute luminal restenosis within the stent, which therapies can increase the risko bleeding
occurs in a substantial minority o patients. T e development o drug-eluting stents (see Chapter 19).
to prevent the smooth muscle proli eration that is responsible or restenosis has
had an important impact on clinical practice. wo drugs are currently being used
in intravascular stents: paclitaxel and sirolimus (see Side Bar DRUG-ELU ING
IN RACORONARY S EN S).
Stent-induced damage to the vascular endothelial cell layer can lead to thrombosis;
patients typically are treated with antiplatelet agents, including clopidogrel ( or
up to 6 months) and aspirin (inde nitely), sometimes in conjunction with intra-
venous heparin and/or GPIIb/IIIa inhibitors (see Chapter 19) administered at the
time o the revascularization procedure. T e inhibition o cellular proli eration by
paclitaxel and sirolimus not only a ects vascular smooth muscle cell proli eration
but also attenuates the ormation o an intact endothelial layer within the stented
artery. T ere ore, antiplatelet therapy (typically with clopidogrel) is continued or
several months a er intracoronary stenting with drug-eluting stents.

CASE 16-5
A 42-year-old woman develops symptoms o angina at rest. She has not had angina
be ore and has no history o cardiovascular disease. A er an exercise stress test and
other tests, she is diagnosed as having variant angina.
a. What is variant angina and what is the main therapeutic goal in treating it?
In variant angina, ocal or di use coronary vasospasm episodically reduces coro-
nary ow. Patients also may display a mixed pattern o angina with the addition
o altered vessel tone on a background o atherosclerotic narrowing. T e principal
therapeutic aim in variant or Prinzmetal angina is to prevent coronary vasospasm
and improve O2 delivery to ischemic myocardium.
b. What are the treatment options to relieve this patients symptoms o angina?
Whereas long-acting nitrates alone are occasionally ef cacious in abolishing epi-
sodes o variant angina, additional therapy with Ca2+ channel blockers usually is
required. Ca2+ channel blockers, but not nitrates, have been shown to in uence
mortality and the incidence o MI avorably in variant angina; they should gener-
ally be included in therapy. Receptor antagonists should be used with caution i
the underlying pathophysiology is coronary vasospasm.

297
SECTION III Modulation of Cardiovascular Function

KEY CONCEPTS
In typical stable angina, the primary therapeutic goal is to reduce myocardial
O2 demand, typically with an organic nitrate, a receptor antagonist, a Ca2+
channel blocker, or a combination o these 3 classes o drugs.
T e therapeutic goal in treating vasospastic angina is to prevent coronary
vasospasm and maintain myocardial O2 supply using a Ca2+ channel blocker, an
organic nitrate, or a combination o these 2 classes o drugs.
Acute coronary syndromes are caused by disruption o an atherosclerotic
coronary plaque causing local platelet aggregation and thrombosis that partially
or totally occludes the coronary vessel.
T e primary therapeutic goal in treating acute coronary syndromes is to prevent
the disruption o coronary blood ow by inhibiting platelet aggregation and
thrombosis, and secondarily reducing myocardial O2 demand.
In addition to pharmacological therapy, long-term prevention o angina in
patients with acute coronary syndrome may require angioplasty, intracoronary
stents, or in some patients, coronary bypass surgery.

SUMMARY QUIZ

QUESTION 16-1 T e antianginal drug o choice to use in a patient with vasospastic


angina who has sinus bradycardia is
a. propranolol.
b. verapamil.
c. diltiazem.
d. amlodipidine.
e. nitroglycerin.

QUESTION 16-2 A patient with chronic exertional angina is re ractory to combination


therapy with a blocker, organic nitrate, and Ca2+ channel blocker. Which o the
ollowing drugs may be use ul in reducing the symptoms o angina in this patient?
a. A PDE5 inhibitor
b. Aspirin
c. Heparin
d. A statin
e. Ranolazine

QUESTION 16-3 T e mechanism underlying the li e-threatening interaction o organic


nitrates and PDE5 inhibitors is that
a. PDE5 inhibitors block the degradation o organic nitrates in the liver.
b. PDE inhibitors block the degradation o cyclic GMP in vascular smooth muscle.
c. PDE inhibitors enhance the conversion o organic nitrates to NO.
d. PDE5 inhibitor degradation is blocked by organic nitrates.
e. PDE5 inhibitor excretion is blocked by organic nitrates.

QUESTION 16-4 T is combination o antianginal drugs should be avoided to circum-


ventan adverse drug interaction:
a. Nitroglycerin and a adrenergic receptor antagonist
b. Nitroglycerin and verapamil
c. Verapamil and a adrenergic receptor antagonist
d. Amlodipine and a adrenergic receptor antagonist
e. Ranolazine and a adrenergic receptor antagonist
298
Drug Therapy o Myocardial Ischemia CHAPTER 1 6

QUESTION 16-5 T e main advantage o isosorbide-5-mononitrate (ISMN) over nitro-


glycerin or treatment o chronic angina is that
a. it is less expensive than nitroglycerin.
b. tolerance does not develop to its antianginal e ects.
c. it can be used in combination with adrenergic receptor blockers.
d. it can be used in combination with Ca2+ channel blockers.
e. it does not undergo rapid rst-pass metabolism allowing once- or twice-daily oral
administration.

SUMMARY QUIZ ANSWER KEY


QUESTION 16-1 Answer is d. Amlodipine is the drug o choice to use in this patient.
Adrenergic blockers, verapamil, and diltiazem will all slow his heart rate by inhibiting
SA nodal activity, which will exacerbate his sinus bradycardia. Although nitroglycerin
will not inhibit SA nodal activity, it is not as e ective as a dihydropyridine in prevent-
ing the symptoms o angina in a patient with vasospastic angina.
QUESTION 16-2 Answer is e. Ranolazine is an antianginal agent reserved or use in
patients who are re ractory to treatment with other antianginal agents. It has a unique
mechanism o action that reduces myocardial O2 demand without a ecting hemodynamics.
It appears to act by blocking the late inward Na+ current (late INa) in ventricular myocytes,
which in turn reduces intracellular concentrations o Na+ during diastole. Lowering intra-
cellular Na+ acilitates e ux o Ca2+ rom myocytes through the Na+-Ca2+ exchanger. T e
reduction in intracellular Ca2+ acilitates relaxation o ventricular myocytes and reduces O2
demand. Ranolazine would be added to the antianginal therapy this patient is receiving.
QUESTION 16-3 Answer is b. PDE5 inhibitors inhibit the degradation o cyclic GMP
in vascular smooth muscle cells which increases the intracellular concentration o
cyclic GMP. T is causes vascular smooth muscle to relax which results in a lowering
o blood pressure. T e organic nitrates act by stimulating the synthesis o cyclic GMP
in vascular smooth muscle which also leads to a lowering o blood pressure. When an
organic nitrate is used within 24 hours o a PDE5 inhibitor, there can be a dangerous
drop in blood pressure because these di erent agents cause an additive or synergistic
increase in vascular smooth muscle cyclic GMP concentrations.
QUESTION 16-4 Answer is c. Because verapamil and blockers have additive e ects
on inhibiting SA node and AV conduction, the combination should be avoided to
prevent severe bradycardia and conduction block. T ese agents can also have additive
e ects on decreasing le ventricular unction which could seriously impair cardiac
output in a patient with heart ailure.
QUESTION 16-5 Answer is e. Isosorbide-5-mononitrate does not undergo signi cant
rst-pass hepatic metabolism allowing it to be administered orally once or twice daily
depending on the ormulation. However, cutaneous administration o nitroglycerin
using ointments or transdermal patches can also be administered or long-lasting e ect
(see able 27-1 in Goodman & Gilmans T e Pharmacological Basis of T erapeutics,
12th Edition). With any sustained administration o an organic nitrate, tolerance will
develop rapidly and it is important to interrupt therapy or 8 to 12 hours each day,
which allows the return o ef cacy. It is usually most convenient to omit dosing at night
in patients with exertional angina either by adjusting dosing intervals o oral or buccal
preparations or by removing cutaneous nitroglycerin. However, patients whose angi-
nal pattern suggests its precipitation by increased le ventricular lling pressures (ie,
occurring in association with orthopnea or paroxysmal nocturnal dyspnea) may ben-
e t rom continuing nitrates at night and omitting them during a quiet period o the
day. o avoid tolerance with isosorbide-5-mononitrate, an eccentric twice-daily dosing
schedule appears to maintain ef cacy. Some patients develop an increased requency o
nocturnal angina when a nitrate- ree interval is employed using nitroglycerin patches;
such patients may require another class o antianginal agent during this period.

299
SECTION III Modulation of Cardiovascular Function

SUMMARYTABLE: DRUGS USED IN THE TREATMENT OF ANGINA AND ERECTILE DYSFUNCTION


TOXICITIES
CLASS AND
SUBCLASSES NAMES CLINICAL USES COMMON UNIQUE; CLINICALLY IMPORTANT
Organic Nitrate Nitroglycerin (glyceryl Treatment o angina Headache (can be severe), Tolerance can develop rapidly but
Vasodilators trinitrate) (reduce preload and relax transient episodes o can be avoided by interrupting
(Nitrovasodilators, Isosorbide dinitrate large coronary vessels) dizziness, and postural therapy or 8-12 hours each day
NO Donors) (ISDN) hypotension, all o which are PDE5 inhibitors ( or erectile
Isosorbide-5- secondary to e ects on the dys unction) can cause extreme
mononitrate (ISMN) cardiovascular system hypotension in patients
Occasional drug rash taking organic nitrates and are
contraindicated
See Table 16-1

Phosphodiesterase Sildenaf l Treatment o erectile Headache, ushing, rhinitis, Extreme hypotension when
5 Inhibitors (PDE5 Tadalaf l dys unction dyspepsia owing to relaxation administered in combination with
Inhibitors) Vardenaf l Less commonly used to o smooth muscle (all o these organic nitrates
treat pulmonary artery are predictable based on their PDE5 inhibitors should not
hypertension e ects on PDE5) be prescribed to patients
Sildenaf l and vardenaf l can receiving nitrates
produce visual disturbances Nitrates should not be
and one-sided hearing loss administered within 24 hours (or
longer) a ter use o a PDE5 inhibitor

Adrenergic Propranolol Treatment o angina See Chapter 7 See Table 16-1 and Chapter 7
Receptor Timolol (slow heart rate, decrease
Antagonists Metoprolol contractility)
Atenolol Other indications are
Many others (see described in Chapter 7
Chapter 7)

Ca2+ Channel Verapamil Treatment o angina Constipation, headache, IVadministration with IV blocker
Antagonists (Ca2+ (reduce a terload, relax ushing, dizziness, and is contraindicated because o risk o
Entry blockers) coronary vessels, slow heart peripheral edema AVblock and/or severe depression
Phenylalkylamine rate, decrease contractility) o ventricular unction
Treatment o hypertension Avoid in patients with ventricular
(see Chapter 15) dys unction, SA or AVnodal
conduction disturbances, or systolic
blood pressures below 90 mm Hg
See Table 16-1
Blocks the P-glycoprotein
transporter

Ca2+ Channel Diltiazem Treatment o angina Headache, ushing, dizziness, Avoid in patients with ventricular
Antagonists (Ca2+ (reduce a terload, relax and peripheral edema dys unction, SA or AVnodal
Entry Blockers) coronary vessels, slow heart conduction disturbances,
Benzothiazepine rate, decrease contractility) or systolic blood pressures
Treatment o hypertension <90 mm Hg
(see Chapter 15) See Table 16-1

Ca2+ Channel Amlodipine Treatment o angina Headache, ushing, dizziness, May aggravate anginal symptoms
Antagonists (Ca2+ Clevidipine (reduce a terload, relax and peripheral edema in some patients when used
Entry Blockers) Felodipine Isradipine coronary vessels) without a blocker because o
Dihydropyridines Nicardipine Treatment o hypertension re ex tachycardia
Ni edipine Nimodipine (see Chapter 15) See Table 16-1
Nisoldipine

Na+ Channel Ranolazine Treatment o angina Prolongs QTc interval Avoid using with other drugs that
Blocker (Late Inward (reduces ventricular prolong QT interval
Na+ Current Blocker) diastolic wall tension, Contraindicated in patients with
reducing O2 demand); used hepatic cirrhosis
in patients re ractory to
other antianginal therapy

300
CHAPTER

Pharmacotherapyof Heart Failure 17


T is chapter will be most use ul a er having a basic understanding o the material in
DRUGS IN THIS CHAPTER
Chapter 28, Pharmacotherapy o Heart Failure in Goodman & Gilmans T e Pharma-
cological Basis of T erapeutics, 12th Edition. In particular, the reader is directed to the Bisoprolol (ZEBETA)
ollowing table: Candesartan (ATACAND)
able 28-3, Vasodilator Drugs Used to reat Heart Failure, which shows drugs used Captopril (CAPOTEN, others)
as vasodilators to treat heart ailure including their mechanism o action and their Carvedilol (COREG, others)
e ects on preload and a erload
Digoxin (LANOXIN, others)
LEARNING OBJECTIVES Dobutamine (DOBUTREX,others)
Know the stages o heart ailure and the treatments that are recommended at Dopamine
each stage. Enalapril (VASOTEC, others)
Understand the rationale or the use o drugs that prevent and slow the Eplerenone (INSPRA, others)
progression o heart ailure. Furosemide (LASIX,others)
Understand the mechanism o action o inotropic drugs and how they are used Hydralazine (APRESOLINE)
to maintain le ventricular unction.
Hydralazine and isosorbide dinitrate
Identi y the major side e ects and adverse drug reactions o the drugs used to (BIDIL)
treat heart ailure. Inamrinone (INOCOR)
Isosorbide dinitrate (ISORDIL, SORBITRATE,
MECHANISMS OF ACTIONS OF DRUGS USED TO TREAT HEART FAILURE (SEE others)
FIGURE 17-1) Labetalol (NORMODYNE, TRANDATE,
DRUG CLASS DRUG MECHANISM OF ACTION
others)
Lisinopril (PRINIVIL, ZESTRIL, others)
Angiotensin-Converting Captopril, enalapril, lisinopril, Prevent the conversion o AngI to
Enzyme (ACE) Inhibitors others (see Chapter 15) active AngII by inhibiting angiotensin Losartan (COZAAR)
converting enzyme (ACE) Metoprolol (LOPRESSOR, TOPROLXL,
Angiotensin Receptor Losartan, candesartan, valsartan, Competitive (but insurmountable) others)
(AT1) Blockers (ARBs) others (see Chapter 15) inhibitors o angiotensin AT1 Milrinone (PRIMACORIV)
receptors
Minoxidil (LONITEN)
Speci c Adrenergic Metoprolol, carvedilol, Block adrenergic receptors in the Nesiritide (NATRECOR)
Receptor Antagonists labetalol, bisoprolol, others heart, thereby protecting the heart
(see Chapter 7) rom the cytotoxic e ects o high
Nitroglycerin (trinitroglycerin, NITROSTAT,
circulating catecholamines NITRO-BID, others)
Blockade o adrenergic receptors Nitroprusside (NITROPRESS, others)
in the juxtaglomerular complex
reduces renin secretion, thereby
Spironolactone (ALDACTONE)
reducing plasma AngII Thiazide diuretics (many, see Chapter 15)
Na+/K+-ATPase Inhibitors Digoxin Inhibit the Na+/K+-ATPase in Valsartan (DIOVAN)
(Cardiac Glycosides) cardiac myocytes which ultimately
increases accumulation o
intracellular Ca2+ by slowing
Ca2+ e ux through the Na+-Ca2+
exchanger (see Figure 17-2)

Diuretics (various Thiazides, loop diuretics Reduce extracellular uid volume


mechanisms, see ( urosemide, others), K+-sparing and ventricular lling pressure by
Chapter 15) diuretics (see Chapter 15) increasing water and sodium excretion

Vasodilators (various Hydralazine, organic nitrates, Reduce a terload and preload


mechanisms, Table 17-1) others (see Chapter 15) ( lling pressure)
(Continued)
301
SECTION III Modulation of Cardiovascular Function

CHARACTERISTICS AND DRUG CLASS DRUG MECHANISM OF ACTION


CAUSES OF HEART FAILURE Aldosterone Spironolactone, eplerenone Block the pathophysiological
antagonists e ects o aldosterone that occur
Congestive heart ailure (CHF) is a complex
with activation o the renin-
clinical syndrome characterized by: angiotensin-aldosterone system
Impaired ventricular per ormance (see Table 28-2 in Goodman &
caused bythe inabilityo the le t Gilmans The Pharmacological Basis o
ventricle to adequately ll (diastolic Therapeutics,12th Edition)
dys unction) or empty(systolic Adrenergic and Dobutamine, dopamine Provide short-term increase in
dys unction) Dopaminergic Receptor cardiac contractility in patients with
Exercise intolerance Agonists severely decompensated CHF
Dopamine also increases renal
Ahigh incidence o ventricular blood ow and increases diuresis
arrhythmias
Type III Milrinone, inamrinone Inhibit degradation o cAMP
Shortened li e expectancy
Phosphodiesterase in cardiac and smooth muscle
There is no single de nitive diagnostictest (PDE3) Inhibitors myocytes which increases cardiac
or CHF (Inodilators) contractility and relaxes vascular
smooth muscle
Pathophysiological mechanisms that lead
to congestive heart ailure include activa- Natriuretic Peptide Nesiritide (Human Stimulates natriuretic peptide
tion o the sympatheticnervous system Receptor Agonist recombinant B-type natriuretic receptors which increases synthesis
and renin-angiotensin-aldosterone axis peptide; BNP) o cGMP in target tissues causing
(see Figure 17-1) natriuresis and vascular smooth
muscle relaxation
Manydiseases can lead to heart ailure,
with the most common in the United
States being:
Coronaryarterydisease
Hypertension
Diabetes mellitus
He a rt fa ilure
Drug abuse
Inotropic a ge nts , digoxin X
ca rdia c output
Digoxin X
a nta gonis ts
Sympa the tic ne rvous Eleva te d ca rdia c filling pre s s ure s
Re nin X sys te m a ctiva tion
SYMPTOMS OF HEART
FAILURE X Re nin
inhibitors Va s odila tors X
Fatigue Angiote ns in I
X X Va s ocons triction

Shortness o breath X ACE


inhibitors
Rapid heart rate Angiote ns in II Na + a nd wa te r re te ntion

Fluid retention resulting in peripheral and X AT1 re ce ptor


pulmonaryedema (congestion) a nta gonis ts X X Diure tics
Aldos te rone X
S pironola ctone
Ca rdia c
X re mode ling

COMORBIDITIES COMMON FIGURE 17-1 Pathophysiologic mechanisms o heart ailure and major sites o drug action.
Congestive heart ailure is accompanied by compensatory neurohormonal responses, including
IN HEART FAILURE activation o the sympathetic nervous and reninangiotensinaldosterone axis. Increased ven-
Coronaryarterydisease (CAD) tricular a terload, due to systemic vasoconstriction and chamber dilation, causes depression in
systolic unction. In addition, increased a terload and the direct e ects o angiotensin and norepi-
Atrial brillation (AF) nephrine on the ventricular myocardium cause pathologic remodeling characterized by progres-
Myocardial in arction (MI) sive chamber dilation and loss o contractile unction. Key congestive heart ailure medications
and their targets o action are presented. ACE, angiotensin-converting enzyme; AT1 receptor, type
Sudden cardiacdeath 1 angiotensin receptor.

302
Pharmacotherapy of Heart Failure CHAPTER 1 7

[Na +]0 = 140 mM, [K+]0 = 4 mM


Exte rior

Inte rior [Na +]i = 10 mM, [K+]i = 150 mM


Ca 2+

Ca 2+-
ATPa s e
P KA
PL
2+
Ca RyR2
SR P KA
S ERCA2 Ca 2+ Ca 2+
P KA L-type cha nne l

Ca 2+ Ca 2+

NCX NCX
(de pola rize d) (pola rize d)
3Na +
3 Na + 3 Na +

Na +, K+
ATPa s e
Na + cha nne l

Na + 2K+

FIGURE 17-2 Sarcolemmal exchange o Na+ and Ca2+ during cell depolarization and repolarization. Na+ and Ca2+ enter the cardiac myocyte
via the Na+ channel and the L-type Ca2+ channel during each cycle o membrane depolarization, triggering the release, through the ryanodine
receptor (RyR), o larger amounts o Ca2+ rom internal stores in the sarcoplasmic reticulum (SR). The resulting increase in intracellular Ca2+ inter-
acts with troponin C and activates interactions between actin and myosin that result in sarcomere shortening. The electrochemical gradient
or Na+ across the sarcolemma is maintained by active transport o Na+ out o the cell by the sarcolemmal Na+,K+-ATPase. The bulk o cytosolic
Ca2+ is pumped back into the SR by a Ca2+-ATPase, SERCA2. The remainder is removed rom the cell by either a sarcolemmal Ca2+-ATPase or a
high-capacity Na+-Ca2+ exchanger, NCX. NCX exchanges 3 Na+ or every Ca2+, using the electrochemical potential o Na+ to drive Ca2+ extrusion.
The direction o Na+-Ca2+ exchange may reverse brie y during depolarization, when the electrical gradient across the sarcolemma is transiently
reversed. Adrenergic agonists and PDE inhibitors, by increasing intracellular cyclic AMP levels, activate PKA, which phosphorylates phosphol-
amban (PL), the subunit o the L-type Ca2+ channel, and regulatory components o the RyR, as well as TnI, the inhibitory subunit o troponin
(not shown). As a result, the probabilities o opening o the L-type Ca2+ channel and the RyR2 Ca2+ channel are doubled; SERCA2 is uninhibited
and accumulates Ca2+ into the SR aster, more avidly, and to a higher concentration; and relaxation occurs at slightly higher [Ca2+]i due to slightly
reduced sensitivity o the troponin complex to Ca2+. The net e ect o these phosphorylations is a positive inotropic e ect: a aster rate o ten-
sion development to a higher level o tension, ollowed by a aster rate o relaxation. m indicates site o cardiac glycoside binding. See the text in
Chapter 28, Pharmacotherapy o Heart Failure in Goodman &Gilmans The Pharmacological Basis o Therapeutics, 12th Edition or the mechanism o
positive inotropic e ect o cardiac glycosides.

RATIONALE FOR
CASE 17-1 PHARMACOLOGICAL
A 64-year-old man who su ered a myocardial in arction at the age o 56 complains THERAPY OF HEART
o di culty sleeping because he wakes up eeling out o breath. Chest sounds reveal FAILURE (Se e Fig u re 17 3)
crackles and rales. He has di culty walking more than 150 eet be ore having to stop to The goal o therapy or patients who are at
catch his breath. He is taking a adrenergic blocker (metoprolol) and an ACE inhibitor risko developing heart ailure (Stages A
(captopril). He also takes ibupro en every day to relieve pain in his knees. and B) is to slowor stop the progression o
a. What Stage o heart ailure is this patient in? the disease, decrease morbidityand mor-
tality, and improve qualityo li e through
T s pat e t s Sta e C (structural sease w th curre t symptoms, see F ure 17-3). the use o inhibitors o the renin-angioten-
H s pr or myocar al arct o cause structural ama e to h s heart wh ch s ow sin-aldosterone system(ACEinhibitors and
mpa r le ve tr cular u ct o e ou h to cause symptoms o heart a lure. ARBs) and adrenergicreceptor blockers.
b. What is causing this patients symptoms? ACEinhibitors, ARBs, and blockers are
T s pat e t has evelope pulmo ary e ema (co est o ) ecause he s reta - also used in patients who have structural
Na+ a water ue to act vat o o the re -a ote s -al ostero e ax s. T e disease with symptoms o heart ailure
pat e t has culty reath at ht whe ly ow ecause the pulmo ary (Stages Cand D), but additional therapies
e ema worse s a mpa rs ve t lat o a as excha e. Heart a lure pat e ts are added to treat symptoms o edema
w th pulmo ary co est o o e sleep more com orta ly the r hea a upper (diuretics), improve hemodynamics (ino-
o y are elevate w th a couple o p llows. T e symptoms o exerc se tolera ce tropes and vasodilators), and acilitate
a short ess o reath are also the result o the pat e ts heart a lure. natriuresis (dopamine and nesiritide).
(Continued)
303
SECTION III Modulation of Cardiovascular Function

TABLE 17-1 Vasodilator Drugs Used to Treat Heart Failure


PRELOAD AFTERLOAD
DRUG CLASS EXAMPLES MECHANISM OF VASODILATING ACTION REDUCTION REDUCTION
Organic nitrates Nitroglycerin, isosorbide dinitrate NO-mediated vasodilation +++ +
NO donors Nitroprusside NO-mediated vasodilation +++ +++
ACE inhibitors Captopril, enalapril, lisinopril Inhibition o AngII generation, BKdegradation ++ ++
AngII receptor blockers Losartan, candesartan AT1 receptors blockade ++ ++
PDE inhibitors Milrinone, inamrinone Inhibition o cyclic AMP degradation ++ ++
K+ channel agonist Hydralazine Unknown + +++
Minoxidil Hyperpolarization o vascular smooth muscle cells + +++
1 antagonists Doxazosin, prazosin Selective 1 adrenergic receptor blockade +++ ++
Nonselective Phentolamine Nonselective adrenergic receptor blockade +++ +++
antagonists
/1 antagonists Carvedilol, labetalol Selective 1 adrenergic receptor blockade ++ ++
Ca2+ channel blockers Amlodipine, ni edipine, elodipine Inhibition o L-type Ca2+ channels + +++
agonists Isoproterenol Stimulation o vascular 2 adrenergic receptors + ++

AngII, angiotensin II; AT1, type 1 angiotensin II receptor; NO, nitric oxide; ACE, angiotensin converting enzyme; PDE, cyclic nucleotide
phosphodiesterase; BK, bradykinin.

c. Why is he taking a blocker and an ACE inhibitor?


Blocker therapy s recomme e pat e ts who have ha a myocar al arc-
t o (see Chapter 16) ecause t re uces mortal ty. Certa lockers are also rec-
omme e ma y heart a lure pat e ts who ca tolerate them ecause t locks
the eleter ous e ects o catecholam es a may lower the r sk o arrhythm as.
ACE h tors are recomme e pat e ts w th Sta e B a C heart a lure to
slow sease pro ress o y h t the e ects o a ote s II.
d. What drugs, i any, should be added, changed, or withdrawn?
o rel eve the symptoms o e ema, uret cs are typ cally a e to the me ca-
t o s o pat e ts Sta e C heart a lure. T e pat e t shoul also e cou sele to
re uce so um take s ce that w ll also co tr ute to u rete t o . NSAIDs
such as upro e may also co tr ute to so um a u rete t o a shoul e
avo e , poss le. T ere s ev e ce that NSAIDs such as upro e may crease
the r sk o car ovascular eve ts such as stroke pat e ts who have ha a MI.

CASE 17-2
You are counseling a 34-year-old woman who is moderately overweight, and has mild
hypertension and type II diabetes. Her ather had his rst myocardial in arction when
he was 45 and died o a subsequent myocardial in arction when he was 52.
a. Is this patient at risk to develop heart ailure and what can she do to lower her risks?
T s woma has several r sk actors or evelop heart a lure, clu o es ty,
hyperte s o , a etes, a her am ly h story o car ovascular sease. She may also
have ysl p em a. She s Sta e A o heart a lure (h h r sk o evelop heart
a lure ut o symptoms). She has o co trol over her e et cs, ut she ca re uce her
r sk o evelop heart a lure y co troll her we ht, loo pressure, a a etes.
b. What medications are recommended or this patient?
T s pat e t may e a le to re uce all o her co trolla le r sks y a opt a
health er l e-style ( e, re ular exerc se a a opt a health er et). I l estyle
(Continued)
304
Pharmacotherapy of Heart Failure CHAPTER 1 7

At Ris k for He a rt Fa ilure He a rt Fa ilure

S ta ge C S ta ge D
S truc tural he art dis e as e Re frac to ry HF
with prio r o r c urre nt
S ta ge B s ympto ms o f HF
S truc tural he art dis e as e P a tie nts with:
S ta ge A but witho ut s ig ns o r -ma rke d HF
At hig h ris k fo r HF but s ympto ms o f HF s ymptoms a t re s t
witho ut s truc tural P a tie nts with: -re curre nt
he art dis e as e o r -known s tructura l hos pita liza tions
s ympto ms o f HF he a rt dis e a s e de s pite a tte mpts to
P a tie nts with: -HF s igns a nd optimize the ra py
-pre vious MI s ymptoms including diure tics
-LV re mode ling a nd guide line -
P a tie nts with: THERAPY
including LV dire cte d me dica l
-hype rte ns ion Go als
hype rtrophy a nd low the ra py
-a the ros cle rotic dis e a s e -Control s ymptoms
e je ction fra ction-
-dia be te s -P re ve nt hos pita liza tion
a s ymptoma tic THERAPY
me llitus -P re ve nt morta lity
va lvula r dis e a s e Go als
-obe s ity
-me ta bolic s yndrome Drug s fo r us e in patie nts -Control s ymptoms
THERAPY -Improve HRQOL
with pre s e rve d EF (EF 50)
or Go als -Re duce hos pita l
P a tie nts : -Diure tics for fluid re te ntion
-P re ve nt HF s ymptoms re a dmis s ions
-us ing ca rdiotoxic drugs -Tre a t comorbiditie s (HTN,
-P re ve nt furthe r ca rdia c -Es ta blis h e nd-of-life goa ls
-with fa mily his tory of AF, CAD, DM)
re mode ling
ca rdiomyopa thy Optio ns
Drug s fo r ro utine us e in
Drug s patie nts with re duc e d EF -Adva nce d ca re me a s ure s
THERAPY -ACE inhibitor or ARB in -He a rt tra ns pla nt
Go als -Diure tics for fluid re te ntion -Chronic inotrope s
a ppropria te pa tie nts -ACE inhibitor or ARB
-He a rt he a lthy life s tyle -Be ta blocke r in -Te mpora ry or pe rma ne nt
-Be ta blocke r me cha nica l s upport
-P re ve nt va s cula r, a ppropria te pa tie nts -Aldos te rone a nta gonis t -Expe rime nta l s urge ry or
corona ry dis e a s e
-P re ve nt LV s tructura l In s e le c te d patie nts Drug s fo r us e in s e le c te d drugs
a bnorma litie s -Impla nta ble patie nts with re duc e d EF -P a llia tive ca re a nd
ca rdiove rte r- -Hydra la zine /IS DN hos pice
Drug s de fibrilla tor (ICD) -ACE inhibitor a nd ARB -ICD de a ctiva tion
-ACE inhibitor or ARB -Re va s cula riza tion or -Digita lis
in a ppropria te pa tie nts va lvula r s urge ry a s
for va s cula r dis e a s e or In s e le c te d patie nts
a ppropria te
DM -CRT
-S ta tins a s a ppropria te -ICD
-Re va s cula riza tion or
va lvula r s urge ry a s
a ppropria te

FIGURE 17-3 Stages in the development o HF and recommended therapy by stage. ACE indi-
cates angiotensin-converting enzyme; AF, atrial brillation; ARB, angiotensin-receptor blocker;
CAD, coronary artery disease; CRT, cardiac resynchronization therapy; DM, diabetes mellitus; EF,
ejection raction; GDMT, guideline-directed medical therapy; HF, heart ailure; HRQOL, health-
related quality o li e; HTN, hypertension; ICD, implantable cardioverter-de brillator; LV, le t
ventricular; and MI, myocardial in arction. Preserved EF is de ned as an EF 50%. Reduced EF is
de ned as an EF 40%. (Modif ed rom Yancy CW et al. Circulation. 2013;128:e240-e327.)

mo f cat o s o ot ully co trol her a etes a hyperte s o , a t a et c


(see Chapter 30) a a t hyperte s ve therap es (see Chapter 15) are recom-
me e . T e a t hyperte s ves that woul e most appropr ate th s pat e t are
ACE h tors or A 1 lockers (see Chapter 15). I she has ysl p em a that s
ot co trolle y et a exerc se, a l p -lower ru s recomme e (see
Chapter 20).

CASE 17-3
You are reviewing the medications o a 67-year-old man who has recently developed
mild pulmonary congestion and peripheral edema. He has a history o mild heart ail-
ure, hypertension, and he occasionally su ers rom angina. T e drugs he is prescribed
to take on a daily basis are aspirin, urosemide, lisinopril, lovastatin, metoprolol, and
verapamil. He is also on a low-salt diet.
a. What are the possible reasons or the pulmonary congestion and peripheral edema?
Desp te e o a ACE h tor (l s opr l), a locker (metoprolol), a a loop
uret c ( urosem e), th s pat e t s ecompe sat a reta u . Its pos-
s le the pat e ts heart a lure s worse , or the pat e t s ot a her to the
low-salt et or s ot tak h s uret c as prescr e .
(Continued)
305
SECTION III Modulation of Cardiovascular Function

b. What can be done to relieve this patients symptoms?


BIDIL AND THE AFRICAN-
AMERICAN HEART FAILURE It s mporta t to f rst el m ate a y poss le causes o the e ema or worse
TRIAL o heart a lure. O e ru that m ht e worse the pat e ts heart a lure s
verapam l, a calc um cha el locker that ca lower le ve tr cular u ct o . It
The Arican-American Heart Failure Trial shoul e poss le to a equately co trol the pat e ts hyperte s o us l s opr l,
showed signi cantlyreduced mortalityand metoprolol, a urosem e. I a calc um cha el locker s ee e to co trol the
morbidityin sel -described Arican-American pat e ts a a, a hy ropyr e calc um cha el locker (see Chapter 15) w th
patients with heart ailure (predominantly m mal e ects o le ve tr cular u ct o s recomme e . It s also mporta t
NYHAClass III) taking standard therapywith to eterm e whether the pat e t s a her to a low-salt et a s tak hs
hydralazine and isosorbide dinitrate (ISDN). uret c a other ru s as prescr e .
This lead to the development o the xed-
dose combination agent, BiDil (hydralazine
CASE 17-4
and ISDN).
The mechanism or the greater e cacyo A 72-year-old A rican-American woman with mild heart ailure comes in or her
this drug combination maybe due to lower annual check-up. During her examination, she complains o occasional dizziness, espe-
vascular responsiveness to nitricoxide (NO) cially when she stands up quickly. She is taking an ACE inhibitor, thiazide diuretic, and
in Arican-Americans. blocker. Her lung sounds are normal and there is no evidence o peripheral edema.
BiDil was the rst race-based drug to be a. What might be the reason or this patients complaint?
approved bythe FDA. She has orthostat c hypote s o wh ch m ht e ue to tak mult ple ru s that
BiDil is recommended or patients sel - ca lower her loo pressure. Althou h all o these ru s are appropr ate a
described as Arican-Americans, with pat e t w th m l heart a lure, t may e ecessary to re uce the os o o e or
moderate to severe symptoms o CHFwho more, espec ally the uret c s ce she has o ev e ce o co est o .
are alreadyon optimal therapywith ACE
b. Are there other heart ailure therapies that might be appropriate or this patient?
inhibitors, blockers, and diuretics.
I her symptoms worse a are ot well co trolle y the ACE h tor a
locker, t m ht e worth a B D l (see S e Bar BIDIL AND HE AFRI-
CAN-AMERICAN HEAR FAILURE RIAL). However, her symptoms o m l
heart a lure appear to e well ma a e so cha es her me cat o s are ot
appropr ate except or re uc or el m at the uret c.

CASE 17-5
A 72-year-old man with atrial brillation is hospitalized because o severe pulmonary
and peripheral edema that is not adequately treated by his current medications that
include losartan, carvedilol, and urosemide. During his stay in the hospital, his edema
is controlled by aggressive intravenous (IV) urosemide administration and a strict
low-salt diet. He is also started on digoxin and spironolactone.
a. What is the rationale or adding digoxin?
D ox s a m l otrope that was o ce use most pat e ts w th co est ve
heart a lure. Cl cal tr als have show that t oes ot re uce mortal ty ut oes
re uce hosp tal zat o rates o pat e ts w th mo erate to severe heart a lure whe
a e to sta ar therapy. By creas the otrop c state o the ve tr cles, t w ll
mprove k ey u ct o a re uce the r sk o rehosp tal zat o ue to e ema.
T s pat e t w ll also e ef t rom ox ecause he has atr al f r llat o , wh ch
ca lea to ve tr cular tachycar a. D ox w ll crease va al to e to the heart
a there y slow or lock co uct o o mpulses pass throu h the AV o e
a protect the ve tr cles rom atr al- uce tachycar a.
b. What are the risks o digoxin therapy in this patient?
D ox has a arrow therapeut c ex, w th tox c ty poss le at therapeut c oses.
L e-threate tox c t es clu e ve tr cular arrhythm as a s us ra ycar a.
c. What are the symptoms o digoxin toxicity?
T e symptoms o ox tox c ty clu e car ac rhythm stur a ces, eurolo cal
e ects, clu var ous v sual stur a ces, a astro test al symptoms. Severe
tox c ty ca e treate w th a eutral z a t ox a t sera Fa ra me t (digibind).
(Continued)
306
Pharmacotherapy of Heart Failure CHAPTER 1 7

d. What is the rationale or adding spironolactone?


Sp ro olacto e s a al ostero e a ta o st that re uces mortal ty a heart a lure-
relate hosp tal zat o s pat e ts w th mo erate to severe heart a lure y h t
the eleter ous e ects o al ostero e o k ey a car ac u ct o (see a le 28-2
Goodman & Gilmans T e Pharmacological Basis of T erapeutics, 12th E t o ).
e. What are the side e ects o spironolactone?
reatme t w th sp ro olacto e s e erally well tolerate , ut there s a r sk o severe
hyperkalem a, espec ally com at o w th a ACE h tor a pat e ts w th
re al mpa rme t. Gy ecomast a s a commo s e e ect o sp ro olacto e that s
ot commo w th the more al ostero e-spec f c a ta o st, eplere o e.

CASE 17-6
A 72-year-old woman who had a coronary artery bypass gra when she was 60, and
has had symptoms o heart ailure or the past 6 years is hospitalized with decompen-
sated heart ailure. Her medications prior to hospitalization included losartan, meto-
prolol, aldosterone, urosemide, and digoxin.
a. What medications should be considered to restore lef ventricular unction?
A pare teral otrope such as o utam e, opam e, or a type 3 phospho ester-
ase (PDE3) h tor s o e ee e or pat e ts Sta e D heart a lure to ma -
ta LVF. T ese a e ts are much more power ul otropes tha ox . T e cho ce
o a a e t epe s o whether the pat e t s hypote s ve, has s s o re al su -
f c e cy, or has ecome re ractory to a other otrope.
b. What are the mechanisms by which inotropic agents exert their e ects on the heart?
All otrop c a e ts act y creas Ca2+ el very to the co tract le apparatus
ve tr cular myocytes, ut throu h ere t mecha sms.
D ox (a car ac lycos e) s a weak orally a m stere otrope that acts y
h t myocar al Na+, K+-A Pase. I h t o o th s pump re uces the a l ty
o the myocyte to remove Ca2+ throu h the Na+-Ca2+ excha er (NCX). T s results
more tracellular Ca2+, more Ca2+ e take up y the sarcoplasm c ret culum
(SR), a more Ca2+ release rom the SR ur su seque t exc tat o -co tract o
cycles (see F ure 28-5 Goodman & Gilmans T e Pharmacological Basis of T era-
peutics, 12th E t o ).
Do utam e a opam e are a re er c a o sts that are a m stere par-
e terally a are reserve or pat e ts Sta e D heart a lure. At the oses use
cl cally to ma ta car ac output pat e ts w th heart a lure, the r pr mary
mecha sm s throu h st mulat o o myocar al a re er c receptors. Act va-
t o o a re er c receptors causes a crease cAMP, wh ch act vates prote
k ase A (PKA). Act vat o o PKA results phosphorylat o o the L-type Ca2+
cha el wh ch creases Ca2+ e try to car ac myocytes. PKA also phosphorylates
prote s o the sarcoplasm c ret culum (SR) mem ra e that crease SR uptake o
Ca2+ a su seque t release o Ca2+ rom the SR.
Do utam e s a m stere as a co t uous IV us o (t1/2 o ~2 m utes) to
pat e ts e -sta e systol c ys u ct o (Sta e D) to ma ta car ac output. It
s a stro pos t ve otrope ecause o ts myocar al 1 receptor a o sm . It also
re uces loa o the heart ecause o ts 2 a re er c vaso lat e ects skeletal
muscle loo vessels. achyphylax s ue to a re er c receptor ese s t zat o
e s to evelop at 2 hours a ecomes s f ca t a er 72 hours wh ch may
requ re a or sw tch to a PDE3 h tor to ma ta car ac output. Some
pat e ts Sta e D heart a lure may e ef t rom co curre t a m strat o o a
locker, ut th s ca re uce the e cacy o o utam e.
Dopam e s pre erre over o utam e pat e ts w th car o e c shock or com-
prom se re al u ct o ecause o ts a l ty to crease mea arter al pressure a
(Continued)
307
SECTION III Modulation of Cardiovascular Function

st mulate atr ures s. T e hemo y am c e ects o opam e are ose- epe e t


NESIRITIDE
ue to ts a l ty act vate a spectrum o receptors the vasculature w th ere t
Nesiritide is recombinant B-type natriuretic a t es (see Chapter 7). At the h h oses o opam e use pat e ts w th heart
peptide (BNP) approved or acute treatment a lure, there s a crease car ac co tract l ty (act vat o o heart 1 receptors),
o patients with decompensated heart ailure. systol c loo pressure s crease (act vat o o per pheral arter al a re er c
BNPis normallyreleased bythe ventricles receptors), ve ous retur s crease (act vat o o ve ous a re er c receptors),
when exposed to high lling pressures that a atr ures s s st mulate .
stretch the cardiacmuscle bers M lr o e a amr o e are use or short-term pare teral otrop c support
BNPis administered bycontinuous IV pat e ts Sta e D heart a lure. T ese a e ts h t type 3 cycl c ucleot e phos-
in usion and lowers preload and a ter- pho esterase (PDE3). Because PDE3 hy rolyzes cAMP car ac a vascular
load without increasing heart rate, and smooth muscle cells, h t o o th s e zyme creases cAMP these cells. I
improves renal unction car ac myocytes, h t o o PDE3 e ha ces Ca2+ el very to the co tract le pro-
Hypotension is a major adverse ef ect o te s throu h the same PKA-me ate pathway as a re er c a o sts. I vascular
BNPand should not be used in patients smooth muscle cells, creases cAMP result PKA phosphorylat o o su strates
with lowblood pressure that cause relaxat o a vaso lat o . T e PDE3 h tors are somet mes re erre
to as o lators ecause o the r com e otrop c a vaso lat e ects. How-
ever, they also crease heart rate a pre spose pat e ts to ve tr cular arrhythm as.
Because tachyphylax s oes ot occur w th the PDE3 h tors, they ca e use to
temporar ly replace o utam e or chro c otrope therapy.
c. What is the pre erred parenteral inotrope i this patient is normotensive?
Do utam e s pre erre ormote s ve pat e ts, ut ecause o tachyphylax s,
sw tch to a PDE3 h tor w ll e requ re a er ~72 hours o o utam e
a m strat o .
d. Which inotropic agent is used i the patient is hypotensive?
Dopam e at h h oses w ll crease loo pressure ecause o ts a re er c
vasoco str ctor e ects o per pheral arter als.

PHARMACOLOGYOF INOTROPIC DRUGS


DRUG MAP PCWP SVR CO PATIENT POPULATION
Digoxin / / AFib or symptomatic patients on
maximal blocker and ACE-I therapy

Dopamine / Pre erred in patients with septic/


(high dose) cardiogenic shock or renal dys unction

Dobutamine / / Pre erred agonist or end-stage


systolic dys unction

Milrinone, / Pre erred in patients on blocker


inamrinone therapy

AFib: atrial brillation; CO: cardiac output; MAP: Mean arterial pressure; PCWP: Pulmonary
capillary wedge pressure; SVR: Systemic vascular resistance

KEY CONCEPTS
Contemporary approaches to treating heart ailure recognize that it is a pro-
gressive disorder that can be staged.
ACE inhibitors and ARBs are rst-line drugs used at all stages to prevent and
slow the progression o heart ailure by inhibiting the renin-angiotensin-aldo-
sterone system.
Blockers are rst-line drugs used a er structural heart disease occurs to pre-
vent the cardiotoxic and arrhythmogenic e ects o high catecholamine levels.
Diuretics are used symptomatically to reduce edema and f uid volume.
Diuretics do not reduce mortality.
(Continued)
308
Pharmacotherapy of Heart Failure CHAPTER 1 7

Vasodilators are used in selected patients in combination with the rst-line drugs
to reduce the load on the heart.
Inotropic agents are used at late stages o heart ailure to improve symptoms by
maintaining adequate cardiac output.
Inotropes do not reduce mortality or slow progression o heart ailure.

SUMMARY QUIZ

QUESTION 17-1 You are counseling a heart ailure patient who is receiving a new pre-
scription or digoxin to treat their atrial brillation. Which o the ollowing are common
side e ects that might indicate digoxin toxicity?
a. Blood in the urine
b. Muscle cramps and joint pain
c. GI disturbances and yellow-green vision changes
d. Photosensitivity and skin discoloration

QUESTION 17-2 A 32-year-old woman has chronic hypertension and a amily history o
cardiomyopathy. She has no history hersel o heart problems and no symptoms o heart
ailure. Which drug is most appropriate to prevent or slow the development o heart ailure
in this patient?
a. Lisinopril
b. Verapamil
c. Digoxin
d. Dobutamine

QUESTION 17-3 A 74-year-old man who sel -describes himsel as being A rican-
American is taking metoprolol, losartan, urosemide, and digoxin to treat his heart
ailure. However, he still has some symptoms o heart ailure. T e most appropriate
drug to add to this patients drugs is
a. captopril.
b. diltiazem.
c. nesiritide.
d. BiDil (a combination o hydralazine and isosorbide dinitrate).

QUESTION 17-4 You are reviewing the discharge instructions o a 52-year-old man
who was hospitalized with a mild myocardial in arction. He has recovered well and
will be able to return to work next week. Be ore his hospitalization, he was tak-
ing lovastatin, enalapril, and low-dose aspirin. T e ollowing drug should be added
to these drugs because clinical trials have demonstrated that it will help reduce his
risk o death:
a. diltiazem.
b. hydrochlorothiazide.
c. carvedilol.
d. digoxin.

QUESTION 17-5 A patient who has recently been hospitalized with severe heart ailure
and has been receiving IV dobutamine starts to have worsening symptoms. What drug
should be used to replace the dobutamine?
a. Norepinephrine
b. Epinephrine
c. Isoproteronol
d. Milrinone

309
SECTION III Modulation of Cardiovascular Function

SUMMARY QUIZ ANSWER KEY


QUESTION 17-1 Answer is c. GI disturbances such as nausea are some o the rst
indications o digoxin toxicity. Yellow-green vision changes are also a common mani-
estation o cardiac glycoside toxicity.
QUESTION 17-2 Answer is a. T is patient has multiple risk actors or development
o heart ailure including hypertension and a amily history o cardiomyopathy. She is
in Stage A o heart ailure (see Figure 17-3). She should be started on an ACE inhibi-
tor such as lisinopril to control her blood pressure and to reduce her risk o developing
structural heart disease (ie, slow or prevent progression to Stage B).
QUESTION 17-3 Answer is d. T is patient is taking appropriate medications or his
heart ailure, including rst-line medications, an ARB (losartan), and a blocker
(metoprolol), which have been shown to reduce morbidity and mortality in heart
ailure. T e loop diuretic ( urosemide) and mild inotrope (digoxin) will help with
symptoms, but do not reduce mortality. Evidence rom clinical trials shows that BiDil
(hydralazine and isosorbide dinitrate) can reduce mortality in A rican-Americans with
heart ailure who are being maximally treated with rst-line agents but who are still
symptomatic.
QUESTION 17-4 Answer is c. Carvedilol or another appropriate blocker (metoprolol
or bisoprolol) should be initiated be ore discharge to reduce mortality in patients who
have had a myocardial in arction. T e blocker should be started with a low dose and
gradually increased over the course o 6 to 8 weeks until the target dose or maximally
tolerated dose is reached.
QUESTION 17-5 Answer is d. T e PDE3 inhibitors, milrinone and inamrinone, are
used to substitute or dobutamine when tachyphylaxis develops to the receptor
agonist. T ey act by increasing intracellular myocardial and vascular smooth muscle
cAMP independent o receptor activation and are not associated with tachyphylaxis.
However, they are associated with li e-threatening cardiac arrhythmias and their
use is restricted to short-term inotropic support. Milrinone is pre erred to inamrinone
because o its greater selectivity or PDE3, short t1/2, and avorable side e ect pro le.

SUMMARY: DRUGS USED TO TREAT HEART FAILURE


TOXICITIES
CLASS AND UNIQUE; CLINICALLY
SUBCLASSES NAMES CLINICAL USES COMMON IMPORTANT
Angiotensin-Converting Captopril First-line agents used in all Hypotension and renal Angioneurotic edema (due
Enzyme (ACE) Inhibitors Enalapril, stages o heart ailure to insuf ciency to increased production
Lisinopril slow disease progression Dry cough due to increased o kinins)
Others (see Chapter 15) and reduce mortality levels o kinins Teratogenic

Angiotensin Receptor Losartan Same as ACE inhibitors; Hypotension and renal Teratogenic
(AT1) Blockers (ARBs) Candesartan indicated in patients who insuf ciency
Valsartan cannot tolerate the side
Others (see Chapter 15) e ects o increased levels
o kinins associated with
ACE inhibitors

Adrenergic Receptor Metoprolol First-line agents used in Bradycardia, hypotension Risk o hypoglycemic
Blockers Carvedilol Stages B, C, and D to slow Can impair LV unction and reactions may be increased
Bisoprolol disease progression and exacerbate heart ailure in diabetics taking insulin
reduce mortality symptoms
Should be avoided in patients
with reactive airway disease
(asthma) or with SA or AVnodal
dys unction or in combination
with other drugs that inhibit AV
conduction, such as verapamil
310
Pharmacotherapy of Heart Failure CHAPTER 1 7

TOXICITIES
CLASS AND UNIQUE; CLINICALLY
SUBCLASSES NAMES CLINICAL USES COMMON IMPORTANT

Na+, K+-ATPase Inhibitor Digoxin Indicated in patients with GI disturbances Can exacerbate
LVdys unction in atrial Neurological disturbances supraventricular arrhythmias
brillation, or patients (con usion, hallucinations, and lead to ventricular
who remain symptomatic other), visual (yellow-green) arrhythmias
despite maximal therapy disturbances Ventricular
with ACE inhibitors and bradycardia, AVblock,
blockers supraventricular tachycardia

Aldosterone Receptor Spironolactone Indicated or patients with Hyperkalemia Gynecomastia (less requently
Antagonists Eplerenone low LVejection raction Can induce diarrhea and other with eplerenone)
When added to standard gastric disturbances Li e-threatening hyperkalemia,
therapy, morbidity and Can cause CNS e ects such thus contraindicated in
mortality are reduced in drowsiness, ataxia, and patients with hyperkalemia or
patients with heart ailure headache those at risk o hyperkalemia
Contraindicated in patients
with peptic ulcers

Vasodilators Hydralazine To reduce preload ( lling Hypotension Depends on agent (see


Organic nitrates pressure) and a terload Chapter 15)
Others (see Table 17-1 (mean arterial pressure)
and Chapter 15)

Diuretics (Many Many agents (see To reduce uid retention, Hypokalemia, hyponatremia, Depends on agent (see
Subclasses) Chapter 15) especially in patients with and other electrolyte e ects, Chapter 15)
peripheral and pulmonary depending on agent (see
edema Chapter 15)

Adrenergic Receptor Dopamine Inotropic agents Arrhythmogenic, increased Tachyphylaxis develops


Agonists Dobutamine Indicated in patients with mortality rapidly requiring short-term
end-stage heart ailure substitution with PDE3
(Stage D) requiring chronic inhibitors
inotropic therapy to
maintain LV unction

Phosphodiesterase Type Milrinone Parenteral short-term Arrhythmogenic, increased


III (PDE3) Inhibitors Inamrinone agents used in end-stage mortality
heart ailure patients who
have become tolerant to
adrenergic agonists

Natriuretic Peptide Nesiritide Indicated in acutely Hypotension


Receptor Agonist decompensated end-stage
heart ailure patients to
increase natriuresis, and
reduce preload and a terload

311
CHAPTER

18 AntiarrhythmicDrugs
T is chapter wi be most use u a er having a basic understanding o the materia
DRUGS IN THIS CHAPTER
in Chapter 29, Anti-Arrhythmic Drugs in Goodman & Gilmans T e Pharmacological
Adenosine (ADENOCARD) Basis of T erapeutics, 12th Edition. In particu ar, the reader is directed to the o owing
Amiodarone (CORDARONE) tab es, and to animations avai ab e in the on ine version o Goodman & Gilmans T e
Pharmacological Basis of T erapeutics:
Digoxin (LANOXIN; see Chapter 17)
Diltiazem(CARDIZEM) ab e 29-1 Drug-Induced Cardiac Arrhythmias, which shows drugs known to cause
arrhythmias (ie, proarrhythmias), the ike y arrhythmogenic mechanism, the treat-
Disopyramide (NORPACE) ment, and the c inica eatures o the proarrhythmia
Do etilide (TIKOSYN) ab e 29-4 Patient-Speci c Anti-arrhythmic Drug Contraindications, which shows
Dronedarone (MULTAQ) conditions and contraindicated drugs
Esmolol (BREVIBLOC) Severa animations in the on ine version o Goodman & Gilmans T e Pharmaco-
Flecainide (TAMBOCOR) logical Basis of T erapeutics i ustrate the e ectrophysio ogy o cardiac ce s and the
Ibutilide (CORVERT) mechanism o action o antiarrhythmic drugs
Lidocaine (XYLOCAINE) LEARNING OBJECTIVES
Mexiletine (MEXITIL) Know the princip es o cardiac e ectrophysio ogy especia y the ion channe s,
Procainamide (PRONESTYL) exchangers, and pumps that are targets o antiarrhythmic drugs.
Propa enone (RYTHMOL) Understand the mechanisms that cause cardiac arrhythmias.
Propranolol (INDERAL; see Chapter 7) Know the common and important tachyarrhythmias and their mechanisms.
Quinidine (QUINIDEX) Understand the mechanisms and c assi cation o antiarrhythmic drugs.
Sotalol (BETAPACE)
Know the princip es o antiarrhythmic drug pharmacotherapy.
Verapamil (CALAN; see
Know the pharmaco ogica , pharmacokinetic, and adverse e ects o speci c
Chapter 16)
antiarrhythmic agents.

THE CARDIAC ACTION


POTENTIAL MECHANISMS OF ACTION OF DRUGS USED TO TREAT CARDIAC ARRHYTHMIAS

In ast-conducting cardiacmyocytes and ANTIARRHYTHMIC PRIMARY MECHANISM


DRUG CLASS DRUG OF ACTION*
cells o the conducting system, Phase 0
depolarization is caused bythe opening o Class IA Quinidine, procainamide, Na+ channel blocker, prolongs
voltage-gated Na+ channels. disopyramide action potential duration (APD)

In slow-conducting cells o the sinoatrial Class IB Lidocaine, mexiletine Na+ channel blocker, rapid
(SA) node and atrioventricular (AV) node, dissociation
which have relatively ewvoltage-gated
Class IC Flecainide, propa enone Na+ channel blocker, slow
Na+ channels, Phase 0 depolarization is dissociation
caused byopening o voltage-gated L-type
Ca2+ channels. Class II Propranolol, sotalol, esmolol Adrenergic blocker
Phase 3 repolarization is primarilycaused Class III Amiodarone, sotalol, ibutilide, Prolongs APD (primarily by K+
byK+ movement out o the cell through do etilide, dronedarone channel blockade)
voltage-gated K+ channels.
Class IV Verapamil, diltiazem Ca2+ channel blocker
Figure 18-1 shows action potentials rom (nondihydropyridine)
di erent regions o the heart, the ion
currents that contribute to these action Miscellaneous Adenosine Adenosine receptor agonist
potentials, and the corresponding e ects
Miscellaneous Digoxin Na+, K+-ATPase inhibitor
o the ion currents on the electrocardio-
gram(ECG). *Note: Most antiarrhythmic drugs have multiple mechanisms o action (Table 18-1).

312
Antiarrhythmic Drugs CHAPTER 1 8

S inus
node Atria
Ve ntricle s

Conducting s ys te m Ve ntricula r
AV node P urkinje fibe r
myocyte
1
S A node Atria AV n o d e 2
Ac tio n 0
po te ntials P ha s e 4 3

Na + curre nt

Ca 2+ L-type
curre nt
T-type
tra ns ie nt ITO1
outwa rd (4-AP -s e ns itive ) (4-AP -s e ns itive ) (4-AP -s e ns itive ) (4-AP -s e ns itive ) (4-AP -s e ns itive )
curre nt ITO2
(Ca 2+-a ctiva te d) (Ca 2+-a ctiva te d) (Ca 2+-a c tiva te d ) (C a 2+-a ctiva te d) (Ca 2+-a ctiva te d)
Io n fluxe s IKs
de laye d
Inward re ctifie rs IKr
(IK)
Outward IKur
ICl
inwa rd re ctifie r, IK1
pa ce ma ke r curre nt, If
Na +-Ca 2+ excha nge
Na +, K+-ATPa s e

QRS
T
P R inte rva l corre s ponds P QT inte rva l corre s ponds to
ECG
to a tria l de pola riza tion/ ve ntricula r de pola riza tion/
re pola riza tion P R QT re pola riza tion

FIGURE 18-1 Action potentials that occur during normal impulse propagation and the time course o the ion currents that generate them.

MECHANISMS OF CARDIAC
TACHYARRHYTHMIAS
Three arrhythmogenicmechanisms are
CASE 18-1 thought tounderlie most tachyarrhythmias:
A 53-year-o d woman visits the emergency department a er osing consciousness Enhanced automaticityresulting

whi e working in her garden. She says she has recent y had episodes o dizziness and romenhanced phase 4 spontaneous
ainting. Her ECG ooks unremarkab e except that the Q interva is pro onged. depolarization (see Figure 29-10,
Goodman &Gilman, 12th Edition)
a. What are the possible causes o her recent ainting spells?
Triggered automaticity, including

T e prolonged Q interval suggests that her ainting spells may be due to torsade
abnormal rhythms associated with
de pointes ( dP) (see Side Bar LONG Q AND ORSADE DE POIN ES).
delayed a terdepolarizations (DADs)
Because her syncope has started airly recently, it is likely that the dP might be
resulting romCa2+ overload, and those
due to a drug that prolongs action potential duration. Drugs that can prolong
associated with earlya terdepolarizations
Q include Class IA and Class III antiarrhythmic drugs (see MECHANISMS OF
(EADs) caused byprolongation o the
AC ION OF DRUGS USED O REA CARDIAC ARRHY HMIAS), as well as
action potential (see Figure 29-6,
many other drugs, including antibiotics (erythromycin, spar oxacin), antipsychot-
Goodman&Gilman, 12th Edition)
ics (chlorpromazine, haloperidol), and antiemetics (domperidone, droperidol).
Another possible cause is hypokalemia, especially i she has been taking diuretics Reentry, which involves the ormation

that cause loss o K+. She might also have a orm o hereditary long Q that has o a sel -perpetuating circuit caused
become mani ested due to a Q -prolonging drug or hypokalemia. It is important to byanisotropicconduction around an
determine the drugs that this patient is currently taking and whether any o them anatomicor unctional barrier (see
have been started recently. It would also be use ul to know whether there is a amily Figures 29-7 and 29-8, Goodman &
history o sudden cardiac death which might indicate she has a hereditary orm o Gilmans The Pharmacological Basis of
long Q syndrome. Therapeutics, 12th Edition)
Understanding the underlying arrhythmo-
(Continued)
genicmechanismcan acilitate the choice
o antiarrhythmicdrugs (see Table 18-1)

313
SECTION III Modulation of Cardiovascular Function

TABLE 18-1 A Mechanistic Approach to Antiarrhythmic Therapy


ARRHYTHMIA COMMON MECHANISM ACUTE THERAPYa CHRONIC THERAPYa
Premature atrial, nodal, or Unknown None indicated None indicated
ventricular depolarizations
Atrial brillation Disorganized unctionalreentry 1. Control ventricular response: 1. Control ventricular response:
AVnode blockb AVnodal blockb
Continual AVnode stimulation and 2. Restore sinus rhythm: DC 2. Maintain normal rhythm: K+ channel
irregular, o ten rapid, ventricular rate cardioversion block, Na+ channel block, Na+ channel
block with recovery >1 second
Atrial utter Stable reentrant circuit in the right Same as atrial brillation Same as atrial brillation
atrium
Ventricular rate o ten rapid and AVnodal blocking drugs especially
irregular desirable to avoid ventricular rate
Ablation in selected casesc
Atrial tachycardia Enhanced automaticity, DAD-related Same as atrial brillation Same as atrial brillation
automaticity, or reentry in atrium Ablation o tachycardia ocusc
AVnodal reentrant Reentrant circuit within or *Adenosine *AVnodal block
tachycardia (PSVT) near AVnode AVnodal block Flecainide
Less commonly: vagal Propa enone
tone (digitalis, edrophonium, *Ablation c
phenylephrine)
Arrhythmias associated with
WPW syndrome:
1. AVreentry (PSVT) Reentry Same as AVnodal reentry K+ channel block
Na+ channel block with recovery >1 second
*Ablation c
2. Atrial brillation with Very rapid rate due to *DC cardioversion K+ channel block
atrioventricular conduction nondecremental properties o *Procainamide Na+ channel block
via accessory pathway accessory pathway with recovery >1 second (AVnodal
blockers can be harm ul)
VT in patients with remote Reentry near the rim o the healed Lidocaine *ICDd
myocardial in arction myocardial in arction Amiodarone Amiodarone
Procainamide K+ channel block
*DC cardioversion Na+ channel block
VT in patients without DADs triggered by sympathetic Adenosine e Verapamile
structural heart disease tone Verapamile Blockerse
Blockerse
DC cardioversion
VF Disorganized reentry *DC cardioversion *ICDd
Lidocaine Amiodarone
*Amiodarone K+ channel block
Procainamide Na+ channel block
Torsades de pointes, EAD-related triggered activity Pacing Blockade
congenital or acquired Magnesium Pacing
(o ten drug related) Isoproterenol
*Indicates treatment o choice.
a
Acute drug therapy is administered intravenously; chronic therapy implies long-term oral use.
b
AVnodal block can be achieved clinically by adenosine, Ca2+ channel block, adrenergic receptor blockade, or increased vagal tone
(a major anti-arrhythmic e ect o digitalis glycosides).
c
Ablation is a procedure in which tissue responsible or the maintenance o a tachycardia is identi ed by specialized recording techniques
and then selectively destroyed, usually by high- requency radio waves delivered through a catheter placed in the heart.
d
ICD, implanted cardioverterde brillator: a device that can sense VT or VF and deliver pacing and/or cardioverting shocks to restore
normal rhythm.
e
These may be harm ul in reentrant VT and so should be used or acute therapy only i the diagnosis is secure. DAD, delayed a terdepolarization;
EAD, early a terdepolarization; WPW, Wol ParkinsonWhite syndrome; PSVT, paroxysmal supraventricular tachycardia; VT, ventricular tachycardia; VF,
ventricular brillation.

314
Antiarrhythmic Drugs CHAPTER 1 8

b. What are your therapeutic options?


LONG QT AND TORSADE
I the ainting is thought to be due to dP, it is important to rst identi y the
DE POINTES
cause o this arrhythmia and remove it, i possible. Drugs are the most likely
cause so it is important to review the patients medications, particularly ones that Aprolongation o the QTinterval on the
she has started taking recently. I she is hypokalemic, this should be corrected ECGis an indication that repolarization o
with supplemental K+. I she is taking a diuretic, her dosing could be reduced or ventricular myocytes is slowed.
she should be changed to a potassium-sparing diuretic. I the patient is suspected Because the outward repolarizing currents
o carrying one o the mutations that results in hereditary long Q syndrome, (the delayed recti er currents) are carriedby
genetic testing that involves sequencing all o these genes may acilitate appropri- variousvoltage-gatedK+ channels, drugsthat
ate choice o therapy. For instance, patients with hereditary long Q whose dP inhibit these channelscanprolongQTinterval.
is brought on by exercise and activation o the sympathetic nervous system may Mutations in genes that regulate these K+
bene t rom -adrenergic blockers. channels can lead to hereditarylong QT
syndrome (LQT).
CASE 18-2 Patients with drug-induced LQT(DILQT) or
hereditaryLQTare at risko developing a
A 47-year-o d man with uncomp icated atria bri ation is prescribed di tiazem. li e-threatening polymorphicventricular
a. What is the mechanistic rationale or using diltiazem to treat atrial tachycardia known as torsade de pointes
brillation? (TdP;twisting o the pointsin French)
which can progress toventricular brillation.
Diltiazem inhibits L-type voltage-gated Ca2+ channels in the SA and AV nodes o
the heart. By inhibiting these channels in the AV node, conduction o impulses TdPis so-named because the QRScom-
through the AV node is slowed or blocked. T is protects the ventricular rate o plexes progressivelyincrease and decrease
depolarization rom being controlled by the aberrantly high rate o depolarization in size giving the illusion o a picket ence
o the atria. that is being twisted around a central axis
(see Figure 29-9 in Goodman &Gilmans
b. What ef ect will diltiazem have on the ECG o this patient? The Pharmacological Basis of Therapeutics,
In a patient with atrial brillation, the ventricular rate can be rapid and irregular. 12th Edition).
T is will result in an ECG in which the RR intervals are relatively short and vari- Treatment o patients who developTdP
able. Diltiazem is expected to slow ventricular rate and reduce irregular ventricular include removing the underlying cause, i
depolarizations which will be seen on the ECG as a prolongation o RR intervals known, and intravenous magnesiumsul-
and an increase in the regularity o RR intervals. In a patient without atrial bril- ate to terminate the arrhythmia.
lation, diltiazem will increase the PR interval (which re ects the time an impulse
takes to travel through the AV node) and increase the RR interval secondary to
slowing the spontaneous depolarization o the SA node.
c. What are the other ef ects and potential risks o using diltiazem?
Because diltiazem blocks L-type Ca2+ channels in the heart and in arterial smooth
muscle cells, the drug has negative inotropic e ects on the heart and lowers blood
pressure because o its vasodilating e ects. In patients with poor ventricular unc-
tion or low blood pressure, this agent could urther reduce cardiac output and
lower mean arterial pressure.
d. I diltiazem is not well tolerated by this patient, what other classes o drugs
might be use ul in treating this patients atrial brillation?
Other classes o drugs that could be used or ventricular rate control include
adrenergic blockers. o treat the arrhythmogenesis o atrial brillation, antiar-
rhythmic drugs that block Na+ channels and/or K+ channels in atrial myocytes
might be use ul (see able 18-1).
e. What other class o drugs are indicated in patients with uncomplicated atrial
brillation?
Patients with atrial brillation are at signi cantly increased risk o stroke and
other thromboembolic events due to stasis o blood in the atria. o reduce the
risk o blood clots, patients with sustained uncomplicated atrial ibrillation
typically take oral anticoagulants such as war arin, dabigatran, rivaroxaban,
or apixaban (see Chapter 19).

315
SECTION III Modulation of Cardiovascular Function

CASE 18-3
A 59-year-o d man sudden y oses consciousness in a shopping ma . A security guard
at the ma cannot detect a pu se and uses an automated externa de bri ator (AED) to
resuscitate the man. He regains consciousness and is taken by ambu ance to a nearby
hospita . In the emergency department waiting room, he oses consciousness again. His
ECG shows that he is in ventricu ar bri ation.
a. What treatment will this patient receive in the ED?
Ventricular brillation is a li e-threatening arrhythmia and must be treated aggres-
sively to restore normal sinus rhythm. T e patient should be de brillated and
should be administered an intravenous antiarrhythmic drug such as amiodarone,
lidocaine, or procainamide to prevent recurrence. O these 3 drugs, IV amiodarone
is the drug o choice or this indication (see able 18-1).
b. What is the mechanistic rationale or the drug(s) used or the acute treatment o
patients in ventricular bbrillation?
Ventricular brillation is characterized by disorganized reentry in the ventricular myo-
cardium. Amiodarone, lidocaine, and procainamide all block the voltage-gated Na+
channel and increase the threshold voltage required to open the channel. Moreover, all
3 agents exhibit state-dependent block o the Na+ channel ( able 18-2), with amioda-
rone binding the Inactive (I) state, procainamide binding the Open (O) state, and lido-
caine binding both the Inactive and Open states (I>O). By blocking the Na+ channel in
the Open and Inactive states, the drugs will pre erentially block rapidly depolarizing
myocytes, leaving those myocytes depolarizing at normal rates relatively una ected.
T us, any reentrant circuits ormed in the ventricles ollowing the electrical de bril-
lation will be selectively inhibited, thereby preventing recurrence o the tachyarrhyth-
mia. In addition to blocking the Na+ channel, amiodarone and procainamide also have
antiarrhythmic e ects through their actions on blocking outward K+ currents.
c. What treatments might be used in the long-term to prevent another episode o
ventricular brillation in this patient?
T e treatment o choice or long-term prevention o ventricular brillation is an
implantable cardioverter-de brillator (ICD; see able 18-1). Such a device can
be programmed to detect ventricular tachyarrhythmias and deliver a shock and/
or pacing to maintain normal sinus rhythm. As an alternative or in addition to an
ICD, drug therapy that includes a Na+ and/or K+ channel blocker could be used.
Amiodarone is a drug o choice or chronic therapy because it has both Na+ and K+
channel blocking activities and has minimal negative inotropic e ects.

CASE 18-4
A 38-year-o d man comp ains o not having the energy to participate in strenuous
sports that are part o his active i esty e. He had been a competitive amateur cyc ist but
now has di cu ty training. An ECG indicates he has atria bri ation.
a. What are the risks and bene ts o starting this patient on antiarrhythmic drug
therapy?
T e risks o therapy include proarrhythmias as well as extracardiac side e ects
associated with long-term drug therapy. Pharmacological therapy that alleviates the
patients atrial brillation may improve quality o li e and reduce the risk o stroke,
but may not reduce mortality.
b. What are the treatment options?
Because o the patients age and good health, nonpharmacological therapies should
be considered including ablation and DC cardioversion (see able 18-1). T ese
have the potential to provide permanent or long-term termination o the arrhyth-
mia. Antiarrhythmic drugs carry signi cant risks, and it might be sa er to avoid any
drug therapy, except or anticoagulant therapy, to reduce the risk o stroke.

316
Antiarrhythmic Drugs CHAPTER 1 8

TABLE 18-2 Major Electrophysiologic Actions of Anitarrhythmic Drugs


Na + CHANNEL BLOCK
STATE
DRUG RECOVERY
1
, SECONDS DEPENDENCE1 APD Ca 2+ CHANNEL BLOCK AUTONOMIC EFFECTS OTHER EFFECTS
Lidocaine 0.1 I> O

Phenytoin 0.2 I

Mexiletine a 0.3

Procainamide 1.8 O Ganglionic blockade : Metabolite prolongs


(especially intravenous) APD

Quinidine 3 O (x) Blockade, vagolytic


Anticholinergic

Disopyramide b 9 O Anticholinergic

Propa enone b 11 O I Blockade (variable


clinical e ect)

Flecainide a 11 O (x) (x)

Blockers: Blockade Na+ channel block in vitro


Propanololb

Sotalolb Blockade

Amiodarone, 1.6 I (x) Noncompetitive Antithyroid action


Dronedarone blockade

Do etilide

Ibutilide

Verapamila

Diltiazem a

Digoxin : Vagal stimulation : Inhibition o Na+,


K+-ATPase

Adenosine :Adenosine receptor : Activation o outward


Activation K current
+

Magnesium ? Mechanism not well


understood

Indicates an e ect that is important in mediating the clinical action o a drug. (x)Indicates a demonstrable e ect whose relationship to drug action
in patients is less well established.
a
Indicates drugs prescribed as racemates, and the enantiomers are thought to exert similar electrophysiologic e ects.
b
Indicates racemates or which clinically relevant di erences in the electrophysiologic properties o individual enantiomers have been reported
(see text). One approach to classi ying drugs is: Class Major action
I Na+ channel block
II blockade
III action potential prolongation (usually by K+ channel block)
IV Ca2+ channel block
Drugs are listed here according to this scheme. It is important to bear in mind, however, that many drugs exert multiple e ects that contribute
to their clinical actions. It is occasionally clinically use ul to subclassi y Na+ channel blockers by their rates o recovery rom drug-induced
block (recovery) under physiologic conditions. Because this is a continuous variable and can be modulated by actors such as depolarization o
the resting potential, these distinctions can become blurred: class Ib, recovery <1 s; class Ia, recovery 110 s; class Ic, recovery >10 s. These class and
subclass e ects are associated with distinctive ECG changes, characteristic classtoxicities, and ef cacy in speci c arrhythmia syndromes.
1
These data are dependent on experimental conditions, including species and temperature. O, open-state blocker; I, inactivated-state blocker;
APD, action potential duration.

317
SECTION III Modulation of Cardiovascular Function

CASE 18-5
A 12-year-o d gir is brought to the ED with intermittent symptoms o dizziness and
syncope. Her ECG shows that she has paroxysma supraventricu ar tachycardia (PSV ).
a. What is the most likely arrhythmogenic mechanism causing the PSV ?
T e most common mechanism causing PSV is a reentrant circuit within or near
the AV node (see Figures 29-7 and 29-8, in Goodman & Gilmans T e Pharmacologi-
cal Basis of T erapeutics, 12th Edition).
b. What is the drug o choice to rapidly terminate the PSV ?
Adenosine (IV) is the drug o choice to rapidly terminate PSV because o its abil-
ity to inhibit conduction through the AV node (see able 18-1). It acts through
purinergic receptors to hyperpolarize AV nodal cells and has a very short hal -li e
(<10 seconds). Other drugs that block AV nodal conduction include Ca2+ chan-
nel blockers (diltiazem and verapamil) and adrenergic blockers, but adenosine
is the drug o rst choice because o its short hal -li e. Drugs that increase vagal
tone (digitalis, edrophonium, phenylephrine) are less commonly used AV nodal
blockers.
c. What can be done to prevent recurrence o PSV in this patient?
Oral AV nodal blocking agents are the drugs o choice to prevent recurrence o
PSV s (see able 18-1). Flecainide and propa enone are also use ul drugs because
o their ability to alter the electrophysiological properties o the ast-conducting
tissue in the reentrant circuit. Selective ablation o tissue that is critical to the main-
tenance o the reentrant circuit (eg, an accessory pathway) is a treatment o choice
because it can result in a permanent cure.

CASE 18-6
A 62-year-o d man who had a myocardia in arction 3 years ago now comp ains o epi-
sodes o ight-headedness. His cardio ogist determines that his symptoms are the resu t
o nonsustained ventricu ar tachycardia (V ), and that he has deve oped moderate
systo ic heart ai ure.
a. What is the likely cause o this patients ventricular tachycardia?
T is patients V is likely due to a reentrant circuit near the site o the healed
in arction (see able 18-1).
b. What are the treatment options or this patient?
T e treatment o choice or this patient is an ICD that can sense when the patient
has V or ventricular brillation and can pace or cardiovert the heart into normal
sinus rhythm (see able 18-1). Alternatively or in addition to the ICD, this patient
might receive an antiarrhythmic drug. Because this patient has heart ailure, anti-
arrhythmic drugs that decrease lef ventricular unction (eg, disopyramide and
ecainide) should be avoided. Drugs that would be appropriate are amiodarone,
dronedarone, and K+ channel blockers such as do etilide.
c. T e patients cardiologist prescribes amiodarone. What are the toxicities that
might be seen with long-term therapy?
As with most antiarrhythmic agents, there are risks o long-term therapy with
amiodarone that must be considered and mitigated, i possible. Although amio-
darone is e ective in treating a wide range o arrhythmias and is one o the ew
antiarrhythmic agents that has been shown to reduce mortality in clinical trials, it
is associated with a large number o extracardiac toxicities. Serious toxicities (some
o which are li e threatening) include pulmonary toxicity, hepatotoxicity, e ects on
thyroid unction, neuromuscular symptoms, dermatological reactions, and e ects
on vision. T e risk o these toxicities is increased because the mean elimination
hal -li e o amiodarone is more than 50 days.

318
Antiarrhythmic Drugs CHAPTER 1 8

KEY CONCEPTS PRINCIPLES IN THE


CLINICAL USE OF
Antiarrhythmic drugs act by a tering cardiac ion f uxes, direct y or indirect y.
ANTIARRHYTHMIC DRUGS
Most antiarrhythmic drugs have mu tip e mechanisms o action.
Antiarrhythmicdrugs can be veryef ca-
Antiarrhythmic drugs can cause arrhythmias and can have other i e-threaten- cious in terminating or preventing cardiac
ing side e ects. arrhythmias, but o ten carrysigni cant
In choosing an antiarrhythmic drug therapy, the risks o therapy, as we as risks, even when used at therapeuticcon-
possib e bene ts, shou d be considered; patients who are asymptomatic may centrations. The undamental principles o
derive itt e bene t rom taking a drug that carries signi cant risks. antiarrhythmictherapyshould be applied
to optimize bene ts and minimize risks:
1. Identi yand remove precipitating actors.
SUMMARY QUIZ Electrolyte disturbances (especially
hypokalemia)
QUESTION 18-1 An 84-year-o d woman being treated or hypertension, mi d heart
Myocardial ischemia
ai ure (NYHA c ass II), and persistent atria bri ation is taking severa di erent
medications: aspirin, sota o , di tiazem, metopro o , captopri , ch orothiazide, and war- Drugs that can cause arrhythmias,
arin. Over the past severa weeks, she has begun to comp ain that she is no onger ab e including manythat prolong QTinterval
to take her dog or ong wa ks because she tires easi y. She a so comp ains o shortness 2. Establish the goals o treatment.
o breath. T e drug-drug interactions most ike y contributing to the patients recent Some arrhythmias should not be
symptoms are treated, as illustrated bythe CAST
a. sota o and di tiazem. clinical trial.
b. di tiazem and metopro o . Some patients with arrhythmias maybe
c. sota o and metopro o . asymptomaticand it maybe dif cult to
establish the bene t o antiarrhythmic
d. a o the above.
drug therapy.
QUESTION 18-2 A 68-year-o d man who had a myocardia in arction a year ear ier Choice o therapyshould be based on
shows premature ventricu ar beats on his ECG during a routine physica examination. the goals o treatment.
He has not had any symptoms re ated to this arrhythmia such as dizziness or syncope. 3. Minimize risks.
What is the best drug therapy or this patient to treat his arrhythmia? Acommon risko antiarrhythmic
a. No drug therapy drugs is proarrhythmias (see
b. An AV noda b ocker Table 29-1 in Goodman &Gilmans The
c. A Na+ channe b ocker Pharmacological Basis of Therapeutics,
12th Edition).
d. A K+ channe b ocker
Monitoring o drug concentrations in
e. A Ca2+ b ocker
the plasma mayminimize some adverse
QUESTION 18-3 A 72-year-o d patient with heart ai ure and atria bri ation is e ects.
receiving digoxin. T e mechanistic rationa e or using digoxin to treat the patients Some antiarrhythmicdrugs maybe
arrhythmia is that contraindicated in speci cpatients (see
a. digoxin inhibits Ca2+ channe s in the AV node. Table 29-4 in Goodman &Gilmans The
Pharmacological Basis of Therapeutics,
b. digoxin inhibits atria depo arizations by its e ects on Na+ channe s.
12th Edition), including manypatients
c. digoxin increases vaga tone. who are at higher risko arrhythmias.
d. digoxin s ows repo arization in cardiac myocytes. 4. The electrophysiologyo the heart is
aMovingTargetand can change in
QUESTION 18-4 What is the rationa e or the use o idocaine in treating a patient with
response to external actors.
ventricu ar tachycardia?
Changes in autonomictone
a. It b ocks Ca2+ channe s in ventricu ar myocytes.
Myocardial ischemia
b. It b ocks Na+ channe s in ventricu ar myocytes.
Myocardial stretch as can occur in heart
c. It b ocks K+ channe s in ventricu ar myocytes.
ailure
d. It b ocks adrenergic receptors in ventricu ar myocytes. Serumelectrolyte disturbances
e. It activates purinergic receptors in the AV node.

319
SECTION III Modulation of Cardiovascular Function

QUESTION 18-5 What is the mechanism o action o sota o in preventing ventricu ar


and atria arrhythmias?
a. It b ocks Ca2+ channe s in ventricu ar myocytes.
b. It b ocks Na+ channe s in ventricu ar myocytes.
c. It b ocks K+ channe s in ventricu ar myocytes.
d. It b ocks adrenergic receptors in ventricu ar myocytes.
e. It b ocks K+ channe s and adrenergic receptors in ventricu ar myocytes.

SUMMARY QUIZ ANSWER KEY


QUESTION 18-1 Answer is d. A o the indicated drug combinations wou d be
expected to have additive e ects in s owing heart rate and depressing e ventricu ar
unction. Because this person has mi d heart ai ure, drugs that depress e ventricu ar
unction can exacerbate her heart ai ure causing or worsening symptoms o atigue
and shortness o breath.
QUESTION 18-2 Answer is a. Because this patient has a benign arrhythmia and is
asymptomatic, no drug therapies are recommended because the risks o therapy out-
weigh any bene ts. A though a Na+ channe b ocker such as f ecainide might be very
e cacious in preventing PVCs in this patient, the CAS study showed a signi cant
increase in morta ity in patients taking this and other C ass IC drugs who have had a
recent MI and who have a reduced ejection raction.
QUESTION 18-3 Answer is c. Digoxin inhibits the Na+,K+-A Pase in cardiac musc e
and in other excitab e tissue, inc uding neurons. It has e ects on the centra nervous
system that resu t in an increase in vaga tone, which inhibits conduction o impu ses
through the AV node. By inhibiting the AV node, the ventric es are protected rom the
high rate o impu ses generated in the atria during atria bri ation.
QUESTION 18-4 Answer is b. By b ocking vo tage-gated Na+ channe s in ventricu-
ar myocytes, the myocytes are ess ike y to depo arize and conduction o impu ses
through the ventricu ar myocardium is s owed.
QUESTION 18-5 Answer is e. Sota o is a racemic mixture that has both adrenergic
b ocking and K+ channe b ocking activities. Both enantiomers have comparab e K+
channe b ocking activity, but the l -enantiomer contributes most o the adrenergic
b ocking activity.

SUMMARY: ANTIARRHYTHMIC DRUGS


TOXICITIES
UNIQUE; CLINICALLY
CLASS AND SUBCLASSES NAMES CLINICAL USES COMMON IMPORTANT
Class IA Antiarrhythmics Procainamide Acute and chronic treatment o 40% o patients Lupus-like syndrome (25-
Sodium Channel Blockers ventricular tachycardia discontinue within 6 50% with chronic) that can
(Prolong QT Interval; Slow Acute and chronic treatment o months o therapy due to be atal; starts with rash
to Intermediate Kinetics) ventricular brillation (VF) side e ects: hypotension, and mild arthralgia
Acute and chronic treatment o nausea
supraventricular arrhythmias Torsade de pointes (TdP)

Quinidine Chronic treatment o atrial Diarrhea (30-50% o Marked long QT (2-8% o


utter/ brillation (AF) patients); diarrhea- patients) with increased
Chronic treatment o ventricular induced hypokalemia may risk o TdP at therapeutic
tachycardia and VF potentiate TdP or subtherapeutic
Thrombocytopenia concentrations
Cinchonism: CNS,
hypotension, syncope, GI

320
Antiarrhythmic Drugs CHAPTER 1 8

TOXICITIES
UNIQUE; CLINICALLY
CLASS AND SUBCLASSES NAMES CLINICAL USES COMMON IMPORTANT
Disopyramide Chronic treatment o atrial Anticholinergic e ects Can worsen heart ailure
utter/AF Long QT (TdP)
Chronic treatment o ventricular
tachycardia and VF

Class IB Antiarrhythmics Lidocaine Chronic treatment o ventricular CNS: seizures and tinnitus CNS: tremor, hallucinations,
Sodium Channel Blockers tachycardia and VF drowsiness, coma
(Little E ect on ECG; Fast Severe interactions with
Kinetics) other antiarrhythmics

Mexiletine Chronic treatment o ventricular Tremor and nausea Severe interactions with
tachycardia and VF other antiarrhythmics

Class IC Antiarrhythmics Flecainide Chronic treatment o AV Blurred vision Li e-threatening CAST


Sodium Channel Blockers nodal reentry in patients w/o Can worsen heart ailure proarrhythmia in patients
(Prolong PR and Broaden structural heart disease with MI
QRS Intervals; Slow Chronic treatment o li e-
Kinetics) threatening ventricular
arrhythmias

Propa enone Chronic treatment o AVnodal Adrenergic blocking Can increase requency
reentry (PSVTs) and AF e ects (worsening o heart or severity o ventricular
Chronic treatment o ventricular ailure and bronchospasm) tachycardias
arrhythmias (modest ef cacy)

Class II Antiarrhythmics Propranolol (and Chronic prevention o TdP Adrenergic blocking See Chapter 7
Adrenergic Receptor others, see Chapter 7) Rate control in AF e ects (worsening o heart
Antagonists ailure and bronchospasm;
see Chapter 7)

Sotalol Chronic treatment o ventricular Adrenergic blocking Prolongs QT interval


tachycardia e ects increasing risk o TdP
Chronic treatment o atrial
utter/AF

Esmolol Rapid, short-term control o Adrenergic blocking Because o short hal -li e (9
ventricular rate in patients e ects (dissipate rapidly min), should only be used
with AF or atrial utter in when drug is removed) or short-term use
perioperative and post-
operative settings

Class III Antiarrhythmics Amiodarone Drug o choice or acute Hypotension and Pulmonary brosis with
Increase Re ractory Period treatment o VF (IV depressed ventricular chronic therapy which can
(Prolong QT) administration) unction with IV be atal (requires periodic
Acute and chronic treatment o monitoring o lung
ventricular tachycardia unction)
Chronic treatment o AF and VF Many other AEs:
corneal microdeposits,
hepatotoxicity,
neuropathies
photosensitivity, thyroid
dys unction
Note: Tissue half-life of
several months

Dronedarone Chronic treatment o atrial GI disturbances Increases mortality in


utter/AF Less than amiodarone patients with severe heart
ailure

Sotalol Chronic treatment o ventricular adrenergic blocking Prolongs QT interval


tachycardia e ects increasing risk o TdP
Chronic treatment o atrial
utter/AF

321
SECTION III Modulation of Cardiovascular Function

TOXICITIES
UNIQUE; CLINICALLY
CLASS AND SUBCLASSES NAMES CLINICAL USES COMMON IMPORTANT
Do etilide Chronic treatment o AF Prolongs QT interval
increasing risk o TdP

Ibutilide Acute treatment o AF Prolongs QT interval


increasing risk o TdP

Class IVAntiarrhythmics Diltiazem Acute and chronic treatment o Hypotension Sinus bradycardia or AV
Ca2+ Channel Blockers AVnodal reentry (PSVTs) block in combination with
(Nondihydropyridine; Acute and chronic control blockers
Inhibit SA and AVNodes; o ventricular rate in atrial
Prolong PR) utter/AF

Verapamil Acute and chronic treatment o Constipation Sinus bradycardia or AV


AVnodal reentry (PSVTs) block in combination with
Acute and chronic control blockers
o ventricular rate in atrial Worsening o heart ailure
utter/AF

Miscellaneous Adenosine Drug o choice or acute Asystole (<5 seconds) Short hal -li e (<5 seconds)
Antiarrhythmics treatment o AVnodal reentry Dyspnea minimizes toxicities
Adenosine Receptor (IVadministration)
Agonist

Miscellaneous Magnesium sul ate Acute treatment o TdP


Antiarrhythmics
Unknown Mechanism

Miscellaneous Digoxin Ventricular rate control in atrial GI, vision, cognitive Arrhythmias
AntiarrhythmicsNa+, brillation dys unction Sinus bradycardia and
K+-ATPase Inhibitor That Modest positive inotrope (see AVblock
Increases Vagal Tone Chapter 17) Severe toxicities can be
treated with antibody
(DIGIBIND)

322
CHAPTER

Drug Therapyof Thromboembolic


Disorders 19
T is chapter will be most use ul a er having a basic understanding o the material in
DRUGS INCLUDED
Chapter 30, Blood Coagulation and Anticoagulant, Fibrinolytic, and Antiplatelet Drugs
in Goodman & Gilmans T e Pharmacological Basis of T erapeutics, 12th Edition. In IN THIS CHAPTER
particular, the reader is directed to the ollowing table and to animations available in Abciximab (REOPRO)
the online version o Goodman & Gilmans T e Pharmacological Basis of T erapeutics: Apixaban (ELIQUIS)
able 30-3 Absolute and Relative Contraindications to Fibrinolytic T erapy. Argatroban (ARGATROBAN)
Several animations in the online version o Goodman & Gilmans T e Pharmacologi- Aspirin (acetylsalicylicacid, ASA)
cal Basis of T erapeutics illustrate the mechanism o action o many anticoagulant,
Bivalirudin (ANGIOMAX)
brinolytic, and antiplatelet drugs.
Clopidogrel (PLAVIX)
LEARNING OBJECTIVES Dabigatran (PRADAXA)
Know the classes o drugs that inhibit platelet unction, their mechanisms o Drotrecogin al a (XIGRIS)
action, and their role in the prevention and treatment o acute coronary syn- Epti batide (INTEGRILIN)
dromes, strokes, and transient ischemic attacks.
Fondaparinux(ARIXTRA)
Know the classes o drugs used as anticoagulants, their mechanisms o action, Heparin
their indications in preventing and treating venous thromboembolism, pulmo-
nary embolism, and strokes. Low-Molecular-Weight Heparins (LMWHs;
enoxaparin [LOVENOX], dalteparin [FRAG-
Know how anticoagulant drug therapy is monitored. MIN], tinzaparin [INNOHEP], ardeparin
Know the mechanism o action o brinolytic agents, and their indications in [NORMIFLO], nadroparin [FRAXIPARIN])
treating ischemic stroke and myocardial in arction. Prasugrel (EFFIENT)
Know the major toxicities and contraindications o each class o drug. Rivaroxaban (XARELTO)
Ticlopidine (TICLID)
MECHANISMS OF ACTION OF DRUGS COMMMONLY USED TO PREVENT OR TREAT Tiro ban (AGGRASTAT)
THROMBOEMBOLISM Tissue plasminogen activator (t-PA;
DRUG CLASS DRUG MECHANISM OF ACTION alteplase [ACTIVASE], reteplase
Antiplatelet Agent Aspirin Irreversibly inhibits platelet
[RETEVASE], tenecteplase [TNKASE])
COX-1 War arin (COUMADIN)
Thienopyridines: ticlopidine, Irreversibly inhibits platelet
clopidogrel, prasugrel ADP receptors (P2Y12)

Abciximab, epti batide, tiro ban Inhibits activated GPIIb/IIIa on


platelets

Parenteral Heparin, low-molecular-weight Inhibits thrombin or actor Xa


Anticoagulant heparins (LMWHs), ondaparinux by activating antithrombin

Bivalirudin, argatroban Directly inhibits thrombin

Activated protein C (drotrecogin al a) Degrades actors Va and VIIa

Oral Anticoagulant War arin (and other coumarins) Vitamin Kantagonist


(inhibits synthesis o vitamin
K-dependent clotting actors)

Dabigatran Directly inhibits thrombin

Rivaroxaban, apixaban Directly inhibits actor Xa

Fibrinolytic Agent t-PA (alteplase) and its derivatives Activates plasminogen


(reteplase, tenecteplase)

323
SECTION III Modulation of Cardiovascular Function

COMMON INDICATIONS CASE 19-1


FOR THE USE OF A 62-year-old man with a history o angina calls 911 complaining o crushing chest
ANTIPLATELET DRUGS pain. T e 911 operator sends an ambulance and instructs the patient to chew an aspirin
Prevention o platelet thrombi a ter while waiting or the ambulance to arrive.
coronaryangioplasty a. Why did the 911 operator have the patient take an aspirin?
Secondaryprevention o myocardial T e patients history o angina suggests he has coronary artery disease and an
in arction, transient ischemicattacks elevated risk o an MI, and his symptoms o crushing chest pain indicate he may be
(TIAs), and stroke having a myocardial in arction. In a patient with long-standing coronary artery dis-
Primaryprevention o myocardial in arc- ease, coronary vessel wall damage resulting rom a ruptured atherosclerotic plaque
tion, TIAs, and stroke in high-riskpatients can expose collagen, vWF, and other subendothelial prothrombotic substances to
the blood (see Figures 19-1 and 19-2); these subendothelial substances will bind and
activate circulating platelets. T e activated platelets will release a number o media-
tors, including thromboxane A2 ( XA2), which will activate other platelets in the
blood vessel and stimulate the platelets to aggregate at the site o the vessel wall dam-
age. Aspirin can inhibit platelet production o XA2 by irreversibly inhibiting plate-
let COX-1. By chewing an aspirin to increase absorption in the GI tract, the patient
will rapidly inhibit platelet activation and aggregation, and slow the development o
a platelet thrombus that may lead to the occlusion o a major coronary vessel.
b. When the patient arrives at the hospital, he is diagnosed as having a myocardial
in arction and undergoes percutaneous angioplasty to clear the clot in his
coronary artery. He also receives a stent to keep the vessel patent. What drugs
will this patient likely receive while in the hospital to prevent recurrence o
a thrombus in the stented coronary vessel?
T e patient will receive aggressive antiplatelet and anticoagulant therapy in the
hospital to reduce the likelihood o platelet and brin clots in the stented coronary
vessel. o prevent platelet thrombi, he would continue to receive aspirin as well
as more e cacious antiplatelet inhibitors, including an ADP receptor antagonist
(ie, clopidogrel or prasugrel) and possibly a GPIIb/IIIa inhibitor (ie, abciximab,
(Continued)

Endo the lial c e lls

Plate le ts
+
COX-1
TxA2
GP IIb/IIIa
IIa
P 2Y1 /P 2Y12
PAR1/PAR4 COX-1 Fibrinoge n
GP IIb/IIIa P GI2

ADP

GP Ia /IIa GP Ib
Co llag e n vWF

S mo o th mus cle c e lls /mac ro phag es

FIGURE 19-1 Platelet adhesion and aggregation. GPIa/IIa and GPIb are platelet receptors that bind to collagen and
von Willebrand actor (vWF), causing platelets to adhere to the subendothelium o a damaged blood vessel. PAR1 and
PAR4 are protease-activated receptors that respond to thrombin (IIa); P2Y1 and P2Y12 are receptors or ADP; when stim-
ulated by agonists, these receptors activate the brinogen-binding protein GPIIb/IIIa and cyclooxygenase-1 (COX-1) to
promote platelet aggregation and secretion. Thromboxane A2 (TxA2) is the major product o COX-1 involved in platelet
activation. Prostaglandin I2 (prostacyclin, PGI2), synthesized by endothelial cells, inhibits platelet activation.

324
Drug Therapy of Thromboembolic Disorders CHAPTER 1 9

P la que dis ruption BIOACTIVATION OF


THIENOPYRIDINES
(TICLOPIDINE,
Tis s ue fa ctor Colla ge n vWF CLOPIDOGREL, AND
PRASUGREL)
All thienopyridines are prodrugs
that require bioactivation byCYPs
P la te le t a dhe s ion and/or esterases
a nd s e cre tion
There is wide interindividual variabilityin
antiplatelet e ects o clopidogrel because
As pirin XCOX-1 the prodrug is primarilyactivated by
CYP2C19 which has a loss-o - unction
TXA2 ADP
polymorphism(CYP2C19*2) and sev-
Ticlopidine eral reduced- unction polymorphisms
X Clopidrogre l (CYP2C19*3, *4, *5)
Pra s ugre l
Ca ngre lor Prasugrel has rapid and nearlycomplete
Thrombin X P la te le t re cruitme nt
a nd a ctiva tion
Tica gre lor
absorption and activation compared with
only15%activation o absorbed clopidogrel
S CH530348 Prasugrel is more e cacious and predictable
E5555 than clopidogrel but is also associated
GP IIb/IIIa a ctiva tion with higher rates o atal and li e-
Abcixima b
X Eptifiba tide threatening bleeding
Tirofiba n
P la te le t a ggre ga tion

FIGURE 19-2 Sites o action o antiplatelet drugs. Aspirin inhibits thromboxane A2 (TxA2)
synthesis by irreversibly acetylating cyclooxygenase-1 (COX-1). Reduced TxA2 release attenuates
platelet activation and recruitment to the site o vascular injury. Ticlopidine, clopidogrel, and COMMON INDICATIONS
prasugrel irreversibly block P2Y12, a key ADP receptor on the platelet sur ace; cangrelor and FOR THE USE OF
ticagrelor are reversible inhibitors o P2Y12. Abciximab, epti batide, and tiro ban inhibit the nal ANTICOAGULANTS
common pathway o platelet aggregation by blocking brinogen and von Willebrand actor
(vWF) rom binding to activated glycoprotein (GP) IIb/IIIa. SCH530348 (now known as vorapaxar Treatment o venous thrombosis and
[ZANTIVITY]) and E5555 (also known as atopaxar) inhibit thrombin-mediated platelet activation pulmonaryembolism
by targeting protease-activated receptor-1 (PAR-1), the major thrombin receptor on platelets.
Prevention o thromboembolismin
epti batide, or tiro ban). o inhibit the ormation o brin clots, the patient would patients with unstable angina or acute
receive heparin or an LMWH. He would also be started on an oral anticoagulant myocardial in arction
(ie, war arin) be ore discharge but kept on the heparinoid until his INR (see Side Prevention o thromboembolismin
Bar LABORA ORY MONI ORING OF AN ICOAGULAN HERAPY) is patients with atrial brillation
within the therapeutic range. Prevention o postoperative deep vein
c. What drugs is this patient likely to receive a er being discharged rom the hospital? thrombosis in patients undergoing
abdominothoracicsurgery, and knee and
o prevent reocclusion o the stented coronary artery, he will need to take low-dose
hip replacements
aspirin, an ADP receptor antagonist (clopidogrel or prasugrel), and war arin. He will
also be started on a adrenergic antagonist to reduce his risk o heart ailure and Cardiopulmonarybypass surgerytoprevent
ventricular arrhythmias. clotting o the oxygenator

CASE 19-2
A 78-year-old man receives a total knee replacement. He lives by himsel about an hours
drive rom the nearest medical acility. Upon discharge, he is given a prescription
or lovenox.
a. What is the rationale or the use o this agent?
Patients having major orthopedic surgeries such as total knee replacement or hip
replacement are at risk o brin clot ormation because o the surgery which acti-
vates the clotting cascade (see Figure 19-3) and subsequent immobilization which
(Continued)
325
SECTION III Modulation of Cardiovascular Function

promotes blood stasis. o minimize the risk o brin clot ormation that could lead
to pulmonary embolism or stroke, such patients receive an LMWH such as lovenox.
Because this agent has very predictable pharmacokinetics, it does not require labo-
ratory monitoring, and it can be sel -administered by subcutaneous injection, and
is thus well suited or this patient who lives a considerable distance rom the nearest
medical acility.
b. What is the mechanism o action o lovenox?
Lovenox is one o several LMWH agents and has the same mechanism o action as
heparin and other heparinoids such as ondaparinux. Heparin and all o the hepari-
noids contain a pentasaccharide structure (see Figure 19-4) that binds and activates
antithrombin, an endogenous anticoagulant (see Figure 30-5 in Goodman & Gilmans
T e Pharmacological Basis of T erapeutics, 12th Edition). Once activated, antithrom-
bin inhibits the clotting actors thrombin ( actor IIa) and actor Xa (see Figure 19-3).
Inhibition o thrombin blocks the conversion o brinogen to insoluble brin, whereas
inhibition o actor Xa inhibits conversion o the inactive prothrombin ( actor II)
zymogen to active thrombin ( actor IIa). Because o the smaller size o the LMWHs
compared to standard heparin, the LMWH-antithrombin complexes are more speci c
inhibitors o actor Xa than thrombin (see Figure 30-5 in Goodman & Gilmans T e
Pharmacological Basis of T erapeutics, 12th Edition).
c. What are the important drug toxicities that this patient should be cautioned to
watch or?
As with all anticoagulants, bleeding is the most important and common toxicity.
Heparin-induced thrombocytopenia (HI ) is also possible with the LMWHs, but
this adverse e ect is much less common than with standard heparin.

CASE 19-3
A 32-year-old woman in good health is discovered to have atrial brillation during a
routine checkup. She does not recall having any symptoms o arrhythmia such as dizzi-
ness or ainting, although she is not able to exercise or as long as she could a ew years
ago. Her cardiologist prescribes war arin.
(Continued)

Endo the lial c e lls

Plate le ts

X II
VIIIa
X IX IXa Xa Va Fibrino g e n

IIa
VIIa
TF
TF Fibrin TF
TF
TF TF
TF
TF

S mo o th mus cle c e lls /mac ro phag es

FIGURE 19-3 Major reactions o blood coagulation. Shown are interactions among proteins o the extrinsic(tissue actor
and actor VII), intrinsic( actors IX and VIII), and common( actors X, V, and II) coagulation pathways that are important
in vivo. Boxes enclose the coagulation actor zymogens (indicated by Roman numerals); the rounded boxes represent the
active proteases. TF, tissue actor. Activated coagulation actors are ollowed by the letter a: II, prothrombin; IIa, thrombin.
326
Drug Therapy of Thromboembolic Disorders CHAPTER 1 9

(or H)

H2 COS O 3 COO H2 COS O 3 H2 COS O 3
6
5
O O O O O
COO
O 4
OH 1
OH O OS O 3 OH O OH
3 2 O O
O

NHCOCH3 OH NHS O 3 OS O 3 NHS O 3
(or S O 3 )

N-a ce tyl Glucuronic N-s ulfa te d Iduronic N-s ulfa te d


glucos a mine a cid glucos a mine a cid glucos a mine
6-O-s ulfa te 3,6-O-dis ulfa te 2-O-s ulfa te 6-O-s ulfa te

FIGURE 19-4 The antithrombin-binding pentasaccharide structure o heparin. Sul ate groups required or binding
to antithrombin are indicated in blue.

a. What is the rationale or using war arin in this patient?


Patients with atrial brillation are at high risk o developing brin clots that can
lead to transient ischemic attacks ( IAs), strokes, and pulmonary embolism.
Because the atria do not contract properly in patients with atrial brillation, blood
can pool in the atria increasing the chance a brin thrombus will orm. War arin is
an oral anticoagulant that can reduce the risk o brin clot ormation.
b. What is the mechanism o action o war arin?
War arin is classi ed as a vitamin K antagonist. It blocks the enzyme that converts
the oxidized orm o vitamin K to the reduced orm (see Figure 19-5). T e reduced
orm o vitamin K is a co actor required in the post-translational modi cation o clot-
ting actors II, VII, IX, and X. T is post-translational modi cation, -carboxylation

(Continued)
Non-functiona l Functiona l
prozymoge ns zygome ns

-gluta myl
ca rboxyla s e

OH O
CH3 CH3
oxidize d vita min K Vitamin K oxidize d vita min K O
cycle
R R
OH O

(VKORC1)
NAD vita min K NADH
re ducta s e
X
CPYs 1A1,
1A2, 3A4 R-wa rfa rin S -wa rfa rin CYP 2C9

6-OH, 8-OH a nd 7-OH-wa rfa rin


10-OH wa rfa rin
wa rfa rin

FIGURE 19-5 The vitamin Kcycle and mechanism o action o war arin. In the racemic mixture o S- and R-enantiomers, S-war arin is more
active. By blocking vitamin Kepoxide reductase encoded by the VKORC1 gene, war arin inhibits the conversion o oxidized vitamin Kepoxide
into its reduced orm, vitamin Khydroquinone. This inhibits vitamin K-dependent -carboxylation o actors II, VII, IX, and Xbecause reduced
vitamin Kserves as a co actor or a -glutamyl carboxylase that catalyzes the -carboxylation process, thereby converting prozymogens to
zymogens capable o binding Ca2+ and interacting with anionic phospholipid sur aces. S-war arin is metabolized by CYP2C9. Common genetic
polymorphisms in this enzyme can in uence war arin metabolism. Polymorphisms in the C1 subunit o vitamin Kreductase (VKORC1) also can
a ect the susceptibility o the enzyme to war arin-induced inhibition, thereby in uencing war arin dosage requirements.
327
SECTION III Modulation of Cardiovascular Function

o speci c glutamic acids, is required or biological activity o these clotting actors.


T us, war arin blocks the nal step in the synthesis o these key coagulation actors.
T erapeutic doses o war arin reduce by 30 to 50% the total amount o each vitamin
K-dependent clotting actor, and reduce the level o -carboxylation (and biological
activity) by 10 to 40%.
c. What kinds o laboratory monitoring are required or patients receiving war arin
and why?
Because many actors can alter the anticoagulant activity o war arin (see Side Bar
FAC ORS LEADING O AL ERED AN ICOAGULAN AC IVI Y OF WAR-
FARIN), it is necessary to regularly monitor anticoagulant status using the labora-
tory test called the prothrombin time (P ) assay. T e P is normalized based on the
activity o one o the assay reagents to give a value known as the INR (international
normalized ratio). Dose adjustments o war arin are used to maintain a patients INR
within a narrow range o values. For most indications, the target INR is 2 to 3, but a
higher target INR (2.5-3.5) is indicated in patients with high-risk mechanical heart
valves. INR values that are too high indicate the patient is receiving too much war-
arin and is at risk o major bleeding events. INR values that are below the desired
range indicate the war arin dosing is too low and the patient is at greater risk o
stroke and pulmonary emboli. Patients who are initiated on war arin therapy require
laboratory monitoring every ew days, but once a stable therapeutic INR is achieved,
the requency o monitoring can be reduced to once every ew weeks.
d. What are the risks and toxicities associated with war arin therapy?
As with all anticoagulant agents, the most common risk is bleeding. T e risk o major
bleeding is less than 3% in patients who are maintained within a therapeutic target INR
o 2 to 3. T e risk o intracranial bleeding increases markedly with INR more than 4.
In patients whose INR is more than or equal to 5, vitamin K1 administration may be
required to restore clotting unction. War arin administration during pregnancy causes
birth de ects and abortion; heparin, LMWHs, or ondaparinux should be used instead
o war arin in pregnant women. Less common toxicities o war arin include skin necro-
sis, purple toe syndrome, and other toxicities that are described in Chapter 30 o
Goodman & Gilmans T e Pharmacological Basis of T erapeutics, 12th Edition.
e. What counseling should this patient receive?
Because this patient is o child-bearing age, she needs to be warned that war arin
causes birth de ects and abortion. She should be changed to a heparinoid i there
is a possibility she is pregnant or will become pregnant. All patients receiving war-
arin should be educated to report any changes in medications (including nonpre-
scription drugs) or ood supplements because o the large number o drugdrug
and ooddrug interactions possible with war arin that alter the pharmacokinetics
and e cacy o war arins anticoagulant properties (see Side Bar FAC ORS LEAD-
ING O AL ERED AN ICOAGULAN AC IVI Y OF WARFARIN). Dietary
changes can also a ect war arin e cacy by altering the intake o vitamin K. For
instance, changing to a diet that is rich in green lea y vegetables (which are high
in vitamin K) can lower the patients INR. T e patient should report any change in
medications, dietary supplements, or signi cant change in diet so that more
requent INR testing can be done and the results used to adjust war arin dosing.
. Are there other oral anticoagulants that this patient might be prescribed instead
o war arin?
Because this patient has nonvalvular atrial brillation, alternative oral anticoagu-
lants that could be considered include the oral direct thrombin inhibitor dabigatran
etexilate, or one o the oral direct actor Xa inhibitors, rivaroxaban or apixaban. An
advantage o these agents over war arin is that they do not require routine labora-
tory monitoring o anticoagulant status because they have predictable pharmacoki-
netics and ew drugdrug or ooddrug interactions.

328
Drug Therapy of Thromboembolic Disorders CHAPTER 1 9

CASE 19-4 FACTORS LEADING TO


While shopping in the grocery store, a 61-year-old man develops symptoms o weakness ALTERED ANTICOAGULANT
on one side o his body and his wi e notices that he has trouble speaking. Suspecting that ACTIVITY OF WARFARIN
he is having a stroke, they go to the nearest hospital which is a 20-minute drive rom the War arin has a narrowtherapeuticindex
store. At the hospital it is determined that the patient has had an ischemic stroke and requiring regular laboratorymonitoring
alteplase is administered. (INR, see Side Bar LABORATORYMONITOR-
a. What is the rationale or administering alteplase to this patient? INGOFANTICOAGULANTTHERAPY) to
maintain adequate anticoagulation and
Alteplase is a brinolytic (clot-buster) agent that breaks down brin clots. In this
minimize risko bleeding
patient, who has an ischemic stroke, alteplase lyses the brin clots that are occlud-
ing blood vessels in the patients brain, thus restoring blood f ow to the ischemic War arinhasmore than 200drugdrugand
regions and preventing irreversible damage. ooddruginteractionsthancan alter dosing
Common actors that can alter war arins
b. What is the mechanism o action o alteplase?
anticoagulant e ects:
Alteplase is the recombinant orm o tissue plasminogen activator (t-PA; see Changes in diet, particularlyincreases
Figure 19-6). Alteplase and its genetically engineered derivatives, reteplase and
or decreases in vitamin K-rich oods
tenecteplase, are serine proteases that cleave plasminogen to plasmin, thus activating
or supplements
plasmins proteolytic activity (see Figure 19-6). Plasmin is the endogenous protease
that is responsible or digesting brin clots (see Figure 30-3 in Goodman & Gilmans Changes in diet or intestinal fora that
T e Pharmacological Basis of T erapeutics, 12th Edition). a ect gut bacteria synthesis o vitamin K
Reduction in absorption o drug rom
c. What must be considered be ore using alteplase to treat a patient with symptoms
the GItract
o stroke?
Increased clearance o drug resulting
During the initial workup o this patient, it must be determined that this patient is
rominduction o hepaticenzymes,
having an acute ischemic stroke rather than a hemorrhagic stroke since the symp-
especiallyCYP2C9
toms are similar. Administering a brinolytic agent to a patient with a hemorrhagic
stroke would be catastrophic. It is also important to determine the time the patient Lowconcentrations o coagulation
rst experienced symptoms because brinolytic therapy is maximally e ective actors due to hepaticdisease,
within the rst 3 hours ollowing the development o symptoms. Because o the risk congestive heart ailure, and other
o major bleeding that can occur with brinolysis o physiological clots, there are a disease states
(continues)
number o contraindications to brinolytic therapy (see able 30-3 in Goodman &
Gilmans T e Pharmacological Basis of T erapeutics, 12th Edition).
(Continued)

Endo the lial c e lls

PAI-1
PAI-2
Plas mino g e n

t-PA 2 -AP

Plas min

Fibrin

S mo o th mus cle c e lls /mac ro phag es

FIGURE 19-6 Fibrinolysis. Endothelial cells secrete tissue plasminogen activator (t-PA) at sites o injury. t-PA
binds to brin and converts plasminogen to plasmin, which digests brin. Plasminogen activator inhibitors-1
and -2 (PAI-1, PAI-2) inactivate t-PA; 2-antiplasmin (2-AP) inactivates plasmin.
329
SECTION III Modulation of Cardiovascular Function

d. What is the most important toxicity associated with alteplase?


FACTORS LEADING TO
ALTERED ANTICOAGULANT Major bleeding is the most important toxicity associated with alteplase and other
ACTIVITY OF WARFARIN brinolytic agents, hence the many contraindications listed in able 30-3 in
Goodman & Gilmans T e Pharmacological Basis of T erapeutics, 12th Edition. Major
(Cont.)
bleeding is caused by the lysis o brin at sites o vascular injury and by systemic
Increased levels o coagulation actors
activation o plasmin that results in degradation o brinogen and clotting actors.
during pregnancy(although war arin
should not be used in most pregnancies e. Are there other indications or f brinolytic agents?
because o its teratogenice ects) Fibrinolytic agents are also indicated or the treatment o acute myocardial in arc-
Geneticpolymorphisms that alter war arin tion although coronary angioplasty, when easible, is the treatment o choice.
pharmacokinetics and pharmacodynamics Fibrinolytic therapy is also indicated or the treatment o pulmonary embolism
also lead to large di erences in dosing associated with deep vein thrombosis.
requirements
Polymorphisms in CYP2C9 that reduce KEY CONCEPTS
the rate o metabolismo war arin and
require dose reduction compared to Antiplatelet drugs are used to prevent platelet activation and aggregation in
wild-type (see Table 30-2 in Goodman arterial blood vessels.
&Gilmans The Pharmacological Basis of Combination therapy with antiplatelet drugs rom di erent classes can be addi-
Therapeutics, 12th Edition) tive or synergistic.
Polymorphisms in VKORC1 (vitamin K Anticoagulants are used to prevent the ormation o brin clots.
epoxide reductase complex, subunit 1)
that reduce the activityo the gene Anticoagulant drugs either reduce the activation o thrombin (directly or indi-
product and require dose reduction rectly) or inhibit thrombin (directly or indirectly).
compared to wild-type (see Table Fibrinolytic drugs act by increasing the amounts o plasmin, the protease that
30-2 in Goodman &Gilmans The breaks down brin.
Pharmacological Basis of Therapeutics, Fibrinolytic drugs are most e ective in breaking down clots when used soon
12th Edition) a er brin clots are ormed.
Bleeding is the most common adverse e ect o antiplatelet agents, anticoagu-
lants, and brinolytic agents; the therapeutic bene t should outweigh risk o
major bleeding.
Combination therapy with antiplatelet agents and anticoagulants may be indi-
cated to prevent thrombus ormation, but the risk o bleeding is increased.

SUMMARY QUIZ

QUESTION 19-1 A patient diagnosed with acute coronary syndrome is prescribed


clopidogrel. What is the mechanism o action o this agent?
LABORATORY a. It vasodilates coronary arteries.
MONITORING OF
b. It irreversibly blocks ADP receptors.
ANTICOAGULANT THERAPY
c. It blocks thrombin receptors on platelets.
Prothrombin time (PT,with values
d. It inhibits thrombin.
converted to the INR) is used to monitor
war arin therapybut is not required or the QUESTION 19-2 Following a trans-Paci c f ight rom Australia, a 57-year-old man
direct inhibitors o thrombin or actor Xa develops deep vein thrombosis in his right leg. He receives heparin to treat the DV
aPTTis used to monitor standard heparin but a er 6 days o heparin therapy he develops heparin-induced thrombocytopenia
therapybut is not required or LMWHor (HI ). What drug is he likely to receive instead o heparin?
ondaparinux; it is also used or monitoring a. Argatroban
bivalirudin and argatroban
b. War arin
Activated clotting time is used to monitor
c. Activated protein C (drotrecogin alpha)
anticoagulant status in patients receiving
high-dose heparin d. Dabigatran
Chromogenic actor Xa activityassays (also e. Rivaroxaban
termed anti actor Xa assays) can be used to (Continued)
monitor heparin and LMWHe cacy

330
Drug Therapy of Thromboembolic Disorders CHAPTER 1 9

QUESTION 19-3 A 52-year-old woman with atrial brillation is prescribed apixaban.


What is the mechanism o action o this oral anticoagulant?
a. It indirectly inhibits thrombin by activating antithrombin.
b. It directly inhibits thrombin.
c. It indirectly inhibits actor Xa by activating antithrombin.
d. It directly inhibits actor Xa.
e. It blocks the synthesis o active clotting actors.

QUESTION 19-4 T ere is signi cant patient-to-patient variability in the dose o


clopidogrel required to achieve therapeutic inhibition o platelet unction, but not or
prasugrel. What is the reason or this variability in clopidogrel, but not prasugrel?
a. T ere are many signi cant ooddrug interactions that a ect the pharmacodynamics
o clopidogrel.
b. T ere are many signi cant drugdrug interactions that a ect the pharmacokinetics
o clopidogrel.
c. T e are genetic polymorphisms in the enzymes that activate clopidogrel to its active orm.
d. T ere are genetic polymorphisms in the enzymes that degrade clopidogrel to its
inactive orm.

QUESTION 19-5 A patient who is hospitalized or an acute myocardial in arction


receives abciximab. What is the mechanism o action and rationale or using this drug?
a. Abciximab degrades brin clots in the occluded coronary arteries.
b. Abciximab inhibits the ormation o brin clots.
c. Abciximab blocks ADP receptors on platelets.
d. Abciximab inhibits the activity o COX1 in platelets.
e. Abciximab blocks interactions o platelet GPIIb/IIIa with brinogen.

SUMMARY QUIZ ANSWER KEY


QUESTION 19-1 Answer is b. Clopidogrel and the other thienopyridines (ticlopidine,
prasugrel) irreversibly block P2Y12 (ADP) receptors on platelets, which inhibits platelet
activation and aggregation. A patient with acute coronary syndrome is at high risk
o acute myocardial in arction caused by platelet thrombus ormation at the site o
ruptured coronary artery plaques (see Chapter 16). Antiplatelet therapy will reduce the
risk o myocardial in arction in such patients.
QUESTION 19-2 Answer is a. T e patient is most likely to receive argatroban which is
a parenterally administered small molecule direct thrombin inhibitor. Alternatives to
argatroban or patients with HI include bivalirudin or ondaparinux. War arin should
not be used initially to replace heparin because there is a risk o developing venous
limb gangrene, but can be started a er the patient has been stably anticoagulated with
argatroban and his platelet count has increased. Activated protein C would not be an
e ective anticoagulant in this setting. T e oral anticoagulants dabigatran (oral direct
thrombin inhibitor) and rivaroxaban (oral direct actor Xa inhibitor) are theoretically
good alternatives to heparin in patients with DV s, but have not yet been tested
clinically or this indication.
QUESTION 19-3 Answer is d. Apixaban is an oral direct inhibitor o actor Xa. Inhibition
o actor Xa blocks the conversion o prothrombin ( actor II) to thrombin ( actor IIa)
(see Figure 19-3). T e anticoagulant drugs that indirectly inhibit thrombin and actor
Xa by activating antithrombin are the heparinoids. T e anticoagulant drug that blocks
the synthesis o active clotting actors is war arin.
QUESTION 19-4 Answer is c. T ere is wide interindividual variability in antiplatelet
e ects o clopidogrel because the prodrug is primarily activated by CYP2C19 which
(Continued)
331
SECTION III Modulation of Cardiovascular Function

has a loss-o - unction polymorphism (CYP2C19*2) and several reduced- unction


polymorphisms (CYP2C19*3, *4, *5). In contrast, prasugrel is activated by esterases
which are not limiting and do not have polymorphisms that a ect prasugrel activation.
QUESTION 19-5 Answer is e. Abciximab is a humanized monoclonal antibody that
binds to the glycoprotein IIb/IIIa (GPIIb/IIIa) complex on the sur ace o activated
platelets (see Figure 19-1). When platelets are activated, GPIIb/IIIa undergoes a con-
ormational change that allows the complex to bind to brinogen in the blood and von
Willebrand actor on the sur ace o subendothelial cells. T is is a key step in anchoring
platelets to the wall o the damaged blood vessel and in orming platelet aggregates.
Abciximab blocks these interactions, thus preventing the ormation o platelet thrombi
even though platelets are activated by mediators such as ADP and xA2. Other agents
that also block GPIIb/IIIa interactions with brinogen include epti batide and tiro ban.

SUMMARYTABLE: DRUGS USED IN THE PREVENTION AND TREATMENT OF THROMBOEMBOLIC DISORDERS


TOXICITIES
CLASS AND SUBCLASSES NAMES CLINICAL USES COMMON UNIQUE; CLINICALLYIMPORTANT
AntiplateletIrreversible Aspirin (ASA) Prophylaxis o pathological GI bleeding
COX-1 Inhibitor arterial thromboses including
MI, stroke, and peripheral
vascular thromboses

AntiplateletIrreversible Ticlopidine Secondary prevention and Nausea, vomiting, Severe neutropenia


ADP Receptor (P2Y12) treatment o MI and stroke diarrhea, bleeding Fatal agranulocytosis with
Antagonist thrombocytopenia
Thrombotic thrombocytopenic
purpura-hemolytic uremic
syndrome (TTP-HUS)

Clopidogrel Secondary prevention o Bleeding; increased risk Thrombocytopenia and


stroke, MI or bleeding i urgent leukopenia (much less o ten than
Peripheral artery disease, acute surgery required due with ticlopidine)
coronary syndrome to irreversible inhibition Wide inter-individual
Used with ASA or prevention o platelets variability because o CYP2C19
o recurrent ischemia in polymorphisms
unstable angina, a ter
angioplasty with stent
placement

Prasugrel Same as clopidogrel; more Bleeding; increased risk Contraindicated in patients with a
ef cacious and predictable or bleeding i urgent history o cerebrovascular disease
antiplatelet e ects than surgery required due because o increased risk o li e-
clopidogrel because o high to irreversible inhibition threatening bleeding
degree o bioactivation o platelets Dose reduction in patients
weighing <60 kg, >75 years o age,
or renal impairment

AntiplateletGPIIb/IIIa Abciximab Percutaneous angioplasty Bleeding, major bleeding Thrombocytopenia in


Inhibitor in patients with coronary in 1-10% o patients ~2% o patients
thromboses; administered depending on intensity o
with aspirin and heparin heparin anticoagulation

Epti batide Acute coronary syndrome Bleeding, major bleeding Thrombocytopenia in


and coronary angioplasty; in 10% o patients 0.5-1% o patients
administered with aspirin (with heparin)
and heparin

Tiro ban Non-Q-wave myocardial Bleeding (similar to


in arction, unstable angina; epti batide)
used with heparin

332
Drug Therapy of Thromboembolic Disorders CHAPTER 1 9

TOXICITIES
CLASS AND SUBCLASSES NAMES CLINICAL USES COMMON UNIQUE; CLINICALLYIMPORTANT

Anticoagulant Heparin Prevent or treat venous Bleeding, with major Heparin-induced


(parenteral)Heparinoid thromboembolism; bleeding in 1-5% in thrombocytopenia (HIT); allergic
immediate anticoagulant patients receiving reactions
e ects when given IV IVheparin; must be
monitored by aPTT

LMWHs Same as heparin; pre erred Bleeding, but monitoring Heparin-induced


(enoxaparin, over heparin because o more not usually required thrombocytopenia (HIT), but less
dalteparin, predictable PK, no monitoring because o predictable common than heparin
tinzaparin, required, less risk o HIT dosing
ardeparin,
nadroparin)

Fondaparinux Same as heparin; pre erred Bleeding Should not be used in renal
over heparin because o more patients because drug is excreted
predictable PK, no monitoring, in urine
much less risk o HIT than
heparin or LMWHs

Anticoagulant Desirudin, As an alternative to Bleeding Lepirudin and desirudin are


(parenteral)Direct bivalirudin, heparinoids, especially in excreted by the kidneys and can
Thrombin Inhibitor argatroban patients with HIT cause bleeding in renal ailure
patients
Patients may develop antibodies to
these proteins

Anticoagulant Drotrecogin Severe sepsis Bleeding


(parenteral)Activated alpha
Protein C

Anticoagulant (oral) War arin Prevent progression or Bleeding, with risk o Contraindicated during pregnancy
Vitamin KAntagonist recurrence o acute DVT or major bleeding <3% in due to birth de ects and abortion
pulmonary embolism ollowing patients with target (use a heparinoid anticoagulant
heparin therapy INR o 2-3; risk o instead)
Prevent venous intracranial bleeding Skin necrosis (rare)
thromboembolism in at-risk increases with INR >4 Purple toe syndrome
patients including those with Antidote to war arin overdose is
acute MI, prosthetic heart valve, vitamin K
atrial brillation, or undergoing
orthopedic or gynecological
surgery

Anticoagulant (oral)- Dabigatran Alternative to war arin or Bleeding (routine No currently available antidote (as
Direct Thrombin Inhibitor etexilate chronic anticoagulation in laboratory monitoring there is or war arin)
nonvalvular atrial brillation is unnecessary)
Prevention o venous
thromboembolism and stroke
a ter knee, hip surgery

Anticoagulant (oral) Rivaroxaban, Alternative to war arin or Bleeding (routine No currently available antidote
Direct Factor Xa Inhibitor apixaban chronic anticoagulation in laboratory monitoring
nonvalvular atrial brillation is unnecessary)
Prevention o venous
thromboembolism and stroke
a ter knee, hip surgery

FibrinolytictPA Alteplase, Treatment o choice or Bleeding, hemorrhagic Contraindications are risk o li e-


reteplase, patients with acute ischemic stroke, serious bleeding in threatening hemorrhage (see
tenecteplase stroke who present within 3 2-4% o patients Table 30-3 in Goodman &Gilmans
hours o symptoms with heparin The Pharmacological Basis of
Treatment o myocardial Therapeutics, 12th Edition)
in arction when coronary
angioplasty is not easible

333
CHAPTER

20 Drug Therapyof Dyslipidemias


T is chapter will be most use ul a er having a basic understanding o the material in
DRUGS INCLUDED IN THIS
Chapter 31, Drug T erapy or Hypercholesterolemia and Dyslipidemia in Goodman &
CHAPTER Gilmans T e Pharmacological Basis of T erapeutics, 12th Edition. In addition to the
Atorvastatin (LIPITOR) material presented here, the 12th Edition contains:
Bezafbrate (not marketed in the United able 31-1 Characteristics o Plasma Lipoproteins which lists the major classes o
States) lipoproteins and their properties
Cholestyramine (QUESTRAN, CHOLYBAR, able 31-2 Apolipoproteins which describes the properties and unctions o apolipo-
others) proteins that have well-de ned roles in plasma protein metabolism
Ciprofbrate (not marketed in the United able 31-6 Assessing 10-Year Risk o CVD Events which provides an algorithm or
States) estimating a patients risk o a cardiovascular disease (CVD) event (ie, myocardial
Clofbrate (no longer available) in arction [MI], stroke, transient ischemic attack [( IA)], peripheral vascular disease,
heart ailure)
Colesevelam(WELCHOL)
able 31-8 Clinical Identi cation o the Metabolic Syndrome which provides levels
Colestipol (COLESTID, others)
o the ve CHD risk actors that de ne metabolic syndrome
Ezetimibe (ZETIA)
able 31-9 Guidelines Based on the LDL-C and otal Cholesterol:HDL-C Ratio or
Ezetimibe/simvastatin (VYTORIN) reatment o Low HDL-C Patients which provides an algorithm or treating patients
Fenofbrate (TRICOR, TRIGLIDE) with low high-density lipoprotein cholesterol (HDL-C) levels
Fluvastatin (LESCOL) able 31-10 Dose (mg) o Stains Required to Achieve Various Reductions in Low-
Gemfbrozil (LOPID) Density-Lipoprotein Cholesterol From Baseline which provides an algorithm or
initiating dosing o statins based on baseline low-density lipoprotein cholesterol
Lovastatin (MEVACOR)
(LDL-C) levels
Niacin (NIACOR, NIASPAN, others)
T e molecular structures o drugs used to treat dyslipidemias
Pitavastatin (LIVALO)
Pravastatin (PRAVACHOL) LEARNING OBJECTIVES
Rosuvastatin (CRESTOR) Understand the mechanisms o action o drugs used to treat dyslipidemia.
Simvastatin (ZOCOR) Know the untoward e ects o drugs used to treat dyslipidemias.
Know which patients with dyslipidemias should be treated and when treatment
should be initiated.
Know which drugs are most e ective in treating patients with di erent
dyslipidemias.
Know which drugs can be used in combination to treat dyslipidemias.
Know the e ects o treatment on lowering the 10-year risk o a CVD event, and
how to set treatment goals based on primary and secondary prevention o a
CVD event.

MECHANISMS OF ACTION OF DRUGS USED TO TREAT DYSLIPIDEMIAS


DRUG CLASS DRUG MECHANISM OF ACTION
HMG-CoA Reductase Atorvastatin, simvastatin, Competitive inhibitors o HMG-CoA reductase, the rate-limiting step in liver
Inhibitors (Statins) rosuvastatin calcium, lovastatin, cholesterol biosynthesis; inhibition o hepatic cholesterol synthesis increases
pravastatin sodium, uvastatin expression o the LDL receptor on hepatocytes which increases the removal LDL
sodium, pitavastatin calcium rom the blood

Bile AcidBinding Cholestyramine, colestipol, Highly positively charged bile acid sequestrants that bind bile acids in the GI
Resins (Bile Acid colesevelam system, thereby depleting the pool o bile acids and stimulating hepatic bile acid
Sequestrants) production which leads to reduced hepatic cholesterol content
(Continued)

334
Drug Therapy o Dyslipidemias CHAPTER 2 0

DRUG CLASS DRUG MECHANISM OF ACTION


Nicotinic Acid Niacin Binds a GPCR (GPR109A) or niacin in adipose tissue that leads to the inhibition o
(Niacin) triglyceride lipolysis by hormone-sensitive lipase, which reduces transport o ree
atty acids to the liver and decreases hepatic triglyceride synthesis

Fibric acid (Fibrates) Gem brozil, eno brate, Unclear, but likely involves activation o PPAR in liver and brown adipose tissue
cipro brate (not marketed in the which stimulates atty acid oxidation, increases lipoprotein lipase, and reduces
United States), beza brate (not expression o apoC-III
marketed in the United States),
clo brate (no longer available)

Cholesterol Ezetimibe Inhibits cholesterol absorption at the brush border o the small intestine by inhibiting
Absorption Inhibitors sterol transport protein Niemann-PickC1-Like1 (NPC1L1) in jejunal enterocytes

CASE 20-1
A 58-year-old man sees his physician or a routine checkup. He is in good health with
no symptoms or complaints. He does not have a amily history o premature heart dis-
ease, although his paternal grand ather su ered a non atal heart attack when he was 67.
Just prior to the checkup, he provides asting blood samples or lipid pro les. His
plasma cholesterols are: total cholesterol, 203 mg/dL; HDL cholesterol, 52 mg/dL; LDL
cholesterol, 130 mg/mL; VLDL cholesterol, 21 mg/dL. His triglycerides are 148 mg/dL.
His asting serum glucose is in the normal range and his blood pressure is 128/75. He is
not currently taking any medications, has never smoked, and usually has 1 or 2 glasses TRADITIONAL MAJOR RISK
o wine with dinner, but consumes no other alcoholic beverages. FACTORS FOR CORONARY
a. What is this patients 10-year risk o a CVD event? HEART DISEASE
able 31-6 in Goodman & Gilmans T e Pharmacological Basis of T erapeutics, 12th Elevated LDL-C(>160 mg/dL; see
Edition provides an algorithm or estimating the 10-risk o coronary vascular Tables 20-1 and 20-2)
disease (CVD) events based on data rom the Framingham Heart Study. T e risk Reduced HDL-C(<40 mg/dL; see
prediction or a CVD event will exceed the risk o a coronary heart disease (CHD) Table 20-2)
event. For this patient, the risk actors and CVD points associated with each risk
Cigarette smoking (within the preceding
(Continued)
30 days)
Die ta ry fa t + chole s te rol
Hypertension (blood pressure 140/90 mm
Hg or use o antihypertensive medication,
LP L Chylomicron regardless o blood pressure)
Chylomicrons
re mna nts
INTESTINE FFA Type 2 diabetes mellitus
Adipos e LP L
tis s ue Advancing age (men >45 years o age or
HL
women >55 years o age)
Familyhistoryo premature coronaryheart
Pe riphe ra l
Bile a cids FFA tis s ue s
disease (CHD) events (men <55 years;
(with LDL women <65years) in a frst-degree relative
re ce ptors )
FFA Obesity(bodymass index[BMI] >25 kg/m2
LP L
and waist circum erence >40 in [men] or
VLDL IDL LDL >35 in [women])
LDL LP L HL
LIVER re ce ptors
ApoE me dia te d
Re mna nt
re ce ptors ApoB me dia te d
ApoE me dia te d

HIGH-RISK DYSLIPIDEMIAS
FIGURE 20-1 The major pathways involved in the metabolism o chylomicrons synthesized
by the intestine and VLDL synthesized by the liver. Chylomicrons are converted to chylomicron
(SEE TABLE 20-2)
remnants by the hydrolysis o their triglycerides by LPL. Chylomicron remnants are rapidly cleared High total cholesterol (240 mg/dL)
rom the plasma by the liver. Remnant receptorsinclude the LDL receptorrelated protein (LRP),
LDL receptors, and perhaps other receptors. Free atty acid (FFA) released by LPL is used by mus- High LDL-C(>160 mg/dL)
cle tissue as an energy source or taken up and stored by adipose tissue. HL, hepatic lipase; IDL, LowHDL-C(<40 mg/dL)
intermediate-density lipoproteins; LDL, low-density lipoproteins; LPL, lipoprotein lipase; VLDL, Severe hypertriglyceridemia (500 mg/dL)
very-low-density lipoproteins.
335
SECTION III Modulation of Cardiovascular Function

actor are: age (10 points); HDL (1 point); total cholesterol (2 points); systolic
PRIMORDIAL PREVENTION
blood pressure (0 points); smoker (0 points), diabetic (0 points). T e total number
GUIDELINES (TO PREVENT
o CVD points is 11, placing his 10-year risk o a CVD event at 11.2%.
DEVELOPMENT OF CVD
RISK FACTORS) b. What, i anything, can be done to reduce this patients risk o a CVD event?
150 min/wko moderate intensityexercise T is patients largest CVD risk is his age, which is not a modi able risk actor.
(walking 20-30 min/d) Another risk actor which cannot be modi ed is his sex; a woman o the same age
with all the same lipid, blood pressure, and history would have a 10-year risk o a
Reducing total calories rom at to less than CVD event o 6.3%. Based on able 31-6 in Goodman & Gilmans T e Pharmacologi-
30%, with saturated and trans at toless cal Basis of T erapeutics, 12th Edition, the modi able risk actors or this patient are
than 7% his blood lipids and his systolic blood pressure. With optimal blood lipids and sys-
Consuming less than 300 mg cholesterol/d tolic blood pressure, his CVD points would be reduced to 6 points and his 10-year
Eat a varietyo oilyfsh at least twice a week risk o a CVD event would be reduced to 4.7%.
Eat oils/ oods rich in -linolenicacid c. What are the treatment options to maximally reduce this patients 10-year
Restricting sugarybeverages to less than CVD risk?
36 oz/wk or a person consuming 2000 T e rst option is or the patient to adopt changes in exercise and diet (see Side
Kcal daily Bar PRIMORDIAL PREVEN ION GUIDELINES). Such li estyle changes can be
e ective in lowering blood pressure and improving blood lipid pro les. I li estyle
changes do not adequately improve blood lipid pro les, it may be prudent to start
the patient on drugs that lower serum cholesterol (based on able 20-1). o reach
optimal levels o total cholesterol and LDL-C in this patient (<200 mg/dL and <100
mg/dL, respectively; see able 20-2), the lowest dose o a statin drug should be su -
cient (see able 31-10 in Goodman & Gilmans T e Pharmacological Basis of T era-
peutics, 12th Edition).
(Continued)

TABLE 20-1 Treatment Based on LDL-C Levels (2004 Revision o NCEP Adult Treatment Panel III Guidelines)
LDL C GOAL NON HDL C THERAPEUTIC THRESHOLD FOR
RISK CATEGORY mg/dL GOAL mg/dL LIFESTYLE CHANGE DRUG THERAPY mg/dL
Very high risk <70a <100 No threshold No threshold (initiate therapy)
Atherosclerosis-induced CHD plus (initiate change)
one o :
multiple risk factors
diabetes mellitus
a poorly controlled single factor
acute coronary syndrome
metabolic syndrome

High risk <100a <130 No threshold No threshold


CHD or CHD Equivalent

Moderately high risk <130 (optional <100) <160 No threshold 130 (100-129)b
2+ risk actors
10-year risk: <1020%

Moderate risk <130 <160 No threshold >160


2+ risk actors
10-year risk <10%

01 risk factor <160 <160 No threshold 190 (optional: 160189)c


a
I pretreatment LDL-C is near or below LDL-C goal value, then a statin dose su cient to lower LDL-C by 30-40% should be prescribed.
b
Patients in this category include those with a 10-year risk o 10-20% and one o the ollowing: age >60 years, three or more risk actors, a
severe risk actor, triglycerides >200 mg/dL and HDL-C <40 mg/dL, metabolic syndrome, highly sensitive C-reactive protein (CRP) >3 mg/L, and
coronary calcium score (age/gender adjusted) >75th percentile.
c
Patients include those with any severe single risk actor, multiple major risk actors, 10-year risk >8%.
A ter attaining the LDL-C goal, additional therapy may be necessary to reach the non-HDL-C goal. CHD, coronary heart disease; CHD equivalent,
peripheral vascular disease, abdominal aortic aneurysm, symptomatic carotid artery disease, >20% 10-year CHD risk, or diabetes mellitus; HDL-C,
high-density-lipoprotein cholesterol; LDL-C, low-density-lipoprotein cholesterol; NCEP, National Cholesterol Education Program.

336
Drug Therapy o Dyslipidemias CHAPTER 2 0

TABLE 20-2 Classif cation o Plasma Lipid Levels (mg/dL)a


Total cholesterol

<200 Desirable

200-239 Borderline high

240 High

HDL-C

<40 Low (consider <50 mg/dL as low or women)

>60 High

LDL-C

<70 Optimal or very high risk (minimal goal or CHD equivalent patients)

<100 Optimal

100-129 Near optimal

130-159 Borderline high

160-189 High

190 Very high

Triglycerides

<150 Normal

150-199 Borderline high

200-499 High

500 Very high


a
2001 National Cholesterol Education Program guidelines. HDL-C, high-density-lipoprotein
cholesterol; LDL-C, low-density-lipoprotein cholesterol. From The Expert Panel, 2002.

d. What is the mechanism(s) by which statins reduce CVD risk?


T e statins competitively inhibit 3-hydroxy-3-methylglutaryl-coenzyme A
(HMG-CoA) reductase, the early rate-limiting step in hepatic cholesterol bio-
synthesis. T ey are the most e ective and best-tolerated agents or treating dys-
lipidemias. By inhibiting hepatocyte cholesterol synthesis, the statins induce an
increase in the expression o LDL receptors on hepatocytes. T e greater number
o LDL receptors on the sur ace o hepatocytes acilitates the removal o LDL
rom the blood, which lowers LDL-C levels. High levels o LDL-C are associated
with a high risk o CVD.
here is also evidence that statin-induced increases in hepatocyte LDL
receptors also enhance the removal o circulating LDL precursors, VLDL,
and IDL (see Figure 20-1). Statin-mediated reduction in hepatic cholesterol
synthesis also reduces hepatic production o VLDL, which can reduce serum
triglyceride levels.
In addition to lowering LDL-C, a number o other potential cardioprotective e ects
have been ascribed to the statins (see Potential Cardioprotective E ects Other than
LDL Lowering, p. 894, in Goodman & Gilmans T e Pharmacological Basis of T era-
peutics, 12th Edition).

337
SECTION III Modulation of Cardiovascular Function

CASE 20-2
A 52-year-old woman with a history o angina, hypertension (blood pressure o
140/80 mm Hg), and type 2 diabetes su ers an acute myocardial in arction and
receives angioplasty and a stent to open a blocked coronary artery. She has been taking
medications or her hypertension and diabetes, but no other medications. She smokes
a pack o cigarettes each day. Be ore she is discharged rom the hospital, her asting
blood lipids are measured: total cholesterol, 240 mg/dL; HDL-C, 30 mg/dL; LDL-C,
160 mg/dL; triglycerides, 220 mg/dL.
a. What is the 10-year risk this woman will have a CVD event?
Based on able 31-6 in Goodman & Gilmans T e Pharmacological Basis of T era-
peutics, 12th Edition, this patients risk actors and CVD points are: age (7 points),
HDL-C (2 points), total cholesterol (4 points), SBP (5 points), smoker (3 points),
and diabetes (4 points). Her total number o CVD points is 25 placing her 10-year
risk o a CVD event at greater than 30%.
b. What are the recommended treatment options to reduce the risk o another
myocardial in arction or other CVD event in this patient?
Patients such as this woman who have had a CVD event are at the greatest risk o
another CVD event and need to be treated aggressively to reduce their risk (see
able 20-1). She should be counseled to adopt a healthy diet and to begin a moder-
ate exercise program (see Side Bar PRIMORDIAL PREVEN ION GUIDELINES).
Her hypertension and diabetes medications should also be reviewed and adjusted
to optimize systolic blood pressure and blood glucose.
Her dyslipidemias should be treated aggressively with goals o reducing her
LDL-C to less than 70 mg/dL (see able 20-1) and C:HDL-C ratio to less than
3.5 (see able 31-9 in Goodman & Gilmans T e Pharmacological Basis of T era-
peutics, 12th Edition). o reach the goal o reducing her LDL-C to less than 70
mg/L (ie, 44%), she should be started on a statin dosage that achieves this goal
(see able 31-10 in Goodman & Gilmans T e Pharmacological Basis of T erapeu-
tics, 12th Edition).
c. She is started on rosuvastatin 10 mg daily. When should she take this dose?
Rosuvastatin has a t1/2 o 20 to 30 hours and can be taken at any time o the day.
Statins with a short t1/2 (<4 hours) such as uvastatin, lovastatin, simvastatin, and
others (see text) should be taken in the evening since hepatic cholesterol synthesis
is maximal between midnight and 2:00 AM.
d. What are the common adverse ef ects o statins?
Myopathy and rhabdomyolysis are the major adverse e ects o statins and can
result in death (1 death per million 30-day prescriptions). T e risk o myopathy
is increased with dose and plasma concentrations. Concomitant use o drugs that
diminish statin catabolism or inter ere with hepatic uptake is associated with 50 to
60% o all cases o myopathy and rhabdomyolysis. T e most common statin inter-
actions are with some o the brates such as gem brozil. Gem brozil inter eres
with hepatocyte uptake o statins by OA P1B1 and its catabolism by glucuronida-
tion. Fibrates such as eno brate do not inter ere with statin glucuronidation, and
are not associated with myopathy when used concomitantly.
Hepatotoxicity has also been associated with statin use and is dose-related. Serious
hepatotoxicity occurs at a rate o 1 case per million person-years o use. Because
there is some risk o liver damage with statins, it is reasonable to measure serum ala-
nine aminotrans erase (AL ) at baseline and subsequently when clinically indicated.

CASE 20-3
A 64-year-old man with highly elevated total cholesterol and LDL-C is not responding
adequately to high-dose statin therapy.
(Continued)
338
Drug Therapy o Dyslipidemias CHAPTER 2 0

a. What additional drug therapies are available to lower this patients serum
cholesterol?
When used in combination with statins, other lipid-lowering drugs such as the bile
acidbinding resins, niacin, brate, and ezetimibe can produce greater reductions
in LDL-C than can be achieved with statins alone (see text: Statins in Combination
With Other Lipid-Lowering Drugs in Goodman & Gilmans T e Pharmacological
Basis of T erapeutics, 12th Edition). riple therapy with resins, niacin, and statins
can reduce LDL-C by up to 70%. Vytorin, a xed combination o simvastatin and
ezetimibe, can reduce LDL-C levels by 60%.
b. What are the considerations in using multiple agents to treat dyslipidemias?
T ere can be adverse drug interactions that occur when statins are combined with
other lipid-lowering agents, as well adverse e ects due solely to each o the indi-
vidual agents. Cholestyramine and colestipol bind to many drugs and inter ere with
their absorption, including some o the statins. T us, it is wise to administer all
drugs either 1 hour be ore or 3 to 4 hours a er a dose o cholestyramine, colestipol,
or other bile acidbinding resins. Patients taking cholestyramine and colestipol
o en complain o bloating and dyspepsia.
T e occurrence o myopathy increases when statin doses o greater than 25% o the
maximal dose (eg, 20 mg o simvastatin or atorvastatin) are used with niacin.
T ere is also an increased risk o myopathy with combination therapy using a statin
and brate (gem brozil or eno brate), but this combination is usually sa e i the
brate is used at its usual maximal dose and a statin at no more than 25% o its
maximal dose. Feno brate is least likely to inter ere with statin metabolism and
appears to be the sa est drug to use with statins. Combined statin- brate therapy
should be avoided in patients with compromised renal unction.

CASE 20-4
A 30-year-old woman makes her rst visit to the doctor in 10 years. Her asting plasma
lipids are: total cholesterol, 210 mg/dL; LDL-C, 140 mg/dL; HDL-C, 40 mg/dL; and
triglycerides, 180 mg/dL. Her asting glucose is 110 mg/dL, her waist circum erence is
40 in, and her blood pressure is 125/80 mm Hg. T is patient has 3 or more risk actors
that identi y her as having metabolic syndrome.
a. What are this patients CHD risk actors that are part o the constellation known
as metabolic syndrome?
T e CHD risk actors that identi y this patient as having metabolic syndrome are
her abdominal obesity, high levels o triglycerides, low HDL-C, and high asting
glucose (see able 31-8 in Goodman & Gilmans T e Pharmacological Basis of T era-
peutics, 12th Edition). T ere is an increased CHD risk associated with the insulin-
resistant, prediabetic state known as metabolic syndrome.
b. What should be the ocus o treatment in this patient?
reatment should ocus on weight loss and increased physical activity to reduce
obesity. In addition, reducing or eliminating alcohol intake and reducing as much
at as possible rom the diet can reduce triglyceride levels. Drug therapy should
ocus on reducing LDL-C, nonHDL-C, and triglycerides, and increasing HDL-C.
T e LDL-C therapy goal should be less than 70 mg/dL (see able 20-1).
c. What drug therapies are most ef ective in treating patients with high triglycer-
ides and low HDL-C levels associated with the metabolic syndrome?
T e brates play an important role in such patients. It is important to monitor LDL
levels when using brates in these patients as LDL levels may rise. I LDL levels rise,
it might be necessary to add a low-dose statin. Many such patients are treated rst
with a statin; then a brate is added. It is important to monitor or myopathy when
using a statin- brate combination (see answer to Case 20-3, question b).
(Continued)
339
SECTION III Modulation of Cardiovascular Function

Niacin is indicated or hypertriglyceridemia and elevated LDL-C, and is especially


use ul in patients with both hypertriglyceridemia and low HDL-C levels. However,
niacin can cause hyperglycemia and, i used in patients with known or suspected
diabetes, blood glucose levels should be monitored weekly until proven to be stable.
Bile acid resins can cause an increase in hepatic triglyceride synthesis, which is o
consequence in patients with baseline triglyceride levels o greater than 250 mg/dL.
d. Because this patient is o childbearing age, what considerations must be made in
choosing an appropriate drug therapy?
T e sa ety o statins during pregnancy has not been established. Women o childbear-
ing age who take statins should use highly e ective contraception (see Chapter 28);
women who wish to become pregnant or who are nursing children should not take
statins. Fibrates should not be used by children or pregnant women. Niacin, at doses
used in humans, has been associated with birth de ects in experimental animals and
should not be taken by pregnant women. T e sa ety o ezetimibe during pregnancy
has not been established; combination products containing ezetimibe and a statin
should not be used by women in childbearing years. T e bile acid sequestrants are not
absorbed systemically but should be used with caution since there have not been stud-
ies in women. No adverse e ects o colesevelam have been seen in animal reproduction
studies and it is classi ed as pregnancy risk actor B. Also, regular prenatal vitamin sup-
plementation may be inadequate as these agents can inter ere with vitamin absorption.

CASE 20-5
A male patients LDL-C levels remain high even with maximal dosing (80 mg daily) o
simvastatin. He is changed to a combination tablet (VY ORIN) containing 80 mg sim-
vastatin and 10 mg ezetimibe.
a. What is the rational or this drug combination?
Statins, which inhibit cholesterol biosynthesis, increase intestinal cholesterol
absorption. Ezetimibe, which inhibits intestinal cholesterol absorption, enhances
cholesterol biosynthesis. Dual therapy with these 2 classes o drugs prevents both
the enhanced cholesterol synthesis induced by ezetimibe and the increase in choles-
terol absorption induced by statins. T is combination provides additive reductions
in LDL-C levels irrespective o the statin employed.
b. What is the molecular target o ezetimibe?
Ezetimibe inhibits luminal cholesterol uptake by jejunal enterocytes, by inhibit-
ing the transport protein NPC1L. Ezetimibe does not a ect intestinal triglyceride
absorption. In human subjects, ezetimibe reduced cholesterol absorption by 54%,
precipitating a compensatory increase in cholesterol synthesis that can be inhibited
with a cholesterol synthesis inhibitor such as a statin.
c. How does ezetimibe reduce plasma LDL-C?
Inhibiting intestinal cholesterol absorption causes a reduction in the incorporation
o cholesterol into chylomicrons. T e reduced cholesterol content o chylomicrons
diminishes the delivery o cholesterol to the liver by chylomicron remnants. Reduced
delivery o intestinal cholesterol to the liver by chylomicron remnants stimulates
expression o the hepatic genes regulating LDL receptor expression and cholesterol
biosynthesis. T e greater expression o hepatic LDL receptors enhances LDL-C clear-
ance rom the plasma. Indeed, ezetimibe reduces LDL-C levels by 15 to 20%.
d. Why not use ezetimibe or monotherapy?
T e maximal ef cacy o ezetimibe or lowering LDL-C is 15 to 20% when used as
monotherapy. T is reduction is equivalent to, or less than, that attained with 10- to
20-mg doses o most statins. Consequently, the role o ezetimibe as monotherapy
in patients with elevated LDL-C levels appears to be limited to the small group o
statin-intolerant patients.

340
Drug Therapy o Dyslipidemias CHAPTER 2 0

KEY CONCEPTS
Patients with any type o dyslipidemia are at increased risk o CHD and
CVD events.
In the absence o vascular disease, type 2 diabetes mellitus, or metabolic syn-
drome, the need or cholesterol-lowering drugs in many patients will be allevi-
ated by maintaining ideal body weight, eating a diet low in saturated at and
cholesterol, and regular exercise.
Li estyle changes are a cornerstone o managing dyslipidemias.
Dyslipidemic patients with a risk or a CVD should be treated to achieve target
lipid values.
Statins should be the rst-line choice when choosing a lipid-lowering drug.
Initiate treatment with statin doses adequate to reduce the patients lipid values
to target goals.
Patients should be counseled regarding the rare but serious side e ects o hepa-
totoxicity and rhabdomyolysis.
For patients with low HDL-C levels, treatment should be based on both LDL-C
levels and the ratio o total cholesterol:HDL-C (see able 31-9 in Goodman &
Gilmans T e Pharmacological Basis of T erapeutics, 12th Edition).

SUMMARY QUIZ

QUESTION 20-1 A patient is prescribed eno brate as part o his therapy to raise his/
her HDL-C levels. Feno brate most likely acts by
a. inhibiting HMG-CoA reductase.
b. binding cell membrane receptors that leads to lowering o adipocyte hormone
sensitive lipase activity.
c. sequestering bile acids in the small intestine.
d. binding PPAR receptors in liver and brown adipose tissue.
e. inhibiting cholesterol uptake rom the small intestine.

QUESTION 20-2 A patient is prescribed niacin to reduce his/her triglyceride


and LDL-C levels. T e side e ect(s) o niacin that most commonly limits patient
compliance include
a. bloating.
b. ushing and dyspepsia.
c. tinnitus.
d. dry cough.
e. chills.

QUESTION 20-3 A patient is prescribed lovastatin to lower total cholesterol and LDL-C.
For maximal e ect, he should take his medication
a. be ore break ast.
b. with break ast.
c. an hour a er break ast.
d. with his evening meal.
e. at bedtime.

341
SECTION III Modulation of Cardiovascular Function

QUESTION 20-4 A patient taking digoxin or his heart ailure is started on lovastatin
and cholestyramine to lower his/her LDL-C levels. T e dose o digoxin
a. should be increased.
b. should be decreased.
c. should be taken 1 hour be ore or 3 to 4 hours a er a dose o cholestyramine.
d. should be eliminated to prevent serious drug interactions.
e. will not be a ected as there are no known drug interactions among these drugs.

QUESTION 20-5 A patient is being started on statin therapy. What should be consid-
ered in choosing which statin to use?
a. Ef cacy in reducing LDL-C
b. Cost
c. Sa ety
d. All o the above
e. None o the above

SUMMARY QUIZ ANSWER KEY


QUESTION 20-1 Answer is d. T e e ects o eno brate and other brates on blood
lipids are mediated by their interaction with the peroxisome proli eratoractivated
receptors (PPARs). Fibrates bind to PPAR, which is expressed primarily in the liver
and brown adipose tissue and to a lesser extent in the kidney, heart, and skeletal muscle.
Fibrates reduce triglycerides through PPAR-mediated stimulation o atty acid oxida-
tion, increased LPL synthesis, and reduced expression o apoC-III. An increase in LPL
would enhance the clearance o triglyceride-rich lipoproteins. A reduction in hepatic
production o apoC-III, which serves as an inhibitor o lipolytic processing and receptor-
mediated clearance, would enhance the clearance o VLDL. Fibrate-mediated increases in
HDL-C are due to PPAR stimulation o apoA-I and apoA-II expression, which increases
HDL levels. Most bric acid agents have potential antithrombotic e ects, including inhi-
bition o coagulation and enhancement o brinolysis. T ese salutary e ects also could
alter cardiovascular outcomes by mechanisms unrelated to any hypolipidemic activity.
QUESTION 20-2 Answer is b. wo o niacins side e ects, ushing and dyspepsia, limit
patient compliance. T e cutaneous e ects include ushing and pruritus o the ace and
upper trunk, skin rashes, and acanthosis nigricans. Flushing and associated pruritus
are prostaglandin-mediated. Flushing is worse when therapy is initiated or the dosage
is increased but ceases in most patients a er 1 to 2 weeks o a stable dose. aking an
aspirin each day alleviates the ushing in many patients. Flushing recurs i only 1 or
2 doses are missed, and the ushing is more likely to occur when niacin is consumed
with hot beverages (co ee, tea) or with ethanol-containing beverages. Flushing is mini-
mized i therapy is initiated with low doses (100-250 mg twice daily) and i the drug
is taken a er break ast or supper. Dry skin, a requent complaint, can be dealt with by
using skin moisturizers, and acanthosis nigricans can be dealt with by using lotions
or creams containing salicylic acid. Dyspepsia and rarer episodes o nausea, vomiting,
and diarrhea are less likely to occur i the drug is taken a er a meal. Patients with any
history o peptic ulcer disease should not take niacin because it can reactivate ulcer dis-
ease. T e most common, medically serious side e ects are hepatotoxicity, mani ested as
elevated serum transaminases, and hyperglycemia.
QUESTION 20-3 Answer is d. Lovastatin is slightly more e ective i taken with the
evening meal than i it is taken at bedtime, although bedtime dosing is pre erable to
missing doses. Hepatic cholesterol synthesis is maximal between midnight and 2:00
AM. T us, statins with t 1/2 4 hours or less (all but atorvastatin and rosuvastatin) should
be taken in the evening.
QUESTION 20-4 Answer is c. Cholestyramine and colestipol bind and inter ere with
the absorption o many drugs, including some thiazides, urosemide, propranolol,
342
Drug Therapy o Dyslipidemias CHAPTER 2 0

L-thyroxine, digoxin, war arin, and some o the statins. T e e ect o cholestyramine
and colestipol on the absorption o most drugs has not been studied. For this reason, it
is wise to administer all drugs either 1 hour be ore or 3 to 4 hours a er a dose o chole-
styramine or colestipol. Colesevelam does not appear to inter ere with the absorption
o at-soluble vitamins or o drugs such as digoxin, lovastatin, war arin, metoprolol,
quinidine, and valproic acid. T e e ect o colesevelam on the absorption o other drugs
has not been tested, but it seems prudent to recommend that patients take other medi-
cations 1 hour be ore or 3 to 4 hours a er a dose o colesevelam.
QUESTION 20-5 Answer is d. T e choice o statins should be based on ef cacy (reduction
o LDL-C), cost, and sa ety. T ree drugs (lovastatin, simvastatin, and pravastatin) have
been used sa ely in clinical trials involving thousands o subjects or 5 or more years. T e
documented sa ety records o these statins should be considered, especially when initiating
therapy in younger patients. Once drug treatment is initiated, it is almost always li elong.

SUMMARY: DRUGS USED IN THE TREATMENT OF DYSLIPIDEMIAS


TOXICITIES
CLASS AND
SUBCLASSES NAMES CLINICAL USES COMMON UNIQUE; CLINICALLY IMPORTANT
HMG-CoA Reductase Atorvastatin The most ef ective and The safety of statins during Hepatotoxicity (1 case per million
Inhibitors (Statins) Simvastatin best-tolerated agents pregnancy has not been person-years o use); it is reasonable
Rosuvastatin to treat dyslipidemias, established; women wishing to measure liver enzymes (ALT) at
Lovastatin especially elevated LDL-C to conceive and nursing baseline and therea ter when clinically
Pravastatin mothers should not indicated
Fluvastatin take statins; during their Myopathy and rhabdomyolysis
Pitavastatin childbearing years, women (1 death per million) prescriptions
taking statins should use (30-day supply); risk increases with
highly ef ective contraception statin dose and other actors including
concomitant administration with drugs
that inter ere with statin catabolism or
hepatic uptake; gem brozil is the drug
most commonly associated with statin-
induced myopathy

Bile AcidBinding Cholestyramine Probably the sa est Bloating and dyspepsia Rare instances o hyperchloremic
Resins (Bile Acid Colestipol lipid-lowering drugs Constipation acidosis because these are
Sequestrants) Colesevelam because they are not Colesevelam may be less administered as chloride salts
absorbed systemically likely to cause dyspepsia, Severe hypertriglyceridemia is
Recommended or bloating, and constipation a contraindication to the use o
patients 11-20 y o age Cholestyramine and colestipol cholestyramine and colestipol because
O ten used as second bind and inter ere with the these resins increase triglyceride levels
agents with statins absorption o many drugs; it
is wise to administer all drugs
either 1 h be ore or 3-4 h a ter
a dose o a bile acid resin

Nicotinic Acid Niacin Favorably af ects all lipid Flushing, pruritus, and Hepatotoxicity, mani ested as elevated
parameters, it is the best dyspepsia limit patient serum transaminases
agent or increasing compliance Hyperglycemia and niacin-induced
HDL-C, and also lowers Rarer episodes o nausea, insulin resistance; in patients with
triglycerides, and reduces vomiting, and diarrhea known or suspected diabetes, blood
LDL-C Should not be taken by glucose levels should be monitored at
pregnant women least weekly until proven to be stable
Concurrent use o niacin and a statin
can cause myopathy; the statin should
be administered at no more than 25%
o its maximal dose; patients should be
instructed to discontinue therapy i u-
like muscle aches occur
Patients with any history o peptic ulcer
disease should not take niacin
Gout is a relative contraindication

343
SECTION III Modulation of Cardiovascular Function

TOXICITIES
CLASS AND
SUBCLASSES NAMES CLINICAL USES COMMON UNIQUE; CLINICALLY IMPORTANT
Fibric Acid (Fibrates) Gem brozil Usually the drugs o Gastrointestinal side ef ects A myopathy syndrome occasionally
Clo brate (no choice or treating severe occur in up to 5% o patients occurs in subjects taking clo brate,
longer available) hypertriglyceridemia, Fibrates should not be used by gem brozil, or eno brate
Feno brate the chylomicronemia children or pregnant women Myopathy may occur in up to 5% o
Cipro brate (not syndrome, hyperlipidemic patients treated with a combination o
marketed in the subjects with type III gem brozil and higher doses o statins
United States) hyperlipoproteinemia Feno brate-statin combinations are
Beza brate (not as well as subjects less likely to cause myopathy than
marketed in the with severe combination therapy with gem brozil
United States) hypertriglyceridemia and statins
(triglycerides >1000 mg/ Renal ailure and hepatic dys unction
dL) who are at risk or are relative contraindications to the use
pancreatitis o brates
Fibrates appear to
have an important role
in subjects with high
triglycerides and low
HDL-C levels associated
with the metabolic
syndrome or type 2
diabetes mellitus

Cholesterol Ezetimibe Monotherapy in patients Bile acid sequestrants inhibit


Absorption Inhibitor with elevated LDL-C levels absorption o ezetimibe, and the
who are statin-intolerant 2 agents should not be administered
together

Ezetimibe/ Combination with Combination products


simvastatin statin provides additive containing ezetimibe and a
reductions in LDL-C, statin should not be used by
greater than that can be women in childbearing years in
achieved with any statin the absence of contraception
as monotherapy

344
SECTION

Infammation, Immunomodulation,
and Hematopoiesis IV
21. Histamine, Bradykinin, and T eir Antagonists 346

22. Prostaglandins, NSAIDs, and Pharmacotherapy o Gout 356

23. Immunotherapeutic Agents 375

24. Pulmonary Pharmacology 389

25. Hematopoietic Agents 405

345
CHAPTER

21 Histamine, Bradykinin,
and Their Antagonists
T is chapter will be most use ul a er having a basic understanding o the material in
DRUGS INCLUDED
Chapter 32, Histamine, Bradykinin, and T eir Antagonists in Goodman & Gilmans
IN THIS CHAPTER T e Pharmacological Basis of T erapeutics, 12th Edition In addition to the material
Acrivastine (SEMPREX-D*) presented here, the 12th Edition contains:
Aprotinin (TRAYSYLOL) able 32-1 Characteristics o Histamine Receptors, which shows the biochemical
Azelastine (ASTELIN) properties, tissue distribution, and representative agonists and antagonists o the
4 histamine receptors (H 1, H 2, H 3, and H 4)
Brompheniramine maleate
(BROMPHEN, others) able 32-2 Preparations and Dosage o Representative H 1-Receptor Antagonists
Carbinoxamine (RONDEC*, others) T e molecular structures o histamine and drugs that are agonists and antagonists
o the various histamine receptors
Cetirizine (ZYRTEC)
Chlorpheniramine (CHLOR-TRIMETON, others)
Clemastine umarate (TAVIST,others) LEARNING OBJECTIVES
Cyclizine (MAREZINE) Understand the role o histamine and bradykinin in di erent physiological and
pathophysiological processes
Cyproheptadine (PERIACTIN)
Desloratadine (CLARINEX,AERIUS) Understand the mechanisms o action o drugs that act as antagonists o the
H 1 receptor
Dimenhydrinate (combination o diphen-
hydramine and 8-chlorotheophylline; Know the therapeutic utility o H 1-receptor antagonists, alone and in combina-
DRAMAMINE, others) tion with other agents
Diphenhydramine (BENADRYL, others) Know the important adverse e ects o H 1-receptor antagonists, and the di er-
ence between rst- and second-generation H 1 antihistamines with regard to
Doxepin (SINEQUAN)
adverse e ects
Ebastine (EBASTEL)
Fexo enadine (ALLEGRA,TELFAST)
Hydroxyzine (ATARAX,others) MECHANISMS OF ACTION OF HISTAMINE RECEPTOR AND BRADYKININ
ANTAGONISTS
Hydroxyzine (VISTARIL)
Icatibant (FIRAZYR) DRUG CLASS DRUG MECHANISM OF ACTION

Ketoti en umarate (ZADITOR) First-Generation Doxepin Act as inverse agonists to


H1-Receptor Antagonists Carbinoxamine reduce the constitutive
Levocabastine (LIVOSTIN) Clemastine umarate activity o H1 receptors and
Levocetirizine (XYZAL) Diphenhydramine compete with histamine at
Dimenhydrinate (combination H1 receptors on target tissues,
Loratadine (CLARITIN)
o diphenhydramine and including smooth muscle,
Meclizine (ANTIVERT,others) 8-chlorotheophylline) capillaries, peripheral nerve
Mizolastine (MIZOLLEN) Pyrilamine endings, and some exocrine
Tripelennamine glands
Olopatadine (PATANOL, PATANASE, others) Chlorpheniramine First-generation H1
Phenindamine (NOLAHIST) Brompheniramine maleate antagonists can cross the
Hydroxyzine blood-brain barrier and
Promethazine (PHENERGAN, others) Cyclizine both stimulate and depress
Pyrilamine (POLYHISTINE-D*) Meclizine the CNS, though central
Promethazine depression is more common
Tripelennamine (PBZ)
Cyproheptadine Many rst-generation
*Trade-namedrugthat containsothermedications. Phenindamine H1 antagonists have
anticholinergic e ects
mediated through muscarinic
receptors (see Chapter 6)
(Continued)

346
Histamine, Bradykinin, and Their Antagonists CHAPTER 2 1

DRUG CLASS DRUG MECHANISM OF ACTION TRIPLE RESPONSE


Second-Generation Olopatadine Act as inverse agonists to OF LEWIS
H1-Receptor Antagonists Acrivastine reduce the constitutive
Intradermal injection o histamine elicits a
Cetirizine activity o H1 receptors and
Levocetirizine compete with histamine at characteristicphenomenon known as the
Azelastine H1 receptors on target tissues, triple response rst described in 1927 by
Levocabastine including smooth muscle, Lewis. The triple response consists o :
Ketoti en umarate capillaries, peripheral nerve Alocalized red spot extending or a ew
Loratadine endings, and some exocrine
millimeters around the site o injection
Desloratadine glands
Ebastine that appears within a ewseconds and
Mizolastine reaches a maximumin ~1 minute
Fexo enadine The initial red spot results romthe
H2-Receptor Antagonists See Chapter 32 Inhibit H2 receptors in the GI
direct vasodilatorye ect o histamine
system which reduces gastric (H1 receptormediated NOproduction)
secretion (see Chapter 32) Abrighter red fush, orfare,extending
~1 cmbeyond the original red spot and
H3-Receptor Antagonists No agents are currently Inhibit H3 receptors
available (several are currently which are presynaptic developing more slowly
in clinical trials) autoreceptors on The fare is due to histamine-induced
histaminergic neurons stimulation o axon refexes that cause
in the CNS and indirect vasodilation
enterochroma n-like
cells o the stomach Awheal that is discernible in 1 to 2 min-
utes and occupies the same area as the
H4 receptor antagonists No agents are currently Inhibit H4 receptor which original small red spot at the injection site
available is expressed on cells with
inf ammatory or immune The wheal refects histamines capacity
unctions to increase capillarypermeability
(edema ormation)
Kallikrein Inhibitor Aprotinin (no longer available) Inhibit the proteinase activity
o kallikreins, thus inhibiting
the production o kallidin and
bradykinin rom kininogens
FIRST-GENERATION
(see Figure 32-4 in Goodman & VERSUS SECOND-
Gilmans The Pharmacological GENERATION H1
Basis of Therapeutics, 12th ANTIHISTAMINES
Edition)
Side e ects are most prominent with rst-
Kinin Receptor Antagonists Icatibant (only approved in Inhibit B1 and B2 (bradykinin) generation H1 antihistamines, which cross
Europe) receptors
the blood-brain barrier and cause sedation.
Some o the rst-generation H1-receptor
antagonists also have anticholinergic
properties that can be responsible or
CASE 21-1 symptoms such as dryness o the mouth
A 24-year-old woman has seasonal allergies in the summer due to various pollens and respiratorypassages, urinaryretention
and grasses Her symptoms include a runny nose and itchy, watery eyes At the begin- or requency, and dysuria (see Chapter 6).
ning o the summer she takes over-the-counter (O C) diphenhydramine or relie o The signi cant sedative e ects o some
symptoms rst-generation antihistamines have led
a. What is the mechanism o action o diphenhydramine in treating this patients to their use in treating insomnia, although
symptoms? better drugs are available (see Chapter 9).
Diphenhydramine is a rst-generation H 1-receptor antagonist. H1 antagonists are most Some rst-generation H1 antagonists
use ul in acute types o allergy that present with symptoms o rhinitis, urticaria, and (eg, dimenhydrinate, cyclizine, meclizine, and
conjunctivitis. T eir e ect is con ned to the suppression o symptoms attributable to promethazine) can prevent motionsickness,
the histamine released by the antigen-antibody reaction. T ese drugs relieve the sneez- although scopolamine is more e ective.
ing, rhinorrhea, and itching o eyes, nose, and throat. A grati ying response is obtained Antiemetice ects o these H1
in most patients, especially at the beginning o the season when pollen counts are antihistamines can be bene cial in
low; however, the drugs are less e ective when the allergens are most abundant, when treating vertigo or postoperative emesis.
exposure to them is prolonged, and when nasal congestion is prominent. (continues)
(Continued)

347
SECTION IV Inf ammation, Immunomodulation, and Hematopoiesis

b. She nds the diphenhydramine causes her to be drowsy. Why does this agent
FIRST-GENERATION
cause drowsiness?
VERSUS SECOND-
GENERATION H1 T e rst-generation H 1 antagonists can cross the blood-brain barrier and can both
stimulate and depress the CNS. T e most requent side e ect in rst-generation H 1
ANTIHISTAMINES (Cont.)
antagonists is sedation and usually accompanies therapeutic doses o the older H 1
ManyH1 antihistamines (both rst and sec- antagonists. Diminished alertness, slowed reaction times, and somnolence are com-
ond generation) are metabolized byCYPs; mon mani estations o the central CNS depression caused by these agents. Concur-
thus, inhibitors o CYPactivitycan increase rent ingestion o alcohol or other CNS depressants produces an additive e ect that
H1 antihistamine levels, leading to toxicity. impairs motor skills. Patients vary in their susceptibility and responses to individual
Certain H1 antihistamines, especially rst- drugs. T e ethanolamines, which include diphenhydramine, are particularly prone
generation drugs, are possible teratogens to causing sedation. Because o the sedation that occurs with rst-generation anti-
or cause symptomatice ects in in ants histamines, these drugs cannot be tolerated or used sa ely by many patients except
resulting romsecretion o the drug into at bedtime. Even then, patients may experience an antihistamine hangover in
breast milk. the morning, resulting in sedation with or without psychomotor impairment. CNS
Cetirizine and loratadine are pre erred stimulation occasionally is encountered in patients given conventional doses o
in pregnant women i H1 antihistamines rst-generation H 1 antagonists; they become restless, nervous, and unable to sleep.
are required, but i theyare not Central excitation also is a striking eature o overdose, which commonly results
e ective, diphenhydramine can be in convulsions, particularly in in ants. Other untoward central actions caused by
used sa elyin pregnant (but not breast- rst-generation H 1 antagonists include dizziness, tinnitus, lassitude, incoordination,
eeding) women. atigue, blurred vision, diplopia, euphoria, nervousness, insomnia, and tremors.
c. What alternatives does she have to treat her allergy symptoms without getting
drowsy?
T e second-generation H 1 antagonists (eg, levocetirizine, cetirizine, loratadine,
desloratadine, exo enadine) are largely devoid o these side e ects because they do
not penetrate the central nervous system (CNS). T us, they usually are the drugs o
choice or the treatment o allergic disorders.
d. She plans to get pregnant and is concerned about whether she can treat her
allergies when she is pregnant. What are her options?
Caution should be used in treating pregnant or lactating women with certain H 1
antihistamines, especially rst-generation drugs, because o their possible terato-
genic e ects or symptomatic e ects on in ants resulting rom secretion o the drug
into breast milk. Cetirizine and loratadine are pre erred i H 1 antihistamines are
required, but i they are not e ective, diphenhydramine can be used sa ely in preg-
nant (but not breast- eeding) women.

CASE 21-2
A 32-year-old man visits his dermatologist because he is very sensitive to mosquito bites
When bitten by a mosquito, he immediately experiences redness at the site o the bite
T e redness gradually grows to about the size o a dime and a small itchy bump orms
a. What is causing the progression in this patients symptoms ollowing a
mosquito bite?
T e mosquito bite is causing the release o histamine rom mast cells in the skin that
results in a classic triple response (see Side Bar RIPLE RESPONSE OF LEWIS)
o redness, are, and wheal characteristic o intradermal injection o histamine.
b. What can be done to treat this patients mosquito bites?
H 1 antihistamines can e ectively attenuate or block all o the reactions to the mos-
quito bite. opical preparations o H 1 antihistamines are available (see able 32-2 in
Goodman & Gilmans T e Pharmacological Basis of T erapeutics, 12th Edition) that
can be applied to the mosquito bite to relieve itching and reduce the redness and
wheal. However, there is a possibility o producing allergic dermatitis due to the
development o drug allergy with topical application o H 1 antagonists. T e patient
may also bene t rom taking an oral antihistamine i he receives a number o bites,
although relie o symptoms may take 1 hour or longer.
(Continued)
348
Histamine, Bradykinin, and Their Antagonists CHAPTER 2 1

CASE 21-3
A 57-year-old woman with a history o allergies to sea ood suddenly develops swelling
o her lips and ace while dining in a restaurant She also begins to eel light-headed,
and her tongue and mouth begin to itch
a. What is likely causing this womans symptoms?
Her symptoms are consistent with an allergic (ie, immediate hypersensitivity) reac-
tion, perhaps due to the presence o sea ood in something she has just eaten at the
restaurant. T e principal target cells o immediate hypersensitivity reactions are
mast cells and basophils. As part o the allergic response to an antigen, immuno-
globulin E (IgE) antibodies are generated and bind to the sur aces o mast cells and
basophils via speci c high-a nity Fc receptors. Antigen bridges the IgE molecules
and via FcRI activates signaling pathways in mast cells or basophils involving tyro-
sine kinases and subsequent phosphorylation o multiple protein substrates within
5 to 15 seconds o contact with antigen. T ese events trigger the exocytosis o the
contents o secretory granules, which contain histamine. A er its release rom
storage granules, histamine plays a central role in immediate hypersensitivity and
allergic responses. T e actions o histamine on bronchial smooth muscle and blood
vessels account or many, but not all, o the symptoms o the allergic response. A
broad spectrum o other in ammatory mediators is released on mast cell activation
including platelet-activating actor (PAF) and metabolites o arachidonic acid such
as leukotrienes C4 and D4, which contract the smooth muscles o the bronchial tree.
Kinins also are generated during some allergic responses.
T e e ects o histamine in an allergic reaction are largely mediated by its activation
o H 1 receptors which are distributed widely in the periphery. Histamine characteris-
tically dilates resistance vessels, increases capillary permeability, and lowers systemic
blood pressure. Vasodilation is the most important vascular e ect o histamine in
humans. Activation o either the H 1 or H 2 receptor can elicit maximal vasodilation,
but the responses di er. H 1 receptors have a higher a nity or histamine and cause
Ca2+-dependent activation o endothelial NOS (eNOS) in endothelial cells; NO
di uses to vascular smooth muscle, increasing cyclic guanosine monophosphate
(cyclic GMP) (see able 321 in Goodman & Gilmans T e Pharmacological Basis of
T erapeutics, 12th Edition) and causes relaxation that results in a relatively rapid and
short-lived vasodilation. By contrast, activation o H 2 receptors on vascular smooth
muscle stimulates the cyclic AMPPKA pathway, causing dilation that develops
more slowly and is more sustained. As a result, H 1 antagonists e ectively counter
small dilator responses to low concentrations o histamine but only blunt the initial
phase o larger responses to higher concentrations o the amine. Histamines e ect
on small vessels results in ef ux o plasma protein and uid into the extracellular
spaces and an increase lymph ow, causing edema. H 1 receptors on endothelial cells
are the major mediators o this response. Increased capillary permeability results
rom histamine activation o H 1 receptors on postcapillary venules, which causes
contraction o the endothelial cells, disrupts interendothelial junctions, and exposes
the basement membrane, which is reely permeable to plasma proteins and uid.
b. T e woman recognizes the symptoms o an allergic reaction and takes some
diphenhydramine, which she always carries with her (as well as an EPI-PEN
autoinjector). T e swelling in her throat continues and she begins having
di culty breathing so she sel -administers epinephrine using the EPI-PEN.
Why did she need epinephrine in addition to diphenhydramine?
During hypersensitivity reactions, histamine is one o the many potent autacoids
released, and its relative contribution to the ensuing symptoms varies widely.
Edema ormation and itch are e ectively suppressed by H 1 antagonists such as
diphenhydramine, but other e ects, such as hypotension, are less well antagonized.
T is may be explained by the participation o other types o H receptors and by
e ects o other mast cell mediators, chie y eicosanoids (see Chapter 22).
(Continued)
349
SECTION IV Inf ammation, Immunomodulation, and Hematopoiesis

Bronchoconstriction, which is causing the patients di culty in breathing, is


reduced little, i at all by antihistamines. T e bene cial e ects o adrenergic
agonists such as epinephrine in allergic states such as asthma and anaphylaxis are
due mainly to relaxing bronchial smooth muscle. A er she sel -administers her
epinephrine, the patient should seek medical attention immediately. T e EPI-PEN
is designed to be an e ective, but short-term measure to allow the user time to
reach an emergency room where additional epinephrine and other drugs such as
corticosteroids may be needed to treat the anaphylactic reaction.

CASE 21-4
A patient who is prone to motion sickness will be taking a small erry to reach an
island destination on her vacation travels
a. She would like to take something to prevent the motion sickness on the erry
ride. What are her options?
Many o the rst-generation H 1 antagonists have antimuscarinic actions in the CNS
that can be used to treat motion sickness (see Chapter 9). Cyclizine and meclizine
are used primarily to prevent motion sickness and vertigo although promethazine
and diphenhydramine are more e ective. None o these agents are as e ective as the
antimuscarinic agent scopolamine, which is the most e ective drug or the prophy-
laxis and treatment o motion sickness. However, the H 1 antagonists are use ul or
milder cases o motion sickness and have ewer adverse e ects. Promethazine is the
most potent and e ective o the H 1 antagonists in preventing motion sickness, and
its additional antiemetic properties may be valuable in reducing vomiting.
b. When should she take the medication?
Whenever possible, the various drugs should be administered ~1 hour be ore the
anticipated motion. reatment a er the onset o nausea and vomiting rarely is
bene cial.
c. What side ef ects should she expect with the H1 antagonists used or motion
sickness?
Sedation is to be expected with all o these agents.

CASE 21-5
A 23-year old woman has a 2-year old son and both she and her son have symptoms
o a cold, including cough, runny nose, sneezing, and watery eyes She nds an O C
cough and cold medication at the grocery store that is advertised to be e ective in
relieving their symptoms
a. When she gets home rom the store she reads the label more care ully and sees
that promethazine is one o the active ingredients in the cough and cold medica-
tion she purchased. She also reads a warning Do Not Use in Children 4 Years
and Younger. What is the reason or this warning?
Use o O C cough and cold medicines (containing mixtures o antihistamines,
decongestants, antitussives, expectorants) in young children has been associated
with serious side e ects and deaths. In 2008, the Food and Drug Administration
(FDA) recommended that they not be used in children younger than 2 years, and
drug manu acturers a liated with the Consumer Healthcare Products Association
voluntarily relabeled products do not use or children younger than 4 years. T e
FDA is reviewing the sa ety o these medicines in children aged 2 to 11 years.
b. What value does promethazine have in this cold medication?
H 1 antagonists are without value in combating the causes o the common cold. T e
weak anticholinergic e ects o the older agents may tend to lessen rhinorrhea, but
this drying e ect may do more harm than good, as may their tendency to induce
somnolence.

350
Histamine, Bradykinin, and Their Antagonists CHAPTER 2 1

KEY CONCEPTS
H 1 antagonists are most use ul in acute types o allergy that present with
symptoms o rhinitis, urticaria, and conjunctivitis
T eir e ect is con ned to the suppression o symptoms attributable to the
histamine released by the antigen-antibody reaction
In the treatment o systemic anaphylaxis, where autacoids other than histamine
are important, the mainstay o therapy is epinephrine; histamine antagonists
have only a subordinate and adjuvant role
Histamine antagonists have limited ef cacy in treating bronchial asthma and
are not used as sole therapy
Because o their e ects on the CNS, the rst-generation H 1 antagonists also
have therapeutic value in suppressing motion sickness or or sedation
Second-generation drugs are usually the drugs o choice or the treatment o
allergic disorders as they are largely devoid o sedating side e ects because
they do not penetrate the CNS and do not have antimuscarinic properties

SUMMARY QUIZ

QUESTION 21-1 T is antihistamine has antiemetic e ects use ul in treating vertigo or


postoperative emesis
a Cetirizine
b Loratadine
c Fexo enadine
d Promethazine
e None o the above

QUESTION 21-2 T is antihistamine is approved or use in children


a Cetirizine
b Meclizine
c Promethazine
d Diphenhydramine
e None o the above

QUESTION 21-3 T e H 1 antihistamines are highly e ective in treating this histamine-


mediated process in humans
a Gastric secretions
b Anaphylactic bronchospasm
c Allergic urticaria
d Bronchial asthma
e Severe angioedema

QUESTION 21-4 T e antihistamine o choice in treating seasonal allergy symptoms in


a pregnant woman is
a cetirizine
b meclizine
c promethazine
d dimenhydrinate
e none o the above

351
SECTION IV Inf ammation, Immunomodulation, and Hematopoiesis

QUESTION 21-5 T e antihistamine o choice in preventing motion sickness is


a cetirizine
b loratadine
c exo enadine
d promethazine
e none o the above

SUMMARY QUIZ ANSWER KEY


QUESTION 21-1 Answer is d. T e antiemetic e ects o some rst-generation H 1
antagonists (eg, dimenhydrinate, cyclizine, meclizine, and promethazine) can be ben-
e cial in treating vertigo or postoperative emesis Dimenhydrinate and meclizine o en
are o bene t in vestibular disturbances such as Mnire disease and in other types o
true vertigo Only promethazine is use ul in treating the nausea and vomiting subse-
quent to chemotherapy or radiation therapy or malignancies; however, other, more
e ective antiemetic drugs (eg, 5-H 3 antagonists) are available (see Chapter 33)

QUESTION 21-2 Answer is a. T e second-generation H 1 antagonistsloratadine,


desloratadine, exo enadine, cetirizine, levocetirizine, and azelastine (intranasal)have
been approved by the FDA or use in children and are available in appropriate lower-
dose ormulations (eg, chewable or rapidly dissolving tablets, syrup) First-generation
antihistamines are not recommended or use in children because their sedative e ects
can impair learning and school per ormance

QUESTION 21-3 Answer is c. H 1 antagonists are most use ul in acute types o allergy
that present with symptoms o rhinitis, urticaria, and conjunctivitis T eir e ect is
con ned to the suppression o symptoms attributable to the histamine released by the
antigen-antibody reaction In bronchial asthma, histamine antagonists have limited
ef cacy and are not used as sole therapy In the treatment o systemic anaphylaxis,
where autacoids other than histamine are important, the mainstay o therapy is epi-
nephrine; histamine antagonists have only a subordinate and adjuvant role T e same is
true or severe angioedema, in which laryngeal swelling constitutes a threat to li e

QUESTION 21-4 Answer is a. Caution should be used in treating pregnant or lactat-


ing women with certain H 1 antihistamines, especially rst-generation drugs, because o
their possible teratogenic e ects or symptomatic e ects on in ants resulting rom secre-
tion o the drug into breast milk Cetirizine and loratadine are pre erred i H 1 antihista-
mines are required, but i they are not e ective, diphenhydramine can be used sa ely in
pregnant (but not breast- eeding) women

QUESTION 21-5 Answer is d. Some rst-generation H 1 antagonists are use ul or


preventing motion sickness and have ewer adverse e ects than scopolamine, the
most e ective drug or the prophylaxis and treatment o motion sickness T ese anti-
histamines include dimenhydrinate and the piperazines (eg, cyclizine, meclizine)
Promethazine, a phenothiazine, is more potent and more e ective than the other anti-
histamines; its additional antiemetic properties may be o value in reducing vomiting,
but its pronounced sedative action usually is disadvantageous

352
Histamine, Bradykinin, and Their Antagonists CHAPTER 2 1

SUMMARY: HISTAMINE RECEPTOR AND BRADYKININ ANTAGONISTS


TOXICITIES
CLASS AND UNIQUE; CLINICALLY
SUBCLASSES NAMES CLINICAL USES COMMON IMPORTANT
First-Generation H1- Doxepin Treatment o allergic disorders, Can cause drowsiness Not recommended or
Receptor Antagonists relieving the symptoms o seasonal and is associated with use in children because
Tricyclic Dibenzoxepins rhinitis and conjunctivitis anticholinergic e ects their sedative e ects can
Limited bene t in bronchial asthma Even small doses (eg, impair learning and school
Use ul adjuncts to epinephrine in the 20 mg) may cause per ormance
treatment o systemic anaphylaxis or disorientation and OTC cough and cold
severe angioedema con usion, but better medicines (containing
Used to treat certain allergic tolerated by patients mixtures o antihistamines,
dermatoses with depression than and other drugs) have
those who are not been associated with
depressed serious side e ects and
Loss o appetite, deaths in young children
nausea, vomiting, Do not use in children
epigastric distress, and <4 years o age
constipation or diarrhea
(taking the drug with
meals may reduce
incidence o GI e ects)
First-Generation H1- Carbinoxamine Treatment o allergic disorders, Possess signi cant Not recommended or
Receptor Antagonists Clemastine umarate relieving the symptoms o seasonal antimuscarinic activity use in children because
Ethanolamines Diphenhydramine rhinitis and conjunctivitis and have a pronounced their sedative e ects can
Dimenhydrinate Limited bene t in bronchial asthma tendency to induce impair learning and school
(combination o Use ul adjuncts to epinephrine in the sedation per ormance
diphenhydramine and treatment o systemic anaphylaxis or About hal o those OTC cough and cold
8-chlorotheophylline) severe angioedema treated acutely medicines (containing
Used to treat certain allergic with conventional mixtures o antihistamines,
dermatoses doses experience and other drugs) have
Diphenhydramine can be used sa ely somnolence and been associated with
in pregnant (but not breast- eeding) cannot be tolerated serious side e ects and
women or used sa ely by deaths in young children
Dimenhydrinate can prevent motion many patients except Do not use in children
sickness at bedtime; patients <4 years o age
may experience
an antihistamine
hangoverin the
morning, resulting
in sedation with or
without psychomotor
impairment
GI side e ects
First-Generation H1- Pyrilamine Treatment o allergic disorders, Central e ects are Not recommended or
Receptor Antagonists Tripelennamine relieving the symptoms o seasonal relatively eeble, but use in children because
Ethylenediamines rhinitis and conjunctivitis somnolence occurs their sedative e ects can
Limited bene t in bronchial asthma in a air proportion o impair learning and school
Use ul adjuncts to epinephrine in the patients per ormance
treatment o systemic anaphylaxis or GI side e ects are quite OTC cough and cold
severe angioedema common medicines (containing
Used to treat certain allergic mixtures o antihistamines,
dermatoses and other drugs) have
been associated with
serious side e ects and
deaths in young children
Do not use in children
<4 years o age

353
SECTION IV Inf ammation, Immunomodulation, and Hematopoiesis

TOXICITIES
CLASS AND UNIQUE; CLINICALLY
SUBCLASSES NAMES CLINICAL USES COMMON IMPORTANT
First-Generation H1- Chlorpheniramine Treatment o allergic disorders, Less prone to produce Not recommended or
Receptor Antagonists Brompheniramine relieving the symptoms o seasonal drowsiness and are use in children because
Alkylamines maleate rhinitis and conjunctivitis more suitable or their sedative e ects can
Limited bene t in bronchial asthma daytime use, but a impair learning and school
Use ul adjuncts to epinephrine in the signi cant proportion per ormance
treatment o systemic anaphylaxis or o patients experience OTC cough and cold
severe angioedema sedation medicines (containing
Used to treat certain allergic Side e ects involving mixtures o antihistamines,
dermatoses CNS stimulation are and other drugs) have
more common than been associated with
rst-generation agents serious side e ects and
rom other chemical deaths in young children
structure groups Do not use in children <4
GI side e ects years o age
First-Generation H1- Hydroxyzine Treatment o allergic disorders, Considerable CNS- Not recommended or
Receptor Antagonists Cyclizine relieving the symptoms o seasonal depressant activity and use in children because
Piperazines Meclizine rhinitis and conjunctivitis antimuscarinic e ects their sedative e ects can
Limited bene t in bronchial asthma Loss o appetite, impair learning and school
Use ul adjuncts to epinephrine in the nausea, vomiting, per ormance
treatment o systemic anaphylaxis or epigastric distress, and OTC cough and cold
severe angioedema constipation or diarrhea medicines (containing
Hydroxyzine is a long-acting (taking the drug with mixtures o antihistamines,
compound widely used or skin meals may reduce and other drugs) have
allergies; its considerable CNS- incidence o GI e ects) been associated with
depressant activity may contribute to serious side e ects and
its prominent antipruritic action deaths in young children
Cyclizine and meclizine are used Do not use in children
primarily to prevent motion sickness <4 years o age
and vertigo although promethazine
and diphenhydramine are more
e ective
First-generation H1- Promethazine Treatment o allergic disorders, Considerable CNS- Not recommended or
Receptor Antagonists relieving the symptoms o seasonal depressant activity and use in children because
Phenothiazines rhinitis and conjunctivitis antimuscarinic e ects their sedative e ects can
Limited bene t in bronchial asthma Loss o appetite, impair learning and school
Use ul adjuncts to epinephrine in the nausea, vomiting, per ormance
treatment o systemic anaphylaxis or epigastric distress, and OTC cough and cold
severe angioedema constipation or diarrhea medicines (containing
Used to treat certain allergic (taking the drug with mixtures o antihistamines,
dermatoses meals may reduce and other drugs) have
Promethazine is the most e ective incidence o GI e ects) been associated with
H1 antagonist in combating motion serious side e ects and
sickness because o its antimuscarinic deaths in young children
e ects Do not use in children
<4 years o age
First-Generation H1- Cyproheptadine Treatment o allergic disorders, Cause drowsiness, Not recommended or
Receptor Antagonists Phenindamine relieving the symptoms o seasonal have signi cant use in children because
Piperidines rhinitis and conjunctivitis anticholinergic e ects their sedative e ects can
Limited bene t in bronchial asthma Can increase appetite impair learning and school
Use ul adjuncts to epinephrine in the and cause weight gain per ormance
treatment o systemic anaphylaxis or OTC cough and cold
severe angioedema medicines (containing
Used to treat certain allergic mixtures o antihistamines,
dermatoses and other drugs) have
been associated with
serious side e ects and
deaths in young children
Do not use in children
<4 years o age

354
Histamine, Bradykinin, and Their Antagonists CHAPTER 2 1

TOXICITIES
CLASS AND UNIQUE; CLINICALLY
SUBCLASSES NAMES CLINICAL USES COMMON IMPORTANT
Second-Generation H1- Olopatadine Usually drugs o choice or treatment Somewhat higher
Receptor Antagonists o allergic disorders, relieving the incidence o drowsiness
Tricyclic Dibenzoxepins symptoms o seasonal rhinitis and than the other
conjunctivitis second-generation H1
Limited bene t in bronchial asthma antagonists
Use ul adjuncts to epinephrine in the
treatment o systemic anaphylaxis or
severe angioedema
Used to treat certain allergic
dermatoses
Second-Generation H1- Acrivastine Usually drugs o choice or treatment Lack signi cant
Receptor Antagonists o allergic disorders, relieving the anticholinergic actions;
Alkylamines symptoms o seasonal rhinitis and penetrate poorly
conjunctivitis into the CNS, and
Limited bene t in bronchial asthma low incidence o side
Use ul adjuncts to epinephrine in the e ects
treatment o systemic anaphylaxis or
severe angioedema
Used to treat certain allergic
dermatoses
Second-Generation H1- Cetirizine Usually drugs o choice or treatment Lack signi cant
Receptor Antagonists Levocetirizine o allergic disorders, relieving the anticholinergic actions;
Piperazines symptoms o seasonal rhinitis and penetrate poorly
conjunctivitis into the CNS, and
Limited bene t in bronchial asthma low incidence o side
Use ul adjuncts to epinephrine in the e ects
treatment o systemic anaphylaxis or
severe angioedema
Used to treat certain allergic
dermatoses
Cetirizine is a pre erred drug in
pregnant or breast- eeding women i
H1 antihistamines are required
Second-Generation H1- Azelastine Usually drugs o choice or treatment Lack signi cant
Receptor Antagonists o allergic disorders, relieving the anticholinergic actions;
Phthalazines symptoms o seasonal rhinitis and penetrate poorly
conjunctivitis into the CNS, and
Limited bene t in bronchial asthma low incidence o side
Use ul adjuncts to epinephrine in the e ects
treatment o systemic anaphylaxis or
severe angioedema
Used to treat certain allergic dermatoses
Second-Generation H1- Levocabastine Usually drugs o choice or treatment Lack signi cant
Receptor Antagonists Ketoti en umarate o allergic disorders, relieving the anticholinergic actions;
Piperidines Loratadine symptoms o seasonal rhinitis and penetrate poorly
Desloratadine conjunctivitis into the CNS, and
Ebastine Limited bene t in bronchial asthma low incidence o side
Mizolastine Use ul adjuncts to epinephrine in the e ects
Fexo enadine treatment o systemic anaphylaxis or
severe angioedema
Used to treat certain allergic
dermatoses
Loratadine is a pre erred drug in
pregnant or breast- eeding women i
H1 antihistamines are required
H2-Receptor Antagonists See Chapter 32 Inhibit gastric secretion See Chapter 32 See Chapter 32
Kinin Receptor Icatibant Treatment o acute episodes o
AntagonistsB2 swelling in patients with hereditary
(Bradykinin) Antagonist angioedema

355
CHAPTER

22 Prostaglandins, NSAIDs, and


Pharmacotherapyof Gout
T is chapter will be most use ul a er having a basic understanding o the material
DRUGS INCLUDED IN THIS
in Chapter 33, Lipid-Derived Autacoids: Eicosanoids and Platelet-Activating
CHAPTER Factor, and Chapter 34, Anti-in ammatory, Antipyretic, and Analgesic Agents:
Acetaminophen (TYLENOL, others) Pharmacotherapy o Gout in Goodman & Gilmans T e Pharmacological Basis of
Allopurinol (ZYLOPRIM, ALOPRIM, others) T erapeutics, 12th Edition. In addition to the material presented here, the 12th
Edition contains:
Alprostadil (CAVERJECT,EDEX,MUSE,
PROSTINVRPEDIATRIC) able 33-1 Eicosanoid Receptors that lists the major classes o eicosanoid receptors
and their signaling characteristics
Aspirin (acetylsalicylicacid, ASA)
T e molecular structures o lipid-derived autacoids, nonsteroidal anti-in ammatory
Balsalazide (COLAZOL, others)
drugs (NSAIDs), and drugs used to treat gout
Bimatoprost (LUMIGAN)
Brom enac(XIBROM)
LEARNING OBJECTIVES
Carboprost tromethamine (HEMABATE)
Understand the mechanisms o action o drugs used as prostaglandin
Celecoxib (CELEBREX)
agonists.
Colchicine
Understand the mechanisms o action o the NSAIDs.
Diclo enac(CATAFLAM, ZIPSOR, FLECTOR,
VOLTAREN, SOLAREZE, CAMBIA, others) Understand the mechanisms o action o the drugs used in the pharmacother-
apy o gout.
Di unisal
Know the untoward e ects o prostaglandin agonists, NSAIDs, and the drugs
Dinoprostone (CERVIDIL, PREPIDIL)
used in the pharmacotherapy o gout.
Etodolac(LODINE)
Know the therapeutic utility o prostaglandin agonists, NSAIDs, and the drugs
Febuxostat (ULORIC) used in the pharmacotherapy o gout.
Fenopro en (NALFON200, others)
Know which drugs can be used in combination, and those that should not
Flurbipro en (ANSAID, OCUFEN, others) be used concomitantly to treat in ammatory disorders, ever, pain, arthritis,
Ibupro en (ADVIL, MOTRIN, NEOPROFEN, and gout.
CALDOLOR, others)
Iloprost (VENTAVIS) MECHANISMS OF ACTION OF PROSTAGLANDINS AND THEIR ANALOGS
Indomethacin (INDOCIN, others) DRUG CLASS DRUG MECHANISM OF ACTION
Ketopro en
PGE1 Alprostadil Activates EP prostaglandin receptors in vascular smooth
Ketorolac(ACULAR, others) muscle causing vasodilation
Latanoprost (XALATAN) The ductus arteriosus in neonates is highly sensitive to
relaxation by PGE1
Meclo enamate (MECLOMEN)
Me enamicacid (PONSTEL) PGE1 Analog Misoprostol Activates EP prostaglandin receptors in the stomach,
increasing mucous secretion and reducing acid secretion
Meloxicam(MOBIC, others) and pepsin content
Mesalamine (ASACOL, LIALDA, APRISO,
PGE2 Dinoprostone Activates EP prostaglandin receptors in the uterus and
DIPENTUM, CANASA) cervix, promoting uterine contractions, cervical ripening,
Misoprostol (CYTOTEC) and cervical dilation
Nabumetone (RELAFEN) PGF2 Analog Carboprost Activates FP prostaglandin receptors in the uterus and
Naproxen (ALEVE, NAPROSYN, others) tromethamine cervix, promoting uterine contractions, cervical ripening,
Latanoprost and cervical dilation
Nepa enac(NEVANAC)
Bimatoprost Activates FP prostaglandin receptors in the eye, causing
Olsalazine Travoprost a decrease in intraocular pressure by increasing aqueous
Oxaprozin (DAYPRO) humor out ow via the uveoscleral and trabecular
(continues) meshwork pathway
(Continued)

356
Prostaglandins, NSAIDs, and Pharmacotherapy o Gout CHAPTER 2 2

DRUG CLASS DRUG MECHANISM OF ACTION DRUGS INCLUDED IN THIS


PGI2 Prostacyclin Activates IP prostaglandin receptors in pulmonary artery CHAPTER (Cont.)
smooth muscle, causing vasodilation
Piroxicam(FELDENE, others)
PGI2 Analog Iloprost Activates IP prostaglandin receptors in pulmonary artery Probenecid (PROBALAN)
Treprostinil smooth muscle, causing vasodilation
Prostacyclin (epoprostenol, FLOLAN)
Rasburicase (ELITEK)
Salicylate (DOANS, MOMENTUM, others)
Salsalate
MECHANISMS OF ACTION OF NSAIDs Sul asalazine (AZULFIDINE, others)
DRUG CLASS DRUG MECHANISM OF ACTION Sulindac(CLINORIL, others)
NSAIDIrreversible COX-1 Aspirin Covalently acetylates Tolmetin (TOLECTIN, others)
and COX-2 inhibitor COX-1 and COX-2
Travoprost (TRAVATAN)
impeding the ability o
these enzymes to make Treprostinil (REMODULIN)
prostaglandins (see
Figure 22-1)
Does not inhibit the LOX
pathway o arachidonic THE INFLAMMATORY
acid metabolism (see RESPONSE
Figure 22-2)
In ammation can be evoked bya wide
Traditional NSAID (tNSAID) Salsalate Reversible, competitive, varietyo noxious agents (eg, in ections,
Salicylate active site inhibitors o antibodies, physical injuries).
Di unisal COX-1 and COX-2 (see
Mesalamine Figure 22-1), though The classicin ammatorysymptoms
Sul asalazine each agent varies in COX include calor (warmth), dolor (pain), rubor
Balsalazide iso orm selectivity (eg, (redness), and tumor (swelling).
Olsalazine meloxicam, diclo enac,
Manymediators are involved in the in am-
Acetaminophen and etodolac are more
Indomethacin COX-2 selective; see matoryresponse:
Sulindac Figure 22-4) Histamine (see Chapter 21)
Etodolac Does not inhibit the LOX Bradykinin (see Chapter 21)
Tolmetin pathway o arachidonic
Ketorolac acid metabolism (see 5-hydroxytryptamine (5-HT,serotonin)
Diclo enac Figure 22-2) Prostanoidsproducedbycyclooxygenase-1
Brom enac
(COX-1) andcyclooxygenase-2(COX-2)
Nepa enac
Me enamic acid (see Figure 22-1), inparticular, PGE2
Meclo enamate andPGI2
Ibupro en Leukotrienes (LTs; see Figure 22-2)
Naproxen
Fenopro en
Platelet-activating actor (PAF; see
Ketopro en Figure 22-3)
Flurbipro en Other soluble mediators including
Oxaprozin complement actor C5a, proin ammatory
Piroxicam
cytokines (TNF,IL-1, IL-2, IL-6, IL-8; see
Meloxicam
Nabumetone Chapters 23and 25), and growth actors
(GM-CSF; see Chapter 25)
NSAIDSelective COX-2 Celecoxib Selective inhibition COX-2, which is rapidlyinduced during
Inhibitor (coxib) o COX-2, preserving
COX-1 synthesis o
in ammation, is the major source o
cytoprotective PGE2 and proin ammatoryprostanoids.
PGI2 by gastric epithelium COX-1 plays a dominant role in producing
proin ammatoryprostanoids during the
initial phase o an acute in ammatory
response.

357
SECTION IV Inf ammation, Immunomodulation, and Hematopoiesis

COOH
Arac hido nic Ac id Endoca nna ba noids :
CYPs COOH
Ara chidonyle tha nola mide
O 11,12-EET 2-a ra chidonoylglyce rol
COOH c yclo -
LOXs oxyg e nas e ca nna ba noid
re ce ptors
O 14,15-EET
COX-2
O 5,6-EET
COOH
5-LOX/COX COX-1 As pirin Glyce ryl
Inhibitors COX-2 tNSAIDs pros ta gla ndins
O 8,9-EET e .g.:
COOH Lic ofe lone
s e le ctive ac e tylate d
COX-2 inhibitors COX-2
e .g.: c oxib s
Epi-lipoxins
Fre e radic al O (S e e Figure 22-2)
attac k COOH
O
HETEs PGG2
Le ukotrie ne s hydro - OOH
pe roxidas e COX-1
Lipoxins COX-2
Is opros ta ne s (S e e Figure 22-2)
O
COOH
O
TXA s ynthas e
OH PGH2

L-PGD PGI s ynthas e


COOH s ynthas e
O COOH
O H-PGD
TxA2
OH s ynthas e
PGF O PGI2
mPGE s ynthas e s ynthas e
c PGE s ynthas e

OH OH
HO
O
COOH COOH

O PGD2
HO OH PGE2 OH

HO

COOH COOH

PGF2 O 15de oxy12,14 PGJ 2


HO OH

TP , EP 1 EP 2 EP 3A-D EP 4 FP DP DP 2 IP

FIGURE 22-1 Metabolism o arachidonic acid (AA). The cyclooxygenase (COX) pathway is highlighted in gray. The lipoxygenase (LOX) path-
ways are expanded in Figure 22-2. Cyclic endoperoxides (PGG2 and PGH2) arise rom the sequential COX and hydroperoxidase actions o COX-1
or COX-2 on AA released rom membrane phospholipids. Subsequent products are generated by tissue-speci c synthases and transduce their
e ects via membrane-bound receptors (blue boxes). Dashed lines indicate putative ligandreceptor interactions. Epoxyeicosatrienoic acids (EETs;
shaded in blue) and isoprostanes are generated via CYP activity and nonenzymatic ree radical attack, respectively. COX-2 can use modi ed ara-
chidonoylglycerol, an endocannabinoid, to generate the glyceryl prostaglandins. Aspirin and tNSAIDs are nonselective inhibitors o COX-1 and
COX-2, but do not a ect LOX activity. Epi-lipoxins are generated by COX-2 ollowing its acetylation by aspirin (see Figure 22-2). Dual 5-LOXCOX
inhibitors inter ere with both pathways.

358
Prostaglandins, NSAIDs, and Pharmacotherapy o Gout CHAPTER 2 2

Arac hido nic Ac id


12 (S )HETE 12LOXs COOH 15LOX2 COOH
12 (R)HETE
Ac e tylate d COX2 15 (S )HPETE
OOH
FLAP 5LOX COX1 (afte r as pirin)
He poxilins
OOH COX2 15 (R)HETE
COOH 5LOX/COX 5LOX
Inhibitors 5LOX
(ne utro phils )
OH e.g.: lic ofe lone (le uko c yte s )
COOH 5 (S )HPETE Pro s tano ids
5LOX (S e e Fig ure 221)
5LOXinhibitors
O e.g.: zile uton
5 (S )HETE 15 (R)e poxyte trane
COOH O
Cys LT
a nta gonists COOH
e.g.: za firluka s t LTA4 12LOX Epoxide
p ra nluka s t LTC4 LTA4 (platelets ) hydrolas es
hydro las e 15 (S )e poxyte trane
monte luka s t s ynthas e OH
OH OH OH 15e piLXA4
COOH 15e piLXB 4
COOH
C 5 H11 S LTC4 LTB 4 HO OH
Cys Gly
COOH
Glu Epoxide
g lutamyl trans pe ptidas e LXA4 hydro las e s
g lutamyl le uko trie nas e OH OH
OH
COOH
COOH
C 5 H11 S LTD4
Cys Gly HO OH LXB 4
dipe ptidas e
OH 12 (S )HETE
12 (R)HETE
COOH
C 5 H11 S Cys LTE4 BLT1 BLT2 ALX

Cys LT1 Cys LT2

FIGURE 22-2 Lipoxygenase pathways o arachidonic acid metabolism. 5-LOXactivating protein (FLAP) presents arachidonic acid to 5-LOX,
leading to the generation o the LTs. CysLTs are shaded in gray. Lipoxins (shaded in blue) are products o cellular interaction via a 5-LOX12-LOX
pathway or via a 15-LOX5-LOX pathway. Biological e ects are transduced via membrane-bound receptors dark (blue boxes). The dashed line
indicates putative ligandreceptor interactions. Zileuton inhibits 5-LOX but not the COX pathways (expanded in Figure 22-1). Dual 5-LOXCOX
inhibitors inter ere with both pathways. CysLT antagonists prevent activation o the CysLT1 receptor.

1-O-a lkyl-2-a cyl-glyce rophos phocholine


O H2C O R
R C O CH O S TIMULUS
H2C O P Choline
O-
e
s
ra
fe l
s cy

P
L
A

A2
n
a
tr

O
R C O-
Lys o-P AF Lys o-P AF
H2C O R H2C O R
HO CH O HO CH O
H2C O P Choline H2C O P Choline
O- O-
H3 C C O - H3 C C CoA
A

O O
c
e

e
ty

s
s F
ra
P yd

CoA
lh

n A
A r

fe
ra -P
F o

lt o
ty s
e Ly
la
s

H2C O R
e

c
A

H3 C C O CH O
O H2C O P Choline
O-
P AF

FIGURE 22-3 Synthesis and degradation o platelet-activating actor (PAF). RCOO is a mixture o atty
acids but is enriched in arachidonic acid that may be metabolized to eicosanoids. CoA, coenzyme A.
359
SECTION IV Inf ammation, Immunomodulation, and Hematopoiesis

A Che mica l s imila rity (a rbitra ry s ca le ) B COX is oform s e le ctivity (log s ca le ) C P la s ma t1/2 (log s ca le )

As pirin Sa licylic a cids Ke torola c Diclofe na c


S odium s a licyla te Flurbiprofe n Ke toprofe n
Diflunis a l Ke toprofe n Fe noprofe n
Ke torola c Ace tic a cids Indome tha cin Ace ta minophe n
Indome tha cin Tolme tin Indome tha cin
Tolme tin As pirin As pirin
Na bume tone Na bume tone Me clofe na ma te
S ulinda c Fe noprofe n Ibuprofe n
Etodola c Me clofe na ma te Ke torola c
Diclofe na c P he nyla ce tic S ulinda c Tolme tin
Lumira coxib a cids Na proxe n Flurbiprofe n
Flurbiprofe n Propionic a cids P iroxica m S odium s a licyla te
Ke toprofe n Ibuprophe n S ulinda c
Fe noprofe n Ace ta minophe n Etodola c
Na proxe n S odium s a licyla te Va lde coxib
Ibuprofe n Diflunis a l Ce le coxib
Me clofe na ma te Fe na mic a cids Me loxica m Diflunis a l
Ce le coxib Dia rylhe te rocylics S ulfon- Diclofe na c Rofe coxib
Va lde coxib a mide s Ce le coxib Etoricoxib
Rofe coxib Va lde coxib Lumira coxib
S ulfone s
Etoricoxib Etodola c Na proxe n
P iroxica m Enolic a cids Rofe coxib Me loxica m
Me loxica m Etoricoxib Na bume tone
Ace ta minophe n Othe rs Lumira coxib P iroxica m

more uns e le ctive more s hort 10 hours long


COX-1 COX-2 1 hr 60 hrs
Bla ck: tNSAID COX-2-s e le ctive s e le ctive s e le ctive
Gray: COX-2 inhibitors NSAIDS

FIGURE 22-4 Classi cation o NSAIDs by chemical similarity (panel A), cyclooxygenase (COX) iso orm selectivity (panel B), and plasma t1/2
(panel C). tNSAIDs, traditional nonsteroidal anti-in ammatory drugs.

TABLE 22-1 Classi cation and Comparison o Nonsteroidal Analgesics


CLASS/DRUG COMPARED TO
SUBSTITUTION PHARMACOKINETICS D0SING d
COMMENTS ASPIRIN
Salicylates

Aspirin (acetyl Peak Cp a 1 hour Antiplatelet 40-80 mg/day Permanent platelet


ester) Protein binding 80-90% Pain/ ever 325-650 mg every COX-1 inhibition
Metabolitesb Salicyluric acid Rheumatic ever 4-6 hours (acetylation)
t1/2c, therapeutic 2-3 hours Children 1 g every 4-6 Main side e ects: GI,
t1/2, toxic dose 15-30 hours hours increased bleeding
10 mg/kg every time, hypersensitivity
4-6 hours Avoid in children with
acute ebrile illness

Di unisal Peak Cp 2-3 hours 250-500 mg every Not metabolized to Analgesic and
(de uoro-phenyl) Protein binding 99% 8-12 hours salicylic acid anti-in ammatory
Metabolites Glucuronide Competitive COX e ects 4-5 times
t1/2 8-12 hours inhibitor more potent
Excreted into breast Antipyretic e ect
milk weaker
Fewer platelet
and GI side e ects
(Continued)

360
Prostaglandins, NSAIDs, and Pharmacotherapy o Gout CHAPTER 2 2

CLASS/DRUG COMPARED TO
SUBSTITUTION PHARMACOKINETICS D0SING d
COMMENTS ASPIRIN
Para aminophenol derivative

Acetaminophen Peak Cp 30-60 minutes 10-15 mg/kg Weak nonspeci c Analgesic and
Protein binding 20-50% every 4 hours COX inhibitor at antipyretic e ects
Metabolites Glucuronide (maximum o 5 common doses equivalent
conjugates (60%); doses/24 hours) Potency may be Anti-in ammatory,
sul uric acid modulated by GI, and platelet
conjugates (35%) peroxides e ects less than
t1/2 2 hours Overdose leads to aspirin at 1000
production o toxic mg/day
metabolite and liver
necrosis

Acetic acid derivatives

Indomethacin Peak Cp 1-2 hours 25 mg 2-3 times/ Side e ects 10-40x more
(methylated Protein binding 90% day; 75-100 mg at (3-50% o patients): potent;
indole) Metabolites O-demethylation night rontal headache, intolerance limits
(50%); unchanged neutropenia, dose
(20%) thrombocytopenia;
t1/2 2.5 hours 20% discontinue
therapy

Sulindac Peak Cp 1-2 hours; 8 150-200 mg 20% su er GI side Ef cacy


(sul oxide hours or sul de twice/day e ects; 10% get CNS comparable
prodrug) metabolite; side e ects
extensive
enterohepatic
circulation
Metabolites Sul one and
conjugates (30%);
sulindac and
conjugates (25%)
t1/2 7 hours; 18 hours
or metabolite

Etodolac Peak Cp 1 hour 200-400 mg Some COX-2 100 mg etodolac


(pyranocar- Protein binding 99% 3-4 times/day selectivity in vitro has similar
boxylic acid) Metabolites Hepatic ef cacy to 650
metabolites mg o aspirin, but
t1/2 7 hours may be better
tolerated
Tolmetin Peak Cp 20-60 400-600 mg 20 mg/kg/day in Food delays and Ef cacy
(heteroaryl Protein binding 99% 3 times/day 3-4 divided doses decreases peak similar; 25-40%
acetate Metabolites Oxidized to or children absorption develop side
derivative) carboxylic acid/ (anti-in ammatory) May persist longer in e ects; 5-10%
other derivatives, synovial uid to give discontinue drug
then conjugated a biological ef cacy
t1/2 5 hours longer than its
plasma t1/2
Ketorolac Peak Cp 30-60 <65 years: 20 Commonly given Potent
(pyrrolizine Protein binding 99% glucuronide mg (orally), then parenterally (60 mg analgesic, poor
carboxylate) Metabolites conjugate (90%) 10 mg every 4-6 IM ollowed by 30 mg anti-in ammatory
t1/2 4-6 hours hours (not to every 6 hours, or 30
exceed 40 mg/24 mg IVevery 6 hours)
hours); >65 years: Available as ocular
10 mg every 4-6 preparation (0.25%);
hours (not to 1 drop every 6 hours
exceed 40 mg/
24 hours)
(Continued)

361
SECTION IV Inf ammation, Immunomodulation, and Hematopoiesis

CLASS/DRUG COMPARED TO
SUBSTITUTION PHARMACOKINETICS D0SINGd
COMMENTS ASPIRIN
Diclo enac Peak Cp 2-3 hours 50 mg 3 times/day Available as topical More potent;
(phenylacetate Protein binding 99% or 75 mg twice/ gel, ophthalmic 20% develop
derivatives) Metabolites Glucuronide and day solution, and oral side e ects, 2%
sul de (renal 65%, tablets combined discontinue use,
bile 35%) with misoprostol 15% develop
t1/2 1-2 hours First-pass e ect; oral elevated liver
bioavailability, 50% enzymes
Fenamates (N phenyl anthranilates)
Me enamic acid Peak Cp 2-4 hours 500-mg load, then Isolated cases o Ef cacy similar; GI
Protein binding High 250 mg every 6 hemolytic anemia side e ects (25%)
Metabolites Conjugates hours May have some
o 3-hydroxy central action
and 3-carboxyl
metabolites (20%
recovered in eces)
t1/2 3-4 hours
Meclo enamate Peak Cp 0.5-2 hours 50-100 mg 4-6/ Ef cacy similar; GI
Protein binding 99% day (maximum o side e ects (25%)
Metabolites Hepatic 400 mg/day)
metabolism;
ecal and renal
excretion
t1/2 2-3 hours
Flu enamic acid Not available in
the U.S.
Propionic acid Intolerance o one Usually better
derivatives does not preclude use tolerated
o another propionate
derivative
Ibupro en Peak Cp 15-30 minutes Analgesia 200-400 mg every 10-15% discontinue Equipotent
Protein binding 99% Anti-in ammatory 4-6 hours due to adverse e ects
Metabolites Conjugates 300 mg/6-8 hours Childrens dosing
o hydroxyl or 400-800 mg Antipyretic: 5-10 mg/
and carboxyl 3-4 times/day kg every 6 hours
metabolites (max: 40 mg/kg/day)
t1/2 2-4 hours Anti-in ammatory:
20-40 mg/kg/day in
3-4 divided doses
Naproxen Peak Cp 1 hour 250 mg 4 times/ 5 mg/kg twice/ Peak anti- More potent
Protein binding 99% (less in day or 500 mg day in ammatory e ects in vitro; usually
Metabolites elderly) twice/day may not be seen until better tolerated;
6-demethyl and Children: 2-4 weeks o use variably
other metabolites anti-in ammatory Decreased protein prolonged t1/2
t1/2 14 hours binding and delayed may a ord
excretion increase risk cardioprotection
o toxicity in elderly in some
individuals

Fenopro en Peak Cp 2 hours 200 mg 4-6 times/ 15% experience


Protein binding 99% day; 300-600 mg side e ects; ew
Metabolites Glucuronide, 4-OH 3-4 times/day discontinue use
metabolite
t1/2 2 hours
(Continued)

362
Prostaglandins, NSAIDs, and Pharmacotherapy o Gout CHAPTER 2 2

CLASS/DRUG COMPARED TO
SUBSTITUTION PHARMACOKINETICS D0SING d
COMMENTS ASPIRIN
Ketopro en Peak Cp 1-2 hours Analgesia 25 mg 3-4 30% develop side
Protein binding 98% Anti-in ammatory times/day e ects (usually GI,
Metabolites Glucuronide 50-75 mg usually mild)
conjugates 3-4 times/day
t1/2 2 hours

Flurbipro en Peak Cp 1-2 hours 200-300 mg/day Available as a 0.03%


Protein binding 99% in 2-4 divided ophthalmic solution
Metabolites Hydroxylates and doses
conjugates
t1/2 6 hours

Oxaprozin Peak Cp 3-4 hours 600-1800 mg/day Long t1/2 allows or


Protein binding 99% daily administration;
Major metabolites Oxidates and slow onset o action;
glucuronide inappropriate or
conjugates ever/acute analgesia
t1/2 40-60 hours

Enolic acid derivatives

Piroxicam Peak Cp 3-5 hours 20 mg/day May inhibit activation Equipotent;


Protein binding 99% o neutrophils, activity perhaps better
Metabolites Hydroxylates and o proteoglycanase, tolerated
then conjugated collagenases 20% develop
t1/2 45-50 hours side e ects; 5%
discontinue drug

Meloxicam Peak Cp 5-10 hours 7.5-15 mg/day Some COX-2


Protein binding 99% selectivity,
Metabolites Hydroxylation especially at lower
t1/2 15-20 hours doses

Nabumetone Peak Cp 3-6 hours 500-1000 mg 1-2 A prodrug, rapidly Shows some
(naphthyl Protein binding 99% times/day metabolized to COX-2 selectivity
alkanone) Major metabolites O-demethylation, 6-methoxy-2- (active metabolite
then conjugation naphthyl acetic acid; does not)
t1/2 24 hours pharmacokinetics Fewer GI side
re ect active e ects than many
compound NSAIDs

Diaryl heterocyclic NSAIDs Evidence or Decrease in GI


(COX-2 selective) cardiovascular side e ects and in
adverse events platelet e ects

Celecoxib [diaryl Peak Cp 2-4 hours 100 mg Substrate or CYP2C9; See the text or
substituted Protein binding 97% 1-2 times/day inhibitor o CYP2D6 an overview o
pyrazone; Metabolites Carboxylic acid Coadministration COX-2 inhibitors
(sul onamide and glucuronide with inhibitors o
derivative)] conjugates CYP2C9 or substrates
t1/2 6-12 hours o CYP2D6 should be
done with caution

Parecoxib Not approved for


Etoricoxib use in the U.S.
Lumaricoxi
a
Time to peak plasma drug concentration (Cp) a ter a single dose. In general, ood delays absorption but does not decrease peak concentration.
b
The majority o NSAIDs undergo hepatic metabolism, and the metabolites are excreted in the urine. Major metabolites or disposal pathways
are listed.
c
Typical t1/2 is listed or therapeutic doses; i t1/2 is much di erent with the toxic dose, this is also given.
d
Limited dosing in ormation given. For additional in ormation, re er to the product in ormation literature.
CNS, central nervous system; COX, cyclooxygenase; GI, gastrointestinal; IM, intramuscularly; IV, intravenously.

363
SECTION IV Inf ammation, Immunomodulation, and Hematopoiesis

MAJOR THERAPEUTIC USES TABLE 22-2 Common and Shared Side Ef ects o NSAIDs
OF NSAIDs SYSTEM MANIFESTATIONS
All NSAIDs, including COX-2 Inhibitors, are GI Abdominal pain
anti-in ammatory, analgesic, and anti- Nausea
pyretic, except or acetaminophen, which is Diarrhea
largelydevoid o anti-in ammatoryactivity. Anorexia
Used as anti-in ammatoryagents in the Gastric erosions/ulcersa
Anemiaa
treatment o musculoskeletal disorders GI hemorrhage a
(the primaryclinical applicationo NSAIDs) Per oration/obstruction a
Rheumatoid arthritis
Platelets Inhibited platelet activation a
Osteoarthritis Propensity or bruising a
Ankylosing spondylitis Increased risk o hemorrhage a
Gout Renal Salt and water retention
Used as analgesics to treat pain o low Edema, worsening o renal unction in renal/cardiac and cirrhotic patients
to moderate intensity Decreased e ectiveness o antihypertensive medications
Decreased e ectiveness o diuretic medications
NSAIDs lackthe unwanted adverse Decreased urate excretion (especially with aspirin)
CNSe ects o opiates (respiratory Hyperkalemia
depression and development o
physical dependence). Cardiovascular Closure o ductus arteriosus
Myocardial in arction b
Coadministration o NSAIDs with Stroke b
opioids can reduce the dose o opiate Thrombosisb
and the likelihood o adverse CNS
e ects. CNS Headache
Vertigo
NSAIDs are particularlye ective in Dizziness
treating dental pain, postoperative Con usion
pain, pain arising romin ammation, Hyperventilation (salicylates)
and menstrual cramps (the latter due
Uterus Prolongation o gestation
torelease o prostaglandins romthe Inhibition o labor
endometrium).
NSAIDs are commonlyused as rst- Hypersensitivity Vasomotor rhinitis
Angioneurotic edema
line agents in treating migraine
Asthma
attacks and can be combined with Urticaria
second-line agents such as the Flushing
triptans, or with antiemetics or Hypotension
relie o nausea. Shock
NSAIDs lackef cacyin treating a
Side e ects decreased with COX-2-selective NSAIDs.
neuropathicpain or pain arising b
With the exception o low-dose aspirin.
romthe hollowviscera.
Can reduce ever in most situations,
but antipyretice ects mayobscure the CASE 22 1
course o illness
A 24-year-old woman experiences severe menstrual cramping each month. T e
Antipyretictherapyis reserved or cramping and pain are relieved when she takes ibupro en.
patients with deleterious ever and
those who experience relie when a. What is causing this patients menstrual cramping?
ever is lowered. T e release o PGs by the endometrium during menstruation may cause severe
Used in neonates to close inappropriately cramps and other symptoms o primary dysmenorrhea.
patent ductus arteriosus, which is very
b. What is the mechanism o action o ibupro en in relieving this patients pain and
sensitive to the dilating e ects o PGE1.
cramping?
Low-dose aspirin reduces the risko serious
Ibupro en and other NSAIDs are e ective in treating menstrual cramping because
cardiovascular events in high-riskpatients,
o their ability to inhibit synthesis o PGs by the endometrium through their
probablybyirreversiblyinhibiting throm-
inhibitory e ects on COX-1 and COX-2. T e selective COX-2 inhibitors also are
boxane A2 (TxA2) production byplatelets
e cacious in this condition.
(see Chapter 19).
(Continued)
364
Prostaglandins, NSAIDs, and Pharmacotherapy o Gout CHAPTER 2 2

c. I this patient becomes pregnant, are there any hazards to her or the etus in
MECHANISMS OF GASTRIC
using NSAIDs or relie o headaches and musculoskeletal pains?
DAMAGE BY NSAIDs
T e use o NSAIDs and aspirin late in pregnancy may increase the risk o post-
partum hemorrhage. T ere ore, pregnancy, especially close to term, is a relative Inhibition o COX-1 in gastricepithelial
contraindication to the use o all NSAIDs. In addition, their use must be weighed cells depresses mucosal cytoprotective PGs,
against potential etal risk, even in cases o premature labor, and especially in cases especiallyPGE2 and PGI2.
o pregnancy-induced hypertension. NSAID use is associated with closure o the COX-2 inhibition in gastricepithelial
ductus arteriosus and impaired etal circulation in utero, particularly in etuses cells mayalso contribute to some loss o
older than 32 weeks gestation. cytoprotective PGs.
Local irritation o gastricmucosa byorally
CASE 22 2 administered NSAIDs.
A 64-year-old man who su ered a myocardial in arction 2 years earlier is taking 80 mg Enhanced generation o lipoxygenase
o aspirin each day on the recommendation o his amily physician. His doctor has advised (LOX) products (eg, LTs; see Figure 22-2)
him that this will reduce his chances o another myocardial in arction or stroke. maycontribute to ulcerogenicityin
patients treated with NSAIDs.
a. What is the mechanism o aspirins cardioprotective e ect in this patient?
T is e ect is due to irreversible acetylation o platelet COX-1, which permanently
and completely suppresses platelet thromboxane A2 ( xA2) ormation, with the CARDIOVASCULAR
consequent inhibition o platelet unction (see Figure 19-1). Platelets, which being HAZARD WITH COX-2
anucleate, have a markedly limited capacity or protein synthesis. T us, the conse- SELECTIVE NSAIDs
quences o inhibition o platelet COX-1 (COX-2 is not expressed in mature platelets)
Inhibition o COX-2 in the vasculature
last or the li etime o the platelet. Inhibition o platelet COX-1dependent xA2
reduces vascular PGI2 (prostacyclin)
ormation, there ore, is cumulative with repeated doses o aspirin (at least as low as
production which is antithromboticand
30 mg/d) and takes ~8 to 12 daysthe platelet turnover timeto recover ully once
antiatherogenic.
therapy has been stopped. Low-dose aspirin therapy does not signi cantly impair
synthesis o PGI2 by vascular endothelium, in part because o the ability o endothe- Inhibition o COX-2 in the kidneyreduces
lial cells to continuously express COX-1 and COX-2. T us, daily low-dose aspirin renal PGI2 and PGE2 production which
blocks the ormation o xA2, which promotes platelet aggregation, while leaving results in hypertension and edema due to
intact the synthesis o PGI2, which inhibits platelet unction (see Figure 19-1). the contribution o these PGs in maintain-
ing arterial pressure homeostasis.
Aspirin reduces the risk o serious vascular events in high-risk patients (eg, those
with previous myocardial in arction) by 20 to 25%. Low-dose (<100 mg/d) aspirin, In addition to the COX-2-selective (coxib)
which is relatively (but not exclusively) selective or COX-1, is as e ective as higher drug celecoxib, several traditional NSAIDs
doses (eg, 325 mg/d). (tNSAIDs) exhibit relative selectivity or
COX-2 (eg, etodolac, diclo enac, meloxi-
b. What are the hazards o daily aspirin therapy? cam; see Figure 22-4) and are likelyto have
Because o impairment o platelet unction, the most serious hazard to daily aspirin similar cardiovascular risks as the coxibs.
therapy is an increased risk o hemorrhage. Placebo-controlled trials reveal that The COX-2-selective NSAIDs (including the
aspirin increases the incidence o serious GI bleeds, re ecting suppression not just tNSAIDs with relative selectivity or COX-2)
o platelet thromboxane, but also reduction in gastroepithelial PGE2 and PGI2. It should not be used in patients with isch-
also increases the incidence o intracranial bleeds. Although bene t rom aspirin emicheart disease or stroke, and should
outweighs these risks in the case o secondary prevention o cardiovascular disease, be used with caution in patients with risk
the issue is much more nuanced in patients who have never had a serious athero- actors or heart disease.
thrombotic event (primary prevention); here, prevention o myocardial in arction For all such COX-2-selective NSAIDs, it
by aspirin is numerically balanced by the serious GI bleeds it precipitates
is advisable to use the lowest possible
Daily aspirin therapy can also increase the risk o gastric ulcers. COX-1 is expressed dose or the shortest possible time.
as the dominant, constitutive iso orm in gastric epithelial cells and is thought to be
the major source o cytoprotective PG ormation. Inhibition o COX-1 at this site
is thought to account largely or the gastric adverse events that complicate therapy
with tNSAIDs.
c. What concomitantly administered drugs might impair the cardioprotective
e ects o aspirin?
Many patients combine either tNSAIDs or COX-2 inhibitors with cardioprotective,
low-dose aspirin. Epidemiological studies suggest that this combination therapy
increases signi cantly the likelihood o GI adverse events over either class o
NSAID alone.
(Continued)
365
SECTION IV Inf ammation, Immunomodulation, and Hematopoiesis

Prior occupancy o the active site o platelet COX-1 by the commonly consumed
tNSAID ibupro en impedes access o aspirin to its target Ser 529 and prevents irre-
versible inhibition o platelet unction. Whether this interaction is clinically impor-
tant has yet to be de nitively answered.

CASE 22 3
A 23-year-old male college student su ering rom a cold started taking several di erent
over-the-counter (O C) cold/ u medications 2 days ago to relieve symptoms. He is
eeling better and decides to attend an end-o -term party with his classmates where
they will be celebrating with beer and other alcoholic beverages.
a. Is this student at risk o any serious drug interactions when taking multiple
O C cold/f u medications?
A large number o nonprescription multisymptom cold/ u medications contain
acetaminophen to reduce ever and relieve the muscle aches that are common symp-
toms o colds and the u. Severe liver damage can occur with more than 4000 mg
o acetaminophen in a 24-hour period. aking the maximum recommended daily
dose o a single multisymptom cold/ u medication will typically be 2400 mg or less
o acetaminophen, but taking maximal doses o multiple cold/ u medication, or a
cold/ u medication in combination with acetaminophen could result in a total dose
that is more than 4000 mg o acetaminophen. In 2009, a Food and Drug Administra-
tion (FDA) advisory panel recommended a lower maximum daily dose o acetamin-
ophen o 2600 mg and a decrease in the maximum single dose rom 1000 to 650 mg.
b. Are there any risks with consuming alcohol while taking O C cold/f u medications?
T e chances o severe liver damage are increased when alcohol is consumed while
taking acetaminophen. Conditions o cytochrome P450 (CYP) induction (eg, heavy
alcohol consumption) or glutathione (GSH) depletion (eg, asting or malnutrition)
increase the susceptibility to hepatic injury with acetaminophen, which has been
documented, albeit uncommonly, with doses in the therapeutic range. T e maxi-
mum daily dose o acetaminophen is 2000 mg/d or chronic alcoholics.
c. What are the symptoms o acetaminophen toxicity?
Symptoms that occur during the rst 2 days o acute poisoning by acetaminophen
re ect gastric distress (eg, nausea, abdominal pain, anorexia) and belie the potential
seriousness o the intoxication (see Case 3-5 in Chapter 3). Plasma transaminases
become elevated, sometimes markedly so, beginning ~12 to 36 hours af er ingestion.
Clinical indications o hepatic damage mani est within 2 to 4 days o ingestion o
toxic doses, with right subcostal pain, tender hepatomegaly, jaundice, and coagulopa-
thy. Renal impairment or rank renal ailure may occur. Liver enzyme abnormalities
typically peak 72 to 96 hours af er ingestion. T e onset o hepatic encephalopathy or
worsening coagulopathy beyond this time indicates a poor prognosis. Biopsy o the
liver reveals centrilobular necrosis with sparing o the periportal area. In non atal
cases, the hepatic lesions are reversible over a period o weeks or months.

CASE 22 4
A 72-year-old woman has developed arthritis in her hands and has dif culty opening
jars and other common tasks because o the pain. She asks her doctor to recommend
a drug to relieve the pain so she can complete her normal tasks.
a. What are the considerations in choosing a drug to relieve this patients symptoms?
NSAIDs are the drugs o choice or treating the symptoms o arthritis because o
their analgesic and anti-in ammatory e ects. Choice and dosing o an NSAID are
usually guided by multiple considerations, including patient age, coincident dis-
eases or allergies, the drugs sa ety and interaction pro le, and cost considerations.
Drugs with a longer duration o action may be pre erable or management o
arthritic pain.
(Continued)
366
Prostaglandins, NSAIDs, and Pharmacotherapy o Gout CHAPTER 2 2

T e choice among tNSAIDs or the treatment o chronic arthritic conditions is


largely empirical. Substantial di erences in response have been noted among indi-
viduals treated with the same tNSAID and within an individual treated with di er-
ent tNSAIDs, even when the drugs are structurally related. It is reasonable to give
a drug or a week or two as a therapeutic trial and to continue it i the response is
satis actory. Initially, all patients should be asked about previous hypersensitivity
to aspirin or any member o the NSAID class. T ereaf er, low doses o the chosen
agent should be prescribed to determine initial patient tolerance. Older patients
may require lower doses than younger patients to reduce the risk o toxicity. Regu-
latory agencies stress use o the lowest e ective dose or the shortest possible dura-
tion o treatment. Alterations in dosing may take several days (>3-4 t1/2) to translate
into clinically detectable changes. Delayed distribution into the synovial compart-
ment may extend this interval. Indeed, some NSAIDs that are short-lived in plasma
are sustained in synovial uid and may a ord su cient relie even when admin-
istered at intervals longer than their plasma t1/2 li e. I the patient does not achieve
therapeutic bene t rom 1 NSAID, another may be tried. For mild arthropathies,
the scheme outlined earlier in this paragraph, together with rest and physical ther-
apy, probably will be su ciently e ective. When the patient has sleeping problems
because o pain or morning sti ness, a larger proportion o the daily dose may be
given at night.
b. What are the expected adverse e ects o NSAIDs or treatment o osteoarthritis
and when would these e ects occur?
able 22-2 lists the common and shared side e ects o NSAIDs. T e most common
side e ects associated with these drugs are gastrointestinal, including anorexia,
nausea, dyspepsia, abdominal pain, and diarrhea. T ese symptoms may be related
to the induction o gastric or intestinal ulcers, which is estimated to occur in
15 to 30% o regular users. Ulceration may range rom small super cial erosions
to ull-thickness per oration o the muscularis mucosa. T ere may be single or
multiple ulcers, and ulceration may be uncomplicated or complicated by bleeding,
per oration, or obstruction. Blood loss can be gradual, leading to anemia over
time, or acute and li e-threatening. T e risk is urther increased in patients with
Helicobacter pylori in ection, heavy alcohol consumption, or other risk actors or
mucosal injury, including the concurrent use o glucocorticoids.
Some adverse e ects may mani est in the rst weeks o therapy; however, the
risk o gastric ulceration and bleeding accumulates with the duration o dosing.
Combination therapy with more than 1 NSAID is to be avoided.
c. What is the mechanism underlying the gastric adverse e ects o NSAIDs, and
what can be done to minimize these e ects in the treatment o arthritis?
Gastric damage by NSAIDs can be brought about by at least 2 distinct mechanisms
(see Side Bar MECHANISMS OF GAS RIC DAMAGE BY NSAIDs). Inhibition o
COX-1 in gastric epithelial cells depresses mucosal cytoprotective PGs, especially
PGI2 and PGE2. T ese eicosanoids inhibit acid secretion by the stomach, enhance
mucosal blood ow, and promote the secretion o cytoprotective mucus in the
intestine. Inhibition o PGI2 and PGE2 synthesis may render the stomach more sus-
ceptible to damage and can occur with oral, parenteral, or transdermal administra-
tion o aspirin or NSAIDs. T ere is some evidence that COX-2 also contributes to
constitutive ormation o these PGs by human gastric epithelium.
Another mechanism by which NSAIDs or aspirin may cause ulceration is by local
irritation rom contact o orally administered drug with the gastric mucosa. Local
irritation allows back di usion o acid into the gastric mucosa and induces tissue
damage. However, the incidence o GI adverse events is not signi cantly reduced
by ormulations that reduce drug contact with the gastric mucosa, such as enteric
coating or e erent solutions, suggesting that the contribution o direct irritation to
the overall risk is minor. It also is possible that enhanced generation o LOX prod-
ucts (eg, L s) contributes to ulcerogenicity in patients treated with NSAIDs.
(Continued)
367
SECTION IV Inf ammation, Immunomodulation, and Hematopoiesis

A number o risk actors or GI complications have been determined in epide-


miological studies. Patients at high risk or GI complications should be prescribed
a gastroprotective agent. Coadministration o the PGE1 analog, misoprostol, or
proton pump inhibitors (PPIs, see Chapter 32) in conjunction with NSAIDs can be
bene cial in the prevention o duodenal and gastric ulceration. For instance, diclo -
enac is available in combination with misoprostol, a PGE1 analog (AR HRO EC).
T is combination retains the e cacy o diclo enac while reducing the requency o
GI ulcers and erosions. Patients at high risk or GI complications also are potential
candidates or a COX-2selective NSAID (see Chapter 32).
d. What are the considerations in choosing a COX-2selective NSAID to reduce GI
complications?
PHARMACOTHERAPY OF
RHEUMATOID ARTHRITIS COX-2selective NSAIDs were developed to improve the GI sa ety o anti-
in ammatory therapy in patients at elevated risk or GI complications. However,
Rheumatoid arthritis is an autoimmune placebo-controlled trials with 3 structurally distinct COX-2selective NSAIDs
disease (see Chapter 23). revealed an increase in the incidence o myocardial in arction, stroke, and throm-
The goal o therapyis symptomaticrelie o bosis. T e risk appears to also extend to those o the older tNSAIDs, which are
pain with NSAIDs, and reduction in disease quite selective or COX-2 (see Side Bar CARDIOVASCULAR HAZARD WI H
activityand retardation o arthritictissue COX-2SELEC IVE NSAIDs). Regulatory agencies in the United States, European
destruction using DMARDs (see Table 22-3 Union, and Australia have concluded that all NSAIDs (with the exception o aspirin,
and Chapter 23). see Case 22-2) have the potential to increase the risk o heart attack and stroke.

TABLE 22-3 Disease-Modi ying Anti-Rheumatic Drugs


PHARMACOTHERAPY
OF GOUT DRUG CLASS OR ACTION REFERENCE CHAPTER NUMBER

Gout results romthe precipitation o urate Small molecules


crystals in tissues and the subsequent Methotrexate Anti- olate 45
in ammatoryresponse.
Le unomide Pyrimidine synthase inhibitor 45
Acute gout usuallycauses pain ul distal
arthritis, but can also cause joint destruc- Hydroxychloroquine Anti-malarial 35
tion, subcutaneous deposits (tophi), and
Minocycline 5-lipoxygenase inhibitor, This chapter, 41
renal calculi and damage.
tetracycline antibiotic
Hyperuricemia is a necessarycondition or
gout, but does not inevitablylead to gout. Sul asalazine Salicylate This chapter, 33
In most patients with gout, hyperuricemia Azathioprine Purine synthase inhibitor 45
is the result o underexcretion o urate,
rather than overproduction. Cyclosporine Calcineurin inhibitor 23

The aims o treatment are to: Cyclophosphamide Alkylating agent 45


Decrease the symptoms o an acute Biologicals
attack
Adalimumab Ab, TNF- antagonist 23
Decrease the risko recurrent attacks
Lower serumurate levels Golimumab Ab, TNF- antagonist 23
The agents used to treat gout are: In iximab IgG-TNF receptor usion protein 23
Drugs that relieve in ammation (anti-TNF)
and pain (NSAIDs, colchicine,
Certolizumab Fab ragment toward TNF- 23
glucocorticoids)
Drugs that prevent the in ammatory Abatacept T-cell co-stimulation inhibitor 23
(binds B7 protein on antigen-
responses to urate crystals (colchicine
presenting cell)
and NSAIDs)
Drugs that inhibit urate ormation Rituximab Ab toward CD20 (cytotoxic toward 46
(allopurinol, ebuxostat) B cells)
Drugs that augment urate excretion Anakinra IL-1-receptor antagonist 23, 46
(probenecid)
IL, interleukin; TNF, tumor necrosis actor.

368
Prostaglandins, NSAIDs, and Pharmacotherapy o Gout CHAPTER 2 2

CASE 22 5
A 6-year-old girl catches a viral in ection that some o her classmates at kindergarten
have recently had. She develops a ever and headache. Her parents are concerned and
want to give her something to relieve her symptoms.
a. What are the considerations in choosing an NSAID or this patient?
T erapeutic indications or NSAID use in children include ever, mild pain, post-
operative pain, and in ammatory disorders, such as juvenile arthritis and Kawasaki
disease. T e choice o drugs or children is considerably restricted; only drugs that
have been extensively tested in children should be used (acetaminophen, ibupro en,
and naproxen).
b. Why is aspirin to be avoided in this patient?
Because o the possible association with Reye syndrome, aspirin and other salicy-
lates are contraindicated in children and young adults aged younger than 20 years
with viral illnessassociated ever. Reye syndrome, a severe and of en atal disease,
is characterized by the acute onset o encephalopathy, liver dys unction, and atty
in ltration o the liver and other viscera. T e etiology and pathophysiology are not
clear, nor is it clear whether a causal relationship between aspirin and Reye syn-
drome exists. However, the epidemiologic evidence or an association between aspi-
rin use and Reye syndrome seemed su ciently compelling that labeling o aspirin
and aspirin-containing medications to indicate Reye syndrome as a risk in children
was rst mandated in 1986 and extended to bismuth subsalicylate in 2004. Since
then, the use o aspirin in children has declined dramatically, and Reye syndrome
has almost disappeared. Acetaminophen has not been implicated in Reye syndrome
and is the drug o choice or antipyresis in children, teens, and young adults.
(Continued)

MECHANISMS OF ACTION OF DRUGS USED TO TREAT GOUT


DRUG CLASS DRUG MECHANISM OF ACTION
NSAID See Side Bar MECHANISMS OF ACTION OF NSAIDS Inhibits COX-1 and COX-2 impeding the ability o these
enzymes to make prostaglandins (see Figure 22-1) which
are involved in in ammation and pain

Glucocorticoid See Chapter 29 See Chapter 29

Microtubule Colchicine A variety o pharmacological e ects, many o which


Disruptor involve inhibition o microtubule and spindle ormation,
including antimitotic e ects (neutrophils and GI
epithelium), inhibition o neutrophil motility, inhibition
o neutrophil and mast cell release o mediators, and
inhibition o endothelial cell expression o adhesion
molecules

Xanthine Oxidase Inhibitor Allopurinol Prevents the synthesis o urate rom hypoxanthine and
Febuxostat xanthine by inhibiting xanthine oxidase
Allopurinol is a substrate or xanthine oxidase and its
product, oxypurinol, is a noncompetitive inhibitor o the
reduced orm o the enzyme
Febuxostat orms a stable enzyme-inhibitor complex
with both the reduced and oxidized orm o the enzyme

Recombinant Urate Oxidase Rasburicase Catalyzes the enzymatic oxidation o uric acid into the
soluble and inactive metabolite, allantoin

Uricosuric Agents Probenecid Inhibits URAT-1, the primary organic anion transporter
involved in reabsorption o urate in the kidney, thus
increasing the rate o excretion o urate

369
SECTION IV Inf ammation, Immunomodulation, and Hematopoiesis

CASE 22 6
A 50-year-old man has occasional episodes o gout that are pain ul and debilitating. He
requires drugs to treat the symptoms o acute gout attacks and to prevent recurrent attacks.
a. What causes this patients pain ul episodes o gout?
Gout results rom the precipitation o urate crystals in the tissues and the subse-
quent in ammatory response. Acute gout usually causes an exquisitely pain ul distal
monoarthritis, but it also can cause joint destruction, subcutaneous deposits (tophi),
and renal calculi and damage. Gout a ects ~0.5 to 1% o the population o Western
countries. It is the most common orm o in ammatory arthritis in the elderly.
T e pathophysiology o gout is not ully understood. Hyperuricemia, while a pre-
requisite, does not inevitably lead to gout. Uric acid, the end product o purine
metabolism, is relatively insoluble compared to its hypoxanthine and xanthine pre-
cursors, and normal serum urate levels (~5 mg/dL, or 0.3 mM) approach the limit
o solubility. In most patients with gout, hyperuricemia arises rom underexcretion
rather than overproduction o urate.
b. What is the therapeutic rationale or the drugs used to treat gout in this patient?
T e aims o treatment are to decrease the symptoms o an acute attack, decrease
the risk o recurrent attacks, and lower serum urate levels (see Side Bar PHAR-
MACO HERAPY OF GOU ). T e drugs that relieve the symptoms o an acute
attack (in ammation and pain) are NSAIDs, colchicine, and glucocorticoids (see
Chapter 29). T e drugs that decrease the risk o recurrent attacks by preventing
in ammatory responses to urate crystals are colchicine and NSAIDs. T e drugs
that inhibit urate crystal ormation by lowering serum urate levels act by reduc-
ing urate ormation (allopurinol, ebuxostat) or augmenting urate excretion
(probenecid).
c. T is patient is prescribed colchicine. What is the mechanism o action o
this drug?
Colchicine is thought to alleviate in ammation rapidly through multiple mecha-
nisms, including the inhibition o neutrophil chemotaxis and activation. Symptoms
usually resolve within 2 to 3 days, but adverse GI events are requent, and toxicity
may include bone marrow depression. T e therapeutic index o colchicine is
narrowespecially in the elderly.
d. How does probenecid increase the excretion o uric acid?
Probenecid and other uricosuric agents increase the rate o uric acid excretion by
inhibiting its tubular reabsorption. T ey compete with uric acid or organic acid
transporters, primarily URA -1, which mediate urate exchange in the proximal
tubule. Certain drugs, particularly thiazide diuretics (see Chapter 15), and immu-
nosuppressant agents (especially cyclosporine, see Chapter 23) may impair urate
excretion and thereby increase the risk o gout attacks. Probenecid generally is well
tolerated but causes mild GI irritation in some patients.

KEY CONCEPTS
NSAIDs are e ective in reducing in ammation (except or acetominophen,
which lacks appreciable anti-in ammatory activity), most orms o low to mod-
erate pain, and ever.
NSAIDs inhibit the ormation o prostaglandins by inhibiting COX-1 and/or
COX-2.
GI disturbances are the most common adverse e ects o NSAIDs, except
the COX-2-selective inhibitors which have reduced e ects on prostaglandin
production by gastric epithelium.
(Continued)

370
Prostaglandins, NSAIDs, and Pharmacotherapy o Gout CHAPTER 2 2

All NSAIDs except aspirin increase the risk o cardiovascular events (myocardial
in arction, stroke, and thrombosis).
Patients at risk o cardiovascular disease should not be treated with COX-2
selective inhibitors.
For patients treated with COX-2selective inhibitors, treatment should be with
the lowest dose possible or the shortest time possible.
Pharmacotherapy o gout includes treatment o pain and in ammation, and
reduction in tissue urate crystals.

SUMMARY QUIZ

QUESTION 22-1 Which NSAID has the least anti-in ammatory ef cacy?
a. Aspirin
b. Acetaminophen
c. Ibupro en
d. Naproxen
e. Celecoxib

QUESTION 22-2 Which NSAID permanently inactivates xA2 synthesis by platelets?


a. Aspirin
b. Acetaminophen
c. Ibupro en
d. Naproxen
e. Celecoxib

QUESTION 22-3 What is the primary mechanism o action o allopurinol in


treating gout?
a. It is an analgesic agent.
b. It is an anti-in ammatory agent.
c. It blocks in ammatory responses to urate crystals.
d. It inhibits urate ormation.
e. It augments urate excretion.

QUESTION 22-4 T e cardiovascular risk associated with celecoxib results rom


a. inhibition o prostaglandin production in the gastric epithelium.
b. inhibition o platelet thromboxane production.
c. e ects on myocardial ion channels.
d. inhibition o prostaglandin in the kidney.
e. enhanced prostacyclin production by vascular endothelium.

QUESTION 22-5 Diclo enac has a t 1/2 in plasma o 1 to 2 hours, yet its therapeutic
e ects in treating rheumatoid arthritis last or much longer. T is prolongation o
therapeutic e ect is due to
a. irreversible inhibition o COX-1 and COX-2.
b. its relative selectivity or COX-2.
c. its accumulation in synovial uid.
d. the ormation o a long-lived active metabolite.
e. CNS e ects unrelated to inhibition o prostaglandin synthesis.

371
SECTION IV Inf ammation, Immunomodulation, and Hematopoiesis

SUMMARY QUIZ ANSWER KEY


QUESTION 22-1 Answer is b. Acetaminophen, is antipyretic and analgesic, but is
largely devoid o anti-in ammatory activity.
QUESTION 22-2 Answer is a. Aspirin is an irreversible inhibitor o COX-1. Because
platelets are anucleate and have minimal capacity or synthesis o new protein, inhibition
o platelet COX-1 lasts or the li etime o the platelet (8-12 days).
QUESTION 22-3 Answer is d. Allopurinol inhibits xanthine oxidase and prevents the
synthesis o urate rom hypoxanthine and xanthine.
QUESTION 22-4 Answer is d. Inhibition o PG production in the kidney, which
increases the likelihood o hypertension and edema, occurs with celecoxib, as well as
nonselective COX inhibitors. T e lack o an e ect on COX-1 production in platelets and
the inhibition o prostacyclin synthesis by COX-2 in vascular endothelium increase the
risk o thrombosis resulting in myocardial in arction and ischemic stroke.
QUESTION 22-5 Answer is c. Diclo enac accumulates in synovial uid a er oral
administration which likely explains why its duration o therapeutic e ect is consider-
ably longer than its plasma t1/2. T e selectivity o diclo enac or COX-2 resembles that
o celecoxib, but this does not explain its prolonged duration o therapeutic e ect.

SUMMARY: PROSTAGLANDINS, NSAIDs, AND PHARMACOTHERAPYOF GOUT


TOXICITIES
UNIQUE; CLINICALLY
CLASS AND SUBCLASSES NAMES CLINICAL USES COMMON IMPORTANT
PGE1 Alprostadil Second-line treatment or When used in neonates
erectile dys unction (see or maintenance o patent
PDE5 inhibitors, Chapter 16) ductus arteriosus, apnea
Temporary maintenance o occurs in ~10%, particularly
patent ductus arteriosus in in those <2 kg at birth
neonates with congenital
heart disease

PGE1 Analog Misoprostol Prevention o NSAID-


induced gastric ulcers (see
Chapter 32)

PGE2 Dinoprostone Induction o mid-trimester


abortion
Induction o labor at term,
and acilitation o labor

PGF2 Analog Carboprost tromethamine Induction o mid-trimester


abortion
Induction o labor at term,
and acilitation o labor
Control o postpartum
hemorrhage not responding
to conventional methods

PGF2 Latanoprost Treatment o open-angle


AnalogLong-acting Bimatoprost glaucoma (see Chapter 47)
Travoprost

PGI2 Prostacyclin Long-term treatment o


(epoprostenol) pulmonary hypertension

PGI2 Analog Iloprost Long-term treatment o


Treprostinil pulmonary hypertension

372
Prostaglandins, NSAIDs, and Pharmacotherapy o Gout CHAPTER 2 2

TOXICITIES
UNIQUE; CLINICALLY
CLASS AND SUBCLASSES NAMES CLINICAL USES COMMON IMPORTANT

NSAIDIrreversible Aspirin (acetylsalicylic Antiplatelet (irreversible GI e ects (see Side Bar Increased risk o GI
COX-1 Inhibitor acid, ASA) COX-1 inhibition) MECHANISMS OF GASTRIC bleeding and intracranial
Pain/ ever DAMAGE BYNSAIDs) bleeding
Anti-in ammatory (See Side Increased bleeding time Associated with Reye
Bar MAJORTHERAPEUTIC Hypersensitivity (see syndrome (see Case
USES OF NSAIDs) Table 22-2) 22-5) (see Table 22-2 or
additional toxicities)

NSAIDTraditional Salsalate Pain (ef cacy depends on GI (see Side Bar Increased risk o
(tNSAID) Salicylate agent) MECHANISMS OF GASTRIC cardiovascular disease
Di unisal Fever (ef cacy depends on DAMAGE BYNSAIDs) (see Table 22-2 or
Mesalamine agent) (See Table 22-2 or additional additional toxicities)
Sul asalazine Anti-in ammatory (ef cacy toxicities)
Balsalazide depends on agent)
Olsalazine (See Table 22-1 and Side Bar
Acetaminophen MAJORTHERAPEUTIC USES
Indomethacin OF NSAIDs)
Sulindac
Etodolac
Tolmetin
Ketorolac
Diclo enac
Brom enac
Nepa enac
Me enamic acid
Meclo enamate
Ibupro en
Naproxen
Fenopro en
Ketopro en
Flurbipro en
Oxaprozin
Piroxicam
Meloxicam
Nabumetone

NSAIDCOX-2Selective Celecoxib Pain (See Table 22-2; decreased Increased risk o


(coxibs) Fever GI side e ects and platelet cardiovascular disease
In ammation e ects compared to (see Table 22-2 and Side
tNSAIDs) Bar CARDIOVASCULAR
HAZARD WITH COX-2
SELECTIVE NSAIDs)

Microtubule Disruptor Colchicine Acute gout attacks GI (nausea, vomiting, Acute intoxication causes
Prevention o acute gout diarrhea, and abdominal hemorrhagic gastropathy
(o -label) pain) Li e-threatening (especially
in combination with P-gp
or CYP3A4 inhibitors):
myelosuppression,
leukopenia,
granulocytopenia,
thrombopenia,
aplastic anemia, and
rhabdomyolysis

Xanthine Oxidase Allopurinol Gout Occasionally induces Serious hypersensitivity


Inhibitor Hyperuricemia secondary to drowsiness reactions preclude urther
malignancies Hypersensitivity reaction use
Calcium oxalate calculi (may occur a ter months o Cutaneous reaction
therapy) Toxic epidermal necrolysis
Fever, malaise, and myalgias Stevens-Johnson
syndrome

373
SECTION IV Inf ammation, Immunomodulation, and Hematopoiesis

TOXICITIES
UNIQUE; CLINICALLY
CLASS AND SUBCLASSES NAMES CLINICAL USES COMMON IMPORTANT

Febuxostat Hyperuricemia with gout Liver unction abnormalities, Increase in gout ares
attacks nausea, joint pain, rash with initiation o therapy
caused by mobilization o
urate rom tissue deposits
Possible increased
cardiovascular risk
Contraindicated in
patients on azathioprine,
mercaptopurine, or
theophylline (drugs
metabolized by xanthine
oxidase)

Recombinant Urate Rasburicase Initial management o Vomiting, ever, nausea, Hemolysis in G6PD-
Oxidase elevated plasma uric headache, abdominal pain, de cient patients,
acid levels in pediatric constipation, diarrhea, and methemoglobinemia,
patients with leukemia, mucositis acute renal ailure,
lymphoma, and solid anaphylaxis
tumor malignancies who Ef cacy may be
are receiving anticancer hampered by antibodies
therapy expected to result produced against the
in tumor lysis and signi cant drug
hyperuricemia

Uricosuric Agents Probenecid Gout Mild GI irritation Probenecid should not


Hypersensitivity reactions be used in gouty patients
(2-4%) usually are mild with nephrolithiasis or
with overproduction o
uric acid
Liberal uid intake
should be maintained
throughout therapy to
minimize the risk o renal
stones
Substantial overdosage
results in CNS stimulation,
convulsions, and death
rom respiratory ailure

374
CHAPTER

ImmunotherapeuticAgents 23
T is chapter will be most use ul a er having a basic understanding o the material in
DRUGS INCLUDED
Chapter 35, Immunosuppressants, olerogens, and Immunostimulants in Goodman &
Gilmans T e Pharmacological Basis of T erapeutics, 12th Edition. In addition to the IN THIS CHAPTER
material presented here, the 12th Edition contains: Abatacept (ORENCIA)
Figure 35-3 Generation o monoclonal antibodies that illustrates the method used to Adalimumab (HUMIRA)
generate monoclonal antibodies Aldesleukin (PROLEUKIN)
T e molecular structures o immunotherapeutic drugs Ale acept (AMEVIVE)
A Case Study: Immunotherapy or Multiple Sclerosis that includes able 35-3 Phar- Alemtuzumab (CAMPATH)
macotherapy o Multiple Sclerosis listing speci c pharmacotherapies or treating Anakinra (KINERET)
multiple sclerosis (MS)
Azathioprine (IMURAN, others)
LEARNING OBJECTIVES Bacillus CalmetteGuerin (BCG; TICEBCG,
THERACYS)
Understand the mechanisms o action o drugs used to suppress the immune
response in organ transplantation and autoimmune diseases. Basiliximab (SIMULECT)
Belatacept (NULOJIX)
Understand the mechanisms o action o drugs used to stimulate the immune
system. Canakinumab (ILARIS)
Cyclosporine (NEORAL, SANDIMMUNE,
Know the untoward e ects o immunotherapeutic drugs.
GENGRAF, others)
Know the drugs used at each step in organ transplantation.
Daclizumab (ZENAPAX)
Know the drugs used in treating di erent autoimmune disorders. Ealizumab (RAPTIVA)
Know which immunotherapeutic drugs are used in combination with Etanercept (ENBREL)
other drugs. Everolimus
Glatiramer acetate (GA; COPAXONE)
MECHANISMS OF ACTION OF DRUGS USED AS IMMUNOSUPPRESSANTS Immune globulin preparations such as ATG
DRUG CLASS DRUG MECHANISM OF ACTION (see Table 23-1)
Glucocorticoids See Chapter 29 Regulate gene expression In iximab (REMICADE)
in lymphocytes and other Inter eron--2b (IFN--2b; INTRONA)
in ammatory cells by binding
Inter eron--1a (IFN--1a; AVONEX,REBIF)
glucocorticoid response elements
in DNA Inter eron--1b (IFN--1b; BETASERON)
Inter eron--1b (IFN--1b; ACTIMMUNE)
Calcineurin Inhibitors Cyclosporine Inhibit the protein phosphatase
Tacrolimus activity o calcineurin, thus blocking Lenalidomide (REVLIMID)
expression o NFAT-regulated Mitoxantrone (NOVANTRONE, others)
cytokines (see Figure 23-1)
MPA(MYFORTIC)
Antiproli erative/ Sirolimus (rapamycin) Sirolimus and everolimus inhibit Muromonab-CD3 (OKT3, ORTHOCLONE
Antimetabolic Agents Everolimus mTOR kinase activity blocking cell- OKT3)
Azathioprine cycle progression o lymphocytes
Some cancer (see Figure 23-1) Mycophenolate mo etil (MMF, CELL-CEPT)
chemotherapeutic agents (see Azathioprine is converted to Natalizumab (TYSABRI)
Chapter 45) 6-mercaptopurine which is
Rilonacept (IL-1TRAP)
Mycophenolate mo etil (MMF) incorporated into DNA as 6-thio-
MPA GTP impairing proli eration o Sirolimus (rapamycin; RAPAMUNE)
lymphocytes Some cancer chemotherapeuticagents
MMF is a prodrug that is converted (see Chapter 45)
to mycophenolic acid (MPA)
which in turn inhibits lymphocyte Tacrolimus (PROGRAF, FK506)
guanine nucleotide synthesis Tacrolimus (PROGRAF, PROTOPIC, others)
(Continued) Thalidomide (THALOMID)

375
SECTION IV Immunomodulatory Agents

THE IMMUNE RESPONSE DRUG CLASS DRUG MECHANISM OF ACTION

The immune systemis composed o 2 Biological Immune globulin preparations Immune globulin preparations
complementarymechanisms, described as Immunosuppressants such as antithymocyte (polyclonal antibodies raised
Polyclonal Antibodies globulin (ATG; see Table 23-1) in animals against human
innate and adaptive immunity, capable o
thymocytes) are cytotoxic
discriminating sel romnonsel (microbes to lymphocytes and impair
and tumors). lymphocyte unction by binding to
Characterisiticso innate (natural) immunity: cell sur ace proteins
Does not require priming Biological Muromonab-CD3 mAb that binds the CD3 cell
Broadlyreactive immunosuppressants sur ace protein on T lymphocytes
Anti-CD3 Monoclonal causing its internalization, thus
Relativelylowa nity Antibody (mAb) preventing antigen recognition
The most active component o and depleting T cells rom the
the immune systemearlyon in bloodstream
an immune response Biological Daclizumab Anti-IL-2 (anti-CD25) recombinant
Major e ectors include complement, Immunosuppressants Basiliximab mAbs that bind IL-2 receptors on
granulocytes, monocytes/macrophages, AntiIL-2 receptor activated T cells
natural killer cells, mast cells, (anti-CD25) mAbs
and basophils Biological Alemtuzumab Humanized mAb that targets
Characterisitics o adaptive Immunosuppressants CD52 on lymphocytes, monocytes,
(learned) immunity: Anti-CD52 mAb macrophages, and natural killer
cells, causing apoptosis
Antigen-specifc
Depends on antigen priming Biological In iximab In iximab, adalimumab, and
Immunosuppressants Etanercept golimumab are antiTNF- (anti
Can be veryhigh a nity Anti-TNF Reagents Adalimumab tumor necrosis actor-) mAbs
Becomes the more dominant component Golimumab that bind TNF-, thus blocking this
o the immune response over time Certolizumab pegol cytokine binding to its receptor
Certolizumab is a Fab ragment o
Important in normal immune response an antiTNF- mAb
to in ection and tumors Etanercept is a usion receptor
Mediates transplant (allogra t) rejection protein containing the ligand-
binding portion o human
and autoimmunity
TNF- receptor used to the Fc
Majore ectorsincludeBandTlymphocytes portion o human IgG1; it binds
Blymphocytes make antibodies TNF- and prevents it rom
binding its receptor
(immunoglobulins)
Tlymphocytes unction as Biological Anakinra Anakinra is a recombinant,
helper, cytolytic, and regulatory Immunosuppressants Canakinumab nonglycosylated orm o the
(suppressor) cells IL-1 Inhibitors Rilonacept naturally occurring IL-1 receptor
antagonist (IL-IRA) that blocks IL-1
Once activated byspecifcantigen in ammatory activity
recognition via cell sur ace receptors, Canakinumab is an antiIL-1 mAb
BandTlymphocytes di erentiate and that blocks this proin ammatory
divide, leading to release o soluble cytokine
mediators (cytokines and lymphokines) Rilonacept is a usion protein that
blocks IL-1 activity by binding it

Biological E alizumab E alizumab is a humanized mAb


Immunosuppressants Ale acept that binds LFA-1 and prevents
Lymphocyte Function- T-cell adhesion, tra cking, and
associated Antigen-1 activation by inhibiting interaction
(LFA-1) and LFA-3 o LFA-1 with intercellular adhesion
Inhibitors molecule (ICAM)
Ale acept is a human LFA-3-IgG
usion protein that binds CD2 on
T cells preventing their activation
by LFA-3

376
Immunotherapeutic Agents CHAPTER 2 3

TCR
IL-2
Re ce ptor
DG P IP 2

P LC 1 P O4
X
P KC IP 3 Prote in Tyros ine
Kina s e s (S rc, Ick,
X-P O 4 fyn, Za p 70)
S irolimus
Ra s Vav or
GRB2/s os [Ca 2+]
FKBP mTOR Cdk 2 IL-2
Prote in
Ra f-1
Cyclophilin
or FKBP Ca lcine urin Prolife ra tion
MEK
Cyclos porine
or Ta crolimus Pi

MAP Kina s e NFATc NFATc


P O4 IL-2 mRNA

Imme dia te
e a rly ge ne s
fos /jun NFATn NFATc

IL 2 Ge ne

FIGURE 23-1 Mechanisms o action o cyclosporine, tacrolimus, and sirolimus on T lymphocytes. Both cyclosporine and tacrolimus bind to
immunophilins (cyclophilin and FK506-binding protein [FKBP], respectively), orming a complex that binds the phosphatase calcineurin and
inhibits the calcineurin-catalyzed dephosphorylation essential to permit movement o the nuclear actor o activated T cells (NFAT) into the
nucleus. NFAT is required or transcription o interleukin-2 (IL-2) and other growth- and di erentiation-associated cytokines (lymphokines).
Sirolimus (rapamycin) works at a later stage in T-cell activation, downstream o the IL-2 receptor. Sirolimus also binds FKBP, but the FKBP-sirolimus
complex binds to and inhibits the mammalian target o rapamycin (mTOR), a kinase involved in cell-cycle progression (proli eration). TCR, T-cell
receptor. (Reproduced with permission from Pattison JM et al. Mechanisms of allograft rejection. In Neilson EG, Couser WG, eds. Immunologic Renal Dis-
eases. Philadelphia, PA: Lippincott-Raven; 1997. http://lww.com.)

TABLE 23-1 Selected Immune Globulin Preparations USES OF


COMMON
IMMUNOTHERAPEUTIC
GENERIC NAME SYNONYMS ORIGIN BRAND NAME DRUGS
Antithymocyte globulin ATG Rabbit THYMOGLOBULIN Immunosuppressants are used to dampen
the immune response in organ transplant
Botulism immune BIG-IV Human BABYBIG and autoimmune disease.
globulin intravenous
Tolerogens are used to induce a state o
Cytomegalovirus immune CMV-IGIV Human CYTOGAM antigen-specifcnonresponsiveness in
globulin intravenous organ transplantation and autoimmune
Hepatitis B immune HBIG Human HEPAGAM B, HYPERHEP B S/D, diseases while maintaining immune unc-
globulin NABI-HB tions that protect against opportunistic
in ections and secondarytumors.
Immune globulin Gamma globulin, Human GAMASTAN S/D
intramuscular IgG, IGIM Immunostimulants are used to augment
the immune systemin some patients with
Immune globulin IVIG Human CARIMUNE NF, FLEBOGAMMA in ections, cancers, and immunodefciency.
intravenous 5%, GAMMAGARD LIQUID,
GAMUNEX, IVEEGAM EN,
OCTAGAM, PRIVIGEN
(Continued)

377
SECTION IV Immunomodulatory Agents

GENERAL PRINCIPLES COMMON


GENERIC NAME SYNONYMS ORIGIN BRAND NAME
OF ORGAN TRANSPLANT
THERAPY Immune globulin IGSC Human VIVAGLOBIN
subcutaneous
Care ullyprepare patient and select best
available match or organ donor. Lymphocyte immune ALG, antithymocyte Equine ATGAM
globulin globulin (equine),
Use a multitiered approach toimmunosup- ATG (equine)
pressive drug therapywith coadministration
o several agents, each directed at a di erent Rabies immune globulin RIG Human HYPERRAB S/D, IMOGAM
molecular target in the allogra t response. RABIESHT
To gain earlyengra tment or to treat estab- Rho(D) immune globulin Rho[D] IGIM Human HYPERRHO S/D, RHOGAM
lished rejection, use greater immunosup- intramuscular
pression than used to maintain long-term
Rho(D) immune globulin Rho[D] IGIV Human RHOPHYLAC, WINRHO SDF
immunosuppression.
intravenous
Care ullyinvestigate each episode o trans-
plant dys unction to identi ycause(s). Rho(D) immune globulin Rho[D] IG Human HYPERRHO S/D MICRODOSE,
microdose microdose MICRHOGAM
An immunosuppressant drug should
be reduced or withdrawn i its toxicity Tetanus immune globulin TIG Human BAYTET
exceeds its beneft.
Vaccinia immune VIGIV Human Generic
globulin intravenous

ADVERSE EFFECTS OF
IMMUNOSUPPRESSANTS
CASE 23-1
General suppression o the immune system
increases the risko opportunisticin ections A 38-year-old male patient requires a kidney transplant because his kidneys have ailed
and secondarytumors. as the result o congenital renal disease. His biological daughter has agreed to donate
one o her kidneys.
Calcineurin inhibitors are nephrotoxic.
Glucocorticoids are diabetogenicand have a. What is typically done prior to an organ transplant such as this?
manyother adverse e ects (see Chapter 29). Organ transplantation therapy is organized around 5 general principles (see Side
Using drugs that have synergistice ects can Bar GENERAL PRINCIPLES OF ORGAN RANSPLAN HERAPY). T e rst
reduce the doses needed or therapeutic principle is care ul patient preparation and selection o the best available ABO
e cacyand limit specifctoxicities. blood typecompatible human leukocyte antigen (HLA) match or organ donation.
Because the organ donor in this case is the patients daughter, the match is likely to
be excellent. By having a good match, the possibility o acute immune rejection is
signi cantly reduced.
b. What drugs are used prophylactically to prevent organ transplant rejection?
Biological agents or induction therapy in the prophylaxis o rejection currently are
used in ~70% o de novo transplant patients and have been propelled by several ac-
tors, including the introduction o the relatively sa e antiIL-2R antibodies and the
emergence o antithymocyte globulin (A G) as a sa er and more e ective alterna-
tive to lymphocyte immune globulin or muromonab-CD3. Induction therapy with
biological agents is used to delay the use o the nephrotoxic calcineurin inhibitors
or to intensi y the initial immunosuppressive therapy in patients at high risk o
rejection (ie, repeat transplants, broadly presensitized patients, A rican American
patients, or pediatric patients).
Biologicals or induction can be divided into 2 groups: the depleting agents and the
immune modulators. T e depleting agents consist o lymphocyte immune globulin,
A G, and muromonab-CD3 mAb (the latter also produces immune modulation);
their e cacy derives rom their ability to deplete the recipients CD3-positive cells
at the time o transplant and antigen presentation. A G is the most requently used
depleting agent. Lymphocyte immune globulin and OK 3 are rarely used because
o poorer e cacy and acute side e ects, respectively. Alemtuzumab, a humanized
anti-CD52 monoclonal antibody that produces prolonged lymphocyte depletion, is
increasingly used o -label as induction therapy in transplantation.
(Continued)
378
Immunotherapeutic Agents CHAPTER 2 3

T e second group o biological agents used or induction are the immune modula-
tors, speci cally the antiIL-2R mAbs. T ese agents do not deplete lymphocytes,
with the possible exception o regulatory cells, but rather block IL-2mediated
-cell activation by binding to the chain o IL-2R. T ese agents include anakinra,
canakinumab, and rilonacept.
c. What is the approach to using immunosuppressants in organ transplants?
Immunosuppressive drugs are used to dampen the immune response in organ
transplantation. A multitiered approach to immunosuppressive drug therapy is
employed. Several agents, each o which is directed at a di erent molecular target
within the allogra response (see MECHANISMS OF AC ION OF DRUGS USED
AS IMMUNOSUPPRESSAN S), are used simultaneously. Synergistic e ects per-
mit use o the various agents at relatively low doses, thereby limiting speci c toxici-
ties while maximizing the immunosuppressive e ect. T erapy typically involves a
calcineurin inhibitor, glucocorticoids, and mycophenolate, each directed at a dis-
crete site in -cell activation. Glucocorticoids, azathioprine, cyclosporine, tacroli-
mus, mycophenolate, sirolimus, and various monoclonal and polyclonal antibodies
all are approved or use in transplantation.
Greater immunosuppression is required to gain early engra ment and/or to treat
established rejection than to maintain long-term immunosuppression. Maintenance
immunosuppression consists o a calcineurin inhibitor (cyclosporine or tacrolimus),
glucocorticoids, and an antimetabolite (azathioprine or mycophenolate). Mycophe-
nolate has largely replaced azathioprine as part o the standard immunosuppressive
regimen a er transplant.
d. What are the risks of immunosuppressant drugs?
Immunosuppressant therapies require li elong use and nonspeci cally suppress the
entire immune system, exposing patients to considerably higher risks o in ection
and cancer. T e calcineurin inhibitors and glucocorticoids, in particular, are neph-
rotoxic and diabetogenic, respectively, thus restricting their use ulness in a variety
o clinical settings. o minimize risks a er engra ment is achieved, lower-dose
maintenance drug protocols are employed. In addition, a drug should be reduced
or withdrawn i its toxicity exceeds its bene t.
e. What might cause organ transplant dysfunction?
T ere can be many causes o transplant dys unction, including rejection, drug toxicity,
and in ection. T ese various problems can and o en do coexist. Organ-speci c
problems (eg, obstruction in the case o kidney transplants) also must be considered.
A general principle o organ transplantation is to investigate each episode o trans-
plant dys unction to identi y the cause(s) and adjust therapy.

CASE 23-2
A 62-year-old woman su ers rom severe rheumatoid arthritis. Her pharmaco-
therapy includes several agents that are also commonly used or organ transplant
pharmacotherapy.
a. What is the therapeutic rationale in using immunosuppressant drugs to treat
this patients arthritis?
Rheumatoid arthritis is an autoimmune disease driven largely by activated cells,
giving rise to cellderived cytokines, such as IL-1 and NF-. Nonsteroidal anti-
inf ammatory drugs (NSAIDs) are widely used in patients with rheumatoid arthri-
tis to relieve symptoms o pain and reduce inf ammation (see Chapter 22), but they
have minimal e ect on disease progression and joint de ormity. DMARDs (disease-
modi ying antirheumatic drugs), on the other hand, reduce the disease activity
o rheumatoid arthritis and retard the progression o arthritic tissue destruction.
T ese include a diverse group o small molecule nonbiologicals and biological
agents (mainly antibodies or binding proteins), as summarized in able 22-3,
(Continued)
379
SECTION IV Immunomodulatory Agents

Disease-Modi ying Antirheumatic Drugs. Many o the agents listed in able 22-3
INDUCTION OF
include the immunosuppressants covered in this chapter, including cyclosporine,
IMMUNOLOGICAL
azathioprine, NF- antagonists (in iximab, adalimumab, etanercept), IL-1 recep-
TOLERANCE BY tor antagonists (anakinra), and costimulatory blockers (abatacept).
COSTIMULATORY
Cyclosporine is used in severe cases o rheumatoid arthritis that have not
BLOCKADE responded to methotrexate. It can be combined with methotrexate, but the levels
Immunological tolerance is the state o o both drugs must be monitored closely. Azathioprine is used in patients with
nonresponsiveness o the immune system severe rheumatoid arthritis. Lower initial doses o azathioprine are used or rheu-
to a specifcantigen or group o antigens. matoid arthritis than used to prevent organ rejection. Biological DMARDs remain
Antigen-specifctolerance would theoreti- reserved or patients with persistent moderate or high disease activity and indica-
callyeliminate the risko opportunistic tors o poor prognosis such as unctional impairment, radiographic bony erosions,
in ections and secondarytumors that occur extra-articular disease, and rheumatoid actor positivity. T erapy is tailored to the
with immunosuppression. individual patient, and the use o these agents must be weighed against their poten-
tially serious adverse ef ects. In iximab is approved in the United States or treating
Induction o antigen-specifcimmune
the symptoms o rheumatoid arthritis and is typically used in combination with
responses byTlymphocytes requires 2
methotrexate in patients who do not respond to methotrexate alone. Etanercept is
signals (re erred to as costimulation; see
approved or treatment o rheumatoid arthritis in patients who have not responded
Figure 23-2):
to other treatments and can be used in combination with methotrexate in patients
An antigen-specifcsignal via the who have not responded adequately to methotrexate alone.
T-cell receptor
Acostimulatorysignal provided by
the interaction o molecules such as
CD28 on the Tlymphocyte and CD80
and CD86 on the antigen-presenting
AP C
cell (APC) 1 MHC
AP C
1 MHC

Ater a Tcell is activated, it expresses addi- Antige n Antige n


tional costimulatorymolecules. 2 CD80/CD86
TCR TCR
CD152is the CD40ligand, which interacts CD28
with CD40as a costimulatorypair. CD28
T Ce ll T Ce ll
CD154 (CTLA4) interacts with CD80 and
CD86 to dampen or downregulate an
immune response.
Tolerance can be achieved byblocking
costimulatoryinteractions.
Abatacept and belatacept are usion AP C
proteins that contain a portion o CTLA4 1 MHC CD40 1 MHC
AP C
CD80/CD86
used to IgG1.
Antige n Antige n
These agents bind CD80 and CD86 on 2 CD80/CD86 CD80/CD86

APCs and blockinteractions with CD28 TCR CD152


(+) CD154
TCR
a ba ta ce pt
onTcells. ()
CD28
be la ta ce pt
T Ce ll T Ce ll

CD28

FIGURE 23-2 T-Cell Activation. Two signals are required for T-cell activation. The upper left
panel shows Signal 1, stimulation via the T-cell receptor (TCR) by the MHC-antigen complex on
the antigen presenting cell (APC). The upper right panel depicts Signal 2, co-stimulatory interac-
tions between CD28 on the T cell (co-stimulatory receptor) and the co-stimulatory ligand on the
APC, CD80/CD86. Signal 1 in the absence of Signal 2 does not result in T-cell activation. The lower
left panel shows additional co-stimulatory interactions that occur after T-cell activation. CD152
interaction with CD40 on the APC enhances T-cell activation (+), whereas CD154 interaction with
CD80/86 attentuates (-) T-cell activation. The lower right panel shows the mechanism of action
of abatacept and belatacept, fusion proteins that contain the CTLA4 domain of CD154. These
agents block co-stimulation of T cells by binding CD80/CD86.

380
Immunotherapeutic Agents CHAPTER 2 3

CASE 23-3
A 24-year-old woman who has had symptoms o abdominal pain, diarrhea, and blood
in the stool is diagnosed with Crohns disease.
a. What is the pathophysiology of Crohns disease and what agents are typically
used to initiate therapy?
Crohns disease is an inf ammatory bowel disease that is associated with activation
o the immune system. Drugs used to treat mild to moderately active Crohns dis-
ease include sul asalazine (an intestinal anti-inf ammatory), budesonide (an enteric
glucocorticoid with local anti-inf ammatory e ects), and oral corticosteroids (see
Chapter 33).
b. What is the rationale for using TNF- antagonists in the treatment of Crohns
disease?
Patients with Crohns disease have elevated levels o NF- in their stools. As a
result, a number o anti- NF agents are used or the treatment o this disorder,
including inf iximab, adalimumab, and certolizumab pegol. Because these agents
are associated with signi cant risks, they are reserved or patients with moderate to
severe Crohns disease who have ailed to respond to conventional therapy. Also see
Case 33-6.

MECHANISMS OF ACTION OF TOLEROGENS


DRUG CLASS DRUG MECHANISM OF ACTION
Selective T-Cell Costimulation Belatacept Contains the CD80- and CD86-binding region o CTLA4 (a CD28 homolog), and
Blocker competitively inhibits CD28 co-stimulatory interactions o T cells with APCs

Selective T-Cell Costimulation Abatacept Contains a congener o the CD80- and CD86-binding region o CTLA4 (a CD28 homolog),
Blocker and competitively inhibits CD28 co-stimulatory interactions o T cells with APCs

MECHANISMS OF ACTION OF IMMUNOSTIMULANT AND IMMUNOMODULATORYDRUGS


DRUG CLASS DRUG MECHANISM OF ACTION
Immunostimulant Thalidomide Multiple mechanisms o action (see Chapter 46); mechanisms o action as an
Thalidomide and Lenalidomide immunostimulatory drug are unclear
Its Analogs

ImmunostimulantBCG Bacillus Calmette Attenuated, live culture o the BCG o Mycobacterium bovis that induces a granulomatous
Guerin (BCG) reaction at the site o administration; mechanisms o action as an immunostimulatory drug
are unclear

Immunostimulants Inter eron--2b Bind to specif c cell-sur ace receptors that initiate a series o intracellular events: induction
Recombinant Cytokines, (IFN--2b) o certain enzymes, inhibition o cell proli eration, and enhancement o immune activities,
and IFNs Inter eron--1b including increased phagocytosis by macrophages and augmentation o specif c
(IFN--1b) cytotoxicity by T lymphocytes

Immunostimulants Aldesleukin Simulates the biological e ects o native IL-2: enhancement o lymphocyte proli eration
Recombinant Cytokines, and growth o IL-2dependent cell lines, enhancement o lymphocyte-mediated
Interleukin-2 (IL-2) cytotoxicity and killer cell activity, and induction o IFN- activity

ImmunostimulantsPassive See Table 23-1 Used to provide nonspecif c or highly specif c immunoglobulins to patients with
Immunization immunodef ciencies, when there is inadequate time or active immunization, or when an
existing disease can be ameliorated by passive antibodies

Immunomodulators Inter eron--1a IFN--1a and IFN--1b have antiviral and immunomodulatory properties: suppress
Recombinant Cytokines, (IFN--1a) proli eration o T lymphocytes, inhibit their movement into the CNS rom the periphery,
1 IFNs Inter eron--1b and shi t the cytokine prof le rom pro- to anti-in ammatory types
(IFN--1b)
(Continued)
381
SECTION IV Immunomodulatory Agents

DRUG CLASS DRUG MECHANISM OF ACTION

Immunomodulators Glatiramer Induces T-helper type 2 cells that enter the CNS; mediates bystander suppression at sites o
Universal Altered Peptide acetate (GA) in ammation
Ligands (APLs)

Immunomodulators Mitoxantrone Intercalates DNA and suppresses cellular and humoral immune response
Anthracenedione-Derivative

ImmunomodulatorsmAb Natalizumab mAb directed against the adhesion molecule 4 integrin, antagonizes interactions o 4-
containing integrin heterodimers on the sur ace o activated lymphocytes and monocytes,
thereby blocking their movement out o the bloodstream

CASE 23-4
A 31-year-old woman is diagnosed with relapsing-remitting multiple sclerosis (MS).
a. What is the pathophysiology of MS?
MS is a demyelinating inf ammatory disease o the CNS white matter that displays
a triad o pathogenic ndings: mononuclear cell in ltration, demyelination, and
scarring (gliosis). T e peripheral nervous system is uninvolved. T e disease, which
may be episodic or progressive, occurs in early to middle adulthood with prevalence
increasing rom late adolescence to 35 years o age and then declining. T e cause is
unknown, but there is a strong genetic component, with a number o immune-related
genetic variants, many o which are shared with di erent autoimmune diseases. In
addition, MS patients have activated cells that are reactive to di erent myelin anti-
gens, and autoantibodies to myelin proteins can be eluted rom CNS plaque tissue.
b. What is the rationale for using -1 interferons in treating this patients
symptoms?
T e -1 inter erons (IFN--1a, IFN--1b) are immunomodulatory and suppress the
proli eration and activation o lymphocytes by inter ering with costimulatory mole-
cules, inhibit -cell movement into the CNS rom the periphery, and shi the cytokine
pro le rom pro- to anti-inf ammatory types. T ese agents reduce the recurrence o
relapsing-remitting attacks by one-third and reduce disease progression.

KEY CONCEPTS
Immunosuppressant drugs are essential in preventing grant rejection in organ
transplantation.
Immunosuppressant drugs also have application in reducing symptoms and
disease progression in a variety o autoimmune diseases.
Immunosuppressant drugs with di erent mechanisms o action are typically
used in combination to achieve greater e cacy and reduce dosing to reduce
risk o drug-speci c toxicities.
General suppression o the immune system increases the risk o secondary
in ections and cancer.
Many immunosuppressants must be used chronically over the li e o the patient,
which increases the risk o adverse e ects.

SUMMARY QUIZ

QUESTION 23-1 Cyclosporine was one o the rst immunosuppressants to be used or


organ transplant therapy. T e mechanism o action o cyclosporine is to
a. intercalate DNA in proli erating cells.
(Continued)
382
Immunotherapeutic Agents CHAPTER 2 3

b. block purine metabolism in proli erating cells.


c. block CD52 on the sur ace o activated cells.
d. block the protein phosphatase activity o calcineurin.
e. block the protein kinase activity o m OR.

QUESTION 23-2 acrolimus is one o the most e ective immunosuppressant drugs in


routine use. T e mechanism o action o tacrolimus is to
a. intercalate DNA in proli erating cells.
b. block guanine nucleotide synthesis in proli erating cells.
c. block CD52 on the sur ace o activated cells.
d. block the protein phosphatase activity o calcineurin.
e. block the protein kinase activity o m OR.

QUESTION 23-3 Sirolimus is used in combination with other drugs in patients receiv-
ing organ transplants. T e mechanism o action o sirolimus is to
a. intercalate DNA in proli erating cells.
b. block guanine nucleotide synthesis in proli erating cells.
c. block CD52 on the sur ace o activated cells.
d. block the protein phosphatase activity o calcineurin.
e. block the protein kinase activity o m OR.

QUESTION 23-4 Mycophenolate mo etil (MMF) is typically used in combination with


other agents or immunosuppression pharmacotherapy. T e mechanism o action o
mycophenolate is to
a. intercalate DNA in proli erating cells.
b. block guanine nucleotide synthesis in proli erating cells.
c. block CD52 on the sur ace o activated cells.
d. block the protein phosphatase activity o calcineurin.
e. block the protein kinase activity o m OR.

QUESTION 23-5 Adalimumab is a monoclonal antibody used in a variety o autoim-


mune diseases. T e molecular target o adalimumab is
a. CD3 on lymphocytes.
b. IL-2 receptor on cells.
c. CD52 on lymphocytes, monocytes, macrophages, and natural killer cells.
d. NF-.
e. IL-1.

SUMMARY QUIZ ANSWER KEY


QUESTION 23-1 Answer is d. Cyclosporin targets intracellular signaling pathways
induced as a consequence o -cell receptor activation. T e drug binds to an immu-
nophilin known as cyclophilin which subsequently interacts with calcineurin to inhibit
its protein phosphatase activity (see Figure 23-1). Calcineurin-catalyzed dephosphory-
lation o the protein nuclear actor o activated lymphocytes (NFA ) is required or
movement o NFA into the nucleus. NFA , in turn, is required to induce a number o
cytokine genes, including that or interleukin-2 (IL-2), a prototypic -cell growth and
di erentiation actor.
QUESTION 23-2 Answer is d. Although structurally unrelated to cyclosporine, tacro-
limus inhibits -cell signal transduction by almost the same mechanism. acrolimus
binds the immunophilin FK506-binding protein-12 (FKBP-12), which is structurally
(Continued)
383
SECTION IV Immunomodulatory Agents

related to cyclophilin, the target o cyclosporine. A complex o tacrolimus-FKBP-12,


Ca2+, calmodulin, and calcineurin then orms and the protein phosphatase activity
o calcineurin is inhibited (see Figure 23-1). T e inhibition o calcineurin prevents
dephosphorylation and nuclear translocation o NFA and inhibits -cell activation.
T us, although the intracellular receptors di er, cyclosporine and tacrolimus target the
same pathway or immunosuppression. Because o perceived slightly greater e cacy
and ease o blood level monitoring, tacrolimus has become the pre erred calcineurin
inhibitor in most transplant centers
QUESTION 23-3 Answer is e. Sirolimus inhibits -lymphocyte activation and proli -
eration downstream o the IL-2 receptor and other -cell growth actor receptors (see
Figure 23-1). Like cyclosporine and tacrolimus, the therapeutic action o sirolimus
requires ormation o a complex with an immunophilin, in this case FKBP-12. How-
ever, the sirolimusFKBP-12 complex does not a ect calcineurin activity. It binds to
and inhibits a protein kinase, designated m OR, which is a key enzyme in cell-cycle
progression. Inhibition o m OR blocks cell-cycle progression at the G1 S phase
transition. In animal models, sirolimus not only inhibits transplant rejection, gra -
versus-host disease, and a variety o autoimmune diseases, but its e ect also lasts
several months a er discontinuing therapy, suggesting a tolerizing e ect. A newer
indication or sirolimus is the avoidance o calcineurin inhibitors, even when patients
are stable, to protect kidney unction.
QUESTION 23-4 Answer is b. MMF is a prodrug that is rapidly hydrolyzed to the
active drug, MPA, a selective, noncompetitive, reversible inhibitor o inosine mono-
phosphate dehydrogenase (IMPDH), an important enzyme in the de novo pathway o
guanine nucleotide synthesis. B and lymphocytes are highly dependent on this path-
way or cell proli eration, while other cell types can use salvage pathways; MPA there-
ore selectively inhibits lymphocyte proli eration and unctions, including antibody
ormation, cellular adhesion, and migration. MMF is typically used in combination
with glucocorticoids and a calcineurin inhibitor but not with azathioprine.
QUESTION 23-5 Answer is d. Adalimumab is one o several anti- NF agents used to
treat autoimmune diseases. NF is a proinf ammatory cytokine that has been implicated
in the pathogenesis o several immune-mediated intestinal, skin, and joint diseases.
Other anti- NF agents include inf iximab (a monoclonal antibody), golimumab (a
monoclonal antibody), certolizumab (a Fab ragment directed toward NF-), and
etanercept (a usion protein that contains the ligand-binding portion o a human NF-
receptor used to the Fc portion o human IgG1). Adalimumab is approved or use in
rheumatoid arthritis, ankylosing spondylitis, Crohns disease, juvenile idiopathic arthri-
tis, plaque psoriasis, and psoriatic arthritis. All anti- NF agents increase the risk or
serious in ections, lymphomas, and other malignancies.

SUMMARYTABLE: IMMUNOMODULATORY DRUGS


TOXICITIES
CLASS AND SUBCLASSES NAMES CLINICAL USES COMMON UNIQUE; CLINICALLY IMPORTANT
Glucocorticoids See Chapter 29 Commonly combined with Extensive use o steroids See Chapter 29
other immunosuppressive o ten results in disabling
agents to prevent and treat and li e-threatening
transplant rejection adverse e ects, including
Treatment o many growth retardation
autoimmune disorders in children, avascular
Used to limit allergic reactions necrosis o bone,
that occur with other osteopenia, increased
immunosuppressive agents risk o in ection, poor
and to block f rst-dose cytokine wound healing, cataracts,
storm caused by treatment hyperglycemia, and
with muromonab-CD3 and to hypertension
a lesser extent ATG
(Continued)
384
Immunotherapeutic Agents CHAPTER 2 3

TOXICITIES
CLASS AND SUBCLASSES NAMES CLINICAL USES COMMON UNIQUE; CLINICALLY IMPORTANT
Calcineurin Inhibitors Cyclosporine Indicated or the prophylaxis Nephrotoxicity (occurs in Because o its potential or
Tacrolimus o solid-organ allogra t the majority o patients), nephrotoxicity, tacrolimus blood
rejection neurotoxicity (eg, levels and renal unction should
Tacrolimus has become the tremor, headache, motor be monitored closely, especially
pre erred calcineurin inhibitor disturbances, seizures), GI when tacrolimus is used with other
in most transplant centers complaints, hypertension, potentially nephrotoxic drugs
Severe cases o rheumatoid hyperkalemia, Coadministration with
arthritis and psoriasis in which hyperglycemia, and cyclosporine results in additive
other therapies have ailed diabetes (combined use or synergistic nephrotoxicity;
o calcineurin inhibitors there ore, a delay o at least 24 h is
and glucocorticoids is required when switching a patient
particularly diabetogenic) rom cyclosporine to tacrolimus
Increased risk o Great care must be taken to
secondary tumors and di erentiate renal toxicity rom
opportunistic in ections rejection in kidney transplant
patients

Antiproli erative/ Sirolimus Sirolimus and everolimus are Sirolimus and I azathioprine and allopurinol
Antimetabolic Agents (rapamycin) indicated or prophylaxis o everolimus in renal are used concurrently, the
Everolimus organ transplant rejection transplant patients is azathioprine dose must be
Azathioprine usually in combination with a associated with a dose- decreased to 25-33% o the usual
Mycophenolate reduced dose o calcineurin dependent increase in dose; it is best not to use these 2
mo etil (MMF) inhibitor and glucocorticoids serum cholesterol and drugs together
MPA Sirolimus has been used triglycerides Coadministration o azathioprine
with glucocorticoids and Azathioprine is associated with other myelosuppressive
mycophenolate in patients with bone marrow agents or ACE inhibitors include
experiencing or at high risk suppression, including leukopenia, thrombocytopenia,
or calcineurin inhibitor leukopenia (common), and anemia
associated nephrotoxicity thrombocytopenia MMF is teratogenic; women
Azathioprine is indicated as (less common), and/or o childbearing age must use
an adjunct or prevention o anemia (uncommon), e ective contraception
organ transplant rejection hepatotoxicity, alopecia,
(but not with MMF) and in GI toxicity, pancreatitis
severe rheumatoid arthritis MMF principal toxicities
are GI and hematologic
Increased risk o
neoplasms and in ections

Biological Immune globulin ATG is used or induction Polyclonal antibodies Serum sickness and
Immunosuppressants preparations immunosuppression, although are xenogeneic proteins glomerulonephritis can occur;
Polyclonal Antibodies such as ATG (see the only approved indication that can elicit major anaphylaxis is a rare event
Table 23-1) is in the treatment o acute side e ects, including Hematologic complications
renal transplant rejection ever and chills with the include leukopenia and
in combination with other potential or hypotension thrombocytopenia
immunosuppressive agents Increased risk o in ection
and malignancy

Biological Muromonab-CD3 Indicated or treatment Major side e ect is the Potentially atal pulmonary
Immunosuppressants o acute organ transplant cytokine release syndrome edema, acute respiratory distress
Anti-CD3 Monoclonal rejection caused by release o syndrome, cardiovascular collapse,
Antibody (mAb) Rarely used or induction and proin ammatory cytokines cardiac arrest, and arrhythmias
rejection therapy because o by activated Tcells Administration o glucocorticoids
toxicity and the availability o Clinical mani estations be ore the injection o
ATG and alemtuzumab include high ever, muromonab-CD3 prevents
chills/rigor, headache, the release o cytokines and is
tremor, nausea, vomiting, standard procedure
diarrhea, abdominal A ully competent resuscitation
pain, malaise, myalgias, acility must be immediately
arthralgias, and available or patients receiving
generalized weakness their f rst several doses o this
Increased risk o in ections therapy
and neoplasms
(Continued)
385
SECTION IV Immunomodulatory Agents

TOXICITIES
CLASS AND SUBCLASSES NAMES CLINICAL USES COMMON UNIQUE; CLINICALLY IMPORTANT

Biological Daclizumab Used or prophylaxis o acute No cytokine-release


Immunosuppressants Basiliximab organ rejection in adult syndrome has been
AntiIL-2 Receptor patients observed with these
(Anti-CD25) Monoclonal antibodies, but
Antibodies (mAbs) anaphylactic reactions
can occur
Lymphoproli erative
disorders and
opportunistic in ections
may occur, but the
incidence appears
remarkably low

Biological Alemtuzumab Approved or use in chronic


Immunosuppressants lymphocytic leukemia
Anti-CD52 Monoclonal Some use in renal
Antibody (mAb) transplantation because it
produces prolonged T- and
B-cell depletion and allows
drug minimization, but
large controlled studies
o e cacy or sa ety are
not available

Biological In iximab Used in treating several About 1 o 6 patients Increase the risk or serious
Immunosuppressants Etanercept immune-mediated intestinal, receiving in iximab in ections, lymphomas, and other
Anti-TNF Reagents Adalimumab skin, and joint diseases experiences an in usion malignancies
Golimumab including rheumatoid arthritis, reaction characterized Patients should be closely
Certolizumab pegol Crohns disease, ankylosing by ever, urticaria, monitored or the signs and
spondylitis, juvenile idiopathic hypotension, and symptoms o in ection during
arthritis, plaque psoriasis, dyspnea within 1-2 h and a ter treatment including
psoriatic arthritis, and the possible development o
ulcerative colitis tuberculosis in patients who
tested negative prior to
initiating therapy
Fatal lymphomas and other
malignancies have been reported
in children, adolescent and
young adult patients treated
with TNF blockers

Biological Anakinra Anakinra is used alone or in


Immunosuppressants Canakinumab combination with anti-TNF
IL-1 Inhibitors Rilonacept agents or management
o joint disease in
rheumatoid arthritis
Canakinumab approved
or treating cryopyrin-
associated periodic
syndromes (CAPS)

Biological E alizumab Pretransplant therapy, Risk o posttransplant Progressive multi ocal


Immunosuppressants Ale acept treatment o psoriasis lymphoproli erative leukoencephalopathy (PML) also
Lymphocyte Function- diseases (PTLD) with has occurred during therapy with
Associated Antigen-1 higher-dose e alizumab e alizumab
(LFA-1) and LFA-3
Inhibitors
(Continued)

386
Immunotherapeutic Agents CHAPTER 2 3

TOXICITIES
CLASS AND SUBCLASSES NAMES CLINICAL USES COMMON UNIQUE; CLINICALLY IMPORTANT

Selective T-Cell Belatacept Prevention o rejection in Increased risk o in ections Risk o PTLD is signif cantly
Costimulation Blocker renal transplantation (such as PML) and increased in patients without
malignancies (including immunity to Epstein-Barr virus
PTLD predominantly (EBV), thus contraindicated in
involving the CNS) patients who are EBVseronegative
Common adverse or unknown serostatus
e ects are anemia,
diarrhea, peripheral
edema, constipation,
hypertension, pyrexia,
gra t dys unction, cough,
nausea, vomiting,
headache, hypokalemia,
hyperkalemia, and
leukopenia

Selective T-Cell Abatacept Treatment o moderate to Increased risk o in ections Increased risk o serious adverse
Costimulation Blocker severe rheumatoid arthritis, (especially in combination events in patients with COPD
juvenile idiopathic arthritis with TNF antagonists) and
malignancies

Immunostimulant Thalidomide Thalidomide is indicated Lenalidomide causes Teratogenic and should never
Thalidomide and Its Lenalidomide or treatment o erythema signif cant neutropenia be taken by women who are
Analogs nodosum leprosum (see and thrombocytopenia pregnant or who could become
Chapter 42) and multiple in almost all patients, pregnant while taking the drug
myeloma, and has orphan which requires weekly Lenalidomide is associated with
drug status or mycobacterial blood counts and dosing a signif cant risk or deep vein
in ections, Crohns disease, adjustments thrombosis
HIV-associated wasting,
Kaposi sarcoma, lupus,
myelof brosis, brain
malignancies, leprosy, gra t-
versus-host disease, and
aphthous ulcers
Lenalidomide is approved
or treating trans usion-
dependent anemia due
to low- or intermediate-
risk 5q minus cytogenetic
myelodysplastic syndromes

ImmunostimulantBCG Bacillus Calmette Indicated or the treatment Hypersensitivity, shock,


Guerin (BCG) and prophylaxis o carcinoma chills, ever, malaise, and
in situ o the urinary bladder immune complex disease
and or prophylaxis o primary
and recurrent stage Ta and/
or T1 papillary tumors a ter
transurethral resection

Immunostimulants Inter eron--2b Indicated in the treatment o Flu-like symptoms, Risk o developing pulmonary
Recombinant Cytokines, (IFN--2b) a variety o tumors, including including ever, chills, and hypertension
Inter erons hairy cell leukemia, malignant headache Risk o cardiovascular
melanoma, ollicular (hypotension, arrhythmias, and
lymphoma, and AIDS-related rarely, cardiomyopathy, myocardial
Kaposi sarcoma, and is also in arction) and CNS (depression,
indicated or in ectious con usion) e ects
diseases, chronic hepatitis B,
and condylomata acuminata
(Continued)

387
SECTION IV Immunomodulatory Agents

TOXICITIES
CLASS AND SUBCLASSES NAMES CLINICAL USES COMMON UNIQUE; CLINICALLY IMPORTANT

Immunostimulants Inter eron--1b Indicated to reduce requency Fever, headache, rash, May increase mortality in patients
Recombinant Cytokines, (IFN--1b) and severity o serious atigue, GI distress, with idiopathic pulmonary f brosis
Inter erons in ections associated with anorexia, weight loss,
chronic granulomatous myalgia, and depression
disease and to delay time
to progression in severe
malignant osteopetrosis

Immunostimulants Aldesleukin Indicated or the treatment o Increased risk o Associated with serious
Recombinant Cytokines, adults with metastatic renal disseminated in ection cardiovascular toxicity resulting
Interleukin-2 cell carcinoma and melanoma due to impaired rom capillary leak syndrome
neutrophil unction (loss o vascular tone and leak o
plasma proteins and uid into the
extravascular space); hypotension,
reduced organ per usion, and
death may occur

Immunostimulants See Table 23-1 Indicated when an individual Plasma-derived products


Passive Immunization is def cient in antibodies carry the theoretical
because o a congenital or risk o transmission o
acquired immunodef ciency, in ectious disease
when an individual with a
high degree o risk is exposed
to an agent and there is
inadequate time or active
immunization (eg, measles,
rabies, hepatitis B), or when a
disease is already present but
can be ameliorated by passive
antibodies (eg, botulism,
diphtheria, tetanus)
Intravenous immunoglobulin
(IVIG) also used or a
variety o autoimmune and
in ammatory diseases

Immunomodulators Inter eron--1a Treatment o relapsing- Flu-like symptoms ( ever,


Recombinant Cytokines, (IFN--1a) remitting MS (RRMS) to chills, myalgia) and
-1 Inter erons Inter eron--1b resolve acute attacks, reduce injection-site reactions
(IFN--1b) the requency o clinical
exacerbations, and slow
progression o disability

Immunomodulators Glatiramer acetate Treatment o RRMS to reduce


Universal Altered (GA) the requency o clinical
Peptide Ligands (APLs) exacerbations and slow
progression o disability

Immunomodulators Mitoxantrone Treatment o worsening Cardiac toxicity; generally FDA recommends that LVEF
Anthracenedione orms o RRMS and secondary can be tolerated only up be evaluated be ore initiating
Derivative progressive MS (SPMS) to to an accumulated dose therapy, prior to each dose, and
reduce requency o relapses o 100-140 mg/m 2 annually a ter patients have
and slow progression f nished treatment to detect
late-occurring cardiac toxicity

Immunomodulators- Natalizumab Treatment o RRMS to reduce Headache, joint pain, Increases the risk o developing
Monoclonal Antibody requency o episodes and injection site reaction, progressive multi ocal
(mAb) slow progression o disability edema, and in usion leukoencephalopathy (PML)
Treatment o moderate to reaction
severe Crohns disease

388
CHAPTER

PulmonaryPharmacology 24
T is chapter will be most use ul a er having a basic understanding o the material in
DRUGS INCLUDED IN THIS
Chapter 36, Pulmonary Pharmacology in Goodman & Gilmans T e Pharmacological
Basis of T erapeutics, 12th Edition. In addition to the material presented here, the 12th CHAPTER
Edition contains: Albuterol (salbutamol; VENTOLIN, PROVEN-
A description o the pathogenesis o asthma, including Figure 36-1: Cellular mecha- TIL, ACCUNEB, others)
nisms o asthma Ambrisentan (LETAIRIS)
A description o the pathogenesis o chronic obstructive pulmonary disease (COPD), Ar ormoterol (BROVENA)
including Figure 36-2: Cellular mechanisms in chronic obstructive pulmonary disease Beclomethasone dipropionate (QVAR)
A description o the various routes o drug delivery to the lungs, including Figure 36-3: Benzonatate (TESSALON, others)
Schematic representation o the deposition o inhaled drugs
Bosentan (TRACLEER)
T e molecular structures o some o the drugs used to treat asthma, COPD, and
Budesonide (PULMICORT,others)
other pulmonary diseases
Budesonide/ ormoterol (SYMBICORT)
LEARNING OBJECTIVES Ciclesonide (ALVESCO, OMNARIS)
Understand the mechanisms o action o bronchodilator drugs used to relax air- Codeine
way smooth muscle and drugs used to prevent bronchoconstriction. Dextromethorphan
Understand the anti-in ammatory mechanism o action o corticosteroids, and DNAase (dornase al a, PULMOZYME)
the role o inhaled and oral corticosteroids in the pharmacotherapy o asthma. Doxapram(DOPRAM, others)
Understand the mechanism o action o mucolytic agents. Epoprostenol (prostacyclin, PGI2; FLOLAN,
Understand the mechanisms o action o antitussive drugs. others)
Understand the mechanisms o action o drugs used to treat pulmonary artery Flunisolide (AEROBID)
hypertension (PAH). Fluticasone (AEROSPAN, FLOVENT)
Know the untoward e ects o the various bronchodilator drugs, corticosteroids, Fluticasone/salmeterol (ADVAIR)
antitussive drugs, and drugs used to treat PAH. Formoterol (FORADIL, others)
Know which patients should be treated and when treatment should be initiated Guai enesin
in patients with asthma, COPD, and PAH.
Hydrocortisone
Know which drugs are most e ective in treating patients with asthma, COPD, Iloprost (VENTAVIS)
and PAH.
Indacaterol (ARCAPTANEOHALER)
Know which drugs can be used in combination to treat asthma, COPD,
Ipratropiumbromide (ATROVENT,others)
and PAH.
Ipratropium/albuterol (COMBIVENT,
DUONEB, others)
MECHANISMS OF ACTION OF DRUGS USED TO TREAT PULMONARY DISORDERS Levalbuterol (XOPENEX)
DRUG CLASS DRUG MECHANISM OF ACTION Metaproterenol (ALUPENT,METAPREL)
2 Adrenergic Albuterol 2 Agonists produce bronchodilation by Methylprednisolone
AgonistsShort- (salbutamol) directly stimulating 2 receptors in airway Mometasone (ASMANEX)
Acting (3-6 h) Levalbuterol smooth muscle (see Figure 24-1), resulting
Metaproterenol in bronchodilation and a rapid decrease in Montelukast (SINGULAIR)
Pirbuterol airways resistance N-acetylcysteine (MUCOMYST,others)
Terbutaline In addition, 2 agonists act indirectly by
Omalizumab
inhibiting the release o bronchoconstrictor
mediators rom in ammatory cells and Pirbuterol (MAXAIR)
o bronchoconstrictor neurotransmitters Prednisolone
rom airway nerves (see Side Bar INDIRECT
ACTIONS OF 2 AGONISTS TO CAUSE Prednisone
BRONCHODILATION) Salmeterol (SEREVENT)
(Continued) (continues)

389
SECTION IV Inf ammation, Immunomodulation, and Hematopoiesis

DRUGS INCLUDED IN THIS DRUG CLASS DRUG MECHANISM OF ACTION


CHAPTER (Cont.) 2 Adrenergic Salmeterol 2 Agonists produce bronchodilation by
Sildena l (REVATIO) AgonistsLong- Formoterol directly stimulating 2 receptors in airway
Acting (Long-Acting Ar ormoterol smooth muscle (see Figure 24-1), resulting
Tadala l (ADCIRCA) Agonists [LABAs]; Indacaterol in bronchodilation and a rapid decrease in
Terbutaline (BRETHINE, others) duration >12 h) airways resistance
In addition, 2 agonists act indirectly by
Tiotropiumbromide (SPIRIVA) inhibiting the release o bronchoconstrictor
Treprostinil (REMODULIN, TYVASO) mediators rom in ammatory cells and
o bronchoconstrictor neurotransmitters
Triamcinolone (AZMACORT,others)
rom airway nerves (see Side Bar INDIRECT
Za rlukast (ACCOLATE) ACTIONS OF 2 AGONISTS TO CAUSE
Zileuton (ZYFLO) BRONCHODILATION)

Methylxanthines Theophylline Mechanisms are uncertain, but nonselective


PDE inhibition is likely to be important in
bronchodilator e ects; adenosine receptor
antagonism may be an important mechanism
in nonbronchodilator e ects (see Figure 24-2)

Muscarinic Cholinergic Ipratropium bromide Competitively inhibit acetylcholine at


Antagonists Tiotropium bromide muscarinic receptors on bronchial smooth
muscle (see Chapter 6)

Corticosteroids Beclomethasone Glucocorticoid receptor agonists that reduce


Inhaled dipropionate in ammation by altering expression o
Corticosteroids (ICSs) Triamcinolone proin ammatory genes and other genes,
Flunisolide primarily in airways cells (see Figures 24-3 and
Budesonide 24-4, and Chapter 29)
Fluticasone
Mometasone
Ciclesonide

Corticosteroids Hydrocortisone Glucocorticoid receptor agonists that reduce


Systemic Methylprednisolone in ammation by altering expression o
Prednisolone proin ammatory genes and other genes in a
Prednisone variety o cells (see Figures 24-3 and 24-4; and
Chapter 29)

Antileukotrienes5- Zileuton Inhibit the ormation o cysteinyl-leukotrienes


Lipoxygenase (5-LOX) (LTs), thus blocking the e ects o LTs on
Inhibitors airways cells (see Figure 24-5 and Chapter 22)

Antileukotrienes Montelukast Inhibit the binding o cysteinyl-leukotrienes


Leukotriene Za rlukast (LTs) to CysLT1 receptors on airways cells (see
Antagonists Figure 24-5 and Chapter 22)

Anti-IgE Monoclonal Omalizumab Humanized monoclonal antibody that blocks


Antibodies the binding o IgE to high-af nity IgE receptors
(FcR1) on mast cells and thus prevents their
activation by allergens; also blocks IgE binding
to low-af nity receptors on in ammatory cells
thus reducing chronic in ammation

Mucolytics N-acetylcysteine N-acetylcysteine reduces disul de bridges that


DNAase (dornase al a) bind glycoproteins to other proteins
DNAse reduces mucus viscosity in sputum o
patients with cystic brosis

Expectorants Guai enesin Appear to act by reducing release and/or


production o mucins, thus thinning mucus
and enhancing mucociliary clearance
(Continued)

390
Pulmonary Pharmacology CHAPTER 2 4

DRUG CLASS DRUG MECHANISM OF ACTION INDIRECT ACTIONS OF


Antitussives Codeine Codeine and other opiates are agonists at 2 AGONISTS TO CAUSE
Dextromethorphan opioid receptors in the central nervous system BRONCHODILATION
Benzonatate (CNS) medullary cough center, and may have
additional peripheral action on cough receptors Prevention o mediator release rom
in the proximal airways (see Chapter 10) human lung mast cells (via 2 receptors)
Dextromethorphan is a centrally active Prevention o microvascular leakage and
N-methyl-D-aspartate (NMDA) receptor
thus the development o bronchial mucosal
antagonist
Benzonatate acts peripherally by anesthetizing edema a ter exposure to mediators, such
the stretch receptors located in the respiratory as histamine and leukotriene D4
passages, lungs, and pleura Increase in mucus secretion romsubmuco-
sal glands and ion transport across airway
Ventilatory Stimulants Doxapram Stimulates carotid chemoreceptors, and
at higher doses it stimulates medullary epithelium; these e ects mayenhance
respiratory centers mucociliaryclearance, and therebyreverse
the de ective mucus clearance ound in
Prostacyclin (PGI2) and Epoprostenol Bind IP receptors on pulmonary vascular asthma
Its Analogs Treprostinil smooth muscle cells, causing an increase in
Iloprost cyclic adenosine monophosphate (cAMP) and Reduction in neurotransmission in human
smooth muscle relaxation (see Chapter 22) airwaycholinergicnerves byan action at
presynaptic2 receptors to inhibit acetyl-
Endothelin-1 Receptor Bosentan Bosentan is an antagonist o both ETA and choline release, which maycontribute to
Antagonists Ambrisentan ETB receptors on vascular smooth muscle
cells, inhibiting ET-1induced pulmonary
their bronchodilator e ect byreducing
vasoconstriction re excholinergicbronchoconstriction
Ambrisentan is a selective ETA receptor
antagonist

Phosphodiesterase-5 Sildena l Inhibiting PDE5 activity in vascular smooth


(PDE5) Inhibitors Tadala l muscle causes an increase in cyclic guanosine
monophosphate (cGMP) leading to vasodilation

2 a gonis t
R1 C C NH
HO OH R2
K+
Ibe riotoxin
Ce ll AC 2 AR
me mbra ne Gs
s
ATP
GTP Gs

Cyclic AMP
Ca 2 +-a ctiva te d K+ cha nne l a ctiva tion
Me thylxa nthine s
P DE P LC-IP 3 -Ca 2 + pa thway a ctivity

P KA Na +/Ca 2 + excha nge


5 AMP
Na +,Ca 2 +-ATPa s e

MLCK

FIGURE 24-1 Molecular actions o 2 agonists to induce relaxation o airway smooth muscle cells.
Activation o 2 receptors ( 2AR) results in activation o adenylyl cyclase (AC) via a stimulatory G protein
intracellular cyclic adenosine monophosphate (cAMP) and activation o protein kinase A (PKA). PKA
phosphorylates a variety o target substrates, resulting in opening o Ca2+-activated K+ channels (KCa),
thereby acilitating hyperpolarization, decreased phosphoinositide (PI) hydrolysis, increased Na+/Ca2+
exchange, increased Na+,Ca2+-ATPase activity, and decreased myosin light chain kinase (MLCK) activity.
2 Receptors may also couple to KCa via Gs. PDE, cyclic nucleotide phosphodiesterase.
391
SECTION IV Inf ammation, Immunomodulation, and Hematopoiesis

INFLAMMATORY CELLS
SIDE EFFECTS OF 2
Eo s ino phil
AGONISTS STRUCTURAL CELLS
Ce ll numbe r
Muscle tremor (direct e ect on skeletal ( a poptos is ) Airway s mo o th
mus cle
muscle 2 receptors) Bronchodila ta tion
Tachycardia (direct e ect on atrial 2 recep- T-lympho c yte
tors, re exe ect romincreased peripheral
Cytokine s ,
vasodilation via 2 receptors) tra ffic
Endo the lial c e ll

Hypokalemia (direct 2 e ect on skeletal


muscle uptake o K+) THEOPHYLLINE Le a k
Mas t c e ll
Restlessness
Hypoxemia (increased V /Q
mismatch Me dia tors
due to reversal o hypoxicpulmonary Re s pirato ry s ke le tal
mus cle s
vasoconstriction)
Mac ro phag e S tre ngth?

Cytokine s

FIGURE 24-2 Theophylline a ects multiple cells types in the airway.

Inflammato ry s timuli
e g, IL-1, TNF-

Corticos te roid

IKK Cytopla s m

GR
NF-B p65
p50

p65 CBP
B GR
p50 HDAC2
HAT Nucle us

Ace tyla tion De a ce tyla tion

Inflammato ry g e ne s
Cytokine s, che mokine s,
a dhe s ion mole cule s,
infla mma tory re ce ptors, Ge ne Ge ne
e nzyme s, prote ins tra ns cription re pre s s ion

FIGURE 24-3 Mechanism o anti-in ammatory action o corticosteroids in asthma. In ammatory genes are activated by in ammatory stimuli
(IL-1, TNF-, etc), resulting in activation o IKK (inhibitor o I-B kinase-), which activates the transcription actor nuclear actor B (NF-B).
A dimer o p50 and p65 NF-B proteins translocates to the nucleus and binds to speci c B recognition sites and also to coactivators, such as
CREB-binding protein (CBP), which have intrinsic histone acetyltrans erase (HAT) activity. This results in acetylation o core histones and conse-
quent increased expression o genes encoding multiple in ammatory proteins. Cytosolic glucocorticoid receptors (GR) bind corticosteroids; the
receptor-ligand complexes translocate to the nucleus and bind to coactivators to inhibit HAT activity in 2 ways: directly and, more importantly, by
recruiting histone deacetylase-2 (HDAC2), which reverses histone acetylation, leading to the suppression o activated in ammatory genes.
392
Pulmonary Pharmacology CHAPTER 2 4

INFLAMMATORY CELLS STRUCTURAL CELLS


SIDE EFFECTS OF INHALED
Eo s ino phil Epithe lial c e lls
CORTICOSTEROIDS
Numbe rs Cytokine s
( a poptos is )
Me dia tors Local side e ects
Dysphonia
T-lympho c yte Endo the lial c e lls Oropharyngeal candidiasis
Cytokine s Cough
Le a k
Systemicside e ects
CORTICOSTEROIDS Adrenal suppression and insuf ciency
Mas t c e ll
Airway s mo o th Growth suppression
Numbe rs mus cle
2 Re ce ptors Bruising
Cytokine s Osteoporosis
Mac ro phag e
Muc us g land Cataracts
Cytokine s Glaucoma
Mucus s e cre tion Metabolicabnormalities (glucose,
insulin, triglycerides)
De ndritic c e ll
Psychiatricdisturbances (euphoria,
Numbe rs depression)
Pneumonia

FIGURE 24-4 E ect o corticosteroids on in ammatory and structural cells in the airways.

CASE 24-1
An 8-year-old boy has occasional episodes o mild asthma while playing basketball
with his riends.
a. What are the symptoms o asthma and some o the underlying pathogenic events
that cause these symptoms?
Asthma is characterized by variable air ow obstruction caused by airway smooth
muscle contraction. It is a chronic in ammatory disease o the airways that is char-
acterized by activation o mast cells, in ltration o eosinophils and helper 2 ( H2)
lymphocytes. Mast cell activation by allergens and physical stimuli releases broncho-
constrictor mediators, such as histamine, leukotriene D4, and prostaglandin D2, which
cause bronchoconstriction, microvascular leakage, and plasma exudation. Increased
numbers o mast cells in airway smooth muscle are a characteristic o asthma.
T e mechanism o chronic in ammation in asthma is still not well understood. It
may initially be driven by allergen exposure, but it appears to become autonomous
so that asthma is essentially incurable. Asthma usually starts in early childhood,
and then may disappear during adolescence and reappear in adulthood. Asthma
severity usually does not change so that patients with mild asthma rarely progress
to severe asthma, and patients with severe asthma usually have this rom the onset,
although some patients, particularly with late-onset asthma, show a progressive loss
o lung unction like patients with COPD.
b. What treatment, i any, should this patient receive to relieve symptoms during an
asthma attack?
Inhaled short-acting 2 agonists are the most widely used and e ective bronchodi-
lators in the treatment o asthma due to their unctional antagonism o broncho-
constriction. When inhaled rom pressurized metered-dose inhalers (pMDIs) or
dry powder inhalers (DPI), they are convenient, easy to use, rapid in onset, and
without signi cant systemic side e ects. In addition to their acute bronchodilator
e ect, these agents are e ective in protecting against various challenges, such as
exercise, cold air, and allergens.
(Continued)
393
SECTION IV Inf ammation, Immunomodulation, and Hematopoiesis

c. What are the mechanisms o action o 2 agonists in treating asthma?


2 Agonists produce bronchodilation by directly stimulating 2 receptors in airway
smooth muscle (see Figure 24-1). 2 Agonists act as unctional antagonists and reverse
bronchoconstriction irrespective o the contractile agent. T is is an important prop-
erty or the treatment o asthma because many bronchoconstrictor mechanisms
(neurotransmitters and mediators) are likely to be contributory. 2 Agonists may
also indirectly cause bronchodilation by inhibiting the release o bronchoconstrictor
mediators rom in ammatory cells and o bronchoconstrictor neurotransmitters rom
airway nerves (see Side Bar INDIREC AC IONS OF 2 AGONIS S O CAUSE
BRONCHODILA ION). Although these additional e ects o 2 agonists may be rele-
vant to the prophylactic use o these drugs against various challenges, their rapid bron-
chodilator action is probably attributable to a direct e ect on airway smooth muscle.
d. I the asthma attacks occur more requently, what changes in therapy might be
appropriate or this patient?
Short-acting inhaled 2 agonists, such as albuterol, should be used as required by
symptoms and not on a regular basis in the treatment o mild asthma; increased use
indicates the need or more e ective anti-in ammatory therapy, that is, corticoste-
roids. Inhaled corticosteroids (ICSs) are recommended as rst-line therapy or all
patients with persistent asthma. T ey should be started in any patient who needs to
use a 2 agonist inhaler or symptom control more than twice weekly. T ey are e ec-
tive in mild, moderate, and severe asthma and in children as well as adults. ICS may be
used in children in the same way as in adults; at doses o 400 g/d or less, there is no
evidence o signi cant growth suppression. T e dose o ICSs should be the minimal
dose that controls asthma; once control is achieved, the dose should be slowly reduced.

CASE 24-2
A 72-year-old woman has chronic di culty in breathing. She has smoked a pack o cig-
arettes every day or the last 55 years and was diagnosed as having COPD 15 years ago.
a. What is COPD and how does it dif er rom asthma?
Chronic obstructive pulmonary disease (COPD) involves in ammation o the
respiratory tract with a pattern that di ers rom that o asthma. In COPD, there
is a predominance o neutrophils, macrophages, and cytotoxic lymphocytes
( c1 cells). T e in ammation predominantly a ects small airways, resulting in
progressive small airway narrowing and brosis (chronic obstructive bronchiolitis)
and destruction o the lung parenchyma with destruction o the alveolar walls
(emphysema). T ese pathological changes result in airway closure on expiration,
leading to air trapping and hyperin ation, particularly on exercise (dynamic
hyperin ation). T is accounts or shortness o breath on exertion and exercise
limitation that are characteristic symptoms o COPD.
In contrast to asthma, the air ow obstruction o COPD tends to be progressive.
T e in ammation in the peripheral lung o COPD patients is mediated by multiple
in ammatory mediators and cytokines, although the pattern o mediators di ers
rom that o asthma. In marked contrast to asthma, the in ammation in patients
with COPD is largely corticosteroid-resistant, and there are currently no e ective
anti-in ammatory treatments or this disease. In addition to pulmonary disease,
many patients with COPD have systemic mani estations (skeletal muscle wasting,
weight loss, depression, osteoporosis, anemia) and comorbid diseases (ischemic
heart disease, hypertension, congestive heart ailure, diabetes). Whether these are
due to spillover o in ammatory mediators rom the lung or due to common causal
mechanisms (such as smoking) is not yet clear, but it may be important to treat the
systemic components in the overall management o COPD.
b. What treatments are available or this patient?
Bronchodilators are the mainstay o treatment in COPD and act to reduce air trap-
ping by dilating peripheral airways. Anticholinergic drugs (ipratropium bromide and
(Continued)
394
Pulmonary Pharmacology CHAPTER 2 4

tiotropium bromide) are the bronchodilators o choice in COPD, and are as e ective as
or even superior to 2 agonists. T eir relatively greater e ect in COPD than in asthma
may be explained by an inhibitory e ect on vagal tone, which, although not necessarily
increased in COPD, may be the only reversible element o airway obstruction and one
that is exaggerated by geometric actors in the narrowed airways o COPD. Anticho-
linergic drugs reduce air trapping and improve exercise tolerance in COPD patients.
Once-daily tiotropium is more e ective than ipratropium 4 times daily. When given
long term, tiotropium improves lung unction and health status and reduces exacerba-
tions and all-cause mortality, although there is no e ect on disease progression. As a
result, tiotropium is becoming the bronchodilator o choice or patients with COPD.
T e long-acting inhaled 2 agonists (LABAs) salmeterol, ormoterol, and ar or-
moterol are also used in COPD therapy. T ese drugs have a bronchodilator action
o more than 12 hours and also protect against bronchoconstriction or a similar
period. LABAs are e ective bronchodilators that may be used alone or in combina-
tion with anticholinergics or ICSs. LABAs improve symptoms and exercise toler-
ance by reducing both air trapping and exacerbations.
Combination inhalers o an anticholinergic and 2 agonist, such as ipratropium/
albuterol, are widely used in patients with COPD. Several studies have demon-
strated additive e ects o these 2 drugs, thus providing an advantage over increas-
ing the dose o 2 agonist in patients who have side e ects.
Combination inhalers that contain a LABA and a corticosteroid (eg, uticasone/salme-
terol, budesonide/ ormoterol) are also used in the treatment o COPD. T ese combina-
tion inhalers are more e ective in patients with COPD than monotherapy with either a
LABA or an ICS. Although patients with COPD occasionally respond to steroids alone,
these patients are likely to have concomitant asthma. Corticosteroids have no objec-
tive short-term bene t on airway unction in patients with true COPD, although these
agents o en produce subjective bene t because o their euphoric e ect. Corticosteroids
do not appear to have any signi cant anti-in ammatory e ect in COPD.
T eophylline, an agent with multiple sites o action (see Figure 24-2), is sometimes
used as a bronchodilator in COPD, but inhaled anticholinergics and 2 agonists are
pre erred. T eophylline tends to be added to these inhaled bronchodilators in more
severe patients and has been shown to give additional clinical improvement when
added to a long-acting 2 agonist.
c. What side ef ects are to be expected with inhaled anticholinergic drugs?
Inhaled anticholinergic drugs are generally well tolerated. On stopping inhaled
anticholinergics, a small rebound increase in airway responsiveness has been
described, but the clinical relevance o this is uncertain. Systemic side e ects a er
ipratropium bromide and tiotropium bromide are uncommon during normal clini-
cal use because there is little systemic absorption. A signi cant unwanted e ect is
the unpleasant bitter taste o inhaled ipratropium, which may contribute to poor
compliance. Nebulized ipratropium bromide may precipitate glaucoma in elderly
patients due to a direct e ect o the nebulized drug on the eye. T is may be pre-
vented by nebulization with a mouthpiece rather than a ace mask. iotropium
causes dryness o the mouth in 10 to 15% o patients, but this usually disappears
during continued therapy. Urinary retention is occasionally seen in elderly patients.

CASE 24-3
A 46-year-old woman has su ered rom severe asthma since she was a girl. She com-
plains o di culty breathing nearly all o the time and occasionally has asthma attacks
that are very severe.
a. What pharmacotherapeutic options are appropriate in this patient to improve
her symptoms?
Because asthma is a chronic in ammatory disease, ICSs are considered as rst-line
therapy in all but the mildest o cases. Corticosteroids inhibit the expression o
(Continued)
395
SECTION IV Inf ammation, Immunomodulation, and Hematopoiesis

Alle rge n Cold a ir


Exe rcis e PAF

As pirin Ma s t ce ll Eos inophil


(in AS a s thma tics ) S O2

Arac hido nic ac id


5-LO INHIBITORS
5-lip oxyg e na s e Zile uton

Cys te inyl-le uko trie ne s


(LTC4 , LTD4 , LTE4 ) LT ANTAGONISTS
Monte luka s t
Pra nluka s t
Za firluka s t
Cys LT1
re c e pto rs

P la s ma
exuda tion

Mucus Bronchocons triction Eos inophil


s e cre tion re cruitme nt

FIGURE 24-5 E ects o cysteinyl-leukotrienes on the airways and their inhibition by antileukot-
rienes. AS, aspirin-sensitive; 5-LO, 5-lipoxygenase; LT, leukotriene; PAF, platelet-activating actor.

multiple in ammatory genes in airway epithelial cells, probably the most important
action o ICSs in suppressing asthmatic in ammation (see Figure 24-3). Cortico-
steroids also increase the transcription o several anti-in ammatory genes. Steroids
have inhibitory e ects on many in ammatory and structural cells that are activated
in asthma and prevent the recruitment o in ammatory cells into the airways (see
Figure 24-4).
For most patients, ICSs should be used twice daily, a regimen that improves com-
pliance once control o asthma has been achieved (which may require 4-times daily
dosing initially or a course o oral steroids i symptoms are severe). Administration
once daily o some steroids (eg, budesonide, mometasone, and ciclesonide) is e ec-
tive when doses o 400 g or less are needed. T e dose o ICS should be the mini-
mal dose that controls asthma; once control is achieved, the dose should be slowly
reduced. ICSs have rapid anti-in ammatory e ects, reducing airway hyperrespon-
siveness and in ammatory mediator concentrations within a ew hours. However,
it may take several weeks or months to achieve maximal e ects on airway hyperre-
sponsiveness, presumably re ecting the slow healing o the damaged in amed air-
way. It is important to recognize that corticosteroids suppress in ammation in the
airways but do not cure the underlying disease; when steroids are withdrawn, there
is a recurrence o the same degree o airway hyperresponsiveness.
For this patients acute severe asthma attacks, inhaled short-acting 2 agonists
are the bronchodilators o choice. T e LABAs, salmeterol, ormoterol, and ar or-
moterol, are important in controlling asthma in patients with chronic symptoms.
T ese drugs have a bronchodilator action o more than 12 hours and also protect
against bronchoconstriction or a similar period. T ey improve asthma control
(when given twice daily) compared with regular treatment with short-acting 2 ago-
nists (4-6 times daily).
(Continued)
396
Pulmonary Pharmacology CHAPTER 2 4

Combination inhalers that contain a LABA and a corticosteroid (eg, uticasone/


salmeterol, budesonide/ ormoterol) are widely used in the treatment o persistent
asthma. In asthma, there is a strong scienti c rationale or combining a LABA with a
corticosteroid because these treatments have complementary synergistic actions. T e
combination inhaler is more convenient or patients, simpli es therapy, and improves
compliance with ICSs because the patients perceive clinical bene t, but there may be
an additional advantage because delivering the 2 drugs in the same inhaler ensures
they are delivered simultaneously to the same cells in the airways, allowing the bene -
cial molecular interactions between LABA and corticosteroids to occur. It is likely that
these inhalers will become the pre erred therapy or all patients with persistent asthma.
In asthmatic patients, anticholinergic drugs are less e ective as bronchodilators
than 2 agonists and o er less ef cient protection against bronchial challenges.
T ese drugs may be more e ective in older patients with asthma in whom there is
an element o xed airway obstruction. Anticholinergics are currently used as an
additional bronchodilator in asthmatic patients not controlled on a LABA. Nebu-
lized anticholinergic drugs are e ective in acute severe asthma, but less e ective
than 2 agonists. Nevertheless, in the acute and chronic treatment o asthma, anti-
cholinergic drugs may have an additive e ect with 2 agonists and should there ore
be considered when control o asthma is not adequate with nebulized 2 agonists.
T eophylline can play a role as add-on therapy to ICS, although it is less e ective as
an add-on than a LABA. Although LABAs are more e ective as an add-on therapy,
theophylline is considerably less expensive and may be the only a ordable add-on
treatment when the costs o medication are limiting.
In patients with mild to moderate asthma, antileukotrienes (see Figure 24-5) cause
a signi cant improvement in lung unction and asthma symptoms, with a reduc-
tion in the use o rescue-inhaled 2 agonists. Antileukotrienes are indicated as an
add-on therapy in patients who are not well controlled on ICS therapy. T e added
bene t is small, equivalent to doubling the dose o ICS, and less e ective than add-
ing a LABA. In patients with severe asthma who are not controlled on high doses o
ICS and LABA, antileukotrienes do not appear to provide any additional bene t.
Omalizumab, a humanized monoclonal antibody that blocks the binding o IgE to
its receptors, is used or the treatment o patients with severe asthma. T e antibody
is administered by subcutaneous injection every 2 to 4 weeks, and the dose is deter-
mined by the titer o circulating IgE. Omalizumab reduces the requirement or oral
and ICSs and markedly reduces asthma exacerbations. Because o its very high cost,
this treatment is generally used only in patients with very severe asthma who are
poorly controlled even on oral corticosteroids and in those with very severe con-
comitant allergic rhinitis.
b. Are there side ef ects or concerns with long-term use o ICSs and LABAs?
Suppression o the hypothalamic-pituitary-adrenal (HPA) axis by a negative eed-
back e ect o corticosteroids on the pituitary gland (see Chapter 29) can occur
with long-term use o oral corticosteroids and high daily doses o ICSs (ie, >2000
g beclomethasone dipropionate or its equivalent daily). Systemic side e ects o
ICSs (see Side Bar SIDE EFFEC S OF INHALED COR ICOS EROIDS) include
dermal thinning and skin capillary ragility (relatively common in elderly patients
a er high-dose inhaled steroids). Other side e ects, such as cataract ormation,
glaucoma, and osteoporosis, are reported but o en in patients who are also receiv-
ing courses o oral steroids. Budesonide, uticasone, mometasone, and ciclesonide
have a lower oral bioavailability than beclomethasone dipropionate because they
are subject to greater rst-pass hepatic metabolism; this results in reduced systemic
absorption rom the raction o the inhaled drug that is swallowed. Ciclesonide is a
prodrug that is converted to the active metabolite by esterases in the lung, giving it
a low oral bioavailability and a high therapeutic index.
ICSs may have local side e ects due to the deposition o inhaled steroid in the orophar-
ynx (see Side Bar SIDE EFFEC S OF INHALED COR ICOS EROIDS). T e most
(Continued)
397
SECTION IV Inf ammation, Immunomodulation, and Hematopoiesis

common problem is hoarseness and weakness o the voice (dysphonia) due to atrophy
o the vocal cords ollowing laryngeal deposition o steroid; it may occur in up to 40%
o patients and is noticed particularly by patients who need to use their voices during
their work (lecturers, teachers, and singers). Oropharyngeal candidiasis occurs in ~5%
o patients. T ere is no evidence or increased lung in ections, including tuberculo-
sis, in patients with asthma. When high doses o an ICS are required, a large volume
spacer is recommended to reduce oropharyngeal deposition and systemic absorption.
Side e ects are not common with inhaled LABA therapy. Muscle tremor is the most
common side e ect, and there are several other side e ects related to stimulation
o extrapulmonary 2 receptors (see Side Bar SIDE EFFEC S OF 2 AGONIS S).
olerance to nonairway 2 receptormediated responses, such as tremor and car-
diovascular and metabolic responses, is readily induced in asthmatic patients, but
tolerance to the bronchodilator e ects o 2 agonists has not usually been ound. T e
reason or the relative resistance o airway smooth muscle 2 responses to desensiti-
zation may re ect the large receptor reserve in airway smooth muscle compared to
other tissues. T e sa ety o LABAs in asthma remains controversial based on large-
scale studies o asthma patients prescribed salmeterol and ormoterol that showed
an excess o respiratory deaths and near deaths. However, concomitant treatment
with an ICS appears to obviate such risk, so it is recommended that LABAs should
only be used when ICSs are also prescribed (pre erably in the orm o a combination
inhaler so that the LABA can never be taken without the ICSs). All LABAs approved
in the United States carry a black box warning cautioning against overuse.

CASE 24-4
A 76-year-old woman who complains o di culty breathing due to pulmonary conges-
tion is diagnosed with pulmonary artery hypertension (PAH).
a. What drugs should be considered in treating this patients PAH?
PAH involves dys unction o pulmonary vascular endothelial and smooth muscle
cells, and results rom an imbalance in vasoconstrictor and vasodilator mediators
(see Figure 24-6). In PAH, there is an increase in the vasoconstrictor mediators
endothelin-1 (E -1), thromboxane A2, and serotonin, and a decrease in the vaso-
dilating mediators prostacyclin (PGI2), NO, and VIP. Vasodilators are the mainstay
o drug therapy or PAH (see Figure 24-6). However, the vasodilators used to treat
systemic hypertension are problematic; they lower systemic blood pressure, which
may result in reduced pulmonary per usion.
Intravenous epoprostenol (prostacyclin, PGI2) is e ective in lowering pulmonary
arterial pressures, improving exercise per ormance, and prolonging survival in pri-
mary PAH (PPAH), but must be administered by continuous IV in usion because o
its short plasma t1/2; this is inconvenient and therapy is very expensive. More stable
prostacyclin analogs (treprostinil and iloprost) can be administered by other routes,
including inhalation.
Bosentan and ambrisentan are oral endothelin receptor antagonists used or the
treatment o PPAH. T ese agents di er in their selectivity or E receptors. Stimula-
tion o E A receptors by E -1 contracts vascular smooth muscle cells and causes pro-
li eration mainly, whereas E B receptors mediate the release o prostacyclin and NO
rom endothelial cells (see Figure 24-6). Bosentan is an antagonist o both E A and
E B receptors, and is ef cacious orally in reducing symptoms and improving mor-
tality in PPAH. Ambrisentan is a selective E A receptor antagonist. T e theoretical
advantage o blocking only E A receptors is that E B receptors may continue to stim-
ulate release o PGI2 and NO, giving a greater therapeutic e ect. However, the clini-
cal ef cacy o ambrisentan is similar to that o bosentan, as are its adverse e ects.
T e oral phosphodiesterase-5 (PDE5) inhibitors, sildena l and tadala l, are e ec-
tive in improving symptoms o PAH because o their ability to induce vasodilation
by elevating intracellular cyclic guanosine monophosphate (cGMP) concentrations
in pulmonary artery smooth muscle (see Figure 24-6).
(Continued)
398
Pulmonary Pharmacology CHAPTER 2 4

A Endo the lial fac to rs influe nc ing B Alte ratio ns in PAH


s mo o th mus cle to ne

Endothe lium

ET-1 P GI2 NO ET-1 P GI2 NO


2+ 2+
Ca Ca

Ca v1.2 ETA IP

VS M
me mbra ne cAMP s GC

Enha nce d Re duce d cyclic


2+
[Ca ]i cAMP cGMP Ca 2 + s igna ling nucle otide s igna ling

Contra ction Re la xa tion Contra ction Re la xa tion


Prolife ra tion Anti-prolife ra tion Prolife ra tion Anti-prolife ra tive
s igna ling

BALANCE IMBALANCE
(favoring contra ction a nd prolife ra tion)
C Drug e ffe c ts in PAH

ETA Exoge nous


a nta gonis ts NO

+
Ca 2
Cha nne l ET-1 P GI2
2+ P DE5
a nta gonis ts Ca a na logs inhibitors

Ca v1.2 ETA IP

s GC

[Ca 2 +]i cAMP cGMP 5'GMP


P DE5
More norma l More norma l cyclic
Ca 2 + s igna ling nucle otide s igna ling

Contra ction Re la xa tion


Prolife ra tion Anti-prolife ra tion

RESTORED BALANCE

FIGURE 24-6 Interactions o endothelium and vascular smooth muscle in pulmonary artery hypertension (PAH).
A. In normal pulmonary artery, there is a balance between constrictor and relaxant in uences that may be viewed as
competition between Ca2+ signaling pathways and cyclic nucleotide signaling pathways in vascular smooth muscle
(VSM). Endothelin (ET-1) binds to the ETA receptor on VSM cells and activates the Gq-PLC-IP3 pathway to increase
cytosolic Ca2+; ET-1 may also couple to Gi to inhibit cyclic adenosine monophosphate (cAMP) production. In depolar-
izing VSM cells, Ca2+ may enter via the L-type Ca2+ channel (Cav1.2). Endothelial cells also produce relaxant actors,
prostacyclin (PGI2) and NO. NO stimulates the soluble guanylyl cyclase (cGC), causing accumulation o cyclic guano-
sine monophosphate (cGMP) in VSM cells; PGI2 binds to the IP prostanoid receptor and stimulates the Gs-adenylyl
cyclase pathway to enhance cAMP accumulation; elevation o these cyclic nucleotides promotes VSM relaxation (see
Chapter 1). B. In PAH, ET-1 production is enhanced, production o PGI2 and NO is reduced, and the balance is shi ted
toward constriction and proli eration o vascular smooth muscle. C. In treating PAH, ETA receptor antagonists can
reduce the constrictor e ects o ET-1, and Ca2+ channel antagonists can urther reduce Ca2+-dependent contraction.
Exogenous PGI2 and NO can be supplied to promote vasodilation (relaxation o VSM); inhibition o PDE5 can enhance
the relaxant e ect o NO by inhibiting the degradation o cGMP, thereby promoting intracellular accumulation o
cGMP and relaxation o VSM. Thus, these drugs can reduce Ca2+ signaling and enhance cyclic nucleotide signaling,
restoring the balance between the orces o contraction/proli eration and relaxation/antiproli eration. Remodeling
and deposition o extracellular matrix by adjacent broblasts is in uenced positively and negatively by the same con-
tractile and relaxant signaling pathways, respectively.

399
SECTION IV Inf ammation, Immunomodulation, and Hematopoiesis

b. What are the side ef ects associated with each o these agents?
Common side e ects o IV epoprostenol are headache, ushing, diarrhea, nausea,
and jaw pain. Inhaled iloprost is associated with the vasodilator e ects o prostacy-
clin, including syncope. It may also cause cough and bronchoconstriction because
it sensitizes airway sensory nerves.
T e endothelin receptor antagonists are generally well tolerated and have similar
side e ect pro les. Adverse e ects include abnormal liver unction tests, anemia,
headaches, peripheral edema, and nasal congestion. Given the risk o serious liver
injury, liver aminotrans erases should be monitored monthly. All endothelin recep-
tor antagonists carry a risk o testicular atrophy and in ertility. Bosentan is poten-
tially teratogenic.
T e PDE5 inhibitors, side e ects are due in part to systemic hypotension (headache
and ushing). Other side e ects include dyspepsia and visual disturbances. Because
o the risk o severe hypotension, concurrent administration with nitrovasodilators
should be avoided (see Chapter 16).

CASE 24-5
A 54-year-old male patient complains o chronic cough. He is not a smoker, does not
have asthma, but has a history o gastroesophageal re ux disease (GERD).
a. What treatments are most appropriate to reduce this patients chronic cough?
Cough is a de ensive re ex and its suppression may be inappropriate because it
could mask the underlying cause. Whenever possible, treat the underlying cause,
not the cough. A likely cause in this patient is GERD. Gastroesophageal re ux is a
common cause o cough through a re ex mechanism and occasionally as a result
o acid aspiration into the lungs. T is cough may respond to suppression o gastric
acid with an H 2 receptor antagonist or a proton pump inhibitor (see Chapter 32),
although even large doses may not always be e ective. Some patients have a chronic
cough with no obvious cause, and this chronic idiopathic cough may be due to air-
way sensory neural hyperesthesia.
Over-the-counter cough medications containing dextromethorphan are largely
ine ective, although they are widely used. Prescription cough suppressants include
opiates such as codeine, and benzonatate, a local anesthetic.

KEY CONCEPTS
Bronchodilators are key drugs in the treatment o asthma and COPD.
When there is a choice o inhaled or oral route or a pulmonary drug (eg, 2
agonist or corticosteroid), the inhaled route is always pre erable, and the oral
route should be reserved or the ew patients unable to use inhalers (eg, small
children, patients with physical problems such as severe arthritis o the hands).
Inhaled 2 agonists are unctional antagonists o bronchoconstriction and
reverse bronchoconstriction irrespective o the contractile agent.
An inhaled 2 agonist is the bronchodilator treatment o choice in asthma
because 2 agonists are the most e ective bronchodilators and have minimal
side e ects when used correctly.
Inhaled anticholinergic drugs are less e ective as bronchodilators than 2 ago-
nists in patients with asthma, but may be as e ective or superior to 2 agonists
in patients with COPD.
ICSs are recommended as f rst-line therapy or all patients with persistent
asthma (ie, patients who require inhaled 2 agonists more than twice a week)
because asthma is a chronic in ammatory disease.
ICSs have limited therapeutic value in patients with COPD except in patients
with concomitant asthma or severe COPD.
400
Pulmonary Pharmacology CHAPTER 2 4

SUMMARY QUIZ

QUESTION 24-1 A 20-year-old woman has occasional asthma symptoms during the
winter when she goes snowshoeing. T e medication o choice to relieve her asthma
symptoms is inhaled
a. albuterol.
b. salmeterol.
c. tiotropium bromide.
d. iloprost.
e. budesonide.

QUESTION 24-2 A 54-year-old man who had occupational exposure to asbestos has devel-
oped COPD and emphysema. T e medication o choice to improve his breathing is inhaled
a. albuterol.
b. salmeterol.
c. tiotropium bromide.
d. iloprost.
e. budesonide.

QUESTION 24-3 An asthma patient with persistent asthma is prescribed a combina-


tion inhaler, budesonide/ ormoterol (SYMBICOR ). T e mechanisms o action o the
2 drugs in this ormulation are
a. a long-acting 2 adrenergic agonist and an anticholinergic.
b. a corticosteroid and a phosphodiesterase inhibitor.
c. an endothelin receptor antagonist and a prostacyclin receptor agonist.
d. a long-acting 2 adrenergic agonist and a corticosteroid.
e. a corticosteroid and an anticholinergic.

QUESTION 24-4 T e mechanistic rationale or using sildenaf l to treat PAH is that it


a. increases cGMP synthesis in pulmonary artery smooth muscle.
b. enhances the e ects o endothelin-1 in pulmonary artery smooth muscle.
c. enhances the e ects o NO in pulmonary artery smooth muscle.
d. enhances the e ects o prostacyclin in pulmonary artery smooth muscle.
e. inhibits the e ects o endothelin-1 in pulmonary artery smooth muscle.

QUESTION 24-5 T e mechanistic rationale or using inhaled ipratropium to treat


COPD is that it
a. activates 2 adrenergic receptor bronchial smooth muscle.
b. binds glucocorticoid receptors in in ammatory cells o the airways.
c. blocks E -1 binding to E A and E B receptors in bronchial smooth muscle.
d. blocks binding o acetylcholine to muscarinic cholinergic receptors in bronchial
smooth muscle.
e. inhibits the hydrolysis o cyclic nucleotides in bronchial smooth muscle.

SUMMARY QUIZ ANSWER KEY


QUESTION 24-1 Answer is a. o relieve occasional asthma symptoms such as those
induced by cold air and exercise, the medication o choice is a short-acting 2 agonist
(eg, albuterol) delivered by inhalation.
QUESTION 24-2 Answer is c. In COPD, inhaled anticholinergic drugs such as tiotro-
pium are the bronchodilators o choice in most patients. Clinical studies have shown
(Continued)
401
SECTION IV Inf ammation, Immunomodulation, and Hematopoiesis

that long-term use o tiotropium improves lung unction and health status, and reduces
exacerbations and all-cause mortality, although there is no e ect on disease progression.
QUESTION 24-3 Answer is d. Budesonide is an ICS that has benef cial anti-in am-
matory e ects in asthma, and ormoterol is a LABA that has bronchodilator and bron-
choprotective e ects. T ese agents have both complementary and synergistic e ects
in patients with persistent asthma. T e combination inhaler is more convenient or
patients, simplif es therapy, and improves compliance with ICS because the patients
perceive clinical benef t. In addition, delivering the 2 drugs in the same inhaler ensures
they are delivered simultaneously to the same cells in the airways, allowing the benef -
cial molecular interactions between LABA and corticosteroids to occur.
QUESTION 24-4 Answer is c. Sildenaf l relaxes pulmonary artery smooth muscle by
blocking the hydrolysis o cGMP by PDE5 (see Figure 24-6). Much o the cGMP in
vascular smooth muscle is synthesized by soluble guanylate cyclase which is activated
by nitric oxide (NO). T us, sildenaf l enhances the vasodilating e ects o NO produced
endogenously by vascular endothelium.
QUESTION 24-5 Answer is d. Ipratropium is a competitive antagonist o endogenous
acetylcholine (ACh) binding to muscarinic cholinergic receptors in bronchial smooth
muscle (see Chapter 6). Activation o muscarinic receptors on bronchial smooth
muscle causes contraction and bronchoconstriction. Multiple and diverse stimuli cause
re ex increases in parasympathetic activity that contribute to bronchoconstriction.
T e e ects o ACh on the respiratory system include not only bronchoconstriction but
also increased tracheobronchial secretion and stimulation o the chemoreceptors o the
carotid and aortic bodies. T us antimuscarinic drugs antagonizes these various pulmo-
nary e ects o acetylcholine released by parasympathetic neurons.
Acetylcholine may also be released rom other airway cells, including epithelial cells.
T e synthesis o acetylcholine in epithelial cells is increased by in ammatory stimuli (such
as NF-), which increase the expression o choline acetyltrans erase, which could contrib-
ute to cholinergic e ects in airway diseases. Muscarinic receptors are expressed in airway
smooth muscle o small airways that do not appear to be innervated by cholinergic nerves;
these receptors may be a mechanism o cholinergic narrowing in peripheral airways that
could be relevant in COPD, responding to locally synthesized, non-neuronal ACh.

SUMMARYTABLE: DRUGS USED IN THE TREATMENT OF ASTHMA, COPD, AND OTHER PULMONARY DISORDERS
TOXICITIES
CLASS AND
SUBCLASSES NAMES CLINICAL USES COMMON UNIQUE; CLINICALLY IMPORTANT
2 Adrenergic Albuterol The most widely used and Side e ects are not Tolerance develops to the
AgonistsShort- (salbutamol) e ective bronchodilators in the common with inhaled bronchoprotective e ects o high-
Acting (3-6 h) Levalbuterol treatment o asthma due to therapy, but quite dose inhaled 2 agonists, but not
Metaproterenol their unctional antagonism o common with oral or IV the bronchodilating e ects
Pirbuterol bronchoconstriction administration (see Side Short-acting inhaled 2 agonists
Terbutaline E ective in protecting against Bar SIDE EFFECTS OF 2 should only be used on demand
various challenges, such as AGONISTS) or symptom control, and i they
exercise, cold air, and allergens are required requently (more than
Bronchodilators o choice in twice weekly), an ICS is needed
treating acute severe asthma

2 Adrenergic Salmeterol Improve asthma control (when Side e ects are not In asthma patients, LABAs should
AgonistsLong- Formoterol given twice daily) compared common with inhaled never be used alone because they
Acting (Long-Acting Ar ormoterol with regular treatment with therapy, but quite do not treat the underlying chronic
Agonists [LABAs] Indacaterol short-acting 2 agonists (4-6 common with oral or IV in ammation; rather, LABAs should
duration >12 h) times daily) administration (see Side always be used in combination
In COPD, LABAs are e ective Bar SIDE EFFECTS OF 2 with ICS (pre erably in a xed-dose
bronchodilators that may be AGONISTS) combination inhaler)
used alone or in combination All LABAs approved in the United
with an anticholinergic or ICS States carry a black box warning
cautioning against overuse in asthma
(Continued)
402
Pulmonary Pharmacology CHAPTER 2 4

TOXICITIES
CLASS AND
SUBCLASSES NAMES CLINICAL USES COMMON UNIQUE; CLINICALLY IMPORTANT
Methylxanthines Theophylline Should be reserved or use as Headache, nausea, At high concentrations, cardiac
add-on maintenance therapy and vomiting (due to arrhythmias
in asthma patients receiving inhibition o PDE4), At very high concentrations, seizures
high- or low-dose ICS or maximal abdominal discom ort, Low doses (5-10 mg/L) largely avoid
2 agonist alone who require and restlessness side e ects
additional bronchodilator e ect Increased acid secretion
Added to inhaled (due to PDE inhibition)
anticholinergics and 2 agonists and diuresis (due to
(LABAs) in patients with severe inhibition o adenosine A1
COPD receptors)

Muscarinic Ipratropium Used as an additional Inhaled anticholinergic Unpleasant bitter taste o inhaled
Cholinergic bromide bronchodilator in asthmatic drugs are generally ipratropium which may contribute
Antagonists Tiotropium bromide patients not controlled on a well tolerated with ew to poor compliance
LABA systemic e ects Bronchoconstriction may occur with
In acute and chronic asthma Tiotropium causes ipratropium
therapy, may have an additive dryness o the mouth in
e ect with 2 agonists 10-15% o patients, but
In COPD, may be as e ective as this usually disappears
or even superior to 2 agonists during continued therapy

Corticosteroids Beclomethasone ICSs are considered as rst-line See Chapter 29 See Chapter 29 and Side Bar
Inhaled dipropionate therapy in all but the mildest and Side Bar SIDE SIDE EFFECTS OF INHALED
Corticosteroids Triamcinolone o asthma patients and should EFFECTS OF INHALED CORTICOSTEROIDS
(ICSs) Flunisolide be started in any patient who CORTICOSTEROIDS All currently available ICSs are
Budesonide needs to use a 2 agonist inhaler Budesonide, uticasone, absorbed rom the lung into the
Fluticasone or symptom control more than mometasone, and systemic circulation, so that some
Mometasone twice weekly ciclesonide are subject systemic absorption is inevitable
Ciclesonide ICSs are much less e ective to greater rst-pass To reduce likelihood o systemic
in COPD and should only be hepatic metabolism e ects, use the lowest dose o
used in patients with severe than beclomethasone inhaled steroid needed to control
disease who have requent dipropionate, thus have the asthma, and use a large-volume
exacerbations reduced adverse systemic spacer to reduce oropharyngeal
e ects deposition

Corticosteroids Hydrocortisone IVsteroids are indicated in acute Adrenal suppression by a Steroid doses a ter prolonged oral
Systemic Methylprednisolone asthma i lung unction is <30% negative eedback e ect therapy must be reduced slowly;
Prednisolone o predicted and there is no on the pituitary gland symptoms o steroid withdrawal
Prednisone improvement with 2 agonist Signi cant adrenal syndromeinclude lassitude,
Short courses o oral steroids suppression a ter short musculoskeletal pains, and,
(30-40 mg prednisolone daily courses o corticosteroid occasionally, ever (see Chapter 29
or 1-2 weeks) are indicated or therapy is not usually a or other toxicities and precautions)
exacerbations o asthma problem, but prolonged
Oral corticosteroids remain suppression may occur
the mainstay o treatment o a ter several months or
several pulmonary diseases, years
such as sarcoidosis, interstitial Long-term oral
lung diseases, and pulmonary corticosteroid side e ects
eosinophilic syndromes include uid retention,
increased appetite,
weight gain, osteoporosis,
capillary ragility,
hypertension, peptic
ulceration, diabetes,
cataracts, and psychosis
(see Chapter 29 or other
toxicities and precautions)

Antileukotrienes Zileuton Antileukotrienes are indicated Associated with rare cases o hepatic
5-Lipoxygenase as an add-on therapy in asthma dys unction; thus liver-associated
(5-LOX) Inhibitors patients who are not well enzymes should be monitored
controlled on ICSs
(Continued)

403
SECTION IV Inf ammation, Immunomodulation, and Hematopoiesis

TOXICITIES
CLASS AND
SUBCLASSES NAMES CLINICAL USES COMMON UNIQUE; CLINICALLY IMPORTANT
Antileukotrienes Montelukast Antileukotrienes are indicated Associated with rare cases o hepatic
Leukotriene Za rlukast as an add-on therapy in asthma dys unction; thus liver-associated
Antagonists patients who are not well enzymes should be monitored
controlled on ICSs Cases o Churg-Strauss syndrome
Cys-LT1 receptor antagonists have been reported
have no role in the therapy o
COPD

Anti-IgEMonoclonal Omalizumab Used or the treatment o patients Major side e ect is an anaphylactic
Antibodies with severe asthma who are response, which is uncommon
poorly controlled even on oral (<0.1%)
corticosteroids and in patients
with very severe concomitant
allergic rhinitis

Mucolytics N-acetylcysteine Used to reduce mucus viscosity


DNAase (dornase N-acetylcysteine is not
al a) recommended or COPD
DNAse is indicated in patients
with cystic brosis i there is
signi cant symptomatic and lung
unction improvement a ter a
trial o therapy

Expectorants Guai enesin Enhance clearance o mucus

Antitussives Codeine Cough suppression Codeine and other opiates Dextromethorphan can cause
Dextromethorphan (Note: Cough is a de ensive are associated with hallucinations at higher doses and
Benzonatate re ex; whenever possible, treat the sedation and constipation has signi cant abuse potential
underlying cause, not the cough) (see Chapter 10) Seizures and cardiac arrest have
Many over-the-counter cough Benzonatate is associated occurred with acute ingestion o
medications containing with dizziness and benzonatate
dextromethorphan are largely dysphagia Severe allergic reactions in patients
ine ective taking benzonatate who are allergic
to p-aminobenzoic acid

Ventilatory Doxapram Increase ventilatory rate Nausea, sweating, anxiety, Should be used with caution i
Stimulants (Note: The use o doxapram to and hallucinations hepatic or renal unction is impaired
treat ventilatory ailure in COPD At higher doses, In COPD, the in usion o doxapram is
has now largely been replaced by increased pulmonary and restricted to 2 h
noninvasive ventilation) systemic blood pressures

Prostacyclin (PGI2) Epoprostenol Continuous IVepoprostenol is Vasodilator e ects


and Its Analogs (prostacyclin, PGI2) e ective in lowering pulmonary (headache, ushing,
Treprostinil arterial pressures, improving syncope) diarrhea,
Iloprost exercise per ormance, and nausea, and jaw pain
prolonging survival in primary Inhaled iloprost may
pulmonary artery hypertension cause cough and
(PPAH) bronchoconstriction
Treprostinil and iloprost can be
given by inhalation

Endothelin-1 Bosentan Oral bosentan is ef cacious Bosentan is generally well Risk o serious liver injury; liver
Receptor Ambrisentan in reducing symptoms and tolerated; adverse e ects aminotrans erases should be
Antagonists improving mortality in PPAH include abnormal liver monitored monthly
unction tests, anemia, Risk o testicular atrophy and
headaches, peripheral in ertility
edema, and nasal Bosentan is potentially teratogenic
congestion

Phosphodiesterase-5 Sildena l E ective in lowering pulmonary Headache, ushing, Should not be used in patients
(PDE5) Inhibitors Tadala l resistance and improving dyspepsia, and visual receiving nitrovasodilators due
exercise tolerance in patients disturbances to risk o severe hypotension (see
with PAH Chapter 16)

404
CHAPTER

HematopoieticAgents 25
T is chapter will be most use ul a er having a basic understanding o the material
DRUGS INCLUDED
in Chapter 37, Hematopoietic Agents: Growth Factors, Minerals, and Vitamins in
Goodman & Gilmans T e Pharmacological Basis of T erapeutics, 12th Edition. In IN THIS CHAPTER
addition to the material presented here, the 12th Edition contains: Copper sul ate (cupricsul ate)
A description o hematopoietic cell growth and di erentiation and the role o hema- Cyanocobalamin (vitamin B12; CALOMIST,
topoietic growth actors in these processes NASCOBAL)
A discussion o iron metabolism and storage in the body Darbepoetin al a (ARANESP)
A discussion o actors that cause iron de ciency anemia Eltrombopag (PROMACTA)
A discussion o vitamin de ciencies that can lead to anemia Epoetin al a (recombinant erythropoietin;
able 37-3 Iron Requirements or Pregnancy EPOGEN, PROCRIT,EPREX)
able 37-2 T e Body Content o Iron Ferrous umarate (HEMCYTE, FEOSTAT,
others)
able 37-5 Average Response to Oral Iron
Ferrous gluconate (FERGON, others)
T e molecular structures o vitamin B12 and olic acid, and their congeners
Ferrous succinate, aspartate, and other
LEARNING OBJECTIVES errous salts
Understand the mechanisms o action o drugs, minerals, and vitamins used to Ferrous sul ate (FEOSOL, others)
stimulate hematopoiesis. Ferumoxytol (FERAHEME)
Know the untoward e ects o drugs used to stimulate hematopoiesis. Folicacid (pteroylglutamicacid [PteGlu],
l -methyl olate)
Know the clinical application o hematopoietic growth actors and how e cacy
o therapy is monitored. Folinicacid (leucovorin calcium, citrovorum
actor)
Know how mineral and vitamin de ciencies can lead to speci c anemia, how
these are diagnosed, and how they are treated with individual agents or combi- G-CSF(recombinant G-CSF, lgrastim;
nations o agents. NEUPOGEN)
Know when iron and vitamin supplementation should be used prophylactically. GM-CSF(recombinant GM-CSF, sargramos-
tim; LEUKINE)
Hydroxocobalamin (vitamin B12)
IL-11 (recombinant IL-11; oprelvekin;
[NEUMEGA]
MECHANISMS OF ACTION OF HEMATOPOIETIC AGENTS
Iron dextran (DEXFERRUM, INFED,
DRUG CLASS DRUG MECHANISM OF ACTION IMFERON)
Erythropoiesis- Epoetin al a Stimulate proli eration and maturation Iron sucrose (VENOFER)
Stimulating Agents Darbepoetin al a o committed erythroid progenitors
Pegylated recombinant G-CSF(peg lgras-
(ESAs) to increase red blood cell (RBC)
production (see Table 25-1 and tim; NEULASTA)
Figure 25-1) Polysaccharide- errihydrite complex(NIFE-
REX,others)
Myeloid Growth GM-CSF (sargrastim) Stimulate proli eration and
Factors G-CSF ( lgrastim) di erentiation o 1 or more myeloid Pyridoxine
Pegylated recombinant G-CSF cell lines, and enhance the unction o Ribo avin
(peg lgrastim) mature granulocytes and monocytes
(see Table 25-1 and Figures 25-1 and Romiplostim(NPLATE)
25-2) Sodium erricgluconate (FERRLECIT)
Thrombopoietic IL-11 (oprelvekin) Enhance megakaryocyte maturation
Growth Factors and increases peripheral blood
platelet counts

Romiplostim Activate thrombopoietin receptor


Eltrombopag which stimulates megakaryopoiesis
(Continued)
405
SECTION IV Inf ammation, Immunomodulation, and Hematopoiesis

DRUG CLASS DRUG MECHANISM OF ACTION


Iron Supplements Ferrous sul ate Provide additional dietary iron needed
Oral Ferrous umarate to correct iron-de ciency anemia
Ferrous succinate, aspartate,
and other errous salts
Ferrous gluconate
Polysaccharide- errihydrite
complex

Iron Supplements Iron dextran Provide iron needed to correct iron


Parenteral Sodium erric gluconate de ciency anemia when oral iron
Ferumoxytol therapy ails
Iron sucrose

Copper Supplement Copper sul ate (cupric sul ate) Corrects copper de ciencies
that inter ere with absorption
o iron and release o iron rom
reticuloendothelial cells that result in
anemia

VitaminsPyridoxine Pyridoxine Pyridoxine improves hematopoiesis in


Ribo avin patients with hereditary sideroblastic
anemia and idiopathic acquired
sideroblastic anemia

VitaminsVitamin B12 Cyanocobalamin (vitamin B12) Co actor required or normal synthesis


Hydroxocobalamin (vitamin B12) o purines and pyrimidines needed
or DNA synthesis, especially in rapidly
dividing cells o the hematopoietic
system (see Figure 25-3)

VitaminsFolic Acid Folic acid (pteroylglutamic acid Co actor required or normal synthesis
and Derivatives [PteGlu], l -methyl olate) o purines and pyrimidines needed
Folinic acid (leucovorin calcium, or DNA synthesis, especially in rapidly
citrovorum actor) dividing cells o the hematopoietic
system (see Figure 25-3)

CASE 25-1
A 64-year-old woman with chronic kidney disease has a low hematocrit and is started
on therapy with epoetin al a.
a. What is the mechanism o action o epoetin al a?
Recombinant human erythropoietin (epoetin al a) is nearly identical to the endog-
enous hormone except that the carbohydrate modi cation pattern o epoetin al a
di ers slightly rom the native protein; this di erence apparently does not alter
kinetics, potency, or immunoreactivity o the drug.
T e endogenous protein is heavily glycosylated and expressed primarily in peri-
tubular interstitial cells o the kidney. A er secretion, erythropoietin binds to a
receptor on the sur ace o committed erythroid progenitors in the marrow and is
internalized. With anemia or hypoxemia, synthesis rapidly increases by 100- old
or more, serum erythropoietin levels rise, and marrow progenitor cell survival,
proli eration, and maturation are dramatically stimulated (see Figure 25-1). T is
nely tuned eedback loop can be disrupted by kidney disease, marrow damage, or
a de ciency in iron or an essential vitamin. With an in ection or an in ammatory
state, erythropoietin secretion, iron delivery, and progenitor proli eration all are
suppressed by in ammatory cytokines. T ere is a clear doseresponse relationship
between the epoetin al a dose and the rise in hematocrit in anephric patients, with
eradication o their anemia at higher doses.
More recently, a novel erythropoiesis-stimulating protein, darbepoetin al a,
has been approved or clinical use in patients with indications similar to those
(Continued)
406
Hematopoietic Agents CHAPTER 2 5

TABLE 25-1 Hematopoietic Growth Factors


ERYTHROPOIETIN EPO
Stimulates proli eration and maturation o committed erythroid progenitors to increase
red cell production
STEM CELL FACTOR SCF, c kit ligand, Steel factor and FLT 3 LIGAND FL
Act synergistically with a wide range o other colony-stimulating actors and interleukins
to stimulate pluripotent and committed stem cells
FL also stimulates both dendritic and NKcells (anti-tumor response)
SCF also stimulates mast cells and melanocytes
INTERLEUKINS
IL-1, IL-3, IL-5, IL-6, IL-9, and IL-11
Act synergistically with each other and SCF, GM-CSF, G-CSF, and EPO to stimulate BFU-E,
CFU-GEMM, CFU-GM, CFU-E, and CFU-Meg growth
Numerous immunologic roles, including stimulation o B cell and T cell growth
IL-5
Controls eosinophil survival and di erentiation
IL-6
IL-6 stimulates human myeloma cells to proli erate
IL-6 and IL-11 stimulate BFU-Meg to increase platelet production
IL-1, IL-2, IL-4, IL-7, and IL-12
Stimulate growth and unction o T cells, B cells, NKcells, and monocytes
Co-stimulate B, T, and LAKcells
IL-8 and IL-10
Numerous immunological activities involving B and T cell unctions
IL-8 acts as a chemotactic actor or basophils and neutrophils
GRANULOCYTE MACROPHAGE COLONY STIMULATING FACTOR GM CSF
Acts synergistically with SCF, IL-1, IL-3, and IL-6 to stimulate CFU-GM, and CFU-Meg to
increase neutrophil and monocyte production
With EPO may promote BFU-E ormation
Enhances migration, phagocytosis, superoxide production, and antibody-dependent cell-
mediated toxicity o neutrophils, monocytes, and eosinophils
Prevents alveolar proteinosis
GRANULOCYTE COLONY STIMULATING FACTOR G CSF
Stimulates CFU-G to increase neutrophil production
Enhances phagocytic and cytotoxic activities o neutrophils
MONOCYTE/MACROPHAGE COLONY STIMULATING FACTOR M CSF, CSF 1
Stimulates CFU-M to increase monocyte precursors
Activates and enhances unction o monocyte/macrophages
MACROPHAGE COLONY STIMULATING FACTOR M CSF
Stimulates CFU-M to increase monocyte/macrophage precursors
Acts in concert with tissues and other growth actors to determine the proli eration,
di erentiation, and survival o a range o cells o the mononuclear phagocyte system
THROMBOPOIETIN TPO, Mpl ligand
Stimulates the sel -renewal and expansion o hematopoietic stem cells
Stimulates stem cell di erentiation into megakaryocyte progenitors
Selectively stimulates megakaryocytopoiesis to increase platelet production
Acts synergistically with other growth actors, especially IL-6 and IL-11

BFU, burst- orming unit; CFU, colony- orming unit; E, erythrocyte; G, granulocyte; M, macrophage;
Meg, megakaryocyte; NKcells, natural killer cells; LAKcells, lymphokine-activated killer cells.

or epoetin al a. It is a genetically modi ed orm o erythropoietin in which


4 amino acids have been mutated such that additional carbohydrate side chains
are added during its synthesis, prolonging the circulatory survival o the drug up
to 24 to 26 hours.
b. What are other indications or erythropoiesis-stimulating agents (ESAs)?
Epoetin al a also is e ective in the treatment o anemia associated with surgery,
AIDS, cancer chemotherapy, prematurity, and certain chronic in ammatory condi-
tions. Darbepoetin al a also has been approved or use in patients with anemia asso-
ciated with chronic kidney disease and is under review or several other indications.
(Continued)
407
SECTION IV Inf ammation, Immunomodulation, and Hematopoiesis

ERYTHROPOIETIN
GM-CS F / IL-3 BFU-E/CFU-E
S CF / FL Re d
blood
ce lls
Totipote nt/pluripote nt
s te m ce lls GM-CS F / G-CS F
Gra nulocyte s

CFU-GM Eos inophils

CFU-GEMM Ba s ophils

GM-CS F / M-CS F

CFU-Me g Monocyte s

Lymphocyte
Proge nitor

P la te le ts
B ce lls NK ce lls
T ce lls

IL-1 / IL-2 / IL-3 / IL-4 / IL-6 Me ga ka ryocyte


IL-6 / IL-11 / THROMBOPOIETIN

FIGURE 25-1 Sites o action o hematopoietic growth actors in the di erentiation and maturation
o marrow cell lines. A sel -sustaining pool o marrow stem cells di erentiates under the in uence
o speci c hematopoietic growth actors to orm a variety o hematopoietic and lymphopoietic
cells. Stem cell actor (SCF), ligand (FL), interleukin-3 (IL-3), and granulocyte-macrophage colony-
stimulating actor (GM-CSF), together with cellcell interactions in the marrow, stimulate stem cells
to orm a series o burst- orming units (BFU) and colony- orming units (CFU): CFU-GEMM (granulo-
cyte, erythrocyte, monocyte, and megakaryocyte), CFU-GM (granulocyte and macrophage), CFU-Meg
(megakaryocyte), BFU-E (erythrocyte), and CFU-E (erythrocyte). A ter considerable proli eration, urther
di erentiation is stimulated by synergistic interactions with growth actors or each o the major cell
linesgranulocyte colonystimulating actor (G-CSF), monocyte/macrophage-stimulating actor
(M-CSF), thrombopoietin, and erythropoietin. Each o these actors also in uences the proli eration,
maturation, and, in some cases, the unction o the derivative cell line (see Table 25-1).

IL-4
T c e ll B c e ll
IL-2

IFN-
Antibody
IL-1 IL-1
IL-3
GM-CS F

Mac ro phag e G-CS F

Bo ne Marrow

FIGURE 25-2 Cytokinecell interactions. Macrophages, T cells, B cells, and marrow stem cells
interact via several cytokines (IL-1, IL-2, IL-3, IL-4, IFN [inter eron]-, GM-CSF, and G-CSF) in
response to a bacterial or a oreign antigen challenge. See Table 25-1 or the unctional activities
o these various cytokines.
408
Hematopoietic Agents CHAPTER 2 5

Highly competitive athletes have used epoetin al a to increase their hemoglobin levels
(blood doping) and improve per ormance. Un ortunately, this misuse o the drug
has been implicated in the deaths o several athletes and is strongly discouraged.
c. What are the therapeutic considerations to optimize therapy with epoetin al a?
During erythropoietin therapy, absolute or unctional iron de ciency may develop.
Functional iron de ciency (ie, normal erritin levels but low trans errin saturation)
presumably results rom the inability to mobilize iron stores rapidly enough to
support the increased erythropoiesis. Virtually all patients eventually will require
supplemental iron to increase or maintain trans errin saturation to levels that will
adequately support stimulated erythropoiesis.
Resistance to epoetin al a therapy is common in patients who develop an in am-
matory illness or become iron de cient, so close monitoring o general health and
iron status is essential. Less common causes o resistance include occult blood loss,
olic acid de ciency, carnitine de ciency, inadequate dialysis, aluminum toxicity,
and osteitis brosa cystica secondary to hyperparathyroidism.
d. What are the possible toxicities and hazards o therapy with epoetin al a?
T e most common side e ect o epoetin al a therapy is aggravation o hyperten-
sion, which occurs in 20 to 30% o patients and most o en is associated with a
rapid rise in hematocrit. Blood pressure usually can be controlled either by increas-
ing antihypertensive therapy, ultra ltration in dialysis patients, or by reducing
the epoetin al a dose to slow the hematocrit response. ESAs should not be used in
patients with preexisting uncontrolled hypertension.
Headache, tachycardia, edema, shortness o breath, nausea, vomiting, diarrhea,
injection site stinging, and u-like symptoms (eg, arthralgias and myalgias) also
have been reported in conjunction with epoetin al a therapy.
During hemodialysis, patients receiving epoetin al a or darbepoetin may require
increased anticoagulation. Serious thromboembolic events have been reported,
including migratory thrombophlebitis, microvascular thrombosis, pulmonary
embolism, and thrombosis o the retinal artery and temporal and renal veins. T e
risk o thrombotic events, including vascular access thromboses, was higher in
adults with ischemic heart disease or congestive heart ailure receiving epoetin al a
therapy with the goal o reaching a normal hematocrit (42%) than in those with
a lower-target hematocrit o 30%. T e higher risk o cardiovascular events rom
erythropoietic therapies may be associated with higher hemoglobin or higher rates
o rise in hemoglobin. T e hemoglobin level should be managed to avoid exceeding
a target level o 12 g/dL.
ESA use is associated with increased rates o cancer recurrence and decreased on-
study survival in patients in whom the drugs are administered or cancer-induced
or or chemotherapy-induced anemia. T e cause(s) o this e ect is presently
unclear, but some studies suggest that tumor cells bearing the erythropoietin recep-
tor are more likely to be a ected by the use o ESAs.

CASE 25-2
A 42-year-old man with leukemia will undergo autologous peripheral blood stem cell
transplant ollowing high-dose chemotherapy.
a. What is the rationale or using myeloid growth actors in this procedure?
T e myeloid growth actors (see Figure 25-1 and able 25-1) are glycoproteins that
stimulate the proli eration and di erentiation o 1 or more myeloid cell lines. T ey
also enhance the unction o mature granulocytes and monocytes. Recombinant
orms o several growth actors have been produced, including granulocyte-
macrophage colony-stimulating actor (GM-CSF), granulocyte colony-stimulating
actor (G-CSF), interleukin-3 (IL-3), macrophage colony-stimulating actor (M-CSF;
also known as colony-stimulating actor, CSF-1), and stem cell actor (SCF).
(Continued)
409
SECTION IV Inf ammation, Immunomodulation, and Hematopoiesis

Recombinant human GM-CSF (sargramostim) and recombinant human G-CSF


( lgrastim) are used in patients undergoing autologous bone marrow and periph-
eral blood stem cell transplant ollowing high-dose chemotherapy to stimulate the
proli eration, di erentiation, and unction o myeloid cell lines that are required to
ght in ection. Sargramostim and lgrastim shorten the duration o severe neutro-
penia and reduce transplant morbidity secondary to bacterial and ungal in ections
without a change in long-term survival or risk o inducing an early relapse o the
malignant process.
b. What is the di erence in the e ects o GM-CSF versus G-CSF?
GM-CSF is capable o stimulating the proli eration, di erentiation, and unction o
a number o the myeloid cell lineages (see Figure 25-1). It acts synergistically with
other growth actors, including erythropoietin, at the level o the burst- orming
unit (BFU). GM-CSF stimulates the colony- orming units (CFUs) CFU-GEMM,
CFU-GM, CFU-M, CFU-E, and CFU-Meg to increase cell production. It also
enhances the migration, phagocytosis, superoxide production, and antibody-
dependent cell-mediated toxicity o neutrophils, monocytes, and eosinophils.
T e activity o G-CSF is restricted to neutrophils and their progenitors, stimulat-
ing their proli eration, di erentiation, and unction. It acts primarily on the CFU-
G, although it also can play a synergistic role with IL-3 and GM-CSF in stimulating
other cell lines. G-CSF enhances phagocytic and cytotoxic activities o neutrophils.
Unlike GM-CSF, G-CSF has little e ect on monocytes, macrophages, and eosino-
phils, and reduces in ammation by inhibiting IL-1, tumor necrosis actor, and
inter eron . G-CSF also mobilizes primitive hematopoietic cells, including hema-
topoietic stem cells, rom the marrow into the peripheral blood. T is observation
has virtually trans ormed the practice o stem cell transplantation, such that more
than 90% o all such procedures today use G-CSFmobilized peripheral blood stem
cells (PBSCs) as the donor product.
c. What are other indications or G-CSF?
G-CSF also is e ective in the treatment o severe congenital neutropenias. In
patients with cyclic neutropenia, G-CSF therapy will increase the level o neutro-
phils and shorten the length o the cycle suf ciently to prevent recurrent bacterial
in ections. Filgrastim therapy can improve neutrophil counts in some patients
with myelodysplasia or marrow damage (moderately severe aplastic anemia or
tumor in ltration o the marrow). T e neutropenia o AIDS patients receiving
zidovudine also can be partially or completely reversed. Filgrastim is routinely
used in patients undergoing peripheral blood stem cell (PBSC) collection or stem
cell transplantation. It promotes the release o CD34+ progenitor cells rom the
marrow, reducing the number o collections necessary or transplant. Moreover,
lgrastim-mobilized PBSCs appear more capable o rapid engra ment. PBSC-
transplanted patients require ewer days o platelet and red blood cell trans usions
and a shorter duration o hospitalization than do patients receiving autologous
bone marrow transplants.
INTERACTIONS OF
VITAMIN B12 AND FOLATE CASE 25-3
IN INTRACELLULAR A 54-year-old woman diagnosed with metastatic breast cancer is undergoing chemo-
METABOLISM (Se e therapy. A er the rst course o chemotherapy, her platelet count drops to very low
Fig ure 25 3) levels. o avoid using platelet trans usions in this patient, she is given a daily injection
o oprelvekin ollowing her second course o chemotherapy.
Conversion o homocysteine to methionine
Conversion o serine to glycine a. What is the mechanism o action o oprelvekin?
Synthesis o thymidylate Oprelvekin is a recombinant version o IL-11, a cytokine that stimulates hemato-
poiesis and enhances megakaryocyte maturation (see Figure 25-1 and able 25-1).
Histidine metabolism
Oprelvekin is approved or use in patients undergoing chemotherapy or non-
Synthesis o purines myeloid malignancies that displayed severe thrombocytopenia (platelet count
Utilization or generation o ormate
(Continued)
410
Hematopoietic Agents CHAPTER 2 5

<20,000/L) on a prior cycle o the same chemotherapy, and it is administered


CAUSES OF VITAMIN
daily until the platelet count returns to greater than 100,000/L. Administration
B12 DEFICIENCY (Se e
o the recombinant cytokine is associated with less severe thrombocytopenia and
reduced use o platelet trans usions in patients who previously demonstrated Fig u re 25 4)
signi cant chemotherapy-induced thrombocytopenia. De ciencyo vitamin B12 can result roma
congenital or acquired de ect in anyone o
b. What are the side e ects o oprelvekin?
the ollowing:
T e major complications o oprelvekin therapy are uid retention and associ-
Inadequate dietarysupply
ated cardiac symptoms, such as tachycardia, palpitation, edema, and shortness o
breath; this is a signi cant concern in elderly patients and o en requires concomi- Inadequate secretion o gastricintrinsic
tant therapy with diuretics. Fluid retention reverses upon drug discontinuation, actor (classicpernicious anemia)
but volume status should be care ully monitored in elderly patients, those with a Ileal disease
history o heart ailure, or those with preexisting uid collections in the pleura, Congenital absence o transcobalamin
pericardium, or peritoneal cavity. Also reported are blurred vision, injection-site II(TcII)
rash or erythema, and paresthesias.
Rapid depletion o hepaticstores by
inter erence with reabsorption o
vitamin B12 excreted in bile

TABLE 25-2 Daily Iron Intake and Absorption


IRON AVAILABLE IRON SAFETY FACTOR GENERAL PRINCIPLES OF
REQUIREMENT (mg/kg) POOR DIET AVAILABLE/ VITAMIN B12 THERAPY
SUBJECT (mg/kg) GOOD DIET REQUIREMENT
1. Vitamin B12 should be given prophylacti-
In ant 67 33-66 0.5-1
callyonlywhen there is a reasonable prob-
Child 22 48-96 2-4 abilitythat a de ciencyexists or will exist:
Dietaryde ciencyin the strict
Adolescent (male) 21 30-60 1.5-3
vegetarian
Adolescent ( emale) 20 30-60 1.5-3 The predictable malabsorption o
Adult (male) 13 26-52 2-4 vitamin B12 in patients who have had
a gastrectomy
Adult ( emale) 21 18-36 1-2 Certain diseases o the small intestine
Mid-to-late pregnancy 80 18-36 0.22-0.45 When GI unction is normal, an oral pro-
phylacticsupplement o vitamins and
minerals, including vitamin B12, may
be indicated; otherwise, the patient
should receive monthlyinjections o
TABLE 25-3 Recommended Dietary Allowances (RDAs) For Iron For Nonvegetariansa cyanocobalamin
(Food and Nutrition Board at the Institute of Medicine, 2001)
2. The relative ease o treatment with vitamin
PREGNANCY B12 should not prevent a ull investigation
AGE MALE (mg) FEMALE (mg) (mg) LACTATION (mg)
o the etiologyo the de ciency.
Birth to 6 months 0.27b 0.27b The initial diagnosis usuallyis sug-
7-12 mo 11 11 gested bya macrocyticanemia or an
unexplained neuropsychiatricdisorder.
1-3 y 7 7
Full understanding o the etiologyo
4-8 y 10 10 vitamin B12 de ciencyinvolves studies
o dietarysupply, GIabsorption, and
9-13 y 8 8 transport.
14-18 y 11 15 27 10 3. Therapyalways should be as speci cas
possible.
19-50 y 8 18 27 9
The use o shotgun vitamin therapyin
51+ y 8 8 the treatment o vitamin B12 de ciency
can be dangerous; there is the danger
a
The RDAs or vegetarians are 1.8 times higher than or people who eat meat.
that su cient olicacid will be given
b
For in ants rom birth to 6 months, the Food and Nutrition Board established an Adequate
Intake (AI; established when evidence is insuf cient to develop an RDA) or iron that is to result in a hematologicrecovery
equivalent to the mean intake o iron in healthy, breast- ed in ants. (continues)

411
SECTION IV Inf ammation, Immunomodulation, and Hematopoiesis

GENERAL PRINCIPLES OF Plas ma Ce ll DNA


VITAMIN B12 THERAPY dUMP
(De oxyuridyla te )
(Cont.) dTMP
CH3 H4 Pte Glu 5 H2 P te Glu
(Thymidyla te )
that can maskcontinued vitamin B12
?
de ciencyand permit neurological
damage to develop or progress. CH3 H4 P te Glu 1 CH3 H4 Pte Glu 1 H4 Pte Glu 5,10-CH2 H4 Pte Glu
+B6
4. Aclassictherapeutictrial with small S e rine Glycine
amounts o vitamin B12 can help con rm CH3 B 12
5,10-CHH4P te Glu
the diagnosis, but acutelyill elderly Homocys te ine
B12 -TcII ?
patients maynot be able to tolerate the
Me thionine
delayin the correction o a severe anemia. 5-CHOH4 Pte Glu
B 12 Purine
Such patients require supplemental FIGLU Synthe s is
blood trans usions and immediate TcII
De oxyade no s yl B 12 10-CHOH4 Pte Glu
therapywith olicacid and vitamin B12
to guarantee rapid recovery.
Me thylma lonyl S uccinyl Gluta mic
5. Long-termtherapywith vitamin B12 must CoA a cid 5-CHNHH4 Pte Glu
CoA
be evaluated at intervals o 6 to 12 months
in patients who are otherwise well
FIGURE 25-3 Interrelationships and metabolic roles o vitamin B12 and olic acid. See Side Bar
I there isanadditional illnessor a condi-
INTERACTIONS OF VITAMIN B12 AND FOLATE IN INTRACELLULAR METABOLISM. FIGLU, ormimino-
tionthat mayincrease the requirement or glutamic acid, which arises rom the catabolism o histidine; TcII, transcobalamin II; CH3H4PteGlu1,
the vitamin(eg, pregnancy), reassessment methyltetrahydro olate.
shouldbe per ormedmore requently.

DIET DIET Live r


1 CH3 H4 P te Glu 1-8 CH3 H4 P te Glu 5
Intrins ic 3-8 g He pa tic B12 S tore s 1 P te Glu 1
2 Fa ctor-B12 1-10 mg
Complex CH3 H4 P te Glu 1 P te Glu 1
5
Live r 4 H2 or H4 P te Glu 1
4 TcI-B12 2 3
TcII-B12 a nd
6 TcIII-B12 Hydrolys is P la s ma binding
"Re le a s ing Re duction prote in
7 Tis s ue
Fa ctor" Me thyla tion
3 B 12
-CH3
+ CH3 CH3 H4 P te Glu 1 H2 or H4 P te Glu 1
5 B12
Me thyl De oxya de nosyl
B12 B12 CH3 H4 P te Glu 5
Tis s ue
FIGURE 25-4 The absorption and distribution o vitamin B12.
De ciency o vitamin B12 can result rom a congenital or acquired FIGURE 25-5 Absorption and distribution o olate derivatives.
de ect in any one o the ollowing: (1) inadequate dietary sup- Dietary sources o olate polyglutamates are hydrolyzed to the
ply; (2) inadequate secretion o intrinsic actor (classic pernicious monoglutamate, reduced, and methylated to CH3H4PteGlu 1 dur-
anemia); (3) ileal disease; (4) congenital absence o transcobala- ing gastrointestinal transport. Folate de ciency commonly results
min II (TcII); or (5) rapid depletion o hepatic stores by inter er- rom (1) inadequate dietary supply and (2) small intestinal disease.
ence with reabsorption o vitamin B12 excreted in bile. The utility In patients with uremia, alcoholism, or hepatic disease there may
o measurements o the concentration o vitamin B12 in plasma be de ects in (3) the concentration o olate binding proteins in
to estimate supply available to tissues can be compromised by plasma and (4) the ow o CH3H4PteGlu1 into bile or reabsorption
liver disease and (6) the appearance o abnormal amounts o and transport to tissue (the olate enterohepatic cycle). Finally,
transcobalamins I and III (TcI and III) in plasma. Finally, the orma- vitamin B12 de ciency will (5) trap olate as CH3H4PteGlu, thereby
tion o methylcobalamin requires (7) normal transport into cells reducing the availability o H4PteGlu 1 or its essential roles in purine
and an adequate supply o olic acid as CH3H4PteGlu1. and pyrimidine synthesis.
412
Hematopoietic Agents CHAPTER 2 5

CASE 25-4
A 22-year-old woman is pregnant with her rst child. She is in good health, but her
pediatrician prescribes an oral iron supplement.
a. What is the rationale or supplemental iron in this patient?
In developed countries, the normal adult diet contains ~6 mg o iron per 1000 calo-
ries, providing an average daily intake or adult men o between 12 and 20 mg, and
or adult women a daily intake o between 8 and 15 mg. Normal iron absorption is
~1 mg/d in adult men and 1.4 mg/d in adult women; 3 to 4 mg o dietary iron is the
most that can normally be absorbed.
Pregnancy and lactation impose a greater requirement or iron with the growth o
the etus and expansion o red cell mass (see ables 25-2 and 25-3). A recent study
ound that 18% o pregnant women in the United States had iron de ciency, with
6.9% in the rst trimester, 14.3% in the second trimester, and 29.7% in the third tri-
mester. Iron de ciency during pregnancy increases the risk o maternal and in ant
mortality, premature birth, low birth weight, and impaired cognitive and behavioral
development o the in ant. In ants, especially those born preterm, with low birth
weight, or whose mothers have iron de ciency, are at risk o iron de ciency due to
their rapid growth.
Whereas iron balance in adult men and nonmenstruating women is reasonably
secure, pregnancy and in ancy represent periods o negative iron balance, and men-
struating women also are at risk o iron de ciency. T e di erence between dietary
supply and requirements is re ected in the size o iron stores, which are low or
absent when iron balance is precarious and high when iron balance is avorable (see
able 37-2, Goodman and Gilmans T e Pharmacological Basis of T erapeutics, 12th
Edition). T us in in ants a er the third month o li e and in pregnant women a er
the rst trimester, stores o iron are negligible. Menstruating women have approxi-
mately one-third the stored iron ound in adult men, indicative o the extent to
which the additional average daily loss o ~0.5 mg o iron a ects iron balance.
b. What are the e ects o iron def ciency in in ants and children?
Iron de ciency in in ants and young children can lead to behavioral disturbances
and can impair development, which may not be ully reversible. Iron de ciency in
children also can lead to an increased risk o lead toxicity secondary to pica and
an increased absorption o heavy metals. Premature and low-birth-weight in ants
are at greatest risk or developing iron de ciency, especially i they are not breast- COMMON CAUSES OF
ed and/or do not receive iron- orti ed ormula. During adolescence there is a FOLATE DEFICIENCY (se e
rapid growth combined with irregular dietary habits and the risk o iron de ciency FIGURE 25-5)
increases again. Adolescent girls are at greatest risk; the dietary iron intake o most
Inadequate dietarysupply.
girls ages 11 to 18 years is insuf cient to meet their requirements.
Small intestinal disease.
c. What are some o the considerations in optimizing the e ects o oral medicinal iron?
In patients with uremia, alcoholism, or
o prevent iron de ciency in pregnant women, doses o 15 to 30 mg o iron per day hepaticdisease there maybe de ects in:
are adequate to meet the daily requirement o the last 2 trimesters. Orally administered
The concentration o olate binding
errous sul ate is the treatment o choice or iron de ciency. Ferrous salts are absorbed
proteins in plasma
about 3 times as well as erric salts, and the discrepancy becomes even greater at high
dosages. Variations in the particular errous salt have relatively little e ect on bioavail- The owo CH3H4PteGlu1 into bile or
ability; the sul ate, umarate, succinate, gluconate, aspartate, other errous salts, and reabsorption and transport to tissue
polysaccharide- errihydrite complex are all absorbed to approximately the same extent. (the olate enterohepaticcycle)
T e e ective dose o all o these preparations is based on iron content. It is essential Vitamin B12 de ciencywilltrap olate
that the coating o the tablet dissolve rapidly in the stomach because iron usually as CH3H4PteGlu, therebyreducing the
is absorbed in the upper small intestine. Bioavailability o iron ingested with ood is availabilityo H4PteGlu1 or its essential
probably one-hal or one-third o that seen in the asting subject. It is always pre er- roles in purine and pyrimidine synthesis.
able to administer iron in the asting state, even i the dose must be reduced because
o GI side e ects. Antacids also reduce iron absorption i given concurrently.

(Continued)
413
SECTION IV Inf ammation, Immunomodulation, and Hematopoiesis

Clinically, the e ectiveness o iron therapy is best evaluated by tracking the reticu-
GENERAL PRINCIPLES OF
locyte response and the rise in the hemoglobin or the hematocrit. An increase in
FOLIC ACID THERAPY
the reticulocyte count is not observed or at least 4 to 7 days a er beginning ther-
1. Prophylacticadministration o olicacid apy. A measurable increase in the hemoglobin level takes even longer.
should be undertaken or clear indications.
Dietarysupplementation is necessary CASE 25-5
when there is a requirement that may
not be met byanormaldiet. During his annual visit to his amily physician, a 68-year-old man complains o numb-
ness in his hands, occasional tremors and loss o balance, and some loss o memory. He
The dailyingestion o a multivitamin is ound to be anemic with abnormally large red cells and other blood cell anomalies.
preparation containing 400 to 500 g He is a strict vegetarian and does not eat eggs or dairy products.
o olicacid has become standard
practice be ore and during pregnancy a. What might be contributing to this patients condition?
to reduce the incidence o neural tube T is patient has symptoms consistent with de ciency o either vitamin B12 or olic
de ects and or as long as a woman is acid. An early sign o de ciency is megaloblastic anemia. Abnormal macrocytic red
breast- eeding. blood cells are produced, and the patient becomes severely anemic. T ese vitamins
In women with a historyo a pregnancy are key co actors in the biochemical pathways that are critical or normal synthe-
complicated bya neural tube de ect, sis o purines and pyrimidines (see Figure 25-3). A de ciency o either vitamin
an even larger dose o 4 mg/d has been impairs DNA synthesis in any cell in which chromosomal replication and division
recommended (MRCVitamin Study are taking place. In addition, a de ciency o either vitamin B12 or olate leads to the
Research Group). decreased synthesis o methionine and S-adenosylmethionine, which inter eres
Patients on total parenteral nutrition with protein biosynthesis, a number o methylation reactions, and the synthesis o
should receive olicacid supplements as polyamines. Because tissues with the greatest rate o cell turnover show the most
part o their uid regimen because liver dramatic e ects with alterations in these pathways, the hematopoietic system is
olate stores are limited. especially sensitive to de ciencies o these vitamins. Neurological lesions are also
common with vitamin B12 de ciency, although the mechanisms are not well under-
Adult patients with a disease state
stood. Neurological signs and symptoms include paresthesia o the hands and eet,
characterized byhigh cell turnover (eg,
decreased vibration and position senses with resultant unsteadiness, decreased
hemolyticanemia) generallyrequire
deep tendon re exes, and in the later stages o vitamin B12 de ciency, con usion,
larger doses, 1 mg o olicacid given
moodiness, loss o memory, and even a loss o central vision. T e patient may
once or twice a day.
exhibit delusions, hallucinations, or even overt psychosis. Because the neurological
The 1-mg dose also has been used in damage can be dissociated rom the changes in the hematopoietic system, vitamin
the treatment o patients with elevated B12 de ciency must be considered in elderly patients with dementia or psychiatric
levels o homocysteine. disorders, even i they are not anemic.
2. As with vitamin B12 de ciency, anypatient
b. Why might this patient be at risk or vitamin B12 or olic acid def ciency?
with olate de ciencyand a megaloblastic
anemia should be evaluated care ullyto T is patient is a strict vegetarian and may be lacking these vitamins because o his diet
determine the underlying cause o the (see Side Bars COMMON CAUSES OF FOLA E DEFICIENCY and CAUSES OF
de ciencystate including: VI AMIN B12 DEFICIENCY). Vegetable products are ree o vitamin B12 unless they
are contaminated with microorganisms that grow in soil, sewage, water, or the intesti-
The ef ects o medications
nal lumen o animals that synthesize the vitamin. Despite this, strict vegetarians rarely
The amount o alcohol intake develop vitamin B12 de ciency because some vitamin B12 is available rom legumes,
The patients historyo travel which are contaminated with bacteria capable o synthesizing vitamin B12, and because
The unction o the GItract vegetarians o en orti y their diets with a wide range o vitamins and minerals.
3. Therapyalways should be as speci cas Although this patient could be de cient in vitamin B12 because o diet, vita-
possible. min B12 de ciency in adults is rarely the result o a de cient diet per se; rather,
it usually re ects a de ect in one or another aspect o the complex sequence o
Multivitamin preparations should be
steps in vitamin B12 absorption that is depicted in Figure 25-4. Achlorhydria and
avoided unless there is good reason to
decreased secretion o intrinsic actor by parietal cells secondary to gastric atrophy
suspect de ciencyo several vitamins.
or gastric surgery is a common cause o vitamin B12 de ciency in adults. Antibod-
4. The potential danger o mistreating a ies to parietal cells or intrinsic actor complex also can play a prominent role in
patient who has vitamin B12 de ciency producing a de ciency. A number o intestinal diseases can inter ere with absorp-
with olicacid must be kept in mind. tion, including pancreatic disorders (loss o pancreatic protease secretion), bacterial
The administration o large doses o overgrowth, intestinal parasites, sprue, and localized damage to ileal mucosal cells
olicacid can result in an apparent by disease or as a result o surgery.
improvement in the megaloblasticane- T is patient is not likely to be de cient in dietary intake o olic acid because o his veg-
mia, inasmuch as PteGlu is converted etarian diet since many ood sources are rich in olate, especially resh green vegetables,
(Continued)
414
Hematopoietic Agents CHAPTER 2 5

yeast, and some ruits. (Liver is also rich in olate but would not be part o a vegetar-
GENERAL PRINCIPLES OF
ian diet.) Generally, a standard US diet provides 50 to 500 g o absorbable olate per
FOLIC ACID THERAPY
day (the recommended daily intake or the normal adult is 400 g), although individu-
als with high intakes o resh vegetables and meats will ingest as much as 2 mg/d. (Cont.)
Folate de ciency is a common complication o diseases o the small intestine that bydihydro olate reductase to H4PteGlu;
inter ere with the absorption o olate rom ood and the recirculation o olate this circumvents the methyl olatetrap.
through the enterohepatic cycle (see Figure 25-5). Because most absorption occurs Folate therapydoes not prevent or alle-
in the proximal portion o the small intestine, it is not unusual or olate de ciency to viate the neurological de ects o vitamin
occur when the jejunum is diseased. Both nontropical and tropical sprues are common B12 de ciency, and these mayprogress
causes o olate de ciency and megaloblastic anemia. In acute or chronic alcoholism, and become irreversible.
daily intake o olate in ood may be severely restricted, and the enterohepatic cycle o
the vitamin may be impaired by toxic e ects o alcohol on hepatic parenchymal cells;
this is the most common cause o olate-de cient megaloblastic erythropoiesis.
c. What treatment is indicated or this patient?
T e general principles o therapy are described in the Side Bars GENERAL PRNCI-
PLES OF VI AMIN B12 HERAPY and GENERAL PRNCIPLES OF FOLIC ACID
HERAPY. Because this patient has symptoms o neurological damage, e ective
therapy must not wait or detailed diagnostic tests. Inasmuch as the patient requires
therapy be ore the exact cause o the disease has been de ned, it is important to
avoid the potential problem o a combined de ciency o vitamin B12 and olic acid.
When the patient is de cient in both, therapy with only 1 vitamin will not provide
an optimal response. Long-standing nontropical sprue is 1 example o a disease in
which combined de ciency o B12 and olate is common. Once the megaloblastic
erythropoiesis has been con rmed and suf cient blood collected or later mea-
surements o vitamin B12 and olic acid, the patient should receive intramuscular
injections o 100 g o cyanocobalamin and 1 to 5 mg o olic acid. For the next 1
to 2 weeks the patient should receive daily intramuscular injections o 100 g o
cyanocobalamin, together with a daily oral supplement o 1 to 2 mg o olic acid.
T e therapeutic response may be monitored by study o the hematopoietic system.
Within 48 hours o the initiation o appropriate therapy, megaloblastic erythropoi-
esis disappears, and as ef cient erythropoiesis begins, the concentration o iron in
plasma alls to normal or below normal values. T e reticulocyte count begins to rise
on the second or third day and reaches a peak by the h to seventh days; the reticu-
locyte index re ects the proli erative state o the marrow. Finally, the hematocrit
begins to rise during the second week. Patients with complicating iron de ciency, an
in ection or other in ammatory state, or renal disease may be unable to correct their
anemia. T ere ore, it is important to monitor the reticulocyte index over the rst sev-
eral weeks. I it does not continue at elevated levels while the hematocrit is less than
35%, plasma concentrations o iron and olic acid should again be determined and
the patient reevaluated or an illness that could inhibit the response o the marrow.
T e degree and rate o improvement o neurological signs and symptoms depend
on the severity and the duration o the abnormalities. T ose that have been present
or only a ew months usually disappear relatively rapidly. When a de ect has been
present or many months or years, ull return to normal unction may never occur.
Once begun, vitamin B12 therapy must be maintained or li e. T is act must be
impressed on the patient and amily, and a system must be established to guaran-
tee continued monthly injections o cyanocobalamin. Intramuscular injection o
100 g o cyanocobalamin every 4 weeks is suf cient to maintain a normal con-
centration o vitamin B12 in plasma and an adequate supply or tissues. Patients
with severe neurological symptoms and signs may be treated with larger doses o
vitamin B12 in the period immediately a er the diagnosis. Doses o 100 g/d or
several times per week may be given or several months with the hope o encour-
aging aster and more complete recovery. It is important to monitor vitamin B12
concentrations in plasma and to obtain peripheral blood counts at intervals o 3 to
6 months to con rm the adequacy o therapy. Because re ractoriness to therapy can
develop at any time, evaluation must continue throughout the patients li e.

415
SECTION IV Inf ammation, Immunomodulation, and Hematopoiesis

KEY CONCEPTS
Recombinant erythropoietin is used routinely or patients with the anemia o
renal insu ciency, inf ammation, and anemia associated with cancer or the
therapy o cancer.
Myeloid growth actors (eg, GM-CSF and G-CSF) are used to hasten the
recovery o granulocytes a er myelosuppressive therapy, to help mobilize
hematopoietic stem cells into the peripheral blood to allow their harvest or
transplantation, and to augment the number o mature leukocytes in the periph-
eral blood so that they can be used in patients with overwhelming in ection.
IL-11 and small molecules that activate the thrombopoietin receptor are used to
treat acquired and hereditary thrombocytopenias.
Iron de ciency is the most common nutritional disorder and can result rom a
variety o causes, including conditions with increased iron requirements such as
pregnancy, in ancy, menstruation, and blood loss.
De ciency o vitamin B12 and olic acid result in a characteristic megaloblastic
anemia, but vitamin B12 de ciency can also lead to serious neurological lesions.
T e megaloblastic anemia that results rom olate de ciency cannot be dis-
tinguished rom that caused by vitamin B12 de ciency; thus it is important to
consider both de ciencies when initiating treatment and monitoring therapy to
avoid irreversible neurologic damage.

SUMMARY QUIZ

QUESTION 25-1 Used prophylactically in pregnant women to prevent neural tube


de ects in the etus:
a. Ribof avin
b. Pyridoxine
c. Iron
d. Vitamin B12
e. Folic acid

QUESTION 25-2 Indicated or use in patients su ering rom cyclic neutropenia to


stimulate neutrophil production:
a. Epoetin al a
b. Filgrastim
c. Darbepoetin al a
d. Romiplostim
e. Oprelvekin

QUESTION 25-3 T erapy with olinic acid is used to treat anemia associated with the
a. antituberculosis drugs isoniazid and pyrazinamide.
b. antiparkinson drug levodopa.
c. antibiotic chloramphenicol.
d. anticancer agent carboplatin.
e. anticancer agent methotrexate.

QUESTION 25-4 De ciency in olic acid is characterized by


a. microcytic, hypochromic anemia.
b. sideroblastic anemia.
c. neutropenia.
(Continued)
416
Hematopoietic Agents CHAPTER 2 5

d. red-cell aplasia.
e. megaloblastic anemia.

QUESTION 25-5 Which o the ollowing agents activates the thrombopoietin receptor
on megakaryocytes?
a. Epoetin al a
b. Filgrastim
c. Darbepoetin al a
d. Romiplostim
e. Oprelvekin

SUMMARY QUIZ ANSWER KEY

QUESTION 25-1 Answer is e. Folate de ciency has been implicated in the incidence
o neural tube de ects, including spina bi da, encephaloceles, and anencephaly. T is
is true even in the absence o olate-de cient anemia or alcoholism. T e daily inges-
tion o a multivitamin preparation containing 400 to 500 g o olic acid has become
standard practice be ore and during pregnancy to reduce the incidence o neural tube
de ects and or as long as a woman is breast- eeding. In women with a history o a
pregnancy complicated by a neural tube de ect, an even larger dose o 4 mg/d has been
recommended.

QUESTION 25-2 Answer is b. Filgrastim is a recombinant orm o G-CSF that is


administered by subcutaneous injection or slow intravenous in usion. Unlike natural
G-CSF, it is not glycosylated and carries an extra N-terminal methionine. T e principal
action o lgrastim is the stimulation o CFU-G to increase neutrophil production (see
Figure 25-1). It also enhances the phagocytic and cytotoxic unctions o neutrophils. Its
primary indication is in the treatment o severe neutropenia a er autologous hemato-
poietic stem cell transplantation and high-dose cancer chemotherapy. Cyclic neutrope-
nia is a congenital neutropenia that is characterized by a neutropenia that recurs every
14 to 35 days, with most patients exhibiting a cycle o 21 days. In patients with cyclic
neutropenia, G-CSF therapy will increase the level o neutrophils and shorten the
length o the cycle su ciently to prevent recurrent bacterial in ections.

QUESTION 25-3 Answer is e. Folinic acid (leucovorin calcium, citrovorum actor) is


the 5- ormyl derivative o tetrahydro olic acid. T e principal therapeutic uses o olinic
acid are to circumvent the inhibition o dihydro olate reductase as a part o high-dose
methotrexate therapy and to potentiate f uorouracil in the treatment o colorectal can-
cer (see Chapter 45). It also has been used as an antidote to counteract the toxicity o
olate antagonists such as pyrimethamine or trimethoprim.
Anemias can be a side e ect o other drug therapies and the vitamin pyridoxine
can correct some o these. Oral therapy with pyridoxine is o proven bene t in cor-
recting the sideroblastic anemias associated with the antituberculosis drugs isoniazid
and pyrazinamide, which act as vitamin B6 antagonists. A daily dose o 50 mg o pyri-
doxine completely corrects the de ect without inter ering with treatment, and routine
supplementation o pyridoxine o en is recommended (see Chapter 42). In contrast, i
pyridoxine is given to counteract the sideroblastic abnormality associated with admin-
istration o levodopa, the e ectiveness o levodopa in controlling Parkinson disease is
decreased. Pyridoxine therapy does not correct the sideroblastic abnormalities pro-
duced by chloramphenicol or lead.

QUESTION 25-4 Answer is e. An early sign o olic acid or vitamin B12 de ciency is
megaloblastic anemia. Abnormal macrocytic red blood cells are produced, and the
patient becomes severely anemic. T is pattern o abnormal hematopoiesis is also termed
pernicious anemia. T e characteristic abnormality in red blood cell morphology is impor-
tant or diagnosis and as a therapeutic guide ollowing administration o the vitamins.

417
SECTION IV Inf ammation, Immunomodulation, and Hematopoiesis

T e anemia characteristic o severe iron de ciency is microcytic, hypochromic


anemia. Copper de ciency also results in microcytic anemia because o the e ects o
copper de ciency on iron metabolism and heme production. Pyridoxine de ciency
results in a characteristic sideroblastic anemia.

QUESTION 25-5 Answer is d. Romiplostim contains 4 copies o a small peptide


gra ed onto an immunoglobulin sca old that binds with high a nity and activates
the thrombopoietin receptor on megakaryocytes. It is approved or use in patients with
immune (idiopathic) thrombocytopenic purpura (I P) who have ailed to respond to
more conventional treatments. Romiplostim was ound sa e and e cacious in
2 randomized controlled studies in patients with I P. Overall, ~84% o patients
responded to the drug with substantial increases in platelet levels, o which approxi-
mately hal were durable (platelets >50,000/L or 6 o the last 8 weeks o study). T e
drug is administered weekly by subcutaneous injection, starting with a dose o 1 g/kg,
titrated to a maximum o 10 g/kg, until platelet count increases above 50,000/L.
Oprelvekin is recombinant IL-11 that increases peripheral platelet counts through
its e ects on the IL-11 receptor (see Figure 25-1 and able 25-1). IL-11 acts synergisti-
cally with thrombopoietin.

SUMMARY: HEMATOPOIETIC AGENTS


TOXICITIES
UNIQUE; CLINICALLY
CLASS AND SUBCLASSES NAMES CLINICAL USES COMMON IMPORTANT
Erythropoiesis-Stimulating Epoetin al a Treatment o anemia, especially Aggravation o Thromboembolism
Agents (ESAs) Darbepoetin al a in patients with chronic hypertension (20-30% o Increased risk o cancer
renal ailure, but also anemia patients) recurrence in patients with
associated with surgery, Iron de ciency (absolute cancer- or chemotherapy-
AIDS, cancer chemotherapy, or unctional); virtually induced anemia
prematurity, and certain chronic all patients will require Seizures with epoetin al a in
in ammatory diseases supplemental iron ~2.5% o patients on dialysis
during rst 90 d o therapy

Myeloid Growth Factors GM-CSF Stimulate myelopoiesis in Higher doses are An acute reaction to rst dose
(sargramostim) patients undergoing autologous associated with bone in sensitive patients includes
bone marrow transplant, and pain, malaise, ulike ushing, hypotension, nausea,
in patients receiving intensive symptoms, ever, vomiting, dyspnea, all in
cancer chemotherapy, and some diarrhea, dyspnea, and arterial PO2
patients with cyclic neutropenia, rash Capillary leak syndrome with
myelodysplasia, aplastic anemia, prolonged administration in
or AIDS-associated neutropenia some patients
Transient supraventricular
arrhythmia, dyspnea, elevation
o serum creatinine, bilirubin,
and liver enzymes

G-CSF ( lgrastim) Stimulation o neutrophil Mild to moderate bone Rare cutaneous necrotizing
Pegylated production (and phagocytic pain with high doses vasculitis
recombinant and cytotoxic unctions over long periods Patients with hypersensitivity
G-CSF o neutrophils) in patients Local skin reactions at to proteins produced in
(peg lgrastim) with severe neutropenia site o subcutaneous Escherichia coli should not
a ter autologous stem cell injection receive the drug
transplantation, high-dose Marked granulocytosis in
cancer chemotherapy, severe patients receiving drug
congenital neutropenia, and over prolonged periods,
other neutropenia but not associated with
Used in peripheral blood stem increased morbidity or
cell (PBSC) collection to promote mortality
release o CD34+ progenitor cells Mild to moderate
splenomegaly with long-
term therapy

418
Hematopoietic Agents CHAPTER 2 5

TOXICITIES
UNIQUE; CLINICALLY
CLASS AND SUBCLASSES NAMES CLINICAL USES COMMON IMPORTANT
Thrombopoietic Growth IL-11 (oprelvekin) Used in patients with Fluid retention and
Factors chemotherapy-induced associated cardiac
thrombocytopenia to increase symptoms, such as
platelet production tachycardia, palpitation,
edema, and shortness
o breath; in elderly
patients it o ten requires
concomitant therapy
with diuretics
Blurred vision, injection-
site rash or erythema,
and paresthesia
Romiplostim Approved or use in patients
Eltrombopag with immune thrombocytopenic
purpura (ITP) who have ailed to
respond to more conventional
treatments
Iron SupplementsOral Ferrous sul ate Treatment o iron de ciency Heartburn, nausea, upper Severe iron poisoning via
Ferrous umarate anemia due to dietary iron gastric discom ort, and accidental ingestion by children
Ferrous succinate, de ciency, blood loss, and as an diarrhea or constipation can be atal; symptoms include
aspartate, and adjunct in patients treated with abdominal pain, diarrhea, or
other errous salts erythropoietin vomiting o brown or bloody
Ferrous gluconate Prophylactic use o oral iron stomach contents containing
Polysaccharide- should be reserved or patients pills; o particular concern are
errihydrite at high risk, including pregnant pallor or cyanosis, lassitude,
complex women, women with excessive drowsiness, hyperventilation
menstrual blood loss, and in ants due to acidosis, and
cardiovascular collapse

Iron Supplements Iron dextran Used when oral iron therapy ails Reactions to intravenous Parenteral iron therapy
Parenteral Sodium erric Common indications are iron iron include headache, should be used only when
gluconate malabsorption (eg, sprue, short malaise, ever, generalized clearly indicated because
Ferumoxytol bowel syndrome), severe oral lymphadenopathy, acute hypersensitivity,
Iron sucrose iron intolerance, as a routine arthralgia, urticaria, including anaphylactic and
supplement to total parenteral and exacerbation o anaphylactoid reactions, can
nutrition, and in patients who rheumatoid arthritis in occur in 0.2-3% o patients
are receiving erythropoietin, some patients
especially in hemodialysis
patients
Copper Supplement Cupric sul ate Supplementation (beginning
with a therapeutic trial) in
patients when a low plasma
copper concentration is
determined in the presence o
leukopenia and anemia
VitaminsPyridoxine Pyridoxine Pyridoxine is used to improve
Ribo avin hematopoiesis in patients with
either hereditary or acquired
sideroblastic anemia (ie,
sideroblastic anemia associated
with the antituberculosis drugs
isoniazid and pyrazinamide)
Ribo avin used in the nutritional
management o patients with
gross, generalized malnutrition to
treat or prevent red cell aplasia

419
SECTION IV Inf ammation, Immunomodulation, and Hematopoiesis

TOXICITIES
UNIQUE; CLINICALLY
CLASS AND SUBCLASSES NAMES CLINICAL USES COMMON IMPORTANT
VitaminsVitamin B12 Cyanocobalamin Treatment o vitamin B12 Once begun, vitamin Cyanocobalamin should never
(vitamin B12) de ciency; de ciency can B12 therapy must be be given intravenously due
Hydroxocobalamin mani est as severe megaloblastic maintained or li e; to rare reports o transitory
(vitamin B12) anemia with neurological it is important to exanthema and anaphylaxis
symptoms including paresthesia monitor vitamin B12 a ter injection; i a patient
o the hands and eet, decreased concentrations in plasma reports a previous sensitivity
vibration and position senses and to obtain peripheral to injections o vitamin B12, an
with resultant unsteadiness, blood counts at intervals intradermal skin test should
decreased deep tendon o 3-6 mo to con rm the be per ormed be ore the ull
re exes, and in the later stages, adequacy o therapy and dose is administered
con usion, moodiness, loss o monitor development o
memory, loss o central vision, re ractoriness to therapy
hallucinations, and overt
psychosis (see Side Bars CAUSES
OF VITAMIN B12 DEFICIENCYand
GENERAL PRNCIPLES OF VITAMIN
B12 THERAPY)

VitaminsFolic Acid and Folic acid Treatment or prevention o olate The megaloblastic Rare reports o reactions
Derivatives (pteroylglutamic de ciency which can cause anemia that results rom to parenteral injections o
acid [PteGlu], olate-de cient megaloblastic olate de ciency cannot olic acid and leucovorin;
l -methyl olate) erythropoiesis and elevated be distinguished rom caution should be exercised
Folinic acid plasma homocysteine that caused by vitamin in patients with a history o a
(leucovorin Prevention o neural tube B12 de ciency (see reaction
calcium, de ects ( olate supplements may Figure 25-3); however, Folic acid in large amounts
citrovorum actor) be required early in pregnancy) olate de ciency is may counteract the
Folinic acid is used to circumvent rarely, i ever, associated antiepileptic e ect o
the inhibition o dihydro olate with neurological phenobarbital, phenytoin,
reductase as a part o high-dose abnormalities; thus and primidone, and increase
methotrexate therapy and to the observation the requency o seizures in
potentiate uorouracil in the o characteristic susceptible children
treatment o colorectal cancer abnormalities in
and as an antidote to counteract vibratory and position
the toxicity o olate antagonists sense and in motor and
such as pyrimethamine or sensory pathways is
trimethoprim incompatible with an
(see Side Bars COMMON CAUSES isolated de ciency o
OF FOLATE DEFICIENCYand olic acid
GENERAL PRINCIPLES OF FOLIC Folic acid and leucovorin
ACID THERAPY) should never be used
or the treatment o
pernicious anemia or
other megaloblastic
anemia secondary to
a de ciency o vitamin
B12 because their
use can result in an
apparent response o the
hematopoietic system,
but neurological damage
may occur or progress i
already present

420
SECTION

Hormones and Hormone


Antagonists V
26. Introduction to Endocrinology: T e HypothalamicPituitary Axis 422

27. T yroid and Antithyroid Drugs 431

28. Estrogens, Progestins, Contraception, and Androgens 441

29. AC H, Adrenal Steroids, and Pharmacology o the Adrenal Cortex 459

30. Endocrine Pancreas and Pharmacotherapy


o Diabetes Mellitus and Hypoglycemia 470

31. Drug T erapy o Mineral Ion Homeostasis and Bone urnover Disorders 480

421
CHAPTER

26 Introduction to Endocrinology: The


HypothalamicPituitaryAxis
T is chapter will be most use ul a er having a basic understanding o the material
DRUGS INCLUDED
in Chapter 38, Introduction to Endocrinology: T e Hypothalamic Pituitary Axis in
IN THIS CHAPTER Goodman & Gilmans T e Pharmacological Basis of T erapeutics, 12th Edition In addi-
Bromocriptine (PARLODEL, CYLOSERT,others) tion to the material presented here, the 12th Edition includes:
Cabergoline (CABERLIN, DOSTINEX,CABASER) A detailed discussion o the hypothalamic-pituitary-endocrine axis
Cetrorelix(CETROTIDE) A description o the pituitary hormones and their releasing actors
Choriogonadotropin alfa (OVIDREL) A discussion o the therapy o growth hormone (GH) de ciency
Follitropin (GONAL-F) A discussion o the glycoprotein hormones: thyroid stimulating hormone ( SH) and
Follitropin (FOLLISTIM, PUREGON) gonadotropins
Ganirelix(ANTAGON) A discussion o the posterior pituitary hormones: oxytocin and vasopressin
Gonadorelin (FACTREL, LUTREPULSE)
LEARNING OBJECTIVES
Goserelin (ZOLADEX)
Understand the unctioning o the hypothalamic-pituitary axis
Histrelin (VANTAS, SUPPRELINLA)
Describe the pharmacotherapy o GH excess and GH de ciency
Lanreotide (SOMATULINELA, SOMATULINE
DEPOT) Develop knowledge o the clinical uses o gonadotropin-releasing hormone
(GnRH) and its analogs
Leuprolide (LUPRON, ELIGARD)
Lutropin alfa (LUVERIS, LHADI)
MECHANISMS OF ACTION OF DRUGS THAT ACT ON THE HYPOTHALAMIC-
Mecasermin (INCRELEX) PITUITARY AXIS
Menotropins (REPRONEX)
DRUG CLASS DRUG MECHANISM OF ACTION
Nafarelin (SYNAREL)
Somatostatin (SST) Octreotide SST analog that inhibits the release o GH rom
Octreotide (SANDOSTATIN, SANDOSTATIN-LAR Analogs the pituitary (see Figures 26-1 and 26-2)
DEPOT)
Lanreotide SST analog that inhibits the release o GH rom
Oxytocin the pituitary (see Figures 26-1 and 26-2)
Pegvisomant (SOMAVERT)
Growth Hormone (GH) Pegvisomant GH receptor antagonist (see Figure 26-3)
Pergolide (PERMAX) Antagonist
Quinagolide (NORPROLAC)
Dopamine (DA) Quinagolide DA receptor agonist; inhibits release o
Recombinant human growth hormon- Receptor Agonists prolactin (see Figure 26-4)
eGH(ACCRETRTROPIN, GENOTROPIN,
HUMATROPE, NORDITROPIN, NUTROPIN, Bromocriptine DA receptor agonist; inhibits release o
OMNITROPE, SAIZEN, SERPSTIM,TEV- prolactin (see Figure 26-4)
TROPIN, ZORBTIVE, NUTROPINEDEPOT) Cabergoline DA receptor agonist; inhibits release o
Triptorelin (TRELSTARDEPOTLA) prolactin (see Figure 26-4)
Urine-derived human chorionicgonadotropin Recombinant GH Recombinant GH Binds to GH receptor (see Figure 26-3)
(hCG; NOVAREL, PREGNYL, PROFASI)
Insulin-like Growth Mecasermin Mimics e ects o GH (see Figure 26-2)
Urofollitropin (uFSH; BRAVELLE, MENOPUR) Factor-1 (IGF-1)

Gonadotropin- GnRH Signals through a speci c GPCR on


Releasing Hormone gonadotropic cells in the anterior pituitary to
(GnRH) Agonists release ollicle-stimulating hormone (FSH) and
luteinizing hormone (LH) (see Figure 26-5)

Goserelin GnRH agonist

Na erelin GnRH agonist

Triptorelin GnRH agonist


(Continued)
422
Introduction to Endocrinology: The HypothalamicPituitary Axis CHAPTER 2 6

DRUG CLASS DRUG MECHANISM OF ACTION


Histrelin GnRH agonist

Leuprolide GnRH agonist

GnRH Antagonists Cetrorelix Antagonizes the e ect o GnRH on


gonadotropic cells

Ganirelix Antagonizes the e ect o GnRH on


gonadotropic cells

Follicle-Stimulating Menotropin FSH preparation that acts on the FSH receptor


Hormone (FSH)

Uro ollitropin (uFSH) FSH preparation that acts the FSH receptor

Follitropin Recombinant FSH that acts on the FSH


receptor

Follitropin Recombinant FSH that acts on the FSH


receptor

Human Chorionic Choriogonadotropin Recombinant hCG that acts on the LH receptor


Gonadotropin (hCG) al a

LuteinizingHormone Lutropin al a Recombinant LH that acts on the LH receptor


(LH)

Posterior Pituitary Oxytocin Acts via speci c GPCR to enhance voltage-


Hormones sensitive Ca2+ channels

Arginine vasopressin See Chapter 15

CASE 26-1
A 43-year-old man o normal height has been diagnosed with acromegaly
a. What is acromegaly and why is this patients height normal?
Acromegaly is the result o excess secretion o GH rom an anterior pituitary
adenoma. T e organization o the posterior and anterior pituitary glands is shown
in Figure 26-1. In adults, the signs and symptoms o acromegaly are arthropathy,
carpal tunnel syndrome, visceromegaly, macroglossia, hypertension, glucose intol-
erance, headache, lethargy, excess perspiration, and sleep apnea. I the epiphyses
are unclosed (presumably the epiphyses are closed in this patient), the GH excess
results in increased longitudinal growth and gigantism.
b. How is GH secretion regulated?
Growth hormonereleasing hormone (GHRH) and somatostatin (SS ), released
rom the hypothalamus, stimulate or inhibit the release o GH rom the pituitary,
respectively (see Figure 26-2). Insulin-like growth actor (IGF-1), a product o GH
action on peripheral tissues, causes negative eedback inhibition o GH release at
the level o the hypothalamus and the pituitary.
c. How does GH cause its myriad o ef ects on various tissues?
GH and prolactin act on speci c receptors in target tissues. T e GH receptor is a
widely distributed cell sur ace receptor that consists o an extracellular hormone-
binding domain, a single membrane-spanning region, and an intracellular domain
that mediates signal transduction (see Figure 26-3).
d. What are the options or treatment o acromegaly in this patient?
reatment options include transphenoidal surgery, radiation, and drugs that inhibit
GH secretion. Increased attention has been given to pharmacological treatment
o acromegaly either as primary treatment or or the treatment o persistent GH
(Continued)
423
SECTION V Hormones and Hormone Antagonists

S ON, PVN PVN


(AVP, OXY) (TRH, CRH, S ST)

Hypo thalamus
ARC
(GHRH, GnRH,
DA)

Po s te rio r
lo be +

Hypo thalamus
Ante rio r Re le a s ing GHRH S ST
fa ctors
lo be

Porta l +
sys te m
Ante rio r Growth
AVP, Ghre lin
pituitary hormone +
OXY
Trophic hormone s
(ACTH, TS H, GH, S toma ch
LH, FS H, prola ctin)

Targ e t Live r Bone Adipocyte Mus cle


tis s ue s

Kidney, ute rus,


ma mma ry gla nd IGF-1

FIGURE 26-1 Organization o the anterior and posterior pituitary gland. S e c o ndary
Hypothalamic neurons in the supraoptic (SON) and paraventricular (PVN) targ e t tis s ue s
nuclei synthesize arginine vasopressin (AVP) or oxytocin (OXY). Most o their
axons project directly to the posterior pituitary, rom which AVP and OXY FIGURE 26-2 Growth hormone secretion and actions. Two
are secreted into the systemic circulation to regulate their target tissues. hypothalamic actors, growth hormonereleasing hormone
Neurons that regulate the anterior lobe cluster in the mediobasal hypo- (GHRH) and somatostatin (SST) stimulate or inhibit the release
thalamus, including the PVN and the arcuate (ARC) nuclei. They secrete o growth hormone (GH) rom the pituitary, respectively.
hypothalamic-releasing hormones, which reach the anterior pituitary via Insulin-like growth actor-1 (IGF-1), a product o GH action on
the hypothalamic-adenohypophyseal portal system and stimulate distinct peripheral tissues, causes negative eedback inhibition o GH
populations o pituitary cells. These cells, in turn, secrete the trophic (sig- release by acting at the hypothalamus and the pituitary. The
nal) hormones, which regulate endocrine organs and other tissues. ACTH, actions o GH can be direct or indirect (mediated by IGF-1).
Adrenocorticotrophic hormone; CRH, corticotropin-releasing hormone; DA, Inhibition, ; stimulation, +.
dopamine; FSH, ollicle-stimulating hormone; GH, growth hormone; GHRH,
growth hormone-releasing hormone; GnRH, gonadotropin-releasing
hormone; LH, luteinizing hormone; TRH, thyrotropin-releasing hormone;
TSH, thyroid-stimulating hormone.

excess a er transphenoidal surgery. Octreotide and lanreotide are 2 SS analogs


that are used widely to treat acromegaly. Both are available as long-acting ormula-
tions that require an intramuscular injection once every 4 weeks.
Pegvisomant is a GH receptor antagonist (see Figure 26-3) that is approved or
the treatment o acromegaly. Pegvisomant binds to the GH receptor, but does not
initiate signaling or stimulate IGF-1 secretion.
e. What adverse ef ects should this patient be warned about as a result o this treatment?
Diarrhea, nausea, and abdominal pain are common with octreotide and lanreotide but
may diminish over time. Patients receiving these drugs may develop gallbladder sludge
or gallstones. Inhibitory e ects on thyroid-stimulating hormone ( SH) secretion may
lead to hypothyroidism and thyroid unction tests should be monitored periodically.
Pegvisomant should not be used in patients with elevated hepatic transaminases
and hepatic unction should be assessed in all patients. Loss o negative eedback o
GH and IGF-1 may increase growth o GH-secreting adenomas.

424
Introduction to Endocrinology: The HypothalamicPituitary Axis CHAPTER 2 6

A GH B GH Hypo thalamus
TRH Dopa mine
GH Pe gvis oma nt

G G G G G G Ante rio r
H H H H H H Prola ctin
pituitary
R R R R R R
J J S uc kling
A A
K K
2 2
No GH
RS -1 s igna ling
S HC STAT5 Targ e t Othe r
Bre a s t
tis s ue tis s ue s
P 3K
FIGURE 26-4 Prolactin secretion and
MAP K
Ge ne expre s s ion actions. Prolactin is the only anterior pituitary
(e g, GF-1) hormone or which a unique stimulatory
releasing actor has not been identi ed.
Thyrotropin-releasing hormone (TRH),
Glucos e Nucle us however, can stimulate prolactin release and
tra ns porte r dopamine can inhibit it. Suckling induces
prolactin secretion, and prolactin a ects
lactation and reproductive unctions but
also has varied e ects on many other tissues.
FIGURE 26-3 Mechanisms o growth hormone and prolactin action and o GH receptor Prolactin is not under eedback control by
antagonism. Left (A): The binding o GH to a homodimer o the GH receptor (GHR) induces peripheral hormones.
autophosphorylation o JAK2. JAK2 then phosphorylates cytoplasmic proteins that activate
downstream signaling pathways, including STAT5 and mediators upstream o MAPK, which
ultimately modulate gene expression. The structurally related prolactin receptor also is a ligand-
activated homodimer that recruits the JAK-STAT signaling pathway. The GHR also activates IRS-1,
which may mediate the increased expression o glucose transporters on the plasma membrane.
The diagram does not re ect the localization o the intracellular molecules, which presum-
ably exist in multicomponent signaling complexes. JAK2, janus kinase 2; IRS-1, insulin receptor
substrate-1; PI3K, phosphatidyl inositol-3 kinase; STAT, signal transducer and activator o tran-
scription; MAPK, mitogen-activated protein kinase; SHC, Src homology containing. Right (B):
Pegvisomant, a recombinant pegylated variant o human GH, contains amino acid substitutions
that increase the af nity or 1 site o the GHR but do not activate its downstream signaling cascade.
It thus inter eres with GH signaling in target tissues.

CASE 26-2
A 56-year-old man with advanced prostate cancer is being treated with leuprolide
a. What is leuprolide and why is it used to treat prostate cancer?
Leuprolide is a synthetic gonadotropin-releasing hormone (GnRH) agonist that
has greater receptor a nity, reduced enzymatic degradation, and is more potent
than the naturally occurring GnRH. Endogenous GnRH is released rom the
hypothalamus and acts on the pituitary to release luteinizing hormone (LH) and
ollicle-stimulating hormone (FSH). LH acts on the testes to promote the secretion
o androgens (see Figures 26-1 and 26-5).
Androgens stimulate the growth o normal and cancerous prostate cells (see
Chapter 46). Standard therapy or advanced prostate cancer is surgical or
pharmacological androgen deprivation therapy (see Chapter 46).
b. How does a GnRH agonist suppress androgen secretion?
T e GnRH agonists bind to GnRH receptors on pituitary gonadotropin-producing
cells, causing an initial release o LH and FSH and a subsequent increase in
testosterone production rom testicular Leydig cells. A er approximately 1 week o
therapy, GnRH receptors are downregulated on the gonadotropin-producing cells,
causing a decline in the pituitary response and a subsequent decline in testosterone
production.
(Continued)

425
SECTION V Hormones and Hormone Antagonists

Hypo thalamus c. What untoward ef ects might this patient expect as a result o the leuprolide
GnRH
therapy?
He should be told to expect hot f ashes, possible decreased bone density, and erectile
Ante rio r dys unction. T ere is an apparent increase in the incidence o pituitary apoplexy, a
LH
pituitary syndrome o headache, neurological mani estations, and impaired pituitary unction;
FS H these e ects mimic ones that usually result rom an in arction o a pituitary adenoma.

CASE 26-3
Targ e t
Gona ds
A 32-year-old woman is diagnosed with a prolactin-secreting pituitary adenoma
tis s ue
a. What are the therapeutic options or this patient?
T e therapeutic options or patients with prolactinomas include transphenoidal
surgery, radiation, and treatment with dopamine (DA) receptor agonists. T e DA
receptor agonists are the treatment o choice since the surgical success rate is 75%
or microadenomas and 33% or macroadenomas.
Targ e t S ex
tis s ue Inhibin
s te roids b. Why are the DA receptor agonists ef ective?
pro duc t
T yrotropin-releasing actor ( RH) stimulates prolactin release whereas DA released
rom the hypothalamus inhibits prolactin release (see Figure 26-4). T e DA recep-
tor agonists decrease both prolactin secretion and the size o the adenoma, thereby
S e c o ndary Acce s s ory Othe r improving the endocrine abnormalities and the neurological symptoms (including
targ e t s ex orga ns tis s ue s visual eld de ects).
tis s ue s
c. What DA receptor agonists are used to treat hyperprolactinemia?
FIGURE 26-5 The hypothalamic-pituitary-
gonadal axis. A single hypothalamic- T e drugs in this class include bromocriptine, cabergoline, and quinagolide. Bro-
releasing actor, gonadotropin-releasing mocriptine normalizes serum prolactin concentrations in 70% to 80% o patients
hormone (GnRH), controls the synthesis and decreases tumor size in more than 50% o patients. ypically, hyperprolac-
and release o both gonadotropins (LH tinemia and tumor growth recur upon cessation o therapy.
and FSH) in males and emales. Gonadal
Cabergoline has a higher a nity and greater selectivity or the D2 receptor than
steroid hormones (androgens, estrogens,
and progesterone) exert eedback inhibi- bromocriptine. It is becoming the pre erred drug in this setting. Cabergoline may
tion at the level o the pituitary and the induce remission in a signi cant number o patients with prolactinomas.
hypothalamus. The preovulatory surge Quinagolide is not approved by the FDA but has been used extensively in Europe
o estrogen also can exert a stimulatory and Canada.
e ect at the level o the pituitary and
the hypothalamus. Inhibins, a amily o d. What adverse ef ects might be expected rom therapy with bromocriptine or
polypeptide hormones produced by the cabergoline?
gonads, speci cally inhibit FSH secretion
by the pituitary. Bromocriptine requently causes nausea, vomiting, headache, and postural hypo-
tension, particularly on initial use. Less requently, bromocriptine may cause nasal
congestion, digital vasospasm, and CNS e ects such as psychoses, hallucinations,
nightmares, or insomnia.
Cabergoline causes much less nausea than bromocriptine, but may still cause diz-
ziness and hypotension. Cabergoline has been linked to valvular heart disease and
echocardiographic assessment is appropriate or patients receiving chronic therapy.

KEY CONCEPTS
T e peptide hormones o the anterior pituitary are essential or the regulation
o growth and development, reproduction, response to stress, and intermediary
metabolism
T e synthesis and secretion o the pituitary hormones are controlled by hypo-
thalamic hormones and by hormones rom the peripheral endocrine organs (see
Figure 26-1)
T e most striking physiological e ect o GH is the stimulation o the longitudinal
growth o bones (see Figure 26-2)
(Continued)

426
Introduction to Endocrinology: The HypothalamicPituitary Axis CHAPTER 2 6

GH excess is treated with the SS analogs octreotide and lanreotide


GH de ciency is treated with recombinant human GH
Analogs o GnRH are used predominantly to treat advanced prostate cancer
(see Chapter 46)

SUMMARY QUIZ

QUESTION 26-1 A 78-year-old woman has received GH injections rom an antiaging


clinic She is likely to experience which o the ollowing e ects?
a Growth o the long bones in her legs
b Carpal tunnel syndrome
c Blurred vision
d Decreased hearing
e A metallic taste

QUESTION 26-2 A 6-year-old boy is diagnosed with central or GnRH-dependent


precocious puberty He is being treated with na arelin, a GnRH analog, as a nasal spray
GnRH analogs are e ective in precocious puberty because they
a downregulate GnRh receptors on pituitary gonadotropes
b block the action o testosterone on muscle cells
c stimulate testicular estrogen synthesis
d antagonize the e ect o luteinizing hormone (LH)
e antagonize the e ect o GnRH release rom the hypothalamus

QUESTION 26-3 A 20-year-old male cyclist is suspected o using per ormance-enhanc-


ing drugs Although his urine testosterone concentration is elevated, his testoster-
one/epitestosterone ( /E) (a measure o exogenous testosterone administration) ratio
is normal He has achieved this state by the use o which o the ollowing?
a Estrogen
b Progestin
c Human chorionic gonadotropin (hCG)
d Low dose testosterone
e Growth hormone (GH)

QUESTION 26-4 A 4-year-old boy with short stature is diagnosed with growth
hormone (GH) de ciency His GH replacement therapy should be continued until
a he is in the 90th percentile on growth charts
b he is in the 60th percentile on growth charts
c his epiphyses are used
d he begins to develop secondary sex characteristics
e he has permanent teeth

QUESTION 26-5 A 22-year-old man is determined to have impaired ertility secondary


to gonadotropin de ciency He is being treated with recombinant FSH T e most common
side e ect with gonadotropin therapy is
a hirsutism
b alopecia
c erectile dys unction
d dry skin
e gynecomastia

427
SECTION V Hormones and Hormone Antagonists

SUMMARY QUIZ ANSWER KEY


QUESTION 26-1 Answer is b. Side e ects associated with the initiation o GH therapy
in adults include peripheral edema, carpal tunnel syndrome, arthralgia, or myalgia,
which occur most requently in patients who are older or obese
QUESTION 26-2 Answer is a. T e intermittent release o GnRH is crucial or the
proper synthesis and release o the gonadotropins; the continuous administration o
GnRH leads to desensitization and downregulation o GnRH receptors on pituitary
gonadotropes and orms the basis or the clinical use o long-acting GnRH agonists to
suppress gonadotropin secretion
QUESTION 26-3 Answer is c. T e administration o hCG can stimulate testosterone
secretion in males with normal Leydig cell unction Epitestosterone is an inactive
epimer o testosterone secreted rom the testes T e normal /E ratio is approximately
one A ratio much larger than this would suggest the exogenous administration o
testosterone hCG administration would stimulate endogenous testosterone secretion
and thus the /E ratio would be close to one
QUESTION 26-4 Answer is c. Although the most pronounced increase in growth
occurs during the rst 2 years o therapy, GH is continued until the epiphyses are used
and may be extended into the transition period rom childhood to adulthood
QUESTION 26-5 Answer is e. T e most common side e ect o gonadotropin therapy
in males is gynecomastia, which occurs in up to a third o patients and presumably
ref ects increased production o estrogens due to the induction o aromatase

SUMMARY: DRUGS THAT ACT ON HYPOTHALAMICPITUITARYAXIS


TOXICITIES
CLASS AND
SUBCLASSES NAMES CLINICAL USES COMMON UNIQUE; CLINICALLY IMPORTANT
Somatostatin (SST) Octreotide Treatment o acromegaly Diarrhea, nausea, Gallbladder sludge
Analogs abdominal pain Hypothyroidism because o
inhibition o TSH secretion

Lanreotide Treatment o acromegaly Diarrhea, nausea, Gallbladder sludge


abdominal pain Hypothyroidism because o
inhibition o TSH secretion

Growth Hormone Pegvisomant Treatment o acromegaly Lipohypertrophy at Should not be used in patients with
(GH) Antagonist injection site elevated hepatic transaminases

Dopamine (DA) Quinagolide Treatment o


Receptor Agonists hyperprolactinemia (Not
approved by FDA, but
available in Europe and
Canada)

Bromocriptine Treatment o Nausea, vomiting, Postural hypotension


hyperprolactinemia headache

Cabergoline Treatment o Less nausea than Postural hypotension


hyperprolactinemia bromocriptine Linked with valvular heart disease

Recombinant GH Recombinant GH Treatment o GH Peripheral edema, Should not be used in patients with
de ciency carpal tunnel syndrome, neoplasia, proli erative retinopathy,
arthralgia, myalgia or acute respiratory ailure

Insulin-like Growth Mecasermin Treatment o impaired Hypoglycemia, Should not be used in patients with
Factor-1 (IGF-1) growth due to mutations lipohypertrophy closed epiphyses or in patients with
in GH receptor or neoplasia
postreceptor pathway

(Continued)
428
Introduction to Endocrinology: The HypothalamicPituitary Axis CHAPTER 2 6

TOXICITIES
CLASS AND
SUBCLASSES NAMES CLINICAL USES COMMON UNIQUE; CLINICALLY IMPORTANT
Gonadotropin- GnRH Problems with availability
Releasing Hormone have limited its use
(GnRH) Agonist

Goserelin Approved or use in Hot ashes Syndrome mimicking pituitary


endometriosis, prostate Decreased bone adenoma in arction, including
and breast cancer density in both sexes headache, impaired pituitary unction
Vaginal dryness and and neurological mani estation
atrophy in women, erectile Contraindicated in pregnant women
dys unction in men

Na erelin Approved or use in Hot ashes Syndrome mimicking pituitary


endometriosis and central Decreased bone adenoma in arction including
precocious puberty density in both sexes headache, impaired pituitary unction
Vaginal dryness and and neurological mani estation
atrophy in women, erectile Contraindicated in pregnant women
dys unction in men

Triptorelin Approved or advanced Hot ashes Syndrome mimicking pituitary


prostate cancer Decreased bone adenoma in arction, including
density in both sexes; headache, impaired pituitary
Vaginal dryness and unction and neurological
atrophy in women, erectile mani estation
dys unction in men Contraindicated in pregnant women

Histrelin Approved or advanced Hot ashes Syndrome mimicking pituitary


prostate cancer and Decreased bone density in adenoma in arction, including
central precocious puberty both sexes headache, impaired pituitary
Vaginal dryness and unction and neurological
atrophy in women, erectile mani estation
dys unction in men Contraindicated in pregnant women

Leuprolide Approved or use in Hot ashes Syndrome mimicking pituitary


endometriosis, uterine Decreased bone adenoma in arction, including
broids, prostate cancer, density in both sexes headache, impaired pituitary
and central precocious Vaginal dryness and unction and neurological
puberty atrophy in women, erectile mani estation
dys unction in men Contraindicated in pregnant women

GnRH Antagonists Cetrorelix Prevention o LH surge Hypersensitivity reactions, including


and premature ovulation anaphylaxis
in ovarian-stimulation Contraindicated in pregnant
protocols (see Chapter 28) women

Ganirelix Prevention o LH surge Hyper-sensitivity reactions,


and premature ovulation including anaphylaxis
in ovarian-stimulation Contraindicated in pregnant
protocols (see Chapter 28) women

Follicle-Stimulating Menotropin Ovarian stimulation


Hormone (FSH) or in vitro ertilization
Development o tests
or the diagnosis o
reproductive disorders

Uro ollitropin Ovarian stimulation


or in vitro ertilization
Development o tests
or the diagnosis o
reproductive disorders
(Continued)

429
SECTION V Hormones and Hormone Antagonists

TOXICITIES
CLASS AND
SUBCLASSES NAMES CLINICAL USES COMMON UNIQUE; CLINICALLY IMPORTANT
Follitropin Ovarian stimulation
or in vitro ertilization
Development o tests
or the diagnosis o
reproductive disorders

Follitropin Ovarian stimulation


or in vitro ertilization
Development o tests
or the diagnosis o
reproductive disorders

Human Chorionic Choriogonadotropin al a Treatment o male Gynecomastia


Gonadotropin (hCG) in ertility and
cryptorchidism

Luteinizing Hormone Lutropin al a Development o tests o


LH surge to determine
ovulation
Diagnosis o reproductive
disorders

Posterior Pituitary Oxytocin Used to induce or


Hormones augment labor or
to prevent post-
partum hemorrhage
(see Chapter 28)

430
CHAPTER

Thyroid and Antithyroid Drugs 27


T is chapter will be most use ul a er having a basic understanding o the material in
DRUGS INCLUDED
Chapter 39, T yroid and Antithyroid Drugs in Goodman & Gilmans T e Pharmacological
Basis of T erapeutics, 12th Edition. In addition to the material presented here, the IN THIS CHAPTER
12th Edition contains: Carbimazole (NEO-MERCAZOLE)
A discussion o the chemistry and biosynthesis o the thyroid hormones. Desiccated thyroid (ARMORTHYROID,others)
Figure 39-1 T yronine, thyroid hormones, and precursors, shows the chemical struc- Iodide
tures o the thyroid hormones Levothyroxine (l -T4, LEVOTHROID, LEVOXYL,
A detailed discussion o thyroid hormone metabolism and the conversion o thyroxine SYNTHROID, UNITHROID, others)
to triiodothyronine in peripheral tissues Liotrix(a thyroxine/triiodothyronine 4:1
A discussion o the actions o thyroid hormone including the non-genomic e ects o mixture; THYROLAR)
thyroid hormones, the e ects o thyroid hormone metabolites, the e ects o thyroid Methimazole (TAPAZOLE, others)
hormones on growth and development, and the thermogenic, cardiovascular, and
Propylthiouracil
metabolic e ects o thyroid hormones
Sodiumiodine (IODOPEN)
able 39-5 Selected Pharmacokinetic Features o Anti-T yroid Drugs, shows selected
pharmacokinetic eatures o propylthiouracil and methimazole Sodiumiodine 131I(HICON, others)
Triiodothyroxine (CYTOMEL,TRIOSTAT,others)
LEARNING OBJECTIVES
Understand the principles o thyroid hormone regulation.
Describe the diagnosis and treatment o hypothyroidism and hyperthyroidism,
including during pregnancy.
Describe the treatment options or well-di erentiated thyroid cancer.

MECHANISMS OF ACTION OF THYROID AND ANTITHYROID DRUGS


DRUG CLASS DRUG MECHANISM OF ACTION
Thyroid Hormone Levothyroxine Binding o 3,5,3-triiodothyronine T3 to thyroid hormone receptors (members o the
Triiodothyroxine nuclear receptor super amily o transcription actors) on target tissues
Liotrix
Desiccated thyroid

Antithyroid Drugs Propylthiouracil Inter eres with the incorporation o iodine into tyrosyl residues to orm
Methimazole iodothyronine (see Figure 27-1)
Carbimazole

Ionic Thyroid Inhibitors Iodide At high concentrations, iodide inhibits the synthesis o iodotyrosines and
Sodium iodine iodothyronines; high concentrations also inhibit the release o thyroid hormones

Radioactive Iodine Sodium iodine 131I Incorporated into iodoamines and deposited into the colloid o thyroid ollicles
where the destructive particles are released

THYROID FUNCTION TESTS


TSH (Thyroid-stimulating hormone) Elevated in hypothyroidism; decreased in
hyperthyroidism

T4 (Thyroxine) Elevated levels suggest hyperthyroidism;


decreased levels suggest hypothyroidism

T3 (Triiodothyronine) Elevated levels suggest hyperthyroidism;


decreased levels suggest hypothyroidism

431
SECTION V Hormones and Hormone Antagonists

SYMPTOMS OF THYROID DISEASE


HYPOFUNCTION HYPERFUNCTION
Fatigue ExophthalmusGraves disease
Lethargy Excessive production o heat
Cold intolerance Increased motor activity
Mental slowness Increased sensitivity to catecholamines
Depression Flushed skin
Dry skin Warm moist skin
Constipation Weak muscles
Mild weight gain Tremulousness
Fluid retention Rapid, orced heart rate
Muscle aches Increased appetite
Irregular menses Weight loss i intake is inadequate
In ertility Heat intolerance
Increased requency o bowel movements

CASE 27-1
A 69-year-old man goes to his amily doctor because he has been eeling atigued and
lethargic. His doctor does a complete evaluation. T is patient had a myocardial in arction
and has a recurrent ventricular arrhythmia. T e patients SH is elevated and his 4 is
slightly decreased. T e doctor suspects hypothyroidism and begins replacement therapy
with levothyroxine.
a. Why is the TSH elevated in this patient?
T e regulation o thyroid unction is shown in Figure 27-2. T yrotropin-releasing
hormone ( RH) is released rom the hypothalamus when circulating concen-
trations o thyroid hormone ( 4 and 3) are low. RH stimulates the release o
pre ormed SH granules rom the pituitary. T e released SH binds to a GPCR
receptor on the plasma membrane o thyroid cells and eventually all phases o thy-
roid hormone synthesis and release are stimulated (see Figure 27-2). When thyroid
hormone levels are low, the negative eedback inhibition on the hypothalamic RH
release is removed and SH secretion is increased.
b. What actors might af ect the levothyroxine dose in this patient?
able 27-1 lists some o the actors that may in uence the dose o levothyroxine.
T is patient has had a myocardial in arction and is being treated or a ventricular
arrhythmia with amiodarone (see Chapter 18). Amiodarone is an iodine-contain-
ing drug which may impair the conversion o 4 to 3.
c. What commonly prescribed drugs are associated with iodine-induced
hypothyroidism?
able 27-2 lists the iodine content o commonly prescribed drugs.
d. What is the goal o levothyroxine replacement therapy?
Levothyroxine is the hormone o choice or thyroid hormone replacement therapy.
With this therapy one relies on the type 1 and 2 deiodinases to convert 4 to 3 to
maintain a steady serum concentration o 3 (see Figure 27-1). T e goal o therapy
is to normalize the serum SH (in primary hypothyroidism) or ree 4 (in second-
ary hypothyroidism) and to relieve the symptoms o hypothyroidism.
(Continued)
432
Thyroid and Antithyroid Drugs CHAPTER 2 7

Ba s ola te ra l Apica l
me mbra ne me mbra ne
Colloid

Na + Na + Tg Tg
A NIS TP O TP O
I I + I + DIT
I I Pe ndrin H2 O 2 or MIT
Tg DIT H2 O 2
Na + De ha loge na s e I I HOI
MIT T4
ATPa s e or T3
K+ DIT EOI
E MIT B C
T4 T3 T4 D
T4 , T3
T3
D1, D2
F
KEY

METABOLIC STEP INHIBITOR


A Iodine tra ns port ClO 4 , S CN
B Iodina tion P TU, MMI
C Coupling P TU, MMI
D Colloid Re s orption Colchicine, Li+, I
E De iodina tion of Dinitrotyros ine
DIT + MIT
F De odina tion of T4 P TU
FIGURE 27-1 Major pathways o thyroid hormone biosynthesis and release. D1 and D2, deiodinases;
DIT, diiodotyrosine; EOI, enzyme-linked species; HOI, hypoiodous acid; MMI, methimazole; MIT,
monoiodotyrosine; PTU, propylthiouracil; Tg, thyroglobulin; TPO, thyroid peroxidase.

Hypotha la mus

S ST
TRH
+

Dopa mine
Glucocorticoids P ituita ry
Re tinoids


TS H

+

Hig h I Thyroid gla nd

T4 and T3

FIGURE 27-2 Regulation o thyroid hormone secretion. Myriad neural inputs in uence hypo-
thalamic secretion o thyrotropin-releasing hormone (TRH). TRH stimulates release o thyrotropin
(TSH, thyroid-stimulating hormone) rom the anterior pituitary; TSH stimulates the synthesis and
release o the thyroid hormones T3 and T4. T3 and T4 eed back to inhibit the synthesis and release
o TRH and TSH. Somatostatin (SST) can inhibit TRH action, as can dopamine and high concen-
trations o glucocorticoids. Low levels o I are required or thyroxine synthesis, but high levels
inhibit thyroxin synthesis and release.
(Continued)
433
SECTION V Hormones and Hormone Antagonists

TABLE 27-1 Factors Inf uencing Oral Levothyroxine Therapy TABLE 27-2 Commonly Used Iodine-Containing Drugs
Drugs and other factors that may increase levothyroxine DRUGS IODINE CONTENT
dosage requirements Oral or local
Impaired levothyroxine absorption
Aluminum-containing antacids Amiodarone 75 mg/tablet
Bile acid sequestrants (cholestyramine, colestipol, colesevelam)
Calcium carbonate (e ect generally small) Calcium iodide syrup 26 mg/mL
Chromium picolinate
Food Iodoquinol (diiodohydroxyquin) 134-416 mg/tablet
Iron salts
Echothiophate iodide ophthalmic solution 5-41 g/drop
Lactose intolerance (single case report)
Phosphate binders (lanthanum carbonate, sevelamer) Hydriodic acid syrup 13-15 mg/mL
Proton pump inhibitors
Raloxi ene Iodochlorhydroxyquin 104 mg/tablet
Soy products (e ect generally very small)
Sucral ate Iodine-containing vitamins 0.15 mg/tablet
Increased thyroxine metabolism, CYP3A4 induction of hepatic
Bexarotene Idoxuridine ophthalmic solution 18 g/drop
Carbamapzepine Kelp/seaweed 0.15 mg/tablet
Phenytoin
Ri ampin Lugols solution 6.3 mg/drop
Sertraline
Impaired T4 T3 conversion PONARIS nasal emollient 5 mg/0.8 mL
Amiodarone
Mechanisms uncertain or multifactorial KI, saturated solution (KISS) 38 mg/drop
Estrogen pregnancy
Ethionamide Topical antiseptics
Tyrosine kinase inhibitors (imatinib, sunitinib) Iodoquinol cream (diiodohydroxyquin) 6 mg/g
Lovastatin, simvastatin
Drugs and other factors that may decrease levothyroxine Iodine tincture 40 mg/mL
dosage requirements
Advancing age (>65 years) Iodochlorhydroxyquin cream 12 mg/g
Androgen therapy in women
Drugs that may decrease TSH without changing free T4 in Iodo orm gauze 4.8 mg/100 mg
levothyroxine treated patients gauze
Met ormin
Povidoneiodine 10 mg/mL

Radiology contrast agents

Diatrizoate meglumine sodium 370 mg/mL

Propyliodone 340 mg/mL

Iopanoic acid 333 mg/tablet

Ipodate 308 mg/capsule

Iothalamate 480 mg/mL

Metrizamide (undiluted) 483 mg/mL

Iohexol 463 mg/mL

Adapted rom Roti E, Cozani R, Braverman LE. Adverse e ects


o iodine on the thyroid. Endocrinologist, 1997, 7:245-254. With
permission. Copyright Lippincott Williams &Wilkins. http://
lww.com.

e. What are the adverse ef ects o levothyroxine therapy?


Adverse e ects o thyroid hormone would generally occur only upon overtreatment
and would be similar to the consequences o hyperthyroidism. An excess o thyroid
hormone can increase the risk o atrial brillation, especially in the elderly, and can
increase the risk o osteoporosis, especially in postmenopausal women.
434
Thyroid and Antithyroid Drugs CHAPTER 2 7

CASE 27-2
A 26-year-old rst trimester pregnant woman is diagnosed with hyperthyroidism.
a. What are the treatment options or this woman?
T yrotoxicosis occurs in approximately 0.2% o pregnancies and is caused by Graves
disease. Graves disease is an autoimmune disorder characterized by increased thyroid
hormone production, di use goiter, and immunoglobulin (Ig)G antibodies that
bind and activate the SH receptor. T e characteristic exophthalmos associated with
Graves disease is an in ltrative opthalmopathy. Antithyroid drugs are the treatment
o choice; radioactive iodine is clearly contraindicated. Both propylthiouracil and
methimazole cross the placenta equally and both may be used sa ely although con-
cern or liver ailure in pregnancy may avor the use o methimazole. Methimazole
is rarely associated with congenital gut anomalies and should be used af er organo-
genesis in the rst trimester. Graves disease of en improves during the course o
pregnancy and it is not uncommon or patients either to be on very low doses or
o antithyroid drugs completely by the end o pregnancy.
b. What is the mechanism o action o methimazole?
Antithyroid drugs inhibit the ormation o thyroid hormones by inter ering with
the incorporation o iodine into tyrosyl residues o thyroglobulin; they also inhibit
the coupling o these iodotyrosyl residues to orm iodothyronines (see Figure 27-1).
c. What other compounds (other than the typical antithyroid drugs) are inhibitors
o thyroid unction?
able 27-3 is a list o compounds that inhibit thyroid unction and the process that
is a ected.
(Continued)

TABLE 27-3 Anti-thyroid Compounds


PROCESS AFFECTED EXAMPLES OF INHIBITORS
Active transport o iodide Complex anions: perchlorate, uoborate, pertechnetate, thiocyanate

Iodination o thyroglobulin Thionamides: propylthiouracil, methimazole, carbimazole


Thiocyanate
Aniline derivatives; sul onamides
Iodide

Coupling reaction Thionamides


Sul onamides
?All other inhibitors o iodination

Hormone release Lithium salts


Iodide

Iodotyrosine deiodination Nitrotyrosines

Peripheral iodothyronine deiodination Thiouracil derivatives

Oral cholecystographic agents

Amiodarone

Hormone excretion/inactivation Inducers o hepatic drug-metabolizing enzymes: phenobarbital, ri ampin, carbamazepine, phenytoin

Hormone action Thyroxine analogs


Amiodarone
?Phenytoin
Binding in gut: cholestyramine

Data adapted rom Meier C.A., Burger A.C. E ects o drugs and other substances on thyroid hormone synthesis and metabolism.
In: Werner and Ingbars The Thyroid, 9th ed. (Braverman L.E. and Utiger R.D. eds.) Lippincott Williams &Wilkins, Philadelphia, 2005.
435
SECTION V Hormones and Hormone Antagonists

d. What adverse ef ects o methimazole therapy should this patient be cautioned


about?
T e most common adverse reaction is a purpuric, urticarial, papular rash which of en
subsides spontaneously. More serious is agranulocytosis although it is rare. A baseline
white blood cell count and di erential should be obtained be ore starting therapy
with antithyroid drugs. Patients should be cautioned to immediately report the devel-
opment o a sore throat or ever, which are of en signs o the presence o leukopenia.
Agranulocytosis is reversible upon discontinuation o the o ending drug.

CASE 27-3
A 58-year-old woman has been diagnosed with well-di erentiated papillary
thyroid cancer.
a. What are the treatment options with this patient?
T e mainstays o therapy or well-di erentiated thyroid cancer (papillary, ollicular)
are surgical thyroidectomy, radioiodine, and levothyroxine to suppress SH. T e
rationale or SH suppression is that SH is a growth actor or these cancers.
b. What is the standard course o treatment or this patient i metastatic disease is
suspected?
Because most well-di erentiated thyroid carcinomas accumulate very little iodine,
stimulation o iodine uptake with SH is required to treat metastases e ectively.
Currently, endogenous SH stimulation is evoked by withdrawal o thyroid hor-
mone replacement therapy (or by the administration o recombinant human SH) in
patients previously treated with near-total thyroidectomy. otal body 131I scanning and
measurement o thyroglobulin when the patient is hypothyroid ( SH >35 mU/L) help
to identi y metastatic disease or residual thyroid bed tissue. Depending on the residual
uptake or the presence o metastatic disease, an ablative dose o 131I is administered,
with a repeat total body scan 1 week later.
SH-suppressive therapy with levothyroxine is indicated in all patients af er treatment
or thyroid cancer.
Anaplastic cancer is the exception: it is highly malignant with survival usually less
than 1 year. Medullary thyroid carcinomas do not accumulate iodine and cannot be
treated with 131I.

KEY CONCEPTS
Replacement therapy or hypothyroidism typically uses oral thyroxine given
once daily.
T e goals o therapy or hypothyroidism are to restore serum SH concentration
to the normal range, and to relieve the signs and symptoms o hypothyroidism.
In pregnancy, the standard replacement dose o thyroid hormone is usually
increased.
Options available or treating hyperthyroid patients include antithyroid drugs
(eg, propylthiouracil and methimazole), radioactive iodine, and surgery.
Radioactive iodine may aggravate ophthalmopathy.
Medical therapy with antithyroid drugs to reduce the concentration o thyroid
hormone is the pre erred approach in younger patients with hyperthyroidism.
In older patients or those with cardiac disease, radioactive iodine is usually
recommended a er the patient has been rendered euthyroid with antithyroid
medication.
(Continued)

436
Thyroid and Antithyroid Drugs CHAPTER 2 7

T e initial treatment o thyroid cancer is surgical. Radioactive iodine is given to


ablate remnant thyroid tissue.
Following resection o well-di erentiated thyroid carcinoma, the goal o thyroxine
therapy is to suppress SH to below normal concentrations, thereby removing the
potential e ect o SH to stimulate the proli eration o the cancer cells.

SUMMARY QUIZ

QUESTION 27-1 A 34-year-old woman is being prepared or thyroidectomy. As part


o the preparation, she is given a solution containing high iodide concentration. She
wonders why she is being treated with something that is added to ood (salt). T e
explanation is that
a. iodide in ood is poorly absorbed.
b. iodide in ood is rapidly taken up by skeletal muscle.
c. low concentrations o iodide are required or thyroxine synthesis, but high concen-
trations inhibit thyroxine synthesis and release.
d. iodide in ood is not utilized by the thyroid gland.
e. high concentrations o iodide block the RH receptor on the pituitary gland.

QUESTION 27-2 A 38-year-old South American woman has a large protrusion on her
neck but is otherwise asymptomatic. Her condition is caused by a dietary de ciency o
a. iron.
b. magnesium.
c. potassium.
d. sodium.
e. iodine.

QUESTION 27-3 A 22-year-old woman has signs o hyperthyroidism. Her doctor


orders blood tests to determine the pituitary regulation o thyroid unction. Which o
the ollowing tests will be most help ul?
a. Iodine
b. 4thyroxine
c. 3triiodothyronine
d. SHthyroid-stimulating hormone
e. RHthyroid-releasing hormone

QUESTION 27-4 A 68-year-old woman has increasing lethargy, slight weight gain, and
the onset o cognitive impairment. A serum SH is slightly elevated and a 4
is within
normal limits. She is a candidate or
a. RH.
b. levothyroxine.
c. methimazole.
d. iodide.
e. radioactive iodine.

QUESTION 27-5 A 25-year-old woman has been treated with thyroxine or hypothy-
roidism. She has become pregnant. She complains now o being constantly atigued.
T e proper course o action would be to
a. do nothing, atigue is normal during pregnancy.
b. increase the iodine in her diet.
c. measure her serum SH during the rst trimester and adjust her thyroxine dose
based on the result.
(Continued)
437
SECTION V Hormones and Hormone Antagonists

d. double her dose o thyroxine.


e. decrease the dose o thyroxine as the need or thyroid replacement therapy
decreases during pregnancy.

QUESTION 27-6 A 48-year-old woman with Graves disease with severe hyperthyroid-
ism is being treated with radioactive iodine. She is also being treated with methimazole
because o
a. the long period o time be ore hyperthyroidism is controlled by radioactive iodine.
b. the expectation that radioactive iodine will be ine ective.
c. methimazole is better absorbed than radioactive iodine.
d. methimazole will counteract the side e ects o radioactive iodine.
e. radioactive iodine is not active unless administered with methimazole.

QUESTION 27-7 A 73-year-old man with a history o surgery or a benign thyroid nod-
ule when he was 50 years old is now euthyroid but has developed a cardiac arrhythmia.
Which o the antiarrhythmic drugs listed below might cause him to develop iodine-
induced hypothyroidism?
a. Quinidine
b. Amiodarone
c. Diltiazam
d. Propranolol
e. Digoxin

QUESTION 27-8 A 53-year-old woman with the diagnosis o Graves disease is being
treated with radioactive iodine. She should be warned o the high likelihood o
a. hyperthyroidism.
b. iodism.
c. hypothyroidism.
d. thyroid nodules.
e. thyroid cancer.

SUMMARY QUIZ ANSWER KEY

QUESTION 27-1 Answer is c. Low levels o iodine are required or thyroid synthesis,
but high levels o iodine inhibit thyroid synthesis and release.

QUESTION 27-2 Answer is e. In some areas o the world, simple or nontoxic goiter is
prevalent because o insuf cient dietary iodine. Normal thyroid unction obviously
requires an adequate intake o iodine; without it, normal amounts o thyroid hormone
cannot be made, SH is secreted in excess (see Figures 27-1 and 27-2), and the thyroid
becomes hyperplastic and hypertrophic. T e enlarged and stimulated thyroid becomes
remarkably ef cient at extracting residual traces o iodine rom the blood and usually
succeeds in producing suf cient thyroid hormones. In more severe iodine de ciency,
adult hypothyroidism and cretinism may occur.

QUESTION 27-3 Answer is d. In patients with normal pituitary unction, serum mea-
surement o SH is the thyroid unction test o choice because pituitary secretion o
SH is sensitively regulated in response to circulating concentrations o thyroid hormone.

QUESTION 27-4 Answer is b. Subclinical hypothyroidism is the presence o a mildly


elevated serum SH concentration and a normal 4 without obvious symptoms. An
elevated SH is especially common among elderly patients. Patients with subclinical
hypothyroidism who may bene t rom levothyroxine to normalize the SH include those
with goiter, autoimmune thyroid disease, hypercholesterolemia, cognitive dys unction,
(Continued)
438
Thyroid and Antithyroid Drugs CHAPTER 2 7

or pregnancy, and those patients who have nonspeci c symptoms that could be due to
hypothyroidism. T e decision to use levothyroxine therapy in these patients must be on
an individual basis because treatment may not be appropriate or all patients.

QUESTION 27-5 Answer is c. T e dose o levothyroxine in the hypothyroid patient


who becomes pregnant usually needs to be increased, perhaps due to the increased
serum concentration o thyroid-binding globulin ( BG) induced by estrogen. Most
physicians measure a serum SH during the rst trimester and then adjust the dose o
thyroxine based on the result. T e increased dosage requirement plateaus at about ges-
tation week 16 to 20, and dosage needs to all back to prepregnancy levels immediately
a er delivery.

QUESTION 27-6 Answer is a. A disadvantage o radioactive iodine therapy is the long


period o time that is sometimes required be ore the hyperthyroidism is controlled.
Methimazole is an antithyroid drug that is the drug o choice in conjunction with
radioactive iodine to hasten recovery while awaiting the e ects o radiation.

QUESTION 27-7 Answer is b. Amiodarone contains 75 mg in each tablet (see


able 27-2). A euthyroid patient with a history o a wide variety o underlying thyroid
disorders may develop iodine-induced hypothyroidism when exposed to large amounts
o iodine present in many commonly prescribed drugs. Among the disorders that
predispose patients to iodine-induced hypothyroidism are treated Graves disease,
Hashimoto thyroiditis, postpartum lymphocytic thyroiditis, subacute pain ul thyroiditis,
and lobectomy or benign nodules. (See the answer to question 27-1.)

QUESTION 27-8 Answer is c. T e chie consequence o the use o radioactive iodine is


the high incidence o hypothyroidism which will require li elong treatment and moni-
toring. Even when elaborate procedures are used to estimate iodine uptake and gland
size, most patients become hypothyroid.

SUMMARY: THYROID AND ANTITHYROID DRUGS


TOXICITIES
CLASS AND UNIQUE; CLINICALLY
SUBCLASSES NAMES CLINICAL USES COMMON IMPORTANT
Thyroid Hormones Levothyroxine Thyroid replacement therapy in Signs o overtreatment Excess thyroxine can
patients with hypothyroidism; would be the same as or increase the risk o atrial
suppression o TSH in patients hyperthyroidism f brillation, especially
with thyroid cancer in the elderly
Risk o osteoporosis in
postmenopausal women

Triiodothyroxine Used when more rapid onset Same as or levothyroxine Same as or levothyroxine
o action is needed such as
myxedema coma or or more
rapid termination o action such
as in preparing a cancer patient
or 131I therapy

Liotrix Mixture o thyroxine and Same as or levothyroxine Same as or levothyroxine


triiodothyroxine

Desiccated thyroid Mixture o thyroxine and Same as or levothyroxine Same as or levothyroxine


triiodothyroxine

Antithyroid Drugs Propylthiouracil Used in treatment o Purpuric urticarial popular rash Agranulocytosis
hyperthyroidism in the Pain, sti ness o joints, Hepatic ailure; requent
ollowing ways: def nitive paresthesia, headache, monitoring o liver unction
treatment; to hasten recovery nausea, skin pigmentation, is recommended
while awaiting e ects o loss o hair
radioactive iodine; preparation
or surgical treatment
(Continued)
439
SECTION V Hormones and Hormone Antagonists

TOXICITIES
CLASS AND UNIQUE; CLINICALLY
SUBCLASSES NAMES CLINICAL USES COMMON IMPORTANT

Methimazole Used in treatment o Same as propylthiouracil Agranulocytosis


hyperthyroidism in the
ollowing ways: def nitive
treatment; to hasten recovery
while awaiting e ects o
radioactive iodine; preparation
or surgical treatment

Carbimazole Same as propylthiouracil; Same as propylthiouracil


available in Europe, not
available in the United States

Ionic Thyroid Inhibitors Iodide Treatment o hyperthyroidism in Allergy to iodine with


Sodium iodine preoperative period anaphylactic reaction
Used in conjunction with
antithyroid drugs and
propranolol in the treatment o
thyrotoxic crisis

Radioactive Iodine Sodium iodine 131I Treatment o hyperthyroidism Contraindicated in pregnancy


Diagnosis o disorders o thyroid High incidence o delayed
unction hypothyroidism
Treatment o thyroid cancer
a ter stimulation o iodine
uptake with TSH Medullary
thyroid cancers do not
accumulate I- and cannot be
treated with 131I
Treatment o toxic nodular goiter

440
CHAPTER

Estrogens, Progestins, Androgens,


and Contraception 28
T is chapter will be most use ul a er having a basic understanding o the material in
DRUGS INCLUDED
Chapter 40, Estrogens and Progestins, Chapter 41, Androgens, and Chapter 66, Con-
traception and Pharmacotherapy o Obstetrical and Gynecological Disorders in Good- IN THIS CHAPTER
man & Gilmans T e Pharmacological Basis of T erapeutics, 12th Edition. In addition to Anabolicsteroids (various 17-alkylated
the material presented here, these chapters in the 12th Edition contain: androgens)
T e biosynthesis and physiological and pharmacological actions o the estrogens, Anastrozole (ARIMIDEX)
progestins, and androgens Bicalutamide (CASODEX,others)
A detailed discussion o gonadotropin and gonadal steroid concentrations during the Clomiphene (CLOMID,SERPPHENE, others)
menstrual cycle, the e ects o cyclical gonadal steroids on the reproductive tract, and
Conjugated equine estrogens (PREMARIN)
the metabolic e ects o estrogens, progestins, and androgens
Conjugated estrogens + medroxyprogesterone
A detailed discussion o ertility induction and the general principles o drug therapy
acetate (PREMPRO,PREMPHASE)
o pregnant women, and the prevention or arrest o preterm labor
Dutasteride (AVODART)
A detailed discussion o the secretion and transport o testosterone, the physiological
and pharmacologic e ects o androgens, and the e ects o androgens at di erent Estradiol + drospirenone (ANGELO)
stages o li e Estradiol + norethindrone (PREFEST)
Estradiol cypionate (DEPO-ESTRADIOL, others)
LEARNING OBJECTIVES
Estradiol micronized (ESTRACE, others)
Describe the mechanisms o action o estrogens, progestins, and androgens.
Estradiol nonmicronized (FEMTRACE)
Understand the therapeutic use and side e ects o postmenopausal hormone Estradiol topical gel (ESTROGEL)
replacement therapy.
Estradiol transdermal (ALORA, CLIMERA,
Understand the therapeutic uses and side e ects o contraceptive therapy. ESTRADERM,VIVELLE, others)
Describe the therapeutic use o selective estrogen receptor modulators, anties- Estradiol vaginal ring (ESTRING)
trogens, and aromatase inhibitors.
Estradiol vaginal tablets (VAGIFEM)
Describe the use o testosterone in the treatment o male hypogonadism. Estradiol valerate (DELESTROGEN, others)
Describe the use o androgen receptor antagonists and 5-reductase inhibitors to Estrogen esterifed esters (MENEST)
treat metastatic prostate cancer and benign prostatic hypertrophy, respectively.
Estropipate (ORTHO-EST,OGEN, others)
Understand the side e ects o the 17-alkylated androgens used by athletes to
Ethinyl estradiol + norethindrone (FEMHRT)
enhance per ormance.
Exemestane (AROMASIN)
Finasteride (PROSCAR)
MECHANISMS OF ACTION OF ESTROGENS, PROGESTINS, AND ANDROGENS Flutamide (EULEXIN)
DRUG CLASS DRUG MECHANISM OF ACTION Formestane
Estrogen Estradiol nonmicronized The biosynthetic pathway or the Fulvestrant (FASLODEX)
Estradiol micronized estrogens is shown in Figure 28-1
Letrozole (FEMARA)
Conjugated equine estrogens and the neuroendocrine control
Estrogen esteri ed esters o gonadotropin secretion in Medroxyprogesterone acetate
Synthetic conjugated estrogens emales is shown in Figure 28-2 (MPA; PROVERA, DEPO-PROVERA)
Estropipate Estrogens exert their e ects by Megestrol acetate (MEGACE, others)
Estradiol transdermal interaction with receptors that
Estradiol topical gel are members o the super amily Mi epristone (RU38486; MIFEPREX)
Estradiol vaginal ring o nuclear receptors; estrogen Nilutamide (NILADRON)
Estradiol vaginal tablets receptors are ligand-activated
Estradiol valerate transcription actors that increase Progesterone micronized (PROMETRIUM)
Estradiol cypionate or decrease the transcription o Progesterone slow-release intrauterine device
Conjugated estrogens + target genes (see Figure 28-3) (PROGESTASERT)
medroxyprogesterone acetate (MPA)
Ethinyl estradiol + norethindrone Progesterone vaginal gel (CRINONE, PROCHIEVE)
Estradiol + norethindrone (continues)
Estradiol + drospirenone
(Continued)
441
SECTION V Hormones and Hormone Antagonists

DRUGS INCLUDED DRUG CLASS DRUG MECHANISM OF ACTION


IN THIS CHAPTER (Cont.) Selective Estrogen Tamoxi en Compounds with estrogenic
Progesterone vaginal insert (ENDOMETRIN) Receptor Modulators Raloxi ene agonist, antagonist, or mixed
Toremi ene activity depending on the species
Raloxi ene (EVISTA) and targeted gene measured (see
See Table 28-1 or brand names and ormula- Chapter 46)
tions o oral contraceptives
Estrogen Antagonists Clomiphene Bind to estrogen receptors and
See Table 28-2 or brand names and ormula- Fulvestrant block estradiol binding
tions o agents used or hormone replace- Clomiphene increases
ment therapy gonadotropin secretion and
stimulates ovulation
Syntheticconjugated estrogens (CENESTIN;
ENJUVIA) Estrogen Synthesis Formestane Formestane and exemestane
Tamoxi en Inhibitors Exemestane act to irreversibly inactivate
Anastrozole aromatase (see Figure 28-1)
Testosterone (HISTERONE, others) Letrozole Anastrozole and letrozole act to
Testosterone buccal tablet (STRIANT) reversibly inhibit aromatase
Testosterone cypionate (DEPO-TESTOSTERONE,
Progestin Progesterone micronized Interact with progesterone
others) Progesterone vaginal gel receptors in target tissues
Testosterone enanthate (DELATESTRYL, others) Progesterone slow-release
intrauterine device
Testosterone gel (ANDROGEL, TESTIM)
Progesterone vaginal insert
Testosterone patch (ANDRODERM) Medroxyprogesterone
Testosterone undecanoate (ANDRIOL) acetate (MPA)
Megestrol acetate
Toremi ene (FARESTON)
Ulipristal (ELLA, ELLAONE) Progesterone Mi epristone Progesterone receptor antagonist
Antagonist

Selective Progesterone Ulipristal Partial agonist at progesterone


Receptor Modulator receptors; antiproli erative
e ects in the uterus and inhibits
ovulation

Oral Contraceptive See Table 28-1 Supresses luteinizing hormone


(LH) surge and prevents
ovulation

Androgen Testosterone The biological e ects o


Testosterone enanthate testosterone are mediated
Testosterone cypionate either directly, by binding to
Testosterone undecanoate the androgen receptor, or
Testosterone patch indirectly by conversion to
Testosterone gel dihydrotestosterone
Testosterone buccal tablet Testosterone can act as an
Anabolic steroids (various estrogen by conversion to
17-alkylated androgens) estradiol (see Figures 28-1,
28-4, 28-5, and 28-6)

Androgen Receptor Flutamide Act as antagonists at the


Antagonist Bicalutamide androgen receptor (see
Nilutamide Chapter 46)

5-Reductase Finasteride Inhibit 5-reductase and block


Inhibitors Dutasteride the conversion o testosterone
to dihydrotestosterone
(see Figure 28-5 and Chapter 46)

442
Estrogens, Progestins, Androgens, and Contraception CHAPTER 2 8

TABLE 28-1 Brand Names and Formulations of Oral Contraceptives


PRODUCT BRAND NAMEa

Combination b monophasic Estrogen (g) Progestin (mg)


Ethinyl estradiol/desogestrel DESOGEN 30 0.15

ORTHO-CEPT 30 0.15
Ethinyl estradiol/drospirenone YASMIN 30 3
Ethinyl estradiol/ethynodiol DEMULEN 1/35 35 1
DEMULEN 1/50 50 1
Ethinyl estradiol/levonorgestrel ALESSE 20 0.1
LEVLITE 20 0.1
LYBREL 20 0.09
NORDETTE 30 0.15
Ethinyl estradiol/norgestrel LO/OVRAL 30 0.3
OVRAL 50 0.5
Ethinyl estradiol/norethindrone BREVICON 35 0.5
FEMCON 35 0.4
LOESTRIN 1/20 20 1
LOESTRIN 1.5/30 30 1.5
NORINYL 1+35 35 1
ORTHO-NOVUM 1/35 35 1
OVCON 35 35 0.4
OVCON 50 50 1
Ethinyl estradiol/norgestimate ORTHO-CYCLEN 35 0.25
Mestranol/norethindrone NORINYL 1+50 50 1
ORTHO-NOVUM 1/50 50 1
Combination biphasic Estrogen (g) Progestin (mg)
Ethinyl estradiol/desogestrel MIRCETTE 20 0.15 (21 tabs)
10 (5 tabs)
Ethinyl estradiol/norethindrone ORTHO-NOVUM 10/11 35 0.5 (10 tabs)
35 1 (11 tabs)
Combination triphasic Estrogen (g) Progestin (mg)
Ethinyl estradiol/desogestrel CYCLESSA 25 0.1 (7 tabs)
25 0.125 (7 tabs)
25 0.15 (7 tabs)
Ethinyl estradiol/levonorgestrel TRI-LEVLEN 30 0.05 (6 tabs)
40 0.075 (5 tabs)
30 0.125 (10 tabs)
(Continued)

443
SECTION V Hormones and Hormone Antagonists

PRODUCT BRAND NAMEa

TRIPHASIL 30 0.05 (6 tabs)


40 0.075 (5 tabs)
30 0.125 (10 tabs)
Ethinyl estradiol/norethindrone ORTHO-NOVUM 7/7/7 35 0.5 (7 tabs)
35 0.75 (7 tabs)
35 1 (7 tabs)
TRI-NORINYL 35 0.5 (7 tabs)
35 1 (9 tabs)
35 0.5 (5 tabs)
Ethinyl estradiol/norgestimate ORTHO TRI-CYCLEN 35 0.18 (7 tabs)
35 0.215 (7 tabs)
35 0.25 (7 tabs)
ORTHO TRI-CYCLEN LO 25 0.18 (7 tabs)
25 0.215 (7 tabs)
25 0.25 (7 tabs)
Combination estrophasic Estrogen (g) Progestin (mg)
Ethinyl estradiol/norethindrone ESTROSTEP 20 1 (5 tabs)
30 1 (7 tabs)
35 1 (9 tabs)
Combination extended cycle Estrogen (g) Progestin (mg)
Ethinyl estradiol/drospirenone YAZ 20 3 (24 tabs)
Ethinyl estradiol/levonorgesterol LYBREL 20 0.09 (28 tabs)
SEASONALE 30 0.15 (84 tabs)
SEASONIQUE 30 0.15 (84 tabs)
10 (7 tabs)
Ethinyl estradiol/norethindrone LOESTRIN 24 20 1 (24 tabs)
Progestin only Estrogen (g) Progestin (mg)
Norethindrone MICRONOR 0.35c
NOR-QD 0.35c
Norgestrel OVRETTE 0.075c
Emergency contraception
Levonorgestrel PLAN B 0.75 2 doses
Ulipristal (progesterone partial agonist) ella; ellaOne 30 mg 1 dose
Unless otherwise indicated, the products are packaged with 21 active (hormone-containing) pills and 7 placebo tablets. For ormulations that
di er rom this standard (eg, multiphasic pills, extended-cycle ormulations), the numbers o tablets o each pill strength are indicated.
a
Some ormulations also contain iron to diminish the risk o Fe-de ciency anemia; these are not listed separately here.
b
Combination ormulations contain both an estrogen and a progestin.
c
Denotes continuous administration o active pills.

444
Estrogens, Progestins, Androgens, and Contraception CHAPTER 2 8

TABLE 28-2 Brand Names and Formulations of Agents Used for Hormone Replacement Therapy
PREPARATION BRAND DOSE mg FREQUENCY
Oral estrogen (tablets)

Conjugated estrogens CENESTIN

ENJUVIA 0.3, 0.45, 0.625, 0.9, 1.25

PREMARIN

Estropipate OGEN 0.625, 1.25, 1.5


1/day
ORTHO-EST 0.75, 1.5

Micronized estrogen ESTRACE 0.5, 1, 2

Estradiol FEMTRACE 0.45, 0.9, 1.8

Esteri ed estrogens MENEST 0.3, 0.625, 1.25, 2.5

Transdermal estradiol

Patch ALORA 0.025, 0.05, 0.075, 0.1 2/week

CLIMARA 0.025, 0.0375, 0.05, 0.06, 1/week


0.075, 0.1

ESCLIM 0.025, 0.0375, 0.05, 0.075, 2/week


0.1

ESTRADERM 0.05, 0.1 2/week

MENOSTAR 0.014 1/week

VIVELLE 0.05, 0.1 2/week

VIVELLE-DOT 0.025, 0.0375, 0.05, 0.075, 2/week


0.1

Gel DIVIGEL 0.25, 0.5, 1 g/packet 1/day

ELESTRIN 0.87 g/pump 1/day

ESTROGEL 1.25 g/pump 1/day

Emulsion ESTRASORB 1.74 g/pouch 2/day

Spray EVAMIST 1.53 mg/spray 1-3 sprays/day

Vaginal estrogen

Estradiol ESTRACE (cream) 0.1 mg/g 1/day, then 1-3/week

ESTRING (ring) 7.5 g/24 hr 1/90 days

FEMRING (ring) 0.05, 0.1 mg/day 1/3 months

VAGIFEM (tabs) 25 g 1/day or 2 weeks, then 2/


week

Conjugated estrogens PREMARIN (cream) 0.625 mg/g 1/day

Oral progestin

Norethindrone AYGESTIN 5 mg

Progesterone (micronized) PROMETRIUM 100, 200 mg 1/day

Medroxyprogesterone PROVERA 2.5, 5, 10 mg


(Continued)

445
SECTION V Hormones and Hormone Antagonists

PREPARATION BRAND DOSE mg FREQUENCY


Oral estrogen plus Estrogen (mg) Progesterone (mg)
progesterone

Estradiol/norethindrone ACTIVELLA 1 0.5

Estradiol/drospirenone ANGELIQ 1 0.5 1/day

Ethinyl estradiol/ FEMHRT 0.5 2.5 g


norethindrone

1 5 g

Transdermal estrogen
plus progesterone

Estradiol/levonorgestrel CLIMARA PRO 0.045 mg/day 0.015 mg/day 1/week

Estradiol/norethindrone COMBIPATCH 0.05 mg/day 0.14 mg/day 2/week

0.05 mg/day 0.25 mg/day 2/week

CASE 28 1
A 49-year-old woman has been 6 months without menstruation. She is now experiencing
severe hot ashes that are impacting her daily li e. She would like some therapy that
would make her more com ortable.
a. What hormone replacement therapy is available or her?
able 28-2 lists the brand names and ormulations used or hormone replacement
therapy. Regardless o the speci c agent or regimen, menopausal hormone therapy
with estrogens should use the lowest dose and shortest duration necessary to
achieve an appropriate therapeutic goal.
(Continued)

H3 C O H3 C O H3 C O
16 -OHa s e OH 3-OH-S DH OH
H3 C H3 C H3 C

HO HO O
De hydroe pia ndros te rone 16 -Hydroxyde hydro- 16 -Hydroxya ndros te ne dione
e pia ndros te rone

17-OH-S DH,
3-OH-S DH a roma ta s e

H3 C O H3 C O H3 C OH
a roma ta s e 17-OH-S DH OH
H3 C

16 -OHa s e
O HO HO
Andros te ne dione Es trone Es triol

16 -OHa s e 3-OH-S DH
17-OH-S DH 17-OH-S DH
3-Hydroxys te roid
de hydroge na s e
H3 C OH H3 C OH
17 17-OH-S DH
H3 C 16
C D a roma ta s e 17-Hydroxys te roid
de hydroge na s e
A B 16 -OHa s e
O 3 HO
16 -Hydroxyla s e
Te s tos te rone Es tra diol

FIGURE 28-1 The biosynthetic pathway or the estrogens.


446
Estrogens, Progestins, Androgens, and Contraception CHAPTER 2 8

hypotha la mic
puls e
A Ne urona l
Activity
(dis cha rge s
ge ne ra tor pe r min)

LH
(ng/ml)
a rc ua te
nuc le us
p ituita ry D 0 1 2 3 4
me d ia l TIME (hrs )
basal
hyp otha la mus
opioid NE
dopa mine +
GABA ++
proge s te rone
B
GnRH

GnRH p ituita ry
ne uron p orta l
va s c ula ture

GnRH g ona d otrop e

C e s troge n
proge s te rone

LH / FS H

e s troge n e s troge n
E proge s te rone proge s te rone
E
ute rus ova ry

FIGURE 28-2 Neuroendocrine control o gonadotropin secretion in emales. The hypothalamic pulse generator located in the arcuate nucleus
o the hypothalamus unctions as a neuronal clockthat res at regular hourly intervals. A. This results in the periodic release o gonadotropin-
releasing hormone (GnRH) rom GnRH-containing neurons into the hypothalamic-pituitary portal vasculature B. GnRH neurons receive inhibitory
input rom opioid, dopamine, and GABA neurons and stimulatory input rom noradrenergic neurons (NE, norepinephrine). The pulses o GnRH
trigger the intermittent release o luteinizing hormone (LH) and ollicle-stimulating hormone (FSH) rom pituitary gonadotropes C, resulting in
the pulsatile plasma pro le D. FSH and LH regulate ovarian production o estrogen and progesterone, which exert eedback controls E.

b. She is being treated with a combination estrogen/progesterone product


(ethinyl estradiol/norethindrone, FEMHRT). Why is the progesterone added to
the estrogen?
T e use o unopposed estrogen or hormone treatment in postmenopausal women
increases the risk o endometrial carcinoma by 5- to 15- old. T is increased risk can
be prevented i a progestin is coadministered with the estrogen.
c. What are the mechanisms o action estrogen and progesterone?
Figure 28-3 describes the mechanism o action o a nuclear estrogen receptor.
Estrogen and progesterone receptors are ligand-activated transcription actors that
increase or decrease the transcription o target genes.
d. What are the risks o hormone replacement therapy that should be discussed
with the patient?
Oral estrogens (and estrogen-progestin combinations) have a small but signi cant
risk o thromboembolic disease in healthy women and in women with preexisting
cardiovascular disease.
(Continued)
447
SECTION V Hormones and Hormone Antagonists

A
TA
TA
ER
ER

GGTCAnnnTGACC
Ag onis t E
Anta g onis t
E
T
T

C
B S RC-1
HDAC1
E
E NCoR
S WI/S NF

ER ER T T
ER ER
GGTCAnnnTGACC
GGTCAnnnTGACC

p300 GTA
E

ER TRAP

GGTCAnnnTGACC

FIGURE 28-3 Molecular mechanism o action o nuclear estrogen receptor. A. Unliganded


estrogen receptor (ER) exists as a monomer within the nucleus. B. Agonists such as 17-estradiol
bind to the ER and cause a ligand-directed change in con ormation that acilitates dimerization
and interaction with speci c estrogen response element (ERE) sequences in DNA. The ER-DNA
complex recruits coactivators such as SWI/SNF that modi y chromatin structure, and coactivators
such as steroid-receptor coactivator-1 (SRC-1) that has histone acetyltrans erase (HAT) activity
and that urther alters chromatin structure. This remodeling acilitates the exchange o the
recruited proteins such that other coactivators (eg, p300 and the TRAP complex) associate on the
target gene promoter and proteins that comprise the general transcription apparatus (GTA) are
recruited, with subsequent synthesis o mRNA. C. Antagonists such as tamoxi en (T) also bind to
the ER but produce a di erent receptor con ormation. The antagonist-induced con ormation also
acilitates dimerization and interaction with DNA, but a di erent set o proteins called corepres-
sors, such as nuclear-hormone receptor corepressor (NcoR), are recruited to the complex. NcoR
urther recruits proteins such as histone deacetylase I (HDAC1) that act on histone proteins to
stabilize nucleosome structure and prevent interaction with the GTA.

T e progestin component in combined hormone-replacement therapy plays a


major role in the increased risk o breast cancer. T e available data suggest that the
excess risk o breast cancer associated with menopausal use appears to abate 5 years
a er discontinuing therapy. T us, hormone replacement therapy or less than or
equal to 5 years is o en prescribed to mitigate hot ashes and likely has a minimal
e ect on the risk o breast cancer.

CASE 28 2
A 22-year-old woman is asking or an oral contraceptive.
a. What are the options available or her?
able 28-1 lists the brand names and ormulations o oral contraceptives. T e major-
ity o available products contain estrogen and progestin in a constant amount o the
estrogen and progestin (monophasic) that are packaged with 21 pills containing active
hormone and 7 placebo tablets. In an e ort to maximize the antiovulatory e ects and

(Continued)
448
Estrogens, Progestins, Androgens, and Contraception CHAPTER 2 8

21
H3 C 20 CH3
H3 C
17 CH3
11
CH3
1
a ce ta te
3 ce llula r pla s ma chole s te rol
HO chole s te rol
s ide cha in
cle ava ge

pre gne nolone 17 OH-pre gne nolone de hydroe pia ndros te rone a ndros te ne diol
(DHEA)
3-hydroxys te roid
de hydroge na s e
pre ge s te rone 17 OH-proge s te rone a ndros te ne dione te s tos te rone

OH
17 -hydroxyla s e 17, 20 lya s e H3 C

H3 C

FIGURE 28-4 Pathway o synthesis o testosterone in the Leydig cells o the testes. In Leydig cells, the 11 and 21 hydroxylases (present in
adrenal cortex) are absent but CYP17 (17 -hydroxylase) is present. Thus androgens and estrogens are synthesized; corticosterone and cortisol
are not ormed. Bold arrows indicate avored pathways.

prevent breakthrough bleeding while minimizing total exposure to the hormones,


some ormulations provide active pills with two (biphasic) or three (triphasic) di erent
amounts o one or both hormone to be used sequentially during each cycle.
Formulations called extended-cycle contraceptives extend the number o active
pills per cycle and thus decrease the duration o menstrual bleeding.

(Continued)

Active Ina ctive


Me ta bolite s Me ta bolite s
DIHYDROTESTOSTERONE ANDROSTERONE
OH O

O HO
H H

5 - 18 OH
17
re ducta s e
19
C D TESTOSTERONE
A B
O 3 5
4

CYP 19 (a roma ta s e )
O
OH

HO
HO H
ESTRADIOL ETIOCHOLANOLONE

FIGURE 28-5 Metabolism o testosterone to its major active and inactive metabolites.
449
SECTION V Hormones and Hormone Antagonists

TESTOSTERONE CYP 19
5 - (a roma ta s e )
re ducta s e

Dihydro te s to s te ro ne Es tradio l

Androge n Androge n Es troge n


Re ce ptor Re ce ptor Re ce ptor

Exte rna l Ge nita lia : Inte rna l Ge nita lia : Bone :


-d iffe re ntia tion d uring g e s ta tion -wolffia n d eve lop me nt -e p ip hys e a l c los ure
-ma tura tion d uring p ub e rty d uring g e s ta tion inc re a s e d d e ns ity
-a d ulthood p ros ta tic d is e a s e s Libido
S ke le ta l Mus cle
Ha ir Follicle s : -inc re a s e ma s s a nd
-inc re a s e d g rowth d uring s tre ng th d uring p ub e rty
p ub e rty
Erythropoie s is
Bone

FIGURE 28-6 Direct e ects o testosterone and e ects mediated indirectly via dihydrotestosterone
or estradiol.

A weekly transdermal patch (OR HO EVRA) releases ethinyl estradiol and norel-
gestromin. Pharmacokinetic data suggests that it provides higher estrogen exposure
than the low-dose oral contraceptives and the FDA has required a warning that this
product may have increased risk o venous thromboembolism.
A vaginal ring (NUVARING) also is available that releases ethinyl estradiol and
etonogestrel.
b. Why do some products contain a progesterone?
T e structural eatures o the various progestins are shown in Figure 28-7. T e
major unctions o the estrogen components are to sensitize the hypothalamus and
pituitary gonadotropes (see Figure 28-2) to the eedback inhibitory e ects o the
progestin and to minimize breakthrough bleeding. T e progestin exerts negative
eedback, which suppresses the LH surge and thereby prevents ovulation and pro-
tects against uterine cancer by opposing the proli erative e ects o the estrogen on
the uterine endometrium.
c. What are progestin-only contraceptives?
T e progestin-only contraceptives contain derivatives o 17-alkyl-19-nortestosterone
but do not contain an estrogen. Although they do inhibit ovulation to some degree,
their ef cacy also re ects changes in the cervical mucosa that inhibit ertilization and
endometrial changes that inhibit implantation. T ey are slightly less e ective than the
combination estrogen/progestin ormulations, particularly when doses are missed,
but provide an alternative in settings when estrogen-containing ormulations are
contraindicated.
Progestins are also used or long-acting contraception. A depot ormulation o
medroxyprogesterone (DEPO-PROVERA) injected subcutaneously or intramuscu-
larly provides e ective contraception or 3 months.
d. What are the risks and side e ects that should be discussed with this patient?
T romboembolic disease, largely due to the estrogenic component, is the most
common serious side e ect. Other side e ects include hypertension, edema, gall-
bladder disease, and elevations in serum triglycerides. Pills containing drospire-
none, which antagonizes the mineralocorticoid receptor, should be accompanied
(Continued)
450
Estrogens, Progestins, Androgens, and Contraception CHAPTER 2 8

Age nts S imila r to Proge s te rone (Pre gna ne s )


CH3

21 CH3 C O
OAc
20 C O
18
17
11
19

O
3
6 CH3
O
P ROGESTERONE MEDROXYP ROGESTERONE
ACETATE

Age nts S imila r to 19-Norte s tos te rone (Es tra ne s )


OH OH
C CH

O O
19-NORTESTOSTERONE NORETHINDRONE

Age nts S imila r to 19-Norge s tre l (Gona ne s )


H3 C H3 C
OH OCOCH3
H2 C H2 C
C CH C CH

O HON
NORGESTREL NORGESTIMATE

FIGURE 28-7 Structural eatures o various progestins.

with care ul monitoring o the serum K+. T e long-acting progestin-only ormu-


lations (DEPO-PROVERA) are associated with decreased bone mineral density.
eenagers and younger women who have not achieved maximal bone density may
be particularly at risk.

CASE 28-3
A 53-year-old woman is diagnosed with breast cancer. T e lump is removed and
lymph nodes are negative or signs o cancer. She is started on tamoxi en, a selective
estrogen receptor modulator (SERM).
a. What are SERMs?
Selective estrogen receptor modulators, or SERMs, are compounds with
tissue-selective actions. T e pharmacological goals o these drugs are to produce
bene cial estrogenic actions in certain tissues (eg, bone, brain, and liver) dur-
ing postmenopausal hormone therapy but antagonist activity in tissues such as
breast and endometrium, where estrogenic actions (eg, carcinogenesis) might be
deleterious. amoxi en is highly ef cacious in the treatment o breast cancer (see
Chapter 46).
b. What are the untoward e ects and risk o tamoxi en therapy?
amoxi en treatment causes a 2- to 3- old increase in the relative risk o deep vein
thrombosis and pulmonary embolism and a roughly 2- old increase in endometrial
carcinoma. T e most common side e ect is hot ashes.

451
SECTION V Hormones and Hormone Antagonists

CASE 28-4
A 24-year-old woman with polycystic ovary syndrome is being treated with clomiphene
or in ertility.
a. What kind o drug is clomiphene and what is its mechanism o action?
Clomiphene is an antiestrogen and is a pure estrogen antagonist in all tissues studied.
Clomiphene binds to estrogen receptors (see Figure 28-3), increases gonadotropin
secretion and stimulates ovulation. It increases the amplitude o LH and FSH pulses
(see Figure 28-2) without changing pulse requency by acting largely at the pituitary
level to block inhibitory actions o estrogen on gonadotropin release and/or somehow
causing the hypothalamus to release larger amounts o GnRH per pulse.
b. What are the untoward e ects o clomiphene?
Clomiphene should not be administered to pregnant women due to reports o
teratogenicity in animals. Its untoward e ects include ovarian hyperstimulation,
increased incidence o multiple births, ovarian cysts, hot ashes, headaches, and
blurred vision. Prolonged use may increase the risk o ovarian cancer.

CASE 28-5
A 23-year-old woman comes into the clinic requesting emergency contraception ollowing
unprotected sexual intercourse 12 hours earlier. She is given ulipristal.
a. What kind o drug is ulipristal and what is its mechanism o action?
Ulipristal unctions as a selective progesterone receptor modulator (SPRM), acting
as a partial agonist at progesterone receptors. It has antiproli erative e ects in the
uterus; however, its most relevant actions to date involve its capacity to inhibit
ovulation. Ulipristals antiovulatory actions likely occur due to progesterone
regulation at many levels, including inhibition o LH release through the hypo-
thalamus and pituitary, and inhibition o LH-induced ollicular rupture within the
ovary. It is e ective up to 120 hours (5 days) a er intercourse.
b. What other treatment options are available or this patient?
Plan B (see able 28-1) contains 2 tablets o the progestin levonorgestrel and may
be obtained in the United States without prescription by women who are 18 years
o age or older. Other options or postcoital contraception include mi epristone
that is not FDA approved or this use but is highly e ective, and copper intrauterine
devices when inserted within 4 days o unprotected intercourse.
c. What side e ects might be expected ollowing ulipristal administration?
T e most severe side e ect in clinical trials using ulipristal has been a sel -limited
headache and abdominal pain.

CASE 28-6
A 46-year-old man is diagnosed with hypogonadism and low serum testosterone.
a. What options are available or his testosterone replacement?
Figure 28-8 shows the androgens that are available or therapeutic use. Even though
ingested testosterone is readily absorbed into the hepatic circulation, the rapid
hepatic catabolism ensures that hypogonadal men generally cannot ingest it in
suf cient amounts and with suf cient requency to maintain a normal serum
testosterone concentration. Most pharmaceutical preparations o androgens,
there ore, are designed to bypass hepatic catabolism o testosterone.
Esteri ying a atty acid to the 17 hydroxyl group o testosterone creates a com-
pound that is even more lipophilic than testosterone itsel . When an ester such
as testosterone enthanthate or cypionate is dissolved in oil and administered
(Continued)
452
Estrogens, Progestins, Androgens, and Contraception CHAPTER 2 8

Te s to s te ro ne
(HISTERONE, othe rs )
OH

Te s to s te ro ne Es te rs O

O CO(CH2 )5 CH3 O COCH2 CH2 O C (CH2 )9 CH3

O O O
Te s to s te ro ne e nanthate Te s to s te ro ne c ypio nate Te s to s te ro ne unde c ano ate
(DELATESTRYL, othe rs ) (DEPO-TESTOSTERONE, othe rs ) (ANDRIOL)

17 -Alkylate d Andro g e ns
OH OH OH
CH3 CH3 CH3 CH3

CH3
O N
HN
O O
H
Me thylte s to s te ro ne
(ORETIN METHYL, othe rs ) Oxandro lo ne S tano zo lo l
(OXANDRIN) (WINSTROL)

OH OH
HO CH3 C CH

F
N
O
O
Fluoxyme s te ro ne Danazo l
(HALOTESTIN) (DANOCRINE)

Othe r 3

OH

O CH3

7 -Me thyl-19-no rte s to s te ro ne Te trahydro g e s trino ne

FIGURE 28-8 Structures o androgens available or therapeutic use.

intramuscularly every 2 to 4 weeks the resulting testosterone concentrations range


rom higher than normal in the rst ew days a er injection to low normal just
be ore the next injection. T e undecanoate ester o testosterone, when dissolved in
oil and ingested orally, is absorbed into the lymphatic circulation, thus bypassing
initial hepatic catabolism.
(Continued)
453
SECTION V Hormones and Hormone Antagonists

A testosterone gel, a testosterone patch, and a testosterone buccal tablet are also
available.
T e 17-alkylated androgens also are less prone to hepatic catabolism; however,
they are less androgenic than testosterone and can cause hepatotoxicity.
b. What are the risks and untoward e ects o testosterone therapy that should be
discussed with this patient?
T e major side e ects o testosterone administration are acne, gynecomastia, and
aggressive sexual behavior. Prolonged administration may be associated with benign
prostatic hypertrophy and prostate cancer. T e 17-alkylated androgens are the only
androgens that cause hepatotoxicity.

CASE 28-7
A 29-year-old male cyclist has been taking testosterone on the advice o his trainer. His
trainer has rationalized that since testosterone is an endogenous substance, it will be
di cult to detect.
a. What side e ects might this athlete expect to experience?
T e pathways o synthesis o testosterone are shown in Figure 28-4. All androgens
suppress gonadotropin secretion when taken in high doses and thereby suppress
endogenous testicular unction. T us, there is decreased endogenous testosterone
and sperm production with resultant diminished ertility. Chronic administration
may result in reduced testicular size. Androgens in high doses can be converted to
estrogens, which may cause gynecomastia. T e 17-alkylated androgens can cause
hepatotoxicity, including cholestasis and peliosis hepatis, blood- lled cysts.
b. Is it sound rationale that testosterone will be di cult to detect?
Endogenous testosterone can be detected by 1 o 2 methods. One is /E ratio, the
ratio o testosterone glucuronide to its endogenous epimer, epitestosterone gluc-
uronide in urine. Administration o exogenous testosterone suppresses secretion
o both endogenous testosterone and epitestosterone and replaces them with only
the exogenous testosterone, so the /E ratio is higher than normal. Both testos-
terone and epitestosterone are measured in urine by gas chromatography-mass
spectrometry.
A second method or detecting the administration o exogenous testosterone
employs gas chromatography-combustion isotope ratio mass spectrometry to detect
the presence o 13C and 12C compounds. Urinary steroids with a low 13C/ 12C ratio are
likely to have originated rom pharmaceutical sources as opposed to endogenous
physiological sources.

CASE 28-8
A 56-year-old man with metastatic prostate cancer is being treated with utamide.
a. What is f utamide and what are its mechanisms o actions?
Flutamide is an androgen receptor antagonist that is used in the treatment o
metastatic prostate cancer (see Chapter 46). When used alone utamide causes an
increased secretion o LH, which in turn increases serum testosterone concentra-
tions. Androgen receptor antagonists are used primarily in conjunction with a
GnRH agonist which during chronic therapy downregulates GnRH receptors caus-
ing a decline in the pituitary response. T e all in LH results in decreased serum
testosterone concentrations. In this setting, utamide blocks the actions o adrenal
androgens, which are not inhibited by GnRH agonists.
b. What are the untoward e ects o f utamide?
Common side e ects o utamide are diarrhea, breast tenderness, and nipple
tenderness. Less commonly nausea, vomiting, and hepatotoxicity occur.

454
Estrogens, Progestins, Androgens, and Contraception CHAPTER 2 8

KEY CONCEPTS
In postmenopausal woman, hormone replacement therapy is most commonly
used to treat vasomotor disturbances (hot ashes).
In postmenopausal women with an intact uterus, a progestin is included in
the hormone replacement therapy to prevent endometrial cancer.
Regardless o the specif c drug(s), hormone replacement therapy should use
the minimum dose and duration or the desired therapeutic end point.
Estrogens and progestins are widely used as combination contraceptives and
are 99% e ective in preventing ovulation.
amoxi en is a selective estrogen receptor modulator widely used or the adjuvant
treatment o breast cancer and or prophylaxis in high-risk women, but treatment
should be limited to 5 years.
Fulvestrant, a pure estrogen antagonist, is used to treat breast cancer.
Clomiphene is an estrogen antagonist used to treat in ertility.
Progestin-only contraceptives are available when estrogen-containing products
are contraindicated.
A levonorgestrel-only product and ulipristal are available or emergency contra-
ception within 72 hours or 120 hours o unprotected intercourse, respectively.
Mi epristone, administered with a prostaglandin, is used as an antiprogestin or
medical abortion.
estosterone delivery systems are designed to avoid hepatic catabolism that
occurs when testosterone is ingested orally.
Androgen receptor antagonists such as utamide are used to treat metastatic
prostate cancer.
Finasteride, a 5-reductase inhibitor, is used to treat benign prostatic hypertrophy.
T e 17-alkylated androgens are the only androgens that cause hepatotoxicity.

SUMMARY QUIZ
QUESTION 28-1 A 30-year-old woman is being started on birth control pills or the
f rst time. She has a history o migraine headaches with aura. T erapy with which one
o the ollowing contraceptive ormulations is contraindicated in this patient?
a. Levonorgestrel
b. Mi epristone
c. Norethindrone
d. Ethinyl estradiol/norethindrone
e. Ulipristal
QUESTION 28-2 A 56-year-old woman with an intact uterus is being treated with
hormone replacement therapy with a product that contains an estrogen/progestin
combination (estradiol/norethindrone). T e progestin is added to
a. decrease the risk o endometrial cancer.
b. decrease the incidence o breast cancer.
c. increase the e ectiveness in treating vasomotor symptoms.
d. decrease the risk o coronary heart disease.
e. decrease the risk o cognitive impairment.
QUESTION 28-3 A 25-year-old woman is being treated with clomiphene or ertility.
Clomiphene acts by
a. blocking estrogen e ects on the uterus.
b. increasing FSH concentrations. (Continued)
455
SECTION V Hormones and Hormone Antagonists

c. blocking progesterone e ects on the ovary.


d. decreasing LH concentrations.
e. increasing progesterone secretion by the ovary.

QUESTION 28-4 A 51-year-old woman is beginning menopause. Hormone replacement


therapy should be considered or therapy o her hot ashes as well as to decrease the risk o
a. coronary heart disease.
b. dementia.
c. thromboembolic disease.
d. bone ractures.
e. liver impairment.

QUESTION 28-5 A 62-year-old man is being treated with testosterone or hypogonad-


ism o old age. A likely side e ect would be
a. hepatotoxicity.
b. dementia.
c. gynecomastia.
d. optic neuritis.
e. peripheral neuropathy.

QUESTION 28-6 A 68-year-old man is being treated with utamide or advanced


prostate cancer. In this setting utamide should be administered with a GnRH agonist
because utamide, when used alone, causes an
a. increased LH secretion.
b. increased estrogen secretion.
c. increased SH secretion.
d. increased progesterone secretion.
e. decreased testosterone secretion.

QUESTION 28-7 A 55-year-old man with benign prostatic hypertrophy is being treated
with f nasteride. Finasteride acts by blocking the conversion o
a. testosterone to estradiol.
b. testosterone to dihydrotestosterone.
c. cortisol to cortisone.
d. cholesterol to pregnenolone.
e. progesterone to androstenedione.

QUESTION 28-8 A 53-year-old woman with breast cancer is being treated with tamox-
i en. amoxi en is a(n)
a. progesterone antagonist.
b. androgen agonist.
c. inhibitor o 5-reductase.
d. inhibitor o aromatase.
e. selective estrogen receptor modulator.

SUMMARY QUIZ ANSWER KEY


QUESTION 28-1 Answer is d. T e combination (estrogen/progestin) contraceptives
are contraindicated in women with a history o thromboembolic disease, cerebrovas-
cular disease, migraine headaches with aura, estrogen-dependent cancer, impaired
hepatic unction or active liver disease, undiagnosed uterine bleeding, and suspected
pregnancy. Levonorgestrel and ulipristal are used as emergency contraception and
456
Estrogens, Progestins, Androgens, and Contraception CHAPTER 2 8

mi epristone is used to terminate pregnancy. Norethindrone is a progestin-only oral


contraceptive. While slightly less e ective than the combination estrogen/progestin
ormulations, norethindrone provides an alternative in settings where estrogen-
containing ormulations are contraindicated.
QUESTION 28-2 Answer is a. In postmenopausal women with an intact uterus, a pro-
gestin is included in hormone replacement therapy to prevent endometrial cancer. T e
structural eatures o various progestins are shown in Figure 28-7.
QUESTION 28-3 Answer is b. Clomiphene is a potent antiestrogen that primarily is used
or treatment o anovulation in the setting o an intact hypothalamic-pituitary axis and
adequate estrogen production. Clomiphene acts by inhibiting the negative eedback e ects
o estrogen at hypothalamic-pituitary levels, increasing ollicular-stimulating hormone
(FSH) concentrations and thereby enhancing ollicular maturation (see Figure 28-2).
QUESTION 28-4 Answer is d. In the Womens Health Initiative (WHI), treatment
o postmenopausal women with conjugated estrogen plus medroxyprogesterone (in
women with a uterus) or with conjugated estrogen alone (in women without a uterus),
there was improved bone density and a decreased risk o bone ractures.
QUESTION 28-5 Answer is c. When administered in high doses, androgens, such as
testosterone can be converted to estrogens, causing gynecomastia (see Figures 28-1 and
28-5). T e 17-alkylated androgens are the only androgens that cause hepatotoxicity.
QUESTION 28-6 Answer is a. Flutamide is a potent androgen receptor antagonist and
has limited e cacy when used alone because it increases LH secretion that stimulates
higher serum testosterone concentrations. It is used primarily in conjunction with a
GnRH agonist in the treatment o metastatic prostate cancer (see Chapter 46).
QUESTION 28-7 Answer is b. Finasteride is an antagonist o 5-reductase that blocks
the conversion o testosterone to dihydrotestosterone, especially in male external geni-
talia (see Figure 28-5).
QUESTION 28-8 Answer is e. amoxi en is a selective estrogen receptor modula-
tor with tissue-selective actions. T e pharmacological goal o tamoxi en therapy is to
produce benef cial estrogenic actions in certain tissues (eg, bone, brain, and liver), but
antagonistic estrogen activity in tissues such as breast and endometrium where estro-
genic actions (eg, carcinogenesis) might be deleterious (see Chapter 46).

SUMMARY: ESTROGENS, PROGESTINS, CONTRACEPTIVES, AND ANDROGENS


TOXICITIES
CLASS AND
SUBCLASSES NAMES CLINICAL USES COMMON UNIQUE; CLINICALLY IMPORTANT
Estrogen Estradiol nonmicronized Menopausal hormone Unopposed use or hormone
Estradiol micronized therapy treatment increases the risk o
Conjugated equine estrogens Combination in oral endometrial carcinoma; this risk is
Estrogen esteri ed esters contraceptives prevented i progestin added to
Synthetic conjugated estrogens (see Table 28-1) therapy
Estropipate Increased risk o thromboembolic
Estradiol transdermal disease
Estradiol topical gel May cause migraine in some
Estradiol vaginal ring women
Estradiol vaginal tablets May reactivate or exacerbate
Estradiol valerate endometriosis
Estradiol cypionate Drospirenone antagonizes the
Conjugated estrogens + mineralocorticoid receptor
medroxyprogesterone and may cause serum K+ to
acetate (MPA) rise particularly in patients on
Ethinyl estradiol + norethindrone potassium-sparing diuretics
Estradiol + norethindrone
Estradiol + drospirenone
(Continued)
457
SECTION V Hormones and Hormone Antagonists

TOXICITIES
CLASS AND
SUBCLASSES NAMES CLINICAL USES COMMON UNIQUE; CLINICALLY IMPORTANT
Selective Estrogen Tamoxi en Treatment o breast Increased risk o endometrial
Receptor Modulators Raloxi ene cancer (see Chapter 46) cancer and thromboembolic
Toremi ene Raloxi ene used to treat disease
osteoporosis

Estrogen Antagonists Clomiphene Used to treat in ertility Ovarian hyper-stimulation,


Fulvestrant due to anovulation increased incidence o multiple
births, ovarian cysts, hot f ashes,
and blurred vision

Estrogen Synthesis Formestane Used to treat breast Hot f ashes Do not increase the risk o uterine
Inhibitors Exemestane cancer (see Chapter 46) cancer or thromboembolic disease
Anastrozole
Letrozole

Progestin Progesterone micronized Used in combination with


Progesterone vaginal gel estrogen as menopausal
Progesterone slow-release hormone therapy (see
intrauterine device Table 28-2)
Progesterone vaginal insert Used either alone
Medroxyprogesterone acetate or in combination
(MPA) with estrogen as oral
Megestrol acetate contraceptives (see
Levonorgestrel Table 28-1)
Levonorgestrel is used as
emergency contraception
a ter intercourse

Progesterone Mi epristone Used in combination with Nausea, vomiting, Vaginal bleeding, abdominal pain,
Antagonist misoprostol (see Chapter and diarrhea uterine cramps
32) or other prostaglandin
or the termination o
early pregnancy

Selective Progesterone Ulipristal Used as emergency Abdominal pain


Receptor Modulator contraception and sel -limited
headache

Oral Contraceptive See Table 28-1 Oral contraception Nausea, edema, Thromboembolic disease
and headache

Androgen Testosterone Therapeutic androgen Diminished ertility


Testosterone enanthate replacement Gynecomastia in high doses17-
Testosterone cypionate Treatment o AIDS-related alkylated androgens are associated
Testosterone undecanoate muscle wasting in men with hepatotoxicity
Testosterone patch
Testosterone gel
Testosterone buccal tablet
Anabolic steroids (see Figure 28-8)

Androgen Receptor Flutamide Treatment o metastatic


Antagonist Bicalutamide prostate cancer (see
Nilutamide Chapter 46)

5-Reductase Finasteride Used to treat benign Impotence is an in requent


Inhibitors Dutasteride prostatic hyperplasia side e ect
Gynecomastia has been reported

458
ACTH, Adrenal Steroids, CHAPTER

and Pharmacologyof the


Adrenal Cortex 29
T is chapter will be most use ul a er having a basic understanding o the material in
DRUGS INCLUDED IN THIS
Chapter 42, AC H, Adrenal Steroids, and the Pharmacology o the Adrenal Cortex in
Goodman & Gilmans T e Pharmacological Basis of T erapeutics, 12th Edition. In addi- CHAPTER
tion to the material presented here, the 12th Edition contains: Aminoglutethimide (CYTADREN)
A detailed discussion o the chemistry, actions, and regulation o the secretion o Corticorelin (ACTHREL)
adrenocorticotropic hormone (AC H) Cosyntropin (COSTROSYN, SYNACTHEN)
A description o the physiological unctions o the adrenocorticosteroids Etomidate (AMIDATE)
A detailed description o the pharmacological e ects o the adrenocorticosteroids Human corticotropin-releasing hormone (CRH)
A depiction o the chemical structures o the adrenocorticosteroids and their syn- Ketoconazole (NIZORAL; see also Chapter 43)
thetic derivatives
Metyrapone (METOPIRONE)
A discussion o inhibitors o the biosynthesis and action o the adrenocorticosteroids
Mi epristone (MIFUPREX,RU-486; see also
Chapter 28)
LEARNING OBJECTIVES
Mitotane (LYSODREN)
Understand the mechanisms o action and the physiological e ects o the
adrenocorticosteroids. Table 29-1 lists available preparations o adre-
Describe the di erences in the anti-in ammatory and sodium-retaining nocortical steroids and their syntheticanalogs
potencies o the glucocorticoids.
Describe the side e ects o prolonged therapy with glucocorticoids.
Understand the e ect o abrupt cessation o glucocorticoid therapy.

MECHANISMS OF ACTION OF ACTH, ADRENOCORTICOSTEROIDS, AND INHIBITORS OF THE SYNTHESIS OF


ADRENOCORTICOSTEROIDS
DRUG CLASS DRUGS MECHANISM OF ACTION
Adrenocorticotropic Hormone Cosyntropin Corresponds to residues 1-24 o ACTH; it stimulates the synthesis and release o
(ACTH) Analog adrenocortical hormones (see Figures 29-1 and 29-2)

Corticotropin-Releasing Corticorelin Binds to speci c membrane receptors on corticotropes in the anterior pituitary
Hormone (CRH) (see Figure 29-2)

Adrenocortical Steroids See Table 29-1 or Binds to speci c receptor proteins in target tissues to regulate the expression o
speci c drugs corticosteroid-responsive genes, thereby changing the levels and array o proteins
synthesized by the various target tissues (see Figure 29-3)

Inhibitors o the Biosynthesis o Ketoconazole In doses higher than those employed in anti ungal therapy, it is an e ective inhibitor
Adrenocortical Steroids o adrenal and gonadal steroidogenesis, primarily because o its inhibition o the
activity o CYP17 (17-hydroxylase)

Metyrapone Inhibitor o CYP11B1 (11-hydroxylase), which converts 11-deoxycortisol to cortisol

Etomidate Inhibitor o CYP11B1 (11-hydroxylase), which converts 11-deoxycortisol to cortisol

Aminoglutethimide Inhibits CYP11A1, which catalyzes the initial and rate-limiting step in the biosynthesis
o all steroids; also inhibits CYP11B1 and CYP19 (aromatase)

Adrenocorticolytic Agent Mitotane Cytolytic action is due to its metabolic conversion to a reactive acyl chloride by
adrenal mitochondria and subsequent reaction with cellular proteins

Antiglucocorticoids Mi epristone At high doses, it inhibits the glucocorticoid receptor

459
SECTION V Hormones and Hormone Antagonists

TABLE 29-1 Available Preparations of Adrenocortical Steroids and Their Synthetic Analogs
NONPROPRIETARY NAME TRADE NAME TYPE OF PREPARATION

Alclometasone dipropionate ACLOVATE Topical


Amcinonide Topical
Betamethasone acetate and sodium CELESTONE SOLUSPAN Injectable
phosphate
Beclomethasone dipropionate BECONASE AQ, QVAR40, QVAR80 Inhaled
DIPROLENE, DIPROLINE AF Topical
Betamethasone sodium phosphate CELESTONE Oral
Betamethasone valerate BETA-VAL, DERMABET, LUXIQ Topical
Budesonide ENTOCORT EC Oral
PULMICORT Inhaled
RHINOCORT Nasal
Ciclesonide ALVESCO, OMNARIS Inhaled
Clobetasol propionate CLOBEX, CORMAX EMBELINE, OLUX, TEMOVATE Topical
Clocortolone pivalate CLODERM Topical
Desonide DESONATE, DESOWEN, VERDESO Topical
Desoximetasone TOPICORT Topical
Dexamethasone Oral
Dexamethasone sodium phosphate MAXIDEX Ophthalmic
Di orasone diacetate PSORCON Topical
Fludrocortisone acetate* Oral
Flunisolide AEROBID, AEROSPAN HFA Inhaled
NASAREL Nasal
Fluocinolone acetonide DERMA-SMOOTHE/FS, FS, SYNALAR Topical
RETISERT Intravitreal implant
Fluocinonide LIDEX, LIDEX-E, VANOS Topical
Fluorometholone FML, FML FORTE Ophthalmic
Fluorometholone acetate FLAREX Ophthalmic
Flurandrenolide CORDRAN CORDRAN SP Topical
Halcinonide HALOG Topical
Hydrocortisone ALA-CORT, HYTONE, NUTRACORT, STIE-CORT, Topical
SYNACORT, TEXACORT

CORTEF Oral
Hydroxycortisone acetate MICORT-HC Topical
CORTIFOAM Rectal
Hydroxycortisone butyrate LOCOID Topical
Hydrocortisone cypionate CORTEF Oral
Hydrocortisone sodium succinate A-HYDROCORT, SOLU-CORTEF Injectable
Hydrocortisone valerate WESTCORT Topical
Methylprednisolone acetate MEDROL Oral
DEPO-MEDROL Injectable
(Continued)

460
ACTH, Adrenal Steroids, and Pharmacology of the Adrenal Cortex CHAPTER 2 9

NONPROPRIETARY NAME TRADE NAME TYPE OF PREPARATION

Methylprednisolone sodium succinate A-METHAPRED, SOLU-MEDROL Injectable


Mometasone uroate ASMANEX Inhaled
NASONEX Nasal
ELOCON Topical
Prednisolone Oral
Prednisolone acetate FLO-PRED Oral
OMNIPRED, PRED FORT, PRED MILD Ophthalmic
Prednisolone sodium phosphate ORAPRED, ORAPRED ODT, PEDIAPRED Oral
Prednisone Oral
Triamcinolone acetonide AZMACORT Inhaled
NASACORT AQ Nasal
KENOLOG Topical
KENOLOG-10, KENALOG-40, TRIESENCE, TRIVARIS Injectable
Triamcinolone hexacetonide ARISTOSPAN Injectable
AZMACORT Inhaled
*
Fludrocortisone acetate is intended or use as a mineralocorticoid.
Note: Topical preparations include agents or application to skin or mucous membranes in creams, solutions, ointments, gels, pastes ( or oral
lesions), and aerosols; ophthalmic preparations include solutions, suspensions, and ointments; inhalation preparations include agents or nasal
or oral inhalation.

ACh 5-HT NE GABA

Zo na
CYP 11B2
Glo me rulo s a IL-1
Hypo thalamus IL-2
Ang II Aldos te rone CRH ne urons IL-6
K+ TNF-

ACTH CRH
CYP 11B1 Immune Sys te m
Zo nae
Fas c ic ulata/ CYP 17 Ante rio r Pituitary Lymphocyte s
Re tic ularis Corticotrope s Ma cropha ge s /Monocyte s
Cortis ol Ne utrophils
ACTH

CYP 17 Adre nal Co rtex


Fa s cicula ta ce lls
Me dulla
DHEA

FIGURE 29-1 The adrenal cortex contains 3 anatomically and unction- Co rtis o l
ally distinct compartments. The major unctional compartments o the
adrenal cortex are shown, along with the steroidogenic enzymes that FIGURE 29-2 Overview o the hypothalamic-pituitary-
determine the unique pro les o corticosteroid products. Also shown adrenal (HPA) axis and the immune in ammatory network.
are the predominant physiological regulators o steroid production: Also shown are inputs rom higher neuronal centers that
angiotensin II (Ang II) and K+ or the zona glomerulosa and ACTH or regulate corticotropin-releasing hormone (CRH) secretion. +
the zona asciculata. The physiological regulator(s) o dehydroepian- indicates a positive regulator, indicates a negative regulator,
drosterone (DHEA) production by the zona reticularis are not known, + and indicates a mixed e ect, as or NE (norepinephrine). In
although ACTH acutely increases DHEA biosynthesis. addition, arginine vasopressin stimulates release o ACTH rom
corticotropes.

461
SECTION V Hormones and Hormone Antagonists

CBG CBG
S

S
IP

S HS P 70
GR
HS P 70 IP GR

HS P 90 HS P 90

nucle us

S S
Tra ns cription
GR GR
GRE GRE
ge ne

mRNA

cytopla s m
pro te in

Alte re d c e llular func tio n

FIGURE 29-3 Intracellular mechanism o action o the glucocorticoid receptor. The gure
shows the molecular pathway by which cortisol (labeled S) enters cells and interacts with the
glucocorticoid receptor (GR) to change GR con ormation (indicated by the change in shape o
the GR), induce GR nuclear translocation, and activate transcription o target genes. The example
shown is one in which glucocorticoids activate expression o target genes; the expression o cer-
tain genes, including proopiomelanocortin (POMC) expression by corticotropes, is inhibited by
glucocorticoid treatment. CBG, corticosteroid-binding globulin; GR, glucocorticoid receptor; GRE,
glucocorticoid-response elements in the DNA that are bound by GR, thus providing speci city to
induction o gene transcription by glucocorticoids; HSP70, the 70-Kd heat-shock protein; IP, the
56-Kd immunophilin; HSP90, the 90-Kd heat-shock protein; S, steroid hormone. Within the gene
are introns (gray) and exons (blue); transcription and mRNA processing leads to splicing and
removal o introns and assembly o exons into mRNA.

PHYSIOLOGIC FUNCTIONS AND PHARMACOLOGIC EFFECTS OF ADRENOCORTICAL STEROIDS


PHYSIOLOGIC FUNCTION
OR ORGAN AFFECTED MECHANISM
Carbohydrate and Protein Stimulates the liver to orm glucose rom amino acids and glycerol and to store glucose as glycogen
Metabolism Decreases glucose utilization in periphery; increases protein breakdown

Lipid Metabolism Redistributes body at; increased at in the back o neck, ace, and supraclavicular area with loss o at in the
extremities
Permissive acilitation o the lipolytic e ect o growth hormone and adrenergic agonists resulting in an
increase in ree atty acids

Electrolyte and Water Balance Mineralocorticoids act on the distal tubules and collecting ducts to enhance reabsorption o Na+, and to
enhance urinary excretion o K+ and H+
Decreases total body Ca2+ stores by lowering Ca2+ uptake rom the gut and increasing Ca2+ excretion by the kidney

Cardiovascular System Mineralocorticoid-induced changes in renal Na+ exacerbate congestive heart ailure
Enhanced vascular reactivity o other vasoactive substances such as norepinephrine and angiotensin II and
exacerbation o hypertension

Skeletal Muscle Corticosteroids are required or the normal unction o skeletal muscle; adrenocortical insuf ciency results in
muscle weakness; in aldosteronism, muscle weakness results rom hypokalemia; glucocorticoid excess results
in muscle wasting termed steroid myopathy
(Continued)
462
ACTH, Adrenal Steroids, and Pharmacology of the Adrenal Cortex CHAPTER 2 9

PHYSIOLOGIC FUNCTION
OR ORGAN AFFECTED MECHANISM

Central Nervous System Adrenocortical steroids exert direct e ects on the CNS such as e ects on mood, behavior, and brain
excitability; rank psychosis has also been noted

Formed Elements o Blood Corticosteroids exert e ects on erythrocytes and produce a decrease in circulating lymphocytes, monocytes,
basophils, and eosinophiles; certain lymphoid malignancies are destroyed by glucocorticoid treatment

Anti-In ammatory and Glucocorticoids pro oundly alter the immune response o lymphocytes and suppress in ammation; they are
Immunosuppressive Actions use ul in managing the in ammatory response to diseases o humoral immunity such as urticarial as well
as diseases o cellular immunity such as transplantation rejection (see Side Bar THE THERAPEUTIC USES OF
ADRENOCORTICAL STEROIDS)

CASE 29-1 THERAPEUTIC USES


OF ADRENOCORTICAL
A 53-year-old woman with rheumatoid arthritis involving the joints o both hands and
STEROIDS
both hips has developed an exacerbation o her disease in the past two weeks. Fi een
years ago she responded well to a short course o prednisone. For the past 3 months, Replacement therapy
she has been receiving methotrexate. She is started back on prednisone now or relie o Acute adrenal Insu ciency
her symptoms. Chronicadrenal insu ciency
a. Why was prednisone chosen? Congenital adrenal hyperplasia
able 29-2 shows the relative potencies and equivalent doses o representative cor- Nonendocrine diseases
ticosteroids. Prednisone is a more potent anti-in ammatory agent than cortisol, Rheumaticdisorders
but not as potent as dexamethasone. Prednisone has much less sodium-retaining
Renal disease
potency than udrocortisone, which has predominant mineralocorticoid activity.
Prednisone is considered to be intermediate in its duration o action compared to Allergicdisease
cortisol and dexamethasone, which are short- and long-acting, respectively. Bronchial asthma and other
o acilitate drug tapering and/or conversion to alternate-day therapy, the inter- pulmonaryconditions
mediate-acting glucocorticoid such as prednisone is pre erred to the longer-acting In ectious diseases
dexamethasone. Ocular diseases
(Continued)
Skin diseases
Gastrointestinal diseases
TABLE 29-2 Relative Potencies and Equivalent Doses of Representative Hepaticdiseases
Corticosteroids
Malignancies
ANTI-INFLAMMATORY Na -RETAINING DURATION
+
EQUIVALENT
COMPOUND POTENCY POTENCY OF ACTIONa DOSE (mg)b Cerebral edema
Cortisol 1 1 S 20
Sarcoidosis
Thrombocytopenia
Cortisone 0.8 0.8 S 25
Autoimmune destruction o
Fludrocortisone 10 125 I c
erythrocytes
Prednisone 4 0.8 I 5
Organ transplantation
Spinal cord injury
Prednisolone 4 0.8 I 5

6-Methylprednisolone 5 0.5 I 4

Triamcinolone 5 0 I 4

Betamethasone 25 0 L 0.75

Dexamethasone 25 0 L 0.75
a
S, short (ie, 8-12 hour biological t1/2); I, intermediate (ie, 12-36 hour biological I1/2); L, long
(ie, 36-72 hour biological t1/2).
b
These dose relationships apply only to oral or intravenous administration, as glucocorticoid
potencies may di er greatly ollowing intramuscular or intraarticular administration.
c
This agent is not used or glucocorticoid e ects.
463
SECTION V Hormones and Hormone Antagonists

T e use o glucocorticoids in a disease such as rheumatoid arthritis is largely


GENERAL PRINCIPLES
empirical and the use o glucocorticoids should ollow the principles in the Side Bar
FOR THE USE OF
GENERAL PRINCIPLES FOR HE USE OF GLUCOCOR ICOIDS.
GLUCOCORTICOIDS
b. What is the mechanism o action o adrenocortical steroids?
The decision to institute therapyrequires
care ul consideration o the relative risks T e glucocorticoids interact with receptors in the various target tissues that are
and bene ts in each patient. members o the nuclear receptor amily o transcription actors (see Figure 29-3).
In the target tissues, proteins are expressed that alter cellular unctions resulting
For anydisease and anypatient, the appro-
in the physiologic unctions and pharmacologic e ects shown in the able
priate dose to achieve a given therapeutic
PHYSIOLOGICAL FUNC IONS AND PHARMACOLOGICAL EFFEC S OF
ef ect must be determined bytrial and
HE ADRENOCOR ICAL S EROIDS.
error and must be reevaluated periodically.
Asingle dose o a glucocorticoid, even a
large one, is virtuallywithout harm ul CASE 29-2
ef ects and a short course o up to 1 week, A 43-year-man has been diagnosed with primary adrenal insu ciency (Addisons
in the absence o speci ccontraindications, disease). He requires replacement therapy with adrenocortical steroids.
is unlikelyto be harm ul.
a. What is the di erence between primary and secondary adrenal insuf ciency?
As the duration o glucocorticoid therapy
extends beyond 1 week, there are time and Adrenal insu ciency can result rom structural or unctional lesions o the adrenal
dose-related increases in the incidence o cortex (primary adrenal insu ciency or Addisons disease) or rom unctional or
disabling and potential lethal ef ects. structural lesions o the anterior pituitary or hypothalamus (secondary adrenal
insu ciency). See Figures 29-1 and 29-2 or the anatomical and unctional struc-
Except in the setting o replacement ture o the adrenal gland and or the regulation o corticosteroids rom the anterior
therapythe glucocorticoids are neither pituitary, respectively.
speci cnor curative.
Abrupt cessation o glucocorticoids b. Why is hydrocortisone chosen to treat this patient?
a ter prolonged therapyis associated Hydrocortisone is the active orm o cortisone (see Figure 29-4). Although some
with the risko adrenal insu ciency, patients with primary adrenal insu ciency can be maintained on hydrocortisone
which maybe atal. and liberal salt intake, most also require a mineralocorticoid such as udrocorti-
sone acetate to maintain water and electrolyte balance.
c. What are the major considerations in maintaining the proper dose o hydrocor-
tisone in this patient?
Glucocorticoid doses o en must be adjusted upward in patients who are also tak-
ing drugs that increase the plasma clearance o glucocorticoids (eg, phenytoin,
ri ampin, or barbiturates). T e stress o concurrent illness may also necessitate a
dose adjustment. During minor illnesses, the glucocorticoid dose should be dou-
TOXICITIES OF bled. I nausea or vomiting precludes the retention o oral medications, the patients
physician should be notif ed, and amily members should be trained to administer
ADRENOCORTICAL
parenteral dexamethasone and then seek medical attention immediately.
STEROIDS
Adrenal insu iciency with abrupt
withdrawal o therapy
Fluid and electrolyte abnormalities
(hypertension)
HO O
Metabolicchanges (hyperglycemia) 11 11-HS D2 11
Immune response (susceptibility
C C
11-HS D1
to in ection)
Co rtis o l Co rtis o ne
Risko pepticulcers
Ac tive Inac tive
Myopathy (binds to MR a nd GR) (binds to ne ithe r MR nor GR)
Behavioral changes FIGURE 29-4 Receptor-independent mechanism by which 11-hydroxysteroid dehydroge-
Cataracts nase con ers speci city o corticosteroid action. Type 2 11-hydroxysteroid dehydrogenase
Osteoporosis (11-HSD2) converts cortisol, which binds to both the mineralocorticoid receptor (MR) and the
glucocorticoid receptor (GR), to cortisone, which binds to neither MR nor GR, thereby protect-
Osteonecrosis (asepticnecrosis o ing the MR rom the high circulating concentrations o cortisol. This inactivation allows speci c
emoral head) responses to aldosterone in sites such as the distal nephron. The type 1 isozyme o 11-HSD
Growth retardation in children (11-HSD1) catalyzes the reverse reaction, which converts inactive cortisone to active cortisol in
such tissues as liver and at. Only ring C o the corticosteroid is depicted.
464
ACTH, Adrenal Steroids, and Pharmacology of the Adrenal Cortex CHAPTER 2 9

CASE 29-3
T e patient in Case 29-1 was treated with 40 mg o prednisone daily or 2 weeks and
then the dose was lowered to 20 mg daily. Attempts to urther lower the dose have
resulted in a are-up o her disease. A er 2 months o prednisone therapy, she became
uncom ortable with her increased weight and the development o roundness in her
ace. Despite warnings not to do so, she stopped the use o her prednisone.
a. What side e ects is she likely to experience?
T e most severe complication o steroid cessation is the li e-threatening condition
o acute adrenal insu ciency characterized by nausea, vomiting, abdominal pain,
hyponatremia, hyperkalemia, weakness, lethargy, and hypotension.
b. How could this situation possibly have been avoided?
Patients who are taking chronic glucocorticoid therapy should be cautioned against
discontinuing the therapy without a signif cant tapering o the dose. T ere is signi -
icant variation among patients with respect to the degree and duration o adrenal
suppression a er glucocorticoid therapy. Patients who have received supraphysi-
ological doses o glucocorticoids or a period o 2 to 4 weeks should be considered
to have some hypothalamic-pituitary-adrenal (HPA) suppression. A gradual reduc-
tion in their glucocorticoid dose is required. Many patients recover rom glucocor-
ticoid-induced HPA suppression within several weeks to months; however, in some
patients the time to recovery can be a year or longer.

CASE 29-4
A 22-year-old woman with the diagnosis o Crohns disease has been success ully
treated with sul asalazine (see Chapter 33). A recent exacerbation o her disease has
necessitated the addition o prednisone to her therapeutic regimen. She has noted,
since starting the prednisone that she has been nervous and has had di culty sleeping.
a. What are the potential toxicities o adrenocortical steroids to warn the
patient about?
T e toxicities o glucocorticoid therapy are listed in the Side Bar OXICI IES OF
ADRENOCOR ICAL S EROIDS. Behavioral changes are particularly common.
T ese might include nervousness, insomnia, changes in mood, and even overt
psychosis. Suicidal tendencies are not uncommon.
b. What are other treatment options available to this patient?
Budesonide, (see Chapter 33) a potent glucocorticoid that is inactivated by f rst-
pass hepatic metabolism, has diminished systemic side e ects commonly associated
with glucocorticoids. T e drug can be administered by a delayed-release capsule
that delivers steroid to the ileum and ascending colon, or as a retention enema or
the treatment o ulcerative colitis.

CASE 29-5
A 36-year-old woman is suspected o having Cushings syndrome. It is recommended
that she have a dexamethasone suppression test.
a. What is a dexamethasone suppression test and how is it interpreted?
o determine i patients with clinical mani estations o hypercortisolism have
biochemical evidence o increased cortisol biosynthesis, an overnight dexamethasone
suppression test has been devised in which patients are given 1 mg o dexamethasone
orally at 11 PM, and plasma cortisol is measured at 8 AM the ollowing morning.
Suppression o plasma cortisol to less than 1.8 g/dL suggests that the patient does
not have Cushings syndrome.

(Continued)
465
SECTION V Hormones and Hormone Antagonists

In a more ormal dexamethasone suppression test, baseline plasma cortisol con-


centrations are measured or 48 hours and then dexamethasone is administered
0.5 mg every 6 hours or 48 hours. T is dose markedly suppresses plasma cortisol
in normal patients, but does not suppress cortisol concentrations in patients with
Cushings syndrome.

CASE 29-6
A 68-year-old man is diagnosed with bacterial conjunctivitis. He is prescribed a combi-
nation eye drop that contains dexamethasone and an antibiotic.
a. Why use an adrenocorticoal steroid in an eye in ection?
Glucocorticoids can dramatically decrease in ammation in the conjunctiva when
administered topically into the conjunctival sac (see Chapter 47).
b. What are the concerns with the use o this combination product?
opical glucocorticoid therapy requently increases intraocular pressure in nor-
mal eyes and exacerbates intraocular hypertension in patients with antecedent
glaucoma. T e glaucoma is not always reversible on cessation o the glucocorti-
coid therapy. Intraocular pressure should be monitored when glucocorticoids are
applied to the eye or more than 2 weeks.

CASE 29-7
A 28-year-old woman is diagnosed with Cushings syndrome rom an ectopic AC H-
producing tumor. She is being treated with metyrapone.
a. What is metyrapone and how does it act?
Metyrapone is a relatively selective inhibitor o CYP11B (11-hydroxylase), which
converts 11-deoxycortisol to cortisol in the terminal reaction in the glucocorticoid
biosynthetic pathway. T us, the biosynthesis o cortisol is markedly impaired.
b. What are the untoward e ects o long-term metyrapone therapy?
Chronic administration o metyrapone can cause hirsutism, which results rom
increased synthesis o androgens upstream rom the enzymatic block. Metyra-
pone can also cause hypertension, which results rom increased concentrations o
11-deoxycortisol. Other side e ects include nausea, headache, sedation, and rash.

KEY CONCEPTS
T e rate o glucocorticoid secretion is determined by uctuations in the release
o AC H by the pituitary corticotropes (see Figure 29-2).
Adrenocortical steroids act by activating glucocorticoid receptors or mineralo-
corticoid receptors in various tissues (see Figure 29-3).
Glucocorticoids are administered in many ormulations (oral, parenteral, and
topical) or disorders that share an in ammatory or immunological basis
(see able 29-1).
Except or replacement therapy or adrenal insu ciency, glucocorticoids are
neither specif c nor curative.
Corticosteroid preparations vary widely in their anti-in ammatory e ects and
duration o action. Some corticosteroids possess mineralocorticoid activity
while others do not (see able 29-2).
Glucocorticoids have several potentially serious side e ects (see Side Bar OX-
ICI IES OF ADRENOCOR ICAL S EROIDS), which necessitates:
Limiting the dose to the minimal needed to achieve a therapeutic e ect.
Periodic reevaluation.
(Continued)
466
ACTH, Adrenal Steroids, and Pharmacology of the Adrenal Cortex CHAPTER 2 9

A single dose o a glucocorticoid is virtually without long-term harm ul e ects.


As the duration o glucocorticoid therapy increases beyond 1 week, adverse
e ects increase in a time- and dose-related manor.
Abrupt cessation o glucocorticoids a er prolonged therapy is associated with
the risk o adrenal insu ciency.

SUMMARY QUIZ

QUESTION 29-1 A 34-year-old man is diagnosed with secondary adrenal insu ciency.
He does not require the administration o a mineralocorticoid because
a. the adrenal medulla is intact.
b. the zona glomerulosa is intact.
c. the zona reticularis is intact.
d. the zona asciculata is intact.
e. 11-hydroxylase is not inhibited.

QUESTION 29-2 A 54-year-old man has been diagnosed with hypertension caused by his
excessive consumption o licorice. Licorice is a known inhibitor o 11-hydroxysteroid
dehydrogenase that is involved in the inactivation o
a. cortisol.
b. aldosterone.
c. AC H.
d. dehydroepiandrosterone.
e. progesterone.

QUESTION 29-3 A 34-year-old woman with Crohns disease has been taking predni-
sone daily or 1 year. She has been experiencing a thickness in the back o her neck.
T is is likely due to a(n)
a. edema.
b. increase in muscle growth.
c. redistribution o lipid.
d. protein deposition.
e. ectopic thyroid tissue.

QUESTION 29-4 A 63-year-old man with type 2 diabetes and osteoarthritis in his le
knee is given an intra-articular injection o a glucocorticoid to decrease pain in his
knee. An elevation in his blood sugar or the next 3 days is likely due to an e ect o the
glucocorticoid on
a. diminished peripheral glucose utilization.
b. increased protein breakdown.
c. activation o lipolysis.
d. increased glucose synthesis in the liver.
e. all o the above.
QUESTION 29-5 A 48-year-old woman with a chronic pulmonary disease is being
treated with an inhaled glucocorticoid product. She is likely to experience which o the
ollowing side e ects?
a. Bu alo hump
b. Acute adrenal insu ciency
c. Hyperglycemia
d. Oral candidiasis
e. Hepatitis
467
SECTION V Hormones and Hormone Antagonists

SUMMARY QUIZ ANSWER KEY


QUESTION 29-1 Answer is b. T e zona glomerulosa is the site o aldosterone synthe-
sis and secretion (see Figure 29-1). In secondary adrenal insu ciency, which usually
involves pituitary or hypothalamic dys unction, mineralocorticoid biosynthesis is
preserved.
QUESTION 29-2 Answer is a. 11-Hydroxysteroid dehydrogenase catalyzes the
conversion o cortisol (active) to cortisone (inactive) (see Figure 29-4). Aldosterone is
resistant to the enzymes actions. T e inhibition o the enzyme by licorice results in a
state o mineralocorticoid excess resulting in hypertension.
QUESTION 29-3 Answer is c. A signif cant untoward e ect o the pharmacological
use o glucocorticoids is the dramatic redistribution o body at. T ere is increased at
in the back o the neck (bu alo hump), ace (moon ace), and the supraclavicular
area, coupled with a loss o at in the extremities. One hypothesis or this e ect is that
truncal adipocytes are more sensitive than peripheral adipocytes to the elevated insulin
concentrations that result rom glucocorticoid-induced hyperglycemia.
QUESTION 29-4 Answer is e. Corticosteroids pro oundly a ect carbohydrate and
protein metabolism, which can be viewed as protecting glucose-dependent tissues such
as the heart and brain. T ey stimulate the liver to orm glucose rom amino acids and
glycerol, and to store glucose as liver glycogen. In the periphery, glucocorticoids dimin-
ish glucose utilization, increase protein breakdown, increase the synthesis o glutamine,
and activate lipolysis, thereby providing amino acids and glycerol or gluconeogenesis.
T e net result is to increase blood glucose concentrations.
QUESTION 29-5 Answer is d. In many patients, inhaled steroids can either reduce
the need or oral corticosteroids or replace them altogether. T e oral bioavailability
o inhaled steroids vary considerably and those with low bioavailability will have less
steroid-induced side e ects. However, the deposition o steroid in the oral cavity may
cause the growth o Candida. T e incidence o this e ect can be reduced by using
a product such as ciclesonide or beclomethasone which are inactive prodrugs that
are activated at their site o action by lung esterases. T ere ore, they are less likely to
adversely a ect the oral cavity.

SUMMARY: ACTH, ADRENAL STEROIDS, AND PHARMACOLOGY OF THE ADRENAL CORTEX


TOXICITIES
CLASS AND SUBCLASSES NAMES CLINICAL USES COMMON UNIQUE; CLINICALLY IMPORTANT
Adrenocorticotropic Cosyntropin Testing the integrity o the
Hormone (ACTH) Analog hypothalamic-pituitary axis

Corticotropin-Releasing Corticorelin Diagnostic testing o Flushing i drug is


Hormone (CRH) hypothalamic-pituitary axis administered as an
intravenous bolus

Adrenocortical Steroids See Table 29-1 or See Side Bar THERAPEUTIC Acute adrenal insuf ciency i
speci c drugs USES OF ADRENOCORTICAL withdrawal o chronic therapy
STEROIDS or the therapeutic is not tapered
use o adrenocortical steroids Pseudotumor cerebri rarely
or selected speci c examples, associated with withdrawal
and the Side Bar GENERAL o therapy
PRINCIPLES FORTHE USE See Side Bar TOXICITIES OF
OF GLUCOCORTICOIDS or ADRENOCORTICAL STEROIDS
principles related to the use o or side e ects o adrenocortical
adrenocortical steroids steroid therapy

(Continued)

468
ACTH, Adrenal Steroids, and Pharmacology of the Adrenal Cortex CHAPTER 2 9

TOXICITIES
CLASS AND SUBCLASSES NAMES CLINICAL USES COMMON UNIQUE; CLINICALLY IMPORTANT

Inhibitors o the Ketoconazole Anti ungal agent used to Hepatic dys unction
Biosynthesis o inhibit steroid biosynthesis in Inhibition o CYPs may lead to
Adrenocortical Steroids patients with hypercortisolism serious drug interactions

Metyrapone Used to impair the biosynthesis Nausea, headache, Precipitation o acute adrenal
o cortisol in patients with sedation, rash insuf ciency
adrenal neoplasms or tumors Chronic administration may cause
producing ACTH ectopically hirsutism

Etomidate Used to treat hypercortisolism Sedation


by intravenous administration
in patients who cannot take
oral medication

Aminoglutethimide Inhibits biosynthesis o cortisol;


recently withdrawn rom the
market by the manu acturer

Adrenocorticolytic Agent Mitotane Used to treat inoperable Severe GI disturbances


adrenocortical carcinoma and ataxia

Antiglucocorticoids Mi epristone Investigational use in high See Chapter 28


doses to treat patients with
inoperable causes o cortisol
excess (see Chapter 28)

469
CHAPTER
Endocrine Pancreas and
30 Pharmacotherapyof Diabetes
Mellitus and Hypoglycemia
T is chapter will be most use ul a er having a basic understanding o the material in
DRUGS INCLUDED IN
Chapter 43, Endocrine Pancreas and Pharmacotherapy o Diabetes Mellitus and Hypo-
glycemia in Goodman & Gilmans T e Pharmacological Basis of T erapeutics, 12th THIS CHAPTER
Edition. In addition to the material presented here, the 12th Edition includes: Acarbose (PRECOSE, others)
A detailed discussion o the regulation o blood glucose Alogliptin (NESINA)
A description o pancreatic islet cell physiology and the synthesis and processing Bromocriptine (CYCLOSET,not yet available
o insulin in the United States for treatment
A discussion o the signaling pathway that is activated by insulin resulting in its of diabetes)
e ects on target cells Colesevelam(WELCHOL)
A description o the pathogenesis o type 1 and type 2 diabetes Diazoxide (PROGLYCEM)
Drugs that are in development or type 2 diabetes Exenatide (BYETTA)
able 43-5 Properties o Insulin Preparations Gliclazide (DIAMICRON, others, unavailable
in the United States)
LEARNING OBJECTIVES Glimepiride (AMARYL)
Understand the mechanisms o action o insulin and the oral antidiabetic drugs. Glipizide (GLUCOTROL, others)
Describe the components or management o the diabetic patient including the Glucagon
goals o therapy. Glyburide (Glibenclamide; MICRONASE,
Describe the pharmacotherapeutic options or the treatment o patients with DIABETA, others)
type 1 or type 2 diabetes. Insulin aspart (NOVOLOG)
Learn about the adverse e ects o insulin and the oral antidiabetic drugs. Insulin determir (LEVEMIR)
Understand the treatment o hypoglycemia. Insulin glargine (LANTUS)
Insulin glulisine (APIDRA)
MECHANISMS OF ACTION INSULIN AND ORAL ANTIDIABETIC DRUGS Insulin lispro (HUMALOG)
DRUG CLASS DRUGS MECHANISM OF ACTION Insulin protamine hagedorn
Insulin Insulin Insulin binds to a plasma membrane receptor that initiates
(NPH, insulin isophane)
a cascade o signaling events, which results in glucose Liraglutide (VICTOZA)
utilization and glycogen synthesis (see Figure 30-1) Metformin (GLUCOPHAGE, others)
Biguanides Met ormin Increases the activity o AMP-dependent protein kinase Miglitol (GLYSET)
(AMPK); activated AMPKstimulates atty acid oxidation,
Nateglinide (STARLIX)
glucose uptake, and nonoxidative metabolism, and it
reduces lipogenesis and gluconeogenesis; the net result Pioglitazone (ACTOS)
is increased glycogen storage in skeletal muscle, lower Pramlintide (SYMLIN)
rates o hepatic glucose production, increased insulin
sensitivity, and lower blood glucose concentrations Repaglinide (PRANDIN)
Rosiglitazone (AVANDIA)
Insulin Glyburide Stimulate insulin release by binding to a speci c site on
Secretagogues Glipizide the cell KATP channel complex and inhibiting its activity Saxagliptin (ONGLYZA)
Sul onylureas Gliclazide (see Figure 30-2) Sitagliptin (JANIVIA)
Glimepiride
Vidagliptin (GALVUS, ZOMELIS, JAIRA, not
Insulin Repaglinide Stimulates insulin release by closing KATP channels in available in the United States)
Secretagogues Nateglinide pancreatic cells (see Figure 30-2)
Voglibose (VOGLIB, not available in the
Nonsul onylureas
United States)
Thiazolidinediones Rosiglitazone Ligands or the peroxisome proli eration activating
Pioglitazone receptor (PPAR) that are involved in the regulation o
genes related to glucose and lipid metabolism
(Continued)

470
Endocrine Pancreas and Pharmacotherapy CHAPTER 3 0

DRUG CLASS DRUGS MECHANISM OF ACTION

GLP-1 Agonists Exenatide Glucagon-like peptide 1 (GLP-1) agonists that activate


Liraglutide the GLP-1 receptor, activate the cAMP-PKA pathway, and
initiate signals through PKC and P13K(see Figure 30-2)

Dipeptidyl Sitagliptin Inhibit the DPP-4 enzyme that is critical or the


Peptidase-4 Saxagliptin inactivation o GLP-1 and GIP (glucose-dependent
(DPP-4) Inhibitors Vidagliptin insulinotropic polypeptide); thus the AUC o GLP-1 and
Alogliptin GIP is increased when their secretion is stimulated by a
meal (see Figure 30-3)

-Glucosidase Acarbose Reduces intestinal absorption o carbohydrates rom the


Inhibitors Miglitol GI tract and blunts the rate o rise o postprandial glucose
Voglibose by inhibiting -glucosidase in the intestinal brush border

Amylin Agonists Pramlintide Synthetic orm o amylin; it acts through the amylin
receptor in the brain causing reduction in glucagon
release, delayed gastric emptying, and satiety

Bile Acid Colesevelam Mechanism by which bile acid binding and removal rom
Sequestrants the enterohepatic circulation lowers blood glucose has
not been established

Agents Used to Glucagon Interacts with GPCR receptor on target cells; the e ects o
Treat Hypoglycemia glucagon on the liver are mediated by cAMP

Diazoxide Hyperglycemia results rom inhibition o insulin secretion


(see Figure 30-2)

Ins ulin
Ins ulin
re ce ptor Ins ulin
Glucos e
cave ola extra c e llula r
flotillin Cav intra c e llula r

Y- -Y P-Y- -Y-P AP S Tra ns loca tion
of GLUT 4 Glucos e
CbI CAP Hexokina s e
S hc CrkII
GTP GLUT4
exch C3G G-6-P
TC10
Ga b1 GDP
P KB
IRS prote ins (Akt) a P KC
1-4
MAP Intra ce llula r
P IP 3 P DK1
kina s e (me mbra ne ) ve s icle
P I3-kina s e

Ce ll growth, Prote in Glycoge n Me ta bolic


diffe re ntia tion, synthe s is synthe s is pa thways
s urviva l

FIGURE 30-1 Pathways o insulin signaling. The binding o insulin to its plasma membrane receptor activates a cascade o downstream signaling
events. Insulin binding activates the intrinsic tyrosine kinase activity o the receptor dimer, resulting in the tyrosine phosphorylation (Y-P) o the recep-
tors subunits and a small number o speci c substrates (light blue shapes): the insulin receptor substrate (IRS) proteins, Gab-1 and SHC; within the
membrane, a caveolar pool o insulin receptor phosphorylates caveolin (Cav), APS, and Cbl. These tyrosine-phosphorylated proteins interact with
signaling cascades via SH2 and SH3 domains to mediate the e ects o insulin, with speci c e ects resulting rom each pathway. In target tissues such
as skeletal muscle and adipocytes, a key event is the translocation o the Glut4 glucose transporter rom intracellular vesicles to the plasma membrane;
this translocation is stimulated by both the caveolar and noncaveolar pathways. In the noncaveolar pathway, the activation o PI3Kis crucial, and PKB/
Akt (anchored at the membrane by PIP3) and/or an atypical orm o PKC is involved. In the caveolar pathway, caveolar protein otillin localizes the
signaling complex to the caveola; the signaling pathway involves a series o SH2 domain interactions that add the adaptor protein CrkII, the guanine
nucleotide exchange protein C3G, and small GTP-binding protein, TC10. The pathways are inactivated by speci c phosphoprotein phosphatases (eg,
PTB1B). In addition to the actions shown, insulin also stimulates the plasma membrane Na+,K+-ATPase by a mechanism that is still being elucidated;
the result is an increase in pump activity and a net accumulation o K+ in the cell. Abbreviations: aPKC, atypical iso orm o protein kinase C; APS, adap-
tor protein with PH and SH2 domains; CAP, Cbl associated protein; CrkII, chicken tumor virus regulator o kinase II; GLUT4, glucose transporter 4; Gab-1,
Grb-2 associated binder; MAP kinase, mitogen-activated protein kinase; PDK, phosphoinositide-dependent kinase; PI3 kinase, phosphatidylinositol-
3-kinase; PIP3, phosphatidylinositol trisphosphate; PKB, protein kinase B(also called Akt); Y, tyrosine residue; Y-P, phosphorylated tyrosine residue.
471
SECTION V Hormones and Hormone Antagonists

ATP-s e ns itive
Kir6.2 K+ K+ cha nne l
Glucos e
Dia zoxide Ca 2 +
S UR1 S ulfonylure a /me glitinide Incre tins
(a c ting via
GP CR-G s -AC)
GLUT De pola riza tion
K+
glucokina s e K+
Ca 2 +
G-6-P ATP
cAMP
S tore d
ins ulin
+
Me ta bolis m
Exocytos is
Plas ma
ins ulin

Pa ncre a tic ce ll

FIGURE 30-2 Regulation o insulin secretion rom a pancreatic cell. The pancreatic cell in a resting state
( asting blood glucose) is hyperpolarized. Glucose, entering via GLUT transporters (primarily GLUT1 in humans,
GLUT2 in rodents), is metabolized and elevates cellular ATP, which inhibits K+ entry through the KATP channel; the
decreased K+ conductance results in depolarization, leading to Ca2+-dependent exocytosis o stored insulin. The
KATP channel, actually a hetero-octamer composed o SUR1 and Kir 6.2 subunits, is the site o action o several
classes o drugs: ATP binds to and inhibits Kir6.2; sul onylureas and meglitinides bind to and inhibit SUR1; all
3 agents thereby promote insulin secretion. Diazoxide and ADP-Mg 2+ (low ATP) bind to and activate SUR1,
thereby inhibiting insulin secretion. Incretins enhance insulin secretion.

Inta ct incre tins Incre tin


me ta bolite s
GLP-1 [7-36]

GIP-1 [1-42] Ca pilla rie s GLP-1 [9-36]

GIP-1 [3-42]

DP P-4
Me a l DP P-4 inhibitor Me a l inhibitor
1
-
P
P
I
L
G
G
No drug No drug
t
c
t
c
a
t
a
n
t
I
n
I
Time Time

FIGURE 30-3 Pharmacological e ects o DDP-4 inhibition. DPP-4, an ectoenzyme located on the luminal side
o capillary endothelial cells metabolizes the incretins, glucagon-like peptide 1 (GLP-1), and glucose-dependent
insulinotropic polypeptide (GIP), by removing the 2 N-terminal amino acids. The target or DPP-4 cleavage is a
proline or alanine residue in the second position o the primary peptide sequence. The truncated metabolites
GLP-1 [9-36] and GIP [3-42] are the major orms o the incretins in plasma and are inactive as insulin secreta-
gogues. Treatment with a DPP-4 inhibitor increases the concentrations o intact GLP-1 and GIP.

472
Endocrine Pancreas and Pharmacotherapy CHAPTER 3 0

TABLE 30-1 Criteria for the Diagnosis of Diabetes


Symptoms o diabetes plus random blood glucose concentration 11.1 mM (200 mg/dL)a or
Fasting plasma glucose 7.0 mM (126 mg/dL)b or
Two-hour plasma glucose 11.1 mM (200 mg/dL) during an oral glucose tolerance testc
HbA1c 6.5%
a
Random is de ned as without regard to time since the last meal.
b
Fasting is de ned as no caloric intake or at least 8 h.
c
The test should be per ormed using a glucose load containing the equivalent o 75 g
anhydrous glucose dissolved in water; this test is not recommended or routine clinical use.
Note: In the absence o unequivocal hyperglycemia and acute metabolic decompensation,
these criteria should be con rmed by repeat testing on a di erent day.
Data adapted from Diabetes Care, 2010; 33:S62S69.

CASE 30-1
A 10-year-old girl is diagnosed with type 1 diabetes. She will start on insulin therapy.
a. How is the diagnosis o type 1 diabetes made?
T e diagnosis o diabetes mellitus is currently based on the correlation o diabetes-
speci c complications with a particular level o glycemia, that is, the level o glyce-
mia at which a diabetes-speci c complication like retinopathy begins to appear. T e
criterion or the diagnosis o diabetes is shown in able 30-1.
b. How should diabetes in this patient be managed? What are the goals o therapy?
T erapy o diabetes has to be individualized or each patient. Figure 30-4 shows the
components o sound diabetes management. able 30-2 lists the indexes o therapy
and the goals or diabetic patients in general.
c. What is hemoglobin A1c (A1C) and how does it di er rom asting blood glucose?
Exposure o proteins to an elevated glucose concentration produces nonenzymatic
glycation o proteins including hemoglobin (Hb). T us, the level o HbA1c (A1C)
represents the average glucose concentration to which the Hb has been exposed
over the past 2 to 3 months.
d. What is the mechanism o action o insulin?
T e pancreatic cell is a highly specialized cell that quickly senses and responds
to the external glucose concentration. T e transport o glucose into the pancreatic
cell via a acilitative transporter (see Figure 30-2) results in increased intracellular
calcium which, in turn, results in exocytotic release o insulin rom storage vesicles.
Once in the circulation, insulin binds, in various tissues, to its plasma membrane
receptor (see Figure 30-1) that activates a cascade o downstream intracellular
signaling events. T e tissues that are considered critical or the regulation o blood
glucose are liver, skeletal muscle, and at.
(Continued)

Ma na ge me nt of
dia be te s

Glyce mic control Tre a t a s s ocia te d S cre e n for/ma na ge


Die t/life s tyle conditions complica tions
Exe rcis e Dys lipide mia of dia be te s
Me dica tion Hype rte ns ion Re tinopa thy
Obe s ity Ca rdiova s cula r
CV dis e a s e dis e a s e
Ne uropa thy
Ne phropa thy
Othe r complica tions

FIGURE 30-4 Components o comprehensive diabetes care.


473
SECTION V Hormones and Hormone Antagonists

TABLE 30-2 Goals of Therapy in Diabetes


INDEX GOALa
Glycemic controlb

A1C <7.0%c

Preprandial capillary plasma glucose 3.9-7.2 mmol/L (70-130 mg/dL)

Peak postprandial capillary plasma glucose 10.0 mmol/L (<180 mg/dL)d

Blood pressure <130/80

Lipidse

Low-density lipoprotein <2.6 mmol/L (<100 mg/dL)

High-density lipoprotein >1.1 mmol/L (>40 mg/dL)g

Triglycerides <1.7 mmol/L (<150 mg/dL)


a
As recommended by the ADA, goals should be individualized or each patient. Goals may be
dif erent or certain patient populations.
b
A1C is primary goal.
c
While the ADA recommends an A1C <7.0% in general, in the individual patient it recommends
an appropriate goal or the individual patient be based on age, duration o diabetes, li e
expectancy, other medical conditions, and cardiovascular disease.
d
One to two hours a ter beginning o a meal.
e
In decreasing order o priority.
In individuals with coronary artery disease, an LDL <1.8 mmol (70 mg/dL) is the goal.
g
For women, some suggest a goal that is 0.25 mmol/L (10 mg/dL) or higher.
Data adapted rom Diabetes Care 33:S11, 2010.

e. What are the options with insulin therapy with this patient?
Insulin preparations are classif ed according to their duration o action into
short-acting and long-acting (see able 45-3 in Goodman and Gilmans T e Phar-
macological Basis of T erapeutics, 12th Edition). In most patients, insulin-replace-
ment therapy includes long-acting insulin (basal) and short-acting
insulin to provide postprandial needs. Most insulin is injected subcutaneously.
Short-acting insulins are the only orm o the hormone used in subcutaneous
in usion pumps.

CASE 30-2
A 53-year-old man is diagnosed with type 2 diabetes. His doctor ollows the treatment
algorithm or the management o type 2 diabetes as shown in Figure 30-5. T e patients
initial A1C is 8.1%. Af er attempts at weight reduction, increased physical activity, and
met ormin pharmacotherapy his A1C is 7.8%.
a. How is the diagnosis of type 2 diabetes made?
T e diagnosis o type 2 diabetes is made using the same criteria as shown in able 30-1.
b. What are the goals of therapy in a patient with type 2 diabetes?
T erapy o type 2 diabetes has to be individualized or each patient. Figure 30-4
shows the components o sound diabetes management and is applicable to type 1
and type 2 diabetes patients. able 30-2 lists the indexes o therapy and the goals or
diabetic patients in general.
(Continued)

474
Endocrine Pancreas and Pharmacotherapy CHAPTER 3 0

Type 2 Dia be te s

As s e s s
A1C

Dia be te s Educa tion


Me dica l Nutrition The ra py
P hys ica l Activity
Me tformin

S cre e n for Complica tions Re a s s e s s Tre a t Co-morbiditie s


Re tina l exa m A1C Dys lipide mia
Urine microa lbuminuria Hype rte ns ion
Ne uropa thy exa m Me tformin + S e cond Age nt Obe s ity
Va s cula r eva lua tion CV dis e a s e
Re a s s e s s
A1C

Me tformin Me tformin
+ +
2 Ora l Age nts Ins ulin

Re a s s e s s
A1C

FIGURE 30-5 Treatment algorithm or management o type 2 diabetes mellitus. Patients


diagnosed with type 2 diabetes, either by asting glucose, oral glucose tolerance testing, or A1C,
should have diabetes education that includes instruction on medical nutrition therapy and physical
activity. Most patients newly diagnosed with type 2 diabetes have had subclinical or undiagnosed
diabetes or many years previously and should be evaluated or diabetic complications (retinal
examination, test or excess protein or albumin excretion in the urine, and clinical evaluation or
peripheral neuropathy and vascular insuf ciency); common comorbidities (hypertension and
dyslipidemia) should be treated. Met ormin is the consensus rst line o therapy and should be
started at the time o diagnosis. Failure to reach the glycemic target, generally an A1C 7% within
3-4 months, should prompt the addition o a second oral agent. Rein orce li estyle interventions at
every visit and check A1C every 3 months. Treatment may escalate to met ormin plus 2 oral agents
or met ormin plus insulin, i necessary.

c. What are the options to lower his A1C in addition to li estyle modif cations?
One option that is commonly ollowed is to add a sul onylurea drug. Other
options include the insulin secretagogues that are nonsul onylureas or the
thiazolidinediones.
d. What are the untoward e ects o sul onylurea drugs?
T e most severe untoward e ect o the sul onylurea drugs is hypoglycemia.
Weight gain is also a common side e ect o this class o drugs. Less common
e ects include nausea, vomiting, cholestatic jaundice, blood dyscrasias, and
dermatological reactions.

CASE 30-3
T e patient in Case 30-2 has an A1C that is 7.5% a er pharmacotherapy with met ormin
plus a sul onylurea.
a. What are the options with this patient?
T e progressive insulin de ciency in type 2 diabetes o en makes it increasingly
dif cult to achieve tight glycemic control solely with oral antihyperglycemic agents
(see Figure 30-5). One option would be the addition o a thiazolidinedione drug
such as rosiglitazone or pioglitazone. I this does not achieve the desired goal then
the addition o insulin is another option.
(Continued)

475
SECTION V Hormones and Hormone Antagonists

b. What are the untoward e ects o the thiazolidinediones?


T e most common side e ect o the thiazolidinediones is weight gain and edema.
O greatest concern are the increased incidence o congestive heart ailure and the
risk o cardiovascular events (myocardial in arction or stroke). reatment with
thiazolidinediones may increase the risk o bone racture in women.
I this patient already has a risk o cardiovascular disease, a better choice might be
to add insulin to his diabetic regimen.
c. What type o insulin should be used?
T e di erent types o insulin are shown in able 43-5 in Goodman and Gilmans
T e Pharmacological Basis of T erapeutics, 12th Edition . Patients with type 2 diabetes
are usually best treated with long-acting insulin such as glargine. Glargine has a
sustained peakless absorption pro le and provides better once-daily insulin coverage
than NPH insulin. Glargine also has a lower risk o hypoglycemia, particularly over-
night, compared to NPH insulin.
d. What other agents can be added?
I the addition o basal insulin still does not reach the target A1C, the addition o a
glucagon-like peptide-1 (GLP-1) receptor agonist may be bene cial. A GLP-1 agonist
such as exenatide or liraglutide stimulates insulin secretion, inhibits glucagon
release, delays gastric emptying, reduces ood intake, and normalizes asting and
postprandial insulin secretion. An alternative is a dipeptidyl peptidase-4 (DPP-4)
inhibitor which results in a rise in the endogenous GLP-1 concentration by inhibit-
ing its plasma clearance.

CASE 30-4
A 49-year-old woman without diabetes is diagnosed with recurring hypoglycemia. T is
is particularly troublesome because she awakes every morning with a headache.
a. What are the options or treating hypoglycemia?
wo agents are available or the treatment o hypoglycemia: glucagon and diazoxide.
b. What is the mechanism o action o glucagon?
Glucagon is used to treat severe hypoglycemia, particularly in diabetic patients
when the patient cannot sa ely consume oral glucose and intravenous glucose is not
available. Glucagon interacts with a GPCR on the plasma membrane o target cells.
c. What is the mechanism o action o diazoxide?
Diazoxide is an antihypertensive with potent hyperglycemic actions when given
orally. T e hyperglycemic action o diazoxide is primarily through the inhibition o
insulin secretion (see Figure 30-2).

KEY CONCEPTS
Insulin is the primary treatment o type 1 diabetes.
T ere are many di erent types o insulin (see able 45-3 in Goodman and
Gilmans T e Pharmacological Basis of T erapeutics, 12th Edition), including
subcutaneous insulin pumps.
Short-acting insulin is the only orm o the hormone used in insulin pumps.
Some drugs may promote hyperglycemia or hypoglycemia (see able 30-3).
ype 2 diabetes is a complex disease that requires multiple orms o treatment
(see Figure 30-5) including diet, exercise, and medications.
T e goals o therapy or type 1 and type 2 diabetes are shown in able 30-2.
Insulin is the cornerstone o treatment o hyperglycemia in hospitalized patients.
(Continued)
476
Endocrine Pancreas and Pharmacotherapy CHAPTER 3 0

TABLE 30-3 Some Drugs That May Promote Hyperglycemia or Hypoglycemia


HYPERGLYCEMIA HYPOGLYCEMIA
Glucocorticoids Adrenergic antagonists
Antipsychotics (atypical, others) Ethanol
Protease inhibitors Salicylates
Adrenergic agonists Non-steroidal anti-in ammatory drugs
Diuretics (thiazide, loop) Pentamidine
Hydantoins (phenytoin, others) ACE inhibitors
Opioids ( entanyl, morphine, others) Lithium chloride
Diazoxide Theophylline
Nicotinic Acid Bromocriptine
Pentamidine Mebendazole
Epinephrine
Inter erons
Amphotericin B
Asparaginase
Acamprosate
Basiliximab
Thyroid hormones

Met ormin is the consensus rst line o therapy or type 2 diabetes and should
be started at the time o diagnosis.
T e most common and serious adverse e ect o diabetes therapy is hypoglyce-
mia. T e more vigorous the attempt to achieve euglycemia, the more requent
are episodes o hypoglycemia.

SUMMARY QUIZ

QUESTION 30-1 A 45-year-old woman with type 2 diabetes is noticing an increase in


her basal insulin dose needed to achieve the same e ect. T is is likely because o
a. increased insulin secretion.
b. insulin resistance in target tissues.
c. improper ormulation o her insulin.
d. increased glucagon secretion.
e. increased consumption o carbohydrates.

QUESTION 30-2 A 69-year-old man with type 2 diabetes is hospitalized because o a


knee replacement. His blood sugar postsurgery is 215 mg/dL. His hyperglycemia is best
managed in the hospital by
a. increasing his met ormin dose.
b. increasing his sul onylurea dose.
c. adding a GLP-1 agent.
d. decreasing his ood intake.
e. subcutaneous or intravenous insulin.

QUESTION 30-3 A 56-year-old woman with type 2 diabetes is being treated with
met ormin and glimepiride. T ese two drugs act synergistically because glimepiride
a. decreases the clearance o met ormin.
b. acts to increase glycogen stores in the liver.
c. stimulates insulin release by binding to a speci c site on cells.
d. increases the activity o AMP-dependent protein kinase (AMPK).
e. decreases glucose reabsorption in the kidney.
477
SECTION V Hormones and Hormone Antagonists

QUESTION 30-4 In the patient in Question 30-3, the met ormin and glimepiride act
synergistically because met ormin
a. decreases the clearance o glimepiride.
b. acts to decrease glycogen stores in the liver.
c. stimulates insulin release by binding to a speci c site on cells.
d. increases the activity o AMP-dependent protein kinase (AMPK).
e. decreases glucose reabsorption in the kidney.

QUESTION 30-5 A 48-year-old man with type 2 diabetes is treated with glargine insulin
once daily and lispro insulin with each meal. T e main clinical di erence in these 2 types
o insulin is in their
a. e ective duration o action.
b. the way they are administered.
c. storage.
d. metabolism by the liver.
e. penetration into the central nervous system.

QUESTION 30-6 T e patient in Question 30-3 has developed onychomycosis and she
is now being treated with f uconazole. She is at increased risk o hypoglycemia because
f uconazole can
a. decrease the hepatic metabolism o met ormin.
b. decrease the hepatic metabolism o glimepiride.
c. increase glucose uptake by skeletal muscle.
d. inhibit insulin secretion.
e. inhibit the action o glucagon.

SUMMARY QUIZ ANSWER KEY

QUESTION 30-1 Answer is b. Insulin sensitivity is a quanti able parameter that is


measured as the amount o glucose cleared rom the blood in response to a dose o
insulin. T e ailure o normal amounts o insulin to elicit the expected response is
re erred to as insulin resistance. T e major insulin-responsive tissues are skeletal muscle,
at, and liver. Insulin resistance in muscle and at is generally marked by a decrease in
the transport o glucose rom the circulation.

QUESTION 30-2 Answer is e. Insulin is the cornerstone o treatment o hyperglycemia


in hospitalized patients. Intravenous insulin is the treatment o choice, but in patients
who are more stable, subcutaneous insulin regimens using a combination o basal and
prandial insulin is the standard.

QUESTION 30-3 Answer is c. Sul onylureas such as glimepiride stimulate insulin


release by binding to a speci c site on the cell KA P channel complex (the sul onylurea
receptor, SUR) and inhibiting its activity (see Figure 30-2).

QUESTION 30-4 Answer is d. Met ormin increases the activity o the AMP-dependent
protein kinase (AMPK). Activated AMPK stimulates atty acid oxidation, glucose
uptake and nonoxidative metabolism, and reduced lipogenesis and gluconeogen-
esis. T e net result is increased glycogen storage in skeletal muscle, lower rates o
hepatic glucose production, increased insulin sensitivity, and lower blood glucose
concentrations.

QUESTION 30-5 Answer is a. See able 45-3 in Goodman and Gilmans T e Pharma-
cological Basis of T erapeutics, 12th Edition . T e major clinical di erence between
glargine and lispro insulin is in their e ective duration o action.
478
Endocrine Pancreas and Pharmacotherapy CHAPTER 3 0

QUESTION 30-6 Answer is b. Hypoglycemia may be more requent in patients taking


a sul onylurea and also taking an azole anti ungal agent. Glimepiride is metabolized by
CYP2C9, which is inhibited by f uconazole.

SUMMARY: DRUGS USED TO TREAT DIABETES MELLITUS AND HYPOGLYCEMIA


TOXICITIES
CLASS AND
SUBCLASSES NAMES CLINICAL USES COMMON UNIQUE; CLINICALLY IMPORTANT
Insulin Insulin Treatment o type 1 and Hypoglycemia
type 2 diabetes Atrophy o subcutaneous at
Lipohypertrophy

Biguanides Met ormin First-line treatment o Nausea, diarrhea, Lower blood concentrations o vitamin B12
type 2 diabetes indigestion, and Lactic acidosis
abdominal pain

Insulin Glyburide Treatment o type 2 diabetes Weight gain Hypoglycemia, cholestatic jaundice, agranulocytosis,
Secretagogues Glipizide Nausea and aplastic anemia, hemolytic anemia, generalized
Sul onylureas Gliclazide vomiting hypersensitivity reactions, and dermatological reactions
Glimepiride Ethanol may enhance the action o sul onylureas
and cause hypoglycemia
Drug interactions with many drugs including azole
anti ungal agents and histamine H2 antagonists

Insulin Repaglinide Treatment o type 2 diabetes Hypoglycemia


Secretagogues Nateglinide Metabolized by liver and should be used cautiously
Nonsul onylureas in patients with hepatic insuf ciency

Thiazolidinediones Rosiglitazone Treatment o type 2 diabetes Weight gain Increased incidence o congestive heart ailure
Pioglitazone and edema Rosiglitazone, but not pioglitazone may increase the
risk o myocardial in arction and stroke
Thiazolidinediones increase the risk o bone racture
in women

GLP-1 Agonists Exenatide Treatment o type 2 diabetes Nausea and Hypoglycemia is rare
Liraglutide vomiting

Dipeptidyl Sitagliptin Treatment o type 2 diabetes Saxagliptin is metabolized by CYP3A4 and dose
Peptidase-4 Saxagliptin should be adjusted when coadministered with
(DPP-4) Inhibitors Vidagliptin drugs that inhibit this enzyme
Alogliptin

-Glucosidase Acarbose Adjuncts to diet and exercise Malabsorption, Hypoglycemia when added to insulin or an insulin
Inhibitors Miglitol in type 2 diabetic patients not atulence, secretagogue
Voglibose reaching glycemic targets diarrhea, and Acarbose can decrease absorption o digoxin
abdominal Miglitol can decrease the absorption o propranolol
bloating and ranitidine

Amylin Pramlintide Used as an adjunct in patients Nausea Hypoglycemia


Antagonists with type 1 or type 2 diabetes Pregnancy category C
who take insulin with meals May alter the pharmacokinetics o drugs that require
GI absorption because o delayed gastric emptying

Bile Acid Colesevelam Treatment o Constipation, Inter eres with the absorption o many commonly
Sequestrants hypercholesterolemia (see dyspepsia, used drugs; most medications should be given
Chapter 20); may be used as abdominal pain, 4 hours be ore colesevelam
an adjunct to diet and exercise and nausea
to treat patients with
type 2 diabetes

Agents Used to Treat Glucagon Used to treat severe


Hypoglycemia hypoglycemia

Diazoxide Antihypertensive drug used to Sodium retention, edema, hyperuricemia,


treat patients with chronic or hypertrichosis, thrombocytopenia, and leukopenia
recurring hypoglycemia

479
CHAPTER
Drug Therapyof Mineral
31 Ion Homeostasis and Bone
Turnover Disorders
T is chapter will be most use ul a er having a basic understanding o the material
DRUGS INCLUDED
in Chapter 44, Agents A ecting Mineral Ion Homeostasis and Bone urnover in
IN THIS CHAPTER Goodman & Gilmans T e Pharmacological Basis of T erapeutics, 12th Edition.
1-hydroxycholecalciferol (1-OHD, alphacal- In addition to the material presented here, the 12th Edition contains:
cidol, ONE-ALPHA) A detailed discussion o the physiology o mineral ion homeostasis
22-Oxacalcitrol (1,25-dihydroxy-22- A description o the hormonal regulation o calcium and phosphorous homeostasis
oxavitamin D3, OCT,maxacalcitol, OXAROL)
A discussion o bone physiology
Alendronate (FOSAMAX)
A detailed discussion o the pharmacological treatment o disorders o mineral ion
Calcipotriene (DOVONEX,others) homeostasis and bone metabolism, including the therapeutic uses o vitamin D, cal-
Calcitonin (CALCIMAR, MIACALCIN) citonin, bisphosphonates, parathyroid hormone (P H), and calcium sensor mimetics
Calcitriol (1,25-dihydroxycholecalciferol; A description o an integrated approach to the prevention and treatment o
CALCIJEX,ROCALTROL) osteoporosis
Calciumsalts
LEARNING OBJECTIVES
Cinacalcet (SENSIPAR)
Understand calcium and phosphorous homeostasis.
Denosumab (investigational)
Dihydrotachysterol (DHT,ROXANE) Describe the roles o P H, calcitonin, and vitamin D in calcium homeostasis.
Doxercalciferol (1-hydroxyvitamin D2, Understand the concept o bone resorption and bone ormation.
HECTOROL) Describe the mechanism o action and untoward e ects o bisphosphonates.
Ergocalciferol (calciferol, DRISDOL) Describe the role o bisphosphonates in the prevention and treatment o
Etidronate sodium(DIDRONEL) osteoporosis.
Ibandronate (BONIVA) Describe the pharmacological management o hypocalcemia and
Lanthanumcarbonate (FOSRENOL) hypercalcemia.
Pamidronate (AREDIA)
Paricalcitol (1,25-dihydroxy-19-norvitamin MECHANISMS OF ACTION OF DRUGS USED IN MINERAL HOMEOSTASIS AND BONE
D2, ZEMPLAR) TURNOVER DISORDERS
Plicamycin (MITHRACIN) DRUG CLASS DRUGS MECHANISM OF ACTION
Risedronate (ACTONEL) Hormones Calcitonin Calcitonin actions are
Teriparatide mediated by the calcitonin
Sevelamer hydrochloride (RENAGEL)
receptor (CTR), which is a
Teriparatide (FORTEO) member o the parathyroid
Tiludronate (SKELID) PTH/secretin sub amily o
GPCRs; the hypocalcemic and
Zoledronate (ZOMETA) hypophosphatemic e ects
o calcitonin are caused by
direct inhibition o osteoclastic
bone resorption
Teriparatide is a synthetic PTH
ragment (1-34); intermittant
exposure to PTH promotes
anabolic actions on bone and
is the basis or teriparatides
use in treating severe
osteoporosis
(Continued)

480
Drug Therapy of Mineral Ion Homeostasis and Bone Turnover Disorders CHAPTER 3 1

DRUG CLASS DRUGS MECHANISM OF ACTION THERAPEUTIC USES


Vitamin D and Vitamin D Calcitriol (biologically active Actions o vitamin D and OF VITAMIN D
Analogs metabolite o Vitamin D); analogs are mediated by the
Prophylaxis and cure of nutritional rickets
(see Figure 31-1) vitamin D receptor (VDR), a
Doxercalci erol nuclear receptor; calcitriol and Treatment of metabolicrickets and
Dihydrotachysterol other drugs in this class act osteomalacia, particularlyin the setting
1-Hydroxycholecalci erol to maintain normal calcium of chronicrenal failure
Ergocalci erol and phosphate in plasma by
Calcipotriene acilitating their absorption Treatment of hypoparathyroidism
Paricalcitol in the small intestine, by Prevention and treatment of osteoporosis
22-Oxacalcitrol interacting with PTH to
enhance their mobilization
rom bone, and by decreasing
their renal excretion

Phosphate Binders Sevelamer hydrochloride Sevelamer is a nonabsorbable


Lanthanum carbonate phosphate-binding polymer
Lanthanum is a poorly
permeable trivalent cation
that binds phosphate

Bisphosphonates Etidronate sodium The bisphosphonates directly


Pamidronate inhibit bone resorption by
Tiludronate either osteoclast apoptosis
Alendronate (etidronate, pamidronate,
Risedronate tiludronate, risedronate,
Zoledronate and zoledronate) or by
Ibandronate inhibition o components o
the cholesterol biosynthetic
pathway that activate proteins
important or osteoclast
unction (alendronate
and ibandronate)

Calcium Sensor Mimetic Cinacalcet Mimic the stimulatory e ect


(calcimimetic) o calcium on the calcium-
sensing receptor (CaSR) to
inhibit PTH secretion by
the parathyroid glands; by
enhancing the sensitivity
o the CaSR to extracellular
calcium, calcimimetics lower
the concentration o calcium
at which PTH secretion
is suppressed

Monoclonal Antibody Denosumab (investigational) Blocks osteoclast ormation


and activation by binding with
high a nity to RANKL and
thereby reducing the binding
o RANKL to RANK; see
Figures 31-2 and 31-3 or details

Miscellaneous Plicamycin (no longer Plicamycin is a cytotoxic


available) antibiotic that also decreases
plasma Ca++ concentrations by
inhibiting bone resorption

481
SECTION V Hormones and Hormone Antagonists

25-OHD Bone lining ce lls

OH

Bone

Os te ocla s ts

CH2

HO
P TH P TH
Ca 2 +, phos pha te Ca 2 +, phos pha te Os te obla s ts Re s orption by
(e s troge n, prola ctin) re building os te ocla s ts

1, 25-(OH)2 D

OH

Os te obla s ts
induction

CH2

HO OH FIGURE 31-2 The bone remodeling cycle. Osteoclast precursors use and are activated to
resorb a lacuna in a previously quiescent sur ace. These cells are replaced by osteoblasts that
FIGURE 31-1 Regulation o 1-hydroxylase deposit new bone to restore the integrity o the tissue. (Adapted with permission from Skerry
activity. Changes in the plasma levels o PTH, TM, Gowen M. Bone cells and bone in remodelling in rheumatoid arthritis. In: Henderson B, Edwards
Ca2+, and phosphate modulate the hydroxyl- JCW, Pettipher ER, eds. Mechanisms and Models in Rheumatoid Arthritis. London: Academic Press;
ation o 25-OH vitamin D to the active orm, 1995, pp 205-220.)
1,25-dihydroxyvitamin D. 25-OHD, 25-hydroxy-
cholecalci erol; 1,25-(OH)2-D, calcitriol; PTH,
parathyroid hormone.

S troma l ce ll/os te obla s t

RANKL
Ina ctive
RANKL/
OP G
OP G complex

RANK

Os te obla s t Os te ocla s t
pre curs or

FIGURE 31-3 Receptor or activating NF-B ligand (RANKL) and osteoclast ormation. RANKL,
acting on RANK, promotes osteoclast ormation and subsequent resorption o bone matrix.
Osteoprotegerin (OPG) binds to RANKL, reducing its binding to RANKand thereby inhibiting
osteoclast di erentiation.

482
Drug Therapy of Mineral Ion Homeostasis and Bone Turnover Disorders CHAPTER 3 1

CASE 31-1
A 56-year-old man with a diagnosis o lung cancer is hospitalized because o lethargy,
weakness, and polyuria. His serum calcium is 13 mg/dL.
a. Why is his serum calcium elevated?
Newly diagnosed hypercalcemia in hospitalized patients is most o en caused by
systemic malignancy, either with or without bone metastasis. P H-related protein
(P HrP) is a primitive, highly conserved protein that may be abnormally expressed
in malignant tissue, particularly in squamous cell and other epithelial cancers. Other
tumors release cytokines or prostaglandins that stimulate bone resorption.
b. How does P H cause hypercalcemia?
T e primary unction o P H is to maintain a constant concentration o calcium and
phosphorous in the extracellular uid. T e principal processes regulated are renal
calcium and phosphorous absorption and mobilization o bone calcium (Figure 31-4).
T e actions o P H on target tissues are mediated by at least 2 receptors: the P H 1
receptor which binds both P H and P HrP; and the P H 2 receptor ound in
vascular tissue, brain, pancreas, and placenta which binds only P H.
c. What are the treatment options or this patient?
Calcitonin may be use ul in managing hypercalcemia. Reduction in serum calcium can
be rapid, although escape rom the hormone commonly occurs within several days.
Intravenous bisphosphonates (pamidronate and zoledronate) have proven very
e ective in the management o hypercalcemia. T ey act by inhibiting bone resorp-
tion (see Case 31-3).
(Continued)

Parathyro ids

Kidney
PTH

Bo ne

Urine Ca 2 +

Urine P la s ma Ca 2 +
phos pha te

Calc itrio l

Inte s tine

P hos pha te Ca 2 +
a bs orption a bs orption

FIGURE 31-4 Calcium homeostasis and its regulation by parathyroid hormone (PTH) and 1,25-
dihydroxyvitamin D. PTH has stimulatory e ects on bone and kidney, including the stimulation
o 1-hydroxylase activity in kidney mitochondria leading to the increased production o
1,25-dihydroxyvitamin D (calcitriol) rom 25-hydroxycholecalci erol, the monohydroxylated
vitamin D metabolite. Calcitriol is the biologically active metabolite o vitamin D.
483
SECTION V Hormones and Hormone Antagonists

d. What is the mechanism o action o calcitonin?


Calcitonin actions are mediated by the calcitonin receptor (C R). T e hypocalcemic
and hypophosphatemic e ects o calcitonin are caused predominantly by direct
inhibition o osteoclastic bone resorption.

CASE 31-2
A 42-year-old man with chronic renal disease is showing signs o decreased bone
density. T is was rst diagnosed as osteomalacia, but later the diagnosis was changed
to renal osteodystrophy.
a. What are osteomalacia and renal osteodystrophy?
Patients with chronic renal disease are at risk or developing osteomalacia (under-
mineralization o bone matrix), which occurs commonly during sustained phos-
phate depletion. Patients with chronic renal disease are also at risk or developing a
complex bone disease called renal osteodystrophy. In this setting, bone metabolism
is stimulated by an increase in P H and by a delay in bone mineralization that is
due to decreased renal synthesis o calcitriol. In renal osteodystrophy, low bone
mineral density (BMD) may accompany high-turnover bone lesions typically seen
in patients with uncontrolled hyperparathyroidism. In another orm o the disease,
low mineral density is accompanied by low bone remodeling activity seen in
patients with adynamic bone disease.
b. What is the role o phosphate binders in managing renal osteodystrophy?
T e therapeutic approach to the patient with renal osteodystrophy o the high-turnover
disease with de cient mineralization is dietary phosphate restriction, generally in com-
bination with a phosphate binder. Highly e ective phosphate binders have been devel-
oped. Sevelamer is a nonabsorbable phosphate-binding polymer that e ectively lowers
the serum phosphate concentration, with a corresponding reduction in the calcium
phosphate product. Lanthanum carbonate is a poorly permeable trivalent cation that is
use ul in treating the hyperphosphatemia associated with renal osteodystrophy.
c. What is the role o vitamin D in the management o renal osteodystrophy?
Renal osteodystrophy associated with low bone turnover is increasingly common
and may be due to oversuppression o P H with aggressive use o either calcitriol
or other vitamin D analog. Current guidelines suggest that treatment with an active
vitamin D preparation (see Figure 31-1) is indicated i serum 25-hydroxy vitamin D
(25-OHD) levels are less than 30 ng/mL and serum calcium is less than 9.5 mg/dL.
However, i 25-OHD and serum calcium levels are elevated, vitamin D supplemen-
tation should be discontinued. I the serum calcium is less than 9.5 mg/dL, treat-
ment with a vitamin D analog is warranted irrespective o the 25-OHD level.

CASE 31-3
A 55-year-old woman is concerned because her mother and sister developed ractures
in their legs and vertebra a er menopause. She wants to know what osteoporosis is
and i she is susceptible when she has never had a broken bone. She is also asking or a
medication and other advice that will prevent osteoporosis. She wants to know i taking
only a calcium supplement will prevent her rom having ractures.
a. How is bone physiology involved in the development o osteoporosis?
Osteoporosis is a condition o low bone mass and microarchitectural disruption that
results in ractures with minimal trauma. Osteoporosis can be categorized as primary
or secondary. Primary osteoporosis represents 2 undamentally di erent conditions:
type I osteoporosis is characterized by loss o trabecular bone owing to estrogen lack
at menopause, and type II osteoporosis is characterized by loss o cortical and tra-
becular bone in men and women due to long-term remodeling inef ciency, dietary
inadequacy, and activation o the parathyroid axis with age. Secondary osteoporosis is
due to systemic illness or medications such as glucocorticoids. (Continued)
484
Drug Therapy of Mineral Ion Homeostasis and Bone Turnover Disorders CHAPTER 3 1

Growth and development o endochondral bone are driven by a process called


modeling. Once new bone is laid down, it is subjected to a continuous process
o breakdown and renewal called remodeling by which bone mass is adjusted
throughout adult li e (see Figures 31-2 and 31-3). In women, loss o estrogen at
menopause accelerates the rate o bone loss. Primary regulators o adult bone mass
include physical activity, reproductive endocrine status, and calcium intake.
Optimal maintenance o BMD requires suf ciency in all 3 areas, and de ciency
o one is not compensated by excessive attention to another. T us, taking only
a calcium supplement is not suf cient to maintain BMD.
b. T is patient is started on ibandronate, a bisphosphonate. What are bisphospho-
nates and how are they used in the prevention and treatment o osteoporosis?
Pharmacological treatment or the prevention and treatment o osteoporosis is
aimed at restoring bone strength and preventing ractures. Bisphosphonates are
the most requently used drugs or the prevention and treatment o osteoporosis.
Bisphosphonates concentrate at sites o active bone remodeling. Because they are
highly charged, bisphosphonates are membrane impermeable but are incorpo-
rated into the bone matrix by uid-phase endocytosis. T e antiresorptive activ-
ity involves 2 primary mechanisms: (1) osteoclast apoptosis and (2) inhibition o
components o the cholesterol biosynthetic pathway. T e rst-generation bisphos-
phonates, medronate, etidronate, clodronate, and tiludronate act by the rst mecha-
nism, whereas the second-generation bisphosphonates, pamidrone, alendronate,
ibandronate, risedronate, and zoledronate act by the latter mechanism.
c. What are the untoward ef ects o bisphosphonates?
Oral bisphosphonates can cause heartburn, esophageal irritation, or esophagitis.
T ese symptoms o en abate when patients take the medication a er an overnight
ast, with tap water, and remain upright.
Serious osteonecrosis o the jaw is associated with the use o bisphosphonates.
Zoledronate has a pro ound e ect on calcium and is capable o causing hypocalcemia.
It has also been associated with decreased renal unction. Patients should have
laboratory tests or renal unction prior to its use and renal unction should be
monitored periodically a er treatment.
d. What are other options or this patient?
eriparatide, a P H ragment, is available to use in treating severe osteoporosis.
In postmenopausal women with osteoporosis, teriparatide increases BMD and
reduces the risk o vertebral and nonvertebral ractures. Candidates or teriparatide
treatment include women who have a history o osteoporotic racture, who have
multiple risk actors or racture, or who ailed or are intolerant o previous
osteoporosis therapy.
In animal studies, teriparatide has been associated with the development o
osteosarcoma.

KEY CONCEPTS
T e steady-state content o calcium in bone re ects the net e ect o bone
resorption and bone ormation.
P H is a polypeptide hormone that helps regulate plasma calcium by a ecting
bone resorption/ ormation, renal calcium excretion/reabsorption, and calcitriol
synthesis (thus GI calcium absorption).
T e primary active metabolite o vitamin D is 1,25-hydroxyvitamin D. T e
enzyme system responsible or the 1-hydroxylation o 25-hydroxyvitamin D
(25-OHD) is located in the proximal tubules o the kidney.
Calcitonin is a hypocalcemic hormone produced and secreted by the thyroid
para ollicular C cells. T e actions o calcitonin generally oppose those o P H.
(Continued)
485
SECTION V Hormones and Hormone Antagonists

In women, loss o estrogen at menopause accelerates the rate o bone loss (see
Figures 31-2 and 31-3).
Patients with chronic renal disease are at risk or developing osteomalacia or
a complex bone disease called renal osteodystrophy.
Osteoporosis is a condition o low bone mass and microarchitectural disrup-
tion that results in ractures with minimal trauma.
Bisphosphonates are the most requently used drugs or the prevention and
treatment o osteoporosis.
Bisphosphonates are associated with severe untoward e ects such as esophageal
distress and osteonecrosis o the jaw.
eriparatide is a P H ragment used or the treatment o severe osteoporosis.

SUMMARY QUIZ

QUESTION 31-1 A 43-year-old man with intestinal bypass surgery is at risk o devel-
oping a de ciency o
a. vitamin D.
b. calcium.
c. phosphorous.
d. vitamin C.
e. parathyroid hormone.

QUESTION 31-2 25-hydroxyvitamin D (25-OHD) is converted to its active orm


1,25-OHD by the enzyme 1-hydroxylase in the proximal renal tubules. 1-hydroxylase
is activated by a dietary de ciency o
a. potassium.
b. iron.
c. calcium.
d. zinc.
e. sodium.

QUESTION 31-3 Calcitonin is a hormone produced and secreted rom the thyroid
para ollicular C cells and is regulated by the serum concentration o
a. potassium.
b. calcium.
c. zinc.
d. iron.
e. sodium.

QUESTION 31-4 A 42-year-old man with lung cancer had a serum calcium measure-
ment 1 month ago that was 9.0 mg/dL. He is hospitalized now with severe muscle
weakness, atigue, anorexia, and constipation. His serum calcium is 13.5 mg/dL. T e
severity o his symptoms is due to the
a. rate o rise in his serum calcium.
b. decrease in the intestinal excretion o calcium.
c. malabsorption o vitamin D.
d. ormation o a parathyroid adenoma.
e. ingestion o large quantities o calcium.

486
Drug Therapy of Mineral Ion Homeostasis and Bone Turnover Disorders CHAPTER 3 1

QUESTION 31-5 A 35-year-old woman with a parathyroid adenoma is being treated


with cinacalcet, a calcium sensor mimetic. T e principal adverse event with this drug is
a. hyperphosphatemia.
b. hypercalcemia.
c. hypervitaminosis D.
d. decreased vitamin D absorption.
e. hypocalcemia.

QUESTION 31-6 A 55-year-old woman is treated with a bisphosphonate drug to


prevent osteoporosis during postmenopause. She should be cautioned about which
o the ollowing adverse events?
a. Congestive heart ailure.
b. innitus.
c. Macular degeneration.
d. Esophagitis.
e. Peripheral neuropathy.

SUMMARY QUIZ ANSWER KEY

QUESTION 31-1 Answer is a. Bile is essential or the absorption o vitamin D.


Patients who have intestinal bypass surgery or otherwise have severe shortening or
in ammation o the small intestine may ail to absorb vitamin D suf ciently to main-
tain normal levels.

QUESTION 31-2 Answer is c. See Figure 31-1. Vitamin D 1-hydroxylase is subject to


tight regulatory control that results in changes in calcitriol ormation appropriate or
optimal calcium homeostasis. Dietary de ciency o vitamin D, calcium, or phosphate
enhances enzyme activity. Conversely high calcium, phosphate, or vitamin D intake
suppresses enzyme activity.

QUESTION 31-3 Answer is b. T e biosynthesis and secretion o calcitonin are regu-


lated by the serum calcium concentration. Calcitonin secretion increases when serum
calcium is high and decreases when serum calcium is low.

QUESTION 31-4 Answer is a. T e degree o hypercalcemia and the rate o rise in


serum calcium concentration largely dictate the extent o symptoms. Chronic elevation
o serum calcium to 12 to 14 mg/dL generally causes ew mani estations, whereas an
acute rise to the same levels may cause marked neuromuscular mani estations owing to
an increased threshold or excitation o nerve and muscle. T e ingestion o large quan-
tities o calcium by itsel generally does not cause hypercalcemia; an exception is the
hyperthyroid patient who absorbs calcium with increased ef ciency.

QUESTION 31-5 Answer is e. Calcimimetics are drugs that mimic the stimulatory
e ect o calcium on the calcium-sensing receptor (CaSR) to inhibit P H secretion
by the parathyroid glands. T e principal adverse event with calcimimetics is
hypocalcemia.

QUESTION 31-6 Answer is d. Oral bisphosphonates can cause heartburn, esophageal


irritation, or esophagitis (see Case 31-3). For patients in whom oral bisphosphonates
cause severe esophageal distress despite counter measures, intravenous zoledronate or
ibandronate o ers skeletal protection without causing adverse GI e ects.

487
SECTION V Hormones and Hormone Antagonists

SUMMARY: DRUGS USED IN MINERAL HOMEOSTASIS AND BONE TURNOVER DISORDERS


TOXICITIES
CLASS AND
SUBCLASSES NAMES CLINICAL USES COMMON UNIQUE; CLINICALLY IMPORTANT
Hormones Calcitonin Calcitonin is use ul in Side e ects o calcitonin Calcitonin is e ective or the
Teriparatide managing hypercalcemia; it is are nausea, hand swelling, initial 6 hours but due to receptor
also e ective in patients with urticaria, and rarely downregulation, patients become
Paget disease and in some intestinal cramping re ractory a ter a ew days
patients with osteoporosis Teriparatide has been associated
Teriparatide is used to treat with osteosarcoma in animals
severe osteoporosis Use o teriparatide should be
limited to 2 years

Vitamin D and Calcitriol Therapeutic uses o vitamin D Hypercalcemia with or without


Vitamin D Doxercalci erol are shown in the Side Bar hyperphosphatemia
Analogs Dihydrotachysterol THERAPEUTIC USES Paricalcitol, and 22-oxacalcitrol
1-Hydroxycholecalci erol OF VITAMIN D reduce serum PTH but have less or
Ergocalci erol Calcipotriene is used as a negligible hypercalcemic activity
Calcipotriene topical preparation to treat
Paricalcitol psoriasis
22-Oxacalcitrol

Phosphate Sevelamer hydrochloride Sevelamer is use ul in treating Side e ects o sevelamer


Binders Lanthanum carbonate hyperphosphatemia in include nausea, vomiting,
hemodialysis patients diarrhea, and dyspepsia
Lanthanum is use ul in
treating hyperphosphatemia
in patients with renal
osteodystrophy

Bisphosphonates Etidronate sodium Treatment o Paget disease Heartburn, esophageal Osteonecrosis o the jaw
Pamidronate Ibandronate is approved irritation esophagitis, Parenteral in usion o pamidronate
Tiludronate or the prevention and abdominal pain, may cause f ushing, f u-like
Alendronate treatment o postmenopausal and diarrhea symptoms, nausea and vomiting
Risedronate osteoporosis Zoledronate is capable o causing
Zoledronate Pamidronate and zoledronate severe hypocalcemia
Ibandronate are use ul in treating
hypercalcemia associated with
malignancy

Calcium Sensor Cinacalcet Approved or the Hypocalcemia


Mimetic treatment o secondary Metabolized by CYP3A4, 2D6,
(calcimimetic) hyperparathyroidism due to and 1A2; coadministration o
chronic renal disease inhibitors o these enzymes should
be cautioned

Monoclonal Denosumab Proposed to treat osteoporosis


Antibody (investigational)

488
SECTION

Drugs A ecting
Gastrointestinal Function VI
32. Pharmacotherapy o Gastric Acidity, Peptic Ulcers,
and Gastroesophageal Re ux Disease 490

33. Drugs Used or the reatment o Bowel Disorders 497

489
CHAPTER
Pharmacotherapyo Gastric
32 Acidity, PepticUlcers, and
Gastroesophageal Ref uxDisease
T is chapter will be most use ul a er having a basic understanding o the material in
DRUGS INCLUDED IN
Chapter 45, Pharmacotherapy o Gastric Acidity, Peptic Ulcers, and Gastroesophageal
THIS CHAPTER Re ux Disease in Goodman & Gilman's T e Pharmacological Basis of T erapeutics, 12th
Cimetidine (TAGAMET,others) Edition In addition to the material presented here, the 12th Edition contains:
Dexlansoprazole (KAPIDEX) A description o the physiology o gastric secretion
Esomeprazole (NEXIUM) T erapeutic strategies or the treatment o speci c acid-peptic disorders
Famotidine (PEPSID, others) able 45-2 which shows the composition and acid neutralizing capacities o popular
Lansoprazole (PREVACID) antacid preparations
Misoprostol (CYTOTEC)
LEARNING OBJECTIVES
Nizatidine (AXID, others)
Identi y the sites in the gastric parietal cell where drugs act to suppress acid
Omeprazole (PRILOSEC, others) secretion
Pantoprazole (PROTONIX) Describe the mechanism o action o proton pump inhibitors, H 2 receptor
Rabeprazole (ACIPHEX) antagonists, and prostaglandin analogs to suppress gastric acid secretion
Ranitidine (ZANTEC, others) Describe the limitations to the use o H 2 receptor antagonists in chronic acid
Sucralfate (CARAFATE, others) suppression
Identi y potential drug interactions with proton pump inhibitors and H 2 receptor
antagonists
Describe the mechanism o action o drugs that enhance gastric cytoprotection
Describe the recommendations or therapy o gastroesophageal re ux disease
(GERD) and peptic ulcer disease
Understand the role o Helicobacter pylori in ection in peptic ulcer disease and
the therapeutic principles or its eradication
Describe appropriate therapy or NSAID-induced ulcers

MECHANISMS OF ACTION OF DRUGS USED TO TREAT GASTRIC ACID DISEASES


DRUG CLASS DRUGS MECHANISM OF ACTION
H2 Receptor Cimetidine Competes with histamine or binding to H2 receptors on the basolateral membrane o
Antagonists Ranitidine parietal cells (see Figure 32-1)
Famotidine
Nizatidine

Proton Pump Lansoprazole The activated orm binds covalently with sul hydryl groups on cysteine in the H+,K+-ATPase
Inhibitors Pantoprazole located on the luminal membrane o the parietal cell to suppress acid secretion
Omeprazole (see Figure 32-1)
Esomeprazole
Dexlansoprazole
Rabeprazole

Prostaglandin Analog Misoprostol Binds to the EP3 receptor on parietal cells decreasing cyclic AMP and gastric acid secretion
(see Figure 32-1)

Cytoprotective Agent Sucral ate Forms a viscous, sticky polymer that adheres to epithelial cells and ulcer craters and is
cytoprotective

Antacids Various over-the-counter Neutralize gastric acid


(OTC) products

Bismuth Various OTC products Binds to the base o ulcers and promotes mucin and bicarbonate production

490
Pharmacotherapy of Gastric Acidity CHAPTER 3 2

Mus ca rinic
Gas trin a nta gonis ts

1 M3
K+ K+
ACh
Cl Cl
CCK2 H2
a nta gonis ts
CCK2 Ca 2 +-de pe nde nt
pa thway
K+
HIST HIST H2 H+, K+
cAMP-de pe nde nt Proton pump
ATPa s e inhibitors
M? ECL c e ll pa thway
NSAIDS H+ Anta cids
ACh EP 3
Parie tal c e ll
Bis muth
C 20 fa tty P GE 2
Me tronida zole
a cids P GI2
Mis opros tol H. pylori Te tra cycline
ENS Cla rithromycin
N ce ll Amoxicillin
Mus ca rinic
a nta gonis ts
M1 EP 3 Mucus

AC h S ucra lfa te
Cyto-
prote ction Ca rbe n-
oxolone
M?
2 P ire nze pine 3 ACh HCO 3
S upe rfic ial e pithe lial c e ll

Mucous laye r Ga s tric lume n


pH 7 pH 2

FIGURE 32-1 Physiological and pharmacological regulation o gastric secretion: the basis or therapy o acid-peptic disorders. Shown are the
interactions among an enterochroma n-like (ECL) cell that secretes histamine, a ganglion cell o the enteric nervous system (ENS), a parietal
cell that secretes acid, and a super cial epithelial cell that secretes mucus and bicarbonate. Physiological pathways, shown in solid black, may
be stimulatory (+) or inhibitory (). 1 and 3 indicate possible inputs rom postganglionic cholinergic bers; 2 shows neural input rom the vagus
nerve. Physiological agonists and their respective membrane receptors include acetylcholine (ACh), muscarinic (M), and nicotinic (N) receptors;
gastrin, cholecystokinin receptor 2 (CCK2); histamine (HIST), H2 receptor; and prostaglandin E2 (PGE2), EP3 receptor. A gray indicates targets o
pharmacological antagonism. A blue dashed arrow indicates a drug action that mimics or enhances a physiological pathway. Shown in gray are
drugs used to treat acid-peptic disorders. NSAIDs are nonsteroidal anti-in ammatory drugs, which can induce ulcers via inhibition o cyclooxy-
genase (see Chapter 22).

CASE 32-1
A 42-year-old woman is seen because o worsening heartburn during the past week She
was rst diagnosed with GERD 1 month ago and treatment was begun with cimetidine
400 mg once daily wo weeks later the cimetidine dosage was increased to 400 mg
twice daily
a. What is the reason or the worsening o her GERD symptoms?
Cimetidine is an H 2 receptor antagonist and tolerance to acid suppressing e ects o
these drugs has been shown to develop as early as 2 to 5 days a er beginning treat-
ment. One mechanism or this phenomenon is secondary to the hypergastrinemia
that stimulates histamine release and overcomes the H 2 receptor blockade.
b. What are the mechanisms o action o H2 receptor blockers and proton pump
inhibitors?
T e mechanisms o action o these drugs is shown in Figure 32-1. T e H 2 receptor
blockers suppress gastric acid secretion by competing with histamine or binding to
H 2 receptors on the basolateral membrane o parietal cells. T e structural similarity
o the H 2 receptor antagonists and histamine is shown in Figure 32-2.
(Continued)
491
SECTION VI Drugs Af ecting Gastrointestinal Function

CH2 CH2 NH2 T e proton pump inhibitors are prodrugs that require activation in an acid environ-
NH N
ment (see Figure 32-3). T e activated orm binds covalently with sul ydryl groups
on cysteine in the H+,K+-A Pase located on the luminal membrane o the parietal cell.
HISTAMINE Despite the short plasma hal -lives (0.5-2 h) o the parent drug, acid suppression
continues or 24 to 48 hours or until new pump molecules are synthesized.
c. What is a rational approach to her treatment?
H3 C CH2 S CH2 CH2 N CNHCH3
A rational choice would be to switch her to a proton pump inhibitor. Although these
HN HN C N drugs can also cause a hypergastrinemia, it does not result in tolerance because the
CIMETIDINE inhibition occurs at the H +,K+-A Pase which is the nal step in gastric acid secretion.
Figure 32-4 shows the general guidelines or medical management o GERD.
d. I the patient is switched to a proton pump inhibitor, o what adverse ef ects
CH2 S CH2 CH2 NHCNHCH3
should she be warned?
O CHNO 2 Proton pump inhibitors are metabolized by hepatic CYPs and they may inter ere
with the elimination o other drugs cleared by this route. Chronic treatment with
CH2 N(CH3 )2
proton pump inhibitors decreases the absorption o vitamin B12. T e loss o gastric
RANITIDINE acidity may a ect the bioavailability o drugs, most notably iron salts. T is may
result in an iron de ciency anemia.
CH2 S CH2 CH2 CNH2

S N NS O 2 NH2 CASE 32-2


N C(NH2 )2 A 56-year-old man is diagnosed with duodenal ulcer complicated by H. pylori in ection He
has been treated with amoxicillin or 2 weeks, but his symptoms o stomach pain persist
FAMOTIDINE
a. Why has his therapeutic regimen not been ef ective? What problems might be the
result o this therapeutic regimen?
CH2 S CH2 CH2 NHCNHCH3
H. pylori, a gram negative rod, has been associated with gastritis and subsequent devel-
S N CHNO 2
opment o gastric and duodenal ulcers, gastric adenoma, and gastric B-cell lymphoma.
CH2 N(CH3 )2
Single antibiotic therapy or H. pylori in ection is not e ective or eradication and may
result in bacterial resistance that is more dif cult to treat than the initial in ection.
NIZATIDINE
b. What would you recommend or the treatment o his condition?
FIGURE 32-2 Histamine and H2 receptor
antagonists. A common therapeutic approach would be to use a proton pump inhibitor plus
clarithromycin plus either metronidazole or amoxicillin or 14 days. Another
common approach would be to use a proton pump inhibitor plus metronidazole
plus bismuth plus tetracycline. T ese regimens are shown in able 32-1.
c. What is the most common problem observed with your recommended regimen?
T e most common problem observed with these therapeutic regimens is poor patient
compliance due to the number o medications that must be taken each day and to
medication-related side e ects.

CASE 32-3
A 64-year-old man is re erred because o stomach pain He has been diagnosed with
osteoarthritis and has been taking a COX-1 inhibitor or the past 3 months Workup
shows that he has a duodenal ulcer that you suspect is a result o his NSAID use
a. Describe the pathogenesis o NSAID-induced ulcers.
NSAIDs diminish prostaglandin ormation by inhibiting cyclooxygenase. T is e ect
can result in enhanced gastric acid secretion (prostaglandins may lower gastric acid
secretion). In addition, prostaglandins stimulate gastric mucin production that pro-
vides a cytoprotective e ect to the gastric mucosa which is diminished by NSAIDs.
b. What are your therapeutic options?
One option is to change this patient to an NSAID that is a selective inhibitor o
COX-2, although this may not completely eliminate the risk o ulcer ormation.

(Continued)
492
Pharmacotherapy of Gastric Acidity CHAPTER 3 2

A OCH2 CF 3 OCH3 O (CH2 )3 OCH3 OCH3


CH3 H3 C CH3 CH3 OCH3

N N N N
N S N S N S N S
O O O O
F 2 HCO
NH NH NH N

OCH3
LANS OP RAZOLE OMEPRAZOLE RABERP RAZOLE PANTOP RAZOLE

H+

OCH3 CYCLIC S ULFENAMIDE S ULFENIC ACID


B
H3 C CH3 OCH3 OCH3
H3 C CH3 H3 C CH3
+ Enzyme SH
N + +
S Enzyme N N
S
N NH S S
N N H2 O N NH OH

OCH3
OCH3 OCH3
ENZYME-INHIBITOR COMPLEX

FIGURE 32-3 Proton pump inhibitors. A. Inhibitors o gastric H+,K+-ATPase (proton pump). B. Conversion o omeprazole to a
sul enamide in the acidic secretory canaliculi o the parietal cell. The sul enamide interacts covalently with sul hydryl groups in
the proton pump, thereby irreversibly inhibiting its activity. The other 3 proton pump inhibitors undergo analogous conversions.

Another option is to try misoprostol although it has numerous gastrointestinal side


e ects and 4-times-daily dosing is inconvenient. Finally, NSAID-induced ulcers can
be managed with acid suppression using either an H 2 receptor antagonist or proton
pump inhibitor. Proton pump inhibitors can e ectively heal active ulcers and pre-
vent recurrence in the setting o continued NSAID administration.

S e ve rity o f GERD Me dic al Manag e me nt

S tag e I Life s tyle modifica tion, including die t,


S pora dic uncomplica te d he a rtburn, ofte n pos itiona l cha nge s, we ight los s, e tc .
in s e tting of known pre cipita ting fa ctor. Anta cids a nd/or his ta mine H2 re ce ptor
Ofte n not the chie f compla int. a nta gonis ts a s ne e de d.
Le s s tha n 2-3 e pis ode s pe r we e k.
No a dditiona l symptoms.

S tag e II Proton pump inhibitors more e ffe ctive


Fre que nt symptoms, with or without tha n his ta mine H2 re ce ptor a nta gonis ts.
e s opha gitis.
Gre a te r tha n 2-3 e pis ode s pe r we e k.

S tag e III Proton pump inhibitor e ithe r once or


Chronic, unre le nting symptoms ; twice da ily.
imme dia te re la ps e off the ra py.
Es opha ge a l complica tions (e g.,
s tricture, Ba rre tts me ta pla s ia ).

FIGURE 32-4 General guidelines or the medical management o gastroesophageal re ux disease


(GERD). Only medications that suppress acid production or that neutralize acid are shown. (Adapted
with permission from Wolfe MM, Sachs G. Acid suppression. Gastroenterology, 2000;118:S24. Copyright
Elsevier.)
493
SECTION VI Drugs Af ecting Gastrointestinal Function

TABLE 32-1 Therapy of Helicobacter pylori Infection


Triple therapy 14 days: Proton pump inhibitor + clarithromycin 500 mg plus metronidazole 500 mg or amoxicillin 1 g twice a day (tetracycline
500 mg can be substituted or amoxicillin or metronidazole)
Quadruple therapy 14 days: Proton pump inhibitor twice a day + metronidazole 500 mg three times daily plus bismuth subsalicylate 525 mg +
tetracycline 500 mg our times daily
or
H2 receptor antagonist twice a day plus bismuth subsalicylate 525 mg + metronidazole 250 mg + tetracycline 500 mg our times daily
Dosages:

Proton pump inhibitors: H2 receptor antagonists:

Omeprazole: 20 mg Cimetidine: 400 mg

Lansoprazole: 30 mg Famotidine: 20 mg

Rabeprazole: 20 mg Nizatidine: 150 mg

Pantoprazole: 40 mg Ranitidine: 150 mg

Esomeprazole: 40 mg

KEY CONCEPTS
Proton pump inhibitors are superior to H 2 receptor antagonists or acid
suppression in patients with GERD and peptic ulcers
olerance to the acid-suppressing e ects o the H 2 receptor antagonists is
commonly observed and limits their continuous use
H. pylori is e ectively eradicated with a combination o acid-suppressing drugs
and multiple antibiotics administered or 10 to 14 days
NSAID-induced ulcers can be e ectively treated with a proton pump inhibitor
even during continued NSAID administration

SUMMARY QUIZ

QUESTION 32-1 A 24-year-old woman presents with symptoms o esophageal re ux


She is 6 months pregnant Which o the ollowing drugs is contraindicated in this
patient?
a Ranitidine
b Lansoprazole
c Misoprostol
d Sucral ate
e Aluminum hydroxide antacid

QUESTION 32-2 A 48-year-old man with a duodenal ulcer disease is treated with
cimetidine A er 6 weeks o treatment, he complains that his stomach pain is returning
and wonders i the dose o cimetidine should be increased T e most likely reason or
the decreased e ectiveness o cimetidine in this patient is
a tolerance
b diminished GI absorption
c enhanced plasma protein binding
d increased hepatic metabolism
e poor patient compliance

494
Pharmacotherapy of Gastric Acidity CHAPTER 3 2

QUESTION 32-3 Esomeprazole has a plasma hal -li e o a ew hours yet suppresses acid
secretion or 24 to 48 hours T e reason or this paradox is
a acid suppression continues until new H +,K+-A Pase molecules are synthesized
b gastrin depletion occurs long a er esomeprazole disappears rom the plasma
c prostaglandin synthesis is enhanced by esomeprazole
d H. pylori is e ectively suppressed by esomeprazole or 24 to 48 hours
e acid suppression continues until new H 2 receptors are synthesized

QUESTION 32-4 A 48-year-old woman has been diagnosed with a duodenal ulcer that
is complicated by H. pylori in ection A suitable therapeutic regimen or this patient
would be
a a single antibiotic
b a single antibiotic plus a proton pump inhibitor
c misoprostol plus a proton pump inhibitor
d an H 2 receptor antagonist
e 2 antibiotics plus a proton pump inhibitor

QUESTION 32-5 A 60-year-old woman has had symptoms o heartburn or 6 months


She rst treated hersel with antacids but as the requency and severity o the pain
increased she switched to over-the-counter omeprazole which she has been taking
daily or 3 months She now comes to your of ce complaining o atigue and lethargy
Her physical examination is unusual only in that her skin is remarkably pale An analysis
o her blood reveals hypochromic, microcytic red blood cells T e most likely explanation
or this nding is
a poor absorption o calcium
b poor absorption o vitamin B12
c poor absorption o olic acid
d poor absorption o iron salts
e a direct e ect o omeprazole on the bone marrow production o red blood cells

SUMMARY QUIZ ANSWER KEY


QUESTION 32-1 Answer is c. Misoprostol is contraindicated in pregnancy because it
can increase uterine contractility
QUESTION 32-2 Answer is a. H 2 receptor antagonists are known to produce tolerance
One cause or this tolerance is the secondary increase in gastrin which stimulates
histamine release and overcomes the H 2 receptor blockade

QUESTION 32-3 Answer is a. Proton pump inhibitors irreversible bind to the H +,K+-
A Pase and suppress acid secretion by the gastric parietal cell until new pump mol-
ecules are synthesized

QUESTION 32-4 Answer is e. H. pylori in ection is e ectively eradicated with an acid-


suppressing drug plus at least 2 antibiotics (see able 32-1) A single antibiotic or acid-
suppressing drug alone is not e ective in eradicating H. pylori

QUESTION 32-5 Answer is d. Proton pump inhibitors are so e ective at the suppres-
sion o gastric acid secretion that the absorption o many substances that require an
acidic environment is a ected One o these substances is iron salts and the resulting
iron de ciency can be mani ested by a hypochromic, microcytic anemia T e absorption
o olic acid and vitamin B12 may also be decreased in the presence o acid-suppressing
drugs but a de ciency in these would not result in a hypochromic, microcytic anemia
(see AccessMedicine, Harrison's Principles of Internal Medicine, 18th Edition Online,
Chapter 57, Anemia and Polycythemia)

495
SECTION VI Drugs Af ecting Gastrointestinal Function

SUMMARY: DRUGS USED TO TREAT GASTRIC ACIDITY, PEPTIC ULCERS, AND GASTROESOPHAGEAL REFLUX DISEASE
TOXICITIES
CLASS AND UNIQUE; CLINICALLY
SUBCLASSES NAMES CLINICAL USES COMMON IMPORTANT
H2 Receptor Cimetidine Promote healing o gastric Diarrhea, headache, drowsiness, Inhibition o hepatic CYPs
Antagonists Ranitidine and duodenal ulcers, atigue, muscular pain, and Delirium and con usion with
treatment o uncomplicated constipation IVuse in elderly patients
GERD, and to prevent Tolerance occurs with long-term use Gynecomastia in men
occurrence o stress ulcers Galactorrhea in women

Famotidine Same as cimetidine Same as cimetidine No inhibition o hepatic CYPs

Nizatidine Same as cimetidine Same as cimetidine No inhibition o hepatic CYPs

Proton Pump Omeprazole Treatment o gastric and Nausea, atulence, abdominal pain, Metabolized by hepatic CYPs
Inhibitors Esomeprazole duodenal ulcers and GERD constipation and may af ect other drugs
Lansoprazole that are metabolized by
Pantoprazole hepatic CYPs
Rabeprazole Decreased absorption o olic
acid, vitamin B12, and iron salts

Prostaglandin Analog Misoprostol Prevention o NSAID- Diarrhea Clinical exacerbation o


induced mucosal injury in ammatory bowel disease
Contraindicated in pregnancy

Cytoprotective Sucral ate Treatment o peptic ulcer Constipation Avoid in patients with renal
Agents disease ailure
May inhibit the absorption o
other drugs

Bismuth OTC products Treatment o peptic ulcer Black tongue and stools Bismuth subsalicylate is
particularly in patients with associated with salicylate
H. pylori in ection (see poisoning
Table 32-1)

Antacids Various OTC OTC or heartburn and See Goodman and Gilman's Gilman's The Decreased drug absorption by
products excess gastric acidity Pharmacological Basis of Therapeutics, increasing gastric pH
12th Edition, Chapter 45, or a
discussion o the common adverse
ef ects o the various antacids

496
CHAPTER

Drugs Used for the Treatment


of Bowel Disorders 33
T e material in this chapter covers the material presented in Chapters 46 and 47 o
DRUGS INCLUDED
Goodman and Gilmans T e Pharmacological Basis of T erapeutics, 12th Edition; this
chapter will be most use ul a er having a basic understanding o these chapters in the IN THIS CHAPTER
12th Edition. In addition to the material presented here, Chapters 46 and 47 o the 12th Adalimumab (HUMIRA)
Edition contain: Alosetron (LOTRONEX)
A description o gastric motility (see Chapter 46) Alvimopan (ENTEREG)
A description o the pathophysiology o constipation (see Chapter 46), including Aprepitant (EMEND)
able 46-3 that provides a classi cation and comparison o representative laxatives,
Azathioprine (IMMURAN)
and able 46-4 that shows the properties o di erent dietary bers
Balsalazide (COLAZIDE)
T e general principles and approaches to the treatment o diarrhea (see Chapter 46)
Bisacodyl (DULCOLAX,CORRECTOL, others)
A general description o irritable bowel syndrome (IBS) (see Chapter 46)
Bismuth (OTCPEPTO-BISMOL)
A pharmacological approach to the management o nausea and vomiting (see
Chapter 46), including able 46-6 that shows the receptor speci city o antiemetic Budesonide (ENTOCORTER)
agents, and able 46-7 that shows some antiemetic regimens used in cancer chemo- Carboxymethylcellulose
therapy and their initial doses Certolizumab pegol (CIMZIA)
A description o the pharmacological management o chronic pancreatitis and Cholestyramine (QUESTRAN, others)
steatorrhea (see Chapter 46)
Cisapride (PROPULSID) available onlyin a
T e pathogenesis o in ammatory bowel disease (IBD) (see Chapter 47), including limited-access program
able 47-1 which details medications commonly used to treat IBD
Cyclosporine
LEARNING OBJECTIVES Dexpanthenol (ILOPAN, others)
Describe the mechanisms o action o laxatives and cathartics, and understand Dicyclomine (BENTYL, others)
their use. Di enoxin (MOTOFEN)
Describe the mechanisms o action o antidiarrheal agents and understand Diphenoxylate (LOMOTIL)
their use. Docusate sodium(COLACE, DOXINATE, others)
Understand the pharmacotherapy o IBS. Domperidone (MOTILIUM)not available
Identi y drugs used to treat nausea and vomiting, and describe the mechanisms in the United States
o action o these drugs. Dronabinol (MARINOL)
Identi y the drugs used to treat IBD, their mechanisms o action, untoward e ects, Glycopyrrolate (ROBINUL, others)
and understand how they are used to treat IBD. Hyoscyamine (LEVSIN, others)
In iximab (REMIDCADE)
MECHANISMS OF ACTION OF DRUGS USED TO TREAT BOWEL DISORDERS Lactulose (CEPHULAC, others)
DRUG CLASS DRUGS MECHANISM OF ACTION Loperamide (IMODIUM, others)
Prokinetic Agents Metoclopramide D2 receptor antagonist may also involve Lubiprostone (AMITIZA)
5-HT4 receptor agonism, vagal, and central Mesalamine (5-aminosalicylicacid, 5-ASA)
5-HT3 antagonism
Methotrexate
Domperidone D2 receptor antagonist Methscopolamine (PAMINE, others)
Cisapride 5-HT4 receptor agonist Methylnaltrexone (RELISTOR)
Prucalopride Metoclopramide (REGLAN, others)
Erythromycin Mimics the e ects o motilin, a peptide that Nabilone (CESAMET)
contracts the upper GI tract Natalizumab (TYSABRI)
Lubiprostone Prostanoid activator o Cl channels Octreotide (SANDOSTATIN)
(Continued) Olsalazine (DIPENTUM)
(continues)

497
SECTION VI Drugs Af ecting Gastrointestinal Function

DRUGS INCLUDED DRUG CLASS DRUGS MECHANISM OF ACTION


IN THIS CHAPTER (Cont.) Methylnaltrexone Peripheral opioid receptor antagonist (see
Alvimopan Chapter 10)
Ondansetron (ZOFRN, others)
Polyethylene glycol (COLYTE, GOLYTELY, Dexpanthenol Enhances acetylcholine synthesis in the gut
others)
Laxatives Saline laxatives Osmotic increase in intraluminal uid and
Prucalopride (RESELOR)available inEurope Magnesium sul ate, increased motility
or use in women with chronicconstipation hydroxide, citrate
Sodium phosphate
Saline laxatives
Senna (SENOKOT,EX-LAX,others) Lactulose Nondigestible sugar which causes an
osmotic increase in intraluminal uid and
Sul asalazine (AZULFIDINE) increased motility

Polyethylene glycol Nondigestible alcohol which causes an


MECHANISMS OF ACTION osmotic increase in intraluminal uid and
increased motility
OF LAXATIVES
Enhanced retention o intraluminal uid by Docusate sodium Anionic sur actant that lowers the sur ace
tension o stool, so tens stool, and permits
hydrophilicor osmoticmechanisms
easier de ecation
Bulk- ormingagents(bran, psyllium, etc);
hydrophiliccolloids (methylcellulose, etc) Bisacodyl Stimulation o enterocytes, enteric neurons,
and GI smooth muscle
Osmoticagents (nonabsorbable
inorganicsalts or sugars) Antidiarrheal Agents Carboxymethylcellulose Absorbs intestinal liquids
Stool-wetting agents (sur actants) Cholestyramine Binds bile acids and some bacterial toxins
and emollients (docusate)
Bismuth Antisecretory, anti-in ammatory and
Decreased net absorption o uid byef ects
antimicrobial e ects
on small- and large-bowel uid and elec-
trolyte transport Loperamide opioid receptor agonist (see Chapter 10)
Diphenylmethanes (bisacodyl) Diphenoxylate
Di enoxin
Anthraquinones (senna and cascara)
Clonidine 2 receptor agonist
Castor oil
Altered motilitybyeither inhibiting seg- Octreotide Somatostatin analog
menting (nonpropulsive) contractions or
Alosetron 5-HT3 antagonist that produces a decrease
stimulating propulsive contractions in GI contractility and increased uid
5-HT4 receptor agonists absorption
Dopamine receptor antagonists Antispasmodics Dicyclomine Anticholinergic (see Chapter 6)
Motilides (erythromycin) Hyoscyamine
Glycopyrrolate
Methscopolamine

Antinausea and Ondansetron 5-HT3 antagonist


Antiemetic Agents

Aprepitant Substance P receptor antagonist

Dronabinol Stimulation o cannabinoid receptors


Nabilone

Agents to Treat Sul asalazine Intestinal anti-in ammatory e ect


In ammatory Bowel Olsalazine (see Chapter 22)
Disease Balsalazide
Mesalamine (5-ASA)

Budesonide Enteric glucocorticoid with local anti-


in ammatory e ects (see Chapter 29)

Azathioprine Impairs purine synthesis and cell


proli eration (see Chapter 23)
(Continued)
498
Drugs Used or the Treatment o Bowel Disorders CHAPTER 3 3

DRUG CLASS DRUGS MECHANISM OF ACTION


Cyclosporine Calcineurin inhibitor (see Chapter 23)

Methotrexate Inhibition o dihydro olate reductase (see


Chapter 46)

In iximab Binds to and neutralizes TNF


Adalimumab (see Chapter 23)
Certolizumab pegol

Natalizumab Binds 4 integrin (see Chapter 23)

CASE 33-1 CAUSES OF DIARRHEA


A 56-year-old man returns to his orthopedic surgeon 2 weeks a er repair o a torn Increased osmoticload within the intestine.
anterior cruciate ligament. He has had a considerable amount o pain and has been Excessive secretion o electrolytes and
taking oral hydrocodone on a regular basis daily. He complains today o constipation water into the intestinal lumen.
mani ested by decreased stool requency and dif culty with his bowel movements.
Exudation o protein and uid romthe
a. What is the likely cause o his constipation? intestinal mucosa.
Figure 33-1 shows the volume and composition o uid that traverses the small Altered intestinal motilityresulting in rapid
and large intestine daily. Opiate analgesics such as hydrocodone reduce propulsa- transit (and decreased uid absorption).
tile activity in the small and large intestine, and decrease intestinal secretions (see
Chapter 10). T ese actions lead to increased water absorption, increasing viscosity
o bowel contents and constipation.
b. What is a rational approach to this type o constipation?
able 33-1 shows a classi cation o laxatives and able 33-2 shows the e ects o
some laxatives on bowel unction. In this case the constipation is speci cally due
to the use o an opioid narcotic or pain. Decreasing the use o the opiate narcotic
should improve the situation. I this is not possible, increasing ber in the diet as
well as good hydration should be encouraged.
c. Is there an alternative i a high ber diet is not ef ective?
Methylnaltrexone is a peripherally acting opioid receptor antagonist that speci -
cally targets the underlying reason or the constipation, without limiting centrally
produced analgesia.

Rate Io n Co nc e ntratio ns
(lite rs /day) (mEq/lite r) Os mo lality
H2 O _ _
Flow Na + K+ Cl HCO 3
Upta ke
9.0 60 15 60 15 va ria ble

6.0

3.0 140 6 100 30 is otonic

1.5

1.5 140 8 60 70 is otonic

1.4

0.1 40 90 15 30 is otonic

FIGURE 33-1 The approximate volume and composition o uid that traverses the small and
large intestines daily. O the 9 L o uid typically presented to the small intestine each day, 2 L are
rom the diet and 7 L are rom secretions (salivary, gastric, pancreatic, and biliary). The absorptive
capacity o the colon is 4 to 5 L per day.

499
SECTION VI Drugs Af ecting Gastrointestinal Function

TABLE 33-1 Classi cation o Laxatives


1. Luminally active agents
Hydrophilic colloids; bulk- orming agents (bran, psyllium, etc)
Osmotic agents (nonabsorbable inorganic salts or sugars)
Stool-wetting agents (sur actants) and emollients (docusate, mineral oil)
2. Nonspeci c stimulants or irritants (with e ects on f uid secretion and motility)
Diphenylmethanes (bisacodyl)
Anthraquinones (senna and cascara)
Castor oil
3. Prokinetic agents (acting primarily on motility)
5-HT4 receptor agonists
Dopamine receptor antagonists
Motilides (erythromycin)

CASE 33-2
A 36-year-old woman presents with a 2-day history o diarrhea and abdominal pain
ollowing a weekend camping trip. She appears acutely ill and is dehydrated.
a. What are the issues to address in this particular patient?
T e presumptive diagnosis in this patient is giardiasis which has resulted rom the
ingestion o Giardia intestinalis in contaminated drinking water (see Chapter 36).
T e diagnosis is made by identi cation o cysts or trophozoites in ecal specimens
or trophozoites in duodenal contents. While waiting on a de nitive diagnosis,
treatment with an antimicrobial such as metronidazole can be started, and her
dehydration should be addressed.
b. What drugs might be used to treat her diarrhea?
An opioid that acts through peripheral opioid receptors such as loperamide or
diphenoxylate/atropine should be e ective in decreasing the diarrhea. Neither drug
should be used or a prolonged period o time.
(Continued)

TABLE 33-2 Summary o Ef ects o Some Laxatives on Bowel Function


SMALL BOWEL COLON
MIXING PROPULSIVE
AGENT TRANSIT TIME CONTRACTIONS CONTRACTIONS MASS ACTIONS STOOL WATER
Dietary ber ? ?

Magnesium

Lactulose ? ? ?

Metoclopramide ? ?

Cisapride ? ?

Erythromycin ? ? ? ?

Naloxone

Anthraquinones

Diphenylmethanes

Docusates ? ? ?

Modi ed rom Kreek, M.J. Constipation syndromes. In, A Pharmacological Approach to Gastrointestinal Disorders. (Lewis, J.H., ed.) Williams &
Wilkins, Baltimore, 1994, pp. 179208. With permission. http://lww.com.
500
Drugs Used or the Treatment o Bowel Disorders CHAPTER 3 3

c. What untoward ef ects might this patient expect?


Loperamide lacks signi cant abuse potential and is more e ective in treating
diarrhea than diphenoxylate. Overdose can result in CNS depression (especially in
children) and paralytic ileus. In patients with IBD loperamide should be used with
caution to prevent the development o toxic megacolon.

CASE 33-3
A 57-year-old man comes to hospital or his rst cycle o chemotherapy or small cell
lung cancer.
a. What drug is a good choice or prophylactic antiemesis therapy or this patient?
Figure 33-2 shows the myriad o signaling pathways that are involved in the control
o emesis and shows that cytotoxic drugs stimulate 5-H 3 receptors in the GI system
and chemoreceptor trigger zone. able 33-3 is a general classi cation o antiemetic
agents and shows drugs that are e ective in treating cytotoxic drug-induced emesis.
Ondansetron is one o the 5-H 3 receptor antagonists that are e ective in this
setting. It should be administered intravenously 30 minutes be ore chemotherapy.
b. Is there any clinical situation in which the dose o ondansetron should
be altered?
Ondansetron is extensively metabolized in the liver by CYP1A2, CYP2D6, and
CYP3A4. Patients with hepatic dys unction have reduced plasma clearance and
some adjustment o the dosage is advisable.
(Continued)

Me mory, fe a r,
HIGHER
dre a d, a nd
CENTERS
a nticipa tion

(me dulla )
EMETIC CENTER S OLITARY TRACT
CEREBELLUM NUCLEUS
(5-HT3 , D2 , M, H1 , NK1 , CB 1 )
CNS (H1 , M)
(are a po s tre ma)
blood-bra in CHEMO-RECEPTOR
ba rrie r TRIGGER ZONE
(5-HT3 , D2 , M1 , CB 1 )
INNER EAR
PERIPHERY va ga l a nd
(motion)
sympa the tic
a minog lyc os id e s STOMACH a ffe re nts
S MALL INTESTINE
S ENS ORY INP UT (5-HT3 )
(pa in, s me ll, s ight)
glos s opha rynge a l
a nd trige mina l
BLOOD-BORNE EMETICS
a ffe re nts
cytotoxic d rug s
op ioid s
LOCAL IRRITANTS
c holinomime tic s
cytotoxic d rug s
ca rd ia c g lyc os id e s P HARYNX
CuS O 4
L-DOPA (ga gging)
ra d ia tion
b romoc rip tine
b a c te ria
a p omorp hine
virus e s
e me tine (ip e ca c )

FIGURE 33-2 Pharmacologists view o emetic stimuli. Myriad signaling pathways lead rom the periphery to the emetic center. Stimulants o
these pathways are noted in italics. These pathways involve speci c neurotransmitters and their receptors (bold type). Receptors are shown or
dopamine (D2), acetylcholine (muscarinic, M), histamine (H1), cannabinoids (CB1), substance P (NK1), and 5-hydroxytryptamine (5-HT3). Some o
these receptors also may mediate signaling in the emetic center.
501
SECTION VI Drugs Af ecting Gastrointestinal Function

TABLE 33-3 General Classi cation o Anti-emetic Agents


TYPE OF VOMITING MOST
ANTI-EMETIC CLASS EXAMPLES EFFECTIVE AGAINST
5-HT3 receptor antagonistsa Ondansetron Cytotoxic drug induced
emesis

Centrally acting dopamine Metoclopramide b Cytotoxic drug induced


receptor antagonists Promethazine c emesis

Histamine H1 receptor Cyclizine Vestibular (motion sickness)


antagonists

Muscarinic receptor Hyoscine (scopolamine) Motion sickness


antagonists

Neurokinin receptor Aprepitant Cytotoxic drug induced


antagonists emesis (delayed vomiting)

Cannabinoid receptor Dronabinol Cytotoxic drug induced


agonists emesis
a
The most e ective agents or chemotherapy-induced nausea and vomiting are the 5-HT3
antagonists and metoclopramide. In addition to their use as single agents, they are o ten
combined with other drugs to improve ef cacy as well as reduce the incidence o side e ects.
b
Also has some peripheral activity at 5-HT3 receptors.
c
Also has some antihistaminic and anticholinergic activity.

c. I the ondansetron is not ef ective in this patient, is there an additional therapy


that may be used?
Dronabinol and nabilone are cannabinoids that react with the cannabinoid receptor
(CB1) and are use ul prophylactic agents in patients receiving cancer chemotherapy
when other antiemetic medications are not e ective.

CASE 33-4
A 42-year-old man with ulcerative colitis is having a are in symptoms with severe
abdominal pain and bloody diarrhea or the past 24 hours. He is being treated with sul-
asalazine but has not been compliant because o a persistent annoying rash.
a. What are options with his therapy?
T e rst option that should be considered is glucocorticoid therapy to rapidly alle-
viate his intestinal in ammation and curtail his diarrhea. I he is volume depleted,
this should also be addressed.
b. What are the side ef ects o sul asalazine therapy and what might be done to
increase this patients compliance?
Figure 33-3 shows the structures o sul asalazine and related agents. Since the side
e ects o sul asalazine are mostly related to the sul apyridine moiety, an option in
this patient would be to switch rom sul asalazine to a newer preparation such as
olsalazine or balsalazide which does not contain sul apyridine.

CASE 33-5
A 37-year-old man with ulcerative colitis has been treated in the past with azathioprine
but a er 6 months o therapy, there was no signi cant improvement in his symptoms.
a. How is azathioprine metabolized?
Figure 33-4 shows the metabolism o azathioprine. Azathioprine is a prodrug that
is converted to mercaptopurine. Mercaptopurine is metabolized by thiopurine
(Continued)
502
Drugs Used or the Treatment o Bowel Disorders CHAPTER 3 3

S ulfas alazine a za thioprine 6-me thyl-me rca ptopurine


HOOC TPMT
H N
HO N N S O2 N

6-me rca ptopurine XO 6-thiouric a cid


HOOC
H N HGPRT
HO NH2 H2 N S O2 N
6-thioinos inic a cid 6-thiogua nine
5-ASA nucle otide s
Me s a la mine S ulfa pyridine
FIGURE 33-4 Metabolism o azathioprine and 6-mercaptopurine.
HGPRT, hypoxanthineguanine phosphoribosyl trans erase; TPMT,
Ols alazine
thiopurine methyltrans erase; XO, xanthine oxidase. The activities o
Na OOC COONa these enzymes vary among humans because genetic polymorphisms
are expressed di erentially, explaining responses and side e ects
HO N N OH when azathioprinemercaptopurine therapy is employed.

Bals alazide
COONa
O
Na OOCCH2 CH2 NH C N N OH

FIGURE 33-3 Structures o sul asalazine and related agents.


The blue N atoms indicate the diazo linkage that is cleaved to
generate the active moiety.

methyltrans erase ( PM ) to 6-methyl-mercaptopurine and by hypoxanthine-


guanine phosphoribosyl trans erase (HGPR ) to 6-thioguanine nucleotides.
b. How might polymorphisms in the metabolism o azathioprine lead to the lack o
response to azathioprine in this patient?
Mercaptopurine has 3 metabolic ates: (1) conversion by xanthine oxidase to
6-thiouric acid, (2) metabolism by PM to 6-methyl-mercaptopurine (6-MMP),
and (3) conversion to 6-thioguanine nucleotides by HGPR (see Figure 33-4).
Polymorphisms o PM in a given individual will determine the ate o mercapto-
purine and the levels o 6-thioguanine and 6-MMP in each patient. Prior to initiating
therapy patients may bene t rom an evaluation o PM activity to establish the
likelihood o an adverse e ect such as suppression o bone marrow, or a er the ther-
apy has been started to measure the 6-thioguanine/6-MMP levels in individuals not
responding to therapy.

CASE 33-6
A 32-year-old woman with the diagnosis o Crohns disease initially had a good
response to glucocorticoid therapy with a one-year remission. However, despite
glucocorticoid therapy, she is now having requent ares o abdominal pain and
bloody diarrhea.
a. What options are available or her treatment o this relapse?
Patients who relapse requently may be treated with immunosuppressive agents such
as azathioprine-mercaptopurine. In iximab is an immunoglobulin that neutralizes
NF and is particularly use ul in closing stulas associated with Crohns disease.
T e combination o in iximab and azathioprine is more e ective than in iximab
alone in induction o remission and mucosal healing in steroid-resistant patients.
(Continued)
503
SECTION VI Drugs Af ecting Gastrointestinal Function

b. How should this patient be monitored or the appearance o serious side ef ects?
T e most serious idiosyncratic reaction to azathioprine is pancreatitis. T e most
serious dose-related adverse reaction to azathioprine is bone marrow suppression.
Circulating blood counts should be monitored closely when azathioprine therapy
is initiated and at 3-month intervals during maintenance therapy. Antibodies to
in iximab can decrease its clinical ef cacy. In iximab is also associated with
increased incidence o respiratory in ections. O particular concern is reactivation
o tuberculosis or other granulomatous in ections with subsequent dissemination.

KEY CONCEPTS
Although many choices exist or treating constipation, most therapies are
empirical and nonspeci c.
Most cases o constipation can be addressed by increasing ber intake, avoiding
constipating medications, and the judicious use o osmotic laxatives on an
as-needed basis.
Stimulant laxatives, although e ective, should be avoided or long-term use.
I no underlying cause can be determined, chronic diarrhea can be treated
empirically with bulk- orming and hygroscopic agents ollowed by opiates such
as loperamide.
5-H 3 receptor antagonists, such as ondansetron, are primary therapy or nausea
and vomiting, especially in the postchemotherapy and postoperative settings.
T e cannabinoids, dronabinol or nabilone, may be e ective or re ractive cases
o nausea and vomiting.
Acute exacerbations o ulcerative colitis are treated with colonic-release
preparations o mesalamine (5-ASA) and glucocorticoids.
Maintenance therapy or patients with ulcerative colitis is with one o the
5-ASA compounds; in patients who relapse, azathioprine-mercaptopurine
may be used.
Monitoring the activity o PM and the metabolites o mercaptopurine may
provide personalized therapy with this drug.
Drugs used in mild to moderately active Crohns disease include sul asalazine,
budesonide, and oral corticosteroids.
In iximab and other biological agents are use ul in closing stulas associated
with Crohns disease, but their use in maintaining patients in remission must be
balanced against the risk o adverse e ects.
Antibiotics, particularly metronidazole, are used or the acute treatment o com-
plications associated with Crohns disease, but are not used as a routine therapy.

SUMMARY QUIZ

QUESTION 33-1 A 23-year-old woman with a history o type 1 diabetes since age
6 presents with nausea and vomiting associated with gastroparesis and delayed gastric
emptying. Which o the ollowing medications would be the most suitable treatment o
the nausea and vomiting in this patient?
a. Dephenoxylate
b. Metoclopramide
c. Ondansetron
d. Dronabinol
e. Loperamide
(Continued)

504
Drugs Used or the Treatment o Bowel Disorders CHAPTER 3 3

QUESTION 33-2 A 64-year-old woman is receiving ondansetron or the prophylactic


treatment o emesis secondary to chemotherapy. Ondansetrons main mechanism o
action is antagonism o
a. central opioid receptors.
b. peripheral H 2 receptors.
c. central dopamine receptors.
d. central 5-H 3 receptors.
e. peripheral muscarinic receptors.

QUESTION 33-3 A 35-year-old woman has severe irritable bowel syndrome character-
ized by requent and pro use diarrhea. She is being treated with alosetron because she
has not responded to other orms o antidiarrheal therapy. T e physician treating this
patient must diligently monitor or
a. ischemic colitis.
b. congestive heart ailure.
c. drug-induced hepatitis.
d. renal ailure.
e. pulmonary brosis.

QUESTION 33-4 A 44-year-old man has been treated or chronic back pain with opioid
narcotics. He has been plagued with constipation that he has managed to treat by increas-
ing the ber in his diet. T is is no longer e ective. Which o the ollowing medications is
likely to be o bene t in this patient?
a. Naltrexone
b. Metoclopramide
c. Methylnaltrexone
d. Bisacodyl
e. Lactulose

QUESTION 33-5 A 53-year-old man with a long history o alcohol abuse and hepatic
cirrhosis is now developing hepatic encephalopathy. Which o the ollowing agents
may be most e ective in reducing the signs and symptoms o this patients hepatic
encephalopathy?
a. Loperamide
b. Bisacodyl
c. Glycerin
d. Docusate sodium
e. Lactulose

QUESTION 33-6 A 27-year-old woman with ulcerative colitis is being treated with
sul asalazine. She should be closely monitored or which o the ollowing common
adverse e ects with this drug?
a. Hearing loss
b. Skin rash
c. Blood in her urine
d. Heart arrhythmia
e. Onset o seizures

QUESTION 33-7 A 42-year-old woman with Crohns disease has been treated
with glucocorticoid therapy. During her initial high dose therapy, she responded
well and the dose was tapered over 3 months. However, she relapsed with severe
(Continued)

505
SECTION VI Drugs Af ecting Gastrointestinal Function

symptoms during the last month o her taper. T e reason or her relapse is most
likely because
a. her taper was too rapid.
b. she is glucocorticoid-responsive.
c. she is glucocorticoid-unresponsive.
d. she was noncompliant with her steroid therapy.
e. she is glucocorticoid-dependent.

QUESTION 33-8 A 56-year-old man with ulcerative colitis is being treated with azathi-
oprine. It is determined that his blood concentration ratio o 6-thioguanine/6-methyl-
mercaptopurine is lower than normal. T e most likely reason or this nding is that he
has increased activity o which o ollowing enzymes?
a. Xanthine oxidase
b. Hypoxanthine-guanine phosphoribosyl trans erase (HGPR )
c. Acetyl trans erase
d. T iopurine methyltrans erase ( PM )
e. Cytochrome P450 3A4

QUESTION 33-9 A 19-year-old woman with ulcerative colitis had to stop taking sul-
asalazine because o a skin rash. She is now taking olsalazine which is also converted
to 5-ASA but she has experienced no side e ects. T e most likely reason or the lack o
side e ects with olsalazine in this patient is because the olsalazine
a. is absorbed in the jejunum a er oral use.
b. does not contain a sul a moiety.
c. has a pH-sensitive release ormulation.
d. is given intravenously.
e. is given by enema.

SUMMARY QUIZ ANSWER KEY

QUESTION 33-1 Answer is b. Metoclopramide is a dopamine (DA) receptor antago-


nist that is commonly used to treat the symptoms o delayed gastric emptying. Another
drug that might be used is cisapride although it is available only through a limited use
program because o its incidence o serious cardiovascular toxicities. Erythromycin is
also a possible alternative.

QUESTION 33-2 Answer is d. Ondansetron is a central 5-H 3


receptor antagonist.
able 33-3 shows a general classi cation o anti-emetic drugs.

QUESTION 33-3 Answer is a. Alosetron is a 5-H 3


receptor antagonist that reduces
GI contractility with decreased colonic transit and increased uid absorption. T e
drug was withdrawn rom the US market (then reapproved by the FDA) because o
an unusually high incidence o ischemic colitis (up to 3 per 1000 patients). T e cause
is not ully established but may result rom the drugs ability to suppress intestinal
relaxation.

QUESTION 33-4 Answer is c. Methylnaltrexone is an antagonist o peripheral


opioid receptors that is e ective in increasing bowel movements in patients with
opioid-induced constipation without limiting centrally produced analgesia.

QUESTION 33-5 Answer is e. Lactulose is a synthetic disaccharide o galactose and


ructose that resists intestinal disaccharidase activity and osmotically draws water
into the intestinal lumen. Patients with severe liver disease have an impaired capac-
ity to detoxi y ammonia coming rom the colon, where it is produced by bacterial
(Continued)
506
Drugs Used or the Treatment o Bowel Disorders CHAPTER 3 3

metabolism o ecal urea. T e drop in luminal pH that accompanies hydrolysis o lactu-


lose to short-chain atty acids in the colon results in trapping o ammonia by its con-
version to the polar ammonium ion. Combined with the increases in colonic transit,
lactulose therapy signi cantly lowers circulating ammonia levels.

QUESTION 33-6 Answer is b. Sul apyridine, the sul a moiety o sul asalazine (see
Figure 33-3), is responsible or most o its adverse reactions. Skin rash is a particu-
larly common reaction with sul a drugs and may be a harbinger o Stevens-Johnson
syndrome.

QUESTION 33-7 Answer is e. Patients who are glucocorticoid-dependent respond ini-


tially to glucocorticoids but then experience a relapse o symptoms as the steroid dose
is tapered. T ese patients are classi ed as glucocorticoid-dependent (see Chapter 29).

QUESTION 33-8 Answer is d. Patients who are rapid metabolizers with respect to
PM shunt mercaptopurine metabolism away rom 6-thioguanine nucleotides and
toward 6-MMP (see Figure 33-4 and Case 33-5).

QUESTION 33-9 Answer is b. Olsalazine does not contain a sul apyridine moiety
(see Figure 33-3).

SUMMARY: DRUGS USED TO TREAT BOWEL DISORDERS


TOXICITIES
CLASS AND SUBCLASSES NAMES CLINICAL USES COMMON UNIQUE; CLINICALLY IMPORTANT
Prokinetic Agents Metoclopramide Ameliorate the nausea and Extrapyramidal e ects, tardive
vomiting associated with GI dyskinesia
dysmotility

Domperidone Gastroparesis

Cisapride Available only through a Cardiac arrhythmia including


limited access program ventricular tachycardia
or GERD, gastroparesis,
pseudoobstruction

Prucalopride Treatment o chronic


constipation in women

Erythromycin Diabetic gastroparesis See Chapter 41

Lubiprostone Chronic constipation and Nausea, headache,


irritable bowel syndrome with diarrhea, allergic
constipation reactions

Opioid Antagonists Methylnaltrexone Opioid-induced constipation


Alvimopan

Enhancer o Acetylcholine Dexpanthenol Treatment o postoperative Mild hypotension Shortness o breath


Synthesis ileus and local irritation

Laxative Saline laxatives Treatment o constipation Increased salt load


Magnesium sul ate, Prolonged use associated with
hydroxide, citrate electrolyte imbalance
Sodium phosphate

Lactulose Treatment o constipation Abdominal Prolonged use associated with


Treatment o hepatic discom ort, electrolyte imbalance
encephalopathy atulence

Polyethylene glycol Treatment o constipation Prolonged use associated with


Bowel preparation or electrolyte imbalance
endoscopy and surgery
(Continued)
507
SECTION VI Drugs Af ecting Gastrointestinal Function

TOXICITIES
CLASS AND SUBCLASSES NAMES CLINICAL USES COMMON UNIQUE; CLINICALLY IMPORTANT

Docusate sodium Stool so tening and promotes


ease o de ecation

Bisacodyl Treatment o constipation Prolonged use associated with


electrolyte imbalance and atonic
colon

Antidiarrheal Agents Carboxymethylcellulose Treatment o diarrhea

Cholestyramine Treatment o bile salt-induced See Chapter 20


diarrhea

Bismuth Treatment o diarrhea Danger o salicylate poisoning


with bismuth subsalicylate

Loperamide Treatment o diarrhea Used with caution in patients with


in ammatory bowel disease to
prevent toxic megacolon

Diphenoxylate Treatment o diarrhea Combined with low High doses cause CNS depression
combined with atropine dose o atropine to and anticholinergic e ects
Di enoxin combined prevent abuse
with atropine

Clonidine Treatment o chronic diarrhea See Chapter 7 See Chapter 7


in diabetics

Octreotide Treatment o hormone- Nausea and bloating Long-term treatment can lead
secreting tumors o the to gallstones and hypo- or
pancreas and GI tract hyperglycemia

Alosetron Treatment o diarrhea in Ischemic colitis in a small number


women with irritable bowel o patients
syndrome (IBS)

Antispasmodics Dicyclomine Treatment o pain associated Light-headedness,


Hyoscyamine with IBS drowsiness,
nervousness

Glycopyrrolate Treatment o pain associated


with IBS

Methscopolamine Treatment o pain associated


with IBS

Antinauseants and Ondansetron Treatment o nausea and Constipation or


Antiemetic Agents chemotherapy-induced diarrhea, headache,
emesis light-headedness

Aprepitant Treatment o chemotherapy- Contraindicated in patients with


induced nausea and emesis li e-threatening QT prolongation
Interacts with other substrates o
CYP3A4

Dronabinol Treatment o chemotherapy- Palpitations, Marijuana highsand abstinence


Nabilone induced nausea and emesis tachycardia, syndrome upon withdrawal
hypotension
(Continued)

508
Drugs Used or the Treatment o Bowel Disorders CHAPTER 3 3

TOXICITIES
CLASS AND SUBCLASSES NAMES CLINICAL USES COMMON UNIQUE; CLINICALLY IMPORTANT
Agents to Treat Sul asalazine Treatment o IBD particularly Headache, nausea, Rash, bone marrow suppression,
In ammatory Bowel ulcerative colitis atigue Stevens-Johnson syndrome,
Disease (IBD) hepatitis, pneumonia: All due to
sul a moiety

Mesalamine (5-ASA) Treatment o IBD particularly Headache, Has been associated with
ulcerative colitis dyspepsia, skin rash interstitial nephritis; renal unction
should be monitored in all
patients

Olsalazine Treatment o IBD particularly Headache, nausea,


ulcerative colitis dyspepsia, diarrhea

Balsalazide Treatment o IBD particularly Headache, nausea,


ulcerative colitis dyspepsia

Budesonide Enteric release o synthetic


steroid used or ileocecal
Crohns disease

Azathioprine Treatment o IBD Fever, rash, Idiosyncratic pancreatitis


arthralgias, nausea, Dose-related bone marrow
and vomiting suppression
Rare cholestatic hepatitis

Cyclosporine Second-line treatment o IBD See Chapter 23

Methotrexate Treatment o steroid- See Chapter 48


resistant IBD

In iximab Treatment o IBD Increased incidence o respiratory


in ections and potential
reactivation o TB

Adalimumab Treatment o IBD Natalizumab reacts with other


Certolizumab pegol immune-modulating drugs to
Natalizumab increase the risk o multi ocal
leukoencephalopathy

509
This page intentionally left blank
SECTION

Chemotherapyof
Microbial Diseases VII
34. General Principles o Antimicrobial T erapy 512

35. Chemotherapy o Malaria 520

36. Chemotherapy o Protozoal In ections:


Amebiasis, Giardiasis, richomoniasis, rypanosomiasis,
Leishmaniasis, and Other Protozoal In ections 530

37. Chemotherapy o Helminth In ections 536

38. Sul onamides, rimethoprim-Sul amethoxazole,


Quinolones, and Agents or Urinary ract In ections 542

39. Penicillins, Cephalosporins,


and Other -Lactam Antibiotics 549

40. Aminoglycosides 559

41. Protein Synthesis Inhibitors and Miscellaneous


Antibacterial Agents 568

42. Chemotherapy o uberculosis, Mycobacterium Avium


Complex Disease, and Leprosy 578

43. Anti ungal Agents 588

44. Antiviral Agents and reatment o HIV In ection 600

511
CHAPTER

34 General Principles of
Antimicrobial Therapy
T is chapter will be most use ul a er having a basic understanding o the material in
CLASSIFICATION OF
Chapter 48, General Principles o Antimicrobial T erapy in Goodman & Gilmans T e
ANTIMICROBIAL AGENTS Pharmacological Basis of T erapeutics, 12th Edition Neither Mechanisms o Action nor
The rst broad classi cation o antimicro- Clinical Summary ables are included in this chapter because this in ormation is pro-
bial agents is according to the microorgan- vided in subsequent chapters
isms theyare active against: antibacterial,
In addition to the material presented here, Chapter 48 o the 12th Edition contains:
antiviral, anti ungal, antiparasitic.
A detailed discussion o the pharmacokinetic basis or antimicrobial therapy
Further classi cation o an antibioticis
based on: A thorough discussion o the types and goals o antimicrobial therapy
The class and spectrumo the
LEARNING OBJECTIVES

microorganisms it kills
Describe the pharmacokinetic basis o antimicrobial therapy
The biochemical pathwayit inter eres
with Identi y the importance o susceptibility testing o antimicrobial agents
The chemical structure o its Identi y the actors that orm the basis or selection o an antimicrobial dose
pharmacophore (the chemical moiety and dosing schedule
o the drug that binds to a microbial Describe the types and goals o antimicrobial therapy
receptor)
Identi y the mechanisms o resistance to antimicrobial agents

DRUGS INCLUDED IN THIS CHAPTER


T e pharmacology o speci c drugs is not included in this chapter T e drugs presented
in the cases are used as examples to illustrate various aspects o antimicrobial therapy;
FACTORS THAT AFFECT THE
the speci c pharmacology o these drugs is presented in the subsequent chapters o
PENETRATION OF A DRUG this section
INTO AN ANATOMICAL
COMPARTMENT
CASE 34-1
Physical barriers (eg, bloodbrain barrier)
Chemical properties o the drug (eg, A 34-year-old woman has a diagnosis o pneumococcal meningitis T e pathogen
octanol-water partition coe cient) is determined to be sensitive to penicillin and she is being treated with high-dose
penicillin
Membrane transporters (eg,
p-glycoprotein) a. How does the chemical nature o a drug a ect its penetration into various ana-
tomic compartments such as the brain?
An important actor is the drugs lipid solubility or its octanol:water partition
coe cient. T e higher the octanol:water partition coe cient, the more likely it is
to penetrate physical barriers erected by layers o cells. T e more charged a drug
molecule, and the larger it is, the poorer its penetration across physical barriers.
b. What is unique about the brain in terms o drug penetration?
T e central nervous system (CNS) is guarded by the bloodbrain barrier. T e
movement o antibiotics across the bloodbrain barrier is restricted by tight junc-
tions that connect endothelial cells o cerebral microvessels to one another in the
brain parenchyma, as well as by protein transporters. Antimicrobial agents that are
polar at physiological pH, such as penicillin G, generally penetrate poorly. Some
drugs, such as penicillin G, are actively transported out o the cerebrospinal uid.
However, the integrity o the bloodbrain barrier is diminished during active bacte-
rial in ection, such as pneumococcal meningitis, leading to an opening o the tight
junctions and the penetration o even polar drugs.
(Continued)
512
General Principles of Antimicrobial Therapy CHAPTER 3 4

8
)
l
m
/
U
F
6
C
10
E ma x
g
o
l
(
4
n
e
d
r
u
b
m
2 EC 50
s
i
n
a
g
r
O
0
0
[Antimicrobia l]

FIGURE 34-1 Inhibitory sigmoid Emax curve. CFU, colony orming units.

c. What other anatomical compartments require special attention or antimicro-


bial therapy?
T e ocular cavity, pulmonary epithelial linings, endocardial vegetations, and pros-
thetic devices such as arti cial heart valves, long-dwelling catheters, arti cial hips,
and devices or internal xation o bone ractures are compartments in which anti-
biotic penetration is poor.
d. How should the drug and its antimicrobial target be viewed?
Antimicrobial molecules should be viewed as ligands whose receptors are microbial
proteins and the relationship between drug concentration and e ect on a popula-
tion o organisms is modeled using the standard Hill-type curve or receptor and
agonist (see Figure 34-1). T e inhibitory concentration 50 or IC50 (also termed EC50
or e ective concentration 50) is a measure o the antimicrobial agents potency; the
Emax is the antimicrobial agents maximum e ect.

CASE 34-2
A 62-year-old man with chronic obstructive pulmonary disease develops a pumonary
in ection that rapidly becomes a systemic bacteremia requiring antibiotic therapy T e
microorganism has been isolated and susceptibility testing has been completed T e
minimum inhibitor concentration (MIC) is 0 5 mg/L and the serum t o the antibiotic
is 3 hours
a. In choosing a drug and dose regimen that will be most e ecitve in this patient
what must be considered?
T e microbiology laboratory plays a central role in the decision to choose a par-
ticular antimicrobial agent over others. T eir rst unction is the identi cation and
isolation o the pathogenic organism. Once this is accomplished, the rational choice
o the class o antibiotics to administer to the patient can be made. T en the labora-
tory per orms susceptibility testing to narrow down the list o possible antimicro-
bials that could be used. Susceptibility testing or bacteria employs antibiotics in
serially diluted concentrations o solid agar or in broth medium that contains a cul-
ture o the test microorganism. T e lowest concentration o the agent that prevents
visible growth a er 18 to 24 hours o incubation is known as the MIC.
In choosing a dose regimen, the rst consideration is to index drug exposure to
the MIC. Second, dose itsel is a poor measure o drug exposure, given between-
patient and within-patient pharmacokinetic variability. T e optimal dose o the
antibiotic or a patient is the dose that achieves inhibitory concentration (IC) o
IC80 to IC90 exposures at the site o in ection. A third consideration is that optimal
microbial kill by the antibiotic may be best achieved by optimizing certain shapes
(Continued)
513
SECTION VII Chemotherapy of Microbial Diseases

o the concentration-time curve (see answer to Case 34-2b below). Last, changes in
susceptibility to drug (ie, drug-resistance) can occur during therapy. T e change in
drug susceptibility o an organism is depicted in Figure 34-2. A shi to the right o
the antimicrobial concentration versus response curve, that is, an increase in the
IC50 (see Figure 34-2A), means that much higher concentrations are now needed to
show a speci c response to the drug and re ects increasing resistance to a particu-
lar antibiotic. A second possible change in the curve is a decrease in Emax (see Fig-
ure 34-2B), such that increasing the dose o the antibiotic beyond a certain point
will achieve no urther e ect. T is occurs because the available microbial target
proteins have been reduced or the microbe has developed an alternative pathway
to overcome the biochemical inhibition.
b. How does the dose administration schedule (once daily or 3 divided doses) a ect
the drug concentration-time pro le?
Figure 34-3A depicts the concentration-time curve o an antibiotic in which the
peak concentration (CPmax), area under the curve (AUC), and the raction o the
dosing interval ( ) or which the drug concentration remains above the MIC
( > MIC) are shown. Figure 34-3B shows concentration-time curves when the
same dose o the antibiotic is given as 3 equal doses administered at 0, 8, and
16 hours. T e AUC0-24 will be similar whether the dose was given once a day or
3 times a day. T us, or the same pathogen, the change in dose schedule does not
change the AUC0-24/MIC. However, the CPmax will decrease by a third when the total
dose is split into 3 doses and administered more requently (see Figure 34-3B). In
addition, the time that the drug concentration persists above MIC ( > MIC) is
slightly increased with the more requent dosing schedule.
(Continued)

A 100

80

60
e
s
n
o
p
s
40
e
R
20

B 100

80

60
e
s
n
o
p
E ma x
s
40
e
R
20

0
0 100
[Antimicrobia l]

FIGURE 34-2 Changes in sigmoid Emax model with increases in drug resistance. Increase in
resistance may show changes in IC50 (panel A: the IC50 increases rom 70 [gray line] to 100 [black
line], to 140 [blue line]) or decrease in Emax (panel B: e cacy decreases rom ull response [gray line]
to 70% [black line]).
514
General Principles of Antimicrobial Therapy CHAPTER 3 4

5 A PATIENTS AT RISK FOR


C ma x = ma ximum conce ntra tion
INFECTIVE ENDOCARDITIS
4
FOR WHICH
PROPHYLACTIC THERAPY
3
IS RECOMMENDED
2 AUC 0-24h = 24h a re a unde r the T ose with prosthetic material
conce ntra tion-time curve used or heart valve repair or
replacement
)
L
1
/
g
T>MIC
T ose who have had previous
m
Microbes MIC
(
n
0 in ective endocarditis
o
i
t
a
T ose with congenital heart
r
t
n
e
5 disease such as unrepaired cya-
c
B
n
notic heart disease, or within
o
c
g
4 6 months o repair o the heart
u
r
D
disease with prosthetic mate-
3 rial, or those with residual
de ects adjacent to the prosthetic
2 C ma x material
AUC 8-16h AUC 16-24h Postcardiac transplant patients
AUC 0-8
1 with heart valve de ects
MIC
T1 >MIC T2 >MIC T3 >MIC
0
0 3 6 9 12 15 18 21 24
Time in hours

FIGURE 34-3 Ef ect o dif erent dose schedules on shape o concentration-time curve. The
total AUC or the ractionated dose in curve B is determined by adding AUC0-8h, AUC8-16h, and
AUC 16-24h, which adds up to the same AUC0-24h in curve A. The time concentration exceeds MIC
is also determined by adding up T1>MIC, T2>MIC, and T3>MIC, which results in a raction greater
than that or curve A.

c. Which indices, AUC/MIC, or CPmax/MIC or > MIC is the most important to


determine the microbial kill?
T is depends on the antimicrobial. Some antibiotics (-lactam antibiotics, eg,
penicillin) kill best when drug concentration persists above MIC or longer dura-
tions o the dosing interval. A drug optimized by > MIC should be dosed more MECHANISMS OF
requently. Indeed the e ectiveness o penicillin is enhanced when it is given as a ANTIMICROBIAL
continuous in usion. RESISTANCE
T e peak concentration is what matters or some antimicrobial agents. Aminogly- Reduced entryo antibioticinto pathogen
cosides are a good example o this class as they previously were given three times a
Enhanced export o antibioticbypatho-
day but experience has shown that aminoglycosides are highly e ective when given
gen e uxpumps (see Side Bar MAJOR
once a day (see Chapter 40).
SYSTEMSOFEFFLUXPUMPSTHATAREREL-
T ere is a third group o antimicrobial agents or which the dosing schedule has no EVANTTOANTIMICROBIALAGENTS)
e ect on e cacy, but it is the cummulative e ect that matters. Daptomycin is an
Release o microbial enzymes that destroy
example o this group o drugs. T ese drugs also have a long postantibiotic e ect
the antibiotic
(PAE). T eir e ect continues long a er antibiotic concentrations decline below
the MIC. Alterations o microbial enzymes that
are required to trans ormprodrugs to the
d. How does the dosing schedule relate to resistance suppression? ef ective moieties
In many instances, the drug exposure required or resistance suppression is much Alteration o target proteins
higher than or optimal kill. Un ortunately, this higher exposure is also associated
Development o alternative biochemi-
with drug toxicity when doses are increased. Although the relationship between
cal pathways to those inhibited bythe
microbial kill and exposure is based on the inhibitory sigmoid Emax model, this
antibiotic
model does not apply to resistance suppression.

515
SECTION VII Chemotherapy of Microbial Diseases

MAJOR SYSTEMS OF CASE 34-3


EFFLUX PUMPS THAT A 54-year-old woman is undergoing hip replacement surgery Her surgeon placed her
ARE RELEVANT TO on a broad-spectrum antibiotic prior to surgery
ANTIMICROBIAL AGENTS
a. What is the goal o this antibiotic use?
Multidrug and toxiccompound extruder
Prophylaxis involves treating patients who are not yet in ected or have not yet devel-
(MATE)
oped disease (see Figure 34-4). It is used in immunosuppressed patients such as those
Major acilitator super amily(MFS) with HIV-AIDS, or are post-transplantation and on antirejection medications (see
transporters Chapter 23). In these patients, antimicrobial prophylactic therapy is administered
Small multidrug resistance (SMR) system based on the well-de ned pattern o pathogens that are major causes o morbidity dur-
Resistance nodulation division (RND) ing immunosuppression. In ections or which prophylaxis is commonly given include
exporters Pneumocystis jirovecii, Mycobacterium avium-intracellulare, oxoplasma gondii,
Candida species, Aspergillus species, Cytomegalovirus, and other Herpesviridae.
ATP-binding cassette (ABC) transporters
Postexposure prophylaxis is sometimes recommended as in ri ampin therapy to
prevent meningococcal meningitis in people who are in close contact with a case,
prevention o gonorrhea or syphilis a er contact with an in ected person, macro-
lide antibiotic therapy a er contact with con rmed cases o pertussis, and those
patients or health care workers inadvertently exposed to HIV in ection.
Prophylactic therapy is also recommended in pregnant patients to prevent
CHEMOPROPHYLAXIS transmission o HIV and syphilis.
USED TO PREVENT
WOUND INFECTIONS b. What principles should be ollowed when using antibiotics to prevent wound
in ections?
SHOULD FOLLOW THESE
PRINCIPLES T ese guidelines are listed in the Side Bar CHEMOPROPHYLAXIS USED O PRE-
VEN WOUND INFEC IONS SHOULD FOLLOW HESE PRINCIPLES. T e sur-
Antimicrobial activitymust be present at geon has ollowed the rst guideline by placing the patient on the antibiotic prior to
the time o wound closure the surgery so there is su cient antimicrobial activity at the time o wound closure.
Antimicrobial therapymust be active
c. T is patient has had a previous episode o in ective endocarditis. Does this make
against the most likelycontaminating
her a stronger candidate or prophylactic therapy?
microorganisms or that type o surgery
T is de nitely changes the situation with regard to prophylactic antibiotic therapy.
In clean surgical procedures, the expected
T e situations in which prophylactic therapy is recommended or patients at risk
incidence o wound in ection is less than
o in ective endocarditis are shown in the Side Bar PA IEN S A RISK FOR
5%, and antibiotics should not be used
INFEC IVE ENDOCARDI IS FOR WHICH PROPHYLAC IC HERAPY IS
routinely
RECOMMENDED.

CASE 34-4
A 59-year-old transient man comes to the emergency department with ever and di -
culty breathing He is diagnosed with pneumonia and a er blood and sputum cultures
are obtained he is started on a -lactam antibiotic T e next day his cultures show that
the pathogen is resistant to the -lactam antibiotic and he is switched to a combination
o a uoroquinolone and an aminoglycoside
(Continued)

Prophyla xis Pre e mptive Empiric De finitive S uppre s s ive

No infe ction Infe ction Symptoms Pa thoge n Re s olution


is ola tion

FIGURE 34-4 Antimicrobial therapy-disease progression timeline. Stages o disease progression are
below the horizontal arrow; categories o antimicrobial therapy are above the arrow.

516
General Principles of Antimicrobial Therapy CHAPTER 3 4

a. Has the -lactam antibiotic caused the pathogen to be resistant?


SPECIAL CIRCUMSTANCES
Mutation and antibiotic selection o the resistant mutant are the molecular basis or THAT FAVOR COMBINATION
resistance or many bacteria, viruses, and ungi. Mutations are not caused by drug ANTIMICROBIAL THERAPY
exposure per se. T ey are random events that con er a survival advantage when
drug is present. Drug resistance may be acquired by mutation and selection with Preventing resistance to monotherapy
passage o the trait vertically to daughter cells. Drug resistance more commonly Accelerating the rapidityo microbial kill
is acquired by horizontal trans er o resistance determinants rom a donor cell, Enhancing therapeutice cacybyuse o
o en rom another bacterial species, by transduction, trans ormation, or conjuga- synergisticantibioticinteractions
tion. Resistance acquired by horizontal trans er disseminates rapidly and widely
Paradoxically, reducing toxicity(ie, when
either by clonal spread o the resistant strain or by subsequent trans ers to other
ull e cacyo a standard antibacterial
susceptible recipient strains.
agent can onlybe achieved at doses that
b. What are the mechanisms o resistance to antimicrobial agents? are toxicto the patient)
Nowadays, every major antimicrobial class is associated with the emergence
o signi cant resistance. T e mechanisms o microbial resistance are shown in the
Side Bar MECHANISMS OF AN IMICROBIAL RESIS ANCE. T e emergence o
antibiotic resistance is associated with 2 actors: evolution and clinical/environmental
practices. Microorganisms develop 1 o the mechanisms o resistance by acquiring
genetic elements that code or the resistance mechanism, by mutating under anti-
microbial pressure, or by constitutive induction o a resistance mechanism.
Small polar molecules, including many antibiotics, enter the pathogen cell through
protein channels called porins. Absence o , mutation in, or loss o a avored porin
channel can slow the rate o drug entry or prevent entry altogether. Or, i the target
is intracellular and the drug requires active transport across the cell membrane, a
mutation or phenotypic change that slows or abolishes this transport mechanism
can con er resistance.
c. How are ef ux pumps in microbial membranes responsible or resistance?
Ef ux pumps (see Side Bar MAJOR SYS EMS OF EFFLUX PUMPS HA ARE
RELEVAN O AN IMICROBIAL AGEN S) are also a prominent mecha-
nism o resistance or parasites, bacteria, and ungi. Drug resistance to most
antimalarial drugs (see Chapter 35), but speci cally to chloroquine is mediated by
an ABC transporter encoded by Plasmodium falciparum multidrug resistance gene 1
(Pfmdr1). Point mutations in the Pfmdr1 gene lead to drug resistance and ailure
o chemotherapy.
d. Give some other examples o mechanisms o antimicrobial resistance.
Another mechanism o drug resistance is or the bacteria to enzymatically
inactivate the drug. Bacterial resistance to -lactam antibiotics (eg, penicillin,
see Chapter 39) is due to production o a -lactamase enzyme which destroys
the antibiotic.
A common mechanism or antimicrobial resistance is either single point or mul-
tiple point mutations that result in a change in amino acid sequence and con orma-
tion o a target protein. T is change leads to a reduced a nity o drug or its target
(eg, methicillin-resistant Staphylococcus aureus [MRSA] caused by production o a
low-a nity penicillin-binding protein).
Rarely an organism not only becomes resistant to an antimicrobial agent but
subsequently starts requiring it or growth (eg, enterococcus, which develops
vancomycin resistance, can, a er prolonged exposure, develop vancomycin-
requiring strains).
Nucleoside reverse transcriptase inhibitors such as zidovudine (see Chapter 44)
are incorporated into the viral DNA chain and cause chain termination. Resistance
emerges when mutations occur at a variety o points in the reverse transcriptase
gene, and phosphorolytic excision o the incorporated chain-terminating nucleo-
side analog is enhanced.

517
SECTION VII Chemotherapy of Microbial Diseases

CASE 34-5
A 35-year-old patient with HIV in ection is started on 3 drugs in combination
a. What are the problems o using combination therapy?
Using multiple antimicrobial agents where only one is required leads to increased
toxicity and unnecessary damage to the patients otherwise protective ungal and
bacterial ora.
b. Why is this patient started on 3 drugs?
T ere are special circumstances where evidence is unequivocal in avor o com-
bination therapy. T ese are listed in the Side Bar SPECIAL CIRCUMS ANCES
HA FAVOR COMBINA ION AN IMICROBIAL HERAPY. Clinical situations
or which combination therapy is used are discussed in the relevant chapters but
include antiretroviral therapy or AIDS (see Chapter 44), antiviral therapy or hepa-
titis B and C (see Chapter 44), and the treatment o tuberculosis (see Chapter 42).

KEY CONCEPTS
Antimicrobial agents should be viewed as ligands whose receptors are microbial
proteins
Knowledge o an antibiotics pharmacokinetic variability (due to such actors as
genetics, age, weight, disease status) leads to better dose adjustments
Proper interpretation o microbial susceptibility testing (minimum inhibitory
concentration, MIC) is the rst step in the selection o an antibiotic dose and
dosing schedule
Maximizing the antibiotic concentration-time curve by selection o the proper
dosing schedule is critical to obtaining optimal microbial kill
Monotherapy o antibiotics is pre erred in order to limit the risk o toxicity and
the emergence o resistance; however, there are circumstances that avor combi-
nation antibiotic therapy
Resistance to antibiotics develops when there is acquisition o genetic elements
that code or one o several resistance mechanisms, or when mutations develop
under antibiotic pressure, or under constitutive induction
Mutations that result in drug resistance are not caused by drug exposure per
se, rather they are random events that con er a survival advantage when drug is
present

SUMMARY QUIZ

QUESTION 34-1 A 43-year-old man is neutropenic and has developed a ever A er


obtaining the appropriate cultures, you decide to start him on a broad-spectrum
antibacterial and anti ungal therapy Your decision ts which o the ollowing goals o
antimicrobial therapy?
a Prophylaxis
b De nitive
c Empirical
d Preemptive
e Post-treatment suppression

518
General Principles of Antimicrobial Therapy CHAPTER 3 4

QUESTION 34-2 ypically the time when the antibiotic concentration is greater than
the MIC is less with once-daily dosing than i the antibiotic were administered in
3 equally divided doses However, aminoglycoside antibiotics are commonly administered
once daily T e e cacy o once-daily dosing o aminoglycosides is due to
a decreased toxicity
b increased renal excretion at higher doses
c less bacterial resistance to the antibiotic
d postantibiotic e ect
e the rate o increase in the initial plasma concentration

QUESTION 34-3 Bacterial resistance to -lactam antibiotics (eg, penicillin) is usually


due to the bacterial production o -lactamase T is results in
a destruction o the antibiotic
b reduced a nity to a bacterial protein
c enhanced ef ux o the antibiotic rom the bacteria
d decreased entry o the antibiotic into the bacteria
e development o an alternative mechanism o bacterial survival in the presence
o antibiotic

QUESTION 34-4 A 35-year-old patient with HIV in ection is being started on 3 drugs
in combination T e primary reason he is started on 3 drugs rather than 1 is
a the drugs only come in combinations o 3
b the potential or reducing toxicity
c preventing resistance to monotherapy
d enhanced antiviral e ect o each drug
e a decrease in urinary excretion o each drug

SUMMARY QUIZ ANSWER KEY

QUESTION 34-1 Answer is c. T is is termed empiric therapy (see Figure 34-4)


Neutropenic patients with ever have a high risk o mortality, and when ebrile, they are
presumed to have either a bacterial or ungal in ection Once the pathogen is identi ed
and susceptibility determined the broad-spectrum therapy can be switched to a speci c
antimicrobial agent

QUESTION 34-2 Answer is d. T e peak antibiotic plasma concentration is what mat-


ters or some antimicrobial agents, such as aminoglycosides, and the persistence above
MIC has less relevance T ese drugs can be administered less requently due to their
long duration o postantibiotic e ect T at is, their antibacterial e ect continues long
a er antibiotic concentrations decline below the MIC

QUESTION 34-3 Answer is a. -lactamase is an enzyme that destroys antibiotics that


contain a -lactam in their chemical structure (see Chapter 39)

QUESTION 34-4 Answer is c. It is standard o care to start patients with HIV in ection
on a regimen o 3 drugs primarily to prevent the development o resistant virus (see
Chapter 44)

519
CHAPTER

35 Chemotherapyof Malaria
T is chapter will be most use ul a er having a basic understanding o the material in
DRUGS INCLUDED IN Chapter 49, Chemotherapy o Malaria in Goodman & Gilmans T e Pharmacological
THIS CHAPTER Basis of T erapeutics, 12th Edition. In addition to the material presented here, Chapter 49 in
Artemisinin and Artemisinincombination the 12th Edition contains:
therapies ables 49-2, 49-3, and 49-4 which provide in ormation about appropriate regimens,
Atovaquone/Proguanil (MALARONE) including adult and pediatric dosages, or the prevention, treatment, and presumptive
Chloroquine (ARALEN) sel -treatment o malaria
Doxycyclinesee Chapter 41 Figure 49-4 which is an algorithm approach to the treatment o malaria
Hydroxychloroquine (PLAQUENIL) LEARNING OBJECTIVES
Mefoquine (LARIUM)
Know the stages o the malaria parasite in the human body.
Primaquine
Classi y antimalarial drugs into those that are e ective against only the blood
Pyrimethamine (DARAPRIM) stages o the parasite, those that are e ective against both the blood and liver
Quinine/Quinidine stages, and those that are e ective against only the liver stages o the parasite.
Sul adoxine/Pyrimethamine (FANSIDAR) Understand the use o antimalarial drugs in clinical context, particularly with
regard to their mechanism o action, therapeutic uses, and toxicities.
Describe the principles and guidelines or the chemoprophylaxis and treatment
o malaria.

CRITERIA FOR THE CRITERIA FOR DIAGNOSIS OF SEVERE MALARIA


DIAGNOSIS OF MALARIA
Prostration Jaundice
Signs and symptoms consistent with
malaria, including ever, chills, malaise, Impaired consciousness/coma Severe anemia
myalgia, headache, severe malaria crite- Respiratory distress Acute renal ailure
ria (see Table CRITERIAFORDIAGNOSISOF
SEVEREMALARIA), plus Multiple convulsions Disseminated intravascular coagulation
Demonstration o the malaria parasite: Circulatory shock Acidosis
Light microscope examination o
Pulmonary edema Hemoglobinuria
stained thin or thicksmear
Rapid diagnostictest Acute respiratory distress syndrome Parasitemia >5%
Molecular or biophysical-based testing Abnormal bleeding

ANTIMALARIAL DRUG MECHANISMS OF ACTION AND RESISTANCE


DRUG MECHANISM OF ACTION MECHANISM OF RESISTANCE
Artemisinin and Production o toxic Not known at this time
Derivatives heme-adducts

Atovaquone Inhibits mitochondrial Nucleotide polymorphisms in the


electron transport in the cytochrome b gene
cytochrome bc1 complex

Proguanil Inhibits dihydro olate Mutations in the amino acid


reductase-thymidylate sequence near the dihydro olate-
synthase reductase binding site
(Continued)

520
Chemotherapy of Malaria CHAPTER 3 5

DRUG MECHANISM OF ACTION MECHANISM OF RESISTANCE CURRENT ARTEMISININ


Pyrimethamine Inhibits Plasmodium Mutations in dihydro olate-reductase COMBINATION THERAPY
dihydro olate-reductase binding site (ACT) REGIMENS
Sul adoxine Inhibition o Plasmodium Mutations in dihydropteroate Artemether-lume antrine*
dihydropteroate synthase synthase gene Artesunate-mefoquine
Chloroquine/ Production o toxic heme Production o a chloroquine e ux Artesunate-amodiaquine
Hydroxychloroquine adducts transporter
Artesunate-sul oxadine-pyrimethamine
Quinine/Quinidine Production o toxic heme Production o an e ux transporter; Dihydroartemisinin-piperaquine
adducts ampli cation o pfmdr 1 gene
*Lumefantrine shares structural similarities
Mef oquine Production o toxic heme Ampli cation o pfmdr 1 gene that with mefloquine and is onlyformulated with
adducts accumulates drug in digestive artemether (COARTEM)
There is also a cytosolic vacuole away rom cytosolic site o
mode o action action

Primaquine Production o reactive Not known at this time


oxygen species

CASE 35 1
A 43-year-old woman develops the signs and symptoms o uncomplicated malaria
caused by Plasmodium falciparum. Her symptoms include ever, chills, malaise, and
headache. A microscopic examination o a blood smear con rms the diagnosis. She is
treated with the combination therapy artemether-lume antrine.
a. Why is she treated with the combination product rather than either artemether
or lume antrine alone?
Artemether-lume antrine is one o several artemisinin-based combination
therapies (AC s). T e artemisinins are very potent and ast-acting antimalarials.
However, the emergence o P. falciparum isolates with an increased tolerance
to artemisinins has recently been ound. T e primary goal o these combina-
tion products is to increase treatment e cacy and reduce selection pressure
or the emergence o drug resistance. T e short t o the artemisinins results in
substantial treatment ailure rates when artemisinins are used as monotherapy.
Combining an artemisinin with a longer-lasting partner drug assures sustained
antimalarial activity.
b. What are the pharmacokinetic properties o lume antrine that make it a good
choice in combination with artemether?
Artemisinins rapidly achieve peak serum concentrations; intramuscular artemether
peaks at 2 to 6 hours, due to a depot e ect at the injection site. Lume antrine has a
large apparent volume o distribution and a terminal elimination t o 4 to 5 days.
Administration with a high- at meal is recommended because it signi cantly
increases lume antrine absorption. T e advantages and disadvantages o the other
combination drugs are discussed in detail in Chapter 49 in Goodman and Gilmans
T e Pharmacological Basis of T erapeutics, 12th Edition.

CASE 35 2
A 23-year-old emale college student is planning a trip to an area o A rica that is endemic
or chloroquine-resistant P. falciparum. She has been advised to take atovaquone-
proguanil or chemoprophylaxis.
a. When should the drug be started?
Chemoprophylaxis or malaria should begin be ore exposure and pre erably be ore
the traveler leaves home. T is is to establish therapeutic blood concentrations
and to detect early signs or symptoms o intolerance so that the regimen can be
(Continued)
521
SECTION VII Chemotherapy of Microbial Diseases

modi ed be ore departing. T e regimen should be taken daily while in the malari-
ous area and or 7 days a er leaving the area. (see able 35-1 below and able 49-2
in Goodman and Gilmans T e Pharmacological Basis of T erapeutics, 12th Edition
or detailed chemoprophylactic regimens.)
b. What side e ects might this patient expect?
T is combination is generally considered sa e with no serious side e ects. T e most
common side e ects are nausea, vomiting, diarrhea, and headache.
c. Why is the combination atovaquone-proguanil used rather than either drug alone?
Resistance to atovaquone when used alone occurs easily. T e addition o proguanil
markedly reduces the requency o atovaquone resistance. However, once atovaquone
resistance is present the e ect o adding the proguanil diminishes.
d. How do the mechanisms o atovaquone and proguanil work synergistically?
Atovaquone acts selectively on the mitochondrial cytochrome bc1 complex to
inhibit electron transport and collapse the mitochondrial membrane potential.
T e antimalarial activity o proguanil is ascribed to cycloguanil (structurally
similar to pyrimethamine), a selective inhibitor o the bi unctional plasmodial
dihydro olate reductase-thymidylate synthetase that is crucial or parasite purine
and pyrimidine synthesis. Inhibition o this enzyme causes inhibition o DNA
synthesis and depletion o olate co actors. T e synergy between proguanil and
atovaquone results rom the ability o proguanil to enhance the mitochondrial
toxicity o atovaquone.

TABLE 35-1 Regimens for the Prevention of Malaria in Nonimmune Individuals (For complete details see Table 49-2 in
Goodman &Gilmans The Pharmacological Basis of Therapeutics, 12th Edition, Chapter 49)
DRUG USAGE COMMENTS
Atovaquone/Proguanil Prophylaxis in all areas Begin 1-2 days be ore travel to malarious areas; take daily at the
same time each day while in the malarious area and or 7 days a ter
leaving such area

Chloroquine Phosphate Prophylaxis only in areas with Begin 1-2 weeks be ore travel to malarious areas; take weekly
chloroquine-sensitive malaria on the same day o the week while in the malarious area and or
4 weeks a ter leaving such area

Doxycycline Prophylaxis in all areas Begin 1-2 days be ore travel to malarious areas; take daily at the
same time each day while in the malarious area and or 4 weeks
a ter leaving such area

Hydroxychloroquine Sul ate Alternative to chloroquine or prophylaxis Begin 1-2 weeks be ore travel to malarious areas; take weekly
only in areas with chloroquine-sensitive on the same day o the week while in the malarious area and or
malaria 4 weeks a ter leaving such area

Mef oquine Prophylaxis in areas with mef oquine- Begin 1-2 weeks be ore travel to malarious areas; take weekly
sensitive malaria on the same day o the week while in the malarious area and or
4 weeks a ter leaving such area

Primaquine Prophylaxis or short duration travel to Begin 1-2 days be ore travel to malarious areas; take daily at the
areas principally with Plasmodium vivax same time each day while in the malarious area and or 7 days a ter
leaving such area

For presumptive antirelapse therapy Indicated or persons with prolonged exposure to P. vivax and
(terminal prophylaxis) to decrease the risk P. ovale or both
o relapse (P. vivax, Plasmodium ovale)

Source: From the United States Centers or Disease Control and Prevention, Health In ormation or International Travel 2010 (Yellow Book).
http://wwwnc.cdc.gov/travel/content/yellowbook/home-2010.aspx. Accessed January 12, 2010.

522
Chemotherapy of Malaria CHAPTER 3 5

CASE 35 3
A 28-year-old emale university student in urkey contracts uncomplicated malaria
with symptoms o ever, chills, malaise, myalgia, and headache. T e director o the
clinic in which you work wants to treat her with chloroquine.
a. Is chloroquine an appropriate drug to treat uncomplicated malaria in the
country o Turkey?
Chloroquine is highly e ective against the erythrocytic orms o Plasmodium
ovale, Plasmodium vivax, Plasmodium malariae, Plasmodium knowlesi, and chlo-
roquine-sensitive strains o Plasmodium falciparum. urkey is a country in which
(Continued)
Ita ly Arme nia Uzbe kis ta n
Ge orgia Aze rba ija n Turkme nis ta n
Fra nce Mongolia
Gre e ce
S pa in Kyrgyzs ta n North
Turkey Afgha nis ta n Kore a
Portuga l
Tunis ia Cyprus Syria Ne pa l China
Morocco Is ra e l Ira q Ira n Ja pa n
Bhuta n
S outh
Alge ria Libya J orda n Kuwa it Kore a
We s te rn Egypt
Sa ha ra Qa ta r Mya nma r
Sa udi India Ta iwa n
Ma urita nia Ara bia UAE Ma ca u
Ma li Nige r Pa kis ta n
S e ne ga l Oma n La os
Ba ngla de s h P hilippine s
Cha d S uda n
The Ga mbia Eritre a Ye me n Tha ila nd
Guine a - Nige ria
Bis s a u Be nin Djibouti Vie tna m Pa cific Is la nds
CAR Ethiopia
Guine a Ma lays ia (Pa la u)
Togo S oma lia
S ie rra Le one Gha na DROC S ri La nka
Libe ria Ke nya Pa pua Ne w Guine a
Ca me roon Rwa nda
Burkina Fa s o Equa toria l Uga nda S inga pore Indone s ia
Burundi S olomon Is la nds
^ Guine a Ta nza nia
Cote d'Ivoire
Ga bon Ma lawi
Angola Va nua tu
Congo Moza mbique
Za mbia Fiji

Na mibia Ma da ga s ca r
Aus tra lia Ne w Ca le donia
Botswa na Zimba bwe
S outh Swa zila nd
Africa Le s otho

New Ze a la nd

Dominica n
Mexico Re public

Be lize
Ha iti
Hondura s
Gua te ma la
Guya na
El Sa lva dor S urina me
Ve ne zue la
Nica ra gua Fre nch
Pa na ma Colombia Guia na
Cos ta Rica
Ecua dor

Ga la pa gos
Is la nds
Pe ru Bra zil

Bolivia

Pa ra guay

Chile
KEY
Ma la ria -Ende mic Are a s
Chloroquine -re s is ta nt Arge ntina Uruguay
Chloroquine -s e ns itive
None

Fa lkla nd
Is la nds

FIGURE 35-1 Malaria-endemic countries in the Americas (bottom) and in A rica, the Middle East, Asia, and the South Paci c (top), 2007. CAR,
Central A rican Republic; DCOR, Democratic Republic o the Congo; UAE, United Arab Emirates. (Reproduced with permission from Anthony S. Fauci,
Eugene Braunwald, Dennis LKasper, et al, Eds. Harrisons Principles of Internal Medicine, 17 ed. McGraw-Hill, Inc., New York, 2008. Figure 203-2, p. 1282.)
523
SECTION VII Chemotherapy of Microbial Diseases

P. falciparum is still chloroquine-sensitive (see Figure 35-1). Although inexpensive


and sa e, chloroquine has, because o the spread o chloroquine-resistant strains o
P. falciparum across most malaria-endemic regions o the world (see Figure 35-1),
been largely replaced by artemisinin-based combination therapies.
b. Describe the mechanism o chloroquines antimalarial activity and the mechanism
that P. falciparum use to become resistance to its use.
Malarial parasites digest hemoglobin in the host erythrocytes and sequester the
heme as an insoluble, chemically inert pigment called hemozoin. It is thought that
chloroquine inter eres with this heme detoxi cation.
A parasite-encoded e ux mechanism may explain the reduced concentrations o
chloroquine in the digestive vacuoles o chloroquine-resistant parasites. T e poly-
morphic gene (pfcrt, or P. falciparum chloroquine resistance transporter) encodes
a putative chloroquine transporter that resides in the membrane o the parasite
digestive vacuole at the site o hemoglobin degradation and chloroquine action.
c. What chloroquine toxicities should this student be cautioned about?
Chloroquine is not recommended or treating patients with epilepsy or myasthenia
gravis, and it can cause hemolytic anemia in patients with G6PD de ciency. It
should be used cautiously in patients with advanced liver disease. Chloroquine
inhibits CYP2D6 and interacts with drugs metabolized by this enzyme.

CASE 35 4
A 34-year-old man has developed uncomplicated malaria in an area that is known to
have chloroquine-resistant P. falciparum (see Figure 35-1) and is being treated with
quinine plus doxycycline.
a. What are the common toxicities o quinine that should be watched or in this
patient?
Quinine has a triad o dose-related toxicities: cinchonism, hypoglycemia, and hypo-
tension. Cinchonism consists o tinnitus, high-tone dea ness, visual disturbances,
headache, dysphoria, nausea, vomiting, and postural hypotension. T ese e ects
disappear soon a er the drug is withdrawn. Hypoglycemia is also common and can
be li e-threatening i not treated promptly with intravenous glucose.
b. What are the more severe and rare toxicities and precautions o quinine use?
Quinine rarely causes cardiac complications (ie, arrhythmias) unless therapeutic
plasma concentrations are exceeded. Severe hemolysis can result rom hypersensitivity
to quinine and therapy should be discontinued immediately i evidence o hemolysis
appears. Quinine should be avoided in patients with tinnitus or optic neuritis and in
patients with cardiac arrhythmias, the same precautions as or quinidine (see Chapter
29 in Goodman and Gilmans T e Pharmacological Basis of T erapeutics, 12th Edition).
c. What are other therapeutic options?
Other possible choices or treatment o P. falciparum malaria in a chloroquine-resistant
area are artemether-lume antrine, atovaquone-proguanil, clindamycin (used in
combination with quinine or quinidine), or mef oquine (except in Southeast Asia).

CASE 35 5
A 34-year-old-man in Cambodia has been treated with me oquine or severe malaria.
He returns to the clinic because his symptoms have not subsided.
a. What is a major limitation to the use o mef oquine in Southeast Asia?
Mef oquine is reserved or the prevention and treatment o drug-resistant P. falciparum
and P. vivax, but is no longer considered rst-line treatment. In areas where malaria
is due to multiple drug-resistant strains o P. falciparum, such as Southeast Asia,
mef oquine is used in combination with an artemisinin compound.
(Continued)
524
Chemotherapy of Malaria CHAPTER 3 5

b. What is the mechanism o action o mef oquine?


Mef oquine binds to intraerythrocytic hemozoin similar to chloroquine. It appears
that pfmdr1 gene ampli cation is a major determinant o mef oquine treatment
ailure and in vitro mef oquine resistance.
c. Describe the major limitations to the use o mef oquine.
Mef oquine is not recommended or use during pregnancy and pregnancy should
be avoided or 3 months a er stopping mef oquine because o its prolonged
elimination. Mef oquine is associated with neuropsychiatric signs and symptoms,
including dizziness, con usion, and headache in 10% (or greater) o patients. Vivid
dreams are common. Severe neurotoxicity such as psychosis or seizures is rare.

CASE 35 6
A 46-year-old man with a primary hepatic stage o P. falciparum is treated with
primaquine.
a. Why was primaquine chosen and what are its major therapeutic uses?
Figure 35-2 shows the li e cycle o malaria parasites and able 35-2 shows the
malarial parasite developmental stages targeted by antimalarial drugs. Primaquine
acts against primary and latent hepatic stages o Plasmodium spp. and prevents
relapses in P. vivax and P. ovale in ections. Primaquine is used primarily or the
(Continued)

Bite from
infe cte d
mos quito

Infe cte d
mos quito

Uninfe cte d
Prima ry a ttack Re la ps e (due to P. viva x or P. ova le ) mos quito

Erythrocytic
s ta ge of infe ction

ERYTHROCYTIC
CYCLE

S chizonts Me rozoite s

FIGURE 35-2 Li e cycle o malaria parasites.


525
SECTION VII Chemotherapy of Microbial Diseases

TABLE 35-2 Malarial Parasite Developmental Stages Targeted by Antimalarial Drugs


EFFECT OF DRUG ON PARASITE VIABILITY
LIVER STAGES BLOOD STAGES
GROUP DRUGS SPOROZOITE PRIMARY HYPNOZOITE ASEXUAL GAMETOCYTE
1 Artemisinins + +

Chloroquine + +/

Mef oquine +

Quinine/Quinidine + +/

Pyrimethamine +

Sul adoxine +

Tetracycline +

2 Atovaquone/ + + +/
Proguanil

3 Primaquine + + +

, no activity; +/, low to moderate activity; +, important activity.

terminal chemoprophylaxis (shortly be ore or immediately a er a patient leaves


an endemic area), and or radical cure o P. vivax and P. ovale in ections because
o its high activity against the latent tissue orms o these Plasmodium species.
Primaquine is o en given together with a blood schizonticide, usually chloroquine,
to eradicate erythrocytic stages o these Plasmodia and to reduce the possibility o
emerging drug resistance.
b. What toxicity is o major concern with primaquine and what precautions are
necessary with primaquine?
T erapeutic or higher doses o primaquine may cause acute hemolysis and hemo-
lytic anemia in patients with G6PD de ciency. About 11% o A rican-Americans
have the A variant o G6PD, and are there ore susceptible to hemolysis caused by
primaquine. G6PD de ciency should be ruled out prior to the administration o
primaquine. Primaquine should not be given to pregnant women.

CASE 35 7
T e student in Case 35-3 asks your general advice regarding malaria therapy.
a. What is your advice to her (beside what you told her in Case 35-3) regarding
chemoprophylaxis or malaria?
Regimens or malaria chemoprophylaxis include primarily 3 drugs: atovaquone-
proguanil and doxycycline that can both be used in all areas, and mef oquine
that can be used in areas with mef oquine-sensitive malaria. Other options
include chloroquine or hydroxychloroquine in the ew areas with chloroquine-
sensitive malaria, and primaquine or short duration travel to areas with
principally P. vivax.
b. She also asks or advice about treatment should she contract malaria.
For uncomplicated malaria, chloroquine is the drug o choice or P. ovale, P. knowlesi,
P. malariae, and chloroquine-sensitive strains o P. vivax and P. falciparum. Primaquine
(Continued)

526
Chemotherapy of Malaria CHAPTER 3 5

must not be given to patients with G6PD de ciency. For uncomplicated malaria caused
by chloroquine-resistant P. falciparum, 4 treatment options are available:
Artemether-lume antrine
Atovaquone-proguanil
Oral quinine with other e ective but slower-acting blood schizonticides such as
doxycycline or clindamycin
Mef oquine
For the treatment o severe malaria, no matter the region where the in ection was
acquired, the recommended treatments are based on intravenous artesunate or
quinidine plus a second drug.
c. She is also concerned about getting pregnant during her stay in Turkey and asks
i there is a recommended treatment or malaria in pregnant patients?
Chemoprophylaxis during pregnancy is complex, and women should evaluate with
expert medical sta the bene ts and risks o di erent strategies. Severe malaria
during pregnancy should be treated with intravenous antimalarial treatment
according to the general guidelines, taking into account the drugs that should be
avoided during pregnancy such as primaquine, mef oquine, and tetracycline.

KEY CONCEPTS
P. falciparum has become progressively more resistant to antimalarial drugs.
E ective chemoprophylaxis regimens include atovaquone-proguanil, doxycycline,
or me oquine.
Chemoprophylaxis should be started be ore exposure, ideally be ore the traveler
leaves home, and continued a er returning rom a malarious region.
First-line therapy or uncomplicated malaria is artemether-lume antrine.
First-line therapy or severe malaria is IV artesunate ollowed by atovaquone-
proguanil, doxycycline, or me oquine.
Primaquine and me oquine should not be used in pregnant women and prima-
quine should not be used in patients with G6PD de ciency.

SUMMARY QUIZ
QUESTION 35-1 A physician is planning to treat a 27-year-old woman with primaquine
or a documented P. vivax in ection. Prior to beginning treatment the physician must
test the patient or
a. elevated serum amylase.
b. glucose-6-phosphate dehydrogenase de ciency.
c. iron de ciency.
d. elevated serum calcium.
e. vitamin B12 de ciency.

QUESTION 35-2 A 35-year-old man will be traveling to an area that is endemic or


malaria. His physician starts him on an oral artemisinin derivative prior to his trip.
T is drug is likely to be ine ective as chemoprophylactic therapy because
a. it is e ective against only the blood stages o P. falciparum.
b. its side e ects are so severe that compliance is unlikely.
c. it is likely to have cross-resistance with chloroquine.
d. its active metabolite dihydroartemisinin has a short plasma t.
e. its e ectiveness a er oral administration is poor.

527
SECTION VII Chemotherapy of Microbial Diseases

QUESTION 35-3 A 42-year-old man is being treated with atovaquone-proguanil or


uncomplicated P. falciparum malaria. T is combination product is pre erred to the use
o atovaquone alone because
a. there is a reduced risk o resistance.
b. proguanil improves the absorption o atovaquone.
c. proguanil decreases the clearance o atovaquone.
d. proguanil decreases the side e ects o atovaquone.
e. atovaquone improves the absorption o proguanil.

QUESTION 35-4 Chloroquine-resistance is now common in most parts o the world.


T e cause o chloroquine-resistance is
a. enhanced metabolism by the P. falciparum parasite.
b. poor absorption in the human GI tract.
c. induced metabolism by the human liver a er repeat dosing.
d. sequestration o the drug in at stores.
e. enhanced ef ux o drug rom the digestive vacuoles o drug-resistant parasites.

QUESTION 35-5 A 19-year-old woman will be traveling to an area that is endemic or


me oquine-sensitive P. falciparum. She is to be treated with me oquine chemoprophy-
lactically. She should be warned about which o the ollowing common side e ect?
a. Nosebleeds
b. innitus
c. Vivid dreams
d. Blurred vision
e. A metallic taste

SUMMARY QUIZ ANSWER KEY


QUESTION 35-1 Answer is b. Primaquine may cause acute hemolysis and hemolytic
anemia in humans with G6PD de ciency. More than 400 genetic variants o G6PD de -
ciency produce a variable response to oxidative stress. About 11% o A rican-Americans
have a variant o G6PD de ciency; primaquine-induced hemolysis can be even more
severe in white ethnic groups, including Sardinians, Sephardic Jews, Greeks, and Iranians
as these populations have a G6PD variant in which 2 amino acid substitutions impair
both enzyme stability and activity.
QUESTION 35-2 Answer is d. Artemisinins have a relatively short plasma t that makes
them unsuitable or prophylactic therapy.
QUESTION 35-3 Answer is a. Resistance to atovaquone alone in P. falciparum develops
easily and it is con erred by a nucleotide polymorphisms in the cytochrome b gene
located in the parasites mitochondrial genome. T ese polymorphyisms appear to a ect
atovaquone binding. T e addition o proguanil markedly reduces the requency o
appearance o atovaquone-resistance.
QUESTION 35-4 Answer is e. Chloroquine acts by inter ering with heme detoxi cation
in parasite digestive vacuoles. A parasite-encoded ef ux mechanism reduces the concen-
tration o chloroquine in the digestive vacuoles o chloroquine-resistant parasites.
QUESTION 35-5 Answer is c. Vivid dreams are common with me oquine. Other tox-
icities, including disturbed sleep, dysphoria, headache, GI disturbances, and dizziness,
can all occur at chemoprophylactic doses. Severe CNS toxicity with me oquine treat-
ment is not common but can be as high as 0.5% and include seizures, con usion, acute
psychosis, and disabling vertigo.

528
Chemotherapy of Malaria CHAPTER 3 5

SUMMARY: ANTIMALARIAL DRUGS


TOXICITIES
CLASS AND UNIQUE; CLINICALLY
SUBCLASSES NAMES CLINICAL USES RESISTANCE COMMON IMPORTANT
Artemisinins Dehydroartemisinin Severe malaria No signi cant cross- Allergic reaction in Reversible decreases
Artemether Uncomplicated resistance to other 1:3000 patients in reticulocyte and
Artesunate malaria drugs neutrophil count
Artemether/ and increases in
Lume antrine transaminases

Atovaquone MEPRONE Rarely used by itsel Resistance i used Nausea, vomiting, diarrhea, May compete with
alone and headache other drugs or plasma
protein binding
Ri ampin and tetracycline
reduce plasma
concentrations

Proguanil Rarely used by itsel Resistance develops Nausea and diarrhea Polymorphism in
i used alone CYP2C amily may alter
metabolism

Atovaquone- MALARONE Prophylaxis and Resistance to Nausea, vomiting, Not indicated or use in
Proguanil treatment o drug- the combination abdominal pain, mild pregnant women due
resistant strains o is uncommon reversible elevations in to limited data
P. falciparum or unless strain is liver transaminases
P. vivax initially resistant to
atovaquone

Pyrimethamine Pyrimethamine- No longer Resistance is Well tolerated in Megaloblastic anemia


sul adoxine recommended extensive therapeutic doses resembling olate
or the treatment de ciency
o uncomplicated Skin rashes rom the
malaria sul a moiety

Chloroquine ARALEN Used to treat P. ovale, Resistance o Well tolerated in Not to be used in
P. vivax, P. malariae, P. falciparum is therapeutic doses patients with epilepsy or
P. knowlesi, and extensive myasthenia gravis
chloroquine-sensitive Hemolysis in patients
P. falciparum with G6PD de ciency

Quinine/Quinidine Not used or Resistance is Cinchonism, Should not be used in


chemoprophylaxis increasing Hypoglycemia patients with tinnitus or
Treatment o Hypotension optic neuritis
P. falciparum rom Use with caution in
chloroquine-resistant patients with cardiac
areas arrhythmias
IVquinidine used to Severe hemolysis
treat severe malaria may occur

Mef oquine LARIUM Used or Used in Vivid dreams, nausea, Contraindicated in


treatment and combination with vomiting, and vertigo patients with seizures or
chemoprophylaxis an artemisinin other neuropsychiatric
compound in area disorders
with high drug Not recommended in
resistance patients with cardiac
disorders

Primaquine Terminal Partial resistance to Well tolerated in Contraindicated in


chemoprophylaxis o P. vivax is reported therapeutic doses patients with G6PD
P. vivax and P. ovale Nausea and vomiting de ciency and pregnant
women

529
CHAPTER
ChemotherapyofProtozoalInfections:
36 Amebiasis, Giardiasis, Trichomoniasis,
Trypanosomiasis, Leishmaniasis, and
Other Protozoal Infections
T is chapter will be most use ul a er having a basic understanding o the material in
DRUGS INCLUDED IN
Chapter 50, Chemotherapy o Protozoal In ections in Goodman & Gilmans T e
THIS CHAPTER Pharmacological Basis of T erapeutics, 12th Edition. In addition to the material
Amphotericin B(see Chapter 43) presented here, the 12th Edition contains:
Efornithine (ORNIDYL) A detailed discussion o protozoal in ections, including amebiasis, giardiasis, toxoplas-
Fumagillin (FUMIDIL) mosis, trichomoniasis, cryptosporidiosis, trypanosomiasis, leishmaniasis, babesiosis,
balantidiasis, Isospora belli, and microsporidia
8-Hydroxyquinolines (iodoquinol; YODOXIN)
Melarsoprol LEARNING OBJECTIVES
Metronidazole (FLAGYL) Understand the most common protozoal in ections, the clinical symptoms,
Milte osine (IMPAVIDO) and the mainstays o therapy.
Nitazoxanide (ALINA) Describe the mechanisms o action o antiprotozoal drugs.
Ni urtimox(LAMPIT) Understand the treatment o giardiasis, amebiasis, and cryptosporidiosis.
Benznidazole (ROCHAGAN) Identi y the therapeutic uses o antiprotozoal drugs.
Paromomycin Describe the toxicities and precautions to the use o antiprotozoal drugs.
Pentamidine
Sodiumstibogluconate
MECHANISMS OF ACTION AND RESISTANCE OF ANTI-PROTOZOAL DRUGS
Suramin
DRUG MECHANISM OF ACTION MECHANISM OF RESISTANCE
Amphotericin Binds ergosterol in protozoal membranes No signi cant protozoal
to orm pores and increase membrane resistance
PROTOZOAL INFECTIONS permeability
DISCUSSED IN E ornithine Inhibition o ornithine decarboxylase Mutations in ornithine
THIS CHAPTER decarboxylase
Amebiasis Fumagillin Inhibition o methionine-aminopeptidase-2
Giardiasis
Melarsoprol Inhibition o many enzymes Mutations in drug transporters
Cryptosporidiosis
Metronidazole Forms highly reactive nitro radical Impaired O2 scavenging
anion that targets DNA and other vital capabilities leading to higher local
biomolecules O2 concentrations and decreased
Inter eres with the pyruvate: erredoxin activation o metronidazole
oxidoreductase (PFOR) enzyme-dependent Also mutations in nitroreductase
electron-trans er reaction, which is essential
to anaerobic metabolism in protozoan
and bacterial species

Milte osine Not completely known Point mutations in transporter


which leads to decreased drug
uptake

Nitazoxanide Nitazoxanide (like metronidazole) inter eres Resistance to nitazoxanide has


with the pyruvate: erredoxin oxidoreductase been induced in Giardia in vitro,
(PFOR) enzyme-dependent electron-trans er but resistant clinical isolates have
reaction, which is essential to anaerobic not been reported
metabolism in protozoan and bacterial
species
(Continued)

530
Chemotherapy of Protozoal Infections CHAPTER 3 6

DRUG MECHANISM OF ACTION MECHANISM OF RESISTANCE


Ni urtimox Ni urtimox is activated by an NADH- Reduced nitroreductase
dependent mitochondrial nitroreductase, expression
leading to the intracellular generation o
nitro radical anions that are thought to
account or trypanocidal e ects; these
radicals orm covalent attachments
to macromolecules leading to cellular
damage that kills the parasites

Benznidazole Same as ni urtimox Same as ni urtimox

Paromomycin Paromomycin binds to the 30S ribosomal


subunit similar to neomycin and
kanamycin (see Chapter 40) and shares
their spectrum o antibacterial activity

Pentamidine In Trypanosoma brucei pentamidine is Although ailure to concentrate


concentrated via an energy-dependent pentamidine is the usual cause
high-a nity uptake system in cells where o resistance, other mechanisms
it is thought to react with a variety o also may be involved
negatively charged intracellular targets

Sodium Pentavalent antimonials act as prodrugs Resistance appears to result rom


Stibogluconate being reduced to the Sb 3+ species; Sb 3+ overexpression o glutathione
induces a rapid ef ux o trypanothione and and polyamine biosynthetic
glutathione rom the cells, and also inhibits enzymes, leading to increased
trypanothione reductase, thereby causing a trypanothione levels, which
signi cant loss o thiol reduction potential conjugate to the drug
in the treated cells

Suramin Suramin is a slow-acting trypanocide No signi cant eld resistance


with an unknown mechanism o
action; parasites take up the drug by
receptor-mediated endocytosis o
the protein-bound drug; once in the
cell, suramin reacts reversibly with a
variety o biomolecules inhibiting many
trypanosomal and mammalian enzymes

CASE 36-1
A 35-year-old woman comes into your o ce complaining o diarrhea and abdominal
pain or the past 3 days. She has recently returned rom a white-water ra ing trip. On
the trip she ell out o the boat and although she had a li e preserver on, she believes
that she swallowed considerable amounts o river water.
a. You suspect giardiasis and want to begin treatment a er obtaining appropriate
specimens. What is giardiasis?
Giardiasis is caused by the protozoan Giardia intestinalis, is prevalent world-
wide, and is the most common intestinal protozoal in ection in the United
States. Giardia is a zoonosis and cysts shed in the eces o animals and humans
can contaminate recreational and drinking water supplies. In ection with
Giardia results in an asymptomatic carrier state, acute sel -limited diarrhea,
or chronic diarrhea.
b. How is the diagnosis o giardiasis made?
T e diagnosis o giardiasis is made by identi cation o cysts or trophozoites in ecal
specimens or o trophozoites in duodenal contents.
(Continued)

531
SECTION VII Chemotherapy of Microbial Diseases

c. A er instructing your patient in how to collect ecal specimens, what is the most
COMMON ROUTES
appropriate therapy to limit the acute diarrhea, to prevent the development o
OF TOXOPLASMOSIS
chronic diarrhea, and to prevent the development o a carrier state?
INFECTION IN HUMANS
Chemotherapy with a 5-day course o metronidazole or a single dose o tinidazole
Ingestion o undercooked meat is usually success ul. Paromomycin has been used to treat pregnant women to avoid
containing cysts any possible mutagenic e ects o the other drugs.
Ingestion o vegetables contami-
d. What are the potential side e ects o metronidazole therapy which this patient
nated with soil containing cysts
should be in ormed about?
Direct oral contact with eces o
T e most common side e ects o metronidazole are headache, nausea, dry mouth,
cats shedding cysts
vomiting, and diarrhea, and are rarely severe enough to discontinue therapy. More
ransplacental etal in ection severe neuropathies may include dizziness, vertigo, and rarely encephalopathy,
with tachyzoites rom acutely seizures, incoordination, ataxia, and numbness or paresthesias o the extremities;
in ected mothers these neuropathies warrant discontinuation o metronidazole.
Metronidazole has a well-documented disul ram-like e ect (see Chapter 9) and
patients should be warned not to drink alcoholic beverages during or within 3 days
o therapy.

CASE 36-2
A 10-year-old boy is brought into the emergency department with a 2-day history o
severe diarrhea. T e child is otherwise well with no chronic diseases but appears dehy-
drated and lethargic. It is summer and the child has been swimming in the public pool
every day or the past week.
a. What organism is most likely causing this childs diarrhea?
Cryptosporidia protozoa cause diarrhea in a number o animal species including
humans. Cryptosporidium parvum and Cryptosporidium hominis account or almost
all in ections in humans. Oocysts in eces may be spread by direct human-to-human
contact or by contaminated water supplies.
b. Are there special populations that are vulnerable to cryptosporidiosis?
T ose at risk include travelers, children in day-care acilities, male homosexuals,
animal handlers, and health care personnel. Immunocompromised individuals are
especially vulnerable.
c. Are there specif c treatments or cryptosporidiosis?
In most individuals, the in ection is sel -limited. However, immunocompromised
individuals may require hospitalization and supportive care. T e most e ective
therapy or cryptosporidiosis in AIDS patients is the restoration o their immune
unction. Nitazoxanide is currently the only drug approved or the treatment o
cryptosporidiosis in the United States.

CASE 36-3
A 56-year-old woman is diagnosed with amebic colitis. She is started on metronidazole
but an in ectious disease consultant recommends adding paromomycin to her thera-
peutic regimen.
a. Why add the paromomycin when Entamoeba histolytica is usually susceptible to
metronidazole?
T e cornerstone o therapy or amebiasis is metronidazole which is the drug o choice
or the treatment o amebic colitis, amebic liver abscess, and any other extraintestinal
orm o amebiasis. Because metronidazole is so well absorbed in the gut, concentra-
tions may not be therapeutic in the colonic lumen such that the drug is less e ective
(Continued)

532
Chemotherapy of Protozoal Infections CHAPTER 3 6

against cysts. Patients with amebic colitis or amebic liver abscess also should receive
a luminal agent such as paromomycin or the 8-hydroxyquinoline compound, iodo-
quinol. Nitazoxanide, a drug approved or the treatment o cryptosporidiosis and
giardiasis, is also active against E. histolytica.
b. What is paromomycin and what toxicities and side e ects should this patient
be warned o ?
Paromomycin (aminosidine) is an aminoglycoside used as an oral agent to treat
E. histolytica in ection. Adverse e ects af er oral administration are rare but include
abdominal pain and cramping, epigastric pain, nausea, vomiting, steatorrhea, and
diarrhea.
Parenteral administration carries the same risks o nephrotoxicity and ototoxicity
as with other aminoglycosides (see Chapter 40).

KEY CONCEPTS
Amebiasis, giardiasis, trichomoniasis, toxoplasmosis, cryptosporidiosis,
trypanosomiasis, and leishmaniasis are common protozoal in ections seen
worldwide.
Protozoa multiply rapidly in their hosts and e ective vaccines are
unavailable.
T erapy o protozoal in ections o en requires multiple drugs.
Antiprotozoal drugs have severe toxicities that require care ul monitoring.
Giardiasis, prevalent worldwide, is the most commonly reported protozoal
in ection in the United States.
richomoniasis is a sexually transmitted disease common in the United States.
reatment o patients with giardiasis or trichomoniasis using either metronida-
zole or tinidazole is usually success ul.

SUMMARY QUIZ

QUESTION 36-1 A 36-year-old woman with the diagnosis o richomonas vaginalis


returns to your o ce one day a er receiving a 2 g oral dose o metronidazole with
complaints o ushing, headache, vomiting, and abdominal pain. She states that her
symptoms began a er she and her husband celebrated their 10th wedding anniversary
at a restaurant. T e most likely cause o her symptoms is
a. chocolate.
b. red wine.
c. cabbage.
d. pork.
e. co ee.

QUESTION 36-2 A 24-year-old man with the diagnosis o late stage trypanosomiasis
caused by . brucei gambiense is being treated with e ornithine. E ornithine is an
inhibitor o ornithine decarboxylase. T e parasite and human enzyme are equally
susceptible to inhibition by e ornithine. T e selective toxicity o e ornithine in parasites
is because the
a. human enzyme is protected within vacuoles.
b. parasite accumulates the drug in high concentrations.
c. human enzyme is turned over more rapidly than the parasitic enzyme.
d. human enzyme contains a di erent molecular site o binding o e ornithine.
e. product o the enzyme in parasites is putrescine.

533
SECTION VII Chemotherapy of Microbial Diseases

QUESTION 36-3 Milte osine is the f rst orally active drug or the treatment o visceral
leishmaniasis. However, because o its teratogenic potential, milte osine is contraindi-
cated in pregnant women. A suitable alternative in a pregnant woman might be
a. metronidazole.
b. umagillin.
c. melarsoprol.
d. sodium stibogluconate.
e. pentamidine.

QUESTION 36-4 Amphotericin B is a highly e ective drug or the treatment o visceral


leishmaniasis. Amphotericin B works in leishmania by
a. inhibiting protein synthesis.
b. binding with and depleting calcium.
c. competing or enzymes that are responsible or olic acid synthesis.
d. inhibiting DNA synthesis.
e. binding with ergosterol precursors in the cell membrane.

QUESTION 36-5 Melarsoprol is the only drug e ective or the treatment o


a. late (CNS) stages o rypanosoma brucei rhodesiense.
b. late (CNS) stages o rypanosoma brucei gambiense.
c. early stages o rypanosoma brucei rhodesiense.
d. early stages o rypanosoma brucei gambiense.
e. oxoplasma gondii.

SUMMARY QUIZ ANSWER KEY


QUESTION 36-1 Answer is b. Patients taking metronidazole may have a disulf ram-
like reaction (see Chapter 9) i they drink alcohol. T e symptoms o en include ush-
ing, headache, vomiting, and abdominal pain.
QUESTION 36-2 Answer is c. T e molecular mechanism o e ornithine clearly is
inhibition o ornithine decarboxylase. E ornithine irreversibly inhibits both mamma-
lian and trypanosomal ornithine decarboxylases, thereby preventing the synthesis o
putrescine, a precursor o polyamines needed or cell division. E ornithine inactivates
the enzyme through covalent binding o an active-site cysteine residue. T e mamma-
lian enzyme is turned over rapidly, whereas the parasite enzyme is stable, and this di -
erence likely plays a role in the drugs selective toxicity. In addition, mammalian cells
may be able to replenish polyamine pools through uptake o extracellular polyamines.
T e parasite lacks e cient polyamine transport mechanisms.
QUESTION 36-3 Answer is d. T e classic therapy or all species o Leishmania is pentava-
lent antimony (sodium stibogluconate). T ough widely used, resistance to this compound
has led to widespread ailure o this drug in India. Other alternatives might be amphotericin
B and paromomycin. Pentamidine is use ul or the treatment o cutaneous leishmaniasis.
QUESTION 36-4 Answer is e. T e mechanism o action o amphotericin B against
leishmania is similar to the basis or the drugs anti ungal activity (see Chapter 43).
Amphotericin complexes with ergosterol precursors in the cell membrane, orming
pores that allow ions to enter the cell. Leishmania have a membrane sterol composition
similar to ungal pathogens, and the drug binds to these sterols pre erentially over the
host membrane cholesterol.
QUESTION 36-5 Answer is a. Despite the act that it causes an o en atal encephalop-
athy in 2 to 10% o patients, melarsoprol has remained the only drug or the treatment
o late (CNS) stages o East A rican trypanosomiasis (sleeping sickness) caused by
rypanosoma brucei rhodesiense. T is disease is 100% atal i untreated.

534
Chemotherapy of Protozoal Infections CHAPTER 3 6

SUMMARY: ANTIPROTOZOAL DRUGS


TOXICITIES
CLASS AND UNIQUE; CLINICALLY
SUBCLASSES NAMES CLINICAL USES RESISTANCE COMMON IMPORTANT
Amphotericin B AMBISOME Visceral leishmaniasis Not o clinical See Chapter 43 See Chapter 43
(This is an anti ungal importance
agent discussed in
Chapter 43)
E ornithine DFMO, ORNIDYL Trypanosomiasis caused Ine ective against Abdominal pain and Pneumonia, ever,
by T. brucei gambiense East A rican headache seizures, diarrhea
trypanosomiasis
Fumagillin FUMIDIL B Keratoconjunctivitis
caused by
Encephalitozoon hellem
8-Hydroxyquinolines Iodoquinol and Intestinal colonization Not o clinical Peripheral neuropathy
clioquinol with E. histolytica importance and myelo-optic
(Pre erred luminal agent neuropathy
is paromomycin to treat
amebiasis)
Melarsoprol MEL B; ARSOBAL Only drug or treatment Fatal encephalopathy
o late (CNS) stage o East in 2-10% o patients,
A rican trypanosomiasis peripheral neuropathy,
hypertension, myocardial
damage
Metronidazole FLAGYL Trichomoniasis, amebiasis, Clinical resistance Headache, nausea, Dizziness, vertigo,
Tinidazole and giardiasis to T. vaginalis, dry mouth, vomiting, seizures, ataxia,
Secnidazole anaerobic bacteria, diarrhea paresthesias
Ornidazole Helicobacter pylori
Benznidazole
Milte osine IMPAVIDO Leishmaniasis Vomiting and diarrhea Contraindicated in
pregnant women
Ni urtimox LAMPIT American Nausea and vomiting Psychic disturbance,
trypanosomiasis polyneuritis, CNS
excitability
Benznidazole ROCHAGAN American Nausea and vomiting
trypanosomiasis
Nitazoxanide ALINA Cryptosporidiosis and Abdominal pain,
giardiasis diarrhea, vomiting,
headache
Paromomycin HUMATIN Amebiasis, used in Abdominal pain, Risk o nephrotoxicity
combination with nausea, vomiting, and ototoxicity with
metronidazole to treat diarrhea parenteral use
amebic colitis and amebic
liver abscess
Pentamidine PENTAM 300 Early stage T. brucei Intravenous use
gambiense associated with
Used as an aerosol hypotension, tachycardia,
(NEBUPENT) to treat headache, hypoglycemia
Pneumocystis carinii
Sodium PENTOSTAM Leishmaniasis Clinical resistance is Pain at IM injection site, Bone marrow suppression,
Stibogluconate common chemical pancreatitis, ECG changes include
elevation o serum T-wave attening and
hepatic enzymes prolongation o QT interval
Suramin BAYER 205 A rican trypanosomiasis Not e ective Nausea, vomiting, Mazzotti reaction
against American malaise, atigue with concomitant
trypanosomiasis onchocerciasis (see
Chapter 37), renal toxicity,
delayed neurotoxicity

535
CHAPTER

37 Chemotherapyof
Helminth Infections
T is chapter will be most use ul a er having a basic understanding o the material in
DRUGS INCLUDED IN
Chapter 51, Chemotherapy o Helminth In ections in Goodman & Gilmans T e Phar-
THIS CHAPTER macological Basis o T erapeutics, 12th Edition. In addition to the material presented
Albendazole (ALBENZAand ZENTEL) here, the 12th Edition contains:
Diethylcarbamazine (DEC) A detailed discussion o helminth in ections and their treatment including nema-
Doxycycline todes (roundworms), cestodes ( atworms), and trematodes ( ukes)
Ivermectin (MECTIZAN, STROMECTOL)
LEARNING OBJECTIVES
Mebendazole (VERMOX,others)
Understand the common helminth in ections, the clinical symptoms, and the
Metrifonate (trichlorfon; BILARCIL) mainstays o therapy.
Niclosamide Describe the therapeutic uses o antihelmintic drugs.
Oxamniquine
Understand the mechanisms o actions o antihelmintic drugs.
Piperazine
Identi y the toxicities and contraindications o antihelmintic drugs.
Praziquantel (BILTRICIDE, DISTOCIDE)
Pyrantel Pamoate (PIN-X,others)
CASE 37-1
A 3-year-old boy is brought into your o ce. His mother is complaining that her child
is scratching between his legs which has caused irritation and redness.
a. Although you suspect a pinworm (Enterobius vermicularis) in ection, how is the
diagnosis made?
O en the worms or their eggs can be seen in the perianal area early in the morning.
A tape test is used to diagnose pinworm. In the morning be ore bathing or using the
toilet a piece o cellophane tape is pressed in the skin around the anus, and removed.
T e tape is then examined with a microscope to look or the worms or their eggs.
b. What is the appropriate treatment o this child?
T e relative incidence o common helminthic in ections in humans worldwide is
illustrated in Figure 37-1. Enterobius, pinworm, is one o the most common hel-
minth in ections in temperate climates, including the United States. Although this
parasite rarely causes serious complications, pruritus in the perianal region can be
severe, and scratching may cause secondary in ection. Salpingitis or even peritonitis
is a rare complication in emale patients.
Pyrantel pamoate, mebendazole, or albendazole as a single dose are highly e ec-
tive in the treatment o Enterobius in ection. A second dose is o en recommended
to be administered 2 weeks a er the rst. T e in ection easily spreads throughout
members o a amily, a school, or an institution, and may require treatment o all
individuals in close contact with an in ected person.

CASE 37 2
A 53-year-old man comes into your o ce with a 3-month history o weight loss,
abdominal pain, and atigue. His history is also signi cant or the consumption o par-
tially cooked sh during a camping trip 6 months previously. A physical examination
reveals a blood pressure o 140/60 mm Hg, pulse o 72 beats/min, and a respiratory rate
o 16 breaths/min. T e remainder o his physical examination is also normal with the
exception o pale nail beds and skin. A complete blood count shows the presence o
a megaloblastic anemia. An examination o a resh stool specimen reveals the eggs and
segments o a worm.
(Continued)
536
Chemotherapy of Helminth Infections CHAPTER 3 7

Infe ctions
(pe rce nta ge of world popula tion)
0 10 20 30

Trichine lla
S trongyloide s
Ta peworms
S chis tos ome s
Ente robius
Fila ria s
Trichuris
Hookworms
As ca ris

0 0.6 1.2 1.8


Numbe r of Infe ctions
(Billion)

FIGURE 37-1 Relative incidence o helminth in ections worldwide.

a. What is the likely diagnosis?


Diphyllobothrium latum, known as sh tapeworm, is ound most commonly in riv-
ers and lakes o the Northern Hemisphere. Eating inadequately cooked in ected sh
introduces the larvae into the human intestine. Although most in ected individuals
are asymptomatic, the most requent mani estations include abdominal symptoms
and weight loss.
b. What is the cause o this patients anemia?
Megaloblastic anemia develops due to a de ciency o vitamin B12 (see Chapter 25),
which is taken up by the parasite.
c. What is an appropriate treatment or this patient?
Praziquantel eliminates the worm and ensures hematological remission.
d. What are the likely side ef ects o this treatment?
Abdominal discom ort, particularly pain, nausea, and diarrhea, may occur shortly
a er taking praziquantel, and is transient and dose-related. Other adverse e ects
include headache, dizziness, and drowsiness, and patients should be warned not to
drive or operate machinery a er taking a dose. Praziquantel is contraindicated in
ocular cysticercosis because the host response can irreversibly damage the eye.

CASE 37 3
As a medical missionary to sub-Saharan A rica you are involved with the administra-
tion o ivermectin to rural communities or the treatment o onchocerciasis caused by
Onchocerca volvulus.
a. What is onchocerciasis?
Onchocerca volvulus is transmitted by black ies near ast- owing streams and rivers
in sub-Saharan A rica. In ammatory reactions, primarily to micro lariae rather
than adult worms, a ect the subcutaneous tissues, lymph nodes, and eyes. Oncho-
cerciasis is a leading cause o in ectious blindness worldwide and results rom the
cumulative destruction o micro lariae in the eyes that occurs over decades.
b. What are the usual therapeutic uses o ivermectin?
Ivermectin is the drug o choice or treatment o onchocerciasis, in adults and
children 5 years or older. It is also given in mass drug administration programs in
(Continued)
537
SECTION VII Chemotherapy of Microbial Diseases

the Americas and in sub-Saharan A rica. Ivermectin is not curative because it has
little e ect on adult Onchocerca volvulus.
A single annual dose o ivermectin is e ective and sa e or mass chemotherapy o
in ections with Wuchereria bancrof i and Brugia malayi. Ivermectin is as e ective
as diethylcarbamazine (DEC) or controlling lymphatic lariasis, and unlike DEC,
it can be used in regions where onchocerciasis, loiasis, or both are endemic.
Ivermectin administration is the drug o choice or treatment o human
strongyloidiasis.
c. Describe the mechanism o action o ivermectin?
Ivermectin immobilizes a ected organisms by inducing a tonic paralysis o the mus-
culature. T is appears to occur by binding to glutamate-activated Cl channels ound
in nematode nerve or muscle cells, which causes hyperpolarization by increasing
intracellular chloride concentration.
Ivermectin has no discernible e ect on adult parasites, even in high doses, but
a ects developing larvae and blocks egress o micro lariae rom the uterus o adult
emale worms.
d. How is ivermectin administered and how is it metabolized?
Peak concentrations o ivermectin in plasma are achieved within 4 to 5 hours a er
oral administration. T e long terminal t o approximately 57 hours re ects a low
systemic clearance and a large apparent volume o distribution. Ivermectin is
extensively metabolized by hepatic CYP3A4 to at least 10 metabolites.
A P-glycoprotein ef ux pump drug transporter located in the endothelium o brain
microvasculature appears to reduce ivermectin penetration into the CNS and may
explain the paucity o CNS side e ects in humans.
e. What are the common side ef ects associated with the use o ivermectin and what
precautions should be considered in the treatment o onchocerciasis?
Ivermectin is well tolerated by unin ected humans. In larial in ections, ivermectin
therapy requently causes a Mazzotti-like reaction to dying micro lariae. T e intensity
o these reactions relates to the micro larial burden.
Ivermectin is not approved or use in children younger than 5 years or pregnant
women.

KEY CONCEPTS
Worms pathogenic or humans are classi ed into roundworms (nematodes),
atworms and ukes (trematodes), and tapeworms (cestodes).
Anthelmintics are drugs that act either locally within the GI tract to cause
expulsion o worms, or systemically against helminthes residing outside the
GI tract.
Acquired resistance to anthelmintics in humans has yet to become a major
clinical problem.
reatment o loiasis or onchocerciasis should proceed with caution in patients
with large parasite burden due to tissue damage caused by destruction o the
micro lariae (Mazzotti-like reaction).

SUMMARY QUIZ
QUESTION 37-1 A 32-year-old man is seen because o megaloblastic anemia o
unknown origin. A er considerable workup, he is discovered to have an intestinal sh
tapeworm, Diphyllobothrium latum. reatment with praziquantel eliminates the worm

(Continued)

538
Chemotherapy of Helminth Infections CHAPTER 3 7

and cures his anemia. T e cause o the anemia in patients with Diphyllobothrium latum
in ection is
a. iron de ciency.
b. vitamin B12 de ciency.
c. olic acid de ciency.
d. vitamin E de ciency.
e. intestinal blood loss.
QUESTION 37-2 A 23-year-old woman returns rom several years in sub-Saharan A rica.
She has lymphadenitis and decreasing eyesight. She is diagnosed with onchocerciasis. Both
ivermectin and diethylcarbamazine (DEC) are known to be ef ective against Onchocerca
volvulus. Why is ivermectin chosen over DEC in the treatment o onchocerciasis?
a. DEC may worsen ocular lesions.
b. DEC is not absorbed rom the GI tract.
c. Ivermectin may be given by intramuscular injection.
d. Resistance to DEC is common.
e. DEC is contraindicated in women.
QUESTION 37-3 A 4-year-old boy is brought into your o ce because o a possible
pinworm in ection. Examination o the anal area con rms the diagnosis o Enterobius
(pinworm). Which o the ollowing drugs would you recommend to treat this in ection?
a. Ivermectin
b. DEC
c. Praziquantel
d. Pyrantel pamoate
e. Metri onate
QUESTION 37-4 A 25-year-old woman immigrant rom Guatemala is diagnosed with
aenia solium (pork tapeworm) upon examination o a stool specimen. ests reveal
that she has cysticercosis although a magnetic resonance image o the brain with a
contrast agent shows no signs o neurocysticercosis. Which o the ollowing drugs
would be appropriate or her treatment?
a. Ivermectin
b. DEC
c. Pyrantel pamoate
d. Metri onate
e. Albendazole
QUESTION 37-5 A 28-year-old man returns to the United States a er living 2 years
in Brazil. He complains o lethargy. Examination reveals mild liver dys unction. He is
diagnosed with Schistosoma mansoni. Which o the ollowing drugs is most appropriate
or treating this patient?
a. Praziquantel
b. Ivermectin
c. DEC
d. Albendazole
e. Metri onate

SUMMARY QUIZ ANSWER KEY


QUESTION 37-1 Answer is b. T e cause o anemia in Diphyllobothrium latum in ections
is a de ciency o vitamin B12, which is taken up by the parasite.
(Continued)

539
SECTION VII Chemotherapy of Microbial Diseases

QUESTION 37-2 Answer is a. DEC is contraindicated or the treatment o onchocer-


ciasis because it causes severe reactions related to micro larial destruction, including
worsening ocular lesions. Such reactions are ar less severe in response to ivermectin.
QUESTION 37-3 Answer is d. In the United States, pyrantel pamoate is sold as an
over-the-counter pinworm treatment. High cure rates are achieved a er a single oral
dose, although it is wise to repeat the treatment a er an interval o 2 weeks. I the child
is in a day-care center, it may be prudent to recommend that other children in the cen-
ter seek consultation with a physician and treatment i necessary. An alternate choice
would be mebendazole.
QUESTION 37-4 Answer is e. Niclosamide is the pre erred drug or treatment o
intestinal in ections with . solium. However, niclosamide poses a risk to people
in ected with . solium because ova released rom drug-damaged gravid worms
develop into larvae that can cause cysticercosis. Albendazole is the drug o choice
or treating cysticercosis. reatment o neurocysticercosis is controversial because
anthelmintic treatment can shrink brain cysts but also can have adverse consequences
leading to seizures and hydrocephalus. Pretreatment with glucocorticoids is advised.
Chemotherapy or neurocysticercosis is appropriate only when it is directed at live
cysticerci and not against dead or dying cysticerci (as evidenced by neuroimaging).
QUESTION 37-5 Answer is a. Praziquantel is the drug o choice or treatment o
schistosomiasis. Metri onate has been used with success in the treatment o Schistosoma
haematobium, but it is not ef ective against S. mansoni.

SUMMARY: ANTHELMINTIC DRUGS


TOXICITIES
CLASS AND UNIQUE; CLINICALLY
SUBCLASSES NAMES CLINICAL USES RESISTANCE COMMON IMPORTANT
Benzimidazoles Albendazole GI nematodes, including Resistance among Mild GI symptoms Liver dys unction in
Mebendazole enterobiasis human nematodes is long-term use
Cystic hydatid disease due to con ned to T. trichiura Albendazole should not
E. granulosus be given to pregnant
Cysticercosis caused by T. solium women

Diethylcarbamazine DEC Lymphatic lariasis Not o clinical Anorexia, nausea, Contraindicated


signi cance headache or treatment o
onchocerciasis because o
Mazzotti reaction

Doxycycline Treatment o bacterial See Chapter 41 See Chapter 41


symbionts o the genus
Wolbachia

Ivermectin MECTIZAN Onchocerciasis Not o clinical Mazzotti-like reaction in


STROMECTOL Lymphatic lariasis signi cance larial in ections
Strongyloidiasis Resistance has been Not approved in children
observed in livestock <5 years or in pregnant
and is o concern women
in mass treatment
programs

Praziquantel BILTRICIDE Most cestodes and trematodes Not o clinical GI symptoms, In neurocysticercosis,
DISTOCIDE that in ect humans signi cance headache, in ammatory
Schistosomiasis and liver uke dizziness reactions may produce
in ections meningismus and seizures

(Continued)

540
Chemotherapy of Helminth Infections CHAPTER 3 7

TOXICITIES
CLASS AND UNIQUE; CLINICALLY
SUBCLASSES NAMES CLINICAL USES RESISTANCE COMMON IMPORTANT
Metri onate Trichlor on Schistosoma haematobium Not o clinical
signi cance

Oxamniquine Second-line drug a ter


praziquantel or S. mansoni
in ection

Niclosamide Second-line drug a ter Not o clinical Poses a risk o causing


praziquantel or T. saginata, signi cance cysticercosis in patients
D. latum; no longer approved with T. solium
or use in the United States

Piperazine Highly ef ective against Ascaris Not o clinical


lumbricoides and Enterobius signi cance
vermicularis, but is superseded
by the better tolerated
benzimidazoles

Pyrantel pamoate PIN-X, others Pinworm, roundworm, and Not o clinical Mild GI symptoms, Not recommended in
hookworm in ections signi cance headache, pregnant women or
dizziness children <2 years

541
CHAPTER
Sulfonamides, Trimethoprim-
38 Sulfamethoxazole, Quinolones, and
Agents for UrinaryTract Infections
T is chapter will be most use ul a er having a basic understanding o the material in
DRUGS INCLUDED IN
Chapter 52, Sul onamides, rimethoprim-Sul amethoxazole, Quinolones, and Agents or
THIS CHAPTER Urinary ract In ections in Goodman & Gilmans T e Pharmacological Basis o T erapeutics,
Ma enide (SULFAMYLON) Methenamine 12th Edition. In addition to the material presented here, the 12th Edition contains:
Nitro urantoin (FURADANTIN, Structural ormulas or each o the drugs in this chapter, in addition to the gures
MICROBID,others) reproduced here
Phenazopyridine (PYRIDIUM, others) able 52-2 which is a compilation o the structural ormulas or selected quinolones
Quinolones (nor oxacin [NOROXIN, and uoroquinolones
others], o oxacin, [FLOXIN, others],
cipro oxacin [CIPRO, others], moxi oxacin LEARNING OBJECTIVES
[AVELOX]) Understand the mechanism o action o sul onamide drugs.
Silver sul adiazine (SILVADENE, others) Identi y the various sul onamide drugs and categorize them according to their
Sul acetamide absorption rom the gastrointestinal (GI) tract.
Sul adiazine Identi y the therapeutic uses and untoward e ects o sul onamide drugs includ-
Sul adoxine (FANSIDAR) ing trimethoprim-sul amethoxazole.
Sul amethoxazole Describe the therapeutic uses, mechanisms o action, and toxicities o quino-
lone antibiotic drugs.
Sul asalazine (AZULFADINE, others)
Sul soxazole Identi y the uses and limitations o antiseptic and analgesic drugs or the treat-
ment o urinary tract in ections.
Trimethoprim-sul amethoxazole (BACTRIM,
SEPTRA, others) T e mechanism o action o sul onamides is shown in Figure 38-2 and the resis-
tance to sul onamides is described in the side bar BAC ERIAL RESIS ANCE O
SULFONAMIDES
T e mechanism o action o quinolines is shown in Figure 38-3 and the resistance to
BACTERIAL RESISTANCE
quinolines is described in the side bar BAC ERIAL RESIS ANCE O QUINOLINES.
TO SULFONAMIDES
Resistance to sul onamides is the conse-
CASE 38-1
quence o altered enzymaticconstitution
o the bacterial cell characterized by: A 56-year-old woman presents with symptoms o her second urinary tract in ection
Alower a nityo dihydropteroate within 2 months. She has no ever and her white blood cell count is not elevated. A
synthesis enzymes. previous culture showed Escherichia coli, and she responded well to the combination o
trimethoprim-sul amethoxazole. A er obtaining the appropriate cultures and sensitivi-
Decreased bacterial permeabilityor active
ties you decide to treat her again with this combination.
ef uxo the drug.
An alternative metabolicpathway or a. What is unique about the mechanism o action o this combination that makes it
synthesis o an essential metabolite. ef ective in treating bacterial in ections?
Increased production o an essential T e sul onamides can be classi ed on the basis o the rapidity with which they are
metabolite or drug antagonist. absorbed and excreted (see able 38-1). T e structural ormulas o selected mem-
bers o this class are shown in Figure 38-1.
T e antimicrobial activity o the combination o trimethoprim and sul amethoxa-
zole results rom its actions on 2 steps o the enzymatic pathway or the synthesis o
BACTERIAL RESISTANCE tetrahydro olic acid (see Figure 38-2). Mammalian cells use pre ormed olates rom
TO QUINOLINES the diet and do not synthesize the compound. rimethoprim is a highly selective
inhibitor o dihydro olate reductase o lower organisms. T is relative selectivity is
Resistance toquinolines maydevelop dur-
vital because dihydro olate reductase unction is essential to all species.
ing therapyvia mutations in the bacterial
chromosomal genes encoding DNAgyrase or b. T e culture o the urine shows again Escherichia coli sensitive to the combination
topoisomerase IV(see Figure 38-3), or byactive o trimethoprim-sul amethoxazole. Why is this step necessary in this patient
transport o the drug out o the bacteria. with a previous urinary tract in ection?
(Continued)
542
Sulfonamides, Trimethoprim-Sulfamethoxazole, Quinolones, and Agents CHAPTER 3 8

4 1 N
H2 N S O 2 NH2 H2 N S O 2 NH
N
S ULFANILAMIDE S ULFADIAZINE

H2 N S O 2 NH H2 N S O 2 NH O
N
N CID
O CH3 H3 C CH3
S ULFAMETHOXAZOLE S ULFIS OXAZOLE

O
H2 N S O 2 NH C CH3 H2 N COOH

S ULFACETAMIDE PARA-AMINOBENZOIC ACID

FIGURE 38-1 Structural ormulas o selected sul onamides and para-aminobenzoic acid. The N
o the para-NH2 group is designated as N4; that o the amide NH2, as N1.

reatment o uncomplicated lower urinary tract in ection (U I) with trimethoprim-


sul amethoxazole o en is highly e ective or sensitive bacteria. T e combina-
tion appears to have special e cacy in chronic and recurrent in ections o the
urinary tract.
Bacterial resistance to trimethoprim-sul amethoxazole is an increasing problem,
although resistance is lower than it is to either agent alone. Resistance is o en due
to the acquisition o a plasmid that codes or an altered dihydro olate reductase.
c. Since the combination trimethoprim-sul amethoxazole a ects olate synthesis,
does it induce olate def ciency in humans? O what untoward e ects should this
patient be warned?
rimethoprim-sul amethoxazole, in recommended dosage, does not induce olate
def ciency in normal persons. T e margin between toxicity or bacteria and that
or humans may be relatively narrow when the cells o the patient are def cient
in olate. In such cases, trimethoprim-sul amethoxazole may cause or precipitate
megaloblastosis, leucopenia, or thrombocytopenia.
About 75% o the untoward e ects involve the skin and are due to the sul a moiety
in sul amethoxazole. Ex oliative dermatitis, Stevens-Johnson syndrome, and epi- P te ridine + P ABA
dermal necrolysis are rare, occurring primarily in older individuals. Glossitis and
stomatitis are relatively common. s ulfona mide s

TABLE 38-1 Classif cation o Sul onamides According to Absorption and Dihydropte roic a cid
Elimination Kinetics gluta ma te
CLASS SULFONAMIDE SERUM t (hours)
Absorbed and eliminated Sulf soxazole 5-6
rapidly Sul amethoxazole 11
Sul adiazine 10 Dihydrofolic a cid
NADP H
Poorly absorbed, active in Sul asalazine trime thoprim
bowel
NADP
Used topically Sul acetamide
Silver sul adiazine Te tra hydrofolic a cid

Long-acting (absorbed rapidly Sul adoxine 100-230 FIGURE 38-2 Steps in olate metabolism
but eliminated slowly) blocked by sul onamides and trimethoprim.
PABA, para-amino benzoic acid
543
SECTION VI Chemotherapy of Microbial Diseases

CASE 38-2
A 42-year-old man has been treated with cipro oxacin or a known Hemophilus inf u-
enza in ection o the respiratory tract or 2 weeks without improvement o symptoms.
Although an initial culture showed sensitivity o the organism to cipro oxacin, a repeat
culture shows that the organism is now resistant.
a. How does resistance develop to the quinolones?
Resistance to the quinolone drugs may develop during therapy via mutations in
the bacterial chromosomal genes encoding DNA gyrase or topoisomerase IV, or by
active transport o the drug out o the bacteria (see Side Bar BAC ERIAL RESIS-
ANCE O QUINOLINES). Resistance has increased a er the introduction o the
f uoroquinolones, especially in Pseudomonas and Staphylococci. Increasing resis-
tance is being observed in C. jejuni, Salmonella, N. gonorrhoeae, and S. pneumoniae.
b. How can the manner in which quinolones are administered af ect the develop-
ment o resistance?
As mentioned in Chapter 34, the pharmacokinetic and pharmacodynamic
parameters o antimicrobial agents are important in preventing the selection and
spread o resistant strains and have led to the concept o the mutation-prevention
concentration, which is the lowest concentration o antimicrobial that prevents
selection o resistant bacteria rom high bacterial inocula. -Lactams are time-
dependent agents without signi cant post-antibiotic e ects, resulting in bacterial
eradication when unbound serum concentrations exceed MICs o these agents
against in ecting pathogens or more than 40 to 50% o the dosing interval. By
contrast, f uoroquinolones are concentration- and time-dependent agents, result-
ing in bacterial eradication when unbound serum area-under-the-curve-to-MIC
ratios exceed 25 to 30. An extended release ormulation o ciprof oxacin exempli-
es this principle.
c. What is the mechanism o action o the quinolones?
T e quinolone antibiotics target bacterial DNA gyrase and topoisomerase IV (see
Figure 38-3). For many gram-positive bacteria (such as S. aureus), topoisomerase
IV is the primary activity inhibited by the quinolones. In contrast, DNA gyrase is
the primary quinolone target in many gram-negative microbes (such as E. coli).
T e quinolones inhibit gyrase-mediated DNA supercoiling at concentrations
that correlate well with those required to inhibit bacterial growth. Mutations o
the gene that encodes the gyrase A subunit polypeptide can con er resistance to
these drugs.
opoisomerase IV separates interlinked (catenated) daughter DNA molecules that
are the product o DNA replication. Eukaryotic cells do not contain DNA gyrase,
but they do contain a conceptually and mechanistically similar type II DNA topoi-
somerase that removes positive supercoils rom DNA to prevent its tangling during
replication. Quinolones inhibit eukaryotic type II topoisomerase only at concentra-
tions much higher than those that inhibit bacterial DNA gyrase.

S ta bilize Bre a k Re s e a l bre a k


pos itive node (+) ba ck s e gme nt on front s ide ()

() () ()
1 2 3

FIGURE 38-3 Model o the ormation o negative DNA supercoils by DNA gyrase. The enzyme binds to 2 segments o DNA
(1), creating a node o positive (+) superhelix. The enzyme then introduces a double-strand break in the DNA and passes the
ront segment through the break (2). The break is then resealed (3), creating a negative () supercoil. Quinolones inhibit the
nicking and closing activity o the gyrase and, at higher concentrations, block the decatenating activity o topoisomerase IV.
(From Cozzarelli NR. DNAgyrase and the supercoiling of DNA. Science, 1980, 207:953-960. Reprinted with permission from AAAS.)
544
Sulfonamides, Trimethoprim-Sulfamethoxazole, Quinolones, and Agents CHAPTER 3 8

CASE 38-3
A 48-year-old man has had chronic urinary tract in ections due to loss o bladder control ol-
lowing a spinal cord injury. He is now being treated with nitro urantoin chronically to sup-
press E. coli which has been the organism most commonly cultured rom his urinary tract.
a. What kind o a drug is nitro urantoin and what is its mechanism o action?
Nitro urantoin is used or the prevention and treatment o in ections o the urinary
tract. Enzymes capable o reducing nitro urantoin result in the ormation o reac-
tive intermediates that damage DNA. T e selective antimicrobial activity is due to
the act that bacteria reduce nitro urantoin more rapidly than do mammalian cells.
T e antibacterial activity is higher in an acidic urine. Nitro urantoin is approved
only or the treatment o urinary tract in ections caused by microorganisms known
to be susceptible to the drug. Resistance to nitro urantoin occurs more requently
than resistance to f uoroquinolones or trimethoprim-sul amethoxazole, making
nitro urantoin a second-line agent or treatment o urinary tract in ections.
b. Should this patient develop a systemic in ection, would nitro urantoin be ef ective?
Nitro urantoin is absorbed rapidly and completely rom the GI tract, but antibacte-
rial concentrations are not achieved in plasma because the drug is eliminated
rapidly. Forty percent o nitro urantoin is excreted unchanged into the urine.
c. What are the common untoward ef ects o nitro urantoin?
T e most common untoward e ects o nitro urantoin are nausea, vomiting, and
diarrhea. Hypersensitivity reactions include chills, ever, leucopenia, granulocy-
topenia, hemolytic anemia (associated with glucose-6-phosphate dehydrogenase
[G6PD] de ciency), cholestatic jaundice, and hepatocellular damage. Chronic active
hepatitis is an uncommon but serious side e ect. Interstitial pulmonary brosis can
occur in patients taking the drug chronically. Nitro urantoin colors the urine brown.

KEY CONCEPTS
Sul onamides inhibit the incorporation o para-amino-benzoic acid (PABA)
into tetrahydro olic acid (see Figure 38-2).
rimethoprim prevents the reduction o dihydro olate to tetrahydro olate (see
Figure 38-2).
T e combination o sul amethoxazole and trimethoprim inhibits 2 sequential
steps in the bacterial synthesis o tetrahydro olate, thus increasing its antibacte-
rial activity and reducing the development o resistance.
Sul onamide drugs (see able 38-1) can be divided into:
T ose that are well-absorbed and rapidly excreted
T ose that are poorly absorbed and are e ective in bowel lumen or
used topically
T ose that are long-acting
Sul onamides have activity against gram-positive and gram-negative organisms.
Resistant strains have become common.
Adverse reactions to sul onamides requently involve the skin and some are serious.
Hypersensitivity reactions to repeated use o sul onamides are common.
Quinolones are e ective orally or a wide variety o microorganisms and are
commonly used to treat urinary tract in ections, travelers diarrhea, and respira-
tory tract in ections.
Urinary antiseptics are commonly used to prevent or suppress urinary tract
in ections in susceptible patients.
Phenazopyridine, a urinary analgesic, is used to treat the symptoms o dysuria
and requency.
545
SECTION VI Chemotherapy of Microbial Diseases

SUMMARY QUIZ

QUESTION 38-1 Sul onamide drugs are selective or sensitive bacteria as compared to
mammalian cells because
a. mammalian cells have the ability to extrude sul onamide drugs.
b. mammalian cells do not take up sul onamide drugs.
c. mammalian cells require pre ormed olic acid.
d. bacterial cells accumulate sul onamide drugs more than mammalian cells.
e. sul onamide drugs inter ere with the synthesis o vitamin B12 in bacterial cells but
not mammalian cells.

QUESTION 38-2 Ma enide is a sul onamide drug that is used topically in burn patients.
I the burn area is extensive and ma enide is suf ciently absorbed, a metabolic acidosis
may occur because the ma enide
a. inhibits carbonic anhydrase.
b. is an acid.
c. inhibits olic acid excretion.
d. depresses respiration and causes the accumulation o carbon dioxide.
e. causes an accumulation o dihydropteroic acid.

QUESTION 38-3 In the combination drug trimethoprim-sul amethoxazole, the trim-


ethoprim moiety is selective or bacterial cells compared to mammalian cells because
the trimethoprim
a. is accumulated in bacterial cells.
b. is a selective inhibitor o dihydro olate reductase in lower organisms.
c. destroys bacterial cell walls.
d. inhibits the incorporation o PABA into dihydropteroic acid.
e. inhibits bacterial protein synthesis.

QUESTION 38-4 A 23-year-old pregnant woman presents with the signs and
symptoms o a urinary tract in ection. She is 3 months pregnant. T e choice o a
trimethoprim-sul soxazole drug over a uoroquinolone drug might be recom-
mended because
a. the likely bacteria involved in a U I during pregnancy are not sensitive to
uoroquinolones.
b. in pregnancy, the uoroquinolone drugs are not absorbed a er oral administration.
c. the uoroquinolones are contraindicated in pregnancy.
d. trimethoprim-sul soxazole achieves much higher plasma concentrations than uo-
roquinolone drugs in pregnancy.
e. trimethoprim-sul amethoxazole also has urinary analgesic e ects.

QUESTION 38-5 A 46-year-old woman has a urinary tract in ection which has become
a systemic in ection with ever and lethargy. Nitro urantoin, an e ective urinary anti-
septic, would not be recommend or her treatment because
a. the bacterial organism is likely not sensitive to nitro urantoin.
b. nitro urantoin is bacteriostatic.
c. e ective concentrations o nitro urantoin are not achieved in the kidney or bladder.
d. e ective concentrations o nitro urantoin are not achieved in the plasma.
e. bacterial resistance to nitro urantoin is common.

546
Sulfonamides, Trimethoprim-Sulfamethoxazole, Quinolones, and Agents CHAPTER 3 8

SUMMARY QUIZ ANSWER KEY

QUESTION 38-1 Answer is c. In sensitive bacteria, sul onamide drugs inhibit the
enzyme responsible or the incorporation o PABA into dihydropteroic acid, the imme-
diate precursor o olic acid (see Figure 38-2). Mammalian cells require pre ormed olic
acid, cannot synthesize it, and are insensitive to drugs that act by inhibiting the synthe-
sis o olic acid.

QUESTION 38-2 Answer is a. Ma enide and its metabolite inhibit carbonic anhydrase,
and the urine becomes alkaline. Metabolic acidosis with compensatory tachypnea and
hyperventilation may ensue.

QUESTION 38-3 Answer is b. rimethoprim is a highly selective inhibitor o dihydro-


late reductase in bacteria; approximately 100,000 times more drug is required to inhibit
human reductase than the bacterial enzyme. T is relative selectivity is vital because this
enzymatic unction is essential to all species.

QUESTION 38-4 Answer is c. T e uoroquinolones are contraindicated in pregnancy.

QUESTION 38-5 Answer is d. Antibacterial concentrations o nitro urantoin are not


achieved in plasma ollowing oral administration o recommended doses because the
drug is eliminated rapidly.

SUMMARY: SULFONAMIDES, TRIMETHOPRIM-SULFAMETHOXAZOLE, QUINOLONES, AND AGENTS FOR URINARYTRACT


INFECTIONS
TOXICITIES
CLASS AND UNIQUE; CLINICALLY
SUBCLASSES NAMES CLINICAL USES RESISTANCE COMMON IMPORTANT
Sul soxazole Rapidly absorbed and Because o high Use with caution in patients
eliminated solubility, it in requently with impaired renal unction
Used in combination causes hematuria or Hypersensitivity reactions
with erythromycin or crystalluria
otitis media

Sul amethoxazole Urinary tract in ections Insoluble acetylated orm


Fixed-dose Slightly slower in the urine may cause
combinations with absorption than crystalluria
trimethoprim sul soxazole

Sul adiazine Absorbed rapidly and Insoluble acetylated orm


acetylated orm is may cause crystalluria
excreted slowly

Sul asalazine (see Poorly absorbed and Side e ects and In ertility in males
Chapter 33) is used or ulcerative toxicities are due to the Should not be used in
colitis and regional sul apyridine moiety patients with known sul a
enteritis drug hypersensitivity

Sul acetamide Used primarily or Should not be used in


ophthalmic in ections patients sensitive to
sul a drugs

Silver sul adiazine Used topically to Resistance is Burning, itching, Should not be used in
reduce the incidence o known to occur and rash patients with known
in ections rom burns hypersensitivity to sul a drugs

547
SECTION VI Chemotherapy of Microbial Diseases

TOXICITIES
CLASS AND UNIQUE; CLINICALLY
SUBCLASSES NAMES CLINICAL USES RESISTANCE COMMON IMPORTANT
Ma enide Used topically to Intense pain at site o Superin ection with Candida
prevent colonization o application may be a problem
burns Due to inhibition o carbonic
anhydrase metabolic
acidosis may occur

Sul adoxine Prophylaxis and Severe skin reactions, eg,


treatment o malaria Stevens-Johnson syndrome
(see Chapter 35)

Trimethoprim Urinary tract in ections The emergence o Skin rashes and other Should not be used in
Sul amethoxazole Treatment o resistant bacteria dermatologic reactions patients with known
Pneumocystis jirovecii in may limit its Glossitis and stomatitis hypersensitivity to sul a
AIDS patients use ulness are common drugs

Quinolones Ciprof oxacin Urinary tract in ections, Resistance may Nausea, vomiting, Headache and dizziness
Of oxacin prostatitis, sexually develop during abdominal discom ort Photosensitivity
Norf oxacin transmitted diseases, therapy Achilles tendon rupture in
Levof oxacin travelers diarrhea, patients >60 years o age
Moxif oxacin respiratory in ections, Should be used with
bone, joint, and so t caution in patients taking
tissue in ections antiarrhythmic agents that
prolong QT interval

Urinary Methenamine Chronic suppressive Combines with sul a drugs


Antiseptics therapy o urinary tract causing mutual antagonism
in ections Mandelate moiety causes
crystalluria

Nitro urantoin Urinary tract in ections; Resistance is more Nausea, vomiting, Colors urine brown
cannot be used or common than to diarrhea Hypersensitivity reactions
systemic in ections f uoroquinolones Interstitial pulmonary brosis
Should not be used in
pregnant women or patients
with impaired renal unction

Urinary Phenazopyridine Urinary analgesia to Colors urine orange or red


Analgesics alleviate symptoms o
dysuria

548
CHAPTER

Penicillins, Cephalosporins, and


Other -Lactam Antibiotics 39
T is chapter will be most use ul a er having a basic understanding o the material in
DRUGS INCLUDED
Chapter 53, Penicillins, Cephalosporins, and Other -Lactam Antibiotics in Goodman &
Gilmans T e Pharmacological Basis o T erapeutics, 12th Edition. In addition to the IN THIS CHAPTER
material presented here, the 12th Edition contains: Penicillins
A detailed discussion o the mechanisms o action and resistance o the penicillins Penicillin G
and cephalosporins Penicillin V
A detailed discussion o each o the penicillins and cephalosporins, including their Methicillin
pharmacokinetics and therapeutic uses in speci c in ections
Oxacillin (BACTOCILL)
Figure 53-3 which shows a comparison o the structure and composition o gram-
Na cillin
positive and gram-negative bacterial cell walls
Ampicillin
able 53-1 which shows the chemical structures and major properties o the various
penicillins Amoxicillin
able 53-2 which shows the chemical structures and dosage orms o selected cepha- Carbenicillin
losporins and related compounds Ticarcillin
able 53-3 which shows the cephalosporin generations and the use ul spectrum o Mezlocillin
each generation Piperacillin
Cephalosporins
LEARNING OBJECTIVES
Cephazolin (ANCEF, KEFZOL, others)
Understand the mechanisms o action o the penicillins, cephalosporins, and
other -lactam antibiotics. Cephalexin (KEFLEX,others)
Understand the mechanisms o resistance o the penicillins, cephalosporins, Ce adroxil (DURACEF)
and other -lactam antibiotics. Cephradine (VELOCEF)
Describe the therapeutic e ects o the penicillins, cephalosporins, and other Ce uroxime (ZANACEF)
-lactam antibiotics. Ce uroxime axetil (CEFTIN)
Describe the untoward e ects and contraindications o the penicillins, cephalo- Ce prozil (CEFZIL)
sporins, and other -lactam antibiotics. Ce metazole (ZEFAZONE)
Loracarbe (LORABID)
MECHANISMS OF ACTION AND RESISTANCE OF PENICILLINS, CEPHALOSPORINS, Ce otaxime (CLAFORAN)
AND -LACTAMASE INHIBITORS Ce triaxone (ROCEPHIN)
DRUGS MECHANISM OF ACTION MECHANISM OF RESISTANCE Ce dinir (OMNICEF)
Penicillins Inhibition o cell wall Inactivated by -lactamases Ce ditoren pivoxil (SPECTRACEF)
peptidoglycan synthesis (see Figure 39-2) Ce tibuten (CEDAX)
(see Figure 39-1) Antibiotic e ux pumps
Binding to penicillin-binding o gram-negative bacteria Ce podoxime proxetil (VANTIN)
proteins (see Figure 53-3 (see Figure 39-3) Ce tizoxime (CEFIZOX)
Goodman and Gilmans The
Pharmacological Basis o
Ce operazone (CEFOBID)
Therapeutics, 12th Edition) Ce tazidime (FORTAZ, others)
Cephalosporins Same as or penicillins Same as or penicillins Ce epime (MAXIPINE)
Other -LactamAntibiotics
Carbapenems Same as or penicillins Resistant to hydrolysis
by -lactamases (see Imipenem(PRIMAXIN)
individual drugs Meropenem(MERREM)
-Lactamase Inhibitors Inactivate certain Not appropriate Doripenem(DORIBAX)
-lactamases Ertapenem(INVANZ)
Aztreonam(AZACTAM)
(continues)

549
SECTION VII Chemotherapy of Microbial Diseases

DRUGS INCLUDED IN THIS Glycope ptide Glycope ptide


polyme r polyme r
CHAPTER (Cont.)
NAM NAM
-Lactamase Inhibitors
Clavulanicacid (AUGMENTUM, combination
with amoxicillin;TIMENTIN, combination
with ticarcillin)
Sulbactam(UNASYN, combination Re a ction s ite
pe nicillins
with ampicillin) ce pha los porins Trans pe ptidas e (PBP)
Tazobactam(ZOSYN, combination D-Ala nine L-Ala nine
with piperacillin) D-Gluta ma te
Glycope ptide Glycope ptide L-Lys ine
polyme r polyme r Glycine
D-Ala nine
NAM NAM NAM =
N-Ace tyl-
MECHANISMS OF mura mic a cid
BACTERIAL RESISTANCE
TO PENICILLINS AND
CHEPHALOSPORINS
Structural di erences in penicillin-binding FIGURE 39-1 Action o -lactam antibiotics in Staphylococcus aureus. The bacterial cell
proteins (PBPs) wall consists o glycopeptide polymers (a NAM-NAG amino-hexose backbone) linked via
Decreased a nity or PBPs bridges between amino acid side chains. In S. aureus, the bridge is (Gly)5-d -Ala between
lysines. The cross-linking is catalyzed by a transpeptidase, the enzyme that penicillins and
Inabilityo antibioticto penetrate
cephalosporins inhibit. NAM, N-acetyl-muramic acid; NAG, N-acetyl-glucosamine.
to site o action
Ef uxpumps that extrude antibiotic
CASE 39-1
(see Figure 39-3)
Enzymatic(-lactamase) destruction A 56-year-old man is brought to hospital with a productive cough and temperature o
o antibiotic(see Figure 39-2) 103F. A chest X-ray reveals pneumonia in the le lower lobe. A er obtaining sputum
and blood cultures he is started on an intramuscular injection o penicillin G. His
symptoms do not improve over the next 24 hours and the cultures return showing
S. pneumoniae not sensitive to penicillin.

CLASSES OF -LACTAMASE a. What is the mechanism o action o penicillin to sensitive bacteria?


ENZYMES T e cell walls o bacteria are essential or their normal growth an evelopment.
Pepti oglycan is a heteropolymeric compoun o the cell wall that provi es rigi
Class Aincludes the extended-spectrum mechanical stability by virtue o its highly cross-linke latticework structure (see
-lactamases (ESBLs) that degrade Figure 39-1). Figure 53-3 in Goodman and Gilmans T e Pharmacological Basis o
penicillins, some cephalosporins, and, T erapeutics, 12th E ition shows a comparison o the structure an composition o
in some instances, carbapenems. (Continued)
ClassBare Zn2+-dependent enzymesthat
destroyall -lactamsexcept aztreonam. O S CH3 1 S ite of a ction of pe nicillina s e
Class Cactive against cephalosporins. R C NH CH CH C 2 S ite of a ction of a mida s e
B A CH3 A Thia zoline ring
Class Dincludes cloxacillin-degrading 2
O C N CH COOH
B -la cta m ring
enzymes. Pe nicillins
1

Pe nicillina s e
Amida s e

O S CH3 O S CH3
R CH NH2 CH CH C R C NH CH CH C
CH3 CH3
O C N CH COOH O C N CH COOH
O OH H

R CH 6-Aminope nicilla nic a cid Pe nicilloic a cids

FIGURE 39-2 Structure o penicillins and products o their enzymatic hydrolysis.


550
Penicillins, Cephalosporins, and Other -Lactam Antibiotics CHAPTER 3 9

Amphiphillic
drug
Cha nne l

Oute r
me mbra ne

Acce s s ory Pe ripla s m


prote in
Cytopla s mic
me mbra ne

Efflux
tra ns porte r

FIGURE 39-3 Antibiotic e ux pumps o gram-negative bacteria. Multidrug e ux pumps


traverse both the inner and outer membranes o gram-negative bacteria. The pumps are
composed o a minimum o 3 proteins and are energized by the proton motive orce. Increased
expression o these pumps is an important cause o antibiotic resistance. (Reprinted with
permission rom Ox ord University Press. Nikaido H. Antibiotic resistance caused by gram-negative
multidrug e ux pumps. Clin In ect Dis, 1998, 27[suppl I]:S32-S41. 1998 by the In ectious Diseases
Society o America. All rights reserved.)

gram-positive an gram-negative cells. In the synthesis o the bacterial cell wall, the
terminal glycine resi ue o the pentaglycine bri ge is linke to the ourth resi ue
o the pentapepti e (d-alanine), releasing the h resi ue (also d-alanine). It is
this step in pepti oglycan synthesis that is inhibite by the -lactam antibiotics an
glycopepti e antibiotics such as vancomycin (see Chapter 41). T ere are a itional
targets or the actions o penicillins an cephalosporins that are collectively terme
penicillin-bin ing proteins (PBP). T e PBPs vary in their a nities or if erent
-lactam antibiotics, although the interactions eventually become covalent.
b. What is the mechanism o bacterial resistance to the penicillins?
-Lactam antibiotics are capable o being inactivate by -lactamases that are
present in large quantities (see Figure 39-2). T e -lactamases are groupe into
4 classes (see Si e Bar CLASSES OF -LAC AMASE ENZYMES). Some Class A
an D enzymes are inhibite by -lactamase inhibitors such as clavulanate an
tazobactam. Bacterial resistance to the -lactam antibiotics may also evelop by
mechanisms other than estruction by -lactamases (see Si e Bar MECHANISMS
OF BAC ERIAL RESIS ANCE O PENICILLINS AND CHEPHALOSPORINS).

CASE 39-2
A 68-year-old woman has contracted an S. epidermidis in ection that is not methicillin-
resistant. An in ectious disease consultant recommends the use o a penicillinase-
resistant penicillin.
a. What are the penicillinase-resistant penicillins?
T e penicillinase-resistant penicillins (oxacillin, icloxacillin, an cloxacillin) are
resistant to hy rolysis by staphylococcal penicillinase (see able 39-1). Na cillin is
a semisynthetic penicillin that is highly resistant to penicillinase an has proven
ef ective against in ections cause by penicillinase-pro ucing strains o S. aureus.
b. What are their therapeutic uses?
T e role o these penicillins as the agents o choice or most staphylococcal isease
is changing with the increasing inci ence o isolates o methicillin-resistant (MRSA)
microorganisms. T is term enotes resistance o these bacteria to all the penicil-
linase-resistant penicillins an cephalosporins. Hospital-acquire strains usually
are resistant to the aminoglycosi es, tetracyclines, erythromycin, an clin amycin
as well. Vancomycin (see Chapter 41) is consi ere the rug o choice or hospital-
acquire resistant strains, although resistance to vancomycin is also emerging.
551
SECTION VII Chemotherapy of Microbial Diseases

TABLE 39-1 Classif cation o The Penicillins and Summary o Their


Pharmacological Properties
DRUG ACTIVITY SPECIAL CIRCUMSTANCES
Penicillin G and V Sensitive strains o Hydrolyzed by penicillinase
gram-positive cocci Ine ective against most strains
o S. aureus

Penicillinase-Resistant: Less sensitive than Agents o rst choice or


Methicillin (discontinued in penicillin G and V penicillinase-producing
the United States), Na cillin, S. aureus and S. epidermidis
Oxacillin, Cloxacillin (not in the that are not methicillin-resistant
United States), Dicloxacillin

Ampicillin, Amoxicillin, Activity extended to gram- Frequently administered with


and Others negative organisms such -lactamase inhibitors to
as H. inf uenzae, E. coli, and prevent hydrolysis by Class A
Proteus mirabilis -lactamases

Carbenicillin (discontinued Activity extended to include In erior to ampicillin against gram-


in the United States) and Pseudomonas, Enterobacter, positive cocci and less active than
Ticarcillin and Proteus spp. piperacillin against Pseudomonas

Mezlocillin, Azlocillin (both Excellent activity against Utility is threatened by the


discontinued in the United Pseudomonas, Klebsiella, emergence o broad-spectrum
States), and Piperacillin and certain other gram- -lactamases
negative organisms Piperacillin retains the activity
o ampicillin against gram-
positive cocci

CASE 39-3
A 7-year-old girl is treated with amoxicillin or an otitis media o 3 days duration. A er
the rst dose, the girls mother notices swelling o the childs lips and mild dif culty
breathing. T e girl is seen in the emergency room where the diagnosis o penicillin
allergy is made, and the amoxicillin is discontinued.
a. T e mother does not recall her daughter ever taking penicillin. Is this possible?
Hypersensitivity reactions are by ar the most common a verse ef ects note
with the penicillins, which probably are the most common cause o rug allergy.
Hypersensitivity reactions may occur with any osage orm o penicillin; allergy to 1
penicillin exposes the patient to a greater risk o reaction i another penicillin is given.
Hypersensitivity reactions may appear in the absence o a previous known exposure
to the rug. A etaile iscussion o hypersensitivity reactions an the management
o the patient potentially allergic to penicillin can be oun in Chapter 53 o the 12th
E ition o Goodman and Gilmans T e Pharmacological Basis o T erapeutics.
b. What is the cause o penicillin allergy?
Penicillins an their break own pro ucts act as haptens a er covalent reaction to
proteins. T e most abun ant break own pro uct is the penicilloyl moiety (major
eterminant moiety, MDM), which is orme when the -lactam ring is opene
(see Figure 39-2). A large percentage o immunoglobulin (Ig)E-me iate reactions
are to the MDM, but at least 25% o reactions are to other break own pro ucts
(minor eterminants). T ese pro ucts are orme in vivo an also can be oun in
solutions o penicillin prepare or a ministration. Anaphylactic reactions to peni-
cillin usually are me iate by IgE antibo ies against the minor eterminants.
c. Is there a means o detecting penicillin allergy prior to giving a dose o penicillin?
Evaluation o the patients history is the most practical way to avoi the use o penicil-
lin in patients who are at the greatest risk o a verse reaction. Most patients who give
a history o allergy to penicillin shoul be treate with another antibiotic. Un ortu-
nately, there is no totally reliable means to con rm a history o allergy to penicillin.
552
Penicillins, Cephalosporins, and Other -Lactam Antibiotics CHAPTER 3 9

CASE 39-4 MECHANISM OF


A 43-year-old man is treated with the third-generation cephalosporin, ce azidime, or BACTERIAL RESISTANCE TO
an in ection with Pseudomonas. THE CEPHALOSPORINS
a. What is the therapeutic distinction in the di erent generations o cephalosporin Inabilityo antibioticto reach site o action
antibiotics? Alterations in penicillin-binding
See able 53-3 in Goodman & Gilmans T e Pharmacological Basis o T erapeutics, proteins (PBPs)
12th E ition an the Summary able at the en o this chapter or in ivi ual rugs. -Lactamases that hydrolyze the
T e rst-generation cephalosporin antibiotics have goo activity against gram- -lactamring
positive bacteria an mo est activity against gram-negative microorganisms. T e
secon -generation cephalosporins have somewhat increase activity against the
gram-negative microorganisms, but are much less active than the thir -generation
agents. T e thir -generation cephalosporins generally are less active against gram-
positive cocci; these agents are much more active against Enterobacteriacae, though
resistance is ramatically increasing ue to -lactamase-pro ucing strains. Fourth-
generation cephalosporins have an exten e spectrum o activity compare to the
thir -generation agents an have increase stability rom hy rolysis by plasmi
an chromosomally me iate -lactamases.
b. Why was this man treated with cef azidime?
Ce azi ime has activity against the Enterobacteriaceae very similar to ce otaxime,
but its istinguishing eature is its excellent activity against Pseudomonas an other
gram-negative bacteria.

CASE 39-5
A 43-year-old man is being treated with aztreonam or a gram-negative bacterial in ection.
a. How is aztreonam di erent rom the penicillins or cephalosporins?
Aztreonam belongs to the subclass o -lactam rugs calle carbapenems. But even
within this subclass, aztreonam is unique. Carbapenems have a broa er spectrum
o activity than most other -lactam antibiotics. Aztreonam has activity only
against gram-negative bacteria; it has no activity against gram-positive bacteria an
anaerobic organisms. However, activity against Enterobacteriaceae an P. aeruginosa
is excellent. It is also highly active in vitro against H. inf uenzae an gonococci.
b. How is aztreonam administered and what precautions should be ollowed?
Aztreonam is a ministere either intramuscularly or intravenously an most o the
rug is recovere unaltere in the urine. T e usual ose o aztreonam shoul be
re uce in patients with renal insu ciency.

CASE 39-6
A 64-year-old woman with type 2 diabetes develops a severe sore throat. She has been
caring or her 3-year-old grandchild who has similar symptoms. She is treated with a
combination o amoxicillin and clavulanic acid because o a concern that the organism
may be resistant to amoxicillin alone.
a. What are -lactamase inhibitors and how are they used to prevent bacterial
resistance to the penicillins?
-Lactamase inhibitors (clavulanic aci , sulbactam, an tazobactam) prevent the
estruction o -lactam antibiotics by -lactamase. T ey are most active against
plasmi -enco e -lactamases; they are inactive at clinically achievable concentra-
tions against the type I chromosomal -lactamase.
b. What other antibiotics are combined with -lactamase inhibitors?
Sulbactam is combine with ampicillin. azobactam is combine with piperacillin.

553
SECTION VII Chemotherapy of Microbial Diseases

KEY CONCEPTS
Penicillins and cephalosporins act by inhibiting bacterial cell wall synthesis.
Bacterial resistance to the penicillins and cephalosporins is a serious clinical
problem.
Penicillinase-resistant penicillins (na cillin, oxacillin, dicloxacillin, and cloxa-
cillin) are less active against microorganisms sensitive to penicillin but are the
agents o rst choice or the treatment o penicillinase-producing S. aureus and
S. epidermidis that are not methicillin resistant.
T e cephalosporins can be divided into rst-, second-, third-, and ourth-genera-
tion agents, with each generation having a di erent spectrum o activity.
T e carbapenems (imipenem, meropenem, doripenem, ertapenem, and aztreo-
nam) are -lactam antibiotics with a broader spectrum o activity than most
other -lactam antibiotics.
-Lactamase inhibitors are added to some -lactam antibiotics to extend their
activity against strains o bacteria that produce -lactamase.

SUMMARY QUIZ

QUESTION 39-1 A 23-year-old woman develops a rash ollowing her rst dose o peni-
cillin V. T e most likely cause o the rash is an allergy to penicillin. What component o
penicillin is the most requent cause o an allergic reaction?
a. T e intact penicillin molecule
b. T e penicilloic acid moiety (major determinant moiety)
c. T e 6-aminopenicillanic acid moiety
d. Other breakdown products (minor determinant moiety)
e. T e side chain attached to the -lactam ring through the amide linkage

QUESTION 39-2 A 48-year-old woman has an in ection that cultures a S. aureus known
to elaborate penicillinase. It is appropriate that she be treated with na cillin because
a. blood concentrations o na cillin are suf ciently high to kill all S. aureus.
b. na cillin is more potent against S. aureus than penicillin G.
c. na cillin is resistant to penicillinase.
d. na cillin is active against methicillin-resistant microorganisms (MRSA).
e. na cillin is a naturally occurring penicillin.

QUESTION 39-3 A 62-year-old man is being treated or gonorrhea with a third-generation


cephalosporin, ce riaxone. Ce riaxone is the drug o choice or gonorrhea rather than
a rst- or second-generation cephalosporin because
a. this third-generation cephalosporin has a shorter plasma hal -li e than the rst- or
second-generation cephalosporins.
b. ce riaxone has a hal -li e o about 8 hours making it e ective as a single dose.
c. it has a much lower incidence o adverse reactions compared to other cephalosporins.
d. it is active against methicillin-resistant bacteria.
e. ce riaxone is well absorbed a er oral administration.

QUESTION 39-4 A 37-year-old woman is being treated with the combination drug,
imipenem and cilastatin. Cilastatin is added to imipenem because the cilastatin
a. inhibits the destruction o imipenem by a renal tubular dipeptidase.
b. inhibits -lactamase.
c. increases the oral absorption o imipenem.
(Continued)
554
Penicillins, Cephalosporins, and Other -Lactam Antibiotics CHAPTER 3 9

d. decreases the incidence o nausea and vomiting caused by imipenem.


e. inhibits the possibility o allergic reactions in patients who have had allergic
reactions to other -lactam antibiotics.

QUESTION 39-5 A 16-year-old boy is treated or an in ection with a strain o staphylo-


cocci known to produce -lactamase with a combination o amoxicillin and clavulanic
acid. Clavulanic acid is added to amoxicillin because it
a. increases the oral absorption o amoxicillin.
b. decreases the renal excretion o amoxicillin.
c. decreases the protein binding o amoxicillin.
d. enhances the binding o amoxicillin to penicillin-binding proteins.
e. inhibits -lactamase.

SUMMARY QUIZ ANSWER KEY

QUESTION 39-1 Answer is b. A large percentage o immunoglobulin E-mediated


reactions are to the major determinant moiety, but at least 25% o reactions are to other
breakdown products which can be ound in solutions o penicillin or administration.

QUESTION 39-2 Answer is c. Na cillin is a semisynthetic penicillin that is highly resistant


to penicillinase and has proven e ective against in ections caused by penicillinase-
producing strains o S. aureus. T ere is an increasing incidence o isolates o methicillin-
resistant microorganisms (eg, MRSA) that are resistant to all penicillinase-resistant
penicillins including na cillin.

QUESTION 39-3 Answer is b. A single dose o ce riaxone is e ective in the treatment


o urethral, cervical, rectal, or pharyngeal gonorrhea. A single dose is e ective because
o its t o ~8 hours allows it to be administered intramuscularly as a single dose.

QUESTION 39-4 Answer is a. Imipenem, which is not absorbed orally, is marketed


in combination with cilastatin, a drug that inhibits the degradation o imipenem by
a renal tubular dipeptidase. Patients who are allergic to other -lactam antibiotics may
have hypersensitivity reactions to imipenem.

QUESTION 39-5 Answer is e. Clavulanic acid is a suicide inhibitor that irreversibly


binds -lactamases produced by a wide range o gram-positive and gram-negative
microorganisms. It has been combined with amoxicillin as an oral preparation and
with ticarcillin as a parenteral preparation .

SUMMARY: PENICILLINS, CEPHALOSPORINS, CARBAPENEMS, AND OTHER -LACTAM ANTIBIOTICS


TOXICITIES
CLASS AND UNIQUE; CLINICALLY
SUBCLASSES NAMES CLINICAL USES RESISTANCE COMMON IMPORTANT
Penicillins Penicillin G and Sensitive strains o Strains o bacteria that Allergic reactions
Penicillin V gram-positive cocci produce -lactamases
and MRSA

Oxacillin Sensitive strains o Methicillin-resistant Allergic reactions


staphylococci that S. aureus (MRSA)
elaborate penicillinase

Cloxacillin Same as oxacillin Same as oxacillin Allergic reactions


Dicloxacillin
Na cillin

Amoxicillin Bacteriocidal or Sensitive to penicillinase Allergic reactions


Ampicillin gram-positive and and MRSA Dose reduced in patients
gram-negative bacteria with renal ailure
(Continued)
555
SECTION VII Chemotherapy of Microbial Diseases

TOXICITIES
CLASS AND UNIQUE; CLINICALLY
SUBCLASSES NAMES CLINICAL USES RESISTANCE COMMON IMPORTANT

Carbenicillin P. aeruginosa and some Sensitive to penicillinase Allergic reactions


Proteus strains that are and MRSA May cause heart ailure
resistant to ampicillin due to administration o
excessive Na+
May inter ere with
platelet unction

Ticarcillin More active against Sensitive to penicillinase Allergic reactions


P. aeruginosa than and MRSA
carbenicillins

Mezlocillin More active against Sensitive to penicillinase Allergic reactions


Klebsiella than and MRSA
carbenicillin; discontinued
in the United States

Piperacillin Activity against Sensitive to penicillinase Allergic reactions


P. aeruginosa, and MRSA
Enterobacteriaceae,
many Bacteroides spp.,
and E. aecalis

Cephalosporins- Cephazolin Sensitive strains o Resistant to bacteria that Allergic reactions


First Generation Cephalexin streptococci and produce -lactamases
Ce adroxil Staphylococci aureus and MRSA
Cephradine

Cephalosporins- Ce uroxime Similar to ce aclor with Resistant to bacteria that Allergic reactions
Second Generation broader gram-negative produce -lactamases
activity and MRSA

Ce oxitin Treatment o certain Resistant to destruction Allergic reactions


anaerobic and mixed by some -lactamases
aerobic-anaerobic
in ections such as pelvic
in ammatory disease
and lung abscess

Ce aclor Used orally Some -lactamase Allergic reactions


Activity is similar to producing organisms
cephalexin are resistant
More active against
H. inf uenzae and
Moraxella catarrhalis

Ce otetan Activity against B. ragilis Resistant to bacteria that Allergic reactions


produce -lactamases
and MRSA

Ce uroxime axetil Prodrug o ce uroxime Resistant to bacteria that Allergic reactions


produce -lactamases
and MRSA

Ce prozil Orally administered Resistant to bacteria that Allergic reactions


More active than rst- produce -lactamases
generation agents against and MRSA
penicillin-sensitive
streptococci, E. coli, P.
mirabilis, Klebsiella spp.
and Citrobacter spp.
(Continued)

556
Penicillins, Cephalosporins, and Other -Lactam Antibiotics CHAPTER 3 9

TOXICITIES
CLASS AND UNIQUE; CLINICALLY
SUBCLASSES NAMES CLINICAL USES RESISTANCE COMMON IMPORTANT

Ce metazole Activity against B. ragilis; Allergic reactions


discontinued in the
United States

Loracarbe Similar to ce aclor More stable against some Allergic reactions


-lactamases

Cephalosporins- Ce otaxime Gram-positive and gram- Resistant to many Allergic reactions


Third Generation negative aerobic bacteria -lactamases

Ce triaxone Similar to ce otaxime Resistant to bacteria that Allergic reactions


Single dose used to treat produce -lactamases
gonorrhea; t is 8 hours and MRSA

Ce dinir Gram-positive and Resistant to bacteria that Allergic reactions


gram-negative in ections produce -lactamases
Indicated or treatment and MRSA
o otitis media, so t tissue,
and respiratory in ections
Inactive against
Pseudomonas and
Enterobacter spp.

Ce xime Treatment o otitis media Resistant to bacteria that Allergic reactions


caused by H. inf uenzae produce -lactamases
and S. pyrogenes and MRSA

Ce ditoren pivoxil Indicated or the Resistant to bacteria that Allergic reactions


treatment o pharyngitis, produce -lactamases
tonsillitis, uncomplicated and MRSA
skin in ections and acute
exacerbations o chronic
bronchitis

Ce tibuten Administered orally Resistant to bacteria that Allergic reactions


Activity limited to S. produce -lactamases
pneumoniae, S. pyrogenes, and MRSA
H. inf uenzae, and
M. catarrhalis

Ce podoxime Administered orally Resistant to bacteria that Allergic reactions


proxetil Similar activity to produce -lactamases
ce epime except it is and MRSA
more active against
Enterobacter and
Pseudomonas spp

Ce tizoxime Similar to ce otaxime Resistant to bacteria that Allergic reactions


except it is less active produce -lactamases
against S. pneumoniae and MRSA

Ce operazone E ective against Resistant to bacteria that Allergic reactions


Pseudomonas produce -lactamases May cause a clotting
and MRSA disorder
May cause a disul ram-
like reaction with alcohol
ingestion

Ce tazidime Excellent activity against Resistant to bacteria that Allergic reactions


Pseudomonas and other produce -lactamases
gram-negative bacteria and MRSA
(Continued)

557
SECTION VII Chemotherapy of Microbial Diseases

TOXICITIES
CLASS AND UNIQUE; CLINICALLY
SUBCLASSES NAMES CLINICAL USES RESISTANCE COMMON IMPORTANT

Cephalosporins- Ce epime Comparable to second- Stable to hydrolysis by Allergic reactions


Fourth Generation and third-generation many -lactamases Dose reduced in patients
agents with renal insuf ciency

Other -Lactam Imipenem Broader spectrum o Resistant to hydrolysis by Seizures in 1.5% o


Antibiotics activity than most other most -lactamases, but patients
-lactam agents not Klebsiella pneumoniae Allergic reactions
carbapenemase (KPC)- Dose reduced in patients
producing strains with renal insuf ciency

Meropenem Broader spectrum o Resistant to hydrolysis by Less risk o seizures than


activity than most other most -lactamases, but imipenem
-lactam agents not KPC-producing strains Allergic reactions
Dose reduced in patients
with renal insuf ciency

Doripenem Broader spectrum o Resistant to hydrolysis by Allergic reactions


activity than most other most -lactamases, but Dose reduced in patients
-lactam agents not KPC-producing strains with renal insuf ciency

Ertapenem Broader spectrum o Resistant to hydrolysis by Allergic reactions


activity than most other most -lactamases, but
-lactam agents not KPC-producing strains

Aztreonam Activity only against Resistant to hydrolysis by Dose reduced in patients


gram-negative most -lactamases, but with renal insuf ciency
bacteria; active against not KPC-producing strains Allergic reactions
Enterobacteriaceae,
P. aeruginosa, and
H. inf uenzae

-Lactamase Clavulanic acid Combined with


Inhibitors amoxicillin and
with ticarcillin

Sulbactam Combined with ampicillin

Tazobactam Combined with


piperacillin

558
CHAPTER

Aminoglycosides 40
T is chapter will be most use ul a er having a basic understanding o the material in
DRUGS INCLUDED IN
Chapter 54, Aminoglycosides in Goodman & Gilmans T e Pharmacological Basis of T era-
peutics, 12th Edition. In addition to the material presented here, the 12th Edition contains: THIS CHAPTER
Figure 54-1 which shows the sites o activity o various plasmid-mediated enzymes Amikacin
capable o inactivating aminoglycosides Gentamicin (GARAMYCIN, others)
able 54-1 which provides the minimal inhibitory concentrations o aminoglycosides Kanamycin
that will inhibit 90% (MIC90) o clinical isolates or several bacterial species Neomycin
able 54-2 which provides an algorithm or dose reduction o aminoglycosides based Netilmicin (NETROMYCIN)
on creatinine clearance
Streptomycin
LEARNING OBJECTIVES Tobramycin (TOBREX,others)
Understand aminoglycoside mechanisms o action and resistance.
Describe the advantages and disadvantages o multiple daily dosing versus once
daily extended-interval dosing regimens or aminoglycosides. MECHANISMS OF ACTION
Describe the rationale and the methods o plasma concentration monitoring OF AMINOGLYCOSIDES
o aminoglycoside therapy. The aminoglycosides are rapidlybac-
Describe the causes and clinical signs o aminoglycoside ototoxicity and nephro- tericidal and their bacterial killing is
toxicity and the best means o monitoring therapy to avoid these serious toxicities. concentration-dependent
Understand the unique clinical di erences among the aminoglycosides. Aminoglycosides exhibit a postantibiotic
ef ect, that is, the bactericidal activity
T e mechanisms o action and resistance o the aminoglycosides are shown in the persists a ter serumconcentration alls
side bars MECHANISMS OF AC ION OF AMINOGLYCOSIDE and MECHANISMS belowthe MIC
OF AMINOGLYCOSIDE RESIS ANCE, respectively.
Duration o postantibioticef ect is also
concentration-dependent
CASE 40-1
Inside the bacterial cell, the aminoglyco-
A 56-year-old woman is in hospital or the treatment o pneumonia. Her case is com- sides bind to polysomes and inter ere with
plicated because she acquired the pneumonia in another hospital while undergoing protein synthesis bycausing misreading
post-surgical rehabilitation or a hysterectomy. She was being treated with a cephalo- and premature termination o mRNAtrans-
sporin at the other hospital, but gentamicin is now added. lation (see Figure 40-1)
a. Why was the aminoglycoside added to her therapeutic regimen?
An aminoglycoside in combination with a -lactam antibiotic is recommended as stan-
dard therapy or hospital-acquired pneumonia in which a multiple-drug resistant gram- MECHANISMS OF
negative aerobe is a likely causative agent. I it is established that the -lactam is active AMINOGLYCOSIDE
against the causative agent, there is generally no need to continue the aminoglycoside.
RESISTANCE
b. Why is the combination o a -lactam antibiotic and an aminoglycoside e ective?
Failure o the aminoglycoside to penetrate
T ese 2 antibiotic classes have distinctly di erent mechanisms o action and their the bacteria cell
e ectiveness should be complementary. -Lactam antibiotics inhibit cell wall synthesis
Inactivation o the aminoglycoside by
(see Chapter 39). Aminoglycosides act at the 30S ribosomal subunit to disrupt the nor-
microbial enzymes
mal cycle o ribosomal unction by inter ering with the initiation o protein synthesis,
also by leading to accumulation o abnormal initiation complexes and premature Lowa nityo the aminoglycoside or the
termination, and by causing misreading o the mRNA template and incorporation o bacterial ribosome
incorrect amino acids into the growing polypeptide chains (see Figure 40-1).

CASE 40-2
A 23-year-old woman is being treated with gentamicin or an enterococci in ection.
wenty- our hours a er beginning treatment it is learned that the organism is resistant
to gentamicin. (Continued)
559
SECTION VII Chemotherapy of Microbial Diseases

FIGURE 40-1 E ects o aminoglycosides on protein synthesis. A. Aminoglycoside (represented by dark grey circles) binds to the 30S
ribosomal subunit and inter eres with initiation o protein synthesis by xing the 30S to 50S ribosomal complex at the start codon (AUG) o
mRNA. As 30S to 50S complexes downstream complete translation o mRNA and detach, the abnormal initiation complexes, the so-called
streptomycin monosomes, accumulate, blocking urther translation o the message. Aminoglycoside binding to the 30S subunit also causes
misreading o mRNA, leading to B, premature termination o translation with detachment o the ribosomal complex and incompletely
synthesized protein or C, incorporation o incorrect amino acids (indicated by the grey X), resulting in the production o abnormal or
non unctional proteins.

a. Does resistance to gentamicin mean resistance to other aminoglycosides?


Resistance to gentamicin indicates cross-resistance to tobramycin, amikacin, kana-
mycin, and netilmicin because the inactivating enzyme is bi unctional and can
modi y all these aminoglycosides. T e chemical structure o streptomycin is su -
ciently di erent rom gentamicin that this inactivating enzyme does not modi y
streptomycin; consequently gentamicin-resistant strains o enterococci may be sus-
ceptible to streptomycin.
Amikacin is a suitable substrate or only a ew o the inactivating enzymes that con-
er gentamicin-resistance; thus, strains that are resistant to multiple other amino-
glycosides tend to be susceptible to amikacin.
A signi cant percentage o clinical isolates o Enterococcus faecalis and E. faecium
are highly resistant to all aminoglycosides.
b. What are the likely mechanisms o aminoglycoside resistance?
See Side Bar MECHANISMS OF AMINOGLYCOSIDE RESIS ANCE. Clinically,
drug inactivation is the most common mechanism or acquired microbial resis-
tance to aminoglycosides. T e genes encoding aminoglycoside-modi ying enzymes
are acquired primarily by conjugation and trans er o resistance plasmids. T ese
enzymes phosphorylate, adenylate, or acetylate speci c hydroxyl or amino groups.
Other mechanisms o aminoglycoside resistance include the inability o the ami-
noglycoside to penetrate bacterial intracellular space and a reduced a nity o the
aminoglycoside or the bacterial ribosome.

CASE 40-3
A 65-year-old man is in hospital or the treatment o community-acquired pneumonia
that is sensitive to an aminoglycoside. He is being treated with 5.1 mg/kg o gentamicin
as a single dose once every 24 hours.
a. Based on a hal -li e o 2 to 3 hours, aminoglycosides have been historically
administered in 3 equally divided doses over a 24-hour period. Is this patients
single dose expected to be e ective?
Numerous studies and meta-analyses demonstrate that administration o the total
dose o aminoglycoside once daily is just as e ective as multiple-dose regimens. In
addition, extended-interval administration costs less, is administered more easily,
and is associated with less nephrotoxicity.
(Continued)
560
Aminoglycosides CHAPTER 4 0

A comparison o high-dose, extended-interval dosing to traditional divided-dose


methods is illustrated in Figure 40-2. Because o the postantibiotic e ect o amino-
glycosides, good therapeutic response can be attained even when the concentrations
o aminoglycosides all below bacterial inhibitory concentrations or a substantial
period o the dosing interval.
b. How does the once-daily dosing a ect aminoglycoside toxicity?
T e high-dose, extended-interval dosing methods or aminoglycosides reduce the
characteristic oto- and nephrotoxicity o these drugs. High-dose, extended-interval
dosing regimens, despite higher peak concentrations, provide a longer period when
concentrations all below the threshold or toxicity than does a multiple-dose
regimen (12 hours vs less than 3 hours total in the example shown in Figure 40-2),
potentially accounting or the lower toxicity o this approach. T is diminished tox-
icity is probably due to a threshold e ect rom accumulation o drug in the inner
ear or in the kidney (see Cases 40-5 and 40-6).
c. Is this once-a-day dosing o aminoglycosides suitable or all patients?
One key exception to the use o extended-interval dosing is or aminoglycoside use as
a combination therapy with a cell wall-active agent in the treatment o gram-positive
in ections, such as endocarditis.
Although schemes exist or adjusting the dosages o aminoglycosides dosed by
extended-interval methods in patients with renal dys unction, some clinicians
pre er to use the traditional multiple-dose regimens in such patients.
d. Af er administration o an aminoglycoside, how should the plasma concentration
be monitored?
For multiple-daily-dosing regimens, both peak and trough plasma concentrations are
determined. T e trough sample is obtained just be ore a dose, and the peak sample is
obtained 60 minutes a er an intramuscular injection or 30 minutes a er an intrave-
nous in usion given over 30 minutes. T e peak concentration documents that the dose
produces therapeutic concentrations, generally accepted to be 4 to 10 g/mL or gen-
tamicin, netilmicin, and tobramycin, and 15 to 30 g/mL or amikacin and streptomy-
cin. T e trough concentration is used to avoid toxicity by monitoring or accumulation
o the drug. rough concentrations should be less than 1 to 2 g/mL or gentamicin,
netilmicin, and tobramycin and less than 10 g/mL or amikacin and streptomycin.
Monitoring o aminoglycoside plasma concentrations also is important when
using an extended-interval dosing regimen, although peak concentrations are not
(Continued)
35
30 eve ry 8h
eve ry 24h
25
Thre s hold
20
L
m
/
15
g

10
5
0
0 2 4 6 8 10 12 14 16 18 20 22 24
Hours

FIGURE 40-2 Plasma concentrations (g/mL) a ter administration o 5.1 mg/kg o gentamicin intravenously to a
hypothetical patient either as a single dose (every 24 hours) or as 3 divided doses (every 8 hours). The threshold or
toxicity has been chosen to correspond to a plasma concentration o 2 g/mL, the maximum recommended. The
high-dose, extended-interval (once-daily) regimen produces a 3- old higher plasma concentration, which enhances
ef cacy that otherwise might be compromised due to prolonged sub-MIC concentrations later in the dosing inter-
val compared with the every-8-hours regimen. The once-daily regimen provides a 12-hour period during which
plasma concentrations are below the threshold or toxicity, thereby minimizing the toxicity that otherwise might
result rom the high plasma concentrations early on. The every-8-hours regimen, in contrast, provides only a brie
period during which plasma concentrations are below the threshold or toxicity.

561
SECTION VII Chemotherapy of Microbial Diseases

determined routinely (these will be 3-4 times higher than the peak achieved with a
multiple-daily-dosing regimen).
T e most accurate method or monitoring plasma concentrations or dose adjust-
ment during an extended-interval dosing regimen is to measure the concentration
o 2 plasma samples drawn several hours apart (eg, at 2 and 12 hours a er a dose).
T e clearance can then be calculated and the dose adjusted to achieve the desired
target range.

CASE 40-4
A 46-year-old man with chronic renal insu ciency is brought into the emergency depart-
ment su ering rom an elevated temperature o 104 F. His medical chart is not available.
It is decided to treat him with gentamicin, but his last creatinine clearance is not known.
a. T e dose o aminoglycoside should be decreased in patients with poor renal
unction. Since this patients creatinine clearance is unknown, what initial dose
should he be started on?
T e concentration o aminoglycoside in plasma produced by the initial dose depends
only on the volume o distribution o the drug. T us (assuming his volume o distri-
bution is normal), this patient can be treated initially with a standard dose o gen-
tamicin, or example, 5 mg/kg given over 30 to 60 minutes. T e next dose (24 hours
later) can be adjusted or his reduced renal unction which should then be known.
b. Why is it important to reduce the dose o aminoglycosides in patients with
decreased renal unction?
T e elimination o aminoglycosides depends almost entirely on the kidney and a
linear relationship exists between the concentration o creatinine in the plasma and
the t o all aminoglycosides in patients with moderately compromised renal unc-
tion. In anephric patients, the t varies rom 20 to 40 times that o normal individu-
als. Because the incidence o nephrotoxicity and ototoxicity is likely related to the
overall drug exposure to aminoglycosides, it is critical to reduce the maintenance
dosage o these drugs in patients with impaired renal unction.
c. How is a maintenance dose determined?
For patients with impaired renal unction, it is common to rely upon algorithms
such as the one shown in able 54-2 in the 12th Edition o T e Pharmacologi-
cal Basis of T erapeutics or those published in other medical texts and re erence
books. It should be understood that such algorithms are only a guideline and that
plasma concentrations must be monitored as discussed in Case 40-3 above and in
Appendix II o the T e Pharmacological Basis of T erapeutics, 12th Edition. Deter-
mination o the concentration o drug in plasma is an essential guide to the proper
administration o aminoglycosides. In patients with li e-threatening systemic in ec-
tions, aminoglycoside concentrations should be determined several times per week
(more requently i renal unction is changing) and should be determined within
24 to 48 hours o any change in dosage.

CASE 40-5
During a course o gentamicin therapy, a 76-year-old man develops ringing in his ears.
a. What are the clinical symptoms o ototoxicity with aminoglycosides?
A high-pitched tinnitus o en is the rst sign o cochlear toxicity. I the drug is not
stopped, auditory impairment may develop a er a ew days. T e a ected individual
is not always aware o the di culty, and it will not be detected except by care ul
auditory examination.
Moderately intense headache lasting 1 to 2 days may precede the onset o vestibu-
lar toxicity. T is is ollowed immediately by nausea and vomiting. Di culty with
equilibrium may develop and persist or 1 to 2 weeks. Prominent symptoms o
(Continued)
562
Aminoglycosides CHAPTER 4 0

vestibular toxicity include vertigo in the upright position, inability to perceive ter-
mination o movement, and di culty in sitting or standing without visual clues.
b. What are the risk actors or producing ototoxicity?
It is generally thought the ototoxicity seen with aminoglycosides is more likely to
occur in patients with persistently elevated concentration o drug in plasma.
Ototoxicity is largely irreversible and results rom progressive destruction o vestib-
ular or cochlear sensory cells. Accumulation within the perilymph and endolymph
occurs predominantly when aminoglycoside concentrations in plasma are high.
Di usion back into the bloodstream is slow; the hal -lives o the aminoglycosides
are 5 to 6 times longer in otic f uids than in plasma. Back di usion is concentra-
tion-dependent and acilitated at the trough concentration o the drug in plasma.
However, studies have not consistently shown an association between ototoxicity
and risk actors such as aminoglycoside plasma concentrations, total dose, duration
o exposure, and renal dys unction.
c. How should aminoglycosides be monitored to avoid ototoxicity?
I it is anticipated that the patient will be treated with an aminoglycoside or more
than 3 to 4 days then plasma concentrations should be monitored to avoid drug
accumulation. See the answer to Case 40-3d or a description o a rational approach
to aminoglycoside plasma concentration monitoring.

CASE 40-6
A 63-year-old woman has been treated with gentamicin or 7 days. During these
7 days, her serum creatinine has risen rom 1.0 mg/dL to 2.2 mg/dL and there is con-
cern or the development o nephrotoxicity. She is scheduled or another 3 days o
gentamicin therapy.
a. What variables are important or the development o aminoglycoside
nephrotoxicity?
Approximately 8 to 26% o patients who receive an aminoglycoside or several days
develop mild renal impairment that is almost always reversible. T e toxicity cor-
relates with the total amount o drug administered and consequently with longer
courses o therapy. Advanced age, liver disease, diabetes, and septic shock have
been suggested as risk actors or the development o aminoglycoside nephrotoxic-
ity. T e nephrotoxic potential varies among individual aminoglycosides. Neomycin
concentrates to the greatest degree in the renal cortex, is highly nephrotoxic, and
should not be administered systemically. Streptomycin does not concentrate in
the renal cortex and is the least nephrotoxic. T e di erence in the nephrotoxic-
ity o gentamicin and tobramycin is slight. Other drugs such as amphotericin B,
cyclosporin, vancomycin, angiotensin-converting enzyme inhibitors, and cisplatin
may potentiate the nephrotoxicity o the aminoglycosides. Clinical studies have not
proven conclusively that urosemide potentiates aminoglycoside nephrotoxicity, but
the volume depletion and K+ wasting that accompany the use o urosemide may
predispose to aminoglycoside toxicity.
b. What are the mechanisms o aminoglycoside nephrotoxicity?
Aminoglycoside nephrotoxicity results rom the accumulation and retention o the
aminoglycoside in the renal proximal tubular cells. T e biochemical events leading
to tubular cell damage and glomerular dys unction probably involve perturba-
tions o the structure o cellular membranes. T e initial mani estation o damage
is excretion o enzymes o the renal tubular brush border. A er several days, there
is a de ect in the renal concentrating ability, mild proteinuria, and the appearance
o hyaline and granular casts. T e glomerular ltration rate is reduced a er sev-
eral additional days. Aminoglycoside renal impairment is almost always reversible
because the proximal tubular cells have the capacity to regenerate.

(Continued)
563
SECTION VII Chemotherapy of Microbial Diseases

c. What methods are available or monitoring aminoglycoside therapy to avoid or


manage nephrotoxicity?
Monitoring plasma trough concentrations, as described in Case 40-3, are used to
avoid toxicity by measuring drug accumulation. rough concentrations should be
less than 1 to 2 g/mL or gentamicin, netilmicin, and tobramycin and less than
10 g/mL or amikacin and streptomycin.

KEY CONCEPTS
Aminoglycosides are bactericidal by disrupting the normal cycle o ribosomal
unctioning thereby inter ering with bacterial protein synthesis.
Resistance to aminoglycosides is most commonly caused by drug-inactivating
enzymes produced by bacteria.
Aminoglycosides are characterized by a postantibiotic e ect, that is, residual
bactericidal activity persisting a er the serum concentration has allen below
the minimum inhibitory concentration (MIC).
High-dose, extended-interval dosing regimens are e ective in most instances
and may reduce the characteristic oto- and nephrotoxicity o aminoglycosides.
Plasma concentration monitoring o aminoglycosides is an essential guide to
the proper administration o aminoglycosides.
All aminoglycosides have the potential to produce irreversible cochlear and
vestibular toxicity.
All aminoglycosides have the potential to produce nephrotoxicity that is almost
always reversible.

SUMMARY QUIZ

QUESTION 40-1 A 36-year-old woman is being treated with gentamicin in hospital or


a gram-negative in ection. Her trough blood concentrations are below the known min-
imum inhibitory concentration (MIC) o the bacteria. An in ectious disease consultant
is not concerned about this because
a. the MIC or most bacteria are notoriously too high.
b. gentamicin plasma concentrations are not an e ective way to monitor therapy.
c. gentamicin has residual bacteriocidal activity which persists a er the plasma con-
centration has allen below the MIC.
d. gentamicin is bacteriostatic.
e. the mechanism o action o gentamicin is inhibition o cell wall synthesis.

QUESTION 40-2 A 64-year-old man is su ering rom a gas gangrene in ection o his right
oot due to a mid-cal arterial clot. T e bacteria involved is Clostridium perfringens. An
aminoglycoside is not a good choice or antibiotic therapy because aminoglycosides are
a. bacteriostatic.
b. not active against anaerobic bacteria.
c. more toxic under anaerobic conditions.
d. only e ective i they are injected directly into anaerobic tissue.
e. metabolized more rapidly under anaerobic conditions.

QUESTION 40-3 A 26-year-old woman has the diagnosis o bacterial meningitis. T e


organism cultured is a gram-negative Pseudomonas susceptible to gentamicin. T ere is
concern about treatment o this patient with gentamicin because
a. aminoglycosides are not generally active against gram-negative bacteria.
b. aminoglycosides can cause seizures at therapeutic concentrations.
(Continued)
564
Aminoglycosides CHAPTER 4 0

c. patients with meningitis have a tendency to be more susceptible to the nephrotoxic


e ects o the aminoglycosides.
d. aminoglycosides are polar and do not penetrate the central nervous system.
e. aminoglycosides are rapidly metabolized in the brain and do not reach therapeutic
concentrations.

QUESTION 40-4 A 40-year-old woman is being treated with intravenous gentamicin 3 times
daily. Her therapy was begun 3 days ago and is expected to continue or an additional
10 days. What is the recommended monitoring o plasma gentamicin concentrations?
a. No plasma concentration monitoring is necessary.
b. Measure peak and trough concentrations several times per week.
c. Measure only peak plasma concentrations.
d. Measure only trough concentration once a er 3 days o therapy.
e. Monitor peak and trough plasma concentrations only i renal dys unction is
apparent.

QUESTION 40-5 A 43-year-old man is being treated with gentamicin and penicillin
mixed in the same IV solution or a serious pulmonary in ection that is sensitive to
gentamicin. A er 3 days o therapy, his temperature remains elevated and his pulmo-
nary congestion is worsening. A possible cause is that the
a. gentamicin is inactivated by the penicillin in the IV mixture in vitro.
b. penicillin enhances the elimination o gentamicin.
c. gentamicin causes a destruction o penicillin in vivo.
d. penicillin inhibits the entry o gentamicin into the bacterial cell.
e. penicillin prevents the ribosomal binding o gentamicin.

QUESTION 40-6 A 23-year-old woman has developed a pulmonary in ection o


unknown cause. Patients in the same hospital have recently developed in ections with
organisms shown to be resistant to gentamicin and tobramycin. An in ectious disease
consultant recommends initially starting the patient on amikacin along with another
antibiotic that is known to inhibit cell wall synthesis. T e choice o amikacin is made
because it is
a. less expensive than gentamicin or tobramycin.
b. less nephrotoxic than gentamicin or tobramycin.
c. less ototoxic than gentamicin or tobramycin.
d. more concentrated in pulmonary tissue than gentamicin or tobramycin.
e. resistant to many o the aminoglycoside-inactivating enzymes.

SUMMARY QUIZ ANSWER KEY

QUESTION 40-1 Answer is c. A postantibiotic e ect, that is, residual bacteri-


cidal activity persisting a er the serum concentration has allen below the MIC,
is characteristic o the aminoglycoside antibiotics; the duration o this e ect is
concentration-dependent.

QUESTION 40-2 Answer is b. Aminoglycosides must penetrate intracellularly to be


e ective. ransport o aminoglycosides across the cytoplasmic membrane is an oxygen-
dependent active process. T us, anaerobic bacteria are resistant to the aminoglycosides
because they lack the necessary transport system.

QUESTION 40-3 Answer is d. Because o their polar nature, the aminoglycosides do


not penetrate into most cells, the CNS, or the eye. Concentrations o aminoglycosides
achieved in the CSF with parenteral administration usually are subtherapeutic. I CNS
therapy with an aminoglycoside is necessary, a preservative- ree ormulation o an
(Continued)
565
SECTION VII Chemotherapy of Microbial Diseases

aminoglycoside is administered directly intrathecally or intraventricularly once daily.


Such treatment may cause local in ammation and can result in radiculitis and other
complications.

QUESTION 40-4 Answer is b. Determination o the concentration o gentamicin in


plasma is an essential guide to its proper administration. In patients with li e-threatening
in ections or which gentamicin may be administered more than or a ew days, it is
recommended that plasma concentrations be determined several times per week or
more requently i renal unction is changing. Plasma aminoglycoside concentrations
should be determined within 24 to 48 hours o a change in dosage.

Peak plasma concentrations o aminoglycosides are used to determine that the dose
produces a therapeutic concentration; trough plasma concentrations are used to moni-
tor drug accumulation. Peak concentrations are not routinely measured during an
extended-interval dosing regimen.

QUESTION 40-5 Answer is a. Aminoglycosides can be inactivated by various penicil-


lins in vitro and thus should not be admixed in solution.

QUESTION 40-6 Answer is e. Amikacin is resistant to many o the aminoglycoside-


inactivating enzymes. It is the pre erred agent or the initial therapy o serious noso-
comial gram-negative in ections in hospitals where resistance to gentamicin and
tobramycin has become a signif cant problem.

SUMMARY: AMINOGLYCOSIDES
TOXICITY
UNIQUE;
CLASS AND CLINICALLY
SUBCLASSES NAME CLINICAL USES RESISTANCE COMMON IMPORTANT

Aminoglycosides Gentamicin Urinary tract in ection: not indicated or Clinical bacterial Nephrotoxicity
routine use resistance is increasingly (reversible),
Pneumonia: in combination with a common irreversible
-lactam antibiotic or the treatment o Aminoglycosides have ototoxicity
hospital-acquired pneumonia in which little activity against When applied
a multiple drug-resistant gram-negative anaerobic bacteria or topically to
aerobe is a likely causative agent acultative bacteria large areas
Bacterial endocarditis: low-dose under aerobic conditions o denuded
gentamicin (3 mg/kg/d in divided Aminoglycoside activity skin, plasma
doses) in combination against most gram- concentrations
with penicillin or vancomycin or positive bacteria is may reach the
treatment due to gram-positive limited and they should toxic range
organisms; in cases o enterococcal not be used as a single
endocarditis, the administration o agent in the treatment
penicillin and gentamicin or 4-6 weeks o in ections caused by
is recommended because o the gram-positive bacteria
unacceptably high relapse rate
with penicillin alone
Sepsis: inclusion o an aminoglycoside
is recommended or the ebrile patient
with granulocytopenia and when
P. aeruginosa is a potential pathogen
Tobramycin Similar to gentamicin Similar to gentamicin Similar to
gentamicin
(Continued)
566
Aminoglycosides CHAPTER 4 0

TOXICITY
UNIQUE;
CLASS AND CLINICALLY
SUBCLASSES NAME CLINICAL USES RESISTANCE COMMON IMPORTANT

Amikacin Broader spectrum than gentamicin Because o its Similar to


Amikacin is less active than gentamicin resistance to many o gentamicin
against enterococci and should not be the aminoglycoside-
used or this organism inactivating enzymes,
Amikacin is active against it is recommended or
M. tuberculosis, including streptomycin- the initial treatment
resistant strains and atypical o nosocomial gram-
mycobacteria negative in ections
in hospitals where
resistance to gentamicin
and tobramycin has
become a signi cant
problem
Netilmicin Its antibacterial activity is broad against It is not metabolized Similar to
aerobic gram-negative bacilli by most o the gentamicin
aminoglycoside-
inactivating enzymes
and may be active
against certain bacteria
that are resistant to
gentamicin
Streptomycin Rarely used because it is less active Gentamicin-resistant Ototoxicity o
against gram-negative aerobes than strains o enterococci streptomycin
other aminoglycosides may be susceptible to is primarily
Streptomycin (or gentamicin) is the streptomycin vestibular and
drug o choice or the treatment irreversible
o tularemia
Kanamycin Its spectrum o activity is limited Similar to
compared with other aminoglycosides; gentamicin
the use o kanamycin has become
obsolete
Neomycin Available or topical administration in Hypersensitivity Similar to
combination with polymyxin, bacitracin, reactions gentamicin
other antibiotics and corticosteroids are reported
Neomycin and polymyxin B have and involve
been used or irrigation o the primarily skin
urinary bladder rashes
when the
drug is
applied
topically

567
CHAPTER

41 Protein Synthesis Inhibitors and


Miscellaneous Antibacterial Agents
T is chapter will be most use ul a er having a basic understanding o the material
DRUGS INCLUDED
in Chapter 55, Protein Synthesis Inhibitors and Miscellaneous Antibacterial Agents in
IN THIS CHAPTER Goodman & Gilmans T e Pharmacological Basis o T erapeutics, 12th Edition. In addition
Azithromycin (ZITHROMAX) to the material presented here Chapter 55 contains:
Bacitracin able 55-2 which provides in ormation about the activity o most o the antibacterial
Chloramphenicol agents covered in this chapter against key gram-positive pathogens
Clarithromycin (BIAXIN, others)
LEARNING OBJECTIVES
Clindamycin (CLEOCINPHOSPHATE, others)
Understand the mechanisms o action and resistance o tetracyclines, macro-
Daptomycin (CUBICIN) lides, vancomycin, linezolid, daptomycin, and quinupristin/dal opristin
Doxycycline Describe the unique toxicities o antibiotics that are inhibitors o bacterial
Erythromycin protein synthesis
Linezolid (ZYVOX) Describe the uses and untoward reactions o vancomycin
Mupirocin (BACTROBAN) Identi y the drugdrug interactions that occur with some o these antibiotics
Polymyxin Understand how linezolid, daptomycin, and quinupristin/dal opristin are used
Quinupristin/dalfopristin (SYNERCID) to treat methicillin-resistant and vancomycin-resistant organisms
Spectinomycin
Teicoplanin
Telithromycin (KETEX)
MECHANISMS OF ACTION AND RESISTANCE OF PROTEIN SYNTHESIS INHIBITORS
Tetracycline
DRUGS MECHANISM OF ACTION MECHANISM OF RESISTANCE
Tigecycline
Tetracycline, Doxycycline, Inhibition o protein synthesis Decreased bacterial cell
Vancomycin (VANCOCIN) Tigecycline by binding to the 30S accumulation o tetracycline
bacterial ribosome subunit Bacterial production o a ribosomal
(see Figure 41-1) protection protein that displaces
drug rom its target
Enzymatic destruction o drug

Chloramphenicol Inhibition o protein synthesis Bacterial production o


by binding to the 50S acetyltrans erase that inactivates
bacterial ribosome subunit the drug
(see Figure 41-2)

Erythromycin, Inhibition o protein synthesis Enhanced e ux rom bacterial cell


Clarithromycin, by binding to the 50S Ribosomal protection by
Telithromycin, bacterial ribosome subunit production o methylase enzymes
Azithromycin (see Figure 41-3) Hydrolysis by esterases
Mutation o 50S ribosomal protein

Clindamycin Inhibition o protein synthesis Ribosomal protection by


by binding to the 50S constitutively produced methylase
bacterial ribosome subunit enzyme
Altered metabolism

Quinupristin/Dal opristin Inhibition o protein synthesis Production o methylase enzyme


by binding to the 50S that prevents binding to target
bacterial ribosome subunit Production o acetyltrans erase that
inactivates the drug
Enhanced e ux rom bacterial cell

568
Protein Synthesis Inhibitors and Miscellaneous Antibacterial Agents CHAPTER 4 1

DRUGS MECHANISM OF ACTION MECHANISM OF RESISTANCE

Linezolid Inhibition o protein synthesis Point mutations in rRNA


by binding to the 50S
bacterial ribosome subunit to
prevent ormation o larger
ribosomal complex

Spectinomycin Inhibition o protein synthesis Mutations in rRNA


by binding to the 30S Modi cations o the drug by
bacterial ribosome subunit adenyltrans erase

Polymyxin As sur ace-active agents, Resistance is rare


polymyxins interact with
phospholipids and disrupt
bacterial cell wall

Vancomycin, Teicoplanin Inhibit cell wall synthesis by Bacterial alteration o drug target
binding to d -alanyl-d -alanine
terminus o cell wall precursor
units (see Figure 41-4)

Daptomycin Binds to bacterial membranes Not ully characterized


resulting in depolarization,
loss o membrane potential
and cell death

Bacitracin Inhibits synthesis o bacterial Pseudomonas, Candida spp.


cell wall and Nocardia are resistant but
mechanism is not known

Mupirocin Inhibition o protein synthesis Production o a bypasssynthetase


by binding to isoleucyl that binds mupirocin poorly
trans er-RNA synthetase

Chlora mphe nicol


P s ite A s ite
P s ite A s ite
50S aa

Aminoa cyl
Na s ce nt
tRNA
polype ptide Na s ce nt 50S
cha in polype ptide
Tra ns fe ra s e cha in
s ite aa Tra ns fe ra s e
Te tra cycline s ite

mRNA
te mpla te mRNA
te mpla te
30S
30S
FIGURE 41-1 Inhibition o bacterial protein synthesis by tetra-
cyclines. Messenger RNA (mRNA) attaches to the 30S subunit o FIGURE 41-2 Inhibition o bacterial protein synthesis by
bacterial ribosomal RNA. The P (peptidyl) site o the 50S ribosomal chloramphenicol. Chloramphenicol binds to the 50S ribosomal
RNA subunit contains the nascent polypeptide chain; normally, the subunit at the peptidyltrans erase site and inhibits the trans-
aminoacyl tRNA charged with the next amino acid (aa) to be added peptidation reaction. Chloramphenicol binds to the 50S ribo-
to the chain moves into the A (acceptor) site, with complementary somal subunit near the site o action o clindamycin and the
base pairing between the anticodon sequence o tRNA and the macrolide antibiotics. These agents inter ere with the binding
codon sequence o mRNA. Tetracyclines inhibit bacterial protein o chloramphenicol and thus may inter ere with each others
synthesis by binding to the 30S subunit and blocking tRNA binding actions i given concurrently. See Figure 41-1 and its legend or
to the A site. additional in ormation.

569
SECTION VII Chemotherapy of Microbial Diseases

A s ite
Na s ce nt
polype ptide
cha in 50S

P s ite

aa Tra ns fe ra s e
s ite
Ma crolide s

mRNA
te mpla te
tRNA
30S

FIGURE 41-3 Inhibition o bacterial protein synthesis by the macrolide antibiotics erythromycin,
clarithromycin, and azithromycin. Macrolide antibiotics are bacteriostatic agents that inhibit pro-
tein synthesis by binding reversibly to the 50S ribosomal subunits o sensitive organisms. Erythro-
mycin appears to inhibit the translocation step such that the nascent peptide chain temporarily
residing at the A site o the trans erase reaction ails to move to the P, or donor, site. Alternatively,
macrolides may bind and cause a con ormational change that terminates protein synthesis by
indirectly inter ering with transpeptidation and translocation. See Figure 41-1 and its legend or
additional in ormation.

A. Polyme riza tion


(NAMNAG)n LCP NAMNAG (NAMNAG)n NAMNAG

Va ncomycin

+
tra ns glycosyla s e s
LCP

B. Cros s linking
(NAMNAG)n
(NAMNAG)n (NAMNAG)n (NAMNAG)n
-La cta m
a ge nts

tra ns pe ptida s e s

KEY

LAla nine NAM = NAce tylmura mic a cid


DGluta ma te NAG = NAce tylglucos a mine
LLys ine LCP = Lipid ca rrie r ba ctopre nol
DAla nine ce ll wa ll
Glycine

FIGURE 41-4 Inhibition o bacterial cell wall synthesis: vancomycin and -lactam agents. Vancomycin inhibits the
polymerization or transglycosylase reaction (A) by binding to the d -alanyl-d -alanine terminus o the cell wall precur-
sor unit attached to its lipid carrier and blocks linkage to the glycopeptide polymer (indicated by the subscript n).
These (NAMNAG)n peptidoglycan polymers are located within the cell wall. Van A-type resistance is due to
expression o enzymes that modi y cell wall precursor by substituting a terminal d -lactate or d -alanine, reducing
vancomycin binding af nity by 1000 times. -Lactam antibiotics inhibit the cross-linking or transpeptidase reaction
(B) that links glycopeptide polymer chains by ormation o a cross-bridge with the stem peptide (the 5 glycines in
this example) o 1 chain, displacing the terminal d -alanine o an adjacent chain. See also Figure 39-1.

570
Protein Synthesis Inhibitors and Miscellaneous Antibacterial Agents CHAPTER 4 1

CASE 41 1
A 56-year-old man has developed a community-acquired skin in ection that is shown
to be methicillin-resistant S. aureus. An in ectious disease consultant recommends ini-
tial treatment with doxycycline.
a. Why doxycycline?
etracyclines, oxycycline, an minocycline have generally retaine excellent lev-
els o activity against staphylococci, inclu ing methicillin-resistant Staphylococcus
aureus (MRSA). Community-acquire strains o MRSA o en are susceptible to
tetracycline, oxycycline, or minocycline.
b. What is the mechanism o action o doxycycline?
etracyclines an glycylcyclines inhibit bacterial protein synthesis by bin ing to the
30S bacterial ribosome an preventing access o aminoacyl tRNA to the acceptor
site on the mRNA-ribosome complex (see Figure 41-1).
c. Another consultant has recommended chloramphenicol. Is this a good choice?
Chloramphenicol use shoul be limite to in ections or which the bene ts o the
rug outweigh the risks o the potential toxicities. In this case, the bene ts o not
outweigh the risks.
Chloramphenicol is potentially toxic to the bone marrow in two ways: (1) a ose-
relate toxicity that presents as anemia, leukopenia, or thrombocytopenia, an
(2) an i iosyncratic response mani este by aplastic anemia. T is latter response
may be atal i the bone marrow aplasia is complete. T e risk o aplastic anemia
oes not contrain icate the use o chloramphenicol in situations in which it may
be li esaving. However, the rug shoul never be use in un e ne situations
or in iseases that are rea ily, sa ely, an e ectively treate with other less toxic
antibiotics.

CASE 41 2
A 23-year-old woman college student is given a prescription or clarithromycin or
a Haemophilus in uenzae in ection.
a. What is the mechanism o action o clarithromycin and how does it dif er rom
that o tetracycline and chloramphenicol?
Clarithromycin acts by bin ing to the bacterial 50S ribosome (see Figure 41-3) to
inhibit bacterial protein synthesis. T is is the same site o action or chloramphenicol
an clin amycin an these agents (macroli es, clin amycin, an chloramphenicol)
may inter ere with each others actions i given concurrently. etracycline bin s to
the 30S subunit o the bacterial ribosome.
b. What atypical in ectious disease is commonly treated with clarithromycin?
Clarithromycin is use in combination with omeprazole (or other proton pump
inhibitor) an amoxicillin or the treatment o peptic ulcer isease cause by
H. pylori (see Chapter 32).
c. What toxicities should this patient be warned against?
Macroli es commonly cause GI upset which may inclu e nausea, vomiting, an
ab ominal pain. Erythromycin, clarithromycin, an telithromycin have been
reporte to cause car iac arrhythmias, inclu ing Q prolongation an ventricu-
lar tachycar ia. Most patients who experience car iac toxicity have un erlying
risk actors, such as here itary long Q syn rome, hypokalemia or hypomagne-
semia, pro oun bra ycar ia, or are receiving certain antiarrhythmic rugs (eg,
quini ine, procainami e, amio arone) or other agents that prolong Q c (see
Chapter 18).
(Continued)

571
SECTION VII Chemotherapy of Microbial Diseases

d. What serious drug-interactions should this patient be advised o ?


Macroli e antibiotics inhibit CYP3A4 an are associate with signi cant rug inter-
actions such as potentiation o the e ects o rugs metabolize by the same pathway.
In ucers o CYP3A4 such as ri ampin may ecrease the serum concentration o the
macroli e antibiotics. Inhibitors o CYP3A4, such as itraconazole, may increase the
serum concentrations o the macroli e antibiotics. Azithromycin appears to be ree
o these rug interactions, but caution shoul be use when using azithromycin in
conjunction with rugs known to interact with other macroli e antibiotics.

CASE 41 3
A 34-year-old woman in hospital has developed an E. aecium in ection that is resistant
to vancomycin. An in ectious disease consultant has recommended she be treated with
quinupristin/dal opristin.
a. Why is quinupristin given in combination with dal opristin?
Quinupristin bin s at the same site (on the 50S bacterial ribosome) as the macro-
li es an has a similar e ect. Dal opristin bin s at a site nearby, resulting in a
con ormational change in the 50S ribosome, synergistically enhancing the bin ing
o quinupristin at its target site.
b. What are the hazards o using this combination?
Quinupristin/ al opristin inhibits CYP3A4. T e concomitant a ministration o
other CYP3A4 substrates may result in signi cant toxicity. Appropriate caution an
monitoring are recommen e or rugs in which the toxic/therapeutic win ow is
narrow or or rugs that prolong the Q c interval.
c. Linezolid is another drug that might be use ul in treating this patient. Why
might this drug be an ef ective alternative?
Linezoli is FDA approve or the treatment o in ections cause by vancomycin-
resistant E. aecium. However, this agent is associate with some signi icant
a verse e ects.
d. What are the hazards o linezolid therapy?
Patients receiving long-term therapy with linezoli have evelope peripheral neu-
ropathy, optic neuritis, an lactic aci osis. T ese e ects are not always reversible.
Linezoli is a weak nonspeci c inhibitor o monoamine oxi ase an patients
receiving concomitant therapy with an SSRI may evelop serotonin syn rome.

CASE 41 4
A 62-year-old man has developed a so tissue in ection ollowing an auto accident.
Cultures are pending.
a. Is empirical treatment with vancomycin warranted?
Vancomycin is requently use in the empirical an e nitive treatment o skin/
so -tissue an bone/joint in ections, where gram-positive organisms inclu ing
MRSA are the lea ing pathogens. Vancomycin an teicoplanin have been use to
treat a wi e variety o in ections, inclu ing osteomyelitis an en ocar itis, cause
by methicillin-resistant an methicillin-susceptible staphylococci, streptococci, an
enterococci.
b. What is the mechanism o action o vancomycin
Vancomycin inhibits the polymerization or transglycosylase reaction by bin ing to
the d-alanyl-d-alanine terminus o the cell wall precursor unit attache to its lipi
carrier thus blocking linkage to the glycoprotein polymer (see Figure 41-4).
(Continued)

572
Protein Synthesis Inhibitors and Miscellaneous Antibacterial Agents CHAPTER 4 1

c. What are the concerns about starting vancomycin empirically?


A major concern or using vancomycin empirically is the evelopment o
resistance. Glycopepti e-resistant strains o enterococci have emerge as major
nosocomial pathogens in hospitals in the Unite States. Resistance to vanco-
mycin is ue to the expression o enzymes that mo i y cell wall precursors by
substituting a terminal d -lactate or d -alanine re ucing vancomycin bin ing by
1000 times.
d. What are the hazards o intravenous vancomycin?
Rapi intravenous in usion o vancomycin may cause erythematous or urticarial
reactions, f ushing, tachycar ia, an hypotension. Extreme f ushing is re erre to as
re man syn rome. T is e ect is ue to irect toxic e ect o vancomycin on mast
cells, causing them to release histamine.

CASE 41 5
Cultures rom the patient in Case 41-4 are methicillin-resistant S. aureus that is also resistant
to vancomycin. T e decision has been made to switch the patient to daptomycin.
a. Why daptomycin?
Daptomycin may be active against vancomycin-resistant strains, although minimum
inhibitory concentrations (MICs) ten to be higher or these organisms than or
their vancomycin-susceptible counterparts.
b. What is the mechanism o action o daptomycin?
Daptomycin is an ionophore that bin s to bacterial membranes resulting in
epolarization, loss o membrane potential, an cell eath.

CASE 41 6
A 6-year-old boy has allen and scraped his knee. His mother goes to the pharmacy
to obtain an antibiotic ointment. T e pharmacist recommends an over-the-counter
preparation containing polymyxin, bacitracin, and neosporin.
a. Why is this combination o drugs available over-the-counter?
Polymyxin an bacitracin are available or the topical treatment o minor skin
in ections. Neosporin is an aminoglycosi e that is also available in topical prepa-
rations. Pro ucts that contain these antibiotics or otic or ophthalmic use are not
available over-the-counter.
b. What are the mechanisms o actions o the miscellaneous antibiotic agents
bacitracin, polymyxin, and mupirocin?
Polymyxins are sur ace-active amphipathic agents that interact with phospholipi s
an isrupt the structure o cell membranes. Bacitracin is an inhibitor o bacterial
cell wall synthesis. Mupirocin inhibits bacterial protein synthesis by reversibly
bin ing an inhibiting isoleucyl trans er-RNA synthetase.

KEY CONCEPTS
Doxycycline is a drug o choice or sexually transmitted diseases, rickettsial
in ections, plague, brucellosis, tularemia, and spirochetal in ections.
Macrolide antibiotics are e ective or treatment o respiratory tract in ections
caused by the common pathogens o community-acquired pneumonia,
including S. pneumoniae, Haemophilus spp., Chlamydia, mycoplasma, and
Legionella.
(Continued)

573
SECTION VII Chemotherapy of Microbial Diseases

Macrolides, except azithromycin, have important drug interactions because


they inhibit hepatic CYPs.
Chloramphenicol is rarely indicated because it can cause irreversible bone
marrow toxicity.
Vancomycin, daptomycin, quinupristin/dal opristin, and linezolid are indi-
cated or the treatment o gram-positive in ections caused by drug-resistant
organisms.
Quinupristin/dal opristin and linezolid are indicated or the treatment o
vancomycin-resistant E. aecium in ections.
Quinupristin/dal opristin, linezolid, and daptomycin are active against
vancomycin-resistant strains o S. aureus.
Polymyxin, bacitracin, and mupirocin are e ective or the topical treatment
o minor skin in ections.

SUMMARY QUIZ

QUESTION 41-1 A-47-year-old man is receiving intravenous vancomycin. He develops


ushing, tachycardia, and hypotension. T e ushing, caused by the rapid in usion o
vancomycin (re erred to as red man syndrome), is caused by
a. testosterone released by sertoli cells.
b. glucagon released by hepatic cells.
c. histamine released by mast cells.
d. dopamine released by endothelial cells.
e. growth hormone released by pituitary cells.

QUESTION 41-2 A 25-year-old woman is receiving tetracycline or the treatment o


acne. She should be warned against the concurrent ingestion o
a. milk.
b. ca eine.
c. chocolate.
d. green lea y vegetables.
e. water.

QUESTION 41-3 T e patient in Question 41-2 should also be warned o


a. discoloration o her hair.
b. discoloration o her urine.
c. an excessive growth o f nger and toe nails.
d. night blindness.
e. photosensitivity.

QUESTION 41-4 A 41-year-old woman is being treated with clindamycin or an in ec-


tion in the skin o her leg that resulted rom a bicycle accident. She should be warned
about what serious side e ect?
a. Discoloration around the in ection site
b. Watery diarrhea
c. Constipation
d. Pain ul urination
e. A change in her hearing

574
Protein Synthesis Inhibitors and Miscellaneous Antibacterial Agents CHAPTER 4 1

QUESTION 41-5 A 63-year-old woman has developed a vancomycin-resistant E. aecium


in ection. It is recommended that she be treated with quinupristin/dal opristin. Which o
this patients medications should be discontinued or at least be monitored closely while
she is on quinupristin/dal opristin therapy?
a. Hydrochlorthiazide
b. Acetaminophen
c. Aspirin
d. Fluoxetine
e. None o the above

QUESTION 41-6 T e patient in Question 41-5 is switched to daptomycin. Which o


this patients medications should be discontinued or at least monitored closely while
she is on daptomycin therapy?
a. Hydrochlorthiazide
b. Acetaminophen
c. Aspirin
d. Fluoxetine
e. None o the above

SUMMARY QUIZ ANSWER KEY


QUESTION 41-1 Answer is c. Rapid intravenous in usion o vancomycin may cause
a ushing reaction re erred to as red man syndrome. T is is not an allergic reaction
but a direct toxic e ect o vancomycin on mast cells, causing them to release histamine.
T is reaction is generally not observed with teicoplanin.

QUESTION 41-2 Answer is a. Oral absorption o most tetracyclines is incomplete.


Absorption is urther impaired by the concurrent ingestion o dairy products, antacids,
aluminum hydroxide gels, calcium, magnesium, iron or zinc salts, and bismuth sub-
salicylate (PEP O BISMOL). T e cations in these products result in chelation with the
tetracycline and decreased absorption.

QUESTION 41-3 Answer is e. etracyclines may produce mild to severe photosensitivity


reactions in the skin o treated individuals exposed to sunlight.

QUESTION 41-4 Answer is b. T e reported incidence o diarrhea associated


with the administration o clindamycin ranges rom 2 to 20%. A number o patients
(up to 10%) have developed pseudomembranous colitis caused by the toxin
rom the organism C. dif cile. T is colitis is characterized by watery diarrhea, ever,
and elevated peripheral white blood cell counts. T is syndrome may be atal.
reatment with metronidazole or oral vancomycin is usually curative, although
relapses may occur.

QUESTION 41-5 Answer is d. Quinupristin/dal opristin inhibits CYP3A4. T e con-


comitant administration o other CYP3A4 substrates, such as uoxetine, should be
used with caution or monitored care ully.

QUESTION 41-6 Answer is e. Daptomycin neither inhibits nor induces CYPs, and
there are no signif cant drugdrug interactions.

575
SECTION VII Chemotherapy of Microbial Diseases

SUMMARY: PROTEIN SYNTHESIS INHIBITORS AND MISCELLANEOUS ANTIBIOTICS


RESISTANCE (SEE SIDE
BAR MECHANISMS TOXICITY
OF ACTION AND
CLASS AND RESISTANCE OF PROTEIN UNIQUE;
SUBCLASSES NAME CLINICAL USES SYNTHESIS INHIBITORS) COMMON CLINICALLY IMPORTANT
Tetracyclines Tetracycline, Firstline therapy o in ections Resistance has limited GI irritation Photosensitivity
Doxycycline, caused by rickettsia, the use o these agents Hepatic toxicity in
Tigecycline mycoplasmas, and Chlamydia or the common bacterial doses >2 g/d
in ections Aggravation o azotemia
Fanconi syndrome with
outdated products
Brown discoloration o
teeth in children or in ants
Depressed bone growth in
premature in ants

Chloramphenicol Chloramphenicol Limited to use in in ections or Resistance is reported Hypersensitivity Bone marrow toxicity
which the bene ts outweigh resulting in aplastic
the potential toxicities anemia
Gray baby syndrome
in neonates

Macrolides and Erythromycin, Respiratory tract in ections Resistance is common GI distress Cardiac arrhythmias in
Ketolides Clarithromycin, caused by Streptococcus patients with risk actors
Telithromycin, pneumonia and H. in uenzae such as prolonged QT
Azithromycin Alternatives or treatment o syndrome
erysipelas and cellulitis Inhibition o CYP3A4,
Chlamydial in ections associated with drug
Pertussis and Campylobacter interactions with other
in ections drugs metabolized by
Helicobacter pylori this CYP
(clarithromycin)
Disseminated mycobacterial
in ections (clarithromycin)

Lincosamides Clindamycin Similar to erythromycin, but Similar to erythromycin, Nausea, Pseudomembranous


more active against anaerobic but clindamycin is not a vomiting, and colitis caused by C. dif cle
bacteria especially B. ragilis substrate or macrolide diarrhea
e ux pumps

Streptogramins Quinupristin/ Bacteriocidal against Similar to macrolides Pain and Inhibition o CYP3A4
Dal opristin streptococci and many phlebitis at which may cause
strains o staphylococci but in usion site toxicity o other drugs
bacteriostatic against E. metabolized by this
aecium enzyme

Oxazolidinones Linezolid Treatment o in ections caused Should be reserved as GI distress Hematologic


by vancomycin-resistant an alternative agent or toxicity including
E. aecium; nosocomial treatment o in ections myelosuppression
pneumonia, caused by caused by multiple drug-
methicillin-susceptible resistant strains
and resistant strains o S.
aureus; community-acquired
pneumonia

Aminocyclitols Spectinomycin Treatment o gonorrhea


resistant to rst-line drugs

Polymyxins Polymyxin B Not absorbed orally; available Emergence o resistance None, probably
Polymyxin E or ophthalmic, otic, and while on therapy is due to poor
topical use in combination documented systemic
with other agents absorption
(Continued)

576
Protein Synthesis Inhibitors and Miscellaneous Antibacterial Agents CHAPTER 4 1

RESISTANCE (SEE SIDE


BAR MECHANISMS TOXICITY
OF ACTION AND
CLASS AND RESISTANCE OF PROTEIN UNIQUE;
SUBCLASSES NAME CLINICAL USES SYNTHESIS INHIBITORS) COMMON CLINICALLY IMPORTANT

Glycopeptides Vancomycin, Used to treat in ections caused Resistant strains o Hypersensitivity Red mansyndrome rom
Teicoplanin by methicillin-sensitive and enterococci have reactions rapid IVin usion
methicillin-resistant strains o emerged as major Auditory impairment
staphylococci, streptococci, nosocomial pathogens
and enterococci

Lipopeptides Daptomycin Treatment o complicated skin Not well-characterized Musculoskeletal toxicity


and so t tissue in ections and including rhabdomyolysis
complicated bacteremia in rare cases

Bacitracin Bacitracin Use restricted to topical Not a major clinical Hypersensitivity reactions
application including problem are rare
ophthalmic and skin
ointments

Mupirocin Mupirocin Available or topical use No cross-resistance Irritation and Polyethylene glycol in the
or treatment o traumatic with other classes o sensitization ointment can be absorbed
skin lesions and impetigo antibiotics at the site o rom damaged skin
secondarily in ected with S. application
aureus or S. pyogenes
Nasal ointment is approved
or the eradication o S. aureus
nasal carriage

577
CHAPTER
Chemotherapyof Tuberculosis,
42 MycobacteriumAviumComplex
Disease, and Leprosy
T is chapter will be most use ul a er having a basic understanding o the material in
DRUGS INCLUDED IN THIS
Chapter 56, Chemotherapy o uberculosis, Mycobacterium Avium Complex Disease,
CHAPTER and Leprosy in Goodman & Gilmans T e Pharmacological Basis o T erapeutics, 12th
Aminoglycosides (streptomycin, amikacin, Edition. In addition to the material presented here, the 12th Edition contains:
kanamycin) (See Chapter 40) able 56-1 Pathogenic Mycobacterial Rapid and Slow Growers (Runyon Classi cation)
Capreomycin (CAPASTAT) able 56-2 Population Pharmacokinetic Parameter Estimates or Antimycobacteial
Clo azimine (LAMPRENE) Drugs in Adult Patients
Cycloserine (SEROMYCIN) able 56-3 Pharmacokinetic Parameters o Ri ampin, Ri abutin, and Ri apentine
Dapsone able 56-5 Drugs Used in the reatment o Mycobacteria Other T an or uberculo-
Ethambutol (MYAMBUTOL) sis, Leprosy, or MAC (Mycobacterium Avium Complex)
Ethionamide (TRECATOR) Figure 56-4 Multimodal distribution o isoniazide (INH) clearance due to NA 2
polymorphisms
Fluoroquinolones (o oxacin, cipro oxacin,
moxi oxacin) (See Chapter 38)
LEARNING OBJECTIVES
Isoniazid (NYDRAZID)
Understand the rationale or combination drug therapy in the treatment o
PA-824 tuberculosis ( B).
Pyrazinamide Know the mechanisms o action and resistance or drugs used to treat B
Ri amycins: Ri ampin (RIFADIN, RIMAC- and leprosy.
TANE, others), Ri apentine (PRIFTIN),
Describe the adverse e ects and drug interactions commonly associated with
Ri abutin (MYCOBUTIN)
anti- B drugs.
TMC-207
Know the principles o anti- B chemotherapy.
Understand the principles o therapy o Mycobacterium Avium Complex
(MAC) disease.
Understand the principles o antileprosy therapy.

MECHANISMS OF ACTION AND RESISTANCE OF DRUGS USED TO TREAT TUBERCULOSIS AND LEPROSY
MECHANISM OF ACTION MECHANISM OF RESISTANCE
DRUG see Figure 42 1 see Figure 42 2
Ri amycins Binds to the subunit o DNA-dependent RNA polymerase to orm a Mutations causing an alteration in the drug target
stable drug-enzyme complex thus suppressing chain ormation in RNA
synthesis

Pyrazinamide In Mycobacterium tuberculosis pyrazinamidase deaminates pyrazinamide Pyrazinamidases with reduced a nity or
to pyrazinoic acid (POA) which is protonated in an acidic environment to pyrazinamide
POAH which is responsible or microbial killing (see Side Bar PROPOSED
MECHANISMS FOR MICROBIAL KILLING BYPYRAZINAMIDE)

Isoniazid Activated in bacilli by catalase-peroxidase (KatG) to an isonicotinoyl Preexistent resistance can be expected in
radical that reacts with mycobacterial NAD and NADP to orm protein pulmonary TB cavities o untreated patients;
adducts; adduct ormation inhibits the ormation o mycolic acid and these resistant mycobacteria are selected and
mycobacterial dihydro olate reductase, thereby inter ering with nucleic ampli ed by isoniazid monotherapy; resistance is
acid synthesis (see Figure 42-3) associated with mutation or deletion o KatG, also
by enhancement o e ux pump activity

Ethambutol Inhibition o arabinosyl trans erase III with disruption o mycobacterial Mutations in the embBgene and possibly by
cell wall enhanced e ux pump activity
(Continued)

578
Chemotherapy of TB, MAC Disease, and Leprosy CHAPTER 4 2

MECHANISM OF ACTION MECHANISM OF RESISTANCE


DRUG see Figure 42 1 see Figure 42 2
Clo azimine Possible mechanisms o action include: Unknown
membrane disruption
inhibition o phospholipase A2
inhibition o K+ transport
inter erence with electron transport

Ethionamide Mycobacteria convert ethionamide to a sul oxide and another toxic Resistance occurs via changes in the enzyme that
intermediate; this results in inhibition o mycolic acid biosynthesis and activates ethionamide
impairment o cell wall synthesis

TMC-207 Targets subunit c o ATP synthase resulting in inhibition o proton pump Mutation o ATP synthase c subunit
and inter erence o energy metabolism

PA-824 Inhibition o mycolic acid and protein synthesis Mutation in target protein

Cycloserine Inhibition o alanine racemase and stopping reactions in which Resistance in clinical isolates has been
d -alanine is incorporated into cell wall synthesis demonstrated although mechanism is unknown

Capreomycin Inhibition o protein synthesis similar to aminoglycosides Resistance develops when given alone; there is
cross-resistance with kanamycin and neomycin

Dapsone Structural analog o para-aminobenzoic acid (PABA) and inhibitor o Mutations in genes encoding dihydropteroate
dihydropteroate synthase in olate pathway (see Figure 42-4) synthase

Approve d Drug s
RNA Polyme ra s e
Fluo ro quino lo ne :
mRNA inhibits DNA synthe s is a nd s upe rcoiling by
ta rge ting topois ome ra se
Rifamyc in:
30S DNA inhibits RNA synthe s is by ta rge ting RNA polyme ra s e
S tre pto myc in:
inhibits prote in synthe s is by ta rge ting the 30S
ribos oma l s ubunit
Mac ro lide s :
ta rge t 23S ribos oma l RNA, inhibiting Mycolic
pe ptidyl tra ns fe ra s e a cid

Ce ll wa ll

Myc o bac te rium Ce ll me mbra ne

Is o niazid and Ethio namide :


inhibit mycolic a cid synthe s is
Ethambuto l:
ADP inhibits ce ll wa ll synthe s is
H+ ATP
Pi Pyrazinamide :
inhibits ce ll me mbra ne synthe s is
a b a ATP
syntha s e Expe rime ntal Drug s
TMC-207 (R207910, TMC):
inhibits ATP syntha s e
PA-824:
inhibits mycolic a cid a nd prote in biosynthe s is ;
pos s ibly a cts via ge ne ra tion of toxic ra dica ls

FIGURE 42-1 Mechanisms o action o established and experimental drugs used or the chemotherapy o mycobac-
terial in ections. Shown at the top are the sites o action o approved drugs or the chemotherapy o mycobacterial
diseases. Ri amycin is used as a generic term or several drugs, o which ri ampin is used most requently. Also included
are 2 experimental drugs now under investigation: TMC-207 and PA-824. Clo azimine, whose mode o action is not
understood, is omitted.
579
SECTION VII Chemotherapy of Microbial Diseases

Drug una ble to Ana e robic conditions le a d to


dorma nt/non-re plica ting s ta te ; Alte ra tion of e nzyme preve nts
pe ne tra te ce ll wa ll conve rs ion of pro-drug to a ctive
drugs tha t block me ta bolic proce s s e s
have no e ffe ct during s ta te of dorma ncy form (pyra zina mide, is onia zid)
(exc e p tions : rifa mycin, fluoroquinolone )

Enzyme

pro-drug
H+ H+
+ +
H+ H H+ H
H+ H+
Muta tions in DNA RNA Polyme ra s e
re pa ir ge ne s le a d to
Low pH re nde rs
multiple drug re s is ta nce
drug ina ctive
(s tre ptomycin)
DNA
Drug exporte d from ce ll
be fore it re a che s ta rge t
(s tre ptomycin, is onia zid, e tha mbutol) Alte ra tion of ta rge t prote in s tructure
preve nts drug re cognition
(rifa mycin, e tha mbutol, s tre ptomycin,
fluoroquinolone, ma crolide )
PROPOSED MECHANISMS
FIGURE 42-2 Mechanisms o resistance o Mycobacteria to di erent chemotherapeutic drugs.
FOR MICROBIAL KILLING Shown are the various mechanisms by which mycobacteria resist antibacterial e ects o the
BY PYRAZINAMIDE currently approved chemotherapeutic agents.
Inhibition o attyacid synthesis type I
leading to inter erence with mycolic
acid synthesis
Reduction o intracellular pH
Disruption o membrane transport by
protonated pyrazinoicacid (POAH)
Is o niazid N-ac e tyl is o niazid
(pro-drug) (ma jor me ta bolite )
CRITERION FOR O
PROPHYLACTIC H H
O N NH2 O N N C CH3
ANTITUBERCULOSIS THERAPY C C H

Mantouxreaction is more than or equal to NAT2


5 mmand those who meet one o Re na l excre tion
N AcCoA CoA N
ollowing criteria:
Recentlyexposed toTB
HIVco-in ection In ho s t
In bac illus
Fibroticchanges on chest radiograms R
Ka tG
Immunosuppressed N
Mantouxreaction is more than or equal to
10 mmand those who meet one o the O C R C O
ollowing criteria: N s ponta ne ous C NH2
O
Recent (5 years) immigrants romarea N
o highTBprevalence N C O
Nicotinoyl ra dica l NH2 Nicotinoyl-NAD a dduct
Children younger than 4 years (a ctiva te d drug) (inhibitor of InhA)
Children exposed to adults withTB NAD+ Nicotinoyl-NADP a dduct
NADP + (inhibitor of DHFR)
Intravenous (IV) drug users
FIGURE 42-3 Metabolism and activation o isoniazid. The prodrug isoniazid is
Residents and employees in high-risk metabolized in humans by NAT2 iso orms to its principal metabolite, N-acetyl
settings isoniazid, which is excreted by the kidney. Isoniazid di uses into mycoplasma where
Anyone with a Mantouxreaction more it is activatedby KatG (oxidase/peroxidase) to the nicotinoyl radical, which reacts
than 15 mm spontaneously with NAD+ or NADP+ to produce adducts that inhibit important
enzymes in cell wall and nucleic acid synthesis. DHFR, dihydro olate reductase.

580
Chemotherapy of TB, MAC Disease, and Leprosy CHAPTER 4 2

Pte ridine + PABA DEFINITIVE


Inhibito rs :
Da ps one ANTITUBERCULOSIS
PAS Dihydro pte ro ate THERAPY
S ulfona mide s s ynthas e (fo lP1/P2)
All active TBcases should be confrmed by
culture with antimicrobial susceptibility
Dihydro pte ro ic ac id
Thymidylate determined.
s ynthas e X (thyX)
Dihydro fo late For the frst 2 months:
s ynthas e (fo lC) Isoniazid (5 mg/kg, maximum300
dTMP
dUMP mg/d), pyridoxine (10 to 50 mg/d)
Dihydro fo lic ac id Ri ampin (10 mg/kg, maximum
600 mg/d)
Thymidylate Dihydro fo late
s ynthas e A re duc tas e (dfrA)
Pyrazinamide (15 to 30 mg/kg,
maximum2 g/d)
5,10-me thyle ne Follow-up therapy, 2 or 3 times per week, or
Te trahydro fo lic ac id
te trahydro fo late next 4 months:
FIGURE 42-4 E ects o antimicrobials on olate metabolism and
Isoniazid (15 mg/kg), pyridoxine (10 to
deoxynucleotide synthesis. 50 mg/d)
Ri ampin (10 mg/kg)

CASE 42-1
On a recent trip to India, a 45-year-old man contracted tuberculosis that was con rmed
by culture. Antimicrobial susceptibility was per ormed and the patient was started on
isoniazid, ri ampin, and pyrazinamide as de nitive therapy or tuberculosis.
a. Why did this patient receive three drugs?
Only combination anti- B therapy is currently recommended or treatment o B.
When anti- B drug monotherapy was administered to B patients, resistance emer-
gence terminated the e ectiveness o these drugs. T e mutation rates to rst-line B
drugs are between 107 and 1010 so that the likelihood o resistance is high to any
single anti- B drug in patients with cavitary B who have ~109 CFU (colony orm-
ing units) o bacilli in a 3-cm pulmonary lesion. However, the likelihood that bacilli
would develop mutations to 2 or more di erent drugs is the product o 2 mutation
rates which makes the probability o resistance emergence to more than 2 drugs
acceptably small. Multidrug therapy has also led to a reduction in length o therapy.
b. What is the mechanism o action o ri ampin?
Ri ampin enters bacilli in a concentration-dependent manner where it binds to the
subunit o DNA-dependent RNA polymerase to orm a stable drug-enzyme com-
plex. Drug binding suppresses chain ormation in RNA synthesis (see Figure 42-1).
c. How should ri ampin therapy be optimized?
Ri ampins bactericidal activity is best optimized by a regimen to achieve a high
AUC/MIC (area under the curve/minimal inhibitory concentration) ratio, but
resistance suppression and ri ampins enduring postantibiotic e ect are best opti-
mized by high CPmax/MIC therapy (see Chapter 34). T ere ore, the duration o time
that the ri ampin concentration persists above the MIC is o less importance than
reaching a high CPmax. T e t o ri ampin is less o an issue in optimizing therapy,
and i patients could tolerate it, higher doses would lead to higher bactericidal
activities while suppressing resistance.
d. For what clinical diseases, other than TB, might ri ampin be use ul?
Ri ampin is also use ul or the prophylaxis o meningococcal disease and Haemoph-
ilus inf uenza meningitis. Combined with a -lactam antibiotic or vancomycin,
ri ampin may be use ul or therapy in selected cases o staphylococcal endocarditis
(Continued)
581
SECTION VII Chemotherapy of Microbial Diseases

or osteomyelitis, especially those caused by staphylococci tolerant to penicillin.


Ri ampin may also be used or the eradication o staphylococcal nasal carrier state.
e. What adverse e ects should this patient be warned o with the use o ri ampin?
Ri ampin is generally well tolerated, but patients should be warned that it will cause
an orange-tan discoloration o skin, urine, eces, saliva, tears, and contact lenses.

CASE 42-2
A 35-year-old woman hospital employee has a routine tuberculin skin test reaction
o 18 mm. She is started on prophylactic isoniazid therapy o 300 mg daily. She is also
started on pyridoxine (vitamin B6).
a. What is the mechanism o action o isoniazid?
Isoniazid is activated within the bacillus to an isonicotinoyl radical by KatG, a
multi unctional catalase-peroxidase (see Figure 42-3). T is radical interacts with
mycobacterial NAD and NAPD to orm adducts that inhibit the activities o
enzymes responsible or the synthesis o mycolic acid, an essential component o
the mycobacterial cell wall. T e adducts also inhibit mycobacterial dihydro olate
reductase and inter ere with nucleic acid synthesis.
b. How is isoniazid metabolized?
Isoniazid is metabolized by hepatic arylamine N-acetyltrans erase type 2 (see
Figure 42-3). Isoniazid clearance in patients can be classi ed into 2 phenotypic
groups: slow and ast acetylators as seen in Figure 56-4 o Goodman and Gilmans T e
Pharmacological Basis o T erapeutics, 12th Edition. Recently the phenotypic groups
have been expanded to ast, intermediate, and slow acetylators. T e number o NA 2*4
alleles accounts or 88% o the variability o isoniazid clearance. Slow acetylators may
be a greater risk or adverse e ects rom isoniazid, sul onamides, and procainamide.
Fast acetylators may have diminished responses to standard doses o these agents but a
greater risk rom bioactivation by NA 2 o arylamine/hydrazine carcinogens.

CASE 42-3
In the patient in Case 42-2:
a. What are the potential side e ects o isoniazid therapy in this otherwise healthy
woman?
Isoniazid is converted to acetylisoniazid which can be converted to acetylhydrazine
and hepatotoxic metabolites by CYP2E1. Rapid acetylators will orm diacetylhy-
drazine which is nontoxic, while slow acetylators or CYP2E1 induction will lead to
more hepatotoxic metabolites. Ri ampin, a potent inducer o CYP2E1, potentiates
isoniazid hepatotoxicity.
Isoniazid also can cause a peripheral neuritis (see answer to Case 42-3c below).
b. What drug interactions should she be warned o ?
Isoniazid is a potent inhibitor o CYP2C19, CYP3A, and a weak inhibitor o
CYP2D6. Isoniazide induces CYP2E1. Drugs metabolized by these enzymes will
potentially be a ected. able 42-1 lists the potential drug interactions that might
occur with isoniazid and their adverse e ects.
c. Why is she also given a vitamin?
As mentioned above in the answer to question a o this case, isoniazid causes a
peripheral neuritis (most commonly paresthesias o the eet and hands). T is
neuropathy is more requent in slow acetylators and in individuals with diabetes
mellitus, poor nutrition, or anemia. T e prophylactic administration o pyridoxine
(vitamin B6) prevents the development o peripheral neuritis even when therapy
lasts as long as 2 years.

582
Chemotherapy of TB, MAC Disease, and Leprosy CHAPTER 4 2

TABLE 42-1 Isoniazid-Drug Interactions via Inhibition and Induction of CYPs


CO ADMINISTERED DRUG CYP ISOFORM ADVERSE EFFECTS
Acetaminophen CYP2E1 inhibition-induction Hepatotoxicity

Carbamazepine CYP3A inhibition Neurological toxicity

Diazepam CYP3A and CYP2C19 Sedation and respiratory


inhibition depression

Ethosuximide CYP3A inhibition Psychotic behavior

Isof urane and enf urane CYP2E1 induction Decreased e ectiveness

Phenytoin CYP2C19 inhibition Neurological toxicity

Theophylline CYP3A inhibition Seizures, palpitation, nausea

Vincristine CYP3A inhibition Limb weakness and tingling

War arin CYP2C9 inhibition Possibility o increased bleeding


(single case reported)

CASE 42-4
A 25-year-old medical resident has developed active B a er being exposed in the hos-
pital. It is known that the patient she was exposed to had a strain o M. tuberculosis that
was resistant to isoniazid. She is started on ri ampin, pyrazinamide, and ethambutol.
a. What is the mechanism o action o ethambutol?
Ethambutol inhibits arabinosyl trans erase III which disrupts the assembly o the
mycobacterial cell wall.
b. I this patients M. tuberculosis is resistant to isoniazid, might it also be resistant
to ethambutol?
Mycobacterial resistance to ethambutol develops via mutations in the embB gene.
However, enhanced e ux pump activity may induce resistance to both isoniazid
and ethambutol. T us, resistance to isoniazid does not necessarily predict resis-
tance to ethambutol.
c. What serious side e ect o ethambutol should this patient be warned about?
About 1% o patients receiving ethambutol experience diminished visual acuity.
Approximately 15% o patients receiving a dose o ethambutol o 50 mg/kg/d will
develop optic neuritis resulting in decreased visual acuity and an inability to dis-
tinguish red rom green. T e incidence o this reaction is proportional to the dose
o ethambutol with approximately 1% o patients who receive a dose o 15 mg/kg/d
experiencing this e ect.

CASE 42-5
A 36-year-old man in Central A rica has developed leprosy. He is being treated with
ri ampin, clo azimine, and dapsone.
a. Why 3 drugs?
T e reasons or using combination therapy in the treatment o leprosy include
reduction in the development o resistance, the need or adequate therapy when
primary resistance already exists, and reduction in the duration o therapy. Ri ampin
is the most bactericidal drug in the regimen. Clo azimine is only bacteriostatic
against Myocobacterium leprae; however, it also has anti-in ammatory e ects.
(Continued)
583
SECTION VII Chemotherapy of Microbial Diseases

b. What is the mechanism o action o dapsone?


TREATMENT OF DRUG-
RESISTANT TUBERCULOSIS Dapsone is a competitive inhibitor o dihydropteroate synthase in the olate pathway
(see Figure 42-4). Dapsone also has anti-in ammatory e ects by inhibiting tissue
Therapyshould be based on evidence damage caused by neutrophils.
o susceptibilityand should include
the ollowing: c. What enzyme def ciency should this patient be tested or be ore starting dapsone
therapy?
At least 3 drugs to which the pathogen
is susceptible, one o theminjectable Dapsone, an oxidant, causes severe hemolysis in patients with glucose-6-phosphate
dehydrogenase (G6PD) de ciency. T us, G6PD de ciency testing should be
Use o 4 to 6 medications or MDR-TB
per ormed prior to the use o dapsone wherever possible.
(multiple drug resistant-TB)
18 months o therapy
CASE 42-6
A 58-year-old man develops active B con rmed by positive culture. An in ectious
disease consultant recommends therapy with isoniazid, ri ampin, and pyrazinamide.
Susceptibility testing reveals that the M. tuberculosis cultured rom this patient is
PRINCIPLES OF THERAPY resistant to isoniazid and ri ampin.
OF MYCOBACTERIUM AVIUM
COMPLEX (MAC) a. What are the considerations in treating this patient?
MACis made up o 2 species: M. intracel- Multidrug-resistant tuberculosis (MDR- B) is present i an isolate is resistant
lulare and M. avium. simultaneously to isoniazid and ri ampin. In documented drug resistance, therapy
should be based on evidence o susceptibility and should include the principles
Ater isolating MAC rompulmonaryspeci- shown in the Side Bar REA MEN OF DRUG-RESIS AN UBERCULOSIS.
mens, frst determine whether disease is
present or i the organismis merelypart o
environmental contamination. CASE 42-7
Criteria or therapy: Following a liver transplant, a 38-year-old man develops a sputum culture that is positive
Positive cultures rom2 sputum or Mycobacterium intracellulare.
specimens, or 1 positive culture rom a. What is the f rst course o action?
bronchoalveolar lavage, or pulmonary
M. intracellulare is 1 o the species that make up MAC, Mycobacterium avium is the
biopsywith positive culture or
other species. T ese bacteria are ubiquitous in the environment and can be encoun-
histopathological eatures
tered in ood, water, and soil. M. intracellulare of en in ects immunocompromised
Clinical evidence o in ection patients. T e rst decision af er isolating MAC rom pulmonary specimens is to
Radiological evidence o in ection determine whether disease is actually present or i the organism is merely part
For newlydiagnosed patients with MAC o environmental contamination (see Side Bar PRINCIPLES OF HERAPY OF
pneumonia, therapyshould include MYCOBAC ERIUM AVIUM COMPLEX).
ri amycin, ethambutol, and macrolide b. I the diagnosis is conf rmed, what is the recommended therapy?
(clarithromycin or azithromycin).
In newly diagnosed patients with MAC pneumonia, therapy with ri ampin, ethambutol,
Therapyshould continue or 12 months and a macrolide antibioticeither clarithromycin or azithromycinis recommended.
a ter last negative culture. T erapy should be continued or 12 months af er the last negative culture.

PRINCIPLE OF KEY CONCEPTS


ANTILEPROSY THERAPY Combination therapy is the desirable approach or mycobacterial disease to
Therapy or leprosyis based on multidrug ensure e ective eradication and to prevent the emergence o resistance.
regimens using the ollowing: Isoniazid, ri amycins, ethambutol, streptomycin, and pyrazinamide are rst-line
Ri ampin agents or the treatment o tuberculosis.
Clo azimine Antimicrobial agents with excellent activity against MAC include ri abutin,
Dapsone clarithromycin, azithromycin, and f uoroquinolones.
Clinical monitoring o patients with mycobacterial in ections is important
because drug interactions and adverse drug reactions are common with the
multiple-drug regimens used.
Considerable progress has been achieved in eliminating leprosy through the use
o multiple-drug chemotherapy, including dapsone, ri ampin, and clo azimine.

584
Chemotherapy of TB, MAC Disease, and Leprosy CHAPTER 4 2

SUMMARY QUIZ

QUESTION 42-1 A 35-year-old man has developed a tuberculin skin reaction that is 20 mm
in size. He is being treated with a triple-drug regimen o isoniazid, ri ampin, and pyra-
zinamide but is concerned because o the number o medications he must take every
day. He is a slow acetylator o isoniazid. He asks why other antibiotics like penicillin are
not e ective in treating tuberculosis. Your answer is that the tuberculosis bacteria
a. have developed a resistance to penicillin.
b. have a rigid cell wall that does not allow penicillin to get inside the cell.
c. destroy the penicillin.
d. multiply too rapidly or the penicillin to be e ective.
e. are anaerobic.

QUESTION 42-2 T e patient in Case 42-1 should be instructed to take his ri ampin on
an empty stomach because
a. ood decreases the peak plasma concentration o ri ampin by one-third.
b. ri ampin may a ect the taste o ood.
c. ood increases the absorption o ri ampin and may cause toxicity.
d. ood increases gastric transit.
e. ood increases gastric acid secretion which may destroy the ri ampin.

QUESTION 42-3 T e plasma concentration o ri ampin in the patient in Case 42-1 is


only above the MIC approximately 25% o the dosing interval. T is is o little concern
because the ri ampin
a. t in this patient is prolonged.
b. bactericidal activity is optimized by a high AUC/MIC.
c. volume o distribution is very small.
d. pharmacokinetics are best described by a two-compartment model.
e. induces CYP3A4.

QUESTION 42-4 A er 1 month o treatment, the patient in case 42-1 is complaining o


numbness and tingling in his eet. T e likely problem is
a. tight shoes.
b. a blood clot in the emoral artery.
c. undiagnosed diabetes mellitus.
d. heart ailure.
e. isoniazid-induced peripheral neuritis.

QUESTION 42-5 T e patient in Case 42-1 is taking other medications or medical


problems unrelated to tuberculosis. Which o the ollowing drugs should be monitored
closely a er starting isoniazid therapy?
a. Aspirin
b. Dexamethasone
c. Phenytoin
d. Met ormin
e. Nitroglycerine

QUESTION 42-6 A 63-year-old woman has developed leprosy and is being treated with
a combination o ri ampin, clo azimine, and dapsone. A de ciency o which o the
ollowing enzymes is o speci c concern with dapsone therapy?
a. N-acetyltrans erase
b. DNA gyrase (Continued)
585
SECTION VII Chemotherapy of Microbial Diseases

c. Pyrazinamidase
d. Glucose-6-phosphate dehydrogenase
e. Glutathione trans erase

SUMMARY QUIZ ANSWER KEY


QUESTION 42-1 Answer is b. T e name Mycobacterium re ers to their waxy appear-
ance due to the composition o their cell walls. More than 60% o their cell wall is lipid,
mainly mycolic acids. T is extraordinary shield prevents many pharmacological com-
pounds, such as penicillin, rom getting into the bacterial cell wall or inside the cell.
QUESTION 42-2 Answer is a. A er oral administration, the ri amycins are absorbed
to variable extents. Food decreases the ri ampin CPmax by one-third. A high- at meal
increases the area under the curve (AUC) o ri apentine by 50%.
QUESTION 42-3 Answer is b. Ri ampins bactericidal activity is best optimized by a
high AUC/MIC ratio. However, the resistance suppression and ri ampins postantibiotic
e ect are best optimized by high CPmax/MIC. T ere ore, the duration o time that the
ri ampin concentration persists above the MIC is o less importance.
QUESTION 42-4 Answer is e. Paresthesia o the eet and hands (peripheral neuritis) is
encountered in about 2% o patients receiving isoniazid 5 mg/kg daily. Neuropathy is
more requent in slow acetylators o isoniazid. T e prophylactic administration o pyri-
doxine (vitamin B6) prevents the development o the peripheral neuritis as well as other
nervous system disorders.
QUESTION 42-5 Answer is c. Isoniazid is a potent inhibitor o CYP2C19, CYP3A, and
a weak inhibitor o CYP2D6. Drugs that are metabolized by these enzymes will poten-
tially be a ected. Phenytoin metabolism by CYP2C19 is inhibited by isoniazid with
potential neurotoxicity resulting.
QUESTION 42-6 Answer is d. Glucose-6-phosphate dehydrogenase (G6PD) protects
red blood cells against oxidative damage. Dapsone, an oxidant, causes severe hemolysis
in patients with G6PD de ciency. G6PD de ciency occurs in nearly hal a billion peo-
ple worldwide. T us, G6PD de ciency testing should be per ormed prior to the use o
dapsone wherever possible.

SUMMARY: DRUGS USED IN THE TREATMENT OF TUBERCULOSIS, MAC, AND LEPROSY


TOXICITY
CLASS AND UNIQUE; CLINICALLY
SUBCLASSES NAME CLINICAL USES RESISTANCE COMMON IMPORTANT
Ri amycins Ri ampin, Available or the treatment o The prevalence o Rash, nausea, Rarely hepatitis
ri apentine, TB either alone, or in xed dose ri ampin-resistant and vomiting Skin and body f uid
ri abutin combinations with isoniazid or isolates are 1 in every 107 discoloration
isoniazid plus pyrazinamide to 108 bacilli
Also use ul or prophylaxis
o meningococcal disease and
H. inf uenzae meningitis

Pyrazinamide Coadministration with isoniazid Pyrazinamide-resistant Elevated blood Hepatic injury


or ri ampin has led to a one-third M. tuberculosis have uric acid levels Gout
reduction in the duration o TB pyrazinamidases with
therapy and a two-thirds reduction in reduced a nity or
TB relapse pyrazinamide
(Continued)

586
Chemotherapy of TB, MAC Disease, and Leprosy CHAPTER 4 2

TOXICITY
CLASS AND UNIQUE; CLINICALLY
SUBCLASSES NAME CLINICAL USES RESISTANCE COMMON IMPORTANT
Isoniazid NYDRAZID Available to treat TB either alone, Prevalence o resistance Hepatic injury due to the
or in combination with ri ampin is about 1 in 106 bacilli; ormation o hepatotoxic
or pyrazinamide preexisting resistance metabolites
can be expected in Peripheral neuropathy
pulmonary TB cavities o that is prevented
untreated patients by the prophylactic
administration o
pyridoxine

Ethambutol MYAMBUTOL Oral administration or the treatment 30-70% o clinical Lowered visual Optic neuritis with loss
o TB, disseminated MAC, and isolates are resistant to acuity, rash, o visual acuity
Mycobacterium kansaii in ection ethambutol drug ever

Clo azimine LAMPRENE Part o multiple drug therapy with Abdominal pain, Body f uid discoloration
ri ampin and dapsone or the diarrhea, nausea,
treatment o leprosy and vomiting

TMC-207 Good activity against M. tuberculosis, Nausea and Full side-e ect pro le is
MAC, M. leprae, Mycobacterium diarrhea not clear at this time
bovis, Mycobacterium marinum,
M. kansaii, Mycobacterium
ulcerans, Mycobacterium ortuitum,
Mycobacterium szulgai, and
Mycobacterium abscessus

PA-824 In vitro it kills non-replicating M. Not known at Not known at this time
tuberculosis under aerobic conditions this time
and replicating bacilli in ambient air

Ethionamide TRECATOR Second-line drug or the Resistance occurs GI upset Severe postural
treatment o TB via mutations in the hypotension
enzyme that activates Mental depression,
ethionamide drowsiness, blurred
vision, diplopia, dizziness,
paresthesias; neurologic
symptoms are relieved by
pyridoxine (vitamin B6)

Cycloserine SEROMYCIN Second-line drug or the treatment Resistance in clinical Neuropsychiatric


o TB; inhibits M. tuberculosis at isolates o M. tuberculosis symptoms are common
concentrations o 5-20 mg/L has been detected in 50% o patients
Good activity against MAC, Contraindicated in
Escherichia coli, Staphylococcus patients with a history
aureus, Nocardia, o epilepsy
and Chlamydia

Capreomycin CAPASTA Second-line drug or the treatment Resistance develops Hearing loss, Should not be
o TB when given alone tinnitus, administered with other
Antimicrobial activity is similar to proteinuria drugs that damage
aminoglycosides cranial nerve VIII

Dapsone Bacteriostatic against M. leprae Resistance occurs Skin rashes Hemolysis in patients with
Highly e ective against rom mutations in G6PD de ciency treated
Plasmodium alciparum genes encoding with 200-300 mg/d
dihydropteroate synthase Rarely psychosis

587
CHAPTER

43 Antifungal Agents
T is chapter will be most use ul a er having a basic understanding o the material
DRUGS INCLUDED IN
in Chapter 57, Anti ungal Agents in Goodman & Gilmans T e Pharmacological
THIS CHAPTER Basis of T erapeutics, 12th Edition. In addition to the material presented here,
Amphotericin B(FUNGIZONE); colloidal the 12th Edition contains:
dispersion (AMPHOTEC, AMPHOCIL); Structural ormulas or each o the anti ungal agents in addition to the gures
liposomal ormulation (AMBISOME); lipid reproduced here
complex(ABELCET)
able 57-1 Pharmacotherapy o Mycoses which lists the drugs used or di erent
Anidula ungin (ERAXIS) mycoses
Butenafne (MENTAX,LOTRIMINULTRA) able 57-7 Pharmacokinetics o Echinocandins in Humans which shows the
Butoconazole (FEMSTAT3, others) pharmacokinetic di erences among caspo ungin, mica ungin, and anidula ungin
Caspo ungin (CANCIDAS)
LEARNING OBJECTIVES
Ciclopiroxolamine (LOPROX,others)
Understand the mechanisms o action and resistance o anti ungal agents.
Clotrimazole (LOTRIMIN, MYCELEX,GYNE-
LOTRIMIN, others) Describe the therapeutic uses o anti ungal agents in the context o treatment
Fluconazole (DIFLUCAN, others) or ungal diseases.
Flucytosine (ANCOBON) Develop knowledge o the common and unique toxicities o anti ungal agents.
Griseo ulvin (GRIFULVINand GRIS-PEG) Understand the drugdrug interactions that can occur with the use o azole
anti ungal agents.
Haloprogin (HALOTEX)
Isavuconazole (BAL8557) (investigational) Know the di erences in treating invasive ungal in ections with systemic drugs
versus super cial in ections with topical anti ungal agents.
Itraconazole (SPORANOX,others)
Ketoconazole (NIZORAL, others)
Mica ungin (MYCAMINE) MECHANISMS OF ACTION AND RESISTANCE OF ANTIFUNGAL DRUGS
Miconazole (MICATIN, ZEASORB-AF, MECHANISM OF ACTION
MONISTAT7, MONISTAT3, MONISTAT1, DRUG (see Figure 43-1) MECHANISM OF RESISTANCE
others) Amphotericin B Binds to ergosterol in ungi Resistance is rare, but may be caused
Na tifne (NAFTIN) membranes to orm pores and by replacement o ergosterol with
increase membrane permeability precursor sterols
Nystatin (MYCOSTATIN, NILSTAT,
NYOTRAN, others) Flucytosine Converted in ungi to 5- uorouracil Resistance is due to a loss o the
(5-FU), which is incorporated into ungal permease necessary or
Oxiconazole (OXISTAT)
RNA and inhibits nucleic acid cytosine transport or a decrease in
Posaconazole (NOXAFIL) synthesis (see Figure 43-2) ungal cytosine deaminase activity
Sertaconazole (ERTACZO) Imidazoles and Impaired ergosterol synthesis Mutation in ERG11, the gene coding
Sulconazole (EXELDERM, SULCOSYN) Triazoles due to inhibition o 14--sterol or the 14--sterol demethylase
demethylase
Terbinafne (LAMISIL, others)
Terconazole (TERAZOL, others) Echinocandins Inhibition o 1,3--d -glucan Mutations in glucan synthase
synthesis in ungal cell wall resulting complex genes
Tioconazole (VAGISTAT1, others)
in loss o structural integrity,
Tolna tate (AFTATE, TINACTIN, others) osmotic instability, and cell death
Undecylenicacid (DESENEX,others); Cal- Griseo ulvin Inhibition o microtubule unction De cient energy-dependent
ciumundecylenate (CALDESENE, CRUEX) and inhibition o ungal mitosis transport system used to get
Voriconazole (VFEND) anti ungal agent to microtubules
Whitfelds Ointment Terbina ne Inhibition o ungal squalene Mutations in squalene epoxidase
epoxidase, blocking ergosterol
biosynthesis, and disruption o
ungal cell membrane

588
Antifungal Agents CHAPTER 4 3

cytos ine
de a mina s e
5-FU Flucytos ine
Me mbrane func tio n
a mphote ricin B
uP RTa s e
Erg o s te ro l s ynthe s is
flucona zole
itra cona zole 5-FUMP 5-FUDP 5-FUTP
voricona zole
na ftifine
te rbina fine ribonucle otide
re ducta s e
5-FdUMP RNA

Ce ll wall s ynthe s is Nucle ic ac id thymidyla te


ca s pofungin s ynthe s is syntha s e
5-fluorocytos ine dUMP dTMP
FIGURE 43-1 Sites o action o anti ungal drugs. Amphotericin B FIGURE 43-2 Action o ucytosine in ungi. Flucytosine is transported
and other polyenes, such as nystatin, bind to ergosterol in ungal by cytosine permease into the ungal cell, where it is deaminated to
cell membranes and increase membrane permeability. The imid- 5- uorouracil (5-FU). The 5-FU is then converted to 5- uorouracil-
azoles and triazoles, such as itraconazole and uconazole, inhibit ribose monophosphate (5-FUMP) and then is either converted to
14--sterol demethylase, prevent ergosterol synthesis, and lead to 5- uorouridine triphosphate (5-FUTP) and incorporated into RNA or
the accumulation o 14--methylsterols. The allylamines, such as converted by ribonucleotide reductase to 5- uoro-2-deoxyuridine-
na ti ne and terbina ne, inhibit squalene epoxidase and prevent 5-monophosphate (5-FdUMP), which is a potent inhibitor o thymidylate
ergosterol synthesis. The echinocandins, such as caspo ungin, synthase. 5-FUDP, 5- uorouridine-5-diphosphate; dUMP, deoxyuridine-
inhibit the ormation o glucans in the ungal cell wall. 5-monophosphate; dTMP, deoxythymidine-5-monophosphate.

CASE 43 1
A 56-year-old woman is diagnosed with mucormycoses involving the maxillary
sinuses. An in ectious disease consultant recommends that she be treated with
amphotericin B.
a. What di erent ormulations o amphotericin B are available or her treatment?
T ere are currently 4 ormulations o amphotericin B commercially available:
conventional amphotericin B (C-AMB), liposomal amphotericin B (L-AMB),
amphotericin B lipid complex (ABLC), and amphotericin B colloidal dispersion
(ABCD). able 43-1 summarizes the pharmacokinetic properties o these di erent
preparations.
b. What are the major di erences in these ormulations?
C-AMB is insoluble in water, but is ormulated or intravenous use by complexing
it with the bile salt, deoxycholate. ABCD orms a colloidal solution when
dispersed in water and provides much lower blood concentrations than C-AMB.
(Continued)

TABLE 43-1 Pharmocokinetic Parameters for Amphotericin B Formulations after Multiple Administrations in Humans
PRODUCT DOSE (mg/kg) CMAX (g/mL) AUC(1-24hr) (g .hr/mL) V (L/kg) Cl (mL/hr/kg)
AmBisome (L-AMB) 5 8335.2 555311 0.110.08 116

Amphotec (ABCD) 5 3.1 43 4.3 117

Ablecet (ABLC) 5 1.70.8 147 1317.7 426188.5

Fungizone (C-AMB) 0.6 1.10.2 17.15 52.8 3815

For details, see Boswell et al. (1998). From Boswell GW, Buell D, Bekersky I. AmBisome (Liposomal Amphotericin B): A comparative review. J Clin
Pharmacol, 1998, 38:583-592. 1998 The American College o Clinical Pharmacology. Reprinted by permission o SAGE Publications.

589
SECTION VII Chemotherapy of Microbial Diseases

In usion reactions o chills and ever are more common with ABCD than with
C-AMB. L-AMB is supplied as a lyophilized powder and has equivalent blood
concentrations as C-AMB. L-AMB is approved or empirical therapy o ever in the
neutropenic host not responding to appropriate antibacterial agents, as well as or
salvage therapy o aspirgillosis and candidiasis. ABLC provides blood concentra-
tions o amphotericin B that are much lower than with the same dose o C-AMB.
ABLC is approved or salvage therapy o deep mycoses.
he lipid ormulations appear to reduce the risk o nephrotoxicity during
therapy. T e cost o the lipid ormulations o amphotericin B greatly exceeds that
o C-AMB.
c. What is the mechanism o action o amphotericin?
T e aniti ungal activity o amphotericin B depends principally on its binding to a
sterol moiety, primarily ergosterol, in the membrane o sensitive ungi. By virtue o
their interaction with these sterols, polyenes, appear to orm pores or channels that
increase the permeability o the membrane, allowing the outward leakage o a vari-
ety o small molecules (see Figure 43-1).
d. What untoward e ects should be watched or in this patient?
Major untoward e ects o amphotericin B are in usion-related reactions such as
ever and chills. T ese are most severe with ABCD, slightly less with C-AMB, even
less with ABLC, and least with L-AMB.
Nephrotoxicity with amphotericin, a major concern, is dose-dependent, usually
transient, and increased by concurrent therapy with other nephrotoxic agents such
as aminoglycosides or cyclosporine. Permanent unctional renal impairment is
uncommon in adults with normal renal unction prior to treatment.
Hypochromic, normocytic anemia commonly occurs during treatment
with C-AMB.

CASE 43 2
A 19-year-old woman with coccidiodal meningitis is being treated with uconazole.
a. Why is f uconazole chosen instead o amphotericin B?
Fluconazole is the drug o choice or the treatment o coccidioidal meningitis
because o good penetration into the cerebrospinal uid and much less morbidity
than with intrathecal amphotericin B.
b. What are the limitations to the use o f uconazole?
Fluconazole is a Category C agent that should be avoided during pregnancy unless
the potential bene t justi es the possible risk to the etus.
Fluconazole is an inhibitor o CYP3A4 and CYP2C9, and drugdrug interactions
(shown in ables 43-2 to 43-5) are a limitation to the use o this drug.

TABLE 43-2 Interaction of Azole Antifungal Agents with Hepatic CYPs


FLUCONAZOLE VORICONAZOLE ITRACONAZOLE POSACONAZOLE

CYP3A4 inhibitor CYP2C9 inhibitor and CYP3A4 inhibitor CYP3A4 inhibitor


substrate

CYP2C9 inhibitor CYP3A4 inhibitor

CYP2C19 inhibitor CYP2C19 inhibitor

590
Antifungal Agents CHAPTER 4 3

TABLE 43-3 Drugs Exhibiting Elevated Plasma Concentrations When Co-Administered with Azole Anti-Fungal Agents

Al entanil Eplerenone Losartan Saquinavir

Alprazolam Ergot alkaloids Lovastatin Sildenaf l

Astemizole Erlotinib Methadone Sirolimus

Buspirone Eszopiclone Methylprednisolone Soli enacin

Busul an Felodipine Midazolam Sunitinib

Carbamazepine Fexo enadine Nevirapine Tacrolimus

Cisapride Gef tinib Omeprazole Triazolam

Cyclosporine Glimepiride Phenytoin Vardenaf l

Digoxin Glipizide Pimozide Vinca alkaloids

Docetaxel Halo antrine Quinidine War arin

Do etilide Haloperidol Ramelteon Zidovudine

E avirenz Imatinib Ranolazine Zolpidem

Eletriptan Irinotecan Risperidone

Mechanism o interaction presumably occurs largely at the level o hepatic CYPs, especially CYPs 3 A4, 2C9, and 2D6, but can also involve
P-glycoprotein and other mechanisms. Not all drugs listed interact equally with all azoles.

TABLE 43-4 Some Drugs That Decrease Azole Concentration When Co-Administered
DRUG FLUCONAZOLE VORICONAZOLE ITRACONAZOLE POSACONAZOLE

Antacids (simultaneous) - + -

Barbiturates + +a

Carbamazepine + + + +

H2 antagonists + +

Didanosine +

E avirenz + +

Nevirapine + +

Proton pump inhibitors - _b + +

Phenytoin - + + +

Ri ampin + + + +

Ri abutin + + +

Ritonavir +
a
Phenobarbital only.
b
Omeprazole and voriconazole increase each others concentrations in plasma; reduce omeprazole dose by 50% when initiating voriconazole
therapy.
Reproduced with permission rom Zonios DI, Bennett JE. Update on azole anti ungals. Sem Respir Crit Care Med. 2008;29:192210.

591
SECTION VII Chemotherapy of Microbial Diseases

TABLE 43-5 Some Additional Contraindicated Azole Drug Combinations


DRUG FLUCONAZOLE VORICONAZOLE ITRACONAZOLE POSACONAZOLE
Al uzosin X X X

Artemether X X

Bepridil X

Clopidogrel X

Conivaptan X X X X

Dabigatran X

Darunavir X

Dronedarone X X X X

Everolimus X X X X

Lopinavir X

Lume antrine X X

Mesoridazine X

Nilotinib X X X X

Nisoldipine use with caution X X X

Quinine X X

Ri apentine X use with caution use with caution

Ritonavir X use with caution

Rivaroxaban X X

Salmeterol X X X

Silodosin X X X

Simvastatin use with caution X

St. Johns wort X

Tetrabenazine X X

Thioridazine X X

Tolvaptan X X X

Tolvaptan X X

Topotecan X

Ziprasidone X X

CASE 43 3
A 26-year-old woman has developed esophageal candidiasis. She is being treated with
oral voriconazole. She is also being treated or a seizure disorder and depression, and
during the course o her anti ungal therapy, she will be taking several other drugs.
a. What is the mechanism o action o voriconazole?
Voriconazole, a triazole, is an inhibitor o 14--sterol demethylase and impairs
the biosynthesis o ergosterol required or the ungal cytoplasmic membrane and
thereby inhibits growth o ungi (see Figure 43-1).
(Continued)
592
Antifungal Agents CHAPTER 4 3

Extra c e llula r s p a c e

Ma nnoprote in Gluca n Chitin

Ce ll wa ll
Gluca n syntha s e
complex
Pe rip la s m
P la s ma me mb ra ne

Cytop la s m
Echinoca ndin
a nti-funga l

FIGURE 43-3 The ungal cell wall and membrane and the action o echinocandins. The strength
o the ungal cell wall is maintained by brillar polysaccharides, largely -1,3-glucan and chitin,
which bind covalently to each other and to proteins. A glucan synthase complex in the plasma
membrane catalyzes the synthesis o -1,3-glucan; the glucan is extruded into the periplasm and
incorporated into the cell wall. Echinocandins inhibit the activity o the glucan synthase complex,
resulting in loss o the structural integrity o the cell wall. A subunit o glucan synthase desig-
nated Fks1p is thought to be the target o the echinocandin. Mutations in Fks1p, coded or by
FSK1, cause resistance to echinocandins.

b. What drug interactions are o concern while this patient is taking voriconazole?
Voriconazole is metabolized by, and inhibits, CYPs 2C19, 2C9, and 3A4. Drug
interactions with voriconazole are summarized in ables 43-2, 43-3, 43-4, and 43-5.
c. What other untoward e ects should this patient be warned o ?
Voriconazole is contraindicated in pregnancy (Category D). Voriconazole also
causes a prolongation o the Q c interval, which can become signi cant in patients
with other risk actors or torsades de pointes.

CASE 43 4
An in ectious disease consultant has suggested that the patient in Case 43-3 could be
treated with caspo ungin.
a. What is caspo ungin and what is its role in the treatment o esophageal candidiasis?
Caspo ungin belongs to the Echinocandin class o anti ungal agents. Caspo ungin is
approved or initial therapy o deeply invasive candidiasis and as salvage therapy or
patients with invasive aspergillosis who are ailing or intolerant o approved drugs such
as amphotericin B or voriconzaole. Caspo ungin is approved or esophageal candidiasis.
b. What is the mechanism o action o the Echinocandin class o anti ungal agents?
Echinocandins inhibit the activity o the glucan synthase complex, resulting in loss
o the structural integrity o the ungal cell wall, osmotic instability, and cell death
(see Figure 43-3).

CASE 43 5
A 47-year-old man has developed an irritated and itchy beard. T e cause is diagnosed
as tinea ( richophyton spp.). He is being treated with griseo ulvin orally.
a. What is the mechanism o action o griseo ulvin and why is it e ective in
treating this patient?
Griseo ulvin inhibits ungal microtubule unction and thereby disrupts assembly
o the mitotic spindle with inhibition o ungal mitosis. It is deposited in keratin
(Continued)
593
SECTION VII Chemotherapy of Microbial Diseases

precursor cells; when these cells di erentiate, the drug is tightly bound to, and per-
sists in keratin, providing prolonged resistance to ungal in ection. T us, the new
growth o hair or nails is the rst to become ree o disease. As the ungus-containing
keratin is shed, it is replaced by normal tissue.
b. T ere are numerous anti ungal creams available, why not use one o these?
opical treatment is use ul in many super cial ungal in ections, that is, those
con ned to the stratum corneum, squamous mucosa, or cornea. opical admin-
istration o anti ungal agents usually is not success ul or mycoses o the nails
(onychomycosis) and hair (tinea capitis), and has no place in the treatment o
subcutaneous mycoses, such as sporotrichosis and chromoblastomycosis.
c. What are the untoward e ects o griseo ulvin therapy?
T e incidence o serious reactions due to griseo ulvin is very low. Headache,
which may be severe, usually disappears as therapy is continued. Hepatotoxicity
has been reported. Leukopenia and neutropenia warrant the monitoring o weekly
blood studies during the rst month o therapy.
Griseo ulvin induces hepatic CYPs and may increase the metabolism o war arin
and may reduce the ef cacy o low-estrogen oral contraceptive agents.

CASE 43 6
A 36-year-old woman has developed vaginal candidiasis and is being treated with
miconazole vaginal suppositories.
a. What are the advantages o using topical anti ungal creams?
Vaginal creams, suppositories, and tablets, e ective or the treatment o vaginal
candidiasis, are all used once a day or 1 to 7 days, pre erably at bedtime to acilitate
retention. Approximately 3 to 10% o the vaginal dose is absorbed. T e most common
side e ect is vaginal burning or itching. A male sexual partner may experience mild
penile irritation.

KEY CONCEPTS
Anti ungal agents have 3 mechanisms o action: inhibit synthesis o cell wall
components, inhibit synthesis o nucleic acids, or inhibit microtubule/mitotic
spindle unction.
Amphotericin B comes in 4 ormulations that have speci c advantages and
disadvantages.
Imidazoles and triazoles are anti ungal agents that share the same spectrum o
activity and mechanism o action.
T e azoles interact with hepatic CYPs as substrates and inhibitors, and drugdrug
interactions may limit their use in certain patients (see ables 43-2, 43-3, 43-4,
and 43-5).
T e echinocandins are well-tolerated anti ungal agents, and are used mainly or
invasive candidiasis and aspergillosis.
Griseo ulvin therapy is used or mycotic disease o the skin, hair, and nails due
to Microsporum, richophyton, or Epidermophyton.
opical anti ungal treatment is use ul or super cial in ections con ned to the
stratum corneum, squamous mucosa, or cornea.
Anti ungal agents are ormulated in creams or vaginal use to treat vaginal
candidiasis.

594
Antifungal Agents CHAPTER 4 3

SUMMARY QUIZ

QUESTION 43-1 A 43-year-old man is being treated with amphotericin B or


esophageal candidiasis. Which o the ollowing serum laboratory tests should
be monitored closely?
a. Bilirubin
b. Cholesterol
c. Glucose
d. Creatinine
e. Sodium

QUESTION 43-2 A 56-year-old man with cryptococcal meningitis is being treated with
ucytosine and amphotericin B. Flucytosine is metabolized to 5- urouracil by ungal
cells which inhibits DNA synthesis. T e selectivity o ucytosine is due to a lack o
which enzyme in mammalian cells:
a. Cytosine deaminase
b. N-acetyltrans erase
c. Fluorine oxidase
d. Uracil phosphoribosyl trans erase
e. T ymidylate synthetase

QUESTION 43-3 A 48-year-old woman with a history o atrial brillation is being


treated with war arin. She has recently developed coccidiodal meningitis and treat-
ment with uconazole is started. A potential drugdrug interaction that may result in
elevated plasma war arin concentration is likely because uconazaole inhibits
a. CYP3A4.
b. CYP2D6.
c. CYP2C9.
d. CYP2C19.
e. CYP4.

QUESTION 43-4 A 38-year-old man is being treated with voriconazole or invasive


aspergillosis. His dose should be adjusted i it is determined that he has this disease:
a. Renal ailure
b. Hepatic cirrhosis
c. Rheumatoid arthritis
d. ype 2 diabetes mellitus
e. Colon cancer

QUESTION 43-5 A 69-year-old man with onychomycosis is being treated with griseo-
ulvin. His treatment has been going on or 1 month and he has been told that it may
last or 1 year. Although he has had no serious side e ects, his compliance is waning.
reatment o onchomycosis with griseo ulvin is prolonged because it is
a. absorbed slowly.
b. metabolized rapidly.
c. metabolized completely and the metabolites are inactive.
d. excreted rapidly in the urine.
e. bound to keratin and requires shedding o ungal-containing keratin.

595
SECTION VII Chemotherapy of Microbial Diseases

QUESTION 43-6 What is an alternative therapy or the patient in question 43-5?


a. Amphotericin B
b. Flucytosine
c. olna ate cream
d. erbina ne
e. Undecylenic acid

SUMMARY QUIZ ANSWER KEY

QUESTION 43-1. Answer is d. Azotemia occurs in 80% o patients who receive


C-AMB or deep mycoses. Lipid ormulations are less nephrotoxic. Nephrotoxicity is
dose-dependent, usually transient, and increased by concurrent therapy with other
nephrotoxic agents.
QUESTION 43-2. Answer is a. T e selective action oucytosine is due to the lack
o cytosine deaminase in mammalian cells, which prevents the metabolism to
5- urouracil.
QUESTION 43-3. Answer is c. Fluconazole is an inhibitor o CYP3A4, CYP2C9, and
CYP2C19 (see able 43-3). War arin is metabolized by CYP2C9 (see Chapter 19).
T e inhibition o CYP2C9 would cause an increase in war arin plasma concentrations
unless the war arin dose is adjusted accordingly.
QUESTION 43-4. Answer is b. Voriconazole is metabolized primarily by CYP2C19
and to a lesser extent CYP2C9 and CYP3A4. Patients with mild-to-moderate cirrhosis
should receive the same loading dose o voriconazole but hal o the maintenance dose.
T ere are no data to guide dosing in patients with severe hepatic insuf ciency.
QUESTION 43-5. Answer is e. Griseo ulvin is deposited in keratin precursor cells;
when these cells di erentiate, the drug is tightly bound to, and persists in, keratin, pro-
viding prolonged resistance to ungal invasion. For this reason, the new growth o the
hair or nails is the rst to become ree o disease. As the ungus-containing keratin is
shed, it is replaced by normal tissue.
QUESTION 43-6. Answer is d. erbina ne accumulates in skin, nails, and at. It is
somewhat more e ective or onychomycosis than itraconazole. Duration o treatment
varies with the site being treated but typically is 6 to 12 weeks. Ef cacy in onychomy-
cosis can be improved by the simultaneous use o amoro ne 5% nail lacquer.

SUMMARY: ANTIFUNGAL DRUGS


TOXICITY
CLASS AND UNIQUE; CLINICALLY
SUBCLASSES NAME CLINICAL USES RESISTANCE COMMON IMPORTANT
Amphotericin Amphotericin B Clinical activity against Candida Some Candida and In usion-related Dose-dependent
(C-AMB) spp., Cryptococcus neo ormans, Aspergillus spp. are ever and chills renal toxicity;
Amphotericin B Blastomyces dermatitidis, resistant, but signi cant ABCD > C-AMB > less with lipid
colloidal dispersion Histoplasma capsulatum, resistance is rare ABLC > L-AMB ormulations
(ABCD) Sporothrix schenckii, Coccidiosis
Amphotericin B spp., Paracoccidioides braziliensis,
liposomal (L-AMB) Aspergillus spp., Penicillium
Amphotericin B lipid marnef ei, and the agents o
complex (ABLC) mucormycosis
Flucytosine ANCOBON Clinically use ul against Resistance during therapy Nausea, vomiting, Depression o bone
Cryptococcus neo ormans, (secondary resistance) diarrhea, marrow leading
Candida spp., and the agents o is an important cause o and severe to leukopenia and
chromoblastomycosis therapeutic ailure when entero-colitis throbocytopenia
ucytosine is used alone
or cryptococcosis and
candidiasis
(Continued)
596
Antifungal Agents CHAPTER 4 3

TOXICITY
CLASS AND UNIQUE; CLINICALLY
SUBCLASSES NAME CLINICAL USES RESISTANCE COMMON IMPORTANT

Imidazoles and Itraconazole In ections due to B. Resistance emerges Drug interactions, Doses o 300 mg
triazoles dermatitides, H. capsulatum, P. gradually during see Tables 43-2, twice daily have
braziliensis, and C. immitis prolonged azole therapy 43-3, 43-4, and led to adrenal
43-5 insu ciency and
hypertension

Fluconazole Orophyarngeal candidiasis, Category C agent


uncomplicated vaginal to be avoided in
candidiasis, cryptococcal pregnancy unless
meningitis, coccidiodal bene t justi es risk
memingitis

Voriconazole Invasive apergillosis, Contraindicated


esophageal canididiasis, initial in pregnancy
treatment o candidenia (Category D)
Hepatotoxicity
has been reported
and liver unction
should be monitored
Prolongation o QTc
interval in patients
at risk or torsade de
pointes

Posaconazole Similar to uconazole, but Drug inter- Not known to


uconazole is the pre erred actions, see prolong QTc interval,
drug due to sa ety, cost, and Tables 43-2, 43-3, but should not
experience 43-4, and 43-5; be administered
GI distress with drugs that are
CYP3A4 substrates
and drugs that
prolong QTc interval

Imidazoles and Clotrimazole, cream, Cutaneous dermatophyte Recurrences common in Local irritation, No serious systemic
triazoles or lotion, powder, in ections, including cutaneous all in ections including vaginal toxicities
topical use aerosol solution, and vulvovaginal candidiasis burning; sexual
and solution; vaginal partner may
cream or tablets; experience penile
troches irritation

Econazole Similar to miconazole Local irritation

Miconazole cream, Treatment o tinea pedis, tinea Vaginal irritation


lotion, powder, cruris, tinea versicolor, and
aerosol solution, vulvovaginal candidiasis
and aerosol powder;
vaginal cream

Terconazole Vaginal suppository and


vaginal cream or treatment o
vaginal candidiasis

Butoconazole Vaginal cream similar to Slower response in


clotrimazole pregnancy requires
a 6-day course in
second or third
trimester

Tioconazole Ointment used or treatment o


vulvovaginal candidiasis
(Continued)

597
SECTION VII Chemotherapy of Microbial Diseases

TOXICITY
CLASS AND UNIQUE; CLINICALLY
SUBCLASSES NAME CLINICAL USES RESISTANCE COMMON IMPORTANT

Oxiconazole Cream and lotion available or


common dermatophytes

Sulconazole Solution or cream available or


common dermatophytes

Sertaconzole Cream or treatment o tinea


pedis

Ketoconazole Cream, gel, oam, shampoo or


common skin dermatophytes,
tinea versicolor, and seborrheic
dermatitis

Echinocandins Caspo ungin Deeply invasive candidiasis Resistance can be Phlebitis at


and as salvage therapy or selected or Candida in usion site
aspergillosis albicans during
Esophogeal candidiasis prolonged therapy

Mica ungin Deeply invasive candidiasis Resistance can be Mild inhibitor


Esophageal candidiasis selected or Candida o CYP 3A4
albicans during
prolonged therapy

Andula ugin Candidemia in non- Resistance can be


neutropenic patients selected or Candida
Esophogeal candidiasis albicans during
prolonged therapy

Griseo ulvin Micro-sized powder; Mycotic diseases o the Headache usually Induction o CYP
Ultra-micro-sized skin, hair, and nails due to disappears and increases the
powder Microsporum, Trichophyton, or as therapy is rate o metabolism
Epidermophyton; itraconazole stopped o war arin requiring
or terbina ne is more ef ective an adjustment o
or onychomycosis war arin dose

Terbina ne LAMASIL, others Onychomycosis, tinea capitus, Low incidence Not recommended
ring worm o GI distress, or treatment
Cream or spray is used to treat headache, or rash o patients with
tinea corporis, tinea cruris, and azotemia or hepatic
tinea pedis ailure
Pregnancy Category B

Ciclopirox PENLAC NAIL Available as a cream, gel, Hypersensitivity No systemic


olamine LACQUER, others suspension, and lotion or accumulation
treatment o cutaneous
candidiasis, tinea corporis,
cruris pedis, and versicolor;
nail lacquer is available or
onychomycosis

Haloprogin HALOTEX (Not Cream or solution is used to


available in the treat tinea pedis, cruris, tinea
United States) corporis, tinea manuum, and
tinea versicolor

Tolna tate AFTATE, TINACTIN Cream, gel, powder, aerosol No toxic or allergic
powder, topical solution, reaction have
topical aerosol liquid are used been noted
or treatment o tinea pedis
(Continued)

598
Antifungal Agents CHAPTER 4 3

TOXICITY
CLASS AND UNIQUE; CLINICALLY
SUBCLASSES NAME CLINICAL USES RESISTANCE COMMON IMPORTANT
Na ti ne NAFTIN Cream or gel ef ective or Local irritation
treatment o tinea cruris and and contact
tinea corporis dermatitis

Butena ne MENTAX, LOTRAMIN Similar to na ti ne


ULTRA

Nystatin MYCOSTATIN, Available in preparations or Bitter taste;


NILSTAT, others treatment o cutaneous or adverse ef ects
vaginal candidiasis are uncommon
Suspension is available or
treatment o oral candidiasis
A liposomal ormulation is
in clinical trials or systemic
candidemia

Undecylenic DESENEX, others Available as a cream, powder,


acid Calcium spray powder, soap, and liquid
undecylenate or the treatment o various
is available as a dermatomycoses, especially
powder tinea pedis

Whit elds An ointment that combines Mild local


Ointment the ungistatic action irritation
o benzoic acid and the
keratolytic action o salicylic
acid; used to treat tinea capitis

599
CHAPTER

44 Antiviral Agents and Treatment


of HIVInfection
T is chapter is a composite o Chapters 58 and 59 in Goodman and Gilmans T e Phar-
macological Basis of T erapeutics, 12th Edition. T e material presented here will be
more use ul a er having a basic understanding o the material presented in these chap-
ters. T is chapter does not contain detailed in ormation about all antiviral or antiretro-
viral agents. For the details o agents not discussed here, see Chapters 58 and 59 in the
12th Edition o Goodman and Gilmans T e Pharmacological Basis of T erapeutics. In
addition to the material presented here, the 12th Edition contains:
able 58-2 which provides the nomenclature o antiviral agents
Figures 58-2, 58-4, 58-6, 59-2, 59-4, and 59-6 which show the chemical structures o
each o the drugs discussed in this chapter
At the beginning o Chapter 59 in Goodman and Gilmans T e Pharmacological Basis
of T erapeutics, 12th Edition, there is an excellent overview o human immunode-
ciency disease (HIV) in ection and its treatment, the pathogenesis o HIV-related
disease, and the principles o HIV chemotherapy
able 59-1 which is a list o antiretroviral agents approved in the United States
ables 59-2, 59-3, and 59-4 which provide detailed in ormation on the pharmacoki-
netics o antiretroviral agents

LEARNING OBJECTIVES
Understand the treatment o herpes virus in ections and the use o antiherpes
drugs.
Know the treatment strategies or chronic hepatitis B and C in ections.
Understand the mechanisms o action and resistance, and the therapeutic use
o the anti-in uenza agents.
Know the principles o HIV chemotherapy.
Know the mechanisms o action and resistance, the untoward ef ects, and the
therapeutic uses o the drugs used to treat HIV in ections.

DRUGS INCLUDED IN THIS CHAPTER


GENERIC NAME TRADE NAME GENERIC NAME TRADE NAME
Antiherpes Agents Antihepatitis Agents

Acyclovir ZOVIRAX Ade ovir dipivoxil HEPSERA

Cido ovir VISTIDE Entecavir BARACLUDE

Penciclovir FAMVIR, others; Famciclovir which is Inter eron al a-N1 WELLFERONa


de-esteri ed to penciclovir

Foscarnet FOSCAVIR, others Inter eron al a-N3 ALFERON N

Fomivirsen VITRAVENE Inter eron al acon-1 INFERGEN

Ganciclovir CYTOVENE, VITRASERT, others Inter eron al a-2B INTRON A

Idoxuridine HERPLEX, DENDRID Inter eron al a-2A ROFERON A

Docosanol Lamivudine EPIVIR, others


(Continued)

600
Antiviral Agents and Treatment of HIV Infection CHAPTER 4 4

GENERIC NAME TRADE NAME GENERIC NAME TRADE NAME

Tri uridine VIROPTIC, others Peginter eron al a-2A PEGASYS

Valacyclovir VALTREX, others Peginter eron al a-2B PEG-INTRON

Valganciclovir VALCYTE, others

Anti inf uenza Agents Other Antiviral Agents

Amantadine SYMMETREL, others Ribavirin VIRAZOLE, REBETROL,


COPEGUS, others

Oseltamivir TAMIFLU Telbivudine TYZEKA

Rimantadine FLUMADINE, others

Zanamivir RELENZA Imiquimod ALDARA

Nucleoside Reverse Transcriptase Inhibitors (NRTIs) Protease Inhibitors

Zidovudine RETROVIR, othersb Saquinavir INVIRASE

Didanosine VIDEX, VIDEC EC, others Indinavir CRIXIVAN

Stavudine ZERIT Ritonavir NORVIR

Zalcitabine HIVIDc Nel navir VIRACEPT

Lamivudine EPIVIRb Amprenavir AGENERASE, PROZEIa

Abacavir ZIAGENb Lopinavir KALETRA, ALUVIAc

Teno ovir disoproxil VIREADb Atazanavir REYATAZ, ZRIVADA

Emtricitabine EMTRIVAb Fosamprenavir LEXIVA, TELZIR

Non nucleoside Reverse Transcriptase Inhibitors (NNRTIs) Tipranavir APTIVUS

Nevirapine VERAMUNE Darunavir PREZISTA

E avirenz SUSTIVA, STORCRINb Entry Inhibitors

Delavirdine RESCRIPTOR En uvirtide FUZEON

Etravirine INTELENCE Maraviroc SELZENTRY, CELSENTRI

Integrase Inhibitor

Raltegravir ISENTRESS
a
Not currently approved in the United States.
b
A number o xed-dose combinations are available: zidovudine + lamivudine (COMBIVIR); zidovudine + lamivudine + abacavir
(TRIZIVIR); abacavir + lamivudine (EPZICOM); teno ovir + emtricitabine (TRUVADA); teno ovir + e avirenz + emtricitabine (ATRIPLA).
c
Lopinavir is available only as a xed-dose combination with ritonavir (KALETRA/ALUVIA).

MECHANISMS OF ACTION AND RESISTANCE OF ANTIVIRAL DRUGS


DRUG MECHANISM OF ACTION MECHANISM OF RESISTANCE
Acyclovir See Figures 44-1 and 44-2; inhibition o DNA synthesis Impaired production o thymidine kinase, altered
by interaction with HSVthymidine kinase and DNA thymidine kinase substrate, or altered DNA
polymerase polymerase

Cido ovir Inhibits viral DNA synthesis by slowing and eventually Resistance in cytomegalovirus (CMV) is due to
terminating chain elongation; the diphosphate acts as mutations in DNA polymerase
a competitive inhibitor o deoxycytidine triphosphate
(dCTP) and as alternative substrate or DNA polymerase

601
SECTION VII Chemotherapy of Microbial Diseases

DRUG MECHANISM OF ACTION MECHANISM OF RESISTANCE

Penciclovir Inhibits viral DNA synthesis by competitive inhibition o Mutations in thymidine kinase or viral
viral DNA polymerase (see Figures 44-1 and 44-2) DNA polymerase

Foscarnet Inhibits viral nucleic acid synthesis by interacting with Mutations in herpes virus DNA polymerase
herpes virus DNA polymerase or HIVreverse transcriptase
(see Figure 44-1)

Fomivirsen First approved antisense therapy or viral in ections;


inhibits CMVreplication through sequence-speci c and
nonspeci c mechanisms including virus binding to cells

Ganciclovir Competitive inhibitor o deoxyguanosine triphosphate CMVresistance is due to either mutations in viral
incorporation into DNA and inhibition o viral DNA phosphotrans erase which reduces intracellular
polymerase (see Figures 44-1 and 44-2) ganciclovir phosphorylation or mutations in viral
DNA polymerase

Idoxuridine The triphosphate inhibits viral DNA synthesis and is


incorporated into viral DNA

Tri uridine The monophosphate inhibits thymidylate synthesis Tri uridine-resistant HSVhave altered thymidine
and the triphosphate inhibits thymidine triphosphate substrate speci city
incorporation into DNA

Ribavirin Alteration o cellular nucleotide pool and inhibition Not documented, but cells that do not
o viral mRNA synthesis phosphorylate ribavirin to its active orm have
been reported

Ade ovir dipivoxil Competitive inhibition o viral DNA polymerase and Point mutations in hepatitis B virus (HBV)
reverse transcriptases polymerase

Entecavir Inhibition o HBVpolymerase (reverse transcriptase) Mutation in HIVreverse transcriptase

Inter eron Activates JAK-STAT pathway that leads to synthesis o Block production or activity o inter eron-inducible
over 2 dozen proteins that contribute to antiviral activity proteins
at various stages o viral penetration (see Figure 44-3)

Lamivudine Inhibitor o DNA polymerase/reverse transcriptase o HIV Point mutations in HBVDNA polymerase
and HBV(see Figures 44-4 and 44-5)

Amantadine Inhibits viral replication by inhibition o viral uncoating Mutation in the RNA sequence encoding or
(see Figure 44-1) the M2 protein transmembrane domain

Oseltamivir Selective inhibitor o viral neuraminidase which leads to Mutations in viral hemagglutinin and/or
virus aggregation at the cell sur ace and reduced virus neuraminidase
spread (see Figure 44-1)

Rimantadine Inhibits viral replication by inhibition o viral uncoating Mutation in the RNA sequence encoding or the
(see Figure 44-1) M2 protein transmembrane domain

Zanamivir Inhibition o viral neuraminidase which leads to virus Mutations in viral hemagglutinin and/or
aggregation at the cell sur ace and reduced virus spread neuraminidase
(see Figure 44-1)

Telbivudine Inhibition o HBVDNA polymerase (reverse transcriptase) Mutation o HBVDNA polymerase


by competing with the natural substrate, thymidine
5-triphosphate (see Figures 44-4 and 44-5)

Teno ovir disoproxil Reverse transcriptase inhibitor o HIVand HBV(see Mutations in reverse transcriptase
Figures 44-4 and 44-5)

Imiquimod Induces cytokines and chemokines with antiviral and None known
immunomodulating e ects

Zidovudine Inhibition o nucleoside reverse transcriptase (see Figures Mutations in reverse transcriptase
44-4 and 44-5)
(Continued)

602
Antiviral Agents and Treatment of HIV Infection CHAPTER 4 4

DRUG MECHANISM OF ACTION MECHANISM OF RESISTANCE


Didanoside Reverse transcriptase inhibitor (see Figures 44-4 Mutations in reverse transcriptase
and 44-5)

Stavudine Reverse transcriptase inhibitor (see Figures 44-4 Mutations in reverse transcriptase
and 44-5)

Abacavir Reverse transcriptase inhibitor (see Figures 44-4 Mutations in reverse transcriptase
and 44-5)

Emtricitabine Reverse transcriptase inhibitor (see Figures 44-4 Mutations in reverse transcriptase
and 44-5)

Saquinavir HIVprotease inhibitor (see Figures 44-5 and 44-6) Accumulation o multiple resistance mutations

Indinavir HIVprotease inhibitor (see Figures 44-5 and 44-6) Accumulation o multiple resistance mutations

Ritonavir HIVprotease inhibitor (see Figures 44-5 and 44-6) Mostly used as a pharmacokinetic enhancer
(CYP3A4 inhibitor) and at the low doses used is not
known to induce resistance

Nel navir Nonpeptidic protease inhibitor (see Figures 44-5 Mutation in HIVprotease
and 44-6)

Amprenavir HIVprotease inhibitor (see Figures 44-5 and 44-6) Mutation in HIVprotease

Lopinavir HIVprotease inhibitor (see Figures 44-5 and 44-6) Mutation in HIVprotease

Atazanavir HIVprotease inhibitor (see Figures 44-5 and 44-6) Mutation in HIVprotease

Tipranavir HIVprotease inhibitor (see Figures 44-5 and 44-6) Resistance requires accumulation o multiple
mutations

Darunavir HIVprotease inhibitor (see Figures 44-5 and 44-6) Resistance requires accumulation o multiple
mutations

Nevirapine Non-nucleoside reverse transcriptase inhibitor (see Mutation in reverse transcriptase; cross-resistance
Figures 44-5 and 44-7) extends to e avirenz and delavirdine

E avirenz Non-nucleoside reverse transcriptase inhibitor (see Mutation in reverse transcriptase; cross-resistance
Figures 44-5 and 44-7) extends to nevirapine and delavirdine

Delavirdine Non-nucleoside reverse transcriptase inhibitor (see Mutation in reverse transcriptase; cross-resistance
Figures 44-5 and 44-7) extends to nevirapine and e avirenz

Etravirine Non-nucleoside reverse transcriptase inhibitor (see Mutation in reverse transcriptase; activity o
Figures 44-5 and 44-7) etravirine is not a ected by mutations that con er
high-level resistance to e avirenz, delavirdine,
and nevirapine

En uvirtide Prevents ormation o a complex critical or membrane Retains activity against viruses that have become
usion and viral entry into the host cell (see Figures 44-5 resistant to antiviral agents o other classes
and 44-8)

Maraviroc A chemokine receptor antagonist that binds to host HIVthat is predominantly CCR5-tropic shi ts
CCR5 receptor to block binding o viral gp120 (see tropism to CXCR4; or there is a mutation in
Figures 44-5 and 44-8) gp120 that allows virus binding in the presence
o inhibitor

Raltegravir Blocks the catalytic activity o HIV-encoded integrase, Mutations in the integrase gene
thus preventing the integration o virus DNA into the
host chromosome (see Figures 44-5 and 44-9)

603
SECTION VII Chemotherapy of Microbial Diseases

A a tta chme nt re le a s e

budding

uncoa ting
a nd tra ns fe r of
vira l DNA to a s s e mbly
hos t nucle us of virion

vira l
DNA prote in
synthe s is
tra ns cription by hos t ce ll
into vira l ribos ome s tructura l
synthe s is mRNA prote ins
of vira l vira l
DNA e nzyme s

re gula tory
prote ins
inhibition of vira l DNA polyme ra s e
a cyclovir, vida ra bine, fos ca rne t, ga nciclovir

B binding to
re le a s e
ce ll s urfa ce
za na mivir
HEMAGGLUTININ
os e lta mivir
e ndocytos is
budding

e ndos ome
H+
M2 prote in intra -
ce llula r
vRNA
tra ns port
RNA re plica tion
a nd a s s e mbly
HEMAGGLUTININ p
fus ion
vRNA cRNA R NA
p
R NA s tructura l
uncoa ting prote ins
M2 R NAp
prote in nons tructura l
synthe s is prote ins
mRNA
a ma nta dine
rima nta dine
ribavirin
FIGURE 44-1 Replicative cycles o DNA (A) and RNA (B) viruses. The replicative cycles o herpes-
virus (A) and in uenza (B) are examples o DNA-encoded and RNA-encoded viruses, respectively.
Sites o action o antiviral agents also are shown. cDNA, complementary DNA; cRNA, comple-
mentary RNA; DNAp, DNA polymerase; mRNA, messenger RNA; RNAp, RNA polymerase; vRNA,
viral RNA. The symbol (T-line) indicates a block to virus growth. A. Replicative cycles o herpes
simplex virus, a DNA virus, and the probable sites o action o antiviral agents. Herpesvirus replica-
tion is a regulated multistep process. A ter in ection, a small number o immediate-early genes
are transcribed; these genes encode proteins that regulate their own synthesis and are respon-
sible or synthesis o early genes involved in genome replication, such as thymidine kinases, DNA
polymerases, etc. A ter DNA replication, the bulk o the herpesvirus genes (called late genes)
are expressed and encode proteins that either are incorporated into or aid in the assembly o
progeny virions. B. Replicative cycles o in uenza, an RNA virus, and the loci or e ects o antiviral
agents. The mammalian cell shown is an airway epithelial cell. The M2 protein o in uenza virus
allows an in ux o hydrogen ions into the virion interior, which in turn promotes dissociation o
the RNP (ribonuclear protein) segments and release into the cytoplasm (uncoating). In uenza
virus mRNA synthesis requires a primer cleared rom cellular mRNA and used by the viral RNAp
complex. The neuraminidase inhibitors zanamivir and oseltamivir speci cally inhibit release o
progeny virus. Small capitals indicate virus proteins.

604
Antiviral Agents and Treatment of HIV Infection CHAPTER 4 4

He rpe s
s implex Cytopla s m

O
H2 C

CH2

O O

O P O O P O

O O
Nucle us
HO O
O H2 C
Ce ll Acyclovir CH2
me mbra ne
He rpe s thymidine O
kina s e
OH O P O

O O
O
O P O
P O O P O
O O CH2
P O O
Acyclovir
monophos pha te O O O
He rpe s DNA O P O
polyme ra s e
O

P P O
O
Acyclovir
diphos pha te P P P O
O
Acyclovir
triphos pha te

FIGURE 44-2 Mechanism o action o acyclovir in cells in ected by herpes simplex virus. A herpes simplex virion is shown attaching to a
susceptible host cell, using its envelope with the cell membrane, and releasing naked capsids that deliver viral DNA into the nucleus, where it
initiates synthesis o viral DNA. Acyclovir molecules entering the cell are converted to acyclovir monophosphate by virus-induced thymidine
kinase. Host-cell enzymes add 2 more phosphates to orm acyclovir triphosphate, which is transported into the nucleus. A ter the herpes DNA
polymerase cleaves pyrophosphate rom acyclovir triphosphate (indicated by the blue arrow in the inset), viral DNA polymerase inserts acyclovir
monophosphate rather than 2'-deoxyguanosine monophosphate into the viral DNA (indicated by black arrows in the inset). Further elongation o
the chain is impossible because acyclovir monophosphate lacks the 3' hydroxyl group necessary or the insertion o an additional nucleotide, and
the exonuclease associated with the viral DNA polymerase cannot remove the acyclovir moiety. In contrast, ganciclovir and penciclovir have a
3' hydroxyl group; there ore, urther synthesis o viral DNA is possible in the presence o these drugs. Foscarnet acts at the pyrophosphate-binding
site o viral DNA polymerase and prevents cleavage o the pyrophosphate rom nucleoside triphosphates, thus stalling urther primer template
extension. The blue bands between the viral DNA strands in the inset indicate hydrogen bonding o the base pairs. (Adapted rom Bal our HH.
Antiviral drugs. N Engl J Med. 1999, 340:12551268. )

CASE 44-1
A 47-year-old man is immunocompromised because o the medications he is taking to
prevent a heart transplant rejection. He develops a herpes simplex virus (HSV) in ec-
tion o his lower lip.
a. What are his treatment options?
In ection with herpes simplex virus 1 (HSV-1) typically causes diseases o the
mouth, ace, skin, esophagus, or brain. Herpes simplex virus 2 (HSV-2) causes
in ections o the genitals, rectum, skin, hands, or meninges. Acyclovir and its ester
prodrug, valacyclovir, are the prototypes o a class o drugs that are phosphorylated
intracellularly to become inhibitors o viral DNA synthesis. Other drugs that use
the same strategy include penciclovir, its prodrug amciclovir, and ganciclovir, and
its prodrug valganciclovir. Cido ovir and oscarnet are available to treat acyclovir-
resistant HSV in ections. Idoxuridine is available or the topical (ophthalmic)
(Continued)
605
SECTION VII Chemotherapy of Microbial Diseases

A B1 B2
Virus e s
A. DNA
B. RNA
1. orthomyxovirus e s a nd re trovirus e s
2. picornavirus e s a nd mos t RNA virus e s
uncoa ting
IFN Effe c ts
1 inhibitio n o f trans c riptio n
a ctiva te s Mx prote in
blocks mRNA synthe s is
nucle us
2 inhibitio n o f trans latio n
a ctiva te s me thyla s e, the re by re ducing
1
mRNA ca p me thyla tion

a ctiva te s 25 oligoa de nyla te synthe ta se


25A inhibits mRNA s plicing
golgi a nd a ctiva te s RNa s e L cle ave s
vira l RNA
3
2
a ctiva te s prote in kina s e P 1 blocks
glycoprote ins prote ins e IL-2a function inhibits initia tion
of mRNA tra ns la tion

a s s e mbly a ctiva te s phos phodie s te ra s e blocks


tRNA function

3 inhibitio n o f po s t-trans latio nal pro c e s s ing


inhibits glycosyltra ns fe ra s e, the re by re ducing
prote in glycosyla tion
4
4 inhibitio n o f virus maturatio n
inhibits glycosyltra ns fe ra s e, the re by re ducing
glycoprote in ma tura tion
5 re le a s e 5 inhibitio n o f virus re le as e
ca us e s me mbra ne cha nge s blocks
budding

FIGURE 44-3 Inter eron-mediated antiviral activity occurs via multiple mechanisms. The binding
o inter eron (IFN) to speci c cell sur ace receptor molecules signals the cell to produce a series o
antiviral proteins. The stages o viral replication that are inhibited by various IFN-induced antiviral
proteins are shown. Most o these act to inhibit the translation o viral proteins (mechanism 2),
but other steps in viral replication also are a ected (mechanisms 1, 3, and 4). The roles o these
mechanisms in the other actions o IFNs are under study. 2'5'A, 2'-5'-oligoadenylates; eIF-2, protein
synthesis initiation actor; IFN, inter eron; mRNA, messenger RNA; Mx, IFN-induced cellular protein
with antiviral activity; RNase L, latent cellular endoribonuclease; tRNA, trans er RNA. (Modif ed rom
Baron, S., Coppenhaver, D.H., and Dianzani, F., et al. Introduction to the inter eron system. In, Inter erons:
Principles and Medical Applications. [Baron, S., Dianzani, F., Stanton, G.J., et al., eds.] University o Texas
Medical Branch Dept. o Microbiology, Galveston, TX, 1992, pp. 115. With permission.)

treatment o HSV keratitis. ri uridine is available or the topical treatment o


keratoconjunctivitis and recurrent epithelial keratitis owing to HSV-1 and HSV-2.
opical tri uridine appears to be e ective in some patients with acyclovir-resistant
HSV cutaneous in ections.
b. What is the mechanism o action o acyclovir?
T e stages o viral replication and possible targets o action o antiviral agents are
shown in able 44-1. Acyclovir inhibits viral DNA synthesis via a mechanism out-
lined in Figure 44-2. Its selectivity o action depends on interaction with 2 viral
proteins: HSV thymidine kinase and DNA polymerase.
c. What are the possible mechanisms o acyclovir resistance?
Acyclovir resistance in HSV has been linked to one o three mechanisms (see able
MECHANISMS OF AC ION AND RESIS ANCE OF AN IVIRAL DRUGS):
(Continued)
606
Antiviral Agents and Treatment of HIV Infection CHAPTER 4 4

1. Triphos pha te compe te s O


with na tive nucle otide s CH3
(s hown is zidovudine 5 triphos pha te ) HN
Da ughte r DNA s tra nd
O O O
O N

O P O P O P HO O Pa re nt RNA s tra nd

O O O

N N+ N

2. Incorpora tion
a nd cha in
te rmina tion

O
CH3
HN
HIV reve rs e tra ns cripta s e

O N
O
O P O

N N+ N

Zidovudine Dida nos ine S tavudine Za lcita bine La mivudine


O O O NH2
NH2
CH3 H CH3
HN HN N N N
N
N OH
O N N O N O N
HO CH2 O N O CH2
HO CH2 HO CH2
HO CH2 5 2
O O O O
H S
H

N3

Aba cavir Te nofovir dis oproxil Emtricita bine


NH2
N H2 N N O
N
HN
N N N N O
N O O F
OH
N O P O O O S
H2 N N
O O O
CH3
CH2 OH
O
FIGURE 44-4 Structures and mechanism o nucleoside and nucleotide reverse transcriptase inhibitors.

impaired production o viral thymidine kinase, altered thymidine kinase substrate


speci city (eg, phosphorylation o thymidine but not acyclovir), or altered viral
DNA polymerase. T e most common resistance mechanism in clinical HSV isolates
is absent or de cient viral thymidine kinase activity.
(Continued)

607
SECTION VII Chemotherapy of Microbial Diseases

Ma raviroc

Ce ll me mbra ne Nucle us

Che mokine Cytopla s m As s e mbly Budding


Double - a nd
a nd CD4
Virion s tra nde d vRNA re le a s e
re ce ptors Hos t
ge nomic uninte gra te d
DNA chromos ome
Ma ture RNA Prote a s e
extra ce llula r inhibitors
RT RT
virion Re gula tion
RNa s e S tructura l prote ins
H gp120, p24,
p18, e tc .
Nucle oca ps id cDNA-RNA Provirus
gp160 complex Inte gra s e Ma tura tion
+
gp41 Re gula tory prote ins
Vira l
ta t, rev,
mRNA ne f, e tc .
Atta chme nt Pe ne tra tion Reve rs e Inte gra tion Tra ns cription Tra ns la tion
a nd fus ion a nd uncoa ting tra ns cription

HIV pa rticle
(ma ture virion)

Enfuvirtide Nucle os ide RT inhibitors , Ra lte gra vir


non-nucle os ide RT inhibitors
FIGURE 44-5 Replicative cycle o HIV-1 showing the sites o action o available antiretroviral agents. Available antiretroviral agents are shown
in blue. cDNA, complementary DNA; gp120 + gp41, extracellular and intracellular domains, respectively, o envelope glycoprotein mRNA, mes-
senger RNA; RNase H, ribonuclease H; RT, reverse transcriptase. (Adapted rom Hirsch MS and DAquila RT. Therapy or human immunodef ciency virus
in ection. N Engl J Med. 1993, 328:16861695.)

d. I he is treated with acyclovir, what adverse events should this patient be


warned about?
Oral acyclovir is associated with nausea, vomiting, diarrhea, rash, and headache.
T e principal dose-limiting toxicities o intravenous acyclovir are renal insu -
ciency and CNS side e ects. T e renal e ects are due to high urine concentrations
o the drug causing crystalline nephropathy. Neurotoxicity is mani ested by altered
sensorium, tremor, myoclonus, delirium, seizures, or extrapyramidal signs.

CASE 44-2
An otherwise healthy 76-year-old woman developed a rash on her side and back 5 days
ago. She now describes considerable pain in the area. Her physician suspects varicella
zoster virus (VZV) in ection, commonly known as shingles.
a. What are the treatment options or this woman?
T e 2 drugs most commonly used or VZV in ections are acyclovir and penciclovir,
or their prodrugs, valacyclovir, and amciclovir, respectively. Both drugs are e ec-
tive i started within 24 hours o the rash.
b. Her physician elects to use amciclovir. Is it likely to be e ective?
In immunocompetent adults with herpes zoster o less than or equal to 3 days
duration, amciclovir (500 mg 3 times a day or 10 days) is at least as e ective as
acyclovir (800 mg 5 times daily) in reducing healing time and zoster-associated
pain, particularly in those more than or equal to 50 years o age. T e act that the
rash started 5 days prior to the start o treatment makes it less likely that the treat-
ment will be e ective.
c. How is amciclovir absorbed and metabolized?
Famciclovir is well absorbed orally and is converted rapidly to penciclovir by
deacetylation. T e bioavailability o penciclovir is approximately 75% ollowing oral
administration o amciclovir.

608
Antiviral Agents and Treatment of HIV Infection CHAPTER 4 4

H O
N
N N N
H Tra ns ition s ta te pe ptidomime tic
O OH prote a s e inhibitor (s a quinavir)
O H
O NH
H2 N

H3 C CH3
CH3

N
HN
O CO

HIV prote a s e
(C 2 a xis of symme try)
Ga g or ga g/pol
pre curs or polype ptide


1 2

1 2

+
CH2
H2 N

+ pK2 = 11.0
H3 N CH C O C O

pK2 = 9.1 O pK1 = 1.8 O pK1 = 2.0
P he nyla la nine Proline

FIGURE 44-6 Mechanism o action o an HIVprotease inhibitor. Shown here is a phenylalanine-proline target peptide sequence
(in blue) or the protease enzyme (in black) with chemical structures o the native amino acids (in lower box) to emphasize
homology o their structures to that o saquinavir (at top).

CASE 44-3
A 23-year-old man with AIDS has begun to develop blurred vision in the le eye. T e diag-
nosis o cytomegalovirus (CMV) retinitis is made. He is started on intravenous oscarnet.
a. What treatment options, other than oscarnet, are available or the treatment o
CMV retinitis?
Other than oscarnet, the treatment options or CMV retinitis include ganciclovir,
omivirsen, and cido ovir. Fomivirsen is given by intravitreal injection or patients
intolerant o or unresponsive to other therapies.
(Continued)
609
SECTION VII Chemotherapy of Microbial Diseases

Da ughte r
DNA s tra nd

Pa re nt
RNA s tra nd

HIV reve rs e
tra ns cripta s e

F 3C
Cl
O

N O
H

Efavire nz binds to a
non-e s s e ntia l s ite dis ta nt from
the e nzyme ca ta lytic s ite.

CH3
O CH CH3
CH3 H
N H
CH3 S O2 N NH

N N N N N
N C
H N
O
Nevira pine De lavirdine

NH2
F3C
Cl N Br
O N

N O N N O
H H

Efavire nz Etravirine

N
FIGURE 44-7 Structures and mechanism o non-nucleoside reverse transcriptase inhibitors.

b. What is the mechanism o action o oscarnet?


Foscarnet inhibits viral nucleic acid synthesis by interacting directly with herpesvi-
rus DNA polymerase (see Figure 44-1). Foscarnet reversibly blocks the pyrophos-
phate binding site o the viral polymerase in a noncompetitive manner and inhibits
cleavage o pyrophosphate rom deoxynucleotide triphosphates.
c. Do patients develop resistance to oscarnet?
Resistant clinical isolates o herpesviruses have emerged during therapeutic use
o oscarnet and may be associated with poor clinical response. Herpesviruses
(Continued)
610
Antiviral Agents and Treatment of HIV Infection CHAPTER 4 4

HIV virus
me mbra ne

Nucle oca ps id
core

HIV fus ion 2 CCR5 che mokine re ce ptor binding s ta bilize s complex
prote in gp41 a nd a llows gp41-me dia te d fus ion of virus me mbra ne
with ta rge t ce ll me mbra ne .

HIV e nve lope N


prote in gp120 N Maraviro c
N

N
1 gp 120 a nchors F
HIV to ta rge t ce ll HN
by binding to CD4. F
O

3 Ma raviroc binds to CCR5, preve nting


gp120 binding, fus ion, a nd e ntry.

CD4 CCR5

FIGURE 44-8 Mechanism o action o the HIVentry inhibitor maraviroc.

Ralte g ravir O
H3 C OH F
N N N
H H
H3 C N N
O N
O H3 C CH3 Provira l DNA
O
Ra lte gra vir binds to HIV inte gra s e,
preve nts DNA s tra nd tra ns fe r.
Ra lte gra vir Ra lte gra vir

Hos t Hos t
chromos oma l chromos oma l
DNA DNA
HIV inte gra s e binds to
provira l 3 LTRs
FIGURE 44-9 Mechanism o action o the HIVintegrase inhibitor raltegravir.

611
SECTION VII Chemotherapy of Microbial Diseases

TABLE 44-1 Stages of Virus Replication and Possible Targets of Action of Antiviral Agents
STAGE OF REPLICATION CLASSES OF SELECTIVE INHIBITORS
Cell entry

Attachment Soluble receptor decoys, antireceptor antibodies, usion protein inhibitors

Penetration

Uncoating Ion channel blockers, capsid stabilizers

Release o viral genome

Transcription o viral genome a Inhibitors o viral DNA polymerase, RNA polymerase, reverse transcriptase, helicase, primase, or integrase

Transcription o viral messenger RNA

Replication o viral genome

Translation o viral proteins Inter erons, antisense oligonucleotides, ribozymes

Regulatory proteins (early) Inhibitors o regulatory proteins

Structural proteins (late)

Post-translational modi cations

Proteolytic cleavage Protease inhibitors

Myristoylation, glycosylation

Assembly o virion components Inter erons, assembly protein inhibitors

Release Neuraminidase inhibitors, antiviral antibodies, cytotoxic lymphocytes

Budding, cell lysis


a
Depends on speci c replication strategy o virus, but virus-speci ed enzyme required or part o process.

resistant to oscarnet have point mutations in the viral DNA polymerase and are
associated with 3- to 7- old reductions in oscarnet activity in vitro.
d. T is patient should be monitored or what major untoward e ects?
Foscarnets major dose-limiting toxicities are nephrotoxicity and symptomatic
hypocalcemia. Acute tubular necrosis, crystalline glomerulopathy, nephrogenic
diabetes insipidus, and interstitial nephritis have been described. Saline loading
may reduce the risk o nephrotoxicity.
Foscarnet is highly ionized at physiological pH, and metabolic abnormalities are
common. T ese include increases or decreases in calcium and phosphate, hypo-
magnesemia, and hypokalemia. Decreased serum ionized calcium may cause pares-
thesia, arrhythmias, tetany, seizures, and other CNS disturbances.

CASE 44-4
A 46-year-old woman is considered immunocompromised because o therapy or a
bone marrow transplant. In uenza A is identi ed in the community in which she lives.
Although she is asymptomatic, she is started on amantadine.
a. Why is she started on amantadine while she is asymptomatic?
Seasonal prophylaxis with amantadine is an alternative in high-risk patients i the
in uenza vaccine cannot be administered. Prophylaxis should be started as soon
as in uenza is identi ed in the community or region and should be continued
throughout the period o risk because any protective e ects are lost several days
af er cessation o therapy.
(Continued)
612
Antiviral Agents and Treatment of HIV Infection CHAPTER 4 4

b. What is the mechanism o action o amantadine?


Amantadine and rimantadine have 2 mechanisms o action (see Figure 44-1). T ey
inhibit an early step in viral replication, probably viral uncoating; or some strains
they also have an e ect on a late step in viral assembly, probably mediated through
altering hemagglutinin processing. T e primary locus o action is the in uenza A
virus M2 protein, an integral membrane protein that unctions as an ion channel.
c. Do patients treated with amantadine develop resistance to the drug?
T e utility o amantadine is limited by the development o resistance. Resistant
variants appear to be pathogenic and can cause disabling in uenza. Resistance to
amantadine results rom a mutation in the RNA sequence encoding or the M2
protein transmembrane domain. Resistant isolates typically appear in the treated
patient within 2 to 3 days o starting therapy.
d. What are the major limitations to the use o amantadine?
T e most common side e ects o amantadine are dose-related CNS and gastrointes-
tinal e ects. T ese include nervousness, light-headedness, di culty concentrating,
insomnia, nausea, and loss o appetite. High plasma concentrations are associated with
serious neurotoxic reactions including delirium, hallucinations, seizures, and coma.

CASE 44-5
A 37-year-old man with chronic hepatitis C virus (HCV) in ection is being treated with
inter eron and ribavirin.
a. What are this patients therapeutic options?
Current standard o care or treatment o HCV in ection is a combination o pegin-
ter eron alpha and ribavirin, which produces a high cure rate in selected virus
genotypes only. Recent advances in the development o highly e ective oral agents
include HCV protease inhibitors and polymerase inhibitors that are selective or
this pathogen. It is likely that treatment recommendations will change radically in
the near uture.
b. What inter eron products are available and how do they di er?
Inter erons (IFNs) are potent cytokines that possess antiviral, immunomodulatory,
and antiproli erative activities. T ree classes o human inter erons with signi cant
antiviral activity currently are recognized: , , and . IFN- and IFN- may be
produced in nearly all cells in response to viral in ection. T ey exhibit antiviral and
antiproli erative actions and stimulate the cytotoxic activity o lymphocytes, natu-
ral killer cells, and macrophages. Because IFNs induce long-lasting cellular e ects,
their activities are not easily predictable rom the usual pharmacokinetic measures.
Attachment o IFN to large inert polyethylene glycol (PEG) molecules (pegylation)
slows absorption, decreases clearance, and provides higher and more prolonged
serum concentrations that enable once weekly dosing. T e pegylated IFNs are gen-
erally better tolerated than standard IFNs.
c. What is the mechanism o action o inter erons?
Following binding to speci c cellular receptors, IFNs activate the JAK-S A signal
transduction pathway and lead to nuclear translocation o a cellular protein that
binds to genes containing an IFN-speci c response element. T is in turn leads to
synthesis o over 2 dozen proteins that contribute to viral resistance mediated at
di erent stages o viral penetration (see Figure 44-3).
d. What is ribavirin and why is it added to this patients treatment regimen?
Ribavirin is a purine nucleoside analog that inhibits the replication o RNA and
DNA viruses. Oral ribavirin in combination with pegIFN al a-2A or -2B has
become the standard treatment or chronic HCV in ection.
(Continued)

613
SECTION VII Chemotherapy of Microbial Diseases

e. What untoward e ects might be expected with this regimen?


Injection o recombinant IFN usually is associated with an acute in uenza-like syn-
drome beginning several hours af er injection. T e principal dose-limiting toxicities
are depression, myelosuppression with granulocytopenia and thrombocytopenia,
neurasthenia, and autoimmune disorders, including thyroiditis and hypothyroidism.
Systemic ribavirin causes dose-related reversible anemia owing to extravascular
hemolysis and bone marrow suppression. Ribavirin is classi ed as FDA pregnancy
category X.

CASE 44-6
A 42-year-old woman with recently diagnosed HIV in ection is being started on medi-
cation to treat her disease.
a. What clinical decisions are important or making treatment recommendations?
Current treatment recommendations center on making 2 important clinical deci-
sions: (1) When to start therapy in treatment-nave individuals; and (2) when to
change therapy in individuals that are ailing their current regimen.
Several expert panels issue periodic recommendations or the use o antiretroviral
drugs or treatment-nave and treatment-experienced adults and children. In the
United States, the Panel on Clinical Practices or treatment o HIV In ection issues
updated guidelines approximately every 6 months; their most recent guidelines can
be accessed at http://www.aidsin o.nih.gov/Guidelines (Department o Health and
Human Services).
b. What principles drive treatment guidelines?
In each clinical setting there is a complex algorithm o possible drug choices
depending on patient and viral characteristics. T e speci c drugs recommended
may change rom year to year as new choices become available and clinical research
data accumulate. However, it is likely that uture treatment guidelines will continue
to be driven by 3 principles: (1) Use o combination therapy to prevent the emer-
gence o resistant virus; (2) Emphasis on regimen convenience, tolerability, and
adherence to chronically suppress HIV replication; and (3) Realization o the need
or li elong treatment under most circumstances.
c. What concerns about drug resistance should be considered in all patients?
T ere is a high likelihood that all untreated in ected individuals harbor viruses with
single-amino acid mutations con erring some degree o resistance to every known
antiretroviral drug because o the high mutation rate o HIV and the tremendous
number o in ectious virions. Drug therapy does not cause mutation but rather pro-
vides the necessary selective pressure to promote growth o drug-resistant viruses
that arise naturally. A combination o active agents there ore is required to prevent
drug resistance, analogous to strategies employed in the treatment o tuberculosis
(see Chapter 42). T e current standard o care is to use at least 3 drugs simultane-
ously or the entire duration o treatment.

CASE 44-7
A 45-year-old man with HIV in ection is being treated with a combination o zidovu-
dine, lamivudine, and abacavir.
a. What are the mechanisms o action o these drugs?
T ese drugs are nucleoside reverse transcriptase inhibitors (NR Is). T eir mecha-
nism o action is summarized in Figures 44-1 and 44-5. T e HIV-encoded, RNA-
dependent DNA polymerase, also called reverse transcriptase, converts viral
RNA into proviral DNA that is then incorporated into a host cell chromosome.
Zidovudine, lamivudine, and abacavir are phosphorylated and their triphosphates
(Continued)
614
Antiviral Agents and Treatment of HIV Infection CHAPTER 4 4

competitively inhibit incorporation o natural nucleotides by reverse transcriptase


and also terminate the elongation o proviral DNA because they are incorporated
into nascent DNA but lack a 3-hydroxyl group.
b. Why is the combination therapy used?
T e combination therapy is used to prevent drug resistance as described in Case 44-6.
c. What are the precautions and possible untoward e ects o this combination?
Patients initiating zidovudine of en complain o atigue, malaise, myalgia, nausea,
anorexia, headache, and insomnia.
Lamivudine is one o the least toxic antiretroviral drugs.
Abacavir is associated with a unique and potentially atal hypersensitivity syn-
drome. T is syndrome is characterized by ever, abdominal pain, and other GI
complaints, a rash, and malaise or atigue. T e presence o a ever, abdominal pain,
and rash within 6 weeks o starting abacavir is diagnostic and necessitates the dis-
continuation o the drug. Abacavir can never be restarted once discontinued or
hypersensitivity because the reintroduction o the drug leads to rapid recurrence o
severe symptoms and may lead to death.

CASE 44-8
A 25-year-old man with HIV in ection is being treated with nevirapine as a single agent.
a. What is the mechanism o action o nevirapine?
Nevirapine is a non-nucleoside reverse transcriptase inhibitor (NNR I) that binds
to a hydrophobic pocket o the HIV reverse transcriptase where it induces a con-
ormational change in the enzyme that greatly reduces its activity (see Figures 44-1
and 44-7). Unlike nucleoside reverse transcriptase inhibitors (NR Is), the NNR Is
do not require intracellular phosphorylation to attain activity.
b. What is the problem in using this drug as a sole agent to treat HIV?
When nevirapine was used as monotherapy, there was a rapid all in plasma HIV
RNA concentrations that was ollowed by a return toward baseline within 8 weeks
because o rapid emergence o resistance. Nevirapine should never be used as a
single agent or as the sole addition to a ailing regimen.

CASE 44-9
A 38-year-old woman with HIV in ection is being treated with a combination product
containing lopinavir and ritonavir.
a. What is the mechanism o action o these drugs?
Lopinavir and ritonavir are HIV protease inhibitors (see Figure 44-6). T ese drugs
prevent the proteolytic cleavage o peptides and proteins that are essential struc-
tural and enzymatic components o HIV. Viral proteases are homodimers; human
proteases (renin, pepsin, gastricsin, cathepsins) contain only 1 polypeptide chain
and are not signi cantly inhibited by HIV protease inhibitors.
b. Why is the combination product used?
Ritonavir is one o the most potent known inhibitors o CYP3A4. Because lopinavir
undergoes extensive hepatic oxidative metabolism by CYP3A4, ritonavir is used to
inhibit the metabolism o lopinavir and increase its concentrations.
c. What precautions should be taken with the combination o lopinavir and ritonavir?
T e most common adverse events with lopinavir/ritonavir are loose stools, diar-
rhea, nausea, and vomiting.
Because lopinavir is dependent on CYP3A4, the concomitant administration o
agents that induce CYP3A4 such as ri ampin and St. Johns wort may lower plasma
concentrations o lopinavir and loss o antiviral e ectiveness.
(Continued)
615
SECTION VII Chemotherapy of Microbial Diseases

Ritonavir, beside being a potent inhibitor o CYP3A4, is also a moderate CYP


inducer at the doses employed in the co ormulation and can adversely decrease
concentrations o some coadministered drugs such as oral contraceptives.

CASE 44-10
A 56-year-old man with HIV in ection has been treated with 3 other antiretroviral
agents. Despite this treatment, he has evidence o HIV replication. reatment with
en uvirtide is recommended.
a. What is the mechanism o action o en uvirtide?
En uvirtide is a synthetic peptide that prevents membrane usion and viral entry
into the host cell (see Figure 44-5). En uvirtide retains activity against viruses that
have become resistant to antiretroviral agents o other classes because o its unique
mechanism o action.
b. Why is en uvirtide recommended only af er treatment with other antiretroviral
agents has ailed?
Most patients develop local injection site reactions to the parenteral administration
o en uvirtide. Given the cost, inconvenience, and cutaneous toxicity o en uvirtide,
it is generally reserved or patients who have ailed all other easible antiretroviral
agents. In act, en uvirtide is FDA approved or use only in treatment-experienced
adults who have evidence o HIV replication despite ongoing antiretroviral therapy.

KEY CONCEPTS
T ere are distinct stages o viral replication. T e classes o antiviral agents that
can act at each stage are shown in able 44-1.
Acyclovir is the prototype o a group o antiherpes viral agents that are phos-
phorylated intracellularly by a virus kinase and subsequently by host enzymes
to become inhibitors o viral DNA synthesis.
Amantadine and rimantadine are anti-in uenza agents that inhibit viral rep-
lication by inhibiting viral uncoating; zanamivir and oseltamivir inhibit viral
neuraminidase.
T e anti-in uenza drugs are used or the prevention and treatment o in uenza
A, and some or in uenza B.
Drugs used to treat chronic hepatitis C virus in ection are the inter erons
and ribavirin.
Inter eron and ribavirin can cure patients with chronic hepatitis B virus in ec-
tion but are associated with a high rate o side ef ects, o en leading to premature
treatment discontinuation.
Several antiretroviral nucleoside or nucleotide analog polymerase inhibitors,
including lamivudine, telbivudine, and teno ovir, have potent anti-HBV activity
and provide alternative therapy to inter eron and ribavirin. Ade ovir, dipivoxil,
and entecavir also provide alternative therapy or chronic HBV in ection.
T ree-drug combinations are the minimum standard o care or HIV in ection.
Starting therapy with only a single agent inevitably provokes the emergence o
drug-resistant virus.
Drugs used to treat HIV in ection generally all into 3 classes:
Nucleoside reverse transcriptase inhibitors (NR Is).
Non-nucleoside reverse transcriptase inhibitors (NNR Is).
Protease inhibitors.
Ritonavir is a protease inhibitor that is a very potent inhibitor o CYP3A4. It is o en
added in combination with other drugs to enhance their pharmacokinetic pro le.
(Continued)
616
Antiviral Agents and Treatment of HIV Infection CHAPTER 4 4

T ere are 2 drugs, maraviroc and en uvirtide, that block the entry o HIV
into host cells; raltegravir prevents the integration o virus DNA into the host
chromosome.
Prescribers o antiretroviral therapy must maintain a comprehensive and cur-
rent und o knowledge regarding the disease and its pharmacotherapy. T e
prescribing o antiretroviral agents to patients with HIV in ection should be
limited to those with specialized training.

SUMMARY QUIZ

QUESTION 44-1 A 48-year-old man with AIDS is diagnosed with CMV retinitis. He is
being treated with intravenous oscarnet. A laboratory blood test or which o the ol-
lowing should be monitored closely?
a. Copper
b. Alanine aminotrans erase
c. Calcium
d. Zinc
e. Bilirubin

QUESTION 44-2 Which o the ollowing drugs is an option or the treatment o CMV
retinitis in the patient in Question 44-1?
a. Acyclovir
b. Gancyclovir
c. Rimantadine
d. Oseltamivir
e. Ribavirin

QUESTION 44-3 A 36-year-old woman has been diagnosed with chronic hepatitis C.
She is prescribed oral ribavirin in combination with which o the ollowing agents or
her treatment?
a. eno ovir
b. elbivudine
c. En uvirtide
d. PegINF alpha-2A
e. Cido ovir

QUESTION 44-4 A 27-year-old man with newly diagnosed HIV in ection is started
on zidovudine alone. T is treatment regimen is unlikely to be therapeutically ef ective
because
a. zidovudine is not active against HIV.
b. zidovudine is metabolized by hepatic CYPs.
c. the side ef ects o zidovudine do not warrant its use.
d. the use o zidovudine is limited by its absorption rom the gastrointestinal tract.
e. resistance o the HIV is likely to develop rom the use o a single agent.

QUESTION 44-5 A 43-year-old woman with HIV in ection is treated with atazanavir
and ritonavir. Ritonavir is added to the regiment because it
a. inhibits CYP3A4, decreases the metabolism o atazanavir, and raises its plasma
concentrations.
b. prevents the renal excretion o atazanavir.
c. increases the oral absorption o atazanavir.
(Continued)
617
SECTION VII Chemotherapy of Microbial Diseases

d. is a more potent inhibitor o HIV replication than atazanavir.


e. It decreases the neurotoxicity o atazanavir.

QUESTION 44-6 A 56-year-old man has been treated or HIV in ection or 10 years.
His HIV is now resistant to all previous antiretroviral drugs and there is evidence
o HIV replication despite his treatment with 3 concomitant drugs. He is now being
treated with en uvirtide because it
a. is less expensive.
b. is less toxic.
c. is more ef ective against HIV.
d. has an unique mechanism o action.
e. can be administered orally.

QUESTION 44-7 A 35-year-old woman with HIV in ection also has developed chronic
hepatitis B. She is being treated with entecavir or her chronic HBV in ection. Because
she has recently developed symptoms o atigue and nausea, she has stopped her ente-
cavir therapy. She is at risk or developing
a. acute renal ailure.
b. acute pancreatitis.
c. acute exacerbation o hepatitis B.
d. chronic hepatitis C.
e. acute neurotoxicity.

QUESTION 44-8 A 28-year-old man with HIV is being treated with lopinavir. He
admits to using nutritional supplements bought over the Internet because it makes him
eel in control o his therapy. T is patient should be cautioned about the ef ect nutri-
tional supplements may have on
a. HIV.
b. the metabolism o lopinavir.
c. the renal excretion o lopinavir.
d. the cardiac toxicity o lopinavir.
e. the skin toxicity o lopinavir.

SUMMARY QUIZ ANSWER KEY


QUESTION 44-1 Answer is c. Foscarnet is highly ionized at physiological pH, and
metabolic abnormalities are common. T ese include increases or decreases in calcium
and phosphate, hypomagnesemia, and hypokalemia. Decreased serum ionized calcium
may cause paresthesia, arrhythmias, tetany, seizures, and other CNS disturbances. Con-
comitant intravenous pentamidine administration increases the risk o symptomatic
hypocalcemia.
QUESTION 44-2 Answer is b. Acyclovir is inef ective therapeutically in established
CMV in ections, but gancyclovir is ef ective or treatment and chronic suppression o
CMV retinitis in immunocompromised patients and or prevention o CMV in trans-
plant patients.
QUESTION 44-3 Answer is d. Oral ribavirin in combination with injected pegINF al a-
2A or -2B has become standard treatment or chronic HCV in ection.
QUESTION 44-4 Answer is e. Starting treatment with only a single antiretroviral drug
inevitably provokes the emergence o drug-resistant virus. A combination o antiviral
agents there ore is required to prevent drug resistance.
QUESTION 44-5 Answer is a. Atazanavir undergoes oxidative metabolism in the
liver primarily by CYP3A4. Ritonavir is one o the most potent known inhibitors o
(Continued)
618
Antiviral Agents and Treatment of HIV Infection CHAPTER 4 4

CYP3A4. Ritonavir inhibits the metabolism o all current HIV protease inhibitors
and is requently used in combination with most o these drugs, except nel navir, to
enhance their pharmacokinetic pro le and allow a reduction in dose and dosing re-
quency o the coadministered drug.
QUESTION 44-6 Answer is d. En uvirtide has a unique mechanism o antiretroviral
action. It prevents viral entry into the host cell. Because o this unique mechanism o
action, it retains activity against viruses that have become resistant to antiretroviral
agents o other classes.
QUESTION 44-7 Answer is c. Severe acute exacerbations o hepatitis B have been
reported in patients who have discontinued anti-HBV therapy. Hepatic unction should
be monitored closely with both clinical and laboratory ollow-up or at least several
months in patients who discontinue anti-HBV therapy.
QUESTION 44-8 Answer is b. Because lopinavir metabolism is highly dependent on
CYP3A4, concomitant administration o agents that induce CYP3A4 may lower plasma
lopinavir concentrations considerably. St. Johns wort is a known inducer o CYP3A4,
leading to lower concentrations o lopinavir and possible loss o antiviral ef ectiveness.

SUMMARY: ANTIVIRAL DRUGS


TOXICITY
CLASS AND UNIQUE; CLINICALLY
SUBCLASSES NAME CLINICAL USES RESISTANCE COMMON IMPORTANT
Antiherpes Acyclovir HSVin ections Impaired production Nausea, diarrhea, Renal insuf ciency and
Agents o thymidine kinase, rash, and headache CNS side e ects, including
altered thymidine kinase altered sensorium,
substrate, or altered DNA seizures, delirium, or
polymerase extrapyramidal e ects

Cido ovir Approved or Resistance in CMVis due Nephrotoxicity


treatment o CMV to mutations in DNA Use in pregnancy
retinitis in HIV-in ected polymerase Category C
patients

Foscarnet Use ul or treatment Mutations in herpes virus Rash, ever, nausea Nephrotoxicity
o CMVretinitis and DNA polymerase and symptomatic
acyclovir-resistant HSV hypocalcemia
and VZVin ections

Fomivirsen CMVretinitis Iritis, vitritis, cataracts

Ganciclovir Chronic suppression CMVresistance is due Rash, ever, nausea, Neutropenia and
o CMVretinitis and to either mutations in vomiting thrombocytopenia
other CMVsyndromes viral phosphotrans erase Phlebitis CNS toxicity, including
in AIDS patients which reduces headache and seizures
intracellular ganciclovir
phosphorylation or
mutations in viral DNA
polymerase

Idoxuridine Topical treatment o Resistance develops Pruritus,


HSVkeratitis readily in vitro and in viral in ammation and
isolates o idoxuridine- edema o eye or
treated patients lids

Penciclovir Treatment o HSVand Mutations in thymidine Headache, diarrhea,


varicella-zoster virus kinase or viral DNA nausea
(VZV) in ections polymerase

Tri uridine Treatment o HSV Tri uridine-resistant HSV Irritation and


keratoconjunctivitis have altered thymidine palpebral edema at
and recurrent substrate speci city injection site
epithelial keratitis
(Continued)
619
SECTION VII Chemotherapy of Microbial Diseases

TOXICITY
CLASS AND UNIQUE; CLINICALLY
SUBCLASSES NAME CLINICAL USES RESISTANCE COMMON IMPORTANT

Antihepatitis Ade ovir dipivoxil Treatment o chronic Point mutations in HBV Headache, Pregnancy Category C
Agents hepatitis B virus (HBV) polymerase abdominal Dose-related
in ections discom ort nephrotoxicity

Entecavir Treatment o chronic Mutations in HIVreverse Headache, atigue, Severe exacerbations o


hepatitis B virus (HBV) transcriptase dizziness, nausea hepatitis B in patients
in ections who have discontinued
anti-HBVtherapy

Inter eron Treatment o Blocked production or Acute in uenza- Granulocytopenia and


condyloma activity o inter eron- like syndrome thrombocytopenia
acuminatum inducible proteins several hours a ter Neurotoxicity mani ested
Chronic HCVin ection injection by somnolence,
Chronic HBVin ection con usion, behavioral
Kaposis sarcoma in disturbance
HIV-in ected patients, Debilitating neurasthenia
other malignancies, and depression; thyroiditis
and multiple sclerosis and hypothyroidism

Lamivudine Treatment o chronic Point mutations in HBV Generally well tolerated


HBVhepatitis and HIV DNA polymerase
in ection

Anti-in uenza Amantadine Prophylaxis against Mutations in the RNA Nervousness


Agents in uenza A sequence encoding or light-headedness,
the M2 protein trans- insomnia, loss
membrane domain o appetite, and
nausea

Oseltamivir Treatment and Mutations in viral Nausea, vomiting Pregnancy Category C


prevention o hemagglutinin and/or
in uenza A and B virus neur aminidase
in ections

Rimantadine Prophylaxis against Mutations in the RNA


in uenza A sequence encoding or
the M2 protein trans-
membrane domain

Zanamivir Prevention and Mutations in viral Wheezing and Not recommended or


treatment o in uenza hemagglutinin and/or bronchospasm treatment o patients with
A and B virus neuraminidase underlying airway disease
in ections because o serious
adverse events

Other Antiviral Ribavirin Oral ribavirin in Resistance not Conjunctival Pregnancy Category X
Agents combination with documented, but irritation, rash, Dose-related anemia due
pegIFN al a-2A or -2B cells that do not transient wheezing to extravascular hemolysis
is standard treatment phosphorylate ribavirin to and bone marrow
or chronic hepatitis C its active orm have been suppression
virus (HCV) in ection reported
Ribavirin aerosol
is or treatment o
RSVbronchiolitis
and pneumonia in
children

Telbivudine Treatment o chronic Mutations o HBVDNA Increased creatine


HBVin ection polymerase kinase, nausea,
diarrhea, atigue,
myalgia, and
myopathy
(Continued)

620
Antiviral Agents and Treatment of HIV Infection CHAPTER 4 4

TOXICITY
CLASS AND UNIQUE; CLINICALLY
SUBCLASSES NAME CLINICAL USES RESISTANCE COMMON IMPORTANT

Imiquimod Topical treatment Local erythema,


o condylomata excoriation, itching,
acuminata burning

Nucleoside Reverse Zidovudine Treatment o HIV Mutations in reverse Fatigue, nausea, Bone marrow suppression
Transcriptase in ection in adults and transcriptase malaise, myalgia, with anemia
Inhibitors (NRTIs) children anorexia, insomnia

Didanosine Treatment o HIV Mutations in reverse Diarrhea Peripheral neuropathy


in ection in adults and transcriptase and pancreatitis
children

Stavudine Treatment o HIV Mutations in reverse Peripheral neuropathy


in ection in adults and transcriptase Fat wasting and
children lipodystrophy
Should not be used
with either didanosine
(increased peripheral
neuropathy and
atal pancreatitis) or
zidovudine (competition
or intracellular
phosphorylation)

Zalcitabine Treatment o No longer marketed


advanced HIV because o severe
peripheral neuropathy

Lamivudine Treatment o chronic Point mutations in DNA Few signi cant adverse
HBVhepatitis and HIV polymerase e ects
in ection

Abacavir In combination with Mutations in reverse Fatal hypersensitivity


other antiretroviral transcriptase syndrome mani ested by
agents or the ever, abdominal pain,
treatment o HIV and rash; the adverse
in ection reaction is linked to the
HLA-B*5701 genotype

Teno ovir disoproxil Treatment o chronic Mutations in reverse Flatulence


HBVin ection and HIV transcriptase; in HBV
in ection in ection, resistance
was not evident over
48 weeks o treatment

Emtricitabine Treatment o HIV Mutations in reverse Prolonged Few signi cant adverse
in ection in adults transcriptase exposure causes e ects
hyperpigmentation

Protease Inhibitors Saquinavir Treatment o HIV Accumulation o multiple Nausea, vomiting, Metabolized by intestinal
in ection in adults resistance mutations diarrhea and hepatic CYP3A4;
Predominantly used substances that inhibit
in combination intestinal CYP3A4 increase
with ritonavir AUC by 3 old
Long-term use associated
with lipodystrophy

Indinavir No longer widely Accumulation o multiple Nephrolithiasis and


used because o resistance mutations nephrotoxicity
nephrotoxicity
(Continued)

621
SECTION VII Chemotherapy of Microbial Diseases

TOXICITY
CLASS AND UNIQUE; CLINICALLY
SUBCLASSES NAME CLINICAL USES RESISTANCE COMMON IMPORTANT

Ritonavir Used in requently Mostly used as a Nausea, vomiting, Reduces the ethinyl
alone; ritonavir is pharmacokinetic and diarrhea estradiol AUC by 40%;
a potent inhibitor enhancer (CYP3A4 alternative orms o
o CYP3A4 and is inhibitor) and at the low contraception should be
used in combination doses use is not known ound
to enhance the to induce resistance
pharmacokinetic
pro le o other
protease inhibitors

Nel navir Treatment o HIV Mutation in HIVprotease Diarrhea Metabolized by


in ection in adults and CYP2C19 and CYP
children 3A4; concomitant
administration o
agents that induce
these enzymes may be
contraindicated

Amprenavir Treatment o HIV Mutation in HIVprotease Diarrhea, nausea, Amprenavir is both an


in ection in adults and vomiting inducer and inhibitor
children, commonly o CYP3A4 and drug
in combination with interactions can occur
ritonavir and be unpredictable
Substances that induce
CYP3A4 may lower
plasma amprenavir
concentrations

Lopinavir Treatment o HIV Mutation in HIVprotease Nausea, vomiting, Metabolized by


in ection in adults and diarrhea CYP3A4; concomitant
children, commonly administration o
in combination with substances that induce
ritonavir this enzyme may lower
plasma concentration o
lopinavir substantially

Atazanavir Used in combination Mutation in HIVprotease Unconjugated Metabolized by CYP3A4;


with ritonavir to treat hyperbilirubinemia the concomitant
HIV-in ected adults Diarrhea, nausea administration o agents
that induce this enzyme
are contraindicated

Tipranavir Used in treatment- Resistance requires Rash Associated with rare atal
experienced adults accumulation o multiple hepatotoxicity
and children whose mutations Tipranavir is a substrate,
HIVis resistant to inducer, and inhibitor o
other protease CYP enzymes
inhibitors

Darunavir Used in combination Resistance requires Rash, GI complaints


with ritonavir to treat accumulation o multiple
HIV-in ected adults mutations

Non-Nucleoside Nevirapine Treatment o HIV Mutations in reverse Rash, ever, atigue Elevated hepatic
Reverse in ection in adults and transcriptase; cross- transaminases
Transcriptase children resistance extends to
Inhibitors (NNRTIs) e avirenz and delavirdine
(Continued)

622
Antiviral Agents and Treatment of HIV Infection CHAPTER 4 4

TOXICITY
CLASS AND UNIQUE; CLINICALLY
SUBCLASSES NAME CLINICAL USES RESISTANCE COMMON IMPORTANT

E avirenz Treatment o HIV Mutations in reverse Headache and CNS symptoms including
in ection in adults and transcriptase; cross- elevated hepatic dizziness, impaired
children resistance extends trans-aminases concentration, dysphoria,
to nevirapine and insomnia; rank psychosis
delavirdine Use should be avoided in
women o childbearing
potential unless 2 orms
o birth control are used
Metabolized by CYP2B6
and CYP 3A4; drugs that
induced these enzymes
should be avoided

Delavirdine Treatment o Mutations in reverse Rash Substrate or and inhibitor


HIVin ection in transcriptase; cross- o CYP3A4 and can alter
adults generally in resistance extends to metabolism o other
combination with nevirapine and e avirenz CYP3A4 substrates
other nucleoside
agents

Etravirine Use in treatment- Rash Inducer o CYP3A4 and


experienced HIV- glucuronosyl trans erase
in ected adults Inhibitor o CYP2C9 and
CYP 2C19
Should not be
administered with
tipranavir/ritonavir
osamprenavir/ritonavir or
atazanavir/ritonavir
Should not be combined
with delavirdine,
e avirenz, or nevirapine

EntryInhibitors En uvirtide Use in treatment- Injection site Not metabolized and no


experienced adults reactions known drug interactions
who have evidence o
HIVreplication despite
antiretroviral therapy

Maraviroc Use in HIV-in ected HIVthat is pre- Little signi cant toxicity
adults who have dominantly CCR5-tropic CYP3A4 substrate;
evidence o CCR5- shi ts tropism to CXCR4; susceptible to drug
tropic virus or there is a mutation in interactions with drugs
gp120 that allows virus that are CYP3A4 inhibitors
binding in the presence or inducers
o inhibitor

Integrase Inhibitor Raltegravir Treatment o HIV- Mutations in the Headache, nausea, Little clinical toxicity
in ected adults integrase gene atigue

623
This page intentionally left blank
SECTION

Chemotherapyof
NeoplasticDiseases VIII
45. Cancer Chemotherapy and Cytotoxic Agents 626

46. argeted Anticancer T erapies 661

625
CHAPTER

45 Cancer Chemotherapy
and CytotoxicAgents
T is chapter wi be most use u a er having a basic understanding o the materia in
DRUGS INCLUDED IN THIS
Chapter 60 Genera Princip es o Cancer Chemotherapy and Chapter 61 Cytotoxic
CHAPTER Agents in Goodman & Gilmans T e Pharmacological Basis of T erapeutics, 12th Edition
2-Mercaptoethanesul onate (mesna; In addition to the materia presented here, Chapters 60 and 61 o the 12th Edition
MESNEX)reacts with acrolein in urine contain:
and protects against severe hemorrhagic A description o the ce cyc e, and ce cyc e checkpoints and regu ation
cystitis in high-dose cyclophosphamide
T e c inica pharmaco ogy and toxicities o speci c agents
regimens
T e mo ecu ar structures o cytotoxic agents used to treat cancer and other diseases
5-Fluorouracil (5-FU)
such as immune disorders
6-Mercaptopurine (6-MP; PURINETHOL,
others) LEARNING OBJECTIVES
6-Thioguanine (6-TG) Understand the mechanisms o action o cytotoxic antineop astic agents on
Altretamine (hexamethylmelamine; tumor ce s (see Figure 45-1)
HEXALEN) Understand the mechanisms o toxicity o cytotoxic antineop astic agents on
Ami ostine (WR-2721; ETHYOL, norma ce s and strategies or reducing toxic e ects
others)thiophosphate cytoprotective
Understand the mechanisms o drug resistance to individua agents and
agent (see cisplatin clinical toxicities)
strategies to avoid resistance
Arsenictrioxide (ATO; TRISENOX)
Know how pharmacogenetics can impact tumor sensitivity and toxicities o
Azacytidine (5-azacytidine) speci c agents
Bendamustine (TREANDA, others) Know the signs o acute toxicity and when chemotherapy shou d be a tered
Bleomycin (BLENOXANE, others) or discontinued
Busul an (MYLERAN, BUSULFEX) Know the risk o ong-term toxicities that can occur with individua chemo-
Carmustine (BCNU; BICNU, GLIADEL) therapeutic agents
Capecitabine (XELODA) Know the therapeutic strategies that can reduce acute and chronic toxicities
Carboplatin (CBDCA, JM-8; PARAPLATIN) Know which c asses o agents are typica y used in treating speci c cancers
Carboxypeptidase G2 (a methotrexate- Know which drugs are used in combination to improve tumor ce ki ing and
cleaving enzyme used to reduce metho- reduce the risk o resistance deve opment
trexate in plasma)
Chlorambucil (LEUKERAN)
Cisplatin (PLATINOL, others) CASE 45-1
Cladribine (2-chlorodeoxyadenosine, 2-CdA; A patient with a chronic ymphocytic eukemia is started on chemotherapy with
LEUSTATIN, others) cyc ophosphamide
Clo arabine (2-chloro-2- uoro- a. What is the mechanism o action o cyclophosphamide?
arabinosyladenine) Cyclophosphamide is a nitrogen mustard and is now the most widely used agent
Cyclophosphamide (LYOPHILIZED o this class o alkylating agents. Alkylating agents have in common the property
CYTOXAN, others) o orming highly reactive carbonium ion intermediates. T ese reactive intermedi-
Cytarabine(1--D-arabino uranosylcytosine, ates covalently link to sites o high electron density, such as phosphates, amines,
cytosine arabinoside, Ara-C; CYTOSAR-U, sul ydryl, and hydroxyl groups. T eir chemotherapeutic and cytotoxic e ects are
TARABINEPFS, DEPOCYT, others) directly related to the alkylation o reactive amine, oxygen, or phosphate groups on
Dacarbazine (DTIC; DTIC-DOME) DNA (see Figure 45-1). T e N7 atom o guanine is particularly susceptible to the
ormation o a covalent bond with bi unctional alkylating agents and may represent
Dactinomycin (actinomycin D; COSMEGEN) the key target that determines their biological e ects (see Figure 45-2). Other atoms
Daunorubicin (daunomycin, rubidomycin; in the purine and pyrimidine bases o DNA, including N1 and N3 o the adenine
CERUBIDINE, others) ring, N3 o cytosine, and O6 o guanine, react with these agents, as do the amino
(continues) and sul ydryl groups o proteins and the sul ydryl groups o glutathione.
(Continued)
626
Cancer Chemotherapy and Cytotoxic Agents CHAPTER 4 5

MECHANISMS OF ACTION AND RESISTANCE OF ALKYLATING AGENTS AND DRUGS INCLUDED IN THIS
PLATINUM COORDINATION COMPLEXES (SEE TABLE 45-1) CHAPTER (Cont.)
MECHANISMS OF MECHANISMS OF Decitabine (2-deoxy-5-azacytidine)
CLASSIFICATION DRUG ACTION RESISTANCE
Docetaxel (TAXOTERE)
Nitrogen Mustards Mechlorethamine HCl Form highly reactive Overexpression of Doxorubicin (ADRIAMYCIN)
Cyclophosphamide carbonium ion nucleotide excision
Ifosfamide intermediates that repair (NER) pathway Epirubicin (ELLENCE, others)
Melphalan alkylate reactive Mutations of p53 Estramustine (EMCYT)
Chlorambucil amines, oxygens, and DNA repair by O6-alkyl,
Bendamustine phosphates on DNA methyl guanine Etoposide (VP-16-213; VEPESID, others)
(Figures 45-1 through methyltransferase Floxuridine (FUdR, fuorodeoxyuridine;
45-4) which leads to (MGMT) FUDR, others)
cell death Activated
intermediates of Fludarabine phosphate (FLUDARA,
cyclophosphamide OFORTA)
can be degraded Gemcitabine (2,2-difuorodeoxycytidine;
by aldehyde dFdC, GEMZAR)
dehydrogenase,
glutathione transferase, High-dose methotrexate with leucovorin
and other pathways rescue (HDM-L)
(see Figure 45-4) Hydroxyurea (HU; HYDREA, DROXIA,
See Side Bar
others)
MECHANISMS OF
RESISTANCE TO Idarubicin (IDAMYCINPFS)
ALKYLATING AGENTS I os amide (IFEX,others)
Ethyleneimines and Altretamine Cytotoxic mechanism Overexpression of NER Irinotecan (CPT-11; CAMPTOSAR, others)
Methylmelamines (hexamethylendiamine) of altretamine is pathway Ixabepilone (IXEMPRA)
Thiotepa unknown (it has no Mutations of p53
alkylating activity DNA repair by MGMT L-asparaginase (L-ASP; ELSPAR)
in vitro but is See Side Bar Leucovorin ( olinicacid, citrovorum actor,
demethylated in liver MECHANISMS OF 5- ormyltetrahydro olate, N5- ormyl FH4)
microsomes to form RESISTANCE TO
formaldehyde) ALKYLATING AGENTS Lometrexol
Thiotepa and its Lomustine (CCNU; CeeNU)
primary metabolite
(TEPA) form DNA Mechlorethamine HCl (MUSTARGEN)
cross-links Melphalan (ALKERAN)
Alkyl Sulfonates Busulfan Alkylate DNA Overexpression of NER Methotrexate (amethopterin; RHEUMA-
through the release of pathway TREX,TREXALL, others)
methyl radicals (see Mutations of p53 Mitomycin (mitomycin-C; MUTAMYCIN,
Figures 45-1 and 45-3) DNA repair by MGMT
others)
See Side Bar
MECHANISMS OF Mitotane (o,p-DDD)
RESISTANCE TO Mitoxantrone (NOVANTRONE, others)
ALKYLATING AGENTS
Nab-paclitaxel (ABRAXANE)
Nitrosoureas Carmustine (BCNU) Exert their cytotoxicity Overexpression of NER Nelarabine
Lomustine (CCNU) through the pathway
Semustine spontaneous Mutations of p53
(6-methoxy-arabinosyl-guanine)
(methyl-CCNU) breakdown to an DNA repair by MGMT Oxaliplatin (ELOXATIN)
Streptozocin alkylating intermediate, See Side Bar Paclitaxel (TAXOL, others)
(streptozotocin) the 2-chloroethyl MECHANISMS OF
diazonium ion, a RESISTANCE TO Pegaspargase (PEG-l -Asparaginase;
strong electrophile, ALKYLATING AGENTS ONCASPAR)
that can alkylate Pemetrexed (MTA; ALIMTA)
guanine, cytidine,
and adenine bases, Pentostatin (2-deoxyco ormycin)
leading to interstrand Pralatrexate (FOLOTYN)
or intrastrand cross-
linking of DNA (see Procarbazine (MATULANE)
Figures 45-1, 45-3, Raltitrexed (TOMUDEX)
and 45-5) (continues)
(Continued)
627
SECTION VIII Chemotherapy of Neoplastic Diseases

DRUGS INCLUDED IN THIS MECHANISMS OF MECHANISMS OF


CLASSIFICATION DRUG ACTION RESISTANCE
CHAPTER (Cont.)
Triazenes Dacarbazine (DTIC) Spontaneous cleavage Overexpression o NER
Romidepsin (depsipeptide, FK228; ISTODAX) Temozolomide o the metabolite pathway
Semustine (methyl-CCNU) methyl-triazeno- Mutations o p53
imidazole-carboxamide DNA repair by MGMT
Streptozocin (streptozotocin; ZANOSAR)
(MTIC) yields a methyl See Side Bar
Temozolomide (TEMODAR) diazonium ion that MECHANISMS OF
Teniposide (VM-26; VUMON) trans ers methyl rather RESISTANCE TO
than ethyl groups to ALKYLATING AGENTS
Thiotepa (triethylenethiophosphoramide; DNA (see Figures 45-1 Simple methylation
THIOPLEX,others) and 45-3) can be bypassed by
Topotecan (HYCAMTIN) Temozolomide is DNA polymerases
a mono unctional
Trabectedin (YONDELIS)not currently methylating agent
approved in the United States
Methylhydrazines Procarbazine Converted by CYP- Overexpression o NER
Tretinoin (all-trans retinoicacid; ATRA) mediated oxidation pathway
Trimetrexate (NEUTREXIN) to a highly reactive Mutations o p53
Valrubicin (VALSTAR) mono unctional DNA repair by MGMT
methylating agent See Side Bar
Vinblastine sul ate ((VELBAN, others) that can cause MECHANISMS OF
Vincristine sul ate (VINCASARPFS, others) chromosomal RESISTANCE TO
damage, resulting ALKYLATING AGENTS
Vinorelbine (NAVELBINE, others) in mutagenesis and Simple methylation
Vorinostat (suberoylanilide hydroxamic carcinogenesis (see can be bypassed by
acid, SAHA; ZOLINZA) Figures 45-1 and 45-3) DNA polymerases

Platinum Cisplatin Platinum-coordinated Reduced uptake


Coordination Carboplatin (CBDCA) ligands are replaced by Cu2+ transporter
Complexes Oxaliplatin by water molecules (CTR1)
MECHANISMS OF in the cell yielding a Increased e ux
CYTOTOXICITY highly reactive aquated through Cu 2+ e ux
molecule that reacts transporters ATP7A
Alkylating agents inter ere with DNA with nucleophilic sites and ATP7B, and MRP 1
integrity and unction, and primarily on DNA and protein e ux transporters
induce cell death in rapidly proli erating orming intrastrand Intracellular
tissues. and interstrand cross- sul hydryls inactivate
links (see Figures 45-1 the drug
Lethalityo DNAalkylation depends on the and 45-3) Overexpression o NER
ollowing: pathway
The recognition o the adduct byDNA See Side Bar
MECHANISMS OF
repair systems. RESISTANCE TO
The creation o DNAstrand breaks by ALKYLATING AGENTS
repair enzymes. Resistance to cisplatin
but not oxaliplatin is
An intact apoptoticresponse. partly mediated by
In nondividing cells, DNAdamage activates MMR system which
a checkpoint that depends on the presence recognizes platinum-
o a normal p53 gene. DNA complexes and
initiates apoptosis
Cells thus blocked in the G1/Sinter ace Oxaliplatin does
either repair DNAalkylation or undergo not display cross-
apoptosis. resistance to cisplatin
Malignant cells with mutant or absent and carboplatin in
some tumors
p53 ail tosuspend cell-cycle progression,
do not undergo apoptosis, and exhibit
resistance to alkylating drugs. Alkylation o the N7 o guanine in DNA exerts several biologically important
Bi unctional agents, in which cytotoxic e ects. Guanine residues in DNA exist predominantly as the keto tautomer and
e ects predominate, cause DNAstrand readily make Watson-Crick base pairs by hydrogen bonding with cytosine residues.
cross-linking. However, when the N7 o guanine is alkylated (to become a quaternary ammonium
(continues) nitrogen), the guanine residue is more acidic and the enol tautomer is avored.
(Continued)
628
Cancer Chemotherapy and Cytotoxic Agents CHAPTER 4 5

TABLE 45-1 Alkylating Agents MECHANISMS OF


TYPE OF AGENT NONPROPRIETARY NAMES DISEASE
CYTOTOXICITY (Cont.)
Nitrogen mustards Mechlorethamine Hodgkins disease
Mono unctional methylatingagents(procar-
Cyclophosphamide Acute and chronic bazine, temozolomide) have greater capacity
I os amide lymphocytic leukemia; or mutagenesisandcarcinogenesis:
Hodgkins disease; non- Simple methylation maybe bypassed by
Hodgkins lymphoma; multiple DNApolymerases, leading to mispairing
myeloma; neuroblastoma;
reactions that permanentlymodi yDNA
breast, ovary, lung cancer;
Wilmstumor; cervix, testis sequence; these can be transmitted
cancer; so t-tissue sarcoma to subsequent generations leading to
mutagenesis and carcinogenesis.
Melphalan Multiple myeloma
Chlorambucil Chronic lymphocytic
leukemia; macroglobulinemia DNA REPAIR MECHANISMS
Methylhydrazine derivative Procarbazine Hodgkins disease Alkylation o a single strand o DNA
(N-methylhydrazine, MIH) (mono-adducts) is repaired bythe nucleo-
tide excision repair (NER) pathway.
Alkyl sul onate Busul an Chronic myelogenous
leukemia, bone marrow DNAstrand cross-links require participa-
transplantation tion o nonhomologous end joining, an
error-prone pathway, or the error- ree
Nitrosoureas Carmustine (BCNU) Hodgkins disease; non- homologous recombination pathway.
Streptozocin (streptozotocin) Hodgkins lymphoma;
Bendamustine glioblastoma The homologous recombination (end join-
Malignant pancreatic ing) pathwayhas multiple components:
insulinoma; malignant Sensors o DNAintegrity(such as p53)
carcinoid Activation signals such as the ataxia-
Non-Hodgkins lymphoma
telangiectasia-mutated (ATM) and
Triazenes Dacarbazine (DTIC; Malignant melanoma; ataxia-telangiectasia and rad-related
dimethyltriazenoi-midazole Hodgkins disease; so t-tissue (ATR) proteins
carboxamide), sarcomas; melanoma Activated DNArepair complex
Temozolomide Malignant gliomas
composed o Fanconi anemia proteins
Platinum coordination Cisplatin, carboplatin, Testicular, ovarian, bladder, and BRCA2, all o which localize at the
complexes oxaliplatin esophageal, lung, head and site o DNAdamage and initiate removal
neck, colon, breast cancer o the cross-linked segment o DNA
Homologous recombination, which
allows re-synthesis o the damaged
T e modi ed guanine can mispair with thymine residues during DNA synthesis, DNAsequence ollowed byreligation o
leading to the substitution o thymine or cytosine. Second, alkylation o the N7 the repaired sequences
creates lability in the imidazole ring, leading to opening o the ring and excision The homologous recombination process
o the damaged guanine residue. Mispairing and imidazole ring opening can lead depends on the presence and accurate
to attempts to repair the damaged stretch o DNA, causing strand breakage. T ird, unctioning o multiple proteins; their
with bi unctional alkylating agents such as nitrogen mustards, the second 2-chlo- absence or mutation, as in Fanconi anemia
roethyl side chain can undergo a similar cyclization reaction and alkylate a second or ataxia telangiectasia, leads to extreme
guanine residue or another nucleophilic moiety, resulting in the crosslinking o sensitivityto DNAcross-linking agents.
two nucleic acid chains or the linking o a nucleic acid to a protein, alterations that Other repair enzymes are speci c or remov-
would cause a major disruption in nucleic acid unction. Any o these e ects could ing methyl and ethyl adducts romthe O6 o
contribute to both the mutagenic and cytotoxic e ects o alkylating agents. T e guanine (MGMT), and or repair o alkylation
extreme cytotoxicity o bi unctional alkylators correlates very closely with inter- o the N-3o adenine and N-7o guanine
strand cross-linkage o DNA. (3-methyladenine-DNAglycosylase).
T e capacity o alkylating agents to inter ere with DNA integrity and unction, For the methylating drugs, nitrosoureas,
and to induce cell death in rapidly proli erating tissues provides the basis or their cisplatin and carboplatin, and thiopurine
therapeutic and toxic properties (see Side Bar MECHANISMS OF CY O OXIC- analogs, the mismatch repair (MMR) path-
I Y). Acute e ects mani est primarily against rapidly proli erating tissues; how- wayis essential or cytotoxicity, causing
ever, certain alkylating agents may have damaging e ects on tissues with normally strand breaks at sites o adduct ormation,
low mitotic indices (eg, liver, kidney, and mature lymphocytes), which usually are creating mispairing o thymine residues,
a ected in a delayed time rame. and triggering apoptosis.
(Continued)
629
SECTION VIII Chemotherapy of Neoplastic Diseases

6-MERCAP TOP URINE Purine P yrimidine HYDROXYUREA


6-THIOGUANINE synthe s is synthe s is
Inhibits ribonucle otide re ducta s e
Inhibit purine ring biosynthe s is
Inhibit DNA synthe s is Ribonucle otide s 5-FLUOROURACIL
Inhibits thymidyla te synthe s is
ALIMTA
METHOTREXATE GEMCITABINE
De oxyribo- CYTARABINE
Inhibit dihydrofola te re duction, nucle otide s FLUDARABINE
block thymidyla te a nd 2-CHLORODEOXYADENOS INE
purine synthe s is CLOFARABINE
Inhibits DNA synthe s is
CAMP TOTHECINS
ETOP OS IDE P LATINUM ANALOGS
TENIP OS IDE ALKYLATING AGENTS
DAUNORUBICIN DNA
MITOMYCIN
DOXORUBICIN TEMOZOLOMIDE
Block topois ome ra s e function Form a dducts with DNA

L-AS PARAGINAS E
RNA
(tra ns fe r, me s s e nge r, ribos oma l) De a mina te s a s pa ra gine
P ROTEIN TYROS INE
KINAS E INHIBITORS, Inhibits prote in synthe s is
ANTIBODIES
EP OTHILONES
Block a ctivitie s of s igna ling pa thways Prote ins TAXANES
VINCA ALKALOIDS
ESTRAMUSTINE
Enzyme s Micro-
tubule s Inhibit function of microtubule s

Diffe re ntia tion ATRA


ARS ENIC TRIOXIDE
HISTONE DEACELYAS E INHIBITORS
Induce rs of diffe re ntia tion

FIGURE 45-1 Summary o the mechanisms and sites o action o some chemotherapeutic agents use ul in neoplastic disease.

A Ac tivatio n
S P HAS E
+ - S P ECIFIC DRUGS
CH2 CH2 Cl + CH2 cytos ine a ra binos ide,
H3 C N H3 C N Cl hydroxyure a , topois ome ra s e
CH2 CH2 Cl CH2
S inhibitors
CH2 CH2 Cl
G1
B Nucle o philic attack o f S P HAS E S P ECIFIC
uns table aziridine ring by e le c tro n do no r S ELF-LIMITING
che ckpoints 6-me rca ptopurine ,
(S H of prote in, N of prote in or DNA ba s e ,
G1 me thotrexa te
=O of DNA ba s e or phos pha te ) G2 G2

CH2 CH2 Cl

CH2 CH2 CH2 Cl M


H3 C N H3 C N
+ CH2 CH2 CH2 O

O N N M P HAS E S P ECIFIC DRUGS


DNA O CH2
GO vincris tine, vinbla s tine, pa clita xe l
N N N N NH2
O
DNA O CH2
N N NH2
O
O CELL CYCLE NON-S P ECIFIC DRUG
DNA
O a lkyla ting drugs, nitros oure a s,
DNA a ntitumor a ntibiotics, proca rba zine,
cis pla tin, da ca rba zine
FIGURE 45-2 Mechanism o action o alkylating agents.
A. Activation reaction. B. Alkylation o N7 o guanine. FIGURE 45-3 Cell cycle speci city o antineoplastic agents.
630
Cancer Chemotherapy and Cytotoxic Agents CHAPTER 4 5

Cyclophos pha mide


O
O
Cl CH2 CH2 O CH2 CIH2 CH2 C N C NH CH2 CH2 Cl
N P CH2 N O
Cl CH2 CH2 NH CH2

he pa tic CYP s M = (ClCH2 CH2 )2 N CIH2 CH2 C N NOH O C N CH2 CH2 Cl


O O
O CH2 O CH2
M P CH2 M P CH2 +
CICH2 CH2 N2 OH
NH CH NH2 CHO
Alkyla tion of
OH Gua nine
4-Hydroxycyclophos pha mide Aldophos pha mide of DNA -NH2 -Lys ine
a lde hyde O CH2 CH2 Cl of Prote in
e nzyma tic de hydroge na s e none nzyma tic
N N
O O O
O CH2 O CH2 O H2 N N N
M P CH2 M P CH2 M P H2 C CH CHO
NH C NH2 NH2 DNA O CH2 O
COOH O
O P hos phor- P ROTEIN NH C NH CH2 CH2 Cl
4-Ke tocyclo- Ca rboxy- a mide Acrole in
O DNA
phos pha mide phos pha mide mus ta rd
Alkyla te d DNA Ca rba moyla te d Prote in
INACTIVE METABOLITES TOXIC METABOLITES
FIGURE 45-5 Degradation o carmustine (BCNU) with generation
FIGURE 45-4 Metabolism o cyclophosphamide. o alkylating and carbamylating intermediates.

b. How are the active alkylating species ormed in vivo?


Cyclophosphamide is well absorbed orally and is activated to the 4-hydroxy inter-
mediate in the liver by CYP2B (see Figure 45-4), with subsequent transport o the
activated intermediate to sites o action. 4-Hydroxycyclophosphamide may be
oxidized urther by aldehyde oxidase, either in the liver or in tumor tissue, to inac-
tive metabolites. 4-Hydroxycyclophosphamide and its tautomer, aldophosphamide,
travel in the circulation to tumor cells where aldophosphamide cleaves spontane-
ously, generating stoichiometric amounts o phosphoramide mustard and acrolein.
Phosphoramide mustard is responsible or antitumor e ects, while acrolein is asso-
ciated with signi cant toxicities.
c. What are the adverse e ects o cyclophosphamide?
Alkylating agents are associated with a wide range o toxicities (see Side Bar OX-
ICI IES OF ALKYLA ING DRUGS). Most alkylating agents (ie, melphalan, chlo-
rambucil, cyclophosphamide, and i os amide) cause dose-limiting toxicity to bone
marrow elements (myelosuppression) and, to a lesser extent, intestinal mucosa.
Cyclophosphamide has lesser e ects on peripheral blood platelet counts than do
the other agents. Nausea and vomiting, which are due to neurotoxic e ects, com-
monly ollow agent administration o nitrogen mustard. High-dose therapy with
cyclophosphamide can lead to organ damage, including GI ulceration, cystitis (see
next paragraph), and, less commonly, pulmonary, renal, hepatic, and cardiac tox-
icities (a hemorrhagic myocardial necrosis). Dose-limiting major organ toxicities
associated with alkylating agents are listed in able 45-2.
Cyclophosphamide and i os amide release a nephrotoxic and urotoxic metabolite,
acrolein (see Figure 45-4), which causes a severe hemorrhagic cystitis in high-dose
regimens. T is adverse e ect can be prevented by coadministration o 2-mercap-
toethanesul onate (mesna), which conjugates acrolein in urine. Mesna does not
negate the systemic antitumor activity o the drug. Patients should receive vigorous
intravenous hydration during high-dose treatment. Brisk hematuria in a patient
receiving daily oral therapy should lead to immediate drug discontinuation. Re rac-
tory bladder hemorrhage can become li e threatening, and cystectomy may be
necessary or control o bleeding. Inappropriate secretion o antidiuretic hormone
(Continued)
631
SECTION VIII Chemotherapy of Neoplastic Diseases

MECHANISMS OF TABLE 45-2 Dose-Limiting Extramedullary Toxicities of Single Alkylating Agents


RESISTANCE TO FOLD INCREASE OVER
ALKYLATING AGENTS DRUG MTD,a mg/m 2
STANDARD DOSE MAJOR ORGAN TOXICITIES
Speci cbiochemical changes implicated in Cyclophosphamide 7000 7 Cardiac, hepatic VOD
the development o resistance to an alkyl-
ating agent when used as a single agent I os amide 16,000 2.7 Renal, CNS, hepatic VOD
include the ollowing: Thiotepa 1000 18 GI, CNS, hepatic VOD
Decreased permeation o activelytrans-
Melphalan 180 5.6 GI, hepatic VOD
ported drugs (mechlorethamine and
melphalan). Busul an 640 9 GI, hepatic VOD
Increased intracellular concentrations o Carmustine (BCNU) 1050 5.3 Lung, hepatic VOD
nucleophilicsubstances, principallythiols
such as glutathione, which can conjugate Cisplatin 200 2 PN, renal
withanddetoxi yelectrophilicintermediates.
Carboplatin 2000 5 Renal, PN, hepatic VOD
Increased activityo DNArepair pathways,
which maydi er or the various alkylating aMaximum tolerated dose (MTD; cumulative) in treatment protocols.
agents: CNS, central nervous system; GI, gastrointestinal; PN, peripheral neuropathy; VOD, veno-
occlusive disease.
Increased activityo the complexNER
pathwaycorrelates with resistance to
most chloroethyl and platinumadducts.
MGMTactivitydetermines response
to BCNUand to methylating drugs has been observed in patients receiving cyclophosphamide, usually at doses greater
such as the triazenes, procarbazine, than 50 mg/kg. It is important to be aware o the possibility o water intoxication,
temozolomide, and busul an. because these patients usually are vigorously hydrated to prevent bladder toxicity.
Increased rates o metabolicdegradation All alkylating agents have toxic e ects on the male and emale reproductive systems,
o the activated orms o cyclophospha- causing an o en permanent amenorrhea, and an irreversible azoospermia in men.
mide and i os amide to their inactive keto Alkylating agents are highly leukemogenic. Acute nonlymphocytic leukemia peaks
and carboxymetabolites byaldehyde in incidence ~4 years a er therapy and may a ect up to 5% o patients treated on
dehydrogenase (see Figure 45-4), and regimens containing alkylating drugs. Leukemia o en is preceded by a period o
detoxi cation o most alkylating interme- neutropenia or anemia, and by bone marrow morphology consistent with myelo-
diates byglutathione trans erases. dysplasia. Melphalan, the nitrosoureas, and the methylating agent procarbazine
Loss o the abilityto recognize adducts have the greatest propensity to cause leukemia, while it is less common a er
ormed bynitrosoureas and methylating cyclophosphamide.
agents, as the result o de ective MMR d. What are the mechanisms by which resistance to cyclophosphamide and other
capability, con ers resistance, as does alkylating agents might develop?
de ective checkpoint unction or virtually
T e Side Bar MECHANISMS OF RESIS ANCE O ALKYLA ING AGEN S
all alkylating drugs.
describes mechanisms by which tumors can develop resistance to the alkylating
Impaired apoptoticpathways (eg, overex- agents. One mechanism o resistance is increased rates o metabolic degradation o
pression o bcl-2) con er resistance. the activated orms o cyclophosphamide to its inactive keto and carboxy metabo-
lites by aldehyde dehydrogenase, as well as by inactivation by glutathione trans er-
ase and other enzymatic pathways (see Figure 45-4).
TOXICITIES OF With all alkylating agents, lethality o DNA alkylation depends on the recognition
ALKYLATING DRUGS o the DNA adduct, the creation o DNA strand breaks by repair enzymes, and an
Se e Tab le 45-2) intact apoptotic response. T e speci c repair enzyme complex utilized will depend
on two actors: the chemistry o the DNA adduct ormed and the repair capac-
Most alkylating agents cause dose-limiting
ity o the cell involved. T e process o recognizing and repairing DNA generally
toxicityto bone marrowelements, with
requires an intact NER complex. As described in the answer to Case 45-1a, the
acute myelosuppression that peaks at
ability o the bi unctional nitrogen mustards to orm interstrand DNA cross-links
6-10 days and recovers in 14-21 days.
increases cytotoxicity over alkylation o single strands by mono unctional alkylat-
Alkylating agents are highlytoxicto divid- ing agents. Alkylation o a single strand o DNA (mono-adducts) is repaired by
ing mucosal cells and to hair ollicles, lead- the NER pathway, while the less requent DNA cross-links require participation o
ing to oral mucosal ulceration, intestinal nonhomologous end joining, an error-prone pathway, or the error- ree homologous
denudation, and alopecia. recombination pathway (see Side Bar DNA REPAIR MECHANISMS).
(Continued)
632
Cancer Chemotherapy and Cytotoxic Agents CHAPTER 4 5

Recognition o extensively damaged DNA by p53 can trigger apoptosis. In nondividing


TOXICITIES OF
cells, DNA damage activates a checkpoint that depends on the presence o a normal
ALKYLATING DRUGS
p53 gene. Cells thus blocked in the G1/S inter ace either repair DNA alkylation or
undergo apoptosis. Malignant cells with mutant or absent p53 ail to suspend cell-cycle (Se e Tab le 45-2) (Cont.)
progression, do not undergo apoptosis, and exhibit resistance to alkylating drugs. Neurotoxice ects range romnausea
and vomiting to more severe central and
peripheral toxicities that include altered
CASE 45-2 mental status, coma, generalized seizures,
Carbop atin, cisp atin, and oxa ip atin are re erred to as p atinum coordination com- and cerebellar ataxia.
p exes, and are wide y used cancer chemotherapy agents Other organ toxicities maysupervene a ter
prolonged or high-dose use; these e ects
a. What is the mechanism o action o these agents?
can appear a ter months or years and may
T e platinum coordination complexes have broad antineoplastic activity and have be irreversible and even lethal:
become the oundation or the treatment o many cancers (see able 45-1). T ey Pulmonary brosis.
covalently bind to nucleophilic sites on DNA and share many pharmacological
attributes with alkylators (see Figure 45-1). Vascular endothelial damage may
precipitate veno-occlusive disease (VOD)
Cisplatin, carboplatin, and oxaliplatin enter cells by an active Cu 2+ transporter,
o the liver, an o ten atal side e ect.
C R1. Inside the cell, the chloride, cyclohexane, or oxalate ligands o the three
analogs are displaced by water molecules, yielding a positively charged and highly Nephrotoxic and urotoxic (ie, toxic
reactive molecule. T e aquated species o the drug can react with electron-rich metabolites in the urine that can
molecules, such as sul ydryls, and with various sites on DNA, orming both damage the bladder and lower urinary
intrastrand and interstrand cross-links. T e N-7 o guanine is a particularly tract) e ects.
reactive site, leading to platinum cross-links between adjacent guanines (GG Unstable alkylating agents (particularly
intrastrand cross-links) on the same DNA strand; guanineadenine cross-links mechlorethamine and the nitrosoureas)
also orm and may contribute to cytotoxicity. Interstrand cross-links orm less have strong vesicant properties and cause
requently. DNA-platinum adducts inhibit replication and transcription, lead to damage toveins with repeated use and,
single- and double-stranded breaks and miscoding, and, i recognized by p53 and i extravasated, produce ulceration.
other checkpoint proteins, cause induction o apoptosis. For cisplatin and carbo- Toxice ects on the male and emale
platin, the mismatch repair (MMR) pathway is essential or cytotoxicity, causing reproductive systems, causing an o ten
strand breaks at sites o adduct ormation, creating mispairing o thymine resi- permanent amenorrhea, particularly
dues, and triggering apoptosis. in perimenopausal women, and an
T e analogs di er in the con ormation o their adducts and the e ects o adduct on irreversible azoospermia in men.
DNA structure and unction. T e oxaliplatin adducts are bulkier and less readily Alkylating agents are highlyleukemogenic.
repaired, create a di erent pattern o distortion o the DNA helix, and di er rom Acute nonlymphocytic leukemia,
cisplatin adducts in the pattern o hydrogen bonding to adjacent segments o DNA.
o ten associated with partial or total
Unlike the other platinum analogs, oxaliplatin exhibits a cytotoxicity that does not
deletions o chromosome 5 or 7, peaks
depend on an active MMR system, which may explain its greater activity in colorec-
in incidence ~4 years a ter therapyand
tal cancer. It also seems less dependent on the presence o high-mobility group
maya ect up to 5%o patients treated
(HMG) proteins that are required by the other platinum derivatives.
on regimens containing alkylating drugs.
b. How does the mechanism o action o these agents di er rom the
alkylating agents?
Cisplatin and other platinum complexes do not orm carbonium ion intermediates
like other alkylating agents or ormally alkylate DNA; however, they covalently bind
to nucleophilic sites on DNA causing DNA strand cross-links, and thus share many
pharmacological attributes with alkylators.
c. What are the common toxicities o these agents?
Many o the toxicities o these agents are similar to the alkylating agents (see Side
Bar OXICI IES OF ALKYLA ING DRUGS; also see able 45-2). T e primary
clinical toxicities di er somewhat among cisplatin, carboplatin, and oxaliplatin.
An important dose-limiting toxicity o cisplatin is renal toxicity. o prevent renal
toxicity, it is important to establish a chloride diuresis by the in usion o 1-2 L
o normal saline prior to treatment. T e appropriate amount o cisplatin then is
diluted in a solution containing dextrose, saline, and mannitol, and administered
over 4-6 hours. Ami ostine is a thiophosphate cytoprotective agent that reduces

(Continued)
633
SECTION VIII Chemotherapy of Neoplastic Diseases

renal toxicity associated with repeated administration o cisplatin, but is not


commonly used.
Ototoxicity caused by cisplatin is una ected by diuresis and is mani ested by tin-
nitus and high- requency hearing loss. T e ototoxicity can be unilateral or bilateral,
tends to be more requent and severe with repeated doses, and may be more pro-
nounced in children.
Marked nausea and vomiting occur in almost all patients receiving cisplatin and
usually can be controlled with 5-H 3 antagonists, NK1-receptor antagonists, and
high-dose corticosteroids (see Chapter 33). At higher doses, or a er multiple cycles
o treatment, cisplatin causes a progressive peripheral motor and sensory neuropa-
thy, which may worsen a er discontinuation o the drug and may be aggravated
by subsequent or simultaneous treatment with taxanes or other neurotoxic drugs.
Cisplatin causes mild to moderate myelosuppression, with transient leukopenia
and thrombocytopenia. Anemia may become prominent a er multiple cycles o
treatment. Electrolyte disturbances, including hypomagnesemia, hypocalcemia,
hypokalemia, and hypophosphatemia, are common. Hypocalcemia and hypomag-
nesemia secondary to tubular damage and renal electrolyte wasting may produce
tetany i untreated. Routine measurement o Mg2+ concentrations in plasma is
recommended. Hyperuricemia, hemolytic anemia, and cardiac abnormalities are
rare side e ects. Anaphylactic-like reactions, characterized by acial edema, bron-
choconstriction, tachycardia, and hypotension, may occur within minutes a er
administration and should be treated by intravenous injection o epinephrine and
with corticosteroids or antihistamines.
Carboplatin is relatively well tolerated clinically, causing less nausea, neurotoxic-
ity, ototoxicity, and nephrotoxicity than cisplatin. Instead, the dose-limiting toxic-
ity is myelosuppression, primarily thrombocytopenia. It is more likely to cause a
hypersensitivity reaction; in patients with a mild reaction, premedication, graded
doses o drug, and more prolonged in usion lead to desensitization. Carboplatin is
an e ective alternative or responsive tumors in patients unable to tolerate cisplatin
because o impaired renal unction, re ractory nausea, signi cant hearing impair-
ment, or neuropathy, but doses must be adjusted or renal unction.
T e dose-limiting toxicity o oxaliplatin is a peripheral neuropathy. An acute orm,
o en triggered by exposure to cold liquids, mani ests as paresthesias and/or dysesthe-
sias in the upper and lower extremities, mouth, and throat. It may be caused by rapid
release o oxalate, with depletion o calcium and magnesium, and responds to in usion
o these electrolytes. A second type o peripheral neuropathy is more closely related to
cumulative dose and has eatures similar to cisplatin neuropathy: progressive sensory
neurotoxicity, with dysesthesias, ataxia, and numbness o the extremities. Hematologi-
cal toxicity is mild to moderate, except or rare immune-mediated cytopenias, and
nausea is well controlled with 5-H 3 receptor antagonists. Oxaliplatin may cause an
acute allergic response with urticaria, hypotension, and bronchoconstriction.
All o the platinum analogs are mutagenic, teratogenic, and carcinogenic. Like
other DNA adduct orming drugs, these agents have been associated with pulmo-
nary brosis and the development o leukemia, months to years a er treatment.
d. What are the mechanisms or resistance and sensitivity to the platinum coordi-
nation complex agents?
Resistance to the platinum analogs likely is multi actorial (see Side Bar MECHA-
NISMS OF RESIS ANCE O ALKYLA ING AGEN S), and the compounds di er
in their degree o cross-resistance. Carboplatin shares cross-resistance with cispla-
tin in most experimental tumors, while oxaliplatin does not.
Repair o platinum-DNA adducts requires participation o the NER pathway.
Inhibition or loss o NER increases sensitivity to cisplatin in ovarian cancer
patients, while overexpression o NER components is associated with poor

(Continued)

634
Cancer Chemotherapy and Cytotoxic Agents CHAPTER 4 5

response to cisplatin or oxaliplatin-based therapy in lung, colon, and gastric


cancer. Higher levels o expression o the NER component ERCC1, in tumor
cells and peripheral white blood cells are associated with a lower response rate in
patients with solid tumors.
Resistance to cisplatin, but not oxaliplatin, appears to be partly mediated through
loss o unction in the MMR proteins (hMLH1, hMLH2, or hMSH6), which recog-
nize platinum-DNA adducts and initiate apoptosis.
In the absence o e ective repair o DNA-platinum adducts, sensitive cells cannot
replicate or transcribe a ected portions o the DNA strand. However, it is clear that
some DNA polymerases can bypass adducts, especially those created by cisplatin.
Oxaliplatin adducts are less easily bypassed. It remains unproven whether these
polymerases contribute to resistance.
Platinum coordination complex drugs enter cells by active transport and cisplatin
resistance related to loss o active uptake has been demonstrated in vitro. T e com-
pounds are actively extruded rom cells by A P7A and A P7B copper transporters
and by multidrug resistance protein 1 (MRP1/ABCC1); variable expression o these
transporters may contribute to clinical resistance. Overexpression o copper e ux
transporters, A P7A and A P7B, correlates with poor survival af er cisplatin-based
therapy or ovarian cancer.

MECHANISMS OF ACTION AND RESISTANCE OF ANTIMETABOLITES (SEE TABLE 45-3)


CLASSIFICATION DRUG MECHANISMS OF ACTION MECHANISMS OF RESISTANCE
Folic Acid Analogs Methotrexate (amethopterin) Binds DHFR and prevents the ormation See Side Bar MECHANISMS OF
(Anti olates) High-dose methotrexate with o FH4 (see Figure 45-6) and allows the RESISTANCE TO ANTIMETABOLITES
leucovorin rescue (HDM-L) accumulation o the toxic inhibitor
Trimetrexate substrate FH2 polyglutamate, thus shutting
down one-carbon trans er reactions
needed or the de novo synthesis o purine
nucleotides, thymidylate, and the synthesis
o DNA and RNA (see Figures 45-1 and 45-3)

Pemetrexed (MTA) Inhibits DHFR, and is readily converted See Side Bar MECHANISMS OF
to polyglutamates, which more potently RESISTANCE TO ANTIMETABOLITES
inhibit glycinamide ribonucleotide
ormyltrans erase (GART) and thymidylate
synthase (TS) (see Figures 45-1 and 45-6)

Pralatrexate Exert their primary inhibitory e ects on TS See Side Bar MECHANISMS OF
Raltitrexed (see Figure 45-6) RESISTANCE TO ANTIMETABOLITES

Lometrexol Primary inhibitory e ects are on GART to See Side Bar MECHANISMS OF
block early steps in purine biosynthesis (see RESISTANCE TO ANTIMETABOLITES
Figure 45-6)

Pyrimidine 5-Fluorouracil (5-FU) Enzymatically converted in cells to the See Side Bar MECHANISMS OF
Analogs Floxuridine (FUdR, f uorodeoxyuridine) nucleotide orms (see Figure 45-7); RESISTANCE TO ANTIMETABOLITES
Capecitabine FUTP is incorporated into RNA, FdUTP is
incorporated into DNA, and FdUMP inhibits
TS (see Figures 45-1, 45-7, and 45-8)

Cytarabine (cytosine arabinoside, Ara-C) Enzymatically converted in cells to its See Side Bar MECHANISMS OF
active orm, the 5-monophosphate RESISTANCE TO ANTIMETABOLITES
ribonucleotide (Ara-CMP), by deoxycytidine
kinase (dCK); Ara-CMP is phosphorylated
to orm Ara-CDP and Ara-CTP, which inhibit
DNA polymerase, thus blocking replication
and repair synthesis, and elongation (see
Figures 45-1 and 45-3)
(Continued)
635
SECTION VIII Chemotherapy of Neoplastic Diseases

CLASSIFICATION DRUG MECHANISMS OF ACTION MECHANISMS OF RESISTANCE


Azacytidine (5-azacytidine) Incorporates into DNA and orms Sensitivity to the azanucleosides
Decitabine (2-deoxy-5-azacytidine) covalent adducts with DNA cytosine correlates with the presence o the
methyltrans erase, leading to its depletion in ux carrier, human equilibrative
and global demethylation o DNA, and nucleoside transporter 1 (hENT1)
tumor cell di erentiation and apoptosis
Decitabine also induces double-strand
DNA breaks

Gemcitabine (2,2-di uorodeoxyuridine) Converted to di uorodeoxycytidine di- See Side Bar MECHANISMS OF
and triphosphates (dFdCDP and dFdCTP) RESISTANCE TO ANTIMETABOLITES
which inhibit DNA synthesis by multiple
mechanisms including incorporation o
dFdCTP into DNA and subsequent DNA
strand termination (see Figure 45-1)

Purine Analogs 6-Mercaptopurine (6-MP) Converted by hypoxanthine guanine See Side Bar MECHANISMS OF
6-Thioguanine (6-TG) phosphoribosyl trans erase (HGPRT) into RESISTANCE TO ANTIMETABOLITES
the corresponding ribonucleotides (6-MP
to 6-thioGMP and 6-TG to T-IMP)
T-IMP inhibits the synthesis o ribosyl-
5-phosphate and is also converted to
6-thioGMP
6-thioGMP is incorporated into DNA
causing strand breaks and base mispairing
(see Figures 45-1 and 45-3)

Fludarabine phosphate Phosphorylated by dCKto the active See Side Bar MECHANISMS OF
triphosphate derivative, which inhibits DNA RESISTANCE TO ANTIMETABOLITES
polymerase, DNA primase, DNA ligase,
and ribonucleotide reductase (RNR), and
becomes incorporated into DNA and RNA;
chain termination occurs when incorporated
into DNA (see Figure 45-1); incorporation
into RNA inhibits RNA unction, RNA
processing, and mRNA translation

Cladribine (2-chlorodeoxyadenosine, A ter phosphorylation by dCKto cladribine See Side Bar MECHANISMS OF
2-CdA) triphosphate, it is incorporated into DNA RESISTANCE TO ANTIMETABOLITES
producing DNA strand breaks, depleting
NAD and ATP, and leading to apoptosis (see
Figure 45-1); it is a potent inhibitor o RNR

Clo arabine Phosphorylated by dCKto the active


(2-chloro-2- uoro-arabinosyladenine) triphosphate derivative and is incorporated
into DNA where it terminates DNA synthesis
and leads to apoptosis; it inhibits RNR

Nelarabine Active metabolite is transported into


(6-methoxy-arabinosyl-guanine) tumor cells, where it is activated by dCK
to Ara-G triphosphate (Ara-GTP), which
is incorporated into DNA and terminates
DNA synthesis

Pentostatin (2-deoxyco ormycin) A transition-state analog o the


intermediate in the adenosine deaminase
(ADA) reaction that potently inhibits ADA,
which leads to accumulation o intracellular
adenosine and deoxyadenosine
nucleotides, which can block DNA
synthesis by inhibiting RNR; also inactivates
S-adenosyl homocysteine hydrolase, can
inhibit RNA synthesis and its triphosphate
derivative is incorporated into DNA,
resulting in strand breakage

636
Cancer Chemotherapy and Cytotoxic Agents CHAPTER 4 5

TABLE 45-3 Antimetabolites


TYPE OF AGENT NONPROPRIETARY NAMES DISEASE
Folic acid analogs Methotrexate Acute lymphocytic leukemia; choriocarcinoma;
(amethopterin) breast, head, neck and lung cancers; osteogenic
sarcoma; bladder cancer
Pemetrexed Mesothelioma, lung cancer
Pyrimidine Fluorouracil (5- uorouracil; Breast, colon, esophageal, stomach, pancreas,
analogs 5-FU), capecitabine head and neck; premalignant skin lesion
(topical)
Cytarabine (cytosine Acute myelogenous and acute lymphocytic
arabinoside) leukemia; non-Hodgkins lymphoma
Gemcitabine Pancreatic, ovarian, lung cancer
5-aza-cytidine Myelodysplasia
Deoxy-5-aza-cytidine Myelodysplasia
Purine analogs Mercaptopurine Acute lymphocytic and myelogenous
and related (6-mercaptopurine; 6-MP) leukemia; small cell non-Hodgkins
inhibitors lymphoma
Pentostatin Hairy cell leukemia; chronic lymphocytic
(2-deoxyco ormycin) leukemia; small cell non-Hodgkins lymphoma
Fludarabine Chronic lymphocytic leukemia
Clo arabine Acute myelogenous leukemia
Nelarabine T-cell leukemia, lymphoma

A Thymidylate s ynthe s is

FH4 Glu n dihydrofola te


re ducta s e

N5 -10 me thyle ne FH2 Glu n


FH4 Glu n thymidyla te +
+ syntha s e TMP
dUMP

B De n ovo purine s ynthe s is

P RP P GAR AICAR
+ + +
a s pa rtate N-10 formyl GAR FH4 Glu n
FH4 Glu n tra ns formyla s e

AICAR IMP
+ +
N-10 formyl AICAR FH4 Glu n
FH4 Glu n tra ns formyla s e

REACTION INHIBITED BY:


me thotrexa te
me thotrexa te polygluta ma te s FH2 Glu n

FIGURE 45-6 Sites o action o methotrexate and its polyglutamates. AICAR, aminoimidazole carboxamide; dUMP, deoxy-
uridine monophosphate; FH2Glu n, dihydro olate polyglutamate; FH4Glu n, tetrahydro olate polyglutamate; GAR, glycinamide
ribonucleotide; IMP, inosine monophosphate; PRPP, 5-phosphoribosyl-1-pyrophosphate; TMP, thymidine monophosphate.
637
SECTION VIII Chemotherapy of Neoplastic Diseases

5
FUR FUMP FUDP FUTP RNA
1
1 Uridine phos phoryla s e
Dihydro 5-FU 3 2 Thymidine phos phoryla s e
3 P hos phoribosyl tra ns fe ra se
7 (P RP P )
5-FU 6 4 Thymidine kina s e
inhibitor of 5 Uridine kina s e
thymidyla te
syntha s e
6 Ribonucle otide re ducta s e
7 Dihydropyrimidine de hydroge na s e
2
4
FUdR FdUMP FdUDP FdUTP DNA
FIGURE 45-7 Activation pathways or 5- uorouracil (5-FU) and 5- oxuridine (FUR). FdUDP, uorodeoxyuridine diphosphate; FdUMP, uorode-
oxyuridine monophosphate; FdUTP, uorodeoxyuridine triphosphate; FUDP, oxuridine diphosphate; FUdR, uorodeoxyuridine; FUMP, oxuridine
monophosphate; FUTP, oxuridine triphosphate; PRPP, 5-phosphoribosyl-1-pyrophosphate.

MECHANISMS OF FdUMP
RESISTANCE TO dUMP dTMP dTTP
ANTIMETABOLITES
Acquired resistance to methotrexate a ects thymidyla te
each known step in methotrexate action: syntha s e
Impaired transport o methotrexate N5-10 me thyle ne FH2 Glu n
into cells FH4 Glu n
Production o altered orms o DHFR Othe r a ctions of 5-FU nucle otide s :
that have decreased a nity or the Inhibition of RNA proce s s ing

inhibitor Incorpora tion into DNA

Increased concentrations o intracellular


DHFRthrough gene ampli cation or FIGURE 45-8 Site o action o 5- uoro-2-deoxyuridine-5-phosphate (5-FdUMP). 5-FU,
altered gene regulation 5- uorouracil; dUMP, deoxyuridine monophosphate; FdUMP, uorodeoxyuridine monophos-
phate; FH2Glun, dihydro olate polyglutamate; FH4Glun, tetrahydro olate polyglutamate; dTMP,
Decreased abilityto synthesize deoxythymidine monophosphate; dTTP, deoxythymidine triphosphate.
methotrexate polyglutamates
Increased expression o a drug ef ux
transporter o the MRPclass (see CASE 45 3
Chapter 5 in Goodman &Gilmans The
A 6-year-o d boy is being treated or acute ymphob astic eukemia (ALL) using
Pharmacological Basis of Therapeutics,
methotrexate with eucovorin rescue
12th Edition or a comprehensive
description o drug ef uxtransporters) a. What is the mechanism o action o methotrexate?
Resistance to pemetrexed is incompletely Methotrexate is a olic acid analog, or anti olate, that inter eres with olic acid
understood but might arise romthe metabolic pathways (see Figure 45-6). Folic acid is an essential dietary actor that
ollowing is converted by enzymatic reduction to a series o tetrahydro olate (FH 4) co actors
Loss o in uxtransport that provide methyl groups or the synthesis o precursors o DNA (thymidylate
and purines) and RNA (purines). Inter erence with FH4 metabolism reduces the cel-
Thymidylate synthesis (TS) ampli cation
lular capacity or one-carbon trans er and the necessary methylation reactions in the
Changes in purine biosyntheticpathways synthesis o purine ribonucleotides and thymidine monophosphate ( MP), thereby
Loss o polyglutamation inhibiting DNA replication (see Figure 45-1).
Resistance tothe cytotoxice ects o 5-FU T e primary target o methotrexate is the enzyme dihydro olate reductase (DHFR;
or FUdRhas been ascribed to the ollowing: see Figure 45-6). Inhibition o DHFR leads to partial depletion o the FH 4 co actors
Loss or decreased activityo the enzymes (N5-10 methylene tetrahydro olic acid and N-10 ormyl tetrahydro olic acid). T e
necessary or activation o 5-FU absence o reduced olates shuts down the one-carbon trans er reactions crucial
or the de novo synthesis o purine nucleotides and thymidylate, and interrupts
Ampli cation o TS
the synthesis o DNA and RNA. In addition, methotrexate, like its physiologi-
Mutation o TSto a ormthat is not cal counterparts (the olates), undergoes conversion to a series o polyglutamates
inhibited byFdUMP
(Continued)
638
Cancer Chemotherapy and Cytotoxic Agents CHAPTER 4 5

(M X-PGs) in both normal and tumor cells. T ese M X-PGs constitute an intra-
MECHANISMS OF
cellular storage orm o olates and olate analogs, and dramatically increase inhibi-
RESISTANCE TO
tory potency o the analog or additional sites, including thymidylate synthase ( S)
and 2 early enzymes in the purine biosynthetic pathway (see Figure 45-6). Finally, ANTIMETABOLITES (Cont.)
the dihydro olic acid polyglutamates that accumulate in cells behind the blocked High levels o the degradative enzymes
DHFR reaction also act as inhibitors o S and other enzymes (see Figure 45-6). dihydrouracil dehydrogenase and
As with most antimetabolites, methotrexate is only partially selective or tumor thymidine phosphorylase
cells and kills rapidly dividing normal cells, such as those o the intestinal epithe- Insu icient concentrations o 5,
lium and bone marrow. Folate antagonists kill cells during the S phase o the cell 10-methylenetetrahydro olate, thus
cycle (see Figure 45-3) and are most e ective when cells are proli erating rapidly. preventing ormation o maximal
levels o the inhibited ternarycomplex
b. What is the rationale or the use o leucovorin?
withTS
o unction as a co actor in one-carbon trans er reactions, olate must rst be
Resistance to Ara-Chas been attributed to
reduced by DHFR to FH 4.
the ollowing:
Introduction o high-dose methotrexate regimens with rescue o host toxicity
Reduced in uxthrough nucleoside
by the reduced olate, leucovorin ( olinic acid, citrovorum actor, 5- ormyl tetra-
carriers (eg, hENT1 transporter
hydro olate, N5- ormyl FH 4), extends the e ectiveness o this drug to treat both
mutation)
systemic and CNS lymphomas, osteogenic sarcoma, and leukemias. T e admin-
istration o leucovorin within 24 hours a er high-dose in usions o methotrexate Reduced activities o catabolicenzymes
allows the use o regimens that produce cytotoxic concentrations o drug in the (deoxycytidine kinase [dCK] is the rate-
CSF and protects against leukemic meningitis. T e toxic e ects o methotrexate limiting activating enzyme)
are rapidly terminated by administering leucovorin which repletes the intracellu- Increased activities o anabolic
lar pool o FH 4 co actors. enzymes
c. How might pharmacogenetics a ect the response o this patient to anti olate Cytidine deaminase which converts
treatment? Ara-Cto nontoxicAraU
T e C677 substitution in methylenetetrahydro olate reductase (M HFR) reduces dCMPdeaminase which converts
the activity o the enzyme that generates methylenetetrahydro olate, the co actor Ara-CMPto inactive Ara-UMP
or S (see Figure 45-6), and thereby increases methotrexate toxicity, especially 5 nucleotidase which degrades
GI toxicity (see Chapter 7 in Goodman & Gilmans T e Pharmacological Basis of Ara-CMP
T erapeutics, 12th Edition or a more comprehensive description o the pharma- Gemcitabine resistance is ound in tumors
cogenetics o methotrexate). T e presence o this polymorphism in leukemic cells with lowlevels o dCKand high levels o
con ers increased sensitivity to methotrexate and might also modulate the toxicity cytidine deaminase.
and therapeutic e ect o pemetrexed, a predominant S inhibitor. Likewise, poly-
The most common mechanismo 6-MP
morphisms in the promoter region o S govern the translation e ciency o this
resistance is de ciencyor complete lacko
message and, by governing the intracellular levels o S in both normal and tumor
the activating enzyme, HGPRT,or increased
cells, modulate the response and toxicity o both anti olates and f uoropyrimidines
alkaline phosphate activity; other mecha-
such as 5-f uorouracil. Other common polymorphisms that a ect methotrexate
nisms or resistance include the ollowing:
pharmacotherapy include those in the genes that code or DHFR and proline trans-
ormylases (see Chapter 7 in Goodman & Gilmans T e Pharmacological Basis of Decreased drug uptake
T erapeutics, 12th Edition). Increased ef uxdue to one o several
active transporters
d. How do other olate antagonists di er rom methotrexate in their mechanism o
action other important properties? Alteration in allostericinhibition o
ribosylamine 5-phosphate synthase
Pemetrexed and its polyglutamates have a somewhat di erent spectrum o biochem-
ical actions. Like methotrexate, pemetrexed inhibits DHFR, but as a polyglutamate, Impaired recognition o DNAbreaks and
it even more potently inhibits glycinamide ribonucleotide ormyltrans erase (GAR ) mismatches due to loss o a component
and thymidylate synthase ( S). Unlike methotrexate, it produces little change in the (MSH6) o MMR
pool o reduced olates, indicating that the distal sites o inhibition ( S and GAR ) Resistance to udarabine is associated
predominate. Its pattern o deoxynucleotide depletion, as studied in cell lines, also with the ollowing:
di ers rom methotrexate, as it causes a greater all in thymidine triphosphate ( P) Decreased activityo dCK(the enzyme
than in other triphosphates. Like methotrexate, it induces p53 and cell-cycle arrest, that phosphorylates and thereby
but this e ect does not seem to depend on downstream induction o p21. Pralatrex- activates the drug)
ate is more e ectively taken up and polyglutamated than methotrexate.
Increased drug ef ux
New olate antagonists that are better substrates or the reduced olate carrier have
Increased ribonucleotide reductase
been identi ed. In e orts to bypass the obligatory membrane transport system and
(RNR) activity
(continues)
(Continued)
639
SECTION VIII Chemotherapy of Neoplastic Diseases

to acilitate penetration o the blood-brain barrier, lipid-soluble olate antagonists


MECHANISMS OF
also have been synthesized. rimetrexate, a lipid-soluble analog, has modest antitu-
RESISTANCE TO
mor activity, primarily in combination with leucovorin rescue. However, it is ben-
ANTIMETABOLITES (Cont.) e cial in the treatment o Pneumocystis jiroveci (Pneumocystis carinii) pneumonia
Resistance to cladribine is associated with where leucovorin provides di erential rescue o the host but not the parasite.
the ollowing:
e. What are the mechanisms o acquired resistance to methotrexate and pemetrexed?
Loss o the activating enzyme, dCK
Resistance to the cytotoxic e ects o olate analogs can develop by a number o
Increased expression o RNR mechanisms that a ect drug transport into tumor cells, mutations in the enzymes
Increased active ef uxbyABCG2 or involved in olic acid metabolism, changes in expression o these enzymes, and
other members o the ABCcassette increased e ux o drug rom tumor cells (see Side Bar MECHANISMS OF RESIS-
amilyo transporters ANCE O AN IME ABOLI ES).

CASE 45 4
A 58-year-o d man is prescribed topica 5-FU to treat precancerous skin esions on his
ace and arms
a. What is the mechanism o action o 5-FU?
5-FU is an analog o thymine. It is enzymatically converted in cells to a nucleotide,
f oxuridine monophosphate (FUMP), by one o several pathways (see Figure 45-7).
FUMP can then ollow several di erent pathways (see Figure 45-7). One pathway
leads to the ormation o the triphosphate FU P, which is incorporated into RNA.
5-FU incorporation into RNA is one mechanism o cytotoxicity as the result o
major e ects on both the processing and unctions o RNA.
In an alternative reaction sequence crucial or antineoplastic activity, FUMP is
reduced to FUDP by RNR to the deoxynucleotide level and then to FdUMP, a
potent inhibitor o thymidylate synthesis ( S; see Figures 45-7 and 45-8). S is
required or the physiological conversion o dUMP to d MP, which is phosphory-
lated to d P and incorporated into DNA (see Figure 45-8). T us, blocking S
prevents DNA synthesis (see Figure 45-1) and is another mechanism o cytotoxic-
ity. In 5-FUtreated cells, both f uorodeoxyuridine triphosphate (FdU P) and
deoxyuridine triphosphate (dU P) (the substrate that accumulates behind the
blocked S reaction) incorporate into DNA in place o the depleted physiological
d P (see Figure 45-7). T e signi cance o the incorporation o FdU P and dU P
into DNA is unclear. Presumably, the incorporation o deoxyuridylate and/or f uo-
rodeoxyuridylate into DNA would call into action the excision-repair process. T is
process may result in DNA strand breakage because DNA repair requires d P, but
this substrate is lacking as a result o S inhibition.
T e olate co actor, N5,10 methylene tetrahydro olate, and FdUMP orm a cova-
lently bound ternary complex with S (see Figure 45-8). T is inhibited complex
resembles the transition state ormed during the enzymatic conversion o dUMP
to thymidylate. T e physiological complex o S- olate-dUMP progresses to the
synthesis o thymidylate by trans er o the methylene group and 2 hydrogen atoms
rom olate to dUMP, but this reaction is blocked in the inhibited complex o S-
FdUMP- olate by the stability o the f uorine carbon bond on FdUMP; sustained
inhibition o the enzyme results.
5-FU also may be converted by thymidine phosphorylase to the deoxyriboside f uo-
rodeoxyuridine (FUdR) and then by thymidine kinase to FdUMP (see Figure 45-7).
T ese complex metabolic pathways or the generation o FdUMP may be bypassed
through administration o f oxuridine (f uorodeoxyuridine; FUdR), which is converted
directly to FdUMP by thymidine kinase (FUdR rarely is used in clinical practice).
b. When used to treat tumors, what are the mechanisms o resistance to the
f uoropyrimidines?
Resistance to the cytotoxic e ects o 5-FU or FUdR has been ascribed to loss or
decreased activity o the enzymes necessary or activation o 5-FU, ampli cation o
(Continued)
640
Cancer Chemotherapy and Cytotoxic Agents CHAPTER 4 5

S, mutation o S to a orm that is not inhibited by FdUMP, and high levels o the
degradative enzymes dihydrouracil dehydrogenase and thymidine phosphorylase.
S levels are controlled by an autoregulatory eedback mechanism wherein the
unbound enzyme interacts with and inhibits the translational e ciency o its own
mRNA, which provides or the rapid S modulation needed or cellular division.
When S is bound to FdUMP, inhibition o translation is relieved, and levels o ree
S rise. T is may be an important mechanism by which malignant cells become
insensitive to the e ects o 5-FU.
Some malignant cells appear to have insu icient concentrations o N 5,10 methy-
lene tetrahydro olate, and thus cannot orm maximal levels o the inhibited
ternary complex with S. Addition o exogenous olate in the orm o N5- ormyl
FH 4 (leucovorin) increases ormation o the complex and enhances responses to
5-FU in clinical trials.
c. What are the important toxicities when the f uoropyrimidines are used to
treat tumors?
T e earliest untoward symptoms during a course o therapy are anorexia and nau-
sea, ollowed by stomatitis and diarrhea, which constitute reliable warning signs
that a su cient dose has been administered. Mucosal ulcerations occur throughout
the GI tract and may lead to ulminant diarrhea, shock, and death, particularly in
patients who are dihydropyrimidine dehydrogenase de cient.
5-FU is inactivated by reduction o the pyrimidine ring in a reaction carried out
by dihydropyrimidine dehydrogenase (DPD), which is ound in the liver, intestinal
mucosa, tumor cells, and other tissues. Inherited de ciency o this enzyme leads to
greatly increased sensitivity to the drug (see Figure 45-7). T e rare individual who
totally lacks this enzyme may experience pro ound drug toxicity ollowing conven-
tional doses o the drug. DPD de ciency can be detected either by enzymatic or
molecular assays using peripheral white blood cells or by determining the plasma
ratio o 5-FU to its metabolite, 5-f uoro-5,6-dihydrouracil.
Myelosuppression is also a major toxicity. Loss o hair, occasionally progressing to
total alopecia, nail changes, dermatitis, and increased pigmentation and atrophy
o the skin may be encountered. Hand- oot syndrome, a particularly prominent
adverse e ect o capecitabine (an orally administered prodrug o 5-FU), consists
o erythema, desquamation, pain, and sensitivity to touch o the palms and soles.
Acute chest pain with evidence o ischemia in the electrocardiogram may result
rom coronary artery vasospasms during or shortly a er 5-FU in usion.
T e signi cant risk o toxicity with f uoropyrimidines emphasizes the need or very
skill ul supervision by physicians amiliar with the action and possible hazards.

CASE 45 5
A 32-year-o d woman is being treated with 6-MP or her eukemia
a. What is the mechanism o action o 6-mercaptopurine?
6-MP is a purine analog. 6-MP and a related 6-thiopurine, 6-thioguanine (6- G),
unction as analogs o the natural purines, hypoxanthine and guanine. T e sub-
stitution o sul ur or oxygen on C6 o the purine ring creates compounds that
are readily transported into cells, including activated malignant cells. Nucleotides
ormed rom 6-MP and 6- G inhibit de novo purine synthesis and also become
incorporated into nucleic acids (see Figure 45-1). Both 6- G and 6-MP are excel-
lent substrates or hypoxanthine guanine phosphoribosyl trans erase (HGPR )
and are converted in a single step to the ribonucleotides 6-thioguanosine-5-
monophosphate (6-thioGMP) and 6-thioinosine-5-monophosphate ( -IMP),
respectively. Because -IMP is a poor substrate or guanylyl kinase, the enzyme
that converts guanosine monophosphate (GMP) to guanosine diphosphate (GDP),
-IMP accumulates intracellularly and in a second step is converted to 6- GMP.
(Continued)
641
SECTION VIII Chemotherapy of Neoplastic Diseases

-IMP inhibits the new ormation o ribosyl-5-phosphate, as well as conversion o


inosine-5-monophosphate (IMP) to adenine and guanine nucleotides. O these,
the most important point o attack seems to be the reaction o glutamine and PRPP
to orm ribosyl-5-phosphate, the rst committed step in the de novo pathway.
6-thioguanine nucleotide is incorporated into DNA, where it induces strand breaks
and base mispairing. Strand breaks depend on the presence o an intact MMR sys-
tem, the absence o which leads to resistance.
b. What are the mechanisms o resistance to the thiopurine antimetabolites?
T e most common mechanism o 6-MP resistance observed in vitro is de ciency
or complete lack o the activating enzyme, HGPR , or increased alkaline phosphate
activity. Other mechanisms or resistance are listed in the Side Bar MECHANISMS
OF RESIS ANCE O AN IME ABOLI ES.
c. What are the primary toxicities o 6-MP?
T e principal toxicity o 6-MP is bone marrow depression, although in general,
this side e ect develops more gradually than with olic acid antagonists; accord-
ingly, thrombocytopenia, granulocytopenia, or anemia may not become apparent
or several weeks. When depression o normal bone marrow elements occurs, dose
reduction usually results in prompt recovery, although myelosuppression may be
severe and prolonged in patients with a polymorphism a ecting thiopurine methyl-
trans erase ( PM ), one o the enzymes responsible or the metabolic degradation
o 6-MP.
PM catalyzes the methylation o the sul ydryl group on 6-MP, which leads
to subsequent oxidation. Enzymatic activity o PM ref ects the inheritance o
polymorphic alleles; up to 15% o the Caucasian population has decreased enzyme
activity (see Chapter 7 in Goodman & Gilmans T e Pharmacological Basis of T era-
peutics, 12th Edition). Low levels o erythrocyte PM activity are associated with
increased drug toxicity in individual patients and a lower risk o relapse a er treat-
ment with 6-MP. esting or these MP polymorphisms prior to treatment is
recommended.
Anorexia, nausea, or vomiting is seen in ~25% o adults, but stomatitis and diarrhea
are rare; mani estations o GI e ects are less requent in children than in adults.
Jaundice and hepatic enzyme elevations occur in up to one-third o adult patients
treated with 6-MP and usually resolve upon discontinuation o therapy. T eir
appearance has been associated with bile stasis and hepatic necrosis on biopsy.
6-MP and its derivative, azathioprine, predispose to opportunistic in ections such
as reactivation o hepatitis B, ungal in ection, and Pneumocystis pneumonia, and
an increased incidence o squamous cell malignancies o the skin. 6-MP has tera-
togenic e ects during the rst trimester o pregnancy, and AML has been reported
a er prolonged 6-MP therapy or Crohns disease.

MECHANISMS OF ACTION AND RESISTANCE OF CYTOTOXIC NAURAL PRODUCTS (SEE TABLE 45-4)
CLASSIFICATION DRUG MECHANISMS OF ACTION MECHANISMS OF RESISTANCE
Microtubule-Damaging Vinblastine sul ate Bind to tubulin blocking its polymerization See Side Bar MECHANISMS OF
AgentsVinca Alkaloids Vinorelbine with tubulin into microtubules, thus RESISTANCE TO CYTOTOXIC
Vincristine sul ate preventing mitotic spindle ormation which NATURAL PRODUCTS
blocks cells in mitosis leading to apoptosis (see
Figures 45-1 and 45-3)

Microtubule-Damaging Paclitaxel Bind to tubulin site (di erent than the vinca See Side Bar MECHANISMS OF
AgentsTaxanes Nab-paclitaxel alkaloids) and promote rather than inhibit RESISTANCETO CYTOTOXIC
Docetaxel microtubule ormation causing aberrant NATURAL PRODUCTS
microtubule structures away rom the centriole
and arrest in mitosis (see Figures 45-1 and 45-3)
(Continued)

642
Cancer Chemotherapy and Cytotoxic Agents CHAPTER 4 5

CLASSIFICATION DRUG MECHANISMS OF ACTION MECHANISMS OF RESISTANCE


Microtubule-Damaging Estramustine Bind to tubulin and microtubule-associated
AgentsEstramustine proteins, causing microtubule disassembly and
antimitotic actions (see Figure 45-1)

Microtubule-Damaging Ixabepilone Bind to tubulin (at a site distinct rom taxanes) See Side Bar MECHANISMS OF
AgentsEpothilones and trigger microtubule nucleation at multiple RESISTANCE TO CYTOTOXIC
sites away rom the centriole (see Figure 45-1) NATURAL PRODUCTS

Topoisomerase inhibitors Topotecan Bind and stabilize the normally transient DNA- See Side Bar MECHANISMS OF
Camptothecin Analogs Irinotecan (CPT-11) topoisomerase I cleavable complex, inhibiting RESISTANCETO CYTOTOXIC
religation and resulting in single-stranded DNA NATURAL PRODUCTS
strand breaks (reversible) and double-stranded
DNA strand breaks (irreversible)
(see Figure 45-1)

Topoisomerase inhibitors Etoposide (VP-16-213) Form a ternary complex with topoisomerase II See Side Bar MECHANISMS OF
Epipodophyllotoxins Teniposide (VM-26) and DNA that prevents resealing o the strand RESISTANCETO CYTOTOXIC
break; the enzyme remains bound to the ree NATURAL PRODUCTS
end o the broken DNA strand, leading to an
accumulation o DNA breaks and cell death (see
Figure 45-1)

Anticancer Dactinomycin Bind double-stranded DNA, which prevents


AntibioticsActinomycins (actinomycin D) DNA-dependent RNA polymerase transcription;
also causes single-strand breaks in DNA through
a ree-radical intermediate or as a result o the
action o topoisomerase II (see Figure 45-2)

Anticancer Antibiotics Doxorubicin Intercalate with DNA, directly inhibiting


Anthracyclines and Daunorubicin transcription and replication
Anthracenediones (daunomycin, Form a tripartite complex with topoisomerase
rubidomycin) II and DNA, which inhibits the religation o the
Idarubicin broken DNA strands, leading to apoptosis (see
Epirubicin Figures 45-1 and 45-2)
Valrubicin Their quinone groups also generate ree radicals
Mitoxantrone which attack DNA and oxidize DNA bases

Anticancer Bleomycin Binds to DNA and the activated complex


AntibioticsBleomycin generates ree radicals that cause oxidative
damage to the deoxyribose o thymidylate and
other nucleotides, leading to single- and double-
stranded breaks in DNA; an excess o breaks
results in apoptosis (see Figure 45-2)

Anticancer Mitomycin Chemical reduction o the quinone and loss


AntibioticsMitomycin (mitomycin-C) o the methoxy group converts mitomycin to
a bi unctional or tri unctional alkylating agent
that inhibits DNA synthesis and orms DNA
cross-links; attempts to repair DNA lead to strand
breaks (see Figures 45-1 and 45-2)

Trabectedin Trabectedin Binds to the minor groove o DNA, allowing the


alkylation o guanine and bending the helix
toward the major groove; the bulky DNA adduct
is recognized by the transcription-coupled
nucleotide excision repair (NER) complex, which
initiates attempts to repair the damaged strand,
converting the adduct to a double-stranded
break

Enzymes L-asparaginase (L-ASP) Catalyzes the hydrolysis o circulating asparagine,


Pegaspargase thus depriving lymphocytic leukemias (which
(PEG-L-asparaginase) lack asparagine synthetase) o asparagine,
leading to cell death (see Figure 45-1)

643
SECTION VIII Chemotherapy of Neoplastic Diseases

MECHANISMS OF TABLE 45-4 Natural Products


RESISTANCE TO CYTOTOXIC TYPE OF AGENT NONPROPRIETARY NAMES DISEASE
NATURAL PRODUCTS
Vinca alkaloids Vinblastine Hodgkins disease; non-Hodgkins
The vinca alkaloids share cross-resistance Vinorelbine lymphoma; testis cancer; breast cancer;
to manytumors and their antitumor lung cancer
e ects are blocked by:
Vincristine Acute lymphocytic leukemia;
Increased expression o the mdr-1 neuroblastoma; Wilmstumor;
gene and its product, P-glycoprotein, a rhabdomyosarcoma; Hodgkins disease;
membrane ef uxtransporter non-Hodgkins lymphoma
MRPand the closelyrelated breast Taxanes Paclitaxel Ovarian, breast, lung, prostate, bladder,
cancer resistance protein, maymediate Docetaxel head and neck cancer
multidrug resistance
Epipodophyllotoxins Etoposide Testis, small cell lung and other lung
Mutations in tubulin or in the relative cancer; breast cancer; Hodgkins
expression o iso orms o tubulin; both disease; non-Hodgkins lymphomas;
changes prevent the inhibitors rom acute myelogenous leukemia; Kaposis
e ectivelybinding to their target sarcoma
The mechanismo resistance to taxanes Teniposide Acute lymphoblastic leukemia in
clinicallyis not known, but several children
mechanisms are possible based on
studies in vitro: Camptothecins Topotecan irinotecan Ovarian cancer; small cell lung cancer;
colon cancer
Increased expression o the mdr-1
gene and its product, P-glycoprotein Antibiotics Dactinomycin (actinomycin D) Choriocarcinoma; Wilmstumor;
rhabdomyosarcoma; testis cancer;
Tubulin mutations (which may Kaposis sarcoma
heighten sensitivityto vinca alkaloids)
An increase in survivin expression, an Daunorubicin (daunomycin, Acute myelogenous and acute
rubidomycin) lymphocytic leukemia
antiapoptotic actor
An increase in aurora kinase Doxorubicin So t-tissue, osteogenic, and other
expression, an enzyme that promotes sarcoma; Hodgkins disease; non-
completion o mitosis Hodgkins lymphoma; acute leukemia;
breast, genitourinary, thyroid, lung, and
Up-regulating the IIIiso ormo tubulin stomach cancer; neuroblastoma and
(the tubulin subunit that taxanes bind other childhood and adult sarcomas
pre erentially)
Echinocandins Yondelis So t-tissue sarcomas; ovarian cancer
Drugs that blockcell-cycle progression
prior to mitosis antagonize the toxic Anthracenedione Mitoxantrone Acute myelogenous leukemia; breast
e ects o taxanes and prostate cancer
Ixabepilone maybe less susceptible to Bleomycin Testis and cervical cancer; Hodgkins
P-glycoproteinmediated multidrug disease; non-Hodgkins lymphoma
resistance when compared to taxanes, but
Mitomycin C Stomach, anal, and lung cancer
resistance might result romthe ollowing:
Mutation o the tubulin binding site Enzymes L-Asparaginase Acute lymphocytic leukemia
Up-regulation o iso orms o tubulin
Anumber o mechanisms o resistance to
campothecins have been suggested based MECHANISMS OF ACTION AND RESISTANCE OF CYTOTOXIC AGENTS WITH DIVERSE
on in vitro studies (their relevance to MECHANISMS OF ACTION (SEE TABLE 45-5)
clinical settings is still unclear) including MECHANISMS OF MECHANISMS OF
the ollowing: CLASSIFICATION DRUG ACTION RESISTANCE
Topotecan is a substrate or Adrenocortical Mitotane (o,p-DDD) Mechanism o
P-glycoprotein, but its ef uxis much Suppressants-Mitotane action has not been
lower than other MDRsubstrates. elucidated
Relatively selective
The MRPclass o transporters have cytotoxicity or
been associated with topotecan and adrenocortical cells,
irinotecan resistance. normal or neoplastic
(continues)
(Continued)
644
Cancer Chemotherapy and Cytotoxic Agents CHAPTER 4 5

MECHANISMS OF MECHANISMS OF MECHANISMS OF


CLASSIFICATION DRUG ACTION RESISTANCE
RESISTANCE TO CYTOTOXIC
Substitutes Urea Hydroxyurea (HU) Inhibits ribonucleoside Tumor cells become NATURAL PRODUCTS (Cont.)
Hydroxyurea (HU) diphosphate resistant to HU
reductase (RNR), through increased Irinotecan resistance mayresult rom
which catalyzes the synthesis o the lacko carboxylesterase that converts the
reductive conversion hRRM2 subunit agent toits active metabolite, SN-38.
o ribonucleotides to o ribonucleoside
deoxyribonucleotides diphosphate
Decreased expression or mutation o
(a rate-limiting step reductase, thus topoisomerase I.
in the biosynthesis o restoring enzyme Enzyme phosphorylation or polyADP
DNA), causing cells to activity ribosylation mayreduce the activity
arrest at or near the o topoisomerase Iand its susceptibility
G1S inter ace through
both p53-dependent
to inhibition.
and -independent Cells exposed to camptothecins
mechanisms (see up-regulate topoisomerase II,
Figures 45-1 and 45-3) an alternative enzyme or DNA
Di erentiating Tretinoin The pharmacology o Resistance arises by strand passage.
AgentsRetinoids (all-transretinoic acid, retinoids is described urther mutation Cellular repair processes maynot readily
ATRA) in Chapter 48 o the usion gene recognize the stabilized drug-DNA-
ATRA induces that abolishes topoisomerase Icomplexes.
di erentiation in ATRA binding;
acute promyelocytic by induction o Resistance to anthracyclines in tumor cells
leukemia (APL) cells by CYP26A1 in liver or has been attributed to the ollowing:
binding the oncogenic leukemic cells; or by Overexpression o transcription-linked
PMLRAR- usion loss o expression
DNArepair
gene (see Figure 45-1) o the PMLRAR-
usion gene Multidrug resistance associated with
expression o MRP transporters and by
Di erentiating Arsenic trioxide (ATO) Mechanisms o ATOs cytotoxic the breast cancer resistance protein
AgentsArsenic antitumor action e ects are
Trioxide and di erentiation antagonized by Increased glutathione peroxidase activity
induction are cell survival signals Decreased activityor mutation o
uncertain but may act emanating rom topoisomerase II
by increasing reactive activation o the PI3
oxygen species kinase cell survival Enhanced abilityto repair DNA
(ROS) by inhibiting pathway (Akt kinase, strand breaks
thioredoxin reductase S6 kinase, and Tumor cells resistant to etoposide and
and inactivating GSH mammalian target o
(see Figure 45-1)
teniposide demonstrate the ollowing:
rapamycin [mTOR])
Ampli cation o the mdr-1 gene that
Di erentiating Vorinostat Antitumor action is encodes the P-glycoprotein drug ef ux
AgentsHDAC (suberoylanilide unclear, but inhibits
transporter
Inhibitors hydroxamic acid) histone deacetylases
Romidepsin (HDACs), increasing Mutation or decreased expression o
(depsipeptide, FK228) histone acetylation topoisomerase II
which enhances gene Mutations o the p53 tumor suppressor
transcription o cell-
cycle regulators, nuclear gene, a required component o the
transcription actors, apoptoticpathway
and pro-apoptotic Several mechanisms o bleomycin resistance
genes (see Figure 45-1) have been described in resistant tumor cells:
High levels o hydrolase activity
CASE 45 6 Apolymorphismo the hydrolase gene,
T e standard curative regimen or Hodgkins ymphoma is known as the ABVD SNPA1450G, that maybe associated
regimen (doxorubicin [ADRIAMYCIN], b eomycin, vinb astine, and dacarbazine) with increased hydrolase activity
Decreased uptake
a. What is the mechanism o action o each agent in the ABVD regimen?
Repair o strand breaks
Doxorubicin (ADRIAMYCIN) is an anthracycline antibiotic derived rom
the ungus Streptomyces peucetius var. caesius. Cytotoxic agents o this class
Drug inactivation bythiols or thiol-rich
proteins
(Continued) (continues)

645
SECTION VIII Chemotherapy of Neoplastic Diseases

MECHANISMS OF TABLE 45-5 Miscellaneous Agents (some of these agents are covered in Chapter 46)
RESISTANCE TO CYTOTOXIC TYPE OF AGENT NONPROPRIETARY NAMES DISEASE
NATURAL PRODUCTS (Cont.)
Substituted urea Hydroxyurea Chronic myelogenous
Resistance to mitomycin has been ascribed leukemia; polycythemia vera;
to the ollowing: essential thrombocytosis
De cient activation Di erentiating agents Tretinoin, arsenic trioxide Acute promyelocytic leukemia
Intracellular inactivation o the
reduced quinone Histone deacetylase inhibitor Cutaneous T-cell lymphoma
(vorinostat)
P-glycoproteinmediated drug ef ux
Protein tyrosine kinase Imatinib Chronic myelogenous
inhibitors leukemia; GI stromal tumors
(GIST); hypereosinophilia
syndrome

Dasatinib, nilotinib Chronic myelogenous


leukemia

Ge tinib, erlotinib EGFR inhibitors: non-small cell


lung cancer

Sora enib Hepatocellular cancer, renal


cancer

Sunitinib GIST, renal cancer

Lapatinib Breast cancer

Proteasome inhibitor Bortezomib Multiple myeloma

Biological response modi ers Inter eron-al a, interleukin-2 Hairy cell leukemia; Kaposis
sarcoma; melanoma;
carcinoid; renal cell; non-
Hodgkins lymphoma;
mycosis ungoides; chronic
myelogenous leukemia

Immunomodulators Thalidomide Multiple myeloma

Lenalidomide Myelodysplasia (5q -


syndrome); multiple myeloma

mTOR Inhibitors Temsirolimus, everolimus Renal cancer

Monoclonal antibodies (see Tables 46-1 and 46-2)

(the anthracyclines and anthracenediones: doxorubicin; daunorubicin; idarubicin;


epirubicin; valrubicin; mitoxantrone) all have quinone and hydroquinone moieties
on adjacent rings that permit the gain and loss o electrons. T ese compounds
can intercalate with DNA, directly a ecting transcription and replication. A more
important action is the ability to orm a tripartite complex with topoisomerase II
and DNA. opoisomerase II is an A P-dependent enzyme that binds to DNA and
produces double-strand breaks at the 3-phosphate backbone, allowing strand pas-
sage and uncoiling o supercoiled DNA. Following strand passage, topoisomerase
II religates the DNA strands. T is enzymatic unction is essential or DNA replica-
tion and repair. Formation o the tripartite complex with anthracyclines or with
etoposide inhibits the religation o the broken DNA strands, leading to apoptosis.
Anthracyclines, by virtue o their quinone groups, also generate ree radicals in
solution, and in both normal and malignant tissues. Anthracyclines can orm semi-
quinone radical intermediates that can react with O2 to produce superoxide anion
(Continued)

646
Cancer Chemotherapy and Cytotoxic Agents CHAPTER 4 5

radicals. T ese can generate both hydrogen peroxide and hydroxyl radicals, which
attack DNA and oxidize DNA bases.
Bleomycin is a member o unique group o DNA-cleaving antibiotics which are
ermentation products o Streptomyces verticillus. Bleomycins cytotoxicity results
rom its ability to cause oxidative damage to the deoxyribose o thymidylate and
other nucleotides, leading to single- and double-stranded breaks in DNA. T e bleo-
mycin molecule is a glycopeptide that contains a metal-binding cage that is bound
to either Fe2+ or Cu 2+. In the presence o O2 and a reducing agent the metaldrug
complex becomes activated and unctions as a errous oxidase, trans erring elec-
trons rom Fe2+ to molecular oxygen to produce oxygen radicals. Bleomycin binds
to DNA, and the activated complex generates ree radicals that are responsible or
abstraction o a proton at the 3 position o the deoxyribose backbone o the DNA
chain, opening the deoxyribose ring and generating a strand break in DNA.
Vinblastine is a vinca alkaloid derived rom the Madagascar periwinkle plant,
Catharanthus roseus. Other vinca alkaloids with antitumor activity include vincris-
tine, vindesine, and vinorelbine. T e vincas bind speci cally to tubulin and block
its polymerization with tubulin into microtubules, thus preventing the ormation
o mitotic spindles. In the absence o an intact mitotic spindle, duplicated chromo-
somes cannot align along the division plate. T ey disperse throughout the cyto-
plasm (exploded mitosis) or may clump in unusual groupings, such as balls or stars.
Cells are blocked in mitosis and undergo changes characteristic o apoptosis.
Dacarbazine (D IC) is a triazene alkylating agent. It is metabolically activated
by hepatic CYPs through an N-demethylation reaction. In the target cell, spon-
taneous cleavage o the metabolite, methyl-triazeno-imidazole-carboxamide
(M IC), yields an alkylating moiety, a methyl diazonium ion. T is reactive species
incorporates methyl groups into DNA. For a detailed description o DNA alkylation
reactions, re er to Case 45-1a.
b. What is the rationale or using combinations o cytotoxic drugs such as the
ABVD regimen compared to single-agent therapies?
Most cytotoxic drugs are used in combination with other cytotoxic agents with
di erent mechanisms o action to enhance tumor cell killing and to avoid the
development o resistance. Drugs in combination can negate the e ects o a resis-
tance mechanism speci c or a single agent, and they may be synergistic because
o their biochemical interactions. Ideally, drug combinations should not overlap in
their major toxicities.

KEY CONCEPTS
Cytotoxic antineop astic agents are structura y and mechanistica y diverse
T e e cacy o many cytotoxic agents depends on intact ce cyc e checkpoint
regu ation and apoptotic mechanisms
Rapid y dividing ce s (inc uding norma ce s such as GI epithe ia ce s and
bone marrow ce s) are typica y more sensitive to the e ects o cytotoxic agents
Common acute toxicities o cytotoxic agents inc ude GI damage and
mye osuppression
Mye osuppression can ead to increased risk o opportunistic in ections and
b eeding disorders
DNA modi ying agents are typica y mutagenic and carcinogenic, resu ting
in increased risk o eukemias and other cancers months or years o owing
treatment
Cytotoxic agents can cause chronic damage to critica organs, inc uding the kid-
ney, iver, and heart
Most cytotoxic chemotherapeutic agents are etotoxic
(Continued)
647
SECTION VIII Chemotherapy of Neoplastic Diseases

Resistance to the e ectiveness o cytotoxic antineop astic agents can deve op


through a variety o mechanisms, inc uding changes in ce u ar in ux and
ef ux, a terations in drug activation and inactivation, and mutations or changes
in drug targets or ce u ar pathways needed or cytotoxicity
Antineop astic agents are o en used in combination or with radiation to mini-
mize the deve opment o resistance and enhance tumor ce ki ing
Individua patient tumor response and organ toxicity to cytotoxic chemother-
apy can be a ected by the patients genetics, as we as many other actors such
as rena and hepatic unction
It is imperative to recognize toxicities ear y, to a ter doses or discontinue
o ending medication to re ieve symptoms and reduce risk, and to provide
vigorous supportive care (p ate et trans usions, antibiotics, and hematopoietic
growth actors)

SUMMARY QUIZ

QUESTION 45-1 A nuc eoside ana og that is enzymatica y converted by deoxycytidine


kinase (dCK) to its active orm, the 5-monophosphate ribonuc eotide is
a eucovorin
b 5- uorouraci (5-FU)
c azacytidine
d gemcitabine
e cytarabine (Ara-C)

QUESTION 45-2 When administered prior to 5-FU, this agent enhances the activation
o and antitumor activity by increasing poo s o PRPP
a Leucovorin
b Oxa ip atin
c Methotrexate
d Cytarabine (Ara-C)
e F uxuridine

QUESTION 45-3 An enzyme derived rom Escherichia coli that deprives eukemia ce s
o a required amino acid is
a pentostatin
b l -asparaginase (l -ASP)
c deoxycytidine kinase (dCK)
d trabectedin
e asparagine synthase

QUESTION 45-4 Pac itaxe and the other taxanes have a centra ro e in the therapy
o ovarian, breast, ung, GI, genitourinary, and head and neck cancers T e cytotoxic
e ect o the taxanes in tumor ce s is the resu t o
a inhibition o microtubu e ormation
b inhibition o microtubu e disassemb y
c enhancement o microtubu e disassemb y
d inhibition o topoisomerase I
e inhibition o topoisomerase II

648
Cancer Chemotherapy and Cytotoxic Agents CHAPTER 4 5

QUESTION 45-5 T e camptothecins, topotecan and irinotecan, are potent, cytotoxic


neop astic agents T ey act by
a interca ating DNA
b a ky ating DNA
c stabi izing the DNA-topoisomerase I c eavab e comp ex
d inhibiting DNA c eavage by topoisomerase I
e inhibiting topoisomerase I binding to DNA

SUMMARY QUIZ ANSWER KEY

QUESTION 45-1 Answer is e. Cytarabine (1--D-arabino uranosy cytosine; Ara-C)


is the most important antimetabo ite used in the therapy o acute mye ocytic eukemia
(AML) Ara-C is an ana og o 2-deoxycytidine with the 2-hydroxy in a position that
hinders rotation o the pyrimidine base around the nuc eoside bond and inter eres
with base pairing T e drug enters ce s via a nuc eoside transporter and is converted to
its active orm, the 5-monophosphate ribonuc eotide (Ara-CMP), by dCK Po ymor-
phisms o dCK may in uence the rate o drug activation in individua patients
Ara-CMP reacts with appropriate deoxynuc eotide kinases to orm diphosphate
and triphosphates (Ara-CDP and Ara-C P) Ara-C P competes with the physio ogica
substrate deoxycytidine 5-triphosphate (dC P) or incorporation into DNA by DNA
po ymerases T e incorporated Ara-CMP residue is a potent inhibitor o DNA po y-
merase, both in rep ication and repair synthesis, and b ocks the urther e ongation o
the nascent DNA mo ecu e T e b ock in e ongation activates checkpoint kinases (A R
and chk-1), which initiate attempts to remove the o ending nuc eotide I DNA breaks
are not repaired, apoptosis ensues

QUESTION 45-2 Answer is c. A number o agents have been combined with 5-FU in
attempts to enhance cytotoxic activity through biochemica modu ation Methotrexate,
by inhibiting purine synthesis and increasing ce u ar poo s o PRPP (see Figure 45-6),
enhances the activation o 5-FU (see Figure 45-7), and increases antitumor activity o
5-FU when given prior to but not o owing 5-FU
Some ma ignant ce s appear to have insu cient concentrations o N 5,10 methy-
ene tetrahydro o ate, and thus cannot orm maxima eve s o the inhibited ternary
comp ex with S Addition o exogenous o ate in the orm o N5- ormy FH 4
( eucovorin) increases ormation o the comp ex and enhances responses to 5-FU
(see Figure 45-8)
T e combination o cisp atin and 5-FU has yie ded impressive responses in
tumors o the upper aerodigestive tract, but the mo ecu ar basis o their interaction is
not we understood Oxa ip atin, which downregu ates S expression, is common y
used with 5-FU
Perhaps the most c inica y important interaction is the enhancement o irradiation
by uoropyrimidines, the mechanistic basis or which is unc ear 5-FU with simu tane-
ous irradiation cures ana cancer and enhances oca tumor contro in head and neck,
cervica , recta , gastroesophagea , and pancreatic cancers

QUESTION 45-3 Answer is b. Most norma tissues are ab e to synthesize l -asparagine


in amounts su cient or protein synthesis, but ymphocytic eukemias ack adequate
amounts o asparagine synthetase, and derive the required amino acid rom p asma
l -ASP, by cata yzing the hydro ysis o circu ating asparagine to aspartic acid and
ammonia, deprives these ma ignant ce s o asparagine, eading to ce death T e
enzyme has become a standard agent or treating ALL l -ASP is used in combination

649
SECTION VIII Chemotherapy of Neoplastic Diseases

with other agents, inc uding methotrexate, doxorubicin, vincristine, and prednisone or
the treatment o ALL and or high-grade ymphomas Resistance arises through induc-
tion o asparagine synthetase in tumor ce s
l -ASP toxicities resu t rom its antigenicity as a oreign protein and its inhibition
o protein synthesis Hypersensitivity reactions, inc uding urticaria and u -b own
anaphy axis, occur in 5-20% o patients and may be ata T ese reactions usua y are
hera ded by the ear ier appearance o circu ating neutra izing antibody and acce erated
enzyme c earance rom p asma In these patients, pegaspargase (a preparation in which
the enzyme is conjugated to 5000-Da units o monomethoxy po yethy ene g yco ) is a
sa e and e ective a ternative T e so-ca ed si ent enzyme inactivation by antibodies
occurs in a higher percentage o patients than overt hypersensitivity and may be associ-
ated with a negative c inica outcome, especia y in high-risk ALL patients

QUESTION 45-4 Answer is b. Pac itaxe and other taxanes (nab-pac itaxe and
docetaxe ) bind speci ca y to the tubu in subunit o microtubu es and antagonize
the disassemb y o this key cytoske eta protein, with the resu t that bund es o micro-
tubu es and aberrant structures derived rom microtubu es appear in the mitotic phase
o the ce cyc e Arrest in mitosis o ows Drugs that b ock ce cyc e progression prior
to mitosis antagonize the toxic e ects o taxanes Resistance to taxanes is associated in
some cancer ce s with increased expression o the mdr-1 gene and its product, P-g y-
coprotein; other resistant ce s have tubu in mutations (these atter ce s may disp ay
heightened sensitivity to vinca a ka oids) T e taxanes pre erentia y bind to the II
tubu in subunit o microtubu es; there ore, ce s may become resistant by upregu ating
the III iso orm o tubu in
T e vinca a ka oids and co chicine derivatives di er rom the taxanes in that they
bind to a di erent site on tubu in site and inhibit microtubu e ormation rather than
inhibit disassemb y
Estramustine binds to tubu in and microtubu e-associated proteins, causing
microtubu e disassemb y and inhibition o mitosis
Ixabepi one and other members o the epothi ones c ass resemb e taxanes in that
they bind to tubu in and trigger microtubu e nuc eation at mu tip e sites away rom
the centrio e T is chaotic microtubu e stabi ization triggers ce cyc e arrest at the
G2-M inter ace and apoptosis Epothi ones bind to a site distinct rom that o taxanes
In vitro studies suggest that ixabepi one is ess susceptib e to P-g ycoprotein-mediated
mu tidrug resistance when compared to taxanes Mechanisms imp icated in epothi one
resistance inc ude mutation o the tubu in binding site and up-regu ation o iso orms
o tubu in

QUESTION 45-5 Answer is c. T e camptothecins bind to and stabi ize the norma y
transient DNA-topoisomerase I c eavab e comp ex
T e DNA topoisomerases are nuc ear enzymes that reduce torsiona stress in
supercoi ed DNA, a owing se ected regions o DNA to become su cient y untang ed
and re axed to permit rep ication, repair, and transcription wo c asses o topoisom-
erase (I and II) mediate DNA strand breakage and resea ing, and both have become
the target o cancer chemotherapies Camptothecin ana ogs inhibit the unction o
topoisomerase I, whi e a number o di erent chemica entities (eg, anthracyc ines,
epipodophy otoxins, acridines) inhibit topoisomerase II opoisomerase I binds cova-
ent y to doub e-stranded DNA through a reversib e transesteri cation reaction T is
reaction yie ds an intermediate comp ex in which the tyrosine o the enzyme is bound
to the 3-phosphate end o the DNA strand, creating a sing e-strand DNA break T is
c eavab e comp ex a ows or re axation o the DNA torsiona strain, either by passage
o the intact sing e-strand through the nick or by ree rotation o the DNA about the
nonc eaved strand Once the DNA torsiona strain has been re ieved, the topoisomer-
ase I resea s the c eavage and dissociates rom the new y re axed doub e he ix
(Continued)

650
Cancer Chemotherapy and Cytotoxic Agents CHAPTER 4 5

T e camptothecins do not a ect the initia DNA c eavage action o topoisomerase


I, but inhibit the DNA re igation step, eading to the accumu ation o sing e-stranded
breaks in DNA T ese esions are reversib e and are not by themse ves toxic to the
ce However, the co ision o a DNA rep ication ork with this c eaved strand o DNA
causes an irreversib e doub e-strand DNA break, u timate y eading to ce death
Camptothecins are there ore S phase-speci c drugs (see Figure 45-3 or a representa-
tion o the ce cyc e) because ongoing DNA synthesis is necessary or cytotoxicity T is
has important c inica imp ications S phase-speci c cytotoxic agents genera y require
pro onged exposures o tumor ce s to drug concentrations above a minimum thresh-
o d to optimize therapeutic e cacy In act, prec inica studies o ow-dose, protracted
administration o camptothecin ana ogs have ess toxicity, and equa or greater antitu-
mor activity, than shorter, more intense courses

SUMMARY: CYTOTOXIC AGENTS USED IN THE TREATMENT OF CANCER


TOXICITIES
CLASS AND UNIQUE; CLINICALLY
SUBCLASSES NAMES CLINICAL USES COMMON IMPORTANT
Nitrogen Mustards Mechlorethamine HCl See Table 45-1 and See Side Bar TOXICITIES OF Acrolein is a toxic metabolite o
Cyclophosphamide text o Chapter 61 o ALKYLATING DRUGS and cyclophosphamide (see Figure
I os amide Goodman &Gilmans The Table 45-3 45-4) and i os amide, and
Melphalan Pharmacological Basis of Major acute toxicities o causes hemorrhagic cystitis
Chlorambucil Therapeutics, 12th Edition mechlorethamine are which can be mitigated by
Bendamustine or uses o speci c agents nausea and vomiting, coadministration o mesna;
lacrimation, and vigorous IVhydration should be
myelosuppression given with high-dose therapy
to prevent bladder toxicity

Ethyleneimines and Altretamine Palliative treatment or See Side Bar TOXICITIES OF Peripheral and central
Methylmelamines (hexamethylendiamine) persistent or recurrent ALKYLATING DRUGS neurotoxicity (ataxia,
ovarian cancer ollowing Main toxicities are depression, con usion,
cisplatin-based myelosuppression and drowsiness, hallucinations,
combination therapy neurotoxicity dizziness, and vertigo)
Nausea and vomiting also Peripheral blood counts and
are common side e ects a neurological examination
and may be dose-limiting should be per ormed prior to
Renal dys unction may the initiation o each course
necessitate discontinuing o therapy
the drug Severe, li e-threatening
orthostatic hypotension may
develop in patients who
receive monoamine oxidase
(MAO) inhibitors concurrently
with altretamine

Thiotepa Primarily used or high- See Side Bar TOXICITIES OF May cause neurotoxic
dose chemotherapy ALKYLATING DRUGS symptoms, including coma and
regimens seizures in high-dose regimens

Alkyl Sul onates Busul an See Table 45-1 See Side Bar TOXICITIES OF Pulmonary brosis, GI mucosal
ALKYLATING DRUGS damage, and hepatic veno-
Major toxic e ects occlusive disease (VOD) are
are related to important toxicities with high-
myelosuppression dose regimens
Occasionally, patients VOD and hepatotoxicity is
experience nausea, increased by coadministration
vomiting, and diarrhea with drugs that inhibit CYPs
Long-term use leads Anticonvulsants must be used
to impotence, sterility, concomitantly to protect
amenorrhea, and etal against acute CNS toxicities,
mal ormation including tonic-clonic seizures,
which may occur several hours
a ter each dose
(Continued)
651
SECTION VIII Chemotherapy of Neoplastic Diseases

TOXICITIES
CLASS AND UNIQUE; CLINICALLY
SUBCLASSES NAMES CLINICAL USES COMMON IMPORTANT
Nitrosoureas Carmustine (BCNU) See Table 45-1 and See Side Bar TOXICITIES OF With the exception o
Lomustine (CCNU) text o Chapter 61 o ALKYLATING DRUGS streptozocin, nitrosoureas
Semustine (methyl-CCNU) Goodman &Gilmans The With streptozocin, cause pro ound and delayed
Streptozocin (streptozotocin) Pharmacological Basis of nausea is requent and myelosuppression with
Therapeutics, 12th Edition mild, reversible renal or recovery 4-6 weeks a ter a
or uses o speci c agents hepatic toxicity occurs in single dose
Carmustine, lomustine, approximately two-thirds Renal ailure with long-term
and semustine are highly o cases (renal toxicity treatment
lipophilic, readily cross the may be cumulative), and Highly carcinogenic and
blood-brain barrier and hematological toxicity mutagenic
are used to treat brain occurs in 20% o patients BCNU in high doses with bone
tumors marrow rescue produces
Streptozocin is used hepatic VOD, pulmonary
exclusively in the brosis, renal ailure, and
treatment o human secondary leukemia
pancreatic islet cell
carcinoma and
malignant carcinoid
tumors

Triazenes Dacarbazine (DTIC) See Table 45-1 and See Side Bar TOXICITIES OF Temozolomide is administered
Temozolomide text o Chapter 61 o ALKYLATING DRUGS cyclically, and hematological
Goodman &Gilmans The DTIC induces nausea monitoring is necessary to
Pharmacological Basis of and vomiting in >90% o guide dosing adjustments
Therapeutics, 12th Edition patients
or uses o speci c agents Myelosuppression is mild
Primary indication and readily reversible
or dacarbazine is within 1-2 weeks
in the combination A u-like syndrome
chemotherapy o consisting o chills, ever,
Hodgkins disease; malaise, and myalgias
modestly e ective against may occur
malignant melanoma and Hepatotoxicity,
adult sarcomas alopecia, acial ushing,
Temozolomide is neurotoxicity, and
the standard agent dermatological reactions
in combination with are less common
radiation therapy or
patients with malignant
glioma and astrocytoma

Methylhydrazines Procarbazine See Table 45-1 and See Side Bar TOXICITIES OF Augments sedative e ects;
text o Chapter 61 o ALKYLATING DRUGS concomitant use o CNS
Goodman &Gilmans The Most common toxic depressants should be
Pharmacological Basis of e ects include leukopenia avoided
Therapeutics, 12th Edition and thrombocytopenia A weak MAO inhibitor
or uses o speci c agents GI symptoms such as Has disul ram-like action,
Currently employed or mild nausea and vomiting EtOH ingestion should be
second-line therapy in occur in most patients; avoided
malignant brain tumors; diarrhea and rash are Highly carcinogenic,
is rarely used in current noted in 5-10% o cases mutagenic, and teratogenic,
practice and is associated with a
5-10% risk o acute leukemia
in combination with other
agents or radiation
Causes in ertility, particularly
in males
(Continued)

652
Cancer Chemotherapy and Cytotoxic Agents CHAPTER 4 5

TOXICITIES
CLASS AND UNIQUE; CLINICALLY
SUBCLASSES NAMES CLINICAL USES COMMON IMPORTANT
Platinum Coordination Cisplatin See Table 45-1 and See Side Bar TOXICITIES OF Cisplatin-induced
Complexes Carboplatin (CBDCA) text o Chapter 61 o ALKYLATING DRUGS nephrotoxicity; to prevent
Oxaliplatin Goodman &Gilmans The Marked nausea and renal toxicity with cisplatin,
Pharmacological Basis of vomiting occur in almost it is important to establish
Therapeutics, 12th Edition all patients and usually can a chloride diuresis by the
or uses o speci c agents be controlled with 5-HT3 in usion o 1-2 L o normal
Oxaliplatin exhibits antagonists, NK1-receptor saline prior to treatment
a range o antitumor antagonists, and high- Ami ostine is a thiophosphate
activity (colorectal and dose corticosteroids cytoprotective agent that
gastric cancer) that di ers Electrolyte disturbances, reduces renal toxicity
rom other platinum including hypo- Ototoxicity
agents, perhaps due magnesemia, Anaphylactic-like reactions
to its MMR- and HMG- hypocalcemia, Aluminum reacts with and
independent e ects hypokalemia, and inactivates cisplatin; the
hypophosphatemia drug should not come in
are common; routine contact with needles or other
monitoring o plasma Mg 2+ in usion equipment that
recommended contain aluminum during its
Ototoxicity preparation or administration
Anaphylactic-like reactions Dose-limiting toxicity
Carboplatin is relatively o carboplatin is
well tolerated, causing myelosuppression
less nausea, neurotoxicity, Dose-limiting toxicity o
ototoxicity, and oxaliplatin is a peripheral
nephrotoxicity than neuropathy
cisplatin Platinum agents cause
leukemia and pulmonary
brosis months to years a ter
administration

Folic Acid Analogs Methotrexate(amethopterin) See Table 45-3 and Primary toxicities o Myelosuppression can
(Anti olates) High-dose methotrexate text o Chapter 61 o anti olates a ect the bone increase risk or spontaneous
with leucovorin rescue Goodman &Gilmans The marrow and the intestinal hemorrhage or li e-
(HDM-L) Pharmacological Basis of epithelium and usually threatening in ection, and
Trimetrexate Therapeutics, 12th Edition resolve within 10-14 days may require prophylactic
Pemetrexed or uses o speci c agents Pemetrexed is also trans usion o platelets and
Pralatrexate Methotrexate is also used associated with a broad-spectrum antibiotics i
Raltitrexed to treat psoriasis and prominent erythematous ebrile
Lometrexol rheumatoid arthritis (see and pruritic rash in 40% Prolonged myelosuppression
Chapter 23) o patients which can may occur in patients
be diminished with with compromised renal
dexamethasone unction and requires
Additional toxicities dosage adjustment based on
include alopecia, creatinine clearance
dermatitis, an allergic
interstitial pneumonitis,
nephrotoxicity (a ter
high-dose therapy),
de ective oogenesis or
spermatogenesis, abortion,
and teratogenesis

(Continued)

653
SECTION VIII Chemotherapy of Neoplastic Diseases

TOXICITIES
CLASS AND UNIQUE; CLINICALLY
SUBCLASSES NAMES CLINICAL USES COMMON IMPORTANT
Pyrimidine Analogs 5-Fluorouracil (5-FU) See Table 45-3 and Anorexia and nausea, Mucosal ulcerations
Floxuridine (FUdR, text o Chapter 61 o ollowed by stomatitis throughout the GI tract may
uorodeoxyuridine) Goodman &Gilmans The and diarrhea constitute lead to ulminant diarrhea,
Capecitabine Pharmacological Basis of reliable warning signs that shock, and death, particularly
Therapeutics, 12th Edition a suf cient dose has been in patients who are DPD
or uses o speci c agents administered de cient
5-FU is rarely used as a Alopecia, nail changes, Myelosuppression (more o ten
single agent dermatitis, and increased with bolus regimens)
5-FU also is a potent pigmentation and atrophy Hand- oot syndrome
radiation sensitizer o the skin may occur (more o ten with in usional
5-FU is used topically or regimens)
premalignant keratoses Coronary vasospasm with
o the skin and multiple 5-FU in usion
super cial basal cell The signi cant risk o toxicity
carcinoma with uoropyrimidines
emphasizes the need or
very skill ul supervision by
physicians amiliar with the
action and possible hazards

Cytarabine (cytosine See Table 45-3 and Acute, severe leukopenia, A ter high-dose Ara-C (1-2
arabinoside, Ara-C) text o Chapter 61 o thrombocytopenia, and weeks) dyspnea, ever, and
Goodman &Gilmans The anemia with striking pulmonary in ltrates may
Pharmacological Basis of megaloblastic changes occur, which may be atal in
Therapeutics, 12th Edition GI disturbances, stomatitis, 10-20% o patients
or uses o speci c agents conjunctivitis, reversible Intrathecal Ara-C (especially
hepatic enzyme with systemic high-dose
elevations, noncardiogenic methotrexate) or high-dose
pulmonary edema, and systemic Ara-C may cause
dermatitis CNS toxicity including ataxia
and slurred speech, seizures,
delirium, myelopathy, or coma,
especially in patients >40 years
o age and/or patients with
poor renal unction

Azacytidine (5-azacytidine) See Table 45-3 and Myelosuppression and Severe nausea and vomiting
Decitabine text o Chapter 61 o mild GI symptoms when given intravenously in
(2-deoxy-5-azacytidine) Goodman &Gilmans The large doses
Pharmacological Basis of
Therapeutics, 12th Edition
or uses o speci c agents

Gemcitabine See text o Chapter 61 o Myelosuppression Longer-duration in usions lead


(2,2-di uorodeoxycytidine, Goodman &Gilmans The Flu-like syndrome, to greater myelosuppression
dFdC) Pharmacological Basis of asthenia, and and hepatic toxicity
Therapeutics, 12th Edition rarely a posterior Long-term regimens
or uses o speci c agents leukoencephalopathy may lead to progressive
syndrome hemolytic uremic syndrome,
Mild elevation in liver necessitating drug
transaminases may occur discontinuation
in 40% o patients and Gemcitabine is a very potent
are reversible radiosensitizer and should not
be used with radiotherapy

(Continued)

654
Cancer Chemotherapy and Cytotoxic Agents CHAPTER 4 5

TOXICITIES
CLASS AND UNIQUE; CLINICALLY
SUBCLASSES NAMES CLINICAL USES COMMON IMPORTANT
Purine Analogs 6-Mercaptopurine (6-MP) See Table 45-3 and Myelosuppression Myelosuppression may be
6-Thioguanine (6-TG) text o Chapter 61 o (develops more gradually severe and prolonged in
Goodman &Gilmans The than with olic acid patients with a polymorphism
Pharmacological Basis of antagonists) a ecting TPMT
Therapeutics, 12th Edition Anorexia, nausea, or Predisposes to opportunistic
or uses o speci c agents vomiting in ~25% o in ection such as reactivation
The combination o adults, but stomatitis and o hepatitis B, ungal in ection,
methotrexate and 6-MP diarrhea are rare and Pneumocystis pneumonia
appears to be synergistic Jaundice and hepatic and an increased incidence o
6-MP is also used or enzyme elevations occur squamous cell malignancies o
Crohns disease (see in up to one-third o adult the skin
Chapter 23) patients treated with 6-MP;
usually resolves upon
discontinuation o therapy

Fludarabine phosphate See Table 45-3 and Myelosuppression (World Depletion o CD4+ T cells
text o Chapter 61 o Health Organization grade Predisposes to opportunistic
Goodman &Gilmans The 3 or 4) in about hal o in ections
Pharmacological Basis of patients Tumor lysis syndrome
Therapeutics, 12th Edition Nausea and vomiting (relatively in requent)
or uses o speci c agents in a minor raction; Altered mental status, seizures,
Also used as an in requently chills and optic neuritis, and coma at
immunosuppressant ever, malaise, anorexia, higher doses and in older
in nonmyeloablative peripheral neuropathy, patients
allogeneic bone marrow and weakness Auto-immune events (acute
transplantation, where hemolytic anemia or pure
it suppresses the host red cell aplasia, prolonged
response and may cytopenias)
encourage alloreactive Myelodysplasia and acute
donor T cells leukemias may arise as late
complications

Cladribine See text o Chapter 61 o Major toxicity is Opportunistic in ections are


(2-chlorodeoxyadenosine, Goodman &Gilmans The myelosuppression common
2-CdA) Pharmacological Basis of Cumulative Tumor lysis syndrome
Therapeutics, 12th Edition thrombocytopenia with
or uses o speci c agents repeated doses
Other toxic e ects include
nausea, high ever,
headache, atigue, skin
rashes

Clo arabine (2-chloro-2- See Table 45-3 and Major toxicity is Clinical syndrome o
uoro-arabinosyladenine) text o Chapter 61 o myelosuppression hypotension, tachyphemia,
Goodman &Gilmans The Elevated hepatic enzymes pulmonary edema, organ
Pharmacological Basis of and increased bilirubin dys unction, ever, are all
Therapeutics, 12th Edition Nausea, vomiting, suggestive o capillary leak
or uses o speci c agents and diarrhea syndrome and cytokine
Hypokalemia and release; evidence o capillary
hypophosphatemia leak should lead to immediate
discontinuation o the drug

Nelarabine (6-methoxy- See Table 45-3 and Myelosuppression Frequent, serious neurological
arabinosyl-guanine) text o Chapter 61 o and liver unction test sequelae such as seizures,
Goodman &Gilmans The abnormalities delirium, somnolence,
Pharmacological Basis of peripheral neuropathy, or
Therapeutics, 12th Edition Guillain-Barr syndrome;
or uses o speci c agents neurological side e ects may
not be reversible
(Continued)

655
SECTION VIII Chemotherapy of Neoplastic Diseases

TOXICITIES
CLASS AND UNIQUE; CLINICALLY
SUBCLASSES NAMES CLINICAL USES COMMON IMPORTANT
Pentostatin See Table 45-3 and Myelosuppression, GI Immunosuppression may
(2-deoxyco ormycin) text o Chapter 61 o symptoms, skin rashes, and persist or several years a ter
Goodman &Gilmans The abnormal liver unction discontinuation
Pharmacological Basis of studies Major renal and neurological
Therapeutics, 12th Edition Neutropenic ever and complications are
or uses o speci c agents opportunistic in ections encountered with high doses
may occur Severe or even atal
pulmonary toxicity may
occur in combination with
udarabine phosphate

Microtubule- Vinblastine sul ate See Table 45-4 and Mild myelosuppression Syndrome o inappropriate
Damaging Agents Vinorelbine text o Chapter 61 o and mild neurotoxicity secretion o antidiuretic
Vinca Alkaloids Vincristine sul ate Goodman &Gilmans The GI disturbances including hormone has been reported
Pharmacological Basis of nausea, vomiting, anorexia, with vinblastine, less
Therapeutics, 12th Edition and diarrhea may occur commonly with vincristine
or uses o speci c agents Severe constipation Extravasation during injection
with vincristine may be may lead to cellulitis and
prevented by a prophylaxis phlebitis
with laxatives and bulk-
orming agents
Alopecia occurs in
~20% o patients given
vincristine, however,
it is always reversible,
requently without
cessation o therapy
Vinorelbine may cause
allergic reactions and mild,
reversible changes in liver
enzymes

Microtubule-Damaging Paclitaxel See Table 45-4 and Myelosuppression A ter treatment with paclitaxel,
AgentsTaxanes Nab-paclitaxel text o Chapter 61 o Peripheral neuropathy myalgias or several days are
Docetaxel Goodman &Gilmans The with paclitaxel is dose- common
Pharmacological Basis of limiting; with high-dose Mucositis is common in 72-
Therapeutics, 12th Edition schedules or prolonged or 96-hour in usions and in
or uses o speci c agents use, a stocking-glove the weekly schedule with
sensory neuropathy can paclitaxel
be disabling Asymptomatic bradycardia
Hypersensitivity reactions and occasional silent
in patients receiving ventricular tachycardia can
paclitaxel in usions o short occur with paclitaxel
duration (1-6 h) are largely With multiple cycles o
averted by pretreatment docetaxel therapy, progressive
with dexamethasone, uid retention can occur
diphenhydramine, and leading to peripheral edema,
histamine H2 receptor pleural and peritoneal
antagonists uid, and pulmonary
Higher rates o peripheral edema in extreme cases;
neuropathy with nab- dexamethasone can prevent
paclitaxel compared this
to paclitaxel but rarely
hypersensitivity reaction
Docetaxel causes
greater neutropenia
than paclitaxel but less
peripheral neuropathy
and less requent
hypersensitivity
(Continued)

656
Cancer Chemotherapy and Cytotoxic Agents CHAPTER 4 5

TOXICITIES
CLASS AND UNIQUE; CLINICALLY
SUBCLASSES NAMES CLINICAL USES COMMON IMPORTANT
Microtubule- Estramustine See text o Chapter 61 o Myelosuppression Possesses estrogenic side
Damaging Goodman &Gilmans The e ects (gynecomastia,
AgentsEstramustine Pharmacological Basis of impotence, elevated risk
Therapeutics, 12th Edition o thrombosis, and uid
or uses o speci c agents retention), hypercalcemia,
acute attacks o porphyria,
impaired glucose tolerance,
and hypersensitivity reactions,
including angioedema

Microtubule- Ixabepilone See text o Chapter 61 o Toxicities similar to those


Damaging Goodman &Gilmans The o the taxanes, namely
AgentsEpothilones Pharmacological Basis of neutropenia, peripheral
Therapeutics, 12th Edition sensory neuropathy,
or uses o speci c agents atigue, diarrhea, and
asthenia

Topoisomerase Topotecan See Table 45-4 and Dose-limiting toxicity o Severe neutropenia and ebrile
inhibitors Irinotecan (CPT-11) text o Chapter 61 o topotecan with all dosing neutropenia (which may
Camptothecin Goodman &Gilmans The schedules is neutropenia, be atal with concomitant
Analogs Pharmacological Basis of with or without diarrhea)
Therapeutics, 12th Edition thrombocytopenia, and Inhibition o
or uses o speci c agents GI side e ects such as acetylcholinesterase activity
mucositis and diarrhea in by irinotecan may occur
some patients within the rst 24 h
Less common but mild causing a cholinergic
topotecan-related syndrome (acute diarrhea,
toxicities include nausea diaphoresis, hypersalivation,
and vomiting, elevated abdominal cramps, visual
liver transaminases, ever, accommodation disturbances,
atigue, and rash lacrimation, rhinorrhea,
Dose-limiting toxicity asymptomatic bradycardia)
with irinotecan is delayed which can be treated
diarrhea which can be with atropine
reduced by treatment
with loperamide (see
Chapter 33)
Common but mild toxicities
with irinotecan include
nausea and vomiting,
atigue, vasodilation or
skin ushing, mucositis,
elevation in liver
transaminases, and alopecia

Topoisomerase Etoposide (VP-16-213) See Table 45-4 and Dose-limiting toxicity o


inhibitors Teniposide (VM-26) text o Chapter 61 o etoposide is leukopenia;
Epipodophyllotoxins Goodman &Gilmans The thrombocytopenia occurs
Pharmacological Basis of less o ten and usually is
Therapeutics, 12th Edition not severe
or uses o speci c agents Nausea, vomiting,
stomatitis, and diarrhea
occur with etoposide in
~15% o patients
Alopecia is common but
reversible
Myelosuppression,
nausea, and vomiting are
teniposides primary toxic
e ects
(Continued)

657
SECTION VIII Chemotherapy of Neoplastic Diseases

TOXICITIES
CLASS AND UNIQUE; CLINICALLY
SUBCLASSES NAMES CLINICAL USES COMMON IMPORTANT
Anticancer Dactinomycin See Table 45-4 and Anorexia, nausea, and Severe injury may occur
Antibiotics (actinomycin D) text o Chapter 61 o vomiting as a result o local drug
Actinomycins Goodman &Gilmans The Hematopoietic suppression extravasation
Pharmacological Basis of with pancytopenia
Therapeutics, 12th Edition Proctitis, diarrhea, glossitis,
or uses o speci c agents cheilitis, and ulcerations
o the oral mucosa are
common
Dermatological
mani estations (alopecia,
erythema, desquamation,
and increased in ammation
and pigmentation) in areas
previously or concomitantly
subjected to X-ray radiation

Anticancer Doxorubicin (ADRIAMYCIN) See Table 45-4 and Myelosuppression, Cardiac toxicity (acute and
Antibiotics Daunorubicin (daunomycin, text o Chapter 61 o stomatitis, alopecia, GI chronic) characterized by
Anthracyclines and rubidomycin) Goodman &Gilmans The disturbances, and rash are tachycardia, arrhythmias,
Anthracenediones Idarubicin Pharmacological Basis of common toxicities dyspnea, hypotension,
Epirubicin Therapeutics, 12th Edition Mitoxantrone causes less pericardial e usion, and
Valrubicin or uses o speci c agents cardiac toxicity, nausea, congestive heart ailure poorly
Mitoxantrone vomiting, and alopecia responsive to digitalis; chronic
than does doxorubicin cardiomyopathy is cumulative
Erythematous streaking dose-related and can occur
near the site o in usion years a ter treatment;
(ADRIAMYCIN are) is concomitant administration o
a benign local allergic dexrazoxane may avert later
reaction and should cardiotoxicity
not be con used with
extravasation

Anticancer Bleomycin See Table 26-4 and Skin toxicities, including Pulmonary toxicity (1-5% o
Antibiotics text o Chapter 61 o hyperpigmentation, patients), which begins with
Bleomycin Goodman &Gilmans The hyperkeratosis, erythema, a dry cough, ne rales, and
Pharmacological Basis of ulceration, and rarely di use basilar in ltrates on
Therapeutics, 12th Edition Raynauds phenomenon X-ray and may progress to li e-
or uses o speci c agents Other toxicities include threatening pulmonary brosis
hyperthermia, headache, (~1% die); risk o pulmonary
nausea and vomiting, and toxicity is related to total dose
a peculiar acute ulminant
reaction in ~1% patients
with lymphomas or
testicular cancer

Anticancer Mitomycin (mitomycin-C) See Table 45-4 and Myelosuppression, A hemolytic uremic syndrome
Antibiotics text o Chapter 61 o characterized by (hemolysis, neurological
Mitomycin Goodman &Gilmans The marked leukopenia and abnormalities, interstitial
Pharmacological Basis of thrombocytopenia pneumonia, and glomerular
Therapeutics, 12th Edition damage resulting in renal
or uses o speci c agents ailure) believed to result rom
drug-induced endothelial
damage; toxicity increases
with dose (28% renal ailure
in total high dose); early
recognition and immediate
discontinuation o drug is
needed to minimize toxicity
Interstitial pulmonary brosis
Heart ailure, especially in
combination with doxorubicin
(Continued)
658
Cancer Chemotherapy and Cytotoxic Agents CHAPTER 4 5

TOXICITIES
CLASS AND UNIQUE; CLINICALLY
SUBCLASSES NAMES CLINICAL USES COMMON IMPORTANT
Trabectedin Trabectedin Approved outside Mild myelosuppression Signi cant hepatic enzyme
the United States or elevations and atigue in one-
second-line treatment third o patients, but with
o so t tissue sarcomas dexamethasone pretreatment
and ovarian cancer transaminase increases are
in combination with less pronounced and rapidly
doxorubicin reversed
Rhabdomyolysis (rarely)

Enzymes L-asparaginase (L-ASP See Table 45-4 and Toxicities resulting rom Hypersensitivity reactions to
Pegaspargase text o Chapter 61 o inhibition o protein L-ASP, including urticaria and
(PEG-L-asparaginase) Goodman &Gilmans The synthesis in normal tissues ull-blown anaphylaxis, occur
Pharmacological Basis of (eg, hyperglycemia due to in 5-20% o patients and may
Therapeutics, 12th Edition insulin de ciency, clotting be atal; pegaspargase is a
or uses o speci c agents abnormalities due to sa e and e ective alternative
de cient clotting actors, in patients who show
hypertriglyceridemia hypersensitivity to L-ASP
due to e ects on Silentenzyme inactivation by
lipoprotein production, antibodies occurs in a higher
hypoalbuminemia) percentage o patients than
overt hypersensitivity and may
be associated with a negative
clinical outcome, especially in
high-risk ALL patients
Pancreatitis
Dysregulated coagulation
(thrombosis and hemorrhage)

Mitotane Mitotane (o,p-DDD) Adrenal cortex cancer Anorexia and nausea in Because o drug-induced
most patients, somnolence adrenal cortex damage,
and lethargy in ~34%, and adrenocorticosteroid
dermatitis in 15-20% replacement therapy is
necessary

Hydroxyurea (HU) Hydroxyurea (HU) See Table 45-5 and Myelosuppression, which Potent teratogen; should
text o Chapter 61 o is rapidly reversible with not be used in women with
Goodman &Gilmans The drug discontinuation childbearing potential
Pharmacological Basis of Desquamative
Therapeutics, 12th Edition interstitial pneumonitis,
or uses o speci c agents GI disturbances, and
dermatological reactions
(including increased
pigmentation and pain ul
leg ulcers)

Di erentiating Tretinoin (all-trans retinoic See Table 45-5 and Dry skin, cheilitis, Di erentiation syndrome
AgentsRetinoids acid, ATRA) text o Chapter 61 o reversible hepatic enzyme or retinoic acid syndrome
Goodman &Gilmans The abnormalities, bone caused by an outpouring
Pharmacological Basis of tenderness, pseudotumor o cytokines and mature-
Therapeutics, 12th Edition cerebri, hypercalcemia, appearing neutrophils o
or uses o speci c agents and hyperlipidemia leukemic origin results in
a syndrome o respiratory
distress, ever, pulmonary
in ltrates pleural and
pericardial e usions, and
mental status changes that
a ect 15-20% o patients and
may have a atal outcome;
mitigated by dexamethasone
pretreatment
(Continued)

659
SECTION VIII Chemotherapy of Neoplastic Diseases

TOXICITIES
CLASS AND UNIQUE; CLINICALLY
SUBCLASSES NAMES CLINICAL USES COMMON IMPORTANT
Di erentiating Arsenic trioxide (ATO) See Table 45-5 and Reversible side e ects Di erentiation syndrome
AgentsArsenic text o Chapter 61 o include hyperglycemia, similar to ATRA in 10% o
Trioxide Goodman &Gilmans The hepatic enzyme elevations, patients which is reversed by
Pharmacological Basis of atigue, dysesthesias, and oxygen, corticosteroids, and
Therapeutics, 12th Edition light-headedness temporary discontinuation
or uses o speci c agents o ATO
Prolongation o QTc interval in
40% o patients (torsades de
pointes is rare); monitor serum
electrolytes, treat hypokalemia,
and avoid using ATO with
other QT-prolonging drugs
(see Chapter 18)

Di erentiating Vorinostat (suberoylanilide See Table 45-5 and Fatigue, nausea, diarrhea, Deep venous thrombosis and
AgentsHDAC hydroxamic acid, SAHA) text o Chapter 61 o and thrombocytopenia pulmonary embolism with
Inhibitors Romidepsin (depsipeptide) Goodman &Gilmans The vorinostat
Pharmacological Basis of Prolongation o QTc interval
Therapeutics, 12th Edition with vorinostat, requiring
or uses o speci c agents caution when used in patients
with underlying cardiac
disease, care ul monitoring
o the QTc interval, and
correction o electrolyte
(K+, Mg ++) abnormalities, and
avoidance o use with other
QT-prolonging drugs (see
Chapter 18)

660
CHAPTER

Targeted Anticancer Therapies 46


T is chapter will be most use ul a er having a basic understanding o the material in
DRUGS INCLUDED IN THIS
Chapter 62 argeted T erapies: yrosine Kinase Inhibitors, Monoclonal Antibodies,
and Cytokines, and Chapter 63 Natural Products in Cancer Chemotherapy: Hormones, CHAPTER
in Goodman & Gilmans T e Pharmacological Basis of T erapeutics, 12th Edition. In 131I-Tositumomab (BEXXAR)
addition to the material presented here, the 12th Edition contains: 90Y-Ibritumomab tiuxetan (ZEVALIN)
A discussion o the use o targeted anticancer therapies in combination with cytotoxic Abarelix(PLENAXIS; withdrawn
agents in Chapter 60 General Principles o Cancer Chemotherapy rommarket)
T e molecular structures o small molecule drugs used in targeted anticancer Abiraterone (ZYTIGA)
therapies
Aldesleukin (IL-2; PROLEUKIN)
LEARNING OBJECTIVES Alemtuzumab (CAMPATH)
Understand the mechanisms o action and clinical uses o small molecule Anastrozole (ARIMIDEX)
inhibitors o protein tyrosine kinases. Bevacizumab (AVASTIN)
Understand the mechanisms o action and clinical uses o monoclonal antibodies Bicalutamide (CASODEX,others)
that target growth actor receptors and other tumor cell antigens. Bortezomib (VELCADE)
Understand the mechanisms o action and clinical uses o monoclonal antibodies Buserelin (SUPREFACT)not available in
that target the VEGF pathway. the United States
Understand the mechanism o action and clinical uses o anticancer drugs that Cetrorelix(CETROTIDE)
target the m OR pathway, and proteosome-mediated protein degradation. Cetuximab (ERBITUX)
Understand the mechanism o action and clinical uses o anticancer drugs that Cyproteronenot available in the
target the IL-2 receptor. United States
Understand the clinical uses o the immunomodulatory analogs (IMiDs) in Dasatinib (BMS-354825; SPRYCEL)
treating multiple myeloma and myelodysplastic syndrome. Degarelix(FIRMAGON)
Understand the mechanism o action and use o hormone therapy to treat Denileukin di titox(ONTAK)
hormone-dependent tumors.
Deslorelinnot available inthe UnitedStates
Know the common and important toxicities o targeted anticancer Dexamethasone
pharmacotherapies.
Erlotinib (TARCEVA)
Know mechanisms o acquired resistance to speci c targeted anticancer
Everolimus (AFINITOR)
therapies.
Exemestane (AROMASIN)
Flutamide (EULEXIN)
MECHANISMS OF ACTION AND RESISTANCE OF TARGETED ANTICANCER DRUGS
Fulvestrant (FASLODEX)
MECHANISM OF MECHANISM OF
DRUG CLASS DRUG ACTION RESISTANCE Ganirelixnot available inthe UnitedStates
Protein Tyrosine Imatinib mesylate Inhibit disease-causing Point mutations in Geftinib (IRESSA)
Kinase Inhibitors Dasatinib tyrosine kinase activity, kinases that lower Gemtuzumab ozogamicin (MYLOTARG)
ATP Inhibitors Nilotinib usually by blocking drug a nity and alter withdrawn romthe USmarket in 2010
Ge tinib ATP binding to the drug e ects on kinase
Erlotinib catalytic site activity GnRH(GONADORELIN)
Lapatinib VEGFR inhibitors Ampli cation o kinase Goserelin (ZOLADEX)
Sunitinib (sunitinib and gene
Sora enib sora enib) block
Histrelin (VANTAS)
Drug e ux, activation
angiogenesis o downstream Imatinibmesylate (STI571; GLEEVEC,GLIVEC)
pathways, and altered Ketoconazole (NIZORAL)
kinase tra cking are
also possible Lapatinib (TYKERB)
Lenalidomide (REVLIMID)
(Continued) (continues)

661
SECTION VIII Chemotherapy of Neoplastic Diseases

DRUGS INCLUDED IN THIS MECHANISM OF MECHANISM OF


DRUG CLASS DRUG ACTION RESISTANCE
CHAPTER (Cont.)
Monoclonal Cetuximab Binds the extracellular
Letrozole (FEMARA) AntibodiesGrowth Panitumumab domain o the growth
Leuprolide (LUPRON, ELIGARD) Factor Receptors Trastuzumab actor which prevents
(EGFR, HER2/neu, receptor activation and
Medroxyprogesterone (DEPO-PROVERA,
VEGFRs, PDGFR) downstream signaling,
others) Antibodies downregulates receptor
Megestrol (MEGACE) expression, and induces
antibody-dependent
Megestrol acetate (MEGACE, others)
cellular cytotoxicity
Na arelin (SYNAREL) (ADCC) (see Table 46-1)
Nilotinib (AMN107; TASIGNA)
Monoclonal Bevacizumab Binds the ligand(s) o
Nilutamide (NILANDRON) AntibodiesGrowth Ranibizumab growth actor receptors,
Oatumumab (ARZERRA) Factor (VEGF) blocking interaction
Antibodies o the ligand with its
Panitumumab (VECTIBIX) cognate receptor (see
Pazopanib (VOTRIENT) Table 46-1)
Prednisone Monoclonal Rituximab Binds CD20 Downregulation o
Raloxi ene (EVISTA) AntibodiesCD20 131
I-Tositumomab antigen on B-cell CD20
Antibodies 90
Y-Ibritumomab lymphoma and Genetic
Ranibizumab (LUCENTIS) tiuxetan chronic lymphocytic polymorphisms o
Rapamycin (sirolimus; RAPAMUNE) O atumumab leukemia (CLL) cells target antigen
which inhibits CD20- Impaired antibody-
Rituximab (RITUXAN) mediated proli eration dependent cellular
Sora enib (NEXAVAR) and di erentiation cytotoxicity
(see Table 46-1) Decreased
Sunitinib (SUTENT)
The radionuclide- complement activation
Tamoxi en (NOLVADEX,others) conjugated antibodies Limited e ects
Temsirolimus (TORISEL) target delivery o on signaling and
radiation to CD20- induction o apoptosis
Thalidomide (THALOMID) containing tumor cells
Toremi ene (FARESTON) (see Table 46-1)
O atumumab causes
Trastuzumab (HERCEPTIN) B-cell lysis (both ADCC
Triptorelin (TRELSTAR, TRELSTARDEPOT) and complement-
dependent
cytotoxicity [CDC])

Monoclonal Alemtuzumab Binds CD52 antigen Genetic


AntibodiesCD52 on B-cell CLL and polymorphisms o
Antibodies T-cell lymphoma, target antigen
causing tumor cell
death through
ADCC and CDC (see
Table 46-1)

Monoclonal Gemtuzumab Binds CD33 on acute Genetic


AntibodiesCD33 ozogamicin myelocytic leukemia polymorphisms o
Antibodies (AML) cells and other target antigen
hemopoietic cells;
ollowing binding to
CD33 and endocytosis,
calicheamicin (the
toxin) is cleaved
rom gemtuzumab
ozogamicin, and
the toxin enters the
nucleus where it
causes double-strand
DNA breaks and cell
death (see Table 46-1)

(Continued)
662
Targeted Anticancer Therapies CHAPTER 4 6

MECHANISM OF MECHANISM OF
DRUG CLASS DRUG ACTION RESISTANCE
Thalidomide and Thalidomide Multiple
Its Derivatives Lenalidomide immunomodulatory
(Immunomodulatory mechanisms (see
analogs, IMiDs) Figure 46-1)

Proteasome Bortezomib Binds the 26S


Inhibitors proteasome and
Inhibits proteasome-
mediated protein
degradation,
particularly IB,
thus preventing
the transcriptional
activity o NF-B
and downregulating
survival responses
Proteasome inhibition
also disrupts
degradation o
other key cell cycle
regulators leading
cells to apoptosis

mTOR Inhibitors Rapamycin Inhibits mTORC1 Resistance is


Temsirolimus in the PI3 kinase incompletely
Everolimus pathway (Figure 46-2) understood but may
arise through the
action o a second
mTOR complex,
mTORC2, which
is una ected by
rapamycins and which
regulates Akt kinase
(see Figure 46-2)

Interleukin-2 (IL-2) Aldesleukin Binds the IL-2 receptor


Receptor Agonists Denileukin di titox on immune e ector
cells which appears
to enhance tumor cell
killing by activated
T cells, NKcells, and
monocytes
Denileukin di titox
is a recombinant
immunotoxin
consisting o IL-2 and
the active ragment o
diphtheria toxin which
binds the high-a nity
IL-2R receptor and
introduces the toxin
into cells causing cell
death by inhibiting
protein synthesis (see
Table 46-1)

Colony-Stimulating See Chapter 25 See Chapter 25


Factors
(Continued)

663
SECTION VIII Chemotherapy of Neoplastic Diseases

MECHANISM OF MECHANISM OF
DRUG CLASS DRUG ACTION RESISTANCE
Glucocorticoids Prednisone Binds to speci c
Dexamethasone physiological
receptors that
translocate to the
nucleus and induce
antiproli erative and
apoptotic responses
in sensitive cells (see
Chapter 29)

Progestins Medroxyprogesterone See Chapter 28


Megestrol acetate

Estrogens and See Chapter 28 See Chapter 28


Androgens

Antiestrogens Tamoxi en Bind to the estrogen Polymorphisms in


Selective Estrogen- Toremi ene receptor (ER) and CYP2D6 that reduce its
Receptor Modulators Raloxi ene exert either estrogenic activity (see Figure 46-3)
(SERMs) or antiestrogenic Cross-talk between
e ects, depending on the ER and HER2/neu
the speci c organ (see pathway
Chapter 28) Altered interactions
between PAX2 and the
ERco-activator AIB-1/
SRC-3

Antiestrogens Fulvestrant Similar to SERMs


Selective but devoid o any
Estrogen-Receptor estrogen agonist
Downregulators e ects
(SERDs) Estradiol antagonist
Reduce the number
o ER molecules in
target cells

Anti-estrogens Anastrozole Block aromatase


Aromatase (CYP19) Letrozole conversion o
Inhibitors Exemestane androgens to
estrogens (see
Figure 46-4)
resulting in estrogen
deprivation

Gonadotropin- GnRH Bind to GnRH


Releasing Hormone Leuprolide receptors on pituitary
(GnRH) Agonists Buserelin gonadotropin-
Na arelin producing cells
Deslorelin leading to GnRH
Histrelin receptor down-
Triptorelin regulation,
Goserelin causing inhibition
o testosterone
production in
Leydig cells
(Continued)

664
Targeted Anticancer Therapies CHAPTER 4 6

MECHANISM OF MECHANISM OF
DRUG CLASS DRUG ACTION RESISTANCE
Gonadotropin- Cetrorelix Antagonize GnRH
Releasing Hormone Ganirelix receptors on pituitary
(GnRH) Antagonists Abarelix gonadotropin-
Degarelix producing cells,
causing inhibition
o testosterone
production in Leydig
cells without causing
the initial testosterone
f are caused by GnRH
agonists

Antiandrogens Cyproterone Bind to androgen


Steroidal Megestrol receptors (ARs)
inhibiting binding
o testosterone and
dihydrotestosterone

Antiandrogens Flutamide Inhibit binding o


Nonsteroidal Bicalutamide testosterone and
Nilutamide dihydrotestosterone
and consequent AR
translocation rom
the cytoplasm to the
nucleus

Estrogens See Chapter 28 High estrogen levels


reduce testosterone
via negative eedback
on the hypothalamic
pituitary axis
May exert a cytotoxic
e ect on prostate
cancer cells by
competing with
androgens or steroid
hormone receptors

Inhibitors o Ketoconazole Ketoconazole inhibits


Steroidogenesis Abiraterone both testicular and
adrenal
steroidogenesis
by blocking CYPs,
primarily CYP17
(17-hydroxylase)
Abiraterone is an
irreversible inhibitor o
both 17-hydroxylase
and C-17,20-lyase
CYP17 activity
(See Figure 46-4)

CASE 46 1
A 34-year-old man is diagnosed with Philadelphia chromosome-positive (Ph +) chronic
myelogenous leukemia (CML).
a. What is the molecular mechanism that causes this orm o cancer?
A single molecular event, in this case the 9:22 chromosomal translocation which
results in the Philadelphia chromosome (Ph +), leads to expression o the Abelson
protooncogene kinase ABL used to BCR (breakpoint cluster region), yielding a
(Continued)
665
SECTION VIII Chemotherapy of Neoplastic Diseases

T c e ll IGF1R
ce ll Othe r TK re ce ptors
me mbra ne
NK c e ll
pro duc tio n IL-2
and INF-
+ D
func tio nality P I3K Ra s
NK Ce ll s tre s s
e ne rgy de ple tion
Anti-apo pto s is a mino a cid s ta tus
c e ll g rowth A
Akt mTORC2
Tumo r A1/Bfl-1
c e ll NF-B
IL-6 Ra pa mycins
FKBP 12
MAP K
Adhe s io n mTORC1
Ang io g e ne s is

IL-6
IL-6 S DF-1 prolife ra tion
IGFL bFGF S 6K1 4EBP
VEGF TNF

B
Capillary tra ns la tion tra ns la tion s urviva l
e ndo the lium a utopha gy me ta bolis m prolife ra tion
Bo ne marrow s tro ma
C me ta bolis m

FIGURE 46-1 Schematic overview o proposed mechanisms o antimy- FIGURE 46-2 Insulin-like growth actor 1 receptor (IGF-1R)
eloma activity o thalidomide and its derivatives. Some biological hallmarks and other tyrosine kinase (TK) growth actor receptors sig-
o the malignant phenotype are indicated in the boxes. The proposed sites nal through multiple pathways. A key pathway is regulated
o action or thalidomide (letters inside circles) are hypothesized to also by phosphatidylinositol-3 kinase (PI3K) and its downstream
be operative or thalidomide derivatives. A. Direct antimultiple myeloma partner, the mammalian target o rapamycin (mTOR). Rapamy-
(MM) e ect on tumor cells, including G1 growth arrest and/or apoptosis, cins complex with FKBPP12 to inhibit the mTORC1 complex.
even against MM cells resistant to conventional therapy. This is due to the mTORC2 remains una ected and responds by upregulating
disruption o the antiapoptotic e ect o BCL-2 amily members, blocking Akt, driving signals through the inhibited mTORC1. The various
NF-B signaling, and inhibition o the production o interleukin-6 (IL-6). B. downstream outputs o the 2 complexes are shown. Phosphor-
Inhibition o MM-cell adhesion to bone marrow stromal cells partially due ylation o 4EBP by mTOR inhibits the capacity o 4EBP to inhibit
to the reduction o IL-6 release. C. Decreased angiogenesis due to the inhi- ei -4E and slow metabolism. 4EBP, eukaryotic initiation actor
bition o cytokine and growth actor production and release. D. Enhanced 4e (ei -4E) binding protein; FKBP12, the immunophilin target
T-cell production o cytokines, such as IL-2 and inter eron- (IFN-), that (binding protein) or tacrolimus (FK506); S6K1, S6 kinase 1.
increase the number and cytotoxic unctionality o natural killer (NK) cells.
VEGF, vascular endothelial growth actor.

constitutively activated protein tyrosine kinase, BCR-ABL, and then the malignant
phenotype. T is is the most common mechanism causing CML.
b. What is the rst-line treatment or CML and what is the rationale or this
pharmacotherapy?
Imatinib and the related compounds dasatinib and nilotinib induce clinical and
molecular remissions in more than 90% o CML patients in the chronic phase o
disease. T ese agents target the BCR-ABL tyrosine kinase and inhibit its activity.
c. What are the mechanisms o resistance to imatinib pharmacotherapy o CML?
Resistance to the tyrosine kinase inhibitors arises rom point mutations in 3 sepa-
rate segments o the BCR-ABL kinase domain. T e contact points between imatinib
and the enzyme become sites o mutations in drug-resistant leukemic cells; these
mutations prevent tight binding o the drug and lock the enzyme in its open con g-
uration, in which it has access to substrate. Most such mutations hold the enzyme
in its open and enzymatically active con rmation. T e most common resistance
mutations a ect amino acids 255 and 315, both o which serve as contact points

(Continued)
666
Targeted Anticancer Therapies CHAPTER 4 6

TABLE 46-1 Monoclonal Antibodies Approved for Hematopoietic and Solid Tumors
ANTIGEN AND TUMOR RADIOISOTOPE-BASED
CELL TARGETS ANTIGEN FUNCTION NAKED ANTIBODIES ANTIBODIES TOXIN-BASED ANTIBODIES
Antigen: CD20

Tumor type: B-cell Proli eration/di erentiation Rituximab (chimeric) 131


I-tositumomab; None
lymphoma and CLL 90
Y-ibritumomab tiuxetan

Antigen: CD52

Tumor type: B-cell CLL and Unknown Alemtuzumab (humanized) None None
T-cell lymphoma

Antigen: CD33

Tumor type: acute Unknown Gemtuzumab (humanized) None Gemtuzumab ozogamicin


myelocytic leukemia

Antigen: HER2/neu
(ErbB2)

Tumor type: breast cancer Tyrosine kinase Trastuzumab (humanized) None None

Antigen: EGFR (ErbB1)

Tumor type: colorectal, Tyrosine kinase Cetuximab (chimeric) None None


NSCLC, pancreatic, breast Panitumumab (humanized)

Antigen: VEGF

Tumor type: colorectal Angiogenesis Bevacizumab (humanized) None None


cancer

CLL, chronic lymphocytic leukemia; EGFR, epidermal growth actor receptor; NSCLC, non-small cell lung cancer; VEGF, vascular endothelial
growth actor.

CH3 CH3
N N
O CH3 O CH3

CYP2D6
(CYP 2B6, CYP 2C9,
CYP 2C19, CYP 3A)
S ULT1A1
OH
Tamoxife n 4-hydroxy-tamoxife n

CYP3A4/5
(CYP 2C9 + othe r CYP3A4/5 S ulfa te
CYP s ) me ta bolite s
CH3 CH3
N N
O H O H

S ULT1A1

CYP2D6

OH
N-de s me thyl-tamoxife n Endoxife n

FIGURE 46-3 Tamoxi en and its metabolites.


667
SECTION VIII Chemotherapy of Neoplastic Diseases

DHEA
s ulfa te

Chole s te rol

17 17, 20
Pre gne nolone 17-hydroxypre gne nolone De hydroe pia ndros te rone
(DHEA)

3 a bira te rone ke tocona zole 3 a bira te rone 3

17 17, 20
Proge s te rone 17-hydroxyproge s te rone Andros te ne dione
A
21 21 17R
Es trone
17, 20
17R
De oxycorticos te rone 11-de oxycortis ol Te s to s te ro ne A
a bira te rone Es tradio l
11 11 5 R

Corticos te rone Co rtis o l Dihydro te s to s te ro ne

18

Aldo s te ro ne

FIGURE 46-4 Steroid synthesis pathways. The enclosed shaded area contains the pathways used by the adrenal glands and gonads. Enzymes
are shown next to their respective biochemical pathways, inhibitors are shown in boxes with T-shaped arrows pointing at the enzymes they
inhibit. 11, 11-hydroxylase; 17,20, C-17,20-lyase (also CYP17); 17, 17-hydroxylase (CYP17); 17R, 17-reductase; 18, aldosterone synthase; 21,
21-hydroxylase; 3, 3-hydroxysteroid dehydrogenase; 5R, 5-reductase; A, aromatase.

or imatinib; these mutations con er high-level resistance to imatinib and nilotinib.


Dasatinib is una ected by mutation at 255 but is ine ective in the presence o
mutation at 315. Nilotinib retains inhibitory activity in the presence o most point
mutations (except at 315) that con er resistance to imatinib.
Other mutations a ect the phosphate-binding region and the activation loop o
the domain with varying degrees o associated resistance. Some mutations, such as
at amino acids 351 and 355, do not a ect response to dasatinib or nilotinib but con-
er low levels o resistance to imatinib.
Molecular studies o circulating tumor cells have detected resistance-mediating
kinase mutations prior to initiation o therapy, particularly in patients with Ph +
acute lymphoblastic leukemia (ALL) or CML in blastic crisis. T is nding strongly
supports the hypothesis that drug-resistant cells arise through spontaneous muta-
tion and expand under the selective pressure o drug exposure. Mutations may
become detectable in the peripheral blood o patients receiving imatinib in the
accelerated phase and in the late (>4 years rom diagnosis) chronic phase o CML,
heralding the onset o drug resistance.
Mechanisms other than BCR-ABL kinase mutation play a minor role in resistance
to imatinib. Ampli cation o the wild-type kinase gene, leading to overexpres-
sion o the enzyme, has been identi ed in tumor samples rom patients resistant
to treatment. T e multidrug resistant (MDR) gene, which codes or a drug e ux
protein, con ers resistance experimentally but has not been implicated in clinical
resistance. Finally, Philadelphia chromosome-negative clones lacking the BCR-ABL
translocation and displaying the karyotype o myelodysplastic cells may emerge in
patients receiving imatinib or CML and may progress to myelodysplasia (MDS)
and to acute myelocytic leukemia (AML). T eir origin is unclear.
(Continued)

668
Targeted Anticancer Therapies CHAPTER 4 6

d. What other cancers and diseases are e ectively treated with imatinib?
Imatinib has e cacy in diseases in which the ABL, kit, or PDGFR protein kinases
have dominant roles in driving the proli eration o the tumor, re ecting the pres-
ence o a mutation that results in constitutive activation o the kinase, either by
usion with another protein or via point mutations. T us, imatinib shows remark-
able therapeutic bene ts in patients with chronic-phase CML (BCR-ABL), GIS
(kit mutation-positive gastrointestinal stromal tumor), chronic myelomonocytic
leukemia (EV 6-PDGFR translocation), hypereosinophilia syndrome (FIP1L1-
PDGFR), and dermato brosarcoma protuberans (constitutive production o the
ligand or PDGFR). It is the agent o choice or GIS patients with metastatic dis-
ease and as adjuvant therapy o c-kit-positive GIS . GIS biology is particularly
instructive, as patients with an exon 11 mutation o kit have a signi cantly higher
partial response rate (72%) than those with no detectable kit mutations (9%).
e. What are the common and important side e ects o therapy with imatinib,
dasatinib, and nilotinib?
Imatinib, dasatinib, and nilotinib cause GI distress (diarrhea, nausea, and vomit-
ing), but these symptoms usually are easily controlled. All 3 drugs promote uid
retention, which may lead to dependent edema, and periorbital swelling. Dasatinib
may cause pleural e usions. Nilotinib may prolong the Q interval, and should
be used with caution in patients with underlying heart disease or arrhythmias,
although ventricular arrhythmias have not been reported. Signi cant myelosup-
pression occurs in requently but may require trans usion support, dose reduction,
or discontinuation o the drug. All 3 drugs in this class can be associated with hepa-
totoxicity. Most nonhematological adverse reactions are sel -limited and respond
to dose adjustments. Af er the adverse reactions such as edema, myelosuppression,
or GI symptoms have been resolved, the drug may be reinitiated and titrated back
to e ective doses.

CASE 46 2
In a number o epithelial cancers, the epidermal growth actor receptor (EGFR) is
overexpressed or is activated by mutations.
a. What role does the EGFR play in these epithelial cancers?
T e EGFR belongs to the ErbB amily o transmembrane receptor tyrosine kinases.
EGFR, also known as ErbB1 or HER1, is essential or the growth and di erentia-
tion o epithelial cells. Ligand binding to the extracellular domain o EGFR amily
members causes receptor dimerization and stimulates the protein tyrosine kinase
activity o the intracellular domain, resulting in autophosphorylation o several
yr residues in the C-terminal domain. Recognition o the phosphotyrosines by
other proteins initiates protein-protein interactions that result in stimulation o a
variety o signaling pathways, including MAPK, PI3K/Akt, and S A pathways (see
Figures 46-2 and 46-5). In epithelial cancers, overexpression (or mutational activa-
tions) o the EGFR is a common nding and, to some extent, creates a dependence
on EGFR signaling in these tumors.
b. What drugs are available to target the EGFR in epithelial cancers?
wo separate classes o drugs that target the EGFR pathway have become important
agents in the therapy o solid tumors. T e EGFR tyrosine kinase inhibitors erlotinib
and ge tinib bind to the kinase domain and block the enzymatic unction o EGFR.
T e monoclonal antibodies cetuximab and panitumumab (see able 46-1) bind
speci cally to the extracellular domain o EGFR. T ey inhibit EGFR-dependent
signaling through inhibition o ligand-dependent activation and receptor dimeriza-
tion, downregulation o EGFR expression, and induction o antibody-dependent
cell-mediated cytotoxicity.
(Continued)

669
SECTION VIII Chemotherapy of Neoplastic Diseases

EGF PDGF

IGF-1 TGF

RAS

PI3K

RAF S MADS

AKT

MEK

TRANS CRIPTION
S uppre s s io n pro life ratio n
o f apo pto s is ang io g e ne s is
me tas tas is

FIGURE 46-5 Growth actor signaling. Binding o agonist ligands to growth actor receptors
causes receptor dimerization and activation o cytosolic protein kinase domains, leading to
activation o multiple signaling pathways. Shown here are the RAS/MAPK/ ERK, PI3K, and SMAD
pathways, each o which is activated by receptors or cross-talk rom adjacent pathways. Their
signals regulate proli eration, metabolism, survival, and the synthesis o other growth actors,
such as the vascular endothelial growth actor (VEGF).

c. What kinds o epithelial cancers are e ectively treated with EGFR-targeted


agents?
Erlotinib is approved or rst-line treatment o patients with locally advanced,
unresectable, or metastatic pancreatic cancer in combination with gemcitabine,
and is also approved or second-line treatment o patients with locally advanced or
metastatic nonsmall cell lung cancer.
Ge tinib initially was approved or the third-line treatment o patients with non
small cell lung cancer based on promising results in 2 small clinical trials. However,
a larger, randomized, placebo-controlled trial ailed to show an e ect on survival,
leading the FDA to restrict its use to patients who have previously received clini-
cal bene t rom the drug. Ge tinib continues to be widely used outside the United
States.
Retrospective analysis o multiple studies has revealed that patients who were
nonsmokers, Asians, or women were most likely to respond to ge tinib. umors
rom these patients requently have characteristic activating mutations in EGFR.
T ese mutations mostly all into 2 groups: (1) small in- rame deletions within exon
19 and (2) the L858R point mutation. rials that randomized never- and light-
smoking patients to rst-line ge tinib or to chemotherapy demonstrated a 70%
response rate to ge tinib in patients with an activating EGFR mutation, compared
with responses in only 1% in patients without an EGFR mutation. In patients with
EGFR-mutant tumors, the response rate to ge tinib was double the response to
standard chemotherapy. T ere ore, mounting evidence supports the use o ge tinib,
and potentially erlotinib, as rst-line therapy in patients selected or the presence o
sensitizing EGFR mutations.
Cetuximab won FDA approval based on an improvement in overall survival when
used in combination with radiation therapy or locally or regionally advanced squa-
mous cell carcinoma o the head and neck (HNSCC). It received a second indica-
tion as monotherapy or patients with metastatic or recurrent HNSCC who had

(Continued)
670
Targeted Anticancer Therapies CHAPTER 4 6

ailed platinum-based chemotherapy. It has become a use ul agent in combination


with cisplatin-based chemotherapy, where it has shown an improvement in survival
compared to chemotherapy alone.
Panitumumab improves progression- ree survival in patients with metastatic
colorectal carcinoma, as demonstrated in patients with EGFR-expressing tumors
who had 2 or more previous therapies.
d. What are the common and important toxicities o EGFR-targeted agents?
T e most common adverse reactions in patients receiving erlotinib and ge tinib
are diarrhea, an acne orm rash, anorexia, and atigue. Most adverse e ects occur
within the rst month o therapy and are tolerable when managed with supportive
medications and dose reductions. Cetuximab therapy is associated with in usion-
related toxicity and skin rash in 75% o treated patients (see able 46-2).
Serious or atal interstitial lung disease, of en associated with symptoms o cough
and dyspnea, occurs with a requency o 0.7 to 2.5%.
Hepatic unction deserves close monitoring because serious or atal hepatic ailure
due to erlotinib has been reported, particularly in patients with baseline hepatic
dys unction. Asymptomatic increases in liver transaminases with ge tinib may
necessitate dose reduction or discontinuation o therapy.
Other rare but serious toxicities with erlotinib therapy include GI per oration, renal
ailure, arterial thrombosis, microangiopathic hemolytic anemia, hand- oot skin
reaction, and corneal per oration or ulceration. Erlotinib therapy may cause rare
cases o Stevens-Johnson syndrome/toxic epidermal necrolysis.
(Continued)

TABLE 46-2 Dose and Toxicity of Monoclonal Antibody-Based Drugs


DRUG MECHANISM DOSE AND SCHEDULE MAJOR TOXICITY
Rituximab ADCC; CDC; apoptosis 375-mg/m 2 IVin usion weekly or In usion-related toxicity with ever, rash,
4 weeks and dyspnea; B-cell depletion; late-onset
neutropenia

Alemtuzumab ADCC; CDC; apoptosis Escalating doses o 3, 10, 30 mg/m 2 IV In usion-related toxicity, T-cell depletion
three times/week ollowed by 30 mg/ with increased in ection; hematopoietic
m 2 three times/week or 4-12 weeks suppression; pancytopenia

Trastuzumab ADCC; apoptosis; inhibition o arrest Loading dose o 4-mg/kg in usion Cardiomyopathy; in usion-related
HER2 signaling with G1 arrest ollowed by 2 mg/kg weekly toxicity

Cetuximab Inhibition o EGFR signaling; Loading dose o 400-mg/kg in usion In usion-related toxicity; skin rash in 75%
apoptosis; ADCC ollowed by 250 mg/kg weekly

Bevacizumab Inhibition o angiogenesis/ 5 mg/kg IVevery 14 days until Hypertension; pulmonary hemorrhage;
neovascularization disease progression GI per oration; proteinuria; congestive
heart ailure

Denileukin diftitox Targeted diphtheria toxin with 9-18 g/kg/day IV or the rst 5 days Fever; arthralgia; asthenia; hypotension
inhibition o protein synthesis every 3 weeks

Gemtuzumab Double-strand DNA breaks and Two doses o 9 mg/m 2 IVseparated In usion-related toxicity; hematopoietic
ozogamicin apoptosis by 14 days suppression; mucosal hepatic (VOD);
skin toxicity
90
Y ibritumomab Targeted radiotherapy 0.4 mCi/kg IV Hematological toxicity; myelodysplasia
tiuxetan
131
I tositumomab Targeted radiotherapy Patient-speci c dosimetry Hematological toxicity; myelodysplasia

ADCC, antibody-dependent cellular cytotoxicity; CDC, complement-dependent cytotoxicity; EGFR, epidermal growth actor receptor;
intravenous; VOD, veno-occlusive disease.
671
SECTION VIII Chemotherapy of Neoplastic Diseases

e. What are the mechanisms o resistance to EGFR-targeted agents?


Patients with nonsmall cell lung cancer who initially respond to erlotinib or
ge tinib have tumors that are dependent on the EGFR signaling pathway. Clini-
cal response to these agents is strongly associated with the presence o sensitizing
mutations in EGFR as described in the answer to part c above, but response is less
strongly correlated with overexpression or ampli cation o EGFR.
umors containing mutations in EGFR initially respond to erlotinib and ge tinib
but eventually progress. Resistance arises through several di erent mechanisms. A
secondary mutation in the EGFR gatekeeper residue, 790M, prevents binding o
drug to the kinase domain and con ers resistance. Irreversible EGFR inhibitors cur-
rently are in clinical development to overcome this mechanism.
Ampli cation o the ME oncogene provides an alternative pathway to clinical
resistance by activating cell growth signals downstream o EGFR. ME -ampli ed
tumors respond in vitro to the simultaneous inhibition o EGFR and ME . Other
potential mechanisms o resistance include activation o downstream mediators, e ux o
drug, and altered receptor tra cking. T e subset o nonsmall cell lung tumors harboring
k-ras mutations and EML4-ALK translocations do not respond to EGFR inhibitors.

CASE 46 3
A 58-year-old woman is diagnosed with metastatic Her2/neu-positive (Her2+)
breast cancer.
a. What is Her2/neu and how does this patients Her2/neu-positive status a ect her
prognosis and the available therapeutic options?
T irty percent o breast cancers overexpress the HER2/neu (ErbB2) receptor due to
gene ampli cation on chromosome 17. Ampli cation o the receptor is associated
with lower response rates to hormonal therapies and to most cytotoxic drugs, with
the exception o anthracyclines. Patients with HER2/neu-ampli ed tumors have
higher recurrence rates af er standard adjuvant therapy and poorer overall survival,
as compared to patients with HER2-nonampli ed tumors. T e internal domain o
the HER2/neu glycoprotein encodes a tyrosine kinase that activates downstream
signaling, enhances metastatic potential, and inhibits apoptosis.
Both antibodies (trastuzumab) and small molecules (lapatinib and others in clinical
trial) have striking antitumor e ects in patients with Her2-positive breast cancer,
and have become essential therapeutic agents in combination with cytotoxic
chemotherapy or this aggressive malignancy.
Currently, trastuzumab is approved or HER2/neu-overexpressing metastatic breast
cancer, in combination with paclitaxel as initial treatment or as monotherapy ol-
lowing chemotherapy relapse. rastuzumab synergizes with other cytotoxic agents
in HER2/neu-overexpressing cancers. Lapatinib is FDA-approved or HER2-ampli-
ed, trastuzumab-re ractory breast cancer, in combination with the uoropyrimi-
dine analog, capecitabine (see Chapter 45). As a small molecule, lapatinib crosses
the blood-brain barrier more readily than inhibitor antibodies and has produced
anecdotal responses in patients with brain metastases and decreased the incidence
o brain metastases in its phase III trial.
b. What are the mechanisms o action o the ErbB2-targeted agents?
rastuzumab exerts its antitumor e ects through several putative mechanisms o
action: inhibition o homo- or heterodimerization o receptor, thereby preventing
receptor kinase activation and downstream signaling; initiation o Fc-receptor-
mediated antibody-dependent cellular cytotoxicity; and blockade o the angioge-
netic e ects o HER2 signaling.
Small molecules can inhibit receptor tyrosine kinase activity o ErbB2 (HER2/neu)
and have antitumor activity in patients who have developed progressive disease

(Continued)
672
Targeted Anticancer Therapies CHAPTER 4 6

on trastuzumab. Lapatinib and other pan-HER inhibitors block both ErbB1 and
ErbB2 and bind to an internal site on the receptor (usually the A P-binding
pocket), compared to the external binding site o trastuzumab. Lapatinib also
inhibits a truncated orm o the HER2 receptor that lacks a trastuzumab-binding
domain. T ese di erences may account or the activity o lapatinib in trastuzumab-
resistant patients.
c. What other types o cancers express HER2/neu?
HER2/neu expression is also ound in subsets o patients with gastric, esophageal,
lung, and other solid tumors, but clinical studies o the e ects o trastuzumab in
these tumors have not yet been completed.
d. What are the important toxicities o the HER2/neu-targeted agents?
T e in usional e ects o trastuzumab are typical o other monoclonal antibodies
and include ever, chills, nausea, dyspnea, and rashes. Premedication with diphen-
hydramine and acetaminophen is indicated.
T e most serious toxicity o trastuzumab is cardiac ailure (see able 46-2); reasons
or cardiotoxicity are poorly understood, although the HER2 antigen is highly
expressed in the developing heart during embryogenesis, and HER2 knockout mice
ail to survive because o cardiomyopathy. Cardiac ailure is a potentially disabling
or atal side e ect unless it is recognized early and the drug is discontinued. Be ore
initializing therapy, baseline electrocardiogram and cardiac ejection raction mea-
surement should be obtained to rule out underlying heart disease, and patients
deserve care ul clinical ollow-up thereaf er or signs or symptoms o congestive
heart ailure, such as cough, weight gain, or edema. When trastuzumab is used as a
single agent, less than 5% o patients will experience a decrease in lef -ventricular
ejection raction, and 1% will have clinical signs o congestive ailure. However, lef -
ventricular dys unction occurs in up to 20% o patients who received the antibody
in combination with doxorubicin and cyclophosphamide. T e risk o cardiac toxic-
ity is greatly reduced with taxanetrastuzumab combinations.
Lapatinib toxicities include mild diarrhea, cramping, and exacerbation o gas-
troesophageal re ux. When lapatinib is combined with capecitabine, diarrhea
becomes a signi cant side e ect in one-third o patients. Lapatinibs inhibition o
ErbB1 (EGFR) causes an acne orm rash in one-third o patients; the rash can be
e ectively controlled in most cases with topical or oral antibiotics and topical ben-
zoyl peroxide gel.
Unlike trastuzumab, lapatinib has not produced a clear signal o cardiac toxicity.
Nonetheless, because it targets ErbB2, lapatinib should be used with caution in
combination with other cardiotoxic drugs and with care ul surveillance in patients
who have underlying heart disease.

CASE 46 4
Angiogenesis is an essential property o many solid tumors, and the target o a number
o anticancer drugs. One clinically important angiogenic pathway involves vascular
endothelial growth actor (VEGF) and its cognate receptors.
a. What is VEGF and how is it important in tumor growth and survival?
Cancer cells secrete angiogenic actors that induce the ormation o new blood
vessels and guarantee the ow o nutrients to the tumor cells. Angiogenic actors
secreted by tumors include VEGF (vascular endothelial growth actor), FGF ( bro-
blast growth actor), GF- (trans orming growth actor ), and PDGF (platelet-
derived growth actor). Multiple tumor types overexpress these angiogenic actors.
T e best studied o the angiogenic actors is VEGF. VEGF initiates endothelial cell
proli eration when it binds to a member o the VEGF receptor (VEGFR) amily, a
group o highly homologous receptors with intracellular tyrosine kinase domains

(Continued)
673
SECTION VIII Chemotherapy of Neoplastic Diseases

that includes VEGFR1 (FL 1), VEGFR2 (KDR), and VEGFR3 (FL 4). T e bind-
ing o VEGF to its receptor activates the intracellular VEGFR tyrosine kinase
activity and initiates mitogenic and antiapoptotic signaling pathways within the
endothelial cell.
b. What agents target the VEGF pathway and what are their mechanisms o action?
Antibodies targeting VEGF, such as bevacizumab, sterically hinder the interaction
o VEGF with its receptor. As an alternative to VEGF antibody therapy, the inves-
tigational drug a ibercept (VEGF rap), a recombinant molecule that utilizes the
VEGFR1-binding domain to sequester VEGF, acts as a soluble decoy receptor or
VEGF. Alternatively, the propagation o proangiogenic signals can be abrogated
by the inhibition o the tyrosine kinase activity o VEGFR. T ree small molecules
(pazopanib, sora enib, and sunitinib) that inhibit the kinase unction o VEGFR-2
have been approved or clinical use.
c. What kinds o tumors are e ectively treated with agents that target the VEGF
pathway?
As a single agent, bevacizumab delays progression o renal-cell cancer, and, in com-
bination with cytotoxic chemotherapy, e ectively treats lung, colorectal, and breast
cancers. Bevacizumab is also approved as a single agent ollowing prior therapy or
glioblastoma.
Sunitinib competitively inhibits the binding o A P to the tyrosine kinase domain
on the VEGFR-2, and also inhibits other protein tyrosine kinases (FL 3, PDGFR-,
PDGFR-, RE , CSF-1R, and c-KI ). Sunitinib has activity in metastatic renal-
cell cancer, producing a higher response rate (31%) and a longer progression- ree
survival than any other approved antiangiogenic drug. Sunitinib also is approved
or treatment o advanced renal-cell carcinoma and GIS that have developed resis-
tance to imatinib as a consequence o c-KI mutations.
Sora enib, like sunitinib, targets multiple protein tyrosine kinases (VEGFR1,
VEGFR2, VEGFR3, PDGFR-, c-KI , FL -3, and b-RAF). Sora enib is the only
drug currently approved or treatment o hepatocellular carcinoma. It is also gener-
ally the pre erred rst-line therapy in metastatic renal-cell cancer.
d. What are the important toxicities o the agents that target the VEGF pathway?
T e main toxicities o bevacizumab (see able 46-2) are shared by all antiangio-
genic inhibitors, including sunitinib and sora enib. Speci cally, patients taking this
class o antiangiogenic agents can experience bleeding, hypertension, proteinuria,
and uncommonly, arterial thromboembolic events and intestinal per oration. How-
ever, because sunitinib is a multitargeted tyrosine kinase inhibitor, it has a broader
side e ect pro le than bevacizumab.
A prominent concern with this class o agents is the potential or vessel injury
and bleeding. Bevacizumab is contraindicated or patients who have a history o
hemoptysis, brain metastasis, or a bleeding diathsis, but in appropriately selected
patients, the rate o li e-threatening pulmonary hemorrhage is less than 2%. T e
sa ety o operating on patients treated with bevacizumab continues to be a major
concern because o the risk o bleeding and poor wound healing; elective surgery
should be delayed or at least 4 weeks rom the last dose o antibody, and treatment
should not be resumed or at least 4 weeks af er surgery.
Other toxicity characteristics o antiangiogenic drugs include hypertension and
proteinuria. A majority o patients receiving bevacizumab require antihyperten-
sive therapy, particularly those receiving higher doses and more prolonged treat-
ment. T e mechanism driving this hypertension is still unclear but may relate, in
part, to decreased endothelial nitric oxide production. T e blood pressure o all
patients on bevacizumab should be care ully monitored and antihypertensives used
when appropriate. Case reports describe patients with poorly controlled hyperten-
sion developing a reversible posterior leukoencephalopathy during bevacizumab
(Continued)
674
Targeted Anticancer Therapies CHAPTER 4 6

treatment. Bevacizumab also is rarely associated with congestive heart ailure,


probably secondary to hypertension. Proteinuria during bevacizumab treatment is
usually an asymptomatic nding and rarely associated with nephrotic syndrome.
T e most dreaded vascular toxicity o antiangiogenic agents is an arterial throm-
boembolic event (ie, stroke or myocardial in arction). o reduce the risk o arterial
thromboembolic events, clinicians should care ully evaluate a patients risk actors
(age >65 years, clotting diathesis, a past history o arterial thromboembolic events)
be ore starting the drug.
GI per oration, a potentially li e-threatening complication o bevacizumab, has
been observed with particular requency (up to 11%) in patients with ovarian
cancer, perhaps related to the presence o peritoneal carcinomatosis and to prior
abdominal surgery. In colon cancer patients, colonic per oration occurs in re-
quently during bevacizumab treatment but increases in requency in patients with
intact primary colonic tumors, peritoneal carcinomatosis, peptic ulcer disease, che-
motherapy-associated colitis, diverticulitis, or prior abdominal radiation treatment.
T e rate o colon per oration is less than 1% in breast and lung cancer patients
receiving the antibody.
Fatigue, the most common side e ect o sunitinib, a ects 50 to 70% o patients and
may be disabling.
Hypothyroidism occurs in 40 to 60% o patients. Bone marrow suppression and
diarrhea also are common side e ects; severe neutropenia (neutrophils <1000/mL)
develop in 10% o patients. Less common side e ects include congestive heart ail-
ure (usually in association with hypertension) and hand- oot syndrome. o moni-
tor or these side e ects, it is essential to check blood counts and thyroid unction
at regular intervals. Periodic echocardiograms also are recommended.
Common adverse e ects o sora enib include atigue, nausea, diarrhea, anorexia,
and rash; uncommonly, bone marrow suppression and GI per oration.

CASE 46 5
A 47-year-old patient diagnosed with renal cancer is being treated with temsirolimus, a
congener o rapamycin (sirolimus).
a. What is the mechanism o action o rapamycin and its congeners?
T e rapamycins inhibit an enzyme complex, m ORC1, which occupies a downstream
position in the PI3 kinase pathway (see Figure 46-2). m OR orms the m ORC1
complex with a member o the FK506-binding protein amily, FKBP12. Among other
actions, m ORC1 phosphorylates S6 kinase and also relieves the inhibitory e ect o
4EBP on initiation actor el -4E, thereby promoting protein synthesis and metabolism.
T e antitumor actions o the rapamycins result rom their binding to FKBP12 and
inhibition o m ORC1. Rapamycin and its congeners have immunosuppressant
e ects, inhibit cell-cycle progression and angiogenesis, and promote apoptosis.
b. What are the toxicities o the rapamycins?
T e rapamycin analogs have very similar patterns o toxicity. T e most promi-
nent side e ects are a mild maculopapular rash, mucositis, anemia, and atigue,
each occurring in 30 to 50% o patients. A minority o patients will develop leu-
kopenia or thrombocytopenia with progressive cycles o treatment, and these
e ects are reversed i therapy is discontinued. Less common side e ects include
hyperglycemia, hypertriglyceridemia, and, rarely, pulmonary in ltrates and inter-
stitial lung disease. Pulmonary in ltrates emerge in 8% o patients receiving evero-
limus and in a smaller percentage o those treated with temsirolimus. In patients
showing minor radiological changes, but without symptoms, drug administration
may be continued. I symptoms such as cough or shortness o breath develop or
radiological changes progress, the drug should be discontinued. Prednisone may
hasten the resolution o radiological changes and symptoms.
(Continued)
675
SECTION VIII Chemotherapy of Neoplastic Diseases

c. What are the mechanisms o resistance to the e ects o the rapamycins?


Resistance to m OR inhibitors is incompletely understood but may arise through
the action o a second m OR complex, m ORC2, which is una ected by rapamy-
cins and which regulates Akt kinase (see Figure 46-2). Experimental work sug-
gests that inhibition o m ORC1 leads to m ORC2 activation o Akt kinase and
the MAP kinase pathway, and these actions may be responsible or incomplete
responses or resistance o rapamycins. Dual m ORC1 and m ORC2 inhibitors are
in clinical development.

CASE 46 6
A 50-year-old woman has a small lump in one o her breasts removed. T e pathology
indicates it is an estrogen receptor-positive (ER+) breast cancer. Her oncologist starts
her on a course o tamoxi en.
a. What is the mechanism o action o tamoxi en in breast cancer and in noncan-
cerous tissues?
amoxi en is classi ed as a selective estrogen receptor modulator (SERM)
(see Chapter 28). SERMs bind to the ER and exert either estrogenic or anties-
trogenic e ects, depending on the speci c organ. amoxi en citrate is the most
widely studied antiestrogenic treatment in breast cancer. T e recent decline in
breast cancer mortality in Western countries is believed to be partly due to the
common use o tamoxi en, especially in the adjuvant setting. However, in addi-
tion to its estrogen antagonist e ects in breast cancer, tamoxi en also exerts
estrogenic agonist e ects on nonbreast tissues, thus in uencing the overall ther-
apeutic index o the drug.
amoxi en is a competitive inhibitor o estradiol binding to the estrogen receptor
(ER), o which there are 2 subtypes: ER and ER. T e 2 ER subtypes have di erent
tissue distributions and can either homo- or heterodimerize. Binding o estradiol
and SERMs to the estrogen-binding sites o the ERs initiates a change in con orma-
tion o the ER, dissociation o the ER rom heat-shock proteins, and inhibition o
ER dimerization. Dimerization acilitates the binding o the ER to speci c DNA
estrogen-response elements (EREs) in the vicinity o estrogen-regulated genes.
Coregulator proteins interact with the receptor to act as corepressors or coactiva-
tors o gene expression. Di erences in tissue distribution o ER subtypes, the unc-
tion o coregulator proteins, and the various transcriptional activating actors likely
explain the variability o response to tamoxi en in hormone receptorpositive (ER+)
breast cancer and its agonist and antagonist activities in noncancerous tissues.
Other organs displaying agonist e ects o tamoxi en include the uterine endome-
trium (endometrial hypertrophy, vaginal bleeding, and endometrial cancer), the
coagulation system (thromboembolism), bone metabolism (increase in bone min-
eral density [BMD]), and liver (alterations o blood lipid pro le).
Metabolism o tamoxi en is complex (see Figure 46-3) and principally involves
CYPs 3A4/5, and 2D6 in the ormation o N-desmethyl tamoxi en, and CYP2D6
to orm 4-hydroxytamoxi en, a more potent metabolite. Both metabolites can be
urther converted to 4-hydroxy-N-desmethyltamoxi en, which retains high a nity
or the ER.
b. What are the clinical uses o tamoxi en?
amoxi en has become a standard agent as a result o its anticancer activity and
good tolerability pro le. amoxi en is prescribed or the prevention o breast cancer
in high-risk patients, or the adjuvant therapy o early-stage breast cancer ollowing
primary tumor excision, and or the therapy o advanced (metastatic) breast cancer
(see able 46-3). It prevents the development o breast cancer in women at high risk
based on a strong amily history, prior nonmalignant breast pathology, or inheri-
tance o the BRCA1 or BRCA2 genes.
(Continued)

676
Targeted Anticancer Therapies CHAPTER 4 6

TABLE 46-3 Clinical Uses for Anti-Estrogen Therapy in ER+ Breast Cancer
DISEASE SETTING
DRUG ADJUVANT premen ADJUVANT postmen METASTATIC premen METASTATIC postmen
Tamoxi en Yes (5 yr) Yes (be ore AI or 2-5 yr) Yes Yes

Fulvestrant No No No Yes (PD on TAM or AI)

Anastrozole No Yes (up ront or a ter TAM) No Yes

Letrozole No Yes (up ront or a ter TAM) No Yes

Exemestane No Yes (up ront or a ter TAM) No Yes

Toremi ene No Yes No Yes

Premen, premenopausal; Postmen, postmenopausal; AI, aromatase inhibitor; PD, progressive disease; TAM, tamoxi en; ER, estrogen receptor.

c. What are selective estrogen receptor downregulators (SERDs) and how do they
di er rom SERMs like tamoxi en?
SERDs, also termed pure anti-estrogens, include ulvestrant and a host o agents in
experimental trials. SERDs, unlike SERMs, are devoid o any estrogen agonist activ-
ity. Fulvestrant is a steroidal antiestrogen that binds to the ER with an a nity more
than 100 times that o tamoxi en. T e drug inhibits the binding o estrogen but also
alters the receptor structure such that the receptor is targeted or proteasomal degra-
dation; ulvestrant also may inhibit receptor dimerization. Unlike tamoxi en, which
stabilizes or even increases ER expression, ulvestrant reduces the number o ER
molecules in cells, both in vitro and in vivo; as a consequence o this ER downregu-
lation, the drug abolishes ER-mediated transcription o estrogen-dependent genes.
Whereas tamoxi en is used in both premenopausal and postmenopausal women
with breast cancer at early and late stages, ulvestrant is reserved or use in premeno-
pausal women with metastatic breast cancer (see able 46-3).
d. What are the mechanisms by which tumors develop resistance to tamoxi en?
Despite its bene ts, initial or acquired resistance to tamoxi en requently occurs.
Polymorphisms in CYP2D6 that reduce its activity lead to lower plasma levels o
4-OH tamoxi en, a potent metabolite (see Figure 46-3), and are associated with
higher risks o disease relapse and a lower incidence o hot ashes. CYP2D6 is also
responsible or the activation o tamoxi en to its active metabolite endoxi en (see
Figure 46-3).
Cross-talk between the ER and HER2/neu pathway also has been implicated in
tamoxi en resistance. T e paired box 2 gene product (PAX2) has been identi ed as
a crucial mediator o ER repression o ErbB2 by tamoxi en. Interactions between
PAX2 and the ER coactivator AIB-1/SRC-3 determine tamoxi en response in breast
cancer cells.
e. What are the toxicities and bene cial e ects that this woman is likely to experi-
ence while on tamoxi en?
T e common adverse reactions to tamoxi en include vasomotor symptoms (hot
ashes), atrophy o the lining o the vagina, hair loss, nausea, and vomiting. T ese
may occur in less than or equal to 25% o patients and rarely are su ciently severe
to require discontinuation o therapy. Menstrual irregularities, vaginal bleeding and
discharge, pruritus vulvae, and dermatitis occur with increasing severity in post-
menopausal women.
amoxi en also increases the incidence o endometrial cancer by 2- to 3- old, par-
ticularly in postmenopausal women who receive 20 mg/d or 2 years or more. In
general, tamoxi en-associated endometrial cancers are reported as low-grade and
(Continued)

677
SECTION VIII Chemotherapy of Neoplastic Diseases

early-stage tumors. Standard practice guidelines rom the National Comprehensive


Cancer Network alert physicians to the evaluation o abnormal vaginal bleeding in
women with an intact uterus.
amoxi en increases the risk o thromboembolic events, which increase with the
age o a patient and also in the perioperative period. Hence, it of en is recom-
mended to discontinue tamoxi en prior to elective surgery. Because tamoxi en
is associated with thromboembolism, some authorities suggest that the pretreat-
ment evaluation o breast cancer patients should include screening or coagulation
abnormalities ( actor V Leiden, protein C, antithrombin de ects) and or a history
o thromboembolic disease. Presence o these risk actors should lead to exclusion
o women rom treatment.
Like estrogen, tamoxi en is a hepatic carcinogen in animals, although increases
in primary hepatocellular carcinoma have not been reported in patients on the
drug. amoxi en causes retinal deposits, decreased visual acuity, and cataracts in
occasional patients, although the requency o these changes is more common in
patients on high doses o drug.
In addition to its ability to prevent recurrence or the development o primary breast
cancer, tamoxi en has other end-organ bene ts related to its partial estrogenic
action. For example, it may slow the development o osteoporosis in postmenopausal
women. Like certain estrogens, tamoxi en lowers total serum cholesterol, low-density-
lipoprotein cholesterol, and lipoproteins, and raises apolipoprotein A-I levels,
potentially decreasing the risk o myocardial in arction.
. Af er receiving tamoxi en therapy or 2 years, the patient is switched to anastro-
zole. What is the mechanism o action o this agent and what is the rationale or
using it in this patient?
Anastrozole is a potent and selective aromatase inhibitor. Aromatase inhibitors
(AIs) block the unction o the aromatase enzyme (CYP19) that converts andro-
gens to estrogens. CYP19 is highly expressed in human placenta and in granulosa
cells o ovarian ollicles, where its expression depends on cyclical gonadotropin
stimulation. Aromatase also is present, at lower levels, in several nonglandular
tissues, including subcutaneous at, liver, muscle, brain, and normal breast, and
in breast-cancer tissue. T e aromatase enzyme is responsible or the conversion o
adrenal androgens and gonadal androstenedione and testosterone to the estrogens,
estrone (E1) and estradiol (E2), respectively (see Figure 46-4). In postmenopausal
women, this conversion is the primary source o circulating estrogens, while
estrogen production in premenopausal women primarily is rom the ovaries. In
postmenopausal women, AIs can suppress most peripheral aromatase activity,
leading to pro ound estrogen deprivation. T is strategy o estrogen deprivation
o ER+ breast cancer cells stands in contrast to the ER antagonist activity that
SERMs and SERDs exert.
AIs now are considered the standard o care or adjuvant treatment o postmeno-
pausal women with hormone receptorpositive breast cancer, either as initial ther-
apy or sequenced af er tamoxi en (see able 46-3).

CASE 46 7
An 84-year-old gentleman has metastatic prostate cancer with symptoms that include
bone pain. His oncologist recommends that he receive hormone therapy.
a. What is hormone therapy?
T e growth o a number o cancers is hormone-dependent or regulated by
hormones. Hormone analogs and antagonists are used or the treatment o both
breast and prostate cancer, as well as several other cancers (see able 46-4). T ese
molecules interrupt the stimulatory axis created by systemic pools o androgens
and estrogens, inhibit hormone production or binding to receptors, and ultimately
(Continued)

678
Targeted Anticancer Therapies CHAPTER 4 6

TABLE 46-4 Hormones and Antagonists


TYPE OF AGENT NONPROPRIETARY NAMES DISEASE
Adrenocortical suppressants Mitotane (o,p-DDD) Adrenal cortex cancer

Adrenocortico-steroids Prednisone (other equivalent Acute and chronic


preparations available) lymphocytic leukemia;
non-Hodgkins lymphoma;
Hodgkins disease; breast
cancer, multiple myeloma

Progestins Hydroxyprogesterone caproate, Endometrial, breast cancer


medroxyprogesterone acetate,
megestrol acetate

Estrogens Diethylstilbestrol, ethinyl Breast, prostate cancer


estradiol (other preparations
available)

Anti-estrogens Tamoxi en, toremi ene Breast cancer

Aromatase inhibitors Anastrozole, letrozole, Breast cancer


exemestane

Androgens Testosterone propionate, Breast cancer


f uoxymesterone (other
preparations available)

Anti-androgen Flutamide, casodex Prostate cancer

GnRH analog Leuprolide Prostate cancer

block the complex expression o genes that promotes tumor growth and survival.
T ese drugs have proven e ective in extending survival and delaying or preventing
tumor recurrence in breast cancer and prostate cancer.
b. What is the goal o hormone therapy in treating metastatic prostate cancer?
Localized prostate cancer requently is curable with surgery or radiation therapy.
However, when distant metastases are present, hormone therapy is the primary
treatment. Standard approaches either reduce the concentration o endogenous
androgens or inhibit their e ects. Androgen deprivation therapy (AD ) is the
standard rst-line treatment. AD is accomplished via surgical castration (bilat-
eral orchiectomy) or medical castration (using gonadotropin-releasing hormone
[GnRH] agonists or antagonists). Other hormone therapy approaches are used in
second-line treatment and include antiandrogens, estrogens, and inhibitors o ste-
roidogenesis (see able 46-4).
AD is considered palliative, not curative, treatment. AD can alleviate cancer-
related symptoms, produce objective responses, and normalize serum prostate
speci c antigen (PSA) in more than 90% o patients. AD provides important
quality-o -li e bene ts, including reduction o bone pain and reduction o rates o
pathological racture, spinal cord compression, and ureteral obstruction. It also
prolongs survival.
c. T is patient will receive androgen deprivation therapy (AD ) using a synthetic
GnRH analog, leuprolide. What is the mechanism o action o GnRH agonists?
T e biosynthesis o androgens, primarily in the testes and adrenals, is described
in Chapter 28, and the regulation o Leydig cell synthetic activity by the hypo-
thalamicpituitary axis is considered there as well. In the United States, the most
common orm o AD involves chemical suppression o the pituitary gland
(Continued)

679
SECTION VIII Chemotherapy of Neoplastic Diseases

with GnRH agonists. Synthetic GnRH analogs have greater receptor a nity and
reduced susceptibility to enzymatic degradation than the naturally occurring
GnRH molecule and are 100- old more potent. GnRH (also termed luteinizing hor-
monereleasing hormone, LHRH) agonists bind to GnRH receptors on pituitary
gonadotropin-producing cells, causing an initial release o both LH and FSH and a
subsequent increase in testosterone production rom testicular Leydig cells. Af er
~1 week o therapy, GnRH receptors are downregulated on the gonadotropin-pro-
ducing cells, causing a decline in the pituitary response. T e all in serum LH leads
to a decrease in testosterone production to castrate levels within 3 to 4 weeks o the
rst treatment. Subsequent treatments maintain testosterone at castrate levels.
d. During the rst ew days o treatment with leuprolide, the patient com-
plains that his bone pain has worsened. What is causing this worsening o his
symptoms?
During the rst ew days o therapy with a GnRH agonist, there is transient rise in
LH, and the resultant testosterone surge may induce an acute stimulation o prostate
cancer growth and a are o symptoms rom metastatic deposits. Patients may
experience an increase in bone pain or obstructive bladder symptoms lasting or
2 to 3 weeks. T e are phenomenon can be e ectively counteracted with concurrent
administration o 2 to 4 weeks o oral antiandrogen therapy (ie, combined androgen
blockade), which may inhibit the action o the increased serum testosterone levels.
Combined androgen blockade (CAB) requires administration o AD with an
antiandrogen. T e theoretical advantage is that the GnRH agonist will deplete
testicular androgens, while the antiandrogen component competes at the receptor
with residual androgens made by the adrenal glands. CAB provides maximal relie
o androgen stimulation. Numerous large trials have compared CAB with AD
monotherapy, with variable results. Several meta-analyses o these trials suggest a
bene t or CAB in 5-year survival but not at earlier time points. oxicity and costs
associated with CAB are higher than with AD alone.
GnRH antagonists have been developed to suppress testosterone while avoiding
the are phenomenon o GnRH agonists. Other than avoidance o the initial are,
GnRH antagonist therapy o ers no apparent advantage compared with GnRH
agonists. T e rst available GnRH antagonist, abarelix, rapidly achieves medical
castration. However, local reactions and anaphylaxis have discouraged its clinical
acceptance and have led to its withdrawal rom the market. A second GnRH antag-
onist, degarelix, is not associated with systemic allergic reactions and is approved
or prostate cancer in the United States.
e. Af er several months o treatment with leuprolide, the oncologist adds an addi-
tional agent, abiraterone to urther reduce growth o the patients tumor. What is
the rationale or adding this agent?
In the castrate state, androgen receptor (AR) signaling, despite low steroid lev-
els, supports continued prostate cancer growth. AR signaling may occur due to
androgens produced rom nongonadal sources, AR gene mutations, or AR gene
ampli cation. Nongonadal sources o androgens include the adrenal glands and the
prostate cancer cells themselves (see Figure 46-4). Androstenedione, produced by
the adrenal glands, is converted to testosterone in peripheral tissues and tumors.
Intratumoral de novo androgen synthesis also may provide su cient androgen or
AR-driven cell proli eration.
Abiraterone is an irreversible inhibitor o both 17-hydroxylase and C-17,20-lyase
CYP17 activity (see Figure 46-4). T e parent compound, abiraterone acetate, is
orally bioavailable and has been well tolerated in castration-resistant prostate
cancer patients as secondary hormone therapy in phase I and II studies. With
continuous administration, abiraterone increases AC H levels, resulting in miner-
alocorticoid excess. Glucocorticoids, such as prednisone, are administered to com-
pensate or inhibition o adrenal steroidogenesis.

680
Targeted Anticancer Therapies CHAPTER 4 6

KEY CONCEPTS
argeted anticancer therapies are designed to block the oncogenic pathways
that cause speci c cancers.
T e e cacy and speci city o targeted anticancer therapies depends on the
expression o the molecular target in cancer cells.
Resistance to targeted therapy can develop because o mutations in the molecu-
lar target, decreased expression o the target, alterations in target pathways, as
well as changes in drug e ux and metabolism.
Dif erent agents (eg, antibodies and small molecule inhibitors) directed at the
same molecular target can have very dif erent spectra o antitumor activity.
T e growth and survival o many cancers, particularly breast and prostate can-
cer, are hormone-dependent or regulated by hormones; hormone therapy can
extend survival and delay or prevent tumor recurrence in many patients.
argeted anticancer therapies are o en used in combination with cytolytic
anticancer agents to achieve higher response rates.

SUMMARY QUIZ

QUESTION 46-1 An important mechanism by which clinical resistance develops to


imatinib in chronic myelogenous leukemia is
a. increased excretion in the urine.
b. increased hepatic metabolism.
c. decreased bioactivation in cancer cells.
d. spontaneous mutations in the BCR-ABL kinase.
e. increased e ux rom tumor cells.

QUESTION 46-2 T e molecular target o trastuzumab is


a. BCR-ABL.
b. VEGF.
c. ErbB1.
d. ErbB2.
e. CD20.

QUESTION 46-3 Rituximab is approved as a single agent or relapsed indolent


lymphomas. Its molecular target is
a. the CD20 antigen on B cells.
b. the CD52 antigen on B cells.
c. the CD33 antigen on B cells and other hematopoietic cells.
d. BCR-ABL in lymphoblasts.
e. the IL-2 receptor on cells.

QUESTION 46-4 Which o the ollowing agents kills multiple myeloma (MM) cells by
indirectly preventing the transcriptional activity o NF-B?
a. T alidomide
b. Lenalidomide
c. Bortezomib
d. Sora enib
e. Sirolimus

681
SECTION VIII Chemotherapy of Neoplastic Diseases

QUESTION 46-5 Aldesleukin is approved or use in metastatic renal-cell cancer and


metastatic melanoma. Its mechanism o action in treating these cancers is to
a. antagonize IL-2 receptors on cells.
b. antagonize IL-2 receptors on epithelial cells.
c. stimulate the proli eration o activated cells.
d. stimulate the proli eration o activated B cells.
e. inhibit the secretion o cytokines by immune ef ector cells.

SUMMARY QUIZ ANSWER KEY

QUESTION 46-1 Answer is d. T e most important mechanism o acquired resistance


to imatinib and other tyrosine kinase inhibitors arises rom point mutations in the
kinase domain o BCR-ABL. Molecular studies o circulating tumor cells have detected
resistance-mediating kinase mutations prior to initiation o therapy, particularly in
patients with Ph + acute lymphoblastic leukemia (ALL) or CML in blastic crisis. T is
nding strongly supports the hypothesis that drug-resistant cells arise through sponta-
neous mutation and expand under the selective pressure o drug exposure. Mutations
may become detectable in the peripheral blood o patients receiving imatinib in the
accelerated phase and in the late (>4 years rom diagnosis) chronic phase o CML,
heralding the onset o drug resistance.
Mechanisms other than BCR-ABL kinase mutation play a minor role in resistance
to imatinib. Ampli cation o the wild-type kinase gene, leading to overexpression o
the enzyme, has been identi ed in tumor samples rom patients resistant to treatment.
T e multidrug resistant (MDR) gene, which codes or a drug e ux protein, con ers
resistance experimentally but has not been implicated in clinical resistance.
QUESTION 46-2 Answer is d. rastuzumab is a humanized monoclonal antibody that
binds to the external domain o HER2/neu (ErbB2).
QUESTION 46-3 Answer is a. Rituximab is a chimeric monoclonal antibody that tar-
gets the CD20 B-cell antigen ( ables 46-1 and 46-2). CD20 is ound on cells rom the
preB cell stage through its terminal dif erentiation to plasma cells and is expressed
on 90% o B-cell neoplasms. T e biological unctions o CD20 are uncertain, although
incubation o B cells with anti-CD20 antibody has variable ef ects on cell-cycle pro-
gression, depending on the monoclonal antibody type. Monoclonal antibody binding
to CD20 generates transmembrane signals that produce autophosphorylation and
activation o serine/tyrosine protein kinases, induction o c-myc oncogene expression,
and expression o major histocompatibility complex class II molecules. CD20 also may
regulate transmembrane Ca2+ conductance through its unction as a Ca2+ channel. It is
unclear which o these actions relates to the pharmacological ef ect o rituximab.
QUESTION 46-4 Answer is c. Bortezomib, an inhibitor o proteasome-mediated protein
degradation, has earned a central role in the treatment o MM. Bortezomib binds to the 5
subunit o the 20S core o the 26S proteasome and reversibly inhibits its chymotrypsin-like
activity. T is event disrupts multiple intracellular signaling cascades, leading to apoptosis.
A most important consequence o proteasome inhibition is its ef ect on NF-B,
a key transcription actor that promotes cell damage response and cell survival. Most
NF-B is ound in the cytosol bound to IB; in this orm, NF-B is restricted to the
cytosol and cannot enter the nucleus to regulate transcription. In response to stress
signals resulting rom hypoxia, chemotherapy, and DNA damage, IB becomes ubiqui-
tinated and then degraded via the proteasome. Its degradation releases NF-B, which
enters the nucleus, where it transcriptionally activates a host o genes involved in cell
survival (eg, cell adhesion proteins E-selectin, ICAM-1, and VCAM-1), as well as
proli erative (eg, cyclin-D1) or antiapoptotic molecules (eg, cIAPs, BCL-2). NF-B is
highly expressed in many human tumors, including MM, and may be a key actor in
tumor cell survival in a hypoxic environment and during chemotherapy. Bortezomib
(Continued)
682
Targeted Anticancer Therapies CHAPTER 4 6

blocks proteasomal degradation o IB, thereby preventing the transcriptional activity


o NF-B and downregulating survival responses.

QUESTION 46-5 Answer is c. Aldesleukin possesses the biological activities o human


native IL-2. IL-2 stimulates the proli eration o activated cells and the secretion o
cytokines rom NK cells and monocytes. IL-2 stimulation increases cytotoxic killing
by cells and NK cells. T e mechanism o tumor cell killing has not been precisely
de ned but is presumed to be the result o enhanced killing by immune ef ector cells.

SUMMARY: TARGETED ANTICANCER DRUGS


TOXICITIES
CLASS AND
SUBCLASSES NAMES CLINICAL USES COMMON UNIQUE; CLINICALLY IMPORTANT
Protein Tyrosine Imatinib mesylate See Table 45-5 Imatinib, dasatinib, and Imatinib, dasatinib, and
Kinase Inhibitors Dasatinib Imatinib, dasatinib, and nilotinib cause GI distress nilotinib can be associated with
ATP inhibitors Nilotinib nilotinib are e cacious (diarrhea, nausea, and hepatotoxicity; in requently cause
Ge tinib in diseases in which the vomiting), f uid retention, myelosuppression
Erlotinib ABL, kit, or PDGFR have which may lead to Interstitial lung disease in <2%
Lapatinib dominant roles in driving the dependent edema, and o patients taking ge tinib and
Sunitinib proli eration o the tumor periorbital swelling erlotinib that may be atal
Sora enib (eg, chronic myelogenous Dasatinib may cause pleural Rare but serious toxicities with
leukemia, GI stromal tumor e usions erlotinib include serious or atal
[GIST]) ref ecting the Nilotinib may prolong QT hepatic ailure, GI per oration,
presence o a mutation interval renal ailure, arterial thrombosis,
that results in constitutive Diarrhea and pustular/ microangiopathic hemolytic
activation o the kinase, papular rash common in anemia, hand- oot skin reaction,
either by usion with another patients taking ge tinib corneal per oration or ulceration,
protein or via point mutations and erlotinib; also GI e ects, Stevens-Johnson syndrome/toxic
Ge tinib and erlotinib are atigue, dry skin, pruritus epidermal necrolysis
used in patients with non- Sunitinib and sora enib
small lung cancer, especially are associated with
cancers associated with vascular toxicities due to
activating mutations in EGFR antiangiogenic e ects
Erlotinib in combination (bleeding, hypertension, and
with gemcitabine is rst- arterial thromboembolic
line treatment o patients events, GI per oration),
with locally advanced, also atigue, diarrhea, skin
unresectable, or metastatic reactions
pancreatic cancer Sunitinib is associated with
Lapatinib is used or HER2- hypothyroidism (50%) and
ampli ed, trastuzumab- severe neutropenia (10%)
re ractory breast cancer
Sunitinib is used or renal-cell
cancer and GIST
Sora enib is used or renal-
cell cancer and is the only
drug currently approved or
treatment o hepatocellular
carcinoma

Monoclonal Cetuximab See Table 46-1 See Table 46-2 See Table 46-2
Antibodies Panitumumab Cetuximab is used or Rash and dermatological Rare but serious adverse
Growth Factor Trastuzumab squamous cell carcinoma o toxicities are common e ects with cetuximab and
Receptors (EGFR, the head and neck (HNSCC), with cetuximab and panitumumab include interstitial
HER2/neu, VEGFRs, EGFR-positive metastatic panitumumab; headache lung disease, hypomagnesemia,
PDGFR) Antibodies colorectal cancer, and in and diarrhea may also occur cardiopulmonary arrest, and
combination with other anaphylactoid reactions
agents or other metastatic The most serious toxicity o
colon cancers trastuzumab is cardiac ailure;
Trastuzumab is used or patients should be evaluated
HER2/neu-ampli ed breast be ore therapy and a terward
cancer or changes in le t ventricular
unction
(Continued)
683
SECTION VIII Chemotherapy of Neoplastic Diseases

TOXICITIES
CLASS AND
SUBCLASSES NAMES CLINICAL USES COMMON UNIQUE; CLINICALLY IMPORTANT
Monoclonal Bevacizumab See Table 46-1 See Table 46-2 See Table 46-2
Antibodies Ranibizumab Bevacizumab is used in Potential or vessel injury, Bevacizumab is associated with
Growth Factor highly vascularized tumors, bleeding, and poor wound arterial thromboembolic events
(VEGF) Antibodies including renal-cell, lung, healing ollowing surgery and GI per oration
colorectal, and breast Hypertension (a majority Bevacizumab is contraindicated
cancers, and glioblastoma o patients require in patients who have a history
Bevacizumab restores antihypertensive therapy) o hemoptysis, brain metastasis,
hearing in patients with and proteinuria (usually or a bleeding diathesis; elective
progressive dea ness due to asymptomatic) surgery should be delayed or at
neuro bromatosis type 2 least 4 weeks rom the last dose o
related tumors antibody, and treatment should
Ranibizumab is used to treat not be resumed or at least 4
wet macular degeneration weeks a ter surgery
(see Chapter 47)

Monoclonal Rituximab See Table 46-1 See Table 46-2 See Table 46-2
AntibodiesCD20 131
I-Tositumomab Rituximab is used as a single The radioimmunoconjugates Rituximab in usion reactions
Antibodies 90
Y-Ibritumomab agent or relapsed indolent cause antibody-related can be li e-threatening, but
tiuxetan lymphomas and with hypersensitivity, bone with pretreatment are usually
O atumumab chemotherapy or di use marrow suppression, and mild and limited to ever,
large B-cell lymphoma and secondary leukemias chills, throat itching, urticaria,
other indolent B-cell non- O atumumabs primary and mild hypotension; severe
Hodgkin lymphomas (NHLs) toxicities consist o mucocutaneous skin reactions,
Rituximab is also used or immunosuppression and including Stevens-Johnson
autoimmune diseases, opportunistic in ection, syndrome are rare
including rheumatologic hypersensitivity reactions May cause reactivation o
disease, thrombotic during antibody in usion, hepatitis B virus or rarely, JC virus;
thrombocytopenic purpura, and myelosuppression should not be administered to
autoimmune hemolytic patients with active in ection
anemias, cryoglobulin- Hypogammaglobulinemia and
induced renal disease and autoimmune syndromes may
multiple sclerosis occur 1-5 months ollowing
O atumumab is approved treatment with rituximab
or treating patients with
chronic lymphocytic
leukemia (CLL)

Monoclonal Alemtuzumab See Table 46-1 See Table 46-2 See Table 46-2
AntibodiesCD52 Used to treat B- and T-cell Acute in usion reactions and CD4+ T-cell counts may remain
Antibodies low-grade lymphomas serious myelosuppression, pro oundly depleted or 1 year
and CLL (all blood lineages) with Alemtuzumab does not combine
signi cant risk o ungal, well with chemotherapy in
viral, and other opportunistic standard regimens because
in ections; patients should o signi cant in ectious
receive antibiotic and complications
antiviral prophylaxis and
be monitored or signs and
symptoms o CMVand other
in ections

Monoclonal Gemtuzumab See Table 46-1 See Table 46-2 See Table 46-2
AntibodiesCD33 ozogamicin Currently is approved in Myelosuppression in all Causes a syndrome that
Antibodies patients >60 years with AML patients, hepatocellular resembles hepatic venoocclusive
in rst relapse damage in 30-40% o disease when patients
patients subsequently undergo
myeloablative therapy or
when ollowing, high-dose
chemotherapy
Prolonged myelosuppression,
particularly delayed recovery o
platelet counts

(Continued)
684
Targeted Anticancer Therapies CHAPTER 4 6

TOXICITIES
CLASS AND
SUBCLASSES NAMES CLINICAL USES COMMON UNIQUE; CLINICALLY IMPORTANT
Thalidomide and Thalidomide See Table 45-5 Sedation and constipation Thalidomide-related peripheral
its Derivatives Lenalidomide Used in newly diagnosed are common with sensory neuropathy (10-30% o
(Immunomodulatory and heavily pretreated thalidomide, but not with patients); long-standing sensory
analogs, IMiDs) relapsed/re ractory multiple lenalidomide loss may not reverse and caution
myeloma (MM) patients Lenalidomide causes is advised in patients with
Lenalidomide also used signi cant leukopenia in preexisting neuropathy
in 5q myelodysplastic 20% o patients Rarely, lenalidomide causes
syndrome (MDS) and CLL Thromboembolic risk hepatotoxicity and renal
increases with both dys unction
thalidomide and CLL patients taking lenalidomide
lenalidomide should receive pretreatment
hydration and allopurinol to
avoid the consequences o tumor
swelling and tumor lysis
Thalidomide is teratogenic, but
lenalidomide is not

Proteasome Bortezomib Initial therapy or MM and as Thrombocytopenia Chronic peripheral neuropathy,


Inhibitors therapy or MM a ter relapse (28%), atigue, peripheral especially in patients with history
rom other drugs, relapsed neuropathy (12%), GI o neuropathy
or re ractory mantle cell distress, anemia and Cardiac toxicity and long QT are
lymphoma, and myeloma neutropenia, limb pain rare

mTOR Inhibitors Rapamycin See Table 45-5 [in previous Mild maculopapular rash, Pulmonary in ltrates and
Temsirolimus chapter] mucositis, anemia, and interstitial lung disease; i
Everolimus Used or renal-cell cancer atigue, each occurring in symptoms such as cough or
and may also be use ul in 30-50% o patients shortness o breath develop or
mantle cell lymphomas and Leukopenia or radiological changes progress, the
other cancers thrombocytopenia with drug should be discontinued
progressive cycles o
treatment in some patients

Interleukin-2 (IL-2) Aldesleukin See Table 45-5 Most patients develop a Most signi cant toxicity with
Receptor Agonists Denileukin di titox Aldesleukin used or pruritic skin rash over most aldesleukin is capillary leak
metastatic renal-cell cancer o the body syndrome which can be li e-
and metastatic melanoma See Table 46-2 or denileukin threatening, but reversible with
Denileukin di titox used di titox toxicities discontinuation o therapy;
or recurrent/re ractory patients should have good
cutaneous T-cell lymphomas hepatic and renal unction be ore
starting therapy
Typical signi cant toxicities with
denileukin di titox include acute
hypersensitivity reactions, vascular
leak syndrome, and constitutional
toxicities (see also Table 46-2)

Colony-Stimulating See Chapter 25 Used as supportive therapy See Chapter 25 See Chapter 25
Factors to restore hematopoiesis
ollowing high-dose and
combination therapies
(see Chapter 25)
(Continued)

685
SECTION VIII Chemotherapy of Neoplastic Diseases

TOXICITIES
CLASS AND
SUBCLASSES NAMES CLINICAL USES COMMON UNIQUE; CLINICALLY IMPORTANT
Glucocorticoids Prednisone See Table 46-4 See Chapter 29 See Chapter 29
Dexamethasone Used as cytotoxic agents
in the treatment o acute
leukemia in children and
malignant lymphoma in
children and adults
Used to control autoimmune
hemolytic anemia and
thrombocytopenia
associated with CLL
Used with radiotherapy to
reduce edema related to
tumors in critical areas such
as the brain and spinal cord

Progestins Medroxyprogesterone See Table 46-4 See Chapter 28 See Chapter 28


Megestrol acetate Used as second-line
hormonal therapy or
metastatic hormone-
dependent breast cancer
and in endometrial
carcinoma previously treated
by surgery and radiotherapy

Estrogens and See Chapter 28 See Table 46-4 See Chapter 28 High-dose therapy has signi cant
Androgens Used in hormone- risk o thromboembolism (see
dependent neoplasms Chapter 28)
such as breast and prostate
cancers

Antiestrogens Tamoxi en See Table 46-4 Vasomotor symptoms (hot Increased risk o thromboembolic
Selective Toremi ene Used or prevention o breast f ashes), atrophy o the lining events
Estrogen-Receptor Raloxi ene cancer in high-risk patients, o the vagina, hair loss, nausea, Increased risk o endometrial
Modulators (SERMs) or the adjuvant therapy o and vomiting cancer
early-stage breast cancer, and Menstrual irregularities,
or the therapy o advanced vaginal bleeding and
breast cancer (estrogen discharge, pruritus vulvae, and
receptor-positive [ER+] dermatitis are more severe in
cancers are most responsive) postmenopausal women

Antiestrogens Fulvestrant Used in postmenopausal Nausea, asthenia, pain,


Selective women as antiestrogen vasodilation (hot f ashes),
Estrogen-Receptor therapy o hormone and headache
Downregulators receptor-positive metastatic Risk o injection site
(SERDs) breast cancer a ter reactions (7%) reduced by
progression on rst-line slow injection
antiestrogen therapy
such as tamoxi en

Antiestrogens Anastrozole See Table 46-4 Vaginal bleeding, vaginal Lower risk o estrogen-
Aromatase (CYP19) Letrozole Adjuvant hormonal therapy discharge, hot f ashes, but related toxicities (including
Inhibitors Exemestane in postmenopausal women lower requency o such side thromboembolism, endometrial
with early-stage breast e ects than tamoxi en cancer) than tamoxi en, but
cancer and as treatment or greater risk o musculoskeletal
advanced breast cancer (ER+ disorders and ractures
and PR+)
(Continued)

686
Targeted Anticancer Therapies CHAPTER 4 6

TOXICITIES
CLASS AND
SUBCLASSES NAMES CLINICAL USES COMMON UNIQUE; CLINICALLY IMPORTANT
Gonadotropin- GnRH See Table 46-4 Vasomotor f ashing, loss Cause an initial release o LH and
Releasing Hormone Leuprolide Used or rst-line androgen o libido, impotence, FSH resulting in a testosterone
(GnRH) Agonists Buserelin deprivation therapy (ADT) gynecomastia, atigue, surge that may cause an acute
Na arelin as palliative treatment to anemia, weight gain, stimulation o prostate cancer
Histrelin alleviate cancer-related decreased insulin sensitivity, growth and a f areo symptoms
Triptorelin symptoms (eg, bone pain), altered lipid pro les, rom metastatic deposits; the
Goserelin produce objective responses, osteoporosis and ractures, f are phenomenon can be
and normalize serum and loss o muscle mass counteracted with concurrent
prostate-speci c antigen administration o antiandrogen
(PSA) in patients with therapy (known as combined
metastatic prostate cancer androgen blockade [CAB])

Gonadotropin- Cetrorelix Used in a similar manner Same as GnRH agonists


Releasing Hormone Ganirelix to GnRH agonists or ADT
(GnRH) Antagonists Abarelix in patients with metastatic
Degarelix prostate cancer, but avoids
the f are phenomenon o
GnRH agonists

Antiandrogens Cyproterone Megestrol Most commonly used as Cause more gynecomastia, Cyproterone is associated with
Steroidal secondary hormone therapy mastodynia, and liver toxicity
in prostate cancer or in CAB hepatotoxicity but less
Cyproterone has in erior vasomotor f ashing, and loss
e cacy compared with o bone mineral density than
other orms o ADT GnRH agonists

Antiandrogens Flutamide See Table 46-4 Flutamide causes diarrhea, Flutamide is associated with risk
Nonsteroidal Bicalutamide Used as secondary hormone breast tenderness, and o hepatotoxicity
Nilutamide therapy (in combination with nipple tenderness; less Nilutamide is associated rarely
other agents) or in CAB commonly, nausea, vomiting, with interstitial pneumonitis
Nilutamide causes mild
nausea, alcohol intolerance
(5-20%), and diminished
ocular adaptation to
darkness (25-40%)

Estrogens See Chapter 28 See Table 46-4 Impotence, loss o libido, Increased risk o myocardial
High-dose estrogen is and lethargy in arctions, strokes, and
used in patients with pulmonary emboli; increased
prostate cancer to reduce mortality
testosterone to castrate
levels via negative eedback
on the hypothalamic
pituitary axis
Estrogen may also exert a
cytotoxic e ect on prostate
cancer cells

Inhibitors o Ketoconazole Used to reduce nongonadal Ketoconazole causes Glucocorticoids are administered
Steroidogenesis Abiraterone (eg, adrenal and tumor) signi cant diarrhea and to compensate or inhibition o
androgen synthesis in hepatic enzyme elevations adrenal steroidogenesis
castration-resistant prostate Abiraterone increases
cancer ACTH levels, resulting in
mineralocorticoid excess

687
This page intentionally left blank
SECTION

Special Systems Pharmacology IX


47. Ocular Pharmacology 690

48. Dermatological Pharmacology 707

689
CHAPTER

47 Ocular Pharmacology
T is chapter will be most use ul a er having a basic understanding o the material
DRUGS INCLUDED IN THIS
in Chapter 64, Ocular Pharmacology in Goodman & Gilmans T e Pharmacological
CHAPTERa Basis of T erapeutics, 12th Edition In addition to the material presented here, the
Abobotulinumtoxin A(DYSPORT) 12th Edition contains:
Acetylcholine (MIOCHOL-E) A detailed description o the anatomy o the eye and extraocular structures
Apraclonidine (IOPIDINE) able 64-1 Autonomic Pharmacology o the Eye and Related Structures
Atropine (ATROPINE-CARE, ISOPTOATROPINE) able 64-8 Vitreous Substitutes
Azelastine (OPTIVAR) T e mechanism o action o vitamin A in retinal physiology and vision
Bepotastine (BEPREVE) Figure 64-7 Structural ormula or -carotene and structural ormulas or the vita-
Betaxolol (BETOPTIC, others) min A amily o retinoids
Bevacizumab (AVASTIN) able 64-9 Ophthalmic E ects o Selected Vitamin De ciencies and Zinc De ciency
Bimatoprost (LUMIGAN, LATISSE) Figure 64-8 Major steps in photoreceptor signaling
Brimonidine (ALPHAGAN, others)
LEARNING OBJECTIVES
Brinzolamide (AZOPT)
Understand the principles o using drugs to treat ophthalmic disorders
Brom enac(XIBROM)
Carbachol (MIOSTAT,ISOPTOCARBACHOL, Know the ocular toxicities o systemic drugs
others) Know the mechanisms o action, clinical uses, and toxicities o ophthalmic
Carteolol (OCCUPRES, others) drugs
Chondroitin sul ate (VISCOAT) Describe how ophthalmic drugs administered topically can cause systemic side
Cromolyn sodium(CROLOM, others) e ects
Cyanoacrylate tissue adhesive (ISODENT, Understand the pathophysiology o glaucoma and the role o pharmacotherapy
DERMABOND, HISTOACRYL) in its management
Cyclopentolate (CYCYLOGYL, others)
Cyclosporin A(RESTASIS) MECHANISMS OF ACTION OF DRUGS FOR OPHTHALMIC USE
Dexamethasone (DEXASOL, others) DRUG CLASS DRUG MECHANISM OF ACTION
Diclo enac(VOLTAREN, others) Autonomic Agents Dipive rin Prodrug that is converted to
Di uprednate (DUREZOL) epinephrine by esterases in the
cornea see Table 47 4, Figure 47 1,
Dipive rin (PROPINE, others) and Chapter 7)
Dorzolamide (TRUSOPT,others)
Apraclonidine Selective 2 adrenergic agonist see
Echothiophate (PHOSPHOLINEIODIDE) Brimonidine Table 47 4 and Figure 47 1)
Emedastine di umarate (EMADINE)
Acetylcholine Cholinergic agonist see Table 47 4,
Epinastine (ELESTAT) Carbachol Figure 47 1, and Chapter 6)
Fibrinogen glue (TISSEEL, EVICEL) Pilocarpine
Fluocinolone ophthalmicimplant (RETISERT) Echothiophate Organophosphate
Fluorescein sodium acetylcholinesterase inhibitor see
Table 47 4, Figure 47 1, and Chapter 6)
Fluorometholone (FML, others)
Fluorouracil Atropine Muscarinic antagonist see
Scopolamine Table 47 4, Figure 47 1, and
Flurbipro en (OCUFEN, others) Homatropine Chapter 6)
Homatropine (ISOPTOHOMATROPINE, others) Cyclopentolate
Tropicamide
Hyaluronate (HEALON, others)
Hydroxyamphetamine + topicamide Phenylephrine Sympathomimetic agent see
(PAREMYD) Naphazoline Table 47 4, Figure 47 1, and
(continues) Tetrahydrozoline Chapter 7)
(Continued)
690
Ocular Pharmacology CHAPTER 4 7

DRUG CLASS DRUG MECHANISM OF ACTION DRUGS INCLUDED IN THIS


Betaxolol 1 Selective adrenergic receptor CHAPTER (Cont.)
antagonist see Table 47 4,
Figure 47 1, and Chapter 7)
Hydroxypropyl cellulose ophthalmicinsert
(LACRISERT)
Carteolol Nonselective adrenergic receptor Hydroxypropylmethylcellulose
Levobunolol antagonists see Table 47 4,
Metipranolol Figure 47 1, and Chapter 7) Indocyanine green
Timolol Ketorolac(ACULAR, others)
Prostaglandin Analogs Latanoprost Prostaglandin analogs that lower Ketoti en umarate (ZADITOR, ALAWAY)
Travoprost intraocular pressure IOP) by Latanoprost (XALATAN)
Bimatoprost acilitating aqueous out ow through
the accessory uveoscleral out ow Levobunolol (BETAGAN, others)
pathway Lodoxamide tromethamine (ALOMIDE)
Carbonic Anhydrase Dorzolamide Inhibit carbonic anyhdrase ound Loteprednol (ALREX,LOTEMAX)
Inhibitors Brinzolamide in the ciliary body epithelium; Metipranolol (OPTIPRANOLOL, others)
this reduces the ormation o
bicarbonate ions, which reduces Mitomycin (MUTAMYCIN)
uid transport and, thus, IOP Naphazoline (AK-CON, ABALON,
NAPHCON, others)
Ophthalmic Dexamethasone Anti in ammatory see
Glucocorticoids Prednisolone Chapters 22 and 29) Nedocromil (ALOCRIL)
Fluorometolone Nepa enac(NEVANAC)
Loteprednol
Rimexolone Olopatadine (PATANOL, PATADAY)
Di uprednate Onabotulinumtoxin A(BOTOX)
Triamcinolone intravitreal)
Fluocinolone implant)
Pegaptanib (MACUGEN)
Pemirolast (ALAMAST)
Nonsteroidal Anti Flurbipro en Anti in ammatory see Chapter 22)
in ammatory Agents Ketorolac Phenylephrine + cyclopentolate
Diclo enac (CYCLOMYDRIL)
Brom enac Phenylephrine + scopolamine
Nepa enac (MUROCOLL-2)
Antihistamines and Mast Emedastine di umarate H1 receptor antagonist Polydimethylsiloxanes (ADATOSIL5000)
Cell Stabilizers Prednisolone (PREDFORTE, others)
Cromolyn sodium Mast cell stabilizer that prevents Povidone iodine (BETADINEOPHTHALMIC)
Lodoxamide tromethamine the release o histamine and other
Ranibizumab (LUCENTIS)
Pemirolast autocoids rom mast cells
Rimexolone (VEXOL)
Nedocromil Mast cell stabilizer with some
antihistamine properties
Scopolamine (ISOPTOHYOSCINE)
Tetrahydrozoline (ALTAZINE, MURINE,
Olopatadine H1 receptor antagonists with mast VISINE, others)
Ketoti en umarate cell stabilizing activity
Bepotastine Timolol (TIMOPTIC, BETIMOL, others)
Azelastine Travoprost (TRAVATAN, TRAVATANZ)
Epinastine
Triamcinolone intravitreal ormulations
Immunosuppressive and Fluorouracil Antineoplastic agents see (TRIVARIS, TRIESENCE)
Antimitotic Agents Mitomycin Chapter 45) that act by limiting Tropicamide (MYDRIACYL)
the healing process and reduce
the risk o scarring Trypan Blue (VISIONBLUE,
MEMBRANEBLUE)
Cyclosporine A Inhibits activation o T cells
see Chapter 23) and decreases Tyloxapol (ENUCLENE)
in ammation in lacrimal gland Verteporfn (VISUDYNE)
Agents Used in Povidone iodine Antiseptic a
Drugs includedinthis chapter have specifc
Ophthalmic Surgery
ophthalmicuses. Antimicrobial agents are listedin
Tables 47-1through 47-3.Their pharmacologyis
discussedinprevious chapters.
(Continued)
691
SECTION IX Special Systems Pharmacology

DRUG CLASS DRUG MECHANISM OF ACTION


Hyaluronate Viscoelastic substances
Chondroitin sul ate
Hydroxypropyl
methylcellulose

Cyanoacrylate Tissue adhesives


Fibrinogen gel

Vitreous substitutes See Table 64 8 Goodman and


Gilmans The Pharmacological Basis of
Therapeutics, 12th Edition

Polydimethylsiloxanes Tamponade o retina

Agents Used in the Onabotulinumtoxin A Prevent acetylcholine release at


Treatment o Strabismus Abobotulinumtoxin A neuromuscular junction
and Blepharospasm

Agents Used to Treat Vertepor n Activation by laser in the presence


Macular Degeneration o oxygen generates ree radicals,
which cause vessel damage
and occlusion o choroidal
neovascularization

Pegaptanib Antagonist o VEGF an inducer o


angiogenesis)

Bevacizumab Inhibitors o vascular proli eration


Ranibizumab and tumor growth by inhibiting
VEGF A see Chapter 46)

Wetting Agents and Tear Tyloxapol Ophthalmic lubricants


Substitutes Carboxymethylcellulose
Hydroxyethyl cellulose
Hydroxypropyl cellulose
Hydroxypropyl methylcellulose
Methylcellulose

TABLE 47-1 Topical Antibacterial Agents Commercially Available or Ophthalmic Use


GENERIC NAME TRADE NAME FORMULATIONa TOXICITY INDICATIONS FOR USE

Azithromycin AZASITE) 1% solution H Conjunctivitis

Bacitracin 500 units/g ointment H Conjunctivitis, blepharitis, keratitis,


keratoconjunctivitis, corneal
ulcers, blepharoconjunctivitis,
meibomianitis, dacryocystitis

Besi oxacin BESIVANCE) 0.6% suspension Conjunctivitis

Chloramphenicol 1% ointment H, BD Conjunctivitis, keratitis

Cipro oxacin hydrochloride 0.3% solution; 0.3% ointment H, D RCD Conjunctivitis, keratitis,
(CILOXAN, others) keratoconjunctivitis,
corneal ulcers, blepharitis,
blepharoconjunctivitis,
meibomianitis, dacryocystitis

Erythromycin ILOTYCIN, others) 0.5% ointment H Super cial ocular in ections


involving the conjunctiva or
cornea; prophylaxis o ophthalmia
neonatorum
(Continued)
692
Ocular Pharmacology CHAPTER 4 7

GENERIC NAME TRADE NAME FORMULATIONa TOXICITY INDICATIONS FOR USE

Gati oxacin ZYMAR) 0.3% solution H Conjunctivitis

Gentamicin sul ate GARAMYCIN, 0.3% solution; 0.3% ointment H Conjunctivitis, blepharitis, keratitis,
GENOPTIC, GENT-AK, keratoconjunctivitis, corneal
GENTACIDIN, OTHERS) ulcers, blepharoconjunctivitis,
meibomianitis, dacryocystitis

Levo oxacin QUIXIN, IQUIX) 0.5% solution H Conjunctivitis

Levo oxacin IQUIX) 1.5% solution H Corneal ulcers

Moxi oxacin VIGAMOX) 0.5% solution H Conjunctivitis

O oxacin OCUFLOX, OTHERS) 0.3% solution H Conjunctivitis, corneal ulcers

Sul acetamide sodium BLEPH-10, 1%, 10%, 15%, and 30% solution; H, BD Conjunctivitis, other super cial
CETAMIDE, ISOPTO CETAMIDE, 10% ointment ocular in ections
others)

Polymyxin B combinationsb Various solutions and ointments Conjunctivitis, blepharitis, keratitis

Tobramycin sul ate c TOBREX, 0.3% solution; 0.3% ointment H External in ections o the eye and
AKTOB, DEFY, others) its adnexa
a
For speci c in ormation on dosing, ormulation, and trade names, re er to the PhysiciansDesk Reference for Ophthalmic Medicines, which is
published annually. bPolymyxin B is ormulated or delivery to the eye in combination with bacitracin, neomycin, gramicidin, oxytetracycline,
or trimethoprim. See Chapters 38 41 or urther discussion o these antibacterial agents. cTobramycin is ormulated or delivery to the eye in
combination with dexamethasone or loteprednol etabonate. H, hypersensitivity; BD, blood dyscrasia; D RCD, drug related corneal deposits.

TABLE 47-2 Antiviral Agents or Ophthalmic Use


GENERIC NAME TRADE NAME ROUTE OF ADMINISTRATION OCULAR TOXICITY INDICATIONS FOR USE

Tri uridine VIROPTIC, others) Topical 1% solution) PK, H Herpes simplex keratitis and
keratoconjunctivitis

Acyclovir ZOVIRAX) Oral, intravenous 200 mg Herpes zoster ophthalmicusa


capsules, 400 and 800 mg Herpes simplex iridocyclitis
tablets)

Valacyclovir VALTREX) Oral 500 and 1000 mg tablets) Herpes simplex keratitisa
Herpes zoster ophthalmicusa

Famciclovir FAMVIR) Oral 125 , 250 , and 500 mg Herpes simplex keratitisa
tablets) Herpes zoster ophthalmicusa

Foscarnet FOSCAVIR) Intravenous Cytomegalovirus retinitis


Intravitreala

Ganciclovir Intravenous, oral Cytomegalovirus retinitis


CYTOVENE) Intravitreal implant
VITRASERT)

Valganciclovir VALCYTE) Oral Cytomegalovirus retinitis

Cido ovir VISTIDE) Intravenous Cytomegalovirus retinitis


a
Of label use. For additional details, see Chapter 44. PK, punctate keratopathy; H, hypersensitivity.

693
SECTION IX Special Systems Pharmacology

TABLE 47-3 Anti ungal Agents or Ophthalmic Use


DRUG CLASS/AGENT METHOD OF ADMINISTRATION INDICATIONS FOR USE
Polyenes

Amphotericin Ba 0.1 0.5% typically 0.15%) topical Yeast and ungal keratitis and
solution endophthalmitis
0.8 1 mg subconjunctival Yeast and ungal endophthalmitis
5 g intravitreal injection Yeast and ungal endophthalmitis
Intravenous Yeast and ungal endophthalmitis

Natamycin 5% topical suspension Yeast and ungal blepharitis,


conjunctivitis, keratitis

Imidazoles

Fluconazole a Oral, intravenous Yeast keratitis and endophthalmitis

Itraconazole a Oral Yeast and ungal keratitis and


endophthalmitis

Ketoconazole a Oral Yeast keratitis and endophthalmitis

Miconazole a 1% topical solution Yeast and ungal keratitis


5 10 mg subconjunctival Yeast and ungal endophthalmitis
10 g intravitreal injection Yeast and ungal endophthalmitis

Of label use. Only natamycin NATACYN) is commercially available and labeled or ophthalmic use.
a

All other anti ungal drugs are not labeled or ophthalmic use and must be ormulated or the given
method o administration. For urther dosing in ormation, re er to the PhysiciansDesk Reference for
Ophthalmic Medicines. For additional discussion o these anti ungal agents, see Chapter 43.

Iris
Ce rvica l
sympa the tic
ne rve

Cilia ry
ga nglion

Optic Pa ra -
ne rve sympa the tic
fibe rs of
Optic oculomotor
cha s m ne rve
b
Optic S upe rior
tra ct ce rvica l
sympa the tic
ga nglion
La te ra l
ge nicula te Infe rior
body cilio-s pina l
a sympa the tic
Optic ce nte r
ra dia tion
Vis ua l
cortex Edinge r-We s tpha l Hypotha la mic
nucle i a nd sympa the tic
Motor ce nte r nucle i of Pe rlia ce nte r
in occipita l cortex

FIGURE 47-1 Autonomic innervation o the eye by the sympathetic a) and parasympathetic
b) nervous systems. (Adapted with permission from Wybar KC, Kerr-Muir M. Baillire Concise Medical
Textbooks, Ophthalmology, 3rd ed. Baillire Tindall, New York, 1984. Copyright Elsevier.)

694
Ocular Pharmacology CHAPTER 4 7

TABLE 47-4 Autonomic Drugs or Ophthalmic Use


DRUG CLASS TRADE NAME FORMULATION INDICATIONS FOR USE OCULAR SIDE EFFECTS
Cholinergic agonists

Acetylcholine MIOCHOL-E) 1% solution Miosis in surgery Corneal edema

Carbachol MIOSTAT, ISOPTO CARBACHOL, 0.01 3% solution Miosis in surgery Corneal edema, miosis, induced
others) Glaucomaa myopia, decreased vision, brow
ache, retinal detachment

Pilocarpine ISOPTO CARPINE, PILOCAR, 0.5%, 1%, 2%, 4%, and 6% Glaucoma Same as or carbachol
PILAGAN, PILOPINE HS, PILOPTIC, PILOSTAT, solution; 4% gel
others)

Anticholinesterase agents

Echothiophate PHOSPHOLINE IODIDE) 0.125% solution Glaucoma Retinal detachment, miosis,


Accommodative esotropia cataract, pupillary block
glaucoma, iris cysts, brow
ache, punctal stenosis o the
nasolacrimal system

Muscarinic antagonists

Atropine ATROPINE-CARE, ISOPTO 0.5%, 1%, and 2% solution; Cycloplegia, mydriasis,b Photosensitivity, blurred vision
ATROPINE) 1% ointment Cycloplegic retinoscopy,a
Dilated unduscopic exam

Scopolamine ISOPTO HYOSCINE) 0.25% solution


Homatropine ISOPTO HOMATROPINE, others) 2% and 5% solution
Cycloplegia, mydriasisb Same as or atropine
Cyclopentolate AK-PENTOLATE, CYCLOGYL) 0.5%, 1%, and 2% solution
Tropicamide MYDRIACYL, TROPICACYL) 0.5% and 1% solution

Sympathomimetic agents

Dipive rin AKPRO) 0.1% solution Glaucoma Photosensitivity, conjunctival,


hyperemia hypersensitivity

Phenylephrine AK-DILATE, MYDFRIN, 0.12%, 2.5%, and 10% Mydriasis, vasoconstriction,


NEO-SYNEPHRINE, others) solution decongestion
Apraclonidine IOPIDINE) 0.5% and 1% solution Ocular hypertension
Brimonidine ALPHAGAN-P, others) 0.1%, 0.15%, and 0.2% Glaucoma, ocular Same as or dipive rin
Naphazoline AK-CON, ALBALON, NAPHCON, solution hypertension
others) 0.012%, 0.03%, and 0.1% Decongestant
Tetrahydrozoline ALTAZINE, MURINE TEARS solution Decongestant
PLUS, VISINE, others) 0.05% solution

and adrenergic antagonists

Betaxolol 1 selective) BETOPTIC, 0.25% and 0.5% suspension


BETOPTIC-S, others) 1% solution
Carteolol ) OCUPRESS, others) 0.25% and 0.5% solution Glaucoma, ocular
Levobunolol ) BETAGAN, others) 0.3% solution hypertension
Metipranolol ) OPTIPRANOLOL, others) 0.25% and 0.5% solution
Timolol ) TIMOPTIC, TIMOPTIC XE, and gel
BETIMOL, others)
a
Of label use. Re er to the PhysiciansDesk Reference for Ophthalmic Medicines or speci c indications and dosing in ormation. bMydriasis
and cycloplegia, or paralysis o accommodation, o the human eye occurs a ter one drop o atropine 1%, scopolamine 0.5%, homatropine
1%, cyclopentolate 0.5% or 1%, and tropicamide 0.5% or 1%. Recovery o mydriasis is de ned by return to baseline pupil size to within
1 mm. Recovery o cycloplegia is de ned by return to within 2 diopters o baseline accommodative power. The maximal mydriatic ef ect o
homatropine is achieved with a 5% solution, but cycloplegia may be incomplete. Maximal cycloplegia with tropicamide may be achieved
with a 1% solution. Times to development o maximal mydriasis and to recovery, respectively, are: or atropine, 30 40 minutes and 7 10 days;
or scopolamine, 20 130 minutes and 3 7 days; or homatropine, 40 60 minutes and 1 3 days; or cyclopentolate, 30 60 minutes and 1 day;
or tropicamide, 20 40 minutes and 6 hours. Times to development o maximal cycloplegia and to recovery, respectively, are: or atropine,
60 180 minutes and 6 12 days; or scopolamine, 30 60 minutes and 3 7 days; or homatropine, 30 60 minutes and 1 3 days; or cyclopentolate,
25 75 minutes and 6 hours to 1 day; or tropicamide, 30 minutes and 6 hours.

695
SECTION IX Special Systems Pharmacology

SYSTEMIC DRUGS WITH OCULAR SIDE EFFECTS


DRUG SIDE EFFECT
Topiramate Angle closure glaucoma

Hydroxychloroquine Central retinal toxicity

Tamoxi en Crystalline maculopathy o the retina

Vigabatrin Progressive and permanent bilateral


concentric visual eld constriction

Digoxin A sign o elevated plasma concentrations is


a yellow halo around objects

Phosphodiesterase PDE5) inhibitors: sildena l, Seeing a bluish haze or experiencing light


vardena l, and tadala l sensitivity

Ethambutol Toxic optic neuropathy with progressive


Chloramphenicol bilateral scotomas and vision loss
Ri ampin

Ri abutin Iridocyclitis

Isotretinoin Dry eye and meibomian gland dys unction

Amiodarone Corneal drug deposits


Indomethacin
Atovaquone, chloroquine, hydroxychloroquine

Chlorpromazine Brown deposits in cornea


Thioridazine

Gold Gold deposition in cornea and conjunctiva


chrysiasis)

Tetracyclines Yellow discoloration o light exposed


conjunctiva

Minocycline Blue gray scleral pigmentation

CASE 47 1
A 48-year-old man with a 10-year history o congestive heart ailure ollowing an acute
myocardial in arction is treated with timolol eye drops or increased intraocular pressure
(IOP) A er using the eye drops or 1 week the patient notices increased atigue and
shortness o breath
a. What kind of drug in timolol?
imolol is a nonselective adrenergic receptor antagonist (see Chapter 7). Its
adverse systemic e ects are bradycardia, negative cardiac inotropy, and decreased
cardiac output. T ese e ects would worsen this patients congestive heart ailure.
b. How is the timolol associated with a worsening of this patients heart failure
when the drug is being administered as drops to the eye?
All ophthalmic medications are potentially absorbed into the systemic circula-
tion (see Figure 47-2; able 47-5), so undesirable systemic side e ects may occur.
opically administered drugs may undergo systemic distribution primarily by nasal
mucosal absorption and possibly by local ocular distribution by transcorneal/
transconjunctival absorption. Following transcorneal absorption, the aqueous
humor accumulates the drug, which then is distributed to intraocular structures
as well as potentially to the systemic circulation via the trabecular meshwork
pathway (see Figures 47-2 and 47-3).
(Continued)
696
Ocular Pharmacology CHAPTER 4 7

TEARS

CONJUNCTIVA

CORNEA S CLERA

AQUEOUS
HUMOR

IRIS CILIARY
BODY

SYSTEMIC CIRCULATION
FIGURE 47-2 Possible absorption pathways o an ophthalmic drug ollowing topical application
to the eye. Solid black arrows represent the corneal route; dashed blue arrows represent the
conjunctival/scleral route; the black dashed arrow represents the nasolacrimal absorption
pathway. (Adapted with permission from Chien DS et al. Curr Eye Res. 1990;9:10511059. Copyright
Taylor &Francis Group, http://www.informaworld.com.)

Systemic absorption a er ocular administration o adrenergic receptor antagonists


can induce all the side e ects ound with direct systemic administration. Absorption
rom the nasal mucosa avoids rst-pass metabolism by the liver, and consequently,
signi cant systemic side e ects may be caused by topical medications, especially
when used chronically. Possible absorption pathways o an ophthalmic drug ollowing
topical application to the eye are shown in Figures 47-2 and 47-3.

TABLE 47-5 Some Characteristics o Ocular Routes o Administration


ROUTE ABSORPTION PATTERN SPECIAL UTILITY LIMITATIONS AND PRECAUTIONS
Topical Prompt, depending on Convenient, economical, relatively Compliance, corneal and
ormulation sa e conjunctival toxicity, nasal mucosal
toxicity, systemic side ef ects rom
nasolacrimal absorption

Subconjunctival, sub Tenons, Prompt or sustained, depending Anterior segment in ections, Local toxicity, tissue injury, globe
and retrobulbar injections on ormulation posterior uveitis, cystoid macular per oration, optic nerve trauma,
edema central retinal artery and/or vein
occlusion, direct retinal drug toxicity
with inadvertent globe per oration,
ocular muscle trauma, prolonged
drug ef ect

Intraocular intracameral) Prompt Anterior segment surgery, Corneal toxicity, intraocular toxicity,
injections in ections relatively short duration o action

Intravitreal injection or device Absorption circumvented, Endophthalmitis, retinitis, age Retinal toxicity
immediate local ef ect, potential related macular degeneration
sustained ef ect

697
SECTION IX Special Systems Pharmacology

S upe rior la crima l pa pilla a nd puncta

La crima l ca na liculi

Orbita l (s upe rior)


pa rt of la crima l gla nd
Pa lpe bra l (infe rior)
La crima l s a c
pa rt of la crima l gla nd

Ducts of la crima l gla nd


Na s ola crima l duct

Ope ning of
na s ola crima l duct

Infe rior la crima l pa pilla a nd puncta

FIGURE 47-3 Anatomy o the lacrimal system.

CASE 47 2
A 17-year-old girl is brought to the emergency room a er ingesting Jimson weed
Her main complaint is the bright lights that are an irritation to her eyes Her pupils
are dilated
a. What is Jimson weed?
Jimson weed is commonly ound in backyards and vacant lots. Its seeds contain
belladonna alkaloids including atropine (see Chapters 3 and 6). T e seeds are
ingested to attain the CNS e ects o atropine including mild hallucinations.
b. Why do antimuscarinic substances cause mydriasis?
T e autonomic innervation o the eye by the sympathetic and parasympathetic
nervous systems is shown in Figure 47-1. T e e ect o cholinergic agents on the
pupil is constriction (miosis) (see able 47-6). T us, the e ect o an anticholin-
ergic agent such as atropine would be dilation (mydriasis). For urther explanation
see Chapter 6.

TABLE 47-6 Ef ects o Pharmacological Agents on the Pupil


CLINICAL SETTING DRUG PUPILLARY RESPONSE
Normal Sympathomimetic drugs Dilation mydriasis)
Normal Parasympathomimetic drugs Constriction miosis)
Horners syndrome Cocaine 4 10% No dilation
Preganglionic Horners Hydroxyamphetamine 1% Dilation
Postganglionic Horners Hydroxyamphetamine 1% No dilation
Adies pupil Pilocarpine 0.05 0.1%a Constriction
Normal Opioids oral or intravenous) Pinpoint pupils

Topically applied ophthalmic drugs unless otherwise noted. a This percentage o pilocarpine is not commercially available and usually is
prepared by the physician administering the test or by a pharmacist. This test also requires that no prior manipulation o the cornea ie,
tonometry or measuring intraocular pressure or testing corneal sensation) be done so that the normal integrity o the corneal barrier is intact.
Normal pupils will not respond to this weak dilution o pilocarpine; however, an Adies pupil mani ests a denervation supersensitivity and is,
there ore, pharmacodynamically responsive to this dilute cholinergic agonist.

698
Ocular Pharmacology CHAPTER 4 7

CASE 47-3
A 53-year-old man has been told by his ophthalmologist that his intraocular pressure is
increased and that he will need to be treated or glaucoma
a. What is the pathophysiology of glaucoma?
T e peripheral anterior chamber angle is an important anatomical structure or
di erentiating 2 orms o glaucoma: open-angle glaucoma, which is by ar the most
common orm o glaucoma in the United States, and angle-closure glaucoma (see
Figure 47-4). Current medical therapy o open-angle glaucoma is aimed at decreas-
ing aqueous humor production and/or increasing aqueous outf ow. T e pre erred
management or angle-closure glaucoma is surgical iridectomy, by either laser
or incision, but short-term medical management may be necessary to reduce the
acute intraocular pressure (IOP) elevation and to clear the cornea prior to surgery.
Long-term IOP reduction may be necessary, especially i the peripheral iris has
permanently covered the trabecular meshwork.

(Continued)

ca na l of S chle mm a nte rior cha mbe r


limbus pupil corne a
A (S chle mms ca na l) a ngle
iris
e pis cle ra l
tra be cula r ve ins
R
T
me s hwork pos te rior
N
O
I
E
cha mbe r
R
le ns
M
E
conjunctiva
G
T
cilia ry
E
N
body
A
S
zonule
vitre ous
R
T
O
N
choroid
I
R
E
M
E
T
re tina l
G
S
E
s cle ra a rte riole s
O
S
P
re tina a nd ve ins
optic ne rve
optic dis k ce ntra l re tina l
a rte ry a nd ve in

B Co rne a e pithe lium


Bowma ns me mbra ne
s troma
De s ce me ts me mbra ne
ca na l of
e ndothe lium S chle mm

a nte rior conjunctiva l


cha mbe r e pithe lium
tra be cula r
me s hwork
Te nons
s cle ra l ca ps ule
s pur +
e pis cle ra
pos te rior
cha mbe r
zonule zonula r
fibe rs
s cle ra
le ns

cilia ry proce s s
cilia ry mus cle Ciliary
cilia ry e pithe lium bo dy
pa rs pla na

FIGURE 47-4 A. Anatomy o the eye. B. Enlargement o the anterior segment, revealing the
cornea, angle structures, lens, and ciliary body. (Adapted with permission from Riordan-Eva P.
Anatomy and embryology of the eye. In, Vaughan &Asburys General Ophthalmology, 17th ed.
[Riordan-Eva P, Whitcher JP, eds.] McGraw-Hill, New York, 2008. Copyright 2008 by The McGraw-Hill
Companies, Inc. All rights reserved.)
699
SECTION IX Special Systems Pharmacology

b. What are the options for therapy in this patient?


able 47-4 lists the autonomic drugs or ophthalmic use. Current pharmacothera-
pies are targeted at decreasing the production o aqueous humor at the ciliary body
and increasing outf ow through the trabecular meshwork and uveoscleral path-
ways. T e goal is to prevent progressive glaucomatous optic-nerve damage with
minimum risk and side e ects rom either topical or systemic therapy. With these
general principles in mind, a stepped medical approach may begin with a topical
prostaglandin (PG) analog. Due to their once-daily dosing, low incidence o sys-
temic side e ects, and potent IOP-lowering e ect, PG analogs have largely replaced
adrenergic receptor antagonists as rst-line medical therapy or glaucoma. Modi-
cations to the chemical structure o PGF2 have produced analogs with a more
acceptable side-e ect pro le.
T e adrenergic receptor antagonists now are the next most common topical
medical treatment. T ere are 2 classes o topical blockers. T e nonselective ones
bind to both 1 and 2 adrenergic receptors and include timolol maleate and hemi-
hydrate (timolol hemihydrate is not available in the United States), levobunolol,
metipranolol, and carteolol.
When there are medical contraindications to the use o PG analogs or receptor
antagonists, other agents, such as a 2 adrenergic receptor agonist or topical car-
bonic anhydrase inhibitor (CAI), may be used as rst-line therapy. Any o these
drug classes can be used as additive second- or third-line therapy.

CASE 47 4
A 60-year-old man wakes up with a crusty discharge in both eyes His ophthalmologist
diagnoses blepharitis
a. What is blepharitis?
Blepharitis is a common bilateral inf ammatory process o the eyelids characterized
by irritation and burning, usually associated with a Staphylococcus spp. in ection.
Local hygiene is the mainstay o therapy; topical antibiotics requently are used,
usually in gel, drop, or ointment orm, particularly when the disease is accompa-
nied by conjunctivitis and keratitis.
Conjunctivitis is an inf ammatory process o the conjunctiva that varies in sever-
ity rom mild hyperemia to severe purulent discharge. T e more common causes o
conjunctivitis include viruses, allergies, environmental irritants, contact lenses, and
chemicals. T e less common causes include other in ectious pathogens, immune-
mediated reactions, associated systemic diseases, and tumors o the conjunctiva
or eyelid.
Keratitis, or corneal inf ammation, can occur at any level o the cornea (eg, epi-
thelium, subepithelium, stroma, and endothelium). It can be due to nonin ectious
or in ectious causes. Numerous microbial agents have been identi ed as causes o
in ectious keratitis, including bacteria, viruses, ungi, spirochetes, cysts, and tro-
phozoites. Severe in ections with tissue loss (corneal ulcers) generally are treated
more aggressively than in ections without tissue loss (corneal in ltrates).
T e mild, small, more peripheral in ections usually are not cultured, and the eyes
are treated with broad-spectrum topical antibiotics. In more severe, central, or larger
in ections, corneal scrapings or smears, cultures, and sensitivities are per ormed,
and the patient is immediately started on intensive hourly, around-the-clock topi-
cal antibiotic therapy. T e goal o treatment is to eradicate the in ection and reduce
the amount o corneal scarring and the chance o corneal per oration and severe
decreased vision or blindness. T e initial medication selection and dosage are
adjusted according to the clinical response, and culture and sensitivity results.
b. What are the treatment options with this patient?
able 47-1 shows the topical antibacterial agents available or ophthalmic use. See
Chapters 38, 39, 40, and 41 or additional details about speci c agents.
700
Ocular Pharmacology CHAPTER 4 7

CASE 47-5
A 33-year-old woman has developed a herpes simplex in ection o her cornea
(viral keratitis)
a. What is the pathophysiology of viral infections of the eye?
Viral keratitis, an in ection o the cornea that may involve either the epithelium or
stroma, is most commonly caused by herpes simplex type I and varicella zoster viruses.
Less common viral etiologies include herpes simplex type II, Epstein-Barr virus, and
cytomegalovirus (CMV). opical antiviral agents are indicated or the treatment o
epithelial disease due to herpes simplex in ection. When treating viral keratitis topi-
cally, there is a very narrow margin between the therapeutic topical antiviral activity
and the toxic e ect on the cornea; hence, patients must be ollowed very closely.
b. What treatment options are available for this patient?
T e various antiviral drugs currently used in ophthalmology are summarized in
able 47-2 (see Chapter 44 or additional details about speci c agents).
T e primary indications or the use o antiviral drugs in ophthalmology are viral
keratitis, herpes zoster ophthalmicus, and retinitis. T ere currently are no antivi-
ral agents or the treatment o viral conjunctivitis caused by adenoviruses, which
usually has a sel -limited course and typically is treated by symptomatic relie
o irritation.
opical glucocorticoids are contraindicated in herpetic epithelial keratitis due to
active viral replication. In contrast, or herpetic disci orm keratitis, which predomi-
nantly is presumed to involve a cell-mediated immune reaction, topical glucocorti-
coids accelerate recovery.

CASE 47 6
A 43-year-old woman who has been receiving chemotherapy or colon cancer is diag-
nosed with a ungal keratitis
a. What treatment is available for this patient?
T e only currently available topical ophthalmic anti ungal preparation is a poly-
ene, natamycin (NA ACYN). Other anti ungal agents may be extemporaneously
compounded or topical, subconjunctival, or intravitreal routes o administration
(see able 47-3). T e pharmacology and structures o speci c anti ungal agents are
given in Chapter 43.
As with systemic ungal in ections, the incidence o ophthalmic ungal in ections
has risen with the growing number o immunocompromised hosts. Ophthalmic
indications or anti ungal medications include ungal keratitis, scleritis, endo-
phthalmitis, mucormycosis, and canaliculitis. Risk actors or ungal keratitis
include trauma, chronic ocular sur ace disease, contact lens wear, and immunosup-
pression (including topical steroid use). In 2005 to 2006, there was a worldwide
epidemic o Fusarium ungal keratitis related to a speci c contact lens solution,
which resolved when it was removed rom the market. When ungal in ection is
suspected, samples o the a ected tissues are obtained or smears, cultures, and sen-
sitivities, and this in ormation is used to guide drug selection.

KEY CONCEPTS
T e eye is relatively secluded rom systemic access by blood-retinal, blood-
aqueous, and blood-vitreous barriers; as a consequence, the eye exhibits some
unusual pharmacodynamics and pharmacokinetic properties
opically administered drugs may undergo systemic distribution primarily by
nasal mucosal absorption and possibly by local ocular distribution by transcor-
neal/transconjunctival absorption
(Continued)
701
SECTION IX Special Systems Pharmacology

Drug absorption rom the nasal mucosa avoids rst-pass metabolism by the
liver, and consequently, signi cant systemic side e ects may be caused by topical
medications
Individual variation may be an important consideration or ocular drug distri-
bution due to drug-melanin binding to melanocytes in the iris
Classical pharmacokinetic theory based on studies o systemically administered
drugs does not ully apply to all ophthalmic drugs administered topically or locally
A er topical instillation o a drug, the rate and extent o absorption are deter-
mined by the time the drug remains in the cul-de-sac and precorneal tear lm,
elimination by nasolacrimal drainage, drug binding to tear proteins, drug metab-
olism by tear and tissue proteins, and di usion across the cornea and conjunctiva
Several drug ormulations prolong the time a drug remains on the sur ace o
the eye and thus acilitate drug absorption
Appropriate selection o an antimicrobial and route o administration is
dependent on the patients symptoms, the clinical examination, and the
culture/sensitivity results

SUMMARY QUIZ

QUESTION 47-1 A 42-year-old man with a seizure disorder has developed glaucoma
It is possible that he has developed glaucoma rom the use o
a phenytoin
b topiramate
c carbamazepine
d lamotrigine
e valproic acid

QUESTION 47-2 A 25-year-old woman with re ractory seizures is started on vigabatrin


She should be counseled about
a the development o glaucoma
b eye lid droop
c conjunctivitis
d permanent constriction o visual elds
e mydriasis

QUESTION 47-3 A 53-year-old woman with type 2 diabetes has noticed that her blood
sugar has been dif cult to regulate a er she started using glucocorticoid eye drops It is
likely that the glucocorticoid is being absorbed into the systemic circulation rom the
a nasal mucosa
b drops that spill rom the eye and enter her oral cavity
c lens
d vitreous humor
e canal o Schlemm

QUESTION 47-4 A 42-year-old man with glaucoma is treated with the 2 adrenergic
receptor agonist apraclonidine Despite being a derivative o clonidine, apraclonidine
does not cause systemic hypotension because it is
a tightly bound to plasma proteins
b metabolized rapidly by the liver
c highly ionized at physiological pH
d not absorbed rom the eye
e rapidly excreted by the kidney
702
Ocular Pharmacology CHAPTER 4 7

QUESTION 47-5 A 48-year-old man experiences a bluish haze and light sensitivity
when he takes which o the ollowing medications?
a etracycline
b Amoxicillin
c Metoprolol
d Ibupro en
e adala l

QUESTION 47-6 A 65-year-old woman is being treated with vertepor n intravenously


ollowed by laser light activation or age-related macular degeneration She should be
cautioned about which o the ollowing side e ects?
a Photosensitization
b Liver impairment
c Kidney impairment
d Chronic ophthalmic discharge
e Yellowing o the sclera

QUESTION 47-7 Vitamin A plays an essential role in the unction o the retina Vitamin A
de ciency inter eres with
a tear production
b vision in dim light
c color distinction
d peripheral vision
e vision in bright light

SUMMARY QUIZ ANSWER KEY


QUESTION 47-1 Answer is b. T e antiseizure drug topiramate requently has been
reported to cause choroidal e usions, thereby anteriorly rotating the ciliary body and
causing angle-closure glaucoma
QUESTION 47-2 Answer is d. T e antiseizure drug vigabatrin causes progressive and
permanent bilateral concentric visual eld constriction in a high percentage o patients
T e mechanism is not known, but vigabatrin is more e ectively transported into the
retina than into the brain, and consequently, elevations o retinal GABA concentrations
may contribute to the vision loss
QUESTION 47-3 Answer is a. opically administered drugs may undergo systemic dis-
tribution primarily by nasal mucosal absorption and possibly by local ocular distribution
by transcorneal/transconjunctival absorption (see Figures 47-2 and 47-3)
QUESTION 47-4 Answer is c. Apraclonidine is a relatively selective 2 adrenergic ago-
nist that is highly ionized at physiological pH and there ore does not cross the blood-
brain barrier
QUESTION 47-5 Answer is e. T e 3 phosphodiesterase (PDE) inhibitorssildena l,
vardena l, and tadala linhibit PDE5 in the corpus cavernosum to help achieve and
maintain penile erection T e drugs also mildly inhibit PDE6, which controls the levels
o cyclic GMP in the retina Visually, this can result in seeing a bluish haze or experi-
encing light sensitivity
QUESTION 47-6 Answer is a. Vertepor n is approved or photodynamic therapy o
the exudative orm o age-related macular degeneration (ARMD) with predominantly
classic choroidal neovascular membranes Vertepor n also is used in the treatment
o predominantly classic choroidal neovascularization caused by conditions such as
pathological myopia and presumed ocular histoplasmosis syndrome
(Continued)
703
SECTION IX Special Systems Pharmacology

Vertepor n is administered intravenously, and once it reaches the choroidal


circulation, the drug is light-activated by a nonthermal laser source Depending on the
size o the neovascular membrane and concerns o occult membranes and recurrence,
multiple photodynamic treatments may be necessary Activation o the drug in the
presence o oxygen generates ree radicals, which cause vessel damage and subsequent
platelet activation, thrombosis, and occlusion o choroidal neovascularization
T e potential side e ects include headache, injection-site reactions, and visual
disturbances T e drug causes temporary photosensitization, and patients must avoid
exposure o the skin or eyes to direct sunlight or bright indoor lights or 5 days a er
receiving it
QUESTION 47-7 Answer is b. Vitamin A de ciency inter eres with vision in dim light,
a condition known as night blindness (nyctalopia), as well as causing xerosis (dryness),
and keratomalacia (corneal thinning), which may lead to corneal per oration Nycta-
lopia may be reversed with vitamin A therapy; however, rapid, irreversible blindness
ensues i the cornea per orates
Following short-term deprivation o vitamin A, dark adaptation can be restored to
normal by the addition o retinol to the diet However, vision does not return to nor-
mal or several weeks a er adequate amounts o retinol have been supplied T e reason
or this delay is unknown

SUMMARY: DRUGS USED FOR OPHTHALMIC DISORDERS


TOXICITIES
Class and Subclasses Names Clinical Uses Common Unique; Clinically Important
Autonomic Agents Dipive rin Second line therapy or Localized irritation and Systemic ef ects o
glaucoma hyperemia epinephrine see Chapter 7)

Apraclonidine Treatment o glaucoma Skin allergy Systemic ef ects i absorbed


see Chapter 7)
Does not cross the blood
brain barrier so systemic
ef ects are less than with
clonidine

Brimonidine Treatment o glaucoma Skin allergy Systemic ef ects i absorbed


see Chapter 7)

Acetylcholine Used to induce Side ef ects and toxicity


intraoperative miosis are generally related to
systemic absorption see
Chapter 6)

Carbachol Used to induce Side ef ects and toxicity


intraoperative miosis are generally related to
systemic absorption see
Chapter 6)

Pilocarpine Historically important Side ef ects and toxicity


glaucoma medication, but are generally related to
rarely used today systemic absorption see
Chapter 6)

Echothiophate Miotic agent that may be Quaternary ammonium


used to treat glaucoma compound that is poorly
absorbed

Atropine Mydriasis, cycloplegia Photosensitivity and Side ef ects and toxicity are
Scopolamine blurred vision generally related to systemic
Homatropine absorption see Chapter 6)

(Continued)
704
Ocular Pharmacology CHAPTER 4 7

TOXICITIES
Class and Subclasses Names Clinical Uses Common Unique; Clinically Important

Cyclopentolate Mydriasis, cycloplegia


Tropicamide Used to treat uveitis

Phenylephrine Mydriasis, vasoconstriction, Side ef ects and toxicity are


decongestant generally related to systemic
absorption see Chapter 7)

Naphazoline Decongestant
Tetrahydozoline

Betaxolol Used to treat glaucoma Side ef ects and toxicity are


Carteolol generally related to systemic
Levobunolol absorption see Chapter 7)
Metipranolol
Timolol

Prostaglandin Latanoprost Used to treat glaucoma Blurred vision, burning, See Chapter 22
Analogs Travoprost stinging, conjunctival
Brimatoprost hyperemia, dry eyes

Carbonic Anhydrase Dorzolamide Used to treat glaucoma Side ef ects and toxicity are
Inhibitors Brinzolamide generally related to systemic
absorption see Chapter 15)

Ophthalmic Dexamethasone Treatment o ocular Side ef ects and toxicity Development o cataracts,
Glucocorticoids Prednisolone in ammatory diseases are generally related to secondary in ections,
Fluorometolone systemic absorption see secondary open angle
Loteprednol Chapter 29) glaucoma
Rimexolone
Di uprednate

Triamcinolone intravitreal) Intravitreal ormulation


or ocular in ammatory
conditions

Fluocinolone Ophthalmic implant used


or treatment o chronic
nonin ectious uveitis

Nonsteroidal Anti Flurbipro en Used to treat ocular Side ef ects and toxicity Occasionally associated with
in ammatory Agents Ketorolac in ammation are generally related to corneal per oration
Diclo enac systemic absorption see
Brom enac Chapter 22)
Nepa enac

Antihistamines and Emedastine di umarate Used to treat allergic Side ef ects and toxicity
Mast Cell Stabilizers Cromolyn sodium conjunctivitis are generally related to
Lodoxamide tromethamine systemic absorption see
Pemirolast Chapter 21)
Nedocromil
Olopatadine
Ketoti en umarate
Bepotastine
Azelastine
Epinastine

Immunosuppressive Fluorouracil Used in glaucoma surgery Can result in thin, ischemic,


and Antimitotic Mitomycin to limit the postoperative avascular tissue that is prone
Agents wound healing process and to breakdown
prevent scarring

Cyclosporine A Treatment o chronic dye eye Localized burning,


tearing, discharge, itching
Visual blurring
(Continued)
705
SECTION IX Special Systems Pharmacology

TOXICITIES
Class and Subclasses Names Clinical Uses Common Unique; Clinically Important

Agents Used in Povidone iodine Ophthalmic solution used or Contraindicated in patients


Ophthalmic Surgery preoperative preparation o that are hypersensitive to
periocular skin iodine

Hyaluronate Viscoelastic substances used Transient elevation o IOP


Chondroitin sul ate to assist in ocular surgery a ter surgery
Hydroxypropyl
methylcellulose

Cyanoacrylate Used as a tissue adhesive


Fibrinogen gel during ocular surgery
Approved or cardiac,
vascular, and general surgery,
but not or the eye

Vitreous substitutes Used or reattachment o the


retina ollowing vitrectomy
and other retinal surgeries
see Table 64 8 Goodman and
Gilman's The Pharmacological
Basis of Therapeutics, 12th
Edition)

Polydimethylsiloxanes Used or long term Glaucoma, cataract


tamponade o the retina ormation, corneal edema,
corneal band keratopathy,
and retinal toxicity

Trypan Blue Used during ophthalmic


surgery to visualize the lens
and to guide the excision o
tissue

Agents Used in Onabotulinumtoxin A Used in the treatment Double vision, lid droop
the Treatment o Abobotulinumtoxin A o strabismus and Potentially li e threatening
Strabismus and blepharospasm distant spread o toxin see
Blepharospasm Chapter 6)

Agents Used to Vertepor n Approved or photodynamic Headache, injection site Temporary photosensitivity
Treat Macular therapy o the exudative reactions, and visual o the skin and eyes
Degeneration orm o age related macular disturbances Patients must avoid direct
degeneration ARMD) sunlight or bright indoor
lights or 5 days

Pegaptanib Treatment o neovascular Side ef ects are related to Monitor patients or increase
wet) ARMD the intravitreal injection IOP and endophthalmitis
Rare cases o anaphylaxis

Bevacizumab Used or the treatment o Risk o hemorrhage and Associated with


Ranibizumab ARMD and of label to treat in ection rom intravitreal cerebrovascular accidents
neovascularization caused injection see Chapter 46)
by other diseases see
Chapter 46)

Wetting Agents and Tyloxapol Arti cial tears and ophthalmic Localized burning and
Tear Substitutes Carboxy methylcellulose lubricants used or the irritation
Hydroxylethyl cellulose management o dry eyes
Hydroxypropyl cellulose
Hydroxypropyl
methyl cellulose
Methylcellulose

706
CHAPTER

Dermatological Pharmacology 48
T is chapter will be most use ul a er having a basic understanding o the material in
DRUGS INCLUDED IN THIS
Chapter 65, Dermatological Pharmacology in Goodman & Gilmans T e Pharmacological
Basis of T erapeutics, 12th Edition. In addition to the material presented here, the CHAPTERa
12th Edition contains: Acitretin (SORIATANE)
A detailed description o the structures o the skin and their pharmacological Adalimumab (HUMIRA)
implications Adapalene (DIFFERIN)
A discussion o antimicrobial therapy o skin disorders Ale acept (AMEVIVE)
A discussion o the use o cytotoxic and immunosuppressive drugs in the treatment Alitretinoin (9-cis-retinoicacid)
o skin disorders (PANRETIN)
ables 65-4 and 65-5 which are a listing o topical and systemic retinoids, respectively Aminolevulinicacid (ALA, LEVULAN
Chemical structures o drugs used to treat dermatological disorders KERASTICK)
Azathioprine (IMURAN, others)
LEARNING OBJECTIVES
Azelaicacid (AZELEX,FINACEA)
Understand how drugs are absorbed through the skin.
Benzyl alcohol
Know the mechanisms o action, therapeutic uses, and toxicities o topical and
Bexarotene (TARGRETIN)
systemic drugs used to treat dermatological disorders.
Bleomycin (BLENOXANE)
Know the principles o photochemotherapy o dermatological disorders.
Calcipotriene (DOVONEX,others)
Know the science behind the use o sunscreen agents.
Capsaicin (ZOSTRIX,CAPSIN)
MECHANISMS OF ACTION Carmustine (BICNU)
DRUG CLASS DRUG MECHANISM OF ACTION Chloroquine (ARALEN, others)
Crotamiton (EURAX)
Glucocorticoids Triamcinolone acetonide See Chapter 29
Triamcinolone hexacetonide Cyclophosphamide
Numerous other agents Cyclosporine (NEORAL, GENGRAF, others)
(Table 48 7)
Dapsone (diaminodiphenylsul one, DDS)
Retinoids Tretinoin Exert their e ect on gene expression Denileukin Di titox(ONTAK)
Isotretinoin by activating 2 amilies o receptors:
Acitretin retinoic acid receptors (RARs) and Doxorubicin (DOXIL, CAELYX)
Tazarotene retinoid X receptors (RXRs) Ealizumab (RAPTIVA)
Bexarotene Bexarotene selectively binds RXRs
Adapalene Etanercept (ENBREL)
Alitretinoin Finasteride (PROPECIA, others)
Vitamin Analogs Calcipotriene Vitamin D analog exerts its e ects Fluorouracil (CARAC, others)
through the vitamin D receptor Hydroquinone (TRI-LUMA)
(VDR); upon binding the VDR, the
drug receptor complex associates Hydroxychloroquine (PLAQUENIL, others)
with RXR and binds to vitamin D Imiquimod (ALDARA)
response elements on DNA
In iximab (REMICADE)
Photochemotherapeutic Methoxsalen Methoxsalen ollowed by ultraviolet Isotretinoin(13-cis-retinoicacid)(ACCUTANE)
Drugs (PUVA, Table 48 4) radiation between 320 340 nm
Ivermectin (STOMECTOL)
(UVA) results in 2 photoreactions:
Type I reactions involve the oxygen Lindane
independent photoaddition o the Malathion (OVIDE)
psoralen to pyrimidine bases in
DNA; Type II reactions are oxygen Mechlorethamine (MUSTARGEN)
dependent and involve the trans er o Mequinol (SOLAGE, in combination with
energy to molecular oxygen, creating tretinoin and vitamin C)
reactive oxygen species (Table 48 4) (continues)
(continued)
707
SECTION IX Special Systems Pharmacology

DRUGS INCLUDED IN THIS DRUG CLASS DRUG MECHANISM OF ACTION


CHAPTER (Cont.) Photodynamic Therapy Aminolevulinic acid Prodrugs that are converted
(Table 48 4) Methylaminolevulinate to protoporphyrin IX in living
Methotrexate (RHEUMATRIX)
cells; in the presence o speci c
Methoxsalen (OXSORALEN, others)not wavelengths o light (Table 48 4)
marketed in the United States and oxygen, protoporphyrin
produces reactive oxygen species
Methylaminolevulinate (MAL, VETVIXIA)
Minoxidil (ROGAINE, others) Antimicrobial Agents Various antibacterial, See Chapters 38, 39, 40, 41, 43, and 44
anti ungal (Table 48 5),
Monobenzone (BENOQUIN) and antiviral agents
Mycophenolate mo etil (CELLCEPT)
Retapamulin Selectively inhibits bacterial protein
Permethrin (KLOUTshampoo) synthesis by interacting at a site
Pimecrolimus (ELIDEL) on the 50S subunit o bacterial
ribosomes
Podophyllin (podophyllumresin)
Quinacrine (ATABRINE) Agents Used to Treat Permethrin Inter eres with insect sodium
In estations transport proteins causing
Retapamulin (ALTABAX) neurotoxicity and paralysis
Retinol (vitamin A)
Malathion Cholinesterase inhibitor (see
Tacrolimus (FK506, PROTOPIC) Chapter 6)
Tazarotene (TAZORAC, others)
Ivermectin Anthelmintic (see Chapter 37)
Thalidomide (TALOMID)
Tretinoin (all trans-retinoicacid; vitamin A Lindane A pesticide that inter eres with
GABA neurotransmitter unction
acid) (ATRALIN, others)
Triamcinolone acetonide (KENALOG-10) Crotamiton Toxic to scabies mite by an unknown
mechanism
Triamcinolone hexacetonide (ARISTOSPAN)
Vinblastine (VELBAN) Benzyl alcohol Inhibits lice rom closing their
respiratory spiracles causing
a
Drugsincludedinthischapter havespecifc asphyxiation
dermatological uses. Antibioticsusedtotreat
Antimalarial Agents Chloroquine Proposed mechanisms o action
acne are listedinthe Side BarANTIBIOTICSUSED
Hydroxychloroquine include stabilization o lysosomes,
TOTREATACNE. Antimicrobialsusedtotreat Quinacrine inhibition o antigen presentation,
cutaneousin ectionsarediscussedinthe cases; inhibition o prostaglandin synthesis,
the pharmacologyo specifcdrugscanbe ound inhibition o inf ammatory cytokine
inChapters35, 39, 40, 41, 43, and44. Sunscreen synthesis, photoprotection,
agentsare listedintheTableSUNSCREENAGENTS. inhibition o immune complex
ormation, and antithrombotic
e ects (see Chapter 35)

Antimetabolites Methotrexate Folic acid analog that competitively


inhibits dihydro olate reductase (see
Chapter 45 and Table 48 8)

Azathioprine Impairs purine synthesis and cell


proli eration (see Chapter 23 and
Table 48 8)

Fluorouracil Inter eres with DNA synthesis


by blocking the methylation o
deoxyuridylic acid to thymidylic acid
(see Chapter 45 and Table 48 8)

Alkylating Agents Cyclophosphamide See Chapter 45 and Table 48 8


Mechlorethamine
Carmustine

Calcineurin Inhibitors Cyclosporine See Chapter 23 and Table 48 8


Tacrolimus
Pimecrolimus

(Continued)
708
Dermatological Pharmacology CHAPTER 4 8

DRUG CLASS DRUG MECHANISM OF ACTION


Other Immunosuppressive Mycophenolate mo etil Inhibits the enzyme inosine
and Anti inf ammatory monoposphatase dehydrogenase
Agents (IMPDH) thereby depleting
guanosine nucleotides essential
or DNA and RNA synthesis (see
Chapter 23 and Table 48 8)

Imiquimod Acts as a ligand at toll like receptors


in the innate immune system and
induces the cytokines inter eron
(IFN ), tumor necrosis actor
(TNF ) and IL 1, IL 6, IL 8, IL 10, and
IL 12 (see Table 48 8)

Vinblastine See Chapter 45 and Table 48 8


Bleomycin
Doxorubicin

Dapsone See Chapter 42 and Table 48 8

Thalidomide See Chapter 23 and Table 48 8

Biological Agents Ale acept Binds to CD2 on the sur ace o


(Table 48 3) T cells, thus blocking a necessary
costimulation step in T cell activation
(see Figures 48 1 and Table 48 3)

E alizumab Binds to CD11a on T cells and


prevents binding o leukocyte
unction associated antigen 1 (LFA 1)
to intercellular adhesion molecule
(ICAM) 1 on the sur ace o antigen
presenting cells, vascular endothelial
cells, and cells in the dermis and
epidermis, thereby inter ering with
T cell activation (see Figures 48 1
and 48 2, and Table 48 3)

Etanercept Mouse human chimera antibody


that binds soluble and membrane
bound TNF and inhibits its action
(see Chapter 23 and Table 48 3)

Inf iximab Mouse human chimera antibody


that binds soluble and membrane
bound TNF and inhibits its action
Induces complement dependent
and cell mediated lysis (see Chapter
23 and Table 48 3)

Adalimumab Human antibody that binds soluble


and membrane bound TNF and
inhibits its action
Induces complement dependent
and cell mediated lysis (see Chapter
23 and Table 48 3)

Agent Used or Treatment Denileukin Di titox Fusion protein composed o


o Cutaneous T Cell diphtheria toxin ragments A
Lymphoma and B and the receptor binding
portion o IL 2; once internalized
by endocytosis the active ragment
o diphtheria toxin is released into
the cytosol, where it inhibits protein
synthesis through ADP ribosylation
(see Chapter 46)
(Continued)
709
SECTION IX Special Systems Pharmacology

MECHANISMS OF DRUG CLASS DRUG MECHANISM OF ACTION


PERCUTANEOUS Sunscreens Various chemicals (see Table Absorb incident solar radiation in
ABSORPTION SUNSCREEN AGENTS) the ultraviolet B (UVB) or ultraviolet
A (UVA) ranges and physical agents
The process o absorption o a topicallyapplied that contain particulate materials
drug consists o : that can block or ref ect incident
Establishment o a concentration gradient energy and reduce its transmission
to the skin
Movement o drug into stratumcorneum
(partition coe cient) Drugs or Androgenic Minoxidil Minoxidil is bioactivated to orm
Alopecia minoxidil sul ate which relaxes
Dif usion o drug across layers o skin (di - arteriolar smooth muscle by
usion coe cient) activating ATP modulated K+
The relationship o these actors is summarized channels (KATP channels) (see
in the ollowing equation: Chapter 15)

Finasteride Inhibits the type II isozyme


J Cveh Km D/X
o 5 reductase, the enzyme
that converts testosterone to
where J is the rate o absorption; Cveh is the dihydrotestosterone (see Chapter 28)
concentration o drug in the vehicle; Km is
the partition coe cient; D is the dif usion Agents Used to Treat Hydroquinone Decrease melanocyte pigment
coe cient; and x is the thickness o the Hyperpigmentation Monobenzone production by inhibiting the
Azelaic conversion o dopa to melanin
stratumcorneum
Mequinol through inhibition o the enzyme
tyrosinase

Miscellaneous Agents Capsaicin Interacts with the transient receptor


potential vanilloid (TRPV1) receptor
on C ber sensory neurons
TRPV1 is a ligand gated nonselective
cation channel
Capsaicin rst stimulates then
desensitizes this channel to
noxious stimuli

La nge rha ns ce ll

EFALIZUMAB ALEFACEP T

ICAM-1 LF3A

CD80/CD86 MHC

LFA1CD2CD28 Ag
CD4
TCR
CD3

CD45RO +

T ce ll

FIGURE 48-1 Mechanisms o action o selected biological agents in psoriasis. Newer biological agents
can inter ere with one or more steps in the pathogenesis o psoriasis, resulting in clinical improvement.
See text in Chapter 65, Dermatological Pharmacology in Goodman &Gilmans The Pharmacological Basis
of Therapeutics, 12th Edition or details. ICAM 1, intercellular adhesion molecule 1; LFA, lymphocyte
unctionassociated antigen; MHC, major histocompatibility complex; TCR, T cell receptor.
710
Dermatological Pharmacology CHAPTER 4 8

Le s iona l ps oria tic s kin


Non-le s iona l s kin
Antige n AP C Re -a ctiva tion Cytokine s e cre tion Lymphocytic
Epide rmis infiltra te

Ke ra tinocyte
Cytokine a ctiva tion Epide rmis
s e cre tion
Migra tion Migra tion
through a ffe re nt Che mokine
lympha tics s e cre tion
Clona l expa ns ion

De rmis Antige n
pre s e nta tion
De rmis
CD45RO +
CD45RA+ CLA+
CD45RO +
CLA Lymph node CLA+ Blood

FIGURE 48-2 Immunopathogenesis o psoriasis. Psoriasis is a prototypical inf ammatory skin disorder in which speci c T cell populations are
stimulated by as yet unde ned antigen(s) presented by antigen presenting cells. The T cells release proinf ammatory cytokines, such as tumor
necrosis actor (TNF ) and inter eron (IFN ), that induce keratinocyte and endothelial cell proli eration. APC, antigen presenting cell; CLA,
cutaneous lymphocyte associated antigen.

CASE 48 1
A 70-year-old woman has developed an in ection rom scratching an area on her leg.
Despite her physicians recommendation or an oral antibiotic, she wishes to have a
cream to put on it.
a. What is the process o absorption o the antibiotic through the skin?
Passage through the outermost layer is the rate-limiting step or percutaneous
absorption. Figure 48-3 shows the structure and layers o the skin, and Figure 48-4
shows the compartments o skin as they relate to cutaneous drug delivery. T e
mechanisms o percutaneous absorption are shown in the Side Bar MECHANISMS
OF PERCU ANEOUS ABSORP ION.
b. What are the important considerations when applying a drug to the skin?
T e important considerations when applying a drug to the skin are shown in
able 48-1. In addition, the ollowing considerations should be taken into account:
dosage, under-application o a drug because o cost considerations o en occurs
when large amounts o skin are treated or a long time; regional anatomical
variation, drug penetration is higher on the ace, intertriginous areas, and
perineum due to stratum corneum thickness (see Figures 48-3 and 48-4). Skin sites
that are naturally occluded by apposing sur aces, such as the axillae, groin, and
in ramammary areas, are vulnerable to drug-related atrophy rom potent topical
glucocorticoids; altered barrier function in disease, in many dermatological diseases,
the stratum corneum is abnormal and barrier unction is compromised. In these
(Continued)

TABLE 48-1 Important Considerations When a Drug Is Applied to the Skin


What are the absorption pathways o intact and diseased skin?
How does the chemistry o the drug a ect the penetration?
How does the vehicle a ect the penetration?
How much o the drug penetrates the skin?
What are the intended pharmacological targets?
What host and genetic actors inf uence drug unction in the skin?

711
SECTION IX Special Systems Pharmacology

TABLE 48-2 Vehicles for Topically Applied Drugs


CREAM OINTMENT GEL/FOAM LOTION/SOLUTION/FOAM
Physical basis Oil in water emulsion Water in oil Water soluble emulsion Solution dissolved drug
base
Lotion suspended drug
Aerosol propellant with
drug
Foam drug with sur actant
as oaming agent and
propellant

Solubilizing medium >31% water (up to 80%) <25% water Contains water soluble May be aqueous or
polyethylene glycols alcoholic

Pharmacological Leaves concentrated drug Protective oil lm on skin Concentrates drug at


advantage at skin sur ace sur ace a ter evaporation

Advantages or patient Spreads and removes Spreads easily Nonstaining Low residue on scalp
easily Slows water evaporation Greaseless
No greasy eel Gives a cooling e ect Clear appearance

Locations on body Most locations Avoid intertriginous areas Foams well or scalp and Solutions and oams are
other hairy locations well accepted on scalp

Disadvantages Needs preservatives Greasy to very greasy Needs preservatives


Stains clothes High alcohol can be drying

Occlusion Low Moderate to high


Increases skin moisture

Composition issues Requires humectants Needs sur actants to Microspheres or


(glycerine, propylene prevent phase separation microsponges can be
glycol, polyethylene Hydrocarbon (VASELINE) ormulated in gels
glycols) to keep moist
when applied
Oil phase with long chain
alcohol or stability and
smooth eel
Has absorption bases
hydrophilic petrolatum

settings, increased percutaneous absorption o potent topical steroids can cause


systemic toxicity, such as hypothalamicpituitaryadrenal (HPA) axis suppression;
vehicle, drug vehicles are summarized in able 48-2 and below in the answer to
Case 1c; age, children have a greater ratio o sur ace area to mass than adults do,
so the same amount o topical drug can result in a greater systemic exposure;
application frequency, topical agents o en are applied twice daily. For certain drugs,
once-daily application o a larger dose may be equally e ective as more requent
applications o smaller doses. For some drugs, the stratum corneum may act as
a drug reservoir that allows gradual penetration into the viable skin layers over a
prolonged period; intralesional administration, intralesional drug administration is
used mainly or in ammatory lesions but also can be used or treatment o warts
and selected neoplasms. Medications injected intralesionally have the advantages o
direct contact with the underlying pathological process, no rst-pass metabolism,
and the opportunity or a slowly absorbed depot o drug.
c. What vehicles are available or topically applied drugs?
Vehicles or the topical administration o drugs are shown in able 48-2. Newer
vehicles include liposomes and microgel ormulations. Liposomes are concentric
(Continued)
712
Dermatological Pharmacology CHAPTER 4 8

La me lla r Tight S ca le de s qua ma tion


body junction

Cornifie d lipid e nve lope


SC
Lipid bilaye rs
Cornifie d ce ll e nve lope with
SG involucrin, foricrin, fla ggrin

Ke ra tohya lin gra nule

Cytoke ra tin K1/K10


SS

De s mos ome

Cytoke ra tin K5/K14


SB

FIGURE 48-3 Structure o the epidermis. The epidermis matures progressively rom the stratum basale (SB) to the stra
tum spinosum (SS), stratum granulosum (SG), and stratum corneum (SC). Important structural and metabolic proteins
are produced at speci ic layers o the epidermis. (Reproduced with permission from Wolff Ket al (eds). Fitzpatricks
Dermatology in General Medicine, 7th ed. McGraw-Hill, Inc., 2008. Figure 45-2.)

S hunt diffus ion


Pe rme a tion
Pe ne tra tion

Ve hicle
re s e rvoir
S tr. corne um Re s e rvior Ba rrie r
~20 mm function function

Binding
Via ble e pide rmis
~100 mm

Re s orption
Cuta ne ous
va s cula ture
Binding Lymph
De rmis ve s s e ls
~1200 mm

Pe rme a tion

Hypode rmis

FIGURE 48-4 Cutaneous drug delivery. Diagrammatic representation o the 3 compartments o the skin as they
relate to drug delivery: sur ace, stratum (Str.), and viable tissues. A ter application o drugs to the sur ace, evaporation,
structural, and compositional alterations, which determine the bioavailability o drugs, occur in the applied ormula
tion. The stratum corneum limits di usion o compounds into the viable skin and body. A ter absorption, compounds
either bind targets in viable tissues or di use within the viable tissue or into the cutaneous vasculature, where
they may be carried to internal cells and organs. (Reproduced with permission from Wolff Ket al (eds). Fitzpatricks
Dermatology in General Medicine, 7th ed. McGraw-Hill, Inc., 2008. Figure 215-1.)
713
SECTION IX Special Systems Pharmacology

spherical shells o phospholipids in an aqueous medium intended to enhance


percutaneous absorption in normal and abnormal stratum corneum. Variations in
size, charge, and lipid content can in uence liposome unction substantially.
rans ersomes are a drug-delivery technology based on highly de ormable, ultra-
exible lipid vesicles that penetrate the skin when applied nonocclusively. Microgels
are polymers intended to enhance solubilization o certain drugs, thereby enhancing
topical penetration and diminishing irritancy.
d. What concerns should the physician have when treating an in ection o the skin
with topical therapy?
Gram-positive organisms, including Staphylococcus aureus and Streptococcus pyogenes,
are the most common cause o pyoderma. Skin in ections with gram-negative bacilli
are rare, although they can occur in diabetics and patients who are immunosup-
pressed; appropriate parenteral antibiotic therapy is required or their treatment.
opical therapy requently is adequate or impetigo, a super cial bacterial in ection
o the skin caused by S. aureus and S. pyogenes. opical therapy o en is employed
or prophylaxis o super cial in ections caused by wounds and injuries.
Deeper bacterial in ections o the skin include olliculitis, erysipelas, cellulitis, and
necrotizing asciitis. Because streptococcal and staphylococcal species also are the
(Continued)

TABLE 48-3 Biological Agents Commonly Used in Dermatology


DRUG ALEFACEPT EFALIZUMAB ADALIMUMAB ETANERCEPT INFLIXIMAB
Structural class Receptor antibody Humanized Human monoclonal Receptor antibody Chimeric monoclonal
usion protein monoclonal antibody antibody usion protein antibody

Components LFA 3 and Fc IgG1 Complementarity IgG1 p75 TNF receptor and Variable region o
determining region Fc IgG1 mouse monoclonal
o mouse monoclonal antibody on
antibody on human IgG1
human IgG1

Binding site CD2 CD11a subunit o TNF TNF TNF


LFA 1

Method o IM SC SC SC IV
administration

Dosing or 15 mg weekly 12 0.7 mg/kg rst week, 80 mg loading dose, 50 mg twice weekly 3, 5 mg/kg at weeks 0, 2,
psoriasis weeks, stop 12 weeks, then 1 mg/kg weekly then 40 mg biweekly months then 50 mg and 6, then every 6 8
then repeat weekly weeks

FDA indications Moderate severe Moderate severe Moderate severe Moderate severe Severe psoriasis;
psoriasis psoriasis psoriasis; moderate psoriasis; moderate moderate severe
severe psoriatic arthritis; severe psoriatic arthritis; psoriatic arthritis;
adult and juvenile adult and juvenile adult rheumatoid
rheumatoid arthritis; rheumatoid arthritis; arthritis; ankylosing
ankylosing spondylitis; ankylosing spondylitis spondylitis; ulcerative
Crohns disease colitis; Crohns disease

Pregnancy B C B B B
category

E cacy in 28 33% 27 39% 53% 47% 76 80%


psoriasis a
a
Probability (%) o Psoriasis Area and Severity Index (PASI) score o 75 a ter 12 weeks o therapy at the dosing described in the table. LFA,
lymphocyte unctionassociated antigen; IgG, immunoglobulin G; TNF, tumor necrosis actor; IM, intramuscular; SC, subcutaneous; IV,
intravenous.
714
Dermatological Pharmacology CHAPTER 4 8

most common causes o deep cutaneous in ections, penicillins (especially -


lactamaseresistant -lactams) and cephalosporins are the systemic antibiotics
used most requently in their treatment (see Chapter 39). A growing concern
is the increased incidence o skin and so -tissue in ections with hospital- and
community-acquired methicillin-resistant S. aureus (MRSA) and drug-resistant
pneumococci. In ection with community-acquired MRSA o en is susceptible to
trimethoprimsul amethoxazole. See previous chapters or the pharmacology o
speci c antibacterial agents.

CASE 48 2
A 35-year-old man has the diagnosis o psoriasis. He is being treated with
methotrexate.
a. What is psoriasis?
Psoriasis is a disorder o T 1 cell-mediated immunity (see Figure 48-2), with the
epidermal changes being secondary to the e ect o released cytokines.
b. Why is methotrexate e cacious in the treatment o psoriasis?
T e antimetabolite methotrexate is a olic acid analog that competitively inhibits
dihydro olate reductase (see Chapter 45). Methotrexate has been used or moderate
to severe psoriasis since 1951. It suppresses immunocompetent cells in the skin, and
it also decreases the expression o cutaneous lymphocyte-associated antigen (CLA)
positive cells and endothelial cell E-selectin, which may account or its ef cacy.
c. What adverse e ects o methotrexate might limit its use?
Doses o methotrexate must be decreased or patients with impaired renal
clearance. Methotrexate should never be co-administered with trimethoprim
sul amethoxazole, probenecid, salicylates, or other drugs that can compete with
it or protein binding and thereby raise plasma concentrations to levels that may
result in bone marrow suppression. Fatalities have occurred because o concur-
rent treatment with methotrexate and nonsteroidal anti-in ammatory agents.
Methotrexate exerts signi cant antiproli erative e ects on the bone marrow; there-
ore, a complete blood count should be monitored serially. Physicians administer-
ing methotrexate should be amiliar with the use o olinic acid (leucovorin) to
rescue patients with hematological crises caused by methotrexate-induced bone
marrow suppression. Care ul monitoring o liver unction tests is necessary but
may not be adequate to identi y early hepatic brosis in patients receiving chronic
methotrexate therapy. Methotrexate-induced hepatic brosis may occur more com-
monly in patients with psoriasis than in those with rheumatoid arthritis. Conse-
quently, liver biopsy is recommended when the cumulative dose reaches 1 to 1.5 g.
A baseline liver biopsy also is recommended or patients with increased potential
risk or hepatic brosis, such as a history o alcohol abuse or in ection with hepa-
titis B or C. Patients with signi cantly abnormal liver unction tests, symptomatic
liver disease, or evidence o hepatic brosis should not use this drug. Many clini-
cians routinely administer olic acid along with methotrexate to ameliorate side
e ects; this does not reduce ef cacy o the methotrexate. Pregnancy and lactation
are absolute contraindications to methotrexate use.
d. What other therapeutic options are available or this patient?
Biological agents (see Chapter 23) are compounds derived rom living organisms
that target speci c mediators o immunological reactions. Classes o biologicals
include recombinant cytokines, interleukins, growth actors, antibodies, and usion
proteins. Currently, 5 biological agents are approved or the treatment o psoriasis
(see able 48-3). Biological therapies modi y the immune response in psoriasis
through (1) reduction o pathogenic cells, (2) inhibition o -cell activation,
(3) immune deviation ( rom a T 1 to a T 2 immune response), and (4) blockade o
the activity o in ammatory cytokines.
(Continued)
715
SECTION IX Special Systems Pharmacology

TABLE 48-4 Photochemotherapy Methods


PUVA PHOTODYNAMIC THERAPY PHOTOPHERESIS
Target Broad cutaneous area Focal cutaneous sites Peripheral blood leukocytes

Photosensitizing agent Methoxsalen (8 methoxypsoralen) Protoporphyrin IX Methoxsalen


Trioxsalen
(4,5,8 trimethylpsoralen)
Bergapten (5 methoxypsoralen)

Method o drug administration Oral Topical cream or solution o a To isolated plasma within
Topical lotion prodrug (aminolevulinic acid or photopheresis device
Bath water methylaminolevulinate)

FDA approved indications Psoriasis Actinic keratosis Cutaneous T cell lymphoma


Vitiligo

Activating wavelength UVA2 (320 340 nm) 417 nm and 630 nm UVA2 (320 340 nm)

Adverse ef ects (acute) Phototoxic reactions Phototoxic reactions Phototoxic reactions


Pruritus Temporary dyspigmentation GI disturbance
Hypertrichosis Hypotension
GI disturbance Congestive heart ailure
CNS disturbance
Bronchoconstriction
Hepatic toxicity
Herpes simplex recurrence
Retinal damage

Adverse ef ects (chronic) Photoaging Potential scarring Loss o venous access a ter
Nonmelanoma skin cancer repeated venipuncture
Melanomaa
Cataractsa

Pregnancy category C C FDA unrated


a
Controversial. CNS, central nervous system; GI, gastrointestinal; UVA, ultraviolet A.

T e appeal o biological agents in the treatment o psoriasis is that they speci cally
target the activities o lymphocytes and cytokines that mediate in ammation
versus traditional systemic therapies that are broadly immunosuppressive or
cytotoxic. hus, the use o these agents theoretically should result in ewer
toxicities and side e ects.
When evaluating the ef cacy o biological agents, it is important to understand the
standard measurement o ef cacy in psoriasis treatment, the Psoriasis Area and
Severity Index (PASI). T e PASI quanti es the extent and severity o skin involve-
ment in di erent body regions as a score rom 0 (no lesions) to 72 (severe disease).
o gain FDA approval or the treatment o psoriasis, a biological agent must decrease
the PASI by 75%. Although such quanti cation is an essential element in controlled
clinical trials, many patients in practice may gain clinically signi cant bene t rom
biological treatment without achieving this degree o PASI improvement.

CASE 48 3
Prior to his treatment with methotrexate, the patient in Case 48-2 was treated with
photochemotherapy.
a. What is photochemotherapy?
Phototherapy and photochemotherapy are treatment methods in which UV or
visible radiation is used to induce a therapeutic response either alone or in the
presence o a photosensitizing drug. o be e ective, the incident radiation must
(Continued)
716
Dermatological Pharmacology CHAPTER 4 8

be absorbed by a target or chromophore in the skinwhich in phototherapy is


ANTIBIOTICS USED TO TREAT ACNE
endogenous and in photochemotherapy must be administered exogenously
(see able 48-4). TOPICAL Clindamycin
T e action spectrum or oral psoralen (eg, methoxysalen) ollowed by ultraviolet light Erythromycin
A (PUVA) is between 320 and 340 nm. wo distinct photoreactions take place. ype I Benzoyl peroxide antibiotic
combination
reactions involve the oxygen-independent photoaddition o psoralens to pyrimidine Sul acetamide
bases in DNA. ype II reactions are oxygen-dependent and involve the trans er o Sul acetamide/sul ur
energy to molecular oxygen, creating reactive oxygen species. T rough incompletely combination
understood mechanisms, these phototoxic reactions stimulate melanocytes and Metronidazole
induce antiproli erative, immunosuppressive, and anti-in ammatory e ects. Azelaic acid

b. What are the potential toxicities o phototherapy and photochemotherapy and SYSTEMIC Tetracycline
how should patients be monitored? Doxycycline
Minocycline
Patients treated with these modalities should be monitored or concomitant use o Trimethoprim
other potential photosensitizing medications be ore initiation o therapy. Such drugs sul amethoxazole
include phenothiazines, thiazides, sul onamides, nonsteroidal anti-in ammatory
agents, sul onylureas, tetracyclines, and benzodiazepines.
T e major side e ects o PUVA are listed in able 48-4. Phototoxicity is character-
ized by erythema, edema, blistering, and pruritus. Ocular toxicity can be prevented
by wearing UVA-blocking glasses the day o treatment. T e risk o nonmelanoma
skin cancer is dose-dependent, with the greatest risk in those receiving more than
250 treatments. A possible association o melanoma and extensive exposure to
PUVA has been reported; however, several other studies have ailed to con rm this
association. As skin cancer may not develop or decades a er exposure, annual skin
examinations should be continued or years a er completion o PUVA.

CASE 48 4
A 17-year-old boy with nonin ammatory acne is treated with a topical retinoid
preparation.
a. What is acne?
Acne is believed to result rom a combination o sebaceous gland hyperplasia, ol-
licular hyperkeratosis, Propionibacterium acnes colonization, and in ammation.
T ough incompletely understood mechanisms, topical retinoids correct abnormal
ollicular keratinization, reduce P. acnes counts, and reduce in ammation, thereby
making them the cornerstone o acne therapy. opical retinoids are rst-line agents
or nonin ammatory (comedonal) acne and o en are combined with other agents
in the management o in ammatory acne.
b. What is the mechanism o action o retinoids?
Retinoids exert their e ects on gene expression by activating 2 amilies o receptors
retinoic acid receptors (RARs) and retinoid X receptors (RXRs)that are members
o the steroid receptor super amily.
Unique therapeutic e ects can be produced by targeting speci c retinoid receptors.
For example, retinoids that target RARs predominantly a ect cellular di erentia-
tion and proli eration, whereas retinoids that target RXRs predominantly induce
apoptosis. Hence, tretinoin, adapalene, and tazarotene, which target RARs, are used
in acne, psoriasis, and photoaging (disorders o di erentiation and proli eration),
whereas bexarotene and alitretinoin, which target RXRs, are used in mycosis un-
goides and Kaposi sarcoma (to induce apoptosis o malignant cells).
c. What retinoid preparations are available or this patient?
opical and systemic retinoids are shown in ables 65-4 and 65-5, respectively, in
Goodman and Gilmans T e Pharmacological Basis of T erapeutics, 12th Edition.

(Continued)

717
SECTION IX Special Systems Pharmacology

TABLE 48-5 Recommended Cutaneous Antifungal Therapy


CONDITION TOPICAL THERAPY ORAL THERAPY

Tinea corporis, localized Azoles, allylamines

Tinea corporis, widespread Griseo ulvin,terbina ne,itraconazole,f


uconazole

Tinea pedis Azoles, allylamines Griseo ulvin, terbina ne, itraconazole,


f uconazole

Onychomycosis Griseo ulvin, terbina ne, itraconazole,


f uconazole

Candidiasis, localized Azoles

Candidiasis, widespread and Ketoconazole, itraconazole,


mucocutaneous f uconazole

Tinea versicolor, localized Azoles, allylamines

Tinea versicolor, widespread Ketoconazole, itraconazole,


f uconazole

d. What adverse e ects might this patient experience rom retinoids?


Acute retinoid toxicity is similar to vitamin A intoxication. Side e ects o systemic
retinoids include dry skin, nosebleeds rom dry mucous membranes, conjunctivi-
tis, reduced night vision, hair loss, alterations in serum lipids and transaminases,
hypothyroidism, in ammatory bowel disease are, musculoskeletal pain, pseudo-
tumor cerebri, and mood alterations. RAR-selective retinoids are more associated
with mucocutaneous and musculoskeletal symptoms, whereas RXR-selective reti-
noids induce more physiochemical changes. Suicide or suicide attempts have been
associated with the use o isotretinoin. T us, all patients treated with isotretinoin
should be observed closely or symptoms o depression or suicidal thoughts.
Adverse e ects o all topical retinoids include erythema, desquamation, burning,
and stinging. T ese e ects o en decrease with time and are lessened by concomi-
tant use o emollients. Patients also may experience photosensitivity reactions
because o enhanced reactivity to UV radiation and have a signi cant risk or
severe sunburn. See Summary Quiz Question 48-3 or a discussion o the avoid-
ance o retinoids during pregnancy.
e. What other drugs are available or the treatment o acne?
Antibiotics are commonly used to treat acne. Commonly used antibiotics are shown
in the Side Bar AN IBIO ICS USED O REA ACNE.

CASE 48 5
A 53-year-old woman has widespread tinea corporis in ection in her skin that is causing
irritation and itching. Her physician has prescribed oral uconazole.
a. What preparations are available or the treatment o cutaneous ungal in ections?
able 48-5 shows the recommendations or cutaneous anti ungal therapy.
(Continued)

718
Dermatological Pharmacology CHAPTER 4 8

b. What are the adverse e ects o f uconazole?


T e azoles interact with hepatic CYPs as substrates and inhibitors, providing myriad
possibilities or the interaction o azoles with many other medications. See Chapter 43
or a complete discussion o the drug interactions with azole anti ungal drugs.
Side e ects in patients receiving more than 7 days o uconazole, regardless o
dose, include the ollowing: nausea, headache, skin rash, vomiting, abdominal pain,
and diarrhea. Use o high doses may be limited by nausea. Reversible alopecia may
occur with prolonged therapy at 400 mg daily. Rare cases o deaths due to hepatic
ailure or Stevens-Johnson syndrome have been reported. Fluconazole is terato-
genic in rodents and has been associated with skeletal and cardiac de ormities in at
least 3 in ants born to 2 women taking high doses during pregnancy. Fluconazole
is a Category C agent that should be avoided during pregnancy unless the potential
bene t justi es the possible risk to the etus.

SUNSCREEN AGENTS
ULTRAVIOLET A (UVA) SUNSCREEN AGENTS ULTRAVIOLET B (UVB) SUNSCREEN AGENTS
Avobenzone (Parsol 1789) p Aminobenzoic acid (PABA) esters (padimate O)
Oxybenzone Cinnamates (octinoxate)
Titanium dioxide Octocrylene
Zinc oxide Salicylates (octisalate)
Ecamsule (MEXORYL SX)

CASE 48 6
A 23-year-old woman is planning a beach vacation. She asks you or advice on
sunscreen agents.
a. What are sunscreens?
Photoprotection rom the acute and chronic e ects o sun exposure is readily avail-
able with sunscreens. T e major active ingredients o available sunscreens include
chemical agents that absorb incident solar radiation in the UVB and/or UVA ranges
and physical agents that contain particulate materials that can block or re ect inci-
dent energy and reduce its transmission to the skin (see the able SUNSCREEN
AGEN S). Many o the sunscreens available are mixtures o organic chemical
absorbers and particulate physical substances. Ideal sunscreens provide a broad
spectrum o protection and are ormulations that are photostable and remain intact
or sustained periods on the skin. T ey also should be nonirritating, invisible, and
nonstaining to clothing. No single sunscreen ingredient possesses all these desir-
able properties, but many are quite e ective nonetheless.
T e major measurement o sunscreen photoprotection is the sun protection actor
(SPF), which de nes a ratio o the minimal dose o incident sunlight that will produce
erythema or redness (sunburn) on skin with the sunscreen in place (protected) and
the dose that evokes the same reaction on skin without the sunscreen (unprotected).
T e SPF provides valuable in ormation regarding UVB protection but is useless in
documenting UVA ef cacy. In 2007, the FDA proposed a consumer- riendly rating
system or UVA products consisting o 1 to 4 stars representing low, medium, high, and
highest UVA protection available in an O C sunscreen product as an indicator o the
products ability to prevent tanning. T e test proposed to determine UVA rating is anal-
ogous to the SPF test used to determine the e ectiveness o UVB sunscreen products.
Except or total sun avoidance, sunscreens are the best single method o protection
rom UV-induced damage to the skin.

719
SECTION IX Special Systems Pharmacology

TABLE 48-6 Agents Used for the Treatment of Pruritus


Pruritoce ptive Pruritus: Itch originating in the skin due to inf ammation or other
cutaneous disease
EmollientsRepair o barrier unction
Coolants (menthol, camphor, calamine)Counter irritants
CapsaicinCounter irritant
AntihistaminesInhibit histamine induced pruritus
Topical steroidsDirect anti pruritic and anti inf ammatory e ects
Topical immunomodulatorsAnti inf ammatories
PhototherapyReduced mast cell reactivity and anti inf ammatory e ects
ThalidomideAnti inf ammatory through suppression o excessive tumor necrosis actor
Neuropathic Pruritus: Itch due to disease o a erent nerves
CarbamazepineBlockade o synaptic transmission and use dependent sodium channels
GabapentinSuppresses neuronal hyperexcitability by inhibiting voltage dependent
calcium channels
Topical anesthetics (EMLA, benzocaine, pramoxine)Inhibit nerve conduction via decreased
nerve membrane permeability to sodium
Neurogenic Pruritus: Itch that arises rom the nervous system without evidence o neural
pathology
ThalidomideCentral depressant
Opioid receptor antagonists (naloxone, naltrexone)Decrease opioidergic tone
Tricyclic antidepressantsDecrease pruritus signaling through alteration in neurotransmitter
concentrations
Selective serotonin reuptake inhibitors (SSRIs)Decrease pruritus signaling through
alteration in neurotransmitter concentrations
Psychogenic Pruritus: Itch due to psychological illness
Anxiolytics (alprazolam, clonazepam, benzodiazepines)Relieve stress reactive pruritus
Antipsychotic agents (chlorpromazine, thioridazine, thiothixene, olanzapine)Relieve
pruritus with impulsive qualities
Tricyclic antidepressantsRelieve depression and insomnia related to pruritus
SSRIsRelieve pruritus with compulsive qualities

CASE 48 7
A 65-year-old woman has a complaint o itching on her arms and legs. Physical exami-
nation shows that she has broken skin on her legs rom scratching.
a. What are the common causes o itching?
T e term pruritus is derived rom the Latin prurire, which means to itch. Pruritus
is a symptom unique to skin that occurs in a multitude o dermatological disorders,
including dry skin or xerosis, atopic eczema, urticaria, and in estations. Itching also
may be a sign o internal disorders, including malignant neoplasms, chronic renal
ailure, and hepatobiliary disease. In addition to treating the underlying disorder,
a general approach to the treatment o pruritus can be made by classi ying pruritus
into one o our clinical categories (see able 48-6).
b. What agents are available or itching when there is no obvious cause?
Agents use ul in treating pruritus are listed in able 48-6.

KEY CONCEPTS
Drugs can be applied to the skin to directly treat disorders o the skin or to
deliver drugs to other tissues.
E ective and sa e use o topical agents requires appreciation o the physical and
physiological variables that in uence the interactions o drugs and the skin,
impacting absorption and transport.
(Continued)
720
Dermatological Pharmacology CHAPTER 4 8

T ere is a superf cial capillary plexus between the epidermis and dermis that
is the site o the majority o the systemic absorption o cutaneous drugs (see
Figure 48-4).
Passage through the outermost layer o skin (stratum corneum) is the rate-limiting
step or percutaneous absorption.
T e unique therapeutic e ects o retinoids can be produced by targeting specif c
retinoid receptors (RXR and RAR).
Except or total sun avoidance, sunscreens are the best single method o
protection rom UV-induced damage to the skin.
Cytotoxic and immunosuppressive drugs are used in dermatology or immuno-
logically mediated diseases such as psoriasis, autoimmune blistering diseases,
and leukocytoclastic vasculitis (see able 48-8).
Biological therapies modi y the immune response in patients with
psoriasis through (1) reduction o pathogenic cells, (2) inhibition o -cell
activation, (3) immune deviation ( rom a T 1 to a T 2 immune response),
and (4) blockade o the activity o in ammatory cytokines (see able 48-3 and
Figures 48-2 and 48-1).

SUMMARY QUIZ

QUESTION 48-1 T e rate-limiting step in the absorption o drugs through the skin
is passage through the
a. dermis.
b. epidermis.
c. stratum corneum.
d. hypodermis.
e. sweat glands.

QUESTION 48-2 A 35-year-old man has a ungal in ection o his toenails. A systemic
anti ungal preparation, griseo ulvin, is prescribed because
a. topical anti ungal preparations are irritating.
b. topical anti ungal agents are expensive.
c. systemic anti ungal agents produce high drug concentrations in the nails.
d. systemic anti ungal agents have no adverse e ects.
e. topical anti ungal preparation are easily washed o the nails.

QUESTION 48-3 A 23-year-old woman with recalcitrant acne is being treated with an
oral dose o isotretinoin. She should be warned against
a. getting pregnant.
b. sun exposure.
c. close contact with her 1-year-old child.
d. ying in an airplane.
e. eating broccoli.

QUESTION 48-4 A 53-year-old woman with psoriasis is treated with methotrexate.


Aside rom monitoring her blood count or evidence o bone marrow suppression,
she should also have care ul monitoring o her
a. kidney unction.
b. liver unction.
c. sexual unction.
(Continued)
721
SECTION IX Special Systems Pharmacology

d. cognitive unction.
e. visual unction.

QUESTION 48-5 A 76-year-old man has severe eczema on his legs or which a potent
glucocorticoid ointment is prescribed. He should be warned not to use an occlusive
dressing as it may result in
a. blistering.
b. decreased kidney unction.
c. behavioral changes.
d. suppression o the hypothalamic-pituitary-adrenal axis.
e. breakdown o the glucocorticoid.

QUESTION 48-6 A 43-year-old woman with severe psoriasis is prescribed adalimumab.


Adalimumab is an inhibitor o
a. -cell activation.
b. IL-2.
c. CYP2D6.
d. adrenergic receptors.
e. NF-.

SUMMARY QUIZ ANSWER KEY


QUESTION 48-1 Answer is c. Passage through the outermost layer (stratum corneum)
is the rate-limiting step or percutaneous absorption. Permeability generally is inversely
proportional to the thickness o the stratum corneum. Drug penetration is higher on
the ace, intertriginous areas, and perineum due to minimal stratum corneum thick-
ness in these regions.
QUESTION 48-2 Answer is c. Systemic therapy is necessary or e ective management
o onychomycosis (see Chapter 43). Griseo ulvin is deposited in keratin precursor cells;
when these cells di erentiate, the drug is tightly bound to, and persists in, keratin, pro-
viding prolonged resistance to ungal invasion. For this reason, the new growth o hair
or nails is the f rst to become ree o disease. As the ungus-containing keratin is shed,
it is replaced by normal tissue.
QUESTION 48-3 Answer is a. Isotretinoin is approved or the treatment o recalcitrant
and nodular acne vulgaris. Systemic retinoids are highly teratogenic. T ere is no sa e
dose during pregnancy. Common mal ormations include cranio acial, cardiovascular,
thymic, and central nervous system (CNS) abnormalities. Although there appears to be
minimal, i any, risk o retinoid embryopathy in etuses conceived by males taking sys-
temic retinoids, it is commonly recommended that men avoid retinoid therapy when
actively trying to ather children.
QUESTION 48-4 Answer is b. Care ul monitoring o liver unction tests is necessary
but may not be adequate to identi y early hepatic f brosis in patients receiving chronic
methotrexate therapy. Methotrexate-induced hepatic f brosis may occur more com-
monly in patients with psoriasis than in those with rheumatoid arthritis. Consequently,
liver biopsy is recommended when the cumulative dose reaches 1 to 1.5 g. A baseline
liver biopsy also is recommended or patients with increased potential risk or hepatic
f brosis, such as a history o alcohol abuse or in ection with hepatitis B or C. Patients
with signif cantly abnormal liver unction tests, symptomatic liver disease, or evidence
o hepatic f brosis should not use this drug.
(Continued)

722
Dermatological Pharmacology CHAPTER 4 8

TABLE 48-7 Potency of Selected Topical Glucocorticoids


CLASS OF DRUGa GENERIC NAME, FORMULATION TRADE NAME
1 Betamethasone dipropionate cream, ointment 0.05% (in optimized vehicle) DIPROLENE
Clobetasol propionate cream, ointment 0.05% TEMOVATE
Dif orasone diacetate, ointment 0.05% PSORCON
Halobetasol propionate, ointment 0.05% ULTRAVATE

2 Amcinonide, ointment 0.1% CYCLOCORT


Betamethasone dipropionate, ointment 0.05% DIPROSONE, others
Desoximetasone, cream, ointment 0.25%, gel 0.05% TOPICORT
Dif orasone diacetate, ointment 0.05% FLORONE, MAXIFLOR
Fluocinonide, cream, ointment, gel 0.05% LIDEX, LIDEX-E, FLUONEX
Halcinonide, cream, ointment 0.1% HALOG, HALOG-E

3 Betamethasone dipropionate, cream 0.05% DIPROSONE, others


Betamethasone valerate, ointment 0.1% BETATREX, others
Dif orasone diacetate, cream 0.05% FLORONE, MAXIFLOR
Triamcinolone acetonide, ointment 0.1%, cream 0.5% ARISTOCORT A, others

4 Amcinonide, cream 0.1% CYCLOCORT


Desoximetasone, cream 0.05% TOPICORT LP
Fluocinolone acetonide, cream 0.2% SYNALAR-HP
Fluocinolone acetonide, ointment 0.025% SYNALAR
Flurandrenolide, ointment 0.05%, tape 4 g/cm 2 CORDRAN
Hydrocortisone valerate, ointment 0.2% WESTCORT
Triamcinolone acetonide, ointment 0.1% KENALOG, ARISTOCORT
Mometasone uroate, cream, ointment 0.1% ELOCON

5 Betamethasone dipropionate, lotion 0.05% DIPROSONE, others


Betamethasone valerate, cream, lotion 0.1% BETATREX, others
Fluocinolone acetonide, cream 0.025% SYNALAR
Flurandrenolide, cream 0.05% CORDRAN SP
Hydrocortisone butyrate, cream 0.1% LOCOID
Hydrocortisone valerate, cream 0.2% WESTCORT
Triamcinolone acetonide, cream, lotion 0.1% KENALOG
Triamcinolone acetonide, cream 0.025% ARISTOCORT

6 Alclometasone dipropionate, cream, ointment 0.05% ACLOVATE


Desonide, cream 0.05% TRIDESILON, DESOWEN
Fluocinolone acetonide, cream, solution 0.01% SYNALAR

7 Dexamethasone sodium phosphate, cream 0.1% DECADRON


Hydrocortisone, cream, ointment, lotion 0.5%, 1.0%, 2.5% HYTONE, NUTRICORT, PENECORT
a
Class 1 is most potent; class 7 is least potent.

QUESTION 48 -5 Answer is d. Chronic use o class 1 (see able 48-7) topical


glucocorticoids can cause skin atrophy, striae, telangiectasias, purpura, and acnei orm
eruptions. Occlusion increases the risk o HPA suppression (see Chapter 29).
QUESTION 48-6 Answer is e. Adalimumab is a human IgG1 monoclonal
antibody that binds soluble and membrane-bound NF-. Like in iximab,
it can mediate complement-induced cytolysis on cells expressing NF. Unlike
in iximab, however, it is ully human, which reduces the risk or development o
neutralizing antibodies.

723
SECTION IX Special Systems Pharmacology

TABLE 48-8 Mechanism of Action for Selected Cytotoxic and Immunosuppressive


Drugs
Methotrexate Dihydro olate reductase inhibitor

Azathioprine Purine synthesis inhibitor

Fluorouracil Blocks methylation in DNA synthesis

Cyclophosphamide Alkylates and cross links DNA

Mechlorethamine hydrochloride Alkylating agent

Carmustine Cross links in DNA and RNA

Cyclosporine Calcineurin inhibitor

Tacrolimus Calcineurin inhibitor

Pimecrolimus Calcineurin inhibitor

Mycophenolate mo etil Inosine monophosphate dehydrogenase


inhibitor

Imiquimod Inter eron induction

Vinblastine Inhibits microtubule ormation

Bleomycin Induction o DNA strand breaks

Dapsone Inhibits neutrophil migration, oxidative burst

Thalidomide Cytokine modulation

SUMMARY: DRUGS USED TO TREAT DERMATOLOGIC DISORDERS


TOXICITIES
UNIQUE; CLINICALLY
CLASS AND SUBCLASSES NAMES CLINICAL USES COMMON IMPORTANT
Glucocorticoids Triamcinolone acetonide Anti inf ammatory and See Chapter 29 See Chapter 29
Triamcinolone immunosuppression (see
hexacetonide Table 48 7)
Numerous other agents
(see Table 48 7)

Retinoids Tretinoin Tretinoin, adapalene, and Topical retinoids Systemic retinoids are
Isotretinoin tazarotene, which target may cause erythema highly teratogenic and their
Acitretin RARs, are used to treat acne, desquamation, use is contraindicated in
Tazarotene psoriasis, and photoaging burning, stinging, and women who are pregnant,
Bexarotene Bexarotene and alitretinoin, photosensitivity contemplating pregnancy,
Adapalene which target RXRs are used Serum lipid elevation is or breast eeding
Alitretinoin to treat mycosis ungoides a common laboratory The use o topical retinoids
and Kapposi sarcoma abnormality o systemic should be avoided during
Acitretin Is used to treat the retinoids pregnancy
cutaneous mani estations o Systemic retinoids may
psoriasis also cause cheilitis, xerosis,
Isotretinoin is approved or cutaneous photosensitivity,
the treatment o recalcitrant photophobia, myalgia,
and nodular acne vulgaris nail ragility, and
increased susceptibility to
staphylococcal in ections

Vitamin Analogs Calcipotriene Vitamin D analog used in the Perilesional irritation and Hypercalcemia and
treatment o psoriasis mild photosensitivity hypercalcuria with high
doses
(Continued)
724
Dermatological Pharmacology CHAPTER 4 8

TOXICITIES
UNIQUE; CLINICALLY
CLASS AND SUBCLASSES NAMES CLINICAL USES COMMON IMPORTANT
Photochemotherapeutic PUVA Approved or the treatment Erythema, blistering, and See Table 48 4
Drugs Methoxsalen o vitiligo and psoriasis pruritus (see Table 48 4)

Photodynamic therapy Approved or use to treat


Aminolevulinic acid actinic keratosis
Methyl aminolevulinate Commonly used to treat
thin nonmelanoma skin
cancers, acne, and photo
damaged skin

Antimicrobial Agents Various antibacterial, Treatment o cutaneous See Chapters 38, 39, 40, See Chapters 38, 39, 40, 41,
anti ungal, and antiviral in ections (see Chapters 38, 41, 43, and 44 43, and 44
agents 39, 40, 41, 43, and 44) (See
Table 48 5 or anti ungal
agents

Retapamulin Approved or topical


treatment o impetigo

Agents Used to Treat Permethrin Treatment o scabies and Irritation and burning
In estations lice in estations in in ants 2
months o age

Malathion Treatment o lice in estation See Chapter 6 See Chapter 6


in children 6 years o age

Ivermectin Oral anthelmintic approved See Chapter 37 See Chapter 37


or the treatment o
onchocerciasis and
stronglyoidiasis but used o
label or treatment o scabies
and lice

Lindane Treatment o lice in estation Neurotoxicity


Contraindicated in
premature in ants and
in patients with seizure
disorders

Crotamiton Used to treat scabies


and lice when lindane
and permethrin are
contraindicated

Benzyl alcohol Approved or


treatment o lice

Antimalarial Agents Chloroquine Used in the treatment See Chapter 35 See Chapter 35
Hydroxychloroquine o cutaneous lupus
Quinacrine erythematosus, cutaneous
dermatomyositis, porphyria
cutanea tarda, and
sarcoidosis

Antimetabolites Methotrexate Used to treat psoriasis See Chapter 45 See Chapter 45

Azathioprine Used o label as a See Chapter 23 See Chapter 23


steroid sparing agent
or autoimmune and
inf ammatory dermatoses

Fluorouracil Used to treat actinic keratosis, See Chapter 45 See Chapter 45


actinic cheilitis, and super cial
basal cell carcinomas
(Continued)
725
SECTION IX Special Systems Pharmacology

TOXICITIES
UNIQUE; CLINICALLY
CLASS AND SUBCLASSES NAMES CLINICAL USES COMMON IMPORTANT
Alkylating Agents Cyclophosphamide Used or the treatment o See Chapter 45 See Chapter 45
Mechlorethamine cutaneous T cell lymphoma
Carmustine

Calcineurin Inhibitors Cyclosporine Cyclosporin is approved or See Chapter 23 See Chapter 23


Tacrolimus the treatment o psoriasis
Pimecrolimus Tacrolimus and
pimecrolimus are approved
or the treatment o atopic
dermatitis in patients 2
years o age

Other Immunosuppressive Mycophenolate mo etil Used to treat inf ammatory See Chapter 23 See Chapter 23
and Anti inf ammatory and autoimmune diseases in
Agents dermatology

Imiquimod Approved or the treatment Irritant reactions Minor f u like symptoms


o genital warts and actinic including edema, vesicles,
keratoses erosions, or ulcers

Vinblastine Vinblastine and doxorubicin See Chapter 45 See Chapter 45


Bleomycin are used to treat Kaposi
Doxorubicin sarcoma
Vinblastine is used to treat
T cell lymphoma
Intralesional bleomycin is
used or recalcitrant warts

Dapsone Approved or use in See Chapter 42 See Chapter 42


dermatitis herpeti ormis and
leprosy

Thalidomide Approved or treatment See Chapter 23 See Chapter 23


o erythema nodosum
leprosum

Biological Agents and Ale acept Treatment o psoriasis See Table 48 3 and See Table 48 3 and
Tumor Necrosis Factor E alizumab Inf iximab is a mouse human Chapter 23 Chapter 23
(TNF) Inhibitors Etanercept chimeric IgG1 monoclonal
Inf iximab antibody
Adalimumab Etanercept is a ully human
TNF receptor usion protein
Adalimumab is a human
IgG1 monoclonal antibody

Agent Used or Treatment Denileukin di tox Used to treat advanced Pain, ever, chills, nausea, Hypersensitivity reaction
o Cutaneous T Cell cutaneous T cell lymphoma vomiting, and diarrhea (hypotension, back pain,
Lymphoma (see Chapter 46) dyspnea, and chest pain)
Capillary leak syndrome
(edema, hypoalbuminemia,
and hypotension)
(see Chapter 46)

Sunscreens Various chemicals (see Used to provide protection Local irritation and skin
Table SUNSCREEN AGENTS) against ultraviolet radiation sensitivity

Drugs or Androgenic Minoxidil Used to treat hair loss Allergic and contact See Chapter 15
Alopecia dermatitis

Finasteride Used to treat hair loss Decreased libido, Pregnant women should
erectile dys unction, and not be exposed to
ejaculation disorder nasteride
See Chapter 28
(Continued)
726
Dermatological Pharmacology CHAPTER 4 8

TOXICITIES
UNIQUE; CLINICALLY
CLASS AND SUBCLASSES NAMES CLINICAL USES COMMON IMPORTANT
Agents Used to Treat Hydroquinone Used to treat hormonally or Dermatitis and ochronosis Monobenzone may
Hyperpigmentation Monobenzone light induced pigmentation cause permanent
Azelaic within the epidermis hypopigmentation
Mequinol

Miscellaneous Agents Capsaicin Approved or the treatment


o minor aches and pains
associated with backpain,
strains, and arthritis
Used o label or
postherpetic neuralgia and
pain ul diabetic neuralgia

Podophyllin Treatment o anogenital Irritation and ulcerative Should not be used during
warts local reactions pregnancy

727
PREFACE
INDEX

Page numbers o owed by , t, or b indicate gures, tab es, or boxes respective y.

A somatic neuromuscu ar junction, 82 , dependence, physica , 264, 266


Abacavir, 601t, 603t, 607 , 608 , 621t 93, 94 MDMA, 265, 266
adverse e ects, 615 synthesis enhancer, dexpantheno , mu tip e simu taneous variab es, 255, 255t
mechanisms o action and resistance, 498t, 507t negative rein orcement, 265, 266
603t, 604 , 607 , 608 , 614615 Acety cho ine receptor (AChR), 97 nicotine, 258259, 259t
structure, 607 Acety cho ine receptor (AChR) agonists, opioids, 259262, 260 , 261t
Abare ix, 665t, 687t 106, 117t oxycodone, 254256, 255 , 264, 266
Abatacept, 368t, 381t, 387t Acety cho inesterase (AChE), 97 , 104 to erance, 255, 256t, 264, 265
ABCB1 transporter, 23t, 24t, 25t, 38 Acety cho inesterase (AChE) Addisons disease, 461 , 465
ABCC1 transporter, 23t inhibitors, 100t Additive drugdrug interaction, 48b
ABCC2 transporter, 23t Acety cho inesterase (AChE) inhibitors, Ade ovir dipivoxi , 600t, 602t, 620t
ABCC3 transporter, 24t reversib e, 114t115t Adenosine
ABCC4 transporter, 24t Acety cysteine, 40t, 42t, 54, 65t antiarrhythmic, 312t, 317t, 318, 322t
ABCC5 transporter, 24t Acety sa icy ic acid. See Aspirin e ectrophysio ogic actions, 317t
ABCC6 transporter, 24t Acid urine, 20b, 2122, 39 paroxysma ventricu ar tachycardia,
ABCDE emergency care, 51, 52t Acitretin, 707t, 724t 314t, 318
ABC emergency care, 63, 64 Acne, 717718 Adenosine receptor agonists, antiarrhythmic,
ABCG2 transporter, 24t retinoids, 707t, 717718, 717b 312t, 322t
ABCG5 transporter, 25t Acrivastine, 347t, 355t adenosine, 312t, 317t, 318, 322t
ABCG8 transporter, 25t Acromega y, 423424, 424 Adies pupi , 698t
Abciximab, 323t, 324325, 331332, 332t Actinomycin D, 630 , 643t, 658t Administration, repeated, 38b, 38
ABC transporters, 23t, 23t25t Action potentia , 90 Administration, routes, 20b, 21t. See also
ABDE treatment, 52t cardiac, 312 , 313 speci c drugs and routes
Abiraterone, 665t, 668 , 680, 687t Activated comp ex (LR*), 6b ocu ar, 697t
Abobotu inumtoxin A, Activated protein C, 333t skin, 710b, 711715, 711t, 713 (See also
ophtha mic, 692t, 706t drotrecogin a a, 323t, 333t Skin absorption)
Absorption, 20b, 20 . See also speci c drugs mechanisms o action, 323t Administrations, routes, 20b, 21t.
and drug classes Active ingredients, 3. See also speci c drugs See also speci c drugs and
chi dren, 67b Active transport, 19b, 19 drug classes
drugdrug interactions, 48t drugs requiring, 20b inha ed, e der y, 74t
e der y, 73b, 73 , 73t74t membrane transporters, 23b intramuscu ar, 21t
poisoning, 52 Acute coronary syndrome, 290t, 291 . See e der y, 74t
ABVD, 630 , 643t, 645647 also Myocardia ischemia neonates, in ants, and
Acamprosate, 177t, 186187, 186t, drugs, 296297 chi dren, 67b
193t, 257 percutaneous coronary interventions, 297 intranasa , e der y, 74t
Acarbose, 471t, 479t Acute myocardia in arction, 290t. See also intravenous, 21t
Acceptors, 2b Myocardia ischemia ocu ar, 697t
Acebuto o , 126t, 128t, 146t Acyc ovir, 600t, 605608, 619t ora , 21t
Acetaminophen, 2829, 5054, 50 adverse e ects, 608 e der y, 73t
actions, 371, 372 antiherpes, 600t601t, 604 605 , in ant bioavai abi ity, 72, 74
drugdrug interactions, 48 605608, 605t, 617618, 619t recta , e der y, 73t
metabo ism, 2829, 50 , 207, 208 CMV retinitis, 617, 618 subcutaneous, 21t
oxycodone-acetaminophen, 207, 208 mechanisms o action and resistance, e der y, 74t
pain target and site o action, 205t 601t, 604 606 , 606607 sub ingua , 22
Acetaminophen toxicity, 2829, 5054, Ada imumab e der y, 73t
52t, 207, 208 Crohns disease, 381 topica (derma ), 710b, 711715, 711t, 713
with a coho , 366 dermato ogic, 709t, 714t, 722, important considerations,
antidote, 42t 723, 726t 711712, 711t
diagnosis, 54 immunotherapy, 368t, 376t, 381, on pharmacokinetics, 20b, 21t
metabo ism and mechanisms, 50 , 5354, 383, 384, 386t rate- imiting step, 721, 722
207, 208 in ammatory bowe disease, 499t, 509t systemic spread, 702, 703
N-acety cysteine therapy, 40t, 42t, mechanisms o action, 368t or topica in ections, 714715
54, 65t mo ecu ar target, 383, 384 vehic es, 712714, 712t
symptoms, 366 rheumatoid arthritis, 368t ADP receptor antagonists, 323t, 332t
treatment, 54 Adapa ene, 707t, 724t c opidogre , 323t, 324, 325, 325t,
Acetazo amide, 225t, 237t Adaptive immunity, 376b 331332, 332t
Acety cho ine (ACh), 81 , 82 , 97 , 112t Addiction, drug, 254265. See also mechanisms o action, 323t
acety cho inesterase hydro ysis, 104 speci c types prasugre , 323t, 324, 325, 325t, 332t
A zheimers disease, 246, 250252 a coho ism, 256257, 256t tic opidine, 323t, 332t
mechanisms o action, 95t benzodiazepine, 257258, 257t Adrena cortex
neuromuscu ar junction, 105 cannabis, 263 anatomy, 461
on nicotinic receptors, 93, 94 cocaine, 262263, 263t zona g omeru osa, 461 , 467, 468
pharmaco ogy, 83t84t, 85, 9294, 99t cocaine + ethano (cocaethy ene), 265, 266 Adrena insu ciency, 461 , 465, 467, 468

728
Adrenergic neuron b ocking agents, abeta o , 126t, 128t, 146t g ucocorticoids, genera princip es, 464b
antihypertensives, 277t, 286t eva butero , 125t, 143t hydrocortisone, 390t, 403t, 460t, 464, 464
guanadre , 277t, 287t evobetaxo o , 126t, 145t mechanisms o action, 459t, 462 , 464
reserpine, 277t, 286t evobuno o , 125t, 144t names and preparations, 460t461t
Adrenergic pharmaco ogy, 139t isdexam etamine, 120t, 141t physio ogic unctions and pharmaco ogic
acebuto o , 126t, 128t, 146t mechanisms o action, 119t120t, 123 e ects, 462t463t
a butero , 125t, 127, 135, 137, 143t mephentermine, 119t, 139t potencies and equiva ent doses, 463t
a uzosin, 119t, 140t metabo ism, 123 rheumatoid arthritis, 463465
1 adrenergic receptor antagonists, 119t, metaprotereno , 125t, 143t therapeutic uses, 463b
121124, 123 , 140t metaramino , 119t, 139t toxicities, 464b
1-se ective adrenergic receptor methamphetamine, 120t, 141t Adrenocortica suppressants, mitotane, 679t
agonists, 119t, 122 , 123 , 139t methy dopa, 119t, 140t cancer, 644t, 659t
2 adrenergic receptor antagonists, 120t, methy phenidate, 120t, 141t pharmaco ogy, 459t, 469t
123 , 141t metiprano o , 125t, 144t Adrenocortico ytic agents, 459t
2-se ective adrenergic receptor metopro o , 126t, 127129, 128t, 145t Adrenocorticotropic hormone (AC H)
agonists, 119t, 122 , 123 , midodrine, 119t, 139t ana og, 459t, 461 , 468t
124125, 139t140t moda ni , 120t, 142t cosyntropin, 459t, 461 , 468t
adrenergic receptor agonists, 83t84t, nado o , 125t, 128t, 144t mechanisms o action, 459t, 461
93, 94, 123 , 136, 137 naphazo ine, 120t, 142t Adriamycin. See Doxorubicin
receptor antagonists, additiona , 120t, nebivo o , 126t, 128t, 146t Adverse drug events, 5657, 57t
123 , 141t norepinephrine, 118t, 135, 137, 139t Adverse e ects, 5b, 912. See also speci c
amphetamine, 120t, 135136, 137, 141t oxymetazo ine, 120t, 136, 137, 142t drugs and drug classes
aprac onidine, 119t, 140t pemo ine, 120t, 141t speci city, 9
ar ormotero , 125t, 144t penbuto o , 128t A nity (KA), 6b, 8
ateno o , 126t, 128t, 146t phenoxybenzamine, 120t, 136, 137, 141t competitive antagonists, 7b, 16, 17
betaxo o , 126t, 128t, 145t phento amine, 120t, 141t A atoxin B1, 41t, 6062, 61
1-se ective adrenergic receptor antago- pheny ephrine, 119t, 139t hepatitis B rom, 61
nists, 123 , 128t, 145t146t pindo o , 126t, 128t, 145t A rican-American heart ai ure therapy, 306,
2-se ective adrenergic receptor agonists, pirbutero , 125t, 143t 306b, 309, 310
123 , 127, 143t144t prazosin, 119t, 121124, 123 , 140t Agonism, quanti cation, 7
adrenergic receptor agonists, procatero , 125t, 143t Agonists, 3b, 8. See also speci c types
123 , 142t proprano o , 125t, 128t, 136, 138, 144t a osteric (a otropic), 4b, 5 , 8
adrenergic receptor antagonists propy hexedrine, 120t, 142t u , 4b, 4 , 7b, 9
with additiona cardiovascu ar e ects, pseudoephedrine, 120t, 142t inverse, 4, 4b, 4 , 9
123 , 128t, 146t ritodrine, 125t, 144t partia , 4b, 4 , 7b, 9
or heart ai ure, 129130, 286t, 302t, 304, sa metero , 125t, 143t primary, 3b, 8
311t (See also Heart ai ure) serotonin, norepinephrine, and dopamine Akathisia, 172t
mechanisms o action, 302t reuptake inhibitors, 147t A bumin, 22b
nonse ective, 123 , 128t, 144t145t sibutramine, 147t A butero
third-generation, 123 , 129t, 146t si odosin, 119t, 140t pharmaco ogy, 125t, 127, 135, 137, 143t
vasodi ating, 126 sympathomimetic agonists, misc., 120t, pu monary, 389t, 402t
bisopro o , 126t, 128t, 145t 123 , 141t142t A c ometasone dipropionate, 460t. See also
bito tero , 125t, 143t synthesis, storage, and re ease, 122 Corticosteroids
brimonidine, 119t, 140t tamsu osin, 119t, 140t A coho . See Ethano
bucindo o , 126t, 146t terazosin, 119t, 140t A coho dehydrogenase (ALDH), 178
bunazosin, 120t, 141t terbuta ine, 125t, 143t disu ram on, 187
carteo o , 125t, 128t, 144t timo o , 126t, 128t, 145t genetic variants, 189, 190
carvedi o , 126t, 128t, 129, 146t transporters, catecho amine p asma A coho ism
catecho amine metabo ism, 121 membrane, 124t detoxi cation, 257
ce ipro o , 126t, 128t, 146t urapidi , 120t, 141t eta e ects, 182b, 187
c assi cation, 123 xy ometazo ine, 120t, 142t negative rein orcement, 265, 266
c onidine, 119t, 124125, 139t yohimbine, 120t, 141t neurochemica system impacts, 185t
dexmedetomidine, 119t, 139t Adrenergic receptors, 122 physio ogica system e ects, 183,
dexmethy phenidate, 120t, 141t Adrenocortica steroid biosynthesis inhibitors 183t185t, 189, 190
dobutamine, 125t, 142t, 308t aminog utethimide, 459t, 469t treatment, 186, 256257, 256t
doxazosin, 119t, 140t etomidate, 459t, 469t A coho ism drugs, 177t, 186187,
epinephrine, 118t, 139t ketoconazo e, 459t, 469t 186t, 193t
esmo o , 126t, 128t, 146t mechanisms o action, 459t acamprosate, 177t, 186187, 186t,
enotero , 125t, 143t metyrapone, 459t, 466, 469t 193t, 257
ormotero , 125t, 144t Adrenocortica steroids, 468t. See also Cor- disu ram, 177t, 178 , 186187, 186t,
guanabenz, 119t, 140t ticosteroids; G ucocorticoids; 193t, 257
guan acine, 119t, 140t speci c agents ethano and methano metabo ism, 178
indoramin, 120t, 141t adrena insu ciency, 461 , 465, 467, 468 na me ene, 177t, 193t
isoetharine, 125t, 143t conjunctivitis, bacteria , 466 na trexone, 177t, 186187, 186t,
isoprotereno , 125t, 142t Crohns disease, 465 203, 257
ketanserin, 120t, 133t, 141t Cushings syndrome, 465466 A coho withdrawa syndrome, 256, 256t

729
A des eukin procarbazine, 629t, 652t A osteric agonists, 4b, 5 , 8
anticancer, 663t, 682, 683, 685t toxicities, 632b633b, 652t A osteric antagonists, 4b, 9
immunostimu ant, 381t, 388t uses, 629t A osteric site, 8
A dicarb, 100t, 115t nitrogen mustards, 627t, 651t A otropic agonists, 4b, 5 , 8
A dosterone antagonists. See K+-sparing activation, in vivo, 631, 631 A otropic antagonists, 4b, 9
diuretics, minera ocorticoid bendamustine, 627t, 651t A otropic site, 8
antagonists ch orambuci , 627t, 651t A -trans retinoic acid (A RA), 645t,
A dosterone receptor antagonists, 269t, 271t, cyc ophosphamide, 627t, 631 , 646t, 659t
272 , 285t 632t, 651t A motriptan, 130t, 147t
ep erenone, 269t, 280281, 285t, i os amide, 627t, 632t, 651t A og iptin, 471t, 472 , 479t
302t, 311t mechanisms o action, 626, 627t, 628, A opecia, androgenic, 710t, 726t
heart ai ure, 302t, 307, 311t 628b, 630 nasteride, 710t, 726t
ep erenone, 302t, 311t mechanisms o resistance, 627t, minoxidi , 710t, 726t
mechanisms o action, 302t 632633, 632b A osetron, 498t, 505, 506, 508t
spirono actone, 302t, 307, 311t mech orethamine HC , 627t, 651t antidiarrhea , 505, 506
spirono actone, 269t, 285t, 302t, me pha an, 627t, 632t, 651t pharmaco ogy, 130t, 147t
307, 311t toxicities, 632b633b, 632t, 651t 1-Hydroxycho eca ci ero , 481t, 488t
A e acept, 376t, 386t uses, 629t 1-Hydroxy ase, 482 , 486, 487
dermato ogic, 709t, 710 , 714t, 726t nitrosoureas, 652t 5-Reductase inhibitors, 442t, 458t
immunotherapy, 376t, 386t carmustine, 632t, 652t dutasteride, 442t, 458t
psoriasis, 709t, 710 omustine, 652t nasteride, 442t, 456, 457, 458t
A emtuzumab mechanisms o action, 627t, 1-Acid g ycoprotein, 22b
anticancer, 662t, 667t, 671t, 684t 628b, 630 1 Adrenergic receptor antagonists, 119t,
immunotherapy, 376t, 378, 386t mechanisms o resistance, 627t, 121124, 140t
A endronate, 481t, 488t 632633, 632b antihypertensives, 277t
A entani , 195t, 208t semustine, 652t doxazosin, 277t, 286t
A uzosin, 119t, 140t streptozocin, 652t prazosin, 277t, 286t
A iskiren, 273t, 282, 283, 286t toxicities, 632b633b, 652t terazosin, 277t, 286t
A itretinoin, 707t, 724t uses, 629t c assi cation, 123
A ka ine urine, 20b, 2122, 39 p atinum coordination comp exes, 628t, mechanisms o action, 119t, 123
A ka oids, natura , 99t 633635, 653t pharmaco ogy, 140t
A ky ating agents, 626635, 627t628t, carbop atin, 628t, 629t, 632t, 653t prazosin, 121124, 123
651t653t. See also speci c types ce cyc e speci city, 630 1-Se ective adrenergic receptor agonists,
and agents cisp atin, 628t, 629t, 632t, 653t 119t, 124125, 139t
activation, in vivo, 631, 631 mechanisms o action, 628b629b, 628t, c assi cation, 123
a ky su onates, 627t, 651t 630 , 633 c onidine, 124125
busu an, 627t, 632t, 651t mechanisms o resistance, 628t, 632b, mechanisms o action, 119t, 123
mechanisms o action, 627t, 634635 pharmaco ogy, 122 , 139t
628b629b, 630 oxa ip atin, 628t, 629t, 653t 2 Adrenergic receptor agonists
mechanisms o resistance, 627t, toxicities, 632b633b, 632t, c assi cation, 123
632633, 632b 633634, 653t mechanisms o action, 119t
toxicities, 632b633b, 632t, 651t uses, 629t, 653t 2 Adrenergic receptor antagonists, 120t,
ce cyc e speci city, 630 toxicities, 631632, 632b633b 123 , 141t
dermato ogic, 708t, 724t, 726 dose- imiting, 631, 632t antihypertensives, 277t, 286t
carmustine, 708t, 724t, 726t triazenes, 652t c onidine, 277t, 286t
cyc ophosphamide, 708t, 724t, 726t dacarbazine, 628t, 629t, 630 , guanabenz, 277t, 286t
mech orethamine, 708t, 724t, 726t 645647, 652t guan acine, 277t, 286t
DNA repair mechanisms, 629b mechanisms o action, 628b, c assi cation, 123
ethy eneimines and methy me amines, 628t, 630 pharmaco ogy, 141t
627t, 651t mechanisms o resistance, 628t, 2-Se ective adrenergic receptor agonists,
a tretamine (hexamethy endiamine), 632633, 632b 119t, 124125, 139t140t
627t, 651t temozo omide, 652t c assi cation, 123
mechanisms o action, 627t, toxicities, 632b633b, 652t mechanisms o action, 119t, 122 , 139
628b629b, 630 uses, 629t pharmaco ogy, 122 , 139t140t
mechanisms o resistance, 627t, A ky Su onates, 627t, 651t Adrenergic receptor agonists, 123 ,
632633, 632b busu an, 627t, 632t, 651t 136, 137
thiotepa, 627t, 632t, 651t mechanisms o action, 627t, air ow resistance, 136, 137
toxicities, 632b633b, 632t, 651t 628b629b, 630 c assi cation, 123
mechanisms o action, 626, 627t628t, mechanisms o resistance, 627t, pharmaco ogy, 83t84t, 93, 94
628, 628b629b, 630 632633, 632b Adrenergic receptor antagonists, 120t,
mechanisms o resistance, 627t628t, toxicities, 632b633b, 632t, 651t 123 , 141t
632633, 632b uses, 629t c assi cation, 123
methy hydrazines, 629t, 652t A ergic reactions, 46t. See also speci c drugs mechanisms o action, 120t
mechanisms o action, 628b629b, and drug classes pharmaco ogy, 141t
628t, 630 case study, 36 -G ucosidase inhibitors, 471t, 479t
mechanisms o resistance, 628t, urticaria, H 1 antihistamines or, 351, 352 acarbose, 471t, 479t
632633, 632b A opurino , 369t, 370372, 373t mechanisms o action, 471t
730
-G ucosidase inhibitors (Cont.) on protein synthesis, 559, 560 Androgen deprivation therapy, 679680
mig ito , 471t, 479t with rena dys unction, 562 Androgenic a opecia, 710t, 726t
vog ibose, 471t, 479t resistance, 559560, 559b nasteride, 710t, 726t
A prazo am, 92, 176t, 180, 191t, 192t streptomycin, 567t minoxidi , 710t, 726t
addiction, 257258, 257t tobramycin, 566t Androgen receptor antagonists, 442t, 458t
withdrawa syndrome, 257258, 257t Amino evu inic acid photodynamic bica utamide, 442t, 458t
A tep ase, 323t, 329330, 329 , 333t therapy, 708t, 725t utamide, 442t, 454, 456, 457,
A tretamine, 627t, 651t Amiodarone, 312t, 321t 458t, 679t
mechanisms o action, 627t, e ectrophysio ogic actions, 317t ni utamide, 442t, 458t
628b629b, 630 iodine-induced hypothyroidism, 435t, Androgens, 442t, 452454, 458t
mechanisms o resistance, 627t, 438, 439 anticancer, 664t, 679t, 686t
632633, 632b ventricu ar bri ation, 316 biosynthesis, 679
toxicities, 632b633b, 651t ventricu ar tachycardia, 318 drugs against (See Antiandrogens)
A vimopan, 498t, 507t Amitripty ine, 150t, 152154, 152 , 168, 169, structures, 453
A zheimers disease, 245246, 246 173t, 205t testosterone, 442t, 458t
acety cho ine de ciency, 246, 250252 Am odipine testosterone bucca tab et, 442t, 458t
A zheimers disease drugs, 239t, 245248, antihypertensive, 277t, 282, 287t, 294 testosterone cypionate, 442t, 458t
247t, 253t myocardia ischemia, 290t, 294295, 298, testosterone enanthate, 442t, 458t
behaviora , 247248 299, 300t testosterone ge , 442t, 458t
cognitive, 246247, 247t Amobarbita , 177t, 181t, 192t testosterone patch, 442t, 458t
donepezi , 239t, 246, 247t, 250252, 253t Amoxapine, 150t, 173t testosterone undecanoate, 442t, 458t
ga antamine, 239t, 246, 247t, 253t Amoxici in, 552t, 555t Anemia
memantine, 239t, 246247, 251, 252, 253t or duodena u cer, 492, 494t mega ob astic, 417418, 537540
rivastigmine, 239t, 246, 247t, 253t Amphetamine, 120t, 135136, 137, 141t. pernicious, 417418
tacrine, 239t, 247t, 253t See also speci c types Anesthesia, genera , 211214
Amanita muscaria, 97, 100 a ertness, 135136, 137 anesthetic state, components, 211b
Amanita phalloides, 97 toxidrome, 53t rst time, 211212
Amantadine Amphotericin B, 530t, 534, 535t, 588t, genera princip es, 211212, 212b
anti-in uenza, 601t, 602t, 604 , 589590 inducing agents, 212 , 213214, 213 ,
612613, 620t adverse e ects, 590 213t (See also Anesthetic agents,
adverse e ects, 613 ormu ations and pharmacokinetics, parentera )
in uenza A prophy axis, 612 589590, 589t, 596t mechanisms, 211b, 212213
mechanisms o action and resistance, mechanisms o action, 588t, 589 , 590 types, 212
602t, 604 613 mechanisms o resistance, 588t Anesthesia, spina , 217219
Parkinsons disease, 239t, 242t, 245, 253t pharmacokinetic properties, 589t Anesthetic adjuncts
Ambenonium ch oride, 97 , 100t, 104105, uses, 596t ana gesics, 211t, 217, 223t
105 , 106 , 114t Amphotericin B (C-AMB), 588t, 589590, benzodiazepines, 211t, 216217, 223t
Ambrisentan, pu monary, 391t, 398400, 589 , 589t, 595, 596, 596t dexmedetomidine, 211t, 223t
399 , 404t Amphotericin B co oida dispersion neuromuscu ar b ocking agents,
Amcinonide, 460t. See also (C-ABCD), 588t, 589590, 589 , 211t, 223t
Corticosteroids 589t, 596t Anesthetic agents, 210224
Amebic co itis, 532533 Amphotericin B ipid comp ex (ABLC), 588t, endotrachea intubation, 216217
paromomycin, 531t, 532533, 535t 589590, 589 , 589t, 596t mechanisms o action, 210t211t
Amethopterin. See Methotrexate Amphotericin B iposoma (L-AMB), 588t, Anesthetic agents, inha ationa , 222t223t
Amikacin, 559566, 559b, 567t 589590, 589 , 589t, 596t b ood:gas partition coe cient, 215216,
Ami oride, 269t, 284t Ampici in, 552t, 555t 215 , 215t, 220, 221
Aminocyc ito s, 569t, 576t Amprenavir, 601t, 603t, 608 , 609 , 622t choices, 214
Aminog utethimide, 459t, 469t Amy in agonists CO2 absorbent, 220, 221
Aminog ycosides, 559567 mechanisms o action, 471t des urane, 210t, 215t, 220, 221, 223t
amikacin, 565, 566, 567t pram intide, 471t, 479t en urane, 210t, 215t, 223t
+ - actam antibiotics, 559, 560 Amyotrophic atera sc erosis (ALS), 239t, ha othane, 210t, 215t, 222t
dosing 249, 253t iso urane, 210t, 215t, 222t
maintenance, 562 bac o en, 239t, 249, 253t minimum a veo ar concentration (MAC),
schedu e, 515, 560561, 561 dantro ene, 239t, 249, 253t 215, 215t
gentamicin, 566t ri uzo e, 239t, 249, 251, 252, 253t nitrous oxide, 210t, 215t, 223t
kanamycin, 567t tizanidine, 239t, 249, 253t pharmacokinetics, 215, 215t
mechanisms o action, 559b Anakinra potencies, 215, 215t
monitoring, p asma concentrations, immunotherapy, 376t, 386t sevo urane, 210t, 215t, 223t
561562, 563, 564565 rheumatoid arthritis, 368t speed o induction, 215216, 215t, 216
neomycin, 567t Ana gesic anesthetic adjuncts, 211t, uptake, 215t, 216, 216
nephrotoxicity, 563564 217, 223t xenon, 211t, 215t, 223t
neti micin, 567t Anandamide, 263 Anesthetic agents, oca , 224t
ototoxocity, 562563 Anaphy axis, 56 administration
penetration, intrace u ar, 564566 Anastrozo e, 442t, 458t routes, 217b, 219
penici in inactivation, 565, 566 anticancer, 664t, 677t, 679t, 686t spina , 217219
postantibiotic e ect, 564, 565 mechanisms o action, 442t, 664t, 678 a ergic reactions, 217, 220, 221

731
articaine, 224t isinopri , 273t, 281, 282, 285t angina, 293t
benzocaine, 224t mechanisms o action, 273t, 274276, 275 atria bri ation, 315, 316
bupivacaine, 224t moexipri , 273t, 285t cardiac action potentia , 312 , 313
Ch oroprocaine, 224t perindopri , 273t, 285t c ass IA (sodium channe b ockers), 312t,
cocaine, 224t quinapri , 273t, 285t 320t321t
dibucaine, 224t ramipri , 273t, 285t disopyramide, 312t, 317t, 321t
dyc onine, 224t structure, 275 procainamide, 312t, 316, 317t, 320t
epinephrine in, 217 trando apri , 273t, 285t quinidine, 312t, 317t, 320t
end artery tissues and gangrene, 220, Angiotensin II (Ang II), e ects, 274 c ass IB (sodium channe b ockers),
221222 Angiotensin II (Ang II) receptor 312t, 321t
idocaine, 224t antagonists, 270t idocaine, 312t, 316, 317t, 319, 320, 321t
mechanisms o action, 217, 218 Angiotensin (A 1) receptor b ockers (ARBs), mexi etine, 312t, 317t, 321t
mepivacaine, 224t 270t, 273t, 285t c ass IC (sodium channe b ockers),
pramoxine, 224t candesartan ci exeti , 273t, 285t 312t, 321t
pri ocaine, 224t diabetics, 274 ecainide, 312t, 317t, 321t
procaine, 224t heart ai ure, 273t, 274, 279, 280, 285t, propa enone, 312t, 317t, 321t
proparacaine, 224t 301t, 310t c ass II ( adrenergic receptor
ropivacaine, 224t ibupro en on, 282, 283 antagonists), 312t, 317t, 321t
su te (methy paraben) in, 220, 221 irbesartan, 273t, 285t esmo o , 312t, 321t
tetracaine, 224t osartan, 273t, 285t proprano o , 312t, 317t, 321t
uses, 217b o mesartan medoxomi , 273t, 285t sota o , 312t, 317t, 321t
on vo tage-gated Na+ channe s, 217 structure, 275 c ass III, 312t, 317t, 321t322t
Anesthetic agents, parentera , 222t te misartan, 273t, 285t amiodarone, 312t, 316, 317t, 318, 321t
context-sensitive ha -time, va sartan, 273t, 280, 282, 283, 285t do eti ide, 312t, 317t, 322t
214, 214 Anidu a ungin, 588t, 589 , 598t dronedarone, 312t, 317t, 321t
di erences, 213t, 214 Anima testing, 8 ibuti ide, 312t, 317t, 322t
etomidate, 210t, 213t, 214 , 222t Antacids, 490t, 496t sota o , 312t, 321t
ospropo o , 210t, 222t Antagonism, 4b, 5 , 89. See also c ass IV (Ca2+ channe b ockers,
on GABAA receptor, 219220, 221 speci c receptors nondihydropyridine), 312t, 322t
ketamine, 210t, 213t, 214 , 217, 222t a osteric (a otropic), 4b, 9 di tiazem, 312t, 315, 317t, 322t
methohexita , 210t, 213t, 222t chemica , 4b, 9, 48b verapami , 312t, 317t, 322t
pharmacokinetics, 212 , 213214 dispositiona , 48b drugdrug interactions, 319, 320
pharmaco ogica properties, 213t drugdrug interaction, 48b e ectrophysio ogic actions, 317t
propo o , 210t, 213t, 214 , 220, mechanisms, 4b, 5 inhibitor concentration, 513514, 514
221, 222t receptor, 48b ong Q and torsade de pointes,
sodium thiopenta , 210t, 213t, 214 , 222t Antagonists, 4b, 5 , 89. See also speci c drugs 313315, 314t, 315b
termination and redistribution, 220, 221 competitive, 4b, 5b, 5 , 7b, 16, 17 magnesium su ate, 317t, 322t
use, 213 unctiona , 4b mechanistic approach, 314t
Anesthetic state, 211b noncompetitive, 4b, 5 Na+-K+-A Pase inhibitors, 312t, 322t
Angina, 290t. See also Myocardia Anthe minthics, 536541. See also digoxin, 312t, 317t, 319, 320, 322t
ischemia He minth in ection drugs paroxysma ventricu ar tachycardia
Angina drugs Anthracenedione derivative immunostimu- (PSV ), 314t, 318
myocardia ischemia (See Myocardia ants, mitoxantrone, 382t, 388t premature ventricu ar beats, 314t
ischemia drugs) Anthracyc ines and anthracenediones, post-MI, 314t, 319, 320
non-myocardia ischemia conditions, 643t, 658t tachyarrhythmia mechanisms, 313b
293t294t daunorubicin, 630 , 643t, 645b, 658t ventricu ar bri ation, 314t, 316
Angioedema. See also speci c drugs doxorubicin, 630 , 643t, 645647, Antibiotics. See Antimicrobia agents
ACE inhibitors, 279 645b, 658t Antibiotics, anticancer, 630 , 643t, 658t
Angiogenesis, 673 epirubicin, 630 , 643t, 645b, 658t actinomycins, dactinomycin, 630 ,
Angiotensin-converting enzyme (ACE) idarubicin, 630 , 643t, 645b, 658t 643t, 658t
inhibitors, 270t, 273t, 285t mechanisms o action, 630 , 643t anthracyc ines and anthracenediones,
acute MI, 279 mechanisms o resistance, 643t, 645b 643t, 658t
adverse e ects, 276 mitoxantrone, 630 , 643t, 645b, 658t daunorubicin, 630 , 643t, 645b, 658t
angioedema, 279 uses, 644t, 658t doxorubicin, 630 , 643t, 645647,
benzapri , 273t, 285t va rubicin, 630 , 643t, 645b, 658t 645b, 658t
captopri , 273t, 279, 285t Antiandrogens, targeted anticancer epirubicin, 630 , 643t, 645b, 658t
cautions, 274276 nonsteroida , 665t, 679t, 687t idarubicin, 630 , 643t, 645b, 658t
diabetics, 274 bica utamide, 665t, 687t mechanisms o action, 630 , 643t
ena apri , 273t, 274276, utamide, 665t, 679t, 687t mechanisms o resistance, 643t, 645b
275 , 285t ni utamide, 665t, 687t mitoxantrone, 630 , 643t, 645b, 658t
ena apri at, 273t, 285t steroida , 665t, 687t uses, 644t, 658t
osinopri , 273t, 285t cyproterone, 665t, 687t va rubicin, 630 , 643t, 645b, 658t
heart ai ure, 272 , 276 , 279, 285t, megestro , 665t, 687t b eomycin, 630 , 643t, 645b, 658t
303304, 310t Antiarrhythmic drugs, 312322 mechanisms o action, 643t
mechanisms o action, 270t, 273t, adenosine receptor agonists, 312t, 322t mechanisms o resistance, 643t,
274276, 275 , 301t adenosine, 312t, 317t, 318, 322t 645b646b

732
Antibiotics, anticancer (Cont.) mood e evation, 152153, 152 Antidiarrhea agents, 498t, 508t
mitomycin, 630 , 643t, 646b, 658t phases o treatment, 153154 a osetron, 498t, 505, 506, 508t
uses, 644t, 658t sites o actions, 152 bismuth, 498t, 508t
Anticho inergic toxidrome, 53t structures, dosing, and adverse e ects, carboxymethy ce u ose, 498t, 508t
Anticho inesterase agents, 89 155t156t cho estyramine, 498t, 508t
Anticho inesterase insecticides, 102, 102b, therapeutic ag, 153 c onidine, 498t, 508t
103 , 103t, 104 . See also Antidepressants, atypica , 150t, di enoxin + atropine, 498t, 508t
speci c types 156t, 174t diphenoxy ate + atropine, 498t,
antidotes, 40t, 65t, 102 atomoxetine, 150t, 174t 500, 508t
pra idoxime, 41t, 42t, 65t, bupropion, 150t, 174t operamide, 209t, 498t, 500501, 508t
100t, 102 mianserin, 150t, 174t mechanisms o action, 498t
Anticoagu ants. See also speci c drugs and mirtazapine, 150t, 174t octreotide, 498t, 508t
drug classes ne azodone, 150t, 174t Antidiarrhea agents, opioid, 209t
indications, 325t reboxetine, 150t, 174t di enoxin, 209t
monitoring, 325, 325t, 330b structures, dosings, and adverse diphenoxy ate (+ atropine), 209t, 498t,
toxicities, 325 e ects, 156t 500, 508t
Anticoagu ants, ora trazodone, 150t, 174t operamide, 209t, 498t, 500501, 508t
direct actor Xa inhibitors, Antidepressants, tricyc ic ( CAs) mechanisms o action, 195t
323t, 333t adverse e ects, 154 Antidotes, 42t, 65t. See also speci c poisons
apixaban, 323t, 328, 331, 333t e der y, 71, 72, 75 Antiemetics. See Antinauseants and
mechanisms o action, 323t pain target and site o action, 205t antiemetics
rivaroxaban, 323t, 328, 333t sites o actions, 152 Antiestrogens, targeted anticancer
direct thrombin inhibitors, 333t Antidiabetic agents, ora aromatase inhibitors, 664t, 679t, 686t
dabigatran etexi ate, 323t, 328, 333t -g ucosidase inhibitors, 471t, 479t anastrozo e, 664t, 677t, 679t, 686t
mechanisms o action, 323t acarbose, 471t, 479t exemestane, 664t, 677t, 679t, 686t
vitamin K antagonist, 333t mechanisms o action, 471t etrozo e, 664t, 677t, 679t, 686t
mechanisms o action, 323t mig ito , 471t, 479t se ective estrogen-receptor
war arin (See War arin) vog ibose, 471t, 479t downregu ators, 664t
Anticoagu ants, parentera amy in agonists u vestrant, 664t, 677t, 686t
activated protein C, 333t mechanisms o action, 471t se ective estrogen-receptor modu ators,
drotrecogin a a, 323t, 333t pram intide, 471t, 479t 664t, 686t
mechanisms o action, 323t biguanides, met ormin, 470t, 479t mechanisms o resistance, 664t
direct thrombin inhibitors, 323t, 333t bi e acid sequestrants, 471t, 479t ra oxi ene, 664t, 686t
argatroban, 323t, 330, 331, 333t co eseve am, 471t, 479t tamoxi en, 664t, 667 , 676678, 677t,
biva irudin, 323t, 333t mechanisms o action, 471t 679t, 686t
desirudin, 333t dipeptidy peptidase-4 inhibitors, 471t, toremi ene, 664t, 677t, 679t, 686t
mechanisms o action, 323t 472 , 476, 479t uses, 677t, 686t
heparinoid, 323t, 325, 333t a og iptin, 471t, 472 , 479t Anti o ates. See Fo ic acid ana ogs
ardeparin, 323t, 325, 333t mechanisms o action, 471t, 472 (anti o ates)
da teparin, 323t, 325, 333t saxag iptin, 471t, 472 , 479t Anti unga agents, 588599
enoxaparin, 323t, 325, 333t sitag iptin, 471t, 472 , 479t amphotericin B, 588t, 589590, 589 , 596t
ondaparinux, 323t, 325, 333t vi dag iptin, 471t, 472 , 479t adverse e ects, 590
heparin, 323t, 325, 327 , 333t drug deve opment, 69 amphotericin B (C-AMB), 595, 596, 596t
ovenox, 325326, 326 GLP-1 agonists, 471t, 472 , 476, 479t amphotericin B co oida dispersion
ow-mo ecu ar-weight heparins, 323t, exenatide, 471t, 472 , 479t (C-ABCD), 596t
324325, 325 , 333t irag utide, 471t, 472 , 479t amphotericin B ipid comp ex
mechanisms o action, 323t mechanisms o action, 471t, 472 (ABLC), 596t
nadroparin, 323t, 325, 333t insu in secretagogues, nonsu ony urea, amphotericin B iposoma
tinzaparin, 323t, 325, 333t 470t, 472 , 479t (L-AMB), 596t
Anticonvu sants. See also speci c drugs and nateg inide, 470t, 472 , 479t mechanisms o action, 588t, 589 , 590
drug classes repag inide, 470t, 472 , 479t mechanisms o resistance, 588t
pain target and site o action, 205t insu in secretagogues, su ony urea, 470t, pharmacokinetic properties, 589t
Anticonvu sants, ma aria, 151t, 175t 472 , 479t butena ne, 599t
carbamazepine, 151t, 164165, 175t adverse e ects, 475 cic opirox o amine, 598t
amotrigine, 151t, 165, 175t g ic azide, 470t, 472 , 479t dermato ogic, 708t, 721, 722, 725t
va proic acid, 151t, 164165, 169, g imepiride, 470t, 472 , 477, echinocandins, 588t, 589 , 598t
173, 175t 478, 479t anidu a ungin, 598t
Antidepressants, 155t156t g ipizide, 470t, 472 , 479t caspo ungin, 588t, 589 , 593, 593 , 598t
actions and e ects, 152154, 152 g yburide, 470t, 472 , 479t mechanisms o action, 588t, 593, 593
adverse e ects, 154 mechanisms o action, 470t, 477, 478 mica ungin, 598t
disposition, 158t mechanisms o action, 470t471t ucytosine, 588t, 589 , 595, 596, 596t
drug-re ated deaths, 51t thiazo idinediones, 470t, 475476, 479t griseo u vin, 588t, 593596, 598t, 721, 722
e der y, 71, 72, 75 adverse e ects, 476 ha progin, 598t
initia treatment phase, 153 mechanisms o action, 470t imidazo es and triazo es, 588t, 589 ,
ong-term e ects, 153 piog itazone, 470t, 475476, 479t 590, 597t
maintenance phase, 154 rosig itazone, 470t, 475476, 479t CYP interactions, 590, 590t

733
drug interactions, 590t592t adrenergic receptor antagonists, 277t, mechanisms o action and resistance,
uconazo e, 588t, 590, 595, 596, 286t (See also adrenergic recep- 602t, 604
597t, 719 tor antagonists) ose tamivir, 601t, 602t, 604 , 620t
itraconazo e, 597t Ca channe b ockers, L-type, 270t,
2+
rimantadine, 601t, 602t, 604 , 620t
posaconazo e, 597t 277t, 287t (See also Ca2+ channe zanamivir, 601t, 602t, 604 , 620t
voriconazo e, 588t, 589 , 590t592t, b ockers) Antima aria s agents, dermato ogic
592593, 595, 596, 597t dihydropyridines, 277t, 282, 287t, 290t, (See Ma aria chemotherapy,
imidazo es and triazo es, topica , 588t, 300t, 306 (See also Ca2+ channe dermato ogic agents)
597t598t b ockers, L-type, dihydropyridine) Antimetabo ites, 635642, 635t636t,
butoconazo e, 597t edema, 282 653t656t
c otrimazo e, 597t mechanisms o action, 277t dermato ogic, 708t, 724t, 725t
econazo e, 597t nondihydropyridines, 277t, 278, 287t 5- uorouraci , 708t, 724t, 725t
ketoconazo e, 598t (See also Ca2+ channe b ockers, azathioprine, 708t, 724t, 725t
miconazo e, 594, 597t L-type, nondihydropyridine) methotrexate, 708t, 715, 721722, Anti-
oxiconazo e, 598t choice, 270, 270t unga agents 724t, 725t
sertaconazo e, 598t c assi cation, 270t o ic acid ana ogs, 635t, 653t
su conazo e, 598t diabetes with, 273274 high-dose methotrexate with eucovo-
terconazo e, 597t direct renin inhibitors/ rin rescue, 635t, 638640, 653t
tioconazo e, 597t vasodi ators, 270t ometrexo , 635t, 653t
mechanisms and sites o action, diuretics, 268t270t, 269272, 283t285t mechanisms and sites o action, 635t
588t, 589 (See also Diuretics) mechanisms o resistance, 635t,
na i ne, 589 , 599t ibupro en on, 282, 283 638b, 640
nystatin, 599t mechanisms o action, 270t methotrexate, 635t, 653t
ophtha mic, 694t, 701 nephrons, 270b pemetrexed, 635t, 653t
terbina ne, 588t, 589 , 596, 598t pregnancy, 277278, 278 pharmacogenetics, 637 , 639
to na ate, 598t renin-angiotensin system inhibitors, pra atrexate, 635t, 653t
undecy enic acid, 599t 270t, 273t, 274276, 274 , 275 , ra titrexed, 635t, 653t
Whit e ds ointment, 599t 279, 281282, 285t (See also toxicities, 653t
Antig ucocorticoids Renin-angiotensin system (RAS) trimetrexate, 635t, 653t
mechanisms o action, 459t inhibitors) uses, 637t
mi epristone, 459t, 469t treatment a gorithm, 275 , 276 mechanisms o action, 653t656t
Antihepatitis agents, 600t602t, 620t vasodi ators mechanisms o resistance, 638b640b,
ade ovir dipivoxi , 600t, 602t, 620t arteria , 278t, 287t (See also 653t656t
entecavir, 600t, 602t, 620t Vasodi ators, arteria ) purine ana ogs, 636t, 641642, 655t656t
hepatitis B, entecavir, 618, 619 arteria and venous, nitroprusside, 6-mercaptopurine, 630 , 636t,
hepatitis C, ribavirin-pegy ated inter- Anti unga agents 278t, 287t 641642, 655t
eron, 613614, 617, 618 Antihypertensive drugs, sympatho ytics, 6-thioguanine, 630 , 636t, 655t
inter erons, 600t602t, 606 , 620t 270t, 277t c adribine, 630 , 636t, 655t
amivudine, 600t, 602t, 607 , 620t adrenergic neuron b ocking agents, Anti- c o arabine, 636t, 655t
mechanisms o action and unga agents 277t, 286t udarabine phosphate, 630 ,
resistance, 602t guanadre , 277t, 287t 636t, 655t
peginter erons, 601t reserpine, 277t, 286t mechanisms o action, 630 , 636t,
Antiherpes agents, 600t602t, 605608, 619t centra y acting adrenergic agents 641642
acyc ovir, 600t601t, 604 605 , methy dopa, 277t, 286t mechanisms o resistance, 636t,
605608, 605t, 617618, 619t 2 adrenergic receptor antagonists, 277t, 639b640b
cido ovir, 600t, 601t, 605, 619t 286t ne arabine, 636t, 655t
docosano , 600t c onidine, 277t, 286t pentostatin, 636t, 656t
omivirsen, 600t, 602t, 609, 619t guanabenz, 277t, 286t toxicities, 642, 655t656t
oscarnet, 600t, 602t, 604 , 605, 609612, guan acine, 277t, 286t uses, 637t, 655t656t
617, 618, 619t mechanisms o action, 277t pyrimidine ana ogs, 635t636t, 640641
gancic ovir, 600t, 602t, 604 , 605, 605 , 1 adrenergic receptor antagonists, 277t 5- uorouraci (5-FU), 630 , 635t,
609, 617, 618, 619t doxazosin, 277t, 286t 638b639b, 638 , 640641
idoxuridine, 600t, 602t, prazosin, 277t, 286t azacytidine, 636t
605606, 619t terazosin, 277t, 286t capecitabine, 630 , 635t, 638
mechanisms o action and resistance, adrenergic receptor antagonists, cytarabine (Ara-C), 630 , 635t, 639b,
601t602t, 606607, 606 oxazosin, 277t 648, 649
pencic ovir, 600t, 602t, 604 , 605, 605 , Anti-IgE monoc ona antibodies, decitabine, 636t
608, 619t oma izumab, 390t, 397, 404t oxuridine (FUdR), 630 , 635t,
tri uridine, 601t, 602t, 606, 619t Anti-in ectives. See Antimicrobia agents 638b639b, 638
va acyc ovir, 601t, 605, 608 Anti-in ammatories, dermato ogic. See gemcitabine, 630 , 636t, 639b
va gancic ovir, 601t Immunosuppressants and anti- mechanisms and sites o action, 630 ,
Antihistamines. See Histamine receptor in ammatories, dermato ogic 635t636t, 638 , 640
antagonists Anti-in uenza agents, 601t, 602t, Anti unga mechanisms o resistance, 635t636t,
Antihypertensive drugs, 269270. See also agents 604 , 620t 639b, 640641
speci c drugs and drug classes amantadine, 601t, 602t, 604 , uses, 637t
Ang II receptor antagonists, 270t 612613, 620t uses, 637t

734
Antimicrobia , ocu ar, topica , 692t693t tic opidine, 323t, 332t choice, 227b
Antimicrobia agents. See also speci c drugs irreversib e, COX-1 inhibitors dosage, 227b
and drug classes aspirin, 323t, 324325, 332t, e bamate, 226t, 237t
dermato ogic, 708t, 725t 371, 372 gabapentin, 225t, 237t
antibacteria s, 708t, 725t mechanisms o action, 323t, 358 genera princip es, 227b
anti unga s, 708t, 721, 722, 725t sites o action, 325 hepatic microsoma enzyme
retapamu in, 708t, 725t Antipro i erative/antimetabo ic agents, interactions, 227t
mechanisms o action, 2b 375t, 385t. See under Bio ogica iminosti benes, 225t, 236t
Antimicrobia therapy princip es, 512519. immunosuppressants carbamazepine, 225t, 226 ,
See also speci c drugs azathioprine, 368t, 375t, 380, 385t 232235, 236t
and disorders evero imus, 375t, 385t oxcarbazepine, 225t, 236t
area under the curve, 514, 515 MPA, 375t, 385t acosamide, 226t, 237t
barrier penetration, 512513, 513 mycopheno ate mo eti , 375t, 383, amotrigine, 225t, 232, 237t
b ood-brain, 512 (See also B ood-brain 384, 385t Lennox-Gastaut syndrome, 232
barrier (BBB)) siro imus, 375t, 377 , 383, 384, 385t evetiracetam, 225t, 234235, 237t
b ood-CSF, 24b, 28 , 94 Anti-protozoa drugs, 530535. See also mechanisms o action, 225t, 225t226t,
- actam antibiotic resistance, 516517 Protozoa drugs 226 227
c assi cation, 512b Antipsychotics Ca channe b ockers, -type, 225t,
2+

combination therapy, 517b, 518 adverse e ects, 172t 225t226t, 227 , 230231
disease progression time ine, 516, 516 e der y, 77b, 168, 169, 171 GABAA receptor synaptic transmission,
dosing schedu e nonpsychotic disorders, 163b 225t, 225t226t, 226
on concentration-time pro e, Antipsychotics, atypica , 151t, Na+ channe inactivation, 225t,
514, 515 160t161t, 175t 225t226t, 226
resistance suppression, 515 aripiprazo e, 151t, 170t, 175t pregaba in, 225t, 237t
drug and dose regimen choice, microbia chemica structures, dosages, and ru namide, 226t, 237t
aboratory, 513514, 514 maintenance, 160t161t succinimides, 225t, 236t
drug penetration, 512513, 512b metabo ism, 170t ethosuximide, 225t, 230231, 234,
ef ux pumps, 515b, 517 o anzapine, 151t, 165, 168, 170t, 235, 236t
empiric therapy, 516 , 518519 171172, 175t methsuximide, 225t, 236t
endocarditis risk and prophy axis, potencies, 162t tiagabine, 226t, 237t
515b, 516 quetiapine, 151t, 171t, 175t topiramate, 226t, 235, 236, 237t,
HIV combination therapy, 518, 519 risperidone, 151t, 168, 169, 171t, 175t 702, 703
indices, most important, 514, 515 Antipsychotics, typica , 151t, 160t, 175t va proic acid, 225t, 231232, 234,
inhibitory sigmoid Emax curve, 513, 513 , asenapine, 151t, 170t, 175t 235, 236t
514, 514 chemica structures, dosages, and with amotrigine, 235
peak p asma concentration, 514, 515, maintenance, 160t zonisamide, 226t, 237t
515 , 519 c ozapine, 151t, 167169, 170t, 172, 175t Antispasmodics, bowe , 498t, 508t
postantibiotic e ect, 515 droperido , 151t, 175t dicyc omine, 498t, 508t
pre-surgery prophy axis, 515b, 516 ha operido , 151t, 159, 163 , 171t, 175t g ycopyrro ate, 498t, 508t
resistance, 515b, 517 i operidone, 151t, 170t, 175t hyoscyamine, 498t, 508t
wound in ection prophy axis, oxapine, 151t, 175t mechanisms o action, 498t
514b, 516 metabo ism, 170t methscopo amine, 498t, 508t
Antinauseants and antiemetics, 498t, 508t mo indone, 151t, 175t Antispasticity agents, cho inergic,
aprepitant, 498t, 502t, 508t pa iperidone, 151t, 170t, 175t 106t, 117t
choice, 501, 501 potencies, 162t Antithymocyte g obu in (A G), 376t, 377 ,
c assi cation, 501, 502t receptor occupancy and c inica 377t, 378, 385t
dronabino , 498t, 502, 502t, 508t response, 163 Antithyroid agents, 431t, 439t440t.
mechanisms o action, 498t sertindo e, 151t, 175t See also T yroid and
nabi one, 498t, 502, 508t ziprasidone, 151t, 171t, 175t antithyroid drugs
ondansetron, 498t, 501, 502t, 505, Antipsychotics, typica , phenothiazines, carbimazo e, 431t, 440t
506, 508t 150t, 174t compounds, 435t
Antip ate et agents ch orpromazine, 150t, 171t, 174t methimazo e, 431t, 435436, 438,
GPIIb/IIIa, 323t, 324325, 332t uphenazine, 150t, 174t 439, 440t
abciximab, 323t, 324325, perphenazine, 150t, 174t propy thiouraci , 431t, 439t
331332, 332t tri uoperazine, 150t, 174t Anti-thyroid hormone
epti batide, 323t, 324325, 332t Antirigidity agents, cho inergic, compounds, 435t
mechanisms o action, 323t 106t, 117t Anti- NF reagents, 376t, 386t.
tiro ban, 323t, 324325, 332t Antiseizure drugs, 225235 See also under Bio ogica
indications, 324t acetazo amide, 225t, 237t immunosuppressants
irreversib e, ADP receptor antagonists, adverse e ects, 227b, 227t ada imumab, 368t, 376t, 381, 383,
323t, 332t agents and princip es, 227b, 228 384, 386t
c opidogre , 323t, 324, 325, 325t, barbiturates, phenobarbita , 225t, 236t certo izumab pego , 376t, 381, 386t
331332, 332t benzodiazepines, 225t, 237t etanercept, 376t, 380, 386t
mechanisms o action, 323t c onazepam, 225t, 237t go imumab, 376t, 386t
prasugre , 323t, 324, 325, 325t, 332t c orazepate, 225t, 237t in iximab, 368t, 376t, 380, 381, 386t

735
Antitussives, 209t, 391t, 404t nevirapine, 601t, 603t, 608 , 610 , Ar ormotero
benzonatate, 209t, 210t, 391t, 404t 615, 622t pharmaco ogy, 125t, 144t
mechanisms o action, 196t structures, 610 pu monary, 390t, 402t
opioid nuc eoside reverse transcriptase inhibitors Argatroban, 323t, 330, 331, 333t
codeine, 209t, 391t, 404t (NR Is), 601t, 603t, 607 , 608 , 621t Arginine vasopressin, 423t, 424 , 429t
dextromethorphan, 391t, 404t abacavir, 601t, 603t, 607 , 608 , 621t Aripiprazo e, 130t, 148t, 151t, 170t, 175t
O C cough medicines, young adverse e ects, 615 adverse e ects, 163
chi dren, 350 didanosine, 601t, 603t, 607 , 608 , 621t mania prophy axis, 165
Antivira agents, 600624 emtricitabine, 601t, 603t, 607 , mechanisms o action, 162163
antihepatitis agents, 600t602t, 620t 608 , 621t Aromatase inhibitors, 664t, 679t, 686t
ade ovir dipivoxi , 600t, 602t, 620t HIV, 614615 anastrozo e, 664t, 677t, 679t, 686t
entecavir, 600t, 602t, 620t amivudine, 601t, 602t, 607 , 621t exemestane, 664t, 677t, 679t, 686t
inter erons, 600t602t, 606 , 620t mechanisms o action and resistance, etrozo e, 664t, 677t, 679t, 686t
amivudine, 600t, 602t, 607 , 620t 602t603t, 604 , 608 , 614615 Arrhythmia drugs. See Antiarrhythmic drugs
mechanisms o action and stavudine, 601t, 603t, 607 , 608 , 621t Arrhythmias, cardiac, angina drugs, 293t
resistance, 602t structures, 607 Arsenic, 41t, 43t, 63, 64, 66t
peginter erons, 601t teno ovir disoproxi , 601t603t, 607 , Arsenic trioxide (A O), 630 , 645t, 646t, 660t
antiherpes, 600t602t, 605608, 619t 608 , 621t Artemether, 520t, 529t
acyc ovir, 600t, 601t, 604 , 605608, za citabine, 601t, 621t Artemether- ume antrine, 520t, 521,
605 , 605t, 617, 618, 619t zidovudine, 601t, 602t, 607 , 527, 529t
cido ovir, 600t, 601t, 605, 619t 608 , 621t Artemisinin-based combination therapies
docosano , 600t ophtha mic, 693t, 701 (AC s), 521
omivirsen, 600t, 602t, 609, 619t other Artemisinin, 520t, 529t
oscarnet, 600t, 602t, 604 , 605, imiquimod, 601t, 602t, 621t artemether, 520t, 529t
609612, 617, 618, 619t ribavirin, 601t, 602t, 620t artemether- ume antrine, 520t, 521,
gancic ovir, 600t, 602t, 604 , 605, 605 , te bivudine, 601t, 602t, 607 , 608 , 620t 527, 529t
609, 617, 618, 619t teno ovir disoproxi , 601t603t, artesunate, 520t, 529t
idoxuridine, 600t, 602t, 605606, 619t 607 , 608 dihydroartemisinin, 520t, 529t
mechanisms o action and resistance, protease inhibitors, 601t, 603t, 608 , 609 , Artesunate, 520t, 529t
601t602t, 606607, 606 621t622t Arthritis. See also Rheumatoid arthritis
pencic ovir, 600t, 602t, 604 , 605, 605 , amprenavir, 601t, 603t, 608 , NSAIDs or, 366368 (See also Nonste-
608, 619t 609 , 622t roida anti-in ammatory drugs
tri uridine, 601t, 602t, 606, 619t atazanavir, 601t, 603t, 608 , 609 , 622t (NSAIDs))
va acyc ovir, 601t, 605, 608 darunavir, 601t, 603t, 608 , 609 , 622t Articaine, 224t
va gancic ovir, 601t indinavir, 601t, 603t, 608 , 609 , 621t ASA. See Aspirin
anti-in uenza, 601t, 602t, 604 , 620t opinavir, 601t, 603t, 615, 622t Asenapine, 151t, 170t, 175t
amantadine, 601t, 602t, 604 , mechanisms o action and resistance, Aspergillus avus, 60, 61
612613, 620t 603t, 608 , 609 Aspirin, 357t, 358 , 360t, 365366, 371, 372
mechanisms o action and resistance, ne navir, 601t, 603t, 608 , 609 , 622t antip ate et, 323t, 324325, 332t
602t, 604 ritonavir, 601t, 603t, 608 , 609 , COX-1 inhibitors or NSAIDs with,
ose tamivir, 601t, 602t, 604 , 620t 615619, 622t 365366
rimantadine, 601t, 602t, 604 , 620t saquinavir, 601t, 603t, 608 , 609 , 621t pain target and site o action, 205t
zanamivir, 601t, 602t, 604 , 620t tipranavir, 601t, 603t, 608 , 609 , 622t pregnancy, 365
entry inhibitors, 601t, 603t, 608 , 611 , virus rep ication stages and targets, 612t prophy axis, dai y, 365
616, 623t Anxio ytics toxidrome, 53t
en uvirtide, 601t, 603t, 608 , 611 , benzodiazepine, 174t (See also Association rate (k+1), 6b
616, 623t Benzodiazepines) Asthma, 393397, 396
maraviroc, 601t, 603t, 608 , 611 , 623t buspirone, 150t, 174t mi d, 393394 (See also
mechanisms o action and Apixaban, 323t, 328, 331, 333t Pu monary drugs)
resistance, 603t APLs, universa , g atiramer acetate, severe, 393398
amcic ovir, 605, 608 382t, 388t symptoms and pathophysio ogy, 393
integrase inhibitor Apomorphine, 130t, 148t A 1 receptor b ockers. See Angiotensin (A 1)
mechanisms o action and Parkinsons disease, 239t, 242t, 253t receptor b ockers (ARBs)
resistance, 603t Apparent dissociation constant (Kapp), 7b Atazanavir, 601t, 603t, 608 , 609 , 622t
ra tegravir, 601t, 603t, 611 , 623t Aprac onidine CP3A4 metabo ism, 617
mechanisms o action and resistance, b ood-brain barrier, 702, 703 Ateno o
601t603t ophtha mic, 690t, 695t, 702, 703, 704t myocardia ischemia, 300t
non-nuc eoside reverse transcriptase pharmaco ogy, 119t, 140t pharmaco ogy, 126t, 128t, 146t
inhibitors (NNR Is), 601t, 603t, Aprepitant, 498t, 502t, 508t Atomoxetine, 150t, 174t
608 , 610 , 623t Aprotinin, 347t Atorvastatin, 334t, 343t
de avirdine, 601t, 603t, 608 , 610 , 623t aP , 330b Atovaquone, 520t, 529t
e avirenz, 601t, 603t, 608 , 610 , 623t Arachidonic acid metabo ism, 358 Atovaquone-proguani , or ma aria
etravirine, 601t, 603t, 608 , cyc ooxygenase pathway, 358 chemoprophy axis, 521522, 522t, 526
610 , 623t ipoxygenase pathways, 358 , 359 chemotherapy, 520t, 527, 529t
mechanisms o action and resistance, Ardeparin, 323t, 325, 333t A P inhibitors. See Protein tyrosine
603t, 608 , 610 Area under the curve (AUC), 514, 515 kinase inhibitors

736
Atracurium, 106t, 107t, 116t adverse e ects, 182 antiseizure, 225t, 237t
Atria bri ation, 315, 316. See also amobarbita , 177t, 181t, 192t c onazepam, 225t, 237t
Antiarrhythmic drugs butabarbita , 177t, 181t, 192t c orazepate, 225t, 237t
ab ation, 314t, 316 mechanisms o action, 180181 anxio ytic, 174t
antiarrhythmics, 314t mephobarbita , 177t, 181t, 192t categories, e imination-based, 178b
competitive amateur ath ete, 316 methohexita , 177t, 192t ch ordiazepoxide, 176t, 191t, 192t
DC cardioversion, 314t, 316 overdose treatment, 189, 190 c onazepam, 176t, 191t, 192t
digoxin, 319, 320 pentobarbita , 177t, 181t, 192t c orazepate, 176t, 191t, 192t
uncomp icated, drugs, 315 pharmaco ogic properties and uses, 181t dependence and withdrawa , 180
war arin, 327 phenobarbita , 177t, 181t, 189, 190, 192t, diazepam, 176t, 191t, 192t, 214
Atria utter, antiarrhythmics, 314t 225t, 236t drug-re ated deaths, 51t
Atria natriuretic peptide, 269t secobarbita , 177t, 180, 192t e der y, 71, 77b
carperitide, 269t, 284t structures, 181t estazo am, 176t, 191t, 192t
nesiritide, 269t, 280, 284t thiopenta , 177t, 181t, 192t, 214 urazepam, 176t, 180, 191t, 192t
Atria tachycardia, antiarrhythmics, 314t to erance and physica dependence, with hepatic cirrhosis, 189, 190
Atropine, 40t, 65t, 96t, 101102, 181182 hypnotics choice, 189, 190, 191t
101t, 112t toxidrome, 53t orazepam, 176t, 189, 190, 191t, 192t
adverse e ects, 8082, 112t trade names, 181t metabo ic re ationships
e ects, dose in, 101t Baroreceptor re ex, e der y, 77b among, 188
mechanisms o action, 96t Barrier penetration, 512513, 513 midazo am, 176t, 191t, 192t, 214
ophtha mic, 690t, 695t, 704t b ood-brain, 512 (See also B ood-brain overdose treatment, umazeni , 41t, 42t,
uses, 112t barrier (BBB)) 65t, 177t, 179180, 192t
Autonomic gang ia drugs, 106t b ood-CSF, 24b, 28 , 94 oxazepam, 176t, 189, 190, 191t, 192t
Autonomic gang ionic b ockade, 107t Basa gang ia quazepam, 176t, 191t, 192t
Autonomic nervous system, 8085, 92 Huntingtons disease, 248, 248 sedative-hypnotics, 176t, 178b, 180,
anatomy, 81 Parkinsons disease, 240241, 241 189190, 191t192t
e ector organs, 83t84t Basi iximab, 376t, 386t temazepam, 176t, 191t, 192t
gang ionic b ockade, 107t BCNU. See Carmustine (BCNU) toxidrome, 53t
parasympathetic, 82 , 85, 92, 107t BCRP transporter, 24t triazo am, 176t, 191t, 192t
periphera divisions, 80, 85b Bec omethasone dipropionate, 390t, 403t Benzonatate
somatic motor and e erent nerves, Beers Criteria, 6970, 78b antitussive, 209t, 210t, 391t, 404t
82 , 92 Be atacept, 381t, 387t mechanisms o action, 391t
sympathetic, 82 , 85, 92, 107t Belladonna atropa toxidrome, 53t Benzoy ecgonine, 262
AV noda reentrant tachycardia (PSV ) anti- Be adonna, 101102 Benztropine mesy ate, 96t, 114t
arrhythmics, 314t Bendamustine, 627t, 651t Benzy a coho , dermato ogic, 708t, 725t
Azacytidine (5-azacytidine), 636t, 654t activation, in vivo, 631, 631 Bepotastine, ophtha mic, 691t, 705t
mechanisms and sites o action, 630 , mechanisms o action, 626, 627t, 628, Best Pharmaceutica s or Chi dren Act o
636t, 638 , 640 628b, 630 2002 (BPCA), 71b
mechanisms o resistance, 636t mechanisms o resistance, 627t, 11-Hydroxysteroid dehydrogenase, 464 ,
toxicities, 637t, 654t 632633, 632b 467, 468
uses, 637t, 654t toxicities, 632b633b, 651t B ockers. See adrenergic receptor
Azathioprine uses, 629t antagonists
dermato ogic, 708t, 724t, 725t Bendro umethiazide, 269t, 284t Betamethasone preparations, 460t. See also
idiosyncratic reaction, 504 Benzapri , 273t, 285t Corticosteroids
immunotherapy, 368t, 375t, 380, 385t Benznidazo e, 530t531t, 535t Betaxo o , 126t, 128t, 145t
in ammatory bowe disease, 498t, Benzo[a]pyrene, 41t, 6365 ophtha mic, 691t, 695t, 704t
502503, 503 , 509t Benzocaine, 224t 1-Se ective adrenergic receptor
metabo ism, 502503, 503 Benzodiazepine antagonists, 128t, 145t146t
rheumatoid arthritis, 368t, 375t, addiction, 257258, 257t mechanisms o action, 123 , 126t
380, 385t withdrawa syndrome, 257258, 257t pharmaco ogy and pharmacokinetics,
u cerative co itis, 502503, Benzodiazepine receptor agonists, 177t, 192t 123 , 128t, 145t146t
506, 507 eszopic one, 177t, 192t 2 Adrenergic receptor agonists, ong-
Aze aic, 710t, 727t za ep on, 177t, 192t acting pu monary, 390t, 402t
Aze astine, 347t, 355t zo pidem, 177t, 178179, 192t adverse e ects, 392b, 398
ophtha mic, 691t, 705t Benzodiazepine receptor antagonists, 192t ar ormotero , 390t, 402t
Azithromycin, 568t, 570 , 576t, 692t umazeni , 41t, 42t, 65t, 177t, asthma, 393394, 396397, 402
Az oci in, 552t 179180, 192t adverse e ects, 392b
Aztreonam, 549t, 553, 558t Benzodiazepines, 191t192t bronchodi ation, indirect
a coho withdrawa , 256257 actions, 391b
B a prazo am, 176t, 180, 191t, 192t, mechanisms o action, 390t, 391 , 394
Baci us Ca mette-Guerin (BCG), 381t, 387t 257258, 257t COPD, 395
Bacitracin, 569t, 573, 577t, 692t amyotrophic atera sc erosis, 249 ormotero , 390t, 402t
Bac o en, 239t, 249, 253t anesthetic adjuncts, 211t, 216217, 223t indacatero , 390t, 402t
Bactrim a ergy, 5657 anticonvu sants, 190 mechanisms o action, 390t, 391b,
Ba sa azide, 357t, 498t, 503 , 509t antidote, umazeni , 42t, 65t, 177t, 391 , 394
Barbiturates, 177t, 180182, 181t, 192t 179180 sa metero , 390t, 402t

737
2 Adrenergic receptor agonists, short-acting, mechanisms o action, 301t etanercept, 376t, 380, 386t
pu monary, 389t, 401, 402t metopro o , 301t, 303304, 306, 310t go imumab, 376t, 386t
adverse e ects, 392b Ce , pancreatic, 471 , 472 , 473 in iximab, 368t, 376t, 380,
a butero , 389t, 402t -Lactam antibiotics, 549558. See also 381, 386t
asthma, 393394 speci c agents and drug classes dermato ogic, 709t, 714t, 715716, 726t
eva butero , 389t, 402t + aminog ycosides, 559, 560 ada imumab, 709t, 714t, 722, 723, 726t
mechanisms o actions, 389t, 391b, amoxici in-c avu anic acid, 555, 558t a e acept, 709t, 710 , 714t, 726t
391 , 394 aztreonam, 549t, 553, 558t e a izumab, 709t, 710 , 714t, 726t
metaprotereno , 389t, 402t - actamase inhibitors, 549t, 553, 558t etanercept, 709t, 714t, 726t
pirbutero , 389t, 402t carbapenems, 549t, 553 in iximab, 709t, 714t, 726t
terbuta ine, 389t, 402t cepha osporins, 549t, 550b, 553555, disease-modi ying anti-rheumatic drugs
2-Se ective adrenergic receptor agonists, 556t558t (See also abatacept, 368t, 381t, 387t
127, 143t144t Cepha osporins) ada imumab, 368t
adverse e ects, 127 c avu anic acid, 555, 558t anakinra, 368t
a butero , 125t, 127, 135, 137, 143t doripenem, 558t certo izumab, 368t
mechanisms o action, 123 , 125t, 127 ef ux pumps, 551 go imumab, 368t
pharmaco ogy and pharmacokinetics, enzyme c asses, 550b in iximab, 368t, 376t, 380, 386t
123 , 128t, 143t144t ertapenem, 558t rituximab, 368t
Adrenergic receptor agonists, 123 , 142t imipenem, 558t IL-1 inhibitors, 376t, 386t
with additiona cardiovascu ar e ects, imipenem-ci astatin, 554555 anakinra, 376t, 386t
123 , 128t, 146t mechanisms o action, 549t canakinumab, 376t, 386t
adverse e ects, 128129 meropenem, 558t ri onacept, 376t, 386t
antihypertensive, 286t penici ins, 549t, 550556, 555t556t LFA-1 & LFA-3 inhibitors
doxazosin, 277t (See also Penici ins) a e acept, 376t, 386t
c assi cation, 123 resistance, 516517, 549t, 550b e a izumab, 376t, 386t
e der y, 77b on Staphylococcus aureus, 550 monoc ona antibody, anti-CD3,
heart ai ure, 311t -Lactamase, 517, 519 muromonab-CD3, 376t,
dobutamine, 302t, 307308, 308t, 311t -Lactamase inhibitors, 549t, 553, 558t 378, 385t
dopamine, 302t, 307308, 308t, 311t c avu anic acid, 558t monoc ona antibody, anti-CD25, 376t,
mechanisms o action, 302t, 307308 resistance, 549t 379, 386t
or heart ai ure, 129130, 286t, 302t, su bactam, 558t basi iximab, 376t, 386t
304, 311t tazobactam, 558t dac izumab, 376t, 386t
intrinsic sympathomimetic activity, -Lactam enzyme c asses, 550b monoc ona antibody, anti-CD52,
15, 16 Bethanecho , 95t, 99t, 112t a emtuzumab, 376t, 378, 386t
mechanisms o action, 125t, mechanisms o action, 95t po yc ona antibodies, immune
127128, 128t pharmaco ogy, 99t, 112t g obu in preparations, 376t, 377 ,
metopro o Bevacizumab, 662t, 671t, 674675, 684t 377t378t, 378, 385t
heart ai ure, 306, 309, 310 Bexarotene, 707t, 724t Biotrans ormation, 20b, 28b
myocardia ischemia, 300t Beza brate, 335t, 344t Biotrans orming enzymes, 29b, 29t
pharmaco ogy, 126t, 127129, 128t, 145t Bica utamide, 442t, 458t, 665t, 679t, 687t Bipo ar disorder, 164166
myocardia ischemia, 289t, 291 , BiDi , 306, 306b Bisacody , 498t, 508t
292294, 300t Biguanides, 470t, 479t Bismuth, 490t, 496t, 498t, 508t
ateno o , 300t Bi e acidbinding resins (sequestrants), 334t, Bisopro o
with nitrates, 294 339, 340, 343t heart ai ure, 301t, 309, 310, 310t
proprano o , 300t cho estyramine, 334t, 342343, 343t pharmaco ogy, 126t, 128t, 145t
timo o , 300t co eseve am, 334t, 340, 343t, 471t, 479t Bisphosphonates, 481t, 488t
nonse ective, 123 , 128t, 144t145t co estipo , 334t, 342343, 343t adverse e ects, 485, 487, 488t
mechanisms o action, 125t126t diabetes me itus, 471t, 479t a endronate, 481t, 488t
pharmaco ogy and pharmacokinetics, mechanisms o action, 334t, 471t etidronate sodium, 481t, 488t
123 , 128t, 144t145t Bimatoprost, ophtha mic, 691t, 705t or hyperca cemia, 481t, 485, 488t
ophtha mic, 690t691t, 695t, 700 Binding, 20 pamidronate, 481t, 488t
pharmacokinetics, 128t in vitro studies, 8 zo edronate, 481t, 485, 488t
pharmaco ogy, 123 , 128t, 142t p asma protein, 22b ibandronate, 481t, 485, 488t
pheochromocytoma, 136, 137 chi dren, 68b jaw osteonecrosis rom, 481t, 485, 488t
third-generation, 123 , 129t, 146t e der y, 74b, 75t mechanisms o action, 481t
mechanisms o action, 126t tissue, 22b23b or Paget disease, 481t, 488t
pharmaco ogy, 123 , 146t e der y, 75t pamidronate, 481t, 488t
vasodi ating, 129t Bioavai abi ity, 20b, 33b. See also speci c risedronate, 481t, 488t
vasodi ating, 126 drugs and drug classes ti udronate, 481t, 488t
Adrenergic receptor antagonists e der y, 73b, 73 , 73t74t zo edronate, 481t, 485, 488t
antiarrhythmic, 312t, 317t, 321t in ant, ora drug, 72, 74 Bito tero , 125t, 143t
esmo o , 312t, 321t vo ume o distribution, 30, 33b, 34b Biva irudin, 323t, 333t
proprano o , 312t, 317t, 321t Bio ogica immunosuppressants, 385t386t B eomycin, 643t, 658t
sota o , 312t, 317t, 321t anti- NF reagents, 376t, 386t dermato ogic, 709t, 724t, 726t
heart ai ure, 286t, 304, 310t ada imumab, 368t, 376t, 381, 383, mechanisms o action, 630 , 643t, 647
bisopro o , 301t, 309, 310, 310t 384, 386t mechanisms o resistance, 645b
carvedi o , 301t, 309, 310, 310t certo izumab pego , 376t, 381, 386t B epharitis, 700
738
B epharospasm agents, 692t, 706t carboxymethy ce u ose, 498t, 508t Bradykinin, 347t
abobotu inumtoxin A, 692t, 706t cho estyramine, 498t, 508t Bradykinin antagonists
onabotu inumtoxin A, 692t, 706t c onidine, 498t, 508t ka ikrein inhibitors, 347t
B ood-brain barrier (BBB), 24b, 28 di enoxin + atropine, 498t, 508t kinin receptor antagonists, 347t, 355t
drug di usion, 94 diphenoxy ate + atropine, 498t, Brain. See also speci c agents and disorders
e der y, 74b 500, 508t signa transduction, 88
in ants, 68b operamide, 209t, 498t, 500501, 508t Braters a gorithm, 275 , 276 , 283
penetration, 512 mechanisms o action, 498t Breast cancer. See also Cancer chemotherapy
pH on di usion, 1920, 19b20b, 19 , 20 , octreotide, 498t, 508t and cytotoxics; speci c agents
35, 3738, 90 antinauseants and antiemetics, 498t, 508t ER+, 676679
B ood coagu ation reactions, 325326, 326 aprepitant, 498t, 502t, 508t Her2/neu-positive, 672
B ood-CSF barrier, 24b, 28 , 94 choice, 501, 501 tamoxi en, 442t, 451, 456, 457, 458t, 664t,
B ood:gas partition coe cient, 215216, c assi cation, 501, 502t 667 , 676678, 677
215 , 215t, 220, 221 dronabino , 498t, 502, 502t, 508t Breast eeding, FDA drug ru es, 72b
BNP, human recombinant. See mechanisms o action, 498t Brimonidine, ophtha mic, 690t, 695t, 704t
Nesiritide (BNP) nabi one, 498t, 502, 508t Brinzo amide, ophtha mic, 691t, 705t
Bone remode ing cyc e, 482 , 485 ondansetron, 498t, 501, 502t, 505, Brom enac, 357t, 373t
Bone turnover disorder drugs, 480489 506, 508t ophtha mic, 691t, 705t
bisphosphonates, 481t, 488t antispasmodics, 498t, 508t Brimonidine, 119t, 140t
a endronate, 481t, 488t dicyc omine, 498t, 508t Bromocriptine, 422t, 426, 428t
etidronate sodium, 481t, 488t g ycopyrro ate, 498t, 508t Parkinsons disease, 242t
ibandronate, 481t, 485, 488t hyoscyamine, 498t, 508t pharmaco ogy, 130t, 148t
mechanisms o action, 481t mechanisms o action, 498t Brompheniramine ma eate, 346t, 354t
pamidronate, 481t, 488t methscopo amine, 498t, 508t BSEP transporter, 23t, 24t, 25t, 38
risedronate, 481t, 488t in ammatory bowe disease, Bucindo o , 126t, 146t
ti udronate, 481t, 488t 498t499t, 509t Budesonide, 381, 460t. See also
zo edronate, 481t, 485, 488t ada imumab, 499t, 509t Corticosteroids
hormones azathioprine, 498t, 502503, 503 , 509t Crohns disease, 465
ca citonin, 480t, 483484, 483 , ba sa azide, 498t, 503 , 509t in ammatory bowe disease, 498t, 509t
486, 487 budesonide, 498t, 509t pu monary, 390t, 402, 403t, 460t
teriparatide, 480t certo izumab pego , 499t, 509t Bumetanide, 269t, 282, 283, 284t
hyperca cemia, 483484 cyc osporine, 499t, 509t Bunazosin, 120t, 141t
mechanisms o action, 480t481t in iximab, 499t, 509t Bupivacaine, 224t
monoc ona antibody, denosumab, mechanisms o action, 498t499t Buprenorphine, 194t, 195t, 200t, 203, 209t
481t, 488t mesa amine, 498t, 503 , 509t or opioid addiction, 261262
phosphate binders, 481t, 484, 488t methotrexate, 499t, 509t or opioid dependence, 203
anthanum carbonate, 481t, 484, 488t nata izumab, 499t, 509t Buprenorphine/na oxone, 261262
mechanisms o action, 481t, 488t o sa azine, 498t, 503 , 506, 507, 509t Bupropion, 150t, 174t
rena osteodystrophy, 482 , 484 su asa azine, 381, 498t, 502, 503 , 505, or smoking, 259
seve amer hydroch oride, 481t, 484, 488t 507, 509t Busere in, 664t, 687t
rena osteodystrophy, 482 , 484 axatives, 498t, 507t508t Buspirone, 130t, 133t, 136, 138, 147t, 150t, 174t
vitamin D and ana ogs, 481t, 482 , bisacody , 498t, 508t Busu an, 627t, 632t, 651t
484, 488t bowe e ects, 500t mechanisms o action, 627t, 628b629b, 630
1-hydroxycho eca ci ero , 481t, 488t c assi cation, 500t mechanisms o resistance, 627t,
22-oxaca citro parica cito , 481t, 488t docusate sodium, 498t, 508t 632633, 632b
ca cipotrio , 481t, 488t actu ose, 497t, 505, 506507, 507t toxicities, 632b633b, 632t, 651t
ca citrio , 481t, 482 , 488t magnesium citrate, 497t, 507t uses, 629t
dihydrotachystero , 481b magnesium hydroxide, 497t, 507t Butabarbita , 177t, 181t, 192t
doxerca ci ero , 481b magnesium su ate, 497t, 507t Butena ne, 599t
ergoca ci ero , 481t, 488t mechanisms o action, 498b, 498t, 499 Butoconazo e, 588t, 597t
intestina bypass surgery, 486, 487 po yethy ene g yco , 497t, 507t Butorphano , 194t, 195t, 200t, 203, 209t
parica cito , 481t, 488t sa ine, 497t, 507t
therapeutic uses, 481b sodium phosphate, 497t, 507t C
Bortezomib, 663t, 681, 682683, 685t opioid antagonists Ca2+ channe b ockers
Bosentan, 391t, 398400, 399 , 404t a vimopan, 498t, 507t adverse e ects, 295
Botu inum toxins, 106t, 117t mechanisms o action, 498t contraindications, 295
abotu inum toxin A, 692t, 706t methy na trexone, 498t, 499, 505, toxidrome, 53t
onabotu inum toxin A, 106t, 117t, 506, 507t Ca channe b ockers, L-type, 270t, 277t, 287t
2+

692t, 706t prokinetics, 497t, 507t chemica structures and cardiovascu ar


rimabotu inum toxin B, 106t, 117t cisapride, 497t, 504, 506, 507t e ects, 295t296t
Bowe disorder drugs, 497508 domperidone, 497t, 507t contraindications, 295
acety cho ine synthesis enhancer, erythromycin, 497t, 507t edema, 282
dexpantheno , 498t, 507t ubiprostone, 497t, 507t mechanisms o action, 277t
antidiarrhea , 498t, 508t mechanisms o action, 497 myocardia ischemia, 290t, 294295,
a osetron, 498t, 505, 506, 508t metoc opramide, 497t, 504, 506, 507t 298299, 300t
bismuth, 498t, 508t pruca opride, 497t, 507t with nitrates, 297

739
Ca2+ channe b ockers, L-type, dihydropyridine Candesartan ci exeti , 273t, 285t Carisoprodo , 177t, 193t
antihypertensive, 277t, 282, 287t, 290t, 300t Candidiasis Carmustine (BCNU), 627t, 652t
am odipine, 277t, 282, 287t, 290t, esophagea , 589 , 590t592t, 594596 degradation, 631
294295, 298, 299, 300t ora , 467, 468 dermato ogic, 708t, 724t, 726t
c evidipine, 277t, 287t, 290t, 300t vagina , 594 mechanisms o action, 627t, 628b, 630
e odipine, 277t, 287t, 290t, 300t Cannabinoid receptors, 263, 358 mechanisms o resistance, 627t,
isradipine, 277t, 287t, 290t, 300t Cannabis, 263 632633, 632b
nicardipine, 277t, 287t Capecitabine, 635t, 654t toxicities, 631 , 632t, 652t
ni edipine, 277t, 287t, 290t, 300t mechanisms o action, 630 , 635t, 638 uses, 629t
niso dipine, 277t, 287t, 290t, 300t uses, 637t, 654t Carteo o , 125t, 128t, 144t
heart ai ure, 290t, 300t, 306 Capreomycin, 579t, 587t ophtha mic, 691t, 695t, 704t
nimodipine, 290t, 300t Capsaicin, 710t, 727t Carvedi o
Ca channe b ockers, L-type,
2+
Captopri , 273t, 279, 285t heart ai ure, 301t, 309, 310, 310t
nondihydropyridine Carbacho , 95t, 99t, 112t MI patients, 279
antiarrhythmic, 312t, 315, 317t, 322t mechanisms o action, 95t pharmaco ogy, 126t, 128t, 129, 146t
di tiazem, 312t, 315, 317t, 322t ophtha mic, 690t, 695t, 704t Caspo ungin, 588t, 589 , 593, 593 , 598t
verapami , 312t, 317t, 322t pharmaco ogy, 99t, 112t Catecho amines. See also speci c
antihypertensive, 277t, 287t Carbamate insecticides, 100t, 102, 115t types and drugs
di tiazem, 277t, 278, 287t, 290t, 300t atropine or, 42t, 65t (See also Atropine) endogenous, 118t, 139t
verapami , 277t, 287t, 290t, 300t, 306 Carbamazepine metabo ism, 121
Ca2+ channe s, 86, 87 , 89t a coho withdrawa , 257 transporters, p asma membrane, 124t
-type, antiseizure drugs on, 225t, anticonvu sants, 151t, 164165, 175t Catecho -O-methy trans erase (COM ),
225t226t, 227 , 230231 antiseizure, 225t, 226 , 232235, 236t 240 , 243, 250, 251
Cabergo ine, 130t, 148t, 422t, 426, 428t or g ossopharyngea and trigemina neu- evodopa metabo ism, 243, 243 , 244
Cadmium, 41t, 43t, 65t ra gias, 233 Catecho -O-methy trans erase (COM )
Ca cimimetics. See Cinaca cet Huntingtons disease, 248 inhibitors, 238t, 242t, 244, 244
Ca cineurin, 383384 or ightning-type pain, 233 CBDCA. See Carbop atin (CBDCA)
Ca cipotriene, 707t, 724t Carbapenems, 549t, 553 CCNU. See Lomustine (CCNU)
Ca cipotrio , 481t, 488t Carbary , 100t, 115t Ce ac or, 556t
Ca citonin, 480t, 483484, 483 , 486, 487 Carbenici in, 552t, 556t Ce adroxi , 556t
hyperca cemia, 480t, 481t, 483484, Carbidopa, 238t, 244 Ce dinir, 557t
483 , 488t Carbidopa/ evodopa, 238t, 242, 250, Ce ditoren pivoxi , 557t
Paget disease, 481t, 488t 251, 252t Ce epime, 558t
P H in, 483 Carbimazo e, 431t, 440t Ce xime, 557t
regu ation, 486, 487 Carbinoxamine, 346t, 353t Ce metazo e, 557t
Ca citrio , 481t, 482 , 488t Carbon dioxide gas, therapeutic, 211t, 223t Ce operazone, 557t
Ca cium sensor mimetics. See Cinaca cet Carbonic anhydrase inhibitors Ce otaxime, 557t
Camptothecin ana ogs, 643t, 657t antihypertensive, 268t, 271t, 272 , 283t Ce otetan, 556t
irinotecan, 630 , 643t, 644b645b, 649 acetazo amide, 268t, 283t Ce oxitin, 556t
651, 657t brinzo amide, 283t Ce podoxime proxeti , 557t
mechanisms o action, 630 , 643t, dich orphenamide, 268t Ce prozi , 556t
649651 dorzo amide, 283t Ce azidime, 553, 557t
mechanisms o resistance, 643t, methazo amide, 268t Ce ibuten, 557t
644b645b ophtha mic, 691t, 705t Ce izoxime, 557t
topotecan, 630 , 643t, 644b645b, brinzo amide, 691t, 705t Ce riaxone, 554, 555, 557t
649651, 657t dorzo amide, 691t, 705t Ce uroxime, 556t
toxicities, 657t Carbop atin (CBDCA), 628t, 653t Ce uroxime axeti , 556t
uses, 644t, 657t mechanisms o action, 628b629b, 628t, Ce ecoxib, 357t, 358 , 363t, 371,
Canakinumab, 376t, 386t 630 , 633 372, 373t
Cancer chemotherapy and cytotoxics, mechanisms o resistance, 628t, 632b, cardiovascu ar risk, 357, 365b,
626660. See also speci c drugs 634635 371, 372
and drug classes toxicities, 632b633b, 632t, 633634, 653t Ce ipro o , 126t, 128t, 146t
a ky ating agents, 626635, 627t628t, uses, 629t, 653t Centra y acting adrenergic agents,
651t653t (See also A ky ating Carboxymethy ce u ose, 498t, 508t antihypertensive
agents) ophtha mic, 692t, 706t 2 adrenergic receptor antagonists,
antimetabo ites, 635642, 635t636t, Carcinogens, 43t, 6062, 60 , 61 . See also 277t, 286t
653t656t speci c types c onidine, 277t, 286t
ce cyc e speci city, 630 a atoxin B1, 41t, 6062, 61 guanabenz, 277t, 286t
cytotoxic natura products, 630 , initiation and promotion, 60, 60 guan acine, 277t, 286t
642t644t, 645647 Cardiac action potentia , 312 , 313 methy dopa, 277t, 286t
cytotoxics with diverse mechanisms o Cardiac arrhythmias, drugs. See Centra nervous system (CNS), 92
action, 644t646t, 659t660t (See Antiarrhythmic drugs distribution, 24b
also Cytotoxics with diverse mech- Cardiovascu ar disease risk drugs, 9192 (See also speci c drugs)
anisms o action) assessment, 335336, 338 neurotransmitters, 86b, 9192, 91t
targeted, 661687 (See also argeted anti- LDL-C treatment guide ines, 336, 336t Ce adroxi , 556t
cancer therapies) prevention guide ines, 335, 335b, 338 Cepha exin, 556t

740
Cepha osporins, 553555, 556t558t Chlamydia, cyc oserine, 579t, 587t carbacho , 95t, 99t, 112t
mechanisms o action, 549t Ch ora hydrate, 177t, 193t carbamate insecticides, 100t, 102
mechanisms o resistance, Ch orambuci , 629t carbary , 100t, 115t
549t, 550b Ch oramphenico , 568t, 569 , 571, 576t, 692t cevime ine, 95t, 112t
Cepha osporins, rst-generation, 556t Ch ordiazepoxide, 176t, 191t, 192t ch orpyri os, 100t, 103t, 115t
ce adroxi , 556t Ch oride channe s, 86 cho ine acety trans erase, 97
cepha exin, 556t 2-Ch oro-2'- uoro-arabinosy adenine. See cho ine esters, 99t
cephazo in, 556t C o arabine (2-ch oro-2'- cho inergic crisis, 105
Cepha osporins, ourth-generation, ce epime, uoro-arabinosy adenine) cisatracurium, 106t, 107t, 117t
558t 2-Ch orodeoxyadenosine (2-CdA. See cyc opento ate hydroch oride, 96t, 114t
Cepha osporins, second-generation, C adribine dantro ene, 106t, 117t
556t557t Ch orophy in, 62 dari enacin, 96t, 101t, 113t
ce ac or, 556t Ch oroprocaine, 224t diazinon, 100t, 103t, 115t
ce metazo e, 557t Ch oroquine dicyc omine hydroch oride, 96t, 113t
ce otetan, 556t dermato ogic, 708t, 725t donepezi , 100t, 115t
ce oxitin, 556t ma aria chemotherapy, 521t, 522t, doxacurium, 106t, 107t, 117t
ce prozi , 556t 523524, 526, 529t edrophonium, 100t, 104105, 104 , 114t
ce uroxime, 556t P. alciparum resistance, 523524, 523 esoterodine, 96t, 101t, 113t
ce uroxime axeti , 556t toxicities, 524 avoxate, 96t
oracarbe , 557t Ch oroquine/hydroxych oroquine, 521t, 522t, ga antamine, 100t, 115t
Cepha osporins, third-generation, 553, 554, 523524, 526, 529t g ycopyrro ate, 96t, 113t
555, 557t Ch oroquine phosphate, 522t homatropine hydrobromide, 96t, 114t
ce dinir, 557t Ch orothiazide, 269t, 284t ipratropium, 96t
ce ditoren pivoxi , 557t Ch orpheniramine, 346t, 354t jimson weed, 101102, 101t
ce xime, 557t Ch orpromazine, 150t, 171t, 174t mecamy amine, 106t, 117t
ce operazone, 557t Ch orpyri os, 100t, 103t, 115t mechanisms o action
ce otaxime, 557t Ch ortha idone, 269t, 284t muscarinic agonists, 95t
ce podoxime proxeti , 557t Cho estero absorption inhibitors muscarinic antagonists, 96t
ce azidime, 553, 557t ezetimibe, 335t, 344t, 350 methacho ine, 99t, 112t
ce ibuten, 557t ezetimibe/simvastatin, 344t, 350 methscopo amine bromide, 96t, 114t
ce izoxime, 557t Cho estero eve c assi cation, 337t mivacurium, 106t, 107t, 117t
ce riaxone, 554, 555, 557t Cho estyramine muscarinic acety cho ine receptors,
Cephazo in, 556t antidiarrhea , 498t, 508t 98t99t
Certo izumab (pego ) on drug absorption, 342343 muscarinic receptor antagonists, 99t, 100,
Crohns disease, 381 Cho ine acety trans erase (ChA ), 97 101b, 101t
immunotherapy, 376t, 381, 386t Cho ine esters, 99t muscarinic receptors, 97
in ammatory bowe disease, 499t, 509t Cho inergic antagonists, pu monary mushrooms, 97, 100
rheumatoid arthritis, 368t muscarinic, 390t, 401402, 403t. neostigmine bromide, 100t, 114t
Cetirizine, 347t, 348, 348b, 351352, 355t See also speci c drugs and neostigmine methy su ate, 100t, 114t
chi dren, 351, 352 drug classes neuroe ector junction, 97
motion sickness, 352 adverse e ects, 395 neuromuscu ar b ocking agents
pregnancy and actation, 348, 348b, asthma, severe, 397 c assi cation, 107t
351352 COPD, 394395 mechanisms, 106t
Cetrore ix, 423t, 429t, 665t, 686t ipratropium bromide, 390t, 402, 403t nicotine gum or ozenge, 106t, 108,
Cetuximab, 662t, 667t, 669672, 683t mechanisms o action, 390t 108b, 117t
Cevime ine, 95t, 112t tiotropium bromide, 390t, 401402, 403t nicotine nasa spray or vapor inha er,
Channe s, ion, 3. See also speci c types Cho inergic crisis, 105 106t, 108, 108b, 117t
Che ators, heavy meta , 66t. See also Cho inergic pharmaco ogy, 95116. nicotine transderma patch, 106t, 108,
speci c agents See also speci c drugs and 108b, 117t
Chemica antagonism, 4b, 9, 48b drug classes nicotinic acety cho ine receptors, 98t
Chemotherapy, cancer. See Cancer acety cho ine, 95t, 97 , 99t, 112t onabotu inum toxin A, 106t, 117t
chemotherapy and cytotoxics; acety cho ine receptors, 97 organophosphates, 100t, 102, 102b,
speci c drugs and drug classes acety cho inesterase, 97 103t, 104
Chi dren. See also speci c drugs and disorders acety cho inesterase inhibitors, 100t oxybutynin, 96t, 99t, 100, 101b, 101t, 113t
gray baby syndrome, 68 a dicarb, 100t, 115t Panaeolus, 97
in ant ora drug bioavai abi ity, 72, 74 a ka oids, natura , 99t pancuronium, 97 , 105 , 106 , 106t,
kernicterus, 68 Amanita muscaria, 97, 100 107t, 116t
medication sa ety and FDA, 71b72b Amanita phalloides, 97 physostigmine sa icy ate, 100t, 102, 114t
morphine ambenonium ch oride, 97 , 100t, pi ocarpine, 95t, 99t, 112t
in ant dosing, 72, 74 104105, 105 , 106 , 114t pi ocarpine hydroch oride, 95t, 99t, 112t
metabo ism, 68 atracurium, 106t, 107t, 116t pipecuronium, 97 , 105 , 106 , 106t,
Chi dren, pharmacokinetics atropine, 96t, 101102, 101t, 112t 107t, 116t
absorption, 67b autonomic gang ionic b ockade, 107t pra idoxime, 41t, 42t, 65t, 100t, 102, 116t
distribution, 68b be adonna, 101102 propoxur, 100t, 115t
excretion, 69b benztropine mesy ate, 96t, 114t Psilocybe, 97
metabo ism, 70b bethanecho , 95t, 99t, 112t psi ocybin, 97

741
pyridostigmine bromide, 100t, 114t mechanisms o resistance, 628t, 632b, C oxaci in, 551, 552t, 554, 555, 555t
rimabotu inum toxin B, 106t, 117t 634635 C ozapine, 130t, 147t, 151t, 167169, 170t,
rivastigmine, 100t, 115t toxicities, 632b633b, 632t, 633634 172, 175t
rocuronium, 97 , 105 , 106 , 106t, uses, 629t, 632t, 653t CO2 absorbent, 220, 221
107t, 117t Cita opram, 130t, 146t, 150t, 174t Cocaethy ene, 265, 266
sarin, 100t, 103t, 115t Citrovorum actor, 406t, 416, Cocaine
scopo amine, 96t, 112t 417418, 420t addiction, 262263
so i enacin, 96t, 101t, 113t C adribine (2-ch orodeoxyadenosine, drug-re ated deaths, 51t
soman, 100t, 103t, 116t 2-CdA), 630 , 636t, 655t ethano with, 265, 266
succiny cho ine, 97 , 105 , 107t, 108, 116t mechanisms o action, 630 , 636t, oca anesthetic, 224t
summary tab e, 112t117t 641642 toxidrome, 53t
tabun, 100t, 103t, 116t mechanisms o resistance, 636t withdrawa , 263, 263t
tiotropium, 96t, 113t C arithromycin, 568t, 570 , 571, 576t Cockcro -Gau t equation, 76b
to terodine, 96t, 101t, 113t duodena u cer, 492, 494t Codeine, 195t, 200t, 206, 208, 208t
trihexyphenidy hydroch oride, peptic u cer disease, 571 antitussive, 209t, 391t, 404t
96t, 114t toxicities, 571 Co chicine, 369t, 370, 373t
trimethaphan, 106t, 117t C audication, 295b Co d medicines. See also speci c drugs and
tropicamide, 96t, 114t C audication treatment, 295b ingredients
trospium ch oride, 96t, 101t, 113t C avu anic acid, 549t, 553, 558t a coho with, 366
varenic ine, 106t, 117t C - channe s, 86 O C, young chi dren, 350
vecuronium, 97 , 105 , 106 , 106t, 107t, C earance (CL), 33b, 35, 35b36b, 36 , 39 Co eseve am, 471t, 479t
108, 116t e der y, 75b, 76 , 76t77t, 77 on drug absorption, 343
Cho inergic toxidrome, 53t rst-order kinetics, 36b Co estipo , on drug absorption, 342343
Cho inesterase inhibitor, 89 in ants and chi dren, 69b Co itis, amebic, 532533
Cho inesterase reactivators, 41t, 42t, 65t, 102, p asma concentration-time curves, paromomycin, 531t, 532533, 535t
104 , 116t 36b, 37 Co itis, u cerative
Chondroitin su ate, ophtha mic, 692t, 706t C emastine umarate, 346t, 353t azathioprine, 502503, 503 , 506, 507
Choriogonadotropin a a, 423t, 427, C evidipine su asa azine, 381, 502
428, 430t antihypertensive, 277t, 287t thiopurine methy trans erase, 502503,
Chromium, 41t, 43t, 66t myocardia ischemia, 290t, 300t 503 , 506, 507
Chronic mye ogenous eukemia, Ph+, C indamycin, 568t, 574, 575, 576t Co ony-stimu ating actors, 663t, 685t.
664669 C obetaso propionate, 460t. See also See also speci c types
rst- ine treatment, 665666 Corticosteroids Combination therapy, antimicrobia ,
mo ecu ar mechanism, 665666 C ocorto one propionate, 460t. See also 517b, 518
Chronic obstructive pu monary disease Corticosteroids Combined androgen b ockade, 680
(COPD), 394395 C o arabine (2-ch oro-2'- uoro-arabinosy - Competitive antagonists, 4b, 5b, 5 , 16, 17
pathophysio ogy, 394 adenine), 630 , 636t, 655t a nity, 7b
treatments, 394395 (See also mechanisms o action, 630 , 636t, Competitive neuromuscu ar b ocking
Pu monary drugs) 641642 agents, 97 , 105 , 106 , 106t, 107t,
Chronic rena ai ure, diuresis, 282, 283 mechanisms o resistance, 636t 116t117t
Chy omicrons, metabo ic pathways, 335 uses, 637t Concentrationresponse (curve), 6b, 6
Cic esonide, 460t. See also Corticosteroids C o azimine, 579t, 583, 587t quanta , 11 , 12
pu monary, 390t, 403t C o brate, 335t, 344t Concentrationtime curves, p asma,
Cic opirox o amine, 598t C omethiazo e, 177t, 193t 36b, 37
Cido ovir, 600t, 601t, 605, 619t, 693t C omiphene, 442t, 452, 455457, 458t dose administration schedu e, 514, 515
Cigarette addiction. See Smoking C omipramine, 150t, 173t e der y, 73
Ci astatin, 554555 C onazepam, 176t, 191t, 192t Conduction abnorma ities
Ci ostazo , 295b amyotrophic atera sc erosis, 249 angina drugs, 293t
Cimetidine antiseizure, 225t, 237t arrhythmia drugs (See Antiarrhythmic
gastric acid disease, 491, 494, 495 C onidine drugs)
structure, 492 antidiarrhea , 498t, 508t Congestive heart ai ure. See Heart ai ure
to erance, 491, 494, 495 antihypertensive, 277t, 286t Conivaptan, 279t, 281, 288t
Cinaca cet, 481t, 486, 487, 488t or opioid addiction, 261 Conjugated equine estrogens, 441t, 445t
hyperparathyroidism, 481t, 488t pharmaco ogy, 119t, 124125, 139t Conjugated estrogen + medroxyprogesterone
hypoca cemia, 481t, 488t C opidogre , 323t, 324, 325, 325t, acetate, 441t
Cinchonism, 524 331332, 332t Conjugation, phase 2, 2829, 28b, 33 ,
Cipro brate, 335t, 344t bioactivation, 22, 28, 325t 35, 39
Cipro oxacin, 544, 544 , 545, 548t, 692t dosing, 22, 28 neonata , 70b
Cisapride, 133t patient-to-patient variabi ity, Conjugation reactions, drug metabo ism, 32t
bowe disorders, 497t, 504, 506, 507t 331332 Conjunctivitis, 700
nausea and vomiting, 504, 506 drug interactions, 28 adrenocortica steroids in, 466
postmarketing survei ance, 4748 mechanisms o action, 323t, 330, 331 Constipation
Cisatracurium, 106t, 107t, 117t C orazepate, 176t, 191t, 192t uid vo ume, 499, 499
Cisp atin, 628t, 653t antiseizure, 225t, 237t axatives, 498b, 498t, 499, 499 ,
mechanisms o action, 628b629b, 628t, C otrimazo e, 588t, 597t 507t508t
630 , 633 C otting cascade, 325326, 326 rom morphine, 92

742
Context-sensitive ha -time, anesthetic, Costimu atory b ockade, 380b, 380 toxicities, 631 , 632b633b, 632t, 651t
214, 214 Cosyntropin, 459t, 461 , 468t dermato ogic, 708t, 724t, 726t
Contraceptives, ora , 448450, 458t Cough, 400 mechanisms o action, 626, 627t, 628,
brand names and ormu ations, 443t444t Cough medicines. See Antitussives 628b, 630
desogestre , 443t COX-1/2 inhibitors mechanisms o resistance, 627t,
drospirenone, 441t, 443t, 444t aspirin, 323t, 324325, 332t 632633, 632b
emergency, 452 COX-1 inhibitors or NSAIDs with, rheumatoid arthritis, 368t
ethiny estradio ormu ations, 441t, 365366 toxicities, 631632, 631 , 632b633b,
443t444t, 457t irreversib e 632t, 651t
evonorgestre , 443t, 444t aspirin, 357t, 358 , 360t, 365366, uses, 629t
mechanisms o action, 441t, 441t442t 371, 372 Cyc oserine
migraine history, 455, 456457 mechanisms o action, 357t, 358 Mycobacterium avium comp ex,
norethindrone, 441t, 443t, 444t COX-1 inhibitors 579t, 587t
norgestre , 443t, 444t mechanisms o action, 323t, 357t, 358 tubercu osis, 579t, 587t
options, 448450 se ective, 357t, 358 Cyc osporine, 368t, 375t, 377 , 380,
progesterone, 450 u cers rom, 492 382383, 385t
progestin-on y, 450, 458t COX-2 inhibitors dermato ogic, 708t, 724t, 726t
risks and adverse e ects, 450451 mechanisms o action, 358 in ammatory bowe disease, 499t, 509t
u iprista , 442t, 444t, 452, 458t pain target and site o action, 205t mechanisms o action, 375t, 382383
Copper supp ement, 406t, 419t se ective ophtha mic, 691t, 705t
Cornea in ammation, 700 cardiovascu ar hazard, 357, 365b, rheumatoid arthritis, 368t, 376t, 385t
Coronary heart disease, risk actors, 371, 372 CYP (cytochrome P-450 super ami y), 29b,
335, 335b ce ecoxib, 357t, 358 , 363t, 371, 29t, 30, 30b31b, 33
Corticore in, 459t, 461 , 468t 372, 373t antiseizure drug interactions, 227t
Corticosteroids, 468t. See also speci c agents etoricoxib, 363t drugdrug interactions, 30b
carbohydrate and protein metabo ism, umiracoxib, 363t genes, 30b
467, 468 menstrua cramping, 364 genetic po ymorphisms, 27t, 28, 31b
Crohns disease, 465 parecoxib, 363t imidazo e and triazo e interactions,
dexamethasone suppression test, 465466 risks, 357 590, 590t
dose tapering, 465 Creatine c earance, oading dose, 34 induction, 31b
genera princip es, 464b Crohns disease agents, 381 reaction cata ysis, 30b, 32t
mechanisms o action, 459t, 462 ada imumab, 381 SSRIs on, 133
names and preparations, 460t461t adrenocortica steroids, 465 CYP2C9, 27t, 30b31b, 595, 596
physio ogic unctions and pharmaco ogic azathioprine, 503504 azo e anti unga s on, 590, 590t
e ects, 462t463t budesonide, 465 po ymorphisms, 10t, 1214, 13
potencies and equiva ent doses, 463t certo izumab, 381 CYP2C19, 27t, 30b31b, 595, 596
rheumatoid arthritis, 463465 in iximab, 381, 503 c opidogre bioactivation, 22, 28, 325t,
therapeutic uses, 463b prednisone, 465, 467, 468 331332
toxicities, 464b su asa azine, 381, 498t, 502, 503 , 505, isoniazid on, 582, 585, 586
Corticosteroids, inha ed pu monary, 507, 509t isoniazid + phenytoin on, 585, 586
390t, 403t Cromo yn sodium, ophtha mic, 691t, 705t prasugre bioactivation, 325t
adverse e ects, 393b, 397398 Crotamiton, 708t, 725t CYP2D6, 27t, 30b31b
asthma, 393397 Cryptosporidia uoxetine, 157
bec omethasone dipropionate, 390t, 403t diarrhea, 532 isoniazid on, 582, 585, 586
bioavai abi ity, 467, 468 nitazoxanide, 530t, 532, 535t paroxetine, 157
budesonide, 390t, 402, 403t, 460t Cupric su ate, 406t, 419t SSRIs on, 133
candidiasis, ora , 467, 468 Cushings syndrome, 466 CYP2E, isoniazid on, 582
cic esonide, 390t, 403t, 460t Cutaneous -ce ymphoma agents, CYP3A4, 27t, 30b31b, 595, 596
COPD, 395 709t, 727t atazanavir metabo ism by, 617
uniso ide, 390t, 403t, 460t Cyanoacry ate, ophtha mic, 692t, 706t azo e anti unga s on, 590, 590t
uticasone, 390t, 403t Cyanocoba amin, 406t, 414415, isoniazid on, 582, 585, 586
mechanisms o action, 390t, 392 , 393 417418, 420t opinavir metabo ism, 618, 619
mometasone, 390t, 403t, 461t Cyc ic nuc eotidegated nonspeci c cation macro ides, 572
triamcino one, 390t, 403t, 461t channe inhibitors, 269t quinupristin/da opristin, 572, 575
Corticosteroids, systemic pu monary, carperitide, 269t, 284t ritonavir on, 617619
390t, 403t nesiritide, 269t, 280, 284t St. Johns wort on, 619
hydrocortisone, 390t, 403t, 460t, 464, 464 Cyc izine, 346t, 350, 352, 354t, 502t CYP3A5, 27t, 30b31b
mechanisms o action, 390t, 392 , 393 Cyc ooxygenase-1 (COX-1), 324, 324 , CYP3A7, 27t, 30b31b
methy predniso one, 390t, 403t, 460t461t 325 , 358 Cyproheptadine, 346t, 354t
predniso one, 390t, 403t, 461t Cyc opento ate, ophtha mic, 690t, 695t, 705t Cyproterone, 665t, 687t
Corticotropin-re easing hormone (CRH), Cyc opento ate hydroch oride, 96t, 114t Cytarabine (cytosine arabinoside, Ara-C),
459t, 461 , 468t Cyc ophosphamide, 627t, 631 , 635t, 648, 649, 654t
corticore in, 459t, 461 , 468t 632t, 651t mechanisms o action, 630 , 635t
mechanisms o action, 459t, 461 activation, in vivo, 631, 631 mechanisms o resistance, 635t, 639b
Cortiso , 11-hydroxysteroid dehydrogenase cancer chemotherapy, 626, 628633 toxicities, 637t, 654t
on, 464 , 467, 468 metabo ism, 631 uses, 637t, 654t

743
Cytochrome P-450 super ami y. See CYP mechanisms o action, 630 , 643t, hydroxych oroquine, 708t, 725t
(cytochrome P-450 645647 quinacrine, 708t, 725t
super ami y) mechanisms o resistance, 645b antimetabo ites, 708t, 724t, 725t
Cytokinece interactions, 408 DDAVP, 279t, 288t 5- uorouraci (FU), 708t, 724t, 725t
Cytokines, recombinant, 381t. See also Deaths, drug-re ated, 51, 51t. See also azathioprine, 708t, 724t, 725t
speci c agents speci c drugs methotrexate, 708t, 715, 721722,
a des eukin, 381t Decitabine (2'-deoxy-5-azacytidine), 724t, 725t
inter eron--2b, 381t 636t, 654t antimicrobia agents, 708t, 725t
inter eron--1a, 381t, 382 mechanisms and sites o action, 630 , antibacteria s, 708t, 725t
inter eron--1b, 381t, 382 636t, 638 , 640 anti unga s, 708t, 721, 722, 725t
inter eron--1b, 381t mechanisms o resistance, 636t retapamu in, 708t, 725t
Cytotoxic natura products, 630 , 642t644t, toxicities, 637t, 654t bio ogic agents, 709t, 714t, 715716, 726t
645647 uses, 637t, 654t ada imumab, 709t, 714t, 722, 723, 726t
Cytotoxics De erasirox, 41t, 65t a e acept, 709t, 710 , 714t, 726t
cancer (See Cancer chemotherapy and De eroxamine, 40t, 42t, 55, 65t e a izumab, 709t, 710 , 714t, 726t
cytotoxics; speci c drugs and drug Degare ix, 665t, 686t etanercept, 709t, 714t, 726t
classes) Dihydroartemisinin, 520t, 529t in iximab, 709t, 714t, 726t
natura products (See Natura products, De avirdine, 601t, 603t, 608 , 610 ca cineurin inhibitors, 708t, 724t, 726t
cytotoxic) mechanisms o action and resistance, cyc osporine, 708t, 724t, 726t
Cytotoxics with diverse mechanisms o 603t, 608 , 610 pimecro imus, 708t, 724t, 726t
action, 644t646t, 659t660t structure, 610 tacro imus, 708t, 724t, 726t
adrenocortica suppressants, mitotane, De irium tremens, 256t capsaicin, 710t, 727t
644t, 659t, 679t -9- HC, 265, 266 cutaneous -ce ymphoma agents, deni-
di erentiating agents Deni eukin di itox eukin di itox, 709t, 727t
arsenic trioxide, 630 , 645t, anticancer, 663t, 671t, 685t g ucocorticoids, 707t, 722, 723, 724t
646t, 660t cutaneous -ce ymphoma agents, 709t, adverse e ects, 723, 724t
HDAC inhibitors, 645t, 660t 727t potency, 723t
romidepsin, 645t, 660t Denosumab, 481t, 488t triamcino one acetonide, 707t,
vorinostat, 645t, 646t, 660t 2'-Deoxy-5-azacytidine. See Decitabine 723t, 724t
retinoids, 645t, 646t, 659t (2'-deoxy-5-azacytidine) triamcino one hexacetonide, 707t,
substitutes urea, hydroxyurea, 630 , 645t, 2'Deoxyco ormycin, 636t, 637t, 723t, 724t
646t, 659t 641642, 656t H 1 b ockers
Dependence, 264, 266. See also Addiction; rst-generation, 718t
D Opioid addiction; speci c drugs second-generation, 718t
Dabigatran etexi ate, 323t, 328, 333t barbiturates, 181182 H 2 b ockers, 718t
Dacarbazine (D IC), 628t, 629t, 630 , benzodiazepines, 180 hyperpigmentation, 710t, 727t
645647, 652t opioid, buprenorphine or, 203 aze aic, 710t, 727t
mechanisms o action, 628b, 628t, Depo arizing neuromuscu ar b ocking hydroquinone, 710t, 727t
630 , 647 agents, 97 , 105 , 107t, 108, 116t mequino , 710t, 727t
mechanisms o resistance, 628t, Depression monobenzone, 710t, 727t
632633, 632b drug-induced immunosuppressives and anti-in amma-
toxicities, 632b633b, 652t acute, 92 tories, other, 709t, 724t, 726t
uses, 629t hyperexcitabi ity a er, 8990, 94 b eomycin, 709t, 724t, 726t
Dac izumab, 376t, 386t drugs (See Antidepressants) dapsone, 709t, 724t, 726t
Dactinomycin, 630 , 643t, 658t Depsipeptide, 645t, 660t doxorubicin, 709t, 726t
Da teparin, 323t, 325, 333t Derma absorption. See Skin absorption imiquimod, 709t, 724t, 726t
Dantro ene Derma administration mycopheno ate mo eti , 709t, 724t, 726t
amyotrophic atera sc erosis, 239t, e der y, 74t tha idomide, 709t, 724t, 726t
249, 253t Dermato ogic disorders vinb astine, 709t, 724t, 726t
pharmaco ogy, 106t, 117t eczema, 720, 722, 723 in estation agents, 708t, 725t
Dapsone impetigo, 714 benzy a coho , 708t, 725t
dermato ogic, 709t, 724t, 726t pruritus, 720, 720t crotamiton, 708t, 725t
eprosy, 579t, 581 , 583586, 587t psoriasis, 709t, 710 , 715716 ivermectin, 708t, 725t
Daptomycin, 569t, 570 , 573, pyoderma, 714 indane, 708t, 725t
575, 577t tinea corporis, 719 ma athion, 708t, 725t
dosing schedu e, 515 topica administration, 714715 permethrin, 708t, 725t
mechanisms o action, 569t, 573 Dermato ogic therapies, 707727 percutaneous absorption, 710t,
Darbepoetin a a, 405t, 406409, 418t a ky ating agents, 708t, 724t, 726 711715, 711t, 713 (See also Skin
Dari enacin, 96t, 101t, 113t carmustine, 708t, 724t, 726t absorption)
Darunavir, 601t, 603t, 608 , 609 , 622t cyc ophosphamide, 708t, 724t, 726t photochemotherapeutic, 707t,
Dasatinib mech orethamine, 708t, 724t, 726t 716t, 725t
anticancer, 661t, 668, 683t androgenic a opecia, 710t, 726t methoxsa en, 707t, 716t, 725t
mechanisms o resistance, 668 nasteride, 710t, 726t photopheresis, 716t
pharmacodynamics, 15 minoxidi , 710t, 726t PUVA, 716717, 716t, 725t
Daunorubicin, 643t, 658t antima aria agents, 708t, 725t trioxsa en, 716t
ch oroquine, 708t, 725t photodynamic therapy, 708t, 716t, 725t

744
Dermato ogic therapies (Cont.) Diazoxide sitag iptin, 471t, 472 , 479t
amino evu inic acid, 708t, 725t hypertension and edema, 278t, 287t vi dag iptan, 471t, 472 , 479t
methy amino evu inate, 708t, 725t hypog ycemia, 471t, 472 , 476, 479t Dimenhydrinate, 346t, 353t
podophy in, 727t Dibucaine, 224t Diphenhydramine, 1922, 40t, 42t, 65t
rate- imiting step, 721, 722 Dic o enac, 357t, 362t, 371, 372, 373t adverse e ects, 8990, 347
retinoids, 707t, 717718, 717b, 724t ophtha mic, 691t, 705t or a ergic reaction, ood, 349350
acitretin, 707t, 724t Dic oxaci in, 551, 552t, 554, 555, 555t antidote use, 42t, 65t
adapa ene, 707t, 724t Dicyc omine (hydroch oride) antihistamine, 36
a itretinoin, 707t, 724t bowe antispasmodic, 498t, 508t e der y, 69
bexarotene, 707t, 724t pharmaco ogy, 96t, 113t H 1 receptor antagonist, 36, 346t,
isotretinoin, 707t, 721, 722, 724t Didanosine, 601t, 603t, 607 , 608 , 621t 347350
pregnancy warning, 721, 722, 724t mechanisms o action and resistance, ipid membrane di usion,
tazarotene, 707t, 724t 603t, 607 , 608 1920, 20
tretinoin, 707t, 724t structure, 607 mechanisms o action, 346t, 347
sunscreens, 710t, 719b, 726t Di enoxin, 209t motion sickness, 350
vitamin ana ogs, ca cipotriene, 707t, 724t Di enoxin + atropine, 498t, 508t pharmacodynamics, 36
Des urane, 210t, 215t, 220, 221, 223t Di erentiating agents pharmacokinetics, 1922
Desiccated thyroid, 431t, 439t arsenic trioxide, 630 , 645t, 646t, 660t pregnancy, 352
Desipramine, 150t, 173t HDAC inhibitors, 645t, 660t toxidrome, 53t
Desirudin, 333t romidepsin, 645t, 660t Diphenoxy ate, 209t
Des oratadine, 347t, 355t vorinostat, 645t, 646t, 660t Diphenoxy ate + atropine, 498t, 500, 508t
Des ore in, 664t retinoids, 645t, 646t, 659t Diphyllobothrium latum, 537540
Desmopressin (DDAVP), 279t, 288t Di usion. See also B ood-brain barrier Dipive rin, ophtha mic, 690t, 695t, 704t
Desonide, 460t. See also Corticosteroids (BBB); B ood-CSF barrier Direct actor Xa inhibitors, 323t, 333t
Desven a axine, 130t, 147t, 150t, 174t aci itated, 19b, 19 apixaban, 323t, 328, 331, 333t
Detoxi cation. See Addiction; speci c drugs ionized mo ecu es, 19b, 19 mechanisms o action, 323t
Dexamethasone, 460t. See also ipid membrane, 1920, 20 rivaroxaban, 323t, 328, 333t
Corticosteroids nonionized mo ecu es, 1920, 19b20b, Direct renin inhibitors, 270t, 273t, 286t
anticancer, 664t, 686t 19 , 20 , 35, 3738 a iskiren, 273t, 282, 283, 286t
ophtha mic, 691t, 705t passive, 19b, 19 mechanisms o action, 273t
Dexamethasone suppression test, 465466 Di orasone diacetate, 460t. See also structure, 275
Dex ansoprazo e, 490t, 496t Corticosteroids Direct renin inhibitors/vasodi ators, 270t
Dexmedetomidine Di unisa , 357t, 360t, 373t Direct thrombin inhibitors, 323t, 333t
anesthetic adjunct, 211t, 223t 2',2'-Di uorodeoxyuridine. See Gemcitabine argatroban, 323t, 330, 331, 333t
pharmaco ogy, 119t, 139t Di uprednate biva irudin, 323t, 333t
Dexmethy phenidate, 120t, 141t ophtha mic, 691t, 705t dabigatran etexi ate, 323t, 328, 333t
Dexpantheno , 498t, 507t Digoxin desirudin, 333t
Dextromethorphan, 209t, 210t, 391t, 404t antiarrhythmic, 312t, 317t, 319, mechanisms o action, 323t
Diabetes me itus 320, 322t Disease mode s, 7
antihypertensives, 273274 atria bri ation, 319, 320 Disease-modi ying anti-rheumatic drugs
diagnostic criteria, 473, 473t e ectrophysio ogic actions, 317t (DMARDs), 368b, 368t, 379380
drugs promoting, 477t heart ai ure, 301t, 306, 307, 308t, 311t bio ogica s
g ucocorticoids, 467, 468 maintenance dose, steady state, abatacept, 368t
hypertension with, 273274 3031, 34 ada imumab, 368t
type 1, 473474 (See also Insu in) pharmacokinetics, 3034 anakinra, 368t
type 2, 473t, 474476, 475 (See also Anti- pharmaco ogy, 308t certo izumab, 368t
diabetic agents, ora ) therapeutic index, 31, 306 go imumab, 368t
Diabetes me itus pharmacotheapy, toxicity, 306, 309, 310 in iximab, 368t, 376t, 380, 386t
470479 vo ume o distribution, 30 rituximab, 368t
antidiabetic drugs (See Antidiabetic Dihydrotachystero , 481b rheumatoid arthritis, 368b, 368t, 379380
agents, ora ) Dihydrotestosterone, 449 sma mo ecu es
comprehensive care, 473, 473 Diisopropy uorophosphate (DFP), 103t, 104 azathioprine, 368t, 375t, 380, 385t
drug deve opment, 69 Di tiazem cyc ophosphamide, 368t
hemog obin A1c, 473, 474t, 475476 antiarrhythmic, 312t, 315, 317t, 322t cyc osporine, 368t, 376t, 380, 385t
insu in, 470474 antihypertensive, 277t, 278, 287t hydroxych oroquine, 368t
therapeutic goa s, 473, 474t e ectrophysio ogic actions, 317t e unomide, 368t
Dia ysis disequi ibrium syndrome, 283t myocardia ischemia, 290t, 300t methotrexate, 368b, 368t, 380
Diarrhea Dimenhydrinate, 352 minocyc ine, 368t
causes, 499b Dimercapro , 41t, 42t, 66t su asa azine, 368t
drugs, 498t, 508t (See also Antidiarrhea Dimercaptosuccinic acid (DMSA), 41t, 66t Disease progression time ine, antimicrobia
agents) Dimethoxymethamphetamine (DOM), 266 therapy, 516, 516
Giardia intestinalis, 500501 Dipeptidy peptidase-4 (DPP-4) inhibitors, Disopyramide
Diazepam, 176t, 191t, 192t 471t, 472 , 476, 479t antiarrhythmic, 312t, 317t, 321t
context-sensitive ha -time, 214 A og iptin, 471t, 472 , 479t e ectrophysio ogic actions, 317t
e der y, 71 mechanisms o action, 471t, 472 Dispositiona antagonism, 48b
Diazinon, 100t, 103t, 115t saxag iptin, 471t, 472 , 479t Dissociation rate (k-1), 6b

745
Distribution, 20 , 21b. See also speci c drugs methyc othiazide, 269t, 284t ropiniro e, 130t, 148t
and drugs classes meto azone, 269t, 284t rotigotine, 130t, 148t
chi dren, 68b po ythiazide, 269t, 284t serotonin, norepinephrine, and dopamine
CNS, 24b quinethazone, 269t, 284t reuptake inhibitors, 147t
de nition, 21b trich ormethiazide, 269t, 284t serotonin synthesis and inactivation, 131
e der y, 74b, 75t treatment a gorithm, 275 , 276 sibutramine, 147t
phases, 21b DMSA, 41t, 66t Dopaminergic (DA) receptor agonists,
poisoning, 52 DNA 130t, 148t
rate, 20 , 36b37b, 37 drug binding, 2b Dopaminergic (DA) receptor antagonists,
tissue, 21b repair mechanisms, 629b 130t, 148t
transmembrane, weak e ectro ytes, 1920, DNase (dornase a a), muco ytic, 390t, 404t Dopaminergic termina , 138139, 138
19b20b, 19 , 20 , 35, 3738 DNA virus rep icative cyc es, 604 Dopexamine, 118t, 139t
Disu ram, 177t, 178 , 186187, 186t, Dobutamine, 125t, 142t Doripenem, 558t
193t, 257 heart ai ure, 302t, 307308, 308t, 311t Dornase a a, muco ytic, 390t, 404t
Diuretics, 268t269t, 269272, 270t, mechanisms o action, 307308 Dorzo amide, ophtha mic, 691t, 705t
283t285t. See also speci c types Docetaxe , 642t, 648650, 656t Dose. See also speci c drugs and drug classes
atria natriuretic peptide, 269t mechanisms o action, 630 , 642t, e ective vs. etha , 44
carperitide, 269t, 284t 648650 Dose-dependent reactions, 46t
nesiritide, 269t, 280, 284t mechanisms o resistance, 642t, 644b Dose-response curves (re ationship), 4 7 ,
carbonic anhydrase inhibitors, 268t, 271t, toxicities, 656t 6b, 6 , 89, 44 , 45
272 , 283t uses, 644t, 656t graded vs. quanta , 45
acetazo amide, 268t, 283t Docosano , 600t toxicity, 44 , 45
brinzo amide, 283t Docusate sodium, 498t, 508t Dosing interva , 31, 38b39b
dich orphenamide, 268t Do eti ide Dosing rate, 31, 35b, 38, 38b, 38 , 39
dorzo amide, 283t antiarrhythmic, 312t, 317t, 322t Dosing schedu e. See also speci c agents
methazo amide, 268t e ectrophysio ogic actions, 317t on concentration-time pro e, 514, 515
c assi cation, 270t, 272 Do asetron, 130t, 147t resistance suppression, 515
excretory and rena hemodynamic DOM, 266 Doxacurium, 106t, 107t, 117t
e ects, 271t Domperidone, 497t, 507t Doxapram, 391t, 404t
heart ai ure, 284t285t, 301t, 304, 311t Donepezi Doxazosin
ibupro en on, 282, 283 A zheimers disease, 239t, 246, 247t, antihypertensive, 277t, 286t
K+-sparing, minera ocorticoid antago- 250252, 253t pharmaco ogy, 119t, 140t
nists, 269t, 271t, 272 , 285t pharmaco ogy, 100t, 115t Doxepin, 150t, 173t, 346t, 353t
ep erenone, 269t, 280281, 285t Dopamine (DA), 118t, 139t Doxerca ci ero , 481b
spirono actone, 269t, 285t, 307 derivatives, 118t, 139t Doxorubicin, 643t, 645647, 658t
K+-sparing, Na+-channe inhibitors, 269t, heart ai ure, 302t, 307308, dermato ogic, 709t, 726t
271t, 272 , 284t 308t, 311t mechanisms o action, 630 , 643t,
ami oride, 269t, 284t Parkinsons disease, 240241, 241 645647
triamterene, 269t, 282, 283, 284t postsynaptic e ect, 136137, mechanisms o resistance, 645b
oop, 269t, 271t, 272 , 284t 138139, 138 Doxycyc ine, 568t, 569 , 571, 574, 575, 576t
bumetanide, 269t, 282, 283, 284t storage and re ease, 122 ma aria chemoprophy axis, 522t
ethacrynic acid, 269t, 284t synthesis, 122 , 239, 240 mechanisms o action, 568t, 571
urosemide, 269t, 280, 284t, 306 Dopamine (DA) agonists, endocrine, MRSA in ections, 571
torsemide, 269t, 284t 422t, 428t DPP-4. See Dipeptidy peptidase-4
mechanisms and sites o action, bromocriptine, 422t, 426, 428t (DPP-4) inhibitors
268t269t, 270b271b, 270t, cabergo ine, 422t, 426, 428t Dronabino
271t, 272 pro actin-secreting pituitary or anorexia, 266
nephrons, 270b adenoma, 426 antidiarrhea , 498t, 502, 502t, 508t
NSAIDs and resistance, 274b quinago ide, 422t, 426, 428t or nausea and vomiting, 502, 502t
osmotic, 268t, 271t, 283t Dopamine (DA) receptors, 122 chemotherapy-re ated, 265, 266
g ycerin, 268t, 283t Dopaminergic pharmaco ogy Dronedarone
isosorbide, 268t, 283t apomorphine, 130t, 148t antiarrhythmic, 312t, 317t, 321t
mannito , 268t, 283t aripiprazo e, 130t, 148t, 162163, 165 e ectrophysio ogic actions, 317t
urea, 268t, 283t bromocriptine, 130t, 148t Droperido , 151t, 175t
renin re ease pathways, 272 cabergo ine, 130t, 148t Drospirenone, 441t, 443t, 444t, 446t
resistance, 274b dopamine, 118t, 139t Drotrecogin a a, 323t, 333t
therapy, 274b dopamine derivatives, 118t, 139t Drugabi ity, 7
thiazide/thiazide- ike, 269t, 271, 271t, dopaminergic receptor agonists, 130t, Drug addiction. See Addiction, drug; Opioid
272 , 284t 148t, 422t, 426, 428t pharmaco ogy; speci c drugs
bendro umethiazide, 269t, 284t dopaminergic receptor antagonists, Drug approva process, FDA, 44 , 45t,
ch orothiazide, 269t, 284t 130t, 148t 4648, 46t
ch ortha idone, 269t, 284t dopexamine, 118t, 139t Drug deve opment, 69
hydroch orothiazide, 269t, 271, eno dopam, 118t, 139t Drugdrug interactions, 48. See also speci c
273274, 282, 283, 284t mechanisms o action, 130t drugs and drug classes
hydro umethiazide, 269t, 284t pergo ide, 130t, 148t on absorption, 48t
indapamide, 269t, 284t pramipexo e, 130t, 148t acetaminophen, 48

746
Drugdrug interactions (Cont.) mu tip e-agent therapy, 339 distribution and vo ume o distribution,
additive, 48b nicotinic acid, 335t, 343t 74b, 75t
antiarrhythmics, 319, 320 mechanisms o action, 335t excretion, 76b, 78t
c assi cation and description, 48b, 48t niacin, 335t, 340, 341, 342, 343t rst-pass c earance, 73t74t
CYP, 30b pregnancy risk, 340 ha - i e, 77
on drug metabo ism, 48, 48b, 48t vytorin, 339, 340 metabo ism, 73t74t, 75b
Drug interactions, 47 . See also speci c drugs Dystonia, acute, 172t p asma concentrationtime curves, 73 , 73t
and drug classes E ectro ytes, weak
Drug overdose, 63, 64 E p asmagastric juice partitioning,
Drug penetration, 512513, 512b Ebastine, 347t, 355t 19b20b, 20 , 2122
Drug receptor, 2b, 3t. See also Receptor, drug; Echinocandins, 588t, 589 , 598t transmembrane distribution, 1920,
speci c drugs and receptors anidu a ungin, 598t 19b20b, 19 , 20 , 35, 3738
Drug-receptor comp ex (LR), 6b caspo ungin, 588t, 589 , 593, 593 , 598t E etriptan, 130t, 147t
Drug-receptor comp ex con ormationa mechanisms o action, 588t, 593, 593 11-hydroxysteroid dehydrogenase, 464 ,
change (LR*), 6b mica ungin, 598t 467, 468
Drugreceptor interactions, 3b5b, 4 , 6b Echothiophate, ophtha mic, 690t, 695t, 704t E imination, 35b. See also C earance;
Drug-re ated deaths, agents, 51t Econazo e, 588t, 597t speci c drugs
Drug target, 2b Eczema, 720, 722, 723 rst-order kinetics, 36b
Drug testing, new drug, 44 , 45, 45t Edema drugs poisoning, 52
D IC. See Dacarbazine (D IC) diuretics (See Diuretics) E imination t1/2, 34b, 37 , 39b
Du oxetine, 130t, 147t, 150t, 174t to vaptan, 279t, 281, 288t e der y, 77
Duodena u cers Edrophonium, 100t, 104105, 104 , 114t E trombopag, 406t, 419t
proton pump inhibitor + c arithromycin + ED A CaNa2, 41t, 42t, 63, 64, 66t Emedastine di umarate
metronidazo e or amoxici in, 492, E a izumab ophtha mic, 691t, 705t
494t, 495, 571 dermato ogic, 709t, 710 , 714t, 726t Emesis. See Antinauseants and antiemetics
proton pump inhibitors, 492, 493, immunotherapy, 376t, 386t Emetic stimu i, 501, 501
494t, 495 psoriasis, 709t, 710 Empiric therapy. See also speci c drugs and
Duration o action, 36 E avirenz, 601t, 603t, 623t disorders
Dutasteride, 442t, 458t mechanisms o action and resistance, antimicrobia , 516 , 518519
Dyc onine, 224t 603t, 608 , 610 Emtricitabine, 601t, 603t, 621t
Dys ipidemia structure, 610 mechanisms o action and resistance,
c assi cation, 337t E ective concentration, ha -maxima (EC50), 603t, 607 , 608
high-risk, 335b, 337t 6 , 7b, 9 structure, 607
LDL-C treatment guide ines, 336t E ective dose, 44 Ena apri , 273t, 274276, 275 , 285t
Dys ipidemia drugs, 334344 vs. etha dose, 44 Ena apri at, 273t, 285t
bi e acidbinding resins, 334t, 339, median (ED50), 44 , 45 Endocannabinoids, 358
340, 343t E ector organs, autonomic, 83t84t Endocarditis risk and prophy axis, 515b, 516
cho estyramine, 334t, 342343, 343t E ector tone Endocrine pancreas and pharmacotherapy,
co eseve am, 334t, 340, 343t parasympathetic, 107t insu in
co estipo , 334t, 342343, 343t sympathetic, 107t secretion regu ation, 470t, 472
mechanisms o action, 334t E erent nerves, autonomic nervous system, 82 signa ing pathways, 470t, 471
cho estero absorption inhibitor E cacious, 7b Endothe in-1 receptor antagonists,
ezetimibe, 335t, 344t, 350 E cacy pu monary, 391t, 404t
ezetimibe/simvastatin, 344t, 350 u , 4b, 4 adverse e ects, 400
bric acid, 335t, 339, 344t re ative, 7b, 7 , 9 ambrisentan, 391t, 398400, 399 , 404t
beza brate, 335t, 344t Ef ux pumps, 515b, 517 bosentan, 391t, 398400, 399 , 404t
cipro brate, 335t, 344t antimicrobia agent, 516b mechanisms o action, 391t
c o brate, 335t, 344t resistance, 515b, 517 pu monary artery hypertension,
eno brate, 335t, 341, 342, 344t Ef ux transporters, 24b 398400, 399
gem brozi , 335t, 344t b ood-brain and b ood-CSF barriers, Endotrachea intubation, neuromuscu ar
HMG-CoA reductase inhibitors, 24b, 28 b ocking drugs, 216. See also
334t, 343t E ornithine, 530t, 535t Neuromuscu ar b ocking agents
adverse e ects, 338 EGFR (ErbB1), 667t En urane, 210t, 215t, 223t
atorvastatin, 334t, 343t E der y, 6869, 72, 75 En uvirtide, 601t, 603t, 608 , 611 , 616, 623t
choice, 342, 343 benzodiazepines, 71, 77b adverse e ects, 616
uvastatin sodium, 334t, 343t diphenhydramine, 40t, 65t, 69 HIV, 617618
ovastatin, 334t, 341, 342, 343t drug response, 77b mechanisms o action and resistance,
mechanisms o action, 334t, 337 inappropriate medications, 603t, 608 , 611 , 616
other drugs with, 339 6970, 78b Eno ic acid derivatives
pitavastatin ca cium, 334t, 343t NSAIDs, 6869, 72, 75 me oxicam, 357t, 363t, 373t
pravastatin ca cium, 334t, 343t optimizing drug regimens, 77b nabumetone, 357t, 363t, 373t
pregnancy risks, 340 therapeutic window, 11 , 12, 12 piroxicam, 357t, 363t, 373t
rosuvastatin ca cium, 334t, 338, 343t tricyc ic antidepressants, 72, 75 Enoxaparin, 323t, 325, 333t
simvastatin, 334t, 343t E der y pharmacokinetics Entacapone
LDL-C treatment guide ines, 336t absorption and bioavai abi ity, 73b, 73 , antivira , 600t, 602t, 620t
mechanisms o action, 334t335t 73t74t Parkinsons disease, 238t, 242t,
or metabo ic syndrome, 339340 c earance, 75b, 76 , 76t77t, 77 244 , 251, 252t
747
Enteric nervous system, 8283 Epoprosteno estropipate, 441t, 445t
Entecavir adverse e ects, 400 ethiny estradio , 441t, 457t
antihepatitis, 600t, 602t, 620t pu monary, 391t, 398400, 399 , 404t gonadotropin secretion, 447
HIV + chronic HBV, 618, 619 Epothi ones, 643t, 644b, 650, 657t hormone rep acement therapy, 441t442t,
Enterobius in ection, 536, 539, 540 Epti batide, 323t, 324325, 332t 445t446t, 446448, 455457, 458t
Entry inhibitors, 601t, 603t, 608 , 611 , Equi ibration, 20 , 36b37b, 37 evonorgestre , 443t, 444t, 446t
616, 623t Equi ibrative transport, membrane mechanisms o action, 441t442t
en uvirtide, 601t, 603t, 608 , 611 , transporters, 23b ora contraceptives, 441t444t, 448450
616, 623t Equi ibrium association constant (KA), 6b, 8 (See also Contraceptives, ora )
maraviroc, 601t, 603t, 608 , Equi ibrium dissociation constant (KD), 6b, se ective estrogen receptor modu ators,
611 , 623t 8, 16, 17 442t, 458t
mechanisms o action and resistance, 603t ErbB2-targeted agents, 672673 ra oxi ene, 442t, 458t
Environmenta toxicity. See oxicity, ER+ breast cancer, 676679 tamoxi en, 442t, 451, 456, 457,
c inica and environmenta Ergoca ci ero , 481t, 488t 458t, 667
Enzymes. See also speci c types Ergot a ka oids and derivatives, 120t, toremi ene, 442t, 458t
biotrans orming, 29b, 29t 141t, 147t synthetic conjugated estrogens, 441t
intrace u ar, 3t Er otinib, 661t, 670672, 683t Estrogen synthesis inhibitors, 442t, 458t
transmembrane, 3t Ertapenem, 558t anastrozo e, 442t, 458t, 664t, 678
xenobiotic metabo izing, 29b, 29t Erythromycin, 568t, 570 , 576t exemestane, 442t, 458t
Epiderma growth actor receptor (EGFR), in bowe disorders, 497t, 507t ormestane, 442t, 458t
epithe ia cancers, 669 Erythropoietin (EPO), 407t, 408 etrozo e, 442t, 458t
Epiderma growth actor receptor (EGFR) Escherichia coli, cyc oserine, 579t, 587t Estropipate, 441t, 445t
targeting agents, 669671 Escita opram, 130t, 146t Eszopic one, 177t, 192t
adverse e ects, 671 Esmo o Etanercept
cetuximab, 662t, 669672, 683t antiarrhythmic, 312t, 321t dermato ogic, 709t, 714t, 726t
c asses, 669 pharmaco ogy, 126t, 128t, 146t immunotherapy, 376t, 380, 386t
epithe ia cancers treated, 670671 Esomeprazo e, 490t, 495, 496t Ethacrynic acid, 269t, 284t
er otinib, 661t, 670672, 683t Estazo am, 176t, 191t, 192t Ethambuto
ge tinib, 661t, 670672, 683t Estradio cypionate, 441t, 457t Mycobacterium avium comp ex,
mechanisms o resistance, 672 Estradio + drospirenone, 441t, 444t, 457t 578t, 583, 587t
Epidermis, 713 Estradio micronized, 441t, 457t tubercu osis, 578t, 583, 587t
Epi epsy Estradio nonmicronized, 441t, 457t Ethano . See also A coho ism
drug choice, 227b Estradio + norethindrone, 441t, 443t, 444t, abstention, 183
ear y diagnosis and treatment, 228 446t, 457t with acetaminophen, 366
genera princip es, 227b Estradio , testosterone e ects, 449 a coho dehydrogenase 2*2, 178 ,
pharmacotherapy (See Antiseizure drugs) Estradio transderma /topica ge , 441t, 189, 190
seizure c assi cation, 228, 229t 445t, 457t antidote, 42t, 50, 65t
treatment princip es, 227b, 228 Estradio vagina ring/vagina tab ets, 441t, cocaine with, 265, 266
Epinastine, ophtha mic, 691t, 705t 445t, 457t coronary heart disease protection,
Epinephrine (E) Estradio va erate, 441t, 457t 182183
adverse e ects, 217 Estramustine, 630 , 643t, 650, 657t drug-re ated deaths, 51t
or a ergic reactions, 349350 Estrogen antagonists, 442t, 458t eta a coho syndrome, 182b, 187
anaphy axis, 56 c omiphene, 442t, 452, 455457, 458t metabo ism, 49 , 178
end artery tissues, gangrene, u vestrant, 442t, 458t metronidazo e with, 533, 534
220, 221222 mechanisms o action, 442t neurochemica system impacts, 185t
in oca anesthetics, 217 Estrogen esteri ed esters, 441t physio ogica system e ects, 183,
metabo ism, 121 Estrogen receptor, 448 183t185t, 189, 190
pharmaco ogy, 118t, 139t Estrogens, 441t, 457t poisoning, 41t, 49 , 65t
synthesis, storage, and re ease, 122 anticancer, 664t, 665t, 679t, 686t, 687t teratogenic e ects, 182b, 187
Epinephrine (E) receptors, 122 biosynthetic pathways, 446 toxicity, 41t, 65t
Epipodophy otoxins, 643t, 657t conjugated equine estrogens, 441t, 445t Ethiny estradio , 441t, 457t
etoposide, 630 , 643t, 645b, 657t conjugated estrogen + medroxyprogester- Ethionamide, 579t, 587t
mechanisms o action, 630 , 643t one acetate, 441t Ethosuximide, 225t, 230231, 234,
mechanisms o resistance, 643t, 645b drospirenone, 441t, 443t, 444t, 446t 235, 236t
teniposide, 630 , 643t, 645b, 657t estradio cypionate, 441t, 457t Ethy ene g yco antidote
toxicities, 657t estradio + drospirenone, 441t, 444t, 457t ethano , 65t
uses, 644t, 657t estradio micronized, 441t, 457t omepizo e, 42t, 65t, 178
Epirubicin, 643t, 658t estradio nonmicronized, 441t, 457t Ethy eneimines and methy me anines,
mechanisms o action, 630 , 643t, estradio + norethindrone, 441t, 443t, 627t, 651t
645647 444t, 446t, 457t a tretamine, 627t, 651t
mechanisms o resistance, 645b estradio transderma /topica ge , 441t, mechanisms o action, 627t,
Epithe ia cancers, EGFR overexpression/ 445t, 457t 628b629b, 630
activation, 669672 estradio vagina ring/vagina tab ets, mechanisms o resistance, 627t,
Ep erenone, 269t, 280281, 441t, 445t, 457t 632633, 632b
285t, 302t, 311t estradio va erate, 441t, 457t thiotepa, 627t, 632t, 651t
Epoetin a a, 405t, 406409, 418t estrogen esteri ed esters, 441t toxicities, 632b633b, 632t, 651t

748
Etidronate sodium, 481t, 488t Fe odipine mechanisms o resistance, 638b639b,
Etodo ac, 357t, 361t, 373t antihypertensive, 277t, 287t 640641
Etomidate, 459t, 469t myocardia ischemia, 290t, 300t toxicities, 637t, 641, 654t
anesthetic, 210t, 213t, 214 , 222t Fenamates uses, 637t, 654t
context-sensitive ha -time, 214 u enamic acid, 362t, 373t FL -3 igand (FL), 407t, 408
parentera anesthetic, 210t, 213t, 222t mec o enamate, 357t, 362t, 373t F uconazo e, 588t, 589 , 590, 597t
sedative-hypnotic, 177t, 193t me enamic acid, 357t, 362t, 373t adverse e ects, 719
Etoposide (VP-16-213), 643t, 657t Feno brate, 335t, 341, 342, 344t ormu ations and actions, 597t
mechanisms o action, 630 , 643t Feno dopam mechanisms o action and resistance,
mechanisms o resistance, 643t, 645b hypertension and edema, 278t, 287t 588t, 590
toxicities, 657t pharmaco ogy, 118t, 139t tinea corporis, 719
uses, 644t, 657t Fenotero , 125t, 143t war arin interaction, 595, 596
Etoricoxib, 363t Fentany F ucytosine, 588t, 589 , 595, 596, 596t
Etravirine, 601t, 603t, 608 , 610 , 623t heroin with, 260 F udarabine phosphate, 630 , 636t, 655t
mechanisms o action and resistance, pharmaco ogy, 194t, 195t, 203, mechanisms o action, 630 , 636t,
603t, 608 , 610 205207, 208t 641642
structure, 610 Ferrous aspartate, 406t, 413414, 419t mechanisms o resistance, 636t,
Evero imus Ferrous umarate, 406t, 413414, 419t 638b640b
anticancer, 663t, 685t Ferrous g uconate, 406t, 413414, 419t toxicities, 642
immunotherapy, 375t, 385t Ferrous sa ts, 406t, 413414, 419t uses, 637t
Excitatory postsynaptic potentia (EPSP), Ferrous succinate, 406t, 413414, 419t F udrocortisone acetate, 460t
autonomic, 106 Ferrous su ate, 406t, 413414, 419t F umazeni , 41t, 42t, 65t, 177t, 179180, 192t
Excretion, 20 Ferti ity agents, 442t, 452, 455457, 458t F u medicines. See also speci c agents and
chi dren, 69b Ferumoxyto , 406t, 419t ingredients
e der y, 76b, 78t Fesoterodine, 96t, 101t, 113t a coho with, 366
pH, 20b Feta a coho syndrome, 182b F uniso ide, 460t. See also Corticosteroids
weak acid, 36, 39 Fexo enadine, 347t, 355t pu monary, 390t, 403t
Exemestane Fibric acid ( brates), 335t, 339, 344t F uocino one, ophtha mic, 691t, 705t
anticancer, 664t, 677t, 679t, 686t beza brate, 335t, 344t F uocino one acetonide, 460t
estrogen synthesis inhibitor, 442t, 458t cipro brate, 335t, 344t F uocinonide, 460t
Exenatide, 471t, 472 , 479t c o brate, 335t, 344t F uorodeoxyuridine. See F oxuridine
Exercise-induced angina, 290t, 291 eno brate, 335t, 341, 342, 344t (FUdR, uorodeoxyuridine)
Exertiona angina, 290t, 291 gem brozi , 335t, 344t F uorometho one, 460t
Expectorants, 390t, 404t Fibrin c ot ormation, 325326, 326 F uorometho one acetate, 460t
Eye Fibrino ysis, 329, 329 F uorometo one, ophtha mic, 691t, 705t
anatomy, 699 Fibrino ytics, tPA, 323t, 333t 5-F uorouraci , 635t, 640641, 654t
autonomic innervation, 694 a tep ase, 323t, 329330, 329 , 333t activation pathways, 638
Eythropoiesis-stimu ating agents (ESAs), mechanisms o action, 323t agents combined with, 648, 649
405t, 418t retep ase, 323t, 333t dermato ogic, 708t, 724t, 725t
darbepoetin a a, 405t, 406409, 418t tenectep ase, 323t, 333t mechanisms and sites o action, 630 ,
epoetin a a, 405t, 406409, 418t Fibrogen ge , ophtha mic, 692t, 706t 635t, 638 , 640
indications, 406409 Fi grastim. See G-CSF ( grastim) mechanisms o resistance, 635t,
iron de ciency rom, 409 Finasteride, 442t, 456, 457, 458t 638b639b, 640641
mechanisms o action, 405t androgenic a opecia, 710t, 726t ophtha mic, 691t, 705t
toxicities and hazards, 409 First-pass e ect, 20b, 35, 3637 topica , precancerous skin esions,
e der y, 73t74t 640641
F nitrog ycerin and nitrates, 22, toxicities, 637t, 641, 654t
Faci itated di usion, 19b, 19 291, 299 uses, 637t, 654t
Faci itated transport, 23b Fish tapeworm, 537540 F uoxetine
Factor Xa inhibitors, direct, 323t, 333t 5-ASA, 498t, 503 , 509t on CYP2D6, 157
apixaban, 323t, 328, 331, 333t 5-F uorouraci (5-FU), 635t, 640641, 654t. Huntingtons disease, 248
mechanisms o action, 323t See also 5-F uorouraci pharmaco ogy, dopaminergic and seroto-
rivaroxaban, 323t, 328, 333t 5-LOX inhibitors, 390t, 403t nergic, 130t, 131133, 131 , 133t,
Famcic ovir, 605, 608 5-reductase inhibitors, 442t, 458t 146t
Famotidine dutasteride, 442t, 458t psychopharmaco ogy, 150t, 157, 174t
gastric acid disease, 490t, 496t nasteride, 442t, 456, 457, 458t F uphenazine, 150t, 174t
structure, 492 FK506-binding protein-12 (FKBP-12), F urandreno ide, 460t. See also
FDA 383, 384 Corticosteroids
chi dren, 71b72b F avoxate, 96t F urazepam, 176t, 180, 191t, 192t
drug approva process, 44 , 45t, F ecainide F urbipro en, ophtha mic, 691t, 705t
4648, 46t antiarrhythmic, 312t, 317t, 321t F utamide
pregnancy and breast eeding, 72b e ectrophysio ogic actions, 317t androgen receptor antagonist action, 442t,
FDA Sa ety and Innovation Act o F oxuridine (FUdR, uorodeoxyuridine), 454, 456, 457, 458t, 679t
2012, 72b 635t, 640641, 654t anticancer, targeted, 665t, 679t, 687t
Febuxostat, 369t, 370, 374t mechanisms and sites o action, 630 , prostate cancer, 442t, 454, 456, 457, 679t
Fe bamate, 226t, 237t 635t, 638 , 640 F uticasone, 390t, 403t

749
F uvastatin (sodium), 334t, 343t neonata , 70b prostag andin ana og, misoprosto , 490t,
F uvoxamine, 130t, 146t, 150t, 174t Funga in ections. See also speci c types 493, 494, 495, 496t
Fo ate de ciency, 412 , 413b, 414415 agents (See Anti unga agents) proton pump inhibitors, 490t, 491 , 492,
pregnancy, 416, 417 aspergi osis, invasive, 595, 596 493 , 496t
trimethoprim-su amethoxazo e, candidiasis dex ansoprazo e, 490t, 496t
543, 543 esophagea , 589 , 590t592t, esomeprazo e, 490t, 495, 496t
Fo ate metabo ism, 543 594596 gastroesophagea re ux disease,
Fo ic acid ana ogs (anti o ates), 635t, 653t ora , 467, 468 492, 493
Fo ic acid and derivatives, 406t, 412 , vagina , 594 iron de ciency rom, 495
415b, 420t coccidioida meningitis, 590, 592t593t, ansoprazo e, 490t, 493 , 496t
absorption and distribution, 412 595, 596 mechanisms o action, 490t, 491 , 495
de ciency, 412 , 413b, 414415 cryptococca meningitis, omeprazo e, 490t, 493 , 496t
o ic acid, 406t, 412 , 414417, 414b, 595, 596 pantoprazo e, 490t, 493 , 496t
415b, 420t eye, 701 rabeprazo e, 490t, 493 , 496t
o inic acid, 406t, 416, 417418, 420t mucormycoses, 589590 structures, 493
genera princip es, 414b onychomycosis, 595596, 721, 722 or u cers, 492, 493, 494t, 495
mechanisms o action, 406t tinea, 593594 vitamin B12 absorption, 492
vitamin B12 interactions, 410b, 412 Furosemide, 269t, 280, 284t, 304t Gastric emptying, neonates and in ants, 67b
Fo ic acid de ciency, 412 , 413b, 414415 Gastric secretion, regu ation, 491
pregnancy, 416, 417 G Gastroesophagea re ux disease (GERD),
Fo inic acid, 406t, 416, 417418, 420t GABA 400. See also Pu monary drugs
Fo ic e-stimu ating hormone (FSH), on GABAA receptor, 226 cimetidine, 491
423t, 429t metabo ism, antiseizure drugs, 225t226t, management guide ines, 493
o itropin , 423t, 430t 226b, 226 pregnancy, 494, 495
o itropin , 423t, 430t GABAA receptor proton pump inhibitors, 492
menotropin, 423t, 429t anesthetics on, 219221 treatments, 400
uro o itropin, 423t, 429t antiseizure drugs on, 225t, G-CSF ( grastim), 405t, 407t, 408 ,
Fo itropin , 423t, 430t 225t226t, 226 409410, 416, 417, 418t
Fo itropin , 423t, 430t binding sites, 178 indications, 409410, 416, 417
Fomepizo e, 41t, 42t, 65t, 178 umazeni , 179 mechanisms o action, 405t, 407t
Fomivirsen, 600t, 619t GABA on, 226 sites o action, 408
antiherpes, 600t, 602t, 609, 619t sedative-hypnotics on, 178 , 179 G-CSF, pegy ated recombinant
CMV retinitis, 609 Gabapentin, 205t, 225t, 237t (peg grastim), 405t, 407t, 408 ,
mechanisms o action and Ga antamine 409410, 418t
resistance, 602t A zheimers disease, 239t, 246, Ge tinib, 661t, 670672, 683t
Fondaparinux, 323t, 325, 333t 247t, 253t Gemcitabine, 630 , 636t, 654t
Formestane, 442t, 458t pharmaco ogy, 100t, 115t mechanisms and sites o action, 630 ,
Formotero Gancic ovir, 600t, 619t 636t, 638 , 640
pharmaco ogy, 125t, 144t antiherpes, 600t, 602t, 604 , 605, 605 , mechanisms o resistance, 635t, 639b
pu monary, 390t, 402t 609, 617, 618, 619t toxicities, 637t, 654t
Forward rate (k+1), 6b CMV retinitis, 609, 617, 618 uses, 637t, 654t
Foscarnet, 600t, 609612, 619t herpes simp ex virus, 605 Gem brozi , 335t, 344t
adverse e ects, 612, 617, 618 mechanisms o action and resistance, Gemtuzumab ozogamicin, 662t, 667t,
antiherpes, 600t, 602t, 604 , 605, 609612, 602t, 604 , 605 672t, 684t
617, 618, 619t Gang ionic b ocking drugs, 106t, 117t Genetic po ymorphisms
CMV retinitis, 609612 Gang ionic stimu ating agents, nicotine, 106t, CYP, 27t, 28, 31b
herpes simp ex virus, 605 108, 108b, 117t drug response, 10t, 1214, 13
mechanisms o action and resistance, Ganire ix, 423t, 429t, 665t, 687t Genotoxic e ects, 46t
602t, 604 , 610612 Gases, therapeutic, 211t, 223t224t. Gentamicin, 559566, 559b, 566t
Fosinopri , 273t, 285t See also speci c agents Giardia intestinalis, 500501
Fosphenytoin, 225t, 236t carbon dioxide, 211t, 223t Giardiasis, 531532
Fospropo o , 210t, 222t he ium, 211t, 224t diarrhea, 500501
Fractiona occupancy ( ), 7b, 8 nitric oxide, 211t, 223t metronidazo e, 530t, 532533, 535t
Frovatriptan, 130t, 147t oxygen, 211t, 223t G argine. See also Diabetes me itus
FUdR. See F oxuridine (FUdR, Gastric acid disease drugs, 490496 vs. ispro, 478
uorodeoxyuridine) antacids, 490t, 496t G atiramer acetate, 382t
Fu agonists, 4b, 4 , 7b, 9 bismuth, 490t, 496t G aucoma, 699700, 699
Fu e cacy, 4b, 4 cytoprotective agents, sucra ate, G ic azide, 470t, 472 , 479t
Fu vestrant, 442t, 458t 490t, 496t G imepiride, 470t, 472 , 477, 478, 479t
anticancer, 664t, 677t, 686t H1 receptor antagonists, 490t, 491 , 496t G imepiride + met ormin, 477, 478
estrogen antagonist, 442t, 458t cimetidine, 491, 492t, 494, 495 G ipizide, 470t, 472 , 479t
Fumagi in, 530t, 535t amotidine, 490t, 492 , 496t GLP-1 agonists, 471t, 472 , 476, 479t
Functiona antagonists, 4b mechanisms o action, 490t, 491 exenatide, 471t, 472 , 479t
Functiona iron de ciency, 409 nizatidine, 490t, 492 , 496t irag utide, 471t, 472 , 479t
Functiona ization, phase 1, 28b, 33 , ranitidine, 490t, 492 , 496t mechanisms o action, 471t, 472
35, 38 to erance, 491, 494, 495 G ucocorticoid receptors, 462

750
G ucocorticoids. See also speci c drugs and des ore in, 664t Guanabenz
drug classes GnRH, 664t, 680, 687t antihypertensive, 277t, 286t
anticancer, targeted, 664t, 679t, 686t gosere in, 664t, 687t pharmaco ogy, 119t, 140t
dexamethasone, 664t, 686t histre in, 664t, 687t Guanadre , 277t, 287t
prednisone, 664t, 679t, 686t eupro ide, 664t, 679t, 680, 687t Guan acine
carbohydrate and protein metabo ism, na are in, 664t, 687t antihypertensive, 277t, 286t
467, 468 triptore in, 664t, 687t pharmaco ogy, 119t, 140t
dependence, 505506, 507 pharmaco ogy and actions Gynecomastia treatment
dermato ogic, 707t, 722, 723, 724t androgen secretion, 425 gonadotropin, 427, 428
adverse e ects, 723, 724t GnRH, 422t, 429t, 680 testosterone, 456, 457
potency, 723t gosere in, 422t, 429t
triamcino one acetonide, 707t, histre in, 423t, 429t H
723t, 724t eupro ide, 423t, 425426, 429t H 1 receptor antagonists
triamcino one hexacetonide, 707t, na are in, 422t, 427, 428, 429t adverse e ects, 46, 347, 350
723t, 724t triptore in, 422t, 429t gastric acid disease, 490t, 491 , 496t
dexamethasone, 664t, 686t Gonadotropin-re easing hormone (GnRH) cimetidine, 491, 492 , 494, 495
dose tapering, 465 antagonists, 423t, 429t amotidine, 490t, 492 , 496t
drug interactions and dosing, 464 anticancer, 665t, 687t mechanisms o action, 490t, 491
drugs against (See Antig ucocorticoids) abare ix, 665t, 687t nizatidine, 490t, 492 , 496t
genera princip es, 464b cetrore ix, 665t, 687t ranitidine, 490t, 492 , 496t
gout, 369t, 370 degare ix, 665t, 687t to erance, 491, 494, 495
immunotherapeutic, 375t, 384t ganire ix, 665t, 687t uses, 6
ophtha mic, 691t, 705t pharmaco ogy and actions H 1 receptor antagonists, rst generation,
dexamethasone, 691t, 705t cetrore ix, 423t, 429t 1922, 346t, 352354t
di uprednate, 691t, 705t ganire ix, 423t, 429t adverse e ects, 8990, 347
uocino one, 691t, 705t Gonadotropins brompheniramine ma eate, 346t, 354t
uorometo one, 691t, 705t or gynecomastia, 427, 428 carbinoxamine, 346t, 353t
otepredno , 691t, 705t neuroendocrine contro , 447 ch orpheniramine, 346t, 354t
predniso one, 691t, 705t secretion, 447 c emastine umarate, 346t, 353t
rimexo one, 691t, 705t Gosere in cyc izine, 346t, 350, 352, 354t, 502t
triamcino one, 691t, 705t anticancer, 664t, 687t cyproheptadine, 346t, 354t
osteoarthritis and diabetes type II, pharmaco ogy and actions, 422t, 429t dermato ogic, 718t
467, 468 Gout, 369t, 370 dimenhydrinate, 346t, 353t
prednisone, 664t, 679t, 686t Gout drugs, 368b, 369370, 369t, 374 diphenhydramine, 36, 346t, 347350
rheumatoid arthritis, 463465 g ucocorticoids, 369t, 370 doxepin, 346t, 353t
topica , 465 microtubu e disruptors, 369t drowsiness, 1920
toxicities, 464b co chicine, 369t, 370, 373t e der y, 69
G ucose-6-phosphate (G6PD), 586 NSAIDs, 369t hydroxyzine, 346t, 354t
G ucose-6-phosphate (G6PD) de ciency urate oxidase, recombinant, 369t ionization and ipid membrane
dapsone with, 584, 585586 rasburicase, 369t, 374t di usion, 1920, 20
primaquine with, 527, 528 uricosuric agents, 369t, 370 mechanisms o action, 346t, 347
G ucuronidation, neonata , 70b probenecid, 369t, 370, 374t mec izine, 346t, 350, 352, 354t
G ucuronosy trans erase, in ants, 68 xanthine oxidase inhibitors, 369t motion sickness, 350, 352
G yburide, 470t, 472 , 479t a opurino , 369t, 370372, 373t phenindamine, 346t, 354t
G ycerin diuretic, 268t, 283t ebuxostat, 369t, 370, 374t pregnancy and actation, 348
G ycopeptides GPIIb/IIIa antip ate et agents, 323t, promethazine, 346t, 350352, 354t
teicop anin, 569t, 570 , 577t 324325, 332t pyri amine, 346t, 353t
vancomycin, 569t, 570 , abciximab, 323t, 324325, vs. second generation, 347b348b
572575, 577t 331332, 332t tripe ennamine, 346t, 353t
G ycopyrro ate epti batide, 323t, 324325, 332t H 1 receptor antagonists, second
bowe antispasmodic, 498t, 508t mechanisms o action, 323t generation, 347t, 355t
pharmaco ogy, 96t, 113t tiro ban, 323t, 324325, 332t acrivastine, 347t, 355t
GM, 405t, 407t, 408 , 409410, 418t G proteincoup ed receptors (GPCRs), 3t, 4 advantages, 348
GnRH. See Gonadotropin-re easing Granisetron, 130t, 147t aze astine, 347t, 355t
hormone (GnRH) Graves disease, 432t, 434t, 435439 cetirizine, 347t, 348, 348b, 351352, 355t
Goiter, 437, 438 Gray baby syndrome, 68 dermato ogic, 718t
Go imumab Griseo u vin, 588t, 593596, 598t, 721, 722 des oratadine, 347t, 355t
immunotherapy, 376t, 386t Growth actor signa ing, 670 . See also speci c ebastine, 347t, 355t
rheumatoid arthritis, 368t growth actors and agents exo enadine, 347t, 355t
Gonadotropin-re easing hormone (GnRH), Growth hormone (GH), 423, 424 vs. rst generation, 347b348b
422t, 429t, 680 antagonist drug, pegvisomant, 422t, 424, ketoti en umarate, 347t, 355t
anticancer, 664t, 680, 687t 425 , 428t evocabastine, 347t, 355t
Gonadotropin-re easing hormone (GnRH) receptor antagonism, 423, 425 evocetirizine, 347t
agonists, 680 recombinant, 422t, 427, 428, 428t oratadine, 347t, 348, 348b,
anticancer, 664t, 679680, 687t secretion and actions, 423, 424 351352, 355t
busere in, 664t, 687t Guai enesin, 390t, 404t mechanisms o action, 347t

751
mizo astine, 347t, 355t adrenergic receptor agonists, 311t Hematopoietic agents, 405420,
nonionization and brain concentrations, dobutamine, 302t, 307308, 405t406t, 419t
20, 38 308t, 311t copper supp ement, 406t, 419t
o opatadine, 347t, 355t dopamine, 302t, 307308, 308t, 311t cupric su ate, 406t, 419t
pregnancy and actation, 348, 348b, mechanisms o action, 302t cytokinece interactions, 408
351352 mechanisms o action, heart, erythropoiesis-stimu ating agents,
H 1 receptors, 4 307308 405t, 418t
H 2 receptor antagonists adrenergic receptor antagonists, 286t, darbepoetin a a, 405t, 406409, 418t
dermato ogic, 718t 304, 310t (See also adrenergic epoetin a a, 405t, 406409, 418t
e der y, 73b receptor antagonists) indications, 406409
mechanisms o action, 347t, 491, 491 bisopro o , 301t, 309, 310, 310t iron de ciency rom, 409
structure, 491, 492 carvedi o , 301t, 309, 310, 310t mechanisms o action, 405t
to erance, 491, 494, 495 mechanisms o action, 301t toxicities and hazards, 409
H 2 receptors, 4 metopro o , 301t, 303304, 306, 310t hematopoietic growth actors, 407t
H 3 receptor antagonists, 347t Braters a gorithm, 270t, 273t, 276 , 285t erythropoietin, 407t, 408
H 3 receptors, 4 diuretics, 284t285t, 301t, 304, 311t (See FL -3 igand, 407t, 408
H 4 receptor antagonists, 347t also Diuretics) inter eukins, 407t, 408
H 4 receptors, 4 ep erenone, 269t, 280281, 285t mechanisms o action, 407t, 408
Haemophilus in uenzae meningitis prophy- mechanisms o action, 301t302t sites o action, 408
axis, ri amycins, 578t, 579 580 , Na+-K+-A Pase inhibitors, 303 , 311t stem ce actor, 407t, 408
581586, 586t. See also Ri amycins digoxin, 301t, 306, 307, 311t iron supp ements, ora , 406t, 419t
Ha cinonide, 460t. See also Corticosteroids mechanisms o action, 301t errous aspartate, 406t, 413414, 419t
Ha - i e (t 1/2), 34b, 37 , 39b natriuretic peptide receptor agonists errous umarate, 406t, 413414, 419t
e der y, 77 mechanisms o action, 302t errous g uconate, 406t, 413414, 419t
Ha -maxima e ective concentration (EC50), nesiritide, 302t, 311t errous sa ts, 406t, 413414, 419t
6 , 7b, 9 nesiritide, 269t, 280, 284t errous succinate, 406t, 413414, 419t
Ha -time, context-sensitive, anesthetic, phosphodiesterase 3 inhibitors, 307, 309, errous su ate, 406t, 413414, 419t
214, 214 310, 311t in ants and chi dren, 413
Ha operido , 151t, 159, 163 , 171t, 175t inamrinone, 302t, 308, 308t, 311t mechanisms o action, 405t
Ha othane, 210t, 215t, 222t mechanisms o action, 302t, 308 optimizing e ects, 413414
Ha progin, 598t mi rinone, 302t, 308, 308t, 311t po ysaccharide errihydrite comp ex,
HDAC inhibitors, 645t, 660t pharmaco ogy, 308t 406t, 413414, 419t
romidepsin, 645t, 660t rationa e, 303b, 305 pregnancy, 413414
vorinostat, 645t, 646t, 660t sites o action, 302 , 303 iron supp ements, parentera ,
HDL-C eve s, 337t to vaptan, 279t, 281, 288t 406t, 419t
HDL-C metabo ism, 335 vasodi ators, 304t, 311t erumoxyto , 406t, 419t
Heart ai ure arteria , 278t, 287t (See also Vasodi a- iron dextran, 406t, 419t
A rican-American, 306, 306b, 309, 310 tors, arteria ) iron sucrose, 406t, 419t
BiDi , 306, 306b mechanisms o action, 301t mechanisms o action, 406t
Braters a gorithm, 270t, 273t, Heavy meta che ators, 66t sodium erric g uconate, 406t, 419t
276 , 285t Heavy meta s, 41t, 43t, 65t66t mye oid growth actors, 405t, 407t, 418t
characteristics and causes, 302b, 302 Helicobacter pylori trip e therapy, 492, G-CSF, 405t, 407t, 408 , 409410, 416,
comorbidities, 302b 494t, 495 417, 418t
hypertension and diabetes risk, 304305 He ium gas, therapeutic, 211t, 224t G-CSF, pegy ated recombinant, 405t,
ibupro en risk, 304 He minth in ection drugs, 536541 407t, 408 , 409410, 418t
pu monary congestion and periphera benzimidazo es, 540t GM-CSF, 405t, 407t, 408 ,
edema, 305306 a bendazo e, 536, 540t 409410, 418t
stages, 303, 303b, 305 mebendazo e, 536, 540t indications and choice, 409410
symptoms, 302b diethy carbamazine, 538540, 540t mechanisms o action, 405t, 407t
verapami on, 306 doxycyc ine, 540t sites o action, 408
Heart ai ure drugs. See also speci c drugs and ivermectin, 537540, 540t thrombopoietic growth actors, 405t, 406t,
drug classes metri onate, trich or on, 541t 407t, 419t
a dosterone receptor antagonists, 311t nic osamide, 539, 540, 541t e trombopag, 406t, 419t
ep erenone, 302t, 311t oxamniquine, 541t IL-11 (opre vekin), 406t,
mechanisms o action, 302t piperazine, 541t 410411, 419t
spirono actone, 302t, 307, 311t pork tapeworm, 539, 540 mechanisms o action, 405t,
or angina, 294t praziquante , 537, 539, 540, 540t 407t, 408
angiotensin-converting enzyme inhibitors, pyrante pamoate, 536, 539, romip ostim, 406t, 417, 418, 419t
272 , 276 , 279, 285t, 303304, 310t 540, 541t vitamins, B12, 406t, 420t
(See also Angiotensin converting He minth in ections absorption and distribution, 412
enzyme (ACE) inhibitors) Brugia malayi, 538 cyanocoba amin, 406t, 420t
mechanisms o action, 270t, 273t, sh tapeworm, 537, 538539 de ciency, 411b, 412 , 413b, 414415,
274276, 275 , 301t incidence, 537 417418
angiotensin receptor b ockers, 273t, 274, onchocerciasis, 537540 o ate interactions, 410b, 412
279, 280, 285t, 301t, 310t (See also pinworm, 536, 539, 540 genera princip es, 411b412b
Angiotensin (A 1) receptor schistosomiasis, 537, 539, 540, 540t hydroxycoba amin, 406t, 420t
b ockers (ARBs)) Wuchereria bancrof i, 538 mechanisms o action, 406t, 420t
752
Hematopoietic agents (Cont.) treatment, 260262 Homatropine hydrobromide, 96t, 114t
vitamins, o ic acid and derivatives, 406t, withdrawa , 260, 261t Hormesis, 6b
412 , 415b, 420t Herpes simp ex virus (HSV) in ection Hormone rep acement therapy, 441t442t,
absorption and distribution, 412 agents (See Antiherpes agents) 445t448t, 446448,
de ciency, 412 , 413b, 414415 eye, 701 455457, 458t
o ic acid, 406t, 412 , 414417, 414b, Hexamethy endiamine, 627t, 651t brand names and ormu ations,
415b, 420t mechanisms o action, 627t, 445t446t
o inic acid, 406t, 416, 417418, 420t 628b629b, 630 estrogen/progestin, 446t, 455,
genera princip es, 414b mechanisms o resistance, 627t, 457, 458t
mechanisms o action, 406t 632633, 632b ethiny estradio /progesterone ormu a-
vitamin B12 interactions, 410b, 412 toxicities, 632b633b, 651t tions, 441t, 446t, 447, 457t
vitamins, pyridoxine, 406t, 419t High-dose methotrexate with eucovorin res- indications and e ects, 456, 457
mechanisms o action, 406t cue (HDM-L), 635t, 638640, 653t mechanisms o action, 441t442t
pyridoxine, 406t, 419t mechanisms and sites o action, 620 , risks, 447448
ribo avin, 406t, 419t 635t, 637 , 638 Horners syndrome, 698t
Hematopoietic growth actors, 407t. See also mechanisms o resistance, 638b, 640 Human chorionic gonadotropin (hCG), 423t,
Hematopoietic agents; pharmacogenetics, 635t, 637 , 638640, 639 427, 428, 430t
speci c types toxicities, 653t choriogonadotropin a a, 423t, 427,
erythropoietin, 407t, 408 uses, 637t 428, 430t
FL -3 igand, 407t, 408 High (narrow) speci city, 5b, 9 Human immunode ciency virus (HIV),
inter eukins, 407t, 408 Histamine 614619. See also Antivira agents;
mechanisms o action, 407t, 408 actions, 3 speci c drugs and drug classes
sites o action, 408 in a ergic reactions, 349350 atazanavir-ritonavir, 617619
stem ce actor, 407t, 408 CNS neurotransmitter, 45 combination therapy, 518, 519, 617, 618
Heme biosynthesis, 58 structure, vs. H 2 receptor drug resistance, 614
Hemog obin A1c, 473 antagonists, 492 en uvirtide, 617618
or monitoring diabetes me itus, 473, trip e response o Lewis, 347b, 348 opinavir, 618, 619
474t, 475476 Histamine receptor antagonists and mast ce rep icative cyc e, 608
Henderson-Hasse ba ch equation, 19b stabi izers, 3. See also H 1 receptor treatment princip es, 614
Heparin, 323t, 325, 327 , 333t antagonists; speci c types treatment recommendations, 614
Heparin-induced thrombocytopenia (HI ), actions and adverse e ects, 46 zidovudine- amivudine-abacavir, 604 ,
325, 330, 331 active ingredient, 3 608 , 614615
Heparinoid anticoagu ants, parentera , 323t, a ergy, 351352 Human so ute carrier (SLC) super ami y
325, 333t drug c ass, 3 transporters, 23b24b, 25t26t
ardeparin, 323t, 325, 333t ophtha mic, 691t, 705t Huntingtons disease, 248249, 248
da teparin, 323t, 325, 333t aze astine, 691t, 705t tetrabenazine or, 239t, 249, 253t
enoxaparin, 323t, 325, 333t bepotastine, 691t, 705t Hya uronate, ophtha mic, 692t, 706t
ondaparinux, 323t, 325, 333t cromo yn sodium, 691t, 705t Hydantoins, antiseizure, 225t, 236t
heparin, 323t, 325, 327 , 333t emedastine di umarate, 691t, 705t osphenytoin, 225t, 236t
ovenox, 325326, 326 epinastine, 691t, 705t phenytoin, 225t, 231, 234, 235, 236t
ow-mo ecu ar-weight heparins, 323t, ketoti en umarate, 691t, 705t Hydra azine, 278t, 287t
324325, 325 , 333t odoxamide tromethamine, 691t, 705t Hydroch orothiazide, 269t, 271, 273274,
mechanisms o action, 323t nedocromi , 691t, 705t 282, 283, 284t
nadroparin, 323t, 325, 333t o opatadine, 691t, 705t Hydrocodone, 195t, 200t, 208t
tinzaparin, 323t, 325, 333t pemiro ast, 691t, 705t structure and activity o , 464
Hepatic metabo ism and c earance. See also Histamine receptors, 3t, 4 to erance, 264, 265
speci c drugs and drug classes Histre in Hydrocortisone, 460t, 464, 464 . See also
e der y, 75b, 76 , 76t77t, 77 anticancer, 664t, 687t Corticosteroids
Hepatic microsoma enzymes. See also GnRH agonist pharmaco ogy, 423t, 429t adrena insu ciency, 461 , 465
speci c types HMG-CoA reductase, 337 drug interactions and dosing, 464
antiseizure drug interactions, 227t HMG-CoA reductase inhibitors (statins), genera princip es, 464b
Hepatitis 334t, 343t mechanisms o action, 460t
agents (See Antihepatitis agents) adverse e ects, 338 physio ogic unctions and pharmaco ogic
u minant, 231 atorvastatin, 334t, 343t e ects, 462t463t
hepatitis B, 61 choice, 342, 343 pu monary, 390t, 403t
hepatitis C, chronic, 613614 uvastatin sodium, 334t, 343t toxicities, 464b
Her2/neu, 672673 ovastatin, 334t, 341, 342, 343t trade names and preparations, 460t
Heroin. See also Opioid pharmaco ogy mechanisms o action, 334t, 337 Hydro umethiazide, 269t, 284t
entany with, 260 other drugs with, 339 Hydro ysis reactions, drug metabo ism, 32t
vs. methadone responses, 260, 260 pitavastatin ca cium, 334t, 343t Hydromorphone, 194t, 195t, 200t, 208t
overdose, 203204 pravastatin ca cium, 334t, 343t Hydroquinone, or hyperpigmentation,
to erance, 260 pregnancy risks, 340 710t, 727t
toxidrome, 53t rosuvastatin ca cium, 334t, 338, 343t Hydroxych oroquine
Heroin addiction, 259262, 260 , 261t simvastatin, 334t, 343t dermato ogic, 708t, 725t
addictabi ity, 259261 Hodgkins ymphoma, ABVD regimen, 630 , ma aria chemotherapy, 521t, 522t,
overdose, 260 643t, 645647 523524, 526, 529t
to erance, 260 Homatropine, ophtha mic, 690t, 695t, 704t rheumatoid arthritis, 368t
753
Hydroxych oroquine su ate, 522t eszopic one, 177t, 192t human chorionic gonadotropin, 423t, 427,
1-Hydroxycho eca ci ero , 481t, 488t za ep on, 177t, 192t 428, 430t
Hydroxycoba amin, 406t, 414415, zo pidem, 177t, 178179, 192t choriogonadotropin a a, 423t, 427,
417418, 420t benzodiazepine receptor antagonists, 428, 430t
Hydroxyethy ce u ose, ophtha mic, umazeni , 41t, 42t, 65t, 177t, insu in- ike growth actor-1, mecasermin,
692t, 706t 179180, 192t 422t, 428t
Hydroxyprogesterone caproate, 679t benzodiazepines, 176t, 178b, 180, uteinizing hormone, utropin a a,
Hydroxyprophy methy ce u ose, 189190, 191t192t (See also 423t, 430t
ophtha mic, 692t, 706t Benzodiazepines) organs, hormones, and targets, 426
Hydroxypropy ce u ose, ophtha mic, carisoprodo , 177t, 193t posterior pituitary hormones, 423t, 429t
692t, 706t ch ora hydrate, 177t, 193t arginine vasopressin, 423t, 424 , 429t
Hydroxypropy methy ce u ose, ophtha mic, c omethiazo e, 177t, 193t (See also Vasopressin)
692t, 706t etomidate, 177t, 193t oxytocin, 423t, 424 , 430t
11-Hydroxysteroid dehydrogenase, 464 , GABAA receptor, 178 , 179 pro actin-secreting pituitary
467, 468 insomnia categories, 178b, 179 adenoma, 426
Hydroxyurea (HU), 630 , 645t, ong-term use, 179 somatostatin ana ogs, 422t, 428t
646t, 659t me atonin congeners, rame teon, 177t, anreotide, 422t, 428t
Hydroxyzine, 346t, 354t 189, 190, 192t octreotide, 422t, 428t
Hyoscyamine, 498t, 508t meprobamate, 177t, 193t Hypothyroidism, 431t, 432434, 432t,
Hyperca cemia, 483484 para dehyde, 177t, 193t 433 , 434t
bisphosphonates, 483 propo o , 177t, 193t diagnosis, 432, 432t, 433
pamidronate, 481t, 488t toxidrome, 53t iodine-induced, 432, 434t, 435t, 438, 439
zo edronate, 481t, 485, 488t Hypoca cemia, cinaca cet, 481t, 488t evothyroxine or, 431t, 432434, 433 ,
ca citonin, 480t, 481t, 483484, Hypoch orhydria, age-re ated, 73b 434t, 437439, 439t
483 , 488t Hypog ycemia subc inica , 437439
ung cancer, 483484, 486, 487 drugs promoting, 477t
vitamin D and ana ogs or, recurring, 476 I
481t, 488t Hypog ycemia agents, 471t, 479t I, 431t, 435, 437439, 440t
131

Hypercortiso ism, 465466 diazoxide, 471t, 472 , 476, 479t Ibandronate, 481t, 485, 488t
Hyperg ycemia. See also Antidiabetic agents; g ucagon, 471t or osteoporosis, 481t, 485, 488t (See also
Diabetes me itus mechanisms o action, 471t Bisphosphonates)
drugs promoting, 477t Hypogonadism, 456, 457 Ibupro en
Hyperparathyroidism agents, Hypotha amic-pituitary axis, 422430, 461 . on antihypertensive drugs, 282, 283
481t, 488t See also speci c hormones and heart ai ure risk, 304
Hyperphosphatemia agents regulators menstrua cramping, 364
anthanum, 481t, 488t organs, hormones, and targets, 426 pain target and site o action, 205t
seve amer, 481t, 488t Hypotha amic-pituitary axis agents, Ibuti ide
Hyperpigmentation agents, 710t, 727t 422430 antiarrhythmic, 312t, 317t, 322t
aze aic, 710t, 727t dopamine agonists, 422t, 428t e ectrophysio ogic actions, 317t
hydroquinone, 710t, 727t bromocriptine, 422t, 426, 428t Icatibant, 347t, 355t
mequino , 710t, 727t cabergo ine, 422t, 426, 428t Idarubicin, 643t, 658t
monobenzone, 710t, 727t pro actin-secreting pituitary mechanisms o action, 630 , 643t,
Hypersensitivity reactions. See also adenoma, 426 645647
speci c drugs quinago ide, 422t, 426, 428t mechanisms o resistance, 645b
immediate, 349350 o ic e-stimu ating hormone, Idiosyncratic reactions, 46t
penici ins, 550 , 552, 554, 555 423t, 429t IDL-C metabo ism, 335
Hypertension o itropin , 423t, 430t Idoxuridine, 600t, 602t, 605606, 619t
criteria, 270t o itropin , 423t, 430t I os amide, 627t, 632t, 651t
diabetes with, 273274 menotropin, 423t, 429t activation, in vivo, 631, 631
drugs (See Antihypertensive drugs) uro o itropin, 423t, 429t mechanisms o action, 626, 627t, 628,
icorice, 467, 468 gonadotropin-re easing hormone agonists 628b, 630
pregnancy, 277278, 278 on androgen secretion, 425 mechanisms o resistance, 627t,
progression, 269270, 270t GnRH, 422t, 429t, 680 632633, 632b
Hyperthyroidism, 432t, 434t, gosere in, 422t, 429t toxicities, 631632, 632b633b,
435439 histre in, 423t, 429t 632t, 651t
Hypertrophic cardiomyopathy, angina eupro ide, 423t, 425426, 429t uses, 629t
drugs, 294t na are in, 422t, 427, 428, 429t IgE antibodies, in anaphy axis, 56
Hyperuricemia, 370 triptore in, 422t, 429t IL-1, 407t, 408
Hypnotics and sedative-hypnotics, gonadotropin-re easing hormone antago- IL-1 inhibitors, 376t, 386t. See also under
176193, 191t193t. See also nists, 423t, 429t Bio ogica immunosuppressants
speci c types cetrore ix, 423t, 429t anakinra, 376t, 386t
adverse e ects, 179 ganire ix, 423t, 429t canakinumab, 376t, 386t
barbiturates, 177t, 180182, 181t, 192t growth hormone antagonist, pegvisomant, ri onacept, 376t, 386t
(See also Barbiturates) 422t, 424, 425 , 428t IL-2, 407t, 408
benzodiazepine receptor agonists, nove , growth hormone, recombinant, IL-3, 407t, 408
177t, 192t 422t, 427, 428, 428t IL-4, 407t, 408

754
IL-5, 407t Immunity monoc ona antibody, anti-CD52,
IL-6, 407t, 408 adaptive, 376b a emtuzumab, 376t, 378, 386t
IL-7, 407t innate, 376b po yc ona antibodies, immune
IL-8, 407t Immunization, passive, 377t378t, 381t g obu in preparations, 376t, 377 ,
IL-9, 407t Immunomodu ators, 381t382t. See also 377t378t, 378, 385t
IL-10, 407t speci c drugs and drug classes risks, 379
IL-11 Immunostimu ants, 381t ca cineurin inhibitors, 375t, 385t
mechanisms o action, 407t, 408 anthracenedione derivative, cyc osporine, 368t, 375t, 377 , 380,
therapeutic, 406t, 410411, 419t mitoxantrone, 382t 382383, 385t
IL-12, 407t Baci us Ca mette-Guerin, 381t mechanisms o action, 375t, 377
I operidone, 151t, 170t, 175t monoc ona antibody, nata izumab, 382t tacro imus, 375t, 377 , 383, 385t
I oprost, 391t, 404t passive immunization, 377t378t, 381t Crohns disease, 381
Imatinib (mesy ate), 661t, 666669 recombinant cytokines, 381t g ucocorticoids, 375t, 384t
adverse e ects, 669 a des eukin, 381t immuno ogica to erance and costimu atory
anticancer, targeted, 661t, 666669, 681, inter eron--2b, 381t b ockade, 380b, 380
682, 683t inter eron--1a, 381t, 382 immunosuppressants and
cancers, 669 inter eron--1b, 381t, 382 anti-in ammatories,
chronic mye ogenous eukemia, Ph +, 661t, inter eron--1b, 381t dermato ogic, 709t, 724t, 726t (See
666668 tha idomide and ana ogs also Immunosuppressants and
mechanisms o resistance, 666668, ena idomide, 381t anti-in ammatories,
681, 682 tha idomide, 381t dermato ogic)
pharmacodynamics, 1415 universa APLs, g atiramer acetate, 382t mechanisms o action, 375t376t
Imidazo es and triazo es, 588t, 589 , Immunosuppressants and anti-in ammatories, organ transp ant, 378379
590, 597t dermato ogic, 709t, rheumatoid arthritis, severe,
CYP interactions, 590, 590t 724t, 726t 380381
drug interactions, 590t592t b eomycin, 709t, 724t, 726t Impetigo, 714
uconazo e, 588t, 590, 595, 596, dapsone, 709t, 724t, 726t Imp antab e cardioverter-de bri ator (ICD)
597t, 719 doxorubicin, 709t, 726t ventricu ar bri ation, 316
itraconazo e, 597t imiquimod, 709t, 724t, 726t ventricu ar tachycardia, 318
posaconazo e, 597t mycopheno ate mo eti , 709t, 724t, 726t Inactive con ormation, 4, 5
voriconazo e, 588t, 589 , 590t592t, tha idomide, 709t, 724t, 726t Inamrinone, 302t, 308, 308t, 311t
592593, 595, 596, 597t vinb astine, 709t, 724t, 726t Indacatero , 390t, 402t
Imidazo es and triazo es, topica , 588t, Immunosuppressants, ocu ar, 691t, 705t Indapamide, 269t, 284t
597t598t 5- uorouraci (FU), 691t, 705t Indinavir, 601t, 603t, 608 , 609 , 621t
butoconazo e, 597t cyc osporine A, 691t, 705t Indomethacin, 357t, 361t, 373t
c otrimazo e, 597t mitomycin, 691t, 705t Indoramin, 120t, 141t
econazo e, 597t Immunotherapeutic agents, 375388 Inducing agents, anesthesia, 212 , 213214,
ketoconazo e, 598t antipro i erative/antimetabo ic, 213 , 213t. See also Anesthetic
miconazo e, 594, 597t 375t, 385t agents, parentera
oxiconazo e, 598t azathioprine, 368t, 375t, 380, 385t Induction speed, anesthesia, 215216,
sertaconazo e, 598t evero imus, 375t, 385t 215t, 216
su conazo e, 598t MPA, 375t, 385t In ants. See Chi dren
terconazo e, 597t mycopheno ate mo eti , 375t, 383, In estation agents, dermato ogic,
tioconazo e, 597t 384, 385t 708t, 725t
Iminosti benes, antiseizure, 225t, 236t siro imus, 375t, 377 , 383, 384, 385t benzy a coho , 708t, 725t
carbamazepine, 225t, 226 , bio ogica immunosuppressants crotamiton, 708t, 725t
232235, 236t adverse e ects, 378b ivermectin, 708t, 725t
oxcarbazepine, 225t, 236t anti- NF reagents, 376t, 386t indane, 708t, 725t
Imipenem, 558t ada imumab, 368t, 376t, 381, ma athion, 708t, 725t
Imipenem-ci astatin, 554555 383, 384, 386t permethrin, 708t, 725t
Imipramine, 150t, 173t certo izumab pego , 376t, 381, 386t In ammatory bowe disease agents,
Imiquimod etanercept, 376t, 380, 386t 498t499t, 509t
antivira , 601t, 602t, 621t go imumab, 376t, 386t ada imumab, 499t, 509t
dermato ogic, 709t, 724t, 726t in iximab, 368t, 376t, 380, 381, 386t azathioprine, 498t, 502503,
Immediate hypersensitivity reactions, IL-1 inhibitors, 376t, 386t 503 , 509t
349350 anakinra, 376t, 386t ba sa azide, 498t, 503 , 509t
Immune g obu in preparations, 376t, 377 , canakinumab, 376t, 386t budesonide, 498t, 509t
377t378t, 378, 385t ri onacept, 376t, 386t certo izumab pego , 499t, 509t
agents, synonyms, and origins, 377t378t LFA-1 & LFA-3 inhibitors cyc osporine, 499t, 509t
mechanisms o action, 376t a e acept, 376t, 386t in iximab, 499t, 509t
toxicities, 385t e a izumab, 376t, 386t mechanisms o action, 498t499t
uses, 377b, 385t monoc ona antibody, anti-CD3, mesa amine, 498t, 503 , 509t
Immune response, 376b. See also Hypersensi- muromonab-CD3, 376t, 378, 385t methotrexate, 499t, 509t
tivity reactions; speci c types monoc ona antibody, anti-CD25, 376t, nata izumab, 499t, 509t
Immune (idiopathic) thrombocytopenic 379, 386t o sa azine, 498t, 503 , 506, 507, 509t
purpura (I P), 406t, 417, basi iximab, 376t, 386t su asa azine, 381, 498t, 502, 503 , 505,
418, 419t dac izumab, 376t, 386t 507, 509t
755
In ammatory response, 357b g yburide, 470t, 472 , 479t Iontophoresis, 203
In iximab mechanisms o action, 470t, 477, 478 Ion trapping, 20b
Crohns disease, 381, 503 Integrase inhibitor Ipratropium (bromide)
dermato ogic, 709t, 714t, 726t mechanisms o action and resistance, 603t COPD, 390t, 402, 403t
immunotherapy, 368t, 376t, 380, 381, 386t ra tegravir, 601t, 603t, 611 , 623t pharmaco ogy, 96t
in ammatory bowe disease, 499t, 509t Interactions. See Drug interactions; pu monary, 390t, 402, 403t
rheumatoid arthritis, 368t, 376t, 380, 386t speci c drugs Irbesartan, 273t, 285t
In uenza agents. See Anti-in uenza agents Interceptor mo ecu es, 62 Irinotecan (CP -111), 643t, 649651, 657t
Inha ed administration. See also speci c drugs Inter erons (IFNs), antihepatitis, 600t602t, mechanisms o action, 630 , 643t,
and drug classes 606 , 620t 649651
e der y, 74t adverse e ects, 614 mechanisms o resistance, 630 , 643t,
Inha ers, combination, 395, 397, 398, hepatitis C, chronic, 613614 644b645b, 649651
401402 IFN di erences, 613614 Iron
Inhibitory concentration (IC), mechanisms o action and resistance, recommended dietary a owances,
513514, 514 602t, 606 , 613 nonvegetarians, 411t
Inhibitory postsynaptic potentia (IPSP), 86, Inter eron--2b, 381t requirement, dai y, 411t
86 , 106 Inter eron--1a, 381t, 382 Iron de ciency
Inhibitory sigmoid Emax curve, 513, 513 , Inter eron--1b, 381t, 382 unctiona , 409
514, 514 Inter eron--1b, 381t rom proton pump inhibitors, 495
Innate immunity, 376b Inter eukin-2 (IL-2) receptor agonists, Iron dextran, 406t, 419t
Inotropic agents. See also Digoxin; anticancer, 663t, 685t Iron poisoning, 5455
Dobutamine; Dopamine (DA); a des eukin, 663t, 682, 683, 685t de erasirox, 41t, 65t
Phosphodiesterase 5 (PDE5) deni eukin di itox, 663t, 671t, 685t de eroxamine, 42t, 55, 65t
inhibitors mechanisms o resistance, 663t Iron sucrose, 406t, 419t
mechanisms o action, 307308 Inter eukins. See also IL entries Iron supp ements, ora , 406t, 419t
pharmaco ogy, 3034, 308t hematopoietic growth actors, 407t, 408 errous aspartate, 406t, 413414, 419t
INR, 328, 330b mechanisms o action, growth actor, 407t errous umarate, 406t, 413414, 419t
Insecticides. See also speci c types Intestina bypass surgery, 486, 487 errous g uconate, 406t, 413414, 419t
a dicarb, 100t, 115t Intrace u ar enzymes, 3t errous sa ts, 406t, 413414, 419t
anticho inesterase, 102, 102b, 103 , Intramuscu ar administration, 21t errous succinate, 406t, 413414, 419t
103t, 104 e der y, 74t errous su ate, 406t, 413414, 419t
antidotes, 40t, 41t, 42t, 65t, 100t, 102 neonates, in ants, and chi dren, 67b in ants and chi dren, 413
carbamate, 100t, 102, 115t Intranasa administration, e der y, 74t mechanisms o action, 405t
treatment, 42t, 65t Intravenous administration, 21t optimizing e ects, 413414
carbary , 100t, 115t Intrinsic ce u ar responses, 2b po ysaccharide errihydrite comp ex, 406t,
diazinon, 100t, 103t, 115t Intrinsic sympathomimetic activity (ISA), 413414, 419t
ma athion, 100t, 102t, 115t 15, 16 pregnancy, 413414
organophosphate, 100t, 102, 102b, 103 , Inverse agonists, 4, 4b, 4 , 9, 16, 17 Iron supp ements, parentera , 406t, 419t
103t, 104 , 115t In vitro binding studies, 8 erumoxyto , 406t, 419t
propoxur, 100t, 115t Iodide, 431t, 440t iron dextran, 406t, 419t
Insomnia Iodine iron sucrose, 406t, 419t
categories, 178b, 179 drugs with, 434t mechanisms o action, 406t
zo pidem, 177t, 178179, 192t high eve s, 437, 438 sodium erric g uconate, 406t, 419t
Insu in hypothyroidism rom, 435t, 438, 439 ISDN, 289t, 300t
g argine, 476 radioactive, 438439 ISMN, 289t, 298, 299, 300t
vs. ispro, 478 radioactive (131I), 431t, 435, Isocarboxazid, 130t, 147t, 150t, 173t
hospita ized, 477, 478 437439, 440t Isoetharine, 125t, 143t
mechanisms o action, 470t, 471 , supp ementation, post-thyroidectomy, Iso urophate, 103t, 104
472 , 473 437, 438 Iso urane, 210t, 215t, 222t
secretion regu ation, 470t, 472 Iodine de ciency (goiter), 437, 438 Isoniazid
sensitivity, 477, 478 Ion channe s, 3, 3t, 86, 86 88 , 89t. See also adverse e ects, 582, 585, 586
signa ing pathways, 471 speci c types and drug classes drug interactions, 582, 583t
Insu in- ike growth actor-1 (IGF-1), Ca , 86, 87 , 89t
2+
mechanisms o action, 578t, 580
422t, 428t C , 86 tubercu osis, 578t, 580 , 582, 583t, 587t
Insu in- ike growth actor-1 (IGF-1) receptor K+, 86, 87 vitamin B6 with, 582
signa ing pathway, 666 igand-gated receptor structure, 88 Isoniazid-resistant tubercu osis, 583
Insu in secretagogues Na+, 86, 87 Isoprostanes, 358
nonsu ony urea, 470t, 472 , 479t vo tage-dependent, 86, 87 Isoprotereno , 125t, 142t
nateg inide, 470t, 472 , 479t Ionic thyroid inhibitors, 431t, 440t Isosorbide, 268t, 283t
repag inide, 470t, 472 , 479t iodide, 431t, 440t Isosorbide-5-mononitrate (ISMN), 289t, 298,
su ony urea, 470t, 472 , 479t sodium iodide, 431t, 440t 299, 300t
adverse e ects, 475 Ionization Isosorbide dinitrate (ISDN), 289t, 300t
g ic azide, 470t, 472 , 479t kidney tubu es, 2122 Isotretinoin, 707t, 721, 722, 724t
g imepiride, 470t, 472 , 477, p asmagastric juice partitioning, Isradipine
478, 479t 19b20b, 20 , 2122 antihypertensive, 277t, 287t
g ipizide, 470t, 472 , 479t Ionized mo ecu es, di usion, 19b, 19 myocardia ischemia, 290t, 300t

756
131
I-tositumomab, 662t, 671t, 684t Lapatinib, 661t, 673, 683t Levetiracetam, 225t, 234235, 237t
Itraconazo e, 588t, 589 , 590, 597t l -asparaginase (L-ASP), 630 , 643t, 644t, Levobetaxo o , 126t, 145t
Ivermectin, 708t, 725t 648650, 659t Levobuno o , 125t, 144t
Ixabepi one, 643t, 644b, 650, 657t Latanoprost, ophtha mic, 691t, 705t ophtha mic, 691t, 695t, 704t
Laxatives, 497t, 498t, 507t, 507t508t Levocabastine, 347t, 355t
J bisacody , 498t, 508t Levocetirizine, 347t
Jaw osteonecrosis, bisphosphonate, 481t, bowe e ects, 500t Levodopa, 238t, 242, 243245, 243 ,
485, 488t c assi cation, 500t 244 , 252t
Jimson weed, 101102, 101t, 694 , docusate sodium, 498t, 508t Levo oxacin, 544, 544 , 545, 548t
698, 698t actu ose, 497t, 505, 506507, 507t Levonorgestre , 443t, 444t, 446t
magnesium citrate, 497t, 507t Levorphano , 194t, 195t, 200t, 209t
K magnesium hydroxide, 497t, 507t Levothyroxine, 431t, 432434, 434t,
KA, 6b, 8 magnesium su ate, 497t, 507t 436439, 439t
Ka ikrein inhibitors, 347t mechanisms o action, 497t498t, 498b, adverse e ects, 434
Kanamycin, 559566, 559b, 567t 498t, 499 actors in, 434t
K+ channe s, 86, 87 po yethy ene g yco , 497t, 507t hypothyroidism
KD, 6b, 8 sa ine, 497t, 507t pregnancy dosing, 437439
Keratitis, 700 sodium phosphate, 497t, 507t subc inica , 437, 438439
unga , 701 LDL-C eve s therapeutic goa s, 432
vira , 701 c assi cation, 337t mechanisms o action, 431t
Kernicterus, 68 treatment, 336t thyroid cancer, 436
Ketamine, LDL-C metabo ism, 335 Lico e one, 358
ana gesics, 217 Lead, 41t, 43t, 5758, 57b, 58 , 63, 64, 65t Licorice, 467, 468
anesthetic, 210t, 213t, 214 , 217, 222t Lead poisoning, 43t, 63, 64, 65t Lidocaine, 224t
Ketanserin, 120t, 133t, 141t, 147t dimercapro , 41t, 42t, 66t antiarrhythmic, 312t, 316, 317t,
Ketoconazo e, 459t, 469t, 588t, 598t dimercaptosuccinic acid, 41t, 42t, 66t 319, 320, 321t
anticancer, 665t, 687t ED A CaNa2, 41t, 42t, 63, 64, 66t e ectrophysio ogic actions, 317t
mechanisms o action, 668 hea th e ects, 57b ventricu ar bri ation, 316
Ketoro ac, 357t, 361t, 373t heme biosynthesis, 58 ventricu ar tachycardia, 319, 320
ophtha mic, 691t, 705t mechanisms, 41t Ligand-gated receptors, 88
Ketoti en umarate, 347t, 355t Le unomide, 368t Lincosamides, 568t, 574, 575, 576t
ophtha mic, 691t, 705t Le ventricu ar dys unction, angina drugs, 293t Lindane, 708t, 725t
Kidney Leishmaniasis, viscera , 534 Linezo id, 569t, 576t
passive di usion, excretion, Lena idomide, 381t Liotrix, 431t, 439t
and pH, 20b anticancer, 663t, 666 , 685t Lipid membrane di usion, 1920, 20
tubu e ionization, 2122 immunostimu ant, 381t Lipopeptides, 569t, 570 , 573, 575, 577t
Kinin receptor antagonists, 347t, 355t Lennox-Gastaut syndrome, 232 Lipoxins, 358
K+-sparing diuretics, 269t, 271t, 272 , 285t Leprosy drugs Lipoxygenase pathways, 358 , 359
minera ocorticoid antagonists c o azimine, 579t, 583, 587t Lirag utide, 471t, 472 , 479t
ep erenone, 269t, 280281, 285t combination therapy, 583584 Lisdexam etamine, 120t, 141t
spirono actone, 269t, 285t, 307 dapsone, 579t, 581 , 583586, 587t Lisinopri
Na+-channe inhibitors, 269t, 271t, 272 , 284t therapeutic princip es, 584b heart ai ure, 306, 309, 310
ami oride, 269t, 284t MC-207, 579t, 587t pharmaco ogy and actions, 273t,
triamterene, 269t, 282, 283, 284t Letha dose, 44 281, 282, 285t
vs. e ective dose, 44 Lispro. See also Diabetes me itus; Insu in
L median (LD50), 44 , 45 vs. g argine, 478
LABAs. See 2 adrenergic receptor Letrozo e Lithium, 151t, 164166, 175t
agonists, ong-acting anticancer, 664t, 677t, 679t, 686t adverse e ects, 164t
Labeta o , 126t, 128t, 146t pharmaco ogy, 442t, 458t e der y, 165166
Lacosamide, 226t, 237t Leucovorin. See High-dose methotrexate ibupro en, 165, 168, 169
Lacrima system, 698 with eucovorin rescue (HDM-L) ithium carbonate, 151t, 175t
Lactation, FDA ru es, 72b Leukotriene antagonists, pu monary, 390t, ithium citrate, 151t, 175t
Lactu ose, 497t, 505, 506507, 507t 403t404t overdose, 166167
Lambert-Eaton syndrome, 104 5-LOX inhibitors, zi euton, 390t, 403t Lithium carbonate, 151t, 175t
Lamivudine, 601t, 602t, 607 , 621t asthma, severe, 396 , 397 Lithium citrate, 151t, 175t
adverse e ects, 615 eukotriene antagonists, 390t, 404t Liver metabo ism and c earance.
antihepatitis, 600t, 602t, 607 , 620t monte ukast, 390t, 404t See also speci c drugs and
mechanisms o action and resistance, za r ukast, 390t, 404t drug classes
602t, 604 , 607 , 608 , 614615 mechanisms o action, 390t, 396 e der y, 75b, 76 , 76t77t, 77
structure, 607 Leukotrienes, 358 L-methy o ate. See Fo ic acid and
Lamotrigine, 225t, 232, 237t Leucovorin ca cium, 406t, 416, 417418, 420t derivatives
anticonvu sants, 151t, 165, 175t Leupro ide LMWH. See Low-mo ecu ar-weight
va proic acid with, 235 anticancer, targeted, 664t, 679t, 680, 687t heparins (LMWHs)
Lanreotide, 422t, 428t prostate cancer, 423t, 425426, 429t Loading dose, 39b
Lansoprazo e, 490t, 493 , 496t Leva butero creatine c earance, 34
Lanthanum, 481t, 488t pharmaco ogy, 125t, 143t Loca anesthetics. See Anesthetic agents,
Lanthanum carbonate, 481t, 484, 488t pu monary, 389t, 402t oca ; speci c agents
757
Lodoxamide tromethamine, ophtha mic, Lymphocyte immune (ALG), 376t, 377 , 378, Ma athion, 89
691t, 705t 378t, 385t dermato ogic, 708t, 725t
Lometrexo , 635t, 653t pharmaco ogy, 100t, 102t, 115t
mechanisms o action, 635t M Mannito , 268t, 283t
mechanisms o resistance, 638b Macro ides and keto ides, 568t, 570 , 576t MAO inhibitors
Lomustine (CCNU), 627t, 630 , 652t azithromycin, 568t, 570 , 576t 5-H concentration a tering, 130t, 147t
mechanisms o action, 627t, c arithromycin, 568t, 570 , 571, 576t ood contraindications, 135, 137
628b, 630 drug interactions, macro ide, 572 with SSRIs, 133
mechanisms o resistance, 627t, erythromycin, 568t, 570 , 576t Maproti ine, 150t, 173t
632633, 632b te ithromycin, 568t, 570 , 576t Maraviroc, 601t, 603t, 608 , 611 , 623t
toxicities, 652t toxicities, 571 Marijuana, 263
uses, 629t Macrophage co ony stimu ating actor medicina e ects, 265, 266
Long Q syndrome, 313315, 315b (M-CSF), 407t, 408 withdrawa syndrome, 263, 263t
Loop diuretics, 269t, 271t, 272 , 284t Macu ar degeneration agents, 692t, 706t Mast ce stabi izers. See Histamine
bumetanide, 269t, 282, 283, 284t bevacizumab, 692t, 706t receptor antagonists and mast ce
ethacrynic acid, 269t, 284t pegaptanib, 692t, 706t stabi izers
urosemide, 269t, 280, 284t, 306 ranibizumab, 692t, 706t MDR1 transporter, 23t, 24t, 25t, 38
torsemide, 269t, 284t vertepor n, 692t, 703, 706t MDR3 (ABCB4) transporter, 25t
Loperamide Magnesium citrate, 497t, 507t Mecamy amine, 106t, 117t
antidiarrhea , 209t, 498t, Magnesium hydroxide, 497t, 507t Mecasermin, 422t, 428t
500501, 508t Magnesium su ate Mech orethamine, dermato ogic, 708t,
mechanisms o action, 498t antiarrhythmic, 317t, 322t 724t, 726t
Lopinavir, 601t, 603t, 615, 622t e ectrophysio ogic actions, 317t Mech orethamine HC , 627t, 651t
adverse e ects, 615 axative, 497t, 507t mechanisms o action, 626, 627t, 628,
CYP3A4 metabo ism, 618, 619 Ma aria chemoprophy axis, 521522, 522t 628b, 630
drug interactions, 615 atovaquone-proguani , 521522, 522t, 526 mechanisms o resistance, 627t,
HIV, 618, 619 ch oroquine phosphate, 522t 632633, 632b
mechanisms o action and resistance, doxycyc ine, 522t toxicities, 632b633b, 651t
603t, 615 genera advice, 526527 uses, 629t
Loracarbe , 557t hydroxych oroquine su ate, 522t Mec izine, 346t, 350, 352, 354t
Loratadine, 8990 me oquine, 522t, 524 Median e ective dose (ED50), 44 , 45
H 1 receptor antagonist, 347t, 348, 348b, primaquine, 522t, 525526 Median etha dose (LD50), 44 , 45
351352, 355t Ma aria chemotherapy, 520529, Medication errors, 5657, 57t
pregnancy and actation, 348, 348b, 520t521t, 529t Medroxyprogesterone, targeted anticancer
351352 artemisinin, 520t, 529t therapy, 664t, 679t, 686t
Lorazepam, 176t, 189, 190, 191t, 192t artemether, 520t, 529t Medroxyprogesterone acetate (MPA) conju-
Losartan, 273t, 285t artemether- ume antrine, 520t, 521, gated estrogen +, 441t
Lotepredno , ophtha mic, 691t, 705t 527, 529t hormone rep acement therapy, 442t, 458t
Lou Gehrigs disease. See Amyotrophic artesunate, 520t, 529t MedWatch, 47
atera sc erosis (ALS) dihydroartemisinin, 520t, 529t Me oquine, ma aria
Lovastatin, 334t, 341, 342, 343t atovaquone, meprone, 520t, 529t chemoprophy axis, 522t, 524
Lovenox, 325326, 326 atovaquone-proguani , 520t, 527, 529t chemotherapy, 521t, 522t, 524, 527, 529t
Low-mo ecu ar-weight heparins (LMWHs), chemoprophy axis, 521522, 522t, 526 Mega ob astic anemia, 417418. See also
323t, 324325, 325 , 333t. See also ch oroquine/hydroxych oroquine, 521t, Vitamin B12 de ciency
speci c agents 522t, 523524, 526, 529t sh tapeworm, 537540
indications, 325t ch oroquine-resistant P. alciparum, Megestro , anticancer, 665t, 687t
ovenox, 325326, 326 523 , 524 Megestro acetate, 442t, 458t
mechanisms o action, 325 cinchonism, 524 anticancer, 664t, 679t, 686t
monitoring, 325, 330b dermato ogic agents, 708t, 725t Me arsopro , 530t, 534, 535t
Low (broad) speci city, 5b, 9 ch oroquine, 708t, 725t Me atonin congeners, 177t, 189, 190, 192t
Loxapine, 151t, 175t hydroxych oroquine, 708t, 725t Me pha an, 627t, 632t, 651t
5-LOX inhibitors, 390t, 403t quinacrine, 708t, 725t mechanisms o action, 626, 627t, 628,
Lubiprostone, 497t, 507t endemic countries, 523, 523 628b, 630
Lumiracoxib, 363t mechanisms o action, 520t521t mechanisms o resistance, 627t,
Lumbar puncture me oquine, 521t, 522t, 524, 527, 529t 632633, 632b
seque ae, 219 pregnancy, 527 toxicities, 632b633b, 632t, 651t
spina anesthesia, 217219 primaquine, 521t, 522t, 525526, 526t, uses, 629t
Lung cancer, hyperca cemia in, 483484, 527, 529t Memantine, 239t, 246247, 251, 252, 253t
486, 487 proguani , 520t, 529t Menotropin, 423t, 429t
ca citonin or, 480t, 483484, 488t pyrimethamine-su adoxine, 521t, 529t Menstrua cramping, 364365
Luteinizing hormone (LH) therapy, 423t, 429t quinine/quinidine, 521t, 524, 527, 529t Meperidine, 195t, 200t, 204, 208t
Lutropin a a, 423t, 430t su adoxine, 521t, 529t, 543t, 548t MAO inhibitors contraindication, 204
Lymphocyte unction-associated antigen treatments o choice, 526527 toxicity, 204, 206, 208
(LFA-1 & LFA-3) inhibitors Ma aria parasite Mephentermine, 119t, 139t
a e acept, 376t, 386t deve opmenta stages, 525, 526t Mephobarbita , 177t, 181t, 192t
e a izumab, 376t, 386t i e cyc e, 525526, 525 Mepivacaine, 224t

758
Meprobamate, 177t, 193t skin, treatment, 714715 Methy xanthines, pu monary, 390t, 403t
Meprone, 520t, 529t tetracyc ines, 571 COPD, 395
Mequino , 710t, 727t Methimazo e, 431t, 435436, 438, mechanisms o action, 390t
6-Mercaptopurine (6-MP), 636t, 439, 440t theophy ine, 390t, 392 , 397, 403t
641642, 655t Methohexita , 177t, 192t Methysergide, 130t, 147t
mechanisms o action, 630 , 636t, anesthetic, 210t, 213t, 222t Metiprano o , 125t, 144t
641642 mechanisms and sites o action, 635t ophtha mic, 691t, 695t, 704t
mechanisms o resistance, 636t, Methotrexate, 635t, 653t Metoc opramide
639b, 642 + 5-FU, 648, 649 antiemetic, 502t, 504, 506
toxicities, 642, 655t anticancer bowe disorders, 497t, 504, 506, 507t
uses, 637t, 655t mechanisms and sites o action, 630 , Meto azone, 269t, 284t
Mercury, 41t, 43t, 5960, 59 , 65t 635t, 637 , 638639 Metopro o
Meropenem, 558t mechanisms o resistance, 635t, heart ai ure, 301t, 303304, 306, 309,
Mesa amine, 357t, 373t 638b, 640 310, 310t
Mesa amine (5-ASA), 498t, 503 , 509t dermato ogic, 708t, 715, 721722, myocardia ischemia, 300t
Metabo ic syndrome, 339340. See also 724t, 725t pharmaco ogy, 126t, 127129, 128t, 145t
Dys ipidemia drugs psoriasis, 715716, 721722 Metronidazo e, 530t, 532534, 535t
Metabo ism, drug, 20b, 28b. See also speci c o inic acid with, 416, 417418 a coho reaction, 533, 534
drugs and drug classes in ammatory bowe disease, amebic co itis, 530t, 532533, 535t
acetaminophen, 2829, 5054, 50 , 52t, 499t, 509t duodena u cer, 492, 494t
207, 208 mechanisms and sites o action, 635t giardiasis, 530t, 532533, 535t
chi dren, 70b mechanisms o resistance, 638b, 640 Metyrapone, 459t, 466, 469t
CYP super ami y, 30, 30b31b, 32t, 33 pharmacogenetics, 637 , 639 Mexi etine
drugdrug interactions on, 48, 48b, 48t rheumatoid arthritis, 368b, 368t, 380 antiarrhythmic, 312t, 317t, 321t
e der y, 73t74t, 75b toxicities, 653t e ectrophysio ogic actions, 316
nuc ear receptors inducing, 33t, 34 uses, 637t Mez oci in, 552t, 556t
phase 1, 28b, 33 , 35, 38 Methoxsa en photochemotherapy, 707t, Mianserin, 150t, 174t
neonata , 70b 716t, 725t Mica ungin, 588t, 589 , 598t
phase 2, 2829, 28b, 33 , 35, 39 6-Methoxy-arabinosy -guanine. Michae is-Menten equation, 35b
poisoning, 52 See Ne arabine Miconazo e, 588t, 594, 597t
reactions, 32t Methscopo amine, 498t, 508t Microtubu e-damaging agents. See also
Meta s, heavy (toxic), 41t, 43t, 65t66t. Methscopo amine bromide, 96t, 114t Co chicine; axanes;
See also speci c types Methsuximide, 225t, 236t Vinca a ka oids
che ators, 66t Methy amino evu inate photodynamic ther- estramustine, 630 , 643t, 650, 657t
Metaprotereno apy, 708t, 725t ixabepi one, 643t, 644b, 650, 657t
pharmaco ogy, 125t, 143t Methy -CCNU. See Semustine Microtubu e disruptors, 369t
pu monary, 389t, 402t (methy -CCNU) co chicine, 369t, 370, 373t
Metaramino , 119t, 139t Methy ce u ose, ophtha mic, 692t, 706t Midazo am, 176t, 191t, 192t, 214
Met ormin, 470t, 479t Methyc othiazide, 269t, 284t Midodrine, 119t, 139t
diabetes me itus type 2 a er, Methy dopa Mi epristone, 442t, 458t, 459t, 469t
hemog obin A1c, 475476 antihypertensive, 277t, 286t Mig ito , 471t, 479t
+ g imepiride, 477, 478 pharmaco ogy, 119t, 140t Migraine
Methacho ine, 99t, 112t Methy enedioxymethamphetamine (MDMA) 5-H in, 133134, 133t
mechanisms o action, 95t 5-H 2 receptor ora contraceptives with, 455, 456457
pharmaco ogy, 99t, 112t a nity, 265, 266 Mi nacipran, 150t, 174t
Methadone, 194t, 195t, 200t, 201203, L-Methy o ate. See Fo ic acid and derivatives Mi rinone, 302t, 308, 308t, 311t
202t, 208t Methy hydrazines, 629t, 652t Mi te osine, 530t, 534, 535t
cancer pain, 201203, 202t mechanisms o action, 628b, 628t, 630 Mi nacipran, 130t, 147t
routes o administration, 202203 mechanisms o resistance, 628t, Minera ion homeostasis disorder drugs,
conversion guide ines, ora morphine to 632633, 632b 480489
methadone, 202t procarbazine, 629t, 652t ca cimimetics, cinaca cet, 481t, 486,
vs. heroin responses, 260, 260 toxicities, 632b633b, 652t 487, 488t
or opioid addiction, 260261 uses, 629t hormones
Methamphetamine, 120t, 141t Methy me amines, 627t, 651t. See also Ethy - ca citonin, 480t, 483484, 483
Methano , 42t, 4950, 49 eneimines and methy me amines teriparatide, 480t
metabo ism and poisoning, 4950, Methy na trexone hyperca cemia, 483484
49 , 178 bowe disorders, 498t, 499, 505, 506, 507t vitamin D and ana ogs, 481t, 488t
Methano antidote constipation, opioid, 199, 498t, 499, 505, 1-hydroxycho eca ci ero , 481t, 488t
ethano , 42t, 50, 65t 506, 507t 22-oxaca citro parica cito , 481t, 488t
omepizo e, 41t, 42t, 65t, 178 mechanisms o action, 498t ca cipotrio , 481t, 488t
Methenamine, 548t opioid receptor antagonism, 195t, 199, 209t ca citrio , 481t, 482 , 488t
Methici in, 551, 552t, 554, 555, 555t Methy paraben, 220, 221 dihydrotachystero , 481b
Methici in-resistant Staphylococcus aureus Methy phenidate, 120t, 141t doxerca ci ero , 481b
(MRSA), 551, 571 Methy predniso one, 460t. See also ergoca ci ero , 481t, 488t
doxycyc ine, 571 Corticosteroids parica cito , 481t, 488t
minocyc ine, 571 pu monary, 390t, 403t therapeutic uses, 481b

759
Minera ocorticoid receptor antagonists o atumumab, 662t, 684t Mu tip e sc erosis (MS)
(K+-sparing), 269t, 271t, rituximab, 662t, 671t, 681, 682, 684t IFN--1a and IFN--1b, 381t
272 , 285t CD33, anticancer, 662t, 684t pathophysio ogy, 382
ep erenone, 269t, 280281, 285t gemtuzumab ozogamicin, 662t, 667t, Mupirocin, 569t, 573, 577t
spirono actone, 269t, 285t, 307 672t, 684t Muromonab-CD3, 376t, 378, 385t
Minimum a veo ar concentration (MAC), mechanisms o resistance, 662t Muscarine
215, 215t CD52, anticancer, 662t, 667t, 671t, 684t mushrooms, 97
Minimum e ective concentration a emtuzumab, 662t, 667t, 671t, 684t pharmaco ogy, 99t
(MEC), 36 mechanisms o resistance, 662t Muscarinic acety cho ine receptors
Minimum inhibitory concentration denosumab, 481t, 488t (mAChRs), 98t99t
(MIC), 513 growth actor antibodies (VEGF), Muscarinic receptor agonists, 99t, 112t.
Minocyc ine anticancer, 662t, 673675, 684t See also speci c types
MRSA, 571 bevacizumab, 662t, 671t, 674675, 684t bethanecho , 95t, 99t, 112t
rheumatoid arthritis, 368t dose, 671t mechanisms o action, 95t
Minoxidi ranibizumab, 662t, 684t Muscarinic receptor antagonists, 99t, 100,
androgenic a opecia, 710t, 726t toxicities, 671t, 674675, 684t 101b, 101t, 112t114t. See also
hypertension and edema, 278t, 287t growth actor receptor, anticancer, speci c types
Miosis, 698, 698t 662t, 683t mechanisms o action, 96t
Mirtazapine, 150t, 174t cetuximab, 662t, 669672, 683t overactive b adder, 100, 101t
Misoprosto , 490t, 493, 494, 495, 496t dose and toxicities, 671t Muscarinic receptors, 82 , 97
gastric acid disease, 490t, 493, 494, EGFR targeting, 669672 atropine, 8082
495, 496t HER2/neu, 667t, 682t muscarine, 97, 99t
GERD in pregnancy, 494, 495 panitumumab, 662t, 669672, 683t Mushrooms, catecho aminergic, 97, 100
with NSAID-induced duodena trastuzumab, 662t, 671t, 672673, 681, Musse s, saxitoxin, 221, 222
u cer, 493 682, 683t MXR transporter, 24t
Mitomycin, ophtha mic, 691t, 705t nata izumab, 382t Myasthenia gravis, 104105, 105
Mitomycin (mitomycin-C), 630 , 643t, Monocyte/macrophage co ony stimu ating Mycobacterium avium comp ex drugs
646b, 658t actor (M-CSF, CSF-1), 407t, 408 cyc oserine, 579t, 587t
Mitotane, 679t Monte ukast, 390t, 404t ethambuto , 578t, 583, 587t
anticancer, 644t, 659t Morphine, 194t, 195t, 200t, 206208, 208t therapeutic princip es, 584b
pharmaco ogy, 459t, 469t constipation rom, 92 MC-207, 579t, 587t
Mitoxantrone conversion guide ines, ora morphine to Mycobacterium intracellulare, 579t, 583, 584,
anticancer, 630 , 643t, 645b, 658t methadone, 202t 584b, 587t
immunostimu ant, 382t in ant dosing, 72, 74 Mycobacterium kansaii
Mivacurium, 106t, 107t, 117t metabo ism ethambuto , 578t, 583, 587t
Mizo astine, 347t, 355t chi dren, 68 MC-207, 579t, 587t
Moda ni , 120t, 142t rst-pass, 206, 207 Mycobacterium spp., MC-207, 579t, 587t
Moexipri , 273t, 285t morphine-6-g ucuronide, 206, 207208 Mycobacterium tuberculosis drugs.
Mo indone, 151t, 175t Morphine-6-g ucuronide, 206, 207208 See ubercu osis drugs
Mometasone, 390t, 403t Mosquito bite, 347b, 348 Mycopheno ate mo eti (MMF)
Monoamine oxidase inhibitors (MAOIs), Motion sickness, 350, 352 dermato ogic, 709t, 724t, 726t
150t, 157, 173t. See also Moxi oxacin, 544, 544 , 545, 548t immunotherapy, 375t, 383, 384, 385t
speci c agents MPA. See Medroxyprogesterone acetate mechanisms o action, 375t, 383, 384
isocarboxazid, 150t, 173t (MPA) Mydriasis, 694 , 698, 698t
meperidine contraindication, 204 MPA immunotherapy, 375t, 385t Mydriasis agents, 694 , 698, 698t
phene zine, 150t, 173t MRP1 (ABCC1) transporter, 23t Mye oid growth actors, 405t, 407t, 418t
se egi ine, 150t, 173t MRP2 (ABCC2) transporter, 23t G-CSF, 405t, 407t, 408 , 409410, 416,
SSRI a er, 157 MRP3 (ABCC3) transporter, 24t 417, 418t
structures, dosing, and adverse MRP4 (ABCC4) transporter, 24t G-CSF, pegy ated recombinant, 405t, 407t,
e ects, 156t MRP5 (ABCC5) transporter, 24t 408 , 409410, 418t
trany cypromine, 150t, 173t MRP6 (ABCC6) transporter, 24t GM-CSF, 405t, 407t, 408 , 409410, 418t
Monobenzone, 710t, 727t MRSA. See Methici in-resistant indications and choice, 409410
Monoc ona antibodies (mAbs) Staphylococcus aureus (MRSA) mechanisms o action, 405t, 407t
anti-CD3, muromonab-CD3, 376t, M A. See Pemetrexed (M A) sites o action, 408
378, 385t m OR, 384 Myocardia in arction
anti-CD25 (anti-IL-2 R), 376t, m OR inhibitors, 663t, 685t aspirin prophy axis, dai y, 365
379, 386t evero imus, 663t, 685t drugs, 324325, 324 (See also
basi iximab, 376t, 386t mechanisms o action, 663t, 675 T romboembo ic disorder drugs)
dac izumab, 376t, 386t mechanisms o resistance, 663t, 676 p ate et adhesion and aggregation, 324,
anti-CD52, a emtuzumab, 376t, rapamycin, 663t, 675676, 685t 324 (See also Antip ate et agents)
378, 386t temsiro imus, 663t, 675676, 685t Myocardia ischemia
CD20, anticancer, 662t, 667t, 684t Muco ytics, 390t, 404t c assi cation, 290t
99
Y-ibritumomab, 662t, 671t, 684t DNase (dornase a a), 390t, 404t percutaneous coronary
131
I-tositumomab, 662t, 671t, 684t N-acety cysteine, 390t, 404t interventions, 297
dose and toxicities, 671t, 684t Mu tip e mye oma, bortezomib, 663t, 681, stents, drug-e uting coronary, 297, 297b
mechanisms o resistance, 662t 682683, 685t symptoms, 289b290b

760
Myocardia ischemia drugs, 289300. See also Na+-K+ exchange, sarco emma , 303 microtubu e-damaging agents: taxanes,
speci c drugs and drug classes Na buphine, 195t, 200t, 209t 642t, 648650, 656t
adrenergic receptor antagonists, 289t, Na me ene, 177t, 193t docetaxe , 630 , 642t, 644b,
291 , 292294, 300t Na oxone, 16, 17, 41t, 42t, 65t, 195t, 648650, 656t
Ca2+ channe antagonists, 290t, 294295, 203204, 205, 207, 209t mechanisms o action, 630 , 642t,
295t296t, 297299, 300t opioid overdose, 203204, 205, 207 648650
c audication, 295b precautions, 203 mechanisms o resistance,
mechanisms o action, 289t290t route o administration, 205, 207 642t, 644b
Na+ channe b ocker, rano azine, 290t, toxicity use, 41t, 42t, 65t nab-pac itaxe , 630 , 642t, 644b,
298, 299, 300t Na trexone 648650, 656t
nitrate vasodi ators, organic, 22, 289t, a coho ism, 177t, 186187, 186t, 203, 257 pac itaxe , 630 , 642t, 644b,
291292, 291b, 291 , 294, or opioid addiction, 262 648650, 656t
297299, 300t opioid receptor antagonism, 193t, 195t, 209t toxicities, 656t
periphera vascu ar disease, 295b Naphazo ine, 120t, 142t uses, 644t, 656t
phosphodiesterase 5 inhibitors, 291b, ophtha mic, 690t, 695t, 705t microtubu e-damaging agents: vinca
292, 300t NAPQI (N-acety -p-benzoquinone imine), a ka oids, 642t, 656t
unstab e angina, 296297 29, 48, 53 mechanisms o action, 630 , 642t
variant angina, 297 Naratriptan, 130t, 147t mechanisms o resistance, 642t, 644b
vasospastic angina with sinus bradycardia, Nata izumab, 382t toxicities, 656t
297, 298 Nateg inide, 470t, 472 , 479t uses, 644t, 656t
Natriuresis, 270b vinb astine su ate, 630 , 642t, 644b,
N Natriuretic agents 644t, 656t
Na+-2C - symporter inhibitors. See T iazide/ atria natriuretic peptide, 269t, vincristine su ate, 630 , 642t, 644b,
thiazide- ike diuretics 280, 284t 644t, 656t
Na+ ba ance disorder agents, 279t, 280281, diuretics (See Diuretics) vinore bine, 630 , 642t, 644b,
281 . See also Diuretics Natriuretic peptide receptor agonists, heart 644t, 656t
Nabi one, 498t, 502, 508t ai ure topoisomerase inhibitors: camptothecin
Nab-pac itaxe , 642t, 648650, 656t mechanisms o action, 302t ana ogs, 643t, 657t
mechanisms o action, 630 , 642t, nesiritide, 302t, 311t irinotecan, 630 , 643t, 644b645b,
648650 Natura products, cytotoxic, 642t644t, 649651, 657t
mechanisms o resistance, 642t, 644b 645647 mechanisms o action, 630 , 643t,
toxicities, 656t antibiotics, anticancer, 630 , 643t, 658t 649651
uses, 644t, 656t b eomycin, 630 , 643t, 645b, 658t mechanisms o resistance, 643t,
N-acety cysteine (NAC), 40t, 42t, 54, 65t mechanisms o action, 643t 644b645b
muco ytic, 390t, 404t mechanisms o resistance, 643t, topotecan, 630 , 643t, 644b645b,
Na+ channe b ockers 645b646b 649651, 657t
antiarrhythmic, 312t, 320t321t mitomycin (mitomycin-C), 630 , 643t, toxicities, 657t
disopyramide, 312t, 317t, 321t 646b, 658t uses, 644t, 657t
ecainide, 312t, 317t, 321t uses, 644t, 658t topoisomerase inhibitors:
idocaine, 312t, 316, 317t, 319, antibiotics, anticancer: actinomycins, epipodophy otoxins, 643t, 657t
320, 321t dactinomycin, 630 , 643t, 658t etoposide, 630 , 643t, 645b, 657t
mexi etine, 312t, 317t, 321t antibiotics, anticancer: anthracyc ines and mechanisms o action, 630 , 643t
procainamide, 312t, 316, 317t, 320t anthracenediones, mechanisms o resistance, 643t, 645b
propa enone, 312t, 317t, 321t 643t, 658t teniposide, 630 , 643t, 645b, 657t
quinidine, 312t, 317t, 320t daunorubicin, 630 , 643t, 645b, 658t toxicities, 657t
myocardia ischemia, rano azine, 290t, doxorubicin, 630 , 643t, 645647, uses, 644t, 657t
298, 299, 300t 645b, 658t trabectedin, 643t, 659t
Na channe s, 86, 87
+
epirubicin, 630 , 643t, 645b, 658t uses, 644t
antiseizure drug-enhanced inactivation, idarubicin, 630 , 643t, 645b, 658t Nausea drugs. See Antinauseants and
225t, 225t226t, 226 , 232 mechanisms o action, 630 , 643t antiemetics
vo tage-gated mechanisms o resistance, 643t, 645b Nebivo o , 126t, 128t, 146t
oca anesthetics on, 217 mitoxantrone, 630 , 643t, 645b, 658t Nedocromi , ophtha mic, 691t, 705t
structure and unction, 218 uses, 644t, 658t Ne azodone, 150t, 174t
Nado o , 125t, 128t, 144t va rubicin, 630 , 643t, 645b, 658t Negative rein orcement, 265, 266
Nadroparin, 323t, 325, 333t enzymes Ne arabine, 636t, 655t
Na are in, 422t, 427, 428, 429t, 664t, 686t l -asparaginase (l -ASP), 630 , 643t, mechanisms o action, 630 , 636t,
Na ci in, 551, 552t, 554, 555, 555t 644t, 648650, 659t 641642
Na i ne, 589 , 599t pegaspargase (PEG-l -asparaginase), uses, 637t
Na+-K+-2C - symporter inhibitors. 643t, 659t Ne navir, 601t, 603t, 608 , 609 , 622t
See Loop diuretics uses, 644t Neomycin, 559566, 559b, 567t
Na+-K+-A Pase inhibitors mechanisms o action, 630 , 642t643t Neosporin, 573
antiarrhythmic, 312t, 322t mechanisms o resistance, 644b646b Neostigmine, 104
digoxin, 312t, 317t, 319, 320, 322t microtubu e-damaging agents Neostigmine bromide, 100t, 114t
heart ai ure, 303 , 311t epothi ones: ixabepi one, 643t, 644b, Neostigmine methy su ate, 100t, 114t
digoxin, 301t, 306, 307, 311t 650, 657t Nepa enac, 357t, 361t, 373t
mechanisms o action, 301t estramustine, 630 , 643t, 650, 657t ophtha mic, 691t, 705t

761
Nephrons, 270b Neurotransmitters, 85, 86b, 86 mechanisms o resistance, 627t,
Nerve gases actions and termination, 89, 90 632633, 632b
organophosphate, 100t, 103t, 116t CNS, 86b, 9192, 91t mech orethamine HC , 627t, 651t
sarin, 100t, 103t, 115t PNS, 83t84t, 86b, 92 me pha an, 627t, 632t, 651t
soman, 100t, 103t, 116t receptors, 89b toxicities, 632b633b, 632t, 651t
tabun, 100t, 103t, 116t Nevirapine, 601t, 603t, 615, 622t uses, 629t
Nerve impu se transmission, 8586, 85b, 86 HIV, 615 Nitrog ycerin
Nerve injury pain, 197b, 197 mechanisms o action and resistance, administration, timing, 291292
Nesiritide (BNP), 269t, 280, 284t, 302t, 311t 603t, 608 , 610 , 615 adverse e ects, 292
Neti micin, 559566, 559b, 567t structure, 610 angina prevention, 291
Neurodegenerative disease drugs, 238253. New drug testing, 44 , 45, 45t rst-pass e ect, 22, 291, 299
See also speci c diseases phases, 45t myocardia ischemia, 22, 289t, 291292,
and drugs Niacin, 335t, 340, 341, 342, 343t 291b, 291 , 300t
A zheimers disease, 239t, adverse e ects, 341, 342 PDE5 inhibitor interactions, 291b, 292,
245248, 253t Nicardipine, 277t, 287t 298, 299
amyotrophic atera sc erosis, 239t, Nicotine pharmaco ogy, 22, 289t
249, 253t addiction, 258259 Nitroprusside, 278t, 287t
Huntingtons disease, 239t, 248249, negative rein orcement, 265, 266 Nitrosoureas, 652t
248 , 253t poisoning, acute, 106t, 108, 108b carmustine, 632t, 652t
Parkinsons disease, 238t239t, 239245, withdrawa syndrome, 258259, 259t omustine, 652t
239b, 242t, 250251, 252t253t Nicotine gum or ozenge, 106t, 108, 108b, mechanisms o action, 627t, 628b, 630
Neuroe ector junction, 97 117t, 259 mechanisms o resistance, 627t,
Neuro eptic ma ignant syndrome, 172t Nicotine nasa spray or vapor inha er, 106t, 632633, 632b
Neuromuscu ar b ocking agents 108, 108b, 117t, 259 semustine, 652t
anesthetic adjuncts, 211t, 223t Nicotine transderma patch, 106t, 108, 108b, streptozocin, 652t
c assi cation, 107t 117t, 259 toxicities, 632b633b, 652t
competitive, 97 , 105 , 106 , 106t, 107t, Nicotinic ACh receptor agonists, 106, 117t uses, 629t
116t117t Nicotinic acid, 335t, 343t Nitrous oxide, 210t, 215t, 223t
depo arizing, 97 , 105 , 107t, 108, 116t mechanisms o action, 335t Nizatidine
mechanisms o actions, 106t niacin, 335t, 340, 341, 342, 343t gastric acid disease, 490t, 496t
Neuromuscu ar junction Nicotinic receptors, 82 , 93, 94 structure, 492
acety cho ine, 105 Ni edipine Nocardia, 579t, 587t
drugs at, mechanisms o action, 106t antihypertensive, 277t, 287t Nociception. See also Opioid
sites o action, 105 myocardia ischemia, 290t, 300t pharmaco ogy; Pain
somatic, neurotransmitters, 82 , 93, 94 Ni urtimox, 531t, 535t acute, 197b, 197
(See also speci c types) Ni otinib Noncompetitive antagonists, 4b, 5
Neuropeptides, 91t mechanisms o resistance, 668 Nonionized mo ecu es
Neuropeptide Y (NPY), 122 pharmacodynamics, 15 b ood-brain barrier, 1920, 19b20b, 19 ,
Neurotransmission, 8094 Ni utamide, 442t, 458t, 665t, 679t, 687t 20 , 35, 3738
a prazo am, 92 Nimodipine, 290t, 300t di usion, 1920, 19b20b, 19 , 20 , 35,
atropine, 8082 Niso dipine 3738
autonomic nervous system, 80, 81 , antihypertensive, 277t, 287t Non-nuc eoside reverse transcriptase inhibi-
82 , 85b myocardia ischemia, 290t, 300t tors (NNR Is), 601t, 603t, 608 ,
b ood-brain barrier, 94 Nitazoxanide, 530t, 532, 535t 610 , 623t
brain signa transduction, 85, 86b, 86 Nitrates de avirdine, 601t, 603t, 608 , 610
cho inergic antagonist, 81 , 82 , 83t84t, rst-pass e ect, 22, 291, 299 e avirenz, 601t, 603t, 608 , 610 , 623t
93, 94, 95t PDE5 inhibitor interactions, 291b, 292, etravirine, 601t, 603t, 608 , 610 , 623t
cho inesterase inhibitor, 89 298, 299 mechanisms o action and resistance,
CNS drugs, 9192 Nitrate vasodi ators, myocardia ischemia, 603t, 608 , 610
diphenhydramine vs. oratadine, 8990 289t, 291 , 300t nevirapine, 601t, 603t, 608 , 610 ,
drug-induced sedation/depression, with Ca2+ channe b ockers, 297 615, 622t
hyperexcitabi ity a er, isosorbide-5-mononitrate, 289t, 298, structures, 610
8990, 94 299, 300t Nonsteroida anti-in ammatory drugs
enteric nervous system, 8283 isosorbide dinitrate, 289t, 300t (NSAIDs), 356373
excitatory and inhibitory, 86, 86 nitrog ycerin, 22, 289t, 291292, 291b, acetic acid derivatives
ion channe s, 86, 86 88 , 89t 291 , 300t brom enac, 357t, 373t
morphine, constipation, 92 PDE5 inhibitor interactions, 291b, 292, dic o enac, 357t, 362t, 371, 372, 373t
muscarinic receptors, 82 298, 299 etodo ac, 357t, 361t, 373t
nerve impu se, 8586, 85b, 86 Nitric oxide gas, therapeutic, 211t, 223t indomethacin, 357t, 361t, 373t
neuropeptides, 91t Nitro urantoin, 545, 546, 547, 548t ketoro ac, 357t, 361t, 373t
neurotransmitters, 85, 86b, 86 , 89, 90 , Nitrogen mustards, 627t, 651t nepa enac, 357t, 361t, 373t
9192 activation, in vivo, 631, 631 su indac, 357t, 361t, 373t
nicotinic receptors, 82 , 93, 94 bendamustine, 627t, 651t to metin, 357t, 361t, 373t
receptors, 89b cyc ophosphamide, 627t, 631 , 632t, 651t adverse e ects, 364t, 365b, 367368
somatic nerves, 82 , 85, 92 i os amide, 627t, 632t, 651t arthritis, 366368
sympathetic vs. parasympathetic mechanisms o action, 626, 627t, 628, ba sa azide, 357t
nervous systems, 82 , 85, 92 628b, 630 choice and dosing, 366367
762
Nonsteroida anti-in ammatory drugs (Cont.) emedastine di umarate, 691t, 705t strabismus and b epharospasm agents,
c assi cation, 360 , 361t363t epinastine, 691t, 705t 692t, 706t
di unisa , 357t, 360t, 373t ketoti en umarate, 691t, 705t abobotu inumtoxin A, 692t, 706t
mesa amine, 357t, 373t odoxamide tromethamine, 691t, 705t onabotu inumtoxin A, 692t, 706t
o sa azine, 357t, 373t nedocromi , 691t, 705t surgery agents, ophtha mic,
ophtha mic, 691t, 705t o opatadine, 691t, 705t 691t692t, 706t
brom enac, 691t, 705t pemiro ast, 691t, 705t chondroitin su ate, 692t, 706t
dic o enac, 691t, 705t antivira s, 693t, 701 cyanoacry ate, 692t, 706t
urbipro en, 691t, 705t autonomic agents, 690t691t, 695t, 700, brogen ge , 692t, 706t
ketoro ac, 691t, 705t 704t705t hya uronate, 692t, 706t
nepa enac, 691t, 705t acety cho ine, 690t, 695t, 704t hydroxypropy methy ce u ose,
rheumatoid arthritis, 380 aprac onidine, 690t, 695t, 702, 692t, 706t
sa icy ate, 357t, 373t 703, 704t po ydimethy si oxanes, 692t, 706t
sa sa ate, 357t, 373t atropine, 690t, 695t, 704t povidone iodine, 691t, 706t
su asa azine, 357t, 368t, 373t, 381 betaxo o , 691t, 695t, 704t vitreous substitutes, 692t, 706t
u cers rom, 492 brimonidine, 690t, 695t, 704t systemic drugs with ocu ar adverse
Non-S segment-e evation myocardia carbacho , 690t, 695t, 704t e ects, 696t
in arction, 290t, 291 . See also carteo o , 691t, 695t, 704t wetting agents/tear substitutes,
Myocardia ischemia cyc opento ate, 690t, 695t, 705t 692t, 706t
drugs, 296297 dipive rin, 690t, 695t, 704t carboxymethy ce u ose, 692t, 706t
percutaneous coronary interventions, 297 echothiophate, 690t, 695t, 704t hydroxyethy ce u ose, 692t, 706t
Norepinephrine (NE), 85, 93, 118t, homatropine, 690t, 695t, 704t hydroxypropy ce u ose,
135, 137, 139t evobuno o , 691t, 695t, 704t 692t, 706t
on bronchia air ow, 135, 137 metiprano o , 691t, 695t, 704t hydroxypropy methy ce u ose,
metabo ism, 121 naphazo ine, 690t, 695t, 705t 692t, 706t
synthesis, storage, and re ease, 122 pheny ephrine, 690t, 695t, 705t methy ce u ose, 692t, 706t
Norepinephrine (NE) receptors, 122 pi ocarpine, 690t, 695t, 704t ty oxapo , 692t, 706t
Nor oxacin, 544, 544 , 545, 548t scopo amine, 690t, 695t, 704t Ocu ar route o administration, 697t
Nortripty ine, 150t, 173t tetrahydrozo ine, 690t, 695t, 705t O atumumab, 662t, 684t
Nuc ear actor o activated ymphocytes timo o , 691t, 695t, 696697, 697 , 704t O ce o Pediatric T erapeutics (OP ) ,
(NFA ), 383 tropicamide, 690t, 695t, 705t FDA, 72b
Nuc ear receptors, 3t. See also carbonic anhydrase inhibitors, 691t, 705t O -target interactions, 5b
speci c types brinzo amide, 691t, 705t O oxacin, 544, 544 , 545, 548t
drug metabo ism, 33t, 34 dorzo amide, 691t, 705t O anzapine, 130t, 147t, 151t, 165, 168, 170t,
Nuc eoside reverse transcriptase inhibitors g ucocorticoids, 691t, 705t 171172, 175t
(NR Is), 601t, 603t, 607 , dexamethasone, 691t, 705t O mesartan medoxomi , 273t, 285t
608 , 621t di uprednate, 691t, 705t O opatadine, 347t, 355t
abacavir, 601t, 603t, 607 , 608 , 621t uocino one, 691t, 705t ophtha mic, 691t, 705t
adverse e ects, 615 uorometo one, 691t, 705t O sa azine, 357t, 373t
didanosine, 601t, 603t, 607 , 608 , 621t otepredno , 691t, 705t in ammatory bowe disease, 498t, 503 ,
emtricitabine, 601t, 603t, 607 , 608 , 621t predniso one, 691t, 705t 506, 507, 509t
HIV, 614615 rimexo one, 691t, 705t NSAID properties, 357t, 373t
amivudine, 601t, 602t, 607 , 621t triamcino one, 691t, 705t Oma izumab, 390t, 397, 404t
mechanisms o action and resistance, immunosuppressives and antimitotics, Omeprazo e, 490t, 493 , 496t
602t603t, 604 , 608 , 614615 691t, 705t Onabotu inum toxin A, 106t, 117t
stavudine, 601t, 603t, 607 , 608 , 621t 5- uorouraci (FU), 691t, 705t Onabotu inumtoxin A, 692t, 706t
structures, 607 cyc osporine A, 691t, 705t Onabotu inumtoxin, A ophtha mic, 692t, 706t
teno ovir disoproxi , 601t603t, 607 , mitomycin, 691t, 705t Onchocerciasis (Onchocerca volvulus),
608 , 621t Jimson weed, 694 , 698, 698t 537540
za citabine, 601t, 621t macu ar degeneration agents, 692t, 706t Ondansetron
zidovudine, 601t, 602t, 607 , 608 , 621t bevacizumab, 692t, 706t antidiarrhea , 498t, 501, 502t, 505,
Nystatin, 599t pegaptanib, 692t, 706t 506, 508t
ranibizumab, 692t, 706t chemotherapy prophy axis, 505, 506
O vertepor n, 692t, 703, 706t mechanisms o action, 498t
Octreotide, 422t, 428t mydriasis, 694 , 698, 698t pharmaco ogy, 130t, 133t, 147t
antidiarrhea , 498t, 508t NSAIDs, 691t, 705t 1-Hydroxycho eca ci ero , 481t, 488t
Ocu ar administration, e der y, 74t brom enac, 691t, 705t 1-Hydroxy ase, 482 , 486, 487
Ocu ar pharmaco ogy, 690706 dic o enac, 691t, 705t Onychomycosis, 595596, 721, 722
absorption pathways, 697 urbipro en, 691t, 705t Ophtha mic, brinzo amide, 691t, 705t
antibacteria s, topica , 692t693t ketoro ac, 691t, 705t Ophtha mic pharmaco ogy. See Ocu ar
anti unga s, 694t, 701 nepa enac, 691t, 705t pharmaco ogy
antihistamines and mast ce stabi izers, prostag andin ana ogs, 691t, 705t Ophtha mic surgery agents, 691t692t, 706t
691t, 705t bimatoprost, 691t, 705t chondroitin su ate, 692t, 706t
aze astine, 691t, 705t atanoprost, 691t, 705t cyanoacry ate, 692t, 706t
bepotastine, 691t, 705t travoprost, 691t, 705t brogen ge , 692t, 706t
cromo yn sodium, 691t, 705t route o administration, 697t hya uronate, 692t, 706t

763
hydroxypropy methy ce u ose, Opioid receptor agonists, 194t195t, mannito , 268t, 283t
692t, 706t 201208, 202t, 208t209t urea, 268t, 283t
po ydimethy si oxanes, 692t, 706t actions and receptor se ectivities, Osteoc ast ormation, RANKL, 482
povidone iodine, 691t, 706t 194t195t Osteodystrophy, rena , 482 , 484
vitreous substitutes, 692t, 706t adjuvant ana gesic use, 205, 207 vitamin D, 481t, 482 , 484, 488t
Opiates. See also Opioid pharmaco ogy; a entani , 195t, 208t Osteoma acia, 484
speci c types codeine, 195t, 200t, 206, 208, 208t Osteonecrosis o jaw, bisphosphonate, 481t,
adverse e ects dosing, 200t 485, 488t
constipation, 198199 entany , 194t, 195t, 203, 205207, 208t Osteoporosis. See also Bisphosphonates
mood, reward, and addiction, hydrocodone, 195t, 200t, 208t denosumab, 488t
197198, 198 hydromorphone, 194t, 195t, 200t, 208t ibandronate, 481t, 485, 488t
respiratory depression, 196b, 199 evorphano , 194t, 195t, 200t, 209t teriparatide, 481t, 488t
e der y, 71, 77b mechanisms o action, 195t types and pathophysio ogy, 482 ,
Opioid addiction, 259262, 260 , 261t meperidine, 195t, 200t, 204, 208t 484485
overdose, 260 methadone, 194t, 195t, 200t, 201203, zo edronate, 481t, 485, 488t
to erance, 260 202t, 208t 22-Oxaca citro , 481t, 488t
withdrawa , 260, 261t morphine, 194t, 195t, 200t, Oxaci in, 551, 552t, 554, 555, 555t
Opioid addiction treatment, 260262 206208, 208t Oxa ip atin, 628t, 653t
buprenorphine and buprenorphine/ oxycodone, 195t, 200t, 208t mechanisms o action, 628b629b, 628t,
na oxone, 261262 oxymorphone, 195t, 200t, 208t 630 , 633
c onidine, 261 propoxyphene, 195t, 200t, 208t mechanisms o resistance, 628t, 632b,
detoxi cation, 260261 remi entani , 195t, 208t 634635
methadone, 260261 su entani , 194t, 195t, 205, 207, 208t toxicities, 632b633b, 633634
na trexone, 262 tapentado , 195t, 209t uses, 629t, 653t
Opioid antagonists, bowe disorders tramado , 195t, 200t, 209t Oxazepam, 176t, 189, 190, 191t, 192t
a vimopan, 498t, 507t Opioid receptor antagonists, 203205, 209t a coho withdrawa , 256
mechanisms o action, 498t actions and receptor se ectivities, 195t Oxazo idinones, 569t, 576t
methy na trexone, 498t, 499, 505, mechanisms o action, 195t Oxcarbazepine, 225t, 236t
506, 507t methy na trexone, 195t, 199, 209t Oxiconazo e, 588t, 598t
Opioid pharmaco ogy. See also speci c drugs na oxone, 195t, 203204, 205, Oxidative reactions, metabo ic, 32t
and drug classes 207, 209t Oxybutynin, 96t, 99t, 100, 101b, 101t, 113t
actions and receptor se ectivities, na trexone, 193t, 195t, 209t Oxycodone, 195t, 200t, 208t
194t195t therapeutic uses, 203204 concerns, 254
adverse e ects Opre vekin, 406t, 410411, 419t dependence, physica , 264, 266
constipation, 198199 Ora administration, 21t. See also speci c agents dose doub ing, 255, 255t
respiratory depression, 196b, 199, 201 bioavai abi ity, in ant, 72, 74 dose-response curves, idea ized,
antidiarrhea s, 195t, 209t e der y, 73t 255, 256
antitussives, 196t, 209t, 210t Ora contraceptives, 441t444t, 448450, ong-acting, 256
dosing data, 200t 458t. See also Contraceptives, ora ; to erance, 255, 256t
drug-re ated deaths, 51t Estrogen antagonists; Estrogens; toxidrome, 53t
mechanisms o action, 195t196t speci c agents Oxycodone-acetaminophen toxicity, 2829,
ana gesia, 196197, 196 , 197 Organic anion-transporting po ypeptide 5054, 50 , 52t, 207, 208
mood, reward, and addiction, (OA P), 24b, 38 Oxygen gas, therapeutic, 211t, 223t
197198, 198 Organophosphates, 53t, 63, 64, 102, 102b, Oxymetazo ine, 120t, 136, 137, 142t
metabo ism, rst-pass, 206, 207 103 , 103t, 104 Oxymorphone, 195t, 200t, 208t
pain diphenhydramine, 40t, 42t, 65t Oxytocin, 424
guide ines, 201t insecticides, 100t, 102, 102b, 103t, 104 , 115t
target and site o action, 205t mechanisms, 100t P
pain, cancer nerve gases, 100t, 103t, 116t PA-824, 579t, 587t
methadone, 201203, 202t poisoning, 63, 64 Pac itaxe , 642t, 648650, 656t
routes o administration, 202203 symptoms, 102, 102b mechanisms o action, 630 , 642t,
pain states, 197b, 197 toxidrome, 53t 648650
to erance, 206, 207 Organ transp ant therapy mechanisms o resistance, 642t, 644b
toxidrome, 53t genera princip es, 378b toxicities, 656t
withdrawa , 260, 261t immunosuppressant risks/adverse e ects, uses, 644t, 656t
Opioid receptor agonist/antagonists and 378379, 378b, 379 Paget disease
partia agonists, 194t195t, 209t immunotherapeutic agents bisphosphonates, 481t, 488t (See also
actions and receptor se ectivities, (See Immunotherapeutic agents; Bisphosphonates)
194t195t speci c agents) ca citonin, 481t, 488t
buprenorphine, 194t, 195t, 200t, pre-transp antation, 378 Pain
203, 209t rejection prophy axis, 378379 breakthrough, 202, 206, 207
butorphano , 194t, 195t, 200t, Orthostatic hypertension, 306, 306b incident, 202
203, 209t Orthosteric site, 3b mechanisms, 196197, 196 , 197
dosing, 200t Ose tamivir, 601t, 602t, 604 , 620t targets and sites o action, 205t
mechanisms o action, 195t Osmotic diuretics, 268t, 271t, 283t Pain, cancer
na buphine, 195t, 200t, 209t g ycerin, 268t, 283t methadone, 201203, 202t
pentazocine, 195t, 209t isosorbide, 268t, 283t routes o administration, 202203
764
Pain management. See also speci c agents Paroxysma ventricu ar tachycardia (PSV ), mechanisms o action, 630 , 636t, 641642
anticonvu sants, 205t 314t, 318 uses, 637t
COX-2 inhibitors, 205t Partia agonist, 4b, 4 , 7b, 9 Pentoxi y ine, 295b
drug targets and sites o action, 205t Passive di usion, 19b, 19 Peptic u cer disease, 571
morphine and opiates, 192 , 196b, kidney, 20b Percutaneous absorption, 710b, 711715,
197, 199, 205t (See also Opioid Passive immunization, 377t378t, 381t 711t, 713
pharmaco ogy) Passive transport, 19b, 19 e der y, 74t
NSAIDs, 205t Patho ogica toxicity, 46t important considerations, 711712, 711t
opioid (See Opioid pharmaco ogy) Patient-contro ed ana gesia (PCA), 202 rate- imiting step, 721, 722
tricyc ic antidepressants, 205t Peak e ect, 36 or topica in ections, 714715
Pain states, 197b, 197 . See also Opioid Peak p asma concentration, 514, 515, vehic es, 712714, 712t
pharmaco ogy 515 , 519 Pergo ide, 130t, 148t
Pa iperidone, 151t, 170t, 175t Pediatric Research Equity Act o 2003 Perindopri , 273t, 285t
Pa onosetron, 130t, 147t (PREA), 71b Periora tremor, 172t
2-PAM, 41t, 42t, 65t Pediatrics. See Chi dren Periphera nervous system (PNS)
Pamidronate, 481t, 488t Pegaspargase (PEG-l -asparaginase), neurotransmitters, 83t84t, 86b, 92
Panaeolus, 97 643t, 659t Periphera vascu ar disease, 295b
Pancreatic ce , 471 , 472 , 473 Peg grastim, 405t, 407t, 408 , 409410, 418t Permethrin, 708t, 725t
Pancuronium, 97 , 105 , 106 , 106t, Peginter eron a pha, 613614 Pernicious anemia, 417418. See also
107t, 116t Peginter erons, 601t Vitamin B12 de ciency
Panitumumab, 662t, 669672, 683t Pegvisomant, 422t, 424, 425 , 428t Peroxisome pro i erator-activated
Pantoprazo e, 490t, 493 , 496t Pegy ated recombinant GCSF receptors (PPARs), 341, 342
Papi ary thyroid cancer, 436 (peg grastim), 405t, 407t, 408 , Perphenazine, 150t, 174t
Para-aminopheno derivative. See 409410, 418t Pesticide poisoning, 89
Acetaminophen Pemetrexed (M A), 635t, 638b, 640, 653t PGE1 ana og, 356t
Parace u ar transport, 19b, 19 mechanisms and sites o action, 630 , pH
Para dehyde, 177t, 193t 635t, 639 di usion, 1920, 19b20b, 19 , 20 , 35,
Parasympathetic nervous system, 82 , mechanisms o resistance, 635t, 3738
85, 92, 107t 638b, 640 tissue uptake, 21b
Parathion, 101t uses, 637t urine, excretion, 20b, 2122
Parathyroid hormone (P H), 483 Pemiro ast, ophtha mic, 691t, 705t Pharmacodynamics, 217. See also speci c
Parecoxib, 363t Pemo ine, 120t, 141t drugs and drug classes
Parica cito , 481t, 488t Penbuto o , 128t active ingredient, 3
Parkinsonism, 172t Pencic ovir, 600t, 602t, 604 , 605, 605 , adverse e ects, 46, 5b, 912
Parkinsons disease, 239245, 250251 608, 619t a nity (KA), 6b, 8
basa gang ia, 240241, 241 Penetration, drug. See Barrier penetration a ergy drug case study, 36
cardina eatures, 239b Penici amine, 42t, 66t antihistamine, 3
dopamine neurotransmission, Penici in G, 552, 555t competitive antagonists, 4b, 5b, 5 , 7b,
240241, 241 Penici ins, 549t, 555t556t 16, 17
dopamine synthesis, neurona , 239, 240 a ergy, 550 , 552, 554, 555 concentrationresponse curve, 6b, 6 ,
pathophysio ogy, 239 aminog ycoside inactivation, 565, 566 11 , 12
prognosis, 241 amoxici in, 552t, 555t de nition, 2b
Parkinsons disease drugs, 239245, 250251, ampici in, 552t, 555t doseresponse curves, 4 7 , 89
252t253t az oci in, 552t drug deve opment, 69
amantadine, 239t, 242t, 245, 253t carbenici in, 552t, 556t drugreceptor interactions, 3b5b, 4
apomorphine, 239t, 242t, 253t c assi cation, 552t e cacy, 4b, 4 , 7b, 7 , 9
bromocriptine, 242t mechanisms o action, 549t, equi ibrium association constant (KA),
carbidopa, 238t, 244 550551, 550 6b, 8
carbidopa/ evodopa, 238t, 242, 250, mez oci in, 552t, 556t equi ibrium dissociation constant (KD),
251, 252t penici inase-resistant, 554, 555 6b, 8, 16, 17
dosing, 242t c oxaci in, 551, 552t, 555t ractiona occupancy ( ), 7b, 8
ear y treatment options, 242243, 242t, dic oxaci in, 551, 552t, 555t genetic po ymorphisms, 10t, 1214, 13
252t253t methici in, 551, 552t ha -maxima e ective concentration
entacapone, 238t, 242t, 244 , 251, 252t na ci in, 551, 552t, 554, 555, 555t (EC50), 6 , 7b, 9
evodopa, 238t, 242245, 243 , 244 , 252t oxaci in, 551, 552t, 555t imatinib, 1415
mechanisms o action, 238t239t penici in G and V, 552, 555t interindividua variabi ity, 12
pramipexo e, 238t, 242t, 245, piperaci in, 552t, 556t inverse agonists, 4, 4b, 4 , 9, 16, 17
250252, 252t resistance, 549t, 550b, 551 ionization, kidney tubu es, 2122
rasagi ine, 239t, 242t, 244 , 245, 253t structures, 550 postmarketing survei ance, 911, 4648
ropiniro e, 238t, 242t, 245, 250, 251, 252t ticarci in, 552t, 556t resistance, 15
se egi ine, 238t, 242t, 244 , 245, 250, toxicity/a ergy, 5556 speci city, 5b, 9
251, 253t Penici in V, 552, 555t therapeutic index, 11, 11 , 39b, 62, 64
to capone, 238t, 242t, 244 , 252t Pentamidine, 531t, 535t therapeutic window, 11, 12, 12 , 16, 17,
trihexyphenidy HC , 242t Pentazocine, 195t, 209t 35b, 36
Paromomycin, 531t, 532533, 535t Pentobarbita , 177t, 181t, 192t war arin sensitivity, 10t, 1214, 13 ,
Paroxetine, 130t, 146t, 150t, 157, 174t Pentostatin, 636t, 656t 329b330b

765
Pharmacogenetics. See also CYP (cytochrome Pharmacokinetics, chi dren si dena , 391t, 398402, 399 , 404t
P-450 super ami y); speci c drugs absorption, 67b tada a , 391t, 398402, 399 , 404t
o ic acid ana ogs, 637 , 639 distribution, 68b Photochemotherapy, 707t, 716t, 725t
high-dose methotrexate with eucovo- excretion, 69b methoxsa en, 707t, 716t, 725t
rin rescue (HDM-L), 635t, 637 , metabo ism, 70b photopheresis, 716t
638640, 639 p asma protein binding, 68b PUVA, 716717, 716t, 725t
methotrexate, 637 , 639 Pharmacokinetics, e der y trioxsa en, 716t
war arin dosing, 10t, 1214, 13 absorption and bioavai abi ity, 73b, 73 , Photodynamic therapy, 708t, 716t, 725t
Pharmacokinetics, 1839. See also speci c 73t74t amino evu inic acid, 708t, 725t
drugs and drug classes c earance, 75b, 76 , 76t77t, 77 methy amino evu inate, 708t, 725t
absorption, 20b, 20 distribution and vo ume o distribution, Photopheresis, 716t
acetaminophen, 2829, 5054, 50 , 52t, 74b, 75t Physio ogica receptors, 2b, 3t
207, 208 excretion, 76b, 78t Physostigmine (sa icy ate), 41t, 100t, 102, 114t
administration (See also Administration) rst-pass c earance, 73t74t antidote, 42t, 65t, 102
repeated, 38b, 38 ha - i e, 77 mechanisms o action, 100t
route o , 20b, 21t metabo ism, 73t74t, 75b Pi ocarpine, 95t, 99t, 112t
bioavai abi ity, 20b, 33b p asma concentrationtime curves, 73 , 73t mechanisms o action, 95t
biotrans orming enzymes, 29b, 29t p asma protein binding, 74b, 75t ophtha mic, 690t, 695t, 704t
b ood-brain barrier, 1920, 19b20b, 19 , tissue binding, 75t pharmaco ogy, 99t
20 , 28 , 35, 3738 Phase 1 unctiona ization, 28b, 33 , 35, 38 uses and toxicities, 112t
c earance, 33b, 35, 35b36b, 36 , neonata , 70b Pi ocarpine hydroch oride, 95t, 99t, 112t
37 , 39 Phase 1 metabo ism, 28b, 33 , 35, 38 Pimecro imus, 708t, 724t, 726t
c inica , 33b34b neonata , 70b Pindo o , 126t, 128t, 145t
c opidogre dosing, 22, 28, 331332 Phase 2 conjugation, 2829, 28b, 33 , Piog itazone, 470t, 475476, 479t
de nition, 2b 35, 39 Pipecuronium, 97 , 105 , 106 , 106t, 107t, 116t
digoxin, 3034, 308t neonata , 70b Piperaci in, 552t, 556t
diphenhydramine, 1920, 40t, 65t (See Phenazopyridine, 548t Piperazines, 352
also Diphenhydramine) Phene zine, 130t, 147t, 150t, 173t Pirbutero
distribution, 20 , 21b Phenindamine, 346t, 354t pharmaco ogy, 125t, 143t
CNS, 24b Phenobarbita , 177t, 181t, 189, 190, 192t pu monary, 389t, 402t
dosing, 3031 Phenoxybenzamine, 120t, 136, 137, 141t Pitavastatin (ca cium), 334t, 343t
drug interactions, 28 Phento amine, 120t, 141t Pituitary adenoma, pro actin-secreting, 426
e imination t1/2, 34b, 37 , 39b Pheny ephrine, 119t, 139t Pituitary g and
excretion, 20 ophtha mic, 690t, 695t, 705t anterior, 424
weak acids, 36, 39 Phenytoin posterior, 424
rst-pass e ect, 20b, 22, 35, 3637 antiseizure, 225t, 231, 234, 235, 236t Pituitary hormones, posterior, 423t, 429t
genetic po ymorphisms, 10t, e ectrophysio ogic actions, 316 arginine vasopressin, 423t, 424 , 429t (See
1214, 13 Pheochromocytoma, 136, 137 also Vasopressin)
interindividua variabi ity, 12 Phi ade phia chromosome-positive (Ph +) oxytocin, 423t, 424 , 430t
drug response and toxicity, 22, 24b, chronic mye ogenous eukemia, pKa, 1920, 19b20b, 20 , 35, 3738
28, 34b 664669 P asma concentrationtime curves,
ionization, p asmagastric juice rst- ine treatment, 665666 36b, 37
partitioning, 19b20b, mo ecu ar mechanism, 665666 e der y, 73 , 73t
20 , 2122 Phosphate binders, 481t, 484, 488t P asmagastric juice partitioning,
ionization state, 1920, 19b20b, 20 anthanum carbonate, 481t, 484, 488t 19b20b, 20 , 2122
ong-term therapy, rationa , 35, 39 mechanisms o action, 481t, 488t P asma protein binding, 22b
metabo ism, 20b, 2829, 28b, 32t rena osteodystrophy, 482 , 484 chi dren, 68b
phase 1, 28b, 33 , 35, 38 seve amer hydroch oride, 481t, 484, 488t e der y, 74b, 75t
movement and avai abi ity Phosphodiesterase 3 (PDE3) inhibitors, heart Plasmodium alciparum treatment, 579t, 581 ,
characteristics, 18b ai ure, 307, 309, 310, 311t 583586, 587t
nitrog ycerin, rst-pass e ect, 22, inamrinone, 302t, 308, 308t, 311t P ate et-activating actor (PAF), 359
291, 299 mechanisms o action, 302t, 308 P ate et adhesion and aggregation,
passive vs. active transport, 19b, 19 mi rinone, 302t, 308, 308t, 311t 324, 324
pH and di usion, 1922, 19b20b, 19 , pharmaco ogy, 308t P ate et drugs. See Antip ate et agents
20 , 35, 3738 Phosphodiesterase 5 (PDE5) inhibitors P atinum coordination comp exes, 628t,
pKa, 19b20b, 20 myocardia ischemia, 300t 633635, 653t
p asma protein binding, 22b nitrate interactions, 291b, 292 carbop atin, 628t, 629t, 653t
rate o distribution and equi ibration, 20 , si dena , 300t ce cyc e speci city, 630
36b37b, 37 tada a , 300t cisp atin, 628t, 629t, 632t, 653t
steady state, 19 , 3031, 34, 38b vardena , 300t mechanisms o action, 628t, 630 , 633
St. Johns wort, 2930, 33 , 34 ocu ar e ects, 703 mechanisms o resistance, 628t, 632b,
tissue binding, 22b23b pu monary, 391t, 404t 634635
transporters adverse e ects, 398, 399 , 400 oxa ip atin, 628t, 629t, 653t
ABC, 23t, 23t25t mechanisms o action, 391t toxicities, 632b633b, 632t,
membrane, 23b24b, 23t25t pu monary artery hypertension, 633634, 653t
SLC super ami y, 23b24b, 25t26t 398400, 399 uses, 629t, 653t

766
P icamycin, 481t, 488t Pravastatin (ca cium), 334t, 343t hydroxyprogesterone caproate, 679t
Podophy in, 727t Prazosin medroxyprogesterone, 664t,
Poisoning. See also oxicity, c inica antihypertensive, 277t, 286t 679t, 686t
and environmenta ; pharmaco ogy, 119t, 121124, 123 , 140t megestro acetate, 664t, 679t, 686t
speci c poisons Precocious puberty, 422t, 427, 428, 429t hormone rep acement therapy,
ABDE treatment, 52t Predniso one 442t, 458t
initia treatment, 63, 64 ophtha mic, 691t, 705t mechanisms o action, 442t, 458t
argest number ata ities, 52t pu monary, 390t, 403t medroxyprogesterone acetate (MPA),
management, 5052 Prednisone. See also Corticosteroids 442t, 458t
initia , 63, 64 adverse e ects, 467, 468 megestro acetate, 442t, 458t
princip es, 5052 cancer progesterone micronized, 442t,
most common substances, 51t chemotherapy, 664t, 679t, 686t 445t, 458t
most requent substances, 51, 51t target, 664t, 679t, 686t progesterone s ow-re ease intrauterine
pharmacokinetics, 52 cessation, adverse e ects, 465 device, 442t, 458t
prevention, chi dren, 6264 Crohns disease, 465, 467, 468 progesterone vagina ge , 442t, 458t
Po yc ona antibodies, 376t, 377 , 377t378t, dose tapering, 465 progesterone vagina insert,
378, 385t pu monary, 390t, 403t 442t, 458t
Po ycystic ovary syndrome, 452 rheumatoid arthritis, 463465 structura eatures, 451
Po ydimethy si oxanes, ophtha mic, Pregaba in, 225t, 237t Proguani , 520t, 529t
692t, 706t Pregnancy, 365 Prokinetics, bowe disorders, 497t, 507t
Po yethy ene g yco , 497t, 507t FDA ru es, 72b cisapride, 497t, 504, 506, 507t
Po ymixins, 573 iron supp ements, 406t, 413414, 419t domperidone, 497t, 507t
po ymyxin B, 569t, 576t Prehypertension, 269270, 270t erythromycin, 497t, 507t
po ymyxin E, 569t, 576t Premature ventricu ar beats, 314t ubiprostone, 497t, 507t
Po ymorphisms, genetic post-MI, 314t, 319, 320 mechanisms o action, 497
CYP, 27t, 28, 31b (See also CYP Pre-surgery prophy axis, 515b, 516 metoc opramide, 497t, 504, 506, 507t
(cytochrome P-450 super ami y)) Pri ocaine, 224t pruca opride, 497t, 507t
drug response, 10t, 1214, 13 , 27t Primaquine Pro actin
Po ymyxin-bacitracin-neosporin, 569t, 573, G6PD de ciency, 527, 528 GH receptor antagonism, 423, 425
576t, 577t ma aria secretion and actions, 424 , 425
Po ysaccharide errihydrite comp ex, chemoprophy axis, 522t, 525526 Pro actin-secreting pituitary adenoma, 426
406t, 419t chemotherapy, 521t, 522t, 525526, Promethazine, 350352, 354t
Po ythiazide, 269t, 284t 526t, 527, 529t antiemetic, 351, 352, 502t
Pork tapeworm, 539, 540 mechanisms o action, 521t co d medication, 346t, 350
Posaconazo e, 588t, 589 , 590, 597t parasite deve opment stage, 526t mechanisms o action, 346t
Postantibiotic e ect (PAE), 515 regimens, 522t motion sickness, 352
cepha osporins, 564, 565 Primary agonists, 3b, 8 Propa enone
Posterior pituitary hormones, 423t, 429t Primary site, 3b, 8 antiarrhythmic, 312t, 317t, 321t
arginine vasopressin, 423t, 424 , 429t (See Prinzmeta angina, 290t, 291 , 297. See also e ectrophysio ogic actions, 317t
also Vasopressin) Myocardia ischemia Proparacaine, 224t
oxytocin, 423t, 424 , 430t Probenecid, 369t, 370, 374t Propionic acid derivatives, 362t363t, 373t
Postmarketing survei ance, 911, Procainamide, 312t, 317t enopro en, 362t, 373t
4648 antiarrhythmic, 312t, 316, 317t, 320t urbipro en, 357t, 363t, 373t
Postsynaptic potentia s, autonomic, 106 e ectrophysio ogic actions, 317t ibupro en, 357t, 362t, 364365, 373t
Potency, 7b, 7 ventricu ar bri ation, 316 ketopro en, 357t, 363t, 373t
Potency, re ative, 7 Procaine, 224t naproxen, 357t, 362t, 373t
Potentiation Procarbazine, 628t, 652t oxaprozin, 357t, 363t, 373t
drugdrug interaction, 48b mechanisms o action, 628b629b, Propo o
mechanisms, 4b, 5 628t, 630 anesthetic, 210t, 213t, 214 , 220,
Povidone iodine, ophtha mic, 691t, 706t mechanisms o resistance, 628t, 221, 222t
Pra atrexate, 635t, 639, 653t 632633, 632b context-sensitive ha -time, 214
Pra idoxime ch oride (2-PAM), 41t, 42t, 65t, toxicities, 632b633b, 652t sedative-hypnotic, 177t, 193t
100t, 102, 116t uses, 629t, 652t use, common, 220, 221
mechanisms o action, 100t, 104 Procatero , 125t, 143t Propoxur, 100t, 115t
uses and toxicities, 41t, 42t, 65t, 100t, Prodrugs, 28b Propoxyphene, 195t, 200t, 208t
102, 116t Progesterone micronized, 442t, 445t, 458t Proprano o
Pramipexo e Progesterone receptor antagonists, antiarrhythmic, 312t, 317t, 321t
Parkinsons disease, 238t, 242t, 245, 250, 442t, 458t e ectrophysio ogic actions, 317t
251, 252, 252t Progesterone receptor modu ator, se ective, myocardia ischemia, 300t
pharmaco ogy, 130t, 148t u iprista , 442t, 444t, 452, 458t pharmaco ogy, 125t, 128t, 136,
Pram intide, 471t, 479t Progesterone s ow-re ease intrauterine 138, 144t
Pramoxine, 224t device, 442t, 458t Propy hexedrine, 120t, 142t
Prasugre , 323t, 324, 325, 325t, 332t Progesterone vagina ge , 442t, 458t Propy thiouraci , 431t, 439t
bioactivation, 325t Progesterone vagina insert, 442t, 458t Prostacyc in (PGI2) and ana ogs, 391t, 404t
mechanisms o action, 323t, Progestins adverse e ects, 400
330, 331 anticancer, 664t, 679t, 686t epoprosteno , 391t, 398400, 399 , 404t

767
i oprost, 391t, 404t neosporin, 573 Pseudoephedrine, 120t, 142t
mechanisms o action, 391t Oxazo idinones, inezo id, 569t, 576t Psilocybe, 97
treprostini , 391t, 404t streptogramins, quinupristin/da opristin, Psi ocybin, 97
Prostag andins and ana ogs 568t, 572, 575, 576b Psoriasis, 715716
mechanisms o action, 356t357t tetracyc ines, 568t, 569 , 576t ada imumab, 709t, 714t, 722, 723
ophtha mic, 691t, 705t doxycyc ine, 568t, 569 , 571, 576t bio ogic agent mechanisms o
bimatoprost, 691t, 705t ood contraindications, 574, 575 action, 710
atanoprost, 691t, 705t MRSA in ections, 571 immunopathogenesis, 711
travoprost, 691t, 705t photosensitivity, 574, 575 methotrexate, 715716, 721722
PGE1, 356t, 372t tetracyc ine, 568t, 569 , 576t Psychopharmaco ogy, 149175. See also
a prostadi , 356t, 372t tigecyc ine, 568t, 569 , 576t speci c drugs and drug classes
PGE1 ana og, 356t, 372t Protein tyrosine kinase inhibitors, anticonvu sants, ma aria, 151t, 175t
misoprosto , 356t, 372t 661t, 683t carbamazepine, 151t, 164165, 175t
PGE2, 356t, 372t dasatinib, 661t, 668, 683t amotrigine, 151t, 165, 175t
dinoprostone, 356t, 372t er otinib, 661t, 670672, 683t va proic acid, 151t, 164165, 169,
PGE2 ana og, 356t, 372t ge tinib, 661t, 670672, 683t 173, 175t
bimatoprost, 356t, 372t imatinib mesy ate, 661t, 666669, 681, antidepressants
carboprost, 356t, 372t 682, 683t actions and e ects, 152154, 152
atanoprost, 356t, 372t apatinib, 661t, 673, 683t adverse e ects, 154
travoprost, 356t, 372t mechanisms o resistance, 661t disposition, 158t
tromethamine, 356t, 372t ni otinib, 661t, 668, 683t phases o treatment, 153154
PGI2, 357t, 372t sora enib, 661t, 674, 683t sites o actions, 152
prostacyc in, 357t sunitinib, 661t, 674, 683t structures, dosing, and adverse e ects,
PGI2 ana og, 357t, 372t Prothrombin time (P ) assay, 328, 330b 155t156t
i oprost, 357t, 372t Proton pump inhibitors, 490t, 491 , 492, antidepressants, atypica , 150t, 156t, 174t
treprostini , 357t, 372t 493 , 496t atomoxetine, 150t, 174t
Prostate cancer dex ansoprazo e, 490t, 496t bupropion, 150t, 174t
utamide, 442t, 454, 456, 457, 679t e der y, 73b mianserin, 150t, 174t
eupro ide, 423t, 425426, 429t esomeprazo e, 490t, 495, 496t mirtazapine, 150t, 174t
metastatic, 678680 gastroesophagea re ux disease, ne azodone, 150t, 174t
Protease inhibitors, 601t, 603t, 621t622t 492, 493 reboxetine, 150t, 174t
amprenavir, 601t, 603t, 608 , 609 , 622t iron de ciency rom, 495 structures, dosings, and adverse
atazanavir, 601t, 603t, 608 , 609 , 622t ansoprazo e, 490t, 493 , 496t e ects, 156t
darunavir, 601t, 603t, 608 , 609 , 622t mechanisms o action, 490t, 491 , 495 trazodone, 150t, 174t
indinavir, 601t, 603t, 608 , 609 , 621t omeprazo e, 490t, 493 , 496t antipsychotics
opinavir, 601t, 603t, 615, 622t pantoprazo e, 490t, 493 , 496t adverse e ects, 172t
mechanisms o action and resistance, rabeprazo e, 490t, 493 , 496t e der y, 77b, 168, 169, 171
603t, 608 , 609 structures, 492, 492 , 493 nonpsychotic disorders, 163b
ne navir, 601t, 603t, 608 , 609 , 622t or u cers, 492, 493, 494t, 495 antipsychotics, atypica , 175t
ritonavir, 601t, 603t, 608 , 609 , vitamin B12 absorption, 492 aripiprazo e, 151t, 170t, 175t
615619, 622t Protozoa drugs, 530535 chemica structures, dosages, and main-
saquinavir, 601t, 603t, 608 , 609 , 621t amebic co itis, 532533 tenance, 160t161t
tipranavir, 601t, 603t, 608 , 609 , 622t amphotericin (B), 530t, 534, 535t metabo ism, 170t
Proteasome inhibitors, 663t, 681, benznidazo e, 530t531t, 535t o anzapine, 151t, 165, 168, 170t,
682683, 685t cryptosporidia diarrhea, 532 171172, 175t
Protein binding, drugdrug e ornithine, 530t, 535t potencies, 162t
interactions on, 48t umagi in, 530t, 535t quetiapine, 151t, 171t, 175t
Protein drug receptors, 2b giardiasis, 531532 risperidone, 151t, 168, 169,
Protein synthesis inhibitors, 568577 eishmaniasis, viscera , 534 171t, 175t
aminocyc ito s, spectinomycin, 569t, 576t mechanisms o action, 530t531t antipsychotics, typica , 151t, 175t
bacitracin, 569t, 577t me arsopro , 530t, 534, 535t asenapine, 151t, 170t, 175t
ch oramphenico , 568t, 569 , 571, 576t metronidazo e, 530t, 532534, 535t chemica structures, dosages,
incosamides, c indamycin, 568t, 574, mi te osine, 530t, 534, 535t and maintenance, 160t
575, 576t ni urtimox, 531t, 535t c ozapine, 151t, 167169, 170t,
macro ides and keto ides, 568t, nitazoxanide, 530t, 532, 535t 172, 175t
570 , 576t paromomycin, 531t, 532533, 535t i operidone, 151t, 170t, 175t
azithromycin, 568t, 570 , 576t pentamidine, 531t, 535t metabo ism, 170t
c arithromycin, 568t, 570 , sodium stibog uconate, 531t, pa iperidone, 151t, 170t, 175t
571, 576t 534, 535t potencies, 162t
drug interactions, 572 toxop asmosis, 532b sertindo e, 151t, 175t
erythromycin, 568t, 570 , 576t Trichomonas vaginalis, 533, 534 ziprasidone, 151t, 171t, 175t
te ithromycin, 568t, 570 , 576t Trypanosoma brucei rhodesiense, 534 antipsychotics, typica , other, 151t, 175t
toxicities, 571 trypanosomiasis, 533, 534 droperido , 151t, 175t
mechanisms o action, 568t569t, Protripty ine, 150t, 173t ha operido , 151t, 159, 163 , 171t, 175t
569 , 570 Pruca opride, 497t, 507t oxapine, 151t, 175t
MRSA, 571 Pruritus, 720, 720t mo indone, 151t, 175t

768
Psychopharmaco ogy (Cont.) structures, dosing, and adverse mechanisms o action, 390t, 392 , 393
receptor occupancy and c inica e ects, 155t methy predniso one, 390t, 403t
response, 163 trimipramine, 150t, 173t predniso one, 390t, 403t
antipsychotics, typica , phenothiazines, Psychosis drugs. See Antipsychotics prednisone, 390t, 403t
150t, 174t Pteroy g utamic acid (PteG u). See Fo ic acid endothe in-1 receptor antagonists,
ch orpromazine, 150t, 171t, 174t and derivatives 391t, 404t
uphenazine, 150t, 174t P H-re ated protein (P HrP), 483 adverse e ects, 400
perphenazine, 150t, 174t Pu monary artery hypertension (PAH), ambrisentan, 391t, 398400, 399 , 404t
tri uoperazine, 150t, 174t 398400, 399 bosentan, 391t, 398400, 399 , 404t
anxio ytics, benzodiazepine, 174t Pu monary drugs, 389404 mechanisms o action, 391t
anxio ytics, non-benzodiazepine, anti-IgE monoc ona antibodies, oma i- pu monary artery hypertension,
150t, 174t zumab, 390t, 397, 404t 398400, 399
buspirone, 150t, 174t anti eukotrienes, 390t, 403t404t expectorants, guai enesin, 390t, 404t
bipo ar disorder, 164166 5-LOX inhibitors, zi euton, 390t, 403t gastroesophagea re ux
e der y, 77b asthma, severe, 396 , 397 disease (GERD), 400
ithium, 151t, 164166, 175t eukotriene antagonists, 390t, 404t methy xanthines, 390t, 403t
adverse e ects, 164t monte ukast, 390t, 404t COPD, 395
e der y, 165166 za r ukast, 390t, 404t mechanisms o action, 390t
ibupro en, 165, 168, 169 mechanisms o action, 390t, 396 theophy ine, 390t, 392 , 397, 403t
ithium carbonate, 151t, 175t antitussives, 209t, 391t, 404t muco ytics, 390t, 404t
ithium citrate, 151t, 175t benzonatate, 391t, 404t DNase (dornase a a), 390t, 404t
overdose, 166167 codeine, 209t, 391t, 404t N-acety cysteine, 390t, 404t
monoamine oxidase inhibitors (MAOIs), dextromethorphan, 391t, 404t muscarinic cho inergic antagonists, 390t,
150t, 157, 173t mechanisms o action, 391t 401402, 403t
isocarboxazid, 150t, 173t asthma, 393397, 396 adverse e ects, 395
phene zine, 150t, 173t 2 adrenergic receptor agonists, asthma, severe, 397
se egi ine, 150t, 173t ong-acting, 390t, 402t COPD, 394395
SSRI a er, 157 adverse e ects, 392b, 398 ipratropium bromide, 390t, 402, 403t
structures, dosing, and adverse ar ormotero , 390t, 402t mechanisms o action, 390t
e ects, 156t asthma, 393394, 396397, 402 tiotropium bromide, 390t,
trany cypromine, 150t, 173t COPD, 395 401402, 403t
psychosis, acute, 160t161t ormotero , 390t, 402t phosphodiesterase 5 inhibitors,
schizophrenia indacatero , 390t, 402t 391t, 404t
chemica structures, dosages, and main- mechanisms o action, 390t, 391b, adverse e ects, 398, 399 , 400
tenance, 160t161t 391 , 394 mechanisms o action, 391t
with drug abuse history, 159, 162163 sa metero , 390t, 402t pu monary artery hypertension, 398
se ective serotonin reuptake inhibitors 2 adrenergic receptor agonists, 400, 399
(SSRIs) (See Se ective serotonin short-acting, 389t, 401, 402t si dena , 391t, 398402, 399 ,
reuptake inhibitors (SSRIs)) adverse e ects, 392b 404t, 703
serotonin/norepinephrine reuptake inhib- a butero , 389t, 402t tada a , 391t, 398402, 399 , 404t
itors (SNRIs), 150t, 154, 157, 174t asthma, 393394 prostacyc in (PGI2) and ana ogs,
de ay o onset, 157 eva butero , 389t, 402t 391t, 404t
desven a axine, 150t, 174t mechanisms o actions, 389t, 391b, adverse e ects, 400
disposition, 158t 391 , 394 epoprosteno , 391t, 398400,
du oxetine, 150t, 174t metaprotereno , 389t, 402t 399 , 404t
mi nacipran, 150t, 174t pirbutero , 389t, 402t i oprost, 391t, 404t
ven a axine, 150t, 154, 157, 174t terbuta ine, 389t, 402t mechanisms o action, 391t
serotonin/norepinephrine reuptake inhib- combination inha ers, 395, 397, 398, treprostini , 391t, 404t
itors (SNRIs), secondary amine 401402 pu monary artery hypertension,
tricyc ics, 150t, 173t COPD, 394395 398400, 399
amoxapine, 150t, 173t corticosteroids, inha ed, 390t, 403t venti atory stimu ants, doxapram,
desipramine, 150t, 173t adverse e ects, 393b, 397398 391t, 404t
maproti ine, 150t, 173t asthma, 393394, 395397 Pu monary edema, 303. See also Edema
nortripty ine, 150t, 173t bec omethasone dipropionate, 390t, 403t drugs; Heart ai ure
protripty ine, 150t, 173t bioavai abi ity, 467, 468 Pupi e ects, 698, 698t
structures, dosing, and adverse budesonide, 390t, 402, 403t, 460t Purine ana ogs, 636t, 641642, 655t656t
e ects, 155t candidiasis, ora , 467, 468 PUVA photochemotherapy, 716717,
serotonin/norepinephrine reuptake cic esonide, 390t, 403t 716t, 725t
inhibitors (SNRIs), tertiary amine COPD, 395 Pyoderma, 714
tricyc ics, 150t, 173t uniso ide, 390t, 403t Pyrazinamide, 578t, 580b, 586t
amitripty ine, 150t, 152154, 152 , 168, uticasone, 390t, 403t Pyridostigmine bromide, 100t, 114t
169, 173t mechanisms o action, 390t, 392 , 393 Pyridoxine, 406t, 419t
c omipramine, 150t, 173t mometasone, 390t, 403t mechanisms o action, 406t
disposition, 158t triamcino one, 390t, 403t pyridoxine, 406t, 419t
doxepin, 150t, 173t corticosteroids, systemic, 390t, 403t ribo avin, 406t, 419t
imipramine, 150t, 173t hydrocortisone, 390t, 403t Pyri amine, 346t, 353t

769
Pyrimethamine-su adoxine, 521t, 529t Rasagi ine, 239t, 242t, 244 , 245, 250, angiotensin (A 1) receptor b ockers
Pyrimidine ana ogs, 635t636t, 251, 253t (ARBs), diabetics, 274
640641, 654t Rasburicase, 369t, 374t c assi cation, 270t
5- uorouraci (5-FU), 630 , 635t, Rate o distribution, 20 , 36b37b, 37 direct renin inhibitors, 270t,
638b639b, 638 , 640641, 654t Reboxetine, 150t, 174t 273t, 286t
azacytidine, 636t, 654t Receptor, drug. See also speci c drugs a iskiren, 273t, 282, 283, 286t
capecitabine, 630 , 635t, 638 , 654t and receptors mechanisms o action, 273t
cytarabine (Ara-C), 630 , 635t, 639b, 648, con ormation-se ective drugs, 4, 4 structure, 275
649, 654t de nition, 2b e ects, 273t, 274
decitabine, 636t, 654t drugdrug interactions, on binding, 48t mechanisms o action, 270t, 273t, 278t
F oxuridine (FUdR), 630 , 635t, unctions, 2b physio ogic actors, 272 , 275b
638b639b, 638 , 654t nervous system, 89b structures, 275
gemcitabine, 630 , 636t, 639b, 654t nuc ear, 3t, 33t, 34 vasodi ators, 270t, 278t
mechanisms and sites o action, 630 , physio ogica , 2b Renin inhibitors, direct, 270t, 273t, 286t
635t636t, 638 , 640 primary (orthosteric) site, 3b, 8 a iskiren, 273t, 282, 283, 286t
mechanisms o resistance, 638b639b, Receptor antagonism, 48b mechanisms o action, 273t
640641 Receptor or activating NF-B igand structure, 275
toxicities, 637t, 654t (RANKL), 482 Renin inhibitors/vasodi ators, direct. See
uses, 637t, 654t Receptor occupancy theory, 6b Direct renin inhibitors
Recta administration, e der y, 73t Renin re ease pathways, 272
Q Rein orcement, negative, 265, 266 Repag inide, 470t, 472 , 479t
Q interva , ong, 313315, 315b Re ative e cacy, 7b, 7 , 9 Repeated administration, 38b, 38
Quanta dose-response phenomenon, Re ative potency, 7 Reserpine, 277t, 286t
44 , 45 Remi entani , 195t, 208t Resistance, 15, 515b, 517
Quazepam, 176t, 191t, 192t Rena epithe ia Na+-channe inhibitors, 269t, antimicrobia , 515b
Quetiapine, 130t, 147t, 151t, 171t, 175t 271t, 272 , 284t - actam antibiotics, 516517
Quinacrine, 708t, 725t ami oride, 269t, 284t - actamase, 517, 519
Quinago ide, 422t, 426, 428t triamterene, 269t, 282, 283, 284t dosing schedu e, 515
Quinapri , 273t, 285t Rena unction ef ux pumps, 515b, 517
Quinethazone, 269t, 284t e der y, 76b, 78t mutations, 517
Quinidine, antiarrhythmic, 312t, 317t, 320t in ants and chi dren, 69b suppression, dosing schedu e, 515
Quinine/quinidine, ma aria, 521t, 524, Rena osteodystrophy, 482 , 484 Respiratory depression, opiate, 196b,
527, 529t vitamin D, 481t, 484, 488t 199, 201
Quino ones, 544, 544 , 545, 548t Renin-angiotensin system (RAS) Retapamu in, 708t, 725t
cipro oxacin, 544, 544 , 545, 548t inhibitors, 270t Retep ase, 323t, 333t
evo oxacin, 548t ACE inhibitors, 270t, 273t, 285t Retinoic acid receptors (RARs), 717
mechanisms o action, 544 , 545 acute MI, 279 Retinoids, 707t, 717718, 717b, 724t
moxi oxacin, 548t adverse e ects, 276 acitretin, 707t, 724t
nor oxacin, 548t angioedema, 279 adapa ene, 707t, 724t
o oxacin, 548t benzapri , 273t, 285t a itretinoin, 707t, 724t
resistance, 544 captopri , 273t, 279, 285t bexarotene, 707t, 724t
Quinupristin/da opristin, 568t, 572, cautions, 274276 cytotoxic actions and pharmaco ogy, 645t,
575, 576b diabetics, 274 646t, 659t
ena apri , 273t, 274276, 275 , 285t isotretinoin, 707t, 721, 722, 724t
R ena apri at, 273t, 285t pregnancy warning, 721, 722, 724t
Rabbit syndrome, 172t osinopri , 273t, 285t tazarotene, 707t, 724t
Rabeprazo e, 490t, 493 , 496t isinopri , 273t, 281, 282, 285t, tretinoin, 707t, 724t
Radioactive iodine (131I), 431t, 435, 306, 309, 310 Rheumatoid arthritis, 380
437439, 440t mechanisms o action, 270t, 273t, Rheumatoid arthritis drugs, 357,
Ra oxi ene 274276, 275 368b, 368t
anticancer, 664t, 686t moexipri , 273t, 285t disease-modi ying anti-rheumatic drugs,
SERM, 442t, 458t perindopri , 273t, 285t 368b, 368t, 379380
Ra tegravir, 601t, 603t, 611 , 623t quinapri , 273t, 285t (See also Disease-modi ying anti-
Ra titrexed, 635t, 653t ramipri , 273t, 285t rheumatic drugs (DMARDs))
Rame teon, 177t, 189, 190, 192t structure, 275 immunotherapeutic agents, 380381
Ramipri , 273t, 285t trando apri , 273t, 285t NSAIDs, 366368, 380 (See also
Ranibizumab, 662t, 684t angiotensin II e ects, 274 Nonsteroida anti-in ammatory
Ranitidine angiotensin receptor b ockers (ARBs), drugs (NSAIDs))
gastric acid disease, 490t, 496t 270t, 273t, 285t prednisone, 463465
structure, 492 candesartan ci exeti , 273t, 285t Ribavirin, 601t, 602t, 620t
Rano azine, 290t, 298, 299, 300t irbesartan, 273t, 285t adverse e ects, 614
Rapamycins osartan, 273t, 285t hepatitis C, chronic, 613614
anticancer, 663t, 675676, 685t o mesartan medoxomi , 273t, 285t Ribo avin, 406t, 419t
congeners, temsiro imus, 663t, 675676 structure, 275 Ri abutin, 578t, 579 , 580 , 586t
mechanisms o action, 662t, 675 te misartan, 273t, 285t Ri ampin, 581586, 586t
toxicities, 675676 va sartan, 273t, 280, 282, 283, 285t adverse e ects, 582

770
Ri ampin (Cont.) S Se ective estrogen receptor modu ators
H. in uenzae meningitis prophy axis, Sa ety (SERMS), 442t, 458t
578t, 579 , 581582, 586t FDA Sa ety and Innovation ra oxi ene, 442t, 458t
mechanisms o action, 578t, 579 Act o 2012, 72b tamoxi en, 442t, 451, 456, 457,
mechanisms o resistance, 578t, 580 medication, chi dren, 71b72b 458t, 667
tubercu osis, 578t, 579 , 581586, 586t Sa butamo toremi ene, 442t, 458t
Ri amycins, 578t, 581586, 586t pharmaco ogy, 125t, 127, 135, Se ective estrogen receptor modu ators
H. in uenzae meningitis prophy axis, 137, 143t (SERMs), targeted anticancer,
578t, 581586, 586t pu monary, 389t, 402t 664t, 686t
tubercu osis, 578t, 586t Sa icy ate, 357t, 373t mechanisms o resistance, 664t
mechanisms o action, 578t, 579 Sa icy ate toxidrome, 53t ra oxi ene, 664t, 686t
mechanisms o resistance, 578t, 580 Sa ine axative, 497t, 507t tamoxi en, 664t, 667 , 676678, 677t,
ri abutin, 578t, 586t Sa metero 679t, 686t
ri ampin, 578t, 579 , 581586, 586t pharmaco ogy, 125t, 143t toremi ene, 664t, 677t, 679t, 686t
ri apentine, 578t, 586t pu monary, 390t, 402t uses, 677t, 686t
Ri apentine, 578t, 579 , 580 , 586t Sa sa ate, 357t, 373t Se ective progesterone receptor modu ator
Rigidity agents, cho inergic, 106t, 117t Saquinavir, 601t, 603t, 608 , 609 , 621t (u iprista ), 442t, 444t, 452, 458t
Ri onacept, 376t, 386t Sargramostim, 405t, 407t, 408 , Se ective serotonin reuptake inhibitors
Ri uzo e, 239t, 249, 251, 252, 253t 409410, 418t (SSRIs), 130t, 131133, 131 , 136,
Rimabotu inum toxin B, 106t, 117t Sarin, 100t, 103t, 115t 138, 146t
Rimantadine, 601t, 602t, 604 , 620t Saxag iptin, 471t, 472 , 479t case question, 136, 138
Rimexo one Saxitoxin, musse s, 221, 222 cita opram, 150t, 174t
ophtha mic, 691t, 705t Schizophrenia de ay o onset, 157
Rimonanbant, 98t drug abuse history, 159, 162163 disposition, 158t
Risedronate, 481t, 488t drug structures, dosages, and mainte- drug interactions, 157
Risperidone, 130t, 147t, 151t, 168, 169, nance, 160t161t uoxetine, 150t, 157, 174t
171t, 175t Scopo amine uvoxamine, 150t, 174t
Ritodrine, 125t, 144t antiemetic, 502t MAO inhibitors with, 133
Ritonavir, 601t, 603t, 608 , 609 , motion sickness, 346t, 350, 352 a er MAOIs, 157
615619, 622t ophtha mic, 690t, 695t, 704t mechanisms o action, 136, 138
adverse e ects, 615 pharmaco ogy, 96t, 112t paroxetine, 150t, 157, 174t
on CP3A4, 617619 Sea ood a ergies, 349350 sertra ine, 150t, 174t
drug interactions, 616 Secobarbita , 177t, 180, 192t sites o actions, 152
mechanisms o action and resistance, Sedation, drug-induced. See also structures, dosing, and adverse e ects,
603t, 608 , 609 , 615 speci c drugs 155t156t
Rituximab hyperexcitabi ity a er, 8990, 94 Se ective -ce costimu ation
anticancer, 662t, 671t, 681, Sedative-hypnotics, 176193, 177t, b ockers, 387t
682, 684t 191t193t. See also Hypnotics and Se egi ine
rheumatoid arthritis, 368t sedative-hypnotics; speci c drugs Parkinsons disease, 238t, 242t, 244 , 245,
toxicities, 684t and drug classes 250, 251, 253t
uses, 684t adverse e ects, 179 psychopharmaco ogy, 150t, 173t
Rivaroxaban, 323t, 328, 333t barbiturates, 177t, 180182, 181t, 192t Semustine (methy -CCNU), 627t, 652t
Rivastigmine, 100t, 115t benzodiazepine receptor agonists, nove , mechanisms o action, 627t,
A zheimers disease, 239t, 246, 177t, 192t 628b, 630
247t, 253t benzodiazepine receptor antagonists, mechanisms o resistance, 627t,
Rizatriptan, 130t, 147t umazeni , 41t, 42t, 65t, 177t, 632633, 632b
RNA virus rep icative cyc es, 604 179180, 192t toxicities, 652t
Rocuronium, 97 , 105 , 106 , 106t, benzodiazepines, 176t, 178b, 180, uses, 629t
107t, 117t 189190, 191t192t (See also Serotonergic (5-H ) 5-H receptor
Romidepsin (depsipeptide), 645t, 660t Benzodiazepines) agonists, 130t, 133134, 133t,
Romip ostim, 406t, 417, 418, 419t GABAA receptor, 178 , 179 134 , 147t
Ropiniro e insomnia categories, 178b, 179 Serotonergic (5-H ) 5-H receptor antago-
Parkinsons disease, 238t, 242t, 245, 250, ong-term use, 179 nists, 130t, 131133, 133t, 147t
251, 252t me atonin congeners, rame teon, 177t, Serotonergic (5-H ) pharmaco ogy
pharmaco ogy, 130t, 148t 189, 190, 192t actions and c inica indications,
Ropivacaine, 224t toxidrome, 53t 131133, 133t
Rosig itazone, 470t, 475476, 479t Seizure disorders adverse e ects, 133
Rosuvastatin (ca cium), 334t, 338, 343t agents and princip es, 227b, 228 a motriptan, 130t, 147t
Rotigotine, 130t, 148t c assi cation, 228, 229t a osetron, 130t, 147t
Route o administration. See Administration, drug choice, 227b buspirone, 130t, 133t, 136, 138, 147t
routes ear y diagnosis and treatment, 228 cita opram, 130t, 146t
Rubidomycin, 643t, 658t genera princip es, 227b c ozapine, 130t, 147t, 170t
mechanisms o action, 630 , 643t, treatment (See Antiseizure drugs) desven a axine, 130t, 147t
645647 Se ective estrogen-receptor downregu ators do asetron, 130t, 147t
mechanisms o resistance, 645b (SERDs), 664t drug interactions, 133
Ru namide, 226t, 237t u vestrant, 664t, 677t, 686t du oxetine, 130t, 147t

771
e etriptan, 130t, 147t structures, dosing, and adverse bupropion or, 259
ergot a ka oids and derivatives, 120t, e ects, 155t negative rein orcement, 265, 266
141t, 147t trimipramine, 150t, 173t withdrawa syndrome, 258259, 259t
escita opram, 130t, 146t ven a axine, 150t, 154, 157, 174t Sodium 2,3-dimercaptopropane su onate
uoxetine, 130t, 131133, 131 , 133t, Serotonin (5-H ) receptor agonists, 130t, (DMPS), 41t, 66t
146t, 157 133134, 133t, 134 , 147t Sodium channe b ockers. See Na+ channe
uvoxamine, 130t, 146t Serotonin (5-H ) receptor antagonists, 130t, b ockers
rovatriptan, 130t, 147t 131133, 133t, 147t Sodium channe s. See Na+ channe s
granisetron, 130t, 147t Serotonin (5-H ) receptors Sodium erric g uconate, 406t, 419t
isocarboxazid, 130t, 147t autoreceptor c asses, 134, 134 Sodium iodide, 431t, 440t
ketanserin, 133t, 147t subtypes, 132 Sodium iodine (131I), 431t, 435, 437439, 440t
MAO inhibitors Serotonin (5-H 2) receptors, MDMA and Sodium phosphate, 497t, 507t
a tering 5-H concentrations, DOM a nity, 265, 266 Sodium stibog uconate, 531t, 534, 535t
130t, 147t Sertaconazo e, 588t, 598t Sodium thiopenta , 210t, 213t, 214 , 222t
oods prohibited, 135, 137 Sertindo e, 151t, 175t So i enacin, 96t, 101t, 113t
mechanisms o action, 130t Sertra ine, 130t, 133t, 136, 138, 146t, Soman, 100t, 103t, 116t
methysergide, 130t, 147t 150t, 174t Somatic motor nerves, 82
mi nacipran, 130t, 147t Seve amer, 481t, 488t neurotransmitters, 82 , 93, 94 (See also
naratriptan, 130t, 147t Seve amer hydroch oride, 481t, 484, 488t speci c types)
o anzapine, 130t, 147t Sevo urane, 210t, 215t, 223t Somatic nervous system, 82 , 85, 92
ondansetron, 130t, 133t, 147t Short-acting inha ed 2 agonists (SABAs). See Somatostatin (SS ), 424 , 433
pa onosetron, 130t, 147t 2-adrenergic receptor agonists, Somatostatin (SS ) ana ogs, 422t, 428t
paroxetine, 130t, 146t, 157 short-acting anreotide, 422t, 428t
phene zine, 130t, 147t Sibutramine, 130t, 147t octreotide, 422t, 428t
quetiapine, 130t, 147t Si dena Sora enib, 661t, 674, 683t
receptor subtypes, 132t myocardia ischemia, 300t Sota o
risperidone, 130t, 147t ocu ar e ects, 703 antiarrhythmic, 312t, 317t, 321t
rizatriptan, 130t, 147t pu monary, 391t, 398402, 399 , 404t e ectrophysio ogic actions, 317t
sertra ine, 130t, 133t, 136, pu monary artery hypertension, 398402 Spasticity agents, cho inergic, 106t, 117t
138, 146t Si odosin, 119t, 140t Specia popu ations, 6778. See also
sibutramine, 130t, 147t Simvastatin, 334t, 343t Chi dren; E der y
sumatriptan, 130t, 133134, 133t, Sinus bradycardia, vasospastic angina with, Speci city. See also speci c agents
134 , 147t 297, 298 adverse e ects, 9
trany cypromine, 130t, 135, Siro imus (rapamycin) de nition, 5b
137, 147t immunotherapy, 375t, 377 , 383, 384, 385t high (narrow), 5b, 9
ven a axine, 130t, 147t mechanisms o action, 375t, 383, 384 ow (broad), 5b, 9
zo mitriptan, 130t, 147t Sitag iptin, 471t, 472 , 479t Spectinomycin, 569t, 576t
Serotonin (5-H ) 6-Mercaptopurine (6-MP), 636t, Spina anesthesia, 217219
in migraine, 133134, 133t 641642, 655t Spirono actone, 269t, 285t
synthesis and inactivation, 131 mechanisms o action, 630 , 636t, adverse e ects, 307
Serotonin, norepinephrine, and dopamine 641642 heart ai ure, 302t, 307, 311t
reuptake inhibitors, 130t, 147t mechanisms o resistance, 636t, Stab e angina, 290t, 291 . See also
Serotonin/norepinephrine reuptake inhibi- 639b, 642 Myocardia ischemia
tors (SNRIs), 130t, 147t, 150t, 154, toxicities, 642, 655t Staphylococcus aureus treatment,
157, 174t uses, 637t, 655t cyc oserine, 579t, 587t
de ay o onset, 157 6-T ioguanine (6- G), 630 , 636t, 655t Statins. See HMG-CoA reductase
desven a axine, 150t, 174t mechanisms o action, 630 , 636t, inhibitors (statins)
disposition, 158t 641642 Stavudine, 601t, 603t, 621t
du oxetine, 150t, 174t mechanisms o resistance, 636t mechanisms o action and resistance,
mi nacipran, 150t, 174t Skin absorption, 710b, 711715, 711t, 713 603t, 607 , 608
secondary amine tricyc ics, 150t, 173t important considerations, 711712, 711t structure, 607
amoxapine, 150t, 173t neonates, in ants, and chi dren, 67b Steady state
desipramine, 150t, 173t rate- imiting step, 721, 722 dosing, 19 , 3031, 34, 38b
maproti ine, 150t, 173t or topica in ections, 714715 repeated administration, 38b, 38
nortripty ine, 150t, 173t vehic es, 712714, 712t Stem ce actor (SCF), 407t, 408
protripty ine, 150t, 173t Skin disorders. See Dermato ogic disorders Stents, drug-e uting coronary, 297, 297b
structures, dosing, and adverse Skin structure, 713 Steroidogenesis inhibitors, 665t, 687t
e ects, 155t SLC super ami y transporters, 23b24b, abiraterone, 665t, 668 , 680, 687t
sites o actions, 152 25t26t ketoconazo e, 665t, 687t
tertiary amine tricyc ics, 150t, 173t S eep apnea, opiates and respiratory depres- Steroid synthesis pathways, 668
amitripty ine, 150t, 152154, 152 , 168, sion, 196b, 199, 201 St. Johns wort
169, 173t S eeping sickness, 534 on CYP3A4, 619
c omipramine, 150t, 173t Sma mo ecu es, rheumatoid arthritis, 368t pharmacokinetics, 2930, 33 , 34
disposition, 158t Smoking, 258259 Strabismus agents, 692t, 706t
doxepin, 150t, 173t addiction, 258259 abobotu inumtoxin A, 692t, 706t
imipramine, 150t, 173t benzo[a]pyrene, 41t, 6365 onabotu inumtoxin A, 692t, 706t

772
Streptogramins, quinupristin/da opristin, adrenergic neuron b ocking agents, antiandrogens, nonsteroida , 665t,
568t, 572, 575, 576b 277t, 286t 679t, 687t
Streptomycin, 559566, 559b, 567t guanadre , 277t, 287t bica utamide, 665t, 687t
Streptozocin (streptozotocin), 627t, 652t reserpine, 277t, 286t utamide, 665t, 679t, 687t
mechanisms o action, 627t, 628b, 630 1 adrenergic receptor antagonists, 277t ni utamide, 665t, 687t
mechanisms o resistance, 627t, doxazosin, 277t, 286t antiandrogens, steroida , 665t, 687t
632633, 632b prazosin, 277t, 286t cyproterone, 665t, 687t
uses, 629t terazosin, 277t, 286t megestro , 665t, 687t
Stroke, tPA or, 323t, 329330, 329 , 333t adrenergic receptor antagonists, antiestrogens, aromatase inhibitors, 664t,
Subcutaneous administration, 21t doxazosin, 277t 679t, 686t
e der y, 74t centra y acting adrenergics anastrozo e, 664t, 677t, 679t, 686t
Suberoy ani ide hydroxamic acid (SAHA), 2 adrenergic receptor antagonists, exemestane, 664t, 677t, 679t, 686t
645t, 646t, 660t 277t, 286t etrozo e, 664t, 677t, 679t, 686t
Sub ingua administration, 22 c onidine, 277t, 286t antiestrogens, se ective estrogen receptor
e der y, 73t guanabenz, 277t, 286t downregu ators (SERDs), 664t
Succimer, 41t, 42t, 66t guan acine, 277t, 286t u vestrant, 664t, 677t, 686t
Succinimides, antiseizure, 225t, 236t methy dopa, 277t, 286t antiestrogens, se ective estrogen receptor
ethosuximide, 225t, 230231, 234, mechanisms o action, 277t modu ators (SERMs),
235, 236t Sympathomimetics, 120t, 123 , 141t142t. 664t, 686t
methsuximide, 225t, 236t See also -Adrenergic receptor mechanisms o resistance, 664t
Succiny cho ine, 97 , 105 , 107t, 108, 116t agonists ra oxi ene, 664t, 686t
Sucra ate, 490t, 496t Sympathomimetic toxidrome, 53t tamoxi en, 664t, 667 , 676678, 677t,
Su entani , 194t, 195t, 205, 207, 208t Synergistic drugdrug interaction, 48b 679t, 686t
Su bactam, 549t, 553, 558t Synthetic conjugated estrogens, 441t toremi ene, 664t, 677t, 679t, 686t
Su conazo e, 588t, 598t Syntopic interaction, 4b, 5 , 8 uses, 677t, 686t
Su adiazine, 543 , 543t, 547t co ony-stimu ating actors, 663t, 685t
Su adoxine, 521t, 529t, 543t, 548t T dose and toxicities, 671t
Su amethoxazo e, 5657 abun, 100t, 103t, 116t EGFR (ErbB1), 667t
a ergy, 5657 achyarrhythmia estrogens, 664t, 665t, 679t, 686t, 687t
urinary tract in ections, 543 , 543t, 547t drugs (See Antiarrhythmic drugs) (See also Estrogens)
Su apyridine, 502, 503 , 505506, 507 mechanisms, 313b g ucocorticoids, 664t, 686t
Su asa azine, 357t, 368t, 373t, 381 acrine dexamethasone, 664t, 686t
adverse e ects, 368t A zheimers disease, 239t, 247t, 253t prednisone, 664t, 679t, 686t
Crohns disease and u cerative co itis, 381, pharmaco ogy, 100t, 115t gonadotropin-re easing hormone
498t, 502, 503 , 505, 507, 509t acro imus, 375t, 377 , 383, 385t agonists, 664t, 679680, 687t
in ammatory bowe disease, 381, 498t, dermato ogic, 708t, 724t, 726t busere in, 664t, 687t
502, 503 , 505, 507, 509t mechanisms o action, 375t, 383, 384 des ore in, 664t
rheumatoid arthritis, 368t ada a GnRH, 664t, 680, 687t
structure, 503 myocardia ischemia, 300t gosere in, 664t, 687t
su apyridine in, 502, 503 , 505, 507 ocu ar e ects, 703 histre in, 664t, 687t
Su ation, neonata , 70b pu monary, 391t, 398402, 399 , 404t eupro ide, 664t, 679t, 680, 687t
Su soxazo e, in ants, 68 pu monary artery hypertension, 398402 na are in, 664t, 686t
Su te, oca anesthetics, 220, 221 Taenia solium, 539, 540 triptore in, 664t, 686t
Su onamides amoxi en, 442t, 451, 456, 457, 458t, 676678 gonadotropin-re easing hormone
absorption and excretion kinetics, 543t anticancer, 664t, 667 , 676678, 677t, antagonists, 665t, 686t
bacteria vs. mamma ian sensitivity, 679t, 686t abare ix, 665t, 687t
546, 547 bene cia e ects, 677678 cetrore ix, 665t, 687t
ma enide, 546, 547, 548t mechanisms o action, 442t, 664t, 676 degare ix, 665t, 687t
pregnancy, 546, 547 mechanisms o resistance, 664t, 677 ganire ix, 665t, 687t
si ver su adiazine, 543t, 547t structure and metabo ites, 667 inter eukin-2 receptor agonists,
structures, 543 toxicities, 677678, 686t 663t, 685t
su acetamide, 543 , 543t, 547t uses, 676, 677t, 686t a des eukin, 663t, 682, 683, 685t
su adiazine, 543 , 543t, 547t amsu osin, 119t, 140t deni eukin di itox, 663t, 671t, 685t
su adoxine, 543t, 548t apentado , 195t, 209t mechanisms o resistance, 663t
su amethoxazo e, 543 , 543t, 547t ape test, 536 monoc ona antibodies: CD20, 662t,
su asa azine, 543t, 547t apeworm 667t, 684t
su soxazo e, 543 , 543t, 547t sh, 537540 99
Y-ibritumomab, 662t, 671t, 684t
Su indac, 357t, 361t, 373t pork, 539, 540 131
I-tositumomab, 662t, 671t, 684t
Sumatriptan, 130t, 133134, 133t, ardive dyskinesia, 172t dose and toxicities, 671t, 684t
134 , 147t arget mechanisms o resistance, 662t
Sunitinib, 661t, 674, 683t drug, 2b (See also Receptor, drug) o atumumab, 662t, 684t
Sunscreens, 710t, 719b, 726t drugabi ity, 7 rituximab, 662t, 671t, 681, 682, 684t
Sympathetic nervous system, 82 , 85, va idity, 7 monoc ona antibodies: CD33, 662t, 684t
92, 107t argeted anticancer therapies, 661687 gemtuzumab ozogamicin, 662t, 667t,
Sympatho ytics, antihypertensive, androgens, 664t, 679t, 686t (See also 672t, 684t
270t, 277t Androgens; speci c types) mechanisms o resistance, 662t

773
monoc ona antibodies: CD52, 662t, 667t, -ce costimu ation b ockers, se ective, 387t etraethy ead, 57
671t, 684t ear substitutes. See Wetting agents/tear etrahydrozo ine, ophtha mic, 690t,
a emtuzumab, 662t, 667t, 671t, 684t substitutes 695t, 705t
mechanisms o resistance, 662t eicop anin, 569t, 570 , 577t T a idomide, 381t
monoc ona antibodies: growth e bivudine, 601t, 602t, 607 , 608 , 620t anticancer, 663t, 666 , 685t
actor antibodies (VEGF), 662t, e ithromycin, 568t, 570 , 576t dermato ogic, 709t, 724t, 726t
673675, 684t e misartan, 273t, 285t immunostimu ant, 381t
bevacizumab, 662t, 671t, 674675, 684t emazepam, 176t, 191t, 192t T eophy ine
dose, 671t emozo omide, 652t asthma, 397
ranibizumab, 662t, 684t mechanisms o action, 628b, 628t, 630 pu monary, 390t, 392 , 397, 403t
toxicities, 671t, 674675, 684t mechanisms o resistance, 628t, T erapeutic drug, 44 , 46, 46t
monoc ona antibodies: growth actor 632633, 632b monitoring, 39b
receptors, 662t, 683t toxicities, 632b633b, 652t toxicity, 44 , 46, 46t
cetuximab, 662t, 669672, 683t uses, 629t T erapeutic gases. See Gases, therapeutic;
dose and toxicities, 671t emsiro imus, 663t, 675676, 685t speci c agents
EGFR targeting, 669672 enectep ase, 323t, 333t T erapeutic index, 11, 11 , 62, 64
HER2/neu, 667t, 682t eniposide (VM-26), 643t, 657t determining, 44 , 45
panitumumab, 662t, 669672, 683t mechanisms o action, 630 , 643t digoxin, 31
trastuzumab, 662t, 671t, 672673, 681, mechanisms o resistance, 643t, 645b narrow, 22b, 31, 39b
682, 683t toxicities, 657t T erapeutic window, 11, 12, 12 , 16, 17
m OR inhibitors, 663t, 685t uses, 644t, 657t c earance, 35b
evero imus, 663t, 685t eno ovir disoproxi , 601t603t, 621t in vitro binding studies, 8
mechanisms o action, 663t, 675 mechanisms o action and resistance, tempora characteristics, 36
mechanisms o resistance, 663t, 676 602t, 603t, 607 , 608 T iazide/thiazide- ike diuretics, 269t, 271,
rapamycin, 663t, 675676, 685t structure, 607 271t, 272 , 284t
temsiro imus, 663t, 675676, 685t erazosin bendro umethiazide, 269t, 284t
progestins, 664t, 679t, 686t antihypertensive, 277t, 286t ch orothiazide, 269t, 284t
hydroxyprogesterone caproate, 679t pharmaco ogy, 119t, 140t ch ortha idone, 269t, 284t
medroxyprogesterone, 664t, erbina ne, 588t, 589 , 596, 598t hydroch orothiazide, 269t, 271, 273274,
679t, 686t erbuta ine 282, 283, 284t
megestro acetate, 664t, 679t, 686t pharmaco ogy, 125t, 143t hydro umethiazide, 269t, 284t
proteasome inhibitors, bortezomib, 663t, pu monary, 389t, 402t indapamide, 269t, 284t
681, 682683, 685t erconazo e, 588t, 597t methyc othiazide, 269t, 284t
protein tyrosine kinase inhibitors, eriparatide, 480t meto azone, 269t, 284t
661t, 683t osteoporosis, 481t, 488t po ythiazide, 269t, 284t
dasatinib, 661t, 668, 683t er ipressin, 279t, 288t quinethazone, 269t, 284t
er otinib, 661t, 670672, 683t ertiary amine tricyc ics. See Serotonin/nor- trich ormethiazide, 269t, 284t
ge tinib, 661t, 670672, 683t epinephrine reuptake inhibitors T iazide/thiazide- ike diuretics (Na+-2C -
imatinib mesy ate, 661t, 666669, 681, (SNRIs), tertiary amine tricyc ics symporter inhibitors), 269t, 271,
682, 683t estosterone, 442t, 458t 271t, 272 , 284t
apatinib, 661t, 673, 683t adverse e ects, 454 T iazo idinediones, 470t, 475476, 479t
mechanisms o resistance, 661t e ects, direct and mediated, 449 adverse e ects, 476
ni otinib, 661t, 668, 683t gynecomastia, 456, 457 mechanisms o action, 470t
sora enib, 661t, 674, 683t hypogonadism, 456, 457 piog itazone, 470t, 475476, 479t
sunitinib, 661t, 674, 683t metabo ism, 449 rosig itazone, 470t, 475476, 479t
steroidogenesis inhibitors, 665t, 687t secretion, hCG, 422t, 427, 428, 429t T ienopyridines, 325t. See also C opidogre ;
abiraterone, 665t, 668 , 680, 687t synthesis, 449 Prasugre ; ic opidine
ketoconazo e, 665t, 687t estosterone bucca tab et, 442t, 458t bioactivation, 325t
tha idomide and derivatives, estosterone cypionate, 442t, 458t mechanisms o action, 323t, 330, 331
663t, 685t estosterone enanthate, 442t, 458t 6-T ioguanine (6- G), 636t, 655t
ena idomide, 663t, 666 , 685t estosterone ge , 442t, 458t mechanisms o action, 630 , 636t,
mechanisms o action, 666 estosterone patch, 442t, 458t 641642
tha idomide, 663t, 666 , 685t estosterone rep acement, 442t, 452454, 453 mechanisms o resistance, 636t
axanes, 642t, 648650, 656t estosterone supp ement T iopenta , 177t, 181t, 192t
docetaxe , 630 , 642t, 644b, adverse e ects, 454 context-sensitive ha -time, 214
648650, 656t detection, 454 T iopurine methy trans erase ( PM ),
mechanisms o action, 630 , 642t, estosterone undecanoate, 442t, 458t 502503, 503 , 506, 507
648650 etrabenazine, 239t, 249, 253t T iotepa, 627t, 632t, 651t
mechanisms o resistance, 642t, 644b etracaine, 224t mechanisms o action, 627t,
nab-pac itaxe , 630 , 642t, 644b, etracyc ine, 568t, 569 , 574, 575, 576t 628b629b, 630
648650, 656t etracyc ines, 568t, 569 , 576t mechanisms o resistance, 627t,
pac itaxe , 630 , 642t, 644b, doxycyc ine, 568t, 569 , 571, 576t 632633, 632b
648650, 656t ood contraindications, 574, 575 toxicities, 632b633b, 632t, 651t
toxicities, 656t MRSA in ections, 571 T rombin inhibitors, direct,
uses, 644t, 656t photosensitivity, 574, 575 323t, 333t
azarotene, 707t, 724t tetracyc ine, 568t, 569 , 576t argatroban, 323t, 330, 331, 333t
azobactam, 549t, 553, 558t tigecyc ine, 568t, 569 , 576t biva irudin, 323t, 333t
774
T rombin inhibitors (Cont.) T yrotoxicosis, 435 irinotecan, 630 , 643t, 644b645b,
dabigatran etexi ate, 323t, 328, 333t T yrotropin-re easing hormone ( RH), 425 , 649651, 657t
desirudin, 333t 426, 432, 433 mechanisms o action, 630 , 643t,
mechanisms o action, 323t T yroxine ( 4) 649651
T romboembo ic disorder drugs, 323333 unction test, 431t mechanisms o resistance, 643t,
anticoagu ants (See Anticoagu ants) hypothyroidism, 432 644b645b
antip ate ets (See Antip ate et agents) iagabine, 226t, 237t topotecan, 630 , 643t, 644b645b,
brino ytics, 323t, 329330, 329 , 333t icarci in, 552t, 556t 649651, 657t
(See also Fibrino ytics, tPA) ic opidine toxicities, 657t
T rombopoietic growth actors, 405t, 406t, antip ate et, 323t, 332t uses, 644t, 657t
407t, 419t bioactivation, 325t epipodophy otoxins, 643t, 657t
e trombopag, 406t, 419t mechanisms o action, 323t, 330, 331 etoposide, 630 , 643t, 645b, 657t
IL-11 (opre vekin), 406t, 410411, 419t igecyc ine, 568t, 569 , 574, 575, 576t mechanisms o action, 630 , 643t
mechanisms o action, 405t, 407t, 408 i udronate, 481t, 488t mechanisms o resistance, 643t, 645b
romip ostim, 406t, 417, 418, 419t imo o teniposide, 630 , 643t, 645b, 657t
T rombopoietin ( PO, Mp igand), 407t, 408 myocardia ischemia, 300t toxicities, 657t
T romboxane A2, 324, 324t, 324 , 325 ophtha mic, 691t, 695t, 696697, uses, 644t, 657t
T yroid and antithyroid drugs, 431440 697 , 704t opotecan, 643t, 649651, 657t
antithyroid compounds, 435t pharmaco ogy, 126t, 128t, 145t mechanisms o action, 630 , 643t,
antithyroid drugs, 431t, 439t440t inea corporis, 719 649651
carbimazo e, 431t, 439t inzaparin, 323t, 325, 333t mechanisms o resistance, 643t,
methimazo e, 431t, 435436, 438, ioconazo e, 588t, 597t 644b645b
439, 440t iotropium, 96t, 113t oremi ene, 442t, 458t
propy thiouraci , 431t, 439t iotropium bromide, 390t, 401402, 403t anticancer, 664t, 677t, 679t, 686t
hyperthyroidism, 432t, 434t, 435439 ipranavir, 601t, 603t, 608 , 609 , 622t toxicities, 686t
hypothyroidism, 431t, 432434, 432t, iro ban, 323t, 324325, 332t uses, 686t
433 , 434t issue binding, 22b23b orsade de pointes, 313315, 315b
ionic thyroid inhibitors, 431t, 440t e der y, 75t antiarrhythmics or, 314t
iodide, 431t, 440t issue distribution, 21b orsemide, 269t, 284t
sodium iodide, 431t, 440t issue injury, pain, 197b, 197 oxicity, c inica and environmenta , 4066.
mechanisms o action, 431t issue per usion, e der y, 74b See also speci c substances
papi ary thyroid cancer, 436 izanidine ABCDE emergency care, 51, 52t
radioactive iodine (131I), 431t, 440t amyotrophic atera sc erosis, ABCDE treatment, 52t
thyroid hormones, 431t, 439t 239t, 249, 253t acetaminophen, 2829, 5054, 50 , 52t,
desiccated thyroid, 431t, 439t pharmaco ogy, 119t, 140t 207, 208
evothyroxine, 431t, 432434, 434t, MC-207 acety cysteine, 40t, 42t, 54, 65t
436439, 439t eprosy, 579t, 587t adverse drug events, 5657
iotrix, 431t, 439t Mycobacterium avium comp ex, a atoxin B1, 41t, 6062, 61
triiodothyroxine, 431t, 439t 579t, 587t anaphy axis, 56
T yroid cancer, papi ary, 436 obacco. See Smoking antidotes, 42t, 65t
T yroid, desiccated, 431t, 439t obramycin, 559566, 559b, 566t arsenic, 41t, 43t, 63, 64, 66t
T yroid disease o capone, 238t, 242t, 244 , 252t atropine, 40t, 65t
hyperthyroidism, 432t, 434t, o erance benzo[a]pyrene, 41t, 6365
435439 barbiturates, 181182 cadmium, 41t, 43t, 65t
hypothyroidism, 431t, 432434, 432t, heroin, 260 carcinogens, 43t, 6062, 60 , 61
433 , 434t, 439t opioid, 206, 207 che ators, heavy meta , 66t
symptoms, 432t oxycodone, 255, 256t chromium, 41t, 43t, 66t
T yroidectomy, 437, 438 o erogens, 381t deaths, drug-re ated, 51, 51t
T yroid unctions tests, 431t o metin, 357t, 361t, 373t de erasirox, 41t, 65t
T yroid hormone, 431t, 439t o na ate, 598t de eroxamine, 40t, 42t, 55, 65t
anti-thyroid hormone compounds, 435t o terodine, 96t, 101t, 113t dimercapro , 41t, 42t, 66t
biosynthesis and re ease o vaptan, 279t, 281, 288t dimercaptosuccinic acid, 41t, 66t
pathways, 433 opica administration, 710b, 711715, diphenhydramine, 40t, 42t, 65t (See also
desiccated thyroid, 431t, 439t 711t, 713 Diphenhydramine)
evothyroxine, 431t, 432434, 434t, important considerations, dose-response re ationships, 44 , 45
436439, 439t 711712, 711t drugdrug interactions, 48, 48b, 48t
iotrix, 431t, 439t rate- imiting step, 721, 722 drug interactions, 47
mechanisms o action, 431t systemic spread, 702, 703 drug overdose, initia management,
secretion regu ation, 433 or topica in ections, 714715 63, 64
triiodothyroxine, 431t, 439t vehic es, 712714, 712t drug-re ated deaths, agents, 51t
T yroid inhibitors, ionic, 431t, 440t opiramate ED A CaNa2, 41t, 42t, 63, 64, 66t
iodide, 431t, 440t antiseizure, 226t, 235, 236, 237t, e ective vs. etha dose, 44
sodium iodide, 431t, 440t 702, 703 ethano , 41t, 49 , 65t
T yroid-stimu ating hormone ( SH) g aucoma rom, 702, 703 ethano antidote, 42t, 50, 65t
unction test, 431t, 432, 437, 438 opoisomerase inhibitors, 643t, 657t FDA drug approva process, 44 , 45t,
pathway and actions, 432, 433 camptothecin ana ogs, 643t, 657t 4648, 46t

775
umazeni , 41t, 42t, 65t, 177t, ravoprost, ophtha mic, 691t, 705t ubercu in skin test reaction, 582, 585, 586
179180, 192t razodone, 150t, 174t ubercu osis drugs, 578587, 578t579t,
omepizo e, 41t, 42t, 65t, 178 reprostini , 391t, 404t 586t587t
iron poisoning, 5455 retinoin capreomycin, 579t, 587t
argest number ata ities, 52t cytotoxic, 645t, 646t, 659t combination therapy, 581
ead, 41t, 43t, 5758, 57b, 58 , 63, dermato ogic, 707t, 724t cyc oserine, 579t, 587t
64, 65t riamcino one, 390t, 403t de nitive therapy, 581b
management, 5052 ophtha mic, 691t, 705t ethambuto , 578t, 583, 587t
management, initia , 63, 64 riamcino one acetonide, 707t, 722, 723, ethionamide, 579t, 587t
medication errors, 5657, 57t 723t, 724t isoniazid, 578t, 580 , 582, 583t, 587t
mercury, 41t, 43t, 5960, 59 , 65t riamcino one hexacetonide, 707t, 722, 723, isoniazid-resistance, 583
meta s, heavy, 41t, 43t, 65t66t 723t, 724t mechanisms o action, 578t579t, 579
methano , 42t, 4950, 49 riamterene, 269t, 282, 283, 284t mechanisms o resistance,
most common substances, 51t riazenes, 628t, 652t 578t579t, 580
most requent substances, 51, 51t dacarbazine, 628t, 629t, 630 , PA-824, 579t, 587t
na oxone, 41t, 42t, 65t (See also Na oxone) 645647, 652t penetration, 585, 586
new drug testing, 44 , 45, 45t mechanisms o action, 628b, 628t, 630 prophy axis criteria, 580b
organophosphates, 53t, 63, 64, 100t mechanisms o resistance, 628t, pyrazinamide, 578t, 580b, 586t
penici amine, 42t, 66t 632633, 632b resistant and mu tidrug-resistant B,
penici in, 5556 temozo omide, 652t 584, 584b
pharmacokinetics, 52 toxicities, 632b633b, 652t ri amycins, 578t, 581586, 586t
physostigmine, 41t, 42t, 65t, 102 uses, 629t mechanisms o action, 578t, 579
postmarketing survei ance, 911, 4648 riazo am, 176t, 191t, 192t mechanisms o resistance, 578t, 580
pra idoxime ch oride, 41t, 42t, 65t, riazo es, 588t, 589 , 590, 597t. See also Imid- ri abutin, 578t, 586t
100t, 102 azo es and triazo es ri ampin, 578t, 579 , 581586, 586t
prevention, chi dren, 6263, 64 rich ormethiazide, 269t, 284t ri apentine, 578t, 586t
sodium 2,3-dimercaptopropane, 41t, 66t Trichomonas vaginalis, 533, 534 tubercu in skin test reaction, 582,
spectrum o e ects, pharmaceutica , 44 ricyc ic antidepressants ( CAs). See also 585, 586
succimer, 41t, 42t, 66t Antidepressants 22-oxaca citro parica cito , 481t, 488t
su amethoxazo e, 5657 adverse e ects, 154 2-PAM. See Pra idoxime ch oride (2-PAM)
therapeutic drug, 44 , 46, 46t e der y, 71, 72, 75 y oxapo , ophtha mic, 692t, 706t
therapeutic index, 62, 64 pain target and site o action, 205t ypica angina, 290t, 291 . See also Myocar-
toxidromes, 51, 53t sites o actions, 152 dia ischemia
types, therapeutic drugs, 46, 46t ri uoperazine, 150t, 174t yrosine kinase ( K) growth actor receptor
urine testing, 5152 ri uridine, 601t, 602t, 606, 619t signa ing pathway, 666
oxidromes, 51, 53t. See also speci c agents rig yceride eve s, 337t
and types rihexyphenidy hydroch oride U
oxop asmosis, 532b Parkinsons disease, 242t U cerative co itis
tPA, 323t, 333t. See also Fibrino ytics, tPA pharmaco ogy, 96t, 114t azathioprine, 502503, 503 , 506, 507
a tep ase, 323t, 329330, 329 , 333t riiodothyronine ( 3) su asa azine, 381, 502, 543t, 547
mechanisms o action, 323t unction test, 431t thiopurine methy trans erase, 502503,
retep ase, 323t, 333t hypothyroidism, 432 503 , 506, 507
tenectep ase, 323t, 333t riiodothyroxine, 431t, 439t U cers
PM , 502503, 503 , 506, 507 rimethaphan, 106t, 117t NSAID-induced, 492
rabectedin, 643t, 659t rimethoprim-su amethoxazo e, proton pump inhibitors, 492, 493, 494t,
ramado , 195t, 200t, 209t 542543, 548t 495 (See also Proton pump
rando apri , 273t, 285t a ergy, 56, 543 inhibitors)
ransendothe ia ux, 28b, 28 bacteria vs. mamma ian sensitivity, U iprista , 442t, 444t, 452, 458t
ransepithe ia ux, 28b, 28 546, 547 Undecy enic acid, 599t
ransmembrane enzymes, 3t o ate de ciency, 543, 543 Unstab e angina, 290t, 291 , 296297. See also
ransport resistance, 543 Myocardia ischemia
active, 19b, 19 , 20b structure, 543 drugs, 296297
parace u ar, 19b, 19 urinary tract in ections, 56, 542543, percutaneous coronary
passive vs. active, 19b, 19 543 , 546, 547, 548t interventions, 297
ransporters. See also speci c types rimetrexate, 635t, 640, 653t Uptake transporters, 24b
ABC, 23t, 23t25t rimipramine, 150t, 173t Urapidi , 120t, 141t
catecho amine p asma membrane, 124t rioxsa en photochemotherapy, 716t Urate oxidase, recombinant, 369t, 374t
ef ux, 24b ripe ennamine, 346t, 353t Urea, diuretics or, 268t, 283t
e der y, 75t rip e response o Lewis, 347b, 348 Urea substitutes, 630 , 645t, 646t, 659t
p asma membrane, catecho amine, 124t riptore in, 422t, 429t Uric acid, 370
SLC super ami y, 23b24b, 25t26t anticancer, 664t, 686t Uricosuric agents, 369t, 370, 374t
uptake, 24b ropicamide, 96t, 114t Uridine diphosphate-g ucuronosy trans erase
rany cypromine, 130t, 135, 137, 147t, ophtha mic, 690t, 695t, 705t (UG ), antiseizure drugs
150t, 173t rospium ch oride, 96t, 101t, 113t and, 227t
rastuzumab, 662t, 671t, 672673, 681, Trypanosoma brucei rhodesiense, 534 Urinary tract in ection agents, 542548
682, 683t rypanosomiasis, 533, 534 ana gesics, phenazopyridine, 548t

776
Urinary tract in ection agents (Cont.) arteria and venous, 278t, 287t Vira in ections
antiseptics nitroprusside, 278t, 287t cytomega ovirus retinitis, 609612,
methenamine, 548t heart ai ure, 301t, 304t, 311t 617, 618
nitro urantoin, 545, 546, 547, 548t Vasopressin, 279t, 288t drugs (See antivira agents)
quino ones, 544, 548t Vasopressin receptor agonists, 288t eye, 701
cipro oxacin, 544, 544 , 545, 548t mechanisms o action, 279t, 280 hepatitis C, chronic, 613614
evo oxacin, 548t V1 receptor, 279t, 288t herpes simp ex virus, 605608
mechanisms o action, 544 , 545 ter ipressin, 279t, 288t HIV, 614619 (See also Human
moxi oxacin, 548t vasopressin, 279t, 288t immunode ciency virus (HIV))
nor oxacin, 548t V2 receptor, 279t, 288t HSV, eye, 701
o oxacin, 548t desmopressin, 279t, 288t in uenza A, immunocompromised,
resistance, 544 Vasopressin receptor antagonists 612613
structures, 543 mechanisms o action, 279t, 280 rep icative cyc es, 604
su adiazine, 543 , 543t, 547t V1aR/V2R, nonse ective, conivaptan, 279t, varice a zoster virus (shing es), 608
su amethoxazo e, 543 , 543t, 547t 281, 288t virus rep ication stages and targets, 612t
trimethoprim-su amethoxazo e, 56, V2 receptor, se ective, to vaptan, 279t, Vitamin A de ciency, 703, 704
542543, 543 , 546, 547, 548t 281, 288t Vitamin B12, 406t, 420t
uncomp icated U I, 542543 Vasospastic angina, sinus bradycardia with, absorption, 412
Urinary tract in ections, 548t 297, 298 proton pump inhibitors on, 492
Urine Vecuronium, 97 , 105 , 106 , 106t, 107t, cyanocoba amin, 406t, 420t
acidi cation and a ka ization, 39 108, 116t distribution, 412
pH, 20b, 2122 Ven a axine, 130t, 147t, 150t, 154, o ate interactions, 410b, 412
testing, 5152 157, 174t genera princip es, 411b412b
Uro o itropin (uFSH), 423t, 429t Venti atory stimu ants, 391t, 404t hydroxycoba amin, 406t, 420t
Ventricu ar bri ation, 314t, 316 mechanisms o action, 406t, 420t
V antiarrhythmics, 314t, 316 (See also Vitamin B12 de ciency, 412 , 413b,
Va acyc ovir, 601t, 605, 608 Antiarrhythmic drugs) 414415, 417418
Va gancic ovir, 601t. See also Acyc ovir imp antab e cardioverter-de bri ator, 316 causes, 411b, 412
Va proic acid Ventricu ar tachycardia sh tapeworm, 537540
anticonvu sants, 151t, 164165, 169, antiarrhythmics, 314t Vitamin D 1-hydroxy ase, 482 , 486, 487
173, 175t imp antab e cardioverter-de bri ator, 318 Vitamin D and ana ogs, 481t, 482 , 484, 488t
antiseizure, 225t, 231232, 234, nonsustained, 318 1-hydroxycho eca ci ero , 481t, 488t
235, 236t Verapami 22-oxaca citro parica cito , 481t, 488t
with amotrigine, 235 antiarrhythmic, 312t, 317t, 322t ca cipotrio , 481t, 488t
Va rubicin, 643t, 658t antihypertensive, 277t, 287t ca citrio , 481t, 482 , 488t
mechanisms o action, 630 , 643t, e ectrophysio ogic actions, 317t dihydrotachystero , 481b
645647 heart ai ure, 306 doxerca ci ero , 481b
mechanisms o resistance, 645b myocardia ischemia, 290t, 300t ergoca ci ero , 481t, 488t
uses, 644t, 658t toxidrome, 53t hyperca cemia, 481t, 488t
Va sartan, 273t, 280, 282, 283, 285t Vi dag iptan, 471t, 472 , 479t intestina bypass surgery, 486, 487
Va vu ar heart disease, angina drugs, 294t Vigabatrin parica cito , 481t, 488t
Vancomycin, 569t, 570 , 572575, 577t antiseizure, 226t, 235, 236, 237t, rena osteodystrophy, 481t, 482 ,
empirica treatment, 572 702, 703 484, 488t
IV, hazards, 573, 574, 575 ocu ar e ects, 702, 703 therapeutic uses, 481b
mechanisms o action, 569t, 572 Vinb astine (su ate), 642t, 656t Vitamin K1, war arin antidote, 14, 42t
resistance, 569t, 573 dermato ogic, 709t, 724t, 726t Vitamin K antagonist, 333t
Vardena mechanisms o action, 630 , 642t, 647 mechanisms o action, 323t
myocardia ischemia, 300t mechanisms o resistance, 642t, 644b war arin (See War arin)
ocu ar e ects, 703 toxicities, 656t Vitamin K cyc e, 327328, 327
Varenic ine, 106t, 117t uses, 644t, 656t Vitreous substitutes
nicotine addiction, 259 Vinb astine su ate, 630 , 642t, 644b, ophtha mic, 692t, 706t
Variant angina, 290t, 291 , 297. See also 644t, 656t VKORC1, 1314, 13
Myocardia ischemia Vinca a ka oids, 642t, 656t. See also VLDL-C metabo ism, 335
Vascu ar endothe ia growth speci c types VM-26. See eniposide (VM-26)
actor (VEGF) mechanisms o resistance, 642t, 644b Vog ibose, 471t, 479t
cancer angiogenesis, 673674 toxicities, 656t Vo tage-dependent ion channe s, 86, 87
monoc ona antibodies targeting, 662t, uses, 644t, 656t Vo tage-gated Na+ channe s
671t, 673675 Vincristine su ate, 630 , 642t, 644b, oca anesthetics on, 217
Vasodi ators, 270t, 278t, 301t 644t, 656t structure and unction, 218
arteria , 278t, 287t mechanisms o action, 630 , 642t Vo ume o distribution (V), 30, 33b, 34b
diazoxide, 278t, 287t mechanisms o resistance, 642t, 644b de nition and equation, 34b
eno dopam, 278t, 287t uses, 644t, 656t digoxin, 30
hydra azine, 278t, 287t Vinore bine, 630 , 642t, 644b, 644t, 656t e der y, 74b, 75t
hypertension and edema, mechanisms o action, 630 , 642t variations, 34b
278t, 287t mechanisms o resistance, 642t, 644b Vomiting drugs. See Antinauseants
minoxidi , 278t, 287t uses, 644t, 656t and antiemetics

777
Voriconazo e, 588t, 590, 592593, Weak e ectro ytes, transmembrane Y
595596, 597t distribution, 1920, 19b20b, Y-ibritumomab, 662t, 671t, 684t
99

adverse e ects, 593, 595, 596 19 , 20 , 35, 3738 Yohimbine, 120t, 141t
drug interactions, 590t592t, 593 Wetting agents/tear substitutes, 692t, 706t
mechanisms o action, 588t, 589 , 592 carboxymethy ce u ose, 692t, 706t Z
Vorinostat (SAHA), 645t, 646t, 660t hydroxyethy ce u ose, 692t, 706t Za r ukast, 390t, 404t
VP-16-213. See Etoposide (VP-16-213) hydroxypropy ce u ose, 692t, 706t Za citabine, 601t, 621t
Vytorin, 344t, 350 hydroxypropy methy ce u ose, mechanisms o action and resistance, 601t
692t, 706t structure, 607
W methy ce u ose, 692t, 706t Za ep on, 177t, 192t
War arin, 323t, 325328, 333t ty oxapo , 692t, 706t Zanamivir, 601t, 602t, 604 , 620t
a tered activity, 329b Whit e ds ointment, 599t Zidovudine, 601t, 602t, 621t
a tered anticoagu ant activity, 329b330b Wo -Parkinson-White syndrome adverse e ects, 615
atria bri ation, 327 arrhythmias, 314t mechanisms o action and resistance, 517,
counse ing, 328 Wound in ection prophy axis, 514b, 516 602t, 604 , 607 , 608 , 614615
ooddrug interactions, 328 structure, 607
indications, 325t X Zi euton, 390t, 403t
mechanisms o action, 327328, 327 Xanthine oxidase inhibitors, gout, 369t Ziprasidone, 151t, 171t, 175t
monitoring, 325, 325t, 328, 330b a opurino , 369t, 370372, 373t Zo edronate, 481t, 485, 488t
post-stenting, 325 ebuxostat, 369t, 370, 374t Zo mitriptan, 130t, 147t
risks and toxicities, 328 Xenobiotic metabo izing enzymes, Zo pidem, 177t, 178179, 192t
therapeutic index, 329b 29b, 29t Zona g omeru osa, 461 , 467, 468
War arin sensitivity, 10t, 1214, 13 Xenon, 211t, 215t, 223t Zonisamide, 226t, 237t
vitamin K (K1), 14, 42t Xy ometazo ine, 120t, 142t

778

You might also like